.
.
Math Central - mathcentral.uregina.ca
Quandaries & Queries
Q & Q
. .
topic card  

Topic:

l

list of
topics
. .
start over

9238 items are filed under this topic.
 
Page
1/1
The diagonals of a polygon 2016-01-29
From Sofia:
What are diagonals and if you drew an octagon and drew all the diagonals how many would there be? Please explain because this is pretty confusing! Thank you!
Answered by Harley Weston.
More on millions and billions 2010-03-11
From julio:
shouldn't the thousands of millions, tens of thousands of millions, and hundreds of thousands of millions be called what they are? technically they are not billions because billions come after 999,999,999,999 . It would sound better to call the numbers by their real name when economists talk about the national deficit besides mathematics is a very precise science that doesn't admit mistakes. Or am I wrong? I know that in other countries is expressed different than here. I have this question to different people and nobody has a "logical" explanation to it. " that's the way is here in the US" does not explain any thing
Answered by Penny Nom.
Foiled by foil 2009-07-20
From RAY:
how u do the foil ?

the question (x+4)(x+5)

Answered by Robert Dawson.
16 golfers 2009-03-18
From Bill:
We have 16 golfers. We golf for 5 days, in foursomes. It should work out that each player plays with each other player 1 time exactly.
I just can't figure it out.

Thank you in advance!

Bill

Answered by Claude Tardif and Victoria West.
A barrel on its side 2008-11-13
From Dave:
Question from Dave:

How many gallons are left in a 36x60 in. barrel (laying on its side) and has 16 in. of gasoline left. I have attached a diagram.

Answered by Harley Weston.
Calculating the area (acreage) of a four sided lot 2007-07-18
From A property owner:
I have a real estate property and the lot size is something I need to find out. I know the lengths of the four sides, but it isn't a rectangle, it is an odd shape. How do I find the acreage?
Answered by Stephen La Rocque.
Lot size / acreage (more than three sides) 2007-05-11
From Martha:
What is the lot size of the following dimensions equivalent to acre. 141.85 x 52.55 x 217.63 x 38.89 x 68.08
Answered by Penny Nom and Stephen La Rocque.
Finding the area of a regular polygon 2007-05-04
From Dana:
We are trying to figure the square footage of a tetradecagon....sort of a round house with 14 sides that are 8 ft' in length. It has a height of 9 ft. How do we figure the square footage of this?
Answered by Stephen La Rocque.
Calculating volume of concrete, dirt, sand, anything. 2006-11-17
From Debbie:
[This article contains a special calculator to help you find the volume (for example, the cubic yards) of material required to fill (or empty) a space if you know the length, width and depth.]
We are wanting to pour a 24'x30' slab that is 5" deep. How many cubic yards of cement do we need?

Answered by Penny Nom.
Billions and more! and even more! 2004-03-01
From Steph:
What comes after undecillion?
Answered by Penny Nom.
Billions and more! 1997-09-15
From Mahabir B. Gupta:
I would like to know how you americans write the number 1 billion.

Do you say "One thousand million"..can you answer by giving me examples?

1,000,000----> 1 million
1,000,000,000---->1 billion

Why is it that in spanish it is different?
Answered by Penny Nom.

Why is lbs the abbreviation for pounds? 1996-10-03
From Nancy Mutdosch:
What does lbs stand for? Why is it the abbreviation for pounds?
Answered by Chris Fisher.
The area of a lot 2022-12-31
From Brian:
To calculate lot size, do you use arc length or chord length? Lot Survey shows: Length 1 = 120.0' Length 2 = 120.0' Width 1 = 61.0' Width 2 = ARC Length 125.58' / CHORD Length 124.10' / Radius = 236.0'
Answered by Harley.
A lock combination 2022-07-19
From Kendal:
If I tried I could probably (I think) figure the answer but it would take a minute and I hoped maybe as math people there may be a simple formula… I just watched a new movie Black Phone, the kid is given 5 numbers for a combination lock in order but with no indication where the stops are. The numbers (in order) are 23317, my question is how many possibilities with a standard combination lock are there…ie. 2-33-17, 23-3-17, 23-31-7…. As you can imagine not a great movie…my curiosity may get the best of me depending on how your app works ;). Thanks for your time.
Answered by Penny Nom.
Strings of letters 2022-04-25
From Javar:
How many different combinations can I make with A 12 selection list but only using A&B

Example:

A,B,A,B,A,B,A,B,A,B,A,B
A,B,B,A,A,B,B,A,A,B,B,A
B,B,A,A,B,A,B,A,B,A,A,B

Answered by penny Nom.
The length of a line at 45 degrees 2022-04-09
From Aiden:
Let's say one line is diagonal and one line is vertical. If they eventually meet at a 45 Degree angle, How long will the diagonal line be compared to the vertical line? (For Example: If The Vertical Line is Exactly 1 Inch.
Answered by penny Nom.
A word problem 2022-01-29
From Ric:
Box of oranges weighs 9 pounds. 2/3 of oranges eaten. Box and remaining oranges weighs 4 pounds. What is weight of box when empty?
Answered by Penny Nom.
The next rational number 2022-01-14
From Sophia:
what comes after 3 1/2?
Answered by Penny Nom.
A Word Problem 2021-12-17
From Bryson:
Yolanda, Brian, and Charlie have a total of in their wallets. Brian has less than Yolanda. Charlie has times what Brian has. How much do they have in their wallets?
Answered by Penny Nom.
The height of a right triangle 2021-12-15
From Slash:
hi how can i find the height of a triangle from the hypotenuse and base. please go into some details with some steps because it is hard to understand. thanks
Answered by Penny Nom.
Simplifying a complex expression 2021-12-15
From Grace:
Write in the form. a+bi, -6-7i+8i^2+4i3-5i^4
Answered by Penny Nom.
Roll of film 2021-11-20
From Brian:
Trying to create a formula to calculate the build up on a roll of film is calculated. Except instead of of a roll, it's essentially a rectangle with two semicircles on each at the end. Example: I start with a 6" diameter core on each side, and I start winding .005" thick film on the core. The distance between the center of those cores, length of the inner rectangle is 7.125. If I continue to do this for a total length of film of 3000 feet, what will the roll length and width be. So what I need is the formula to perform this type of calculation. Can you help me? I want to be able to plug the formula in a spread sheet and to be able to input a core diameter, a film thickness and a total length and the length between the center of the two cores(which is the inner rectangle's length I'm assuming). Thanks
Answered by Harley Weston.
The sides of a 5-pointed star inscribed in a circle 2021-09-18
From Scott:
Is there a calculation for the length of each side of a five-pointed star? I am not worried about area but if I have a circle with a 7.5 foot radius and place a star inside it, how can I measure the lines creating the star?
Answered by Harley Weston.
The dimensions of a rectangle 2021-09-09
From Ash:
I need to know how to figure out the dimensions of a rectangle. I know that the area is 30ft and the perimeter is 26ft. I'm 15 and I have no idea what I'm supposed to do.
Answered by Harley Weston.
30 % off 2021-09-07
From Corrine:
When I am shopping for clothing that is for sale, the advertisement may say the item is 30% off. The original price was $70. How much would it now be? (how do I figure that out?) Thank you!
Answered by Penny Nom.
Fibonacci numbers 2021-08-25
From John:
Make a single column of numbers. Start with two numbers of your choice. The third number is the sum of the previous two, the fourth number is the sum of numbers two and three, and so on until you have ten numbers in the column. Add up all ten numbers. Now, take the seventh number and multiply it by eleven. This product will equal the sum of the ten numbers. The same result will occur regardless of the first two numbers chosen. The question is why does the 7th number multiplied by 11 always equal the sum of the ten numbers?
Answered by Penny Nom.
A word problem 2021-07-28
From Isaac:
The average mass of a group of children is 50 kilograms. Todd, who has a mass of 62 kilograms, then joins the group. This raises the average mass of the group to 52 kilograms. How many children were in the original group
Answered by Penny Nom.
New pool liner 2021-07-14
From Kareena:
I have a rectangular prism shaped pond that are 4ft by 8ft by 2ft. My pond is lined on the sides and bottom with a pool liner so that it doesn't leak (the top of the pool does NOT have a liner). I am redoing my pool and would like to increase the size of the pond. Each dimension will increase by the same amount, and the size of the pool liner will double. How much I am increasing each dimension of the pond? Give an exact answer and than an approx. answer rounded to the nearest tenth.
Answered by Penny Nom.
3.1 miles in 4 minutes 2021-06-21
From Jean:
I am a school bus driver. My route directions and time schedule have me traveling 3.1 miles in 4 minutes. How fast do "they" expect me to drive to accomplish this? Thank You
Answered by Penny Nom.
UFO speed in MPH from nearest galaxy in one year 2021-05-22
From Joe:
If the next closest galaxy is 15 quadrillion miles away and there are 8760 hours in one year; how fast would a UFO have to fly to travel from the next galaxy, in MPH in one year ?
Google calculator gives this result: 1.7123288e+12
I don't understand what the "e+12" means.
How is that pronounced, ex. trillion, quadrillion etc..?

Answered by Harley.
2.5 percent of 1.5 billion 2021-05-13
From Anish:
What is the 2.5 percent of 1.5 billion?
please state if there are any general formulae of this calculation.

Thank you

Answered by Penny Nom.
An octagonal drum 2021-05-11
From Louie:
I teach carpentry on an Indian reservation.
My students want to make an 18 inch drum frame.
They want to have 8 sides or an octagon shape to the drum frame.
What angle I have to set my miter saw to cut the angles to fit the octagon together.
I want to use this formula by changing the diameter of the frame.

Answered by Harley Weston.
More on a roll of paper 2021-05-07
From Stephen:
Can you help with the equations to calculate the length of paper required to achieve a target outside diameter when wrapped around a core please?

The inside diameter of the cardboard core is 76mm
The thickness of the wall of the cardboard core is 5mm
The thickness of the paper is 138microns
The desired outside diameter of the finished roll is 320mm

I hope you can help me with a solution, Thank you

Answered by Harley Weston.
Word problem 2021-05-07
From Donelle:
Freddy is three times as old as Mary. Mary is twice as old as Jack. Jack is 11 years old. How old is Freddy?
Answered by Penny Nom.
The maximum area of a garden 2021-04-28
From Lexie:
suppose you want to make a rectangular garden with the perimeter of 24 meters. What's the greatest the area could be and what are the dimensions?
Answered by Penny Nom.
The volume of a spherical bowl 2021-04-23
From Danny:
How many gallons of water in a glass bowl with the dimensions of 10 inches high and 13.5 inches wide?
Answered by Penny Nom.
Rates & Ratios 2021-04-11
From Kenneth:
Is it correct to think that a rate can have units that are the same? Usually rates have units that are different to each other.
Answered by Penny Nom.
Four friends in a race 2021-04-06
From Angel:
Four friends ran a race:
Ben finished seven seconds ahead of Mike.
Noel finished three seconds behind Sam.
Mike finished five seconds behind Noel.
What will be the formula to support their place?

Answered by Penny Nom.
Scale drawing 2021-02-10
From Elias:
a football field is 160m long and 120m wide . Find the distance between its opposite corner flag posts . use the scale of 1cm:20m
Answered by Penny Nom.
Simplify 2021-02-05
From Leslie:
To simplify improper fractions, do you convert it to a mixed number or leave as an improper fraction?

My daughter had a quiz in which she was supposed to simplify her answers. So, for example, on one question the answer she got was 11/10 and on another question she came up with 17/12. The teacher marked these as incorrect because they were mixed numbers. Her teacher wrote on her paper, "**name**, I even gave this back to you and told you to go through and simplify your answers!" We (her father and I) feel that the teacher wanted her to convert her answers, not simplify them but I haven't been able to find a definitive answer online. So, does simplifying improper fractions mean converting them to mixed numbers? Thank you for your help!


Answered by Penny Nom.
A scale factor 2021-02-02
From Layla:
Beth planned to travel from Battery Park to Times Square. The distance between the two places is 2.48 inches on the map of New York City. If the scale factor used for the map was 1 mile = 0.4 inch, find the actual distance between the two tourist attractions.
Answered by Penny Nom.
A scrambled word 2021-01-19
From john:
How many ways can you scramble the work "0axaca" if two a's must never be adjacent?
Answered by Penny Nom.
A scenario for a linear equation 2021-01-19
From raven:
I have to create a scenario for the following equation y=-1x+45 and then find what each part of the equation represents
Answered by Penny Nom.
Velocity and acceleration 2021-01-09
From yen:
The motion of a stone projected upward is given by the relation s=112t - 16t^2. What are the values of velocity and acceleration when t=3 sec?
Answered by Penny Nom.
10^1,000,000 2020-12-18
From G:
Looking for the name of a really big number. its a 1. with a Million zeroes after it.
Answered by Harley Weston.
0^(a+bi) 2020-12-03
From Douglas:
I realize raising 0^a = 0 if a>0 and undefined if a<=0.

If have read that 0^bi is undefined for all b.

What I don't understand is why 0^(a+bi) = 0 if a and b are not equal to zero.

Is this purely by definition or is there a logical reason why this is the case?
(I have taken Complex Analysis, so have a fairly good understanding of complex numbers.)

Answered by Penny Nom.
Comparing two job offers 2020-11-17
From Selena:
Raja has been offered two jobs.
Each of these jobs takes 24 weeks to complete.
One job pays $3440 every 8 weeks. The other job pays $2700 every 6 weeks. Raja wants to accept the job that pays more per week.
Show how to use equations to help Raja make her choice.

Answered by Penny Nom.
The distance between opposite vertices in an octagon 2020-11-14
From joe:
If the dimension from one flat part of an octagon to opposite flat is 500 mm what is the largest diameter (point to point) of the octagon?
Answered by Harley Weston.
Golf for 28 golfers over 4 days 2020-11-14
From David:
hi, I was wondering if you could help me with a golf draw, I have 28 players and wish to play a four round foursomes over 4 days
I would like to minimize the amount of games that they play together.

Answered by Harley Weston.
Divisibility by 8 2020-11-10
From Sariyah:
Of the positive integers between 1000 and 10000 that are divisible by 8, how many have a 5 in the hundreds place?
Answered by Harley Weston.
The dimensions of a rectangle 2020-11-02
From Chie:
A rectangular piece of property has an area of 987m² and a perimeter of 136m. Find its LENGTH and WIDTH.
Answered by Penny Nom.
Running around a circular track 2020-10-31
From kaylee:
Keiko and Kendra are training for an international track competition. Kendra is running one lap on the outside of a circular track with a diameter of 1000 feet. Her constant rate of speed is 12 mi/h. Keiko is running one lap on the outside of a nearby circular track with a diameter of 1125 feet. She is also running at 12 mi/h. Find the time, in minutes, each runner needs to complete one lap.
Answered by Harley Weston.
Equivalent fractions 2020-10-31
From Lexi:
Which of the following fractions is equivalent to -84 / -90?
A. -14 / 15
B. -42 / 45
C. 14 / 15
D. 42 / 45

Answered by Penny Nom.
An analytic proof that a quadrilateral is a parallelogram 2020-10-26
From Apollo:
Prove analytically that if ABCD is a parallelogram in which points P and Q trisects the diagonal AC, then BPDQ is a parallelogram.
Answered by Penny Nom.
Two equations with fractions 2020-10-22
From mia:
I am supposed to look for x and y. my teacher didn't explain very well.
please help : ((
2x - y = 5
x/4 + y/3= 2

Answered by Penny Nom.
2xy = 2x*2y 2020-10-22
From Amber:
Um i am wondering what this equation equals. Thank you for your time and help.

2xy=2x*2y

Answered by Penny Nom.
Solving an inequality 2020-10-18
From nunya:
3(4 + 2n) > 2n - 16
Answered by Penny Nom.
A word problem with three variables 2020-10-18
From Yesenia:
Carmen, Austin, and Trey have a total of $128 in their wallets. Trey has $10 more than Carmen. Austin has 4 times what Trey has. How much do they have in their wallets?
Answered by Penny Nom.
A confusing problem 2020-10-01
From Kgaugelo:
a triangle has the vertices A(-4;-3),B(x;y)and C(2;1). Determine the coordinates of B if the equation of the line BC is y=5x-9
Answered by Harley Weston.
A combination lock 2020-09-29
From Rourke:
how many possible combinations are there in a 4 digit lock with 6 numbers which can repeat themselves.
Answered by Penny Nom.
Factoring an interest formula 2020-09-29
From Kenneth:
Hello

P + Pr has a common factor of P so it can be expressed as P(1 + r) after the P is factored out.. How does the "P" get in front of (1 + r)?

P/P + Pr/P = 1 + r What step is used to show that P is added in front of (1 + r)?

I thank you for your reply.

Answered by Harley Weston.
A Parabolic Arch 2020-09-21
From Malen:
A hotel entrance makes a parabolic arch that can be represented by the quadratic function, y= -x^2-8x+24, where y is the height of the arch and x is the distance from wall to wall in the feet. What is the distance between the two walls of the arch.
Answered by Harley Weston.
my prcentage of years on earth is 2020-09-21
From Bill:
This Tuesday is my birthday and I will be 69. and I just wanted to know life ear is say earth is 6 billion years old I'm just curious to what percentage does my existence on earth reptresent I do realize that it's a flash in the pan
Answered by Penny Nom.
9 games with 3 possible outcomes 2020-09-13
From Daniel:
Hi
I’d like to know how many different outcomes would be in a Soccer week played if there is 9 different games 2 teams and 3 outcomes Win, tie or lose

Thank you

Answered by Penny Nom.
Simultaneous equations 2020-09-05
From Paa:
a+b=8
a+c=13
b+d=8
c-d=6
find the value of each letter

Answered by Penny Nom.
Slope 2020-09-04
From Mark:
I'm trying to calculate slope over 157'.I have a drop of 9 1/8 inches. What fraction of an inch of fall do I have every 10 feet
Answered by Penny Nom.
The angular speed of a wheel 2020-09-03
From Catrina:
A car is moving at a rate of 75 miles per hour, and the diameter of its wheels is 2.6 in. Find the angular speed of the wheels in radians per minute.
Answered by Penny Nom.
4 digits to unlock a padlock 2020-08-28
From Sophie:
I have forgotten the order in what the numbers go for my padlock! The numbers are, 0125, they don't have any repeats like, 0001, 0002. What are all possible the combinations? Help!
Answered by Penny Nom.
A diameter of a circle 2020-08-19
From Apple:
One end of the the diameter of the circle with equation x^2+y^2-4x+4y=2 is (3,1). Find the coordinates of the other end of the diameter.
Answered by Penny Nom.
A window problem 2020-08-18
From Richard:
Hello,
I was hoping your math specialists could help me with some formulas. I have shapes with specific known variables need to calculate others.  

Example:
We make a straight legged arch, This shape has a width a overall height and a leg height, The leg height is always less then the overall, And the top is arched. We have the width, Height, and leg size.

  Need to calculate the length of the curve and sq ft of the shape.

Answered by Harley Weston.
A word problem with no algebra 2020-08-11
From Julie:
At 3pm, Bruce headed north at 30 km/hr. 2 hours later, Lucy headed south at 40 km/hr. At what time will they be 340 km apart
Answered by Penny Nom.
More on the last 2 digits of a^k = N 2020-08-07
From Jay:
I've seen solution methods for finding the last 2 digits of a^k = N, where a and k are known. My question is:

What if the number N is known and the last 2 digits are, say, ...62. Is there a way to find, for any given a, the value of k which will result in a number where the last 2 digits are ...62? and if that is possible, then do the same for a number with the last n digits (say 7 or 53 or something)?

Answered by Harley Weston.
Filling holes with stone dust 2020-07-30
From Zach:
I’m looking for amount of stone dust needed for thirteen holes with a 3ft depth and 12in diameter with a 4x4 post. I would need the measurement in KG.
Answered by Harley Weston.
A tank, 5/9 full of water 2020-07-28
From Nat:
A rectangular tank contains 1200cmsquare of water when it is 5 out of 9 full . Its base area is 80cmsquare . Find the height of the tank.
Answered by Penny Nom.
Five rational numbers between -4 and -3 2020-07-28
From Priyanshi:
Five rational numbers between -4 and 3
Answered by Penny Nom.
Choosing 4 balls from 9 2020-07-22
From sandiso:
What is the probability associated with each possible random sample of size 4 from the population consisting of these nine balls
Answered by Penny Nom.
4sin(2x)cos(2x)+1=0 2020-07-02
From Sheila:
4sin(2x)cos(2x)+1=0
Answered by Penny Nom.
Scale factor 2020-06-26
From Kinza:
Hi, my question is The perimeter of a triangle is reduced from 75cm to 20cm. What is the linear scale factor?
Answered by Penny Nom.
A word problem involving linear equations 2020-06-22
From Allie:
The length of a rectangular swimming pool is p m and its width is q m. Its given tht the length of the swimming pool is twice its with. if the perimeter of the swimming pool is 150 m, find the values of p and q this is in the chapter linear equations
Answered by Penny Nom.
A word problem with fractions 2020-06-21
From Sthantsa:
I'm stuck with this question, I don't know where to begin because i really don't Know where to start.

Jessie and Thulani each has a sum of money. Jessie’s amount is 2/5 that of Thulani’s. If Thulani were to give Jessie R198, then his remaining amount will be 6/8 that of Jessie’s amount.
How much does Jessie have originally??

Answered by Penny Nom.
The substitution method 2020-06-10
From Hardebolar:
simultaneous equation by substitution method

.a-b=2/3
a+b=1 2/3

Answered by p.
An octagonal pool deck 2020-05-30
From lauchie:
need help on cut sizes and cut degrees on octagon pool deck for a 24 foot round pool
Answered by Harley Weston.
A hexagonal planter 2020-05-28
From Callie:
Hello, I'm having trouble cutting my angles for my 2 x 4 planter. I did exactly as I've watched and read and mine is just not turning out! Ok, so I want 6 or 8 piece hexagon. With an interior of about 10". So I understand that the boards can be any length, but its the 30 degree or 22 1/2" angles that matter. So. How do I figure how long my 2 x 4 is with 6. and another 8 pieces of 2 x 4 to make this planter and at what angle. So does the length determine the angle? Thank you in advance! i don't want to waste anymore wood! And how to figure length, angle and center circumference?
Answered by Harley Weston.
Are all quadrilaterals the same? 2020-05-27
From eva:
are all quadrilaterals the same?
Answered by Penny Nom.
A triangular garden 2020-05-24
From yoonji:
the 3 sides of a triangular garden measure 200 ft., 250 ft., and 300 ft., respectively. the garden is to be divided by a line bisecting the longest side and drawn from the opposite vertex. what is the length of the bisecting line?
Answered by Penny Nom.
A sand castle in the shape of a truncated cone 2020-05-23
From Summer:
A sand castle is in the shape of a truncated cone as shown. Calculate the length of the diameter of the base.
The diameter of the top of the truncated cone is 20cm
The height is 30cm
The slopes are 32cm

Answered by Penny Nom.
Drawing a counter from 2 bags 2020-05-21
From Sephy:
Question 4: Two bags, 1 and 2, each contain equal size counters.
Bag 1 contains a pink counter, yellow counter and white counter. Bag 2 contains counters labelled 1, 4, 5 and 7.
A counter is drawn at random from bag 1 and a counter is drawn at random from bag 2.
If the counter from bag 1 is pink, the number on the counter from bag 2 is increased by 1
If the counter from bag 1 is yellow, the number on the counter from bag 2 is decreased by 5
If the counter from bag 1 is white, the number is halved.
Find the probability of scoring a number below 3

Answered by Penny Nom.
Forming the largest cylinder 2020-05-20
From Amanda:
How do I find the maximum surface area and volume of a cylinder made up of ONE 8.5x11 piece of paper?
Answered by Penny Nom.
Maximizing the volume of a cone 2020-05-18
From Ella:
Hello, this is question - 'If you take a circle with a radius of 42cm and cut a sector from it, the remaining shape can be curled around to form a cone. Find the sector angle that produces the maximum volume for the cone made from your circle.'
Answered by Penny Nom.
An angle i a triangle 2020-05-16
From Ogunjobi:
Two goal post are 8m apart a footballer is 34 m from one post and 38m from the other within what angle must he kick the ball if he is to score
Answered by Penny Nom.
The flight of a football 2020-05-06
From Michael:
Football is thrown from a 10-yard line. It reaches its highest height of 20 yards. It lands on the 50-yard line after 2 seconds. What is the equation of the parabola that models this throw? I really need help as I've been on this for the longest amount of time.
Answered by Penny Nom.
Signs at 1/3 mile intervals 2020-05-03
From Heather:
The butterfly garden in St. Thomas gardens is a three mile loop at every 1/3 there is a sign along the loop to mark each 3rd of a mile along the loop if Melanie is standing next to the 7th sign at what mile is she?
Answered by Penny Nom.
The volume of a stone 2020-05-02
From KHOSI:
A rectangular container is 9cm wide,17cm long and contains water to a depth of 11cm. A stone is placed in the water and the water rises 2,2cm .Find the volume of the stone
Answered by Penny Nom.
The volume under an an uneven surface 2020-05-01
From Solon:
I am trying to fill an area of floor which has an uneven surface. I need to know the volume of fill. If you imagine a rectangular room (4 corners) that measures 189 inches by 240 inches and the depth at each corner is different.
Let us say corner #1 has a depth of 1", corner #2 has a depth of 0", corner #3 has a depth of 2" and corner #4 has a depth of 4".
How would i find the volume of said floor area?

Any assistance would be greatly appreciated!

Answered by Harley Weston.
A system of equations. 2020-04-27
From solomon:
xy + x =28
x + y +4

Answered by Penny Nom.
Power notation 2020-04-26
From Aditi:
The power notation of -1 / 128
Answered by Penny Nom.
The width of a rectangle of land 2020-04-26
From Teresa:
If a rectangle shape piece of land is 813 feet long. How many feet width would be 2 acres?
Answered by Penny Nom.
How many boxes will fit in a closet? 2020-04-24
From Jordan:
Janelle has some boxes that are 43cm long, 23cm wide and 18cm tall. She wants to store them in a closet in her basement that is 2.2m tall, 1.1m wide, and 0.8m deep. What is the maximum number of boxes that Janelle can possibly fit in the closet?
Answered by Penny Nom.
Building a house in one year 2020-04-22
From Fify:
If it takes 8 men to build a house in 450 days. How many men would it take to build the house in 365 days? Is there a specific method to calculate this please?
Answered by Penny Nom.
An arch in the form of a semi-ellipse 2020-04-20
From Anggelica:
an arch in the form of a semi-ellipse is 8 feet wide at the base and has a height of 4ft. how wide is the arch 1foot above the base?
Answered by Penny Nom.
Gravel for a French Drain 2020-04-17
From Ray:
Hello,

** I am wanting to add 2 French Drain's in my yard. The 1st drain will be 25 ft. long, 8 in wide and 18 in deep. The pipe I'm using is 4in. in diameter. I'm trying to figure out how many yards of clean gravel I will need for this project?

** The 2nd French Drain will be 100 ft in Length, 10in wide and 18 in deep. I will also be using the 4in in diameter pipe for this drain. I am needing to see how many yards of clean gravel, I'll need for this drain? Any assistance would be greatly appreciated!! Thank you!!

Answered by Harley Weston.
An isosceles triangle 2020-04-16
From jo:
Find the angle between the two sides of length 5 in an isosceles triangle that has one side of length 9 and two sides of length 5.
Answered by Penny Nom.
Why is a square a rectangle but a rectangle is not a square? 2020-04-15
From A parent:
Why is a square a rectangle but a rectangle is not a square? Explain your answer.
Answered by Penny Nom.
Two chords 2020-04-14
From Frank:
How to find the length of the radius of a circle if you know the lengths of two chords
Answered by Penny Nom.
A hexagon constructed from two triangles 2020-04-13
From sunny:
Triangles AEC and FDB are equilateral triangles and EA= DF= 12cm. The polygon at the center of the star is a regular hexagon. What is the area of the hexagon?
Answered by Penny Nom.
A rectangular garde 2020-04-10
From Denise:
In a rectangular garden a person says "if I made that bed 2 feet wider and 3 feet longer it Would have been 64 sq,ft bigger. But if it had been 3 feet wider and two feet longer It would have been 68 sq.ft bigger.
What is the length and width of the garden bed?

Answered by Penny Nom.
Setting up a linear equation 2020-04-10
From Jessie:
Michaela and Aleah play on the same basketball team. In one game, Michaela scored one fifth of the teams points and Aleah scored one tenth of the teams points. Together, they scored a total of 42 points. How many points did the team score?

I can't figure out how to put together the equation with the given variables. I have to show the work and I also need to know how to make that kind of equation. Thanks!

Answered by Penny Nom.
A pond in a garden 2020-04-09
From Jin:
c) I have a large square pond set inside a square garden: both the pond and the garden have sides which are a whole number of metres, and outside the pond, the garden is grassed over. The area covered by grass is 188 square metres. Find the area of the pond. (5 marks)
Answered by Penny Nom.
Form a square and a triangle from a wire 2020-04-08
From Raahim:
2. A 2 meter piece of wire is cut into two pieces and once piece is bent into a square and the other is bent into an equilateral triangle. Where should the wire cut so that the total area enclosed by both is minimum and maximum?
Answered by Penny Nom.
A linear system 2020-04-02
From Dadzie:
x+s=8→1
y+x=13→2
y-p=6→3
s+p=8→4

Answered by Penny Nom.
A stem-and-leaf plot with decimals 2020-04-02
From Renee:
I am an AP Statistics student who is reviewing for the exam.

On one of my review worksheets, we have to create a stem-and-leaf plot for values with 2 decimal places (i.e. 9.14, 8.14, 8.74, 8.77, 9.26, 8.1, 6.13, 3.1, 9.13, 7.26, and 4.74).

How would I make the stem-and-leaf plot for this? I've already searched your database and nothing popped up.

If you could help me, that would be great. Thank you!

Answered by Penny Nom.
A circle in a square 2020-04-02
From Sabra:
Hello, I have twins in the 7th grade that are learning how to find circumference of circles. I am struggling with figuring out how to help my children to find the answer to this question:

Tye has a square piece of yellow felt that has an area of 81 square inches. She wants to cut the largest circle possible from the material to create a sun for her art project. What is the area of the felt circle? Use 3.14 for pi. Round to the nearest hundredth if necessary.

I wasn't sure if I am to find the area of the circle through finding the area of the square or what steps we needed to follow to find the answer. Once I can see how one is worked out step-by-step, I usually can help them along with questions that are similar.

Answered by Penny Nom.
The center and radius of a circle 2020-03-25
From Riley:
so it says find the center and the radius of the below circle
x^2+(y-6)^2=121

i need help learning how to solve this

Answered by Penny Nom.
A hemisphere and a cylinder 2020-03-23
From Monalizsha:
Wat is the volume and the surface area of the hemisphere and cylinder (both are together) The height is 12cm of the cylinder and the diameter is 6cm
Answered by Penny Nom.
Aaron and Bill's ages 2020-03-12
From Heather:
My granddaughter had this third grade math problem. Aaron is 4 times as old as his brother Bill. In ten years Aaron will be twice Bills age. What are their ages?
Answered by Penny Nom.
Six-digit telephone numbers 2020-03-12
From Lulamile:
What is the probability that a six-digit telephone number has no repeated digits? The telephone number cannot begin with a zero
Answered by Penny Nom.
A times table in base 5 2020-03-11
From Felix:
Design a time table for numbers 1-10 in base 5
Answered by Penny Nom.
Fireman Fred needs to stay fit 2020-03-02
From Chamundeswari:
How do I explain this problem to a year 3 kid?

Fireman Fred needs to stay fit for his job. He runs 4.5km, 4 mornings, a week . if it takes him 4 minutes and 15 seconds on average to run 1km, how much time does he spend running over 4 weeks?

Answered by Penny Nom.
The volume of a berm 2020-02-28
From jason:
how much dirt is in an 20'w 12't 1350'l berm
Answered by Penny Nom.
A straight line graph 2020-02-26
From Penehafo:
I have a problem of drawing a straight line graph by the equation y=2x-1
Answered by Penny Nom.
The height of an isosceles triangle 2020-02-23
From Reagan:
I need to find the height of an isosceles triangle with a base of 6 and sides of 4 units. How do I find it?
Answered by Penny Nom.
Degrees, minutes and seconds 2020-02-21
From Jonathan:
If a cone has an angle of 22 degrees, when i place it flat on a surface, the new resulting central angle is now at 68.69123834, but how come when i saw it on my friend it say 68 degree and 40 minutes, what is this minute?
Answered by Penny Nom.
Multiplication base 8 2020-02-19
From Tamesha:
Let’s count with the symbols 0,1,2,3,4,5,6,7 (base 8)
A). Create a multiplication table for this base
B). Compute the following products using the multiplication algorithm
I). 7605 times 3713
II). 63725 times 40627

Answered by Penny Nom.
The volume of a cylinder 2020-02-12
From Mack:
I need to find how close the actual volume was of a cylinder (with both ends closed) to being optimized. Can you please send step by step response?

Height = 11cm
Circumference = 24.4cm

Thanks,
Mack

Answered by Penny Nom.
Sipping juice from a Tetra Pak 2020-02-11
From Anjhelic:
Karen is sipping juice from a 1 in. by 3 in. by 6 in tetra pack at the rate of 0.5in³/sec. How fast is the height of juice in the pack decreasing?
Answered by Harley Weston.
The area of a triangle from two angles and a side 2020-02-10
From Chinmoy:
How to measure the area of a triangle with two angles and length of the included side known?
Answered by Harley Weston.
A parabolic arch 2020-02-06
From icyy:
how high is the parabolic arch of span 20 feet and height of 16 feet, at a distance 5 feet from the center?
what equation will I be going to use? thank you

Answered by Penny Nom.
The side length of a hexagon 2020-02-05
From Rob:
I have a hexagon that is 8 feet wide how long would the sides be?
Answered by Penny Nom.
What was the original price? 2020-02-01
From Gracie:
If you buy something for $6.30 and it’s 30%off what was the original price?
Answered by Penny Nom.
A system of inequalities 2020-02-01
From Isaac:
Austin has x nickels and y dimes, having a maximum of 15 coins worth a minimum of $1 combined. No more than 4 of the coins are nickels and no less than 11 of the coins are dimes. Solve this system of inequalities graphically and determine one possible solution.
Answered by Penny Nom.
Translation by a vector 2020-01-30
From Jude:
M(3,-2) is the image of the point M under a translation by the vector (-5,2). Find the coordinates of M
Answered by Penny Nom.
A question about fractions 2020-01-26
From Awotile:
A fraction whose denominator is more than the numerator is double when the numerator is increased by 6 and the denominator is increased by 5 find the original fraction?
Answered by Penny Nom.
A point on a line 2020-01-25
From Samved:
The line 3x/5 - 2y/3 + 1 = 0 contains the point (m, 2m-1). Calculate the value of 'm'.
Answered by Penny Nom.
Filling a 48 cubic meter tank 2020-01-19
From kamohelo:
An empty tank for storing water from a borehole has a volume of 480m3 if it is filled by a pump that pump water at a rate of 16L/s how many hours will it take the pump to fill this tank
Answered by Penny Nom.
The angle of a countersunk screw 2020-01-19
From Barbie:
I need to be able to verify the angle used for the head of a countersunk screw.
I have the diameter of the head, diameter of the shank and height between the two. I assume it would be considered a frustum.
For example:
A standard 90 degree metric flat head screw in an M2 diameter has a head diameter of 3.65mm, the actual thread diameter is 1.98mm and the height of the head is 1.20mm.
How can I prove that it is a 90 degree angle?

Answered by Harley Weston.
Investigating y = (-2)^x 2020-01-13
From Gonzalo:
This is not precisely a maths question, but it is formulated based on my maths curiosity. Fidgetting with my new graphic calculator, I started graphing things and had the idea to graph $y=(-2)^x.$ The result surprised me, and I thought a little bit about it, stored it on the back of my brain, and promised myself to look deeper into it someday.
Answered by Harley Weston.
What is the smallest 4 digit number? 2020-01-10
From Ullas:
What is the smallest 4 digit number?
Answered by Penny Nom.
The difference between two number is 7 2020-01-07
From Jaypercey:
the difference between two number is 7 find the two number if the larger 3times the smaller
Answered by Penny Nom.
What do you call a 43-sided polygon? 2020-01-06
From Alniko:
What do you call a 43-sided polygon and what is its interior angle's total measure?
Answered by Penny Nom.
The equation of a line 2020-01-05
From coco:
Good afternoon If relationship between total cost and the number of units made it linear ,and if costs increases by 7$ for each additional unit made ,and if the total cost of 10 units is 180$ .
Find the equation of the relationship b/n total cost (Y) & number of unit made (x).

Answered by Penny Nom.
Dividing the tips 2020-01-02
From Pat:
trying to figure out division of tips I have 3 full time employees 40 hrs a week and 1 part time worker 12 hours a week what percentages do i give them to give them appropriate tips :-)
Answered by Harley Weston.
A circle, a point, and a tangent line 2019-12-31
From Faisal:
A circle has radius 10 units and passes through the point (5,-16). The x-axis is a tangent to the circle. Find the possible equations of circle?
Answered by Penny Nom.
Sabrina's age 2019-12-31
From Nakyi:
The sum of Sabrina’s age and twice her sister’s age is equal to their mom’s Age. If Sabrina’s sister is two years older than her and their mom is 37, how old is Sabrina?
Answered by Penny Nom.
A scale drawing 2019-12-28
From Boris:
If the scale drawing of a car park is drawn to a scale of 1cm representing 7m and the car is 133m by 93m, find the length of the drawing
Answered by Penny Nom.
10 miles in 2 minutes 2019-12-20
From james:
how fast would you be traveling to go 10 miles in 2 minutes?
Answered by Harley Weston.
Volume of a tree 2019-12-19
From Maria:
I have trees for sale. I need to calculate the volume of a tree from base which is wider than the stem.The base will be used for the making of hurleys, similar to hockey sticks so the wider base is crucial.
Answered by Penny Nom.
Odds 2019-12-16
From Craig:
I’d like to know what are the odds of a lotto machine giving out a specific 4 digit number as a quick pick?
I know there are 10,000 possible 4 digit combinations. So let’s say the number is 3587 what are the odds the lotto machine spits it out EXACTLY that way on a quick pick?

Answered by Penny Nom.
More on lampshades from a cone 2019-12-14
From Guy:
from an earlier post re; lampshade from a cone
Being math ignorant I cannot figure out how this equation makes sense

\[\frac{2\pi \times 33 - 2\pi \times 20}{2\pi \times 33} = \frac{13}{33}\]
imagine these as fractions multiplied by 360 to calculate a section of a circle to form a cone
Thx

Answered by Penny Nom.
The equation of a line 2019-12-10
From Jourdan:
Suppose a mining company will supply 96000 tons of ore per month if the price is 80 dollars per ton but will supply 70500 tons per month if the price is 20 dollars per ton. Assuming the supply function is of the form y = m x + b , find the slope, m and y-intercept, b
Answered by Penny Nom.
Roll two dice and record the difference 2019-12-09
From Barbara:
Suppose you roll two dice 100 times. Each time you record their difference (always subtracting the smaller one from the bigger one to get a positive difference). The possible values you get are 0,1,2,3,4 and 5. You record the frequency of each value in the following table:
Difference of two dice 0 1 2 3 4 5
Observed frequency 12 31 26 13 10 8
Let your null hypothesis be that the dice are fair, and the alternative hypothesis be that they are not fair. Using a confidence level of α = 0.10, test the null hypothesis by a goodness-of-fit test.

Hint: begin by
completing table:
x 0 1 2 3 4 5
f(x)

Answered by Penny Nom.
Two people are clapping their hands 2019-12-06
From Yukti:
Two people are clapping their hands, but with different frequencies. Ramya claps her hands 17 times per 3 minutes, and Kumar claps his hands 31 times per 5 minutes. They start clapping at the same time. How many times does each one clap before they clap together at the same time again?
Answered by Penny Nom.
3/8 and 0.375 2019-12-05
From Zamyiah:
how does 3/8 turn into 0.375 i just don't understand it the question is hard. please answer in time thank you for your time.
Answered by Penny Nom.
The area and volume of a wedge 2019-11-30
From Jasmine:
what is the volume and surface area of a wedge with definition
Answered by Penny Nom.
An exterior angle of a parallelogram 2019-11-29
From A student:
one angle of a parallelogram is four times the measure of the exterior angle adjacent to it. find the interior angles
Answered by Penny Nom.
Caleb age and Juliet age add up to 25 years. 2019-11-22
From tolulope:
Caleb age and Juliet age add up to 25 years. eight years ago,Caleb was twice as old as Juliet. how old are they now.
Answered by Penny Nom.
The length and width of 1 1/3 acres 2019-11-22
From Ben:
I’m trading to figure out what the length and width of 1 1/3 of an acre would be in a rectangle or in a perfect square
Answered by Penny Nom.
cos (3pi/2 -x) 2019-11-20
From sam:
1. express the following as a trigonometric function of angle x for cos (3pi/2 -x) the answer in the textbook is -sin x but i am unsure of the process to get the answer.
Answered by Penny Nom.
A four person race 2019-11-19
From Juvy:
Ben finished seven seconds ahead of Mike.
Noel finished three seconds behind of Sam.
Mike finished five seconds behind Noel.
In what order did the friends finish the race?

Answered by Penny Nom.
A hexagonal planter 2019-11-19
From Colleen:
I need to build a hexagon planter around a 32” square box. How long is each side of the hexagon?
Answered by Penny Nom.
Beginning algebra 2019-11-19
From Maya:
Two numbers have a sum of 57. Three times the first number is 13 less than the second number. Find the numbers.
Use let statements to help you solve. (Grade 9 math)
-Maya (help would be appreciated)

Answered by Penny Nom.
The sum of the ages of Ruth and her mother is 77 years. 2019-11-06
From Lydia:
The sum of the ages of Ruth and her mother is 77 years. The difference in their ages is 27 years. How old is each?
Answered by Penny Nom.
Algebra 2019-11-06
From Krizhia:
How is y= 1/2 * (x-1)² be x= √2y + 1. Please show me the steps
Answered by Penny Nom.
Liza works for a call center 2019-10-31
From Timothy:
Liza knows that in the long term, she has a 65% chance of making a sale when calling on customers. One morning, he makes Six calls.
a. What is her probability of making three calls?
b. What is her probability of making sales fewer than three?

Answered by Penny Nom.
The divisors (aliquot parts) of a positive integer 2019-10-31
From Kenneth:
Is there a fast and simple method that can be used to determine the aliquot parts, whole, positive numbers that can divide into another whole, positive number?
Answered by Harley Weston.
Alex rents a car for one day 2019-10-31
From Arshia:
Alex rents a car for one day. The charge is $25 plus $0.20 per mile. Alex wants to spend exactly $110. How many miles can he drive?
Answered by Penny Nom.
The largest integer k so that that 3335^k is a divisor of 3336! 2019-10-29
From Brandon:
Hello my name is Brandon and I having difficulties in solving the following question: Given the number 3336! = 1.2.3.4........................3336 . What's the greater positive integer number k with the condition that 3335expk is a divisor of 3336!
Thanks for your help.

Answered by Penny Nom.
Even 5 digit palindromes 2019-10-24
From daniel:
How many palindromes of 5 digits are even?
Answered by Penny Nom.
The center of a circle 2019-10-24
From Patrick:
I'm trying to find a quick calculation to find the center of the circle that will touch the 3 points of an isosceles triangle.
Known dimensions are length of the triangle base and the height from the top point to the middle of the base.
Thanks in advance,
Kind regards,
Patrick

Answered by Penny Nom.
The perimeter of a piece of a circle 2019-10-18
From Arwin:
The radius of a circle is 17 cm. A chord XY lies 9 cm from the centre and divides the circle into two segments. Find the perimeter of the minor segment.
Answered by Penny Nom.
How do you find the length and the width if all you had was area? 2019-10-16
From Ali:
So, How do you find the length and the width if all you had was area?
Answered by Penny Nom.
Simultaneous equations 2019-10-16
From deepak:
This is too complicated please help me
8/x - 10/y = 1 and x+y=9

Answered by Penny Nom.
The difference of two numbers is 42 2019-10-14
From Patrick:
The difference of two numbers is 42 and the larger number is 8/5 of the smaller number. Find the two numbers.
Answered by Penny Nom.
A binomial probability question 2019-10-08
From venni:
The Medassist Pharmaceutical Company receives large shipments of aspirin tablets and uses this acceptance sampling plan: Randomly select and test 24 tablets, then accept the whole batch if there is only one or none that doesn’t meet the required specifications. If a particular shipment of thousands of aspirin tablets actually has a 4% rate of defects, what is the probability that this whole shipment will be accepted?
Answered by Penny Nom.
The binomial distribution 2019-10-01
From PY:
An on-line game called ‘Shop Quiz’ is held by an e-commerce platform, from Monday to Friday every week. It consists of 8 multiple choice questions (MCQ) and each question has four options (A, B, C, D). Only one option is the correct answer. People who are able to correctly answer all 8 questions are winners and will be awarded a number of on-line shopping credits. Let X represent the number of questions that a person can answer correctly in a ‘Shop Quiz’.

1) Explain why Binomial distribution might NOT be a suitable distribution for the random variable X.

Mr. Saul likes playing the quiz, however, he is afraid that he might not have the necessary knowledge to answer the quiz questions. (The quiz questions cover a variety of topics including science, history, entertainment, sports and geography, etc.) Therefore, he tries to win the game by simply guessing the answers to each question.

Answered by Penny Nom.
Four less than three times a number is 20 2019-09-29
From Linda:
four less than three times a number is 20,what is the number?
Answered by Penny Nom.
The volume of a fire hose 2019-09-18
From Carlou:
How many liters of water in a 2.5 inches diameter of fire hose with a 20 meters in length
Answered by Penny Nom.
4" x 4" square tiles 2019-09-18
From Jill:
how many 4" x 4" square tiles would I need for 50 square feet?
Answered by Penny Nom.
Why does the circle have a larger area? 2019-09-16
From A student:
given a circle and square of same perimeter, why does the circle have a larger area

I’m looking for an intuitive explanation rather than a computational one. Thanks

Answered by Harley Weston.
Mandy walks to work 2019-09-15
From Kenneth:
Mandy walks to work at a constant rate. One-third of the way to work, she passes a bank. Three-fourths of the way to work, she passes a book store. At the bank her watch reads 7:52 A.M., and at the book store it reads 8:02 A.M. At what time is Mandy one-half of the way to work?
Answered by Harley Weston.
A one foot rise in water level 2019-09-15
From Stuart:
If a lake were to rise one foot - how many linear feet would it spread onto the shore line ? What is the correlation - 1 foot of a rise = 10 feet onto the shore ? Does it depend on the lake size - if so Lake Ontario in Canada.
Answered by Penny Nom.
The amount of concrete in a foundation 2019-09-11
From Lorinda:
How do you calculate ( in cubic yards) how much concrete is needed for the foundation of a rectangular building 40 ft by 85 ft if the foundation walls are 8 inches thick and 7 ft 6 inches high?
Answered by Penny Nom.
The number of integers that satisfy some conditions 2019-09-10
From Ryu:
Using only the digits 0 1 2 3 4 5 6 7 8 9 without repetition, how many integers are:
(a) Greater than 999, less than 10,000, start with an even digit and are even?
(b) Greater than 999, less than 10,000, start with an odd digit and are odd?
(c) Greater than 999, less than 10,000, start with an even digit and are odd?
(d) Greater than 999, less than 10,000, start with an odd digit and are even?

Answered by Penny Nom.
Problem solving 2019-09-08
From Arunsi:
When a fifth of a certain number is added to 3 the result is 17. What is the number?
Answered by Penny Nom.
Vectors 2019-09-06
From Kayla:
Hi! Due to time constraint issues, we skipped vectors in Pre-Calc last year. However, I'm attempting to complete my AP Physics summer homework and there are 3 problems with vectors. As I explained, I have no idea even where to begin and as such I need some help.

Problem 1:
A plane flying at 90° at 45 m/s is blown toward 0° at 62 m/s by a strong wind. Find the plane's resultant velocity. I have tried all sorts of online calculators but none are providing me with a formula I can use to solve this problem on my own.

Problem 2:
If you walk 367 m North and 785 m West what is your total displacement from your original location?
What is the problem asking for when it says displacement?

Problem 3:
A plane travels on a heading of 127.0° at a velocity of 25 km/hr. What are the horizontal and vertical components of the plane's velocity? As you know already, I have never done vectors so I don't know what horizontal and vertical components of velocity are.

Thank you So Much for your help!! I am struggling!!

- Kayla

Answered by Penny Nom.
Using the Laws of Exponents 2019-09-06
From Samiya:
i have been struggling with the laws of exponents so im having trouble with this question. the question is to write (b*exponent2*c)(b*exponent5*c*exponent4*)
Answered by Penny Nom.
Average savings 2019-09-06
From Puchu:
A student was saving an average of £2 per week for 25 weeks. He then found a part time job and after total of 45 weeks his average savings rose to £9 per week How much did he save over 20 weeks after finding job?
Answered by Penny Nom.
The volume of a berm 2019-09-06
From John:
I need to construct a berm:
8' tall
5' across the top
20' at the base
125' long
How much soil will I need

Answered by Harley Weston.
Can one equation with two variables be solved algebraically? 2019-08-30
From Don:
Can one equation with two variables be solved algebraically?
Answered by Penny Nom.
400 in 7.5 billion 2019-08-28
From Emma:
What is a better way to express 400 in 7.5 billion? My daughter has a rare condition, I want to know the answer to say “she is 1 in a ...”
Answered by Penny Nom.
David Is 6 years older than his sister Delia 2019-08-27
From Abigail:
David Is 6 years older than his sister Delia. In 10 years the sum of There age 52. how old are David and Delia today
Answered by Penny Nom.
A flat top cone 2019-08-25
From Mary:
I want to make a paper model of a flat top cone with the radius of the base is 12.5 cm and the radius of the apex is 7 cm.The height is 15 cm and the radius of the apex is 5 cm. The height is 15 cm.
Can you help me with the angle and construction, please?
Thanks.

Answered by Penny Nom.
The length of half an acre 2019-08-23
From Angeles:
How many Feet Long In Half An Acre (Rectangle)
Answered by Penny Nom.
A problem with logarithms 2019-08-23
From Iriogbe:
Find the value : if log3 (a-6) =2b and log2 (a-7) =3b
Answered by Penny Nom.
The average height of a wall 2019-08-15
From Jeff:
Trying to figure the average height of a sloped foundation wall.
The lowest point is 24 inches or 2 feet.
The highest point is 116 inches or 9’ 8 inches.
Can this be figured without the degree of slope or can we assume it is Equaled sloped from point a to point b. Thanks

Answered by Penny Nom.
A cone of maximum volume 2019-08-14
From Refilwe:
The slant height of a cone is 10cm. Determine the radius of the base so that the volume of the cone is a maximum
Answered by Penny Nom.
The radius of a circle of given area 2019-08-14
From shelby:
What's the radius of a circle that has an area of 803.84cm2? I also need examples of how you got the answer.
Answered by Penny Nom.
Conversing from tons to cubic yards 2019-08-03
From Kathleen:
Is there a simple formula to convert the following from tons to cubic yards:
Topsoil
Sand base
Fill dirt

Answered by Penny Nom.
Filling a hole 2019-08-01
From Jackie:
Filling a hole 25ft round 3ft deep how much dirt is needed?
Answered by Penny Nom.
A combination lock 2019-08-01
From markbo:
A combination lock has 0-9 plus Y on its keypad. A code is generated by selecting 4 digits from those keys (0-9 and Y) with no repeated choices, but sequence does not matter
(i.e. 1-2-3-4 is the same as 4-3-2-1, and the same as 3-4-1-2 and counts as one combination)

How many different 4-digit codes are possible in this scenario?

Answered by Penny Nom.
Combination lock 2019-07-31
From Antony:
Hello.
So my 3yr old daughter has had a bit of a play with the 3 dial combination lock on my backpack and locked it closed.
I am looking for all the possible combinations to go through a process of elimination to find the combination she has set.
To be clear. There are three dials, each with the numbers 0-9. I have read 10x10x10=1000 options but not sure if that is correct .. feels too small?
Many thanks in advance.

Answered by Penny Nom.
The volume of a well 2019-07-30
From Jordan:
How much water could my well hold if the depth of the well is 30 feet feet and the diameter of the well is 24 feet ?
Answered by Penny Nom.
Packing chocolates in boxes 2019-07-22
From CHESKA:
1.levy can pack the chocolate bars either with 35 bars in a box,with 42 in a box, or with 49 in a box. what is the least number of chocolate bars that he has?
2. the blue light of a Christmas light blinks every 2 second. the red light blinks every 3 second and the yellow light blinks every 5 second. how many times will the three lights blink at the same tims in one hour?
3.a wooden straightedge can exactly measure an 825 cm rope, a675 cm rope, and 375 cm rope. what is the least possible length of the straightedge? how many times will it measure each rope?

Answered by Penny Nom.
Listing three digit combinations 2019-07-17
From Larry:
Using numbers 0-9, what are all the possible combinations of choosing 3 numbers regardless of order and no 3 numbers being used more than 2 times in the combination. So no 1,1,1, 2,2,2, 3,3,3 etc. Order does not matter. Could you please send me a listing of all those combinations?
Answered by Penny Nom.
The maximum volume of a cone 2019-07-14
From A student:
find the maximum volume of a cone if the sum of it height and volume is 10 cm.
Answered by Penny Nom.
Volume of a spherical cap 2019-07-12
From Shanto:
In a jar of water, we kept a sphere with radius r. Then 75% of its volume went under water. Find out the distance of the top of the sphere from the surface of the water.
Answered by Penny Nom.
Price per square meter to price per square inch 2019-07-09
From Al:
If something costs $2.5 square meter, how much would it cost in square inches? How do i convert costs per square inch, into square meters?
Answered by Harley Weston.
Distance traveled 2019-07-03
From J:
If a fly flies at 15 feet per a second between two walls closing in upon themselves. Wall A is moving to the right at 3 feet a second and wall B is moving to the left at 15 feet a minute, how far will the fly fly before the walls touch?
Answered by Penny Nom.
Forty foot and 9 spacers 2019-06-28
From Christopher:
Ok I've got forty foot and 9 spacers one each end and 7 between what is the distance between the seven in the middle
Answered by Penny Nom.
The volume of a frustum 2019-06-24
From Abdulganiy:
A right pyramid on a base 10cm square is 15m high
a)find the volume of the pyramid
b)if the top 6m of the pyramid is removed what is the volume of the remaining frustum?

Answered by Penny Nom.
The area of a patio 2019-06-23
From Miguel:
Stephanie has 60 feet of metal fencing material to only fence 3 sides of a rectangular patio. A tall metal wall serves as the fourth side of the rectangle. write a function for the area the garden (A) in terms of width in feet (x) What measures of the width would give an area of at least 150 feet?
Answered by Penny Nom.
The area of a triangle in acres 2019-06-23
From Karen:
if I have a triangle that measures 78 ft x 54 ft x 50 ft, how do I figure out what part of an acre that is??
Answered by Harley Weston.
A tube through a board at 45 degrees 2019-06-15
From Guy:
I need to insert a tube 5/8" diameter into a board at a 45 degree angle. What size hole must I drill for the tube to fit snugly?
Answered by Harley Weston.
Yards, feet and gallons 2019-06-12
From Ralph:
How many gallons of water in a 10 yard by 10 yard container 2 1/2 feet deep?
Answered by Penny Nom.
150 litres has been drawn from a tank 2019-06-12
From Eke:
When 150 litres has been drawn from a tank, it is 3/8 full, how many litres will the tank hold?
Answered by Penny Nom.
Conversions, feet, kilometers and acres 2019-06-10
From Erika:
A lake has 113 feet deep and an area of 115 acres, what is the volume of the lake in cubic kilometer?
Answered by Harley Weston.
Two power generators 2019-06-07
From A student:
At 100% efficiency two generators would produce 750 MW of power. At efficiencies of 65% and 75%, they produce 530 MW. At 100% efficiency, what power would each produce?
Answered by Harley Weston.
A car drives over a cliff 2019-06-06
From Victor:
A car was going 90 mph towards 90foot drop. What would the distance would the car travel horizontally ,from the cliff edge?
Answered by Penny Nom.
An inline lock with four rows 2019-06-05
From Edward:
I have a inline lock with four rows of numbers 0-9.The numbers cannot be repeated. Can you please print the possible codes so I can go through them one at a time. The lock and the cable cost $300. Don't want to pitch it. You guys are lifesavers, can't wait to hear back. Thanks.
Answered by Penny Nom.
How many lines of symmetry does a circle have? 2019-05-28
From raji:
how many lines of symmetry does a circle have???
Answered by Penny Nom.
Maximize monthly revenue 2019-05-23
From a student:
A real-estate firm owns 100 garden type apartments. At RM400 per month, each apartment can be rented. However, for each RM10 per month increase, there will be two vacancies with no possibility of filling them. What rent per apartment will maximize monthly revenue?
Answered by Penny Nom.
Two circles 2019-05-23
From Arman:
Two concentric circles have their centres at point C. The radius of the smaller circle is 8 cm. The length of chord AB is 26 cm and is tangent to the smaller circle. What is the circumference of the larger circle?
Answered by Penny Nom.
Do all angles have to be equal to a number? 2019-05-15
From Malik:
Do all angles have to be equal to a number? By all angles I mean adjacent, vertical, supplementary, complementary.
Answered by Penny Nom.
What is the original price? 2019-05-15
From Tyler:
if 45% of an item is $90.00, then what is the original price
Answered by Penny Nom.
Pennies, nickels, dimes, and quarters 2019-05-09
From william:
Ann has 177 coins (pennies, nickels, dimes, and quarters)
Her total value is $19.90
She has the same number of nickels as dimes.
She has the same number of quarters as dimes.

How many of each does she have?

Answered by Penny Nom.
Subdividing land 2019-05-09
From Reuben:
This is the measurements of my plot, A-B 46.7M, B-C 193.1, C-D 198.5 & D-A 208.25 (Clockwise naming of sides) angle A at 90 degrees. My questions is how do i subdivide this plot from the bottom having lines running parallel to C-D, eg two 2acre plots. the the remaining part becomes my compound (Uper part at line A-B)
Answered by Harley Weston.
The volume of a wall 2019-05-08
From Peter:
I’m building a dry stack stone wall which is 36” wide at the base, 42” tall and 24” wide at the top. What would be the volume for a 36” long segment ?
Thanks, Peter

Answered by Penny Nom.
Why Mean? 2019-05-08
From Jill:
A group of teachers were trying to figure out why the”mean” is called mean - do you know??
Answered by Penny Nom.
Volume and Surface area of a sphere 2019-05-03
From Caitlin:
Why does the surface area formula of a sphere have a squared radius while the volume of a sphere has a cubed radius?
Answered by Penny Nom.
A repeating decimal divided by a whole number 2019-05-01
From Emily:
How do you divide repeating decimals by whole numbers?
Answered by Penny Nom.
A hemispherical bowl 2019-04-29
From Innocent:
Abc is the section through a hemispherical bowl centre O radius 15cm. It contains water whose maximum depth is 8cm. What is the width of the water surface?
Answered by Penny Nom.
A circle inscribed in a triangle 2019-04-28
From Daksh:
O is the centre of the inscribed circle in a 30°-60°-90° triangle ABC right angled at C. If the circle is tangent to AB at D then the angle COD is-
Answered by Penny Nom.
A dot on a child's car wheel 2019-04-28
From julia:
A wheel of a child’s car revolves at 10 rpm. A painted dot on the circumference of the wheel is 12 cm from the centre of rotation of the wheel. How far does the dot travel in 5 s, to one decimal place?
Answered by Penny Nom.
The length of an arc 2019-04-28
From Patrick:
If an arch is 48 inches wide at the base and 30 inches tall at its apex, what is the length of the arch?
Answered by Penny Nom.
A thread around a cylinder 2019-04-28
From Ally:
A cylinder has a diameter of 3.5 cm. How many times will a thread 143mm go round the cylinder?
Answered by Penny Nom.
Two linear equations with a fraction 2019-04-26
From Asha:
2x=-3y …1
x-2+-4/3(y+1) ...2


note: … is equation

Answered by Harley Weston.
Constructing a hexagonal pyramid 2019-04-25
From REBECCA:
I'm doing a project for my Honors Geometry class and I have to build a regular hexagonal pyramid. I was given the volume of .75 gallons, but I don't know how to figure everything else out. Please help?
Thanks,
Rebecca

Answered by Harley Weston.
Three friend's ages 2019-04-25
From Lemicio:
Janet is 10 years older than Karin, who is twice the age of Josef. If their combine age is 55, how old is each one now?
Answered by Penny Nom.
80% of an item is $200.00 80% of an item is $200.00 2019-04-25
From kayla:
if 80% of a item is $200.00 what is the original price
Answered by Penny Nom.
Is every rectangle a rhombus? 2019-04-25
From Danny:
Is a rectangle a rhombus??? True or False
Answered by Penny Nom.
The area of a triangle 2019-04-25
From troy:
The base and the height of a right triangle are represented by the expressions below. The area of the triangle is 28 square units. Find the lengths of the base and the height of the triangle

base= x+4
height=x-6

Answered by Penny Nom.
(3d) Whether line segment intersects triangle 2019-04-13
From rj:
Well hello,

Lets say I have a triangle,

Not any ordinary triangle, but a triangle that is represented in 3 dimensions. I also have a line segment.

Lets say that this triangle has points A(0,0,0), B(1,0,0), C(0,1,1)

And the line segment has points D(0,1/2,0), E(1,1/2,1)

In what way can I find if the line segment and triangle intersect? Their intersection point?

I don't prefer matrices, because they don't make any sense, but ... if that's the only way to solve this, then do what's necessary.

Thank you in advance.

Answered by Penny Nom.
An equilateral triangle 2019-04-10
From ethan:
In equilateral triangle RST, R has coordinates (0, 0) and T has coordinates of (2a, 0). Find the coordinates of S in terms of a.
Answered by Penny Nom.
The area of a lot 2019-04-06
From Robert:
Can you help me determine the rough square footage of this lot please(attached)? I think it is nearly 7000 square feet. I am sure I need the degrees of of the corners but I don’t have that. An estimate would be all I need. Thank you for you help. Rob
Answered by Harley Weston.
The height of a building 2019-04-05
From Hayley:
A young man is 36 meters away from a building and looking at a bird sitting on top of the building. The man knows he is 85 meters away from the bird. How tall is the building?
Answered by Penny Nom.
4^x=2^x+6 2019-04-05
From Olivia:
How do you find the (b,n) if the solution of the equation 4^x=2^x+6 can be expressed at logb(n) where b and n are both prime numbers?
Answered by Penny Nom.
Salary Plus Commission 2019-04-05
From Herschel:
Fire Fighting Equipment pays salespeople as follows: $452 per week plus a commission of .9% on sales between $15,000 and $25000, with 1.1% paid on sales in excess of $25,000. Find the gross earnings for an salesperson whose Total Sales are $28,400. (No commission is paid on the first $15,000 of sales)
Answered by Penny Nom.
Internal acute angles 2019-04-02
From karan:
Prove that a convex polygon cannot have more than three acute internal angles.
Answered by Penny Nom.
The volume of a pile of dirt 2019-04-01
From Genevieve:
We have a pile of dirt approximately 6 feet high by 15 feet wide by 80 feet long.
We need to know how many cubic yards of dirt are in this pile. Thank you

Answered by Harley Weston.
Percentages 2019-03-30
From Mari:
I am trying to find out what percentage of our worlds population does the youtuber Pewdiepie have for subscribers? So he has 91 million subscribers and there are roughly 7.53 Billion people in the world. So what percent of our worlds population does Pewdiepie have for subscribers? Am I asking this correctly? I am just curious for marketing reasons and how much of the worlds market share does Pewdiepie have for his youtube channel? I just do not know how to set up the problem to find the answer?
Answered by Penny Nom.
The volume of water in a pipe 2019-03-21
From Abati:
How can I calculate the volume of water in a pipe with below details;

Pipe diameter - 6 inches
Length of pipe - 127meters

Answered by Penny Nom.
A scale factor 2019-03-19
From Miracle:
A man in a photograph is 2cm tall. His actual height is 1.8m. Write a scale statement and determine the scale factor.
Answered by Penny Nom.
Slope and percentage 2019-03-19
From Tom:
I need a slope of 2 Inches every 30 feet what is the percentage to put on my pipe lazer. thank you
Answered by Penny Nom.
The area of a triangle 2019-03-19
From Dylan:
The area of a triangle is 108in squared. A base and corresponding height are in the ratio 3:2. Find the length of the base and the corresponding height
Answered by Penny Nom.
A dice game 2019-03-17
From Remo:
So, little game my kid came up with.

Dice are standard d6.

I roll a die, he rolls a die. If he gets the same number as me OR a bigger number he wins. So far so good. Used the opportunity to get him to calculate the probabilities of him winning, see if I could get him interested into looking at this game from a more academic perspective. He did fine.

Then we upped the ante. He rolls one die, I roll two dice. I his die has a number that is equal or greater than both of mine, he wins.
We had a bit of trouble solving that one, and had to resort to a table with all the results ( 6x36) to figure out where we stood.

Then again we upped. He rolls two dice, I roll two dice. If any one of his dice has a number equal to or greater than both of mine he wins. I only win if one of my dice is greater than both of his. And we got stuck. Figured I would need to find a formula to resolve this one, as our earlier solution ( table everything ) suddenly is 36x36, giving well over a thousand possible results. And if we ever go bigger than that then we *really* are going to need another way to calculate it.

Answered by Harley Weston.
When is John's next birthday on a Friday? 2019-03-15
From Darek:
The question is the following:
Last Friday 4th of April of 2014 John was 40 years old. How old will John be the next time when his birthday fell on Friday?
Thanks for your help

Answered by Penny Nom.
A circle with a circular hole 2019-03-11
From Sue:
My little one is wondering if a circle with a circular hole could be described as an irregular semicircle.
As it has 2 sides but is not the standard shape. Could you share your thoughts ??

Answered by Penny Nom.
A parabolic curve on a bridge 2019-03-09
From monica:
how do i fint the formula for my parabola with the provided information:
the golden gate bridge has a parabola
(343,160) = coordinate
(0,0)= vertex

Answered by Penny Nom.
A triangle of maximum area 2019-03-07
From Tom:
Triangle ABC is such that AB=3cm and AC=4cm.
What is the maximum possible area of triangle ABC?

Answered by Penny Nom.
Misuse of greater than 2019-03-07
From Kenneth:
I have an old business mathematics textbook. The authors have indicated that the following expressions indicate multiplication:

? is 2/3 greater than 90; ? is 2/3 smaller than 90. They also indicated that the following expression would indicate division: 30 is 2/3 greater than ? and 30 is 2/3 smaller than ?.

How can these phrases indicate multiplication and division? How can 60 be 2/3 greater than 90 and also smaller than 90 as indicated above. What were the authors thinking? I have added the page from the book that indicates what I have explained in my message Kenneth

Answered by Harley Weston.
Random assignments 2019-03-02
From Ninoshka:
Liu and Michi plan to sign up for a drawing class next term. Drawing is offered during the first 4 periods of the day, and students are assigned randomly to class.
What is the probability that Liu and Michi will have drawing together?

Answered by Penny Nom.
3 months loss and then a profit 2019-03-02
From Ida:
Let say, I have 10k of capital and I ran business for 4 months. The first months i loss 50%, the second month i loss 50% and so do the third month. The fourth month my profit is 50%

So can I calculate the balance of money that i have out of my capital?

Answered by Penny Nom.
Double the number of pennies each week 2019-02-28
From Donald:
If I save a penny a week,double it every week,How many weeks will it take to reach 1,000,000 dollars
Answered by Penny Nom.
The top half of a cone 2019-02-27
From danica:
What portion of the height from top that the cone be cut into two equal volume?
Answered by Penny Nom.
Weighing stars and hearts 2019-02-25
From Zach:
Two stars and one heart weigh 38 units. One star and one heart weigh 27 units what is the weight of each?
Answered by Penny Nom.
Running around a circular track in opposite directions 2019-02-24
From carmi:
how many times will lita and rose pass each other in 15 minutes if lita and rose start jogging on a 110 meters circular track. they begin at the same point but in opposite direction. lita at 8/3 meters per second and rose at 7/3 meters per second.
Answered by Penny Nom.
A word problem 2019-02-24
From Briana:
When 4 times the number increased by 30, the answer is the same as when 70 decreased by the number. What was my number?
Answered by Harley Weston.
Loan payment formula 2019-02-24
From Kenneth:

I have a question regarding the loan payment formula shown below.

Calculating the Payment Amount per Period
The formula for calculating the payment amount is shown below.

Simple Amortization Calculation Formula

A = P X r(1 + r)n over (1 + r)n - 1

where

A = payment Amount per period

P = initial Principal (loan amount)
r = interest rate per period

n = total number of payments or periods

Is this formula/calculation a condensed version of a longer calculation? I am curious to know how the (1 +r)n - 1 was developed from the longer calculation. For example, r(1 + r)n may have been (r + rn)n. The n's are exponents.

I thank you for whatever helpful explanation that may be provided.

Kenneth


Answered by Harley Weston.
The Polynomial Remainder Theorem 2019-02-23
From pasandi:
f(x) is a quadratic polynomial. when f(x) is divided by (x-1),(x-2) & (x+2) the remainders respectively are -1, 4 and 2 how to find the f(x) in a question like this?
Answered by Penny Nom.
Adding and multiplying two numbers 2019-02-13
From katie:
what 2 numbers add to get 7 and multiply to get 2?
Answered by Penny Nom.
Margin with commission as part of selling price 2019-02-13
From Anni:
I need to find selling price to put us at 24% margin.
15% commission is based on selling price and is included in our cost.
I'm trying to do it excel and keep getting a circular reference.
Total cost= cost + commission (15% of selling price)
Selling price=total cost + 24% margin

Answered by Penny Nom.
A linear equation 2019-02-11
From Nick:
What's a linear equation?
Answered by Penny Nom.
A cylindrical drum 2019-02-10
From Ronald:
I have a cylinder that is 32 inches round by 25 inches deep how much water can it hold
Answered by Penny Nom.
The exponential form of sqrt(13) 2019-02-10
From isaebl:
what is the exponential form of sqrt(13)
Answered by Penny Nom.
Fractions with roots 2019-02-06
From DEBMALYA:
5/√2+√3 –1/√2–√3
Answered by Penny Nom.
An ice cream cone 2019-02-05
From Eyuel:
a scoop of ice cream has a 3-inch radius how tall should the ice cream cone at the same radius be in order to contain all of the ice creams inside the cone
Answered by Penny Nom.
A 4 digit combination lock 2019-02-02
From Brijanshu:
I have a lock that contain only four no. 6,7,8 & 9 . How many possible password of four digit containing these Number.
Answered by Penny Nom.
Macy's coin collection 2019-01-30
From Sariah:
Macy's coin collection has 2 pennies for every nickel. If she has 60 coins that are pennies or nickels, how many nickels does she have?
Answered by Penny Nom.
An equilateral triangle inscribed in a circle 2019-01-29
From Penny:
What is the length of each side of the largest equilateral triangle that fits inside a 3 inch diameter circle?
Answered by Penny Nom.
y=lnx+(1+ln2)/2 and y=x^2 2019-01-28
From Mike:
Prove that y=lnx+(1+ln2)/2 and y=x^2 touch each other.

The course is about logarithm and root functions... how should I solve this problem?

Answered by Penny Nom.
A scale factor 2019-01-27
From Hannah:
What is the scale factor of 36 meters and 1.5 centimeters?
Answered by Penny Nom.
The sum of three consecutive even integers 2019-01-27
From Pauleen:
Translate this into algebraic expression
The sum of three consecutive even integers, if x is the first even integer

Answered by Penny Nom.
The supplement of a complement 2019-01-25
From Andrei:
The value of the supplement of the complement of 8° is ?°.
Answered by Penny Nom.
What angle s its own supplement? 2019-01-25
From Ciara:
What is the angle of it's own supplement?
Answered by Penny Nom.
A median of a triangle 2019-01-24
From Anna:
For triangle J(1,4) K(-3,-1) L(3,4), find an equation for the median from vertex J
Answered by Penny Nom.
(2018^2019)-(2019^2018) 2019-01-23
From Nazrul:
Find the remainder when (2018^2019)-(2019^2018) is divided by 4? Please show me the process.
Answered by Harley Weston.
A triangle made from 12 matchsticks 2019-01-20
From Palesa:
How many different types of triangles can be made with a perimeter of 12 matches?
Answered by Penny Nom.
Sale price and employee discount 2019-01-18
From Margaret:
What is the formula to find the 75% off full price item that was already marked 30% off?
Answered by Penny Nom.
A cylinder with a cone on top 2019-01-16
From Berania:
A water tower in New York City has the shape of a cylinder with a cone on top. The cylinder has a diameter of 12 feet and a height of 15 feet. The roof has an inclination angle of 25 degrees.
(a) Determine the height of the cone to the nearest tenth of a foot.
(b) Determine the overall volume of the tower to the nearest cubic foot.

Answered by Penny Nom.
7 spheres on a hexagonal tray 2019-01-14
From herm:
what is the length of each side of a hexagonal tray, with the height of each side 0.75 inch, to hold seven spheres, each with a diameter of 3.00 inches? The spheres are placed such that each side of the hexagon is touched by one sphere at its midpoint (and the seventh sphere is place in the center of the "ring" of the other six spheres.
Answered by Harley Weston.
A binomial expansion 2019-01-05
From shifali:
if the binomial expansion of (m-nx)^3 is 1+9x+-- find the values of m & n
Answered by Penny Nom.
A quadrilateral inside a square 2019-01-02
From Swetha:
In 2*2 square ABCD, E is the mid point of the side AD. F is a point in BE.CF is perpendicular to BE. Find the area of the quadrilateral CDEF
Answered by Penny Nom.
A rectangle problem 2019-01-01
From ahamed:
In rectangle ABCD, angle BCD is trisected. CE,CF meet the sides AB,AD at E,F. BE=6cm, AF=2cm so, find the area of the rectangle ABCD?
Answered by Penny Nom.
An octagonal deer blind 2018-12-31
From Alan:
My grandson and I would like to making a 6'x6' octagon floor for a deer blind. He is eight yrs old and the math is outside my wheelhouse. We need to know the length of the boards and angles of the corners please.
Thank you, Sincerely,
Alan

Answered by Penny Nom.
Two integers X and Y are selected at random 2018-12-31
From Kenneth:
Two integers X and Y are selected at random from the integers 1 to 8.If the same integer may be selected twice,find the probability that;
1)X-Y=2
2)X-Y is 5 or more
3)X is greater than Y

Answered by Penny Nom.
Non-repeating decimals 2018-12-28
From Grace:
What is non repeating decimal
Answered by Penny Nom.
Solve for x in Y=1.5x + log (1+x) 2018-12-23
From Asadollah:
Please restate below expression in terms of y

Y=1.5x + log (1+x)

Thanks

Answered by Penny Nom.
Ten games with two possible outcomes for each game 2018-12-22
From Jin:
I'm trying to make a list to view all the possibility.
An example is if I have 2 games there's two outcomes (home/away)
I will use number to represent the teams. 0-1,2-3
All possible out comes are 0-2,0-3,1-2,1-3 right?

MY QUESTION If I have 10 games each have 2 outcomes (home/away) to win
What is all possible outcome.
0-1,2-3,4-5,6-7,8-9,10-11,12-13,14-15,16-17,18-19
Could u please provide me with a list of all possible out comes

Thanks you

Answered by Penny Nom.
cos(2x) = sin(x) 2018-12-18
From Liz:
Determine all values of x in [0,2pi] that solve the equation Cos(2x)=sinx
Answered by Penny Nom.
Simultaneous equations with fractions 2018-12-14
From zaheer:
solve simultaneous equations and give answer in fractional form
3x - 2 = 4y +5/3
y + 7 = 2x + 4
would really appreciate some help on this please

Answered by Penny Nom.
The height of a tower 2018-12-12
From Pandey:
Hello sir plz solve this question. If the shadow of a tower is found to be 10.5m longer when the sun's altitude is 45°AND60°. Find the height of the tower.
Answered by Penny Nom.
The area and weight of sod 2018-12-11
From Susan:
I have equipment that will cut sod 30 inches across and 75 Feet in length? how much will this be in square foot age ? Yards? Approximate weight?
Answered by Penny Nom.
Angular velocity 2018-12-05
From Kolade:
Krusty the Clown and Sideshow Bob have covered themselves in velcro, and shot themselves out of a large cannon so that they stick to a huge rotating velcro wheel (don’t try this at home). As shown in Figure 1(a), Krusty is now stuck to a point 3.00 m from the center of the wheel while Bob is stuck to another point 5.00 m from the center. The wheel is rotating counterclockwise at 10.0 rpm. Conveniently a set of axes is suspended just in front the wheel (Krusty asked for axes, like the ones you would use to chop wood, so that he and Bob could chase each other with them in the usual wholesome entertainment he is known for. Unfortunately, the set designer misunderstood.) The axes do not rotate with the wheel.
(a) Find Krusty’s and Bob’s angular velocities, in radians per second.

Answered by Penny Nom.
A puzzling equation 2018-12-04
From Manny:
Hello,

Can you explain why this is wrong:
-1 = (-1)^3 = (-1)^{6/2} = ((-1)^6)^{1/2} = (1)^{1/2} = 1 ??

Thank you

Answered by Penny Nom.
How many solutions does 3 (x + 4) = 3 x + 4 have? 2018-12-01
From Milan:
how many solutions does 3 (x + 4) = 3 x + 4 have?
Answered by Penny Nom.
y = |x| - 1 2018-11-30
From alexis:
what does this mean y=|x|-1
Answered by Penny Nom.
Two cones formed from two circular sectors 2018-11-30
From APARNA:
The central angles of two sector of a circle are 60° and 120° . Using these sectors two cones are made. If the radius of the smaller cone is 5 cm, find the radius and base area of the larger cone.
Answered by Penny Nom.
0.025% of 575 billion 2018-11-28
From JoAnne:
What is .025% of 575 billion dollars?
Answered by Penny Nom.
1 acre by 3 inches deep 2018-11-23
From Cheri:
How many yards of soil does it take to cover 1 acre in 3 inches of soil?
Answered by Penny Nom.
Divisibility by 3 2018-11-20
From Ray:
There is a rule that a number is divisible by 3 if the sum of its digits are divisible by 3 (for example, 81=8+1=9 {divisible by 3} and 33=3+3=6 {again, divisible by 3}) I know this works but I don't know why! Please help.
Answered by Penny Nom.
Terry was two-thirds as old as Nia 2018-11-19
From Makenzy:
four years ago, terry was two-thirds as old as nia. now terry is three-fourths as old as nia. how old is each now
Answered by Penny Nom.
What was the original number? 2018-11-18
From jaydaleigh:
Mark was thinking of a number. Mark adds 11 to it, then doubles it and gets an answer of 40.5. What was the original number?
Answered by Penny Nom.
Price before VAT and a discount 2018-11-15
From Carol:
I bought a computer for £330.
The selling price included VAT at 20%.
Then I was allowed a staff discount of £30.
What was it’s original price before the VAT and staff discount?

Answered by Penny Nom.
The original cost when you only know a percentage 2018-11-13
From john:
I was wonder how to find the Original cost when you don't know original cost

The amount paid was 13.82 that is suppose to represent 20% because Insurance pays 80% Thank You

Answered by Penny Nom.
A line cuts a curve 2018-11-11
From roxanne:
Hello, i need to ask a question, do you mind explaining and writing the formula of how to solve equations such as "find the set values of k for which the line y-2x-5 cuts the curve y=x^2 +kx+11" please
Answered by Penny Nom.
The price of eggs rises 10 cents per dozen 2018-11-10
From Maddie:
Hi! I'm currently struggling with a word problem that says: "If the price of eggs rises 10 cents per dozen, one will be able to get 2 dozen fewer eggs with $6.00 than was possible at the lower price. What is the lower price? We are working on completing the square and all of the problems I've done so far in this chapter require using that method. If possible and needed, could you explain how to set up the equation? Thank you so much! I appreciate all your help!!
Answered by Penny Nom.
Speed 2018-11-06
From Dolores:
if I drove 3/4 mile in 32 seconds, what was my speed ?
Answered by Penny Nom.
Why 1/2 in the area of a circular sector? 2018-10-26
From Ariel:
In the equation for area of a sector=1/2r^2theta why is it 1/2? Please explain conceptually not algebraically.
Answered by Penny Nom.
Why 1/3 in the volume of a cone? 2018-10-20
From Keith:
In the formula for the volume of a pyramid, Volume = 1/3 * base * height, where does the 1/3 come from, intuitively?
Answered by Penny Nom.
Angular speed 2018-10-18
From Akidima:
A bicycle wheel rotates uniformly through 2.0 revolutions in 4.0 seconds . What is the average angular speed of the wheel
Answered by Penny Nom.
Filling a candy box 2018-10-16
From Stephanie:
My child had the question: A candy box has the following measurements. 10x6x4Find the total volume. (We got that one right 240in3) But... then it said “How many 1/2 inch cubed pieces of candy can fit in the box? This is where we disagree. Please help.
Answered by Penny Nom.
Covering 6 acres with dirt 2018-09-27
From Scott:
How many cubic yards of dirt would come from 6 acres, 6 ft. deep?
Answered by Penny Nom.
The point-slope equation for a line 2018-09-23
From Sally:
The equation of the line in point-slope form for the line passing through the point ( 4, -6) and parallel to the line y = -6x + 2. Describe the steps.
Answered by Penny Nom.
An inequality with no solution 2018-09-20
From Alena:
I'm trying to help with a homework problem. Can you please help me?

2kx - 3k < 2x + 4 +3kx

We are solving for "k" with no solution.

Answered by Penny Nom.
Compatible numbers 2018-09-19
From Jessica:
Hello, I'm having trouble helping my daughter she is in 5th grade and I tell you her math is so much different then the math I learned in school and I been out of school for quite some time so it's a little hard for me.
Anyway her math is very confusing for me they are saying use compatible numbers to find two estimates.
I have no idea what that even mean I tried using Google but still don't get it . The problem is 12 divided by 478 using compatible numbers to find two estimates.... please help

Answered by Penny Nom.
The sum and difference of two numbers 2018-09-17
From Feni:
The sum of two numbers is 4. The difference of the two numbers is 2. What are the two numbers?
Answered by Penny Nom.
A gas and oil mixture 2018-09-16
From April:
For a new motor bike, the recommended ratio of gas to oil is 27:1 by volume. How much gas and how much oil are needed to fill a 14 L tank on a new motor bike?
Thank you

Answered by Penny Nom.
Predicting the number of fish in a lake 2018-09-16
From April:
A researcher catches 30 fish in a pond, attaches a marker to their fin and releases them. The next day she places a large net in the water and catches 9 fish. 3 have the marker attached to their fin. Can she predict the number of fish in the pond???
Thank you

Answered by Penny Nom.
The volume of a hotel soup pan 2018-09-15
From Melanie:
If the hotel pan measures 12 inches long x 5 inches wide x 6 inches tall, how many quarts of soup will the hotel pan hold?
Answered by Penny Nom.
Velocity 2018-09-14
From Adrian:
If Jill was traveling at 300 miles in 4 hours due South what was his velocity in miles per second.
Answered by Penny Nom.
Covering a region with black Mexican Rock 2018-09-14
From Lenton:
I have a 48 sq. feet area that I need to cover.

How many cubic ft would I need to cover 48 sq. ft.?

Answered by Penny Nom.
Exponential form 2018-09-14
From Saquanna:
What is 25 in exponential form
Answered by Penny Nom.
Splitting a commission 2018-09-13
From Beth:
If Sales Prices is $170975 and commission is 6% but listing agent gets 60% of the 6% and selling agent gens 40% of the 6% what is the actual percentage that each would actually receive?
Answered by Penny Nom.
Exponential form 2018-09-12
From Grace:
hi, I can't figure out how to do 735 in exponential form. note that im only in 5th grade and its not supposed to be in the scientific notation or whatever. okay, thanks!
Answered by Penny Nom.
A locus of points 2018-09-03
From Ericka:
Find the locus of points three times as far from (0,4) as from (2,0)
Answered by Penny Nom.
Golf schedule for 12 players over 8 days 2018-09-03
From William:
We have a golf trip with 12 golfers

we are playing 8 rounds together......we want to mix the group up at best possible w/o too much overlap.......
thoughts? the math is way over my head...

even if we play the first 6 rounds with different partners, then the last 2 can be repeats ...or whatever the math works out

Answered by Penny Nom.
A 3500 acre ranch 2018-08-30
From Dee:
I am in the middle of writing the description of the size of a ranch in my story. It is 3500 acres. How does that equate into mileage? The main ranch house is at one end, and the foreman's ranch house at the other end.
Answered by Penny Nom.
Working with percentages 2018-08-26
From Kenneth:
16 is 4% of what amount? The answer can be determined by the following unusual manner: 4% = 16 1% = 4 100% = 400 Can the following solutions for the following be determined by using the same type of calculation? 16 is ?% of 400 ? is 4% of 400 I thank you for your reply.
Answered by Penny Nom.
Robbie and fran run laps 2018-08-26
From khansa:
robbie runs 3 laps for unknown number of days. fran runs 3 times as many laps each day for unknown number of day. what is the formula for their combined total number of laps
Answered by Penny Nom.
A rotating wheel 2018-08-24
From Kay:
A wheel with a radius of 7 inches is rotating at 4 revolutions per second. What is the linear speed v, in feet per second?
Answered by p.
Two intersecting tubes 2018-08-15
From Tommy:
Hi, I am trying to determine a mathematical model for two metal tubes joining at various degrees for weld.
For instance, if I am trying to join the end of a tube to the side of another at a 90 degree angle, it will be a simple profile cut out of the joining tube.
Where it gets tricky is if you want to join the new tube at a given angle.
It would be very helpful if you could give insight as to how I can solve this problem or an equation I could work off of.
Thanks for the help!!

Answered by Edward Doolittle.
Two spinners 2018-08-13
From Atina:
A spinner has four equal sectors and a number is written on each sector; 1, 2, 3 and 4. A two-digit number is formed by spinning two times. The number on the first spinning makes the first digit and the number on the second spinning makes the second digit. For example, 2 on the first spinning and 1 on the second spinning make the number 21.
(a) Give the sample space S for the experiment.
(b) Consider the following events : E = odd number; F = number smaller than 35; G = prime number. Give the subset of outcomes in S that defines each of the events E, F, and G.
(c) Describe the following events in terms of E, F, and G and find the probabilities for the events.
• getting an even integer less than 35.
• getting an odd number or an prime.
• getting an even number greater than or equal to 35 that is a prime number.
• an odd number smaller than 35 that is not a prime number.
(d) Are E and F mutually exclusive events? Give a reason for your answer.

Answered by Penny Nom.
A cube in a cylinder 2018-08-13
From Geli:
A cylinder with a radius r and height 2r+4 contains a cube with edge length r√2. What fraction of cylinder volume is taken up by the cube
Answered by Penny Nom.
Amy and Joey on a circular track 2018-08-13
From Zoe:
Amy and Joey are situated on a circular track 400 feet around.
Joey starts running at a rate of 10 feet per second.
Amy waits for a full minute, then starts running from the same point (in the same direction) at 12 feet per second.

How many seconds elapse before Amy passes Joey on the track?

Answered by Penny Nom.
Form a cone from a circle sector 2018-08-12
From Tinashe:
A 216 sector of a circle of radius 5cm is bent to form a cone. Find the radius of the base of the cone and its vertical angle.
Answered by Penny Nom.
How many students passed the test? 2018-07-20
From florian:
In Mr. Edgecomb's math class, 30 students took an exam on statistics. If the average passing grade was 84, the average failing grade was 60, and the overall average was 80, how many students passed the test?
Answered by Penny Nom.
Linear feet and cubic feet 2018-07-16
From Dave:
How do I convert: 1 foot wide x 15 inches high x 19 inches deep

We are trying to figure out how to convert the linear footage into cubic footage the above are the figures represent just 1 linear feet of several thousand

Any thing helps,

Thanks in advance

Dave

Answered by Penny Nom.
A sulphuric acid solution 2018-07-12
From Tan:
How much 96% h2so4 to add into sulphuric acid 30% in order to make it to 50% sulphuric acid?
Answered by Penny Nom.
A circle inscribed in a regular polygon 2018-07-12
From Naveen:
The radius of inscribed circle for n sided regular polygon of a side a is? Please with proof
Answered by Penny Nom.
The price of a watermelon 2018-07-07
From errin:
the price of the watermelon is directly proportional to its weight. If a watermelon that weighs 22 pounds cost $5.40, how much will a 17.5 pound watermelon cost?
Answered by Penny Nom.
The surface area of a levee 2018-07-04
From John:
Figuring out the surface area in sq. Ft. Of a levee

Height:10’
Top of levee width: 6’
Width of base (from toe to toe): 40’
Length of levee: 1,250’

Answered by Penny Nom.
Creating a number sentence 2018-06-29
From trish:
First I have to solve the problem. Then write a number sentence for the problem. The problem: By installing a $120.00 thermostat that reduces the temperature setting at night a family hopes to cut its annual fuel bill for heating oil by 8%, and recover the cost of the thermostat in fuel savings after 2years. What was the family's annual fuel bill before installing the thermostat? Thanks can't figure this one out!
Answered by Penny Nom.
Soil in a raised bed 2018-06-24
From Georgia:
Good Afternoon,

I'm planning on building a raised bed garden measuring 6 feet long x 4 feet wide x 12 inches deep. I'm not sure how to calculate the amount of soil to fill it up. I will have to purchase the soil in 2 cubic feet bags.

I've read several articles about this topic on various gardening websites, and one of them stated it would take a ton of soil to fill up an 8 foot long x 4 foot wide x 12 inch deep raised bed garden.

I'd greatly appreciate it if you could help me out with this quandary.

Thank you for your prompt and courteous response.

Kind Regards,
Georgia

Answered by Penny Nom.
Three consecutive whole numbers 2018-06-22
From whaha:
the sum of three consecutive whole numbers divided by 4 is equal to 6. find the numbers.
Answered by Penny Nom.
The base for a circular pool 2018-06-10
From Adam:
How many sheet of 4ft by 8ft foam would I need to lay under a pool 15dt in diameter
Answered by Penny Nom.
A tangent to a circle 2018-06-10
From Bellle:
Determine the equation of the circle which has its center at (3,1) and tangent 3x-4y+5=0
Answered by Penny Nom.
The inverse square law 2018-06-05
From Amy:

Question about inverse square law ;

Hi, I'm trying to understand some nuances about this law and
have been reading about it a lot online and trying out some homework
for personal interest (hobby ) - not school related.

I understand that the simple formula is ; 1/d^2

I was wondering about what appears to be an oddity to me, that
I came across when I was working with a test example I found at this
link ;

http://www.softschools.com/formulas/physics/inverse_square_law_formula/82/

The problem that caught my interest was this one on that page ;

"1) If a bright flashlight has a light intensity of 15.0 candela at a distance 1.00 m from the lens, what is the intensity of the flashlight 100.0 m from the lens?"

So I have a few different questions about this, but the most important
one is, what does it mean if we replace the "1" in ; 1/d^2 with a different value such as a ratio in the form of a decimal value, so instead we have something like this ;

0.75/d^2

???

Does this ratio represent a curve then?

I am thinking that the "1" in the normal formula represents a straight line

Some of the links I was researching ;

http://wisptools.net/book.php?c=3&s=2

http://www.softschools.com/formulas/physics/inverse_square_law_formula/82/

https://www.nde-ed.org/GeneralResources/Formula/RTFormula/InverseSquare/InverseSquareLaw.htm


Answered by Penny Nom.
Are those two equations the same? 2018-06-05
From Francisco:
Solve for y: 2(x+3)=y x=5
Solve for y: (x+3)2=y x=5
Are those two equations the same?

Answered by Penny Nom.
The volume of a berm 2018-06-04
From Mike:
Have a berm that is 700’ long, 29’ y’all and needs a 20’ top with 5-1 slope on one side and 3-1 slope in the other
Answered by Penny Nom.
Two odd integers 2018-06-04
From Pleroma:
The product of two alternate odd integers exceeds three times the smaller number by 12. What is the larger number?
Answered by Penny Nom.
$96 is 66 2/3% of what dollar amount? 2018-05-31
From mary:
$96 is 66 2/3% of what dollar amount?
Answered by Penny Nom.
Taking 6 pills 2018-05-29
From Dave:
My motives are simple, I want to get revenge on my brother who, unbeknown to me, repeatedly cheated when we played on-line Scrabble! Can you help? - he has posed this question knowing that I'll struggle to get the solution. Question: He is taking 6 pills a day, if he takes the pills in a different order each day, how long will it be before he takes the pills in the same order on the same date again? I think leap years might come in to it! Will I confess to using you guys? - I'll think about that one.
Answered by Penny Nom.
Monthly Pension 2018-05-28
From Kenneth:

Hello:

I have a question regarding how the units cancel in the following:

A company uses the following calculation to determine the monthly payment for its employees as they retire:

1.1% * average 5 years highest salary * years of service

If an employee worked 33 years and his average 5 year salary is $30,705.17, what is his monthly payment?

I used the following to determine his monthly payment, but I am not sure how the units cancel especially the years.

1.1% * $30,705.17 * 33 years divided by 12 months/year

I determined his payment as $928.83 per month, but the year units do not cancel unless the 1.1% is per year.

Is the correct calculation 1.1%/year * $30,705.17 * 33 years divided by 12 months/year so that the units cancel properly?

The $\$30,705.17$ is an average for 5 years of working. It is not $30,705.17 per year.

I thank you for your assistance.


Answered by Penny Nom.
What is the diameter of the observable universe? 2018-05-27
From peter:
The diameter of the observable universe is calculated to be 92 billion light years. What would that distance be in miles given a light year is 6 trillion miles?
Answered by Penny Nom.
An equation with rational coefficients 2018-05-26
From Anagh:
If x and y are rational number and
(x+y) + (x - 2y)√2 = 2x -y + (x-y-1) √5

Then find the value of x and y.

Answered by Penny Nom.
The area, radius and slant height of a cone 2018-05-18
From joette:
If you have a cone how to find the slant height when given the area and radius?
Answered by Penny Nom.
Making a cylinder from a metal plate 2018-05-18
From Rahul:
I want to make a cylinder which inside diameter is 600mm and thickness is 5mm and height is 800mm...so my question is that how much plate I need to make this happen
Answered by Penny Nom.
mg to ml 2018-05-17
From Carolynne:
I need to convert 12 mg to ml
density is 12.5mg/5ml
I can give dose of 1mg per pound . My dropper is in ml.
How much in ml can I give

Answered by Penny Nom.
Subdividing a rectangle into squares 2018-05-13
From jeverzyck:
A rectangular board is 108cm wide and 156 cm long Equal squares as large as possible are ruled off this board.Find the size of the square.How many squares are there?
Answered by Penny Nom.
Rods,poles and perches 2018-05-13
From roy:
i have some land and its measured in rods poles and perches
how cann i convert to understand it more

Answered by Penny Nom.
Two expressions for the perimeter of a a rectangle 2018-05-13
From Slayde:
A rectangular house has one side 4 metres longer than the other. Create two different expressions for its perimeter. The expressions should be given in factorised and expanded form.
Answered by Penny Nom.
An egg falls from rest 2018-05-10
From Amy:
An egg falls from rest a distance of 75cm to the floor. Neglecting air resistance, at what speed does it hit the floor?
Answered by Penny Nom.
The square root 3 in exponential form 2018-05-09
From Paige:
how do you write the square root 3 in exponential form
Answered by Penny Nom.
Converting decimals to common fractions 2018-05-08
From Ruth:
what does the 0 represent in .018 versus the decimal .180? As in, changing these to fractions? Thank you.
Answered by Penny Nom.
The volume o a hemisphere 2018-05-06
From Sambox:
When I have a hemisphere with a volume of 700Pi m^3 what is the radius?
Answered by Penny Nom.
Soccer fans and cricket fans 2018-05-06
From Chibo:
In a survey of 60 sport fans, 45 enjoy soccer, 35 enjoy cricket and 10 do not enjoy either soccer or cricket, but some other sports.What percentage of sports fans enjoys both soccer and cricket?
Answered by Penny Nom.
Gauss' Addition of whole numbers. 2018-04-30
From Brad:
I found this on your site. Question: what is the sum of the first 100 whole numbers?? Is there a different formula if the numbers begin at a number other than one? For example What is the series I want to add is goes from 7 - 53?
Answered by Harley Weston.
Spreading soil over 10 acres 2018-04-29
From Joe:
If you have 54000 cubic yards of soil and want to spread it over 10 acres of land, by how many inches would the ground level rise?
Answered by Penny Nom.
The weight of a cylinder 2018-04-27
From Jared:
What is the formula for the weight of a cylinder (not hollow) made of cast iron if it has a height of 5.25 inches and a diameter of 1.75 inches?
Answered by Penny Nom.
Throwing a football 2018-04-26
From Abby:
Daring Danny's threw a football at an angle of 40 degrees to the horizontal. The football hit the ground at 36 feet. Danny is 5 feet 1 inch tall. Find the initial velocity

I have been trying to figure this problem out for forever! Please help! I am really confused because I don't know time either and you are supposed to find that as well as velocity.

Answered by Penny Nom.
A rectangular prism with rounded corners 2018-04-25
From Jackie:
Rectangular prism that is 40 by 20 with rounded corners with a radius of 2.5.

What would the surface area be?

Answered by Penny Nom.
More on the curvature of the Earth 2018-04-23
From will:
the formula for figuring the earth's curve goes against logic, looking at a fixed point and backup 1mi. the point drops 8" then 16" in the next mi. and 32" in the third mi. why shouldn't it be 24" why is the 8" per mi. squared can you tell me in laymens terms why this is it goes against logic it would seem the correct wat would be to add up 8" per mile as you back up from the fixed point 8"- 16"- 24"- 32" not 8/16/32/64"
Answered by Harley Weston.
Seven tangent circles 2018-04-23
From Domenick:
How to calculate the circumscribed and inscribed circles formed by seven .019685 diameter circles arranged in a circle with all seven circles tangent to each other?
Answered by Penny Nom.
A volume expression for a pyramid 2018-04-23
From Shaheer:
Do a three sided pyramid and a square pyramid have the same formula if you want to calculate the volume?
Answered by Penny Nom.
Nickels, dimes, and quarters 2018-04-20
From Patti:
A copy machine accepts nickels, dimes, and quarters. After 1 hour, there are 30 coins total, and their value is $5.10. How many nickels, quarters, and dimes are in the machine? (Hint: There are exactly five different solutions.)
Answered by Penny Nom.
How to find the area of triangle? 2018-04-19
From g.a:
how to find the area of triangle
Answered by Penny Nom.
A cone formed from a circular sector 2018-04-18
From Jessica:
A circle has a radius of 7.5cm. A sector with an angle of 240 degrees is cut out from the sector. If the sector is folded to form a cone. Find the length of the cone.
Answered by Penny Nom.
Pool Digit Combinations 2018-04-18
From Jacob:
I need to know all the possible 3 digit combinations from the following pool of digits: 1st pool (1,2,3), 2nd pool (4,5) and the 3rd pool (7,8,9,0). Each pool has to be represented in the combinations.
The numbers can be repeated as long as they are not of the same set. Example for not repeating: 123 is ok but not 321 or 231,132,213, etc.

The question is somehow related to Mike's but the difference is the digits are allocated into pools. Your help can assist me solve a mystery. Thanks.

Answered by Penny Nom.
A circle inscribed in a quarter circle 2018-04-16
From abhijeet:
ABC is a quarter circle and a smaller circle is inscribed in it. if AB = 1cm then find the radius of smaller circle
Answered by Penny Nom.
A triangle in a cube 2018-04-16
From Sohel:
A cube has an edge of 4 cm. A triangle is formed by joining the middle point of one face of the cube to the ends of a diagonal of the opposite face . What will be the area of the triangle ?
Answered by Penny Nom.
The mass of a durian 2018-04-14
From REEVASSHINIE:
A watermelon with a mass of 5 kg and two durians with the same mass are placed on an electronic balance.If the reading shown on the balance is m kg,write an expression for the mass of a durian
Answered by Penny Nom.
The length of a diagonal of a box 2018-04-13
From Valentina:
A packaging box for a metal rod is 7.5 inches along a diagonal of the base the height of the box is 18 inches what is the lengh of a diagonal of the box?
Answered by Penny Nom.
Covering a region with mulch 2018-04-13
From Roberta:
I have a fall zone area of approx. 700 sq ft. that I have to cover with a 6in depth of mulch. How much mulch would I need to purchase?
Answered by Penny Nom.
Graham's money box 2018-04-12
From Tasnim:
Graham's money box consists of sh.10 and sh.5 coins. There are 24 coins and it's total value is 150. Find how many of each coins are there in the box
Answered by Penny Nom.
The sides of a circle 2018-04-10
From Reid:
Hello,
I recently was wondering about whether or not a circle has an infinite number of sides, and I ended up searching it on your website. I saw that you guys found the question to be too ambiguous or something of that nature, and I thought about your process of reasoning involving vertices and such. I soon realized that I may have come up with the solution to the question, but I want to confirm it with you guys. Allow me to explain:
A circle, unlike any other typical 2 dimensional polygon, can sustain an infinite number of straight lines coming in contact with only one point on it. A square, for example, cannot. A square only has 4 locations that can sustain a such a line, each of those being its corners. The flat edge of a square cannot support a tangential line, because the line would either be crossing the edge of the square or coming into contact with multiple points along its edge.
This concept is consistent in every 2 dimensional polygon: pentagons sustain 5 locations for tangential lines, hexagons 6, nonagons 9, etc. The reason a circle has an infinite number of sides is simply the fact that it must have an infinite number of "corners", assuming it can be defined as a polygon like any of the shapes I described above.
Corners can only exist assuming there is two sides coming into contact with one another. If there is an infinite number of corners in the circle, which is apparent due to the above reasoning, there must be an infinite number of sides coming into contact with each other. Thus, a circle is a polygon that consists of an infinite number of sides coming into contact with each other.
Please review this reasoning and let me know if it is solid.
Thank you!

Answered by Penny Nom.
Rolling 2 dice 2018-04-06
From Jose:
Find the probability p that the sum is as stated when a pair of dice is rolled.

(a) Even and doubles.
p=

(b) Even or doubles.
p=

Answered by Penny Nom.
A side of a square 2018-04-06
From Angie:
I am dyslexic and have problems with spelling and math I'm trying to figure 4.610 Acres Square trying to figure out how many feet or yards of one side if that makes sense to you thank you I'd really appreciate the help
Answered by Penny Nom.
The price of a sheet of metal 2018-04-02
From Nydia:
What is the formula to calculate the following:

I nee to find out what the sheet price be. The dimensions are 2000x3000 mm into ft they are 78x1181. The sqft price I have is $22.16. How do I figure out the sheet price from that?

Answered by Penny Nom.
A circle problem 2018-03-30
From ahamed:
the diameters of a circle AB and CD intersect each other at O perpendicularly. Then a chord DF is drawn. AB and DF intersect at E. DE=6cn and EF=2cm.find the area of thw circle
Answered by Penny Nom.
The volume of a tent with a hexagonal base 2018-03-27
From shohel:
A tent has its base in the shape of a regular hexagon whose sides are 10m . If the height of the tent is 12m , then find its volume.
Answered by Penny Nom.
Distance between a chord and its arc on a circle 2018-03-23
From Doug:
Specifically, what is the subject distance for the Earth orbiting for 27 days. Assume the orbit of the Earth to be a circle have a 93 million mile radius. Assume the angle of arc to be (27/365) x 360 degrees. Thank you.
Answered by Penny Nom.
A circle and two chords 2018-03-21
From Ella:
In a circle, a 30 unit chord is 5 units closer to the center than a 20 unit chord. Find the radius.
Answered by Penny Nom.
Walking a fraction of a mile 2018-03-20
From Jamie:
Who walked a fraction of a mile that is closer to neither 0 nor 1 ? Explain
Avery 1/6
me.Nunez 5/6
Ms. Chang 1/3
Mr. O’Leary 4/8
Miss Lee 4/6

Answered by penny Nom.
A circle of circumference 4*pi*r 2018-03-19
From Ankith:
If the circumference of a circle is 4*pi*r, what is its area?
i) 4*pi*r^2
ii) 2*pi*r^2
iii)3*pi*r^2
iv) pi*r^2

Answered by Penny Nom.
A penny is thrown from the top of a building 2018-03-16
From Zoraida:
A penny is thrown from the top of a 26.7-meter building and hits the ground 3.39 seconds after it was thrown. The penny reached its maximum height above the ground 0.89 seconds after it was thrown.

a. Define a quadratic function, h, that expresses the height of the penny above the ground (measured in meters) as a function of the number of seconds elapsed since the penny was thrown, t. b. What is the maximum height of the penny above the ground?

Answered by Penny Nom.
The volume of a cube 2018-03-15
From Eli:
An outdoor art display is a metal cube with edge length 39 feet. What is the volume of the cube in cubic​ feet? In cubic​ yards
Answered by Penny Nom.
An equilateral triangle inscribed in a circle 2018-03-15
From Olatundun:
An Equilateral Triangle Of Side 20cm Is Inscribed In A Circle.Calculate The Distance Of A Side Of The Triangle From the Centre Of The Circle.
Answered by Penny Nom.
A rope fom a dock to a boat 2018-03-14
From Tracey:
A boat is tied with a rope to a dock that is 16 feet tall. Along, the water, the water the boat is 17 feet from the dock. How long is the rope connecting the boat to the dock? Let c represent the length of the rope.?
Answered by Penny Nom.
Solve sinX=0.703X for X 2018-03-13
From PARAM:
sinX=0.703X
Answered by Penny Nom.
The sum and difference of two numbers 2018-03-13
From samima:
Two numbers have a difference of 0.85 and the sum 1.What are the numbers?
Answered by Penny Nom.
0.366 x cos square (02 degree 17 mins 27 seconds) 2018-03-12
From michael:
what is 0.366 x cos square (02 degree 17 mins 27 seconds)
what is 0.366 x cos square (88 degree 26 mins 45 seconds)

Answered by Penny Nom.
Graphing y = 2x - 1 2018-03-12
From Himanshu:
Hi I Himanshu I having 1 doubt on 1 question this question of graph how to draw y=2x-1
Answered by Penny Nom.
The dimensions of a rectangle 2018-03-03
From Mia:
A wire of length 36cm is bent to form a rectangle. Find the dimensions of the rectangle.
Answered by Penny Nom.
Expand (5x-9)(5x+9) 2018-03-01
From adil:
expand the following:
(5x-9)(5x+9)

Answered by Penny Nom.
An item on sale costs 55% of the original price. 2018-02-28
From Debra:
An item on sale costs 55% of the original price. The original price was $51 .
Answered by Penny Nom.
Is a rectangle a parallelogram? 2018-02-28
From ellie:
is a rectangle a parallelogram?
Answered by Penny Nom.
The limit of (e^x-1)^(1/x) as x tends to 0 2018-02-27
From ARGHA:
Find the limit of (e^x-1)^(1/x) as x tends to 0.
Answered by Penny Nom.
A cyclist reduces his speed 2018-02-27
From Katherine:
Suppose the winner had reduced his average cycling rate by .1 mile per hour. By how much would his time have changed?

(the cyclist's rate is 23.66 miles per hour and the answer to the question before this one was that he cycled 2,292 miles in 96.87 hours)

Answered by Penny Nom.
Billions and trillions 2018-02-25
From Sheila:
How many 50 billion equal 100 Trillion ?
How many 20 billion equal 100 Trillion?
How many 500 million equal 100 Trillion?
How many 50 million equal 100 Trillion?
How many 50 thousand equal 100 Trillion?

Answered by Penny Nom.
$525 is shared among three friends 2018-02-25
From Iyanla:
If $525 is shared among three friends: Paul Katherine and Brandon so that Paul gets $15 less than Brandon and Katherine gets 4 times as much as Paul, how would I solve this equation?
Answered by Penny Nom.
Multiplies and factors 2018-02-24
From Lil:
Is a multiple the same as a factor?
Answered by Penny Nom.
Dividing a region in half 2018-02-24
From mandy:
There is a line through the origin that divides the region bounded by the parabola y=4x−5x2y=4x−5x2 and the x-axis into two regions with equal area. What is the slope of that line?
Answered by Penny Nom.
The slope of a line 2018-02-22
From Steve:
my current slope is 6%. I want to know how many inches higher I will be at 18' Thanks
Answered by Penny Nom.
2.5 gallons of liquid, one half an inch deep 2018-02-21
From Cain:
How many square feet would 2.5 gallons of liquid cover at half an inch deep?
Answered by Penny Nom.
nC0 + nC1 + nC2 + .... + nCn = 2^n 2018-02-19
From bristal:
(QQ) Prove, nC0 + nC1 + nC2 + .... + nCn = 2^n.
Answered by Penny Nom.
-2 (a + 4) = 18 2018-02-18
From rebecca:
-2 (a + 4) = 18
how do i do this with showing it distributive property ( symbolically)
Thank you!

Answered by Penny Nom.
A footballer angle 2018-02-14
From Kim:
Two goal posts are 8m apart. A footballer is 34m from one post and 38m from the other. Within what angle must he kick the ball if he is to score a goal.
Answered by Penny Nom.
Solar cells 2018-02-12
From karla:
Solar (photovoltaic) cells convert sunlight directly into electricity. If solar cells were 100% efficient, they would generate about 1000 watts of power per square meter of surface area when exposed to direct sunlight. With lower efficiency, they generate proportionally less power. For example, 10% efficient cells generate 100 watts of power in direct sunlight. Suppose you want to supply 22 kilowattss of power to a house by putting solar panels on its roof. For solar cells with the average power of 49 watts per square meter of solar panels, how many square meters of solar panels would you need? Assume you can make use of the average power from the solar cells
Answered by Penny Nom.
Related rates 2018-02-11
From angelo:
hi admin please help me answer this question. thank you! At a certain instant of time, the angle A of a triangle ABC is 60 degrees and increasing at the rate of 5degrees per second, the side AB is 10cm and increasing at the rate of 1cm per second, and side AC is 16cm and decreasing at the rate of 1/2 cm per second. Find the rate of change of side AB?
Answered by Penny Nom.
Water in a cone 2018-02-10
From Shuvo:
The diagram shows a vertical cross-section of a container in the form of an inverted cone of height 60 cm and base radius 20 cm. The circular base is held horizontal and uppermost. Water is pursed into the container at a constant rate of 40 cm3/s.
Show that, when the depth of water in the container is x cm, the volume of the water in the container is (πx^3)/27 cm3.
Find the rate of increase of "x" at the instant when "x" = 2.

Answered by Penny Nom.
3 yards 6 inches times 7 2018-02-08
From Tamara:
3 yards 6 inches × 7
Answered by Penny Nom.
A parallelogram, a rhombus and a rectangle 2018-02-07
From Sambo:
what do parallelogram, rhombus and rectangle have in common?
Answered by Penny Nom.
The wholesale price given the retail price and the markup 2018-02-07
From Matthew:
Retail $99
Mark-up is 80%

What is the equation to find the original (wholesale) price (55)?

thanks

Answered by Penny Nom.
Similar rectangles 2018-01-31
From Kathy:
A rectangular picture frame is 14 inches long and 4 inches wide. Which dimensions could a similar picture frame have.
8 L X 21 W
35 L X 15 W
49 L X 14 W
7 L X 3 W

Answered by Penny Nom.
Positive and negative values of a function 2018-01-30
From Grayson:
f(x)=x^6-x^4
Interval: ( negative infinity, negative one )
Test Value: negative two
Function Value f(x): positive forty eight
Interval: ( negative one, zero )
Test Value: negative one
Function Value f(x): zero
Interval: ( zero, positive one )
Test Value: positive one
Function Value f(x): zero
Interval: ( positive one, positive infinity )
Test Value: positive two
Function Value f(x): positive forty eight

What is the sign of f(x) for each Interval?

Answered by Penny Nom.
Pricing a piece of plate steel 2018-01-29
From Simo:
I have a piece of plate that is 2500mm x 8000mm x 20mm thick and costs $7693.00

what is the formula to work out what 1000mm x 1000mm is worth?

Answered by Penny Nom.
Simplify 1/2^i 2018-01-28
From Deepak:
How do I simplify this complex equation:

z=1/(2^i)

Answered by Penny Nom.
The equation of a line 2018-01-23
From Gloria:
Write the equation of a line in standard form through the point (1, 6) and perpendicular to the line 2x –y = 9.
Answered by Penny Nom.
y as an expression in terms of x 2018-01-20
From Mark:
What is y as an expression in terms of x ?
Answered by pennt.
The product of the coordinates of a point on a line 2018-01-17
From Sophia:
For any point on line l, the product of the x-coordinate and the y-coordinate is less than or equal to zero , what is the answer?
Answered by Penny Nom.
The decimal form of a fraction 2018-01-12
From Tom:
Prove that the decimal representation of the quotient of 2 integers must repeat (if it does not terminate).
Answered by Penny Nom.
When are the hands of a clock perpendicular? 2018-01-11
From Nazrul:
How many times in a day are the hands of a clock perpendicular to each other? How can I find the times? Please help me.
Answered by Penny Nom.
xy+x=2y. Solve for y 2018-01-10
From Hugh:
xy+x=2y. Solve for y
Answered by Penny Nom.
A fraction of a mile 2018-01-06
From Sierra:
There are 5280 feet in a mile. what fraction of a mile is represented by 660 feet?
Answered by Penny Nom.
2-color combinations 2018-01-05
From Sabrina:
How many 2-color (pair) combinations can be made from 12 individual colors... is it (11x12) or (optional): (11x12)x2?
Answered by Penny Nom.
Compatible numbers 2018-01-02
From Natalie:
I'm in 4th grade and two weeks ago my class learned how to use compatible numbers. I was REALLY confused but I didn't want to say I didn't understand because I just wanted to go home. Tomorrow we have the Topic 7 Test and I'm stuck on a problem that requires using compatible numbers. My mom apparently hasn't heard of compatible numbers and she can't really help me too much. Here's the problem : The local library places new books in a section with 31 shelves. Each shelf fits 18 books. Use compatible numbers to estimate the number of books that the library can fit on the shelves.
Answered by Penny Nom.
Speed in kilometers per hour 2017-12-21
From Cody:
if a train travels 400 meters in 30 seconds how many kilometers per hr is it traveling
Answered by Penny Nom.
A puzzling sequence 2017-12-19
From Alan:
My son has the following sequence to work out the formula for the nth term (and fing 5th and 6th terms) and I have tried a variety of ways but can't help!

1/2, 5/6, 1, 11/10

I have converted all to 15/30, 25/30, 30/30, 33/30 and so think the next two terms are 37/30 and 45/30 but I have no idea how to prove or if I am right!

Any help appreciated.

Answered by Penny Nom.
Retiring athletes numbers 2017-12-18
From Robin:
I am wondering if you can explain in layman terms how to figure out how many different ways the numbers 00-99 can be arranged? The discussion that sparked this question revolved around the practice of retiring athletes numbers and how many actual number combinations there are. So if I were to use 00, 01, 02 03, 04, 05, 06, 07, 08, 09, 1, 2, 3, 4, 5, 6, 7, 8, 9, 10, 11, ....99. I say there are only 109 different numbers. I'm being told I'm wrong that there are way more than that. Please help. Thank you, Robin
Answered by Penny Nom.
4444^4444 2017-12-10
From Sashi:
4444 to power of 4444=?
Please share the result with simplification.

Answered by Penny Nom.
Needing the area of an irregular Pentagon 2017-12-06
From Dale:
I'm needing to find the square footage of an irregular pentagon.
I've seen the other questions similar to this one, but without a great deal of difficulty can't get you the angle degrees, or the length between opposing angles.
I'm hoping that a different bit of information can help. So here goes:
The longest side is 67ft long.
One of its legs is 39ft long, and the other is 18ft long.
The leg off of the other end of the 39ft leg is 35ft long.
The leg off of the other end of the 18ft is 49ft long, and meets the other end of the 35ft side.
The information that I DO have, which I'm hoping makes the difference, is that the 39ft and the 18ft sides are parallel.
Also, while the angles at each end of the 67ft side are not right angles, they are fairly close to it, approximately 80 degrees at the 39 ft side corner, and 100 degrees at the 18ft side corner.

Dale.

Answered by Harley Weston.
A range hood 2017-12-04
From Chuck:
I'm building a custom range hood and can't seem to get the angles correct where the front and side panels intersect.
I saw a similar post but there's no way I can do the calculations for a Wolfram Alpha!

Here are the dimensions that I have (in inches)...

Base - Front 29 7/16" x side 19 3/16"
Top - Front 10" x side 9"
Front Panel Length (from base to top on an angle) 21"
Side Panel Length (from base to top on an angle) - 17 9/16" Vertical distance from the base to the top 14 1/4 inches.

Any help finding the bevel/miter of the two front corners where the sides meet the front panel would be greatly appreciated.

I will need the angle for the saw in degrees.

Thank you in advance!

Regards, Chuck

Answered by Harley Weston.
The derivative of f(x)=2^x/x 2017-11-28
From Chhavi:
f(x)=2^x/x.
Find f'(x).

Answered by Penny Nom.
Exponential form 2017-11-25
From Sherry:
What is the exponential form of 3sqrt 15?
Answered by Penny Nom.
Heat equation 2017-11-23
From Max:
What does du\dt=a(triangle)^2u mean. Can it be solved for t.
Answered by Penny Nom.
How many miles did he drive in one hour? 2017-11-21
From Ava:
Graham drove 42 1/3 Miles in 1 1/3 hours. How many miles did he drive in one hour?
Answered by Penny Nom.
An expression for the area of a triangle 2017-11-19
From hari:
why is area of a triangle drawn from centre of a circle i.e radii forming the sides is 1/2 *r^2 sin theta where r is radius
Answered by Penny Nom.
Jesse has dimes and nickels in his piggy bank. 2017-11-18
From Dee:
Jesse has dimes and nickels in his piggy bank. The number of nickels is 3 times the number of dimes. If the total amount is $8.50, how many coins are there in total?
Answered by Penny Nom.
Blair is 12 years younger than Serena right now. 2017-11-18
From Dee:
Blair is 12 years younger than Serena right now. Three years ago Serena’s age was five times Blair’s age. What are their ages?
Answered by Penny Nom.
Splitting twice 2017-11-11
From Terry:
4 people are splitting 1/3 of something. 1 is to receive twice as much as the other 3. what it the fractional interest for each?
Answered by Penny Nom.
The graph of graph y= -2x-1 2017-11-11
From Natividad:
How do you graph y= -2x-1?
Answered by Penny Nom.
A triangle with sides measuring 6, 8 and 10 inches 2017-11-11
From rukhsar:
A certain triangle has sides, which are, respectively, 6 inches, 8 inches, and 10 inches long. A rectangle equal in area to that of the triangle has a width of 3 inches. The Perimeter of the rectangle, expressed in inches, is
Answered by Penny Nom.
math 2017-11-11
From rukhsar:
A certain triangle has sides, which are, respectively, 6 inches, 8 inches, and 10 inches long. A rectangle equal in area to that of the triangle has a width of 3 inches. The Perimeter of the rectangle, expressed in inches, is
Answered by Penny Nom.
Mathematical ideas in everyday life 2017-11-08
From Ricita:
Mathematical ideas to solve various problems of our everyday life ,environment related problems.
Answered by Penny Nom.
Scale factor 2017-11-03
From Hannah:
I do not understand it...... What is the scale factor for 9 cenimeters = 1 meter
Answered by Penny Nom.
Formula when i know the GST total 2017-10-31
From Duncan:
HI - I've added up the GST from a pile of receipts and would like to know what the total cost of goods would be without GST. I don't want to add up 100's of receipts again with the cost before GST but I do need this total cost. i.e.) I know I paid $400 GST, what would the formula be to find out the cost of what I purchased before taxes? Your help is much appreciated!
Answered by Penny Nom.
formula when i know the GST total 2017-10-31
From Duncan:
HI - I've added up the GST from a pile of receipts and would like to know what the total cost of goods would be without GST. I don't want to add up 100's of receipts again with the cost before GST but I do need this total cost. i.e.) I know I paid $400 GST, what would the formula be to find out the cost of what I purchased before taxes? Your help is much appreciated!
Answered by Penny Nom.
11% of a central angle 2017-10-22
From Hope:
How do you turn a percentage in to a central angle eg. 11%
Answered by Penny Nom.
A tangent to a curve 2017-10-22
From Jasem:

Suppose that
f(x)=(3x-3)^1/2.

(A) Find an equation for the tangent line to the graph of f(x) at x=2

(B) Find all values of xx where the tangent line is horizontal, and enter them as a comma-separated list (e.g., 2,-3,6). If there are none, enter none.

Values of x


Answered by Penny Nom.
Sampling distribution 2017-10-16
From Esther:
I need 4digits combination of 235689 and without replacement. Thanks
Answered by Penny Nom.
Salary plus commission 2017-10-16
From Natasha:
Veronika works at Future Shop and earns ₱10.50/h plus 6% commission on sales. Last week Veronika worked 40 hours. What was Veronika’s weekly gross salary if her total sales were ₱2050?
Answered by Penny Nom.
The distance between the origin and a moving point on a graph 2017-10-16
From Paulina:
Find the rate of change of the distance between the origin and a moving point on the graph of y=x^2 +1 if dx/dt=2 centimeters per second
Answered by Penny Nom.
27000001 2017-10-09
From Tulashiram:
If a× b =27000001, then what is the value of a & b ?
Answered by Penny Nom.
Five bales 2017-10-08
From John:

two answers were given... I believe both to be incorrect.
five bales weighed two at a time: weightx2 given: 110,112,113,114,115,116,117,118,120, and 121. It stated all combinations ie: ab,ac,ad,ad,bc,bd,be, etc. to arrive at the above 10 combined weights. I come up with the following...

Bale A= 54lbs, Bale B=56, Bale C=58, Bale D=59. and Bale E=62 pounds.

A+B=110, A+C=112, A+D=, B+C=114, B+D=115 ,A+E=116 ,C+D=117, B+E=118 ,C+E=120, and D+E=121


Answered by Penny Nom.
An enclosure for some remote control boats 2017-10-03
From Roberto:
I am trying to build a "ring" for our remote control boats, using irrigation pipe. I'd like to use full 20' long sticks for the long sides and use eight of what we call 45s, which are actually 135' fittings to make the six short sides. If I know the length of the two long sides, what would be the formula to find the exact lengths of the short sides?

Thank you.

Answered by Penny Nom.
The volume of a cone without calculus 2017-10-02
From Akash:
How to find the volume of a cone without the knowledge of calculus?
Answered by Penny Nom.
The square root of 6 is irrational 2017-10-02
From John:
http://mathcentral.uregina.ca/QQ/database/QQ.09.06/sylvia1.html
In the initial assumption of that proof, root 6 is assumed to be a/b where a and b have no common factors, but why does having a common factor make it irrational?

Answered by Penny Nom.
The importance of a comma 2017-09-30
From carolyn:
the equation 2y + 53 = 165 represents the situation joel spent 165 dollars on a pair of jeans and two shirts at the same price what does y represent
price of a pair of jeans
price of a shirt
amount joel spent on the shirts
number of shirts joel bought

Answered by Penny Nom.
Macy and Camile's ages 2017-09-30
From carolyn:
macy is 2 years older than Camille twice the sum of their ages is 52 which equation can be used to find camilles age x
Answered by Penny Nom.
How many pet dogs? 2017-09-26
From Michelle:
In 2016 there were about 135 million households in the United States. 36.5% of the households own at least one dog. On average, these households have 1.6 dogs. How many millins of pet dogs were in the United States in 2016?
Answered by Penny Nom.
Four multiples of 10 2017-09-23
From Laudacir:
Four multiples of 10 are added together.the total is a 3 digit number with three consecutive digits. What could the four number be?
Answered by Penny Nom.
Changing a mixed number to a decimal 2017-09-23
From karla:
to change an improper fraction into a decimal 5 3/4 equals
Answered by Penny Nom.
Walking up and down a hill 2017-09-19
From Justina:
At 2:20 p.m. Jack is at the top of the hill and starts walking down at the exact same time that Jill, who is at the bottom of the hill, starts walking up. If they maintain the uphill speed of 2mph and downhill speed of 4mph and the distance from the bottom to the top of the hill is 1.5 mi, at what time will Jack and Jill meet?
Answered by Penny Nom.
Up a hill and down again 2017-09-17
From justina:
Jack and Jill travel up a hill at a speed of 2mi/h.they travel back down the hill at a speed of 4mi/h.what is their average speed for the entire trip? express your answer as a mixed number
Answered by Penny Nom.
How many guests were present? 2017-09-17
From SM:
How many guests were present at an Italian dinner if every 2 guests shared a bowl of salad, every 3 guests shared a bowl of pasta, and every four guests shared a bowl of meatballs, and there were 65 bowls used altogether?
Answered by Penny Nom.
Peanuts in 5 bags 2017-09-17
From Maria:
There are a total of 270 peanuts divided up into five different bags. The first two bags contain 174 peanuts between the two of them. The second and third bags contain 130 peanuts between the two of them. The third and fourth bags contain 86 peanuts between the two of them. The fourth and fifth bags contain 42 peanuts between the two of them. How many peanuts are in each bag?
Answered by Penny Nom.
An equilateral triangle inscribed in a circle 2017-09-15
From sumit:
what is the area (in sq. unit) of an equilateral triangle inscribed in the circle x^2+y^2-4x-6y-23=0.
Answered by Penny Nom.
Complementary angles 2017-09-15
From SM:
The measure of angle A is 60 degrees more than its complement. Find the measure of angle A.
Answered by Penny Nom.
An impossible problem 2017-09-15
From Fay:
Given math homework problem of: Gary and Larry given 2 numbers and told to add together. Gary subtracted and got 14. Larry multiplied and got 799. I tried substitution:
X-Y=14 and X x Y=799
X= Y+14
(Y+14) Y=799 and here I'm stuck at Y squared+14 Y= 799
using substitution I got 47x17=799 but not 14b when subtracted. HELP?????

Answered by Penny Nom.
The angle between two planes 2017-09-12
From Arthi:
Hello. hope you are having a good day.

I am stuck on a maths question in which it is asking, how to find the angle between two given planes. These planes are in a 3D shape such as a prism.
How can I do this?
Please reply as soon as possible

Thank You

Answered by Penny Nom.
Quadrilateral ABCD is inscribed in a circle 2017-09-11
From Joie:
Quadrilateral ABCD is inscribed in a circle such that side DA is the diameter. AB=2m., BC=4m., CD=6m., angle BAD=75.93degrees. Find the area of the quadrilateral.
Answered by Penny Nom.
A pool with a dome 2017-09-11
From Bruce:
A round 24 foot pool. A dome 3 feet above pool in center. How many feet across is the dome touching top rail straight across of pool?
Answered by Penny Nom.
Simultaneous equations with fractions 2017-09-09
From Farah:
Hi, my name is Farah. I hope you can help me with this question . X/2 + g/5= 3 and 2g - f = 10
Answered by Penny Nom.
The volume of an attic 2017-09-07
From paul:
trying to determine volume of an attic base is rectangular 29 ft by 37 ft peak runs parallel to longest base side and is 7 ft long roof lines run from each end of the peak to the respective corners the peak is 3.5 ft above the base
(so seen from above there are 5 visible lines: the peak and 1 line to each of the 4 corners)

Answered by Harley.
Simultaneous equations 2017-09-02
From keto:
x-y=2,x^2+xy=12
Answered by Penny Nom.
Annual salary 2017-09-01
From Lisa:
I am applying for a job that pays $16.00 per hour, plus 5 hours of overtime every 2 weeks. I live in California, where overtime is 1.5 times the regular rate of pay after you work 8 hours a day. What would the gross annual salary be with all the overtime hours added in?
Answered by Penny Nom.
A rope formed in a semicircle 2017-08-29
From Janna:
How do I find the total length of a piece of rope formed in a semicircle that has a diameter of 8 inches?
Answered by Penny Nom.
A car on a rough road and a concrete road 2017-08-27
From Ken:
A car is driven on a rough road at a speed of 25km and then on a concrete road with a speed of 70kph. The car has 5.5 hours to travel 225km. How long did the car travel on concrete road?
Answered by Penny Nom.
A 4 sided lot 2017-08-15
From Micheal:
If one side of .4 acre is 25 feet what would the other three sides be
Answered by Penny Nom.
Two circles 2017-08-13
From Ladines:
Find the equation of the circle passing through points of intersections of circles x²+y²=4y and x²+y²=2x and the center is on line y=2
Answered by Penny Nom.
A rectangle and a circle 2017-08-12
From Ramzan:
a diagram shows the shape of rectangular framework with length (2x+20)cm and with (y+10)cm . The outline is made of wire . If a circle with diameter 56 cm is to be made from the wire and the area of the rectangle is 420 square centimeter ,find the possible lengths and width of the rectangle
Answered by Penny Nom.
1 billion x 1 billion 2017-08-11
From Kona:
1 billion x 1 billion
1 trillion x 1 trillion

Answered by Penny Nom.
(x-18)/7=6 2017-08-08
From David:
(x-18)/7=6
Answered by Penny Nom.
A pair of linear inequalities 2017-08-08
From Hempstone:
Solve the inequalities 3x-5<2x+5>4x+9
Answered by Penny Nom.
A lock with 3 tumblers 2017-07-29
From Carol:
I have a lock that has 3 tumblers. The numbers are 0 - 9. I cannot recall what I set the combination to. How many combinations will there be with three numbers?
Answered by Penny Nom.
Factorials 2017-07-27
From dinesh:
Is there any shortcut formula for multiply series numbers. like
1x2x3x4x5.......x100 =?

Answered by Penny Nom.
A semi ellipse 2017-07-25
From Ian:
The arch of the bridge is in the shape of semi ellipse,with its major axis at the water level.suppose the arch is 20ft. High in the middle,and 120 ft. Across its major axis. How high above the water level is the arch,at a point 20 ft.from the center (horizontally). Round of 2 decimal places
Answered by Penny Nom.
An arched window 2017-07-24
From Gerry:
Looking to make a full size template for an arched window. Need increments every 16". The radius is 138' 0 9/16" , the chord is 226" and the rise at center of chord is 43"
Answered by Penny Nom.
3 consecutive multiples of 11 2017-07-22
From nisha:
using the multiples formula shown at ypur site how can we solve finding 3 consecutive multiples of 11 whose sum is 363
Answered by Penny Nom.
A police officer spots a speeding car 2017-07-20
From Josh:
Marie, a police officer, spots a speeding car and starts chasing it. The speeding car travels at a speed of 130 feet per second. Marie's car reaches a constant speed of 145 feet per second 1725 feet from the start of the chase. During that time, the speeding car has traveled 3150 feet. Write a system of equations to represent this situation.
Answered by Penny Nom.
Catherine, Molly and Tom 2017-07-19
From liezel:
what is the answer of this problem, If Catherine and molly has 320pesos altogether, Molly and tom have 360 in all. Tom and Catherine have 240pesos altogether.How much does each of them have?
Answered by Penny Nom.
Parallel and Perpendicular lines 2017-07-17
From Karen:
Find an equation for the line with the given properties. Perpendicular to the line 7x + 2y = -6; containing the point (-2, -1)

Find an equation for the line with the given properties. Parallel to the line 5x - 3y =-6; x intercept = 3

Answered by Penny Nom.
Squares and rectangles 2017-07-15
From Tront:
So, there's a general rule that all squares are rectangles but not all rectangles are squares. Im trying to find a term that would describe this relationship. I've found that if all of A is B but not all B is A then I'd say that A is a subset of B, but is there a term that describes the relationship as a whole? I don't want to describe the components, I want to describe the relationship as a whole.
Answered by Penny Nom.
Exponential form 2017-07-06
From estela:
what is exponential form of 9×1000
Answered by Penny Nom.
Of cows and men 2017-07-04
From Akash:
In a group of cows and man the number of legs is 32 more than the twice the number of heads.The number of cows is?
Answered by Penny Nom.
The equation of a circle 2017-07-03
From Chen:
Find the center radius and equation of a circle in standard form given the following conditions:

1. Tangent to 3x+2y=0 at the point (0,0) and passing through (1,-1) and (6,0)

Answered by Penny Nom.
y = g(x + 1) 2017-06-27
From Shamsudiin:
HI there i am currently doing a level maths AS

I don't understand how to graph this Question y=g(x+1) i really need help please please respond to this problem

Answered by Penny Nom.
How fast am I travelling? 2017-06-24
From corey:
if i travel 2 miles in 10 seconds, how fast am i travelling please
Answered by Penny Nom.
The number of truckloads to fill a hole 2017-06-18
From Jennifer:
How many full size truck bed pickup loads do in need to get to fill a hole that is 18' x 30' that is 4" deep?
Answered by Penny Nom.
Long division with decimals 2017-06-16
From Kenneth:
Hello:

Why is it recommended to move the decimal from a divisor when dividing so that the divisor is a whole number? I know how to divide by moving the decimal from the divisor, but I cannot explain the reason.

I thank you for your reply.

Answered by Penny Nom.
The surface area of a hexagonal pyramid 2017-06-16
From Emilie:
Hello, I need help with this question:

A pyramid has a regular hexagonal base with side lengths of 4 and a slant height of 6. Find the total area of the pyramid.

Thank you in advance if you can help me...

Answered by Penny Nom.
Three consecutive natural numbers 2017-06-11
From Anita:
Three consecutive natural numbers are such that the square of the middle number exceeds the difference of the square of the other two by 60. Find the numbers.
Answered by Penny Nom.
A man drives 48km in 45 minutes 2017-06-03
From Jamal:
A man drives from Ibadan to Oyo a distance of 48km in 45 minutes. If he drives at 72km/h where the surface is good and 48km/h where the surface is bad. Find the number of kilometers of good surfaces
Answered by Penny Nom.
Simultaneous equations with fractions 2017-06-02
From Jamal:
1/x + 1/y =5 and 1/y - 1/x =-1
Answered by Penny Nom.
The maximum area of a rectangle with a given perimeter 2017-06-02
From Bob:
How would I go about finding the maximum area of a rectangle given its perimeter (20m, for example)?
Answered by Penny Nom.
A cylindrical water tank with a leak 2017-06-01
From Alexander:
if a cylindrical water tank has diameter of 90cm and leak starts at the bottom 10 litres in an hour. how long will it take for the level to fall by 20cm
Answered by Penny Nom.
Five rational numbers between -2 and -1 2017-05-31
From Prince:
Five rational numbers between -2 and -1
Answered by Penny Nom.
3rd Grade math problem 2017-05-30
From Angela:
Following is a problem from my 3rd grader's homework assignment:
Solve. Draw a diagram to represent the situation.
Rachel has 20 one-inch beads. She wants to use all of them to make a square picture frame. What will be the length of each side?
Find Perimeter, Area, and Side Length

Answered by Penny Nom.
A square peg in a round hole 2017-05-29
From bill:
bill wants to put a 2 foot square peg into a round hole What must be the circumference of the hole?
Answered by Penny Nom.
Algebra 2017-05-28
From Kerim:
I need to solve this equation.
y=at^2 - 2at
x =2a√t

Express y in terms of x and a

Give in the form y=(x^p/ma^3) - (x^q/na) where p,q,m and n are integers

Answered by Penny Nom.
A polygon inscribed in a circle 2017-05-27
From Levan:
This was the closest to what I am trying to solve. http://mathcentral.uregina.ca/QQ/database/QQ.09.06/s/dj1.html

So in the answer linked, we figure out what is "c".

But what if we know "c" and want to find out "n" based on specific "r=1".

It might be simple math, but I have not had any relationship with math for 20 years now but this question puzzles me for a reason.

Answered by Penny Nom.
Initial velocity 2017-05-23
From Annelle:
a body is thrown vertically downward from a height above the ground. determine the initial velocity and the height of the starting point if after 4 secs it reaches the ground with velocity of 68 m/s.
Answered by Penny Nom.
A trapezoid inscribed in a circle 2017-05-17
From Kameron:
I have been given a challenge problem that states that Diameter AB is drawn in a circle if 10 inch. Chords AC and BD are drawn so that each is of length 12inch and ACDB is a trapezoid. Find the height, in inches, of the trapezoid
Answered by Penny Nom.
A limit 2017-05-14
From Soham:
Find out the value of the following:
lim (x+x^2 +x^3 +.........x^n–n)/(x–1)
x→1

Answered by Penny Nom.
Two wires between two buildings 2017-05-11
From Bill:
I need to find the height of the intersection point of two building wires< br /> Building A is 860 feet high and building B is 480 feet high. The wires existent from the top of one building to The bottom of the other < br /> The distance between is 32 feet
Answered by Penny Nom.
What is the regular price? 2017-05-11
From Saira:
If the sale price is 55.50$ and the discount is 12% what is the regular price
Answered by Penny Nom.
Canada's population as a percentage 2017-05-11
From bethmarie:
What percentage of the world population (7.5 billion) is Canada's population (36 million). Thank you very much!
Answered by Penny Nom.
Yards per second to miles per hour 2017-05-11
From Scott:
If a soccer ball travels 30 yards or roughly .017 miles in 2 seconds how fast was the ball going in mph?
Answered by Penny Nom.
Slicing an inverted bowl at various heights 2017-04-30
From Joel:
Find a formula to calculate the circumferences of an inverted bowl at various heights.

E.g. Take an inverted bowl with a diameter of "x" cm and a depth / height of "y" cm.

How can I calculate the circumferences at distances of various heights above the base?

Alternately, what would be the formula to calculate the distance of the line segment resulting from a line which intersects both sides of a parabola in which that line is drawn parallel to the tangent of the vertex of the parabola at any given distance from the vertex?

Answered by Penny Nom.
Jamie has twice as many video games as Mel. 2017-04-27
From John:
Jamie has twice as many video games as Mel. Together, the boys have 24 games. how many games do each boy have
Answered by Penny Nom.
1 acre as a 2 foot wide strip of land 2017-04-25
From alan:
How can one convert 2 acres (a square - area) of land into a strip of land that is 2 feet wide and (?) feet long?
Answered by Penny Nom.
Filling cinder blocks with concrete 2017-04-24
From Katherine:
I am looking to construct a 10 X 10 dance floor using 8 x 16 cinder blocks.

I have figured that I will need about 120 blocks, however I am not sure how much concrete I will need to fill the total empty space.

Thank you

Answered by Penny Nom.
Selling price and markup 2017-04-24
From Julie:
What would the cost of an item be, if the sell price was $50 and the markup 48%
Answered by Penny Nom.
The length of a train 2017-04-24
From Stefano:
If a freight train is coming towards you and you were traveling at 85 miles an hour and it takes 35 seconds for the train to pass you how long is the train

Figure 50mph for train

Answered by Penny Nom.
Baseball Cards 2017-04-20
From Raquel:
A minor league baseball team gave away baseball cards to each person entering the stadium. One group receives 28 baseball cards. A second group received 68 baseball card. If each person entering the stadium receive the same number of cards, what was the greatest possible number of baseball cards that each person could have received?
Answered by Penny Nom.
A word problem about the dimensions of a lot 2017-04-16
From Charlotte:
If a house is 66x78 ft, how many feet out from the house around would be an acre to not include the house
Answered by Penny Nom.
Forming a cone from a circle 2017-04-15
From Tasha:
A sector of a circle subtends an angle of 216 degrees at the centre, If this sector is used to form a cone of vertical height ,8cm, calculate the base radius of the cone
Answered by Penny Nom.
Express x/a -2a/x =-1 in terms of a. 2017-04-14
From tony:
express x/a -2a/x =-1 in terms of a.
Answered by Penny Nom.
The cost of a piece of sheet metal 2017-04-14
From Mohammed:
I bought a 2 mm sheet plate of 1.2m × 2.4m in fijian $95. I used 1.95m × 0.4m from that sheet plate. What will be my total cost for the sheet plate I used. It was a mild steel plate.
Answered by Penny Nom.
Why does 10x10x10 give 1,000.0000000000001? 2017-04-12
From Randolph:
Hi, On your calculator I found that a box 10 by 10 by 10 inches has a volume of 1,000.0000000000001 cubic inches. Can you explain the numeral 1 thirteen places past the decimal? Thanks, Randy
Answered by Penny Nom.
A rectangular box filled with oil 2017-04-11
From morpal:
we want to calculate volume of rectangular box height 1275mm and lengths 2400mm and breadth 1270mm and oil s. gravity is .950 how much kg. of oil i can store
Answered by Penny Nom.
Rational numbers between -2 and -3 2017-04-08
From Medha:
Find 12 rational numbers between-2 and -3
Answered by Penny Nom.
A circle inscribed in a 30-6-90 triangle 2017-04-07
From Kameron:
i have been given a problem with a 30-60-90 triangle and a circle inscribed with a radius of 2 and was told to find the perimeter of the triangle
Answered by Penny Nom.
The height of a isosceles trapezoid 2017-04-03
From Riham:
Hi
How can I find the height in an isosceles trapezoid if I have the measurements of all of its sides ? Thank u in advance.

Answered by Penny Nom.
How to find last two or three digits of a large exponent like 17^256? 2017-03-30
From Raginee:
How to find last two or three digits of a large exponent like 17^256?
Answered by Penny Nom.
A puzzle with some integers 2017-03-30
From Glen:
Use 7,7,5,5 to equal 3 only using each # once and each math function once
Answered by Penny Nom.
13 - 3x = x times the square root of 5 2017-03-29
From Kevin:
A student I am tutoring was sent this question from his teacher.

Solve for x: 13 - 3x = x times the square root of 5

Answered by Penny Nom.
Is the square root of 2 plus the square root of 2 irrational? 2017-03-29
From haya:
how can i prove the the square root of 2 plus the square root of 2 is an irrational number?
Answered by Penny Nom.
How much money do I make per sq ft? 2017-03-28
From jackie:
If I make $12.00 per hour and I cover 750 sq ft in one hour, how much money do I make per sq ft?
Answered by Penny Nom.
Is a large integer divisible by 2^n? 2017-03-28
From Sahand:
You are given number x that is very large (that large that it can't be divided by hand) can we find out that x is divisible by 2^n or not?
Answered by Penny Nom.
Quadratic Equations 2017-03-28
From Natalie:

Hi Maths Central,

I am having trouble with quadratic equations. The question is:

Calculate the length 'l' of a rectangular pool by creating and solving a quadratic equation.

1. The rectangular pool is of equal area to a square pool, and a circular pool (5.70m^2).
2. The length of the rectangular pool must be 2m longer than the width 'w'.

Thank you so, so much for your help, Maths Central!


Answered by Penny Nom.
Differentiate y = x^x^x 2017-03-19
From Nafis:
differentiate y = x^x^x
Answered by Penny Nom.
2^a, 2^b, 1^1 and 1^0 2017-03-17
From Émile:
If you had 2^a = 4 and 2^b = 4 you could assume that a = b right? Yet we can observe that 1^1 and 1^0 both equal 1, yet 1 = 0 isn't true. So if given a log base 1 number 1 the would the answer be 1 AND 0?
Answered by Penny Nom.
What is the grade of the incline? 2017-03-12
From Jon:
If I am climbing 16 feet in elevation over 80 feet horizontal. What is the grade of the incline
Answered by Penny Nom.
A regular tetrahedron 2017-03-12
From Jeramie:
When we have a regular tetrahedron, is it always true that all the angles of the triangles measure 60 degrees?
Answered by Penny Nom.
How much does the fish weigh? 2017-03-08
From Jeri:
Fish is caught that weighs 5 pounds plus half its weight. How much does the fish weigh?
Answered by Penny Nom.
More on calculating the area (acreage) of a four-sided lot 2017-03-06
From Kerri:

Can you please show the formula used to calculate areas as provided on this page:

Calculating the area (acreage) of a four-sided lot


Answered by Penny Nom.
Exponential form 2017-03-04
From michael:
I was teaching my student about exponent when we stumbled across this problem.

"write (-125)^-3 in exponent form"

The problem is with the number "-125". I think -125 = (-5)^3. But one of my student thought that it is -125= -5^3.
Both of them is equal to -125 but they totally different in structure. I wonder which one is correct and why it is. I am worried if they get this wrong, it might spell problem in the future when we start substituting the numbers with variable.
Thank you very much for reading my question and answering it.

Answered by Penny Nom.
Tons of sand 2017-03-01
From Elizabeth:
Hello my arena is 132.23 feet width by 132.23 feet length. I need to put 4 inches of sand. How many tons of sand do I need ?
Answered by Penny Nom.
The inside diameter of a silo 2017-02-28
From JOHN:
I need to find the inside diameter of a cement silo with an outside circumference 118 ft and a wall thickness of 7 inches. thank you John Kerr.
Answered by Penny Nom.
Building a hexagon enclosure 2017-02-26
From Kenneth:
Hello, I have a question for you. I have a special needs daughter and I want to install a ceiling fan into her room. The fan is 42 inches in diameter. I would like to build a wooden hexagon enclosure to protect the fan. I would like the enclosure to be at least 2 inches wider than the fan on each side measuring from flat surface across to flat surface. How do I figure out the length of each panel so that I can miter them and assemble it? Thank you for your time.
Answered by Penny Nom.
Covering a ceiling with tablecloths 2017-02-24
From DEANNA:
I am trying to cover a 16 x 31 foot ceiling with tablecloths. For sale, are 90 x 132 inch tablecloths for $4.97 a piece and 60 x 102 inch tablecloths for $3.85 a piece. Which would be my cheapest option to cover the entire space?
Answered by Penny Nom.
Filling a pool with two hoses 2017-02-20
From Charlene:
A large hose will fill a pool in 40 minutes while it takes a smaller hose 60 minutes to fill the same pool. The same pool takes 80 minutes to drain if the drain is open. Suppose one day both the hoses are turned on, but by accident the drain was left open. How long would it take to fill the pool that day?
Answered by Penny Nom.
Covering the ceiling of a garage 2017-02-20
From DeAnna:
I am trying to drape a 15 x 30 garage with yards of fabric. If the yard of fabric is 60 inches wide, would 10 1/4 yards be enough for my space?
Answered by Penny Nom.
A total amount of $27,500 invested at 5% and 7% 2017-02-19
From Brad:
Kristina invests a total of $27,500 in two accounts paying 5% and 7% simple interest, respectively. How much was invested in each account if, after one year, the total interest was $1,645.00.
Answered by Penny Nom.
A block base for a plywood rectangle 2017-02-18
From Joe:
How many cinder blocks 16x8 would i need to build a solid base for a plywood base 54inches wide and 30 inches deep Thank you
Answered by Penny Nom.
Vinculum 2017-01-27
From Dick:
What is the line or bar over a Roman Numeral called?

Eg. V with the bar over it representing 5,000 .

Answered by Penny Nom.
Feet per second to miles per hour 2017-01-27
From ron:
if a vehicle travels 50 feet in 2.5 seconds what is the speed in mph. 3600 sec in hour /5280 feet in a mile i get .68181818. how do i figure the 2.5 seconds. all the calculators I've used show 34 mph is that correct or am i forgetting the 2.5 second. please help.
Answered by Penny Nom.
The volume of a slab 2017-01-21
From Russell:
Working with a segment of a circle. The O.D. is 3.440" the length of the chord is 3.130 I need information on the volume of the largest area. I would like to know (in thousands of an inch) how tall would the largest area need to be to equal .1143 cubic inches If you need more info I can send it Thanks in advance
Answered by p.
Angular speed 2017-01-21
From Kenzi:
A wheel of radius 14 inches is rotating 45°/sec. What is the linear speed in in/sec and the angular speed in RPM? (Round your answers to three decimal places)

I already found the linear speed. I got 10.996 in/sec. I don't know how to find the angular speed.

Answered by Penny Nom.
The average rate of change of cot(t) 2017-01-18
From Brianna:
Hello! It's been a while since I've taken a math course, and I'm stuck on a problem in my calculus course.
The question is this:

Find the average rate of change of the function over the given interval.
h(t)=cot(t) a) [5pi/4, 7pi/4]

Answered by Penny Nom.
Five equations 2017-01-16
From Muhammed:
A x 4 = E
B ÷ 4 = E
C + 4 = E
D - 4 = E
___________
& A + B + C + D = 100
what is the value of E

Answered by Penny Nom.
One millionth of one percent of 90,000,000,000 2017-01-16
From Gene:
What absolute number is one millionth of one percent of 90,000,000,000
Answered by Penny Nom.
How do I establish the original List Price? 2017-01-15
From Bob:
(1) I have a net price item at $4,664.66 but do not have the discount percentage. I have been told I received 40%? How do I establish the original List Price and to confirm what discount I received?
Answered by Penny Nom.
A circle inscribed in an isosceles triangle 2017-01-14
From Sal:
There is a right isosceles triangle. Inscribed inside of it, is the largest possible circle. Ho do you find the value of the radius?

I want to find out a way of only using the rules/laws of geometry, or is that not possible.

Answered by Penny Nom.
8^3/2(2+2) 2017-01-13
From Mary:
8^3/2(2+2)
Answered by Penny Nom.
A rectangle with a given perimeter and area 2017-01-11
From Joe:
i have a math question for homework that i don't understand. can you draw a rectangle that has 18cm in perimeters and 20cm in areas. i tried but i can't.
Answered by Penny Nom.
An angle and its complement 2017-01-10
From Ysa:
If the measure of an angle is twice the measure of its complement, what is the measure of the angle?
Answered by Penny Nom.
Renting an electric cart 2017-01-07
From Desiree:
Vacation Rentals rents electric carts for travel on their grounds. The economy plan rents carts for $26 per day and the premium plan rents a carts for $12 per day plus a one time fee of $126.05
For how many whole days would it be cheaper to rent from the economy plan then from the premium plan?

Answered by Penny Nom.
A tolerance limit is 0.5% 2017-01-06
From Joe:
Dear Sir/Madam,

How can I describe the following.

My volume tolerance limit is 0.5%,regardless of quantity.

Say , My total volume output is 344,714 Litres and benchmark volume is 348,194 Difference of 3480Litres, this would give me a percentage of 0.999.

Which is above my tolerance.
How can I explain this high tolerance limit even though volume difference is minimal.

Regards

Joe

Answered by Penny Nom.
A parabolic arch 2017-01-05
From Rand:
It is most likely already been answered but I can not seem to find the right key words for the search engine?.

What I am looking for is, if you have an have arch/arc and you know the degree of slope and the height of the arch/arc from ground lvl; how do you factor the decreasing angle/#’s to get the distance tween the two feet on the assumption that the arch/arc is curved all the way to ground lvl?

a. where the legs widen continuously (till they hit ground) so yes parabolic &
b. where the legs come down straight after a ½ circumference run.

What I am focusing is the parabolic.

Many Thanks

Answered by Harley Weston.
Plans for dog kennels 2017-01-04
From Pat:
So... I'm trying to draw up a plan for my new kennel buildings. They are going to built out of grain bins. I know, cool,... right. Anyway the kennels will be along the outside of the circle and be shaped like wedges. the center will be a play area. All of the kennel sides are 6 ft in length so the size of the gate will determine how many kennels I can fit in each bin. If I want to put an odd number of kennels in and I cant draw a straight line across to make all of the wedges the same size... how do I figure out the angles...degrees ? maybe i'm making it too hard but i'd like to give the dogs as much space as i can. I did up some ... attached.
Answered by Harley Weston.
A rational number between -4 and -5 2017-01-03
From Anshika:
Write rational number between -4 and -5
Answered by Penny Nom.
Water leaking from a trough 2016-12-28
From Kathryn:
A trough is 6 m long, and has uniform cross-section of an equilateral triangle with sides 1 m. Water leaks from the bottom of the trough, at a constant rate of 0.1 m3/min. Find the rate at which the water level is falling when the water is 0.2m deep.
Answered by Penny Nom.
Simultaneous equations with fractions 2016-12-21
From Kimi:
I am stuck on this,can someone please help me????

1/2x+1/3y=11
8x+2/5y=102
Can someone please explain how to solve questions like these?? Or else I will never learn...
Thank you

Answered by Penny Nom.
Two concentric circles 2016-12-21
From shrestha:
Two concentric circles have radii of 14 cm and 7 cm respectively. Find the area of space between them.
Answered by Penny Nom.
The area and radius of a circle 2016-12-19
From india:
How do you find the radius of a circle if only the area is given to you?
Answered by Penny Nom.
Joining a fitness center 2016-12-18
From Salem:
A fitness center offers a special price for group-class students. The cost of registration is 30 dollars and then you have to pay 25 dollars per month. What is the rule of the equation in the light of
c=---m+----?

Answered by Penny Nom.
1/1-cosine(2x) - 1/1+ cos(2x) 2016-12-14
From Sean:
1/1-cosine(2x) - 1/1+ cos(2x)
Answered by Penny Nom.
Divisibility of 3n^5+7n 2016-12-14
From Parag:

Question from Parag, a student:

if n is a natural number,then 3n^5+7n is divisible by
a)2
b)3
c)5
d)7

i got the answer but still i need a valid alternate approach.


Answered by Penny Nom.
3^2x-5=27 2016-12-14
From danaja:
solve for the following 3^2x-5=27
Answered by Penny Nom.
How far apart are the boats? 2016-12-13
From Halley:
Two boats leave port at the same time. Boat A travels east at a speed of 12 km/hr. Boat B travels southwest at a speed of 14 km/hr. After two hours, how far apart are the boats? North is 0 degrees. How do I figure this out. Thanks
Answered by Penny Nom.
x^2 = -16 2016-12-12
From A student:
x to the second power = -16

what number solves the equation?

Answered by Penny Nom.
The area of a city block 2016-12-08
From Miura:
A city block is three times as long as long as it is wide.if the distance around the block is 0.48 kilometers, what is the area of the block in square meters?
Answered by Penny Nom.
A sales tax of 7% 2016-12-07
From Kenneth:
Hello:

If a sales tax of 7% is placed on every $1.00 of merchandise for sale, is it correct to indicate the tax as 7%/ per $1.00 or 7%/$1.00? If the calculation is expressed as 7%/$1.00 X $5.00, the tax is not $0.35 but 0.35. Is a tax rate of 7% incorrectly represented as 7%/$1.00 since the dollar unit cancels from the multiplication?

I thank you for your reply.

Answered by Penny Nom.
The sum of three integers is 30 2016-12-06
From Vaishnavi:
_+_+_:30. Blanks are to be filled by(1,3,5,7,9,11,13,15) and numbers can be repeated.
Answered by Penny Nom.
Profit and loss on two shirts 2016-12-06
From Mahesh:
A man purchases one shirt and one T-Shirt for Rs. 6000. He sells the shirt at a profit of 20% and T-Shirt at a loss of 10%, as a result he gains 2% on whole transaction. what is the cost price of the T-Shirt ?
Answered by Penny Nom.
Two boys on the same route 2016-12-06
From Abbey:
Starting from the same spot, one boy completed a route in 9 minutes and another boy completed it in 15 minutes. When will both boys be at the same starting point at the same time again?
Answered by Penny Nom.
Three friends bought 13 books 2016-12-06
From alexis:
Marina bought 4 books José bought as many books as Ben bought together the 3 friends bought 13 books how many books did Ben buy
Answered by Penny Nom.
Gallons and cubic inches 2016-12-05
From Theresa:
If I did not know the cubic inches in a liquid gallon how would I find it? In other words what is the formula for this?
Answered by Penny Nom.
A pie chart 2016-12-05
From vickie:
Determine the central angle needed to form a pie chart for the following housing characteristic data: 63% owner occupied, 27% renter occupied, and 10% vacant
Answered by Penny Nom.
Covering a floor with plank flooring 2016-12-04
From sandra:
my bathroom is 5'x6'.2"
i want to purchase plank flooring which covers 19.8 sq ft per box.
how many boxes would i need.

Answered by Penny Nom.
The diagonals of a cube 2016-12-02
From Peter:
when u have a cube and the inner diagonal is 41.6 how long is the outside diagonal
Answered by Penny Nom.
Building a pyramid 2016-12-01
From Jesse:
I'm trying to build a 30"by30"by 18.25"tall pyramid,what should my cutting measurements be?
Answered by Harley Weston.
Acres and miles 2016-11-30
From Carolyn:
If I am running a pipeline of 1,100 miles, how many one acre plots will it take to cover this distance?
Carolyn

Answered by Penny Nom.
Is every rectangle a rhombus? 2016-11-28
From Robert:
not every rectangle is a rhombus.true or false? if false please leave a short explanation. thank you:)
Answered by Penny Nom.
A stone walkway around a garden 2016-11-27
From Abigail:
A garden measures 12 feet by 16 feet. The family would like to add a stone walkway around the garden. This would increase the total area of to 302.25 square feet. what will be the width of the of the walkway
Answered by Penny Nom.
Two tangent circles 2016-11-27
From mikee:
find the equation of a circle tangent to the circle x2­+y2=4 and with the center at (0,5)
Answered by Penny Nom.
A circle inscribed in an equilateral triangle 2016-11-27
From jo:
what is the radius of the inscribed circle of an equilateral triangle with altitude 12 units?
Answered by Penny Nom.
2^x= -2x - 11 2016-11-27
From Kathy:
I don't know how to start this problem:
2^x= -2x - 11

Answered by Penny Nom.
The weights of two boxes 2016-11-25
From dearbhla:
One box weighs 3 3/7kg. Another box weighs 1 5/6 as much. How heavy is the second box?
Answered by Penny Nom.
Simultaneous equations 2016-11-25
From Rohan:
x-2/3 + y - 1/4 = 13/12
2-x/2 + 3 + y/3 = 11/6

Answered by Penny Nom.
Radius angle and arc length 2016-11-24
From pavidthra:
Length or arc 11 and angle of subtended 45.need to find a radius
Answered by Penny Nom.
Multiplication and repeated addition 2016-11-23
From Anandmay:
Hello I was looking closely at early arithmetic where I found how we discovered properties of Arithmetic. Like:2 x 3 = 3 x 2. This can be proved by considering a 2-D figure(actually,quadrilateral) having length consisting of 2 boxes of 1-by-1 dimensions and breadth of 3 boxes of the same dimensions. Now,consider it again,but,this time,length of 3,and breadth of 2 of such 1 by 1 boxes. We now notice that we can fit the 2 types of rectangles formed on each other precisely. So the multiplicative property of commutativity is true for all natural numbers as we can generalize the result(in our mind,for self satisfaction).

Now,can you find me a nice satisfactory reason of why a fraction times a natural number equals the number times the fraction? I mean, for example,i can understand the meaning of 3 x 2/3 to be three times 2-3rd,that is, 2/3+2/3+2/3.Fair enough. But here is the problem:By definition and actual meaning of multiplication, a x b means the repeated sum of b,done 'a' times. So what is the meaning of doing 2/3 x 3?The repeated addition of 3 how many times??2/3 times??Not making sense,right?And even we have not proved yet the commutative property of numbers INCLUDING fractions.So how can we resolve this problem and make these things meaningful?
Thanks

Answered by Penny Nom.
Why are equivalent fractions equal? Part 2. 2016-11-22
From Anandmay:
Hi! I saw one of your answered questions:Why are equivalent fractions equal? The same question I had for why i searched and ended up entering this wonderful site. The answer was so much experimental based.I mean,for example,it was explained how 3/4 was equal to 6/8 by dividing a unit into 4 equal parts and taking four of the parts,and then dividing the same unit into 8 equal parts and then taking 6 of them.The result was that both the taken parts were equal. (That is,3 out of 4 equal parts were equal to 6 of the 8 equal parts.) But,this was very much non-generalized and experimental-based result(proof). And we all know that in mathematics,we confirm some particular thing for all the numbers iff we generalize it.

So,can you explain more clearly 'HOW' the 3 parts of the 4 equal parts and the 6 of the 8 equal parts of the unit equal(that is the same)?I don't want experimental proof.I want clear proof,and thus please give a generalized proof for all fractions and their equivalents.
Thanks!

Answered by Penny Nom.
A square inside a circle inside a square 2016-11-13
From Jeff:
Greetings,

I came across a question from a textbook from years ago. I've been trying to solve it, but am not sure if my approach is correct.

There are 2 squares (1 inner, 1 outer) & 1 circle.

The inner square is the largest square that will fit inside the circle. It has an area of 1 unit.

The circle is the biggest that will fit in the outer square.

What is the area of the outer square?

Answered by Penny Nom.
The amount of material remaining on a roll 2016-11-11
From Yoh:

Question from Yoh:

Hello,
I am trying to find impressions on a roll (either full roll or partial).
Let's say I have the following information.
- Outer Diameter of roll
- Inner Diameter of roll (cardboard core)
- thickness per layer
- Each cut off length (impression length)

Now let's say a roll has a 40in outer diameter, the cardboard core has a 3.75in diameter and a thickness of .002. Each impression has a cut off of 14inches.
With this the roll will have approximately 2,700 impressions. How would I find the remaining impressions if the outer diameter of the roll is only 6.5inches?

Thank you.


Answered by Penny Nom.
A fish and eight parakeets 2016-11-11
From Cindy:
Cindys fish weighs 8 times as much as her parakeet. Together the pets weigh 63 ounces. How much does the fish weigh?
Answered by Penny Nom.
An equation with fractions 2016-11-09
From Tina:
How do you solve x+1/3=x+2/5. X+1 divided by 3= x+2 divided by 5. What's x=
Answered by Penny Nom.
Volume of liquid remaining in a tilted cylinder 2016-11-08
From Brian:
I am trying to determine the amount of a liquid remaining in a 55 gallon drum when it is tilted at 45 degrees and the liquid level is low enough so that the liquid does not completely cover the bottom of the drum.

Your help is greatly appreciated.

Answered by Harley Weston.
The area of a rectangle 2016-11-05
From Ivan:
If the vertices of a rectangle are a(1,3),b(3,-4),and c(1,-4),find it area.
Answered by Penny Nom.
A key for a lock with five tumblers 2016-11-02
From Aeriel:
A lock works by having a key turn a sequence of tumblers. Consecutive tumblers have different heights, and in order to unlock the lock, the sequence of heights on the key must exactly match those of the lock. The picture below shows a lock with four tumblers. How many different keys can be made for a lock that has 7 tumblers with 5 possible heights each?
Answered by Penny Nom.
A problem with a recipe 2016-10-31
From Ted:
Recipe calls for 3 1/2 c of sugar & 4 c of water. I only have 3c of sugar how much water do I use
Answered by Penny Nom.
Three circles 2016-10-30
From yolani:
two equal circles with centre A (1;1)and B (4;5)touch a third circle with centre P as shown in the diagram. If P ,A and B Lie on a straight line find the coordinates of P
Answered by Penny Nom.
A pattern rule for a sequence 2016-10-28
From Grace:
Here's a Grade 7 question, we need to find the pattern rule:

1 - 1
2 - 3
3 - 6
4 - 10
......
It may involve exponents, but we can't seem to figure it out.
thank you.
Grace.

Answered by Penny Nom.
Parallel lines 2016-10-26
From Jordan:
(7,-5) 3x -4y=5
Answered by Penny Nom.
A word problem at the fair 2016-10-25
From Rick:
Have 48 tickets to spend, have to spend them all in exactly 10 events. Rides cost 6 tickets games cost 4 tickets & treats cost 3 tickets
Answered by Penny Nom.
The volume of a hexagonal pyramid 2016-10-21
From Elizabeth:
Find the height of a hexagonal pyramid with a volume of 1320 ft ^3. The area of the base is 120 ft^2.
Answered by Penny Nom.
How many marbles does Haley have? 2016-10-19
From Pat:
Sabrina has 1/4 the marbles as Haley.
They have 305 total. How many marbles does Haley have?

Answered by Penny Nom.
y = mx + c and Ax + By + C = 0 2016-10-18
From Erick:
please explain to me how to convert an equation from the form y=mx+c to Ax+By+C=o form.
Answered by Penny Nom.
The degree of 2x + 7 2016-10-17
From Deejay:
How about when the given has no degree and the other has a constant only, For example 2x + 7, where's the leading term and what is its leading coefficient?
Answered by Penny Nom.
Travelling upstream and downstream 2016-10-17
From Kathleen:
It took a boat 2 hours to reach town A going upstream. The way back was 1h20 min. What is the speed of the boat in still water if the speed of the stream is 3 mph?
Answered by Penny Nom.
A circle formed from bending a wire 2016-10-16
From Deszaree:
The same piece of a wire is bent to form a circle
calculate:
a, the radius of the circle
b, the area of the circle

Answered by Penny Nom.
A probability question concerning eggs 2016-10-16
From Brian:
Two participants are alternatively selecting eggs from a basket of 6 hard-boiled eggs and 6 uncooked eggs. Alternatively they are smashing the eggs on their foreheads. One participant gets 4 hard boiled eggs in a row and the other person gets 4 uncooked eggs in a row. What is the probability of this occurring (I watched this happen).

Thank you

Brian

Answered by Penny Nom.
A linear relationship 2016-10-16
From Bianca:
x represents the number of hours since 8am. y represents the number of children the school nurse has seen during the school day. The nurse has seen 3 children by 8am and 27 children by noon.
How many children has the nurse seen at 2pm?
If the nurse has seen 18 children, what time is it?

Answered by Penny Nom.
An airplane schedule 2016-10-14
From Stephanie:
Starting at 8:00a.m., a passenger plane leaves from an airport every 6 minutes and a cargo plane leaves every 7 minutes. When is the next time these kinds of planes will leave at the same time?
Answered by Penny Nom.
Selina saved $6 on a 20% off sale 2016-10-14
From Myra:
I don't know how to help my child answer this question - Selina bought a shirt on sale that was 20% less than the original price. The original price was $6 more than the sale price. What was the original price?
Answered by Penny Nom.
A five digit number 2016-10-12
From Massa:
Explain why any four digit number is less than any five digit number?
Answered by Penny Nom.
How far is the boat from the harbour? 2016-10-06
From Karabo:
a boat sails 30 km due east from a harbour. Then it sails 40 km due north. How far is it from the harbour?
Answered by Penny Nom.
A triple product 2016-10-03
From sam:
Let a and b are vectors such that
Vector a = (1,1,2)
Vector b = (2,-1,1)
And let vector c be a unit vector such that triple product of a,b,c is minimum . We have to find the value of c.

[Thoughts]
I thought triple product of a b, c means the volume occupied by parallelepiped. And we have to do volume minimum

Answered by Penny Nom.
The equation of a line 2016-10-01
From Miriam:
write the equation of the line that passes through (6,7) and is perpendicular to 3x+5y=0
Answered by Penny Nom.
The equation of a circle 2016-09-30
From Trent:
write the equation of circle B with center B(-2,3) that passes through (1,2)
Answered by Penny Nom.
Hexagonal pyramid bevel angles 2016-09-30
From Peter:
I have seen your response to a similar question from Steve which Chris and Harley responded to, however I am not familiar with some of the terms. Is there a formula that I can enter my details in which will give me the specific angles I require. The item I am constructing is much larger than the one you responded to previously. Thank you for any assistance you are able to provide.
Answered by Penny Nom.
A 2 million gallon tank 2016-09-28
From Kimberly:
I have a tank that is shaped like a cylinder, and it has to hold 2 million gallons of water. There is a height limit of 30 ft. I know the equation is V=pi x r squared x h but I don't know if I need anything For the square foot of water.
Answered by Penny Nom.
Jenny, Charles and Chase have some butterflies 2016-09-26
From Kyla:
Jenny cares 7 times as many butterflies as Charles. Charles catches 2 times as much as Chase. All together they have 560 butterflies. How many butterflies does each person have? Please help, I am very confused on how to solve the problem.
Thank you!

Answered by Penny Nom.
2,006-1-2-3-4-...-48-49-50 2016-09-24
From Mimi:
Compute the following:

2,006-1-2-3-4-…-48-49-50

Answered by Penny Nom.
The length of stock on a roll 2016-09-23
From Ken:
I have a roll that is 13.5 inches wide ( OD) , and is wound on a 3 inch core. the thickness of the material is 3.5 thick.
Do you know of a formula that can calculate the linear feet on that roll??

Answered by Harley Weston.
Standard form 2016-09-22
From anna:
ok. i have b=2/3x+5/7. im supposed to change it to standard form. how do i do that?
Answered by Penny Nom.
The width of a 300 foot long rectangular acre 2016-09-21
From Debbie:
If 2 sides of a 1 acre plot is 300 feet each what are the other 2 sides? The acre is a rectangle.
Answered by Penny Nom.
The distance around a warehouse 2016-09-21
From Chuck:
How far do I walk when I walk around a 751,000 sq. ft. warehouse?
Answered by Penny Nom.
Two equations 2016-09-17
From Nilesh:
How to solve (x-y)^1/2 + 3(x-y) = 30 ; xy + 3(x-y) = 11?
Answered by Penny Nom.
Integrate (x^2 - 4x + 4) ^4/3 2016-09-15
From Ifah:
Hi i have questions please answer
Integral 2 sampai 3 dari (x² - 4x + 4) ^4/3 dx

Answered by Penny Nom.
20/2 + 2 * 2^2 + 6 = 8 2016-09-15
From Simone:
20/2 + 2 * 2^2 + 6 = 8

insert brackets to make this true

Answered by Penny Nom.
The equation of a ski slope 2016-08-31
From Hannah:
The information provided is
---A ski slope has a slope of -0.2. You start at an altitude of 10,000ft. Then the question is "Write the equation giving altitude (a) as a function of horizontal distance moved (d).
I know this is probably a very simple question but I didn't take a math class my senior year and now I'm a freshman in college, so some of it is very frustrating to me.
Any help is much appreciated, thank you!

Answered by Penny Nom.
The dimensions of a rectangle 2016-08-31
From Hazym:
How do I find the length(which is 15m longer than the breadth) and the breadth of a rectangle just by its perimeter which is 70 m?
Answered by Penny Nom.
Exponential form 2016-08-31
From Lily:
Hi! I was wondering, I was having a little issue understanding this one problem. It was "write 81 in exponential 2 different ways" and I already got 9(2) but I was wondering what the second one could be! Thanks! -Lily
Answered by Penny Nom.
Todd Matt and Jeff 2016-08-30
From Lisa:
Three brothers spent different amounts of money on their recent vacation.
Matt spent $25 less than Todd.
Jeff spent 3 times as much as Matt.
How much did each brother spend if they spent a total of $275?

Check answer required
T= Todd

Answered by Penny Nom.
Two sequences 2016-08-20
From Ting:
The first four terms of a sequence are 4, 20, 56 and 120. By comparing with the nth term of sequence 1,5,14 and 30 which is 1/6n(n+1)(2n+1), find a formula for nth term. Can you precisely show out how you do this questions ( with the steps please)
Answered by Penny Nom.
I bought 3 books for $150.00 2016-08-20
From Shavon:
if i bought 3 books for $150.00 and 1 of the books cost 50% more than both other books combined what's the price of the more expensive book?
Answered by Penny Nom.
A suspension bridge 2016-08-18
From sai:
The main cable of a suspension bridge has the shape of a parabola. The cables are strung from the top of two towers, 61 metres apart, each 15.25 metres high above the roadway. The cable is 1.5 metres above the roadway at the point that is directly between the towers.

h(x)=a(x-30.5)^2+1.5
where a is the constant and its value to be determined.

a) determine the value of the constant a by using that the fact that height of the suspension cable, h(x) is 15.25 metres at each tower. Hence, write the updated model equation.

Answered by Penny Nom.
The ages of 10 children 2016-08-17
From Diann:
Sum total of the ages of 10 children 10 children is 110, what are their ages if all are 2 years apart?
thank you

Answered by Penny Nom.
A search area 2016-08-13
From tammy:
if your searching an area and you go 300 km from point A and search 380° what or how much area would you search?
Answered by Penny Nom.
Covering an octagonal region with sand 2016-08-13
From Lynn:
How much sand is needed to fill a 20 ft. wide octagon area 6 inches deep?
Answered by Penny Nom.
The area of a circle 2016-08-12
From sandeep:
if circumference of a circle is πd what is the area?
Answered by Penny Nom.
A ship and a light 2016-08-11
From Kirby:
A ship is sailing due west when a light is observed with heading 297°50' from the ship. After the ship has travelled 2250 feet, the light heads 311°55' from the ship. If the course is continued, how close will the ship approach the light?
Answered by Penny Nom.
A 15% discount 2016-08-09
From eunice:
brian brought a printer sale at the store. the discount was 15%, and brian saved 550. what was the printer original price?
Answered by Penny Nom.
Two lots 2016-08-08
From Deniece:
I have 2 lots that are joined together. One lot is 38' X 105' and the other is 45' X 105'. What is the total in acre size. Or, how close are these lots to make one full acre? I want to sell both lots and want to advertise using the total lot size. Thank you.
Answered by Penny Nom.
Carlos, Mark, Jeremy, and Alan collect Yugioh cards 2016-08-08
From justin:
Carlos, Mark, Jeremy, and Alan all collect Yugioh cards. Mark has twice as many cards as Jeremy. Carlos has 3 times as many cards as Jeremy, and Alan has collected 12 less than 3 times as many cards as Jeremy. Together the four players have collected 456 cards. How many does each person have?
Answered by Penny Nom.
The diameter of a cylindrical tin 2016-08-04
From Scott:
a cylindrical tin is 15 inches high and holds 5 gallons of paint when it is full. Find the diameter of the tin.
(1 gallon = 0.1605 cubic feet)

Answered by Penny Nom.
The modulus of a complex number 2016-07-29
From Cheyenne:
There's a question on my Summer Assignment I cant figure out. Here it is:

Find the absolute Value of the complex number. -5i

Please help?

Answered by Penny Nom.
The circumcentre of a triangle 2016-07-28
From Raj:
If the vertices of triangle ABC are A(-4,-1) B(1,2) C(4,-3) then find coordinates of circumcentre of triangle
Answered by Penny Nom.
The dimensions of a rectangle 2016-07-25
From Krunox:
A rectangle is twice as long as it is wide. Its perimeter (the sum of the lengths of its sides) is 60 cm. Find its length and width.
Answered by Penny Nom.
A cubic equation 2016-07-25
From Chinmay:
x³-2x²-7x+5
Answered by Penny Nom.
The number of sides of a polygon 2016-07-23
From Shriya:
All the angles of a polygon are either 155° or 140°.
There are twice as many angles of 155 °or 140°.
Find the number of sides of the polygon

Answered by Penny Nom.
The temperature inside and outside a greenhouse 2016-07-22
From Mehzad:
Every day, the temperature in a greenhouse is at its low temperature, 70 degrees Fahrenheit, at 2 a.m. and at its high temperature, 84 degrees Fahrenheit, at 2pm. Its temperature increases linearly between 2 am and 2pm , and decreases linearly from 2 pm and to 2am. The outside temperature follows the same linear patterns, but has a low temperature of 60 degrees Fahrenheit at 2 am and a high temperature of 78 degrees Fahrenheit at 2 pm. At which of the following times will the temperature inside and outside the greenhouse be the same?

a) 8:00 am
b) 12:00 noon
c) 4:00 pm
d) The two temperatures will never be the same

Answered by Penny Nom.
Two equations with fractions 2016-07-22
From kanesh:
p/2+q/3=3

p/4+2q/3=3

Answered by Penny Nom.
Which term of this sequence has value 8? 2016-07-20
From Lauren:
Hi there

Which term in the sequence -2; -5/3; -4/3 ; -1; ... has a value of 8.

Since term 2 and 3 of this sequence contain fractions which can be converted to recurring decimals. What is the best way to work out the common difference here.

I do however understand that to work out the nth term of an arithmetic series, the following formula Tn = a + (n-1)d. In this series a = -2

Answered by Penny Nom.
A congruence theorem for two right angled triangles 2016-07-17
From Sayari:
Hello. Is it possible for two right angled triangles to have the same length of base and height and a different hypotenuse? If not so, then why in the congruence criteria RHS the hypotenuse is given more importance? It can also be like- 'two triangles are congruent if they are right angled and have the same base and height.' Thank You.
Answered by Penny Nom.
The composition of a function with itself 2016-07-17
From Mel:
If f(1 + 3x) = 1 * x, solve f(f(x))
Answered by Penny Nom.
If x+y=7 and x^2+y^2=42, then xy equals? 2016-07-17
From Mehzad:
If x+y=7 and x^2+y^2=42, then xy equals?
Answered by Penny Nom.
A train from Erie to New York 2016-07-14
From carol:
a train leaves Erie with twice as many women as men. At York, 17 men get on and 16 women . How many men and women were originally on the train?off. There are now the same number of men as women
Answered by Penny Nom.
Lisa went for a run 2016-07-14
From Olive:
Lisa ran 3km north and then 4 km east what was her total distance
Answered by Penny Nom.
A 1.4 acre square 2016-06-28
From Jamie:
Dear Faithfullness,

I was previously to be purchasing a square 1 acre plot, which I believe was to have 209feet, 4 sides perimeter.

How big would a 1.4 acre site be?
In sq feet length ie original 209ft + (the adeed 0.4)

Answered by Penny Nom.
Triakis 2016-06-27
From Gordon:
Some authorities treat "triakis" (and related terms "dyakis", "tetrakis", etc.) as attached prefixes; others treat them as separate adjectives. Thus I see one of the Catalan solids described as both "triakistetrahedron" and "triakis tetrahedron". Which usage is correct?
Answered by Chris Fisher.
Slope 2016-06-27
From Ashiq:
What is 1% slope if length 101 feet?
Answered by Penny Nom.
An equation involving pi 2016-06-26
From Diaz:
How would you solve :

( pie*X)/ 4 = -pie/2

I have no idea.what to do!

Answered by Penny Nom.
Simultaneous equations 2016-06-25
From sena:
2x/3+3y/2=-1
4x-5y=22

simultaneous equation

Answered by Penny Nom.
An open ball is an open set? 2016-06-23
From Aqeel:
Theorem: The open ball is an open set?
Answered by Harley Weston.
Cubic feet of soil 2016-06-19
From H:
A garden is 8 ft x 7 ft that needs to raised 6 inch's How many cubic feet of soil required ? What am I missing ?

Thanks

H

Answered by Penny Nom.
Quadrilateral 2016-06-14
From Muhib:
Is there a quadrilateral with 0 sets of parallel sides.
Answered by Penny Nom.
The difference between the ares of two rectangles 2016-06-09
From Ingrid:
I am trying to help my son with an area question.
I have the answer, from the solutions, but I cannot figure out how to teach him.

Question:

Two rectangles have lengths 13cm and 19cm respectively.
Their total area is 376cm squared.
If both their widths are whole numbers, what is the difference in their areas?

I know that this is solvable once I determine the widths of the rectangles ,
but how do I go about finding that?

Thanks for your help

Answered by Chris Fisher and Harley Weston.
Implicit differentiation 2016-06-06
From Pranay:
Is a circle x^2+y^2=2 a function? If it is not a function, why is it possible to do implicit differentiation on it?
Thanks.

Answered by Penny Nom.
The lateral side length of a cone 2016-06-05
From Diane:
Question is find the lateral side length of a right cone with area of 372 sq. cm and base circle radius of 9 cm.
Answered by Penny Nom.
John and Max walk to meet each other 2016-05-29
From abhay:
The speeds of John and Max are 30 km/h and 40 km/h.Initially Max is at a place L and John is at a place M. The distance between L and M is 650 km. John started his journey 3 hours earlier than Max to meet each other. If they meet each other at a place P somewhere between L and M, then the distance between P and M is?
Answered by Penny Nom.
A barn roof 2016-05-29
From Joe:
Is it possible to build a barn roof (irregular pentagon?) with a 12' base and the other 4 sides 4' each? Thanks.
Answered by Penny Nom.
Placing a sphere in a cylinder of water 2016-05-27
From Fiona:
Hi the question i have is, if I have a cylinder filled with water and the volume of the water is 150Pi what would the rise in the level of the water be if you put a sphere into the cylinder with a volume of 500/3Pi? Sorry if it's a bit long winded
Answered by Penny Nom.
A cylinder in a cube 2016-05-25
From Sonia:
A cylinder has the same height and diameter as the sides of a cube use 3.14~ which has the greater volume
Answered by Penny Nom.
The area of a 4-sided lot 2016-05-25
From prasad:
How to find area of a land whose sides are 41ft,33ft,32.3ft and 33.2 ft. Pl give me the formula and proof.
Answered by Penny Nom.
Two pieces of rope 2016-05-21
From Render:
Fred cuts a 12- inch piece of rope. Then he cuts a second piece of rope that is 3/2 longer than the first piece. Is the second piece shorter or longer than the first piece? Explain.
Answered by Penny Nom.
A trough with a triangular cross section 2016-05-21
From Clarice:
A trough having an equilateral triangle end sections has sides equal to 0.4 m and 7m long.what is the volume of the liquid in the container if the depth of the water is one half the depth of the trough?
Answered by Penny Nom.
Euler's polyhedron formula 2016-05-20
From Antonia:
why does not cylinders follow Euler's polyhedron formula ( V-E+F=2) ?? if a cylinder has 3 faces, 2 edges and 0 vertices it doesn't follow Euler's formula???
Answered by Penny Nom.
The tens digit of a number is five times the unit digit 2016-05-19
From sanjeev:
The tens digit of a number is five times the unit digit. Find the number?
Answered by Penny Nom.
Filling a pit with glass pebbles 2016-05-17
From Sam:
I need to know how many pounds of glass pebbles are needed to fill a 24 inch across circular fire pit, if 5 pounds covers 4"H x 4"W x2"D? Thank u for any assistance, Sam
Answered by Penny Nom.
A puzzle embedded in a table top 2016-05-13
From Aaron:
I want to make a table with a puzzle embedded in it. The table top would be a 36" circle and the puzzle is 20"x27" I'm thinking that it's not going to fit, but not sure. Any help would be appreciated.
Thanks,
Aaron

Answered by Penny Nom.
The volume of a truncated rectangular pyramidal pond 2016-05-13
From Paul:
How do you calculate a partially filled truncated rectangular pyramid if you always know the bottom rectangle, the maximum height top rectangle perimeter, but have a varying height. Similar to filling up a pond you know the current height and dimensions at the max rectangle how do you calculate it half full i.e. 10x30 outside perimeter with a 2x8 base and a max height of 6ft how do you calculate it at 3ft without re-measuring the top perimeter. Thanks, Paul
Answered by Harley Weston.
A pair of equations with fractions 2016-05-11
From tiya:
hello, i want to know how to solve this question.

m/6+2n/3=6
-m/10=2n/5=2

Answered by Penny Nom.
A 70% off sale 2016-05-11
From virginia:
If the sale price is $129.99 @ 70% off - what was the original price?
Answered by Penny Nom.
At what time will Sara catch up with Jake? 2016-05-07
From Laura:
At 11:00 a.m., Jake started driving along a highway at constant speed of 50 miles per hour. A quarter of an hour later, Sara started driving along the same highway in the same direction as Jake at the constant speed of 65 miles per hour. At what time will Sara catch up with Jake?
Answered by Penny Nom.
A rectangular field 2016-05-07
From Jan:
A rectangular field is 63m long and 21m wide. A fence of 2m in height is needed to fence the field and fencing is also required to divide the field in three square sections. There are three wooden gates measuring at 1.5m to the three sections of the field. What is the total surface area of the fence needed for fencing and diving the field?
Answered by Penny Nom.
A 3 hour river cruise 2016-05-06
From Jan:
A 3 hour river cruise goes 15km upstream and then back again.The river has a current speed of 2km/h.what is the boat's speed and how long was the upstream journey?
Answered by Penny Nom.
Converting ml to g 2016-04-28
From Lerato:
I would like to know which formula should I use to convert ml to g?
Answered by Penny Nom.
Solve 2^2x + 3(2^x) - 4 = 0 2016-04-27
From Lloyd:
Solve the equation 2^2x + 3(2^x) - 4 = 0
Answered by Penny Nom.
One million is six percent of what number? 2016-04-26
From Colleen:
One million is six percent of what number?
Thanks!

Answered by Penny Nom.
Dimes and pennies 2016-04-26
From Renae:
Charity has 42 more pennies than dimes. Hope has 7 times as many pennies than dimes. Hope and Charity have the same number of dimes. Together they have $5.46. How many dimes and pennies do each have?
Answered by Penny Nom.
sin 2x - sin x = 0 2016-04-24
From lilly:
sin 2x - sin x = 0 0
Answered by Penny Nom.
A triangle inscribed in a circle 2016-04-22
From Olyana:
I am struggling with this question! Help!

So, there is a circle. In the circle, there is an equilateral triangle inscribed. Each side of the triangle is 20. There is no other info given, other than the triangle is inscribed in the circle and the sides of the triangle are 20. I am supposed to find the radius of the circle! Please help!

Answered by Penny Nom.
600(1+0.05)^7 2016-04-21
From chelsey:
I am unsure on how to solve this problem, 600(1+0.05) 7 ?
Answered by Penny Nom.
A sector of a circle 2016-04-21
From mustafa:
In a sector of a circle, the arc length is equal to half the perimeter of a sector.find the area of a sector in terms of r
Answered by Penny Nom.
15.86 acres 2016-04-21
From Ronald:
If a rectangular piece of land is 1320 ft. on one side, how many feet would be required on the other side to equal 15.86 acres?
Answered by Penny Nom.
A square of side length 4x-3 units 2016-04-20
From Bryan:
Find an expression that represents the area of a square with side length 4x-3
Answered by Penny Nom.
External and interior angles of a regular polygon 2016-04-19
From pearl:
a polygon has n sides.The exterior angle is 8 times the interior angle
find the value of the interior angle
find the value of n

Answered by Penny Nom.
The derivative of x! 2016-04-16
From Sang:
How to find the derivative of x! and integral of x!
Answered by Penny Nom.
Heights at various points along an arc 2016-04-15
From Isiah:
So I am working on a problem with a few friends; you have an arc with the center of its chord at 0,0. We also have a known sagitta and a known radius of curvature. How do we calculate the height extending in both the positive and negative directions?
Central sag: 2.48
Chord length: 9.6

Answered by Penny Nom.
Sloping my yard away from my house 2016-04-15
From Harold:
I need to slope a area of my yard away from my house and garage. The length of my house is 80 feet long and i need to slope it 22 feet away. I want to raise the dirt at the foundation by 6 inches and slope it down to nothing at the width of 22 feet. How much soil would i require?
Answered by Penny Nom.
A scale factor 2016-04-13
From tonya:
What is the scale factor for the two figures regular octagon with 15 and 9 written in decimal form
Answered by Penny Nom.
Mrs. Gibson has 3 children 2016-04-12
From MJ:
Mrs. Gibson has 3 children. The older ones are twins. The youngest one is 3 years younger than the twins. If the sum of their ages is 102, how old is the youngest one?
Answered by Penny Nom.
The area and the perimeter of a rectangle 2016-04-12
From kimberly:
is there a relationship between the are and the perimeter of a rectangle? explain.
Answered by Penny Nom.
The diagonal of a 10 acre square 2016-04-11
From Derrick:
I own 100 acre square of land. I want to run from farthest corner of property to farthest corner, Diagonally. How many miles would I have traveled?
Answered by Penny Nom.
Conversion of an equation 2016-04-11
From Marie:
How to you convert the equation of a line from x-intercept form (y=(x-x1)(x-x2)) to general form (y=ax^2+bx+c) if it involves square roots?

For example:
Convert this equation to general form: y=(x+the square root of 5)^2

Thanks for the help!

Answered by Penny Nom.
A road trip word problem 2016-04-10
From Marc:
While on a road trip I imagined this random word problem. Suppose I start a trip of 75 miles. My initial speed is 75 miles per hour. After every mile traveled I decrease my speed by one mile per hour. After the first mile I decrease my speed to 74 miles per hour and so on for each subsequent mile traveled. How long will it take to complete the 75 mile journey?
Answered by Penny Nom.
The area of a circle and a square 2016-04-10
From Richard:
I cannot understand why if you have a perimeter say of 4 units as the circumference of a circle then you reshape it into a square you lose very approximately some 20% of its volume but the perimeter length stays the same.
Can you please explain this to me as to why the volume decreases.

Answered by Penny Nom.
A stained glass cone lamp 2016-04-09
From Edwin:
In making a 16" dia. cone lamp (stained glass), how many square feet of glass do I need.
Answered by Penny Nom.
Setting up a pool 2016-04-09
From BETH:
i am trying to cover an area for a 15x15 foot round pool with play sand before I set up the pool. My playsand bag is 50 lbs , says it covers .5 cubic feet per bag. How many bags do I need?
Answered by Penny Nom.
Pizza delivery 2016-04-03
From Andrew:
Suppose you own a local pizza restaurant, Pizza with Pizzazz, and in order to compete with the big pizza chains in your area you are considering an advertising campaign offering customers a free pizza if their pizza is not delivered in 30 minutes or less. Even though your pizza restaurant is known for its fast and friendly delivery service, you are not sure if you can afford to give away too many pizzas for free. Looking over your past 12,421 pizza deliveries you find out 97% of the pizzas were delivered in 30 minutes or less.

Calculate the probability that all 100 randomly selected pizza deliveries will have been made in 30 minutes or less? Hint: Use the multiplication rule. (Round 4 decimals)

I'm really stumped on how exactly to do this problem. I don't need the answer just the steps broken down to show me how to get it! It would be gladly appreciated if you could help!

Answered by Penny Nom.
Maximizing the area of a two lot region 2016-04-03
From yousef:
A man wishes to enclose two separate lots with 300m of fencing. One lot is a square and the other a rectangle whose length is twice its width. Find the dimensions of each lot if the total area is to be a minimum.
Answered by Penny Nom.
The last two digits of a phone number 2016-04-03
From Joshua:
I want to find the last two digits of a phone number whose first eight numbers of a ten digit phone numbers are known.
Answered by Penny Nom.
The volume of a cone 2016-03-31
From Odum:
Find the volume of a cone with radius 6.5cm and height 12.6cm
Answered by Penny Nom.
When does sqrt(x^2+y^2)=x+y? 2016-03-30
From Margret:
when does sqrt(x^2+y^2)=x+y
Answered by Penny Nom.
A tilted cylinder tank 2016-03-28
From Astri:
I have a problem determining the fluid level for tilted cylinder tank. Actually I've found some solution here for horizontal cylinder tank. What if the cylinder is tilted at a certain degree, for example 30 degree from the vertical axis.
Thank you.

Answered by Penny Nom.
The remainder theorem 2016-03-27
From Pratyasha:
A quadratic polynomial when divide by x+2 leaves a remainder of 1 and when divided by x-1 leaves a remainder of 4. What will be the remainder if it is divided by (x+2)(x-1)?
Answered by Penny Nom.
The volume of a box with a hexagonal base 2016-03-27
From Frank:
how many gallons of water in an 8" hexagon shaped box. 6" sides x 12.5" high ? The walls are 1" thick , beveled @ 30 degrees, so inside dimensions are 6"
Answered by Penny Nom.
A square based box and a cylinder 2016-03-26
From rajpal:
i m trying to calculate an area required to hold 4.4 mln cubic metres of volume. I used square and I get below result

A box shape that has dimensions 1500 meters (1.5km) by 1500 meters (1.5km) by 2 meters depth has a volume of:
= 4,500,000 cubic meters (4.5 mln cubic meters)

but when I use a cylinder shape, i get below result

A cylindrical shape that has a radius of 850 meters and a depth of 2 meters has a volume of:
= 4,539,600 cubic meters (4.54 mln cubic meters approx)

why is it that the cylinder, though of same depth holds more volume than a square? kindly clarify.

Answered by Penny Nom.
An isosceles triangle inscribed in a circle 2016-03-25
From NIHAL:
A isosceles triangle is inscribed in a circle having sides 20cm,20cm,30cm. find the radius of circle
Answered by Penny Nom.
Polynomial division 2016-03-25
From Ashley:
(4x-4+6x^3-x^2)/(3x-2)

The instructions say to "perform the division".

Answered by Penny Nom.
Travelling in a spacecraft 2016-03-25
From julie:
if i was travelling in a spacecraft at the speed of light and i turned on my headlight would i see the beam ?
Answered by Robert Dawson.
A rotating schedule for a softball team 2016-03-25
From Caroline:
I couldn't find a similar question, hopefully there isn't one!

I have a softball team of 18 people (7 girls and 11 boys), but am limiting each game to 13 players (6 girls and 7 boys). The season is 10 games long. How do I create a rotating schedule which allows members to play evenly? I tried separating the boys and girls to create rotation for each but I got confused. Some are unable to attend all games but for the simplicity of a rotation schedule can you please help?

Answered by Robert Dawson and Victoria West.
Biking and jogging 2016-03-17
From alondra:
Ricky went on a bike ride. After 6 miles he got a flat tire and had to jog home. He jogs 3mph slower than he bikes, so the jog took 1 hour longer than the bike ride. At what rate did he travel each way?
Answered by Penny Nom.
The perimeter of a rectangle 2016-03-17
From Linda:
Noel correctly adds the length of three sides of a rectangle and gets 88 cm her brother Ryan correctly adds the lengths of three sides of the same rectangle and gets 80cm. What is the number of cm in the perimeter of the rectangle
Answered by Penny Nom.
A block wall 2016-03-15
From Robert:
I have a wall 40 ft. long and 4 ft. tall as well as another one that is 30 ft by 3.5 ft. tall that I am trying to build. how many 6 inch by 12 inch landscaping blocks do I need?
Answered by Penny Nom.
The sides of a triangle 2016-03-15
From Hitarth:
Why we cannot construct a triangle ofngiven sides 5cm,5cm and 10cm?
Answered by Penny Nom.
Angles 2016-03-12
From Laurynn:
What are angles in general (please include the 'angle of incline')

Thank you
Laurynn

Answered by Penny Nom.
The integral of a sum 2016-03-10
From Rahul:
How to solve definite integral of a sum. The specific problem is as follows,
Integral of ( 1+ sum of x^k, k=1 to k=n), x=0 to x=b *dx.
The answer is b + sum of b^(k)/k, k=2, to k=n+1. I understand only the integral of first term. But integral of the sum I do not understand at all.

Answered by Penny Nom.
A circle and a chord 2016-03-10
From Mujtaba:
The distance of a chord of length 16 cm, from the centre of the circle of diameter 20 cm
Answered by Penny Nom.
A word problem 2016-03-10
From Sam:
Jo is 10 cm taller than Ruth. Pam is 16cm taller than Ruth .Rod is 8cm taller than Pam. How much taller than Jo is Rod?
Answered by Penny Nom.
Can you construct a regular 21-gon? 2016-03-08
From Peter:
hi, i know that if i use a compass to draw a circle, i can use the same setting to mark 6 intervals along the circle circumference, how would i go about marking out 21 equally spaced points around the circumference?

if there an online calculator i can us, or some other interesting trick?

tia

peter

Answered by Robert Dawson.
Tiling a floor 2016-03-05
From joanne:
How many floor tiles 20x20 inch do I need for area of 8x 12 ft.?
Answered by Penny Nom.
The area of a rectangle 2016-03-03
From Lucas:
The area of a rectangle is given by A=x2+18x+72
-use factoring to find an expression for the dimensions of the rectangle.
-if the area of the rectangle is 7 square feet, what are the possible values of x?
-what are the dimensions of a rectangle?

Answered by Penny Nom.
Corrosion inhibitor in a pipeline 2016-03-02
From Edu:
I have a 12 in pipeline and 5 miles long. I need to coat it with corrosion inhibitor how much chemical will I need
Answered by Penny Nom.
Compressed Earth blocks 2016-03-02
From John:
how many 14"x10"x4" blocks do I need for a 1300sq foot house 8 foot high
Answered by Penny Nom.
A percentage problem 2016-03-02
From jehanzaib:
if x is a percentage: e.g. 40% than what does "x/(1+x)" = 29%. what is the explanation.
Answered by Penny Nom.
A rectangular prism 2016-03-02
From Paula:
Trying to help with a math problem. I don't know how to set up the equation for this.

A rectangular prism has a volume of 7.875m3rd and a height of 3.5 m. What is the area of the base of the prism?

Thanks for your help!

Answered by Penny Nom.
An algebra problem with a sphere 2016-02-29
From Sania:
If the number of square centimeters on the surface area of a sphere is equal to the number of cubic centimeters in its volume find its diameter
Answered by Penny Nom.
A deck that is half an ellipse 2016-02-28
From Steve:
On your website, I was reading a question and your response from a girl named Angela in which you provided a formula by which her father, a welder, could figure out points on an arc corresponding to equal 3' intervals on a 30' chord where the vertex was 1' off the chord. Is there an equivalent formula when working with an ellipse? I suspect this change will make the calculations significantly more complex. I am building a deck that is half an oval, and would like to be able to mark out the perimeter by measuring the distance from regular intervals on the primary access to a corresponding point on the perimeter. I will then connect the points on the perimeter and cut a reasonably smooth arc. The length of the primary access will be 22' and width of the deck at the vertex is 9'. I would like to be able to know the distance from the primary axis to a point on the perimeter at equal intervals of 6" along the primary axis. Can you help?
Answered by Penny Nom.
A 13 game bet 2016-02-28
From farah:
I'm trying to calculate the odds of a 13 game bet. Each game has 2 teams with 3 possible outcomes (win,draw,lose).

What are the odds.

Answered by Penny Nom.
A word problem 2016-02-27
From KRK:
amrutha thinks of a number, multiplies it by 5 & 6, devides the result by 3 and takes away the number first thought of to get an answer 36. find out the number
Answered by Penny Nom.
Solve for x and y 2016-02-27
From ntshidi:
Y=1/2x+4and1/4x-6
Answered by Penny Nom.
A place value drawing 2016-02-24
From Kristin:
Make a place value drawing of a number that has double the number of tens as ones and three times the number of thousands as hundreds write the number.
Answered by Penny Nom.
The sum of the angles of a triangle 2016-02-24
From Sophia:
Does every triangle add up to 180 degrees? (Such as a unique triangle)
Answered by Penny Nom.
The evaluation of a 3 by 3 determinant 2016-02-19
From Kristen:
What is the step-by-step process on how to evaluate the determinant of a 3*3 matrix, using the expansion method (not the diagonal method)
Answered by Penny Nom.
The interior and external angles of a regular polygon 2016-02-17
From percy:
a regular polygon has n sides .The size of each interior angle is eight times the size of each exterior angle .
1.find the size of each exterior angle
2.calculate the value of n

Answered by Penny Nom.
Shooting a ball at a target 2016-02-16
From Thys:
Hi
I have a problem with the formula that i use .(for programming)
I have looked all over the web to find a solution but no luck.
I have a cannon that shoots a ball at a target
I use this formula to calculate what my initial velocity must be to hit the target at a angle of 30 degrees and a distance of 15m (the cannon and target position is known) It works perfectly if both is at same height but if one is higher or lower it miss.

In an example I am working with the range is 30m, the angle is 45 degrees and the target is 10m higher than my position.
Please help
Formula = V0 = √RG / Sin(2α)

Answered by Harley Weston.
Speed and velocity 2016-02-14
From kechi:
A boat sails south through Chesapeake Bay. It takes 10 hours to sail 110 kilometers. Describe the boat's motion. Calculate the boat's speed and velocity, showing the calculations. Explain how the boat's speed is different from its velocity
Answered by Penny Nom.
Perpendicular lines 2016-02-14
From Brendon:
points A (-4; 6) , B (1;-2), C (5;-3) and D (4; 1) are given prove that AC and BD are perpendicular to each other
Answered by Penny Nom.
Calendar arithmetic 2016-02-14
From Jenalee:
January 1, 2001 is Julian Day 2 451 911 (the number of days that have passed since Day 0, January 1, 4713 BC).

If Julian Day 0 was a Monday, what day of the week was January 1, 2001?

Answered by Victoria West.
Two equations in two unknowns 2016-02-13
From Anumba:
4x + 2y = 4
7x - y = -11

Answered by Penny Nom.
Todd is 3 years older than his brother Jack 2016-02-13
From Victoria:
Todd is 3 years older than his brother Jack. If Jack is x years old and Todd is y years old, write a rule that relates their ages over time. When Jack is 28 years old, how old will Todd be?
Answered by Penny Nom.
Travelling with and against he current 2016-02-09
From Lloyd:
A boat travelled 210 miles downstream and back. The trip downstream took 10hrs, the trip back took 70hrs
1. what is the speed of the current?
2. what is the speed of the boat in still water?

Answered by Penny Nom.
A 2 digit number 2016-02-08
From Lloyd:
The sum of digits of a certain 2 digit number is 7. Reversing its digits increases the number by 9. Find the number
Answered by Penny Nom.
Backfilling a rectangular hole for a shop foundation 2016-02-08
From Jeremy:
Hello, I'm going to be backfilling a rectangular hole for a shop foundation and need to know how much concrete and gravel I'm going to need to fill it in. The shop foundation is 38 feet wide by 30 feet long and 4 feet deep, there is also a concrete perimeter going inside the foundation that measures 16 inches wide and goes all the way around the inside of the foundation
Answered by Penny Nom.
The equation of a vertical line 2016-02-07
From Kagiso:
The equation for a vertical line that goes through the point (7; -3) is?
Answered by Penny Nom.
38 is what percent of 400? 2016-02-07
From chris:
38 is what percent of 400?
Answered by Penny Nom.
An exercise with factorials 2016-02-07
From vidhi:
find the value of n: (1-1/2) (2-2/3) (3-3/4)...(15-15/16) = n!/16
Answered by Penny Nom.
The midpoints of the sides of a quadrilateral 2016-02-05
From Andrea:
The segments, joining, in order the midpoints of consecutive sides of a quadrilateral form a parallelogram.
Answered by Penny Nom.
When will Scott and Jill have earned the same amount? 2016-02-04
From kalia:
Scott starts with $10 and earns $5 and hour. Jill starts with $40 and earns $3 and hour. when will the two have the same amount of money in the same hours?
Answered by Penny Nom.
The height of a triangle 2016-02-03
From Brooklyn:
How do I figure out the height of a triangle when it only tells me the base of he triangle? And vice versa.
Answered by Penny Nom.
Golf for 6 - 2 three balls 2016-02-01
From Andy:
We are shortly having a golf holiday in Phoenix. There are 6 people playing 6 rounds, so we will play in 2 three balls. Is there a combination of pairings that ensures everyone plays with each other the same number of times. Many thanks for your help ! Andy
Answered by Victoria West.
Which term of the series 2+7+12+...is 152? 2016-02-01
From francis:
whice term of the series 2+7+12+...is 152?
Answered by Penny Nom.
Simplify 2016-02-01
From Sabira:
Simplify step by step
5x/8 + 7y/18 = 6

Answered by Penny Nom.
A quadrilateral inside a square 2016-01-30
From Subrahmanya:
In a square ABCD of side 6 units P, Q are mid points of BC, CD respectively. The line segments BQ, DP intersect in R then find the area of the quadrilateral ABRD using only plane geometry.
Answered by Chris Fisher.
Consecutive angles of a parallelogram 2016-01-28
From Hanna:
The consecutive angles of a parallelogram measures
Answered by Penny Nom.
Two increases of 20% 2016-01-27
From Z:
How do I find how many books are in a library given a problem like this... The number of books increase after buying 20% more in May. The number increased again by 20% after they were given 300 books. How many books did it start with?.
Answered by Penny Nom.
Two increases of 20% 2016-01-27
From Z:
How do I find how many books are in a library given a problem like this... The number of books increase after buying 20% more in May. The number increased again by 20% after they were given 300 books. How many books did it start with?.
Answered by Penny Nom.
Exponential form 2016-01-26
From Jufe:
What is the exponential form of ³√11x²
Answered by Penny Nom.
A chord of a circle, the central angle and the radius 2016-01-26
From Nishan:
If chord length is given along with angle then how to calculate the radius.
Answered by Penny Nom.
Exponential growth 2016-01-26
From Paul:
The value of a stamp was 0.03 in 1851. The value in 2011 is $50,000. If the value increased exponentially, what is the annual growth rate?
Answered by Penny Nom.
I was paid 67% of the total owed 2016-01-25
From Julie:
I was paid $27000. This is 67% of the Total Owed. The Total Owed is unknown. How do I calculate the amount of the Total Owed?
Answered by Penny Nom.
A system of linear equations 2016-01-24
From kareem:
my name is kareem
and i am a student i have a math puzzle and i tried to solved it but it always have same mistake
x-y=9
x+z=12
z-n=14
y+n=2

Answered by Penny Nom.
A quadratic equation with equal roots 2016-01-23
From abdiwahid:
X^2+2ax+4x+9a=0 is quadratic equation which has equal roots. Find the possible value of a?
Answered by Penny Nom.
The length of an 80 acre piece of land 2016-01-21
From Eugene:
Have to clear a piece of land 80 acres by 500 feet wide. What is the length of the property?
Thanks Gene

Answered by Penny Nom.
Find two values of x satisfying f(x)=1 where f(x)=sin(ln(x)) 2016-01-21
From Ruth:
I'm trying to find two values of x satisfying f(x)=1 where f(x)=sin(ln(x))
Answered by Penny Nom.
The volume of a silo 2016-01-18
From Clinton:
Can you show me how to calculate the ground corn capacity in a steel silos of 10' x 10' high and 10' x 20' high and 10' x 24' high and 10' x 28' high
Answered by Penny Nom.
A Max/Min problem with an unknown constant 2016-01-17
From Guido:
Question:

The deflection D of a particular beam of length L is

D = 2x^4 - 5Lx^3 + 3L^2x^2

where x is the distance from one end of the beam. Find the value of x that yields the maximum deflection.

Answered by Penny Nom.
A limit of a rational function 2016-01-16
From selvamani:
F (x) = x^3+3x^2-9x-2 / x^3-x-6 and limit x tends to 2, f (x) exist then limit x tends to 2, f (x) is equal to ? How to answer this problem.
Answered by Penny Nom.
The perimeter of a one quarter acre square 2016-01-15
From Kerry:
If you have a perfect square that is one quarter of an acre what are the length of the sides and how do you arrive at the solution? thank you, Kerry.
Answered by Penny Nom.
Complex numbers in standard form 2016-01-15
From Michael:
express the following complex numbers in standard form (2+3i)+(5-2i)
Answered by Penny Nom.
Solve 1/3 ( x-5 ) + 4 =1/6 ( 2 - 3 x ) +1 for x 2016-01-15
From edwin:
1/3 ( x-5 ) + 4 =1/6 ( 2 - 3 x ) +1
Answered by Penny Nom.
A rectangular enclosure 2016-01-15
From Amber:
A rancher wants to use 300ft of fencing to enclose a rectangular area of 4400 square feet. What dimensions should the rectangle be?
Answered by Penny Nom.
Countable and uncountable sets? 2016-01-15
From wilson:
what are the countable and uncountable sets?
Answered by Penny Nom.
A triangular shaped garden 2016-01-13
From Demetrie:
Dwayne's garden is triangle-shaped with two equal sides and a third side that is 4 ft more than the length of an equal side. If the perimeter is 49 ft, how long is the longest side?
Answered by Penny Nom.
Sue's favourite number 2016-01-12
From Maria:
This is my son's homework. He is 7. Can you help?

Sue has a favourite number.
It is an even number.
It has 3 digits.
The digits add up to 15.

Find 10 numbers that could be Sue's number?

Answered by favourite number.
The perimeter of a triangular plot of land 2016-01-11
From Janelle:
I need to fence off 6.5 acres. The plot is triangular shape. How many feet would I be fencing off?
Answered by Penny Nom.
Water flowing through a rectangular drain 2016-01-08
From Tiffany:
the cross section of drain is a rectangle 30 cm wide. If water 3.5 cm deep flows through the drain at a rate of 22 cm/s, how many litres of water will flow through in one minute?
Answered by Penny Nom.
The area of a rectangle 2016-01-08
From ranjan:
if we make a rectangle with a 100cm of wire, then its area will be?
Answered by Penny Nom.
Integration of dx/(x^2+1)^3 2016-01-07
From Ishank:
Integration of dx/(x^2+1)^3
Answered by Penny Nom.
A pipe is installed with a 2% grade 2016-01-07
From peter:
a sewer ditches to be 148 feet long. the pipe is installed with a 2% grade. what would be total amount of fall be?
Answered by Penny Nom.
Three consecutive integers 2016-01-07
From ronn:
Three consecutive integers whose sum is 186
Answered by Penny Nom.
Two transport companies 2016-01-06
From Brynleigh:
The Sugar Sweet Company will choose from two companies to transport its sugar to market. The first company charges $5096 to rent trucks plus an additional fee of $200.75 for each ton of sugar. The second company charges $5500 to rent trucks plus an additional fee of $175.50 for each ton of sugar. For what amount of sugar do the two companies charge the same? What is the cost when the two companies charge the same?
Answered by Penny Nom.
A ratio word problem 2016-01-06
From Nina:
The length and the width of the rectangle is in the ration 5:3. Its area is 375m2. Find its perimeter.
Answered by Penny Nom.
A graph of the water level of a river 2016-01-05
From emoni:
Suppose that the water level of a river is 34 feet and that it is receding at a rate of 0.5 foot per day.
Answered by Penny Nom.
Four consecutive integers 2016-01-05
From sherica:
my teacher gave us an assignment and it says here that; THERE ARE FOUR CONSECUTIVE INTEGERS.IF THE SMALLEST ONE IS x ,FIND THE SUM OF THE FOUR INTEGERS. that is the question ...thanks hope that u can answer it
Answered by Penny Nom.
The height of a parabolic arc 2015-12-30
From Tom:
Is there an algebraic means to determine the highest point of a parabolic arc if the base and perimeter are known?
Answered by Penny Nom.
Book sales 2015-12-30
From Sandra:
Your finance text book sold 56,500 copies in its first year. The publishing company expects the sales to grow at a rate of 20.0 percent for the next three years, and by 8.0 percent in the fourth year. Calculate the total number of copies that the publisher expects to sell in year 3 and 4.
Answered by Penny Nom.
How can he raise his average to 90? 2015-12-29
From Raj:
Harris has an 87 in a class where daily works count 20%, quizzes count 20% and tests count 60%.
If there are only two test scores what must the score on his second test to raise his average to 90?

Answered by Penny Nom.
A relative maximum and a relative minimum 2015-12-28
From kemelo:
show for the following function f(x)=x+1/x has its min value greater than its max value
Answered by Penny Nom.
A circle is inscribed in a hexagon 2015-12-28
From Lalitesh:
A circle is inscribed in a regular hexagon ABCDEF
Prove that AB+CD+EF=BC+DE+FA

Answered by Penny Nom.
A troublesome word problem 2015-12-28
From Linda:
James is now 3/5 John's age. Three years ago, he was 1/2 John's age. Three years from now, he will be 2/3 John's age.
How old are they now?

Answered by Penny Nom.
Constructing simultaneous linear equations 2015-12-28
From Deborah:
Linda thinks of a two-digit number. The sum of the digits is 8. If she reverses the digits, the new number is 36 greater than her original number. What was Linda's original number?

Thank you!

Answered by Penny Nom.
A diameter of a circle 2015-12-27
From sahil:
Find the equation of diameter of circle x2 +y2-6x +2y-8 which passes through the origin.
Answered by Penny Nom.
The angles of a triangle 2015-12-17
From Faith:
Does the measure of angle determine the length of its side? For example two angles are congruent then the sides are also congruent because from my understanding the angle determine the shape of triangle.
Answered by Penny Nom.
A bus trip 2015-12-15
From Michelle:
I do not know what equation(s) should be used in order to obtain the answer(s) to this word problem.

A bus was rented for a trip, the cost was dived equally amongst the passengers. During the trip, someone mentioned that if there were 9 more passengers, they would have paid each 5$ less. Another mentioned that if there were 6 less passengers, they would have each paid 5$ more. What is the cost of renting the bus and how much must each passenger pay.

Answered by Penny Nom.
Proof that an erroneous algebraic statement is false 2015-12-14
From Berteanu:
I need help with this proposition:
"It exists x a real number that for every y real number 5*x-2*y*y=1
This is false.
Let x be from R.
And I need an y real number that 5*x-2*y*y!=1
Please,could you help me?

Answered by Penny Nom.
Sierpinski Pyramid 2015-12-14
From victor:
Is a tetrix (a Sirpienski Pyramid/Tetrahedron) its own hyperplane?...alternatively, is it a hyperplane?
Answered by Robert Dawson.
Linear equations in two variables 2015-12-13
From priya:
I have problem in solving these equations please help me today itself very urgent:
I)2x+y=y
II)pie*x+y=9

Answered by Penny Nom.
The dimensions of a rectangle given its area and aspect ratio 2015-12-13
From colum:
Can you work out the length and width of a rectangle where you know only the area and aspect ratio
Answered by Penny Nom.
Constructing a cross country district schedule 2015-12-13
From David:
Retired math teacher trying to help son-in-law schedule a cross country district schedule. The would like to have four tri-meets each week for five weeks and one week of dual meets. The score the tri-meets as three duals, i.e. 1v2, 2v3, 1v3 constitutes one tri-meet. After many hours, not sure its possible, but I have forgotten a great deal! Thanks
Answered by Victoria West.
The line of intersection of two planes 2015-12-09
From Rabz:
Determine the vector equation for the intersection between the two given planes 2x+7y equal to 14,x+z equal to 7
Answered by Penny Nom.
The original price before a discount and taxes 2015-12-09
From Michelle:
Good day,

I would like to know if there is a particular equation to validate the following information:

I had to find the original cost of an item sold for 759.36$ before a 25% discount and 13% taxes. I know we must use separate question to obtain the answer but I am unsure how to write the equation to validate my answer.

Answered by Penny Nom.
20 golfers playing a 20 week season 2015-12-07
From Jeff:
We have 20 golfers playing a 20 week season. We play in foursomes. How best to set foursomes so everyone can play with every without playing with the same person multiple times?
Answered by Victoria West.
Travelling in opposite directions 2015-12-06
From Roselyn:
Jenny and Mark commute to work, travelling in opposite directions. Jenny leaves the house at 8 AM, and averages 35 kph. Mark leaves at 8:15 AM, and averages 40 kph.At what time will they be 40 kilometers apart?
Answered by Penny Nom.
Compacted and uncompacted soil 2015-12-05
From J.H.:
I am negotiating with a company to sell them land that they would use for excavating soil to be used in construction. I would like to value the land based on the volume of soil they are likely to remove. I have seen several answers, giving the calculation based on 43,650 sq. ft./acre x number of acres x number of feet in depth, divided by 27 to convert the number to cu. yds. However, that is the number when compacted. They pay $.50/yd. of soil removed, but that yd. is not compacted. How do you calculate the number of yds. of uncompacted soil in 1 acre of land?
Answered by Harley Weston.
A word problem with percentages 2015-12-05
From Caedryn:
After giving 25% of her stickers away, May had 50% as many stickers as Rose. If they had 200 stickers altogether at first, how many stickers did Rose have?
Answered by Penny Nom.
A schedule for a league with 15 teams 2015-12-05
From Zac:
Hello,

I have a league schedule question. We have a league where we have say have 15 teams. Each team is to play each other twice during a 10 week session.
However each team tries only to play 3 matches(teams) per night(week).
How would we set up a random schedule for the 10 week session?

Answered by Victoria West.
Drawing an arc 2015-12-04
From hassan:
how to draw a curve long 1m with an angle of 22.5?
Answered by Penny Nom.
A tangent line to a parabola 2015-12-02
From pei:
Given that the line y=mx-5 is a tangent to the curve y=2x^2+3 find the positive value of M.
Answered by Penny Nom.
A parabolic arch 2015-11-30
From Muhammad:
An arch over a road has a parabolic shape it is 6 meter wide at the base and is just tall enough to allow a truck 5 meter high and 4 meter wide to pass
a): assuming that the arch has an equation of the form y=a(x)^2+b use the given information to find a & b. explain why this assumption is reasonable.
b): sketch the graph of arch equation

Answered by Penny Nom.
The sweep angle of a video camera 2015-11-25
From Franc:
a television camera is 30ft from the sideline of a basketball court 94 ft long. The camera is located 7 ft from the mid court. through what angle must it sweep in order to cover all action on the court?
Answered by Penny Nom.
The angle at the vertex of an isosceles triangle 2015-11-25
From Karan:
I have been given an isosceles triangle. the Top angle is what i have to find out and the two sides adjacent to it are both 4.9 cm. i have been told that the area of the triangle is 4 cm^2. i have no idea how to work this out, any ideas?
Answered by Penny Nom.
The measure of an angle in terms of its complement 2015-11-22
From Pam:
Can you please help me so I can help my daughter the equation is the measure of angle v is 4 time the measure of its complement what is the measure of angle v when the equation is 4x+x=90
Answered by Penny Nom.
20 million as a percentage of 7.3 billion 2015-11-18
From Nick:
Based on the estimate of the worlds current population (7.3 billion people), I'm trying to work out what percentage of that figure 20 million people would amount to?

(Apologies in advance that I've no idea if the 7.3 billion figure is an American Billion or UK billion)

Thanks for any help.

Nick.

Answered by Penny Nom.
Archimedes, Euclid and "Circular Reasoning" 2015-11-15
From Ron:
I have read about Archimedes and his work with sphere in cylinder and cone in cylinder and the volume relationships. Did he or any others also extend this to regular based polygon based regular like pillars, and columns? The ratio of 1/3 to 1 whole holds true with all regular based columns as example: a regular pyramid having a regular hexagon base inside a regular hexagon column of equal height.
Answered by Chris Fisher.
A common tangent to two general parabolas 2015-11-15
From Kind:
Hi, I want to find the common tangent of two general parabolas, but i don't know if it's possible or not.
If it's possible, please make a tutorial.

The first parabola equation : Ax^2 + Bxy + Cy^2 + Dx + Ey + F = 0.
The second parabola equation : Gx^2 + Hxy + Iy^2 + Jx + Ky + L = 0.

I need this because i want to find the equation of Beloch fold. (Huzita - Hatori 6th axiom) However if you know any other method to find Beloch folds equation, I am open for any suggestions.

Answered by Chris Fisher.
A schedule for 12 high school golf teams 2015-11-15
From Jason:
I have 12 high school golf teams (A - L). There will be 4 games each week for 4 weeks. For each game, 3 teams play at a time (A vs B vs C). I would like it where no team plays the same team twice. Week four is my issue as I seem to always have a team or two playing the same team again. Is it possible to have each team play each other only once?
Answered by Victoria West.
Dealing with surds 2015-11-14
From Agnes:
simplify (1-√3)(1÷3+√3)
Answered by Penny Nom.
The diagonal of a rhombus 2015-11-14
From Om:
In a rhombus ABCD, angle A=60° and side AB=6 cm. Then diagonal BD is ?
Answered by Penny Nom.
The area of a domed roof 2015-11-13
From Brandon:
I have a tank with a 13' radius that has a domed roof of 3.5' tall How do I figure out the area in SQFT?
Answered by Penny Nom.
A volume in cubic yards 2015-11-12
From James:
The first formula for figuring cubic yards of concrete need is length in feet, times width in feet, times depth in actual inches, divided by 320. Such as, 150x 50 feet x 4inches divided by 320 = cubic yards. How and why does this formula work?
Answered by Penny Nom.
A concrete container 2015-11-12
From Karstin:
The exterior of a concrete container will be 10 feet by 8 feet by 4 feet tall. The walls and the bottom are 6 inches thick. What will it cost to construct it if concrete is $98.95/cubic yard?
Answered by Penny Nom.
Fencing around a rectangular field 2015-11-11
From Darlene:
Question from Darlene, a parent: A farmer has 10,000 meters of fencing to use to create a rectangular field. He plans on using some of the fencing to divide the rectangular field into two plots of land by constructing a fence inside the rectangle that is parallel to one of the sides. Let X be the width of the rectangular field. Write an equation to express the area of the field as a function of X. Find the value of X that maximizes the area of the field.
Answered by Penny Nom.
Roof Square footage 2015-11-11
From Todd:

Question from Todd:

Good Day.
I have to figure out the square footage of a quonset style roof that's not playing by the rules The building dimensions (rectangular) are 63'x135' the height of the roof is 9.25'. It not an entire Quonset, It's that style of roof,(curved). There are concrete block walls 10' up to the metal roof.

Thank you!

Todd


Answered by Harley Weston.
The combination to a locker 2015-11-11
From Jason:
Hi,
I can't seem to open my locker cause I forgot the combination, the numbers are 1-8 Without repeat for 4 number for example 1234 or 2345 for numbers 1-8 without any repeat.

Answered by Harley Weston.
The derivative of x^1/3 2015-11-08
From omar:
hi can help me
am teacher ask me about x^1/3 Derivation definition .

Answered by Penny Nom.
Is 22/7 bigger or smaller than 3.14? 2015-11-06
From Natalie:
Is 22/7 bigger or smaller than 3.14?
Answered by Harley Weston.
A schedule for 8 golfers over 5 days 2015-11-04
From Ged:
Hope you can help with the following to save next years arguments ! Schedule for 8 golfers playing as pairs in 2 X 4 balls on 5 days. No player to pair same player more than once and to have played with each player In the 4 ball. Thanks.
Answered by Victoria West.
Factoring a trinomial 2015-11-02
From Eric:
2v(squared) +11v +5
Answered by Penny Nom.
Doubling the area of a pamphlet 2015-10-31
From Mariam:
A pamphlet has the following dimensions: 4 cm by 6 cm If the area of the pamphlet is doubled, what will be the length of each side if sides can be integer values only?
Answered by Penny Nom.
A 15 team tournament 2015-10-28
From Joseph:
We have a 15 team tournament. Each team plays all other teams. We need a schedule were each game has a work team. Thus at each fields we have two teams playing and one working. All teams need to play each other and also be a work team. Any help would be appreciated!!
Answered by Victoria West.
An 8 Team Basketball League Schedule 2015-10-28
From Janice:
We have an eight team league that play at four time slots each week. It's a seven week season. How do we schedule so that each teams plays each other at least once?
Answered by Victoria West.
The uniqueness of the lub 2015-10-25
From Aishwarya:
Prove that if the lub and glb of a set exist then they are unique
Answered by Penny Nom.
An example of an irrational number 2015-10-22
From Allison:
It ask me to find an example of an irrational number less than -5 and I don't understand what the difference from a rational number and an irrational number besides the fact that a rational number can be repeated and shown in a simple fraction and an irrational number can't be written in a simple fraction.
Can you help me?

Answered by Penny Nom.
The sum of two repeating decimals 2015-10-22
From Kaitlin:
Here is the question I am struggling with:

Is the sum of two repeating decimals always repeating decimal? Explain your answer and give an example.

I answered this question thinking that no you cannot but only when adding 0.9 repeating, but my professor said this was incorrect.

Thanks for the help!
-Kaitlin

Answered by Penny Nom.
A schedule for a pool league 2015-10-21
From Hazel:
I have to make a schedule for my pool league. I have twenty teams each of which must play all other teams. I have ten tables which must also be rotated to all teams play on all tables.
Answered by Victoria West.
Simplify -3(x-7) 2015-10-21
From geraldine:
simplify: -3(x-7)
Answered by Penny Nom.
Scheduling a social curling league 2015-10-20
From Tyler:
I'm scheduling a social curling league where all skips will play with all thirds, all seconds, all leads. And all thirds will do the same and so on. We have 6 teams of 4 and will be doing 6 rotations. Is it possible that all skips will play with all other players from other positions without anyone doubling up?
My initial thoughts were (1,1,1,1) (2,2,2,2) (3,3,3,3) (4,4,4,4)(5,5,5,5)(6,6,6,6)... then rotate the thirds up and seconds down but don't know what to do with the leads and even with just 3 positions there's doubling (1,2,6,x)(2,3,5,x)(3,4,4,x)...4s have doubled (4,5,3,x)(5,6,2,x)(6,1,6,x) as you can see I'm having problems
If you could let me know if this is even possible it would be greatly appreciated

Answered by Victoria West.
6 teams randomly arranged in groups of 3 2015-10-19
From Anthony:
im trying to make a game scheduled for 6 teams using a 1vs2vs3 ; 4vs5vs6 format. all 6 teams randomly arranged in groups of 3. each team will play all teams at least twice, im trying to see how to make the number of times each team play another the same. schedule should be arranged as such 1vs2vs3 4vs5vs6 so that (1vs2, 1vs3, 2vs3) and (4vs5, 4vs6, 5vs6) i am limited to having only 2 sets of 3 at a time with 6 teams.
Answered by Victoria West.
An 8 team schedule 2015-10-19
From Dean:
I need an 8 team schedule for 2 time slots where 3 teams are on the ice per slot. I understand that there will be 2 teams on a bye per day. How do make sure that the byes are fair? Can you help me out?
Answered by Victoria West.
A six team league to play a 20 game schedule 2015-10-19
From Kelly:
Hi there. Some of your posts are close, but my problem is evening out the home and away games. We have 6 teams in our league and we are going to play a 20 game schedule, playing everyone 4 times. We will play Fridays and Saturdays, each weekend. Of course the ideal would be to have each team have a home and an away game each weekend, but I realize that is impossible, but I would like it as close as possible. Any help would be much appreciated!! We also were considering opening with a home and away - Friday then Saturday with the same teams. Then carrying on with playing everyone else and then possibly finishing up with the same home and home
Answered by Victoria West.
Acres of cattle waste 2015-10-19
From Chris:
In my first math problem, I found out that 100 cows will produce 21900 ft3 of waste over a year. However I don't know how to do the second part. How many acres of land would a farmer need to spread the waste on if he wanted it to be more that 6 inches deep? Thank you.
Answered by Penny Nom.
Six teams playing five games 2015-10-18
From Aidan:
I have six teams that will be playing five games.
I want each team to play each game once and each team to play each other team once.
Is this possible or should I add an extra round and let some teams play each other twice?

Answered by Victoria West.
Exponential form 2015-10-17
From Will:
What is exponential form of -8?
Answered by Penny Nom.
4 games in 4 time slots with 8 teams 2015-10-16
From Paul:
I'm trying to set up a game matrix for my kids that will have 4 games in 4 time slots with 8 teams. And every team is playing a different game in a different time slot. I provided the initial matrix and I am trying to ensure that no 2 teams play each other more than once. That's my problem.
       8 pm 9 pm 10 pm 11 pm
Crokinole 1 vs 2
Trivial Pursuit 3 vs 4
Darts 5 vs 6
Pictionary 7 vs 8
Please help, I've been at it for hours.

Answered by Robert Dawson and Victoria West.
11 golfers over five rounds 2015-10-15
From Leo:
11 golfers over five rounds. Will golf as 4-4-3. How to set it up so everyone plays at least once with each player.
Answered by Victoria West.
We are six golfers going away for 3 days of golf 2015-10-13
From Mike:
We are six golfers going away for 3 days golf. We are playing in 2 no threeballs playing concurrently. How can we arrange things so that everyone plays averydody else at least once. Is this possible.?
Answered by Victoria West.
The flight of a ping pong ball 2015-10-13
From Abigail:
Hello,

Wondering if there is a way to figure out the quadratic equation of half of a parabola?
Doing an assignment about finding quadratic equations for separate bounces of a ping pong ball, but the first bounce is incomplete (as the ball has presumably been dropped, hit the ground and then went on to make a complete bounce).
Please see attachment for references to the diagram that I am using.

Not sure whether I would have to look at it from a different perspective.

Thanks,

Abi

Answered by Harley Weston.
The area of two lots 2015-10-13
From Lynn:
I have 2 lots that have mobile homes on them... I was told they were 100 x 200 lots.. I have tried to calculate it on here but was not able to understand how to exactly do it. I would like to know if I have a 1/2 acreage or exactly what the 2 lots are?
Answered by Penny Nom.
2.236... 2015-10-13
From Ann:
2.236...
What is the most specific category of numbers does this fall into? Rational or Irrational? Does the .... mean that it repeats?

Answered by Harley Weston.
How can the probability be zero? 2015-10-13
From anagha:
Hi,
thank you for answering (beforehand)
The other day I was just thinking of circles and I came across this peculiar thing. Most of'em say that there are infinite number of points on the circumference. and, if it is true, if we spin the radius, the probability that it will point a a particular point is 1/infinity, right?
It didn't make any sense to me and so, I thought there might be an explanation or something of the kind...?

Answered by Harley Weston.
Mathematics 2015-10-13
From anagha:
Hi,
thank you for answering (beforehand)
The other day I was just thinking of circles and I came across this peculiar thing. Most of'em say that there are infinite number of points on the circumference. and, if it is true, if we spin the radius, the probability that it will point a a particular point is 1/infinity, right?
It didn't make any sense to me and so, I thought there might be an explanation or something of the kind...?

Answered by Harley Weston.
Squash scheduling 2015-10-01
From Ed:
I have 5 teams and 2 courts. Every Thursday we play at four different times (6,7,8,9 o clock)

Each team will play each other 5 times over 25 weeks.

Everyone should get to play evenly at the best time (6:00) and the worst time (9:00)

Answered by Victoria West.
I was wondering what xy means. 2015-09-30
From Eric:
I was wondering what xy means. My two things I think it means are the two digits of x and y or x times y. thanks for your time, Eric
Answered by Penny Nom.
The length of a rectangle 2015-09-26
From Tris:
Hi,
I'm trying to figure out the length of a rectangle but i forgot how. So, the width is 5cm and the perimeter is 34cm.

Answered by Penny Nom.
The dimensions of a rectangle 2015-09-25
From Erick:
A rectangular strip of land has a perimeter of 124meters and an area of 385 square meters. Find its possible dimensions
Answered by Penny Nom.
2p^2+4p=16. How do i get p? 2015-09-22
From Dohie:
How do i solve this?

2p^2+4p=16. How do i get p?

Thanks

Answered by Penny Nom.
A 6-person team that has 9 players 2015-09-21
From Sheri:
I am organizing a 6-person team that has 9 players. How do I schedule them fairly over a 9 week league?
Answered by Victoria West.
A hockey roster of 19 kids 2015-09-21
From Brian:
We have a hockey roster of 19 kids. We only want to play 15 kids for each game. 4 players will not play each game. There are 16 games total. We want to rotate who doesn't play evenly. Can this be done? How many games will each player sit during the season?
Answered by Robert Dawson and Victoria West.
Square roots 2015-09-21
From mariana:
I have read various articles on how to find the square root of irrational numbers and every article out there seems to be very confusing.
i read you answer to LUKOW about irrational numbers and i am still quite confused. Say i want to find the square root of 326. i know that it is between 18 and 19 because 18 is the square root of 324 and 19 is the square root of 361 im just very confused about the rest of the process. Please help! ( if possible i would appreciate two examples. thanks)

Answered by Penny Nom.
32 golfers playing 6 rounds 2015-09-19
From scott:
i am looking for a schedule for 32 golfers to play 6 rounds and never play with the same person twice.
I made one for 28 golfers but i am struggling with 32.

Answered by Victoria West.
A word problem 2015-09-17
From Suszanne:
Multiplying a number by x yields the same result as dividing the number by 0.125. What is the value of x?
Answered by Penny Nom.
Two price reductions and a coupon 2015-09-16
From Suszanne:
Mary purchased an item for 45% off the original price, plus an additional 20% off the sale price.She also had a $5 off coupon, which the salesclerk applied after these two discounts. Mary's final purchase price for the item was $50. She paid no sales tax, what was the original price of the item?
Answered by Penny Nom.
Seven switches with 3 stages each 2015-09-15
From jeff:
We are trying to solve a combination problem. If I have 7 things that can be in 3 states (say on/partially on/off), and all 7 have to a state. How many different combinations can there be? It doesn't seem like we should treat it as 21 independent things, but not sure how to break the problem apart.
Answered by Penny Nom.
20 golfers for 6 days 2015-09-11
From Dave:
have 20 golfers for 6 days. Each player should pair with 18 golfers used a system on your website which is good for 5 days, but cannot get the 6th day. Thanks for your help Dave
Answered by Victoria West.
18 golfers playing four rounds 2015-09-09
From Paul:
I have 18 golfers playing four rounds. I want two 3 balls and three 4 balls each day. I don't want anyone to play in a 3 ball more than once and I don't want anyone to play with the same person twice. I realise everybody can't play with everybody, I just want the best you can do.
Answered by Victoria West.
Scheduling for a bridge club 2015-09-09
From margot:
Can you help me with scheduling our bridge club?
8 rounds are played with four teams together.
There are 16 teams in total, so our schedule should look like eight 4x4 boxes.
(Each round the four teams play each other over the course of the evening)

Answered by Victoria West.
Division and units 2015-09-08
From Kenneth:
Hello:

Would there be a reason to divide a number without units by another number with units? Here is an example:

35 divided by $5.00 or 35/$5.00

The answer cannot have units ($) because $5 X 7 does not equal 35 but instead $35.00.

$35.00 divided by 5 or ($35.00/5) and $35 divided by $5.00 or ($35.00/$5.00) make more sense.

I thank you for your reply.

Answered by Robert Dawson.
HCF and LCM 2015-09-05
From Ally:
the HCF of the two numbers is 3, and the LCM is 15. what could the two numbers be?
Answered by Penny Nom.
A schedule for eight teams and four stations 2015-09-04
From Joel:
We are having a youth activity where we have eight teams and four stations. At each station there will be two teams competing against each other. We need each team to go through all four stations, but without ever competing against the same team, and, of course, without ever doing the same station twice.

If this possible? Thank you!

Answered by Victoria West.
A tangent line to a circle 2015-09-03
From Jamie:
Use the Greek method to find an equation of the tangent line to the circle x^2+y^2-4x+6y+4=0 at the points (3,2square root 2-3.
Answered by Penny Nom.
The weight of a pumpkin 2015-09-01
From Lisa:
A pumpkins weighs 10 pounds plus half of 10 pounds plus half it's own weight, what does the pumpkin weigh?
Answered by Penny Nom.
The diameter and circumference of a circle 2015-09-01
From Tracey:
Hello,
I own a custom workroom, I am figuring out fabric quantities or an estimate. When at the job site, I forgot to measure the diameter of the semi circle shape that I have to make a cushion for!!!
If the circumference of the semi circle is 165" what would the diameter be, maybe half this measurement????

I appreciate any help...I basically need the width from left to right to see if I can fit the cushion on one width of fabric that is 52" wide!

Answered by Penny Nom.
Working backwards 2015-08-27
From Heather:
Tom took a collection of tiles from a box. Sue took 13 and Bob took half of those remaining. Tom had 11 left. How many did they start with?
Answered by Penny Nom.
A schedule for an 8 person team 2015-08-27
From Julie:
We have 8 people on our team, but only 4 can play every week. We have 32 weeks, total. How can we divide this so everyone plays evenly ?
Answered by Robert Dawson.
A procedure for finding a square root 2015-08-27
From Tracey:
Is there a method/process to determine the square root of a large number?

For example, Tiananmen Square's area is 396 900m sq., what is the side length?


Thank you,
Tracey

Answered by Harley Weston.
Factor x^2 + y^2 2015-08-26
From Jerome:
Factor: x^2+y^2
Answered by Penny Nom.
A conditional probability problem 2015-08-24
From Faustina.:
Please I have an exam tomorrow. And I have tried by best in solving this Question. An urn contains 10 white, 5 yellow and 10 black marble. A marble is chosen at random from the urn and it is noted that it is one of the black marble, what is the probability that it is yellow?
Answered by Penny Nom.
Four equations in 3 unknowns 2015-08-23
From damien:
solve:
S x U - T = 44
T - U x S = 36
U + T - S = 7
U x U / S = 4

Answered by Penny Nom.
Reinforced steel mesh in a circular foundation 2015-08-22
From Padmesh:
in a circular foundation we are laying a reinforced steel mesh . i.e., like chords in both ways. the circle diameter is 2.91m and rods are placed 0.1m spacing. so I want to find the number of rods present in that circle. here by I am attaching an auto cad drawing for your reference
Answered by Penny Nom.
The distance over a Quonset 2015-08-20
From jane:
total base of hemisphere is 30 ft apex height is 20 feet what is total length over dome
Answered by Penny Nom.
A probability question 2015-08-19
From Omar:
Hi my name omar ,
I have an exam tomorrow and i want answer of this question
Thank you

Q:the probability distribution function of a discrete random variable X is given by
P(X=x)=kx^3
For X=0,1,2,3
a)the value of the constant k .

Answered by Penny Nom.
20 golfers playing four rounds 2015-08-17
From Paul:
I have a party of 20 golfers playing four rounds in fourballs. I realise that everyone will have 7 players that they do not play with, however I want to ensure that no one plays with the same person twice and that the Captain plays with the 12 players of his choice.
Answered by Victoria West.
Parallelogram area 2015-08-04
From Nigel:
To find the area of a parallelogram you multiply the base by the height of the parallogram, the height being determined by an imaginary line drawn at right angles to the base. However if I made a parallelogram with joints at each corner and maipulated it to an upright position where the base and sides were at 90 degrees to each other,, I could then simply multiply the base times the height to get the area. Since the sloping side of the parallelogram does not vary in my imaginary jointed model, why can I not simply multiply the base by the sloping side to get the area in the same way that I multiply the base times the height of a square to find its area ? I cannot find the answer to this on the internet although I have searched.
Answered by Chris Fisher.
Rotating speakers through a 10 week schedule 2015-08-03
From Timothy:
I want 15 students to speak 7 times each over the course of 10 weeks.
They each will have 5 minutes.
Thanks

Answered by Victoria West.
An oil field barrel 2015-08-02
From Michael:
I have a storage tank that is 12' diameter x 22.5' high
it's capacity is 450 oil field barrels (42 gallon)
Which would hold 19,036 us gallons
I'm trying to find out how many inches it would take to make 1 barrel
I was able to get this far by using your volume calculator, but haven't been able to come up with the rest!
Thank you very much for your time, and I really appreciate this site and the tools you all have made available to everyone for free!
Hope to hear back from you soon and God bless you all!!

Answered by Penny Nom.
The length of a shadow 2015-08-01
From maaz:
Hello
I am having trouble with this question:

Lizzie, who is 6 feet tall, stands in her driveway at night, exactly 24 feet from the base of a spotlight, and casts a shadow that is 12 feet long if her friend Hannah who is 5 feet tall decides to stand next to lizzie how long will her shadow be?

Answered by Penny Nom.
The distance between two boats 2015-07-26
From mohammad:
Two boats leave the same dock at the same time,one traveling due east at 10 mph and the other due north at 24mph,how many miles apart are the boats after 3hours ?
Answered by Penny Nom.
Scale 2015-07-24
From Kenneth:

Hello:

Can you explain mathematically how the following scales become inches to feet?
For example, how does the scale of 1:160 become 1/16" = 1'?

(1:160) (1/16" = 1 foot) scale
(1:87) (1/8" = 1 foot) scale

I saw more of these different scale examples at the following:

http://www.oakridgehobbies.com/hobby-shop/modeling-in-scale-dimensions-f-a-q-s.html

I thank you for your reply!


Answered by Robert Dawson and Harley Weston.
Mary, Julie and Peter when on a trip 2015-07-10
From Edward:
How do you solve and set up a problem where: Mary pays one third of the cost of a trip, Julie paid half of what Mary paid, and Peter paid the remaining $75,
Answered by Penny Nom.
Golf for 12 with a proviso 2015-07-07
From Ken:
I hope you can help me. I'm trying to put together a draw for 12 players in 3 groups over 4 days. One proviso, 2 players must play together in each round because they require the use of a golf cart. We are due to play in September this year.
Thanking you in advance for any assistance you can give.

Answered by Victoria West.
Why are bags of soil measured in litres? 2015-06-19
From Cheryl:
Why are bags of soil measured in litres?
Answered by Penny Nom.
My final exam score 2015-06-19
From Deirdre:
Hi,
I got 69% on a question worth 20%
68% on a question worth 20%


And the final exam was worth 60%

My final grade overall was 63%

Did I do very bad in the exam????

Answered by Penny Nom.
1 + 1 + 1 + 1 + 1 + 1 + 1 + 1 + 1 + 1 + 1 + 1 x 0 + 1 = ? 2015-06-18
From Sharon:
1 + 1 + 1 + 1 + 1 + 1 + 1 + 1 + 1 + 1 + 1 + 1 x 0 + 1 = ?

I got 1 as my answer despite BODMAS making it 12 because logic tells me I ought to place brackets around the first set of repeated addition. Could you please clarify this for me? Thank you 😊

Answered by Harley Weston.
A waste oil tank 2015-06-13
From Angela:
a waste oil tank is 5 feet wide and 20 feet long, the empty tank on your truck holds 5000 gallons. If 7.48 gallons are in every cubic foot, what is the maximum depth the oil can be to completely fit into your truck?
Answered by Penny Nom.
The amount before profit and overhead 2015-06-12
From kelly:
I have an existing amount of $22,750.00. this includes the profit at 10% and overhead at 10%. How do I remove the profit and overhead in order to come up with the original starting figure?
Answered by Penny Nom.
The area of an isosceles triangle 2015-06-11
From Pradipta:
Find the area of an isosceles triangle having two equal sides of 20 cm each and angle between them 45 degree ? ( solve without trigonometric function).
Answered by Chris Fisher.
A fantasy football league 2015-06-11
From Jason:
Hello. We have a fantasy football league with 14 teams. The season is 13 weeks long and we would like to figure out a way to have two divisions of seven teams each with each team playing their inter-division opponents twice. Is there a way to set this up? I cannot seem to find a way to do it. Thanks!
Answered by Victoria West.
8 golfers playing 5 round 2015-06-11
From Simon:
We have 8 golfers playing 5 rounds, and want to make the pairings as even as possible (ie all play against everyone else)

2 of the players however must play together each day - due to a buggy being required.

If anyone has a formula to work this out I'd be appreciated

Answered by Victoria West.
An equilateral triangle inscribed in a circle 2015-06-11
From Casey:
I have an equilateral triangle inscribed in a circle - this triangle has been bisected to give me 2 right triangles. I know the length of the line bisecting the equilateral triangle is 36 inches. How do I figure out the circumference of the circle and the length of the sides of the equilateral triangle?
Answered by Penny Nom.
Is 22/7 is an irrational number? 2015-06-07
From Sayak:
Is 22/7 is an irrational number
Answered by Penny Nom.
The surface area of a round bar 2015-06-06
From chirag:
Please tell me the formula for finding round bar surface in square inches/ mm
Answered by Penny Nom.
Manipulating an equation 2015-06-05
From Leigh:
Hi, I'm having trouble trying to find out how the equation y=mx+b for b comes up with the answer b=y-mx. Could you please assist me with this
Answered by Penny Nom.
A two digit number 2015-06-05
From Madison:
Trini took some photos. The number of photos she took is a two digit number. The sum of the digits is 11. The tens digit is 3 more than the ones digit. How many photos did Trini take?
Answered by Penny Nom.
The surface area of a triangular prism 2015-06-03
From Alex:
Find the surface area of a triangular prism with Isoceles triangles and dimensions 4 feet by 6 feet
Answered by Penny Nom.
The volume of a pipe 2015-06-02
From gordon:
What is the formula to find out how many gallon of water in a length of pipe tried your answer on 8inch at 50feet did not work out
Answered by Penny Nom.
A tangent to y = x^3 2015-05-31
From Brayden:
Show that a tangent line drawn to the curve y=x^3 at the point (d,f (d)), where d>0, forms a right triangle with the x and y axes in quadrant 4 whose area is (2/3)d^4.
Answered by Penny Nom.
A pentagon inscribed in a circle 2015-05-30
From Victoria:
find the area of a regular pentagon inscribed in a circle with radius 3 units
Answered by Penny Nom.
A stone walkway around a pool 2015-05-29
From Jerri:
I have a rectangular pool 15x30 and it will require stone placed around it 4 feet out and 4 inches deep how much will I need? Thanks, jerri
Answered by Penny Nom.
The perimeter of a rectangle 2015-05-28
From Imran:
The length of a rectangle is x+3 centimetres.
The width of the rectangle is x-1 centimetres.

Find an expression in terms of x for the perimeter of the rectangle.
The perimeter of a rectangle The perimeter of a rectangle
Give your expressions in it's simplest form.
The perimeter of a rectangle The perimeter of a rectangle
Thank You.

Answered by Penny Nom.
Two concentric circles 2015-05-28
From Shannan:
Two concentric circles have radii of 24cm and 26cm. What is the length of the chord that is tangent to the inner circle? Include a sketch
Answered by Penny Nom.
The perimeter of a quarter circle 2015-05-21
From Bethany:
The perimeter of a quarter circle is 3.57. What is the quarter circle's radius?
Answered by Penny Nom.
Golf for 14: 2 fourballs and 2 threeballs 2015-05-20
From martin:
I have to organize 14 players to play four rounds of golf. Each round comprise two fourballs and two three balls .

What's the best combination to ensure that each persons plays with as many of the others as possible without playing with somebody more than twice?

Answered by Victoria West.
Manipulating an expression using logarithms 2015-05-18
From Katie:
Solve x=At^be^-ct for b in terms of c.
The e is Euler's number, not a letter or random placeholder.
v I've tried 2 methods on my calculator, and I am unsure as to why they give me different answers, and which one is correct.
For some reason, when placing a multiplication sign in between the A and the first t, I get a completely different answer. Why is this?
Also, which is the right answer for the question I have?
Thank you!
Katie :)

Answered by Penny Nom.
An isosceles right triangle 2015-05-17
From Ari:
In a 45-45-90 triangle find the ratio of a leg to the hypotenuse
Answered by Penny Nom.
An octagonal frame around a pool 2015-05-17
From James:
I have a 20' pool and need to put a frame around it using 2by 4 what r the lengths and angle cuts
Answered by Penny Nom.
The intersection of a plane and a cone 2015-05-16
From Tom:
Is there a way to derive an equation that describes the perimeter of the intersection of a plane and a cone regardless of the angle of the plane to the cone. Assume that the plane does not cut through the base of the cone, the x, y, z location of the vertex is known, the distance from the vertex to the plane through the axis is know., and that the angle of the cone is known.
Answered by Chris Fisher.
A pile of mulch 2015-05-15
From Justin:
I have a pile of mulch that measures 19 feet wide at the bottom and has a height of 6 feet. I need to know how many yards of mulch I have.
Please help and help with the formula for further use. Thank you!

Answered by Penny Nom.
Two lorries approaching an intersection 2015-05-15
From Nuraini:
Two straight roads intersect at the right angles. Lorry A, moving on one of the roads, Approaches the intersection at 50mi/h and lorry B, moving on the other roads, approaches the intersection at 20mi/h. At what rate is the distance between the lorry changing when A is 0.4 mile from the intersection and B is 0.3 mile from the intersection?
Answered by Penny Nom.
An angle of depression problem 2015-05-14
From haxir:
find the height of the balloon directly above a town if the measure of angle of depression of another town 5km from the first town is 20°!6`
Answered by Penny Nom.
A calculus optimization problem 2015-05-14
From Ali:
Given an elliptical piece of cardboard defined by (x^2)/4 + (y^2)/4 = 1. How much of the cardboard is wasted after the largest rectangle (that can be inscribed inside the ellipse) is cut out?
Answered by Robert Dawson.
Folding a rectangle to preserve the aspect ratio 2015-05-09
From Anna:
If you fold a rectangular piece of paper in half and the resulting rectangles have the same aspect ratio as the original rectangle, then what is the aspect ratio of the rectangles?
Answered by Penny Nom.
Repeated decimals 2015-05-09
From Vir:
Years ago I (re?)discovered 'cyclic division'. For example: if you arrange the number along a circle and put the number 142857 at the centre all the numbers taken cyclically, starting with 1, are fully divisible by 37. Whatever the starting point of this number, it remains fully divisible by 37. what is more, the number can be formed by taking the digits clockwise as well as anti-clockwise..This I call "full cyclic divisibility". In many cases, only clockwise cyclic divisibility is possible. But I have not come across a case where ONLY anti-clockwise divisibility occurs. Thus clockwise cyclic divisibility seems to be favoured. Could this be construed as a sign of chirality in mathematics?..
Answered by Chris Fisher.
A word problem 2015-05-08
From JAMES:
A businessman bought a number of similar articles for a total of sh. 6000. Three of the articles turned out to be defective with no resale value. He sold the remaining articles at 12.5% each above the cost price making a total profit of sh 480. How many articles had he bought?
Answered by Penny Nom.
A triangle inscribed in a circle 2015-05-07
From R2D2:
A triangle is inscribed in a circle of radius 10. If two angles are 70 degrees and 50 degrees find the length of the side opposite the third angle?
Answered by Chris Fisher.
A circle is tangent to the y-axis 2015-05-06
From Elynna:
What does it mean when a circle is tangent to the y-axis?

Thanks,

Answered by Penny Nom.
Sales tax 2015-05-05
From Adalyn:
Hello i've been having some trouble with math, The question is asking for a GST and a PST of a 30.00 dollar shirt. I don't know how to find the GST or the PST please help
Answered by Penny Nom.
The number of possible musical notes using an n-key instrument 2015-05-04
From Farihin:
Lets say that i have keys, and each key is for notes of a musical instrument, So i wanted to find out the number of notes i can get for a certain number keys, of course in the form of an equation. Notes can use as many keys, it can use 1, or 2, or 3, or even 100.
Notes in real life is not as such, but ignore reality. I tried doing this but i can't seem to find a formula for it. For example, i have 4 keys, say A, B, C, and D. so, for notes that uses one key are 4, which is A, B, C, and D themselves. for notes that uses two keys are 6,
AB, AC, AD, BC, BD and CD.
for notes that uses three keys are 4,
ABC, ABD, ACD and BCD.
lastly for notes that uses all four keys is 1, ABCD.
So, the total will be 4+6+4+1=15#

The nth term for the first equation is n, the second is [(n^2)-n]/2 the third and the fourth, i don't know but the final answer should be like,
n + [(n^2)-n]/2 + [3rd] + [4th]

Sorry for the long question though...

Answered by Penny Nom.
13 games, 3 possible outcomes 2015-05-01
From Kennedy:
Hello, hope it finds you well.
Am not that good with numbers, can you please help me get all the possible outcomes of 13 games, with a home win, draw or an away win as three possible results for each of the 13 games.

Thanks in advance

Answered by Penny Nom.
The method of elimination 2015-05-01
From oreanna:

Question from oreanna, a student:

How do u solve 2x+9y=3

7x-4y=-25 in elimination


Answered by Penny Nom.
A square and a rectangle 2015-05-01
From nick:
why is the area of a square of a given circumference greater than the area of a rectangle with the same circumference ?
Answered by Penny Nom.
Filling a pool with dirt 2015-05-01
From Mike:
I have a hole which a 24 ft pool in it is 10" deep in the the centre and goes to 1" inch at the edge want to fill it in with dirt how many yards of dirt would I need to fill it in
Answered by Penny Nom.
The volume of a sphere 2015-04-30
From Cassidy:
How do you find the radius of a sphere that has volume 36pI?
Answered by Penny Nom.
The volume of a cube 2015-04-30
From megan:
What's the volume of a cube with a side length of 6?
Answered by Penny Nom.
Locating the center of a circle that forms an arc 2015-04-23
From Ken:

Find the Cartesian coordinates of the center of an arc with the given location of the beginning and end points and radius length. Not the midpoint of the circumference but the actual point that the arc
is drawn around.

I know their are two answers depending on the direction of the arc. Unless we assume that all arcs are drawn counter clock wise.

Thanks
Ken


Answered by Harley Weston.
A word problem with fractions 2015-04-23
From Cecilia:
if one third of a pole is red, one quarter is white and 15m is black what is the height of the entire pole
Answered by Penny Nom.
Expand and Simplify (3x+5)(5x-9) 2015-04-23
From chloe:
Expand and Simplify (3x+5)(5x-9)
Answered by Penny Nom.
Two concentric circles 2015-04-21
From Juniper:
Two concentric circles have radii of 4 cm and 8 cm. A segment is drawn so that it is tangent to the smaller circle and a chord of the larger circle. How long is the segment?
Answered by Penny Nom.
A circle inscribed in a square 2015-04-19
From michael:
A circle is drawn in a square PQRS. If the total area of the shaded portion of the square is 42 cm square, calculate the radius of the circle
Answered by Penny Nom.
An inequality 2015-04-19
From mr.:
Simplify the inequality √x>x and represent on a number line
Answered by Penny Nom.
A wireless fence 2015-04-18
From Dave:
I'm buying a wireless fence to keep my pet in my yard. It has a half acre range. In a straight line how far would that be?
Answered by Penny Nom.
Two cars approaching an intersection 2015-04-16
From Engabu:
Car A is traveling west at 50km/h & car B is traveling north at 60km/h. both are headed for the intersection of the two roads. At what rate are the cars approaching each other when car A is 3km & car B is 4km from the intersection?
Answered by Penny Nom.
How many boats total are in the marina? 2015-04-15
From Carla:
In a marina, 3/4 of the boats are white, of the remainder 4/7 are blue, the rest are red. There are 9 red boats. How many boats total are in the marina? My answer is 84 because if 1/4 of the boats are divided into sevenths, that makes the whole marina a 28 part item. 3/28 are red so 84 is the total number of boats. My child thinks I am wrong.
Answered by Penny Nom.
Two airplanes 2015-04-14
From john:
two planes leave an airport at the same time, one going northwest (N35*W)at 400 mph and the other going east at 332 mph. How far apart are the planes after 4 hours to the nearest mile?
Answered by Penny Nom.
Constructing a box of maximum volume 2015-04-14
From Margot:
I need to do a PA for maths and I'm a bit stuck. The PA is about folding a box with a volume that is as big as possible. The first few questions where really easy but then this one came up.

8. Prove by differentiating that the formula at 7 does indeed give you the maximum volume for each value of z.

Answered by Penny Nom.
Team selection 2015-04-13
From Peter:
I manage a football team with a list of 28 players.
Only 24 can play each game.
The season is 16 games.
How do I fairly select 24 players each week.

Answered by Robert Dawson.
The area of a circle given the circumference 2015-04-12
From F:
How do you find the area of a circumference of 6?
Thanks.

Answered by Penny Nom.
Excavating a basement 2015-04-12
From Randy:
The cost of excavation is $4.50/ cu yd. Find the cost of excavating a basement 87 ft long, 42 ft wide, and 8 ft deep.
Answered by Penny Nom.
An 8 pointed star inscribed in a circle 2015-04-10
From Kermit:
How do you find the area of the star that is formed by two squares and surrounded by a circle. The only given information is that the radius of the circle is 10.
Answered by Penny Nom.
The perimeter of a quarter circle 2015-04-10
From Riley:
How do you find the perimeter of a quarter circle when the radius is 56?
Answered by Penny Nom.
A word problem with fractions 2015-04-09
From Lorraine:
If the numerator of a certain fraction is doubled and the denominator is increased by 1, the fraction becomes 1/2.

If the numerator of the original faction is squared and the denominator is decreased by 2, the fraction becomes equal to 1.

Let x be the numerator and let y be the denominator of the original fraction. Write down two simultaneous equation in x and y.

Solve these equations to find two possible values for the given fraction.

Answered by Penny Nom.
The area of the ring between two concentric circles 2015-04-08
From Conner:
The area of the ring between two concentric circles is 25pi/2 square inches. The length of a chord of the larger circle tangent to the smaller circle is?
Answered by Robert Dawson.
A circle graph 2015-04-06
From Davis:
i don't know how to answer this question. please help!

a survey about student government shows the following results

110 like the program
120 think the program is unnecessary
210 plan on running for student government next year

if this information is put in a circle graph,what is the central angle for the students who like the program?

Answered by Penny Nom.
On what day of the week did 1994 begin? 2015-04-05
From tayyaba:
there were 365 days in the year 1993 the first day of the year was friday. on what day of the week did 1994 begin?
Answered by Penny Nom.
A tangent to y=x^3 passes through (0,2) 2015-04-05
From Kevin:
Given that the curve y=x^3 has a tangent line that passes through point (0,2). Find the equation of the tangent line
Answered by Penny Nom.
The weight of a steel block 2015-04-04
From Mark:
the weight of a steel cube 37 inches X 28 inches X 7 inches
Answered by Penny Nom.
n^2 is a multiple of 100 2015-03-30
From Rahul:
I have to prove that n^2 is a multiple of 100 is necessary or Sufficient condition (or both) for n being multiple of 10
Answered by Penny Nom.
Dealing cards to Melissa and Robert 2015-03-24
From Karena:
92 cards in all. Melissa has 3 times as many cards as Robert. How many cards does Robert have?
Answered by Penny Nom.
An isosceles triangle inscribed in a circle 2015-03-23
From Rachel:
Triangle ABC is an isosceles triangle inscribed in circle O. If each leg of the triangle is 13cm and the altitude to the base of triangle ABC is 5cm, find the radius of the circle.
Answered by Penny Nom.
16 golfers for 18 weeks 2015-03-22
From Rose:
We could use a schedule to include the following:
16 golfers
4 foursomes
Each person plays once a week for 18 weeks.
No one plays in same foursome more than 1 or 2 times, or the least amount of time.

Thank you for your kindness.

Answered by Victoria West.
8 golfers playing in 2x2 balls in 5 games 2015-03-18
From Brian:
We are 8 golfers who will be playing in 2x2 balls in 5 games. is there a formula to minimize the chances of playing with a partner or opponent too often please ?
Answered by Victoria West.
The weight of two dogs 2015-03-17
From Renee:
Darlene's dog weighed 5 times as much as Leah's dog. Together, the dogs weighed 84 pounds. How much did each dog weigh? Write an equation and solve
Answered by Penny Nom.
A region defined by three lines 2015-03-16
From Lucinda:
find the area of the figure formed by the line y=-5x+9 and the x-axis and y-axis.
Answered by Penny Nom.
A cone of maximum volume 2015-03-16
From Mary:
I have to use a 8 1/2 inch by 11 inch piece of paper to make a cone that will hold the maximum amount of ice cream possible by only filling it to the top of the cone. I am then supposed to write a function for the volume of my cone and use my graphing calculator to determine the radius and height of the circle. I am so confused, and other than being able to cut the paper into the circle, I do not know where to start. Thank you for your help! -Mary
Answered by Robert Dawson.
Average rate for a return trip 2015-03-14
From Michael:
Jeffrey drove to a town 25 miles away at an average speed of 50 miles per hour. The return trip along the same route took 20 minutes longer than the trip to town. Determine the average speed in miles per hour for Jeffrey's round trip.
Answered by Penny Nom.
The side length of a gazebo 2015-03-14
From Dennie:
From middle of side to middle of opposite is 14'6". What is the length of the side?
Answered by Penny Nom.
A limit 2015-03-13
From arthur:
sin(x-1)/x^+x-2
Answered by Penny Nom.
A fishfinder 2015-03-13
From Dave:
I have a fishfinder that has a 20 degree cone on bottom of boat going to the bottom of the lake. How do I know the size of base diameter of the cone on the lake bottom depending on depth... such as 10 feet deep, for example?
Answered by Penny Nom.
Olivia and Ray walk to school 2015-03-12
From Rebeccah:
Olivia and Ray walk to school. Olivia walks 1/4 of a mile to school. Her walk is 2/3 of the distance that walks Ray to school. What is the total distance,in miles,that Ray walks to school?
Answered by Penny Nom.
Two sunflowers 2015-03-12
From Maurice:
Sam and sally planted two sunflowers.Each sunflower grows 1cm each day. Sally's sunflower is 4cm high. Sam's is 12cm high. How high will each sunflower be when Sam's sunflower is twice the height of Sally's?
Answered by Penny Nom.
Constructing a cone 2015-03-12
From Levi:
While the mathematics at the welding shop page was very helpful there is one vital part missing. If I have never built a cone how do I figure out how much bigger the diameter of my circle has to be when laying flat on the floor verses the diameter when it has been pulled into a cone.
Answered by Harley Weston.
The height of an equilateral triangle 2015-03-12
From anna:
I am anna and i am in 7th grade.
i am trying to find the height of and equilateral triangle, all sides equaling 4 inches

Answered by Penny Nom.
The volume of a trench 2015-03-11
From Kim:
A trench 60m long and 15m deep, 12m wide at the top and 8m wide at the bottom.How many cubic meters of earth have been removed?
Answered by Penny Nom.
The area of a quadrilateral 2015-03-11
From Joel:
Diagonal ac of quadrilateral ABCD is 60cm and the lengths of perpendiculars to It from the opposite vertices are 4.2cm and 5.8 cm find the area of the quadrilateral ABCD
Answered by Penny Nom.
What is 1000 billion billion? 2015-03-10
From LeRoy:
what does this mean? 1000 billion billion
Answered by Penny Nom.
Angles of elevation and depression 2015-03-08
From Timmy:
Joel is walking down a street and sees a 115 ft tall building in front of him. He stops 190 feet from the base of the building at the tip of the building's shadow. Round answers to three decimal places.

A. If there was a piece of rope from the top of the building to Joel, how long would it be?
B. What is the angle of elevation from Joel to the top of the building?
C. Margaret says that she could find the angle of depression from the top of the building to Joel by subtracting the angle of elevation from 90°. Is she correct? Explain.

Answered by Penny Nom.
Extraneous solutions 2015-03-07
From Emily:
I have a question about Extraneous Solutions, Because I was recently researching to figure out on how to determine that a solution is extraneous and many of the answers talked about how if a answer is negative that it should always be extraneous but now I found out that positive solutions can also be considered as extraneous solutions so that is why I am really confused Extraneous Solution's.
I would really appreciate it if you could clearly explain to me how to determine an extraneous solution from a normal solution.
Thanks,
Emily

Answered by Chris Fisher.
A lot on a cul-de-sac 2015-03-06
From ana:
Julie lives on a cul-de-sac and wants to know if the city calculated the area of her plot of land correctly (see diagram below). We can assume the cul-de-sac is a circle and it has radius 40 feet.The boundary of the lot along the cul-de-sac (the arc from A to B) is 78.21 feet. The city found the area to be 7012.5 square feet–are they correct?
Answered by Penny Nom.
A man is standing on top of a building 2015-03-06
From kiki:
a man of height 1.5m standing on top of a building of height 48.5m views another building across the square. he observe that the angle of depression of the bottom of the building is 40 degrees and the angle of depression of the top of the building is 5 degrees. Both buildings stand on the same level
a) calculate the distance of the man from the base of the building across the square measured along the line of sight

Answered by Penny Nom.
Two parallel tangents to a circle 2015-03-05
From Samantha:
The equation of a circle is x^2+y^2=25. Determine the equation of the parallel tangent lines to this circle, for which the slope is 4/3.
Answered by Penny Nom.
The length of a ramp 2015-03-05
From Jaylynn:
Hi, I've been trying to figure out how long my ramp would have to be in order to reach a height of 3.5 feet at a 30 degree angle for a sugar glider enclosure?
Answered by Penny Nom.
Bricks around a fire pit 2015-03-05
From Jayson:
I have a round fire pit. It measures 25 inches in diameter. I have 12 inch long square bricks to go around it . My question is what degree do I cut the ends of these bricks to make them fit around this circle? The brick dimensions are 12"Lx6"Wx4"D.
Answered by Harley Weston.
Istanbul to Lagos 2015-03-03
From Victor:
A plane flies from Istanbul to Lagos at 800km/h and then returns to Istanbul at 900km/h. The return trip takes 1 hour less than the flight to Lagos. What is the distance between Istanbul and Lagos?
Answered by Penny Nom.
The radius of a cylinder 2015-02-26
From Rose:
Hi. I want to know how to find the radius of cylinder.
When I have height (35cm) and area of the curved surface(880sq.cm).
I know the formula is 2πr(h+r). But I can't understand how to find it's radius.
Please help me. I really need your help.

Answered by Penny Nom.
Two equations with fractions 2015-02-26
From Pulane:
Hi math centre I've been trying to solve these equations for days now please help (6/x)-(1/y)=4
(9/x)+1=(-2/y)

Please help me solve them simultaneously
Thank you

Answered by Penny Nom.
Two concentric circles 2015-02-25
From Manasi:
The area between 2 concentric circles is 6 times the smaller circle. Radius of small circle is 7 cm. Find the difference in the circumference of the bigger circle ad the smaller circle?
Answered by Penny Nom.
3 over "x" plus 5 2015-02-23
From Kenny:
3 over "x" plus 5. I don't know this is either a polynomial and if so how would you define it.
Answered by Harley Weston.
f(x)=(x^2-1)/(x-1) 2015-02-21
From Ahmed:
Is f(x)=[(x^2-1)/(x-1) and x=2 at x=1] differentiable at x=1 ? Why ?
Answered by Penny Nom.
A region described by inequalities in cylindrical coordinates 2015-02-21
From Rahul:
I am not able to visualize a solid given by inequalities as under (in cylindrical coordinate system) 0= I know that (can visualize) first 2 inequalities say it is a cylinder of infinite length. It seems that third inequality poses the restriction to the height as well as it makes us to remove a cone.
But I just can not understand it clearly.
Thanks in advance!
Regards,
Rahul

Answered by Penny Nom.
Column vectors 2015-02-21
From jill:
A has co- ordinates (7,-1) and B has co-ordinates (3,5).write these as column vectors
Answered by Penny Nom.
A limit 2015-02-19
From genc:
Lim (27x^3-1) / (3x-1)
X-> 1/3

Answered by Harley Weston.
The height of a building 2015-02-18
From prateek:
the height of a flagpole on top of a building is 6m. the angle of elevation of the top and bottom of the flagpole are 45 and 30 respectively. find the height of the building
Answered by Penny Nom.
An isosceles triangle and an arc 2015-02-18
From Sreeharsha:
The diagram shows an isosceles triangle ABC in which BC = AC = 20 cm, and angle BAC = 0.7 radians. DC is an arc of a circle, centre A. Find, correct to 1 decimal place,
(i) the area of the shaded region, [4]
(ii) the perimeter of the shaded region. [4]

Answered by Penny Nom.
5000 gallons in cubic yards 2015-02-17
From Miguel:
If I have a cube shaped tank in the ground that holds 5000 gallons and I want to take it out and fill the hole in with dirt, how much dirt will I need in yards?
Answered by Penny Nom.
Tiling a room 2015-02-16
From Owen:
nicolas room is 56m2 she wants to put tiles down which are 50cm by 50cm each cost £4 how much money will she spend
Answered by Penny Nom.
Two equations 2015-02-16
From nigel:
2x+1/2y=1
6x-3/2y=21

Answered by Penny Nom.
A 330 foot long fence 2015-02-15
From Marsha:
I am putting up a fence 330 feet long and want to put the t post every 4 feet how many t post do i need?
Answered by Penny Nom.
Distance, time and rate for three people 2015-02-15
From Mubashir:
Steven left Town A and walked towards Town B at a speed of 100m/min. At the same time, Jason and Melvin started from Town B and walked towards Town A at a speed of 80m/min and 75m/min respectively. If Steven met Melvin six minutes after passing Jason, find the distance between Town A and Town B.
Answered by Penny Nom.
Bearings and a rectangular plot 2015-02-14
From Mubashir:
A,B,C,D are the four corners of a rectangular plot marked on level ground. Given that the bearing of B from A is 040 degrees and that the bearing of C from A is 090 degrees. Calculate the bearing of
(a)B from C
(b)A from C
(c) D from C

Answered by Penny Nom.
Stopping distance 2015-02-11
From Audrey:
A Harley Davidson motor cycle weighing 650 lbs with a rider weighing 175 bs was going 40 miles per hour. He had to make a sudden stop . Slammed on brakes. How long a distance for him to stop?
Answered by Penny Nom.
A word problem with fractions 2015-02-11
From Omokayode:
A vehicle tank was 3/5 full of petrol. When 21 litres of fuel was added it was 5/6 full. How many litres of petrol can the tank hold?
Answered by Penny Nom.
A fence post problem 2015-02-08
From Nicole:
In Susan's rectangular garden plot, she put 12 posts on the long sides and 8 posts on the short sides. How many posts did she use altogether? (The answer is not 40.)
Answered by Penny Nom.
A circle and a chord 2015-02-07
From Dakota:
If the diameter of a circle is 48 cm, find the shortest distance from a chord of length 34 cm to the center of the circle
Answered by Penny Nom.
Speed, distance and time 2015-02-06
From Terry:
How many seconds would it take to travel 0.18 km at a speed of 130km per hour?
Answered by Penny Nom.
The product of a 2-digit number and a 3-digit number 2015-02-06
From Nathaniel:
The product of a 2-digit number and a 3-digit number is about 50 000 what are the products
Answered by Penny Nom.
2x^2-2x+1=4x+1 2015-02-06
From Caitlyn:
2x^2-2x+1=4x+1
Answered by Penny Nom.
The center and radius of a circle 2015-02-06
From ariana:
I need to find the center and the radius of this circle. I don't know how to put 9/2 than square it into a fraction.

x^2 + 2x + y^2 +9y=5

Answered by Penny Nom.
The volume of a cone 2015-02-05
From Linda:
How do you calculate the volume of a cone that is 25cm high and has an angle of 20 degrees?
Answered by Penny Nom.
A parallelogram inscribed in a circle 2015-02-04
From narendra:
how can we prove that a circular parallelogram is a rectangle?
Answered by Chris Fisher.
Two cars pass each other going in opposite directions 2015-02-03
From Caroline:
Two cars pass each other going i opposite directions. One car is going 100 km/h and the other is going 80 km/h. How far apart will the two cars be after driving 45 minutes at these rates of speed?
Answered by Penny Nom.
A train from Vancouver to Whistler 2015-02-02
From Tea:
Serena and Nina are taking a train from Vancouver to Whistler, a 160km trip. If the average speed is decreased by 16km/hr, the trip will be 1/2 hours longer. What is the average speed of the train? Thanks :)
Answered by Penny Nom.
Jack and Jill climb two poles 2015-02-02
From APRIL:
Two vertical poles are 3 meters apart. Jack is climbing one pole while Jill climbs the other. If the distance between Jack and Jill is 5 meters, how much higher is Jill than John?
Answered by Penny Nom.
Ratio of gas to oil 2015-02-02
From Chris:
I would like to find the gas oil ratio for a 2 cycle engine. I have the gallons of gas and the ounces of oil used by the engine, but I want to find what the ratio is. For example I used 5.675 gallons of gas and 5 ounces oil what is the ratio? Please explain step by step, and I understand there are 128oz in a gallon.
Answered by Harley Weston.
The volume of Lake Utah 2015-01-29
From Hannan:
Lake Utah has a surface area of 3,846 square miles and an average depth of 10.5 feet. In cubic miles, how much water does it hold? How would you approach this question? Where would you start?
Answered by Robert Dawson and Penny Nom.
128/(-16)/(-2) 2015-01-28
From jackie:
128/(-16)/(-2) I was wondering if you can show me how to work this question out
Answered by Harley Weston.
What is our cost? 2015-01-27
From Johanna:
I have a pieces of 3/4" sch40 (round tube) 240" long at $70. What is our cost. We also add 35% to our cost for selling price. How do I figure this out?
Answered by Penny Nom.
The height of an obelisk 2015-01-27
From Fabian:
Two observers on level ground position themselves 50 meters apart so that an obelisk is due north of position A and due west of position B. The angle of elevation to the top of the obelisk at position A and B is measured to be 29.9 degrees and 37.5 degrees respectively. Use this information to estimate the height of the obelisk.
Answered by Penny Nom.
Is a square a trapezoid? 2015-01-26
From Katie:
Can a trapezoid sometimes be a square?
Answered by Penny Nom.
Russell is 6 years older than Samantha 2015-01-26
From Kelli:
Russell is 6 years older than Samantha. In 4 years the sum of their ages will be 78. How old is Russell now?
Answered by Penny Nom.
A road around a garden 2015-01-25
From hahha:
a garden 150 m long and 80 m wide has a road 4 m wide all around on the out side find the cost of paving at the rate of 12 per s.q m
Answered by Penny Nom.
Two concentric circles 2015-01-25
From nazneen:
select the correct answer from the given four options

given are two concentric circles. radii of outer circle & inner circle are r1 & r2 respectively. the areas of inner circle & shaded ring are equal. the radii r1 & r2 are related by?

1. r1 = r2
2. r1 = r2*square root 2
3. r1 = r2*square root 3
4. r1 = 2r2

Answered by Penny Nom.
A schedule for observing our peers 2015-01-23
From Jennifer:
I am a sales trainer. Each month we need to observe one of our peers and give feedback. What this means is every month, not only am I observing someone, but someone is also observing me. For example, in January, Amy observes Tracy, Tracy observes Rachel, Rachel observes Ryan, and so on. The last person would then observe Amy, bringing it full circle. There are 10 trainers on my team. How do I set it up to where each trainer observes another trainer only once over the course of the year (and in return, each trainer is only being observed once by each person)? I know they will eventually repeat but would that start in the 10th month?
Answered by Robert Dawson.
A schedule for 5 employees working 2 at a time 2015-01-23
From Cynthia:
We have 5 employees and need only 2 to work on Saturdays. what is the best way to fairly schedule everyone. PS. from time to time a sixth person will be thrown in to make things really interesting.
Answered by Victoria West.
Scheduling meetings with pairs of people 2015-01-22
From Jacey:
I am trying to figure out a formula/system to pair up a list of people so they meet with each other every month, but they rotate who they meet with. Right now I have 13 people and I would like to just type in their names and then have the system put each person with someone else every month and rotate so no one gets the same person twice. Can this be done?
Answered by Victoria West.
Math in the universe 2015-01-21
From GA:
I want to find out examples in universe for this equation:
sin(x)/cos(x)=1 or tan(x)=1. I know the mathematical answer of pie/4 or 45 degree angle.

What I need to know is to find examples of this. I found an example in construction, for example, which is the shape of attic in the house.

I want to find examples in universe like in solar system, in quasars or jets coming out of them, in galaxies, in stars, in black holes, etc

Do you know of any such examples?

Answered by Robert Dawson.
The volume of a swimming pool 2015-01-21
From kirara:
Find the volume of water in a swimming pool with vertical ends and sides. The length measured at the water line is 50 ft. And the breadth is 20 ft. The bottom of the swimming pool is a plane sloping gradually downward so that the depth of the water at one end is 4 ft. And at the other end is 8 ft.
Answered by Penny Nom.
The height of a building 2015-01-20
From Emily:
A man 2m high observes the angle of elevation to the top of the building to be 70 degrees. And the angle of depression to the bottom of the building to be 19 degrees. How tall is the building?
Answered by Robert Dawson.
Largest cone in a sphere 2015-01-15
From Alfredo:
What is the altitude of the largest circular cone that may be cut out from a sphere of radius 6 cm?
Answered by Penny Nom.
An input/output table 2015-01-13
From Rose:
How do you create an input/output table when the rule is: the output is a +5?
Answered by Harley Weston.
$xx,xxx million 2015-01-12
From sherry:

as a financial editor, for years I've seen $\$$ amts like this:
$\$xx,xxx$ million (ie, with a comma, not dec point)
Isn't that thousands of millions, which would equal 'billions'?


Answered by Penny Nom.
An investment for 20 months 2015-01-12
From rose:
Darnell wants to have 2700$ to purchase a tv in 20 months. what will he need to deposit today if he earns 5% annual interest compounded monthly
Answered by Penny Nom.
h+h+h = -9 2015-01-12
From juanita:
How to solve h+h+h = -9
Answered by Penny Nom.
The length of a roll of paper 2015-01-12
From Peggy:
roll of paper 3ft wide and roll equals 500 sq ft how long would the roll be?
Answered by Penny Nom.
The volume of a lake 2015-01-10
From Peter:
If a lake is 8 acres in size and 10 feet deep how many gallons of water does it contain
Answered by Penny Nom.
Mutually prime composite numbers 2015-01-09
From nivedita:
Two composite number that is mutually prime?
Answered by Penny Nom.
Scheduling 12 for lunch 2015-01-05
From Sabrina:
I have 12 staff members that need to have lunch together 1 time each week. I am trying to pair them in a way that we each have the same amount of lunches in the same amount of weeks. Please Help! Thanks!
Answered by Victoria West.
The degree of a polynomial 2015-01-05
From Nichole:
How do I determine the degree of polynomials? I've searched this on sites but they are all so confusing! Is there a simple explanation or way to find what the degree is? Some examples are: 6x^4 10x^2yz^5 and 3m^2n^7-10m^8. I also have to say that I am under the impression that this symbol ^ means the number after it is an exponent.
Answered by Robert Dawson and Harley Weston.
A silo is composed of a hemisphere and a cylinder 2015-01-03
From Ami:
A silo is composed of a hemisphere and a cylinder. The total height for the silo is 20m and the diameter of the silo is 8m. What is the volume of the silo to the nearest cubic meter?
Answered by Penny Nom.
Filling a tub 2015-01-02
From Mr:
The cold tap can fill a bath in 5minutes,the hot tap in 15,and the waste pipe empty the full bath in 10 minutes. All three are fully open for 2 minutes,after which the waste-pipe is closed. How much longer will it take to fill the bath?
Answered by Penny Nom.
An isosceles triangle 2014-12-31
From ahmadmahmoud:
an isoseles triangle is such that each of the base angles is twice the vertical angle.Find the angles of the triangle
Answered by Penny Nom.
1.5% of 1 trillion 2014-12-30
From Kgosi:
How much is 1.5% of 1 trillion?
Answered by Penny Nom.
Rates, percentages and units 2014-12-30
From Kenneth:
Hello:

If percentages have no units, why are some percentages called rates, as in interest rate, or perhaps a tax rate of 7% as an example? A rate has units of different quantities.

I thank you for your reply.

Answered by Robert Dawson.
The perimeter of a quarter circle 2014-12-26
From Maisy:
This is the question that I got for a math worksheet.
The radius of a quarter circle is 5 miles. What is the quarter circle's perimeter?
Use 3.14 for pi.

Answered by Penny Nom.
A trip to Ottawa from Toronto 2014-12-22
From Kristy:
A trip to Ottawa from Toronto will take 4 1/2 hours. Assuming we are 2/3 the way there, how much longer will the trip take?
Answered by Penny Nom.
A tangent to a circle 2014-12-22
From azman:
Find the equation of the tangent line to the circle x^2+y^2=9 at the point P (2, square root of 5).
Answered by Penny Nom.
A line segment of length root 5 2014-12-15
From angela:
On the dot grid below, draw and label a line segment with length square root 5 the dot grid is 8 by 10
Answered by Penny Nom.
A cone is 2/3 full of sand 2014-12-14
From Janice:
A cone with a radius of 3.5 cm and a height of 12 cm is 2/3 full of sand.
What is the volume of the sand inside?

Answered by Penny Nom.
A right triangle 2014-12-13
From Katie:
the shorter leg of a right triangle measures 2 feet less than the longer leg. the hypotenuse measure 10 feet. find all three sides
Answered by Penny Nom.
Planar curves 2014-12-13
From ann:
what does planar curve mean in your definition of a cone?
Answered by Penny Nom.
A circle through 3 points 2014-12-04
From priyanka:
Find the equation of a circle passing through the origin and intercept on axes are a and b.
Answered by Penny Nom.
1999 equations in 1999 unknowns 2014-11-28
From Subrahmanya:
Solve the following system of 1999 equations in 1999 unknowns :

x1+x2+x3=0, x2+x3+x4=0……., x1997+x1998+x1999=0,

x1998+x1999+x1=0, x1999+x1+x2=0.

(in x1,x2,...................,x1999 the numbers 1,2,.............,1999 are subscripts)

Answered by Penny Nom.
Exponential notation 2014-11-27
From Jaqui:
May sound super easy, but how would you SHOW that you simplified 3x3 to the power of 2 (the little 2)?

thank you!

Jaqui

Answered by Penny Nom.
What is 20 and 3/5 of 10? 2014-11-26
From Jeannine:
What is 20 and 3/5 of 10? Question is written exactly this way. Is this 20 3/5 times 10 (103/5 times 10), or is it 20 and (3/5 times 10)?
Answered by Penny Nom.
Mark's book case 2014-11-24
From anne:
mark has 312 books in his case. he has 11 times as many fiction books as non fiction books. how many fiction books does mark have?
Answered by Penny Nom.
Cutting a hexagon with a sliding miter saw 2014-11-23
From Joseph:
I need the angles in degrees for a hexagon one side is 18inches on the outside the problem I am having is in the angle of the cut and I am using a sliding miter saw which only goes from0 degrees to 45 degrees so I need help in getting the angle of the cut
Answered by Harley Weston.
Doubling the dimensions of a cone 2014-11-21
From Hannah:
If the volume of a cone of height 10 cm is 261.8 cm3, show that this volume is increased by a factor of 8 if the dimensions of the cone are doubled.
Answered by Penny Nom.
Two quadratic equations 2014-11-20
From rajesh:
Sir,
I would like to discuss on question-
-Find the value of variable X and Y, equations are
X2+Y2+2XY=25
X2+Y2-2XY=1

Sir, my point is equation looks not so easy.My opinion is there many (more than 4)value possible for both X and Y.
Kindly help me.
Waiting for your response.
regards,
rajesh

Answered by Penny Nom.
An age related word problem 2014-11-18
From Cole:
Susan and Russell are brother and sister. Susan says that in two years she will be twice Russell's current age. Russell says that he's only three years younger than Susan. How old is each of the siblings?
Answered by Penny Nom.
How many square feet in 10 gallons of liquid at 1/4 in thick? 2014-11-14
From Rich:
How many square feet in 10 gallons of liquid at 1/4 in thick
Answered by Penny Nom.
(x-3)^2-(x+3)^2 2014-11-13
From Bernice:
(x-3)^2-(x+3)^2
Answered by Penny Nom.
1/6k=-11 2014-11-13
From Marty:
Using the balance method, what is the answer to 1/6k=-11, trying to find what k equals. Thanks
Answered by Penny Nom.
An irregular convex octagon 2014-11-13
From james:
I have an irregular convex octagon, alternating between 4 large edges, say 'a' mm long and 4 small edges, say 'b' mm long, is there a formula available so that I can work out the minimum size sit a circle with a radius of 34.25mm inside it thank you
Answered by Chris Fisher.
The area of a circle in terms of the circumference 2014-11-04
From Sarah:
How do you find the area of a circle if you already have the circumference? There was one answer to this question already but it didn't make sense to me, because they rounded up pi which you can't do so the answer is incorrect. thanks
Answered by Penny Nom.
A quadratic function modelling the heightof a flea jump 2014-11-02
From Hannah:
A flea jumps from the ground to a height of 30 cm and travels 15 cm horizontally from where it started. Suppose the origin is located at the point from which the flea jumped. Determine a quadratic function in vertex form to model the height of the flea compared to the horizontal distance travelled
Answered by Penny Nom.
Jason's speed 2014-10-31
From jason:
the speed of Jason is 12 times faster than everyone. The speed of Jason is 108mph. Write the equation and solve to find the speed of everyone else.
Answered by Penny Nom.
Prime factorization in exponent form 2014-10-31
From Emma:
I need to find out how to make a prime factorization of 120 in exponential form.
Answered by Penny Nom.
What is the speed of the boat in still water? 2014-10-30
From Adam:
The current in a stream moves at a speed of 2 km/h. A boat travels 38 km upstream and 38 km downstream in a total time of 3 hr. What is the speed of the boat in still water?
Answered by Penny Nom.
Two quadratic polynomials 2014-10-28
From khushboo:
The zeroes of a quadratic polynomial x^2+ax+b are 'c' and 'd' and the zeroes of a quadratic polynomial x^2+cx+d=0 are 'a' and 'b'. find the numerical value of a+b+c+d where a, b, c and d are non zero integers.
Answered by Penny Nom.
The region between two concentric circles 2014-10-27
From Ray:
two circles, concentric; Given the length of chord of outer circle that is tangent to inner circle. what are the areas of both? how to calculate?
Answered by Penny Nom.
logbase4 (sin x) = -0.5 2014-10-26
From Susan:
Solve for x: logbase4 (sin x) = -0.5
Answered by Penny Nom.
6 bolt holes around a circle 2014-10-26
From Bobby:
What Is The Cord Dimension For 6 Holes On A 24" Bolt Circule
Answered by Penny Nom.
Revolutions per minute 2014-10-24
From Edward:
Hello; I have a 28.2 inch diameter tire; do not worry about engine RPM or gear ratios, please tell me what the RPM of that tire is at 8 MPH and 64 MPH. Thank you.

Sincerely; Edward

Answered by Penny Nom.
A 10-foot piece of PVC pipe 2014-10-21
From David:
A plumber has a 10-foot piece of PVC pipe. How many 9/5-foot pieces can be cut from the 10-foot piece ?
Answered by Penny Nom.
Three consecutive odd integers 2014-10-17
From Maddy:
3 times the largest of three consecutive odd integers is 13 more than the sum of the other two. What are the integers?
Answered by Penny Nom.
Water in a spherical container 2014-10-17
From Natalie:
A spherical container with radius 10cm being filled with water such that the water level, h cm, in the container increase at a rate of 0.5 cm s^(-1). Show that the area of the horizontal surface of water, A cm^(2), in the container is given by A = π(20h - h^(2) ).
Answered by Penny Nom.
A poorly worded question 2014-10-14
From Marjorie:
How do I figure out the following? Dylan's dog weights 12 times more than his pet rabbit. The dog and the rabbit weight 104 pounds. How much does Dylan's dog weigh? Would you explain in detail how I would arrive at the answer so I can convey same to my grandson who is in the 5th grade. Thanks.
Answered by Penny Nom.
A linear system 2014-10-14
From pheter:
4/x - 1/y = 3 .... equation (1)
6/x - 2/y = 5 .... equation (2)

Answered by Penny Nom.
A fact family containing 4, 4 and 16 2014-10-14
From Leslie:
I am helping my son and the question is 4,4,16 in fact family what does that mean
Answered by Penny Nom.
How does pir^2 = 1/4pid^2? 2014-10-14
From al:
Hi I cant work out the algebra. How does pir^2 = 1/4pid^2 Thanx
Answered by Penny Nom.
Building an 8 sided box 2014-10-14
From Michael:
I want to build an 8 sided box. The North, East, South, and West sides I want them to be 4 feet in length.
The NE, SE, SW, and NW sides I want to be 2 feet in length. What angles do I need to cut my angles.
Thanks Michael

Answered by Penny Nom.
15 - (5-x)/ x = 30 2014-10-13
From AMANDA:
15 - (5-x)/ x = 30

I thought you had to multiply both sides by x in order to remove the fraction?

15x - 5x - x2 = 30x

then combine 10x - x2 = 30x

-x2 =20x

then take the sqrt of each side...

what am I doing wrong??

Answered by Penny Nom.
Pre algebra 2014-10-12
From mohamed:
i want to know the meaning of pre algebra
Answered by Penny Nom.
A quarter mile row of trees 2014-10-09
From junna:
a farmer would like to plant trees on one side of his lot which is 1/4 of a mile in a length His friend advised him that if he would plant the trees 5 feet farther apart,he would need 44 fewer trees.If the farmer would take the advice, how many trees would he plant?
Answered by Penny Nom.
Why express fractions in lowest terms? 2014-10-09
From John:
I teach in a Faculty of Education. A colleague in the university asked me about fractions and lowest terms. I am quoting the person below, and would appreciate your insights into the question/thought.

"One I'm struggling with is why (for example) 6/18 is not considered as good an answer to a fraction question as 1/3. The traditional response is that 6/18 is not in lowest terms so the question has not been finished until the fraction is reduced, but what actually makes the lowest terms answer the better one? Is it convention? Is there a way to explain why simplest form answers in fractions are right and and anything else is considered incorrect without alluding to some need for this 'good habit' elsewhere in math or science? Is there a real-life reason?" Thank you for any insights and if you have nothing to offer to this query, that is fine too. I appreciate your consideration. John

Answered by Robert Dawson.
One zero to a quadratic expression 2014-10-06
From Kevin:
Find a value of n such that f(x) = 3^x2 + nx + 6 only has one zero, is this the only solution to the question?
Answered by Penny Nom.
A tennis ball in a triangular prism 2014-10-06
From Anna:
Hi, I need to find the the base and the height of a triangular prism, but I only know the diameter of a tennis ball that fits exactly in the triangular container.. The diameter for the ball is 8.50cm. Can you help me find the formula? Thank you!
Answered by Penny Nom.
Trig functions and the unit circle 2014-10-02
From Jake:
I was wondering what conclusions can be drawn about the trigonometric functions and how they work about the circle. Can you also please give me an explanation for it? Thank you.
Answered by Penny Nom.
A lost combination 2014-09-28
From Kim:
I cannot remember my combo to this simple lockbox. It is 3 digits- from 0-9. They can be used more than once . I thought it was 117..but no joy. I am fairly sure it was using those digits 1, and 7. can you give me a list of all possible combinations, using 1s and 7s, for the 3 number code? Thank you and I am in awe of people who have math skills, for I certainly do not.
Answered by Penny Nom.
A rectangle and a triangle 2014-09-27
From rajesh:
The perimeter of a rectangle is equal to the perimeter of a right-angled triangle of height 12 cm if the base of the triangle is equal to the breadth of the rectangle, what is the length of the rectangle?
Answered by Penny Nom.
A wire is bent to form a right triangle 2014-09-25
From Lorena:
A wire 24 dm is bent to form a right triangle. The ratio of the shorter leg to the hypotenuse is 3:5. Find the length of the longer leg?
Answered by Penny Nom.
Two equations in x and y 2014-09-25
From seyilogo:
solve y=2x - 3 and (4x - 2y) / x + y = 1 simultaneously
Answered by Harley Weston.
Continuity on a closed interval 2014-09-21
From Pragya:
The trouble I'm having is as follows :
a continuous function is most of the times defined on a closed interval, but how is it possible to define it on a closed interval ,because to be continuous at endpoints of the interval the function's limit must exist at that endpoint,for which it has to be defined in its neighborhood,but we don't know anything about whether the function is always defined in the neighborhood.
Please help...

Answered by Penny Nom.
The perimeter of a parcel of land 2014-09-18
From Shirley:
What is the perimeter of a parcel of land that is 564 acres square
Answered by Penny Nom.
Meetings for 77 students 2014-09-18
From Adam:
We have a new group of students starting, and would like to set up a series of small group meetings so each student meets each other, hopefully without meeting the same people twice.

We have 77 students. In theory I would like to have 11 meeting slots, with 10 groups in each slot, with up to 8 students in each group. Is there a way to do this?

I have wondered if it might be easier to do 9 meeting slots, with 9 groups per slot, with 9 students per group. Thanks!

Answered by Victoria West.
Two equations in two variables 2014-09-18
From Susan:
(28x + 36y) - [20000 - .75(28x + 36y) + 60000] = 5000
x + y = 10000
solving two equations involving variables

Answered by Penny Nom.
How much did each dog weigh? 2014-09-16
From Fred:
LJ owned two dogs that weighed a total of 140 pounds. If one dog weighed 10 pounds more than the other, how much did each dog weigh
Answered by Penny Nom.
A tangent to a curve passing through a point not on the graph 2014-09-15
From Aquilah:
For the curve y = x2 + 3x, find the equations of all tangent lines for this graph that also go through the point (3, 14).
Answered by Penny Nom.
A six team track schedule 2014-09-15
From Paul:
I need to make a six (6) team track schedule. It needs to be five (5) triangular (3 team) meets with each team playing each team twice
Answered by Victoria West.
Two chords in a circle 2014-09-13
From Carlos:
Find the length of the radius of a circle in which a chord of length 6 units is twice as far from the center as a chord of length 12 units
Answered by Penny Nom.
The average of 9 numbers is 45 2014-09-12
From rizza:
The average of 9 numbers is 45. If 90 and 88 are added to the set of numbers. What will be the resulting average?
Answered by Penny Nom.
The average age of a group of 10 students 2014-09-11
From sujan:
The average age of a group of 10 students was 20. The average age increased by 2 years when two new students joined the group . What is the average age of the two new students who joined the group?
Answered by Robert Dawson and Penny Nom.
The perimeter and diagonal of a rectangle 2014-09-10
From niniha:
the perimeter of a rectangle is 68cm.if the diagonal is 26, find its dimensions
Answered by Penny Nom.
A second grade word problem 2014-09-09
From Beth:
How do I explain to my second grader how to find the answer to the problem:
My tens digit is 8 more than my ones digit. My ones digit can not be zero.

Answered by Penny Nom.
Problem solving 2014-09-08
From hubert:
a car left the house and travelled at an average speed of 60 kph.how many minutes will it take for the car to reach the school which is 8 km away from the house
Answered by Penny Nom.
A secret number 2014-09-08
From jai:
it is smaller than half of 100, it is more than 4 tens and less than 5 tens, the tens digit is two more than the ones digit , together the digits have a sum of 6..... What is my secret number?
Answered by Penny Nom.
The diagonals of a cube 2014-09-04
From Rukshanth:
How many diagonals does a cube have?
Answered by Harley Weston.
The sum of two numbers is 52 2014-09-01
From Blake:
The sum of two numbers is 52 and the difference is 10. What are the numbers? i used to be real good at this stuff?
Answered by Penny Nom.
Interior angles in a parallelogram 2014-08-30
From xavier:
so, one of my math homework questions requires me to know how to find out how to find the interior angles in a parallelogram, the question is, "how many interior angles does a parallelogram have?"
Answered by Penny Nom.
Water usage in Ames 2014-08-29
From Abigail:
The City of Ames water treatment plant treats 7 million gallons of water per day on average to meet water demands. Hypothetically, if water is withdrawn from a groundwater system with an area of 51.6 square miles (approximate area of Ames and surrounding area), what is the depth of water in inches that would have to be added to the groundwater system annually to replace the water that is withdrawn by the city?
Answered by Penny Nom.
Three collinear points 2014-08-28
From jhanavi:
p,q,r are three collinear points.p and q are(3,4) and (7,7) respectively and PR is equal to 10 units.find coordinates of R
Answered by Penny Nom.
Logarithms 2014-08-28
From effaryna:
how logarithms is applied in everyday life?
Answered by Penny Nom.
Using a bucket to fill a tub 2014-08-27
From Charlotte:
A bathtub needs to be filled with water. To fill the tub, a bucket needs to be filled then dumped into the tub. If the bucket is a cylinder with a radius of 6 in. and a height of 12 in, how many buckets will it take to fill the 5 ft x 3 ft x 3 ft tub.
Answered by Penny Nom.
Cutting a round cake so that it doesn't dry out 2014-08-26
From James:
I'm wondering if there's a simple way to calculate the area between two parallel chords of a circle equidistant from its diameter, or if I have the area, to find the distance between the two chords.
Here's my "problem". You may have heard of the way of cutting a round cake so that it doesn't dry out - make two parallel cuts (chords) the length of the cake, take the middle piece, then push the two pieces together.
So I know the area of a 12" cake, and I want say, exactly an eighth of the cake. How wide do I cut the centre piece? Now to get even more difficult, the next day I want another eighth from the centre. How wide do I cut the next pieces, and so on...? Thanks, James

Answered by Harley Weston.
A car passing a bus 2014-08-24
From Athish:
the driver of the car is travelling at 36 km/hr and spots a bus 80 m ahead of him after 1 hr the bus is 120 m behind the car what is the speed of the bus
Answered by Penny Nom.
The perimeter of a triangle 2014-08-21
From Phyllis:
What is the perimeter of a five acre triangle with one side being 250 feet and the second side would be at a 90 degree angle.
Thank you.

Answered by Penny Nom.
Filling three holes with stones 2014-08-20
From mark:
how many tonnes of hardcore/crushed stone would it take to fill 1 hole 9ft diameter 5ft deep and 2 holes both 3ft diameter and 5ft deep
Answered by Penny Nom.
Revisiting 8 golfers in two foursomes 2014-08-19
From steve:

I noticed you had an answer on how can you put 8 golfers in even foursomes over 7 games so that everyone plays with everyone equally. However the formula showed the following sequence that was used but did not go past the second day. Can you change the 0s and 1s into the numbers 1 to 8 instead. Here is what you have written:
Day 0 : (0, 0, 0, 0, 1, 1, 1, 1)
Day 1 : (0, 0, 1, 1, 0, 0, 1, 1)
Day 2 : (0, 1, 0, 1, 0, 1, 0, 1)
Day 3 : (0, 1, 1, 0, 1, 0, 0, 1)
Day 4 : (0, 0, 1, 1, 1, 1, 0, 0)
Day 5 : (0, 1, 0, 1, 1, 0, 1, 0)
Day 6 : (0, 1, 1, 0, 0, 1, 1, 0)

Thank you so much for your anticipated help.

Steve


Answered by Harley Weston.
A barge of triangular cross section 2014-08-18
From tushar:
a barge of triangular cross section is 20m long 12 m wide and 6m deep.its floats in SW at a draft if 4m find its displacement
Answered by Penny Nom.
4 rounds of golf as three foursomes 2014-08-18
From Christopher:
For our golf trip, we will be playing 4 rounds of golf as three foursomes. We would like to have each person play with everyone at least once. Is this possible?
Answered by Victoria West.
The equation of a circle 2014-08-14
From jennifer:
hi there My name is Jennifer and residing in Denmark. I am a student and I wrote to you because i am having trouble in finding out the equation for the circle using (x-a)^2 + (y-b)^2 =3D r^2.The diameter of this circle is d=3D 44,514 cm. I have attached a drawing of my problem..thanks
Answered by Penny Nom.
Distance, speed and time 2014-08-13
From abdel:
Mac walked to a park from the college at 5 km/h. 10 minutes later, Steve started riding his bike at 15 km/h to the same park to meet Mac. They arrive at the same time at the park. How far is the park from the college?
Answered by Tyler Wood.
Speed, distance, time 2014-08-13
From Aarti:
a theif running at 8 km/hr is chased by a policeman whose speed is 10km/hr.if the thief is 100 m ahead the policeman then what will b time required for the policeman to catch the thief?
Answered by Tyler Wood.
Golf for 10 2014-08-12
From Fred:
We have 10 golfers playing 4 rounds: one 4-some and two 3-somes each round. Can you suggest the best distribution of players?
Answered by Victoria West.
Golf for 15 2014-08-11
From Brian:
I am trying to arrange 5 rounds of golf for 15 golfers. We will play 5 in a group, 3 groups per round, 5 rounds. I would like everyone to play with everyone else. Secondary objective would be for no one to play with anyone else more than 2 times.

Thank you.

Answered by Victoria West.
Multiplication in base five 2014-08-07
From Alise:
Hi Maths Central!

I was wondering if you could help me with multiplying in base 5. I have read a question previously answered by you in regards to this but got fairly confused by it and was hoping you could help me with my question.

My question is: Multiply 1422 base 5 by 21 base 5 keeping these numbers and answer in base 5.

Would you also be able to explain how to get to the answer?

Thanks!

Answered by Penny Nom.
4821x14y is an 8-digit number divisible by 72 2014-08-06
From RAYA:
if 4821x14y is an 8-digit number divisible by 72. How many values can x and y take?
Answered by Penny Nom.
A chord and an arc of a circle 2014-08-06
From Luis:
Hello I am trying to figure out how to get chord lengths of quarter points for a circle. I work with large diameter circles and I need to be able to find chord length from 0 degrees to 90 degrees, 0 degrees to 270 degrees, 270 degrees to 180 degrees and 90 degrees to 180 degrees. Can you please use 100ft. diameter as example and work out the problem. I'm sorry if I use wrong terminology, I'm really bad at math . Thank you in advance for your time and help I really appreciate it.
Answered by Harley Weston.
The area of a quadrilateral 2014-08-06
From Rahul:
find the area of the quadrilateral whose side measure9cm,40cm,28cm,15cm and in which the angle between the first 2 sides is a right angle
Answered by Penny Nom.
The dimensions of a rectangle 2014-08-03
From Jamaica:
if the perimeter of a rectangle is 30m and its area is 56 m2.find its length and width?
Answered by Penny Nom.
A 6 team sports schedule 2014-08-02
From Daniela:
Hello,

My husband and I are volunteering at our church's Jrs camp and we were given the task of making the schedule for sports. There will be 6 teams and a total of 3 games for two days. We would like for every team to play each other but we cannot figure it out. Pls help!

Thanks.

Answered by Victoria West.
A word problem with fractions 2014-08-01
From nathanielcabalan:
The denominator of a fraction is one more than the numerator. If the numerator is increased by three, the resulting fraction is one more than the original fractional. Find the original fraction.
Answered by Penny Nom.
A word problem concerning averages 2014-07-26
From Bernie:
The average weight of a team of children is 50kg, Todd who weighs 56 kg then joins the team. The average weight of the new team is 51. How many children in the original team?
Answered by Penny Nom.
60% as much as $30. 2014-07-18
From Kenneth:
Question from Kenneth:

Hello:

Do you know why some "as...as" phrases indicate multiplication?

For example, ? is 60% as much as $30. This is equivalent to 60% X $30 = $18. How does this make sense?

I saw this example in an old textbook on business mathematics, but the author did not explain why it is equivalent to multiplication.

I thank you for your reply.

Answered by Robert Dawson.
Selecting a meal at a school cafeteria 2014-07-18
From Gil:
A school cafeteria offers 2 main entrees, 3 fruits, 4 vegetables, and 3 drinks. If a plate consists of 1 entree, 2 vegetables, 1 fruit, and 1 drink, how many different plates can a cafeteria manager make? Please explain how you derived at answer. Thank you
Answered by Penny Nom.
Expand and simplify [x-3][x+3] 2014-07-16
From symion:
Expand and simplify [x-3][x+3]
Answered by Penny Nom.
An octagon around a tree 2014-07-16
From floyd:
I want to build an octagon 4 feet from base of tree from 4x4's. What length do I cut my pieces
Answered by Penny Nom.
16 golfers over 15 weeks 2014-07-11
From Steve:
I need a schedule for 16 golfers over 15 weeks, where each golfer is paired with a different golfer each week and plays 4 matches per week against 4 different pairs (each pair does not have to play each week)
Answered by Penny Nom.
A 14 team golf league 2014-07-11
From Pat:
We have a 14 team golf league that plays a 16 week schedule. Playing once per week, I need a schedule where each team plays every other team at least one time.
Answered by Victoria West.
A 5 sided lot 2014-07-10
From mia:
have the feet numbers of three sides, but the forth side is in a "V" so it has two figures. Question: can i add the two figures of the "v" together to get one figure, so then I would have the figures for all four sides.
Answered by Harley Weston.
4/9 of a set of story books cost Rs 630. 2014-07-09
From YASIN:
If 4/9 of a set of story books cost Rs 630. What will the rest of the set cost?
Answered by Penny Nom.
A ladder against a wall 2014-07-09
From thabo:
A ladder 6.5m long,leans against a wall so that the top of the ladder is 4.8m from the ground.what is the angle of elevation of the ladder to the top of the wall
Answered by Penny Nom.
A rotation schedule for 14 teams 2014-07-08
From Daniel:
I am organizing team activities for a summer camp. We have 14 teams and 7 rotations and 7 different games. every team will play a different game each rotation. Where I am having trouble is I would like each team to play a different team each rotation without a repeat. Is that possible? if so what is the formula?
Answered by Victoria West.
14 golfers, 7 men and 7 women 2014-07-06
From John:
I have to schedule 14 golfers, 7 men and 7 women, into 4 teams each day, playing for 3 days. So each player plays with as many different players as possible for 3 games.
Answered by Victoria West.
An antenna is atop a building 2014-07-05
From Andrew:
If an antenna is atop a building 70 feet tall and the antenna has a beamwidth of 30 degrees how far from the building does the beam first touch the ground?
Answered by Penny Nom.
The method of elimination 2014-07-05
From leo:
please explain how can i solve this problem

3x-6y=-38
6x-9y=44

using elimination and simultaneous method thank you :)

Answered by Penny Nom.
Quarters 2014-07-04
From kamal:
how many quarters in the sum of 8 and 12 ?
Answered by Penny Nom.
Sum of 22, product of 125 2014-07-03
From marj:
find two real numbers whose sum is 22 and whose product is 125.
Answered by Paul Betts, Robert Dawson and Penny Nom.
The width of a rectangle 2014-07-02
From john:
if the area of a rectangular field is 90 sq. m and its length its is 19 m find its width
Answered by Penny Nom.
Covering a 12 inch by 12 inch square hole 2014-07-02
From Patricia:
I am putting in a new bathroom fan. I am wondering if a new light with a 15 inch diameter will cover the existing square hole which is 12 by 12 inches? If the existing hole is 11 1/2 by 11 1/2 inches?

Also, if the 15 inch diameter does not cover the 12 by 12 hole, what size diameter would?

Thank you.

Answered by Penny Nom.
A six digit number 2014-06-30
From anupam:
hi,
my daughter asking a solution for the below question, kindly help.

Write the smallest 6 digit number wit digits ( 2,4,3,0,5,7).

plz this is an urgent requirement.

regards,
anupam

Answered by Penny Nom.
Angular speed 2014-06-29
From andrea:
a wheel having a radius of 10cm rotates such that the linear speed at its rim is 30mls. what is the angular speed of the wheel in rpm?
Answered by Penny Nom.
Differentiate ln[x(2x-4)^1/2] 2014-06-28
From Igwe:
If y=In[x(2x-4)^1/2],find dy/dx at x=3
Answered by Penny Nom.
A triangle on the surface of the Earth 2014-06-26
From Christine:
A, B and C are three towns, the bearing of B and C from A being 310 degrees and 220 degrees, and their distances from A are 510km and 700km respectively. Find the bearing of B from C to the nearest minute.
Answered by Robert Dawson.
The largest odd number using the digits 08672 2014-06-24
From audrey:
what is a 5 digit greatest odd number out of this given digits? 08672
Answered by Robert Dawson.
12 golfers 2014-06-23
From Derek:
I have a golf group that is set up as follows.12 players broken into 2 teams of 6.We play as a 4 ball ie 2 from each group.I am looking for a formular to have the 2 players from each group not play with each other again & also both of the 2 in each group not play with others in the other group more than once.I don't believe it is possible,so will accept as little duplication as possible-many thanks-Derek
Answered by Victoria West.
An oval pool 2014-06-21
From steve:
I have a 16' x 28' oval pool that is buried 24" deep inground. The dig site is dug 2' wider all the way around the pool. I need to back fill this area with stone. I want to fill this area with 6 to 8" of stone. How many tons of stone will this take?

Thanks you
Steve

Answered by Penny Nom.
Simultaneous equations 2014-06-20
From rana:
solve the simultaneous equations
a)3x=7y
12y=5x-1

Answered by Penny Nom.
How many kilometers to a longitudinal degree? 2014-06-19
From cherrielyn:
Assuming that earth is a sphere of radius 6380 km, what is the difference in the latitudes of two cities 270 miles apart positioned on the same meridian?

Thank you in advanced po! :)

Answered by Robert Dawson.
A reversed curved on a railroad track 2014-06-19
From cherrielyn:
Assuming that earth is a sphere of radius 6380 km, what is the difference in the latitudes of two cities 270 miles apart positioned on the same meridian?

Thank you in advanced po! :)

Answered by Penny Nom.
Two regression lines 2014-06-14
From catherine:
Given a set of data values ,we can get two regression lines .Explain.
Answered by Robert Dawson and Penny Nom.
A model of the sun and the Earth 2014-06-14
From Lynn:
There is a model of the sun that is 2 1/2 inches in diameter. This is the only clue we have! How far away is the earth from this model?
Answered by Penny Nom.
A small solid silver pyramid 2014-06-13
From Jenesis:
For Awards Night at Baddeck High School, the math club is designing small solid silver pyramids.The base of the pyramids will be a 2 in by 2 in square. The pyramids should not weigh more than 2.5 pounds. One cubic foot of silver weighs 655 pounds. What is the maximum height of the pyramids?
Answered by Penny Nom.
The area of a circle of circumference 32.69 meters 2014-06-11
From coco:
Find the area of a circle with a circumference of 32.69 meters.
Answered by Penny Nom.
Forming a cylinder from a square 2014-06-08
From kalyani:
the curved surface of a cylinder is a square of diagonal 2*square root of 2. What is the area of the base of the cylinder?
Answered by Penny Nom.
Another conditional probability problem 2014-06-08
From PAVITHRA:
Two dice are thrown. What is the probability that sum of the numbers appearing on the die is eleven, if five is appear on the first die
Answered by Penny Nom.
Bags of sand 2014-06-06
From Robert:
We have a section that is 20ft long by 14ft wide and 4inches deep, the bags of sand are .5 cubic ft. How many bags of sand do we need to fill this space evenly.
Answered by Penny Nom.
Factorials 2014-06-06
From penny:
What is factorial? For eg. Like 2!, 3! 4! Etc.
Answered by Penny Nom.
20 teams and 10 competition stations 2014-06-04
From Joel:
I have 20 teams. There are 10 competition stations. Each station will accommodate exactly 2 teams to compete with each other at a time.

Each of the 20 teams needs to compete at each of the 10 stations exactly one time.

Here's the catch I cannot solve: how to schedule it so that each team plays EVERY OTHER TEAM once and only once, while never repeating a station.

Answered by Victoria West.
10 golfers, playing 7 rounds in 2 x 3balls and 1 x 4ball 2014-06-04
From adrian:
Hi, We are off to Scotland on our yearly golfing trip soon. Question is we have 10 golfers, playing 7 rounds in 2 x 3balls and 1 x 4ball. I would like to ensure that each golfer plays with the other guys an equal amount of times, guess 3ish? and we share the amount of times each player plays in the 4ball. Hope you can help?
Answered by Victoria West.
How do we use logarithms in our daily life 2014-06-01
From ss:
how do we use logarithms in our dailylife
Answered by Penny Nom.
6 inches of dirt in a baseball field 2014-06-01
From Mary:
The baseball field is 20,000 Square feet, 6 inched deep I need to know how many cubic yards of dirt this will generate.
Answered by Penny Nom.
How do I make fractions into decimals? 2014-05-31
From jay:
Hi,
I wanted to know how do I make fractions into decimals example 1 3/8?

Answered by Penny Nom.
An equilateral triangle with vertices in 3 parallel planes 2014-05-31
From prajay:
How to construct an Equilateral Triangle whose vertices lie on three parallel lines, if the distances of two lines are 'a' and 'b' units from the middle line.What is the length of the side of the Equilateral Triangle ?
Answered by Chris Fisher.
A word problem with fractions 2014-05-30
From shadab:
A container of milk is 4/5 full. When 10l milk is poured into it, the container becomes 9/10 full. What is the capacity of the container ?
Answered by Penny Nom.
A rotating schedule for 9 individuals 2014-05-29
From Josh:
I need to set up a rotating schedule for 9 individuals. The individuals will be pairing each week in groups of two. Thus, every week there will be 4 groups of two individuals and one individual not paired with anyone (i.e. a "bye week"). The rotating schedule should not repeat any pairing and it will continue until each individual has been paired with every other individual once.
Thank you

Answered by Victoria West.
A word problem with fractions 2014-05-23
From Tanzeela:
There are 1200 pupils in a school. 2/3 of them are girls. 1/4 of the boys are overweight. How many boys are overweight?
Answered by Penny Nom.
Overlapping sets 2014-05-23
From daniel:
motors inc manufactured 325 cars with automatic transmission,216 with power steering ,and 89 with both these options. How cars were manufactured if every car has at least one option?
Answered by Penny Nom.
The volume of a wedge 2014-05-21
From steve:
I need to figure the volume of a wedge / right triangle, the dimensions of the right triangle is L1(10'-5"), L2 (10'-6"), H (14'-9.5") height of L1 is 0'-0" height at intersect L2/H is 3'-3"
Answered by Penny Nom.
The volume of hexagonal pyramid and hexagonal prism 2014-05-18
From visshwam:
how to find volume of hexagonal pyramid and hexagonal prism? my measurements are each side of pyramid's triangle is 1"x2"x2".

hexagon prism is 2"x1" each side

Answered by Penny Nom.
14 golfers for 13 weeks 2014-05-17
From Judi:
I want to schedule each of my 14 golfers to play each of the other girls one time during a 13 week period. We would golf in foursomes , threesomes or twosomes to make this happen. Can you send me a grid that would do this scheduling? I'm at a loss as to how to do it!!
Answered by Victoria West.
The weight of salt in a basket 2014-05-17
From Sumit:
I want to determine how many kg of salt or XYZ thing can be filled in a basket
Length = 4.5"
Breadth = 5"
Height = 6"

Even its formulae

is kg quantity can be different for semi solid thing

Answered by Penny Nom.
The equation of a line 2014-05-16
From Michael:
Find the equation of a line that passes through (2,-1); the sum of the x- and y-intercepts is 2. (There are two answers)
Answered by Penny Nom.
A hexagon with different side lengths 2014-05-16
From Grace:
Sorry if this is a simple question, but I can't seem to understand how to solve it. I am Trying to find the area of a hexagon with measurements 6x7x6x6x7x6, or 2 sets of opposing sides with 6, and one set with 7. Thank you.
Answered by Penny Nom.
Golf for 14 players over 12 weeks 2014-05-16
From Judi:
How would I develop a playing schedule using foursomes and one twosome for 14 players who will play 12 weeks and each person would play all but one of the others?
Answered by Victoria West.
A problem with numbers 2014-05-14
From Justina:
Okay so I don't understand how to equal this equation to 10 using bedmas. I've been stuck on it for a few days now. Is there different ways you can show me Using these same numbers?

5 2 4 3 1 = 10

Help! Thanks.

Answered by Robert Dawson and Penny Nom.
18 golfers for 5 rounds with a teaching Pro 2014-05-14
From Maree:
We are taking 18 golfers on Tour with a teaching Pro. There are 5 rounds of golf and we want everyone to play with the Pro once, but everyone else to play with as many different people as possible
Answered by Victoria West.
A circle a square and a rectngle 2014-05-12
From mazhar:
suppose the length and breadth of the rectangle are 5 cm and 10 cm respectively and M is a point along the corner of the circle. what is the radius of the circle?(diagram is given..but i didn't mention it..actually the diagram looks like a circle inscribed in a square and the right bottom corner one rectangle will be given ,it is touches to circle at a point M that I've already mentioned and the dimensions of that rectangle also I've mentioned) please help me out..
Answered by Penny Nom.
Travelling upstream and downstream 2014-05-12
From Mohanad:
Howie Sorkin can travel 8 miles upstream in the same time it takes him to go 12 miles downstream. His boat goes 15 mph in still water. What is the rate of the current?
Answered by Penny Nom.
A 12 week poker tournament 2014-05-12
From Jamie:
Looking to schedule a 12 week poker tournament with 18 players ...playing 2 separate 9 person tables...I'm looking to have a balanced schedule where everyone player with each other an equal amount of time
Answered by Victoria West.
Mary's height as a percentage of Michael's height 2014-05-11
From Lauren:
Mary is 1.4m tall and her brother Michael is 1.2m tall.
Express Mary's height as a percentage of her brother's.

Answered by Penny Nom.
The length of a shadow 2014-05-09
From vijay:
A girl of height 80 cm is running away from the base of a lamp-post at a speed of 2 m/s. If the lamp is 4 m above the ground, find the length of her shadow after 5 seconds.
Answered by Penny Nom.
A question concerning two lines 2014-05-09
From Hela:
The equation of the line L is 6x+5y=3, and the equation of line Q is 5x-6y=0, which statement below about the line is true?
A) same y intercept
B)parallel
C) Same x intercept
D)perpendicular

Answered by Penny Nom.
How many boards did the carpenter start with? 2014-05-08
From Yanly:
A carpenter has several boards of equal length. he cuts 3/5 of each board. After cutting the boards, the carpenter notices that he has enough pieces left over to make up the same length as 4 of the original boards. How many boards did the carpenter start with?
Answered by Penny Nom.
A schedule for 2 teams of 10 2014-05-07
From John:
How can I set up a schedule for 2 teams of 10 on a team to play each individually against each other over a 10 day span at 10 different course with 10 different start times each day. I was looking to not repeat any competition. As an example, Team A would consist of players 1-10, Team B would consist of players 11-20. I would want to have 10 start times a day for 10 days where a member of team A plays a member of team B each day without repeating any matches and any of the same courses and not repeating any of the same start times. Any help in this would be GREATLY appreciated!!!!
Answered by Victoria West.
A 5-person golf flight 2014-05-05
From Jeremy:
can you have a 5-person golf flight and play only 3 matches so that one person plays one match each?
Answered by Victoria West.
Golf for 8 2014-05-05
From Ivan:
We have an eight member golf team. We field one foursome each week for 16 weeks of league play. How do I roate the eight members so each team member plays with each team member and all play an approximately equal number of times?
Answered by Victoria West.
4 golfers in twosomes 2014-05-04
From Lori:
4 golfers want to play as 2 somes every week, but rotate the 2 players so we all play together With each other evenly thru out the summer
Answered by Victoria West.
Golf :15 players over 4 days 2014-05-02
From michael:
We are playing with 15 players over 4 days. We plan to have 3 people in each group. A total of 5 groups with 3 players each. Can I arrange for each player to play with other players only 1 time?
Answered by Victoria West.
1÷[1-√2(order of surd is 4)] 2014-05-02
From Anoushka:
if t=1÷[1-√2(order of surd is 4)] , then t=?
Answered by Penny Nom.
11 golfers going on a golf trip for 4 rounds 2014-05-01
From David:
I have 11 golfers going on a golf trip for 4 rounds of golf. I want to make sure that everyone gets a chance to play with everyone at least once but not more than twice. Also, as we are n = 11, we will be composed of 3-4-4 everyday. I am trying to ensure that as few as people as possible play in the group of 3 (golf seems to be more enjoyable for whatever reason in foursomes). i have tried 3 differemt times and can seem to figure it out. Can you assist?
Answered by Victoria West.
2 teams of six playing each other 4 times 2014-05-01
From Steve:
I have 2 teams of six (Team A & B)- playing each other 4 times.

Is it possible to have a player from Team A play with a different partner from Team A for each of the 4 rounds, and likewise for Team B. Whilst no player from either team plays an opponent more than twice during the 4 rounds?

Answered by Victoria West.
If x+y+z=1 then x^2 + y^2 + z^2 = ? 2014-04-30
From Nitin:
If x+y+z=1 then x^2 +y^2+z^2 = ? ?
Answered by Robert Dawson.
Profit and loss 2014-04-29
From Anoushka:
A man sells two tables of the same price. On one he makes a profit of 10 percent and on the other he suffers a loss of 10 percent.Find his loss or gain percent on the whole transaction.
Answered by Robert Dawson.
A triangular chicken pen 2014-04-27
From Cierra:
Margaret has two lengths of fence, 20 meters and 24 meters, for two sides of a triangular chicken pen. The third side will be on the north side of the barn. One fence length makes a 75° angle with the barn. How many different pens can she build if one fence is attached at the corner of the barn? What are all the possible lengths for the barn side of the pen?

Not sure what they are asking here... please show step by step what to do! Thank you so much!

Answered by Penny Nom.
The volume and radius of a sphere 2014-04-27
From grace:
How do you find the radius of a sphere when all you know is the volume?
Answered by Penny Nom.
The derivative of sin(x) 2014-04-26
From Lucky:
f(x)=Sin(x), by first principle its f'(x)...show me how to solve such problem.
Answered by Penny Nom.
A schedule for 9 teams on 3 fields for 18 days 2014-04-25
From Scott:
I need to schedule 9 teams on 3 fields for 18 days with no buys. Two teams play while other practices in field(little guys). Two fields have backstops and other is open field. Would like to have even time on each diamond if possible. Thank you
Answered by Victoria West.
Solve for theta 2014-04-25
From ALASTAIR:
Hi, The question asks Solve for 'theta' cos2theta=sintheta x costheta. Substituting for cos2th either 1-2sin^2th or 2cos^2th does not give an equation in either sine or cosine alone how do I solve this please?
Answered by Penny Nom.
An octagonal pad 2014-04-25
From George:
Hi, I need to pour a cement pad in the shape of an octagon that allows for 12" of clearance around the tank I will be putting on it. The tank has a radius of 16'.
Answered by Penny Nom.
A trig problem 2014-04-25
From srishti:
Consider the points P= (-sin(a-b),-cosa), Q=(cos(a-b),sina), R=(cos(a-b+c),sin(a-c), where 0
Answered by Chris Fisher.
What was the selling price? 2014-04-25
From Con:
Apple built 40 32GB wifi iPads, gave away two in a contest, and sold the rest at two times the manufacturing price. If the company's total profit was $10,782, what is the selling price of one of these iPads?
Answered by Penny Nom.
What time was it when joe's brother passed him? 2014-04-25
From Nathan:
joe left home in his bike at 10:00 am, traveling 21 km/h. At noon, his brother set out after him on his motorcycle, following the same route. if the motorcycle traveled at 63 km/h, what time was it when joe's brother passed him?
Answered by Penny Nom.
Two 9 hole golf leagues 2014-04-23
From Sally:
I need help scheduling two 9 hole leagues. Here are the variables:
- Leagues alternate each week starting on the front and back
- League A starts at 4:30 with 7 groups and League B starts at 5:15 with 8 groups (all use carts and must be teed off by 5:30)
- Regular play must continue, customers who have started before league play begins have preference to continue their round whether they are playing 9 or 18 holes.
Thank you!!!

Answered by Victoria West.
Golf for 6 2014-04-21
From barbara:
We are a group of 6 golfers. We play in groups of 3 for 5 days. How can I arrange the groups that everybody plays with everybody else at least once? Thanks.
Answered by Victoria West.
Golf with 3 teams of 20 2014-04-21
From Joshua:
Thank in advance for this great service

Golf - new - 3 teams of 20 - to create 20 unique groups of 3

For example use 3 teams of 3

1a 2a 3a
1b 2b 3b
1c 2c 3c

So groups are
1a 1b 1c
2a 2b 2c
3a 3b 3c

Groups can only have one a, one and one c - and that combo should be unique

Answered by Victoria West.
A 22 team golf league 2014-04-21
From Terry:
We have a team league at our local golf course. We have 22 teams, we would like it so that each team plays the other team only once. I have tried the scenario.
1,21;2,20;3,19;4,18;5,17;6,16;7,15;8,14;9,13;10,11;12,X X+1=1 scenario
2, 1;3,21;4,20,5,19,6,18;7,17;8,16;9,15;10,14;11,13;1,X
using formula from other post in your forums.

I can get all the possible playing arrangements. The big quandary is setting up the schedule so that each week, each team would start on a different hole.

I can get the first 11 weeks scheduled very easily, it is just trying to get the last 11 weeks scheduled?

Any help would be greatly appreciated.

Answered by Victoria West.
Golf League Schedule for 15 weeks and three flights 2014-04-21
From bigdog:
I need help setting up a 12 team (3 flights with 3 teams in each flight) 15 week league play schedule. Each team must play teams in there perspective flight twice during the 15 week season.
Answered by Victoria West.
A normal distribution problem 2014-04-19
From Melanie:
This is the question:
The lifetime of a certain type of car tire are normally distributed. The mean lifetime of a car tire is 40,000 miles with a standard deviation of 5,000 miles. Consider a sample of 10,000 tires. A) How many tires would you expect to last between 35,000 and 45,000 miles? b) How many tires would you expect to last between 30,000 and 40,000 miles? c) How many tires would you expect to last less than 40,000 miles? d)How many tires would you expect to last more than 50,000 miles? e) How many tires would you expect to last less then 25,000 miles? f) What tires would you want on your car and explain your reasoning Not at all sure that we've done any of this correctly and not sure how to determine how many tires will last less than 25,000 miles. Any help is appreciated.

Answered by Penny Nom.
An input output table 2014-04-19
From Stephanie:
I have an input output table with x/y filled out except one x and one y. I need help. How do I find X?
Answered by Penny Nom.
Simultaneous equations with fractions 2014-04-19
From Maryam:
I looked at your example of simultaneous equations with fractions and applied it to my question from an educate exam papers but I couldn't get it to work. The question is:

x/8 - y = -5/2
3x + y/3 = 13

Answered by Penny Nom.
Robyn and Shaun swim 2014-04-18
From kajinthan:
robyn swims four 35m lengths.shaun swims eight 15m widths. how much further does robyn swim?solve this maths
Answered by Penny Nom.
A frustum of a pyramid with a square base 2014-04-18
From tuba:
a pyramid has a base of 10 m and is 15 m high.what is the volume? if 6m is removed from top what is the volume of the remaining frustum?
Answered by Penny Nom.
A circle is divided into three sectors 2014-04-17
From atolagbe:
the area of a circle is 154cm square. it is divided into three sectors such that two of the sectors are equal in size and the third sector is three times the size of the other two put together. calculate the perimeter of the third sector. take pi=22/7?
Answered by Penny Nom.
Barrels of oil 2014-04-16
From daryl:
Looking for an accurate number to use when calculating barrels per inch in a a tank with the following dimensions:
Nominal diameter - 15'-6", nominal height - 16ft, nominal capacity - 500 barrels. These are commonly referred to as 500x16 tanks.
Currently we are using 2.6 barrels per inch. We need 180 barrels for a full load so I'm trying to figure out how many total inches I need to load 180 bbl. This has been a matter of debate for some time. Thanks for your help.

Answered by Harley Weston.
The area bounded by the X-axis and y=x^(2)-4 from -5 to 0 2014-04-15
From Lexie:
Determine the area that is bounded by the following curve and the x-axis on the interval below. (Round your answer to three decimal places)

y=x^(2)-4, -5 ≤ x ≤ 0

The answer is 32.333 but I have no idea how to get there.

Answered by Penny Nom.
A tangent of the curve (x/a)^n+(y/b)^n =2 2014-04-15
From sudhir:
the equation of tangent of the curve (x/a)^n+(y/b)^n =2. at(a,b) is
Answered by Penny Nom.
A conditional probability problem 2014-04-13
From Chiluya:
Two fair dice are rolled. what is the probability that the number on the first die was at least as large as 4 given that the sum of the two dice was 8?
Answered by Penny Nom.
The cost before the sales tax 2014-04-13
From Juanda:
Hello,

I know the customer cost with tax and I know the sales tax.
How do I find out the customer cost prior to the added sales tax?

Thank you

Answered by Penny Nom.
A sand trench around a pool 2014-04-13
From steve:
How sand is needed to back fill a trench around a 24ft dia. pool that is 26" deep by 2ft wide. Making the outside dia. 26ft.

Thank You

Answered by Penny Nom.
Manipulating an equation 2014-04-12
From Paul:
S=4wl+2wh and solve for h
Answered by Penny Nom.
A slab of wet clay 2014-04-12
From Karen:
How many square feet ( at 1/4 inch thickness) are in 100 lbs. of wet clay?
Answered by Penny Nom.
A golf league with 20 players over 17 weeks 2014-04-12
From Chris:
I have a golf league 20 players and 17 weeks. We don't have teams so I just want to have everyone to play with each other the about same number of time and, hopefully, randomly distributed throughout the season. We have had situations in the past where two people play together for three weeks in a row and then not anymore for the rest of the season. From other posts, I see how to schedule the first 5 weeks without repeats but I can’t figure out an algorithm to for the rest of the season. Thanks!
Answered by Victoria West.
Decimals,fractions and percentages 2014-04-11
From Frances:
Ask an expert to find out their real life usage of your topic... that is the question. meaning what is your real life usage of using decimals,fractions and percentages. This is all part of my math assignment to ask an expert real life usage... so please reply back as soon as possible
Answered by Penny Nom.
A golf league consisting of 14 two man teams 2014-04-10
From Kevin:
Hello, I need to make a schedule for a golf league consisting of 14 two man teams playing over the course of 20 weeks. I understand that each team will play at least once and I believe 7 teams will play each other twice over the course of the season. How would I generate this schedule. Thank you, Kevin Perry
Answered by Victoria West.
How fast did Jessica grow? 2014-04-10
From jasmine:
when she was 14 jessica was 5 feet, 4inches tall . at 18 she is now 6 feet, 2inches tall .how fast did jessica grow during this time
Answered by Robert Dawson.
The surface area of a circular dome 2014-04-10
From Shafiqah:
Is this a dome's surface area formula??
{{2 × π r × h square units}}
Is the surface area of the floor for the dome is calculate too in this formula?

Thanks for answering. =)

Answered by Robert Dawson and Penny Nom.
Two rectangular prisms 2014-04-10
From hannah:
two rectangular prisms have a combined volume of 432 cubic feet Prism A has half the volume prism b has what are the volumes of prism a and prism
Answered by Penny Nom.
Two cones 2014-04-09
From c.j:
what is the length of the radius of the LARGER cone(the LARGER cone has a slant height of 15) when the SMALLER cone has a radius of 8 and a slant height of 12ft ,please help.
Answered by Penny Nom.
Two circles that touch each other externally 2014-04-08
From Ameya:
Two circles of radii a and b (a > b) touch each other externally. ST is a common tangent touching the circles at S and T respectively, then ST^2 is equal to
Answered by Chris Fisher.
The surface area of a rectangular prism 2014-04-08
From Manraj:
Find the surface area of a rectangular prism with dimensions 4, 5, and 7 units.
Answered by Penny Nom.
The sides of a triangle 2014-04-06
From Michael:
I am supposed to sove for the length of side "b" of an irregular triangle. I am given the following:
Side a: 65'
Side b: Find this length
Side c: 50'
Angle A: unknown
Angle B: unknown
Angle C: 52 degrees
I am supposed to use the law of cosines to solve for side "b" and my teacher says there is no mistake in the "givens" for the problem. I do not see how this can be done using the law of cosines and i have not figured out how to sove for angle B to use the law of cosines.

Answered by Penny Nom.
Cutting a hexagon from a disk 2014-04-05
From Paul:
I am a machinist and sometimes need to make a hex from round material.
If I know the distance of the flat sides opposite one another of my hex, how can I calculate the size of material I need to turn to give me the right diameter to finish the part with six sides?

Answered by Penny Nom.
A 4 digit number 2014-04-04
From LIM:
"A" is a 4 digit number formed by all the numbers from 1 to 4. When "A" is divided by 9, the remainder is the biggest possible value. What is the biggest value of A?
Answered by Chris Fisher.
The locus of a point 2014-04-04
From srishti:
A point P moves such that the difference between its distance from the origin and from the axis of x is always a constant c . what is the locus of the point?
Answered by Penny Nom.
3/4lbs of nails fills a container 2/3 full 2014-04-04
From Tyler:
If 3/4lbs of nails fills a container 2/3 full, then how many lbs of nails will fill the container?
Answered by Penny Nom.
Golf for 8 playing 7 rounds 2014-04-03
From Evan:
We are going on a golf trip to Scotland and are trying to work out the pairings. There are 8 of us playing 7 rounds. We would like to be partnered up in 2's with everyone once and have evenly split up foursomes. Thanks for the help
Answered by Victoria West.
Evaluate x-(3x+5) if x=-2 2014-04-03
From Destiny:
I wanted to know the value of x-(3x+5) if x=-2
Answered by Penny Nom.
Shadows of a father and son 2014-04-02
From esada:
the father and son cast a shadow of 11 feet and 8 feet,respectively if the son is 4'8'' in height,how tall is the father?
Answered by Penny Nom.
Is this a right triangle? 2014-04-01
From amina:
decide whether or not the following points are the vertices of right triangle (-9,2),(-1,-2),(-9,11)
Answered by Penny Nom.
A schedule for an 8 man golf team playing 4 each week 2014-04-01
From Harold:
We have an 8 man golf team and wish to schedule only 4 to play each week and to have each member play with one of the other 7 at least once. What is the simplest method. Number of weeks could be up to 18 or so.
Answered by Victoria West.
A cubical tank 2014-04-01
From renzo:
A cubical tank contains 2048 cubic ft of water when HALF-FULL. Find the total area of the tank and the length of its diagonal
Answered by Penny Nom.
Reducing the size of a poster by a scale factor of 1/3 2014-04-01
From deeshon:
eric is making a smaller copy of a poster that measures 20.4 cm wide by 34.2 cm long.he reduces the original by a scale factor of 1/3.what are the dimensions of the copy
Answered by Penny Nom.
Golf with 20 players and one compulsory bye 2014-03-31
From Wayne:
I'm trying to run an event with 20 players, I would like if at all possible to limit amount of people playing repetitively together. (Mix it up and play with different people). Here's the conditions. I want groups of 4 over 5 rounds. On each round I want 4 players to drop out and have a bye (to help run the game) but I dont want them helping a group they've played in if I can help it. Is this possible ?
Answered by Victoria West.
A schedule for 6 people 2014-03-29
From John:
How do I set up a schedule where six people are here for ten of twenty days. Arranged in rotating groups of three, so everyone works with everyone else. Everyone works with everyone else at least once and everyone works ten days.
Answered by Victoria West.
A cyclic quadrilateral 2014-03-28
From Carly:
Suppose ABCD is a cyclic quadrilateral, i.e A, B, C, and D are the points on a circle, given in order going around the circle. Show that if we join each of A, B, C, and D to the orthocentre of the triangle formed by the other three, then the resulting line segments all intersect in a common midpoint.
Thank you.

Answered by Chris Fisher.
Percentage increase of the population of a town 2014-03-28
From hunter:
The population of a certain town in 1984 was 2900 people. The percent increase was 2.5% each year. What is the town's population in 2002?
Answered by Penny Nom.
12 golfers, 6 rounds 2014-03-27
From Bill:
I have a group of 12 golfers playing 6 rounds of golf. I am looking for a pairings schedule that allows each golfer to play at least one round in a foursome with each of the other eleven players.

Can you provide me with a solution?

Answered by Victoria West.
A table of values 2014-03-27
From Marisol:
A table value X 0,1,2,3,4,5,6 and Y 3,7,11,15,19,23,27 . what relationship of X to Y is represent by which equation? A . y= 4x. B y =x+3 . C y =4x+3 or . D y =4x-3
Answered by Penny Nom.
12 golfers, 24 weeks 2014-03-27
From George:
I have 12 golfers and want to set up a season schedule that will avoid duplication of playing partners as much as possible. The saeson is 24 weeks long. Do you have a formula?
Answered by Victoria West.
An inequality 2014-03-27
From James:
a step on a stairway is at least 8 inches high. write and solve an inequality to represent the maximum number of steps between floors that are 10 feet apart
Answered by Penny Nom.
13 golfers in groupings of 4, 3, 3 and 3 2014-03-26
From Lennart:
How can I organise 13 players playing in groupings of 4,3,3,3 for 3 rounds of golf in a way that all play with all others but a minimum of times with the same players(?)If possible, nobody should play in the 4 ball more than once. I would be grateful for an answer/proposal! Thank you, LR
Answered by Victoria West.
A word problem with fractions 2014-03-26
From lois:
a man has a stack of boards, he cuts 3/5 off each board, after he cuts the boards he finds he has enough pieces left to make 4 boards the length of the original boards. how many boards did he start with.
Answered by Penny Nom.
Justin and his father went fishing 2014-03-26
From Confused:
Justin and his father went fishing and together caught 12 fish. Three times the number of fish that Justin caught exceeds 12 by as much as 5 times the number that his father caught exceeds 8. How many fish did each catch?
Answered by Harley Weston.
The faces of a triangle 2014-03-24
From Westun:
Your answer says that a triangle has 0 faces but isn't the part of the shape you see the face!? Because i am having trouble with that so could you help me!
thanks! :)

Answered by Penny Nom.
9 golfers in groups of 3 2014-03-22
From Andrew:
We are a group of 9 golfers.
We play 4 rounds in 3 groups of 3.
What would be the most equitable where everyone plays with another at least 1 time...?. Thank you.

Answered by Victoria West.
Riding on a Ferris wheel 2014-03-21
From Claire:
A Ferris wheel has a radius of 72 feet and its center is 85 feet above the ground. At the top of the Ferris wheel, Sam is in the topmost car. If the Ferris wheel makes two complete revolutions per minute, how high above the ground will Sam be after 10 seconds?
Answered by Penny Nom.
Three digit padlock combinations 2014-03-20
From Edwin:
Would appreciate if you can provide me with the list of different combinations that can be obtained from a three digits number. I lost the access code of a three digits number padlock (and need to open it order to get important documents
Answered by Penny Nom.
Ms. Werenich's nickels, dimes and quarters 2014-03-19
From London:
Ms. Werenich has twice as many dimes as nickels and 4 more quarters than nickels. If she has $4.50, how many of each coin does she have? What would the let statements be of this problem and can you solve it?
Answered by Penny Nom.
Find two numbers with a sum of -8 and a difference of 4 2014-03-19
From Elisabeth:
Find two numbers with a sum of -8 and a difference of 4. Let x be the greater number and y be the lesser number.
Answered by Penny Nom.
A formula for determining concrete volume in cu. yd. 2014-03-19
From Steven:
I know a couple of formulas for determining concrete volume in cu.yd. the one I use most frequently is length in feet x width in feet x thickness in inch. Divided by 324. It works every time. Where does the number 324 come from.
Answered by Penny Nom.
14 teams divided into two equal divisions 2014-03-18
From C:
We have 14 teams divided into two equal divisions. We want each team to play each team in their division one time and four teams from the other division one time. What is the schedule
Answered by Victoria West.
A circle inscribed in a right triangle 2014-03-16
From akshaya:
A circle with centre O and radius r is inscribed in a right angled triangle ABC. If AB=5 cm, BC=12 cm and < B=90*, then find the value of r.
Answered by Penny Nom.
2 rectangular prisms are similar by a scale of 0.75 2014-03-16
From gina:
2 rectangular prisms are similar by a scale of 0.75. The volume of the smaller prism is 27cm3 (cubed). How many cubic centimeters is the volume of the larger prism?
Answered by Penny Nom.
Two overlapping arcs in a square 2014-03-15
From Jean:
I have a square with side 4 cm. There are two overlapping arcs going from vertex to diagonal vertex. The other two vertices are the center of the arcs, which are shaded. How do I find the area of the shaded arcs? The overlapping arcs when shaded resemble a long thin football
Thank you for your help.

Answered by Penny Nom and Walter Whiteley.
The area of a 5 sided lot 2014-03-15
From Michael:
Question from michael:

This lot is in feet. 59x154x109x188x137 per the plot plan

Answered by Harley Weston.
Three consecutive whole numbers 2014-03-13
From Abhishek:
Three consecutive whole numbers are such that if they be divided by 5,3 and 4 respectively, the sum of the quotient is 40. What are the numbers?
Answered by Chris Fisher.
A chord length of 2 cm 2014-03-13
From prema:
I am having only chord length as 2cm. how to find arc length and degree
Answered by Penny Nom.
A cable around the Earth 2014-03-13
From fikile:
By how much must an equatorial cable be extended in order that it runs 1meter above the ground?
Answered by Penny Nom.
A parabolic suspension bridge 2014-03-11
From jeffrey:
the towers of a parabolic suspension bridges 200 meter long are 40 meter high and the lowest point of the cable is 10 meter above the roadway.Find the vertical distance from the roadway to the cable at 50 meter from the center.
Answered by Penny Nom.
8 golfers for 7 rounds 2014-03-11
From Brian:
We have 8 guys playing 7 rounds of golf. We want to play with each player as a partner only once, sounds simple but i can not come up with the match ups beyond the first 4 matches. I believe the numbers should work perfectly where each player can play with another as a partner only once each throughout the week. Please help.
Answered by Robert Dawson.
The ratio of boys to girls is 5:3 2014-03-10
From jane:
The ratio of the number of boys to the number of girls in the concert hall is 5:3. After additional 28 boys entered the concert hall, the ratio of the number of boys to the number of girls become 3:1 How many girls are their in the concert hall?
Answered by Penny Nom.
24 golfers playing 3 rounds 2014-03-10
From Jim:
How can I rotate 24 golfers playing 3 rounds of golf so that none of them play with the same person twice.
Answered by Victoria West.
A circle which is tangent to two perpendicular lines 2014-03-09
From MJ:
I'm a College Student taking up Bachelor of Secondary Education on Math Subject. And I'm struggling for my research about Circles. I done solving the said topic particularly on this question:

"What are the possible equations of a circle being tangent to a pair of perpendicular lines, having the origin as the Point of Intersection and the C (h, k), where h, k ∈ℤ"

But I can't get what would be the process that I must do in order to jive to my idea/goal for that problem.
Please check my idea that the numerical coefficients of the equation is equal to the radius of the circle. Thanks in advance! :)

Answered by Penny Nom.
A word problem 2014-03-09
From Joanne:
Ashley weighs 5kgs more than Joyce. Their total weight is 41 kgs. What is the weight of Ashley?
Answered by Penny Nom.
A schedule for 6 teams over 12 weeks 2014-03-09
From Paul:
So I have a question. I noticed you came up with ways to create schedules and we are having a really hard time making our lineup correctly. Basically we have 6 teams that will play over a 12 week period every Monday night. The time slots are 1 hr each playing at 6, 7 and 8. We would like each team to play 3 games at each time slot over the 12 weeks. Is this something that can actually be accomplished?
Thanks so much for your help!

Answered by Eric Venables.
Eleven golfers 2014-03-08
From Leon:
I have a group of 11 golfers wanting to play 10 rounds of golf in grouping of 4,4,3 .What is the best solution so that everyone plays each other as many times as possible
Answered by Victoria West.
Simple interest 2014-03-08
From lynn:
I need to find what the interest will be for: 5000.00 @ 16.5% annual rate, and the same for 10.8 % for 5000.00 annual rate all one question

thank you for your aid

Answered by Penny Nom.
The volume of scale in a pipe 2014-03-08
From Sid:
What is the estimated volume of scale (Liters) if you have a 12 inch ID pipe,300 feet long with scale 1/2 inch thick. Please show me the method and formula. Thank you for your assistance.
Answered by Penny Nom.
The angle of depression 2014-03-08
From Ranger_minor:
A woman of height 1.4 metres standing on the top of a building 34.6 metres high views a tree some distance away. she observes that the angle of depression of the bottom of the tree is 35 degrees and the angle of depression of the top of the tree is 29 degrees.

assume that the building and the tree are on level ground :
1). calculate the distance of the woman from the top of the tree measured along her line of sight.

Answered by Penny Nom.
Determine the selling price 2014-03-08
From augustine:
knowing an auctioneer charges a 7% fee and I know I want $74,400 what is the formula to determine selling price.
Answered by Penny Nom.
The equation of a circle 2014-03-07
From Balraj:
I have to draw a circle x^2+y^2 =8x . please tell me how to understand these coordinates ? please elaborate , on how to understand these types of equations.
Answered by Penny Nom.
What are the possible lengths of the hypotenuse? 2014-03-07
From audrey:
The three sides of a right angles triangle measure x-2, x+5, and 2x-1 in length. What are the possible lengths of the hypotenuse?

... I'm doing the equation c2=a2+b2 and subbing in the numbers but nothing makes sense

Answered by Penny Nom.
Mixing oil and gas 2014-03-07
From Geoff:
What amount of oil would I need to mix on 50-1 basis for a-4litre tank (13.57ozs) capacity tank. Thank you
Answered by Penny Nom.
The inverse of y = 5^x 2014-03-07
From Melody:
how do you find the inverse of y = 5^x
Answered by Penny Nom.
Parallel chords 2014-03-06
From Raj:
AB andCD are two chords which are in the same side of the circle.The length of AB=12 and CD= 24.if the distance between them is 4 then find their radius. Pls help me I just believe on you guys I am having my exam after 3 days
Answered by p.
A circle graph 2014-03-06
From Caitlyn:
I need to find the measure of the central angle that represents the amount of time spent on each activity.
My question includes a circle graph with the following: Sleep 31%, Other 15%, Entertainment 18%, Errands 7%, Work 20%, and Food 9%.

Answered by Penny Nom.
A speed word problem 2014-02-24
From kiss:
Hank bicycles 5 km/h slower than Kelly. In the time that it takes Hank to bicycle 42 km, Kelly can bicycle 57 km. How fast does each bicyclist travel?
Answered by Penny Nom.
A word problem 2014-02-23
From Anh:
The Texas State Legislature is comprised of state senators and state representatives. The sum of the number of senators and representatives is 181. There are 119 more representatives than senators. How many senators and how many representatives make up the Texas Legislature?
Answered by Penny Nom.
A word problem with fractions 2014-02-23
From alex:
In a certain fraction the number by n the denominator is greater than the number by3.If 2 is added to both the numerator and denominator ,the fraction is increased by 6/35..Find the fraction.
Answered by Penny Nom.
x^3+(1/x^3)=18√3 2014-02-23
From Asiv:
if x^3+(1/x^3)=18√3; then what is the value of x?
Answered by Penny Nom.
9 golfers divided into 3 teams 2014-02-22
From Steve:
We have 9 golfers, divided into 3 teams each day. Play is over 6 days. Can you devise a schedule to ensure pairings allow for all golfers to play at least once with all participants? Also, that the pairings have one player from another team.
Answered by Victoria West.
A circle through three points 2014-02-22
From Mohammad:
Find the equation of circle passing through the origin and the points (a,b) and (b,a). Find the length of chords that it cuts off from the axes.
Answered by Chris Fisher.
Finding the retail price 2014-02-22
From Judy:
An item is currently on sale at 37% off the retail price. The item now sells for $350.

My Question:

What is the retail price and what is the formula or steps to figuring that out?

Answered by Penny Nom.
A limit with trig functions 2014-02-22
From pearl:
(what is the value of limit of x as it approaches 0 of sin8x divided by cos6x)
Answered by Penny Nom.
8 team schedule over 7 days 2014-02-21
From Andrea:
I need to organize 8 teams. Each team has to go through 8 games against another team, but not against the same as before.
Answered by Victoria West.
2.3% of $17.3 trillion 2014-02-21
From C:
What is the annual interest on $17.3 trillion at a rate of 2.3%. Thank you
Answered by Penny Nom.
Gradient 2014-02-21
From Nancy:
calculate the gradient of the line AB when the line AB is passing through the points A (2,1,) and B ( 4, -3).)
Answered by Penny Nom.
A 7 person golf schedule 2014-02-21
From Claude:
Golf Holiday: 7 guys, play a 4-some and a 3-some each day for 10 days. Want to have the best balance of each player playing with every other player the same number of times and each player playing in an equal number of 4-somes and 3-somes.
Answered by Victoria West.
A fountain surrounded by a walk 2014-02-18
From Maddie:
A square Water Fountain is Surrounded by a walk 2 yards wide. If the Area of the Fountain is 2/3 of the total area of the Fountain and the Walk. What are the Outside Dimensions of the Walk
Answered by Penny Nom.
The graph of y=2x+4 2014-02-18
From tyler:
y=2x+4 graph
Answered by Penny Nom.
Paul is making banana bread 2014-02-17
From John:
Paul is making banana bread. The number of cups of bananas he uses is proportional to the number of loaves of bread. Paul uses 11 1/4 cups of bananas to make 5 loaves of bread. Which equation represents the relationship between c, the number of cups of bananas, and b, the number of loaves of bread?
A) C=4/9 b
B) C=2b
C) C= 2 1/4 b
D) C=5b

Answered by Penny Nom.
Selecting a card 2014-02-16
From Akarsh:
Cards numbered 1 to 1000 were put in a box.Ali selects cards at random.What is the probability that Ali selects a card containing at least one'3'?
Answered by Penny Nom.
The growth of a tree 2014-02-16
From Emily:
If a tree is 10 ft tall after 2 years and 25 ft tall after 5 yrs , find the rate of change of growth Of the tree if y is the height in ft and x is the number of years.
Answered by Penny Nom.
What is the exponential form of 1/square root of 6v? 2014-02-16
From Prince:
What is the exponential form of 1/square root of 6v
Answered by Penny Nom.
Simultaneous fractional equations 2014-02-15
From benjamin:
hi math central. benjamin here. during class, i had problem with this topic. normally i wont have problems with math but this topic i just too hard for me. please help i am having exam and test next week on this topic

here is the question:
using substitution method, solve the simultaneous equation.
(x+1)/(y+2)=0.5

(x-2)/(y-1)=1/3

Answered by Penny Nom.
The grade of a road as a percent 2014-02-14
From Emily:
What is the grade of the road shown below expressed as a percent rounded to one decimal place?
Rise is 40feet
Run is 380 feet

Answered by Penny Nom.
Rolls of window film 2014-02-14
From Travis:
This question is probably close to the same question as "roll of paper"

We have Rolls of Window Film that we are trying to figure out an equation for a spreadsheet that we can use to "inventory" our window film.

We use a caliper tool to measure the thickness of the roll in millimeters.

the core thickness = 1.90mm
Full Roll thickness(including core) = 9.08mm to 9.12mm
Film thickness = 0.06

Full Roll of Film is supposed to average 1200" of film

What equation could we use to get the approximate inches left remaining on the roll if we measured the roll including the core with the Caliper tool in Millimeters?

Answered by Harley Weston.
Some mixed nuts 2014-02-12
From Andrew:
How many kg of mixed nuts that contain 30% peanuts must Paul add to 6kg of mixed nuts that contain 38% peanuts to make a mixture that contains 33% peanuts?
Answered by Penny Nom.
A word problem 2014-02-11
From cyril:
In a rectangle,the length is 1.5 times the breadth.if the length is shortened by 3 m and the breadth is lengthened by 3 m, the rectangle will be a square. Calculate the dimensions of the rectangle.
Answered by Penny Nom.
Water in a conical funnel 2014-02-11
From Marcus:
Water is running out of a conical funnel at the rate of 1 inch^3/sec. If the radius of the base of the funnel is 4 in. and the altitude is 8 in., find the rate at which the water level is dropping when it is 2 in. from the top.
Answered by Penny Nom.
A ball in a box 2014-02-09
From kenneth:
I have 2 questions.What is the radius of the largest ball that will fit into a box that has the inside dimensions of 6feet by 8 feet by 10feet. and the second question is how many cubic feet of concrete will you need to complete a patio that is 20 feet long by 15 feet wide by 5 inches in hight.
Answered by Penny Nom.
8 golfers, 11 rounds 2014-02-09
From Clare:
I am leading a group of 8 golfers who will be playing 11 rounds each at the end of next month and have been tasked with coming up with a schedule that allows, as close as possible, that has everyone playing an equal number of rounds with each other. Do you have any suggestions?
Answered by Penny Nom.
The volume of a cone 2014-02-08
From hibba:
why is the volume of right circular cone divided by 3?
Answered by Penny Nom.
Can a cylinder be considered a circular prism? 2014-02-08
From Davina:
Can a cylinder be considered a circular prism?
Answered by Penny Nom.
Pizza for 24 2014-02-07
From Delaney:
if 8 people eat 2/3 of a honey chicken pizza how many pizzas are need to feed 24?

How is this problem solved. Thank you!

Answered by Penny Nom.
A number with all the digits different 2014-02-07
From Jon:
I'm not a mathematician but just curious. Is there a name for a number of up to ten digits where each digit is different, i.e the equivalent of an isogram in linguistics? My online banking gives me a random 8-digit number each time I log on and it is rare to get one of the type I refer to. There must be calculable odds, but I'm only allowed one question!
Thanks in advance
Jon

Answered by Harley Weston.
Prime factorization 2014-02-06
From Kadeejah:
Write the prime factorization of 37 in exponential form
Answered by Penny Nom.
Walking up and down a hill 2014-02-06
From flo:
michelle walks 4 km to the top of a hill at 3 km/h then immediately walks back down at an average speed of 5 km/h what is her average speed for the 8km walk
Answered by Penny Nom.
A triangle has angels in the extended ratio of 2:5:8 2014-02-06
From Rubina:
a triangle has angels in the extended ratio of 2:5:8. find the measure of all three angles?
Answered by Penny Nom.
The volume of a frustum 2014-02-02
From mike:
volume of frustum R23", r 18", h 16"
Answered by Penny Nom.
Conics 2014-02-01
From Kassidy:
Hey, I have searched through all the questions about conics and how people use them in the real world, but none of them were very specific on how they are applied and the process, why it's so important etc. I have a project due asking these questions and it's been very difficult finding the right answer, if you could name jobs, how they are use and specifically applied that would be greatly appreciated.
Answered by Penny Nom.
The dimensions of a room 2014-02-01
From jenna:
The length of a rectangular room is 2m more than the width. Find the dimensions if the perimeter of the room is 34m
Answered by Penny Nom.
Related rates 2014-01-30
From Veronica:
A container is the shape of an inverted right circular cone has a radius of 1.00 inches at the top and a height of 5.00 inches. At the instant when the water in the container is 1.00 inches deep, the surface level is falling at the rate of -2.00 inches/second. Find the rate at which the water is being drained.
Answered by Penny Nom.
A man and a kite 2014-01-29
From Veronica:
A man flies a kite at a height of 120 meters. The wind carries the kite horizontally away from him at a rate of 8 meters/second. How fast is the distance between the man and the kite changing when the kite is 130 meters away from him?
Answered by Penny Nom.
A bizarre percentage problem 2014-01-29
From matthew:
by what percent was the child's sodium level beneath the normal range? the child's sodium level was 107 where the normal is 136-145
Answered by Harley Weston.
Two nonlinear equations 2014-01-26
From Naryn:
(1÷x) + (1÷y) = (7÷12)
xy = 12

Answered by Penny Nom.
An inequality 2014-01-25
From LANELL:
this is a problem to solve: 1/3 + 2/7 >=x/21 -- part of the answer is (-oo) not exactly that similar--it is on a calculator as a symbol- sure you know what it is I am talking about- the x will be a number
Answered by Penny Nom.
Problem solving 2014-01-24
From ngozi:
There are 4 children in the Owens family. Jamie 1 1/2 times as all as Kelly and he is 6 inches taller than Olivia. Sammy is 56 inches tall, which is 2 inches taller than Olivia. Find Jamie's height
Answered by Penny Nom.
Kate, Tess and Nina 2014-01-23
From Sam:
Kate is 16 yrs. younger then Tess. Nina is 8 yrs.older then Kate. The sum of their ages is 189. How old is each?
Answered by Penny Nom.
The fourth side of a quadrilateral 2014-01-23
From joanna:
left vertical measurement 2560mm
right vertical measurement 1850mm
base horizontal measurement 1750mm
question - what will the 4th measurement be please.
using a scale drawing I make it approx 1900mm but require an accurate measurement

regards

Joanna

Answered by Penny Nom.
A word problem involving fractions 2014-01-23
From Pamela:
John put 1/4 of his flowers in a vase
He gave 1/2 to of the flower to his wife
He gave 1/5 of the flowers to his daughter
He has 8 flowers left
How many flowers did John start with

Answered by Penny Nom.
Reducing the area of a parking lot 2014-01-19
From Robin:
The school is reducing the area of its 20yd by 50yd parking lot by removing a strip at one end of the parking lot and another strip of the same width from one side. The new area of the parking lot is 504 said yes. Find the width of the removed strip
Answered by Penny Nom.
Mixing water and wine 2014-01-19
From manish:
a man mixes three part of water to one part of wine.When he consumes one part of mixture he realise that the drink is too dilute and hence adds one part of wine to the remaining quantity what is the ratio of water to wine in the new mixture.
Answered by Penny Nom.
Mark's books 2014-01-18
From Jason:
Mark has 312 books in his bookcases he has 11 times as many fiction books as nonfiction books how many fiction books does mark have?

26
212
286
301

Answered by Penny Nom.
A rectangular shaped property 2014-01-17
From Donna:
If I I have property that is rectangle shaped. I I know the ends are 100 feet how many feet would the sides be to make 5.4 acres
Answered by Penny Nom.
An inequality 2014-01-17
From Rosie:
4x/x-2<3

four x divided by x-2 is less than or equal to 3

Answered by Penny Nom.
$9.50 in coins 2014-01-15
From dana:
you have $9.50 in coins. it has twice as many nickels as quarters and three times as many dimes as nickels. how many of each coin do you have?
Answered by Penny Nom.
Five cubes 2014-01-15
From Bob:
Rick has five cubes. When he arranges them from smallest to largest, the difference between the heights of any two neighbouring cubes is 2 cm. The largest cube is as high as a tower built from the two smallest cubes. How high is a tower built from all five cubes?
Answered by Penny Nom.
Watching a bouncing ball through a window 2014-01-15
From Bob:
A ball falls from the roof, at a height of 10m. After each impact on the ground it bounces back up to 4/5 of its previous height. How many times will the ball appear in front of a rectangular window whose bottom edge is at a height of 5m, and whose top edge at a height of 6m?
Answered by Penny Nom.
Is this operation associative? 2014-01-14
From patrick:
Associative test: Can you explain the following to me?

Is the following operation associative?: x*y=x+y+1
1) x*(y*z)=x*(y+z+1)=x+(y+z+1)+1=x+y+z+2
2) (x*y)*z=(x+y+1)*z=(x+y+1)+z+1=x+y+z+2

The answer is yes as 1) = 2)

My specific questions are:
1) How x*(y*z)=x*(y+z+1)=x+(y+z+1)+1 ?
2) How (x+y+1)*z=(x+y+1)+z+1?

Thank you!!

Answered by Penny Nom.
A 7% solution 2014-01-14
From imran:
what amount of H2SO4 should be added to make 5mL of 0.7% H2SO4 solution
Answered by Penny Nom.
The train fares are proportional to the square root of distance traveled 2014-01-11
From manish:
train fares are proportional to the sq .root of distance traveled. the train fare is Rs. 24.4 for a distance of 140 kms. by how much % does the total fare increase if the same distance is covered by breaking the journey after every 35 kms.
Answered by Penny Nom.
On what day of the week was July 4, 1904? 2014-01-11
From Madi:
Hi guys,
I have a question. July 4, 1903, was a Tuesday. On what day of the week was July 4, 1904? If you could give me an equation that would work for any question like this, that would be GREAT!
Your fellow math scholar (in training),
Madi

Answered by Penny Nom.
A rectangle inscribed in a circle 2014-01-10
From Marian:
A 16 cm by 12 cm rectangle is inscribed in a circle. Find the radius of the circle.
Answered by Penny Nom.
4x^3-7x^2-2x 2014-01-10
From Alvi:
4x^3-7x^2-2x
Answered by Penny Nom.
Radio direction finders 2014-01-09
From Nathan:
Radio direction finders set up two points A and B, which are 2.5 miles apart on an east-west line. From A, it is found that the bearing of a signal from a radio transmitter is N36.33 degress E, while from B the bearing of the same signal is N53.67 degrees W. Find the distance of the transmitter from B.
Answered by Penny Nom.
A box-and-whisker plot with one whisker 2014-01-09
From Susan:

Question from Susan, a teacher:

I read your answer to the question about could a box & whisker have NO whiskers.
It made sense to me. However I am preparing for my advanced class and I have a sample graph comparing box plots of data.
Several of the box have only 1 whisker. There is no explanation for this. Can you help me out. I cannot think of a data set that would produce this. Thanks!


Answered by Penny Nom.
Two tuna boats 2014-01-09
From amanda:
Two tuna boats start from the same port at the same time, but they head in opposite directions. The faster boat travels 10 knots per hour faster than the slower boat. At the end of 8 hours, they were 272 nautical miles had each boat traveled by the end of the 8 hour period?
Answered by Penny Nom.
When would one flip the inequality sign? 2014-01-09
From Natasha:
Would one flip the inequality symbol in this equation: (explain why) (-9a)/(-9) > 81
And please explain in what circumstances one would flip the inequality sign
THANKS!

Answered by Penny Nom.
A tire rolls 100 feet in a straight line 2014-01-07
From Alanna:
A tire has a diameter of 26 inches. If it goes 100 feet in a straight line, how many complete revolutions did it make?
Answered by Penny Nom.
A scalene triangle 2014-01-07
From cherry:
Hi
I have been given a scalene triangle of sides -
8m, 6m, and 10m ..
Please help me to find out the area and help me to find out the height ..

Answered by Penny Nom.
An equilateral triangle inscribed in a circle 2014-01-06
From Anonymous:
An equilateral triangle with sides 6 inches is inscribed in a circle. What is the diameter of the circle?
Answered by Penny Nom.
Units digit 2014-01-03
From gracie:
9^23 units digit
Answered by Penny Nom.
4 children 2013-12-31
From hope:
4 children; Peter is 7 years old & is 4 years younger than Dave. Dave is 9 years older than Paul. Paul is 11 years younger than Frances. Please list the children in order from oldest to youngest?! thank you and anxiously await your answer, Hope
Answered by Penny Nom.
The slope of the line y=-2x+5 2013-12-29
From victoria:
what is the slope of the line with the equation y=-2x+5
Answered by Penny Nom.
Distance from a point to a line 2013-12-28
From Sue:
I need to find the distances from the west line on 1st Take Point and 2nd Take Point.
Thanks,
Sue

Answered by Penny Nom.
ug/l and ppb 2013-12-27
From Wally:
convert 40.5 ug/l to ppb?
Answered by Penny Nom.
A linear equation 2013-12-26
From khushboo:
A farmer sowed wheat and paddy in two fields of total area 5000 sq.m. Write a linear equation and draw a graph to represent the same?
Answered by Penny Nom.
The area of a quadrilateral 2013-12-21
From khushboo:
Find the area of quadrilateral pqrs in which angle QPS=90°,PQ=12cm, PS=9cm, QR=8cm and SR=17cm. (Hint: PQRS has two parts)
Answered by Penny Nom.
Analytic Geometry 2013-12-18
From fRitz:
if (x,4) is equidistant from (5,-2) and (3,4), find x.
Answered by Penny Nom.
Schedule for a 4 team basketball league 2013-12-08
From Sommer:
I need a schedule for my kids basketball league. There are 4 teams. We will play every Saturday for 4 weeks with 4 games per Saturday. Week 5 will be the playoffs where 1st will play 4th and 2nd will play 3rd. Week 6 will be winners of week 5. Please help!!
Answered by Victoria West.
A circle insubscribed in an isosceles trapezoid 2013-12-08
From Bob:
A circle is insubscribed in an isosceles trapezoid, with parallel lengths of 8cm and 18cm. What is the lengths of sloping edges and why?
Answered by Robert Dawson.
A word problem 2013-12-08
From Malissa:
I am attempting to help my son with his math word problem, please help thank you .

Kylie and Ethan kept scrapbooks of their grandfather's stories from world war II. Kylie has 6 more than half the number of pages Ethan has. If Kylie has 21 pages in her scrapbook, how many pages are in Ethan's scrapbook?

Answered by Penny Nom.
conical lamp stand/staved wood 2013-12-07
From Henry:
need to make lamp stand that is wooden staved; need it to be 25 inches at bottom and 10 inches at top; need to know angles for staves to be cut; the lamp stand will be rounded on a lathe and will be 40 inches tall John Lucas built one and it is pictured on his web page. thank you for any help/direction; I checked out the answered for cone shaped objects on your page but didn't find what I could use. thanks again. Henry--woodturner, parent teacher student . . . . .
Answered by Harley Weston.
How do I graph y=-2x-1? 2013-12-07
From cindy:
How do I graph y=-2x-1?
Answered by Penny Nom.
f(x)= a+b*lnx 2013-12-04
From Kevin:
How do I find the curve, algebraically on paper, for f(x)= a+b*lnx with data points (6,1303) (9,1639). I can do it with LnReg on a calculator but I want to know how to get the same results or close to the same results by working it out on paper. When I run it through the LnReg on the calculator I get, f(x)=181.792+828.678 Ln x. How did it achieve this solution?
Answered by Penny Nom.
Problem solving with fractions 2013-12-03
From Jennie:
Paul has 2/3 as many postcards as Shawn. Shawn has 3/5 as many postcards as Tim. If the 3 boys have 280 postcards, how many more postcards does Tim have than Paul?
Answered by Penny Nom.
Exponential form 2013-12-01
From Alina:
My question says to express each number in exponent form in as many different ways as I can; 16, 81 and 64. How do I do this?
Answered by Penny Nom.
Factor 2013-12-01
From Rosa:
How do you factor 9x^2 - 12y^2?
Answered by Penny Nom.
What is the slope of this line? 2013-12-01
From Charlene:
What is the slope of this line?
Answered by Penny Nom.
0.35 as a fraction 2013-11-28
From Laura:
How do you write 0.35 as a fraction or mixed number?
Laura

Answered by Penny Nom.
People on a bus 2013-11-28
From jeric:
3/10 of the passenger in a bus were men, 2/7 of the remainder were women and the rest were children, if there were 50 passengers in the bus, how many more children than women were there?
Answered by Penny Nom.
The area of a parallelogram 2013-11-28
From shivam:
a parallelogram has sides 34 cm and 20 cm .one of its diagonal is 42 cm. calculate its area
Answered by Harley Weston.
A sale price for a TV 2013-11-28
From tom:
a TV can be purchased for $278 normally is $328. what is the percent of the present cost of this TV will you be saving?
Answered by Penny Nom.
Base 5 arithmetic 2013-11-27
From samuel:
Good day sir, please i don't understand when you say 4x4=13 in base five? In fact, am always having difficulties in addition, subtraction, division and multiplication of number in the same base other than base ten. Please can you give me one example each with details explainations?
Answered by Penny Nom.
2 concentric circles 2013-11-27
From Dimaris:
The radius of the outer circle of 2 concentric circles is x. An equilateral triangle inscribed in the outer circle also circumscribes the inner circle. What is the radius of the inner circle in terms of x?
Answered by Penny Nom.
Problem solving 2013-11-26
From Moira:
Amy has £5 more than Ben. Ben has twice as much as Chris. In total they have £20 how much do they each have
Answered by Penny Nom.
Three roommates share the rent 2013-11-23
From Cortlin:
Because of different bedroom sizes, three roommates decide to split the rent this way: Roommate A pays 75% as much as B does, and C pays 40% as much as B does. What fractional part and percent of the rent does each roommate pay?
Answered by Penny Nom.
A schedule for 32 golfers 2013-11-22
From Scott:
I would like a schedule for 32 guys playing in 8 foursomes for 13 week. I would like it so that no one plays in the same foursome more than once or twice. Is this possible to do??? Can you let me know at bogeysm@yahoo.com if you can do it and if you will do it.
Answered by Victoria West.
Admiring handblown glass ornaments 2013-11-21
From Ruby:
We are admiring hand blown glass ornaments 5% are unmarketable due to defects what is the probability the 3rd ornament is the first defective one
Answered by Penny Nom.
Why can't we add or subtract speed? 2013-11-21
From kashish:
why we cannot add or subtract "speed" / directly in any given distance , speed , time questions?
Answered by Chris Fisher.
Scheduling 10 people for 5 rounds 2013-11-20
From Keith:
Using golf as an example, I would like to schedule 10 people for 5 rounds with 2 groups of 4 and 1 twosome each round. I would like to minimze the number of times any one player plays with another. Can you provide an optimal schedule? Thank you for your help.
Answered by Victoria West.
The area of an irregular octagon 2013-11-18
From zafar:
if i have a rectangular octagon then how i can find its area 1. The length of rectangle is 42 and the width is 21 and and the base length is 42-12 =30 i mean the length of base =30
Answered by Penny Nom.
Evaluate log3 14 2013-11-17
From Tash:
Use the Change of Base Formula and a calculator to evaluate the logarithm, correct to six decimal places. Use either natural or common logarithms.

log3 14

Answered by Penny Nom.
Solve 0=-2(x-5)^2+4 2013-11-17
From Leila:
How do you find the solution to 0=-2(x-5)^2+4
Answered by Penny Nom.
Inflection Point 2013-11-15
From Z.:
I was wondering if the function F(x)= x/ln(x) had an inflection point? If yes, why it's not visible on the graph of the function?
Thanks.

Answered by Penny Nom.
n log n = 36 * 10 ^ 12 2013-11-12
From shihab:
How to find value of n in this equation :

n log n = 36 * 10 ^ 12

Answered by Penny Nom.
One train overtaking another 2013-11-11
From kiran:
two trains of equal length are running in parallel tracks in the same direction with the speed of 60km/hr and 90km/hr respectively the latter completely crosses the former in 30 seconds the length of each train is in mtr
Answered by Penny Nom.
Matrices 2013-11-10
From Ricky:
Why must we have brackets around matrices. Why don't we just neatly write an array of entries cross-wise and up and down to indicate a particular matrix?
Answered by Chris Fisher.
A lever 2013-11-09
From Durgesh:
A farmer uses a 2 m long lever to lift large rocks from the ground of his field . He places a fulcrum 20cm from the rock end of the lever and then stands on the other end, levering out the rock. If the farmer weighs 100 kg , what is the heaviest mass of the ro ck that he can lift using this method? (how to solve)
Answered by Penny Nom.
The popcorn box problem 2013-11-07
From Dave:
We know that calculus can be used to maximise the volume of the tray created when cutting squares from 4-corners of a sheet of card and then folding up.

What I want is to find the sizes of card that lead to integer solutions for the size of the cut-out, the paper size must also be integer. EG 14,32 cutout 3 maximises volume as does 13,48 cutout 3.

I have done this in Excel but would like a general solution and one that does not involve multiples of the first occurence, as 16, 10 cutout 2 is a multiple of 8,5 cutout 1.

Answered by Walter Whiteley.
Substitution type simultaneous equations 2013-11-03
From Kayla:
I am having problems with substitution type simultaneous equations, when the variable you are substituting is a algebraic one:
y=x^2-3x+4 and 3x-2y=1
I have rearranged 3x-2y=1 to get x=(1+2y)/3 but when I substitute this x value into the other equation, I get the wrong answers!
Would appreciate any help! Thank you.

Answered by Penny Nom.
Water flowing out of a tank 2013-11-03
From Carolyn:
The flow of water out of a hole in a tank is known to be proportional to the square root of the height of water above the hole. That is,

dV/dt (proportional to) sq root (h)

The tank has a constant cross-sectional area A, show that the height of water in the tank is given by

h = ((-kt+C)/2)^2

If the tank is 9 metres high, and it takes 5 hours for it to drain from full to half full, how much longer will we have to wait until it is completely empty?

Answered by Penny Nom.
The depth of a lot 2013-11-02
From Gini:
what is the depth of a lot with 45 feet frontage containing 1080 sq yards
Answered by Penny Nom.
A ratio problem 2013-10-31
From indu:
students of a school have been into 2 groups in the ratio 6:11, if the group has 870 students, how many students are there in all the school?
Answered by Penny Nom.
An extension using concrete blocks 2013-10-30
From david:
hi how many concrete blocks would i need for an extension 50 ft long ,20 ft wide and 12ft height
Answered by Penny Nom.
A sloped concrete wall 2013-10-30
From Brad:
We are planning on pouring concrete in a wall form that is sloped. One end is 73 inches tall, and it tapers to 24 inches tall over a distance of 348 inches. It is flat on bottom and sides and is 12 inches thick. Thanks.
Answered by Penny Nom.
Is-5/-11 a positive rational number? 2013-10-30
From tazneem:
Is-5/-11 a positive rational number
Answered by Penny Nom.
A plane 2013-10-27
From Thomas:
Why is this 3d plane flat?
if m ≠ n
arcsin(sin(m*arctan(x/y))), arcsin(cos(n*arctan(x/y))),
Pi-arcsin(sin(m*arctan(x/y)))-arcsin(cos(n*arctan(x/y)))
This is using radians.
These are angles of a triangle, when m and n are the same it is a right triangle. All three inputs are not equal to each other. The last input is pi minus the first two inputs, which makes a complete triangle.
I was kind of hoping to find more lissajous curves. It doesn't make sense to me that this is a flat plane, a diagonal one at that.

I'm not a student or anything. I'm just wasting time.

Answered by Chris Fisher.
The perimeter of a rectangle 2013-10-24
From sana:
if the length is given 70 m and the breadth is given as 40 m find the perimeter of the rectangle
Answered by Penny Nom.
A stem and leaf plot 2013-10-24
From mary:
Hi
I'm trying to answer my child's question on stem and leaf plot. For grade 4, the question is where do you put 57 in this list

6    0
5    4 8 8 9 4 6 4

Answered by Penny Nom.
Tim is standing in a hole 2013-10-23
From Marvel:
Tim is standing in a hole that is 4 feet deep. Ben asks him how much deeper is he going to dig the hole. Tim replies he will dig to the level that is 4 feet 2 inches deeper and that then the top of his head will be the same distance below ground level that it is now above. How tall is Tim?
Answered by Penny Nom.
Extraneous solutions 2013-10-22
From tom:
i need an equation where x=-2 is the correct answer and x=-3 is an extraneous solution. can you provide me with such an equation??
Answered by Harley Weston.
Together, Tom and Max have 72 football cards 2013-10-21
From Maureen:
Together, Tom and Max have 72 football cards. Tom has 2 more than 4 times as many cards as Max has. How many football cards does Tom Have?
Answered by Penny Nom.
Peter bought pizza and drinks 2013-10-19
From sd:
Peter spent $30 on pizza and 3/4 of the remainder on drinks. He has 2/9 of the original amount left. How much had he at first?
Answered by Penny Nom.
A string wrapped around three circles 2013-10-19
From Jim:
A recent puzzle ' find the length of a string around 3 touching 1 meter diameter circles ' gave this answer : the string touches 120° (or pi/3 meters) of each circle. Then 3(1+pi/3) = 3+pi meters is the required length. I do not see how it was determined that the string touches 120° or pi/3 meters?
Please explain . Thank-you , Jim

Answered by Penny Nom.
What the dog weighs 2013-10-17
From linda:
Sam has a dog and rabbit together they weigh 104 lbs the dog weighs 12 times more then the rabbit what is the dogs weight
Answered by Penny Nom.
A square inscribed in a circle 2013-10-14
From Jenn:
Hello! I am about to buy a 7'9" round rug, but I want to have it cut down into a square. What's the largest square I can obtain from this? Thank you!
Answered by Penny Nom.
Four tangent circles 2013-10-09
From Nilesh:
Four circular cardboard pieces, each of radius 7cm are placed in such a way that each piece touches two other pieces. How to find the area of the space enclosed by the four pieces?
Please let me know.

Answered by Robert Dawson.
Maximize the volume of a cone 2013-10-09
From Conlan:
Hi I am dong calculus at school and I'm stumped by this question:

A cone has a slant length of 30cm. Calculate the height, h, of the cone if the volume is to be a maximum.

If anyone can help me it would be greatly appreciated.

thanks.

Answered by Penny Nom.
A line through (5, 1) 2013-10-08
From allison:
find an equation in standard form for line passing through point (5,1) and perpendicular to line x=4
Answered by Penny Nom.
Two bikers race on a circular track 2013-10-03
From bryan:
two bikers race on a circular track. biker a can circle the track in 8 minutes, and biker b can circle the track in 6 minutes. From the start of the race, how many minutes will it be before biker b overtakes biker a.
Answered by Chris Fisher.
A triangle problem 2013-10-02
From raneem:
ABC is a triangle in which : BC=20cm. M(<B) =29 and m(<C)=73 . D is the midpoint of BC Find the length Of AB and AD approximated to 2 decimal places
Answered by Penny Nom.
Problem solving 2013-10-02
From naomi:
My son having trouble a math problem please help with this question a bag of cough drops contains 36 drops. the fraction of each flavor is shown in the table strawberry 5/12 honey 1/12 cherry 1/6 mint1/9 and lemon 1/4 which flavor is the greast in the bag
Answered by Penny Nom.
Group the addends so that you can add mentally 2013-10-01
From monica:
the questions in the homework says change the order or group the addends so that you can add mentally ,find the sum and tell which property you used 120+37+280 and 25+25+30
Answered by Penny Nom.
Two teams of 4 playing 4 rounds of golf 2013-09-29
From Tom:
We would like to play a ryder cup format with two teams of 4 playing 4 rounds of golf. Three of the rounds would be pairs playing each other and one round would be singles. How can we set up the foursomes so we balance the number of times we play with every other golfer.
Answered by Victoria West.
Cubic centimeters, milliliters and grams 2013-09-26
From Laura:
How do I find the volume of a cube with dimensions of .78 cm, .3 cm and .22 cm? Then find the volume in ML and the mass if filled with water?
Answered by Penny Nom.
12 golfers for 7 rounds 2013-09-25
From Dennis:
I have 12 golfers playing a round of golf for seven days. How can I set-up teams so that everyone plays with everyone at least once in those seven days. Thanks!
Answered by Victoria West.
A schedule for 2, 7 team divisions 2013-09-24
From Sampson:
I'm looking to set up a weekly schedule within a 22 week segment of 2 7 team divisions where every team plays every other team in their divison 3 times and have 4 cross over games with 4 of the other 7 teams in the other division. I've yet to figure this out. Thanks for your time.
Answered by Victoria West.
6 golfers in threesomes over 7 rounds 2013-09-24
From David:
I have 6 golfers, (2 threesomes) who will be playing 7 rounds of golf. What is the best way to pair the 6 players so they play with as many different players over the 7 rounds?
Answered by Victoria West.
A pool, a school, a park and home 2013-09-24
From Selena:
The distance from the park to the pool is the same distance as the distance from your house to school. The school is between the pool and your house. Prove that the distance from the park to the school is the same distance from the pool to your house.
Answered by Robert Dawson.
25 people are attending a 5 day meeting. 2013-09-20
From Lisa:
25 people are attending a 5 day meeting. There are 5 tables, with each table seating 5 (or 6). I want each person to sit at a different table every day with completely different people.
Is this possible. Can do I figure this out?
I have each of the people a letter designation from A to Y. Each table have a number from 1 to 5.
Thank you!

Answered by Victoria West.
A sequence of operations, in order 2013-09-20
From jen:
multiply 46x23
add 342
divide by 70
subtract 5
I not sure what the answer is..i need to know the answer..thanks..

Answered by Penny Nom.
A rectangular hyperbola 2013-09-19
From Soumya:
Is the graph of an inverse variation a RECTANGULAR HYPERBOLA? If it is, then how can be the equation of a rectangular hyperbola be xy=constant , whereas in the books it is written that the graph of a rectangular hyperbola is- x^2-y^2=a^2?
Answered by Chris Fisher.
The number of cords on a pile of wood 2013-09-18
From Gary:
a cord of wood is 4 feet by 4 feet by 8 feet, how much wood is a row that is 16 in by 5 foot by 32 feet?
Answered by Penny Nom.
If the earth was made of a ball of string 2013-09-18
From Tammy:
My 9 year old son asked me a question and I have no idea how to work it out! Please can you help?

If the earth was made of a ball of string and you unravelled the string how long would it be? Assuming the string is 5mm thick.

Answered by Penny Nom.
The volume of an attic 2013-09-18
From David:
I have small equipment hut that has a small attic space that I want to blow insulation into it.
The attic size is shaped like a triangle that is about 8 feet by 10 feet by 7 inches at the highest point.
I need to estimate the volume in square feet or cubic feet to know how many bags of insulation to blow into the space.
Thanks

Answered by Penny Nom.
A concrete walk around a pool 2013-09-18
From poornima:
circular Swimming pool is surrounded by a concrete walk 4 feet wide, if the area of the walk is 11/25 of the area of the pool, then the radius of the pool in feet is
Answered by Penny Nom.
Covering a styrofoam ball with fabric 2013-09-11
From Alice:
I am trying to create a pattern in order to cover a styrofoam ball with fabric. The pattern needs to be done in such a way as to have only a center seam, rather than several vertical lines as in slicing a melon.
Answered by Chris Fisher.
Intersection points and diagonals 2013-09-11
From Kenneth:
I was wondering, is there a relationship between the number of sides of a regular n-gon and the number of times its diagonals intersect?
Ie:
4 sides=1 intersect (center, diagonals form an 'X' )
5 sides=5 intersects (diagonals form a star)
6 sides=13 intersects
And so on.

Answered by Chris Fisher.
Miles per minute to miles per hour 2013-09-08
From Adam:
Convert 250 miles per min to miles per hour
Answered by Penny Nom.
A contest with 31 people 2013-09-06
From Kevin:
There is a contest with 31 people and only one winner each week.
We play for 17 weeks.
What are the odds of winning at least one week?
The chances of winning each week are independent from previous weeks.

Answered by Robert Dawson.
A garden path 2013-09-06
From mary:
garden path has 8 white slabs, four of the slabs are one third grey, what area of the path is grey
Answered by Penny Nom.
A word problem 2013-09-05
From Thomas:
I'm trying to help my 9th grade child with their homework. The problem is the Length of a rectangle is twice as long as it is wide. The perimeter is 68m. Find It's length and width
Answered by Penny Nom.
2^x . 5^3 = x^4 2013-09-05
From Zuhdina:
Hi, I got a question, 2^x . 5^3 = x^4 I want to know the value of x without using calculator, I've already done some questions in logarithm and I always stuck in the same model equation-I think if someone shows me the way, I can do the other 'stuck equations'
Answered by Penny Nom.
6 golfers in threesomes 2013-09-04
From Gregg:
We have 6 golfers playing in threesomes for three rounds of golf. I would like each golfer to play equally with the other 5. How can this be done.
Answered by Victoria West.
A Hole-in-one on all four par 3's 2013-09-04
From Robert:
There are four (4) par 3's at a specific 18 hole course. One member has the honor of making a Hole-in-one on all four par 3's. It has taken him 8 years to do it! How does one go about computing the odds of this TREMENDOUS accomplishment? Respectfully, Robert laffal
Answered by Victoria West.
7 golfers, a threesome and a foursome 2013-09-04
From Scott:
I'm leading a golf trip with 7 guys, and we're playing 5 rounds of golf together with a threesome and a foursome at each course..

I want to set up a schedule where each person plays at least two rounds with each of the other 6 guys.

Is that possible? I can't seem to find the right combinations.

Thanks for your help!

Answered by Victoria West.
14 golfers in two person teams 2013-09-04
From Stewart:
How to rotate 14 golfers in two person teams without duplication until all have been paired at least once. Play is once per week.
Answered by Victoria West.
A two digit number 2013-09-04
From JEFFERSON:
The sum of the digits is 6. When the digits are interchange, the new number represented is 3 time the 10 digit of the original number. Find the original number?
Answered by Penny Nom.
How many nickles do you have? 2013-09-04
From Monica:
You have 15 more dimes than nickels. If you have $2.55, how many nickels do you have?
Answered by Penny Nom.
Ordering crushed stone 2013-09-03
From Prakash:
Dear Sirs, I am working in a Soft Landscaping contracting company. If I need to purchase crushed stone with the size 50-70mm for $53,429 m^2$ area, how many 20feet containers should I need to order to my suppliers? The 20foot container has internal dimensions 5,897 mm by 2,348 mm by 2,285 mm and the $53,429 m^2$ area is to be covered by 10 cm of stone.
Answered by Harley Weston.
A schedule for 18 golfers 2013-08-29
From Brian:
After searching the data base which helped me solve at least 3 other issues I have one more un answered question. I have 18 golfers split into 6 threesomes for 4 days of golf. What is my best option for a schedule mixing them all up as much as possible over the course of the 4 days
Answered by Victoria West.
An algebraic equation 2013-08-28
From walter:
i'm a parent helping my daughter can you do a step by step on this equation been out of school for years thanks (4w-28)+(11w+13)=180 thanks
Answered by Penny Nom.
More than half of the people said... 2013-08-26
From Lucy:
Hi,
The question is:
Samoa 23%
Brazil 41%
Sri Lanka 52%
Senegal 39%
Armenia 37%
Croatia 20%
Why is the following statement true for the data above: More than half of the people said they were interested in Sri Lanka. The reason I don't understand the problem is that because the percentages add up to be greater than 100%, that means that the 500 people(as stated in the question) who participated in the survey got to pick more than once. Therefore, it is not 52% out of 100% anymore. Thanks for taking the time to answer my question.

Answered by Robert Dawson.
Golden Ratio 2013-08-26
From Mark:
Please Help. I'm trying to help my Child and I have no clue on this math question.

Rectangular shapes with a length to width ratio of approximately 5 to 3 are pleasing to the eye. The ratio is know as the golden ratio. A designer can us the expression 1/3(5w) to find the length of such a rectangle with width 6 inches.

Answered by Robert Dawson and Penny Nom.
More on the scheduling of 16 golfers in 4 rounds 2013-08-26
From Peter:
I see your formula for 16 players 4 rounds.
Is it possible to get the players into different groups each day.
E.G Player 1 in group 1 the first day,
Group 2 the second day,
Group 3 the third day,
Group 4 the fourth day.
Then so on through the players.

Answered by Victoria West.
Tesseract 2013-08-22
From Dan:
I am not a mathematician. This seems to me an intuitively simple enough problem that I very much need an answer to from someone who's mathematics are better than mine. Please help.

The question is: for a tesseract of side length = 1 what is the distance of the center of each cube from the center of the tesseract ?

I think I have calculated the distance of each vertex from the center, and of the center of each edge from the center, but the question above baffles me.

(anyone not having a clue what I am talking about can brush up here http://en.wikipedia.org/wiki/Tesseract )

Thanks in advance - Dan V

Answered by Robert Dawson.
Constructing a triangle 2013-08-22
From Nazrul:
The base, the difference of the angles adjoining the base and the sum of the other two sides are given. How can I draw the triangle?
Please help me.

Answered by Chris Fisher.
Euclid's Parallel Postulate 2013-08-20
From Justin:
Hello there,

I was wondering is Euclid's Fifth Parallel Postulate of parallel lines never intersecting, undecidable or essentially undecidable?

Thank you so much for any help you can provide!

Answered by Robert Dawson.
Similar rectangles 2013-08-17
From Mattie:
The dimensions of a rectangular-shaped picture frame are 14 inches long and four inches wide. Which dimensions below represent another frame that i geometrically similar>

a. l=49 in. and w=14in.
b. l=7in and w=3in
c. l=21in and w=8in
d. l=24in and w=14in

thanks

Answered by Penny Nom.
Equal ordinate and abscissa 2013-08-15
From sonit:
the slope of tangent to the curve y=(4-x^2)^1/2 at the point, where the ordinate and abscissa are equal, is
Answered by Penny Nom.
Two perpendicular lines 2013-08-15
From Hanna:
The problem says, "Write the equation in standard form for the line that is perpendicular to the graph of y=5x+1 with a y-intercept of 4." I have no clue how to do this. Please help me!
Answered by Penny Nom.
Golf for 11 2013-08-14
From Don:
2 foursomes and 1 threesome for 6 rounds of golf
Answered by Victoria West and Harley Weston.
The volume of a pond 2013-08-14
From Kelly:
I have a pond that measures 335 feet wide, 385 feet long and 18 feet deep with a 4:1 slope on all four side. How do I figure out how many gallons of water this pond will hold?
Answered by Harley Weston.
Using trig to find the height of a hill 2013-08-14
From Anna:
From the top of a hill, the angles of depression of two successive milestones on a level road, which leads straight away from the hill, are 5degrees and 15degrees respectively. Fine the height of the hill.
Suggestion: BE is drawn perpendicular to AD. Find BE, then BD, finally CD.

Thanks :)

Answered by Penny Nom.
A system of equations 2013-08-13
From Nina:
3x-2y=8
-x+3y=5
What is the solution of the system of equations

Answered by Penny Nom.
An equation in slope-intercept form 2013-08-12
From Brittney:
write an equation in slope-intercept form of the line that is parallel to the grapg of y=-3/4x+6 passes through -8,-5
Answered by Penny Nom.
A mixture problem 2013-08-11
From MJ:
How many liters of pure acetic acid must be mixed with 3 liters of a 30% acetic acid to obtain a mixture of 65%?
Answered by Penny Nom.
If I know only the tax amount, how do I figure out total cost? 2013-08-11
From Patric:
If I know only the tax amount, how do I figure out total cost?
Answered by Penny Nom.
Differentiate x^x - 2^sinx 2013-08-09
From tarun:
derivative of x^x - 2^sinx
Answered by Penny Nom.
Four equations 2013-08-08
From may:
HI how to solve this 4 equations?
A+C = 0
-4A+B-8C+D=1
3A+16C-8D=-29
-12A+3B+16D=5

Answered by Robert Dawson.
Practical uses of trigonometry 2013-08-06
From tharindu:
use of trigonometry
Answered by Penny Nom.
Slicing through a cone to form an ellipse 2013-08-06
From Pulkit:
we get an ellipse on slicing through a cone. Is there a relation between central axis of the cone and this ellipse?
Does it pass through the any of the foci of the ellipse?

Answered by Chris Fisher.
A golf tournament for 12 people 2013-08-01
From Brad:
I've looked all over the web for a solution to this (including searching the archives here) to no avail.

We're doing a golf tournament with twelve people. Each person has a "handicap" that demonstrates their skill level. We have these numbers for each of the 12 players. We are trying to figure out how to schedule two rounds of golf with the most balance possible.

- We are playing two "real" rounds of golf (18 holes). There are four people per group.
- We want to split each round into two 9 hole games, which gives us a total of 4 games
- Halfway through (after 9 holes), you switch partners within the foursome that you're playing in.
- We want everyone to play with 4 unique different teammates

So say you start the first 18 hole round as a group of Players 1, 2, 3, and 4. Players 1 and 2 are a team, and players 3 and 4 are a team. After 9 holes, Player 1 would either be paired up with Player 3 or 4, and the other two players would pair up as well. Then the next day we will mix up the foursomes and do the same thing. In terms of just pure scheduling this is simple. But I'm trying to figure out how to best balance the handicaps (simple average of each team's two handicaps works here). Assuming I'll need some sort of program or spreadsheet. Any help or even a push in the right direction would be a tremendous help. Thanks in advance to anyone who wants to give this a shot.

Answered by Victoria West.
Golf for 16 2013-07-31
From Graylin:
I am going on a golf trip and we have 16 players and need to know a formula that will allow us to all play together at some point in the 4 rounds we are playing. If not possible please provide best case. Thanks.
Answered by Victoria West.
16 golfers 2013-07-31
From Pat:
16 golfers...3 nine hole rounds a day...for 4 days. what are the best pairings for the fewest repeats of partners?
Answered by Victoria West.
Dilution of concentrated acid 2013-07-31
From Adom:
I have 500mls of 90% conc. of an acid. Now I need to prepare 20% concentration from this (say 100mls of this). How much of the stock solution do I take and how much water to add? Thanks!
Answered by Penny Nom.
Two parallel chords of a circle 2013-07-30
From Don:
two parallel chords of a circle AB and CD on the same side from the center measure 6cm and 11 cm respectively. The distance between the 2 chords is 3 cm. Find the radius
Answered by Penny Nom.
A mixture problem 2013-07-29
From Lucy:
Hi, For the following question:

Ten pounds of mixed nuts contain 50 percent peanuts. How many pounds of peanuts must be added so that the final mixture has 60 percent peanuts?(A) 2.5 (B) 5 (C) 6 (D) 10 (E) 12.5,

my math book gives my the following explanation: Try choice (C): 6 pounds of peanuts. The original 10-pound mixture contained 50 percent peanuts, or 5 pounds. Now you have 11 pounds of peanuts in a 16-pound mixture, more than 60 percent. This result automatically eliminates choices (D) and (E) as well.Why would this result automatically rule out D and E? How can you tell? Thank you for taking the time to answer my question.

Answered by Robert Dawson and Penny Nom.
A word problem 2013-07-29
From Gontsejalo:
Your firm held a dance competition. 900 people attended the dance competion.The cost of admission was 20rand per person and 30,50rand per couple.Twice as mant 20rand tickets as 30,50rand tickets were sold.find the amount of money obtained from the tickets sales
Answered by Penny Nom.
The area of a semicircle 2013-07-28
From Lucy:
Find the area of the the figure described: a semicircle with arc length 3 pi.
Answered by Penny Nom.
A triangle construction 2013-07-27
From Nazrul:
Two angles and the difference of the lengths of their opposite sides of a triangle are given. How can I draw the triangle. Please help me.
Answered by Chris Fisher.
Ann has a bag of marbles. 2013-07-25
From Carmen:
Ann has a bag of marbles. she gave one half to her friend Tina and then one third to her friend Tom. Now left with 15. How many did she had originally?
Answered by Robert Dawson and Penny Nom.
GST and PST 2013-07-22
From Bev:
Total sales revenues are 116391.38 this amount includes 5% GST and 80% of this amount includes a 7% provincial tax. The other 20% is PST exempt. GST is included in all. How do I figure the PST I owe?
Answered by Penny Nom.
What is the value of |2((i)^(1/2))|? 2013-07-22
From Delilah:
What is the value of |2((i)^(1/2))| ?
i.e. absolute value of 2 multiplied by square root of i.

Answered by Penny Nom.
What is the smallest number? (i.e. the closest number to zero) 2013-07-22
From Charlie:
What is the smallest number? (i.e. the closest number to zero)
Answered by Harley Weston.
18/y=30/25 what number does y represent 2013-07-20
From celeste:
18/y=30/25 what number does y represent
Answered by Penny Nom.
Solve x = ln(x) 2013-07-19
From Udayan:
Sir/Madam, How to find the solutions of the equation, x = ln(x)
Answered by Penny Nom.
10 guys playing 8-9 hole rounds 2013-07-19
From Jason:
I have 10 guys playing 8 9 hole match-play rounds. I need to have everybody play the other 9 men, but need to limit them to only a 9 hole match between them. If we need to go 3-3-4 or 2-4-4 for a better solution, that may be possible.
Answered by Victoria West.
1,990 cubic yards in gallons 2013-07-16
From Tayler:
You have 1,990 cubic yards of concrete to pour. If the average swimming pool holds 25,000 gallons, how many pools can Be poured with the 1,990 cubic yards of concrete.

1 cubic yard= 27 cubic feet

Answered by Penny Nom.
A quadratic with no real roots 2013-07-15
From Pranav:
If ax^2+bx-5=0 has no real roots. Then d sign of 1) a+b-5 2) (25a-5b-5)(4a+2b-5) will be what ??
Answered by Penny Nom.
150 litres has been drawn from a tank 2013-07-14
From henry:
When 150 litres has been drawn from a tank, it is 3/8 full, how many litres will the tank hold?
Answered by Penny Nom.
Dirt to fill a pool 2013-07-13
From Neil:
I had a 24 foot diameter pool. The perimeter of it was at ground level. The pool sloped 1 foot deeper to the middle. In other words "a 1 ft. dish. How many cubic yards of dirt do I need to fill this hole?
Answered by Penny Nom.
Simultaneous equations 2013-07-10
From Warren:
solve this simultaneous equation:
xy=4
2x+3y=14

Answered by Penny Nom.
Distance as a function of acceleration 2013-07-10
From Tom:
If you start at a stoplight and your acceleration is 16t - t^2, how far have you gone after 8 seconds?
Answered by Penny Nom.
Scheduling a 12 person fishing 2013-07-09
From Don:
Greetings Math Central!
I searched the index, but could not find a similar quandary.
Here's my predicament:
I am seeking help in scheduling a 12 person fishing outing.
We have four boats.
I would like to schedule three fishermen to a boat.
We fish 2 1/2 days, split into morning and afternoon sessions.
Thursday afternoon, Friday morning, Friday afternoon, Saturday morning, and Saturday afternoon
We switch boats after lunch, with each fisherman moving to a different boat, with different partners.
I would like to have each fisherman fish with as many others in the group as possible, with a minimum of partner duplication.
I would like to have each fisherman fish in all of the four different boats over the course of the trip at least once.

Would you please suggest a boat / fishing partner schedule that fits my parameters as closely as possible.

Thank you!

Don

Answered by Victoria West.
Diego and his dog 2013-07-09
From William:
john went hunting and walked at a speed of 4 km/h. one hour later, Diego and his dog went to meet john with a speed of 6 km/h and the dog was running between john and diego until they meet up with a speed of 18 km/h. After Diego meets john, how much distance did the dog run?
Answered by Penny Nom.
4 couples golfing 2013-07-06
From Brian:
We have 4 couples going on a 4 day golf vacation playing 4 rounds of golf. I have spent hours trying to set up a schedule that allows the 4 spouse to play together, and then each spouse to play with one of the other spouses (men with women) for the] other 3 rounds.
I would like the foursomes to be different as possible. Also, no-one should RIDE in a cart with the same person more than once.
I am not a math guy so I try to do this by working it out on paper, over and over again. It ain't working!!
If you can help, I am very thankful.
Brian

Answered by Victoria West.
The angle of elevation of the sun 2013-07-03
From Maurice:
A vertical pole with a length of 7m cast a shadow with a length of 5m. Calculate the angle of elevation of the sun and include a diagram.
Answered by Penny Nom.
8 golfers playing 8 games 2013-07-03
From Johan:
We have 8 golfers playing 8 games, and we want to schedule them so that they play together equal amount of times, or as close as you can get to that. They will be playing in two four balls each day. Anyone sorted this one out yet, I will really appreciate a solution? Thanks Johan
Answered by Victoria West.
13 golfers playing 5 rounds 2013-07-03
From Alan:
i have 13 golfers playing 5 rounds. want everyone to play together at least once.
Answered by Victoria West.
13 golfers 2013-07-02
From James:
13 golfers playing in 4 groups [4,3,3,3 per group] for four rounds. Can it be scheduled so that no two people play together more than twice?
Answered by Victoria West.
A league schedule for 6 hockey team 2013-07-02
From Giovanni:
Hello, I try to make a league schedule for 6 hockey team. Each team will play 36 games during the regular season.
I cannot find a proper schedule generator. Can you help me please?
Thank you

Answered by Victoria West.
Fantasy football league 2013-06-18
From Chris:
Hello, I am the commissioner of a fantasy football league. In this week, we play all 17 weeks of the season. It's a 12 team league, and we play each team twice. To do that, we have 5 "double header" weeks, which means you play 2 different teams. I am struggling to come up with a schedule to make this work. Does anyone have a solution?
Answered by Victoria West.
A 3 acre plot 2013-06-17
From Chad:
I have 3 acres 100ft at the top and bottom what is the length of each side to make up the 3 acres
Answered by Penny Nom.
{(1+x)^1/3-1/3X(1+x)^-2/3}/(1+x)^2/3 2013-06-17
From STEPHEN:
{(1+x)^1/3-1/3X(1+x)^-2/3}/(1+x)^2/3
Answered by Penny Nom.
An isosceles tiangle 2013-06-16
From Izzy:
what's the height of an isosceles triangle which has a base of 50 m, and both of the other sides are 25 m?
Answered by Penny Nom.
16 golfers in 2 groups of 8 2013-06-15
From Derek:
we are 16 golfers.2 groups of 8."A" group & "B" group.We are playing 4 rounds,4 players at a time,2 from each group.We don't want to play with or against the same person twice.IE we want to play with 7 different people each day.Is it possible ?-thanks-Derek
Answered by Victoria West.
4=(1+.08)^x 2013-06-15
From Samantha:
I can't seem to understand how to solve for x when it is in power form, here is the equation:

4=(1+.08)^x

Answered by Penny Nom.
Surrounding a pipe with concrete 2013-06-11
From jeanne:
I need 2 inches of cement around a 20 inch pipe that is 20 feet long. Thank you.
Answered by Penny Nom.
Two numbers whose difference is 16 2013-06-08
From Johan:
Two numbers whose difference is 16 and whose sum is 120
Answered by Penny Nom.
A kennel for a beagle 2013-06-03
From david:
Hi, I'm building some beagle kennels and I am in need of help with an angle problem. I need to place a roof on my kennel with a drop of 2inches across 3ft 10inches. the posts on the right side will be 5ft and the post on the left will be 4ft 10in. the posts are 4x4 and the space to be covered is 3ft 10in from the outside of the 4x4. Please help, thanks.
Answered by Penny Nom.
Four circles 2013-05-29
From varsha:
four circular cardboard pieces each of radius 7cm are placed in such a way that each piece touches two other pieces. find the area enclosed by the four pieces.
Answered by Penny Nom.
21 golfers 2013-05-27
From Linda:
A database search for a question on how to schedule 20 golfers in groups of 4 was very helpful ( Mona's answer referred to by C. Fisher 9-3-2008) I'm very glad I found this sight. Our problem is we will have 21 golfers in the second half of the season. Is it possible to figure out that schedule? I assume we will have to include byes, but I'm not sure how to go about doing that. Thank you.
Answered by Victoria West.
Simson line 2013-05-23
From Nazrul:
From any point p lying on the circumcircle of the triangle perpendiculars PD , PE are drawn on BC,CA respectively. The line segment ED intersects at the point 0 ; prove that PO ⊥ AB ( PO is perpendicular to AB ).
Answered by Chris Fisher.
Two overlapping circles 2013-05-22
From Alexandra:
There are two overlapping circles. The two non-overlapping regions have areas A and B. As the area of overlap changes, the values of A and B also change. Prove that no matter how big and small the overlap is, the difference between A and B is always the same.
Answered by Penny Nom.
The square footage of a 17 inch by 17 inch tile 2013-05-18
From Leroy:
How do I calculate the square footage of a 17inch by 17 inch tile
Answered by Penny Nom.
r=10cos(t) in cylindrical coordinates 2013-05-18
From r.m:
question from calculus exam: what is the figure obtained having eqn.r=10cos(t) in cylinderical coordinates? i know it is a cylinder with center (5,0) ,but can't the equation represent two cylinders ,one with center (5,0) and the other with center (-5,0). thanks for any help.
Answered by Penny Nom.
The percentage of fat in a container of yogurt 2013-05-17
From Katy:
There is a question that I' having trouble with. It says: "A container of yogurt contains 228 g of water, 54 g of carbohydrates, 12 g of protein and 6 g of fat, what percentage of the yogurt is fat?" How do you convert 6 g into percentage?
Answered by Penny Nom.
Cutting three sections of pipe 2013-05-16
From Michael:
A plumber cuts three sections of pipe from a 12’ length of ABS pipe, the lengths of the sections are 33 3/8”, 56 5/8” and 39 7/8”. What is left over from the full length, if the saw cut is 1/8” wide?
Answered by Penny Nom.
8 3/4 as a decimal 2013-05-15
From Luisa:
How to make 8 3/4 into a decimal
Answered by Penny Nom.
HCF and LCM 2013-05-15
From Kelly:
If HCF and LCM of two numbers is 7 and 20 respectively, then the number/s is/are….
Answered by Penny Nom.
A circle and a chord 2013-05-14
From yashvardhan:
A circle which radius 13cm. Find the length of the chord which is 5 cm away from center
Answered by Penny Nom.
An equation in two variables 2013-05-14
From Steve:
Verify solutions to an equation in two variables. 4x-2y=8 (3, 2)
Answered by Penny Nom.
Dice 2013-05-14
From Julia:
What is the probability of rolling(as many times as necessary) a die and getting a total number(sum) when all the rolls are added together of 11?
Answered by Lorraine Dame.
A 2-digit number 2013-05-13
From A teacher:
Find a “2-digit number” where the sum of the 1st digit (on the Left) and the square of the 2nd digit equal the same number.
Answered by Lorraine Dame, Harley Weston.
Compound interest 2013-05-10
From Kyla:
I am doing correspondents and cannot figure out how to explain the solution to this question, I do not understand how they came to this answer and need it explained step by step so I can complete the following questions in this unit, please help!!! Calculating compound Interest

Johnny invest $800 that pays 8% compounded quarterly for 5 years. How much is the investment worth at the end of the 5th year?

Answered by Penny Nom.
Water use in a rectangular flush tank 2013-05-10
From milo:
A rectangular flush tank 22" by 71/4 contains water to depth of 17" how many gallons of water will be saved if a conservation device reduces the capacity to 3/5 of this amount? And reduced to the nearest tenth
Answered by Penny Nom.
20 golfers over 5 days 2013-05-09
From Russ:
we have 20 golfers over 5 days, with 4 people per foursome which makes 5 groups each day. I would like to each person to play with everyone at least once if possible. Do you have a formula for this?
Answered by Victoria West.
siin (A) and sin (A/2) 2013-05-09
From shanaia:
given that sin A=4/5 and A is obtuse.find sin (A/2)
Answered by Penny Nom.
A triangle and an incircle 2013-05-09
From Max:
On my Geometry Test about tangent, chord, and secant lengths, my teacher gave an extremely difficult problem.
It was a Circle inscribed in a Triangle with all triangle sides being tangents and lengths were given. My class was told to find the length of each segment of the line. The points on each line were the vertexes of the triangle, and the point where the line hits the circle.
Please explain how someone could do this.

Answered by Chris Fisher.
The surface of a solid 2013-05-07
From mustafa:
I am doing a research paper on solid surface applications . Part of the project is to find four at least in our world today and explain what there purpose is. I really need help in this area because I've been searching the internet for where solid surface are used in our world today and I really can't find anything.. Thanking you in advance, mustafa
Answered by Penny Nom.
Drawing a pentagon 2013-05-06
From jacob:
hello. i am trying to draw a template for a project. but i don't have a protractor. i'm trying to draw a normal pentagon with 2cm sides. what i'm trying to figure out is the distance in centimeters from point a, to c, in a straight line. this is needed so that i can measure and create the shape without a protractor, and keep the angles and sides correct. if you can help me with the measurement, or help me another way, any help would be greatly appreciated. thank you.
Answered by Penny Nom.
A silver figurine 2013-05-03
From Renee:
I have a silver figurine, the density of silver is 10.5g/cm3. if the figurine weighed 149 g. to determine the volume, he dropped it into a cylindrical glas of water. the diameter of the glass is 6 cm and the figurine was pure silver, by how much should the water level in the glass rise?
Answered by Penny Nom.
A word problem 2013-05-03
From Shaneaya:
Keikio spent the day bird watching and counted 34 more birds in the morning than in the afternoon. If she counted a total of 76 birds, how many birds did she count in the afternoon?
Answered by Penny Nom.
The rings of a bull's eye 2013-04-28
From Math:
The bull’s eye of a target is a circle 20 cm in diameter. It is surrounded by four rings, each 15 cm wide. Find the area of each ring.
Answered by Penny Nom.
What is the original price? 2013-04-25
From Dylan:
shirts on sale for 25% off, the sale price is 12.75. What is the original price?
Answered by Penny Nom.
How do i reverse this formula? 2013-04-20
From MK:
I have 22,000,000 worth of gold and wants to want to resell the gold back with a profit. Each transaction of gold will have a transaction fee of 15%, so if i resell the gold back at 22,000,000, I will only get back 18,700,000 after 15% transaction cut. The current formula is X-15%X = Y where X = 22,000,000 and Y = 18,700,000 Will u be able to come up with a new formula if we don't know the value of X but we know the value of Y
Something like this X-15%X = 22,000,000. How do i reverse this formula to find the value of X
Thanks

Answered by Penny Nom.
A circle, a chord and an arc 2013-04-16
From Tim:
Morning,
I was told the chord is 3000mm and the depth of chord is 300mm, I can find the radius,(1500 squared + 300 Squared) Divided by (300 + 300) but don't know how to calculate the length of the circle defined by the chord. Regards
Tim

Answered by Penny Nom.
A cyclic rhombus 2013-04-16
From Marisa:
I know that the only rhombus that can be inscribed in a circle is a square, but why is that? I've been racking my brain and the internet for solutions, but have found no logical explainations in relation to the arc degrees and angles. Please help.
Answered by Chris Fisher.
8 golfers, 3 rounds 2013-04-15
From justin:
I have 8 golfers going on a trip. We have 3 rounds to play. We would like to play with everyone (with or against.) Is this possible? If so could you send me the formula please and thank you!
Answered by Robert Dawson.
How do i write log_8(P) =7 in exponential form? 2013-04-14
From nancy:
How do i write this in exponential form log8P =7
Answered by Penny Nom.
4 linear equations with 3 unknowns 2013-04-12
From Marian:
how to solve for 3 unknowns in 4 simultaneous equations
Answered by Penny Nom.
The area of a right triangle 2013-04-10
From derek:
The legs of a right triangle measure 4 and 6 inches. The area of the triangle is
Answered by Penny Nom.
Sharing in a car rental 2013-04-09
From Marilyn:
8 persons are planning to share equally the cost of a rental car. If 1 person withdraws from the arrangement and the others share equally the entire cost of the car, then the share of each of the remaining persons is increased by--------------?
Answered by Penny Nom.
10 golfers playing four rounds 2013-04-09
From Terry:
I've searched "golf" and "golf 10" but don't see 10 golfers playing four rounds (each round having one foursome plus two threesomes, and no "flights" as in one earlier answer); each player playing at least once with a maximum number of the other nine over the four rounds?
Answered by Victoria West.
The equation of a line 2013-04-04
From Miranda:
My coordinates provided are (6, 6) and (-8, 9) and i need to put this into ax=bx=c format. I remember the equation to find the slope of y2-y1/x2-x1 to get the slope. However when i put that back into the y=mx+b format it still leaves me with a lot of variables. Please help :)
Answered by Penny Nom.
A gravel pile in the shape of a triangular pyramid 2013-04-04
From Casey:
Hello
Right now I am stuck and I feel embarrassed because I feel like the answer is so easy I should know it.

I am working on a project and need to find a volume of gravel it will take to occupy this triangular prism like area. I am not sure what formulas I should use whether it be that for the volume of a pyramid or something more complex? Basically it forms a right triangle at one side then from there all points slope to one singular point about 10412mm away.
I am attaching a picture drawn up in paint with the actual dimensions to clear up any confusion.

Thank you for any help. Casey

Answered by Penny Nom.
Question 2013-04-04
From Casey:
Hello
Right now I am stuck and I feel embarrassed because I feel like the answer is so easy I should know it.

I am working on a project and need to find a volume of gravel it will take to occupy this triangular prism like area. I am not sure what formulas I should use whether it be that for the volume of a pyramid or something more complex? Basically it forms a right triangle at one side then from there all points slope to one singular point about 10412mm away.
I am attaching a picture drawn up in paint with the actual dimensions to clear up any confusion.

Thank you for any help. Casey

Answered by Penny Nom.
16 golfers over 15 days 2013-04-02
From Mark:
16 golfers, each plays the other 15 golfers once each over 15 days. played in four foursomes each day. What is schedule that allows each golfer to play with other golfers an equal number of times?
Answered by Victoria West.
Every vertex of a cube is assigned a number +1 or -1 2013-04-01
From Shankar:
Every vertex of a cube is assigned a number +1 or -1 . Every face has a number that is the product of all the numbers in its corners. Then the 14 numbers are summed up ( all the vertices and faces ). Can the sum be 0?
Answered by Chris Fisher.
The distance between 2 lines in 4D 2013-04-01
From samane:
how can i measure the distance between 2 lines in 4D??
Answered by Chris Fisher.
We can't write sinx and cosx as a finite polynomial. 2013-03-31
From rimoshika:
prove that we can't write sinx and cosx as a finite polynomial.
Answered by Walter Whiteley.
Simultaneous equations with fractions 2013-03-31
From Terence:
5/x-6/y=1 17/x+30/y=16 I been spending whole day to solve this question. Would be very grateful if you can help I try The denominator value is a equations term which make is simultaneous equations so hard.
Answered by Penny Nom.
The area of an isosceles right triangle 2013-03-30
From Michelle:
We have a triangle it is a right isosceles. The sides touching the right angle are 10 cm. They hypotenuse is 14.1 (the base) we are trying to find the height of this so we can figure out the area of a shaded sector of a circle.
Answered by Chris Fisher.
A shortest distance problem 2013-03-28
From LYNDELL:
I have a right triangle and know the length of all sides. How do I calculate the shortest distance from the vertex of the 90 degree angle to the hypotenuse?
Answered by Penny Nom.
Tangents to the curve y = x^3 2013-03-24
From Ethan:
How many tangent lines to the curve y = x^33 pass through the point (2, 4)? For each such line, and the exact coordinates of the point of tangency on the curve.
Answered by Penny Nom.
Trigonometry 2013-03-23
From Tizoc:
I am in a trig class and I have a conflict. When solving the length of a side, I know what trig function to use, but I do not know what angle to use in a calculator. To make this a little more understandable, if I have all the angles available in a right triangle and I use the tangent function, how do I know what to use?
Heres what I do not know what to put in my calculator: Tan(?)
Thanks in Advance!

Answered by Penny Nom.
Cumulative Frequency Math Question 2013-03-20
From Primadonna:

Hi,

Please help me solve this question.

Thank you so much.

The cumulative frequency table below shows the length of time that 30 students spent text messaging on a weekend.

Minutes Used Cumulative Frequency
31-40 2
31-50 5
31-60 10
31-70 19
31-80 30

Which 10-minute interval contains the first quartile?

(1) 31–40

(2) 41–50

(3) 51–60

(4) 61–70


Answered by Penny Nom.
Five players in a 4-player partnership game 2013-03-19
From Dee:
I have 5 players who are playing in a 4-player partnership game & want to rotate the 5th person in in such a way that each person is each other person's partner for a game. How can such a rotation be done?
Answered by Penny Nom.
The dimensions of a rectangle 2013-03-19
From stephanie:
the length of a rectangle is 5m more than twice its width, and the area of the rectangle is 88m^2 find the demensions of the rectangle
Answered by Penny Nom.
Six-digit passwords 2013-03-19
From Judy:
Hello:
Can you please explain why the answer to the following question is 10^6? What is the number of possible six-digit passwords when using the digits 0 through 9, with repetition allowed.
Thank you.

Answered by Penny Nom.
Tiling a floor 2013-03-18
From whitley:
Question from Whitley, a student:

How many square feet of tile do you need to cover the floor of a room that is 20ft, 25ft, 15ft, 20ft, 5ft, and 5ft

I made a replica of the floor

Answered by Penny Nom.
An algebraic exercise 2013-03-18
From autumn:
given: f(x)=x/(x^2+2) determine: [f(x-h)-f(x)]/h
Answered by Penny Nom.
If 4 cooks can bake 8 pies in 6 hours????? 2013-03-18
From Kenneth:
Hello:

I want to determine which quantities are directly and inversely proportional in order to determine the answer for the following:.

If 4 cooks can bake 8 pies in 6 hours, 2 cooks can bake how many pies in 4 hours?

Answer: 2 2/3 pies

Can someone fully explain what I need to know in order to determine what is directly and inversely proportional in the example above?

I thank you for your reply.

Answered by Robert Dawson.
Row echelon form 2013-03-15
From Shawn:
Please help me put matrix in row echelon form:

1 9 8 0
5 8 1 35
1 -4 -1 17

Answered by Penny Nom.
Driving a rental car 2013-03-13
From teresa:
Driving a rental car x miles cost y= 0.25x + 25 dollars.

(a) How much would it cost to rent the car but not drive it?
(b) How much does it cost to drive the car 1 additional mile?
(c) What is the y-intercept of y= 0.25x + 25? What does it represent?
(d) What is the slope of y= 0.25x + 25? What does it represent?

Answered by Penny Nom.
A 3 digit number puzzle 2013-03-11
From Bonnie:
My number has 3 digits. My number is not a multiple of 100. My number is a multiple of 50. My number is a factor of 1000. What is my number?
Answered by Penny Nom.
A chart for y=mx+b 2013-03-11
From jacinda:
how to make a chart for y=mx+b
Answered by Penny Nom.
Fencing a parcel of land 2013-03-11
From Koleen:
How much fencing do I need for an 56,850 parcel of land for a 15 stable horse barn
Answered by Penny Nom.
6 miles in 28 minutes what's my average speed? 2013-03-05
From lynda:
6 miles in 28 minutes what's my average speed?
Answered by Penny Nom.
f(x) + f ''''(x)=0 2013-03-05
From Andreea:
Hei. I don’t speak lot of english but here is my question,hope u understand: f(x) + f ````(x)=0. so, my question. what is f(x), where f ````(x) is f(x) derivative by four time ? i tried to find the answer and i knew f(x) is something like that f(x)=e^x*sinx but miss something.
Answered by Brennan Yaremko.
What is the subtotal & GST? 2013-03-04
From REBECCA:
Hi, I have a total of $300. What is the subtotal & GST? How do I work backwards?
Answered by Penny Nom.
Markup and discount to get a sale price 2013-03-02
From Warren:

If you are an ardent shopper you will be aware of the discount (% decrease) marketing strategy to produce the sale price. I began to wonder just how much mark-up (% increase) was added initially to an item in order to deliver the discounted sale price.

It was simple to construct a formula to convert one to the other.

For example a 30% (mark down) decrease in price required a 42.86% (mark up) to increase the price initially to Tag price. However when I started apply the formulae to a number of circumstances I noticed some very interesting circumstances associated with this relationship

For example a 99.999999% decrease is numerically equal to a 9,999,999,925% increase (huge ratio difference) while conversely a 0.1% decrease is numerically equal to a 0.10010010% increase (Infinitesimally small ratio difference) so the patterns appears logarithmic not linear.

However what really caught my attention was this pattern

See http://www.ozsmart.com/doc/percentage.htm

My questions is why does the decline in the ratio of increases in required mark up (%) price peak at 50 and 51% precisely and then expand again as the percentage move higher

Any explanation would be appreciated


Answered by Harley Weston.
A linear programming problem 2013-02-27
From Kelley:
A manufacturer of skis produces two types: downhill and cross-country. Use the following table to determine how many of each kind of ski should be produced to achieve a maximum profit. What is the maximum profit? What would the maximum profit be if the time available for manufacturing is increased to 48 hours.
  Downhill Cross-country time available
manufacturing time per ski 2 hrs 1 hr 40 hr
finishing time per ski 1 hr 1 hr 32 hr
profit per ski $70 $50  

Answered by Penny Nom.
100 animals 2013-02-25
From Jessica:
You have exactly $100.00 to spend. You must get 100 animals.
The chicks cost $0.10 each.
The pigs cost $2.00 each.
The sheep cost $5.00 each.
You must get some of each animal.
How many of each animal can you get?

Answered by Penny Nom.
The tangent to a circle at a point on the circle 2013-02-22
From Andrew:
What is the equation of the line tangent to the circle with equation x^2+y^2=25 at the point (-4,3)
Answered by Penny Nom.
Fractions over fractions 2013-02-22
From Kenssa:
Question from Kenssa, a parent:

Simplify: 3-(7/x)-(6/x^2) / (3/x^2)+(5/x)-2

Answered by Harley Weston.
Your profit should be 20% of your cost 2013-02-21
From anis:
if the customer says your profit should be 20% of your cost..

1) my selling value should be cost +20% of cost = SP

or

2)cost / 0.80 = SP which is correct...if the second answer is right how should i justify my customer

Answered by Penny Nom.
Solve sin 3x = -0.1254 with x between o and 360 degrees 2013-02-21
From David:
sin 3x = -0.1254 0
Answered by Harley Weston.
Solve sin 3x = -0.1254 with 0 2013-02-21
From David:
sin 3x = -0.1254 0
Answered by Harley Weston.
Solve 8 logx > x 2013-02-19
From Fulvio:
Hi, could you solve this equation in algebraic form:

8logx>x

Answered by Harley Weston.
A trig word problem 2013-02-18
From Amy:
Susan notices an unusual rock formation at an angle of 30 degrees to the right of her direction of travel as she is white water rafting. as she continues in a relatively straight path for 20 m, the rock formation appears to be at 45 degrees to the right. she cotninues until the rock formation is directly to her right. how far is she fromt he rock formation?
Answered by Penny Nom.
Extraneous solutions 2013-02-18
From Eileen:
(5x+4)^1/2-3x=0
Answered by Penny Nom.
Related rates 2013-02-17
From Ishaak:
A hemispherical bowl is filled with water at a uniform rate. When the height of water is h cm the volume is π(rh^2-1/3 h^3 )cm^3, where r s the radius. Find the rate at which the water level is rising when it is half way to the top, given that r = 6 and the bowl fills in 1 minute.
Answered by Penny Nom.
Swimming upstream and downstream 2013-02-15
From Kimberly:
a beaver swims down to its lodge 4 times faster than it swims upstream. how many times faster is its speed in still water than the speed of the stream?
Answered by Penny Nom.
Chocolate bars and puzzles 2013-02-15
From sandy:
Alan bought a box of puzzle and two bar of chocolate for $28.
Jan bought 3 boxes of puzzle and a bar of chocolate similar to Alan for $69.
a) Find the cost of a box of puzzle
b) Find the cost of a bar of chocolate

Answered by Penny Nom.
An electron in a TV tube 2013-02-15
From anu:
an electron in a TV tube is beamed horizontally at a speed of (50^6) m/sec. towards the face of a tube 40 cm away about how far will the electron drop before it hits? no information has been provided of initial height from where it is beamed.
Answered by Robert Dawson.
A word problem involving toys 2013-02-14
From sandy:
Each boy gets 5 toys.Each girl gets 3 toys.There are 150 pupils.
The boys had 74 more toys than girls.
How many boys?
How many girls?

Answered by Penny Nom.
A curve in 3-space 2013-02-14
From pardeep:
we have to show that the curve r(t)=(cos t)i+(sin t)j+(1-cos t)k ,0<=t<=2pie; is an ellipse by showing it to an intersection of a right circular cylinder and a plane. i got the eqn. of the cylinder but did not get the eqn of plane.
Answered by Harley Weston.
Dropping supplies from an airplane 2013-02-14
From Claire:
An airplane flying at an altitude of 3500 feet is dropping supplies to researchers on an island. The path of the plane is parallel to the ground at the time the supplies are released and the plane is traveling at a speed of 300 mph.
a) write the parametric equations that represent the path of the supplies
c)How long will it take for the supplies to reach the ground?
d) how far will the supplies travel horizontally before they land?

Answered by Penny Nom.
Is a line lying in a plane an open or a closed region? 2013-02-13
From pardeep:
please help with this question on the open and closed intervals :: Is a line lying in a plane an open or a closed region? my teacher says it is a closed region reasoning out that it contains all it boundary points. please help how is it so?
Answered by Harley Weston.
The height of an aerial 2013-02-12
From ASIT:
A vertical aerial stands on a horizontal ground. A surveyor positioned due east of the aerial measures the elevation of the Top as 48 degree. He moves due south 30metre and measure the elevation as 44 degree. determine the height of the aerial
Answered by Penny Nom.
A cubical shipping container 2013-02-12
From Chantel:
The length, width and height of a cubical shipping container are all decreased by 1.0 ft, thereby decreasing the volume of the cube by 37 ft3(feet-cubed) What was the volume of the original container? (recall: volume of cube with a side length x=x3) Define the variable, write a quadratic equation and solve by factoring.
Answered by Penny Nom.
The equation of a circle 2013-02-11
From mhd:
Complete the equation of the circle centered at(0,4) with radius 3
Answered by Penny Nom.
Simultaneous equations 2013-02-10
From Michael:
2P + 1/3V =8
3P - 2/V=5

Answered by Penny Nom.
The height of a sheet metal cone 2013-02-09
From Charles:
Sheet metal cone.
I need a cone with a finished base of 38.19719 diameter The cone is to be from a 48 diameter round with the wedge cut out. The best calculation I have is the arc is 286.479. (correct?)

Could you verify this arc angle but more so what is the cone height?

Answered by Harley Weston.
Brian is x years old 2013-02-06
From Sabrina:
Brian is x years old.
Peter is 4 years older than Brian.
Amy is 2 years younger than Brian.
The total of their ages is 26 years.
Work out the value of x.

Answered by Penny Nom.
What is the domain of f(x)=sin(ln(x))/ln(x)? 2013-02-06
From Behrooz:
Hi, the following problem may be interesting: What is the domain of f(x)=sin(ln(x))/ln(x)? Be careful, domain is not obvious. Best regards Behrooz
Answered by Penny Nom.
Exponential form 2013-02-05
From Sandra:
What is 37,008 in exponential form? Help me please!!!
Answered by Penny Nom.
We have 10 golfers and four rounds of golf in two days 2013-02-05
From Steve:
We are having a golf tournament. We have 10 golfers and four rounds of golf in two days. We are going to break the group of 10 into two flights. Flight A and Flight B. We would like all members in Flight A to play one nine hole match against each other. All players in Flight B to play one nine hole match against each other. Could we play in groups of 4,3 and 3? What is the best way to make the pairings work?
Answered by Robert Dawson.
Integration from 0 to 2pi of 1/(3cos x + 2) dx 2013-02-04
From ankit:
Integration from 0 to 2pi of 1/(3cos x + 2) dx
Answered by Harley Weston.
A relation on a collection of sets 2013-02-04
From raji:
suppose R be a non empty relation on a collection of sets defined by ARB if and only if A intersection B is nullset.then.....the correct answer is R is symmetric and not transitive...my doubt is how it is symmetric
Answered by Penny Nom.
7x-1/4-1/3[2x -1-x/2]=19/3 2013-02-03
From M:
7x-1/4-1/3[2x -1-x/2]=19/3
Answered by Penny Nom.
Multiplication in base two 2013-02-01
From Michael:
multiply in the indicated base

110two*11two

Answered by Penny Nom.
The fourth side of an irregular polygon 2013-02-01
From Emran:
I have a irregular polygon. I know 3 of the 4 sides, and 2 of the angles. A-B is 285, B-C is 149, and C-D is 310. Angle B is 135 degrees. and Angle C is 45 degrees. Is there a formula to solve for the final side? Thanks.
Answered by Penny Nom.
The multiplication table for the different bases 2013-02-01
From sylvia:
I am having a difficult time trying to figure out how to fill in the multiplication table for the different bases. i don't know how to get the numbers.
Answered by Penny Nom.
The area of a rectangle 2013-01-29
From Bali:
if the length of the rectangular field is decreased by 5m, its area is decreased by 70 sqm. if the width is increased by 6m, the area is increased by 180 sqm. determine the length and width of the rectangular field?
Answered by Penny Nom.
w varies jointly with the square of u and the cube of d 2013-01-26
From Desiree:
i don't understand how to answer this 'w varies jointly with the square of u and the cube of d'
Answered by Penny Nom.
We have 2 teams of 6 couples each that play 6 games 2013-01-25
From Denise:
We put together a game night where we have 2 teams of 6 couples each that play 6 games. We haven't been able to figure out an arrangement that allows each couple to play each game with a different couple from the opposite team (i.e. Team A couples play every game with a different couple from Team B). Is this possible? It works with 2 teams of 5 couples each.
Answered by Chris Fisher.
Use 9,10,6,8, and 4 to make 10 2013-01-25
From Able:
How do you calculate using BEDMAS 9,10,6,8,4 to equal 10
Answered by Penny Nom.
A geometry word problem 2013-01-24
From Matthew:
"G", "A" and "B" are collinear. "A" is between "G" and "B". "GA" Is 14 less than 3 times "AB". "GB"=46 Find "GA"

How would you solve this?

Answered by Penny Nom.
Golf for 17 2013-01-23
From Richard:
I have looked through the examples on your site but can't find one that suits. I am fiddling with one to see if I can get a result; but not confident. I have 17 players playing 6 rounds in 3 groups of 4 and 1 group of 5. Two players will not be included in any 5 ball group. Can you help me please.
Answered by Victoria West.
Golf for 18 for 18 weeks 2013-01-22
From Al:
We have 18 two man golf teams and we play once a week for 18 weeks. We need a formula to schedule these teams so that they play each other one time only.
Answered by Victoria West.
A railroad embankment 2013-01-22
From clint:
the pennsylvania railroad found it necessary,owing to land slides upon the roadbed,to reduce the angle of inclination of one bank of certain railway cut near pittsburgh,pa.,from an original angle of 45 degrees to a new angle of 30 degrees. the bank as it originally stood was 200 ft.long and had a slant length of 60 ft.. find the amount of the earth removed, if the top level of the bank remained unchanged.
Answered by Penny Nom.
Plotting y=f(x)+2 2013-01-21
From Gavin:
how would you plot y=f(x)+2
Answered by Penny Nom.
How many students have neither brown hair or hazel eyes? 2013-01-20
From Julie:
There are 28 students in a class. 15 have brown hair, 17 have hazel eyes, and 10 have both. How many students have neither brown hair or hazel eyes?
Answered by Penny Nom.
Maximize profit 2013-01-19
From Chris:
A firm has the following total revenue and total cost function.
TR=100x-2x^2
TC=1/3x^3-5x^2+30x
Where x=output
Find the output level to minimize profit and the level of profit achieved at this output.

Answered by Penny Nom.
An equilateral triangle inscribed in a circle 2013-01-17
From Nicole:
How do you find the shaded region of a circle if an unshaded equilateral triangle in inscribed in it. The only other things I know about the problem are that the side lengths of the equilateral triangle are 14 inches.
Answered by Penny Nom.
The weight of a pipe full of water 2013-01-16
From joe:
how do you figure out the weight of a pipe full of water
Answered by Harley Weston.
A balance puzzle 2013-01-14
From Emily:
I'm stuck on this problem and feel that it can't work! The problem is: there is a door with a balance scale on it. In order to open the door, you have to use the 5 weights to balance the scale. On the left side you have 2 boxes to place the weights and on the right side you have 3. You're 5 weights are as follows: 1lbs, 2lbs, 3lbs, 4lbs, 5lbs. How do you place the weights so that the scale is balanced?
Answered by Penny Nom.
Percent as a fractional relationship 2013-01-14
From Kenneth:
What does the following indicate?

"A percent is another way of expressing the ratio or fractional relationship of two numbers."

I know what a ratio is, but the term "fractional relationship" confuses me.

I thank you for your reply.

Answered by Penny Nom.
A large ditch 2013-01-11
From steve:
hi
i have a large ditch to make a road entrance accross, the ditch is a green field on one side and a highway on the other side the ditch depth is 1.8m on the field side and 3.2 on the highway side the culvert pipe is 1,2m in diameter are you with me so far? the entrance from the highway will be 26m wide tapering in to 6m - ,20 m in from the highway there will be a concrete slab of 150mm over the top

the fall from highway to field must not exceed 1/8

any ideas how much stone it will need

Answered by Harley Weston.
The side lengths of a triangle 2013-01-07
From Katie:
How do I find the side lengths of a triangle knowing the base and its height?

Thank you,

Answered by Penny Nom.
A trig identity 2013-01-04
From Tehmas:
Prove sinC+sinD=2sin(C+D/2)cos(C-D/2)
Answered by Harley Weston.
The quadratic formula 2013-01-03
From itsel:
Find the discriminant ans use it to determine the use the quadratic formula to solve the equasion -2x^2+3x+2=0
Answered by Penny Nom.
Mod versus Rem in Turing 2013-01-01
From Eric:
I am a teacher teaching computer science using Turing. I am having difficulty understanding why one would use the mod operator versus the rem remainder operator.

Mod seems to make the resulting sign depend on the sign of the divisor, whereas rem makes the resulting sign depend on the dividend.

Examples:

11 mod 5 = 1 and 11 rem 5 =1
-11 mod 5 = 4 and -11 rem 5 = -1
11 mod -5 = -4 and 11 rem -5 =1
-11 mod -5 = -1 and -11 rem -5 = -1

What I can't understand is why this would matter. For example, -11 / 5 = -2.2 and 11 / -5 = -2.2 get the same result.
So how is a remainder dependent on the sign of one of the parts? What benefit would using one over the other have?

Any insight would be most helpful!

Eric

Answered by Harley Weston.
A triangular island 2012-12-29
From Udit:
A long time ago Mr Gibson found an island shaped as a triangle with three straight shores of length 3 km,4 km and 5 km. He declared an 'exclusion zone' around his island and forbade anyone to come within 1 km of his shore. What was the area of his exclusion zone?
Answered by Penny Nom.
Solve 2x^4+x^3-37x>26x^2+12 using a graphical approach. 2012-12-29
From Tehmas:
Solve 2x^4+x^3-37x>26x^2+12 using a graphical approach.
I don't know how to find the x-intercepts or y-intercepts in this form.

Answered by Penny Nom.
A 12 acre parcel of land 2012-12-28
From John:
What are the dimensions of a 12-acre parcel if it is 3 square acres deep and 4 square acres wide?
Answered by Penny Nom.
Increasing the size of a gift box 2012-12-27
From Tehmas:
The dimensions of a gift box are consective positive integers such that the height is the lowest integer and the length is the greatest integer. If the height is increased by 1cm, the width increased by 2 cm, and the length increased by 3 cm, then a larger box is constructed and the volume is increased by 456 cm^3. Determine the dimensions of each box.
Answered by Harley Weston.
Two polynomials are divisible by x - 2 2012-12-26
From Tehmas:
Determine the values of m and n so that the polynomials 2x^3+mx^2+nx-3 and x^3-3mx^2+2nx+4 are both divisible by x-2.
Answered by Penny Nom.
A 4 digit phone lock using the digits 4, 2, 3 and 0 2012-12-24
From Ragu:
I would like to know formula for searching a combination of 4 digit. eg.

I knew my Phone Lock combination is my car plate number 4230. But this time, i used another way around instead of 4230.

But I am sure, there is only this 4 digits only. 4 & 2 & 3 & 0

Answered by Penny Nom.
The last 4 digits of a cell phone number 2012-12-22
From Jacob:
If the last 4 digits of a cell phone number, XXX-XXX-ABCD, are restricted where: A cannot be 0 or 1 and B cannot be 0 / how many possible 4 digit combinations are left?
Answered by Penny Nom.
3x6)+(12divided by2)-8= ? 2012-12-17
From christina:
hi my question that's been bothering me is what's
(3x6)+(12divided by2)-8= ?????

Answered by Penny Nom.
Milling round stock to square stock 2012-12-17
From Bryan:
Question from Bryan:
I want to know what the smallest diameter round is that will make a 3-1/4" square? Is there a formula for that? I am milling round stock into square.
Thank you.

Answered by Harley Weston.
An integral 2012-12-16
From Slavena:
integration of (lnx)^2 / x dx
Answered by Penny Nom.
An area bounded by lines 2012-12-16
From sidra:
find area bounded by functions:
y=x
y=2x
and y=5-x

Answered by Penny Nom.
A max/min problem 2012-12-14
From bailey:
A right angled triangle OPQ is drawn as shown where O is at (0,0). P is a point on the parabola y = ax – x^2 and Q is on the x-axis.

Show that the maximum possible area for the triangle OPQ is (2a^3)/(27)

Answered by Penny Nom.
The perimeter of an equilateral triangle 2012-12-14
From shreyaarora:
if the area of on equilateral triangle is 24/3cm,then what is its perimeter
Answered by Penny Nom.
A project on reclaiming water 2012-12-11
From shannon:
I'm doing a report to reclaim water off of our campus facility to store in a cistern to use to flush toilets.

In Southeast Wisconsin and average of 82 inches of rain and snow fall annually. I want to collect that off of the roof of our school building. The roof is 37128 square feet. how many gallons annually could I collect?

Answered by Penny Nom.
A parabolic bridge 2012-12-09
From Elizabeth:
1) The figure below shows a bridge across a river. The arch of the bridge is a parabola and the six vertical cables that help support the road are equally spaced at 4-m intervals. Figure B shows the parabolic arch in an x-y coordinate system, with the left-end of the arch at the origin. As indicated in Figure B, the length of the leftmost cable is 3.072 m.

I'm suppose to find the (x-h)^2=-4a(y-k) equation for this word problem and I really do not know where to begin.

Afterwards, I need to find the lengths of the other cables and the maximum height of the arch of the road as well which I am very confused about

Answered by Penny Nom.
Mark has 312 books 2012-12-07
From paul:
Mark has 312 books. He has 11 times as many fiction books as non fiction books. How many fiction books does he have?
Answered by Penny Nom.
An arrow is shot straight up in the air 2012-12-06
From heather:
The skywalk is a glass bottomed platform that hangs over the edge of the Grand Canyon and is suspended 4000 feet above the floor of the canyon. If a tourist shot an arrow straight up in the air from the observation platform, the arrow's height could be modeled by H(t) =-16^2 + 240 + 4000 where H(t) is the height of the arrow above the canyon floor in feet t seconds after being shot, t>0.
(a) Find H(5) and explain its meaning

Answered by Penny Nom.
The angles of elevation and depression 2012-12-03
From Chelsey:
a person on a balcony of one building looks towards a second building. if the angle of elevation to the top of the second building is 25 degrees, the angle of depression to the bottom of the second building is 17 degrees, and the balcony of the first building is 22 feet above the ground, what is the height of the second building?
Answered by Penny Nom.
Transposition error 2012-12-03
From Carol:
What is the correct term for when a child writes 61 instead of 16, or 83 for 38--when they switch the place values of a number?
Answered by Chris Fisher.
A circle inscribed in a triangle 2012-11-28
From Angie:
Circle O is inscribed in triangle ABC. Angle A = 50 and angle B = 60. Find arc XY in dgrees
Answered by Penny Nom.
Sinx=logx+x^2 2012-11-28
From yasmin:
Sinx=logx+x^2
Answered by Harley Weston.
The area of a triangle given its vertices 2012-11-27
From elizabeth:
how to calculate the area of a triangle using points ABC a (2,5) b ( -4,-3) c ( 6, -1)
Answered by Penny Nom.
Two sides of a triangle 2012-11-25
From Nazrul:
In the triangle ABC, AB>AC and the bisector AD of angle BAC intersects BC at D. How can I prove that angle ADB is an obtuse angle.
Answered by Penny Nom.
What is the probability that the sixth draw will be a red ball? 2012-11-23
From Patty:
The probability of drawing a red ball out of a bag containing one red ball and one black ball is 1/2. An experiment is conducted where these two balls are placed in a bag before drawing a ball out. A ball is drawn 5 times from the bag containing the two balls. Each of the first five times the red ball was drawn out. what is the probability that the sixth draw will be a red ball?
Answered by Penny Nom.
Golf for 2 groups of 4 for six rounds 2012-11-21
From Andy:
You have very kindly responded to a couple of questions I asked previously and I have tried to find the answer to my current query on the various links etc. My question: There are 8 of us going on a Golf Tour and we will be playing 2 groups of 4 for six rounds of golf. Is there a suitable schedule that works to ensure everyone plays with each other the same number of times (or as close as possible). Many thanks again for your help.
Answered by Victoria West.
A function problem 2012-11-18
From nahla:
f: IN --> IN
n --> f(n)
for every n that belongs the IN : fof(x) = 4n - 3
and for every n that belongs to IN f(2^n) = 2^(n+1) -1

Calculate f(993)

Answered by Penny Nom.
The square of any odd number, decreased by 1, is divisible by 8 2012-11-16
From bailey:
Prove that the square of any odd number, decreased by 1, is divisible by 8
Answered by Penny Nom.
A path around a pond 2012-11-16
From bailey:
hi there,
A circular pond is surrounded by a path 1 metre wide. The area of the path is 1/4 of the area of the pond.
Find the radius of the pond.

Thanks

Answered by Penny Nom.
Factor (6a^2-5a+1)(8a^2-6a+1)(12a^2-7a+1) 2012-11-16
From bailey:
Factorise f(a)=(6a^2-5a+1)(8a^2-6a+1)(12a^2-7a+1) thus find [f(a)]^1/2
Answered by Penny Nom.
How much should I price the earrings? 2012-11-13
From Sharon:
I want to sell some earrings and make a $12.00 profit on them I have to pay a 35% commission on them. How much should I price the earrings?
Answered by Penny Nom.
-5<2x+1<-1 2012-11-11
From Rahul:
How does -5<2x+1<-1 changes to 1<|2x+1|<5. What is the rule? Can we apply absolute value function throughout the first inequality i.e. can we say |-5|<|2x+1|<|-1|. But then it becomes 5<|2x+1|<1 which is impossible because 5 is not less than 1.
Answered by Penny Nom.
What's your secret? 2012-11-11
From ryan:
Tuscan school has a fair. At the fair, Pia plays "what's your secret? The game starts with 3 players who are told the same secret. Each of those 3 players tells another 3 player, and so on. At the end of the 3rd round, everyone who knows the secret will compare what they have heard. What number in exponent form represents the number of people who know the secret after 3 rounds? How many people is that?

Thank you very much.

P.S. I got 3 cube. which is 27

But when i map it out I end up with 39.

Answered by Penny Nom.
Practical uses of trigonometry 2012-11-11
From Michael:
Where can I find books or information on real life function of sine and cosine?
Answered by Penny Nom.
A mathematical expression with the answer 7 2012-11-10
From emily:
hey um i need to find and problem that fallows bedmas that has one division one multiplication and one sub and one add and one brackets and one exponents that has the answer of number 7
Answered by Penny Nom.
Converting a mixed number to a decimal 2012-11-10
From Mahaveer:
Convert mixed fraction to decimals: 4 2/6
Answered by Penny Nom.
The derivative of y = sin (30º + x) 2012-11-07
From Saskia:
derivative of y = sin (30º + x)
Answered by Harley Weston.
Joe and Chris each bought a six pack of cola 2012-11-01
From Loulou:
Joe and Chris each bought a six pack of cola. Joe gave 2/3 of his away and Chris gave half as many as Joe. how many more colas did Chris have than Joe
Answered by Penny Nom.
Filling a box with wood chips 2012-10-31
From lee-Anne:
Each bag of wood chippings will fill 4m3 how bags would be needed?
Volume of box is 572 cm3
L=11CM
W= 6.5CM
H=8CM

Answered by Penny Nom.
Frosting a three layer cake 2012-10-30
From johnathan:
Each layer of a 3 layer cake is a cylinder with height 7.5 cm. The bottom layer has diamegter 25 cm. The middle layer has dimegter 22.5 cm. The top layer has diameter 20 cm. The surface of the cke is frosted. What area of the cake is frosted. I get 1695.56 but the answer in the book is 2081.3
Answered by Penny Nom.
The sum of a number p and twenty-one 2012-10-30
From mayra:
the sum of a number p and twenty-one
Answered by Penny Nom.
Introductory algebra 2012-10-30
From kevon:
if x = 7 is used in the expression 2x + 5 what is the output
Answered by Penny Nom.
birdhouse in the shape of a rectangular prism 2012-10-29
From Tiffany:
Hello, I am Tiff M. from NYC.
I was solving a math problem about adding,subtracting,multiplying or dividing fractions and mixed numbers. I am not getting this question below.
* Laura is building a birdhouse in the shape of a rectangular prism. The base of the birdhouse has an area of 5 square inches. The height of the birdhouse is 2 7/12 inches. Laura calculated the volume of the bird house to be 10 7/12in. Is she correct?
If not, what did she do wrong?

Answered by Robert Dawson.
Forming an algebraic expression 2012-10-27
From ashley:
A sidewalk has a constant width W and comprises several short sections having lengths 12, 14,and 10 feet. How do I find the simplified expression that gives the number of square feet of sidewalk?
Answered by Penny Nom.
An implicit differentiation problem 2012-10-26
From Katie:
find y' of x^2y-2y^3=3x+2y
Answered by Harley Weston.
How fast is the distance between the aircraft and the car increasing? 2012-10-24
From Steven:
At a certain instant an aircraft flying due east at 240 miles per hour passes directly over a car traveling due southeast at 60 miles per hour on a straight, level road. If the aircraft is flying at an altitude of .5mile, how fast is the distance between the aircraft and the car increasing 36 seconds after the aircraft passes directly over the car?
Answered by Penny Nom.
A label to cover a plastic cup 2012-10-23
From Kevin:
I'm trying to make a label to cover the entire outer area or a plastic cup. I know there must be a way to figure out the dimensions needed, but I can't seem to figure it out. The circumference of the bottom of the cup is 21.4cm and the circumference at the top of the cup is 29.8cm. The cup is 14.5cm tall. What should the height of the arc from the plane connecting the two ends of the 21.4cm arc. I attached a diagram where x is the value I'm looking for. I'm guessing there is some simple relationship between the length of a line and the arc needed to turn that line into a perfect circle, but I don't know what it is. Can you figure this out and share it with me? Thanks.

-Kevin

Answered by Penny Nom.
Differentiation rules 2012-10-23
From Morgan:
Use the derivative rules to differentiate each of the following:
1. f(x)=1/x-1
2. f(x)= sqrt(x)

Answered by Penny Nom.
Some 6 digit numbers 2012-10-23
From Mason:
How many different 6 digit numbers can you make using the digits 1 ,2 5, 6, 7, and 9? How many of these six-digit numbers are divisible by 6?
Answered by Penny Nom.
A 2 acre plot 2012-10-22
From bob:
If a plot of land on the south side is 290.16 feet wide, how many feet on the west and east side to the north to make a 2 acre plot?
Thank You Bob

Answered by Penny Nom.
Solving for a/b 2012-10-21
From Jenny:
If a + 3b is equal to 175% of 6b, what is the value of a/b?
Answered by Penny Nom.
Converting a fraction to a decimal 2012-10-20
From A Teacher:
My students want an easy way to convert a fraction to a decimal, can I get one from you?
Answered by Penny Nom.
Two congruent circles in a rectangle 2012-10-20
From Alexander:
Have you ever solved a problem, in which you have a rectangle, from which you need to cut the largest two circles of equivalent diameter? I bisected a rectangle diagonally, but the circles, while tangent to two of the sides, are not tangent to eachother. Can you devise a method for two equivalent circles, that are tangent to two sides, are also to eachother?

Take for example a piece of paper, Each if the two largest circles has a diameter that is greater than the distance to the midpoint of the diagonal bisector of the rectangle.

Answered by Chris Fisher.
A proof by contraposition 2012-10-19
From Rahul:
I am not able to understand the following,
To prove that if for all e>0, |x|0, then |x|>=e. I understand the approach very well but I do not understand why if |x|=e then |x|>=e. If it is so then why not |x|= Thanks in advance!'
Rahul.

Answered by Penny Nom.
Whites and Sulphurs butterflies 2012-10-19
From Arlene:
The Whites and Sulphurs are one major family of butterflies. There are about 720 varieties. Eleven times more Whites and Sulphurs live outside North America, mostly tropical Asia and Africa, than live inside North America. How many of this family of butterflies live in North America? How many live in the rest of the world?
Answered by Penny Nom.
The nth term of a sequence 2012-10-19
From Emily:
I don't understand how to do this question:
Nth term for 9, 7, 5, 3, 1
Is there a fast way to do nth term for this question? Or in fact any question?

Answered by Penny Nom.
Revolutions per second 2012-10-18
From Crystal:
Suppose a car wheel is 26 inches in diameter.
a) If the tire completes one rotation, how far would the car travel?
b) If the car is traveling at 60 miles per hour, how fast is the car wheel spinning in revolutions per second?

Answered by Penny Nom.
Passing out graph paper 2012-10-18
From Andrea:
mr karnowski has a stack of 125 sheets of graph paper.He is passing out g sheets to each of his students. What is the expression for the number of sheets he will pass out to the first 5 students?
Answered by Penny Nom.
Percentiles 2012-10-17
From Kenneth:
Question from Kenneth:

Hello:

What is a common calculation used to determine percentiles?

For example, five employees have the following salaries:

Office worker 1 - $25,000

Office worker 2 - $27,000

Office worker 3 - $30,000

Office worker 4 - $32,000

Office worker 5 - $35,000

What is the percentile rank of office worker 3 who earns $30,000?

Here's what I know: Add the number of salaries. Total: 5

Add the smallest number of salaries less than $30,000. There are two.

Now, divide 2 by 5 and multiply by 100. 2/5 * 100 = 40

I think the office worker making $30,000 is in the 40th percentile. and I'm not sure what this ranking indicates.

I thank you for any helpful reply and or different calculation.

Answered by Harley Weston.
The degree measure of the central arc of a circle 2012-10-17
From Crystal:
On a circle with radius of 12 cm is an arc of length 20 cm. What is the degree measure of the central angle used to make this arc?
Answered by Penny Nom.
The height of a triangle 2012-10-17
From Brian:
the base of my triangle is 12 metres and the two sides are 8.45 metres can you help me find the height of the triangle
Answered by Penny Nom.
Walking home from school 2012-10-15
From margie:
My sister can walk from school to home in 40 minutes. I can walk from school to home in 30 minutes. Today I stayed for some extra help, and my sister was already 2/5 of the way home when I started. If I walk at my usual speed, can I catch my sister before she gets home? If yes exactly what fraction of the trip have we covered when I catch her? If no exactly what fraction of the trip have I covered when my sister gets home?
Answered by Penny Nom.
A question about a parallelogram 2012-10-14
From Renu:
Question from renu, a parent:

ABCD is a parallelogram. BP and DQ are two parallel lines cutting AC at P and Q respectively. prove that BPDQ is a parallelogram

Answered by Harley Weston.
A tank with an inner walled compartment 2012-10-12
From don:
I have a tank 20 feet diameter, 19' 8" tall with an inner walled compartment that has a 7' 6" radius arc with in the tank. I need to figure out the volume of the inner area and the volume of the larger area.
Answered by Harley Weston.
A word problem involving a fraction 2012-10-12
From Derrick:
If the numerator and denominator of a fraction are both decreased by 1 the fraction becomes 2/3. If the numerator and denominator are both increased by 1 the fraction will be 3/4. Find the original fraction. How to do?
Answered by Penny Nom.
GST in New Zealand 2012-10-10
From Colleen:
I work out my claimable gst each month but need to know the total sales or purchases amounts for filling in my six monthly paper work. How do I do that. I tried doing the opposite of calculating the gst i.e. x3 and /23 but it is slightly different to my total.
Answered by Penny Nom.
A four digit number 2012-10-09
From cyndi:
find this 4 digit number: the tens digit is 1/3 of the hundreds digit; the 2 digit number formed by the tens and ones digits i the largest prime number smaller than 40; the sum of all 4 digits is the same as the number of face on an icosahedron ? what is the number?
Answered by Penny Nom.
Three piles of top soil 2012-10-07
From Steve:
I need your help please, I am looking to purchase some top soil and keep getting conflicting answers.
There are 3 piles and here are the sizes;
Pile #1: 203 feet around and 21.29 feet high.
Pile #2: 195 feet around and 18.75 feet high.
Pile #3: 150 feet around and 17.98 feet high.
I look forward to hearing back from you asap.
Thank You!
Steve

Answered by Harley Weston.
A problem that yields a linear equation 2012-10-07
From Eshraj:
one fourth of a whole number is three more than one fifth of the next whole number.find the whole number
Answered by Penny Nom.
How many oranges? 2012-10-06
From Rodger:

Question from Rodger, a teacher:

How Many Oranges?
Marjorie, Colleen, and Bob picked oranges together. They were tired after picking a large number of oranges and so fell asleep. Marjorie woke first and decided to take her third of the oranges and head home. She counted the oranges, and found that the number was divisible by 3 if she took one of the oranges first. Marjorie took one orange and then took one-third of the remaining oranges. She left quietly.
Bob awoke a short time later, noticed that Colleen was sleeping, and presumed Marjorie had gone for a walk. He decided to head home because he wasn't feeling well. Not realizing that anyone had left, he counted the oranges and found that the number of oranges was two more than a number that could be easily divided by 3. So he took the two extra oranges and then one-third of what was left. He left without waking Colleen and without waiting for Marjorie to return.
Colleen finally awoke and wondered where her friends had gone. She assumed they were both busy somewhere nearby. Before she left, she counted the oranges and was surprised at how small the number was. She found that the number of oranges was one more than a multiple of three, so she took the extra orange and one third of those remaining.

1. What was the total number of oranges the friends picked?
2. How many oranges were left behind when Colleen departed?
3. Is more than one answer possible for the total number of oranges? If so, what other numbers are possible? Explain.
4. Based on the values you found in problem 3, determine a reasonable solution to the problem. Explain your answer.
5. Is it possible that five oranges remained after Colleen took her share? Explain.
6. Could there have been six oranges left after Colleen departed? What about 46 oranges initially? Explain
7. Find a pattern that gives all the possible numbers of oranges that the friends picked. Explain your pattern

5. Is it possible that five oranges remained after Colleen took her “share”? Explain. 6. Could there have been six oranges left after Colleen departed? What about 46 oranges initially? Explain 7. Find a pattern that gives all the possible numbers of oranges that the friends picked. Explain your pattern

Answered by Penny Nom.
Two circle problems 2012-10-05
From shahad:
question 1
find an equation for the circle through the point (0,0) and (6,0) that a tangent to the line y=-1
question 2
find an equation for the circle through the point (0,0) and (17,7) whose center lies on the line 12x-5y=0

Answered by Penny Nom.
Painting 400 drums 2012-10-05
From Jordan:
This cylinder is given:

Cylinder with a height of 84 cm and a diameter of 54 cm.

Question: The exterior of the drum (cylinder), except the base, is to be painted blue. calculate how many litres of paint will be needed to paint 400 drums if one litre of paint covers 8.8meters Squared. Answer to nearest tenth of a litre.

Answered by Harley Weston.
\mbox 2012-10-05
From Jordan:
This cylinder is given:

Cylinder with a height of 84 cm and a diameter of 54 cm.

Question: The exterior of the drum (cylinder), except the base, is to be painted blue. calculate how many litres of paint will be needed to paint 400 drums if one litre of paint covers 8.8meters Squared. Answer to nearest tenth of a litre.

Answered by Harley Weston.
Four tangent circles 2012-10-04
From renu:
inside of a circle K of radius length measure R,three circular discs A,Band C each of radius r are placed so that each touches the other two and K . express R in terms of r. in the space between K, A and B , another circular disc D is placed which just touches K, A and B. if the radius is s, show that (6+root3)s=(2+root3)r
Answered by Penny Nom.
Find an algebraic expression 2012-10-03
From Karen:

Question from Karen, a parent:

Find an algebraic expression relating the input and output.

input output
   
3 1
9 3
15 5
21 7
27 9

Answered by Penny Nom.
Solve 8t-r=12t for t 2012-09-29
From Monejah:
I need help solving 8t-r=12t for t
Answered by Penny Nom.
A problem involving simple interest 2012-09-27
From Tash:
The amount of money in a single year of an investment after P dollars were initially invested is A = P + Prt, where r is the rate of simple interest. What expression describes P? How much money was initially invested if the account has $1000 one year after the initial investment and the interest rate was 5%?
Answered by Penny Nom.
More on marbles in a jar 2012-09-27
From josh:

Question from josh, a student:

Suppose you have a jar containing 100 red marbles and 100 white marbles. A) If you draw 5 marbles in a row, throwing each marble across the room as you draw it, what is the probability that at least one of them was red? B) If you draw 101 marbles in a row, throwing each one across the room as you draw it, now what is the probability that at least one of them was red?

I saw that this answer was already answered but "The probability that at least one is red is 1 minus the probability that they are all white." makes no sense to me can you please explain i thought that each time a marble is taken out the amount left is different can you please explain better


Answered by Robert Dawson.
The perimeter of a rectangle 2012-09-19
From aretha:
calculate the perimeter of a rectangle with the long side 462ft x 9inches and the short side 50% less.
Answered by Penny Nom.
Changing the sign of an inequality 2012-09-19
From Bryauna:
Why do you change the signs in inequalities?!
Answered by Penny Nom.
Equivalent sets 2012-09-13
From asif:
show that (-1,1)~(1,1) or give its counter example
Answered by Harley Weston.
Equivalent fractions 2012-09-13
From lily:
if i had 1/3 and i had to find the problem an they gave me 20 what will be the denominator
Answered by Penny Nom.
Cones, pyramids, cylinders and prisms 2012-09-13
From Roy:
I read on this page that a pyramid is a special kind of cone, but a cone is not a pyramid. Does this apply to cylinders. Is a prism a special kind of cylinder, but a cylinder is not a prism?
Answered by Robert Dawson.
The dimensions of a rectangle 2012-09-12
From Becky:
A rectangle has an area of 48 square inches and perimeter of 32 inches. Find the dimensions of the rectangle.
Answered by Penny Nom.
An equilateral triangle and a regular hexagon in a circle 2012-09-11
From Heidemarie:
The vertices of an equilateral triangle with side length of 10 sqrt 3 cm lie on a circle. Find the side length of the regular hexagon whose vertices lie on the same circle.
Answered by Penny Nom.
The units digit 2012-09-11
From Shaunice:
Simplify each of the following expressions and determine what the last digit would be. a. (24^15) 4
Answered by Penny Nom.
Golf for 12 2012-09-10
From John:
To have 12 players in groups of 4 playing in 4 games with different people each day
Answered by Victoria West.
The height of a building 2012-09-08
From Lin:
How do surveyors determine a height of a building 150 feet away with an observation angle at 40 degrees? What is the elevation of that top floor?
Answered by Penny Nom.
A profit of 35% 2012-09-07
From Jack:
If a product cost me $10.00 and I want a profit of 35% what is my sell price? How do I figure the sale price to make a 35% profit on the sale?
Answered by Penny Nom.
The volume of fluid in a tank 2012-09-06
From Dave:
I have vertical round flat bottom tanks of various dimensions. It is easy to find total volume but I would like to know how to figure out the coversion factor to go from how many centimeters of liquid is in the tank to the number of total cubic meters of volume. So if my float gauge on the tank says there is 50 cm of fluid in the tank, how many cubic meter of total volume is there?
Answered by Harley Weston.
The hypotenuse 2012-09-06
From Jeevan:
how can i find the height and base of a right angle triangle if i have the hypotenuse only ?
Answered by Penny Nom.
(2y+1)/3=(1y-1)/2 2012-09-06
From jenna:
(2y+1)/3=(1y-1)/2
Answered by Penny Nom.
A container of split peas 2012-09-06
From Laurie:
I have a container that is 471 square inches (length*width) and is 2.3 inches high. I want to cover the bottom with 1/2 inch of split peas, in order to anchor escort cards (place cards) for a wedding. How many pounds of split peas do I need to cover the area to a height of 1/2 inch? Thanks
Answered by Penny Nom.
Row echelon form 2012-09-05
From Jennifer:
Hello, my name is Jennifer. I am a 12th grader in Pre-Calculus and I was wondering if you may help me with Row Echelon Form.

x + 2y - 3z = -5
-2x - 4y - 6z = 10
3x + 7y - 2z = -13

Answered by Penny Nom.
Angular velocity 2012-09-05
From Kyra:
A belt connects a pulley of radius 8cm to a pulley of radius 6cm. Each point on the belt is travelling at 24 cm/sec. Find the angular velocity of each pulley.
Answered by Penny Nom.
A tangent to f(x) = 1/x 2012-09-04
From Steven:
Consider the graph of the function f(x) = 1/x in the first quadrant, and a line tangent to f at a point P where x = k. Find the slop of the line tangent to f at x = k in terms of k and write an equation for the tangent line l in terms of k.
Answered by Penny Nom.
Two cubes 2012-09-04
From alexis:
a cube has one face that is equal to the total surface area of another cube. Find the ratio of their volumes
Answered by Penny Nom.
Exponential form 2012-09-04
From angie:
How do i write 9^-5 in exponential form?
Answered by Penny Nom.
The number of solutions of sinx = x^2 + x +1 2012-09-01
From anamika:
number of solutions of

sinx = x^2 + x +1
graphically?

Answered by Penny Nom.
The area of a rectangle 2012-08-31
From mario:
The area of a triangle when on side is x and the other is x+6
Answered by Harley Weston.
Exponntial form 2012-08-30
From Robin:
My math teacher has asked the question:

"Write mmmxxjmyy2 in exponential form"

How would you solve this problem?

Answered by Penny Nom.
Golf for 20, two teams of 10 2012-08-30
From Mark:
Hi I have to teams of 10 people playing Ryder cup format(against each other) Can I have all 20 people playing with someone different over 3rounds of golf while remaining on there respective team I.e. players 1 to 10 must team up with a player of that group against a pair for players 11 to 20
I hope you can solve this for me
Thanks a bunch Mark

Answered by Victoria West.
12 golfers, 4 rounds 2012-08-29
From Paul:
We have 12 golfers who will play four rounds of golf - how can we organize the foursomes so that each player has the most exposure to the other 11 players?
Answered by Victoria West.
A schedule for 6 teams and 37 games 2012-08-29
From Giovanni:
Hello, i try to prepare a schedules for 6 teams, and every teams will play 37 games before the playoff. How to make a schedules? thanks
Answered by Victoria West.
Minutes and seconds 2012-08-29
From Casey:
I have to write a variable equation. The questions says there are 60 seconds. but we need to write and equation to solve for minutes. Is it 1/60 or 1/s
Answered by Robert Dawson.
Making a wind sock 2012-08-28
From John:
I am trying to build a wind sock and need to be able to lay the shape out on cloth. I need the wind sock front opening (diameter) to be 3 1/2" and the rear opening diameter to be 1". The windsock needs to be 9 1/2" long. I tried using the example of the person trying to make a crayfish trap but got confused and could not figure out my numbers. Any help would be greatly appreciated.

Thanks

John

Answered by Penny Nom.
Expected value 2012-08-25
From melanie:
The probabilities are 0.24 , 0.35 , 0.29 , and 0.12 that a speculator will be able to sell a subdivision lot within a year at a profit of P120, 500 , at a profit of P80,000 , at a profit of P40,000 , or at a lost of P60,000 respectively . What is his expected profit ?
Answered by Penny Nom.
Probability 2012-08-23
From Christine:
In a study of alcoholics, it was found out that 40% had alcoholic fathers and 6% had alcoholic mother. Fourty-two percent had at least one alcoholic parent. What is the probability that a randomly selected alcoholic will ...
Answered by Penny Nom.
A 2-digit number 2012-08-23
From patty:
Write the 2-digit number that matches the clues. My number has a tens digit that is 8 more than the ones digit. Zero is not one of my digits. My number is
Answered by Penny Nom.
The difference of the numbers on two dice 2012-08-19
From dilys:
Two fair dice are thrown. Find the probability that the difference of the two numbers is divisible by 4?
Answered by Chris Fisher and Lorraine Dame.
The volume of a sphere 2012-08-18
From Rohit:
why isn't volume of a sphere = Area of a semicircle x the circumference. i.e. if we revolve a semicircle around its axis we get a sphere
Answered by Penny Nom.
Kirkman schoolgirl problem 2012-08-18
From Anthony:
need to have the number from 1-15 in 6 rounds of 3 numbers without the numbers be repeating in the same row . eg round 1
1 2 3
4 5 6
7 8 9
10 11 12
13 14 15

2nd round
1 4 7
10 13 2
5 8 11
14 3 6
9 12 15
Answered by Chris Fisher.

A rotating schedule 2012-08-16
From Dusty:
Ok, Im trying to create a rotating schedule. There are four jobs and four people but, one of four people cant preform one job. How can I figure out a rotating schedule for this issue?
Answered by Victoria West.
Golf for 16 in 7 rounds 2012-08-16
From Elizabeth:
Hello, I am trying to sort out golfing for 16 guys for 7 rounds. However the twist is that the teams are broken down into 8 old boys vs. 8 young boys. Ideally, each of the players would only play each other once within their own team and then twice within all of the matches. Is that possible? Thanks for your help, my brain is hurting!
Answered by Victoria West.
The length and depth of a chord 2012-08-16
From Tim:
Can I find the radius of a circle, if I have a chord of 2400mm, and a depth between the center of the chord and the perimeter of the circle of 150mm ?
Thanks
Tim

Answered by Penny Nom.
Problem solving 2012-08-15
From Sugavanas:
If length of a rectangle exceeds its width by 5 m. if the width is increased by 1m and the length is decreased by 2 m, the area of the new rectangle is4 sq m less than the area of the original rectangle. Find the dimensions of the original rectangle.
Answered by Penny Nom.
A tapestry rod on a curved wall 2012-08-14
From Marlyn:
I have a curved wall with a radius of 6'. I am trying to have a 36" rod made to hang a tapestry and need to figure out the degree measure of the arc. Can you help me please?
Answered by Penny Nom.
The exponential function form f(x)=a^x 2012-08-13
From Lucy:
Hi,
Why does the "a" value in the exponential function form f(x)=a^x have to be negative?

Answered by Penny Nom.
Two altitudes of a scalene triangle 2012-08-13
From grace:
Two of the altitudes of a scalene triangle ABC have length 4 and 12. If the length of the third altitude is also an integer, what is the biggest that it can be? Justify all of your conclusions.
Answered by Chris Fisher.
The intersection of normal subgroups 2012-08-13
From nokks:
let (G,*) BE A Group and prove that the intresection of normal subgroups of G is itself a normal subgroup of G
Answered by Penny Nom.
The volume of half an ellipsoid 2012-08-10
From Darcy:
Hi, I need to calculate the volume of half an ellipsoid, split horizontally along the long axis. In effect, an oval shaped mound of soil. So the bottom would be an ellipse. I have the length, width and height measurements but not the formula. Thanks!
Answered by Robert Dawson and Walter Whiteley.
A 4 digit number 2012-08-09
From May:
The number is given as ABCD. The first digit is a quarter of the last digit. The second digit is one less than the first digit. When the number ABCD is multiplied by four its digits appear in the reverse order. None of the digits are the same. What is the number. Pleas help because my child need help and I can not do this. Thank you.
Answered by Penny Nom.
The difference between two numbers is 7. 2012-08-09
From May:
The difference between two numbers is 7. When the two numbers are multiplied the answer is 60. How do I work out the two numbers,I tried solving for one unknown and substitution and can not do it. Pleas show me how to do this.
Answered by Penny Nom.
A garden in the back yard 2012-08-04
From Lucy:
Your boss says that his wife has put an 18 × 51 foot garden in along the whole back end of their back yard. He says that this has reduced the back-yard lawn area by 24%. What are the total dimensions of his back yard? What are the dimensions of the remaining lawn area? I found this question on purplemath(http://www.purplemath.com/modules/percntof3.htm). I still don't understand how to solve it. Can you walk me through the process please? Thank you in advance for taking the time to answer my question.
Answered by Penny Nom.
A plane travelling with the wind and against the wind 2012-08-03
From Gwen:
The time of a plane trip of 450 miles, with the wind is 3/5 the time of the return trip against the wind. If the plane travels 120 mph in still air, what is the wind speed?
Answered by Penny Nom.
5 + 5 + 5 - 5 + 5 + 5 - 5 + 5 x 0 = 2012-07-29
From Tom:
5 + 5 + 5 - 5 + 5 + 5 - 5 + 5 x 0 =
Answered by Harley Weston.
Three right triangles 2012-07-26
From Jora:
I am having a lot of difficulties with this question.

Name: Jora
Subject: Math
Who are you: Student

Answered by Chris Fisher.
The distance between overlapping circles 2012-07-26
From Jeff:
I have two circles of different size that overlap one another: Circle #1 has an area(A) of 731,475, so I can calculate its radius as 482.6. Circle #2 has an area(A) of 502,517, so I can calculate its radius as 400. If I know that the area where they overlap is 179,271, how can I calculate the distance between the midpoints of these two circles?
Answered by Chris Fisher.
John's electronic store 2012-07-25
From Jora:
Electronic Store John opened an electronic store in December. During his first month, He sold 10 LCD TVs and 20 Plasma TVs. His income during that month was $12800. In January, he sold 25 LCD TVs and 40 Plasma TVs for an income of $27600. In February he projects to sell 30 LCD TVs and 50 Plasma TVs. If John expenses are $28900 in February, how much money will he have after paying the expenses?
Answered by Penny Nom.
A regular hexagon inscribed in a circle 2012-07-25
From jim:
A regular hexagon with an area of 24√3 is inscribed in a circle. What is the area of the circle?
Answered by Penny Nom.
The area of a rectangle 2012-07-24
From michael:
the dimension of the rectangle are (x+5) and (x-3) find its area
Answered by Penny Nom.
A pile of topsoil 2012-07-23
From Perry:
I have a pile of topsoil that is 42ft long and 16ft tall shaped like a cone how many yards of topsoil do I have.
Answered by Penny Nom.
Fence post holes 2012-07-19
From Gerry:
Hello, I'm digging 30 8" dia holes, 5 ft deep for fence posts that are 4"x 4" Can you please help me figure out how much stone dust I should order for all 30 holes. Thanks
Answered by Penny Nom.
A truncated cone: the central angle 2012-07-17
From Tom:
I have researched several sites, including this one and am fairly confident I can do the calculations required to produce the two radii and the slant height for a truncated cone. I a somewhat confused by the central angle. Some sites indicate that it should never exceed 180 degrees while others do not. Different examples on your site seem to use this in two different ways in constructing a truncated cone. Sometimes the angle seems to indicate the section to be removed, while in other examples the angle seems to indicate the section to be saved. Since the two angles will always total 360 I am confused about how to use the info to calculate the minimum rectangle required to contain the pattern. I'm guessing that in some cases I may need to use the chord or sagitta to determine the desired height and width of the material. Any help is greatly appreciated. Thank you. Tom
Answered by Penny Nom.
2 threesomes for golf over 4 days 2012-07-16
From Jim:
How to arrange for 2 threesomes for golf over 4 days equitably.
Answered by Victoria West.
Probability and curling rings 2012-07-16
From Fatima:
In the olympic event of curling, the scoring area consists of four concentric circles on the ice with radii of 6 inches, 3 feet, 4 feet, and 6 feet If a team member lands a (43 pound) stone randomly within the scoring area, find the probability that it ends up centred on a.red b.white c.blue

I am explaining the figure also in words The smallest Circle is of White color then a blue colour circle then again a White colour circle then the last big circle of red color The answer for a bit is 5/9 I tried it with many ways but I am not getting the answer Please clearly tell me what is n(S)and what is n(E).

Answered by Penny Nom.
A volume of revolution 2012-07-15
From Tewodros:
Let f(x) = e^x and g(x) = x^1/2 both be defined on [0,1]. Consider the region bounded by f(x), g(x), x = 0, x = 1. Rotate this region about the y-axis and determine the volume using the shell method.
Answered by Harley Weston.
The angular elevation of the sun 2012-07-14
From VINEET:
WHAT IS ANGULAR ELEVATION OF THE SUM
Answered by Penny Nom.
A sand box with 14 sides 2012-07-10
From Tom:
Want to build a sand box for a pool that is fourteen foot round wanted to make it in fourteen pieces of wood what angle would each piece be and how long would each piece be
Answered by Harley Weston.
The height of an isosceles triangle 2012-07-10
From ken:
I am trying to determine the various heights of an isosceles triangle, if each has the same base dimension and varies in the degree of the base (equal) angles. What is the method to do this? As an example, of the base is 10, and the two equal angles are each 45 degrees, what is the height? With the same base (10), but with the two equal angles at 60 degrees, what is the height? And with the same base (10) and the two equal angles at 75 degrees, what would be the height?
I know how to calculate the degrees of the third angle (add the degrees of the known angles, and subtract from 180); but am unsure if that is needed for figuring the overall height. And to be clear; I am not looking for the length of the sides of the triangle, but the height from the base to the top point.
Thank you!

Answered by Chris Fisher.
2x+underroot2=3x-4-3underroot2 2012-07-09
From Eshraj:
2x+underroot2=3x-4-3underroot2
Answered by Penny Nom.
Six people divided into three groups of two 2012-07-09
From Fatima:
Six people call them A,B,C,D,E,F are randomly divided into three groups of two,find the probability of the below event(do not impose unwanted ordering among groups) E andF are in the same group I solved it but I have a doubt that it is wrong . My answer is 576 Please help to solve this problem.
Answered by Lorraine Dame and Penny Nom.
Finite probability 2012-07-08
From Fatima:
Tamika selects two different numbers at random from the set{8,9,10}and adds them.Carlos takes two different numbers at random from the set {3,5,6}and multiplies them.What is the probability that Tamika's result is greater than Carlo's result
Answered by Penny Nom.
A cubical tank 2012-07-07
From jake:
a storage tank is in the form of a cube. when it is full of water,the volume of water is 15.625 m cube. if the present depth of water is 1.3m, find the volume of water already used from tank
Answered by Penny Nom.
An octagonal sandbox around a pool 2012-06-12
From Linda:
My pool is 15 feet across and it is round.

How do I measure to cut wood to build a sandbox around it? was thinking it will look like an octagon.

Thanks

Answered by Harley Weston.
Two equations involving fractions 2012-06-12
From Fatima:
Hi ,teacher gave two question to my daughter as follows
Solve 2/x+3=(1/xx-9)-(1/x-3)and
Solve (4/x-2)-(x/x+2)=16/xx-4


Please help me
Thanks & regards fatima

Answered by Penny Nom.
Expanded exponential form? 2012-06-12
From Ramon:
express the numbers 990 614 000 005 in expanded exponential form.
Answered by Harley Weston.
Scheduling an academic competition 2012-06-12
From Mike:
I need to schedule 18 high school teams to play each other in a "Jeopardy!"-like academic competition. Teams compete against each other 6 at a time (simultaneously) in 3 different rooms. What will the minimum amount of matches (I believe 6 ?) we would have to play to ensure that each team plays every other team exactly twice?

And more importantly: what would that schedule look like?

The best I've come up with is having most teams playing twice, but with 2 pairs of teams having to play either only once or sometimes 3 times.

Thanks in advance for any help you can give! :-)

Mike

Answered by Chris Fisher.
The position of an aircraft relative to the airport 2012-06-08
From Dennis:
"Air traffic controllers usually describe the position of an aircraft relative to the airport by altitude, horizontal distance, and bearing.  Suppose an aircraft is at altitude 500m, distance 15km, and bearing 35 degrees east of north.  What are the x,y, and z components (in meters) of the position vector. The x-axis is east, the y-axis is north and the z-axis is vertical.
Answered by Penny Nom.
A multinomial theorem 2012-06-08
From Anuj:
if (1+x+xsquare) whole raise to 20 is a0 + a1x + ........ + a20xraise to 40, then find a0 + a2 + a4 + .......... + a38.
How do you solve this???????

Answered by Robert Dawson.
Long division 2012-06-07
From Emily:
Simplify the expression using long division (9x^2-41x-6)/(x-4)
Answered by Penny Nom.
A reel of cotton thread 2012-06-05
From Grace:
91 meters of cotton goes round the cotton reel. About how many times does the cotton go round the reel?
Give your answer to the nearest ten.
(The Diameter of the cylinder is 3cm)

Answered by Penny Nom.
29 golfers 2012-06-04
From robert:
i have 29 golfers and have to make 8 teams for a golf scramble there will be 5 teams of 4 and 3 teams of 3 what is the fairest way to make up the teams?
Answered by Victoria West.
16 golfers, 4 rounds 2012-06-04
From Gerry:
16 golfers, 4 rounds - best pairings so that everyone plays with each other at least once.
Answered by Harley Weston.
A cyclist a jogger and a fly 2012-06-04
From Emd:
A cyclist & a jogger are 20 miles apart. Cyclist goes 20 mph & jogger 7mph towards each other. A fly starts on nose of cyclist & flies at 20 mph from cyclist to jogger & back & forth until they meet. How far has the fly travelled?
Answered by Penny Nom.
A parabola 2012-06-04
From Madeline:
In a parabola, I need to know what "a" b and c determine. I think that a determines the width of the parabola, but I am not exactly sure what b, and c do.
Answered by Robert Dawson.
Translation of a graph 2012-06-02
From Sean:
the information is thus: $f(x)=3(x+1)^2-2$ I wish to translate up 2 units and move 2 units right.
Answered by Penny Nom.
An octagonal home 2012-05-29
From Jan:
Lot size is 30' by 30' will a 1,165sq ft. Octagon home fit on lot? I don't know the wall size's only the total size of building? Thank You for have this? If this won't fit what size in sq ft will fit?
Answered by Penny Nom.
Golf 16 2012-05-29
From marknp:
16 players only want tp play with one player each one time
Answered by Victoria West.
The volume of a wedge 2012-05-28
From olwethu:
volume of wedge that have side 3,3 cm and 6,2 cm and 20 cm long?
Answered by Penny Nom.
The area of a semicircle 2012-05-28
From Rebecca:
how do you find the area of a semi circle and then how do you add the area of a semi circle and the area of a rectangle?
Answered by Penny Nom.
Multiples 2012-05-28
From Kenneth:
If I understand correctly , a multiple is a product of two numbers. For example some of the multiples of 6 are 6, 12, 18, 24, 30, etc. I just multiplied 6 by 1, 2, 3, 4, 5, etc.

Are the multiples of a fraction, for example, 2/3, determined in the same way? Are they 2/3, 4/3, 6/3, 8/3, 10/3, etc., or are they instead, 2/3, 4/6, 6/9, 8/12, 10/15, etc.?

Or do fractions have no multiples?

Answered by Penny Nom.
Division in base eleven 2012-05-27
From Zoe:
How do you divide numbers that are in base 11?

For example;

9A7A6A divided by A

Answered by Penny Nom.
Finding the circumcentre 2012-05-24
From Glen:
Hi, I'm having trouble finding the circumcentre of these 3 co-ordinates- A(1,1) B(10.5,1) and C(6,6). I understand what circumcentre is but I cannot work out where it is. Thanks very Much, Glen
Answered by Chris Fisher.
A 10 inch circle using 2x4s 2012-05-19
From Ralph:
I want to form a 10" circle with 4"high pieces of 2 x4's. If each 2x4 piece sit next to each other,What degree would I have to cut each side of the 2x4's, and how many would I need to form a 10 inch circle. I know there is a formula for this out there somewhere.
Answered by Harley Weston.
How fast am i driving? 2012-05-18
From bozenga:
How fast am I driving if I cover 40 feet in .1 seconds (1/10th of a second)?
Answered by Penny Nom.
Romeo and Juliet live 40 miles apart 2012-05-18
From Abigaíl:
Romeo and Juliet live 40 miles apart. Romeo jogs 8mph, while Juliet “fast walks” at 4mph. They leave their houses at the same time and journey toward each other.
a) How many hours after they start will they meet?
b) Approximately how many miles from Romeo’s house is their meeting point?

Answered by Penny Nom.
Math, time and rate 2012-05-18
From Abigaíl:
Romeo and Juliet live 40 miles apart. Romeo jogs 8mph, while Juliet “fast walks” at 4mph. They leave their houses at the same time and journey toward each other.
a) How many hours after they start will they meet?
b) Approximately how many miles from Romeo’s house is their meeting point?

Answered by Penny Nom.
Three tennis balls 2012-05-17
From eiman:
Three tennis balls are placed inside a container . the tennis balls are tangent to the top, bottom,and sides of the container . find the volume of air inside the container surrounding the tennis balls if the diameter of the tennis balls is 6 inches
Answered by Chris Fisher.
16 golfers 2012-05-17
From Nohemi:
Can 16 golfers each play in foursomes for 6 days? I found the answer for 5 days, but is it possible to do it for 6.
Answered by Robert Dawson.
The area of a triangle 2012-05-17
From Emilee:
hi. How do i find the area of a triangle if the vertices are (-3,0) (-2,2) and (3,0). I graphed it and it came out a triangle. How do I find the area of it in an easy way?
Answered by Penny Nom.
Slope 2012-05-16
From Kodie:
What is the easiest way to find slope?
Answered by Penny Nom.
A lamp shade 2012-05-15
From Fleur:
I am helping my child make a cone lampshade, the measurements are as follows in "cm's"
21 cm = height
8 cm = top diametre
40 cm = bottom diametre

Please could you give the pattern (cut out) dimensions for final cut.

Answered by Penny Nom.
Multiplying binomials 2012-05-14
From Jordan:
How do I solve the equation:
3(x+4)(x-6)

Answered by Penny Nom.
Sharing the profit 2012-05-14
From Mafiza:
A and B started a business by investing $6000 and $8000 respectively. At the end of the year, a profit of $2100 is made. How much amount will A get as his share of profit?
Answered by Penny Nom.
A normal distribution problem 2012-05-13
From Alysia:
The scores on a test taken by 1000 students are normally distributed with a mean of 66 and standard of deviation of 12. If the college wishes only the top 8% of people to get an A, what would the cutoff score be for the A's?
Answered by Penny Nom.
Solve for x 2012-05-12
From Desiree:
3x -5^2=8
Answered by Penny Nom.
Water in a hemispherical bowl 2012-05-12
From Jimmy:
A bowl of water, being a hemisphere has a radius of 36cm. Water is filled in the bowl and has depth of 18 cm. Find the surface area of the water?
Answered by Penny Nom.
Golf for 6 2012-05-12
From Steve:
We have six golfers playing in two threesomes for six rounds of golf. Can you help solve this so that everyone plays with everyone else the same amount of times or close to it? Thanks!
Answered by Victoria West.
Drawing an isosceles triangle 2012-05-10
From Nazrul:
How can I draw an isosceles triangle whose each angle adjacent to the base is twice the vertex angle?
Answered by Chris Fisher.
Cycling and running 2012-05-07
From liz:
a biathlete travels 20 miles in 2.25 hours. She cycles part of the way at 12 mph and runs the rest at 5 mph. How far did she run?
Answered by Penny Nom.
A 25% sulfuric acid solution 2012-05-03
From Scott:
In order to make a 25% solution with 96% Sulfuric Acid, how much of the Sulfuric do I add to DI water to make up 2000ml??
Answered by Penny Nom.
Golf: 8 players over 4 days in 4-balls 2012-05-03
From JimB:
I cannot seem to get this to work. Previous answers cover 2-balls, but we are 4-balls.

What we really want is for each player to play with each other, at least once and no more than twice.
Ideally, each would also play equally in the first and the second 4-ball of the day.

Answered by Victoria West.
The maximum distance from the vertex of a triangle 2012-05-02
From David:
There are three towns A,B,and C, equi-distant apart. A car is 3 miles from town A, and 4 miles from town B. (ie, somehwere outside of the triangle which the three towns form) What is the maximum distance that the car can be from town A?
This was asked as quiz question in my local pub last Sunday.
The answer is 7. How do I prove it?
Best regards. David in Denton.

Answered by Robert Dawson and Chris Fisher.
The dimensions of a rectangle 2012-04-30
From Nikki:
Name the dimensions of a rectangle that has the same area as this triangle? I found the area of the triangle, which is 30. What do I have to do in order to complete this math problem?
Answered by Penny Nom.
Third side of a irregular triangle. 2012-04-27
From Darya.:
How to find third side of irregular triangle if c = 17m, b = 28m and area S = 210m^2.
Answered by Penny Nom.
The parameterization of a parabola 2012-04-27
From Shawna:
I am having problems finding the parameterization of a parabola. The question I was given is: Find the work done if a particle moves from the points (-2,4) to (1,1) along the parabola y=x^2, while subject to the vector force of F=(x^3y,x-y). So how would I start with finding the parameterization of a parabola?
Answered by Penny Nom.
A circle inscribed in a triangle inscribed in a circle 2012-04-26
From Maty:
How do i find the area of a triangle inscribed a circle while another smaller circle is circumscribed by the same triangle and the radius is 8.
Answered by Chris Fisher.
The exponential form of log8=P 2012-04-25
From Kristina:
what is the exponential form of log8=P
Answered by Penny Nom.
The circumcentre and orthocentre of a triangle 2012-04-25
From Nazrul:
The circumcentre and orthocentre of a triangle is given. A point on a side of the triangle given. A vertex of the triangle on the circumcircle is also given. How can I draw the triangle? Please Help me. Thank you for answering my previous questions.
Answered by Chris Fisher.
How to find the base length of a isosceles triangle if only the sides are given? 2012-04-25
From aqilah:
how to find the base length of a isosceles triangle if only the sides are given?
Answered by Chris Fisher.
A puzzle with a triangle and 57 2012-04-25
From sharon:

Question from sharon, a parent:

We have been battling with this for days and cant find any solution -

Using each of the following numbers once, fill all the spaces in the triangles so that each side of the triangle adds up to 57

numbers are 11,12,13,14,15,16,17,18,19

diagram is -

      0      
    0   0    
  0   57   0  
0   0   0   0

 

Any ideas ? this has to be in in just over 24 hrs ......help!


Answered by Chris Fisher.
A sloped yard 2012-04-24
From William:
Trying to calculate fill dirt for leveling a sloped yard to place a slab. Slab is 30'x40', yard slopes from corner #1 south 40' to Corner #4 drop is 11". From corner #1 east 30' to corner #2, drop is 11 1/4". From corner #1 southeast at a 45 degrees, 50' to corner #3, drop is 19 3/4". Corners numbered in a clockwise direction.
Answered by Penny Nom.
A parabolic bridge 2012-04-24
From Adiba:
A bridge constructed over a bayou has a supporting arch in the shape of a parabola .Find the equation of the parabolic arch if the length of the road over the arch is 100 meters and the maximum height of the arch is 40 meters. I did the problem but not sure is it correct . I did like this f(0)=0we get c=40 if we took quadratic equation in x for a down ward parabola then how to find b and a please show me the answer of either a or b .Or I should use the standard form of parabola y=a(x-h)^2+k then how to find a,h,k please help me
Answered by Penny Nom.
2x^2+5x+2 2012-04-23
From eddie:
2x^2+5x+2
Answered by Penny Nom.
A common chord to two circles 2012-04-22
From Nicole:
What is a common chord between two circles and how is it found in the problem: Two circles intersect and have a common chord, the radii of the circles are 13 and 15, the distance between the circle's centers is 14, find the common chord.
Answered by Penny Nom.
Gray siding and stone chimneys 2012-04-22
From lupita:
Fifty houses are on the north side of a river. The 2nd house has gray siding, and every 5th house after that also has gray siding. The 11th house has a stone chimney; every 3rd house after that also has a stone chimney. How many of the 50 houses have stone chimneys but do not have gray siding?
Answered by Penny Nom.
The sarsen circle 2012-04-21
From Firdous:
I found the circumference but after what to do I am not getting the question is.The diameter of the sarsen circle is 33 meters,since there were originally 30 sarsen stones located on the circumference, how far apart wold the centres of the stones have been?Round to the nearest tenth of a meter.
Answered by Penny Nom.
The area of a quadrilateral 2012-04-21
From Rajat:
calculate the area ABCD in which AB is 48'1'',BC is 98'4'',CD is 61'4'',DA is 102'10'',AC is 110'3'',BD is 116'9''.
Answered by Penny Nom.
Golf for 12 2012-04-20
From glenn:
need to schedule 12 players for a 20 week season
Answered by Victoria West.
Golf for 12 2012-04-20
From Bram:
We are heading on a golf trip with 12 guys, playing 6 rounds. Can you please provide the formula whcih which have the least duplication.
Thanks in advance,
Bram

Answered by Victoria West.
Liters and gallons 2012-04-20
From ann:
the fuel tank of jacks car hold 72 litres when full jack uses half a tank of fuel he travels240 miles how many miles does he cover for each gallon of fuel
Answered by Penny Nom.
16 golfers 2012-04-20
From steve:
i have 16 golfers, 4 in "A" flight , 4 in "B" flight , 4 in "C" and 4 in "D"

We are playing 4 rounds (4 teams of 4) and every group must have a player from the A B C an D flight, and in no round may 2 people have already played together!

Answered by Victoria West.
What was the class average? 2012-04-20
From olwale:
a class which consist of 15 boys and 10 girls sit for a test. the average score of the boys is 20 and the girls' is 25. find the average score of the class.
Answered by Penny Nom.
5 machine make 5 gadgets in 5 minutes 2012-04-20
From Charles:
5 machine make 5 gadgets in 5 minutes,
so how long does it take for 100 machines make 100 gadgets?

Answered by Penny Nom.
Golf for 12 2012-04-18
From Brent:
I have a golf group that is set up as follows: 12 players, broken into 2 teams of 6. I am looking for a formula to have one player from team A play against one player for team B each round, not repeat the match, and ride with as little duplication as possible. I know that it is not statistically possible with these numbers and will have at least one round that has duplication.
Answered by Robert Dawson.
Golf for 10 2012-04-18
From Bob:
Question from Bob:

Hi,
I am trying to set up pairings for 4 rounds of golf with only 10 golfers (2 foursomes and 1 twosome) and trying to minimize duplications. Any formulas or help?

Answered by Victoria West and Harley Weston.
An equation of the form y=mx 2012-04-17
From Samiya:
what would be an equation of the form y=mx with the points (-2, 6) and (1, -3) for the line?
Answered by Penny Nom.
A pyramid on a regular hexagon 2012-04-14
From deepak:
There is pyramid on a base which is a regular hexagon of side 2a cm. If every slant edge of this pyramid is of length 5a/2 cm, than what will be the volume of this pyramid
Answered by Penny Nom.
A tangent line to a circle 2012-04-14
From Novelyn:
find an equation of the line tangent to a circle with equation x^2+y^2+6x-8y-27=0 at the point P(1,-2)
Answered by Penny Nom.
A 14 man golf league 2012-04-12
From Paul:
14 man golf league. Need to have alternating 2 man teams for 18 weeks. Need to rotate foursomes so no one is stuck in twosome more than once. Can you help?
Answered by Robert Dawson and Victoria West.
A golf league with 2 man teams 2012-04-11
From Cosmo:
wanting to set up a golf league with 2 man teams. There are 14 A players and 14 B players.We want each A player to team with each B player once and each A player to play against each other A player once.Is this possible
Answered by Victoria West.
Two cars approach a right-angled intersection 2012-04-10
From Michael:
Two cars approach a right-angled intersection, one traveling south a 40km/h and the other west at 70km/h. When the faster car is 4km from the intersection and the other case if 3km from the intersection, how fast is the distance between the car cars changing?
Answered by Penny Nom.
The height of a cylindrical container 2012-04-10
From Rodney:
a cylindrical container has a radius of 6cm. the container is holding 255 cm3 of water. rounded to the nearest quarter inch, what is the minimum height of the container?
Answered by Penny Nom.
Golf for 28 2012-04-10
From EARL:
HI, I HAVE 28 GOLFERS AND I NEED A SCHEDULE FOR 5 DIFFERENT DAYS THERE WILL BE 7 GROUPS OF 4 PLAYERS EACH DAY. ALL PLAYERS WOULD LIKE TO PLAY WITH ONE ANOTHER ONCE AND NOT PLAY WITH EACH MORE THAN ONCE IF POSSIBLE OR LIMIT THE TIMES A GOLFER PLAYS WITH ANOTHER TO A MINIMUM IF POSSIBLE. (PLEASE SEND AN EXAMPLE OF THE SCHEDULING OF THE GROUPS FOR THE TEE OFF ORDER)
THANKS FOR ANY SOLUTIONS YOU CAN PROVIDE.
EARL

Answered by Robert Dawson.
A semi-circular roof gutter 2012-04-09
From Kyle:
A semi-circular roof guttering contains some water.
The cross-section of the guttering has a diameter of 10 centimetres. The surface of the water collected in the guttering is 8cm. What is the depth of water in the gutter?

Answered by Penny Nom.
A maximization problem 2012-04-09
From Nancy:
After an injection, the concentration of drug in a muscle varies according to a function of time, f(t). Suppose that t is measured in hours and f(t)=e^-0.02t - e^-0.42t. Determine the time when the maximum concentration of drug occurs.
Answered by Penny Nom.
The spread of a rumor 2012-04-09
From Roohi:
The function f(t) = a/(1+3e^(-bt)) has also been used to model the spread of a rumor. Suppose that a= 70 and b=3 0.2. Compute f(2), the percentage of the population that has heard the rumor after 2 hours. Compute f'(2) and describe what it represents. Compute lim t approaches infinity and describe what it represents.
Answered by Penny Nom.
A number puzzle 2012-04-07
From Kenya:
my number is less than 25
my number of tiles will make only one rectangle
my number is odd
my number is a factor of 36

Answered by Penny Nom.
Water is flowing from tank A to tank B 2012-04-06
From Noel:
At First, Tank A was completely filled with water. The tap from Tank A was then turned on and water flowed out at a constant rate. The water which flowed out was collected into Tank B. At the end of 6 minutes, Tank A was 2/3 full. After a further 8 minutes, Tank A had 2.4 liters of water left while Tank B was completely filled with water. Find the capacity of Tank B.
Answered by Penny Nom.
A 12 foot long board 2012-04-05
From Lesley:
If I have a board 12 feet long and am building shelves 2 1/4 feet in length, how many inches are left?
Answered by Jaymi Peterson and Penny Nom.
A rectangular prism 2012-04-05
From Tori:
How do you find the base height if the base width is 10ft. The height is 7ft. And the volume is105ft.
Answered by Penny Nom.
The area of a right triangle 2012-04-03
From Shelli:
How do you find the area of a right triangle if you only know that the hypotenuse is 10 and that the base and height are unknown but are equal?
Answered by Penny Nom.
Radioactive decay 2012-04-02
From Christy:
Hello, how do I go about answering this question?

Should I be using the formula Ce^(kt)?

Given that the decay constant for Radium is -.000428/year, how long, to the nearest year, does it take a sample to decay to 15% of its present mass?

Answered by Penny Nom.
1+2+4+8....= -1 2012-04-02
From Andy:
In this minutephysics video, it's claimed that 1+2+4+8....= -1 Is this true, and if so, how?
< href="http://www.youtube.com/watch?v=kIq5CZlg8Rg">http://www.youtube.com/watch?v=kIq5CZlg8Rg

Answered by Robert Dawson.
A circle drawn around a equilateral triangle 2012-04-01
From BIMAL:
what is the diameter of a circle drawn around a equilateral triangle of size 6 cm
Answered by Penny Nom.
A lot with a slope 2012-03-30
From Carlos:
I have a lot with a slope. On the right I need to bring it up 2', From that point to the left is 112' which at this point I need to bring it up 5' for it to be level . The distance front to back is 80'. What is the cubic yards of this area that I need to fill in.
Answered by Harley Weston.
Golf for 17 2012-03-29
From Richard:
Hi Guys,
I have a scheduling problem which I don't think you have covered before.
Apologies if you have!
I have 17 golfers due to play 4 rounds of golf.
Each round will consist of 3 threeballs and 2 fourballs (ie. 17 golfers in 5 groups)!
Is it possible to come up with a schedule where each golfer plays
with different partners in each round?

Answered by Robert Dawson.
The period T of a pendulum 2012-03-27
From Ashley:
The period T of a pendulum is given in terms of its length, l, by T=2pi sqrt(l/g) where g is the acceleration due to gravity(a constant)
a. find dT/dl
b. what is the sign of dT/dl
c. what does the sign of dT/dl tell you about the period of the pendulums?

Answered by Penny Nom.
A uniform probability density function 2012-03-26
From Noeline:
The label on a bottle of liquid detergent shows contents to be 12 ounces per bottle. The production operation fills the bottle uniformly according to the following probability density function:
f(x) = 8 for 11.975 ≤ x ≤12.10
and
f(x) = 0 elsewhere

a. What is the probability that a bottle will be filled with 12.02 or more ounces?
b. What is the probability that a bottle will be filled between 12 and 12.05 ounces?
c. Quality control accepts production that is within .002 ounces of number of ounces shown on the container label. What is the probability that a bottle of this liquid detergent will fail to meet the quality control standard?

Answered by Penny Nom.
The equation of a line 2012-03-26
From Danielle:
Hello, could you help me figure out what the equation of the line is in the form of Ax+By=C with the points (-2,0) and (0,3)? Thank You!
Answered by Penny Nom.
Painting a pool 2012-03-25
From Salai:
A swimming pool measures 8m long, 6m wide and 1.5 m deep. The special paint for the pool cost $6per square metre. How much will it cost to paint the interior surfaces of the pool ?
Answered by Penny Nom.
A cubic inequality 2012-03-25
From Victoria:
The flight path of a bumblebee above the ground can be modelled by the function f(x)= 2x^3- 17x^2+ 11x + 130. Where x is the time in seconds and f(x) represents the height in inches above the ground. The entrance to the bee's hive is located 100 inches above the ground. Determine when the bumblebee's height is greater than 100 inches. The bee's height was monitored from 0 to 25 seconds inclusive. (over the domain 0
Answered by Penny Nom.
Angle measures 2012-03-21
From jogiboy:
how can i get the sine theta if the given is 3.14/3
Answered by Penny Nom.
How long will it take lee to catch up with jodie? 2012-03-21
From kirsten:
for example, if jodie is on the bus travelling at 10 m.p.h and lee is walking at 4 m.p.h how long will it take lee to catch up with jodie?? what is this mathematical name for this
Answered by Harley Weston.
A steamroller with a 3 foot diameter and a 6 foot width makes 20 revolutions 2012-03-21
From Robert:
A steamroller with a 3 foot diameter and a 6 foot width makes 20 revolutions. How much ground area does it pave?
Answered by Penny Nom.
Gravel, sand, cement and an equilateral triangle 2012-03-21
From saheed:
how many tonnes of gravel and sand,bags of cements will be required to concrete 5feet depth/deep of 4feet equilateral triangle?
Answered by Harley Weston.
Two overlapping circles 2012-03-21
From Monty:
If you have a 3.75" radius circle overlapping a 5" radius circle with their centers 3" apart what would be the area of the non-overlapped portion of the small circle?
Answered by Penny Nom.
The area of an irregular hexagon 2012-03-20
From Jacob:
I am having trouble finding the area of an irregular hexagon with 5 right angles. It has 5 given sides, with the lengths as follows:

8.2 ft.
6 ft.
4.5 ft.
2.2 ft.
6 and one third ft.

Answered by Chris Fisher.
Remainders 2012-03-18
From Leslie:
Question: Janice has some marbles. When she divides them into groups of 4, she has 2 marbles left over. If she divides them into groups of 5, she will also have 2 left over. What is the least number of marbles she can have?
Answered by Penny Nom.
The length is inversely proportional to the width 2012-03-18
From David:
For a given set of rectangles, the length is inversely proportional to the width. In one of these rectangles, the length is12 and the width is 6. For this set of rectangles, calculate the width of a rectangle whose length is 9
Answered by Penny Nom.
A vertical line passing through (10,4) 2012-03-16
From Isabelle:
Hi! My question is this:

Write the equation of each line described:
A vertical line passing through (10,4)

I would really appreciate your help!
Thanks,
Isabelle

Answered by Penny Nom.
Forming a triangle from 3 line segments 2012-03-15
From rustom:
A point X is selected at random from a line segment AB with midpoint 0. Find the probability that the line segments AX, XB, and A0 can form a triangle.
Answered by Penny Nom.
Probability 2012-03-14
From tom:
Let A and B be events with p(A ∪ B)= 7/8, p(A ∩ B)= 1/4, p(A' )= 5/8
Find:
a. p(A)
b. p(B)
c. P(A ∩ B')
...problem for me is that if p(A') = 5/8 then P(A) should be 1/4...so that the p(A' ∪ A) = p(A ∪ B) = 7/8...however, the book says the answer for p(A) is 3/8...does that mean, my interpretation is wrong (because if it is, then all my answers would be crap)...or is it just another typographical error?

Answered by Penny Nom.
The dimensions of a rectangle 2012-03-13
From irvyn:
if you are only given the area and the perimeter how do you find the dimension?
Answered by Penny Nom.
A trapezoidal prism filled with sand 2012-03-12
From Maria:
could a Trapezoid that is 80 inches high with a 54 inch width top and a 89 inch width bottom and a 80 inch depth hold 58 cubic yards of sand?
Answered by Robert Dawson.
Circles 2012-03-11
From Deniz:
Two circles are externally tangent and the lengths of their diameters are 4 and 6. Find the length of the segment joining the centers of the circles.
Answered by Penny Nom.
Buying school supplies 2012-03-09
From Jada:
Hi, my daughter is having difficulties with a problem she is in the 5th grade, and insisted that it was a guess and check problem, but she really doesn't understand, and I don't either. Can you help us out?

Danielle is buying school supplies. She has $7.25 to spend.
Notebook: $0.75
Pencils: $1.95
Pens: $2.50
Planner: $5.00
Please help by explaining the guess and check strategy for a problem like this. Thank you.

Answered by Penny Nom.
A clock takes 3 seconds to strick the hour of 3 o'clock 2012-03-09
From saptarshi:
[ A clock takes 3 seconds to strick the hour of 3 o'clock. How many seconds will it take to strick 9 o'clock?]
Answered by Robert Dawson.
A water tank is in the shape of a truncated cone 2012-03-08
From Victoria:
Suppose you have a water tank in the shape of a truncated cone. The larger diameter is K, the smaller diameter is K/2, and the height is also K (all measured in meters). The force of gravity on an object of mass, m, is F=9.8m (measured in Newtons). The density of the water is 1000 kilograms per cubic meter. What is the volume of the tank and how much mass will the tank have when it is full?
Answered by Penny Nom.
A lost combination 2012-03-06
From pam:
I lost the combination for my luggage that I haven't used for many years. It is a 3 digit combination. I'm pretty sure only these numbers could have been used: 0,1,2,3,7,8,9. Can you give me a calculated list of all these combinations?
Answered by Penny Nom.
The dimensions of a conical tent 2012-03-04
From yash:
a conical tent is to accommodate 11 people.Each person must have 4m square of space on the ground and 20m cube at air to breathe.Find the height and radius of the conical tent.26202
Answered by Penny Nom.
Tina is older than Catherine by 1 year and 7 months 2012-03-03
From jennifer:
Tina is older than Catherine by 1 year and 7 months. Karen is younger than Tina by 2 years and 11 months. If Karen is 7 years and 2 months old, find out Catherine's age.
Answered by Penny Nom.
Percentage of water in a soil sample 2012-03-01
From avi:
what i need is the formula in getting the percentage from gram. situation:
i need to know how many percent will i declare in getting the moisture of a soil sample i weighed 2,000 grams of soil before i dried it out in the oven, after few hours the recovery of the dried out soil is 1,700 grams. How many percent will be the water content? kindly send me a formula in computing this.
thank u very much!

Answered by Penny Nom.
The area and perimeter of a rectangle 2012-03-01
From RAJIB:
PEREMETER OF A RECTANGLE IS 24. FIND THE AREA ?
Answered by Penny Nom.
The height of an equilateral triangle 2012-02-29
From Carley:
Hi my name is Carley. I am an 8th garder. What us the height of an equilateral triangle si the sides are 18 cm?
Answered by Penny Nom.
Mr. Schneider's mass 2012-02-29
From Khiya:
Mr. Schneider's mass is 2 digits. The first digit is 3 times the second digit. Both digits are odd numbers, and his mass is greater than 35 kg. What is Mr. Schneider's mass?
Answered by Penny Nom.
Measuring the liquid in a horizontal tank 2012-02-28
From Philip:
I have a steel gas tank that is 3' dia X 5' length. The total volume is 1000 litres. But how much is left when I use a stick and measure 6" from the bottom or 12" or 24" ?? Is there a formula to use for this task?

Thanks.

Answered by Harley Weston.
The weight of a steel plate 2012-02-27
From david:
what is the weight of a steel plate that is 1/2" thick, 20" wide, and 45"long if the steel weigh .28 lb per cubic inch.
Answered by Penny Nom.
Tiling a floor 2012-02-27
From jamie:
how many 16in by 16in blocks would it take to cover a 16ft by 16ft floor?
Answered by Penny Nom.
Parametric equations 2 2012-02-25
From Kathy:
If the angle at time 0 is not 45 degrees how can you find the initial velocity? Ball thrown from height of 7 feet. Caught by receiver at height of 4 feet after traveling 90 feet down the field. Find initial velocity. You had a similar problem answered but the angle was 45 degrees so the cos and sin were equal and the equations were simpler to work with. Thank you!
Answered by Harley Weston.
Expected value 2012-02-24
From Lara:
A life insurance company sells a $250,000 1-year term life insurance policy to a 20-year-old male for $350. According to the National Vital Statistics Report the probability that the male survives the year is 0.998734. Compute and interpret the expected value of this policy to the insurance company.
I'm very confused on how to do this. I assume x is the profit the insurance company makes in the year of $350 but beyond that I don't know what to do. Thanks.

Answered by Penny Nom.
Integral 1/(25-x^2)^3/2 2012-02-22
From John:
Integral 1/(25-x^2)^3/2
Answered by Harley Weston.
The volume of an irregular tetrahedron 2012-02-22
From Brittany:
If I am given an irregular tetrahedron with the coordinates of the 4 points how do I find the volume? For example I am asked to find the volume and only given the points A(-4,-3,5), B(2,-1,2), C(0,-5,0), and D(-2,0,0) can you shown me the working and formular to find the volume?
Answered by Robert Dawson.
Dt[sin t tan (t^2+1)] 2012-02-21
From Ayu:
Ayu
Dt[sin t tan (t^2+1)]
derivatives

Answered by Harley Weston.
A 23 acre airport runway 2012-02-21
From Ron:
I have a 23 acre plot. If it were in a straight line, would it be enough distance for a plane to take off and land if the plane needed 1 mile?
Answered by Robert Dawson and Penny Nom.
The 10000th day of her life 2012-02-21
From derrick:
Ella was born early in the morning on a Friday.She got married on the 10000th day of her life. What day of the week was it?
Answered by Penny Nom.
Tim sells 6 times as many oranges as apples 2012-02-20
From Bronwen:
Tim sells 6 times as many oranges as apples. If he sell 490 in total one morning, how many of each type were sold?
Answered by Penny Nom.
A circle inscribed in a quarter circle 2012-02-20
From sonam:
ABC is a quarter circle and a circle is inscribed in it and if AB=1cm than find radius of a small circle.
Answered by Penny Nom.
Demarkus is paving a new driveway 2012-02-19
From Anita:
Demarkus is paving a new driveway for a local YMCA volunteer club. The driveway is 90 yards long and 6 yards wide. If the concrete cost $2.50 per square foot, how much will it cost Demarkus to cover the driveway?
Answered by Penny Nom.
If x ix directly proportional to y then ... 2012-02-19
From Crystal:
I understand that we say x and y are proportional, then y = kx.
Generally when we say x is proportional to y, we think that when x increases y increases.
Now I am questioning whether that is actually true.
If k is negative, then when x increases, y DECREASES.
My question is does this mean this statement is incorrect:
If x is proportional to y, when x increases, y increases proportionally.
Is it more correct to say:
If x is proportional to y, when x increases, y either increases proportionally or decreases proportionally.
Along the same line of reasoning, when x is inversely proportional to y, we think that when x increases y decreases, but that is true only if k is positive.
Is that correct?

Answered by Harley Weston.
Fractions and proportions 2012-02-16
From Kenneth:
Hello:

I read the following in an old textbook:

Display the numbers 27, 18, 26, and 39 so as to form a proportion.

Which set of equal fractions is correct?

18/27 = 26/39

18/26 = 27/39

I thank you for your reply.

Answered by Penny Nom.
Margie threw a ball 2012-02-16
From mary:
at 9:45 Margie threw a ball upwards while standing on a platform 35ft above the ground. The height after t seconds follows the equation: h(t)= -0.6t^2 +72t+35
a) what will be the maximum height of the ball?
b)how long will it take the ball reach its maximum height??

Answered by Harley Weston.
Figures with dots 2012-02-15
From A:
Okay so my daughter came home and had this question for homework from her teacher. I have no idea what it means HELP!!
The question shows a figure that is a square. It has 5 dots on the top, bottom and on the sides looks similar to this:
. . . . .
.       .
.       .
.       .
. . . . .
they want to know 3 more ways to show the number of dots in the figure. They have already shown one with just counting them and the other one that they showed was circling the four corner dots and then taking the dots in between with a number sentence set up like this: 4+4*(5-2) = 4+4*3= 16. They also besides giving 3 ways want us to write a number sentence and an algebraic sentence to show the number of dots in the figure with "n" sides. Please help! So confusing ! I don't know if my square turned out or not but it should be 5 across the top and bottom and on the sides and no dots in teh middle it is just the outside.

Answered by Penny Nom.
A word problem 2012-02-15
From Bo:
Jane guessed a number. Tom guessed a number that was double the sum of 1 more than twice the number guessed by Jane. Sum of the numbers guessed by both were 27, what are the numbers?
Answered by Penny Nom.
36 golfers 2012-02-15
From Steve:
We will have 36 golfers playing 4 rounds of golf and would like to have different foursomes each round. In other words, no player will play with another more than once.
Answered by Harley Weston.
A cylindrical container 2012-02-14
From KRIS:
YOU ARE PaCKAGING 4 EQUAL SIZED PAINT CANS WITH A RADIUS 8CM AND A HEIGHT OF 20CM, IN A CIRCULAR CONTAINER.What is the surface area of the packing container?
Answered by Penny Nom.
Jack and Kate set off in their cars ... 2012-02-13
From akiri:
Jack and Kate set off in their cars from the same point to travel the same journey. Jack has a start of eight minutes before Katie sets off. If Jack travels at 50mph and Katie travels at 75mph, how many miles will be travelled when the two cars are level?
Answered by Penny Nom.
A storage box with a slanted roof 2012-02-13
From Sophia:
Hi!
I have another problem.

The diagram shows a side view of a box which is used to store small logs of wood for burning in a fire place. The slopping lid has an overhand of 15cm.
a) Calculate the total length of the slopping lid to the nearest cm.
b) When the lid is open above ground will the end of the lid be?

P.S. Please see attached.

Answered by Penny Nom.
Simplify the expression 2012-02-12
From Christy:
Hi guys, ok so I'm having problems with solving this equation since it's been so long I've done precal.
I have to find critical points so I have to find the derivative first. This is what I've done so far s(t)= (t-6)^4 * (t+1)^2
t'= 4(t-6)^3*(1)*(t+1)^2+ 2(t+1)(1)*(t-6)^4
so this is where I got stuck, which is the algebra part. How can I simplify this to get the answer to figure out the critical points.

Answered by Penny Nom.
A 3 to 1 ratio of girls to boys 2012-02-12
From amanda:
At a recent school dance it was observed that there was a 3 to 1 ratio of girls to boys. If 204 students attended the dance, how many boys, and how many girls were there?
Answered by Penny Nom.
A circle and a chord 2012-02-11
From Sophia:
The diagram shows a circle with a chord that is 10cm long. The middle of the chord is 12cm from the centre of the circle. Calculate the radius. Chord length is 10cm. The distance from the centre to chord is 12 cm.
Answered by Penny Nom.
Can the result of the calculation be 2? 2012-02-09
From Thomas:
You have the digits 2, 2, 3, 3, 4, 4, 5, 5, 6, 6, 7, 7, 8, 8, 9, 9 in a random order side by side and put randomly between two digits a colon, so that a division comes into existence. Can the result of the calculation be 2?
Answered by Robert Dawson and Chris Fisher.
Litres in a box 2012-02-09
From vicky:
how much litre water can be filled in a box of 12 inches *24 inches *12inches
Answered by Penny Nom.
Two circles 2012-02-08
From crisfe:
find the point where the common cord of the circles x2+y2=25, x2+y2-12x-6y+35=0 process there line centers. what point they intercepts?
Answered by Penny Nom.
Wallpapering a room 2012-02-08
From jimmy:
Ms. Frank is going to wallpaper a living room with dimensions 24 feet long, 18 feet wide, and 8 feet high. What surface area does Ms. Frank plan to wallpaper?
Answered by Penny Nom.
The curvature of the earth 2012-02-08
From sean:
Question from sean, a student:

Two people 1.8 metres tall walk directly away from each other until they can no longer see each other (due to the curvature of the earth, which has a radius of about 6378 km).
A) Find a function relating the height of two identical objects with the distance between them using the scenario above as an example.
B) Sketch this function (you may use Graphmatica if you wish). Over what domain and range does the function exist?
C) Describe this relation in practical terms.

Answered by Harley Weston.
A road around a garden 2012-02-07
From Manu:
Inside a square garden of side 158m, a road 4m wide is built all around. What is the area of the remaining part of the garden?
Answered by Penny Nom.
The volume of a round tube 2012-02-07
From I.C:
I need to find out how long a 10" round tube needs to be in order for there to be .95 cubic feet of air space
Answered by Harley Weston.
Expressing 0.5% 2012-02-06
From Kenneth:
Hello:

It is correct to express 0.5% as 5 tenths of one percent.

Is it also correct to express 0.5% as 5 tenths of one-hundred percentage points or 5 tenths of one percentage point?

I think it would be 5 tenths of one percentage point because 5 tenths of one-hundred percentage points would be 50 percentage points, but I'm not sure.

I thank you for your reply.

Answered by Penny Nom.
Notation for the second derivative 2012-02-06
From Shafira:
In all math textbooks, it is written that d/dx ( d/dx) (y)= d2y/dx2. Why do they write it as d2y/dx2, not as d2y/d2x2?
Answered by Robert Dawson.
A quartic polynomial that is a perfect square 2012-02-05
From archit:
If P(x)=x^4+ax^3+bx^2-8x+1 is a perfect square then (a+b)=?
Answered by Penny Nom.
A rectangular block and a sphere 2012-02-02
From snehal:
a solid rectangular block of metal 49 cm. by 44cm. 18cm. is melted and formed into a solid sphere, find the radius of the sphere[22/7]
Answered by Penny Nom.
There is a police officer in pursuit of suspect 2012-02-01
From Jay:
There is a police officer in pursuit of suspect. They suspect is driving on a straight road traveling at 79 mph, the officer is 233 yds away, traveling at 92 mph, the suspects car 5 ft long and the officers car is 5.3 ft long, how long will it take to have the front of the officers car, equal to the front of the suspects car down to the tenth of a second.
Answered by Harley Weston.
If a parallelogram is a cyclic quadrilateral then it is a rectangle 2012-02-01
From Kim:
Show that if a parallelogram is a cyclic quadrilateral then it is a rectangle. Hint: Observe that in a parallelogram ABCD we always have Triangle ABC is congruent to Triangle CDA.
Answered by Robert Dawson and Chris Fisher.
Properties of real numbers applied to subsets 2012-02-01
From Mark:
Hello - The questions that I have for you is do the properties of real numbers (such as the associative, commutative, identity, inverse, and distributive law) apply to ALL the subsets of real numbers? In other words, do all those properties work for the Natural Numbers? The Whole Numbers? And so on and so forth. I understand that they are all real numbers, but for instance: the identity is whenever you add zero to a number, you get that number back. But does that work with, say, with only the odd numbers? Zero isn't odd so can that property actually apply to JUST the odd numbers? Any consideration would be greatly appreciated!
Answered by Robert Dawson.
The dimensions of a rectangle 2012-01-31
From Dazz:
Hi I was wondering how to find the length of a rectangle if you know the area is 484 square inches And the length is 10 square inches or 22 square inches?
Thanks~Dazz

Answered by Penny Nom.
A two digit number 2012-01-29
From Christine:
Five times the sum of the digits of a two digit number equals the number. If the digits are reversed, it becomes nine more than the original number. What is the original number? I really don't understand this question, help me please
Answered by Penny Nom.
Building a tipi 2012-01-29
From Lacy:
Hi there! We are building a tipi for our children. We want to build a large one about 15ft tall with a base of about 15 feet diameter. I am trying to figure out how much canvas we need to accomplish this. I graduated about 20 years ago and am struggling. Please help if you can.
Answered by Penny Nom.
The volume of a dugout 2012-01-28
From Jan:
is there a formula for calculating the cubic yardage removed from a dugout for example that is forty feet wide by one hundred and twenty feet long by twelve feet deep with a three to one slope? It is the slope that makes it a little tricky. Thank you.
Answered by Penny Nom.
Tons of fill dirt 2012-01-27
From Brian:
about how many tons of fill dirt would I need to level a area 30 foot by 30 foot and 6 foot deep
Answered by Penny Nom.
The area of a triangle 2012-01-27
From mary:
I have a scalene triangle and the base is 33m, side is 52m and the unparallel side is 25m but i drew a dotted line down and that s 20m

how do i figure the area?

thanks

Answered by Penny Nom.
1 + 3 + 3^2 ...+3^(n-1) = 3^n - 1/2 2012-01-27
From Vicki:
I am trying to find out how to do show how this proof was worked.
Here is the end result 1 + 3 + 3^2 ...+3^(n-1) = 3^n - 1/2

This equation was used to find the number of white triangles in the Sierpinski Triangle

Answered by Walter Whiteley.
Rolling three dice 2012-01-26
From Aishwarya:
A red, a blue and a green die are all thrown at the same time. Display all the possible outcomes in a suitable way. Find the probability of obtaining:-
1. A total of 18 on the three dice
2. a total of 4 on the three dice
3. a total of 10 on the three dice
4. a total of 15 on the three dice
5. a total of 7 on the three dice
6. the same number on each die..

Answered by Robert Dawson.
The difference in the latitudes of Lynchburg and Myrtle Beach 2012-01-26
From Sarah:
Assuming that the earth is a sphere of radius 6378 kilometers, what it is the difference in the latitudes of Lynchburg, Virginia and Myrtle Beach, South Carolina, where Lynchburg is 400 kilometers due north of Myrtle Beach?
Answered by Robert Dawson.
A circular cylinder circumscribed about a right prism 2012-01-25
From Noriz:
A cylinder is circumscribed about a right prism of altitude 12.6cm. Find the volume of the cylinder if the base of the prism is an isosceles triangle of sides 3cm by 3cm by 2cm.
Hope you can help me with this.

Answered by Penny Nom.
Great circle course 2012-01-25
From Hervé:
On the earth, the mathematical formula giving the distance between two points, and the initial course for a boat on the great circle is well known. I need to find the inverse formula, ie knowing an initial position on earth, and the initial course of the boat, and the distance to run on the great circle, the formula gives the final position (longitude and latitude).
Answered by Robert Dawson.
Today is Emma's Birthday. 2012-01-25
From Mike:
Today is Emma's Birthday.
Emma's mother is four times as old as Emma is today.
In four years, Emma's mother will be three times as old as Emma.
How old is Emma today?

Answered by Penny Nom.
A fountain 2012-01-24
From kris:
A fountain has a radius of 14 meters to its outer edge. Their is an inner ring in the center of the fountain, where a statue of Sir Isaac Newton stands, that does not contain water. The inner ring has a diameter that is 6m less than the diameter of the outer ring of the fountain. What is the circumference of the inner ring? What is the area that is covered by water in the fountain?
Answered by Penny Nom.
Four pizzas 2012-01-23
From kris:
a pizza company wishes to put 4 medium pizzas in a box to sell as a party pack. The box they want to use the square and has dimensions of 60cm by 60cm. They need you to help them calculate the dimensions of the pizza that will fit in the box. Calculate the following: area, radius, diameter, circumference
Answered by Penny Nom.
A problem in logic 2012-01-19
From Ghader:
The puzzle below I know the answer to; because someone told me! My question is: how could I answer it using logic, maths, etc. what field of inquiry does this kind of problem fall into? Puzzle: You are in a room with two men: one is a compulsive liar and the other is a compulsive honest. There are two doors: one leading to heaven, the other to hell. The two men know which door leads to where. You want to go to heaven but are allowed only ONE question from one or the other of the two men. What would that question be? [The question to ask, from either man, is: "if I asked the OTHER man which door leads to heaven, which door would he point?". You would then choose the other door.] Has this answer got any basis in logic or maths at all?
Answered by Robert Dawson.
-8(3x+4)+6x=4(x-8 )+4x 2012-01-18
From bianca:
-8(3x+4)+6x=4(x-8 )+4x
Answered by Penny Nom.
A limit 2012-01-18
From Neil:
The limit of [(1/x)^3 - 1/8]^1/3 all over (x - 1/2) as x approaches 1/2 to positive infinity. How to answer that?
Answered by Harley Weston.
The difference in latitudes of two cities 2012-01-18
From Renee:
Assuming that the earth is a sphere of radius 6378 kilometers, what is the difference in latitudes of two cities, one of which is 600 kilometers due north of the other?
Answered by Robert Dawson.
A circle inscribed in a regular octagon 2012-01-16
From Eric:
I have a circle inscribed in a regular octagon. How do I determine the length of one side of the octagon if I know the radius of the circle (2.75 inches) ?
Answered by p.
Exponential form 2012-01-15
From Helen:
How do you write (1/x) in exponential form?
Answered by Penny Nom.
The derivative of x^-(1/2) 2012-01-14
From Eric:
I have an problem figuring out the derivative of the negative square root of x i.e. x^-(1/2) using the first principle.
Could someone please show me?
Thanks in advance!

Answered by Harley Weston.
How many gallons of water to fill my pool? 2012-01-13
From Don:
How many gallons of water do you need to fill a 22 feet long and 13 feet wide rectangular pool if it is 3 feet deep on one end and 7 feet deep on the other end?
Answered by Penny Nom.
Four apples and two oranges cost Rs. 30... 2012-01-13
From nasr:
Four apples and two oranges cost Rs. 30, and one apple and 3 oranges costs Rs.15.How much does each apple and each oranges cost?
Answered by Harley Weston.
Lost in the woods 2012-01-12
From Liz:
I am lost in the woods. I believe that I am in the woods 3 miles from a straight road. My car is located 6 miles down the road. I can walk 2miles/hour in the woods and 4 miles/hour along the road. To minimize the time needed to walk to my car, what point on the road should i walk to?
Answered by Harley Weston.
The radius of a circle 2012-01-12
From Janie:
Find the radius of a circle knowing that a chord of 24.6 inches has a corresponding arc of 70°.
Answered by Penny Nom.
Running 5 km 2012-01-12
From Fayeann:
What amount of time will it take a person running 7 m/s to travel a distance of 5 km?
Answered by Penny Nom.
What is the height of the triangle? 2012-01-12
From Hailey:
The area of a triangle is 77ft2. if the base is 11ft. how tall is the height?
Answered by Penny Nom.
A volume of revolution 2012-01-11
From john:
find volume of solid generated by revolving the region in the first quadrant bounded by the curve y squared=x cubed, the line x=4 and the x-axis about the line y=8. The answer in the back of the book is 704 pi divided by5
Answered by Penny Nom.
An equilateral triangle and some circles 2012-01-10
From tushar:
draw an equilateral triangle with side 6cm.draw 3circles with radii 3cm on each angular point of triangle.draw common tangent on each of two circles
Answered by Penny Nom.
A picture in a frame 2012-01-10
From nnadozie:
a picture, which measures 46cm by 38cm, is surrounded by a frame, which is 2cm wide, find the area of the frame.
Answered by Penny Nom.
Designing aluminum cans 2012-01-08
From swathi:
A company manufactures aluminum cans. one customer places an order for cans that must be 15 cm high, with a volume of 1200cm cube.

a) use the formula V= pie r squared h to determine the radius that the company should use to manufacture these cans

b) graph the function that corresponds to 0 = pie r squared h minus V to determine the radius.

Answered by Penny Nom.
A wire spiral 2012-01-07
From Pinar:
I am trying to help my daughter with very challenging maths questions which sometimes I get stuck If someone help me with one of them I would appreciate it.

Jane is making a spiral out of wire. She bends the wire after 1 cm, then bends the wire after 2cm, then 3cm and continues in this manner. After 4 bends she used 15 cm of wire For each bend how many cm wire was used? After 6 bends how many cm wire will she have used? How many bends will she have made if she uses 66 cm of wire?

I would appreciate if somebody would help me with this.

Thank you!
Pinar

Answered by Penny Nom.
The dimensions of a rectangle 2012-01-06
From joseph:
The perimeter of a rectangle is 72 inches. The length of the rectangle is three more than 2 times the width of the rectangle. What is the length of the rectangle? Write and solve an equation to determine the dimension of the rectangle.
Answered by Penny Nom.
A straight line distance 2012-01-06
From Margaret:
If you traveled 300 miles east and 275 miles north, how many miles would you save by going in a straight line?
Answered by Penny Nom.
Two shapes 2012-01-06
From muhammad:
Question from muhammad, a student:

a boy has two pieces of wire each 99cm long .he bends them into the shapes shown below.calculate the radius of each figure,giving each answer to two decimal places.figures are one is semicircle and other on is half of the semicircle.

Answered by Harley Weston.
A parabolic arch 2012-01-04
From Swathi:
A plan for an arch in the shape of a parabola is drawn on a grid with a scale of 1m per square. The base of the arch is located at the points (0,0) and (15,0). The maximum height of the arch is 18m.

a)Determine the quadratic function that models that arch

b)State the domain and range of the function

Answered by Penny Nom.
Lotto tickets 2012-01-03
From Ashley:
Hi. I play the lotto a lot, and have yet to win. How many combinations can I make with 6 numbers 1-53? I am trying to come up with a dollar amount so I know how much I would have to spend to guarantee a win. Thank you in advance. (I'm no good at math)
Answered by Robert Dawson and Penny Nom.
Triangles, area and perimeter 2012-01-01
From Panny:
I saw a question asked on your board without an answer.
How do I find the area of a triangle any triangle if I only know the perimeter..

Answered by Penny Nom.
Sum and difference of squares 2011-12-31
From Anne:
Se x e y são números reais distintos, então:
a) (x^2 + y^2) / (x - y) = x + y
b) (x^2 + y^2) / (x - y) = x - y
c) (x^2 - y^2) / (x - y) = x + y
d) (x^2 - y^2) / (x - y) = x - y
e) Nenhuma das alternativas anteriores é verdadeira.

Answered by Harley Weston.
Limits and composition 2011-12-30
From Rahul:
I want to know about limit proofs of composite functions. Like limit of log of a function equals log of limit of the function
Answered by Penny Nom.
Triangular numbers that are square 2011-12-27
From Luke:
The “TnS” is a positive integer which is both Triangular number and Square number. For example, 36 is a “TnS” number since 36=1+2+3+4+5+6+7+8 (Triangular number) and 36=6x6 (Square number). What is the next “TnS” which is greater than 36?
Answered by Penny Nom.
The graph of a quadratic function 2011-12-27
From Thomas:
sketch a quadratic function with zeros at -3 and 1
Answered by Penny Nom.
The volume of a frustum of a cone 2011-12-24
From CV:
If I know the height, taper, and volume of a frustum cone, what are its Radii?

Calculating frustum cone volume is straight forward.
v=Pi/3*h*(R*R+r*r+R*r) where;
v=volume;
h=frustum height;
R=major radius;
r=minor radius;
Pi=3.1415926;
t=taper, ('slant angle' where t=0 is a disk)

Here is the tricky part.
Knowing 'h', 't', & 'v';
Calculate one or both of 'R' & 'r'.

Specifically, what is the formula for 'R=' or 'r='?

Answered by Penny Nom.
Three sides of a triangle 2011-12-24
From saba:
the three sides of a triangular lot have lengths 10,11and 13cm,respectively. find the measure of its largest angle and the area of the lot?
Answered by Penny Nom.
The volume of a cylinder 2011-12-22
From luke:
what is the volume of a cylinder that is 30 mm in diameter and 70 cm high?

can you please write the equation out in full for me to help understand it better. thanks

Answered by Penny Nom.
The weight of 36 inch steel pipe 2011-12-22
From Keith:
What is the weight of 36 inch steel pipe 3/4 wall per foot in lbs.
Answered by Penny Nom.
Water is flowing into a cup 2011-12-19
From Tim:
A cup has a radius of 2" at the bottom and 6" on the top. It is 10" high. 4 Minutes ago, water started pouring at 10 cubic " per minute. How fast was the water level rising 4 minutes ago? How fast is the water level rising now? What will the rate be when the glass is full?
Answered by Penny Nom.
The number of gallons in a box 2011-12-18
From matt:
i need to know how many gallons a 16 inch square box will hold
Answered by Penny Nom.
Displacement 2011-12-17
From faisal:
a helicopter takes off and travels 300 km due north then returns to its landing field. what is its displacement?
Answered by Penny Nom.
Rounding 2011-12-14
From seth:
A decimal number rounded changed to 23.7 after it was rounded. Give a decimal number that is less than 23.7 and another that is greater than 23.7 that each round to 23.7. Explain to what place each number was rounded.
Answered by Penny Nom.
Augmented matrix 2011-12-14
From Britney:
Write Augmented matrix and use row reduction to solve.
I need major help with this problem! Tomorrow is my semester test and I have no idea how to do this.
2x+5y=8
y-x=10

Answered by Robert Dawson.
The circumference and area of a circle 2011-12-13
From Mable:
A circle that going 70mi across using 22/7 I need the area,radius, and the circumference and how to set up the steps can you help?
Answered by Penny Nom.
i+i^2+i^3+i^4...+i^102 2011-12-13
From megan:
how do you solve i+i^2+i^3+i^4...+i^102
Answered by Robert Dawson.
Extended real numbers 2011-12-12
From Justin:
Hi there, I was wondering does +infinity=+infinity in the extended real number system? Basically, I was wondering does +infinity=+infinity since -infinity and any extended real number (except +infinity) are less than +infinity?

Sincerely,

Justin

Answered by Robert Dawson.
Exponential form 2011-12-12
From Zoey:
What is the exponential form of 1/8??
Answered by Penny Nom.
Simple interest 2011-12-11
From sandeep:
Use the formula I = Prt to solve.

Damon deposits $500 into a savings account that pays simple interest at a rate of 0.65% per year. How long will it take Damon to earn $130 in interest?

Answered by Penny Nom.
factor 6x2-19x+3 2011-12-10
From pedro:
factor 6x2-19x+3
Answered by Penny Nom.
Jennifer wants to fence in her backyard 2011-12-09
From Val:
Jennifer wants to fence in her backyard. A yard is rectangular and is 15 feet by 12 feet. One 12-foot side is against the house so it does not need fencing. How many feet of fencing does Jennifer need?
Answered by Penny Nom.
A 3000 gallon tank 2011-12-08
From Delores:
I need to know what size to build a tank to hold 3000 gallons of water one side needs to be 8 ft width X 6 ft height how long does it need to be?
Answered by Penny Nom.
What does 2xy mean? 2011-12-07
From dana:
I don't know what is 2xy what does that mean? is it 2x+2y or 2x times y Please help! thank you for your time!! :)
Answered by Penny Nom.
The height of a flagpole 2011-12-07
From Grail:
at a certain point the angle of elevation of the top of a flagpole which stands on a level ground is 35 degree. 75 ft. nearer the pole, the angle of elevation is 50 degree. How high is the pole
Answered by Penny Nom.
Two trains passing each other 2011-12-07
From Tamkeen:
two trains 245m and 315m long are travelling toward each other at 90km/h and 54km/h respectively on parallel lines . how long do the train take to pass one another train the time they meet each other?
Answered by Penny Nom.
How many rpm does a 3.5 in. diameter wheel turn at 7 miles per hour? 2011-12-06
From Al:
how many rpm does a 3.5 in. diameter wheel turn at 7 miles per hour
Answered by Penny Nom.
Uniform acceleration 2011-12-06
From Android:
At a certain instant, two cars A and B are 2000 ft apart. At this instant, car A is traveling at 15 miles/hr. and accelerating at 3 ft/s^2 while B is moving towards A at 30 miles/hr. and an acceleration of 2 ft/s^2.

Find the ff:
a.) The possible time of their collusion in minutes.
b.) The distance in ft. each has traveled before collusion.

Answered by Penny Nom.
Squares and triangles 2011-12-06
From Liaqath:
You have squares and triangles.
Altogether there are 33 sides.
How many squares do you have?
How many triangles do you have?

Answered by Penny Nom.
The arc of a bridge 2011-12-05
From Marioneta:
Two same columns placed symmetrically, 40 m apart. The height of columns is 8 m. If the origin of the coordinate system is placed at the foot of the left column the equation of the arc of the bridge: f (x) =-1/80x^2+1/2x+8

What is the maximum height of a boat sailing under the bridge and identify its path.

Answered by Penny Nom.
A cube of ice is melting 2011-12-05
From Emily:
a cube of ice (i.e.) each side is of the same length) is melting at a rate such that the length of each side is decreasing at a rate of 5cm per hour. how fast is the volume of the cube decreasing (in cubic cm per hour) at the instant the length of each side is 25cm?
Answered by Penny Nom.
What is the altitude of the balloon? 2011-12-04
From C:
Chelcy and Jorgeare 2.32 miles apart. The observe a hot air balloon directly overhead between them Chelcy's angle of elevation is 28 degrees. Jorge's Angle of elevation is 37 degrees. What is the altitude of the balloon?
Answered by Penny Nom.
Two circles 2011-12-04
From Luke:
Two fixed circles intersect at A and B.
P is a variable point on one circle.
PA and PB when produced meet the other circle at M and N respectively.
Prove that MN is of constant length.
Thanks!
p.s. I also sent the question with a figure via email.

Answered by Chris Fisher.
Two parallel chords 2011-12-04
From violet:
Two parallel chords of a circle AB and CD on the same side of the centre measure 6 cm and 11 cm respectively. Find the radius.
Answered by Chris Fisher and Walter Whiteley.
Repeating a medication 2011-12-02
From ivan:
One person received the recommendation of a doctor taking the drug "Passaneura" every 10 hours. After a few days, noted that the time you should take the medication had not been repeated once. How many days since the first dose, will have to pass the time until recur
Answered by Penny Nom.
Solve for theta if 8cos^2 theta-3=1 2011-12-02
From Katherine:
Hi, I have just learned to solve trigonometric problems for theta and have one specific question in order to find the solutions to my homework. I will use one example for this question. If I have 8cos^2 theta-3=1 I first divide by 8 and get cos^2theta=3/8 then I have cos theta= plus or minus the square root of 3/8 Then I assume that I plug in inverse cos (the square root of 3/8) to my calculator. How do I find the four solutions (we are typically supposed to find four, I believe?) Can you help me with finding the solution to this problem? Thank you!
Answered by Penny Nom.
The total number of roles that a user can be assigned 2011-12-02
From Colin:
Hi, I want to define an equation for calculating the total combination of roles that a user can be assign in a system.

Here is the background: There are two types of roles that a user can be assigned. These are object roles and data roles. There must be at least 1 object and 1 data role in the system. A user must be assigned at least 1 object role and at least 1 data role. A user can be assigned as many object roles and data roles as are created in the system. There is no upper limit to the number of roles that a user can be assigned except for the total number of object roles and data roles that exist. The number of object roles and data roles in the system are independent. Can someone please assist? Thanks.

Answered by Penny Nom.
A suspension bridge 2011-11-30
From jennifer:
suspension bridges like the golden gate bridge, are used to span large distances. when the main curved cables are attached to the deck by vertical cables they will end up in the shape of parabola. assume that we need to build a bridge that spans 2,400 feet. the two towers 165feet tall each where placed at 400feet from either side. the lowest point of the center of the bridge at 10feet. vertical suspension cables where placed at 25foot interval. how many feet of cable are needed to connect the deck to the main cables between the two towers? show all working.
Answered by Penny Nom.
Four carpenters can build eight houses in 10 days. 2011-11-23
From Kenneth:
Four carpenters can build eight houses in 10 days. Two carpenters can build how many houses in 15 days?
Answered by Penny Nom.
A railway bridge over a road is in the shape of a parabola 2011-11-23
From Brennen:
A railway bridge over a road is in the shape of a parabola, and the bridge is 3 m high in the middle and 6 m wide at its base. A truck that is 2m wide is approaching the bridge. What is the maximum height that the truck can have and still pass under the bridge? Explain.

Thank you!!

Answered by Penny Nom.
The average age of 300 cars 2011-11-22
From Jason:
I have 300 used cars built from 1970 to 1980, I would like to know the average age by total volume. If I provide the qty built per year, can I determine the average age of the lot (300) by total volume?
Total built 50 56 30 10 30 25 64 9 12 14
Year built 1971 1972 1973 1974 1975 1976 1977 1978 1979 1980

Thanks

Answered by Penny Nom.
The height of a flag pole 2011-11-21
From Micah:
A flagpole stands vertically at the edge of a roof of a building 200 ft high. the angle of elevation of the pole from a point 100 ft from the bottom of the building is 67 degrees. Find the length of the pole in meters.
Answered by Penny Nom.
Water pouring into a conical tank 2011-11-21
From Patience:
Hi my name is patience and I'm having a problem with this question.
Water pours into a conical tank of semi vertical angle 30 degrees at the rate of 4 cm^3/s, where h is the depth of the water at time t. At what rate is the water rising in the tank when h = 10 cm?
Thank you

Answered by Penny Nom.
Reducing by a scale factor of 1/3 2011-11-21
From Sandra:
A rectangle with the dimensions of 6.3 cm and 5.8 cm is reduced by a scale factor of 1/3. What are the dimensions of the new image?
Answered by Penny Nom.
A triangle problem 2011-11-20
From May:
In triangle ABC, AB=AC, angle A=20,
D lie on AB making DC=AD,
E lie on AC making angle EBC=70.
Find angle DEB.

Answered by Chris Fisher.
15 pigs and 6 pens 2011-11-16
From Mike:
I have 15 pigs and 6 pens, I have to put them in odd numbers, how do I do it.
Answered by Chris Fisher.
The 3, 4, 5 right triangle 2011-11-15
From rishil:
to the nearest degree,what is the measure of the smalles angle in a rightt triangle with sides of length 3,4 and 5?
Answered by Penny Nom.
I started with Euler's identity and manipulated it 2011-11-14
From anonymous:
I started with Euler's identity and manipulated it
e^i*pi=-1
e^-i*pi=(-1)^-1
e^-i*pi=-1
e^-i*i*pi=(-1)^i
e^--pi=(-1)^i
e^pi=(-1)^i
type it in in a calculator and you get e^pi=23.1406926... and (-1)^i=0.0432139183... What did I do wrong?

Answered by Robert Dawson.
Lines tangent to y^2=4x 2011-11-11
From Reuchen:
Find equations of the lines tangent to y^2=4x and containing (-2,1).
Answered by Penny Nom.
The dimensions of a rectangle 2011-11-11
From Ivan:
The length of a rectangle is 31 centimeters less than five times its width. Its area is 72 sq centimeters. Find the dimensions of the rectangle.
Answered by Penny Nom.
A log equation in exponential form 2011-11-11
From Sarah:
Express the equation in Exponential form

log2 to the 16th power=4

Answered by Penny Nom.
The slope of a line 2011-11-10
From Sara:
Line k lies in the xy-plane.
The x-intercept of line k is -4.
And line k passes through the midpoint of the line segment whose endpoints are (2,9) and (2,0).
What is the slope of line k?
Give the answer as a fraction.

Answered by Penny Nom.
What was the cost price of the article? 2011-11-10
From tashana:
By selling an article for $320,the profit was 20% of the cost price.what was the cost price of the article.
Answered by Penny Nom.
The perimeter of a quarter circle 2011-11-10
From sierra:
how do you figure the perimeter of a quarter circle that has a radius of 12
Answered by Penny Nom.
A word puzzle 2011-11-10
From kevin:
ABCD
ABCD
ABCD
ABCD
-------
EBEA

What does each letter stand for? I've been at it for days and still have nothing.
Thank you,
Kevin

Answered by Robert Dawson.
Uses of logarithms 2011-11-09
From BEA:
what is the use of logarithmic functions in real life or daily life?
Answered by Penny Nom.
0.5 acres of land 2011-11-09
From Kim:
I have .5 acres of land. I want to divide it into 20x20 squares. How many squares will I have?
Answered by Penny Nom.
The volume of a berm 2011-11-04
From Roy:
I need to know how many cubic yards are in a berm 100 ft long 8 ft high and 32 ft wide at the bottom? Looking from the end of the berm it would look like a trangle. Thanks for the help.
Answered by Penny Nom.
A right angled triangle 2011-10-31
From bijo:
how can i find the tangent point at a circle with origin as center with radius r and the tangent pass through a given point P? I also want to find out the third point of a right angled triangle given other two points?
Answered by Chris Fisher.
The area of a triangular garden 2011-10-31
From tasha:
a garden has sides of 6m,8m and 10m find the area
Answered by Chris Fisher.
A scale drawing 2011-10-30
From aretha:
i have a scale drawing of a house floor plan, don't understand how to find the answer. the drawing is 1in: 3ft. need to find the length / width/and the actual length/width of the living room,kitchen, 3bedrooms deck,sitting room, entry, and closet
Answered by Penny Nom.
Modular arithmetic 2011-10-30
From Kim:
Hello,
I am editing a resource for students, and I think some of the answers may be incorrect. The text I was given and my questions are in the attachment. Any help you could give would be appreciated.
Thanks,

Kim

Answered by Harley Weston.
The LCM of 6,15 and x is 90 2011-10-30
From Richie:
Hi there,
There are 2 parts to the question.
#1. Factorise 90. This is easy, 90=2x3x3x5

#2
LCM of 6,15 and x is 90. What are possible values of x if x is odd?

From #1, since 90=6x15, how can this be used to work out possible values of x?

Thanks in advance, Richie

Answered by Penny Nom.
The average speed for the whole journey 2011-10-30
From Shahzad:
a train travels 68 Km at an average speed of 51Km/h. It then travels another 20Km at an average speed of 40Km/h before reaching its destination. Calculate the average speed for the whole journey?
Answered by Penny Nom.
A mythical soccer ball 2011-10-27
From Joel:
We've been working on this problem diligently and can't seem to come up with the answer book's answer. We think it may be wrong, yet want to check it with an expert. Here goes. The school's new soccer balls are covered with 64 regular hexagonal panels. Each hexagon measures 2 inches between opposite corners and 1.5 inches between opposite sides. What is the total surface area of the soccer ball?
Answered by Robert Dawson and Lorraine Dame.
The square root of z=3+4i 2011-10-27
From dianah:
how to find the square roots of complex number, z=3+4i
Answered by Robert Dawson.
3 fish 2011-10-27
From Ann:
You have 3 fish weighing a total of 21 lbs. The largest fish weighs twice as much as the second fish and the third fish weighs half the amount of the second fish. So how much does the third fish weigh?
Answered by Penny Nom.
A spherical ball in a conical wine glass 2011-10-26
From Jules:
A heavy spherical ball is lowered carefully into a full conical wine glass whose depth is h and whose generating angle (between the axis and a generator) is w. Show that the greatest overflow occurs when the radius of the ball is (h*sin(w))/(sin(w)+cos(2w)).
Answered by Claude Tardif.
The number of People who Know Each Other 2011-10-21
From Ted:
I'm trying to prove that at a party where there are at least two people, there are two people who know the same number of other people there.
Answered by Robert Dawson and Penny Nom.
37 miles in 3 seconds 2011-10-20
From Joyce:
Jake can travel 37 miles in 3 seconds,please calculate jake's rate of speed.(round to 2 decimal places)
Answered by Penny Nom.
Implicit differentiation 2011-10-20
From Monica:
Find dy/dx in terms of x and y, if sin(xy)=(x^2)-y.
Answered by Penny Nom.
The end of a swimming pool 2011-10-17
From Jim:
"The end of a swimming pool is a rectangle inclined 45° to the horizontal. The edge at the surface is 12 feet long and the submerged edge is 10 feet long.

Could you draw a diagram for me ? I don't understand the Problem.

Answered by Penny Nom.
The area of a circle 2011-10-17
From Winney:
If the diameter of a semicircle is 3 feet what is the area.
Answered by Penny Nom.
One central circle and three tangent circles 2011-10-16
From Margaret:
You have one central circle and three or more circles tangent to the outside of the circle of varying radii. You know the x,y coordinates of the centers of the other circles. If you now remove that central circle (and pretend you never knew where it was), can you calculate its center in x,y coordinates?
Answered by Chris Fisher.
Expanding the size of a table 2011-10-16
From Ericka:
You are working as a carpenter in an industrial Shop. A customer came to you and inquired about the size of the table which would be fitted in her room. She informed you that she had already a 1.5 x 1m table in her room but she wanted to maximize the space by adding the same amount to is length and width. She is planning to occupy a 3 square meter place on her room for her to work comfortably. She is requesting you a written recommendation before she asked to make a table. What amount should be added to both sides to maximize a 3 square meter area?
Answered by Penny Nom.
A 6% commission 2011-10-15
From Tamara:
if you earned a gross wage of $810, which includes a salary of $90 and a 6% commission how much is the net sales?
Answered by Penny Nom.
Expanded form 2011-10-12
From Chris:
Write this number in expanded form
4 326 180 501

Thanks!!!

Answered by Penny Nom.
Twice an even number 2011-10-10
From Ron:
The number of dollars that Tracy has in her pocket is an odd number. Hal gave Tracy $2 more than she already had in her pocket. The number of dollars Tracy now has cannot be _________.
a) $60 b) $62 c) $64 d) 68

Answered by Penny Nom.
Building a custom range hood 2011-10-08
From Bill:
I'm building a custom range hood for a customer with special order material that matches their newly installed cabinets and I need it to be perfect. The hood is basically a pyramid but the 4th side is the flat wall at the back and a flat, rectangular top. I need to calculate the bevel and miter of the three sides but I never was very good with geometry functions (although I am fairly good with other math fields). I either need the calculations from you at least (shudder) a formula or set of formulas so that I can calculate them myself.
Answered by Harley Weston.
What is the speed of the boat in still water? 2011-10-07
From Hannah:
The speed of a stream is 3 mph. A boat travels 10 miles upstream in the same time it takes to travel 16 miles downstream. What is the speed of the boat in still water
Answered by Penny Nom.
Three consecutive odd integers 2011-10-07
From Muneera:
Find 3 consecutive odd integers so that 3 times their sum is 5 more than 8 times the middle one?
Answered by Penny Nom.
Linear equations 2011-10-06
From Hannah:
I am confused about a question on my homework, here is an example on what I need to do:

You own a sightseeing boat tour. Kids' admission costs $5, adults' admission costs $8. During that day you get (made-up number) $521 with 71 admissions. How many of each passenger boarded?

I tried something like
5x + 8y = 521
x + y = 71

But, I still don't understand how to solve it! Help?

Answered by Penny Nom.
Two great circles 2011-10-06
From Jean:
"Two great circles lying in planes that are perpendicular to each other are drawn on a wooden sphere of radius "a". Part of the sphere is then shaved off in such a way that each cross section of the remaining solid that is perpendicular to the common diameter of the two great circles is a square whose vertices lie on these circles. Find the volume of this solid."

I don't understand the geometry of the problem. Can you please explain the problem and if possible draw a diagram for me ?

Answered by Chris Fisher.
Six golphers for 27 weeks 2011-10-04
From ken:
I have 6 golfers alternating playing in a group of 4 each week for 27 weeks. How can I create a schedule so that each golfer both plays equally and plays against the other golfers an equal amount of time
Answered by Victoria West.
A reservoir has the shape of an inverted cone 2011-10-03
From Roger:
a reservoir has the shape of an inverted cone whose cross section is an equilateral triangle. if water is being pumped out of the reservoir at a rate of 2m^3/sec, at what rate is the depth of the water changing when the depth is 40 meters?
Answered by Penny Nom.
A trapezoid inscribed in a circle 2011-10-02
From Greg:
A trapezoid is inscribed within a circle. The two interior angles who share the longest side are 70 and 80. The arc whose chord is the longest side has a length of 120. Find the other two interior angles of the trapezoid, and the other three arc lengths.
Answered by Chris Fisher.
Drawing three coins from 10 coins 2011-09-29
From Michelle:
Three coins are chosen at random from a purse containing 6 dimes & 4 quarters. What is the probability that the selection will contain at least one quarter?
Answered by Penny Nom.
3-digit, daily numbers 2011-09-29
From Margaret:
I need the list/group for a raffle that would contain the combinations for a 3 digit (000-999) daily number. There would be 100 tickets sold with 10 3-digit numbers from each numerical group beginning from 000's, 100's 200's, 300's,...900's. No duplicates/replacement. What would be the list of possible combinations of each group of 100 tickets, keeping each different group/list of combinations in separate blocks of 100 tickets without duplication. So I could use 1 list of 100 this year and the next list of combos next year and so forth. Please help!

Example: 000,197,245,367,445,569,618,777,842,964

What would be the possible lists of each 100 tickets? Thank you

Answered by Penny Nom.
Three men sharing a task 2011-09-27
From Olaniyan:
Three men do a piece of work, the first does 7/15, the second does 5/6 of the remainder while the third does the rest.if the rest done by the third person equals 24 units, find the whole piece of work?
Answered by Penny Nom.
A hemispherical bowl with a lead ball inside 2011-09-27
From Jean:
"(a) Water is being poured into a hemispherical bowl of radius 3 inch at the rate of 1 inch^3/s. How fast is the water level rising when the water is 1 inch deep ?

(b) In (a), suppose that the bowl contains a lead ball 2 inch in diameter, and find how fast the water level is rising when the ball is half submerged."

Answered by Penny Nom.
90 cm of pipe 2011-09-26
From margie:
A 90cm pipe is cut into two pieces so that one pieces is 6cm shorter than the other. how long was each piece?
Answered by Penny Nom.
A tank is 3/8 full 2011-09-24
From Olaniyan:
When 150 liters has been drawn from a tank, it is 3/8 full; how many liters will the tank hold?
Answered by Penny Nom.
The sides of a regular octagon of area 800 sq. ft. 2011-09-21
From Susan:
Help please. I am building an octagon tree house to live in. I would like the total square footage to equal 800 to 1,000 sq ft. How long should each side of the octagon be.
tx, susan

Answered by Penny Nom.
Place value 2011-09-20
From Ivana:
the digit in the ones place is a hand with out a thumb(4) 2) the digit in the thousands place is 2 less than the digit in the one's place. (4-2= 3) the digit in the ten's place is the sum of 2 and 6 (2+6= 4) the digit in the hundreds place is what you get when you get when you subtract 5 from the digit in the ten's place (ten's-5= what is the number?
Answered by Penny Nom.
The volume of a hexagonal prism 2011-09-19
From Denise:
I am being asked to find the volume of a hexagonal prism, base edges are 3cm, and a height of 15cm. The book that I am working from says that the answer is 77.94cm(cubed) but no matter what the equation I use I always get the answer 116.9/117. Please help! Thank you!
Answered by Penny Nom.
5 crayons 2011-09-17
From John:
How many four color combinations are possible with 5 crayons
Answered by Penny Nom.
A rectangle of largest possible area 2011-09-16
From mary:
Steven has 100 feet of fencing and wants to build a fence in a shape of a rectangle to enclose the largest possible area what should be the dimensions of the rectangle
Answered by Penny Nom.
A four digit number 2011-09-16
From gina:
i am having a hard time figuring this out.. i believe the answer is 3105 but the 0 is throwing me off... i am a 4 digit number between 3100 and 3200, none of my digits are even, the sum of my digits is 9, the largest digit is in the ones place, what number am i ??
Answered by Penny Nom.
A rectangular wall 2011-09-12
From Jimi:
The area of a rectangular wall of a barn is 110 square feet. It's length is 12 feet longer than twice its width. Find the length and width of the wall of the barn.
Answered by Penny Nom.
Blocks in a wall 2011-09-12
From JOHN:
HOW MANY BLOCK WILL IT TAKE TO BUILD A 28*48 HOME, ON A FOOTER THAT IS DONE ALREADY, I WANT TO GO TEN HIGH.
Answered by Harley Weston.
A beginning algebra question 2011-09-10
From Tobby:
A=1/2 solve for h.area of a rectangle with b and h
Answered by Penny Nom.
Decartes' circle theorem 2011-09-07
From Joseph:
Given 3 circles of diameters 50, 55, and 60 units, Place them externally tangent. What is the diameter of the outer circumscribing circle, tangent to all 3 circles? I can attach a pdf if this description is not clear. Not urgent, this has nothing to do with current assignments, just wondering if I have developed the best methods?
Answered by Chris Fisher.
The derivative of f(x) = (x+1)^1/2 2011-09-05
From Carla:
Find the derivative using the limit process of
f(x) = (x+1)^1/2

Answered by Harley Weston.
Geometry Related Careers 2011-09-01
From Richard:
Good morning,

I am hoping to find a list of careers that would relate to geometric constructions and tessellations.

I have searched the internet some and some of the sites I have found are a bit questionable or dated. Are there sites that Math Central would recommend?

I could not find anything when I did a search on the site.

Thanks in advance,

RVD

Answered by Walter Whiteley.
4 x 2 2011-08-30
From Lucjan:
how to sketch a diagram that shows 4x2 expression?
Answered by Penny Nom.
Exponential form? 2011-08-30
From Emily:
I have a 6th grader that needs to find out what is the exponential form of 461,885,811
Answered by Penny Nom.
A coin toss game 2011-08-29
From Aniish:
A and B alternately toss a coin.The first one to turn up a head wins.If no more than 5 tosses each are allowed for a single game,find the probability that the person who tosses first will win the game?What are the odd's against A losing if she goes first?
Answered by Penny Nom.
A border around a flower garden 2011-08-28
From jessica:
A Landscaper, who just completed a rectangular flower garden measuring 6 feet by 10 feet, orders 1 cubic yard of premixed cement, all of which is to be used to create a border of uniform width around the garden. if the border is to have a depth of 3 inches, how wide will the border be? ( 1 cubic yard=27 cubic feet)
Answered by Penny Nom.
An anti-freeze solution 2011-08-27
From aniish:
The radiator of a jeep has a capacity of 4 gallons. It is filled with an anti-freeze solution of water and glycol which analyzes 10% glycol.what volume of the mixture must be drawn off and replaced with glycol to obtain a 25% glycol solution? all percentages are by volume
Answered by Penny Nom.
25 billion dollars 2011-08-26
From Bill:
If you have $25billion dollars, what would the value of this amount become if you inflated it at 4% per year for 55 years?=C2=A0 My TVM calculator will not go this high.
Answered by Penny Nom.
The cost of petrol 2011-08-26
From Sam:
Hi, i'm confused about this question from my textbook, here goes: The cost of petrol for a 240 km journey for a car which runs 12 km on each litre of petrol is $24. What would be the cost of petrol for a 500 km journey for a van which runs 11km on each litre of petrol? please answer in detail and thanks in advance :)
Answered by Penny Nom.
Driving to meet Mary 2011-08-24
From Denise:
mary leaves St Paul traveling 2000 miles @ 55 MPH; John leaves LA traveling 2000 miles @ 45 MPH how many miles will John had traveled to meet mary?
Answered by Penny Nom.
The length of an arc 2011-08-24
From nuel:
Is there any formula to calculate the arc length from only the chord length and angle between the chord and tangent of the arc at its endpoint ?
Answered by Penny Nom.
Exponential form 2011-08-23
From kiara:
The African bush elephant is the largest land animal and weighs about 8 tons . Write the amount in exponential form .
Answered by Robert Dawson.
Romeo throws a pebble at Juliet's wondow 2011-08-22
From Natalie:
There is a picture of Romeo trying to attract Juliet's attention without her nurse who is in a downstairs room, noticing. he stands 10m from the house and lobs a small pebble at her bedroom window (3.5m high). Romeo throws the pebble from a height of 1m with a speed of 11.5m/s at an angle of 60degrees to the horizontal. I have already found that it take 1.74seconds to reach the window and that it does in fact hit Juliet's window however i cannot work out the speed of the pebble when it hits the window! The answer is 9.12m/s but I cannot reach this answer. Hope you can help me :)
Answered by Robert Dawson.
Three tangent circles 2011-08-21
From maribie:
three discs are tangent externally distances between their centers are 23cm, 15cm, and 20cm. find their radii.t
Answered by Penny Nom.
A picture with a frame 2011-08-19
From Aniish:
A picture 8inches by 12inches is placed in a frame of uniform width. If the area of the frame equals the area of the picture find the width of the frame?
Answered by Penny Nom.
Three tangent circles 2011-08-19
From hanniel:
two coin are tangent to a third coin internally and are tangent to each other externally. The distance between their centers are 14 mm, 17mm, and 5mm. find their radii
Answered by Penny Nom.
3.8 x 2.2 2011-08-18
From Isaac:
Question from isaac, a student: I am struggling in math & I need help on multiplying 3.8 x 2.2. Can you helpme
Answered by Melanie Tyrer and Harley Weston.
Ordering fractions 2011-08-17
From Myon:
My problem is putting fractions from least to greatest and they are 1/4 2/5 1/3 4/6 1/2 8/9 3/4 1/1 2/6 1/10 thanks
Answered by Penny Nom.
Two line segments in the plane 2011-08-15
From Tim:
For this problem I actually have tried to visualise the image in my head many times. This question makes my head spin.

Four points lie in a plane. They are partitioned into two pairs so that the sum of the lengths of the segments joining the points of each pair has the minimal possible value. Prove that these segments have no common points.

Answered by Chris Fisher.
A cyclic quadrilateral 2011-08-15
From Tim:
Hi! I've been working on this for a while and I'm quite stuck. If anyone can help that would be great. The sides BC and AD of a quadrilateral ABCD are parallel. A circle meets the side AB at B and E and the side CD at C and F. Prove that the quadrilateral AEFD is cyclic.
Answered by Chris Fisher.
A ratio as a fraction in lowest terms 2011-08-12
From jonecia:
write this ratio as a fraction in lowest terms 707days of 112days?
Answered by Melanie Tyrer.
Infinite Logarithmic Series 2011-08-08
From Sourik:
Dear Expert,

In my Amithabha Mitra and Shambhunath Ganguly's "A Text Book of Mathematics" I found the formula of log (1+x) where the base is e and x lies in between -1 and +1.As I want to learn Mathematics,I am not satisfied with the mere statement of the formula.Please help giving me the full proof.
Thanking you,
Sourik

Answered by Robert Dawson.
A concrete octagonal pad 2011-08-04
From Tina:
I have a project where I need to construct a concrete octagonal pad. Each side is 12' and it is 5" deep. Please tell me how to calculate square yards of concrete. Thank you.
Answered by Penny Nom.
A 3 foot circle using 1/2" cord 2011-08-04
From Cindy:
To make a 3 foot circle using 1/2" cord - how many lineal feet of cord do I need?
Answered by Robert Dawson.
A lock box 2011-08-03
From Tom:
I need to open a lock box from my parent's estate. The combination is a simple 4 number lock. The last two numbers are 33. Numbers can be used more than once.What are the possible combinations? WP
Answered by Harley Weston.
Ben's favourite numbers 2011-08-03
From Luke:
Ben challenges you to guess his favourite numbers by using the following three clues: -Each number is a four-digit odd number. -Each is a palindrome -The digits of each number add up to 10 -None of the numbers are divisible by 5
Answered by Robert Dawson.
A cone with an oval as a base 2011-08-03
From Emily:
Hi, I was wondering how to calculate the surface area of a cone with an oval as a base (which I think is referred to as an elliptic cone or something like that). I have the both the maximum and minimum radius as well as the height, but I don't have a slant height and I'm not sure how to calculate it and then calculate surface area. I would really appreciate some help. Thanks!
Answered by Robert Dawson.
A point on a circle 2011-08-02
From George:
I know the center location (x,y) of the circle, I know the radius of the circle, I know the location (X,Y) of one point on the circle, and I know the angle (in degrees not radians) between the known point location (X,Y) and an unknown point location (let's call it (A,B) for reference). What formula(s) can I use to find out the coordinate position of (A,B)?
Answered by Chris Fisher.
An equation in the form Ax+By=C 2011-07-28
From Mike:
I need to write an equation in the form Ax+By=C The points I was given are (5,0) and (-3,5)

I know how to find the slope I got -5/8 and wrote it out as y=(-5/8)x+b however I'm not quite sure where to go from here.
Any help would be greatly appreciated.

Answered by Penny Nom.
A stained glass lamp 2011-07-25
From Guy:
Like Kay, I also work in stained glass, but in 3-dimensions. I am frequently asked to replicate lamp shades in stained glass where the diameter of the top is different (narrower) from the diameter of the bottom (which is wider). Some people want 5, 6, 7, 8, 10, 12, 16, ,,, n-sided shades. Is there a formula I can use to determine the width of the sides using the angle, if I remember correctly, I think it's called theta. In other words, is there a formula where I can plug in the angle which describes the arc of the circle. For instance, if someone asks for a 7 sided shade, plugging in 51.43 (360/7). I could then use that to determine the width at the top and bottom rings to create the appropriate trapezoids. I've visited a few sites so the formula looks like its a function of sin & cos but they are presented like proofs for teaching. Your site appears to want to actually answer questions without making the inquirer feel stupid.
Answered by Harley Weston.
The height of a fluid in a horizontal tank 2011-07-24
From jason:
Same set up as many others, cylindrical tank on its side, but I am interested in defining the change in volume and/or fluid level as a function of time at a constant volumetric outflow. I plan on hooking a pump to the tank so "gpms' will be constant. I have a couple different sized tanks and pumps so I want a general equation. Thanks for your help.
Answered by Harley Weston.
A cylinder fits perfectly in a box 2011-07-18
From Brix:
A cylinder fits perfectly in a box. 76 cubic inches of oil is poured into the space between the cylinder and the box, filling the space halfway. If the box is 3ft wide, how tall is the cylinder in inches?
Answered by Stephen La Rocque.
A regular hexagon inscribed in a circle 2011-07-18
From Courtney:
If ABCDEF is a regular hexagon inscribed in a circle of radius r, prove that the length of each side of the hexagon equals r.
Answered by Penny Nom.
A rectangle is inscribed in a circle 2011-07-17
From Alexea:
A rectangle is inscribed in a circle of diameter 15in. Express the perimeter as a function of the width x.
Answered by Penny Nom.
A linear equation with fractions 2011-07-13
From Simon:
I would like to ask if you would be able to explain : linear equation: 2 1/2 (X-1) - x+3 /3 = 4 , the first step shows multiplying by 2 then multiplying by 3. etc. What I don't understand is where the 2 in multiplying by 2 comes from? ( the three is pretty obvious being x+3 /3 (to get rid of the divided by) ) .
Thanks for your help.

Answered by Penny Nom.
Fencing around a pool 2011-07-12
From Anita:
The measurement of a swimming pool are 32 feet by 18 feet. How much fencing is needed if the fence is to be built 7 feet from each side of the pool?
Answered by Penny Nom.
A group of 40 golfers 2011-07-11
From Doug:
We are a group of 40 golfers who play in 10 x foursomes playing 3 rounds. Can we all play with different players each round yet maintain the foursomes format.
Answered by Victoria West.
Golf for 14 2011-07-11
From Doug:
We have 14 players playing over 4 days. can we play with different players every day and keep the 3 x foursomes and 1 x pair format
Answered by Victoria West.
Two piles of cards 2011-07-09
From maribie:
a standard deck of playing cards contains 26 black cards and 26 red cards, or 52 cards in all. a deck is randomly divided into two unequal piles, such that the probability of drawing a red card from the small pile is 1/3. at the same time, the probability of drawing a black card from the larger pile is 5/14. how many cards are in the larger pile?
Answered by Penny Nom.
Golf for 12 2011-07-07
From Jim:
I have a golf trip scheduled with a total of 12 guys. We are playing three rounds of golf. For round 1 Players A B C D must play together. Is there a formula that would allow me to set up the other 8 foursomes with no repeats or minimal repeats? Thanks.
Answered by Victoria West.
concentric circles 2011-07-06
From maribiie:
two circles are concentric. the tangent to the inner circle forms a chord of 12cm in the larger circle. find the area of the "ring" between the two circles?
Answered by Penny Nom.
Achilles and a turtle 2011-07-01
From Jean:
Achilles and a turtle are having a race. The turtle starts 45m ahead of Achilles and Achilles is twice as fast as the turtle. If turtle runs at 1m/s,how far would the turtle have run before he is outrun by Achilles?
Answered by Penny Nom.
A column and a sloped ceiling 2011-06-10
From Tom:
Hi! I am a woodworker who needs some help finding a formula. I have to make a cylinder shape column that butts up against a sloped ceiling using bending plywood. I need to figure out how to cut the shape on this flat piece of material. The diameter of the column is 54.375". The tallest point is 9 13/16" and the shortest point is 4 7/16". Any help would be greatly appreciated.
Answered by Harley Weston.
Construct a 30 degree angle 2011-06-10
From Ella:
they give you a simple straight line and ask you to construct an angle of 30 degrees at one end of the line using only a ruler and a compass?
Answered by Penny Nom.
A line tangent to f(x)=1/x 2011-06-05
From Michael:
A line tangent to f(x)=1/x in the first quadrant creates a right triangle with legs the x-axis and the y-axis. Prove that this triangle is always 2 square units regardless of where the point of tangency is.
Answered by Penny Nom.
Tiles for a back splash 2011-06-04
From Catherine:
How man 3 11/16 tiles do I need for 28 square foot for a back splash
Answered by Penny Nom.
A reel of paper 2011-06-03
From keith:
if a full reel of paper has 7750 metres on it with a diameter of 1240mm and a core diameter of 100mm how can work out the thickness of each strip of paper therefore allowing me to work out the metreage of the roll when its half in diameter at say 620mm
Answered by Penny Nom.
The volume of a decagonal prism 2011-05-29
From Susan:
How do you find the volume of a decagonal prism?
Answered by Penny Nom.
How tall is the pole? 2011-05-27
From cynthia:
20 foot rope is tied 2 feet from the top of a pole and secured 8 feet from the base of the pole....How tall is the pole?
Answered by Penny Nom.
How many meters long should the race be? 2011-05-26
From Lee:
Nathan can walk at a rate of two meters per second while David can easily go three-and-a-half meters per second. David offers Nathan a 45 meter head start. How many meters long should the race be in order for Nathan to win by a nose
Answered by Penny Nom.
16 golfers and 8 rounds 2011-05-26
From Gerry:
16 golfers and 8 rounds. To have each player play with as many others as possible.
Answered by Victoria West.
pi/2 + 12/13 2011-05-24
From Jen:
If you have (pi/2) and you want to add it to (12/13), how would you go about it?
Answered by Chris Fisher.
Problem solving with fractions 2011-05-21
From emilyd:
I had a packet of sweets. I put 1/5 of the sweets into Container A, 1/3 of them in Container G, 10 sweets in container C and the rest in container D. If the number of sweets in container D is equal to 75% of the total number of sweets in container A and B, how many sweets are there altogether?
Answered by Penny Nom.
What is the angle of decent? 2011-05-20
From Taten:
So you have an airplane with an altitude of 5.8KM. It has 46KM till the run way. What is the angle of decent?
Answered by Penny Nom.
A pipe in a ditch 2011-05-19
From Jim:
How many cubic feet of stone will it take to fill to six inches around a 4 inch pipe in a ditch 128 ft long x 20 in wide?
Answered by Penny Nom.
The length of a belt around three pulleys 2011-05-18
From Grant:
I need to calculate the belt length around these pulleys, please can you help or refer me?

Known variables
D - Large Pulley Diameter
d - Small Pulley Diameter
c - Center Distance between D and d
T - Tension Pulley Diameter
x - Horizontal Distance between T and d' Centers
y - Vertical Distance between T and d's Centers
I need to calculate the belt length around these pulleys.


Kind Regards,
Grant

Answered by Harley Weston.
Exponential form 2011-05-18
From Kara:
How do I write this in exponential form?

log 1/8 = -3

Answered by Penny Nom.
A rational number 2011-05-17
From owais:
according to definition of irrational number "a number which cannot be expressed in p/q form is called irrational number" square root of 2 is a irrational number but if i round off the digit correct to two decimal places so it becomes 1.41 and we can easily convert into p/q form so it is a rational number ???
Answered by Penny Nom.
What was his salary before the raise? 2011-05-16
From Annette:
Need to show all my work and round to the nearest dollar. If James received a 9% raise and is now making $102.00 per year, what was his salary before the raise?
Answered by Penny Nom.
Two whole numbers 2011-05-11
From yolanda:
The sum of two whole numbers is 12.If the sum of the squares of those numbers is 74,what are the two numbers?
Answered by Penny Nom.
Golf with 36 players 2011-05-09
From Terry:
What is the calculation to have 36 players play in a different foursomes each week. I would guess this would take 12 times but do not know how to figure this out
Answered by Victoria West.
The dimensions of a rectangle 2011-05-08
From yolanda:
If the length of a rectangle is 3ft less than twice its width and the area of the rectangle is 54ft^2,what are the dimensions of the rectangle?
Answered by Penny Nom.
The volume of a triangular shaped aquarium 2011-05-07
From Jeff:
HI, I searched the site but really couldn't find my answer, at least that I could understand! I have a triangular shaped aquarium ( think of a cube, except its an equal triangle shape). all 3 sides are 48 inches, and it is 34" tall. Can you please tell me what the volume is in gallons? Thanks!
Answered by Penny Nom.
A circle inscribed in a triangle 2011-05-07
From Aishwarya:
The angles of a triangle are 50, 60, and 70 degrees, and a circle is touches the sides at A, B, C. Calculate the angles of the triangle ABC.
Answered by Penny Nom.
(4x-5)(3x+2)=0 2011-05-04
From Corina:
solve: (4x-5)(3x+2)=0
Answered by Penny Nom.
The linear thermal expansion of steel 2011-05-03
From Scot:
Several questions on your site deal with the linear thermal expansion of steel. Such as how much will a piece of steel grow in length if it is heated. My question is similar but I would like to know if there is a different calculation to determine how much the diameter of a round bar will grow when heated. Can you tell me how I can calculate how the diameter of .500" round steel will increase for every degree of temperature change? If a bar is raised from 60 degrees F to 120 degrees F how much will the diameter change?
Answered by Robert Dawson.
An exclusion zone around a triangle II 2011-05-03
From Aishwarya:
A long time ago Mr gibson found an island shaped as a triangle with three straight shores of length 3km, 4km and 5km. He declared an exclusion zone around his island and forbade anyone to come within 1km of his shore. What was the area of his exclusion zone? This question was answered but did not understand the explanation.
Answered by Robert Dawson.
A scale of 1:160 2011-05-02
From Dave:
a 4 foot by 8 foot sheet of plywood is how many acres in a scale (1:160)
Answered by Penny Nom.
How many students are 25 and under? 2011-05-01
From ella:
use the five steps of problem solving to answer the following question Enrollment at a local university is 8,500. There are one third as many students over the age of25. there are 25 and under. how many students are 25 and under and how many are 25 and older(p 5)
Answered by Penny Nom.
Three tangent circles 2011-05-01
From mark:
Three circles of radii 24 cm, 32 cm, and 42 cm are externally tangent to each other (each is tangent to the other two). Draw a diagram and using the Law of Cosines find the largest angle of the triangle formed by joining their centres.
Answered by Penny Nom.
A trough of water 2011-05-01
From alvin:
A trough is formed by nailing together, edge to edge, two boards 11 ft. in length, so that the right section is a right triangle. If a 15 gallon of water are poured into the trough and if the trough is held level so that a right section of water is an isosceles right triangle, how deep is the water? (231cu.in=1 gal.)
Answered by Penny Nom.
The area of a rectangle 2011-04-30
From Hayley:
A rectangles width is exactly twice its length. The area of the rectangle is 450 square feet. What are the length and width of the rectangle?
Answered by Penny Nom.
Why use percentiles? 2011-04-29
From Kenneth:
I need some clarification regarding the word "percentile."

Here is an example and what I know: If an investor had twenty stocks in his portfolio and five of those stocks paid an annual dividend of $200.00 or more, the five stocks are in the 25 percentile.

Why is this word necessary? If my example and explanation is correct, why not simply indicate that 25% of his stocks paid an annual dividend of $200.00 or more?

Answered by Penny Nom.
Deciding where to bowl 2011-04-29
From ella:
the bowling alley on wedge street charges $5.50 for shoe rentals plus $2.50 per game. lawerence bowing alley charges $4.00 shoe rentals and $3.00 per game . how many games should they bowl to make wedge street cheaper than lawerence street
Answered by Penny Nom.
cos(x) = -1/(square root of 2) 2011-04-27
From Shelby:
Find exact value of x for 1 <(or equal to) x < 2pi a) cos(x) = -1/(square root of 2)
Answered by Penny Nom.
The radius of a cylinder 2011-04-27
From Jazmin:
Hi, I don't understand how to find the radius in a cylinder with only the surface area (143.7) and the height (0.8)? I know that the formula is 2pir2+2pirh, but I don't see how to isolate the r? I appreciate your help.
Answered by Penny Nom.
The length of a chord 2011-04-25
From G:
A 120 degree central angle intercepts a circle at the points A and B. The radius of the circle is 10 cm. Find the length of chord AB.
Answered by Penny Nom.
(27^r^-1)^-2/3 2011-04-24
From Yung-Ju:
(27^r^-1)^-2/3
Answered by Penny Nom.
A right angled triangle 2011-04-22
From jeremy:
Solve the right triangle with the given sides and angles. a=3.0 cm and b=1.6 cm
Answered by Penny Nom.
Dividing the profits 2011-04-19
From Tracy:
I need to know how to come up with the answers to the following problems. I need to be able to show my work. So can you help me on how to come up with the answers to questions like these. Thanks.

Three individuals form a partnership and agree to divide the profits equally. X invests $9,000, Y invests $7,000, Z invests $4,000. If the profits are $4,800, how much less does x receive compared to having the profits divided in proportion to the amounts invested by X, Y, and Z?

In printing an article of 48,000 words, a printer decides to use two sizes of type. Using the larger type, a printed page contains 1,800 words. Using a smaller type, a page contains 2,400 words. The article is allotted 21 full pages in a magazine. How many pages must be in smaller type? ___________

thank you

Answered by Penny Nom.
How many cats are needed to catch 60 rats in 1 hour? 2011-04-19
From Melody:
If 3 cats can catch 3 rats in 3 minutes, how many cats are needed to catch 60 rats in 1 hour?
Answered by Penny Nom.
Solve x(x-3) = 5 (x+4) 2011-04-19
From Ashley:
I have to solve using the quadratic formula. And my answer should be in simplest exact form. And I can't approximate.
I really need help to prepare for a test ....
x(x-3) = 5 (x+4)

Answered by Penny Nom.
An antiderivative of the square root of (8t + 3) 2011-04-19
From Caitlyn:
I know how to take an antiderivative. But this one's stumping me. I need it to finish a problem. What's the antiderivative of the square root of (8t + 3)

~Caitlyn=

Answered by Penny Nom.
Ramon has 6 puppies 2011-04-17
From Mary:
Ramon has 6 puppies. Five of the 6 weigh the same and one puppy is heavier. If Ramon had a balance scale, what is the least number of times he could weigh the puppies to be sure he could determine which puppy was heavier?
Answered by Penny Nom.
Find the rate at which the searchlight rotates 2011-04-17
From Meredith:
A searchlight is position 10 meters from a sidewalk. A person is walking along the sidewalk at a constant speed of 2 meters per second. The searchlight rotates so that it shines on the person. Find the rate at which the searchlight rotates when the person is 25 meters from the searchlight.
Answered by Penny Nom.
Cone story problem 2011-04-16
From Chelsea:
There is a cone with a story problem that states only that the radius is the same as the altitude and that the slant height of the cone is 6cm. I have no idea how to figure this problem out.
Answered by Penny Nom.
Dividing a polygon into triangles 2011-04-16
From Foxie:
You have a given regular polygon with n vertices and you divide it into triangles(using the vertices of the polygon) which each share at least one side with the polygon. How many distinct ways can you divide the polygon if its vertices are numbered? For n=3 it's 1 way, for n=4 it's 2 ways for n=5 5 ways, I'm not quite sure but think that for n=6 it's 12 ways... thanks in advance!
Answered by Claude Tardif.
The volume of a triangular pyramid 2011-04-15
From Chelsea:
There is a triangular pyramid which is an equilateral on the base with all sides 16.5 and the height is 25. I have to find the volume. Please help!
Answered by Penny Nom.
A concrete walk around a pool 2011-04-15
From Skipper:
I want to know the answer if you were to want a pool to be 20 ft by 30 ft and wanted a 5 ft border around it what would be the dimension of the whole area plus concrete walk
Answered by Penny Nom.
Arrangements of ten letters 2011-04-15
From Mustafa:
In how many ways can the ten letters of the word GELATINOUS be arranged in a line so that the vowels are in alphabetical order (not necessarily together) when read from left to right?
Answered by Penny Nom.
A golf league of 60 golfers 2011-04-14
From Keith:
I have a golf league of 60 golfers and I want to have each golfer play in a different group of 4 each week for 24 weeks. How can I do this
Answered by Victoria West.
Coefficient of variation 2011-04-14
From Braden:
When determining coefficient of variation (CV) or %CV is it possible to calculate %CV for two variables? For instance can %CV be used to determine the precision of 5 data points on a graph using the X and Y coordinates? or does %CV need to be determined for each variable separately?
Answered by Robert Dawson.
The volume of a hopper 2011-04-13
From michael:
I have a tank that is 72" x 72" at the top and 10" x 10" at the bottom and is 31" tall. I need to calculate the cubic feet of the tank
Answered by Penny Nom.
Decreasing numbers 2011-04-13
From Hadi:
A number is called a "decreasing number" if each digit in the number is less than the digit to its left. For example, 87420 is a deceasing number. How many five-digit decreasing numbers are there?
Answered by Claude Tardif.
2 cubic foot bags of mulch 2011-04-11
From Ron:
if I buy 2 cubic foot bags of mulch, how bags would I need to equal 1 yard? (13 bags?)
Answered by Penny Nom.
x/6 + 4/5 = x/9 2011-04-10
From Khadija:
x/6 + 4/5 = x/9
Answered by Penny Nom.
Real-life applications of trigonometry 2011-04-10
From Angela:
I am a teacher and I desire to show the students the real-life application of trigonometry. Of course, one application is to use a clinometer and find the heights of various things. However, I am trying to provide a real-life scenario which also answers the question "why" the height of the object needs to be found. Not being an engineer, I do not know the specifics examples, but I want my information to be accurate and my example to to be as real-life as possible. I mean, I can say that someone wants to know the height of a flagpole; however, I also want to answer the question "why" they want to know this. I would like to give an actual real-life scenario. Do you know of some? Thanks!
Answered by Penny Nom.
8 golfers playing 2 groups of 4 each day for 5 days 2011-04-08
From Chris:
Hi,
8 golfers playing 2 groups of 4 each day for 5 days. Trying to work out how they can organise playing at least once, and no more than twice, with everyone over the 5 days. We always come up with someone playing 3 times with another in one of the 4's!
Hope you can help!
Regards.

Answered by Victoria West.
The degree measure of an arc in a circle 2011-04-08
From Deb:
How do I find the degree measure of an arc in a circle with the Length of 14 and the radius of 70?
Answered by Penny Nom.
The volume of a flower pot 2011-04-07
From kp:
I have 2 flower pots with the following dimensions that I am trying to calculate the volume of Soil I will need to fill them. pot #1 29"tall, top of pot 31.5 inches across( radius of 15.5") the bottom of the pot is 21"across (radius 10.5) Pot #2 29"tall, top of pot 26 across (radius 13") the bottom of the pot 17'"a cross (radius 8.5")
thank you
KP

Answered by Penny Nom.
Eliminate y 2011-04-07
From Lynn:
2x + y = 8
y + 3z =5
z + 2w =1
5w + 3x = 9

Form three equations with y eliminated

Answered by Penny Nom.
A conical container and a spherical balloon 2011-04-06
From Steven:
Water is running out of a conical container 12 feet in diameter and 8 feet deep (vertex down) and filling a spherical balloon. At the instant the depth of the water in the cone is 4 feet, the radius of the sphere is approximately 4 feet.

The rate of change of the depth of the water in the cone at the instant is approximately ______________ times the rate of change of the radius of the balloon.

Answered by Penny Nom.
Two ships 2011-04-05
From Gevork:
Ship A is sailing due south at 16 mph. At the same time, a second ship B, 32 miles south of A, is sailing due east at 12 mph.
(a) at what rate are they approaching or separating at the end of one hour?
(b) At what rate are they approaching or separating at the end of two hours?
(c) When do they cease to approach each other and how far apart are they at that instant.

Answered by Penny Nom.
The rate of change of the area of a parallelogram 2011-04-05
From Gevork:
Let a parallelogram have sides of 8 and 12 and let vertex angle A be decreasing at a rate of 2degrees per minute. Find the rate of change of the area of the parallelogram when angle A equals 30 degrees.
Answered by Penny Nom.
Converting millimeters to centimeters 2011-04-04
From Pat:
I need to convert 67 mm to centimeters how do I do this?
Answered by Penny Nom.
The perimeter of a rectangle 2011-04-04
From aspen:
the perimeter of a rectangle is twice the sum of its length and its width. the perimeter is 40 meters and its length is 2 meters more then twice its width. what is the length?
Answered by Penny Nom.
Tennis schedule 2011-04-04
From Toni:
I need to create a doubles tennis schedule for 6 players playing over a 32 week schedule where no player repeats with a partner.
Answered by Victoria West.
Volume of concrete 2011-04-03
From john:
I have 1200 square foot that i need to fill at 4 inches deep. how many cubic yards of concrete will i need? and how do I figure this?
Answered by Penny Nom.
Designing a tin can 2011-03-31
From Tina:
A tin can is to have a given capacity. Find the ratio of the height to diameter if the amount of tin ( total surface area) is a minimum.
Answered by Penny Nom.
The rectangle is reduced to 90% of its original size 2011-03-31
From stephon:
A rectangle has a length of 4cm and a width of 8cm.

If this rectangle is reduced to 90% of its original size what will be the new length

Answered by Penny Nom.
[(90+36-4) ÷ 2] x 15 = 2011-03-30
From ken:
[(90+36-4) ÷ 2] x 15 =
Answered by Penny Nom.
A transversal 2011-03-30
From Jillian:
I don't understand finding angels in a transversal
Answered by Penny Nom.
sin x = -0.25 2011-03-29
From Wayne:
How do you solve for x in the equation sin x = -0.25

the answer is 3.394 and 6.030 but I don't know the steps they used to calculate this

Answered by Penny Nom.
A circle in a square in a circle in a square 2011-03-29
From George:
A circle within a square which is inside a larger circle which is also within a square. (a circle in a square inside a circle in a square) Equation of the smaller circle is: x ^ 2 x y ^ 2 = 25. What are the dimensions of the larger square? Been 40 years, trying to help my son.
Answered by Penny Nom.
Two boats 2011-03-29
From Mamie:
Two speed boats leave the port of San Francisco traveling in opposite directions at the same time.One boat travels 8 knots per hour faster than the other.After one day's travel they are 1,920 nautical miles apart.What are the speeds of the two boats.
Answered by Penny Nom.
Linear inequalities 2011-03-25
From Alice:
OK. I'm in algebra 1 and the linear inequality is
y<2x-1
y>2
I don't completely know how to graph y>2 and I'm getting really frustrated
Please help me in any way possible!
Thank You!

Answered by Penny Nom.
A stone is dropped into a lake 2011-03-24
From AnneMarie:
A stone is dropped into a lake, creating a circular ripple that travels outward at a speed of 25 cm/s. Find the rate at which the area within the circle is increasing after 4s.
Answered by Penny Nom.
Write 3 1/5 in decimal form 2011-03-24
From Stephenie:
converting fractions????

3 1/5 into decimal???

Answered by Penny Nom.
The number of points on a line is equivalent to that of a surface 2011-03-24
From Gary:
I I was reading about how the number of points on a line is equivalent to that of a surface. This was done by taking any point on a line then taking alternating digits and making them as points on an x and y axis therefore points on a surface.The problem is as i see it if you just take a line then hold it over a surface you have just put the points on the line in a one to one correspondence with the points directly under it on the surface.Now you have all the rest of the surface which cannot be mapped onto the line since it is already used up.What am i missing?
Answered by Penny Nom.
A true or false trig question 2011-03-24
From Abeth:
True or False: Since cot (theta) = cos (theta)/sin (theta), if cot (theta) = 1/2, then cos (theta) =1 and sin (theta)=2. My answer before was true, but not my answer is false. Can you give me a solution on this matter. thanks.
Answered by Penny Nom.
If 25 items are evenly spread over 7 days 2011-03-23
From michael:
What is the formula to calculate:
If 25 items are evenly spread over 7 days, but one day has 3 items less than the other days, what is the number if items for each of the remaining 6 days?

Answered by Penny Nom.
What is x to the power of 0? 2011-03-23
From Jason:
What is x to the power of 0?
Answered by Robert Dawson.
What does 163,100,000 look written in words? 2011-03-22
From Alicia:
What does 163,100,000 look written in words?
Answered by Penny Nom.
Find his income on the fourth week of August. 2011-03-22
From Abeth:
The average weekly income of MJ for the first three weeks of August was 3,600 and his average weekly income for four weeks is 3950. Find his income on the fourth week of August.
Answered by Penny Nom.
Tony and his friends 2011-03-21
From Mieke:
Using the following statements and the information given, how much money did each friend have.
Mike says, "I have $41.00 more than Tony."
Jessica says, "I have 6 times as much as Tony plus $2.00."
Ashley says, "I have 13 times the amount Tony has."
Sara says, "I have the 8 times the amount Tony has."
The total together they had was $304.00

Answered by Penny Nom.
A verbal problem 2011-03-20
From tom:
mrs. jhonson planned to spend $78 for fabric to make draperies. She found her fabric on sale at 20% less per yard than she expected and was able to buy her drapery fabric plus 4 extra yards for bedspread for $83.20. how much fabric had she planned to buy and what was the original cost per yard?
Answered by Stephen La Rocque.
6(M-1/9)=55/12 2011-03-15
From Colleen:
6(M-1/9)=55/12
Answered by Penny Nom.
How much per ft. does it cost? 2011-03-14
From Linda:
350 lbs. of coil at $1.80 per lb equals $630. each coil is 780 ft. how much per ft. does it cost?
Answered by Penny Nom.
A cubic inch 2011-03-13
From wyatt:
what is a cubic inch?
Answered by Penny Nom.
Three consecutive odd numbers 2011-03-11
From George:
Write an equation three consecutive odd numbers whose sum is 75
Answered by Penny Nom.
Regular and irregular polygons 2011-03-10
From thaera:
the difference between regular shape and irregular shapes.
Answered by Penny Nom.
The interior angles of a pentagon 2011-03-10
From Daima:
I need help!
The interior angle of a pentagon i s5 ( I hope ) or 540 degrees.
The interior angle of a pentagon is what? Explain to me please.

Daima

Answered by Penny Nom.
Fencing a park 2011-03-07
From taniel:
Find the number of feet of fence needed to fence a park that is 1 3/8 mi long and 5/8 wide.
Answered by Penny Nom.
Vertices and sides of a polygon 2011-03-04
From RINA:
My questions is : would a vertices and sides of a polygon be the same number.  For example, a octagon it has 8 points and 8 sides. Am i correct?
Answered by Robert Dawson.
ln(12 x) = 6 2011-03-03
From Carolyn:
solve the equation

ln12x = 6

Thank you

Carolyn

Answered by Penny Nom.
A family of circles 2011-03-01
From steffi:
Find the equation of the family of the circle passing through the the point of intersection of x^2+ y^2 -4x-28=0 and x^2 +y^2 -4x-20+52=0; the member tangent to x=7.
Answered by Penny Nom.
A two digit number 2011-03-01
From jhoanna:
the sum of the digits of a two digit number is 10. The value of the number is 16 times the units digit.Find the number.
Answered by Penny Nom.
Find the dimension of the original rectangle 2011-02-28
From sandra:
the perimeter of a rectangle is 40 cm. if the length were doubled and the width halved, the perimeter would be increased by 16cm. Find the dimension of the original rectangle
Answered by Robert Dawson.
A camera's line of sight 2011-02-26
From MJ:
A rocket that is rising vertically is being tracked by a ground level camera located 3 mi from the point of blast off when the rocket is 2 mi high its speed is 400mph At what rate is the (acute) angle between the horizontal and the camera's line of sight changing
Answered by Penny Nom.
At what rate is the grain pouring from the chute? 2011-02-26
From MJ:
Suppose that grain pouring from a chute forms a conical heap in such a way that the height is always 2/3 the radius of the base. At the moment when the conical heap is 3 m high, its height is rising at the rate of 1/2 m/min. At what rate (in m^3/min) is the grain pouring from the chute?
Answered by Penny Nom.
A word problem 2011-02-26
From Barb:
there are five times as many children as adult at a playground. There are 60 people on the playground.

write an algebra equation that represents this situation. Be sure to define your variables.(start with statement "Let X= ___")

Answered by Penny Nom.
A circle is inscribed inside an isosceles trapezoid 2011-02-25
From priyam:
a circle is inscribed inside an isosceles trapezoid (with parallel sides of length 18 cm and 32 cm) touching all its four sides. find the diameter of the circle. thanks for help!!
Answered by Penny Nom.
Slicing a rectangular parallelepiped 2011-02-23
From tom:
a) Pass a plane containing a vertex of a rectangular parallelepiped and a diagonal of a face not containing that vertex to cut a pyramid from the parallelepiped. What fraction of the volume of the parallelepiped is the volume of the pyramid thus cut off?

b)Pass a plane through a cube of edge 8 in so that the section formed will be a regular hexagon. Through each side of the hexagon pass two planes, one plane containing one of the two vertices of the cube which are farthest away from the plane of the hexagon, the other plane containing the diagonally opposite vertex of the cube. Find the volume of the solid bounded by these planes.

c)three of the edges of a rectangular parallelepiped that meet in a point are also the lateral edges of a pyramid. What fraction of the parallelepiped is this pyramid...?

you know the my only problem is how to illustrate this one...if only i could ...can you help me....all I need is the drawing and I can do the rest ..it's fine if you'll only answer any of these but I'll be thankful if you will do the three :)....

Answered by Penny Nom.
6685 is 67.5% of the total number 2011-02-22
From Donna:
I know 6685 is 67.5%. How do I calculate the total number.
Answered by Penny Nom.
Cutting the top off a conical tent 2011-02-22
From tom:
how far from the top must you cut a conical tent in order to cut the cloth in half...
Answered by Penny Nom.
Triangles with perimeter 16 cm 2011-02-22
From Chong:
How many triangles (up yo congruence) with perimeter 16 cm and whose lengths of its side are integers?
Answered by Chris Fisher.
How much gravel do I need to make it level? 2011-02-22
From Robert:
I have an area that is 55 feet long x 75 feet wide and goes from 0 to 13 inches deep. How much gravel do I need to make it level?
Answered by Penny Nom.
An acre divided into lots 2011-02-18
From charlie:
If you have a square acre an divide it up in 2 foot by 3 foot lots how many lots would there be
Answered by Stephen La Rocque and Penny Nom.
Making a truncated cone 2011-02-18
From lisa:
We need to make a cone that has the following dimensions.
14-3/8" diameter on large end
13-3/8" diameter on small end
4" tall

What are the dimensions I need to cut in order to make a cone with one seam?

Answered by Stephen La Rocque.
Tom has 12 times as much money as Bill 2011-02-18
From Chris:
Tom has 12 times as much money than Bill. Together they have $325 dollars. How much does each boy have?
Answered by Stephen La Rocque.
Two conical tanks 2011-02-17
From rustom:
Two vertical conical tanks (both inverted) have their vertices connected by a short horizontal pipe. One tank, initially full of water, has an altitude of 6 ft. and a diameter of base 7 ft. The other tank, initially empty, has an altitude of 9 ft., and a diameter of base 8 ft. If the water is allowed to flow through the connecting pipe, find the level to which the water will ultimately rise in the empty tank (Neglect the water in the pipe.)
Answered by Penny Nom.
The diagonals of a parallelogram 2011-02-17
From ginger:
Find the diagonals of parallelogram where the the side is 40 and the bottom side is 15.
Answered by Penny Nom.
Points on a graph 2011-02-15
From Maggie:
Which set of ordered pairs can be used to graph y = -2x-1.

A.(-5,9),(1,-3),(4,9)
B. (-2,-5),(1,-1),(5,9)
C.(-2,-5),(0,-1),(4,-3)
D. (-5,9),(-2,3),(4,-9)

I cannot figure it out. Could you help me?

Answered by Penny Nom.
Prove sin x = sin (pi - x) 2011-02-15
From Janet:
Prove sin x = sin (pi - x)
Answered by Penny Nom.
(1-1/2)(2-2/3)(3-3/4)...(15-15/16)=n!/16 2011-02-15
From Fiona:
Could you help me find the value of n: (1-1/2)(2-2/3)(3-3/4)...(15-15/16)=n!/16
Answered by Penny Nom.
What is the 6001st digit that Sam wrote? 2011-02-15
From Sarah:
Sam wrote the natural numbers from 1 to 5280 in order. What is the 6001st digit that Sam wrote?
Answered by Penny Nom.
Factoring with fractions 2011-02-15
From Megan:
Hi there, I'm working on factoring polynomials but this question has me quite puzzled.. im a college student in my first year.

(x^2-a^2/xy)(xy/x+a)

Answered by Penny Nom.
The equation of a circle 2011-02-14
From Cristela:
find the equation and all the information in General Form and Standard Form of the Circle that will passed trough the point (2,3) (6,1) (4,-3)
Answered by Stephen La Rocque.
An irregular hexagon 2011-02-09
From Emma:
Hello, I have math question about how to find the area of an irregular hexagon.
It is shaped like a backwards L and the side lengths are as follows
11cm
8cm
5cm
5cm
6 cm
3cm
Can you please help me find the area of this?
Emma

Answered by Penny Nom.
Two tangent circles 2011-02-09
From xhesika(jessica):
Two circles of radius 10 are tangent to each other.A tangent is drawn from the centre of one of the circles to the second circle.To the nearest integer find the area of the shaded region.
Answered by Penny Nom.
Throwing a football 2011-02-08
From Janet:
a football is thrown into the air from 2 meters high. After 1 second it is 6.9 meters high. After another second it is 2 meters high. How high is it after 1.02 seconds and .05 seconds? when will it reach 5.136 meters? I am not sure how to extract the numbers I need and what equations to use them in?
Answered by Stephen La Rocque.
Reversing the digits 2011-02-08
From rejie:
if the digits are reverse , the number is 9 less than the original number.what is the equation?
Answered by Penny Nom.
Rearrange for x 2011-02-06
From sue:
(x+7)/3 = (3xy+2x)/4 rearrange for x
Answered by Penny Nom.
A tangent to a circle 2011-02-06
From debz:
what is the formulae of the tangent to a circle.... our teacher gave us a lot of homework.. and she ask us to find the formulae by ourselves..
Answered by Penny Nom.
I have nine players, three rounds of golf 2011-02-05
From Pat:
My problem is I have nine players three rounds of golf, please could you give me the best formula so that no two players play twice together
Answered by Victoria West.
What is the height of the building? 2011-02-05
From Myra:
The angle of elevation is 45 degrees. The distance from George's foot to the top of the building is 50m. What is the height of the building?
Answered by Penny Nom.
Calibrating a conical tank 2011-02-05
From Bill:
Hi, I have a round tank with tapered sides where I know the diameter at the top and bottom. Is there a formula I can use to calculate the volume by measuring from the bottom up the side (at the angle of the side) to any given point? Thanks, Bill
Answered by Stephen La Rocque and Penny Nom.
Daily life example of logarithms 2011-02-04
From owais:
what are the application or daily life example of logarithms?
Answered by Harley Weston.
The nearest one and a half and the nearest inch 2011-02-02
From Miranda:
REASON:sarah gave the same answer when asked to round 4 seven eighth inches to the nearest one and a half and the nearest inch. explain why sarah is correct?
Answered by Penny Nom.
A cone with a specific angle 2011-02-01
From John:
Hi my daughter came with a seemingly easy question (which to me it was not) How to make/calculate a cone of a specific angle from top to bottom radius.
Answered by Penny Nom.
1 and four eights inches 2011-02-01
From Miranda:
WRITING TO EXPLAIN:If a line is measured as 1 and four eighth inches long,explain how you could simplify the measurement?
Answered by Penny Nom.
An even multiple of 27 2011-02-01
From parth:
the 6 digit # 63x904 is an even multiple of 27 what is X
Answered by Penny Nom.
A fence around a water tank 2011-02-01
From Heath:
I am building a fence around a water tank. the fence is to be in the shape of a normal octagon. The tank has a circumference of 57 ' 6''. I would like the fence to be 3 ft from the tank at the skinny point . How would I calculate(for the simple guy) where to set each of my 4x4 posts at the 8 corners. Any help would be greatly appreciated.
Answered by Harley Weston.
22 people playing 6 rounds of golf 2011-01-31
From angi:
I'm having a problem scheduling matches for a golf vacation. We have 22 people playing 6 rounds of golf. We know that we will need to have two 3 man pairings each day. Is there a way to make sure that is not a great deal of duplication. I did see how Penny answered a question in 2005 with the circles. Is there a way mathematically to do this or through excel?
Answered by Victoria West.
The area of a rectangle 2011-01-31
From T.S.:
for which value of w does the rectangle below have an area of 864 square units if sides are width=w+15 and length=w-15
Answered by Penny Nom.
A player runs from second base to third base 2011-01-30
From Marie:
A baseball diamond is a square with side 90 feet in length. A player runs from second base to third base at a rate of 18 ft/sec. At what rate is the area of the trapezoidal region, formed by line segments A, B, C, and D changing when D is 22.5
Distance A is the players distance from first base when running from 2nd to third. Distance D is his distance from 3rd base. Distance C is the distance from 3rd to 3rd to Home. Distance B is the distance from Home to First. I have found dA/dt in a previous problem.

Answered by Penny Nom.
The perimeter of a semicircle 2011-01-29
From Keith:
I have the perimeter of a semi circle of 37 but how do I calculate the diameter from this. This is all the information I have.
Answered by Penny Nom.
(7cubed * 8(exponent6))exponent 6 2011-01-29
From Shalaine:
What is the exponential form of (7cubed•8(exponent6))exponent 6?
Answered by Penny Nom.
The sides of an equilateral triangle 2011-01-27
From Cristal:
Hello :) How can i find the sides of an equilateral triangle when only given the height 10m?
Answered by Penny Nom.
Scale factor 2011-01-25
From Mackenzie:
hi my problem is:

a man that is 6 feet tall in real life and only .5 inches in a = photograph. what is the scale factor and scale of the photograph?

another:

a girl in the same photograph is 4.8 feet tall. How tall is she in the = picture?=

Answered by Chris Fisher.
The area of isosceles triangle 2011-01-24
From imraan:
how to find the area of isosceles triangle by knowing only its sides
Answered by Penny Nom.
16 golfers 2011-01-22
From Jack:
I am responsible for setting up groups for 16 golfers for 7 days, is there a formula to calculate playing partners so that everyone gets to play with each person?
Answered by Victoria West.
Multiplication and order 2011-01-21
From Janet:
IN A MULTIPLICATION SENTENCE BASED ON A DRAWING..IS THERE ANY ORDER FOR THE MULTIPLICAND ANG MULTIPLIER FOR THE ANSWER TO RIGHT?
Answered by Robert Dawson.
Four digits are to be randomly selected 2011-01-20
From sarahbear:
Suppose four digits are to be randomly selected (repetitions allowed) Find the probability that
A. 5562 is selected
B.0000 is selected
C. All four digits are the same
D. 2 is the first digit selected

Answered by Penny Nom.
Points with distance 5 from the point (-2, -1) 2011-01-19
From Alexa:
Find all points having a x-coordinate of 2 whose distance from the point (-2, -1) is 5.
Answered by Penny Nom.
The diameter of a cylinder 2011-01-18
From Gayle:
Given a cylindrical container that holds 45 imp. gallons and is 42 inches high, find the diameter
Answered by Penny Nom.
Flipping a cone over 2011-01-17
From Fionna:
The height of your cone is "x". Holding the cone so that the vertex is at the bottom, fill it half full with water. The depth of the water is "1/2x". Put a lid over the top, and flip it over. What is the new depth of the water, now that the cone is flipped?
Answered by Penny Nom.
A rectangle 2011-01-16
From meg:
if a rectangle has a length that is 3cm less than 4 times its width and its perimeter is 19cm, what are the dimension?
Answered by Penny Nom.
The greatest value of 5 – | x – 3| 2011-01-16
From pranav:
What is the greatest value of 5 – | x – 3 | for any integral value of x. --< br /> PRANAVM
Answered by Penny Nom.
A cylindrical tank 2011-01-16
From tom:
A cylindrical tank, 1 ft in diameter, length 6 ft , is placed so that its axis is horizontal. How many pounds of water will be used in filling it to a depth of 9in. , if water weighs 62.4 lbs. per cu. ft.?
Answered by Penny Nom.
16 golfers 2011-01-16
From Kenneth:
Our group will play 12 rounds. We are 16 golfers.
How can I schedule 12 days of differing foursomes limiting repeats in the foursomes

Answered by Victoria West.
2x = 2+2x 2011-01-15
From james:
2x=2+2x can you help me find what x epauls?
Answered by Penny Nom.
Matt is thinking of two numbers 2011-01-14
From mary:
Matt is thinking of two numbers. One number is 9 more than the other number. The sum of the numbers is 45. What are the numbers?
Answered by Penny Nom.
Integrating ln^3x/x 2011-01-14
From ken:
y=ln^3x/x from x=1 to x=11
Answered by Penny Nom.
The volume of a smokestack 2011-01-13
From tom:
The outer protective smokestack of a steamship is streamlined so that it has uniform oval section parallel to the deck. The area of this oval section is 48 sq. ft . If the length of the stack is 15 ft. and the stack is raked aft so that its axis at its upper end is horizontally 4.2 ft from the lower end, find the volume enclosed by the stack.
Answered by Penny Nom.
The height of a lamp 2011-01-13
From Dorothy:
I need to find the lamp height with casting shadow (base line of triangle where a boy 1.6m tall stands 3m from base of street lamp and has a 2m shadow. In other words, think of a right angle triangle with zero height starting at left, then 2m to right stands boy (1.6m high). Angle (hypotenuse) increases up to top of street lamp with 'x' height and 3m base.
Answered by Penny Nom.
The angle between a line and a plane 2011-01-12
From tom:
what are the angles of the diagonal of a rectangular parallelepiped 2in by 3 in by 4 in makes with the faces...
You know this is a problem that I can't figure out ...I don't know where the angle and the diagonals where? can you help me with this one?

Answered by Penny Nom.
The weight of a tank 2011-01-11
From tom:
A tank, open at the top is made up of thin sheet iron 1in. thick. The internal dimensions of the tank are 4 ft. 8in. in long ;3 ft. 6 in.in wide ;4ft 4 in. in deep Find the weight of the tank when empty, and find the weight when full of salt water. (Salt water weighs 64 lbs. per cu. ft., and iron is 7.2 times as heavy as salt water)
Answered by Harley Weston.
Identify each polynomial by its degree and number of terms 2011-01-10
From betty:
Write in standard form and identify each polynomial by its degree and number of terms. How do you do this?
Answered by Penny Nom.
Set notation 2011-01-10
From Ron:
The question asked in a handout:

8k - 5 < 35, keI

I believe the answer to k is 5. But what does the k e(E) I mean in this?

Answered by Harley Weston.
Semicircles and the Pythagorean Theorem 2011-01-09
From Jas:
Okay well, in math we are learning about the pythagorean theorem and we have to do a math journal on the question:

****Can you replace the squares (that are put on the sides of a right triangle) with semicircles and still get the same answer??

I do not understand because i tried doing an example and comparing it with a normal way of doing it and I didnt get the same answer!

Answered by Penny Nom.
Tiling a swimming pool 2011-01-09
From rustom:
(a) Find the volume of water in swimming pool with vertical ends and sides . The length measured at the water line is 50 ft. and the breadth is 20 ft. The bottom of the swimming pool is a plane sloping gradually downward so that the depth of the water at one end is 4 ft. and 8ft. at the other end. (b) If the sides, ends, and bottom of the swimming pool are constructed of tile blocks whose glaze surface dimensions are 3in by 6in. , and if the ends and sides of the pool extend 2ft.above the water level, find the number of blocks used if 1/20 of the surface area is covered by sealing material.

I got the (a) question but I don't know the (b) question which have the answer of 16,136 blocks. I hope I can get the procedure for this, THANK YOU!

Answered by Penny Nom.
Tan(x+pi)-tan(pi-x)=2tan(x). 2011-01-09
From Steven:
Verify the identity:
Tan(x+pi)-tan(pi-x)=2tan(x)

Answered by Penny Nom.
10 golfer playing 5 rounds 2011-01-09
From Jeff:
need pairing for 10 golfer playing 5 rounds of golf.
Answered by Victoria West.
A 400 gallon oil tank 2011-01-08
From theresa:
we have a oil tank that holds 400 gallons of oil currently there is 11/16 volume in the tank how many gallons are there?
Answered by Harley Weston.
A sector of a circle 2011-01-07
From Alice:
the radius of a circle is 7cm the angle of the sector is 68 how do you find the
Answered by Penny Nom.
What is its speed in miles per hour? 2011-01-06
From Miriam:
An airliner travels 125miles in 15minutes. What is its speed in miles per hour?
Answered by Penny Nom.
Rondo the snail 2011-01-05
From Kristen:
He trained his snail, Rondo, to crawl around a 6 x 8 rectangle. Rondo stayed exactly 3 cm from teh rectangle at all times. What is the total area that Rondo surrounded with his slime trail?
Answered by Penny Nom.
A golf holiday 2011-01-04
From Andy:
You kindly responded to a question I previously sent with regard to a Golf Holiday and I wonder if I might ask another question please. There are now 7 players playing 5 rounds of golf and I wonder if it is possible to devise a schedule where everyone plays with each other a minimum of two times i.e. they will play twice with some and three times with others. As always, many thanks for your kind assistance.
Answered by Victoria West.
Line segments 2011-01-04
From Izsak:
What is the formula to find the number of segments that can be named by a given number of points on a line?
Answered by Penny Nom.
A tangent line 2011-01-03
From Amanda:

Question from Amanda, a student:

an equation of the line tangent to y=x^3+3x^2+2 at its point of inflection is
(A) y=-6x-6
(B) y= -3x+1
(C) y= 2x+10
(D) y=3x-1
(E) y=4x+1


Answered by Penny Nom.
Exponential form 2011-01-02
From jumana:
find any five examples of exponential form from real life situation
Answered by Penny Nom.
A circle inside a square 2010-12-31
From Jenn:
A circle of a diameter of 2 is drawn inside a square of 4. The circle's center is at the center of the square. To the nearest tenth, what is the perimeter of the area of the square, not including the circle inside.
Answered by Penny Nom.
The third side of a triangle 2010-12-28
From andrea:
in triangle abc, a-b=5 and the area is 50cm2.If angle c- 90degrees, find the length of c.
Answered by Penny Nom.
A polynomial remainder 2010-12-25
From Asterdine:
When a polynomial f(x) is divided by x+1 and x+2, the remainders are 3 and 7 respectively. Find the remainder when f(x) is divided by (x+2)(x+1).
Answered by Penny Nom.
The thermal expansion of steel 2010-12-20
From roger:
Knowing that the coefficient of thermal expansion of steel is 6.5E-06 in/in/deg F. How do you calculate the loads applied as a result of the expansion?
Answered by Robert Dawson.
An air duct in the form of a circular cylinder 2010-12-19
From ed:
an air duct in the form of a circular cylinder has a cross section if diameter 16 in. the distance between the bases is 20 ft and the elements are inclined at an angle of 50 degrees to the bases. find the amount of magnesia required to protect the duct with magnesia covering 1/2 in thick? tnx
Answered by Stephen La Rocque.
Golf pairings 2010-12-17
From AL:
I have a golf tourney where we have fourteen players and are playing five times/rounds of golf. I am trying to pair up two threesomes and two foursomes so we don't play with the same person twice or the least amount of times.
Answered by Victoria West.
Draw a figure with 16 triangles using only 6 line segements 2010-12-16
From Jill:
How can you draw a figure with 16 triangles using only 6 line segements?
Answered by Stephen La Rocque.
The height of an iceberg 2010-12-16
From Sharon:
if an iceberg can be seen 10% or 70 meters above the water, what is the measurement in meters of the iceberg below the surface of the water?
Answered by Robert Dawson and Penny Nom.
The perimeter of a quarter circle 2010-12-15
From kim:
how do you figure the perimeter of a quarter circle that has a radius of 7 inches? I would like to have the explanation of how you figure it out. Thank you
Answered by Penny Nom.
Area and volume of a parallelepiped 2010-12-14
From ryan:
Got a question, I'm really confused what the question mean and how the diagram would look like... someone please help... Here's the question.

The base of a parallelepiped is a rectangle 4m by 6m. If its lateral edge is 8m and is inclined at an angle 45 degrees to a 6m edge of the base, find the total area and volume of its parallelepiped.

My dilemma is what does it mean by the lateral edge of 8m is inclined at 45 degrees to a 6m edge of the base? Can someone post a diagram to help me solve the promble. Thanks in advance.

Answered by Chris Fisher.
Ordering fractions 2010-12-14
From serita:
1/2,2/4,4/8 how do you do least to greatest
Answered by Penny Nom.
An inequality 2010-12-13
From jimmy:
1.7b-1.1 <2.3
Answered by Stephen La Rocque.
Angular speed 2010-12-12
From Jason:
7 in. pulley traveling @ 175ft/sec. What is the rpm?
Answered by Stephen La Rocque and Penny Nom.
A railway cut 2010-12-12
From jade:
the pennsylvania railroad found it necessary,owing to land slides upon the roadbed,to reduce the angle of inclination of one bank of certain railway cut near pittsburgh,pa.,from an original angle of 45 degrees to a new angle of 30 degrees. the bank as it originally stood was 200 ft.long and had a slant length of 60 ft.. find the amount of the earth removed, if the top level of the bank remained unchanged.
Answered by Penny Nom.
Angle of elevation 2010-12-10
From PANKAJ:
angle of elevation of the sun perpendicular72 and base 88 find angle
Answered by Penny Nom.
The length is twice its breadth 2010-12-09
From jenu:
The length of the rectangular field is twice its breadth. A man jogged around it 5 times and covered a distance of 3 km. what is the length of the field?
Answered by Penny Nom.
80 miles in 40 minutes 2010-12-08
From james:
i travel 8o miles in 40 minutes. how fast am i going in mph
Answered by Penny Nom.
A rectangular solid 2010-12-08
From ryan:
Good day! Have a question on rectangular solids. We were asked to find the dimensions of a rectangular solid, we were only given the volume of the rectangular solid, the total area, and the altitude. I tried using the volume of the rectangular solid and was able to get the area of the base, but I don't know how to get its length and its width. Hope you could help me again. Thanks!
Answered by Penny Nom.
Sales as a function of advertising 2010-12-08
From Adori:
The sales S(in thousands if units) of a product after x hundred dollars is spent on advertising is given by S=10(1-e^kx). Find S as a function of x if 2500 units are sold when $500 is spent on advertising.
Answered by Penny Nom.
A water tank has the shape of a right circular cone 2010-12-07
From mike:
A water tank has the shape of a right circular cone with height 12 feet and radius 8 feet. Water is running into the tank so that the radius r (in feet) of the surface of the water is given by r=0.75t where t is the time (in minutes) that the water has been running. the volume V of the water is given by V=1/3 pi r^2h. Find V(t) and use it to determine the volume of the water when t=5 minutes.
Answered by Penny Nom.
A television camera at a basketball court 2010-12-06
From ryan:
Can you just please give me an idea on how the diagram would look like. I'm really confused.

A television camera is 30 ft. from the sideline of a basketball court 94 ft long. The camera is located 7 ft from the midcourt. Through what angle must it sweep in order to cover all action on the court?

Answered by Penny Nom.
Simultaneous equations 2010-12-05
From ryan:

Question from ryan, a student:

3         4
--   -    --      =     1      (1)
x         y

7       2             11
--   -  --        =   --       (2)
x       y              12


Answered by Chris Fisher and Stephen La Rocque.
Fifty votes were cast in a class election 2010-12-03
From nympha:
Fifty votes were cast in class election, Beth got 1/5 of the votes. Helen got as many votes as Jane and Peter put it together. Peter got 1/3 as much votes as Jane. How much votes did each of the four candidates received?
Answered by Stephen La Rocque.
Angular speed 2010-12-03
From Julie:
If the angular speed of a circle is 360 degrees divided by the time it took to rotate, how do I calculate ones that have greater rotations than the amount of time? I mean, you calculate the angular speed of a merry-go-around that spins once every 5 seconds by doing 360degrees/5 sec, but how do you calculate a merry-go-around that has 16 rotations every 4 seconds?
Answered by Penny Nom.
Simplify the logarithmic expression 2010-12-02
From Adori:
Use the properties of logarithms to simplify the logarithmic expression. Log(1/250) base 5
Answered by Penny Nom.
Two chords in a circle 2010-12-02
From girma:
one chord of a circle is 8cm long and it's distance from the center is 4cm long.what will be the length of another chord, of the same circle ,which is 2cm from the center
Answered by Penny Nom.
Find all the roots 2010-12-02
From gagan:
find all the roots of z^5-3z^4+2z^3+z^2-3z+2
Answered by Stephen La Rocque and Penny Nom.
A normal distribution problem 2010-12-02
From Racquel:
I am stuck on this question. I am not sure if using the z score will help me get the answer I need. Here is the question?

The average length of time per week that students at this university spend on homework is normally distributed with a mean of 18 hours and a standard deviation of 3 hours. If Diane spends more time on homework each week than 75% of students, what is the minimum time she must spend?

Answered by Penny Nom.
How many gallons of water will it take to fill a classroom? 2010-12-01
From deena:
How many gallons of water will it take to fill a classroom that is 26 feet long by 19 feet wide and 9 feet high? How many tons of water will that be?
Answered by Stephen La Rocque.
Can determine if it is scalene, isosceles, or equilateral 2010-12-01
From Jessie:
find the measures of the sides of triangle KPL and classify each triangle by its sides. my first problem would be K(-3,2) P(2,1) L(-2,-3) ...The three points they give you are the vertices of the triangle and you need to match them up. Draw the triangle and write in the vertices and the related point with the vertex. You will then do the distance formula three times to find the distance of all three sides. Once you have the three sides you can determine if it is scalene, isosceles, or equilateral...using the distance formula how do i solve this?
Answered by Penny Nom.
Linear inequalities with absolute values 2010-12-01
From ryan:
Good day!
We we're given this assignment in algebra concerning linear inequalities with absolute value. In this number I'm confused how to isolate the absolute value, considering there is a variable in the absolute value and there is a variable in the outside.

2+5 |4-3t|+t ≤ -3-4t

Answered by Penny Nom.
A rectangular piece of plywood 2010-11-30
From Austin:
A rectangular piece of plywood is trimmed to make it square by cutting a 4cm strip off the top and a 2cm strip off one side. If the area of the original piece is 74cm2 greater than the area of the square, find the dimensions of the rectangle.
Answered by Penny Nom.
6 rounds of golf 2010-11-29
From Andy:
I wonder if you are able to assist me with scheduling pairings for a forthcoming Golf Tournament ?

There are 7 players and we will be playing 6 rounds - each round will be a 3 ball and a 4 ball. I am trying to work out a schedule that means everyone plays with each other roughly the same number of times. Also, we are looking to ensure everyone plays in roughly the same number of 3 and 4 balls. Any suggestions would be much appreciated - thank you.

Answered by Chris Fisher.
A punch-code lock box 2010-11-29
From Doug:
I am a real estate agent that uses punch-code lock boxes on houses to store the keys. I need to crack the code on one of our boxes, so I need a list of the possible combinations.
Rules:
4 digit code
Numbers 0-9
Each number can only be used once
Order of the numbers does not matter.

With advice from this site, I have calculated that there should be 210 possibilities. 10x9x8x7 (5040) gives me the total number possibilities and 4x3x2x1 (24) gives me the total number of ways to arrange those numbers. So if I divide the possibilities by the arrangement, I get 210. (5040 / 24 = 210) GREAT!!!

Now, how do I develop a list of those 210 combinations?

Thanks for your help.

Answered by Robert Dawson.
A binomial random variable 2010-11-29
From yvette:
a binomial random variable has a mean equal to 200 and standard deviation of 10. find the values of n and p.
Answered by Penny Nom.
A sector of a circle 2010-11-29
From Mel:
Find the area of a sector of a circle that has a central angle of 13pi/18 and a radius of 12cm. Round answer to nearest 10th degree
Answered by Penny Nom.
What is its speed in still air? 2010-11-28
From Shelby:
An Airplane flies a certain distance in 3 hours going WITH a 40km/hr wind. It travels the same distance in 5 hours when going against the wind. What is it's speed and how long would it take to travel the same distance in still air? Distance = speed x time
Answered by Penny Nom.
What is the 5% GST 2010-11-27
From Louise:
I am doing our business books and could you please tell me how to figure out how to calculate what the 5% GST would be on a total. Such as in example.....$78.79 total figure.....what would be the GST portion. How do you figure it out. ??
Thanks

Answered by Stephen La Rocque.
Sides and diagonals of a polygon 2010-11-25
From nicky:
The sum of sides of two polygons is 12 and the sum of their diagonals is 19. What kind of polygon are they?
Answered by Penny Nom.
A man made circular lake 2010-11-25
From ailish:
A man made circular lake has a diameter of 338 m. A bridge is to be constructed across the lake in such a way that it is 119 m away from the center of the lake. How long is the bridge?
Answered by Penny Nom.
The perimeter of a rectangle 2010-11-24
From shericka:
a rectangle has a perimeter of 60cm. its length is 5 times greater than its width. what is its length?
Answered by Penny Nom.
How long will you and your friend be able to talk? 2010-11-23
From beket:
you and your friend leave your home town at the same time your friend travels by train 45 mph directly east while you travel by train 50 mph going directly south you are talking on cell phone but you plan charges extra if the phones are 100 miles apart. How long will you and your friend be able to talk before the charges increase?
Answered by Penny Nom.
Two girls agree to mow a lawn 2010-11-23
From Milorad:
Two girls agree to mow a lawn 60m by 80m so that the first girl cuts one half by cutting a uniform strip around the lawn. How wide a strip should the first girl cut?

I've been struggling with the question above, mainly with the equation and I have set it up as 2400=(60-2x)(80-2x), if this is incorrect I would like to know where I've gone wrong and also would like a full answer to see it properly done. I need an algebraic answer and not one with a diagram.

Answered by Penny Nom.
i^i 2010-11-21
From trale:
Can we use e^ix=cosx+isinx for finding i^i like that: x= pi/2 => e^(ipi/2)=0+i then [e^(ipi/2)]^i=i^i.then we find i^i= 0,207879576.... is it true? can we give value for x for free?thank you.
Answered by Harley Weston.
What is the largest prime number? 2010-11-20
From vadali:
what is the largest prime number?
Answered by Penny Nom.
A product of 1 000 000 2010-11-19
From Sharron:
There are many pairs of numbers that when multiplied together give a product of 1 000 000. However, there is one pair, and only one pair, of whole numbers that contain no zero digits and has a product of 1 000 000. What are these two numbers?
Answered by Stephen La Rocque.
A 35% profit 2010-11-19
From Ana:
You have a stamp collection and make a 35% profit by selling it on E-Bay for $2700. Find the original cost of the collection by using the formula:New value × 100 / 100 + percent change = Original value
Answered by Stephen La Rocque.
A 20" octagon 2010-11-19
From CHARLES:
I have a 20" octagon and it has 8 parts. In inches how big is each part. I was told each part was 5.86".The top of the octagon breaks into 3 equal parts don't equal 20".When i did my math i thought it should be 6 5/8" which is closer to 20"
I am trying to cut a octagon out of a 20"circle

Answered by Stephen La Rocque.
Intersecting polygons 2010-11-17
From Laura:
How can I estimate the amount of intersection (area or better a normalized number) between two polygons if I have the cartesian coordinates of their vertices?

Thank you in advance,
regards,
Laura

Answered by Robert Dawson.
A scale drawing 2010-11-17
From chandler:
A parking lot 200 meters wide; 1 centimeter:25 meters, find the length on the scale drawing and the scale factor
Answered by Penny Nom.
Simple interest 2010-11-17
From Abeth:
At what simple interest rate will a sum of money triple itself in 5 years time?
Answered by Penny Nom.
Will the ball clear the tree? 2010-11-14
From MK:
Sam hits a golf ball with a five-iron a distance of 120m horizontally. A tree 45m high and 35m in front of Sam is directly in the path of the ball. Will the ball clear the tree if the ball makes a parabolic curve and has a maximum height or 80m?
Answered by Brennan Yaremko.
What numbers can I be? 2010-11-13
From john:
my thousands digit is 2 times my ones digit.my hundreds digit is my ones digit plus 2. my tens digit is the difference between my thousands and hundreds digit what numbers can i be?
Answered by Penny Nom.
The perimeter of a semicircle 2010-11-11
From gayathri:
the area of semicircle is 1925sq.cm find perimeter
Answered by Stephen La Rocque.
The area of an equilateral triangle 2010-11-11
From ginny:
If a equilateral triangle has side lengths of 41.33m how would i calculate the area? if i don't know the height of the triangle?
thank you.

Answered by Penny Nom.
Three consecutive odd integers 2010-11-07
From karen:
find three consecutive odd integers such that twice the sum of the second and the third is 43 more that three times the first
Answered by Penny Nom.
z^5 - 3z^4 + 2z^3 + z^2 - 3z + 2 2010-11-06
From Kumar:
would you please solve this problem, related to complex numbers.

Find all the roots of :

z^5 - 3z^4 + 2z^3 + z^2 - 3z + 2

Answered by Robert Dawson and Penny Nom.
Nickels as dimes 2010-11-06
From Exie:
Clara has the same number of nickels as dimes in each jar. there are 15 cent in one jar and 30 cents in the other. how many of each coin does she have in each jar? I'm home schooling my granddaughter and this problem has me stumped..could you please help? thanks
Answered by Robert Dawson.
Bbippities, bops, and boos 2010-11-05
From Stacey:
six bippities equal three bops,and six bops equal two boos. How many boos are there in one bippity,one bop, and one boo??
Answered by Stephen La Rocque.
Nickels and dimes 2010-11-05
From kebede:
John has 20 fewer nickels than dimes. The total of the nickels and dimes if $12.35. ow many of each coin does she have?
Answered by Penny Nom.
Terminal zeros 2010-11-04
From morgan:
if I have to multiply 1*2*3*4*5*6*7*8*9*10*11*12*13*14*15*16*17*18*19*20 how many terminal zeros do i get?
Answered by Penny Nom.
Four tires and a spare 2010-11-04
From stacey:
a car is driven 40,000 miles using four tires and a spare. the tires are rotated so that each tire travels the same number of miles. What is the number of miles traveled by each tire??
HINT: the answer is not 8,000 miles each..

Answered by Penny Nom.
A 400 gallon drum 2010-11-04
From Jerry:
Question from Jerry:

I want to build A tank. able to hold 400 gallons of asphalt sealer it will be round . basically like A old fuel oil drum but made with heavy material . math ? was I want tank to hold 400 gallons I think the length of 60" would be perfect. I don't know what diameter or radius of tank needs to be . to make it A 400 Gallon tank

Answered by Harley Weston.
How far must the pitcher travel to get to the ball? 2010-11-04
From ken:
A baseball player bunts a ball down the first base line. It rolls 35ft at an angle of 26 degrees with the first base path. The pitchers mound is 60.5 ft from the plate. How far must he travel to get to the ball.
Answered by Penny Nom.
What is the speed of each car? 2010-11-04
From Jennifer:
two cars are 420 miles apart and traveling towards each other along the same road. They meet in 3.5 hours. One car is traveling 15mph slower than the other. What is the speed of each car? This must be solved using a system of equation, I have no idea?
Answered by Stephen La Rocque.
The sides of a trapezoid 2010-11-03
From Bobbi:
Isosceles Trapezoid base=60”; two 45⁰angles at base, two 135⁰ angles at the top, vertical distance between base & top=16”. What is the length of the top line and the two equal side lines?
Answered by Penny Nom.
The weight of some dirt 2010-11-03
From Maggie:
I am wanting to know how many tons of dirt we r going to need to fill a basement. The measurements we r going with r 15ftx15ftx8ft.
Answered by Penny Nom.
39-4+6 divided by 3 to the power of 2 = 3 2010-11-02
From Emma:
Help me and my family cannot figure this out, it the last question on my paper. I need to know how to use place the brackets to make the following problem true.

39-4+6 divided by 3 to the power of 2 = 3

we have tried for three days and cannot get it.......help!

Answered by Robert Dawson and Claude Tardif.
A fourth degree polynomial function with real coefficients 2010-10-30
From Ryan:
Question from Ryan, a student: What is the fourth degree polynomial function with real coefficients that has -1,1 and I as zeros and such that f(x)=160
Answered by Harley Weston.
The slope of a hole 2010-10-28
From William:
What is the slope of a hole that is 7.4 mm in dia by .5mm in depth.
Answered by Penny Nom.
Measuring the heights of tall objects 2010-10-28
From Inder:
what is used in real world to measure heights of tall objects? besides Clinometer or shadows, what else is their that is used to measure tall objects.
Answered by Robert Dawson.
A box full of oranges 2010-10-28
From Mary:
A box full of oranges weighs 9 pounds. After a family ate 2/3 of the oranges, the remaining oranges and the box weighed 4 pounds. The weight of the box when empty is how many pounds?
Answered by Penny Nom.
A train passes a car 2010-10-24
From Mary:
A train, whose speed is 75 mph, takes 3 minutes to pass a car that is going 60 mph in the same direction. How long is the train?
Answered by Stephen La Rocque.
The base of an isosceles triangle 2010-10-24
From Brian:
how can I find the base of an isosceles triangle from the height and the perimeter?
Answered by Penny Nom.
Convert to multiplication 2010-10-23
From angelina:
how to covert 5+5+5=15 to multiplication
Answered by Penny Nom.
A polynomial with integer coefficients 2010-10-23
From Adori:
Question from Adori, a student: Find a polynomial with integer coefficients that has the given zeros: 2/3, 4, root of 3i, root of -3i.
Answered by Harley Weston.
A water trough 2010-10-22
From Jasmine:
A water trough is 8 m long and its cross-section is an isosceles trapezoid which is 90 cm wide at the bottom and 120 cm wide at the top, and the height is 30 cm. The trough is not full. Give an expression for V , the volume of water in the trough in cm^3, when the depth of the water is d cm.
Answered by Stephen La Rocque.
A rectangle with rounded corners 2010-10-21
From Connie:
How do you find surface area if the rectangle is 100ft. by 200ft. with quarter circles in the corners with a radius of 20ft. ?
Answered by Penny Nom.
What are the speeds of the boats? 2010-10-21
From Adori:
Two boats leave the port of San Francisco traveling in opposite directions at the same time. One boat travels 8 knots per hour faster than the other. After one day's travel they are 1920 nautical miles apart. What are the speeds of the boats?
Answered by Penny Nom.
John and Mary are 200 miles apart 2010-10-20
From Sarah:
two people, john and mary, are 200 miles apart and leave at the same time to meet. They are driving toward each other at constant rate on the same straight road. John is traveling at 45 mph and Mary is traveling at 35 mph. When they meet, how many miles will they be from john's house?
Answered by Penny Nom.
Cleaning a ditch 2010-10-20
From bill:
how many yards of dirt will be removed from a ditch 200 feet long 6 feet wide at top x 3 feet wide at bottom and 4 foot slopes
Answered by Penny Nom.
Powers 2010-10-20
From dylan:
how do you write 20736 in exponential form .same for 1728 and 50625.

is there a formula to figure out how to express large know numbers in exponential form.

Answered by Penny Nom.
The equation of a circle 2010-10-20
From Silvan:
Hi, I just want to find the x,y values for the circumference of a circle...

Lets take a clock having its centre at (0,0) in a graph. I just want to know how to find the (x,y) co-ordinates for the curved path or the surface of the circle.. Is there any formula to directly align the curved path or the circumference of the circle in a graph for a known radius of a circle..

I feel it will be useful for me to draw a clock in a graph... :-)

Answered by Penny Nom.
An exponential equation 2010-10-19
From Sandi:

Question from sandi, a student:

I have several questions similar to this one and was wondering if you could walk me through this one. I'm totally lost on how to do it.
Paramecia reproduce by splitting in two. In a laboratory flask, a colony of paramecia had an initial population of 500. Each day, the population of the paramecia was counted. The results are as listed.
Time (in days)------Population
0-----------------------500
1-----------------------600
2-----------------------720
3-----------------------864
4----------------------1037
5---------------------1244
6---------------------1493
7----------------------1792
8---------------------2150
1.)Using graphing calculator make a scatter plot of the data in table.
I think I did this part right I set my window at Xmin=0 Xmax=10 Xscl=1 Ymin=0 Ymax=2500 Yscl=100 Xres=1
2.) Determine an exponential equation to represent the population as a function of time without using a graphing calculator.I have no clue how to do this.
3.)Suppose the flask and food supply is large enough to support the trend of the population growth. Estimate the population of the colony when the time is 10 days.


Answered by Penny Nom.
The equation of a line 2010-10-19
From Sava:
I have to write an equation and sketch a graph for the line that meets the given conditions:

A line with slope -15/5 that passes through the point (-2.5,4.5)

Answered by Penny Nom.
A scale factor 2010-10-17
From david:
We have to rectangles with measurements and we need to find the scale factor. The rectangles are W=6 L=4 for first and second is W=4 and L=3. thanks for your help
Answered by Penny Nom.
An algebra problem with fractions 2010-10-16
From Morgan:
Hi my math teacher has given us this question for our grade nine math class and I always get confused about multiple x algebra, could you help me?

6x+4        3x+2
------   =  ------
  3               5



I have no problems until it comes to combining integers, or x's

Answered by Penny Nom.
The angles in an m-gon and genrealizations 2010-10-16
From Michael:
Hello: In answer to a student's question, someone named Penny from your organization provided a proof that the sum of the interior angles of a triangle in the plane is pi radians (or 180 degrees).

I am interested (and I'm sure many other people would be too) in 3 potential generalizations of this basic fact in plane geometry:

Answered by Walter Whiteley.
Geocaching 2010-10-15
From Clint:
I am stuck on a puzzle for Geocaching which is a GPS game. I know it is a number matrix but don't know how to start.

[1 -7 16 -3 -9; 1 -3 18 -20 -33] x [1 8 -9 3 5; 0 1 0 4 2; 0 0 1 2 1; 0 0 0 1 -1; 0 0 0 0 1] = ?

Answered by Stephen La Rocque.
A linear system 2010-10-15
From Sandy:
HARD!! algebra (Linear Systems) question? An Airplane cruises at 120km/h in still air. One day, when the wind was blowing steady from the west, the airplane travelled west (flying into the wind) and landed at its desination after 1.5h. The airplane's return trip (flying with the wind) was only 1.0 h long. On both legs, the airplane travelled at its cruising speed. (meausred with respect to the air surrounding the airplane).

Part A
Write a System of equations that would allow you to determine the speed of the wind blowing that day and the distance travelled each way

Part B
When solving the system of equations graphically, what equations would you enter into the equation editior (to make graphs on your calculator)

Part C
Indicate what the X and the Y represent

Answered by Stephen La Rocque.
Two jets leave denver at 9 am. 2010-10-15
From Carl:
two jets leave denver at 9 am. One flying east at a speed of 50 km per hour greater than the other which is traveling west. At 11 am the planes are 2500 km apart. Find their speeds.
Answered by Stephen La Rocque.
Simplifying fractions 2010-10-13
From Alice:
14times (9divied3) + 12
______________________
18 divied (26-24)

don't get this.......

Answered by Penny Nom.
The volume of a truncated pyramid 2010-10-12
From A:
what is the formula to find out volume of a pyramid with individual length and breadth? For example bottom size is 6.5m x 6.5m, top size 1.5m x 4.2m and height 0.8m. please send me the formula.
Answered by Stephen La Rocque.
30,000 US gallons of water 2010-10-12
From mike:
If a swimming pool can hold 30,000 US gallons of water - is it possible to calculate the number of yards of dirt it would take to fill the hole?
Answered by Penny Nom.
Cost of a driveway 2010-10-11
From arlene:
What is the total cost of a driveway that is 25' wide and 45' long with 4" thickness if the concreat cost 92.00 per cubic yard and labor is 70.00 a cubic yard?
Answered by Stephen La Rocque.
Electronic cars around a circular track 2010-10-11
From Taylor:
Cory and Melissa are racing electronic cars around a circular track. They begin at the same time going in the same direction. Cory's car completes a revolution in 35 secs, while Melissa's car completes a revolution in 30 secs. How long will it take them before both cars reach the starting point again simultaneously?
Answered by Stephen La Rocque and Penny Nom.
What is the maximum weekly profit? 2010-10-10
From Joe:
A local artist sells her portraits at the Eaton Mall. Each portrait sells for $20 and she sells an average of 30 per week. In order to increase her revenue, she wants to raise her price. But she will lose one sale for every dollar increase in price. If expenses are $10 per portrait, what price should be set to maximize the weekly profits? What is the maximum weekly profit?
Answered by Stephen La Rocque and Penny Nom.
x/200+x/400+x/600+x/800 2010-10-08
From Ashishthombre:
step by step LCM of x/200+x/400+x/600+x/800
Answered by Penny Nom.
2x − 1 ( 1 − 2x) = 1 ( 2x − 2x) 2010-10-08
From MC:
Solve 2x − 1 ( 1 − 2x) = 1 ( 2x − 2x) for x

I am befuddled! Thank you for any explanation you can give that show me the steps. I know the order in which I should tackle it, but am so confused regarding how to assume x. Thank you!

Answered by Penny Nom.
Tiles on a bedroom floor 2010-10-07
From Rochelle:
John's bedroom floor is square in shape. He used 625 tiles, with a side length of 200 mm, to tile the whole floor. Calculate the area and dimensions of the bedroom
Answered by Penny Nom.
Exponential form 2010-10-06
From celina:
what is the exponential form of 4x*4x
Answered by Penny Nom.
Two cars 2010-09-30
From Amanda:
Two cars, 142 miles apart, start moving towards each other at the same time. One is moving 3 times as fast as the other. If they meet 1.7 hrs later, find the average speed of the slower car in miles per hour?
Answered by Penny Nom.
Rectangular arrays 2010-09-29
From Kim:
My 4th grader has started working on rectangular array that shows 2 factors of a number.

The problem reads on centimeter grid paper draw as many arrays as you can for each of the #'s 2,3,4, 5,6,7,,11,12,15 and 16. That would be great except I have no idea what they are talking about and I cannot offer any help to my daughter.

Answered by Penny Nom.
A Taylor polynomial for (lnx)/x 2010-09-29
From Dave:
I have a series problem that I cannot solve. The problem asks for you to compute a Taylor polynomial Tn(x) for f(x) = (lnx)/x. I calculated this poly out to T5(x) and attempted to use this to identify a pattern and create a series in order to calculate Tn(x). However, the coefficients on the numerator out to F5prime(x) are as follows: 1, -3, 11, -50, 274... Ok, so the negative is an easy fix -> (-1)^n-1. But the other coefficients are stumping me. I can't see any sort of pattern there and I've tried every trick I know. Is there another way to go about this? Thanks!
Answered by Chris Fisher.
limit as x approaches a of ((x^(1/2))-(a^(1/2)))/(x-a)? 2010-09-29
From emily:
limit as x approaches a of ((x^(1/2))-(a^(1/2)))/(x-a)?
Answered by Penny Nom.
A limit 2010-09-27
From norma:
I have a problem like this one but I can get it right. please help me to answer find the constant a such that the function is continuous on the entire line.
g(x)= {x^2 - a^2 / x-a if x is not = a
         {6 if x = a

Answered by Penny Nom.
An equation involving logarithms 2010-09-27
From mindy:
Hi,
I am trying to find x, the equation is:
log_2(log x + 2 (log x)^(1/2) + 1) - 2 log_4((log x)^(1/2) + 1) = 1

Thank you very much for your time. =)

Answered by Harley Weston.
4√3 + 2√1/3 2010-09-25
From Michelle:
How do you solve 4√3 + 2√1/3? Please show step by step.
Answered by Penny Nom.
20% profit on the selling price 2010-09-24
From ata:
My cost is 100 and I want to make 20% profit on my selling price (not 20% of the cost). So, what should be calculation formula? Let's say my cost is 493.71 and I want to make 53.28% profit on the selling price; so how to determine the selling price?
Answered by Penny Nom.
The third vertex of a triangle 2010-09-23
From nikos:
Hi, I would really appreciate any help in this:
"Suppose you have a triangle (any triangle, no hypotheses allowed), let's call it ABC. You know the coordinates of the two vertexes, A(x1,y1), B(x2,y2) and the area of the triagnle, let's call it D. Can you find the third vertex of this triangle based on these facts?"

Thanks a lot!

Answered by Harley Weston.
A reverse PST calculation 2010-09-23
From Manning:
Hi, I'm a manufacturing contractor in Saskatoon. The way to calculate PST for us is using total contract amount x 70% then 5%. For example, contract amount of $100 x 70% x 5% = $3.5 (PST). The subtotal is using contract amount + PST. $100 + $3.50 = $103.50.

If you only know the subtotal is $103.50 including PST, how do you reverse calculation to get the contract amount?

Thank you very much for your help.

Answered by Harley Weston.
A Squared Number That's Negative 2010-09-22
From David:
What is the only number that when it's squared becomes negative?
Answered by Stephen La Rocque.
Limiting Cases in Geometry 2010-09-22
From Niki:
Consider a rectangle inscribed in a circle with a radius or R. What are the possible perimeters for the rectangle?
Answered by Stephen La Rocque.
|x+ 3| = |x-1| 2010-09-21
From Jack:
|x+ 3| = |x-1|
Answered by Penny Nom.
What is the average speed? 2010-09-21
From Cindy:
A boat travels downstream m nautical miles at d knots. It travels upstream m nautical miles at u knots. What is the average speed for the entire trip?
Answered by Penny Nom.
Limits 2010-09-20
From rodrigo:
what's the limit of x+1/x-1, when x tends to 1?
Answered by Janice Cotcher.
Find the point of contact of the circles 2010-09-20
From Sandra:
If the center of circle A = (-1,-3) and the radius of it is √20, the center of circle B = (5,9) and the radius of it is √80. Find the point of contact of the circles A and B.
Answered by Janice Cotcher.
Constructing a tipi 2010-09-20
From mike:
we are thinking about making a lightweight tipi tent but we need to know what the angles and lengths of each side that we will need to cut. The height of the tipi wants to be about 2.2m span at the widest point wants to be about 3.2m we want to make it based upon a 6 sided(hexagon) shape thanks for your help
Answered by Harley Weston.
May Lee's cake 2010-09-18
From Wafa:
May Lee bought a cake which is circular in shape. Her sister ate a quarter of the cake. Given that the area of the top surface of the remaining portion is 520 square cm, find the diameter of the cake.
Answered by Penny Nom.
Elimination and substitution 2010-09-18
From Lauren:
Solve one using the method of substitution and the other with the method of elimination.

v a. y=5x+4
x=2y+1

b. 4x+3y=7
6x-3y=13

Answered by Penny Nom.
The circle centered at (2,-4) with radius 3 2010-09-18
From lupe:
Consider the circle centered at (2,-4) with radius 3. Write the equation of this circle sketch its graph, and find the exact coordinates of its intercepts. x-intercept y-intercept using the quadratic equation?
Answered by Penny Nom.
Continuity 2010-09-18
From Carina:
Hi. My name's Carina and I'm currently a sophomore in high school. I'm having a lot of difficulties in AP Calculus with continuity, one-sided limits, and removable discontinuities. Basically, I have no idea how to do them or even what they are. I read the lesson but I still don't get it. Can someone put it in simpler terms so I can understand how to complete my questions? Thank you!
Answered by Robert Dawson.
Six nines 2010-09-16
From Steph:
It's sort of one question. We have to use six nines to get the numbers 1-30. I got all but 22, 23, 24, and 30. Like 15 is 9+9-(9+9+9)/9.
Answered by Penny Nom.
A strip of land 2010-09-15
From Walter:
How many linear feet of strip of land 15 feet wide does it take to make 1 acre?
Answered by Penny Nom.
La racine carrée et l'exposant une demie 2010-09-14
From Alain:
Bonjour. Je cherche une explication sur l'équivalence entre les exposants fractionnaires et les racines nième. Par exemple, comment prouve-t'on que la racine carrée correspond à  l'exposant une demie? merci
Answered by Pierre-Louis Gagnon et Claude Tardif.
Two numbers 2010-09-14
From Sania:
math homework help-Felicia is thinking of two numbers. One of them is 2 more than 3 times the other one. The difference between the squares of these numbers is 112. What is the smaller number?
Answered by Penny Nom.
Choose any three digits that are less than ten 2010-09-12
From cecille:
my son have a homework, but i don't understand the question. here's the problem:
choose any three digits that are less than ten.
make all the two digit number you can using those three digits.
add up the two digit numbers you created and divided by the sum of the original three digits.
record your answer. then do it again with another three digits. write your observation about the answer.

Answered by Claude Tardif.
The length of a vector 2010-09-12
From kyrie:
Find the possible values of the constant a such that
| ai + 4aj + 4k | = 13
I know the answer is 3 but not sure how to get to it

Answered by Penny Nom.
Two perpendicular chords 2010-09-11
From edwin:
two perpendicular chords AB and CD intersect at P. if X,Y are their midpoints and M the centre of the circle, prove that MP=XY. I do not a have clue on how to do it so can you please help me with it
Answered by Robert Dawson.
An algebraic equation with fractions 2010-09-10
From leah:
If 6/x - (x-1)/2 = 4 then the LCD is 2x, right? so you get 12/2x - (x2-x)/2x = 8x/2x, right? then what do you do with the 2x when you turn it into a quadratic equation?
Answered by Penny Nom.
Mrs. Johns graded 121 math tests. 2010-09-10
From Tums:
Mrs. Johns graded 121 math tests. It took her 3 hours to grade all of the tests. Each test took the same amount of time to grade. About how many test did Mrs. Johns grade in 2 hours?
Answered by Penny Nom.
A building and a flag pole 2010-09-09
From paul:
A flag pole and a building stand on the same horizontal level. From the point p at the bottom of the building,the angle of elevation of the top t of the flag pole is 65 degrees. From the top q of the building the angle of elevation of the point t is 25 degrees.If the building is20 meters high. Calculate the distance pt
Answered by Penny Nom.
A logarithmic equation 2010-09-08
From Rohit:
x^2 + k*ln(x) - c - k = 0

Where k and c are constants.

Answered by Penny Nom.
Decimal form 2010-09-08
From Tracy:
What is the decimal form of 0.14 x 10 the the first power?
Answered by Penny Nom.
The new diameter is 50 per cent larger than the original. 2010-09-04
From mahwish:
. A circular logo is enlarged to fit the lid of a jar. The new diameter is 50 per cent larger than the original. By what percentage has the area of the logo increased?
Answered by Tyler Wood.
How are decimals used in every day life? 2010-09-01
From Keith:
how are deciminal used in every day life
Answered by Tyler Wood.
Maximizing the volume of a cylinder 2010-08-31
From Haris:
question: the cylinder below is to be made with 3000cm^2 of sheet metal. the aim of this assignment is to determine the dimensions (r and h) that would give the maximum volume. how do i do this? i have no idea. can you please send me a step-to-step guide on how t do this? thank you very much.
Answered by Penny Nom.
Three cars 2010-08-30
From Anil:
3 cars are moving at speed of 4 kmph,5.5 kmph and 8 kmph in a circular track.The circular track has a distance of 11 km. What is the time taken for all the 3 cars to meet at the starting point ?
Answered by Stephen La Rocque.
A truck goes south and then east 2010-08-26
From MARK:
A truck starts at point A and drives South 3 miles. Then it turns left at Point B and drives east four miles to point C. How many miles is the truck at point C from point A if it were to drive directly from point A to Point C.My book says the answer is 5 miles. How did they come up with the answer?
Answered by Robert Dawson.
Exponential form 2010-08-24
From Courtney:
What the Exponential form? For Ex. X2*X5*X4??
Answered by Penny Nom.
A max min problem 2010-08-19
From Mark:
a rectangular field is to be enclosed and divided into four equal lots by fences parallel to one of the side. A total of 10000 meters of fence are available .Find the area of the largest field that can be enclosed.
Answered by Penny Nom.
A vertical asymptote 2010-08-19
From Kendall:
If a compound function is defined as 1/x for x<0 and 5 for x>=0 does it have a vertical asymptote at x=0? According to the definition in my book I think it does because the left-hand limit is infinity and it only takes one side to cause a vertical asyptote. Thanks!
Answered by Harley Weston.
Mr Thorpe's volume 2010-08-15
From thomas:
A circular pool has a diameter of 3m and is filled to a depth of 70 cm. Mr Thorpe climbs into the pool and submerges for ten seconds while his son measures the new depth at 72 cm. Calculate Mr Thorpe’s volume.
Answered by Penny Nom.
Digging a pond 2010-08-14
From Jody:
I am digging a pond that is 100' X 60' X 10' deep in the centre. All four sides will have 3-1 slopes on them. How do I calculate how may yards of material will have to be removed?
Answered by Tyler Wood.
How many labels are left on a roll? 2010-08-13
From Melissa:
Is there a simple way to calculate how many labels are left on a roll? I thought there was a way to look at the individual label length, the label thickness and the core diameter and then measure the overall diameter to calculate how many pieces were left, but I'm struggling. I've seen a tool before- I thought we called it a circle calculator, but those parameters were all that I needed to input to spit out the correct count. I never saw what was behind the scenes for this calculation. Ie factoring in revolutions or the number of pieces on each layer, etc. Please let me know if you have any suggestions. Any assistance will be greatly appreciated!!! Thanks!
Answered by Penny Nom and Tyler Wood.
80,000 yards of dirt 2010-08-12
From Jeremy:
If I have an area 50' x 1200', what would be the elevation with 80,000 yards of dirt on it.
Answered by Robert Dawson.
There are 12 balls and one balls weight is different than the others 2010-08-12
From david:
if there was 12 balls and one balls weight is different than the other and use a balance 3 times is there a possible way to find which weigh different
Answered by Robert Dawson.
Differentiable on an interval 2010-08-12
From Dave:
Hi I was wondering if a function can be differentiable at its endpoint. For example if I have Y = X^2 and it is bounded on closed interval [1,4], then is the derivative of the function differentiable on the closed interval [1,4] or open interval (1,4). They always say in many theorems that function is continuous on closed interval [a,b] and differentiable on open interval (a,b) and an example of this is Rolle's theorem. Thank you for your help.
Answered by Robert Dawson.
An octagonal shaped cake 2010-08-12
From Sara:
Hi, I have tried many polynomial equations and substitution methods but i cant seem to get the right answer. Can you please provide some guidance?
Question:
A square cake tin is 24cm x 24cm x 7cm but a cook wants to make an octagonal shaped cake so has cut out 4 pieces of rectangular card board and placed one on each corner on the interior of the tin. The cake has to be a regular octagon so pieces must be just right. What is the size of each piece of cardboard to make the cake a regular octagon? Thankyou.

Answered by Tyler Wood.
Two cars on a circular track 2010-08-12
From david:
If two cars starts to drive at point 1 and go in a circle when will they meet? (one cars speed is faster the other ) all I need is a formula
Answered by Tyler Wood.
Disposing of some concrete 2010-08-12
From Kathy:
We are needing to dispose of concrete that we are tearing out of our backyard and I need to figure the dimensions and weight. I figured we have approx. 419 sq ft of concrete at 4" thick. Could you tell me the how much that comes to in cubic ft, cubic yards and weight. So I can get some estimates on pricing to haul off and dispose of. thank you
Answered by Harley Weston and Tyler Wood.
Algebraic Equations 2010-08-10
From Justin:
the amount of money that Mika has if she has some quarters

how fast Rya runs if she runs 5 mi/h slower than Danae
please help,
thanks, justin

Answered by Robert Dawson.
The integral of (x^2*exp(x)/(exp(x)-1)^2 2010-08-09
From sujoy:
please find this integral for me

int(x^2*exp(x)/(exp(x)-1)^2

Answered by Robert Dawson.
The height of a triangle 2010-08-05
From Ann:
The height of a triangle is 4cm less than the base. The area is 198cm^2. Find the height.
Answered by Stephen La Rocque.
Two overlapping circles 2010-08-04
From Husen:
two circles of radius 5 cm intersect each other .the distance between their centers is 5root 2.find the area of the portion common to the two circles
Answered by Penny Nom.
A square field has an area of 289 meter squared 2010-08-02
From daniel:
a square field has an area of 289 meter squared. How do you calculate the dimensions
Answered by Stephen La Rocque and Penny Nom.
A rational equation 2010-08-02
From Bailey:
3/x
+
x/x+2
______
-2/x+2

Answered by Penny Nom.
A shrunken car 2010-07-31
From Diana:
Car manufacturers have outdone themselves. They have discovered how to shrink a car to make parking easier. At the touch of a button, a car shrinks by 2 feet in the width and 4 feet in the length. The original car base is 24 square feet longer than that of the shrunken car. If the original cars length is twice the width, find the dimensions of both cars. Identify the unknowns carefully.
Answered by Penny Nom.
70 cubic meters of mulch 2010-07-30
From zsuzsi:
i have 70 cubic meters of mulch to move and an 8 tonne truck to use how much tonnage is to 1 square meter so that i can work out how many trips i need to make
Answered by Penny Nom.
The suspension cables of a bridge 2010-07-29
From Mike:
what is the formula for the suspension cables of a bridge. The towers are 200 ft above the roadway The towers are 3400 ft apart The cable if at 8ft in the middle of the span
Answered by Robert Dawson.
The volume of earth to be moved 2010-07-29
From Michael:
Volume of earth moved @ 500' across, 1 to 3 degree slope, 30' deep X 50' across at the bottom.
Answered by Stephen La Rocque and Harley Weston.
The shadow of a building 2010-07-28
From vera:
building casts a shadow 210 ft. long,40 degree angle.How tall is the building?
Answered by Penny Nom.
The area of a hexagon 2010-07-28
From Katie:
Hi i have to find the area of a hexagon with only the side length 2cm given would you be able t help?
Answered by Walter Whiteley.
A spaceship playhouse 2010-07-26
From Dave:
I would like to build a spaceship playhouse for my grandson I want it to be about 36" around and I want to use 5/4 decking boards that measure 5 1/2" wide how do I figure out how to lay out a base pattern to nail to (what angle do I need to cut and how many boards will it take to go around the circle.
Answered by Penny Nom.
The tonnage of some gravel 2010-07-22
From Gregory:
i have a road that is 8 miles long 20 feet wide and 4 inches deep what is the tonnage for gravel i would need
Answered by Penny Nom.
Reflection in the line y = x 2010-07-21
From tousif:
Find the 2 x 2 matrix which represents a reflection in the line y = x? Please help....
Answered by Penny Nom.
I need to learn to think mathematically 2010-07-20
From Student:
I need to learn to think mathematically, and like math while doing it, got any ideas, help.
Answered by Walter Whiteley.
Numbers that can be formed using the digits 1,2,3,4,5,8,9 2010-07-19
From Donessa:
write the number that is 2000 more than the difference between the largest and the smallest number that can be formed using the digits 1,2,3,4,5,8,9
Answered by Tyler Wood.
An 18ft round pool 2010-07-19
From christine:
will an 18ft round pool fit on a 12x16 ft rectangular cement slab?
Answered by Tyler Wood.
Simplify (2x)3 2010-07-16
From emily:
how to Simplify (2x)3
Answered by Melanie Tyrer and Penny Nom.
Converting kilograms to cubic metres 2010-07-16
From Ilai:
I want to convert kg to cubic metre
Answered by Penny Nom.
Fibonacci and induction 2010-07-12
From James:
I'm trying to prove by induction that F(n) <= 2^(n-1) where f(1)=f(2)=1 and f(k)=f(k-1)+f(k-2) for k >=3 is the Fibonacci sequence
Answered by Stephen La Rocque and Tyler Wood.
An octagon shaped bench 2010-07-09
From rob:
i am trying to build a octagon shaped bench to fit inside a 69 inch round hot tub so that the tip of each point touches the edge of the circle where it will be fastened.
Answered by Stephen La Rocque.
Thickness of sand 2010-07-07
From RHONDA:
In an 18 x 33 ft area how much is 4 yards of sand in inches to cover this area.
Answered by Penny Nom.
777777 times 111111 2010-07-07
From Chew:
What is 777777 times 111111 without using a calculator?
Answered by Chris Fisher.
Maximize the floor area 2010-07-07
From shirlyn:
A rectangular building will be constructed on a lot in the form of a right triangle with legs of 60 ft. and 80 ft. If the building has one side along the hypotenuse, find its dimensions for maximum floor area.
Answered by Penny Nom.
12 coins 2010-07-07
From Eden:
We have 12 coins same in size and shape and also weight. But among these, one coin which has same size and shape, but not in weight and we can't know that this coin was lighter or heavier. We must decide which coin was lighter or heavier among these 12 coins in 4 times of weighting with scale.
Answered by Stephen La Rocque and Tyler Wood.
A collection of baseball cards 2010-07-01
From Martha:
Cody has a collection of baseball cards. Is it possible for him to place the cards into 5 boxes so that exactly 3 of the boxes contain an odd number of cards and the total number of cards in two of the boxes is the same as the total number in the other 3 boxes? Explain.
Answered by Robert Dawson, Penny Nom and Claude Tardif.
Fillig the pool 2010-06-29
From saurabh:
At certain swimming pool certain pipes can fill it in 2 hours, another can fill it in 5 hours,and third pipe can empty the pool in 6 hours. with all three pipes turned on at the same time and starting with an empty pool , how long will it take to fill the pool
Answered by Robert Dawson.
The equation of a parabola 2010-06-29
From Tiffany:
Find the equation of a parabola that passes through the points (-2,3), (-1,1) and (1,9)
Answered by Stephen La Rocque.
A play tent 2010-06-28
From Susan:
Hi!!! I am making a play tent as seen at the link below and need to figure out how to get the dimensions for the cone shape. The one shown has 4 different seams, but I guess I can get away with just one seam to sew it together (?) I need it to go over a hula hoop as that is what I am using for the round support at the top. My hula hoop is 35" in diameter from outside edge to outside edge. I would like the height of the teepee to be around 30" from the center to the peak. Thanks so much, oh my gosh, I have been fretting for 2 days about this and my hair is about to fall out!!!! Please contact me if you need any additional info.
~Susan

http://www.landofnod.com/family.aspx?c=52&f=4100

Answered by Penny Nom.
1 cubic yard of top soil 2010-06-28
From Bob:
If you were to place 1 cubic yard of top soil onto a 15' X 24' Garden area and it was spread evenly would the amount be a dusting of 1" or about 1'?
Answered by Penny Nom.
The product of 2 consecutive integers 2010-06-28
From Sharon:
the product of 2 consecutive integers is -6 less than the square of the larger number. find the smaller number
Answered by Penny Nom.
An algebraic fraction 2010-06-28
From luis:
3x-2//4-3x=12
Answered by Stephen La Rocque.
The intersection of two graphs 2010-06-27
From Austin:
In depth explanation of how to find the intersection points of functions y=-4t+300 and y=-4.9(t-5)^2+300
Answered by Stephen La Rocque.
Thermal expansion of a steel beam 2010-06-25
From chris:
If a 50 ft steel beam can expand up to 4inches when heated to 1000f How much will a 162 ft and six inches steel beam expand under the same conditions?
Answered by Robert Dawson.
The diameter of a circle 2010-06-24
From John:
Piece of wire 72cm in length bent to make semicircle and diameter.Find length of diameter.Answer given 28cm.How is this arrived at?Looked at circumference in various ways but failed to understand how 28 cm was arrived at.Thanks for the help.
Answered by Robert Dawson.
Rays and angles 2010-06-24
From cristina:
what is the formula for finding the number of angels that can be named by a given number of rays with the same endpoint?
Answered by Robert Dawson.
(x^3 + 11x) is divisible by 6 2010-06-24
From PT:
Given that x is a non-zero integer, how do you show that for all values of x, (x3 + 11x) is divisible by 6?

I know it works but how do I answer the "all values of x" part?

Thanks in advance!

Answered by Robert Dawson.
The equation of a circle 2010-06-23
From Michelle:
Write and equation for the circle with a center, (0,0) and a diameter of 12
Answered by Penny Nom.
An octagonal building 2010-06-23
From Nancy:
I have to submit my approximate sq. meters of an Octagon home in Costa Rica. All I know is all sides are equal. One structure has all sides at 16 feet and the second one has all sides equal at 8 ft. I then have to convert it to sq. meters. Can anybody help me with this? Thank you so much, Nancy
Answered by Penny Nom.
A linear relationship 2010-06-22
From Laura:
The monthly cost of driving a car depends on the number of miles driven. Lynn found that in May it cost her $300 to drive 480 miles and in June it cost her $460 to drive 800 miles.

Question: Express the monthly cost C as a function of the distance driven d, assuming that a linear relationship gives a suitable model.

Answered by Robert Dawson.
Selling price, cost and profit 2010-06-22
From Alex:
The profit on a certain television set is 16.5% of the cost price. If the profit is $330, Find:
a) the cost price
b)the selling price of the television

Answered by Penny Nom.
The height of a triangle 2010-06-21
From Taylor:
how do i find the height of a triangle with a base of 3 and a side of 5 the book says 4 but i cant get that
Answered by Penny Nom.
3 equidistant points on a circle 2010-06-21
From Brian:
We have to line up 3 pins on a lifting bridle to be 120 degrees apart from each other to connect into a lifting assembly.
I know pi*d. I know straight lines connecting these points to each other will result in an equilateral triangle. How does one find these points without using a protractor? There must be a formula (formulae) to work this out mathematically.
r = 10.5 inches
pi*d = 65.9736
3 segments( arcs) = 21.9912 which is the distance along the circumference that the points are from each other.

Answered by Penny Nom.
Geometry in the Woods 2010-06-16
From cleo:
what geometric ideas can you find in the woods
Answered by Walter Whiteley.
A loan 2010-06-16
From melody:
LOAN: 1000
TERM: 2 years
INTEREST: 2% (per month)
AMOUNT: ???

what is he formula of solving this and what is the answer..

Answered by Robert Dawson.
Exponential form 2010-06-16
From charlotte:
what is the exponential form of 6x6x6x5x5x5x5=
Answered by Robert Dawson.
1.2731568 2010-06-16
From angel:
is 1.2731568 a natural number a whole number a irrational number or a inter
Answered by Robert Dawson.
Expand and simplify 2010-06-15
From Alaa:
Expand and simplify
(x-9)(x+2)

Answered by Penny Nom.
How many gallons do I need to finish filling the tank? 2010-06-14
From Bruce:
I have a tank on it's side. 44" dia. by 72" long, I'm 6" from having tank full. How many gallons do I need to finish filling tank?
Answered by Harley Weston.
A max/min problem 2010-06-12
From valentin:
What is the maximum area of an isosceles triangle with two side lengths equal to 5 and one side length equal to 2x, where 0 ≤ x ≤ 5?
Answered by Harley Weston.
The area of a circle 2010-06-12
From grier:
please,
what is the area of a semicircle with arc length 3pi?

Answered by Penny Nom.
Distribution with Fractional Terms 2010-06-11
From Vincent:
Compounded semiannually. P dollars is invested at annual interest rate r for 1 year. If the interest iscompounded semiannually, then the polynomial P(1 + r/2)^2 represents the value of the interest after 1 year. Rewrite this expression without parentheses. Evaluate the polynomial if P=$200 and r = 10%.
Answered by Janice Cotcher.
Roosters, hens and chicks 2010-06-10
From Kamarah:
If a rooster is worth 5 coins and a hen is worth 3 coins and chicks 3 together are worth 1 coin, how many roosters, hen and chicks totaling 100 can be bought with 100 coins?

notes:
you must buy at least one of each type of fowl
no fractional fowl
chicks are purchased in multiple of 3
find 3 solutions

Answered by Penny Nom.
Graphical Representation of Complex Numbers 2010-06-08
From Anas:
why do we write complex number a+ib as (a,b)?
Answered by Janice Cotcher.
Standard Deviation from a Proportion 2010-06-08
From silvestre:
A large company finds that approximately 17% of all purchases are returned for credit. If the company sells 100,000 different items this year, about how many items will be returned. Find the Standard Deviation.
Answered by Janice Cotcher.
The tires on Helen's new car 2010-06-08
From Rich:
Helen bought a new car with four tires and a full-size spare. If she rotated the tires so that each tire would have the same amount of wear, then how many miles were on each tire when her odometer showed 40,000 miles?
Answered by Penny Nom.
Going to the bike shop 2010-06-07
From Omi:
Marie rode her bicycle from her home to the bicycle shop in town and then walked back home. If she averaged 6 miles per hour riding and 3 miles per hour walking, how far is it from her home to the bicycle shop if her total travel time was 1 hour?
Answered by Penny Nom.
An input-output table 2010-06-07
From Brian:
Make an Input -output table to represent the function. Use 0,1,2,3 as the Domain. f(x)= 3+7x
Answered by Harley Weston.
Using the limit definition 2010-06-06
From Meagan:
Using the limit definition find the derivative of 3/(2x^2)
Answered by Harley Weston.
4 > -3m-7 >= 2 2010-06-05
From pamela:
The question is
4>-3m-7>=2

Answered by Penny Nom.
11 trillion divided by 309,418,000 2010-06-05
From Dion:
I am struggling to calculate what 11 trillion divided by 309,418,000 is.
Answered by Penny Nom.
(x + a)^2 2010-06-04
From a:
If the 3rd , 4th, 5th terms in the expansion of (x + a)powe "n" are 720, 1080, 810 respectively, then find the values of x, a and n.
Answered by Penny Nom.
Four circles in a square 2010-06-04
From Daniela:
four circles are drawn in a square such that the circles are tangent to each other as shown. find the area of the shaded region. It the goes on to show a diagram with a square and four circles drawn in it. The length of a side of the square is 24. Please help me!
Answered by Penny Nom.
Compound interest 2010-06-01
From Kenneth:
Hello: I have a question regarding the following compound interest and future value calculation.

Year 1 P + rP equals balance after the first year.
Year 2 (P + rP) + r(P + rP) equals balance after the second year.
Year 3 ? equals balance after the third year.

This question is in two parts.

1. What would follow for year three?

2. I know that a pattern will develop. What will it be so that I can determine the extended pattern for following years ?

Answered by Penny Nom.
A cylindrical vessel with belled ends 2010-06-01
From George:
surface area of a vessel with a cylindrical length of 60 feet, a diameter of 10 feet, and with belled ends of 5-foot radius
Answered by Penny Nom.
graph y=(2x^2-3x)e^ax 2010-05-31
From James:
graph y=(2x^2-3x)e^ax
Answered by Robert Dawson.
x/a +y /b =a+b : x/a^2+ y/b^2 =2 2010-05-30
From smithu:
x/a +y /b =a+b : x/a2+ y/b2 =2 solve by using elimination method , cross multiplication, substitution method
Answered by Penny Nom.
Integration of sin^3 (2x) 2010-05-29
From ascher:
how do you integrate this equation ∫ sin^3 (2x) dx
Answered by Robert Dawson and Penny Nom.
A fuel tank with a broken gage 2010-05-28
From Kinsley:
Hello, I have a fuel tank with a broken gage. It is a 2000 gallon tank. The dimensions are 64" dia. x 146" long. The fuel gage doesn't work, so the only measurement I can do is put a stick in it to see what is left. Currently it is at 19". PLEASE NOTE: the tank is laying on its side, meaning the long side is on the ground. So, how do I determine how much fuel is in the tank by the inches remaining? Thank you very much.
Answered by Tyler Wood.
Weighing a 1000 lb cylinder on a 500 lb scale 2010-05-28
From Jerry:
I am trying to weigh a large possibly 1000 lb cylinder on a 500 lb scale, is this possible? If so what is the actual formula to figure out the correct weight? I am told it is possible with the fulcrum method but have not been able to find a formula. Thanks Jerry
Answered by Robert Dawson.
Two problems 2010-05-27
From debbie:

Question from debbie, a parent:

hi, i have a daughter and she asked me a maths question I cannot solve. I was just wondering if you can give me the answers plus the working out so I could explain to my daughter,

1. The leftmost digit of a six-digit number N is 1. If this digit is removed and then written as a rightmost digit, the number thus obtained is three times N .Find N.

2. Four friends are racing together down a flight of stairs. A goes 2 steps at a time, B 3 steps at a time. C 4 steps at a time and D 5 steps at a time. The only steps which all four tread on are the top one and the bottom one. How many stairs in the flight were stepped on exactly once?


Answered by Penny Nom.
Graphing a linear inequality 2010-05-26
From Valeie:
When graphing a linear inequality, how do you know if the inequality represents the area above the line.
Answered by Penny Nom.
Liquid in a horizontal tank 2010-05-26
From Brian:
I have a 50' long tank by 12' diameter it is lying down on it side with 12" of Liquid in the bottom i need to know how many gallons of fluid this is? Please Help, I was able to calculate out the Triangle but not the Sector!
Answered by Harley Weston.
A square inscribed in a circle 2010-05-25
From Middle:
what is the perimeter of a square inscribed in a circle of radius 5.0 inches?
Answered by Penny Nom.
What is the speed of the current? 2010-05-25
From tien:
a boat travels upstream, against a current, at a speed of 4 miles per hour. On the return trip, the boat travels downstream, with the current , at 10 miles per hour. the speed of the current remains constant. what is the speed of the current?
Answered by Penny Nom.
How wide a strip has he cut when the lawn is half mowed? 2010-05-24
From Arib:
rectangular lawn has length 40 m and width 30 m. David starts cutting from the outside in. How wide a strip has he cut when the lawn is half mowed?
Answered by Penny Nom.
What is the distance from A to C via B? 2010-05-24
From vivianne:
The distance from A to B is d km and that from B to C is x km. if a bus maintains an average speed of 50km/hr between A and B and 60km/hr between B and C, it takes 3 hours to travel from A to C. If it maintains 60km/hr between A and B and 50km/hr between B and C, the journey takes 8 minutes less. What is the distance from A to C via B?
Answered by Penny Nom.
Math in the workplace: nursing 2010-05-24
From Taylor:
How is math used in the workplace - Nurse Anesthetist?
Answered by Penny Nom.
An optimization problem 2010-05-23
From Marina:
Hello, I have an optimization homework assignment and this question has me stumped..I don't even know

A hiker finds herself in a forest 2 km from a long straight road. She wants to walk to her cabin 10 km away and also 2 km from the road. She can walk 8km/hr on the road but only 3km/hr in the forest. She decides to walk thru the forest to the road, along the road, and again thru the forest to her cabin. What angle theta would minimize the total time required for her to reach her cabin?
I'll do my best to copy the diagram here:

                             10km
Hiker_ _ _ _ _ _ _ _ _ _ _ _ _ _ _ _ _ _ _ _ _Cabin
      \                           |                              /
       \                          |                             /
     f  \                      2km                          /
         \                        |                           /
theta   \___________________________ /
                            Road


Answered by Penny Nom.
The number of gallons of water in a basement 2010-05-23
From George:
A basement with dimensions 68 ft long by 42 ft wide by 10 ft deep is flooded to the ceiling calculate the number of gallons of water in the basement. 70
Answered by Penny Nom.
Which values of x satisfy (x-2) / (x+4) <7 2010-05-23
From Rocco:
Which values of x satisfy (x-2) / (x+4) <7
Answered by Harley Weston.
Divisibility by 3 2010-05-23
From Cathleen:
To math central. I have to do a maths extension question that I don't understand. At first I thought I did. It is about the dividing by three. In one part of the question, it asks me to show that the rule of division by three does not work for 23142 with a little 5 down the bottom. What doe base 5 mean? We first thought that the little 5 down the bottom meant multiplying y the power of five. Can you please tell me what it means so I can finish this question?
Answered by Penny Nom.
The area of a triangle 2010-05-22
From jose:
haw many square feet are in a triangle 30 feet high with a 100 ft base?
Answered by Tyler Wood.
How many bags will it take to fill this? 2010-05-22
From David:
A garden lot..360 square feet and 6 inches deep..you want to fill this with mulch and use 2 cubic. foot bags...how many bags will it take to fill this? Thank you...Dave
Answered by Tyler Wood.
How far is the runner from the starting point? 2010-05-22
From Richard:
A marathon runner runs 6 miles south and then 8 miles east. How far is the runner from the starting point?
Answered by Tyler Wood.
Extraneous solutions 2010-05-22
From Joe:

Question from Joe, a parent:

w+3    2w
----- - ----- = 1
w2-1   w-1

W2 is = w squared

                            -4
The answer is w= ------
                             3

but have no idea how this was solved. any help is appreciated. Thanks.


Answered by Penny Nom.
A circular oil slick of uniform thickness 2010-05-22
From Susan:
Hi, I have this problem on a homework assignment and just can't seem to figure it out:
A circular oil slick of uniform thickness is caused by a spill of 1 m^3 of oil. The thickness of the oil is decreasing at the rate of .001m/h. At what rate is the radius of the slick increasing when the radius is 8.

Answered by Penny Nom.
The low leg height of a shutter 2010-05-20
From brian:
I work for a shutter company and am in need of a formula to figure out what the low leg height would be if given the width of shutter, the high point of arch top and the radius. example would be a 18" wide shutter with a 80" high leg on the right side and a 30" radius. I would need a way to figure what the low leg height (left side of shutter) would be. Or if given width, low leg height and radius what the high side would be? If any of this can be given in laymen's terms it would be much appreciated. Thanks, Brian
Answered by Harley Weston.
The volume of a test tube 2010-05-20
From Nick:
A maths question. I have a test tube (basically a cylinder but with a rounded bottom end) and the measurements are: the diameter or width of the tube is 31mm and the total length of the whole tube is 204mm. The question is:
(a) what's the volume of this test tube (rounded to the nearest cubic mm) and
(b) what's the capacity of this test tube (rounded to the nearest mL). I need to show the full mathematical workings out of the two, not just the answer. The problem is the circular bottom end of the tube, it isn't flat, that would make it rather "easy"or straightforward but this is a half-circle (like all test tubes are)!!

Answered by Penny Nom.
Percentage increase 2010-05-20
From Rebecca:
Question from Rebecca:

In 2008 my land was valued at $84,000. In 2009 my land was re-valued at $235,000.00. What is the percent increase between the two values? I want to be able to express "the value was increased by x%."

Answered by Harley Weston.
A probability tree 2010-05-20
From Sandy:
Children’s meals are being prepared for a large gathering. There is an equal number of chicken, hamburger, and fish meals. Each meal comes with a toy: a ball, a parachute toy, or a Frisbee. There are twice as many balls as Frisbees and an equal number of parachutes and Frisbees.

Use a probability tree to show the sample space and determine the probability that a meal consists of chicken or fish and comes with a ball as a toy.

Answered by Penny Nom.
(x+1)(x+2)(x+3)/(x+1)(x+2) 2010-05-19
From Nazrul:
Simplify : (x+1)(x+2)(x+3)/(x+1)(x+2)
Which answer is correct:
(i) x+3
(ii) (x+2)^2(x+3)
Please help me.

Answered by Harley Weston.
A fact family 2010-05-19
From PRISCILLA:
ELLIE HAS 12 BOOKS. SHE GIVES AWAY 3 BOOKS. ELLIE HAS _LEFT. _ - _ = _ _- _ = _ _+ = _ _ + _ = _
Answered by Robert Dawson.
How far am I from the starting point? 2010-05-18
From jilayna:
if i walk 5 miles north, 7 miles east, and 3 miles north again.to the nearest tenth of a mile ,how far,in a straight line, am i from my starting point
Answered by Penny Nom.
A scale model of a city 2010-05-18
From Briana:
Well I have to create a three dimensional city for a geometry project and it requires us to use precise scal factor and well we want to build a medium sized city but our teacher said that If we make or buildings to high our houses would look like super small compared to the tall building were trying to figure out if we were to build a medium sized city what scale factor should we use to make the houses big enough to see and design?
Answered by Walter Whiteley and Tyler Wood.
A conical pile of gravel 2010-05-15
From Chuck:
If I have a conical pile of gravel 50 feet across at the base and a height of 65 feet and the slope of the side is approximately 60 degrees, how do I calculate the cubic yards?
Answered by Robert Dawson.
88 litres of oil in a cylinder 2010-05-15
From fella:
88 litres of oil are poured into a drum 40 cm in diameter. find the depth of the oil in the drum.
Answered by Penny Nom.
The value of 1 acre 2010-05-13
From Rebecca:
If 38,121 sq feet equals $19,846.58 at 75% of its current value, what would the value of 1 acre be at 100% of value?
Answered by Penny Nom.
The circumference of a circle 2010-05-12
From Morgan:
find the circumference of a circle with radius 8. im having trouble i just don't understand. thanks for your help i really do appricate it.
Answered by Penny Nom.
The sum of twice a number and 6 is 8 2010-05-12
From MAE:
THE SUM OF TWICE A NUMBER AND 6 IS 8 . FIND THE NUMBER
Answered by Penny Nom.
The product of two consecutive even integers 2010-05-11
From Terry:
The product of two consecutive even integers is 14 more than their sum. Find the integers.
Answered by Penny Nom.
Modelling an underpass 2010-05-11
From Sue:
An engineer at the Ministry of Transport is creating the plans for a new road. This road will cross the path of a busy railway track so it will be necessary to built an underpass for this road. This underpass will be in the shape of a parabolic arch.

The specifications include:
-the road must be at least 10 m wide and it must have shoulders at least 2 m wide on either side of the road.
-there must be a clearance of 3 m over all areas of the road.

What quadratic equation could you use to model this bridge.

Answered by Penny Nom.
The area of a triangle 2010-05-10
From Saeed:
Show that:
Area of triangle = half of the perimeter of the triangle times the radius of the inscribed circle !!!!!!!!!!!!!!!

Answered by Penny Nom.
The altitude of a triangle 2010-05-08
From kylie:
the vertex angle of an isosceles triangle is 57 degrees 24 minutes and each of its equal sides is 375.5 feet long. find the altitude of the triangle
Answered by Penny Nom.
The dimensions of a driveway 2010-05-07
From Josh:
Dave and Jane have a new rectangular driveway. The perimeter of the driveway is 168 feet. The length is 12 feet longer than three times the width. What are the dimensions of the driveway?
Answered by Penny Nom.
An angle in a triangle 2010-05-06
From Morgan:
Question from Morgan, a student:

                 t
        8   t      t 8
          t    78   t
       t               t
  t                        t
t   x                       t
ttttttttttttttttttttttttt
              10

I'm having trouble solving for x I'm not sure where to start ( the ones in the middle of the triangle are both degrees) thanks in advance for your help i really do appreciate it
Answered by Penny Nom.

A 22 litre bag of topsoil 2010-05-06
From Blanche:
I do a fair amount of gardening when the weather permits. Just wondered if anyone call tell me what a 22 litre bag of topsoil would weigh and how to convert that. My husband says 20 lbs. I say noway I think it's more. Thanks to whomever can help me. :)
Answered by Penny Nom.
Line of sight 2010-05-06
From David:
I live in St. Joseph, Michigan and there is an ongoing argument regarding line-of-sight over the horizon.

Standing on a 200 foot high bluff here, people swear they can see the top of the Willis (nee Sears) Tower in Chicago, which is about 1653 feet high.

It is my contention that this is actually a "refracted reflection" and not direct-line-of-sight.

So, to settle the argument, I'd sure like some simple explanation for this, even if-and I hope not-I am incorrect.

David

Answered by Harley Weston.
A loop 2010-05-05
From Amir:
what is loop theory in algebra?
Answered by Chris Fisher.
The equation of a circle 2010-05-04
From crystal:
find the standard form of the circle with center (-2,3) and tangent to the line y=1
Answered by Penny Nom.
Interest 2010-05-04
From RW:
If you earned 55.00 for a year in your savings acct. The account pays 0.5 % interest. How much money did you initially have in the savings account.
Answered by Penny Nom.
A wishing well 2010-05-04
From Cassie:
I'm trying to construct a Wishing Well made of treated 2x4 wood, so the actual measurement is 3 1/2 by 1 1/2. The well is going to be three feet in diameter. I'm trying to construct this as circular as possible, what angle should the wood panels be cut at?
Answered by Penny Nom.
Tiling a floor 2010-05-03
From guadalupe:
how many 16 x 16 inch square tiles fit into a 10 x 10 foot space?
Answered by Harley Weston.
The rejection region 2010-05-01
From Mong:
In Tests of Hypothesis, what is the decision when the test statistic we compute from the sample information is exactly equal to the critical value? that is, we reject null hypothesis or do not reject it? Thanks!
Answered by Chris Fisher.
The diameter of a circle 2010-04-30
From Jimmy:
If I have a straight line with a known length and decide to make a circle out of this know length how would I calculate what the diameter of the circle would be
Answered by Penny Nom.
How many terms are in the simplified expression? 2010-04-29
From Anu:
The expression (x+y+z)^2006 + (x-y-z)^2006 is simplified by expanding it and combining like terms.
How many terms are in the simplified expression?

I've spent hours on this. pls help!!!
Also, is answering these questions a paid job for you or is it volunteer work?
I'd like to join an ask and answer site for math, (when I grow up)
How does one go about doing that?

Answered by Robert Dawson.
The rate of change of y with respect to x 2010-04-29
From Tom:
I just had a quick calc question about wording that wasn't ever addressed in class. When the book says "the rate of change of y with respect to x", should it be considered how fast y is changing in comparison to x?

I ask because the textbook says that "y is changing 3 times faster than x, so the rate of change of y with respect to x is 3." I'm use to rate being like velocity, as in units of distance per units of time. All we're told in class is that it's the slope of the tangent line, I was hoping you could clarify for me what exactly is meant by the wording of a "rate of change of something with respect to something else". More specifically, what "rate" and "with respect to" mean within this context?

Thanks for your time

Answered by Harley Weston.
A trig equation 2010-04-28
From Steve:
2cos^2(X)-2sin^2(x)+1=0
Answered by Harley Weston.
The width of a picture frame 2010-04-28
From Daniel:
The frame of a picture is 28 cm by 32 cm outside and is of uniform width. What is the width of the frame if 200 cm ^2 shows?
Answered by Penny Nom.
A circle inscribed in a square inscribed in a circle 2010-04-28
From jouniella:
A square is inscribe to the first circle, then another circle is inscribe to the square. Find the ratio of the 2 circles.
Answered by Penny Nom.
A rectangular laundry room 2010-04-28
From Cynthia:
a laundry room is shaped like a rectangle. the area of the room is 6 square yards. the perimeter is 10 yards. The room is longer than it is wide. How wide is the room? How long it the room?
Answered by Penny Nom.
cos(x) = sin(x - 1) 2010-04-28
From alex:
In the equation cos x = sin x-1 for -pi/2 A: solve for x graphically
B: solve algebraically and prove the solution is correct.

Alex

Answered by Penny Nom.
y = - log(x) 2010-04-28
From Alex:
y= - log(x), where y = 4.3
solve for x.

Answered by Harley Weston.
Sales taxes in Quebec 2010-04-28
From carole:
I work at a company where we use transport companies and we often get credits on their invoices. I need to know how to subtract the sales taxes from these credit amounts. (5% TPS and 7.5% TVQ). Is the equation: Amount / 1.05 = then this amount / by 1.075=?
Answered by Penny Nom.
One in twenty as a percentage 2010-04-27
From jim:
Please advise how I work out a gradient of say 1-20 one in twenty as a percentage %
Answered by Penny Nom.
A rectangular prism 2010-04-27
From Savannah:
A rectangular prism is made of exactly 8 cubes. Find the length, width, and height.
Answered by Penny Nom.
Rolls of sod 2010-04-27
From judy:
My yard is 35 feet by 29 feet. I want to put sod on the entire yard. The rolls of sod are 9 square feet. How many rolls of sod will I to complete the job?
Answered by Penny Nom.
Polyhedra and polygons 2010-04-26
From Brandon:
What is the difference between a polyhedra and a polygon?
Answered by Penny Nom.
Tiles 2010-04-25
From Pat:
how many 6 inch tiles will I need to purchase for an area that is currently covered with 351 4 inch by 4 inch tiles. Each box of 6 inch tiles states it contains 16 pieces and covers 4 square feet.
Answered by Penny Nom.
A rectangular garden 2010-04-25
From Billy:
Tanisha wants to make a rectangular garden with a perimeter of 38 feet. What is the greatest area possible that tanisha can make the garden?
Answered by Penny Nom.
The height of a flag shaft 2010-04-25
From Sarah:
A man standing 20metres away from a tower observes the angles of elevation to the top and bottom of a flag shaft standing on the tower as 62degrees and 60degrees respectively. Calculate the height of the flag shaft.'
Answered by Penny Nom.
Commission 2010-04-24
From TAHIR:
I want to pre-determine the amount of commission on sale of my product which will be the amount that I shall have to pay .
For example, the price is $ 600and commission 5% .
1) I calculate commission on 600 @ 5% = $ 30
When added $ 600 the retail price = $ 630
2) But on $ 630, 5% commission = $ 31.50
So we make a loss of $ 1.50 .
3) Again if we add this $ 1.50 to $ 630 the price = 631.50
But the commission when calculated does not tally
and we make a loss of $ 0.075.
Please let me know how to determine the commission beforehand so that it matches our accounts.

Answered by Penny Nom.
Algebraic fractions 2010-04-22
From rory:
3x/(x²-64)+4/(x²-6x-16)=
Answered by Robert Dawson and Harley Weston.
A table of values 2010-04-22
From Donna:
Complete the table by determining the value of each letter. Explain what rule is used to relate the numbers in the second column (with the heading of y) with those in the first column (with the heading of x).

x | y
3 | 5
9 | 23
4 | 8
6 | A
7 | B
C | 11
D | 2

I can't figure this out, can you help me?

Answered by Robert Dawson.
If (x, 4) is equidistant from (5, -2) and (3, 4), find x. 2010-04-21
From abeth:
If (x, 4) is equidistant from (5, -2) and (3, 4), find x.
Find the point on the y - axis that is equidistant from (-4, -2) and (3, 1).

Answered by Penny Nom.
The length and width of a table 2010-04-19
From Linda:
The length of the top of a table is 2 feet longer than 4 times the width. The area is 72m squared. Find the dimensions of the table.
Answered by Penny Nom.
A champagne tower 2010-04-16
From gabriel:
thanks for having something like this up online. My question is how do i build a Champagne Tower that has 290 glasses and 15 stories high.
Please help me out. this is for a clarity function.

Answered by Robert Dawson.
Four less than three times a number is 14 2010-04-16
From David:
My daughter has two questions that we could use some help with.

First ?
Four less than three times a number is 14

Secound?
The quotient of -15 and w

Answered by Penny Nom.
The area of semi circle 2010-04-16
From sagir:
how to find the area of semi circle?
Answered by Penny Nom.
A tangent line to a circle 2010-04-15
From Rhonda:
The Greek method for finding the equation of the tangent line to a circle used the fact that at any point on a circle the line containing the reauis and the tangent line are perpendicular. Use this method to find an equation of the tangent line to the circle x^2+y^2=9 at the point (1,2 square root of 2).
Answered by Penny Nom.
Complex roots 2010-04-14
From Lan:
If b and c are real numbers so that the polynomial (x^2) + bx + c has 1 + i as a zero, find b + c.
Answered by Penny Nom.
Polar coordinates 2010-04-14
From Lan:
Given the rectangular equation (x^2)+2x+(y^2)+y=0, find the polar equation.
Answered by Harley Weston.
Rational root theorem 2010-04-14
From deciree:
How could i get the value of x with this equation: 1/13x^3+12x/13=1/2 Thank you!
Answered by Harley Weston.
A conical pile of gravel 2010-04-13
From Chassity:
The gravel pile is 120' around at the base and goes up 20' high at the peak. How many tons or yards of gravel in that pile?
Answered by Penny Nom.
What are the dimensions of the box? 2010-04-13
From Steven:
The longest rod that will just fit inside a rectangular box, if placed diagonally top to bottom, is 17 inches. The box is 1 inch shorter and 3 inches longer than it is wide. How much must you cut off the rod so that it will lie flat in the bottom of the container? What are the dimensions of the box?
Answered by Robert Dawson.
Two problems 2010-04-13
From Dorothy:
1. Explain why the number 123, 456, 789, 101, 112 cannot be a perfect square. (Hint: What is the units digit?)

2. A substance doubles in volume every minute. At 9:00A.M., a small amount is placed in a container. At 10:00A.M., the container is just full. At what time was the container one-eighth full?

Answered by Robert Dawson.
Two overlapping circles 2010-04-12
From Scott:
There are two circles, big circle with radius R and small one with radius r. They intersect and overlap in such a way that the common area formed is 1/2 pi r^2 (half the area of the small circle). The Question is: suppose we have known the radius r of the small circle, and the distance between the two circle centers, what should the radius R of the large circle be?
Answered by Chris Fisher.
Integrate the ((4th root of x^3)+1) dx 2010-04-12
From Bridget:
integrate the ((4th root of x^3)+1) dx
Answered by Tyler Wood.
Volume of a cylinder 2010-04-12
From Louis:
A hockey puck has a diameter of 7.5 cm and a thickness of 2.5 cm. What is its Volume? A copper pipe with an inside diameter of 1cm. is 4m long. What volume of water (in cm3) can it hold?
Answered by Tyler Wood.
6 items are taken 3 at a time 2010-04-12
From Kristen:
Explain the three different scenarios that could occur and would result in the following number of outcomes when 6 items are taken 3 at a time. Give real-life examples.

a. 63 = 6 x 6 x 6 = 216 possibilities

b. 6! / (6 - 3)! = 6 x 5 x 4 = 120 possibilities

c. 6! / (6 - 3)! 3! = 20 possibilities

Answered by Tyler Wood.
A rectangle problem 2010-04-12
From Charlene:
The ratio of the width to the length of a certain rectangle is equal to the ratio of its length to the difference between its length and twice its width. If the area of the rectangle is 25cm^2, then what are its dimensions? Present a non- algebraic solution?
Answered by Tyler Wood.
The perimeter of a rectangle 2010-04-12
From Charlene:
The area of a rectangle is 390m^2. If its length is increased by 10m and its width is decreased by 6m, then its area does not change. Find the perimeter of the original rectangle.
Answered by Tyler Wood.
Vapor trails 2010-04-12
From Frank:
I'm not sure if this is a proper question to ask so if I have misdirected my question I apologize and no response is expected. I am trying to figure out a way to measure vapor trails from my back yard in Phoenix Arizona. If I used a compass and spread each point of the compass to the start and finish of the vapor trail I would have the angle of an isosceles triangle. The other two angles would be identical. The height of from the inverted base of the triangle to my standing spot on the ground would be about 35,000 feet. I'm thinking that there should be a way to figure out the length of the inverted base (vapor trail) but I'm devoid of mathematical skills and can't seem to figure out how to do this. Is it possible to figure out the length of a vapor trail using this method or do you have an easier way to accomplish the task?

Any help you could offer would be most appreciated.

Thanks....Frank

Answered by Harley Weston.
Exactly two lines of symmetry 2010-04-11
From debbie:
i am looking for a quadrilateral with exactly two lines of symmetry. please help! thank you.
Answered by Tyler Wood.
A cube with twice the volume 2010-04-11
From Woojin:
From a given cube create another cube with twice the volume. the cube: 5cm each.
Answered by Tyler Wood.
Working backwards 2010-04-11
From adel:
alice gave reggie and freddie as much money as each had already.reggie in turn gave alice and friddie as much money as each already had in negotiable treasury notes. freddie gave alice and reggie as much money as they each had. in the end they each had the same sum, 24000.00.how much money did they each in the beginning?
Answered by Penny Nom.
One red ball and one black ball 2010-04-11
From Sammy:
The probability of drawing a red ball out of a bag containing one red ball and one black ball is ½. An experiment is conducted where these two balls are placed in a bag before drawing a ball out. A ball is drawn five times from the bag containing the two balls. Each of the first five times the red ball was drawn out. What is the probability that the sixth draw will be a red ball? Explain your answer.
Answered by Penny Nom.
The derivative of y=x^x 2010-04-09
From David:
So, its David, and I was wondering about the derivative of y=x^x. I have often seen it be shown as x^x(ln(x)+1), but when I did it through limits it turned out differently. Here's what I did:
It is commonly know that df(x)/dx of a function is also the limit as h->0 of f(x+h)-f(x)/h. To do this for x^x you have to start with lim h->0 ((x+h)^(x+h)-x^x)/h. The binomial theorem then shows us that this is equal to lim h->0 (x^(x+h)+(x+h)x^(x+h-1)h+...-x^x)/h
This is also equal to lim a->0 lim h->0 (x^(x+a)+(x+h)x^(x+h-1)h...-x^x)/h.
Evaluating for a=0 you get lim h->0 (x^x+(x+h)x^(x+h-1)h...x^x)/h
Seeing as the last 2 terms on the numerator cancel out you can simplify to a numerator with h's is each of the terms, which you can then divide by h to get:
lim h->0 (x+h)x^(x+h-1)... which when evaluated for h=0 gives us: x(x^(x-1)). This statement is also equal to x^x.
This contradicts the definition of the derivative of x^x that is commonly shown. So, my question is: can you find any flaws in the logic of that procedure? I do not want to be shown how to differentiate x^x implicitly because I already know how to do that.

Answered by Robert Dawson.
Backfilling the area around the outside of a pool 2010-04-07
From Craig:
I need to get dirt to fill an area 25ft x 25ft (think of a giant sandbox) HOWEVER, there is a pool 18 ft round in the middle of it. We need to backfill the area around the outside of the pool 6" deep. How much dirt will I need?
Answered by Penny Nom.
The area of a field of view 2010-04-07
From Frank:
I know we have a field of view of 90 degrees and the distance we can cover is 6 miles. What is the area we can cover/view?
A formula would be nice to have since the diameter or distance of the wedge area can change and the degree of the field of view can also change.
Thank you and I look forward to hearing from you.
Frank

Answered by Penny Nom.
A model for the population of a Canadian city 2010-04-07
From jess:
Let t^2 represent t-squared. The population of a Canadian City is modelled by: f(t) = 12t^2 + 800t + 40,000, where t is the time in years. When t= 0, the year is 2007.
a) According to the model, what will the population be in 2010?
B) In what year is the population predicted to be 300,000?

Answered by Penny Nom.
Does 1= 0.9999....? 2010-04-07
From Asia:
Does 1= 0.9999....? There seems to be different opinions on this.
Answered by Robert Dawson.
An isosceles trapezoid is inscribed in a circle 2010-04-06
From Abby:
An isosceles trapezoid whose bases have lengths 12 and 16 is inscribed in a circle of radius 10. The center of the circle lies in the interior of the trapezoid. Find the area of the trapezoid
Answered by Penny Nom.
Related rates and a rectangular sponge 2010-04-06
From Heather:
A rectangular sponge is increasing its length at 4cm/min, decreasing its width at 2cm/min, and increasing its height at 3cm/min. When its length, width and height are 40, 30, and 20 respectively, find the rate of change of volume and surface area.
Answered by Penny Nom.
A max min problem 2010-04-06
From Terry:
The vertex of a right circular cone and the circular edge of its base lie on the surface of a sphere with a radius of 2m. Find the dimensions of the cone of maximum volume that can be inscribed in the sphere.
Answered by Harley Weston.
The derivative of cos^3x 2010-04-06
From Erson:
Find y' of the given function: y = cos^3x.
Answered by Harley Weston.
Body measurements 2010-04-06
From Amirul:
Recently I'm proposing my research question to my teacher for my extended essay. I'm an IB student. My research question is regarding the estimation of human in buying trousers through reference of neck. What does the relation between the diameter of the neck and the diameter of the waist? I want to see how far does the estimation theory is true for different type of people with different BMI(body mass index)..

But teacher said that it is golden ratio...so nothing interesting... =( really??? But i search on net.... state that my idea seems do not have any relation with the golden ratio so far..... i just want ask you... am I able to perform in my extended essay if i continue with this research question??

Answered by Robert Dawson.
The volume of a berm 2010-04-05
From jim:
a berm 6 ft wide x 112 ft long 18 inches high at center x 18 inches down to 0 front and back as a taper...how much soil do I need?
Answered by Penny Nom.
A regular hexagon and an equilateral triangle in a circle 2010-04-05
From Beth:
A regular hexagon and an equilateral triangle are both inscribed in the same circle so that the hexagon and the triangle share three vertices. The radius of the circle is 10cm. What is the difference between the area of the hexagon and the area of the triangle?
Answered by Chris Fisher.
Sand falling off a conveyer 2010-04-02
From Katherine:
sand is falling off a conveyer onto a pile at the rate of 1.5 cubic feet per minute. The diameter of the base is approximately twice the altitude. At what rate is the height of the pile changing when it is 10 feet high?
Answered by Penny Nom.
Pyramids and parallelograms 2010-04-01
From Vivi:
Hi, my name is Vivi, and math teacher is giving a geometry test. I tend to forget how to find the surface area and volume of triangular pyramids and parallelograms. Do you have a way of remembering how to find the surface area?
Answered by Harley Weston.
A ratios problem 2010-04-01
From Jules:
1a) If 2/3 of the boys and 3/4 of the girls at a party are dancing, what is the ratio of boys to girls, if all the dancing couples consist of a boy and girl?

b) If the total number of the boys and girls of the party is 153, how many boys are at the party?

Answered by Penny Nom.
Total tax 2010-03-31
From Keli:
if the gst in Ontario is 5% and the pst is 8% the total tax would be 13%. How come the total tax is 1.13? Shouldn't it be 0.13?
Answered by Robert Dawson and Tyler Wood.
Tiling a floor 2010-03-31
From shane:
a floor in a house is 12'6" in width and 10'4" in length. Tiling the floor with each tile 5" on each side. First express the square footage into an improper fraction. Second express the area of each tile in square feet. Third how many tiles needed to tile the floor. Fourth explain how answers relate to real world
Answered by Harley Weston.
The diagonals of a rectangle 2010-03-30
From Tabatha:
The diagonals of a rectangle are 8 units long and intersect at a 60 degree angle. Find the dimensions of the rectangle.
Answered by Penny Nom.
A system of equations 2010-03-30
From samantha:
the equations of two lines are 6x-y=4 and y=4x +2.what is the value of x in the solution for this system of equations?
Answered by Penny Nom.
A 25% discount 2010-03-29
From Susan:
If something costs 1.00/per lb. and that price includes a 25% discount. How do you calculate what the price was before the discount?
Answered by Penny Nom.
Sample size 2010-03-29
From Rae:
What sample size was needed to obtain an error range of 2% if the following statement was made? "75% of the workers support the proposed benefit package. These results are considered accurate to within + or - 2%, 18 out of 20 times. This seems like a straight forward question but I'm getting it wrong. Could you please help me out even just the set up would be appreciated so I can see if that's where I'm going wrong. Thanks
Answered by Harley Weston.
The sides of a 30-60-90 triangle 2010-03-29
From maris:
Find the sides of a 30-60-90 triangle whose perimeter is 6. The solution must use systems of equations using 3 variables.
Answered by Penny Nom.
The perimeter of a lake 2010-03-29
From Bill:
I am trying to figure out the approx. perimeter of a 22 acre lake. Can I solve this by calculating the sq. ft. in 22 acres and then take the square foot to find out the sides of a 22 acre square and then add the sides together to get the perimeter?
Answered by Penny Nom.
The area of a polygon 2010-03-28
From John:
Question from John, a student:

How do you find the area of a polygon with any given amount of sides?

Answered by Harley Weston.
Coins in 3 parking meters 2010-03-27
From Zena:
There are 227 coins altogether in 3 parking meters. The second meter has 37 more coins than the first meter. The third meter has twice as many coins as the second. How many coins in each meter?
Answered by Penny Nom.
The height of a hill 2010-03-26
From Amber:
A surveying team determines the height of a hill by placing a 12-foot pole at the top of the hill and measuring the angles of elevation to the bottom and to the top of the pole. They find the angels of elevation. Describe how to find the height of the hill.
Answered by Penny Nom.
A radio tower 2010-03-26
From Alex:
The height of a radio tower is 450 feet, and the ground on one side of the tower slopes upward at an angle of 10 degrees. How long should a guy wire be if it is to connect to the top of the tower and be secured at a point on the sloped side 110 feet from the base of the tower?
Answered by Harley Weston.
y = kx 2010-03-26
From harold:
if y =kx and y =4/15 and x=2/3, what is value of k
Answered by Penny Nom.
Conditional probability 2010-03-26
From Sandy:
In a certain school, it is known that 80% of the students use the internet for school projects, 60% use e-mail on a regular basis, and 90% use the internet for school projects or for e-mail on a regular basis. A student from this school is selected at random

Determine the probability that the student used e-mail, given that the student used the internet for school projects.

Sandy

Answered by Penny Nom.
A ladder against a wall 2010-03-25
From amber:
the distance from the bottom of the ladder to the building is18 foot less than the length of the ladder.How high up the building is the top of the ladder if that distance is 1 ft less than the length of the ladder?
Answered by Penny Nom.
A proof by induction 2010-03-25
From SAMUEL:
use mathematical induction to proof that each statement is true for every positve integer n 1/1.2+1/2.3+1/3.4+......1/n(n+1)=n/n+1
Answered by Robert Dawson.
A roll of paper 2010-03-25
From keith:
how many yds of paper for a roll with a width of 6.875 and a core of 3 in and a diameter of 40in?
Answered by Robert Dawson.
The angle of intersection between two lines 2010-03-25
From Madhumitha:
How to find the angle of intersection between a line x=249 and another line which is of the form y=m*x+c; Line1 has a slope of infinity so what does the angle of intersection turn out to be? Or how can i find it?
Answered by Chris Fisher.
The line passing through (2,6) and (2,9) 2010-03-24
From antoinette:
the equation of a line passing through the points (2,6) and (2,9)?
Answered by Penny Nom.
A rectangle inscribed in a circle 2010-03-24
From sadiq:
here is the question, in my mathematics book there is equation of the area of the rectangle inscribed in a circle having equation x^2+y^2=a^2 and the area of rectangle is 4xy=4x(a^2-b^2)^1/2 i don't know what is b but a is surely the radius (i want the derivation for the area of rectangle).
Answered by Harley Weston.
How many 20"X20" tiles do I need to cover 90 sq. ft? 2010-03-23
From Karen:
How many 20"X20" tiles do I need to cover 90 sq. ft.
Answered by Penny Nom.
Modelling the roof of a house 2010-03-23
From Sandi:
This question has been haunting my dreams at night. It states the cross section of the roof of a house is modelled by the function y= -5\12|x-12|+5, where y>0 or y. Now I'm suppose to graph the function. b) Find the slope, height, length of sides, and base of the isosceles triangle. c)Explain what transformation must be applied to the graph of y=|x| to obtain the graph of y= -5\12|x-12|+5, where y>0 or y=0. 1st I put the equation into my calculator and graphed it. I got 10 for the height. We have had 2 sub teachers this past week. The first one told me the height should be 12 the other one told me 5. So I'm really confused. If I use my 10 units as height and the Pythagorean Theorem I get my sides to be 26 units the base 48 and my slope I calc to be 5/12. Both teachers are pretty sure I'm wrong and they are right but we all have different answers. Please help put this problem to rest so I can rest soundly at night. Thank you for your time.
Answered by Penny Nom.
Factorials 2010-03-23
From Leah:
When should you use factorials?
Answered by Robert Dawson.
A champagne pyramid 2010-03-22
From Kathy:
I have 680 champagne glasses and my pyramid can only be 15 levels high. How many glasses need to be on each level? Is there a formula I can use?
Answered by Robert Dawson.
Sand in an hourglass 2010-03-20
From Luke:
question
Answered by Harley Weston.
A quadrilateral with 4 known sides and 1 known angle 2010-03-19
From samuel:
Name: Samuel
Status: Student

I have a quadrilateral with 4 known sides and 1 known angle, and I'm trying to evaluate the other angles of my quadrilateral.

By the law of cosines, I can easily find my opposite angle (using the diagonal as a basis for the equation).

However, to find the two remaining angles, I have found no other way so far than to use the other diagonal, which can be found with the equation attached (from geometry atlas).

Is there any simpler way?

Answered by Robert Dawson and Harley Weston.
Binomial probability 2010-03-19
From Rana:
According to one study, it has been claimed that 65% of all single men in Montreal would welcome a woman taking the initiative in asking for a date. You decide to challenge this study by performing a little experiment of your own. You randomly select 15 single men from around Montreal and ask them if they would be comfortable with a woman asking them out for a date. If the claim made by the study was true, what is the probability that:
a) Exactly 9 men would say yes?
b) Exactly 6 men would say no?
c) More than 11 men would say yes?
d) At most 4 men would say no?

Answered by Penny Nom.
Water in a culvert 2010-03-18
From Chip:
Hello, I have a problem to which I know there must be an analytical solution - but as it has been 50 years since I studied math I can't remember quite how to do it!! I have a 12" round culvert in my yard that runs water all year. I would like to be able to calculate the flow through the culvert by measuring the depth of water flowing through and measuring the speed of the current. I can measure the depth and the speed, but I forget how to calculate the cross section of the flow. If you could help I would greatly appreciate it. Thank you
Answered by Penny Nom.
Factors and zeros of a polynoial 2010-03-17
From Susan:
find a polynomial function of degree three with -3 as a zero of multiplicity 2 and 4 as a zero of multiplicity 1.
Answered by Tyler Wood.
Angular velocity 2010-03-17
From Lindsay:
Determine the angular velocity, in radians per second, of a 14-inch car wheel, if the car is traveling at 60 miles per hour
Answered by Harley Weston.
What is the speed of the current? 2010-03-17
From Carl:
If you can row 3 mph in still water and paddle 3 miles upstream and return in 3.6 hours then what is the speed of the current?
Answered by Penny Nom.
A rectangular box packed with cylinders 2010-03-16
From Laura:
There is a cylinder which has a diameter of 10cm and height of 10 cm. Work out the volume of the cylinder. Then in a packed box of 20 cylinders (height of box 10cm) work out the volume inside the box that is not filled by the cylinders give your answer in terms of pi.
Answered by Penny Nom.
Decimals and fractions 2010-03-16
From fay:
why do we use decimals in place of fractions in everyday life?
Answered by Robert Dawson.
A quadratic equation 2010-03-15
From rachelle:
under certain condition, it is found that the cost of operating an automobile as a function of speed is approximated by a quadratic equation. use the data shown below to find the function. then use the function to determine the cost of operating the automobile at 60mph.
speed
(in miles per hour)
operating cost
per mile(in cents)
10 22
20 20
50 20

Answered by Penny Nom.
Gina's weight was 4/7 of Helen's weight 2010-03-14
From Eileen:
At first, Gina's weight was 4/7 of Helen's weight. Then, Gina gained 3kg while Helen lost 6kg. They are the same weight now. What is Helen's weight now?
Answered by Penny Nom.
Repeating Decimal 2010-03-14
From Gerald:
Find the 1987th digit in the decimal equivalent to 1785/9999 starting from decimal point. Can you give us a short but powerful technique in solving this problem? thanks so much..
Answered by Chris Fisher.
A red boat and a blue boat 2010-03-13
From SERENA:
Two fishing boats set out at 2pm for some afternoon fishing. Both boats pulled out from the same dock heading in the same direction. The average speed of the blue boat is 30mph slower then twice the speed of the red boat. In two hours, the blue boat is 20 miles ahead of the red boat. Find the rate of the blue boat.
Answered by Penny Nom.
Arc length and Chord length 2010-03-13
From Darryl:
Is there a formula to determine the chord length of an arc knowing only the arc length and the arc depth (sagitta)? I know you can't find the radius with only these two inputs, but can you find the chord length?
Answered by Harley Weston.
The perimeter of a rectangle 2010-03-12
From makiah:
A rectangle has a perimeter of 12 meters.If each side is a whole number of meters, what are the possible dimensions for the length and width?
Answered by Penny Nom.
A 14 side well house cover 2010-03-12
From Kenneth:
I am 35 yr I am wanting to build a well house cover. I'm trying to figure out how long the pieces need to be and what angle they need to be for a 4 ft dia with 14 side well house. I would love an answer but would also like to know how to figure it in the future. Thanks Kd
Answered by Harley Weston.
Selling lettuce 2010-03-11
From Mike:
Day 1 started with 80 heads of lettuce and x were sold.
Day 2 started with two times the number of heads of lettuce that was left over. Again, only x were sold.
Day 3 started with 3 times the number of lettuce from what was left over from day 2 and all was sold.
What was the # of lettuce sold on each day?

Answered by Penny Nom.
The nth derivative of x^(n-1) log x 2010-03-10
From shambodeb:
This is a successive differentiation problem by Leibnitz theorem

If y = xn-1 log x ; Proof nth derivative y(n) = (n-1)!/x

Answered by Harley Weston.
A 6m ladder is placed against a wall 2010-03-09
From Trevor:
A 6m ladder is placed against a wall making a 56 degree angle with the ground. How far up the wall does the ladder reach?
Answered by Harley Weston.
Selecting 3 students from 16 2010-03-09
From amper:
A class has 12 boys and 4 girls. If three students are selected at random from the class, what is the probability that they are all boys?
Answered by Penny Nom.
An irregular octagon 2010-03-09
From Gayle:
Question from Gayle:

I am building an irregular shaped octagon wooden box.
The measurements are 291/2 inches by 211/2 inches.
Sides are 12 inches.
It will be 36 inches high.

What would the cutting angles degrees be?

Answered by Harley Weston.
Log[f(x)] 2010-03-08
From sourabh:
What is the number of solutions of the equation 9 x 2 - 18 x + 5 = 0 for x, such that the expression log10[(x+1)(x+2)] exists?
Answered by Harley Weston.
The side length of a square 2010-03-08
From tara:
am trying to help my brother's daughter in maths, tomorrow she has maths exam 1. Area of sq. picture is 441 sq.cm what is the length of its side.
Answered by Penny Nom.
0.999 ^ (500) 2010-03-07
From debra:
I just need to know how to solve the following problem without using a calculator: .999 ^ (500). I know the answer is .606, I just want to do it by hand since I can't use a calculator on my test.
Answered by Penny Nom and Claude Tardif.
The integral of X^3/the square root of 1-x^2 dx 2010-03-07
From William:
The integral of X^3/the square root of 1-x^2 dx.
Answered by Harley Weston.
Two overlapping circles 2010-03-07
From Hayden:
I have two circles of equal size. The radiuses of the circles are 30ft. The two circles are positioned 40ft apart and I need to find the area where they overlap.
Answered by Harley Weston and Tyler Wood.
Cooling 2010-03-07
From Lori:
If a house is always at 20 degrees celsius. Water (tea) boils at 100 degrees celsius. 5 minutes later the tea was 70 degrees celsius. Use an equation to predict the temperature after 20 minutes.
Answered by Tyler Wood.
The line joining (3,4) and (-5,-6) 2010-03-06
From Patric:
Find the condition that the point (x,y) may lie on the line joining the points (3,4) and (-5,-6)
Answered by Penny Nom.
The distance travelled by a minute hand 2010-03-06
From Patric:
How much distance will the minutes hand of length 14mm of a clock cover in moving from 5 to 10?
Answered by Penny Nom.
Redwood Park and Glendale Park 2010-03-05
From Shaun:
Question from Shaun, a student:

Redwood and Glendale Parks are each surrounded by 100 yards of fencing. The are of Redwood Park is 25 square yards more than the area of Glendale Park. If both parks are rectangles, what is the measurement of each rectangle?

Answered by Harley Weston.
How many people in total came to the museum that day? 2010-03-05
From simon:
1354 people attended exhibition A and 1427 people attended exhibition B. 79% of people attended both exhibitions. How many people in total came to the museum that day
Answered by Penny Nom.
Standard form 2010-03-05
From Laura:
How do I change 3y=4x+1 into standard Ax+By=C standard form. This is my son's schoolwork. I homeschool him. I got the answer to the problem as 3y=4x+1, but it needs to be written in standard form and I know it needs to read 4x-3y=-1, but how do I get that answer into standard form. I need to know how to explain it to him. Can you show me? Thank you so much.
Answered by Penny Nom.
The base area of rectangular prism 2010-03-05
From Desiree:
How do you find the base area of rectangular prism that has 4 cm by 4cm by 6cm ?
Answered by Penny Nom.
The area of a new parking lot 2010-03-04
From nayle:
The owner of a parking lot at the beach wants to increase the parking area of the lot. If the owner increases each of the dimensions of the rectangular lot by half its original value, by what factor will the parking lot increase?
Answered by Penny Nom.
The volume of a frusta of a hexagonal based pyramid 2010-03-04
From sarah:
Volume of a frusta of a hexagonal based pyramid
Answered by Penny Nom.
At what time would Joan catch Jo? 2010-03-04
From Rusty:
If Jo left his house at 6:30am traveling at 5mph and Joan left the same house 7:00am traveling at 8mph, at what time would Joan catch Jo?
Answered by Penny Nom.
Lissajous curve 2010-03-03
From Nikki:
I'm interested in information about a particular mathematical figure. My memory is that it is called a "liciju figure", but obviously my spelling of this is incorrect because a google search of this and it's variants has revealed nothing. I believe it's related to the Moebius strip and probably connected with radio waves. It is used as the logo for our national broadcaster (The Australian Broadcasting Corporation) and you see exactly what I'm talking about by going on their website: www.abc.net.au. I have tried contacting them directly, but have received no response in over a month now!
Answered by Harley Weston.
A related rates problem 2010-03-03
From Amanda:
A circle is inscribed in a square. The circumference of the circle is increasing at a rate of 6 inches per second. As the circle expands, the square expands to maintain the tangency. Determine the rate at which the area of the region between the circle and square is changing at the moment when the cricle has an area of 25(pi) square inches.
Answered by Penny Nom.
The dimensions of a rectangle 2010-03-03
From Kirsten:
A rectangle is twice as long as it is wide. Its area is 128 square meters. What are the dimensions of the rectangle?
Answered by Tyler Wood.
A diagonal of an octagon 2010-03-02
From cody:
I have an octagon 3ft tall by 3ft wide. What are the lengths of the diagonal lines to the vertical and horizontal lines (b and b)?
Answered by Penny Nom.
Sketch the graphs of the following 2010-03-02
From musaf:
without detailed plotting of points,sketch the graphs of the following showing relevant information on the graphs:
a) y=(x-3)2 +5
b)y=4x-x2

Answered by Penny Nom.
An octagon garbage storage bin 2010-03-02
From sharon:
I am trying to make a octagon garbage storage bin and i need to know the measurement of the outer and iner sides i can't get them to fit together what am i doing wrong
Answered by Penny Nom.
Two adjacent rectangular dog pens 2010-03-01
From Janet:
Clint is constructing two adjacent rectangular dog pens. Each pen will be three times as long as it is wide, and the pens will share a common long side. If Clint has 65 ft of fencing, what are the dimensions of each pen?
Answered by Penny Nom.
Zeros of a polynomial 2010-03-01
From Gavin:
Suppose the polynomial R(x) = a_9x^9+a_8x^8+...+a_1x+a_0 has real coefficients with a_9≠0. Suppose also that R(x) has the following zeros:
2,
3,
i

Using this info, answer the following.

a. What is another zero of R(x)?
b. At most how many real zeros of R(x) are there?
c. At most how many imaginary zeros of R(x) are there?

p.s. I used _ for subscript
thanks so much

Answered by Harley Weston.
2+3 =10 and 7+2=63 and ... 2010-03-01
From brenda:
if 2+3 =10 and 7+2=63 and 6+5 =66 and 8 +4 = 96 then 9+7+ ??????
its driving me crazy !!

Answered by Penny Nom, Claude tardif and Melanie Tyrer.
How many CDs and videos did the store sell? 2010-03-01
From dawn:
A used book store started selling CDs and videos. In the first week,the store sold 40 used CDs and videos,at 4.00 per CD and 6.00 per video.The sales for both CDs and videos totaled 180.00 she wrote a system of equations to represent the situation.Then she graph the system of equations

thanks- Dawn

Answered by Tyler Wood.
The binomial distribution 2010-02-27
From Jessica:
A small mobile phone retailer has found that one of their phones has a 12% probability of being faulty and a replacement having to be provided for the customer. They have just received a trial order for 10 phones from their biggest customer who will take their business elsewhere if 20% or more items are faulty.

i) what is the probability that they will lose their biggest customer?

Answered by Penny Nom.
The hypotenuse 2010-02-27
From Dannielle:
how do you find the hypotenuse if a=8 and b=6?
Answered by Penny Nom.
A 25 foot ladder reaches a window 20 feet high 2010-02-27
From Tanner:
a 25 foot ladder reaches a window 20 feet high. how far is the ladder from the building? how far must the foot of the ladder be moved to lower the top of the ladder by four feet?
Answered by Tyler Wood.
Proportionality 2010-02-27
From Nazrul:
If a is proportional to b then how can I prove that (a^2-b^2) is proportional to ab. Please show me the process in details.
Answered by ms. Dame and Tyler Wood.
Four dots are randomly placed on an 8x8 grid 2010-02-26
From bobbym:
Four dots are randomly placed on an 8x8 grid, compute the probability that no row or column contains more than one dot.
Answered by Claude Tardif.
If the earth were covered completely with water 2010-02-25
From don:
If the earth were covered completely with water (over Everest) how many gallons would it be??
Answered by Penny Nom.
The test for some disease is 99% accurate 2010-02-24
From baaba:
Assume that the test for some disease is 99% accurate. If somebody tests positive for that disease, is there a 99% chance that they have the disease?
Answered by Chris Fisher.
A linear relationship 2010-02-24
From Mel:
This table shows the linear relationship between an aeroplane's height (above sea level) and the outside air temperature.
h (cm) 120 340 480 600 1040

T (degrees C)

14.28 12.96 12.12 11.4 8.76
Find the gradient and vertical intercept of the function. Write the formula for T.

Answered by Penny Nom.
A mixture problem 2010-02-24
From Nathan:
How many gallons of 60% solution must be added to 40 gallons of a 10% solution to produce a 20% solution?
Answered by Penny Nom.
Is 0.1234567891011.... periodic? 2010-02-23
From Adriana:
0.1234567891011.......Is this fraction periodic ?
Answered by Penny Nom.
Dividing between between two charities 2010-02-22
From Latoya:
a 149,000 estate is to be divided between two charities so that one charity receives 18,000 less than the other. How much will each charity receive?
Answered by Penny Nom.
How many triangles...? 2010-02-22
From deciree:
Given 12 points, no 3 of them on a line with 6 red, 4 blue and 2 green points.
a) How many triangles have vertices all the same color?
b)How many triangles have vertices with each vertex a different color?
c)How many triangles have at least one green vertex?

thanks to lorraine!

Answered by Penny Nom.
A paper tube 2010-02-21
From shailesh:
How can i estimate weight of paper tube from size. i have paper tube inner diameter : 22mm, thickness : 2mm, length : 14mm. so how can i got weight of paper tube?
Answered by Harley Weston and Tyler Wood.
The game of 24 2010-02-20
From haley:
hello.... we have these very hard make 24 stumpers. i am stuck on one.... how can you make 24 using the numbers 7-7-13-18?? thanks!
Answered by Claude Tardif.
15 men can do a piece of work in 7 days 2010-02-20
From Kenneth:
If 63 books cost $126, what will 125 books cost?

If 15 men can do a piece of work in 7 days, in how many days can 21 men do the same work?

Answered by Penny Nom.
Frank makes picture frames 2010-02-20
From Carol:
Frank makes picture frames his revenue from sales is R=$13.60x for frames sold. The cost making frames is C=$5.80x+$120 for frames made. What is the minum number of frames Frank must make and sell in order for his revenue to be greater than his costs.
Answered by Penny Nom and Melanie Tyrer.
An algebraic equation with fractions 2010-02-19
From Ingrid:
What is the solution set of the equation x over x plus 4 = 1 over x plus 3 = 28 over x to the 2nd power - x- 12?
Answered by Penny Nom.
Triangles 2010-02-19
From Wiliam:
A box contains one 2 inch rod, one 3 inch rod, one 4 inch, and one 5 inch rod. What is the maximum number of different triangles that can be made using these rods as sides?
Answered by Penny Nom.
1/6 x 5 x 12 = what 2010-02-19
From dana:
1/6 x 5 x 12 = what
Answered by Lorraine Dame.
7 odd numbers 2010-02-19
From mike:
Hello there, Problem: find 7 odd numbers who sum is equal to 30. Hint*My professor said it's possible to add 7 odd numbers and get 30 for an answer! and he said we can use Negative and rational numbers but no decimal or fractions. So is there anyway of solving this? Please get back to ASAP! thanks.
Answered by Harley Weston and Tyler Wood.
The vertices of an equilateral triangle 2010-02-19
From Emma:
If two of the vertices of an equilateral triangle are (2,1) and (6,5), what are the two possible coordinates of the third side of the triangle? Answer in radical form.
Answered by Tyler Wood.
A prism and a pyramid 2010-02-19
From Juanita:
What is the relationship between the volume of a prism and a pyramid with congruent base areas and heights?
Answered by Tyler Wood.
The equation of a circle 2010-02-18
From AHMED:
find the equation to the circle with centre at the point (1,-1) and touching the straight line 5x+12y=7.
Answered by Penny Nom.
A magic square 2010-02-18
From Mika:
place the integers from -5 to +10 in the magic square so that the total of each row, column, and diagonal is 10.
Answered by Tyler Wood.
Substitution method 2010-02-17
From Melodie:
7x - 4y = -10
4y= x-2

Answered by Tyler Wood.
The center of a rectangular room 2010-02-16
From Diana:
Consider a rectangular room, 15 feet wide, 30 feet long and 12 feet high. What is the exact distance from any of the 8 corners of the room and its geometric center? Can you write a generic formula for such a distance? And if we keep the same proportions on all dimensions, can you write an expression for the same distance as a function of the floor perimeter?
Answered by Penny Nom.
The dimensions of a flag 2010-02-15
From dawn:
a state flag was twice as long as it was wide and had an area of 288 sq inches, what were its, dimensions
Thanks

Answered by Penny Nom.
The difference of the two numbers 2010-02-15
From Steve:
The difference of the two numbers 'abcdef ' and ' fdebca ' is divisible by 271. prove that b = d and c = e.
Answered by Claude Tardif.
What time did susan go to bed? 2010-02-14
From Sophia:
Susan woke up at 7:42 Am to get ready for school. She slept for 10 and 1/3 hours. What time did she go to bed?
Answered by Stephen La Rocque.
Can a line segment curve over two planes? 2010-02-14
From Graham:
I am working on a math fair project. Can a line segment curve over two planes? Such as, if I had a three dimensional L bracket and I drew a line segment on it with a marker starting on the bottom of the L and had it curve around the corner and up the top, would it still be considered one line segment? Or is that two line segments? Is there a rule that a line segment can only occupy one plane? Thank you. Graham
Answered by Chris Fisher.
Substitution 2010-02-14
From Izayah:
what is 3 ways to find the meeting point of y=2x+4 and 3x+y=7 by making 2 table and using a graph
Answered by Stephen La Rocque.
Birds and cats 2010-02-13
From Sophia:
While visiting a pet store, you notice that there are only birds and cats in the cages. You can't help but wonder how many of each animal there is in the yard. But when you ask the store manager how many of each animal he has, he refuses to give a direct answer. He says there are 16 animal heads and 42 animal feet. How many birds and cats are there in the pet store? It said to show all your work. Explain in words how you found your answer. Tell why you took the steps you did to solve the problem.
Answered by Penny Nom.
Least common denominator 2010-02-13
From Priscila:
3/8 + 4/5 + 7/3 + 9/10 = ?

Thank you for your assistance.

Priscila

Answered by Penny Nom.
Integration of cos (x^2+y^2) dx dy 2010-02-12
From anna:
How do you evaluate the integration of integration of cos (x^2+y^2) dx dy?
Answered by Tyler Wood.
Melting snow 2010-02-11
From Cyndi:
How long will it take for 5 feet of snow in an area of 17 by 12 to melt at 35 degrees and cloudy.
Answered by Robert Dawson.
A square corner 2010-02-11
From Trevor:
I am building a new house and wish to set it out on site with the use of profile boards and string. I want to be certain it is correct in terms of squareness. I have a vague idea that the square on the hypotenuse should be equal to the sum of the squares of the other two sides.

I get a little lost here and need some help. The building is a rectangle measuring 40x30 feet to exterior brickwork. I guess that the length of the hypotenuse should be exactly the square root of the combined squares of the two sides.

Using the above measurements could you give me calculations from nuts to soup as to the correct length of the diagonal. And what adjustments are needed if everthing is not in accord.

Trevor.

Answered by Robert Dawson.
An isosceles triangle 2010-02-11
From Kim:
I am given the length of the two legs of an isosceles triangle (8), and the base angles are 30 degrees...I am asked to find the area of the triangle with only this information
Answered by Tyler Wood.
|-3|-|-4| 2010-02-10
From Nick:
I have another question. This is how i solved it and i know its suppose to be a negative.

z= |-3|-|-4|

Answered by Penny Nom.
The angle at which the road is inclined 2010-02-10
From abeth:
a mountain road drops 5 m for every 22 m of road. Calculate the angle at which the road is inclined to the horizontal to the nearest degree.
Answered by Penny Nom.
The other two sides of a right angle triangle 2010-02-09
From ayesha:
how to find the other two sides of right angle triangle when length of one side and angle of other side is given i.e 45
Answered by Penny Nom.
A dirt berm 2010-02-05
From casey:
My question is two fold. I want to build a dirt berm that is 95 feet long and 8 feet high which has a 45 degree angle on each side of the slope. My first question is how wide will the berm end up being? Second question is how many cubic yards of dirt will I need to get? Thanks!
Answered by Harley Weston.
I want to convert my glaze percents into grams 2010-02-05
From linda:
I am a potter. I want to convert my glaze percents into grams. ex. 20% of 20 lbs would equal how many grams? is there a formula to figure this?
Answered by Penny Nom.
The perimeter of a semicircle 2010-02-04
From Sarah:
The perimeter of a semicircle is doubled when the radius is increased by 7. Find the radius of the semicircle.
Answered by Tyler Wood.
Fencing a garden 2010-02-04
From kris:
How many meters of fencing do i need to enclose a garden if the yard is 26 meters long and half as wide?
Answered by Penny Nom.
Bending a wire to form shapes 2010-02-04
From Geraldine:
a wire bent into the shape of a square encloses an area of 25cm squared. then the same wire is cut and bent into two identical circles. what is the radius of one of the circles round to the nearest hundred
Answered by Penny Nom.
A sector of a circle 2010-02-04
From trisha:
Given a circle with a radius of 6 inches. Find the area of the sector that is formed by an arc of 36 degrees. Round your answer to the nearest tenth of an inch.
Answered by Penny Nom.
Consecutive even integers 2010-02-03
From cassandra:
find 2 consecutive even integers whose sum is 34

find 3 consecutive even integers such that 3 time the middle interger is 10 more than the greatest.

Answered by Tyler Wood.
Mixing oil 2010-02-03
From Andrew:
A dealer wishes to produce 300 gallons of oil worth 40cents a quart. He has oil worth 36cents a quart and oil worth 52cents a quart. How many quarts of each should he mix?
Answered by Penny Nom.
Two bus routes 2010-02-03
From kiyah:
from 4:30 pm to 6:30 pm the route 1 bus stops every 12 min at the gym's bus stop. the route 2 bus stops there every 15 min. if both buses are now at the stop and schedule is kept, how long will it be before both buses will be at the stop again?
Answered by Penny Nom.
A snail on an elastic band 2010-02-03
From Joe:
A snail starts crawling from one end along a uniformly stretched elastic band. It crawls at a rate of 1 foot per minute. The band is initially 10 feet long and is instantaneously and uniformly stretched an additional 10 feet at the end of each minute. The snail maintains his grip on the band during the instant of each stretch. At what points in time is the snail
(a) closest to the far end of the band, and
(b) farthest from the far end of the band?

Answered by Robert Dawson.
A piece of string 8 inches long is cut in two pieces 2010-02-02
From Susan:
If you have a piece of string 8 inches long and cut it in two pieces. Then take the pieces and forms each into a square such that the area of each squares added together equals 2. How long are the sides of the squares?

Susan

Answered by Penny Nom.
Scale factor 2010-02-02
From Darlene:
A towers actual height is 300m. An architect makes a scale drawing using a scale factor of 0.0001

A) how tall is the tower in the drawing?

B) if the drawing was 4cm high, what would the scale factor be?

Answered by Penny Nom.
1/a + 1/b + 1/c < 1 2010-02-01
From Mrityun:
suppose a,b and c are natural numbers such that 1/a + 1/b + 1/c < 1. Prove that

1/a + 1/b + 1/c < = 41/42.

Answered by Penny Nom and Claude Tardif.
Everything in a cube 2 miles long? 2010-01-31
From Naresh:
in a book, i got to read this :

Is it possible to pack the entire population of earth and everything that was created by humankind in a cube whose edges are 2 miles long ?

Is it possible? Thanks.

Answered by Stephen La Rocque.
The height of a roof 2010-01-31
From carl:
Width of my roof I am building is 5M at baseline, and the pitch is 40%. What will the height be, and how can I work this out in the future.
Answered by Penny Nom.
An impossible isosceles triangle 2010-01-31
From Hailey:
An isosceles triangle has one angle that measures 50 degrees and another that measures 70 degrees. Why can't this triangle be drawn?
Answered by Penny Nom.
A 3D cardboard cupcake 2010-01-30
From Margaret:
Hi,
I'm an art student and I'm attempting to build a 3D cupcake out of cardboard. I want it to have a circular base of 8.5 inches and sides that are 7'' tall and slope outward so the top of the base is 29'. The top with the frosting would be a detachable lid made from a cone with a base circumfrance of 29". My problem is how to cut the side so they will slope out, I'm pretty sure there needs to be a curve, however I don't how to calculate the degree of it. Ideally I want the sides to be a single piece of cardboard. I also don't know how to calculate the arc needed to make a cone who's circular base's circumfrence is 29''.
Thanks,
Margaret

Answered by Stephen La Rocque.
A decrease in travel time 2010-01-30
From Mike:
if a 100 mile trip averages 50 miles per hour, how much distance does one need to travel at 90 miles per hour to decrease travel time by 10%
Answered by Stephen La Rocque.
Aggregate must for a road base 2010-01-30
From Bryan:
How many cubic yards of aggregate must be ordered for a road base 16 in. thick and 2 miles long with a top width of 30 ft, if the side slopes are one on one, or 45 degrees
Answered by Stephen La Rocque.
An integral 2010-01-29
From amr:
find the value of the following integral:- (1+x)power (-1/2) dx
Answered by Robert Dawson.
Working with decimals 2010-01-29
From linda:
Write 3 numbers that are between the following:

2.15 meters and
2.17 meters

Answered by Harley Weston.
An isosceles triangle 2010-01-28
From Jazzy:
An isosceles triangle is a 2 congruent (equal) sides. Of the third side is three times the length of the congruent side(s), and the perimeter is 75 cm, find the length of all three sides of the triangle
Answered by Penny Nom.
Linear growth 2010-01-27
From S.:
I. Linear Growth:
Suppose that the town has a fixed increase in population growth number of population increase each year.

Choose the amount of population growth each year = 200 (Hint: Choose a whole number for your growth rate, rather than a percent.)

a) Fill in the following chart:

Year (t) Population (P)
t = 0
(2010) 1000

t = 1
(2011) 1000+200=1200

t = 2
(2012) 1000+400=1400

t = 3
(2013) 1000+600=1600

b) Find a linear equation in the form P = mt + b (y = mx + b), which gives the population, P, t years from 2010.

.
.
.

Answered by Melanie Tyrer.
Packing spheres in a given region 2010-01-27
From MISAEL:
IN A VOLUME GENERATED BY TWO TRAPEZOIDS WITH DIFFERENT SURFACE AREAS BUT SHARING THE BOTTOM PLANE: HOW CAN I CALCULATE THE "N" SPHERES WHICH THIS VOLUME CAN CONTAIN? HOW CAN I CALCULATE THE VOLUME OF SPACES IN RELATIONSHIP WITH VOLUME OF SPHERES OR THE TOTAL VOLUME?
Answered by Robert Dawson.
The perimeter of a rectangle 2010-01-27
From debbie:
how many dimensions are there to get a perimeter of 400 feet for a rectangle?
Answered by Penny Nom.
What is the length of my sidewalk? 2010-01-27
From Les:
Name: Les
Who are you: Student

I have a sidewalk 4"thick,4ft. wide,and I have 2cubic yards of concrete. What is the length of my sidewalk?

Answered by Penny Nom.
Growth rates 2010-01-26
From Bhavya:
Dear Sir/Ma'am,

I read in the text book that the growth rates of these 3 functions are as below- n2/3 < n/lg n < n0.99

I tried the substitution method to check the correctness of this. But it gets really tough as n increase.

Is there any simpler way to understand the correctness of these growth rates?

Thanks and Regards,
Bhavya

Answered by Robert Dawson.
Mill 2010-01-26
From Kenneth:
Is a mill, as it pertains to property taxes, regarded as a unit of measure like a meter or an hour, etc?
Answered by Robert Dawson.
A 30% markup and a 20% sale 2010-01-25
From stephanie:
A car is marked up 30% from its wholesale price.During a sale the retail price is reduced by 20% .What percent higher is the sale price from the wholesale price.
Answered by Penny Nom.
A cylinder is cut at an angle of 20 degrees 2010-01-25
From sanjay:
If I have a cylinder with 40mm diameter and I will cut it at an angle 20 degrees so what would be the increased radius or dia.??? of the cylinder
Answered by Penny Nom.
A tunnel from Toronto to Montreal 2010-01-25
From Dave:
I want to make a tunnel from Toronto to Montreal (for example)

Something like this http://mathcentral.uregina.ca/QQ/database/QQ.09.09/h/grant1.html
-------------
My coordinates are 45.442455,-73.861340 (Montreal) and 43.442455, -79.861340 (Toronto)

I need to know how to find arc distance, chord distance and radius What websites can i find for this subject Google has many but they are useless (blah blah) websites
LOL
Thanks

Answered by Chris Fisher and Robert Dawson.
A triangle in a rectangle 2010-01-25
From Sangya:
The perimeter of the rectangle is 21 inches. The perimeter of the inscribed triangle is 21 inches. Find the dimensions of the rectangle
Answered by Robert Dawson.
Sample variance 2010-01-25
From shodhan:
A series of 100 measurements of a physical quantity have been made that show a random fluctions characterized by a sample variance of 2% of teh mean value. If the series is lengthed to 1000 measurements made under the same conditions, estimates the sample variance of the larger set of data.
Answered by Robert Dawson.
Solving a triangle 2010-01-25
From Paige:
how do i solve a triangle with one angle of 73 degrees, one angle of 32 degrees, and one side of 23cm?
Answered by Harley Weston.
An altitude of the triangle intersect a side of the triangle 2010-01-23
From Sarah:
In what type of triangle does an altitude of the triangle intersect a side of the triangle at its midpoint??
-Sarah

Answered by Walter Whiteley.
The binomial theorem 2010-01-21
From Laura:
Using the fact that (1 + x)^4 * (1 + x)^9 = (1 + x)^13 show (4C0 * 9C4 + 4C1*9C3 + 4C2*9C2 + 4C3*9C1 + 4C4*9C0) = 13C4
Answered by Harley Weston.
The volume of a silo 2010-01-21
From heather:
The height of the silo is 30ft and the face that rests against a barn is 10 ft wide. If the barn if 5 ft from the center of the silo what is the capacity of the silo?
Answered by Penny Nom.
Odds and evens in an n by n+1 table 2010-01-21
From Shankar:
The boxes of an n * (n+1) table ( n rows and n+1 columns) are filled with integers. Prove that one can cross out several columns ( not all of them !) so that after this operation all the sums of the numbers in each row will be even.
Answered by Robert Dawson.
The inradius of an icosahedron 2010-01-21
From fnavroth:
Suppose you have an equilateral triangle. The area of the triangle is exactly 1200 square centimetres. Now suppose you have twenty of those triangles. It's possible to assemble those twenty triangles into a closed three-dimensional shape, a regular polyhedron. What would be the volume, in cubic centimetres, of the largest sphere that could fit inside the shape?
Answered by Chris Fisher.
Angle of incline 2010-01-20
From Alan:
how do I fnd the angle of an incline with a measurement of 0.042 with an adjacent of 1.2mtrs?. Thank You
Answered by Penny Nom.
Loading a headboard in a uhaul 2010-01-20
From gina:
I have a uhaul 9'10" long 4"9"wide 4'7"height I have a headboard 74 inches in height would it fit the box diagonally
Answered by Penny Nom.
divide the product of 'x' and '3y' by '2z' 2010-01-19
From shradheya:
write all fall using variable ,number, and basic operation

1. divide the product of 'x' and '3y' by '2z'.

Answered by Penny Nom.
The length of a sidewalk 2010-01-19
From Les:
I have a sidewalk 4"thick,4ft. wide,and I have 2cubic yards of concrete. What is the length of my sidewalk?
Answered by Penny Nom.
How high up the building does the ladder reach? 2010-01-19
From rosenda:
A 10 meter ladder is 6 meter from the base of a building. How high up the building does the ladder reach?
Answered by Robert Dawson.
A cone circumscribed about a given hemisphere 2010-01-19
From Neven:
The cone of smallest possible volume is circumscribed about a given hemisphere. What is the ratio of its height to the diameter of its base?
(G.F.Simmons, Calculus with Analytic Geometry, CH4 Applications of Derivatives)

Answered by Chris Fisher.
Setting up an algebraic equation 2010-01-18
From jerry:
the 1st of 4 books has twice as many pages as the 3rd. The 2nd book has 30 pages less than the 1st. The 4th has 42 pages more than the 1st and 2nd books combined. there are 567 pages in total. How many pages are in each book.
Answered by Penny Nom.
How far apart are the two girls? 2010-01-18
From benny:
Debby and john are looking up at their house from the backyard. From Debby's point of view, the top of the house is at an angle of elevation of 40 degrees From Johns point of view, directly closer to the house, it is 60 degrees. The house is 15m high. How far apart are the two girls?
Answered by Robert Dawson.
A push-button padlock 2010-01-17
From Vince:
Hi,

I have a push-button padlock using ten buttons (1-2-3-4-5-6-7-8-9-0). Five digits must be pushed in order to open the lock. Each digit can only be used once. Order is not supposed to matter.

How many different possible combinations? And what are they?

Answered by Harley Weston.
A linear equation with the variable on both sides 2010-01-16
From dog:
4x-9=7x+12
Answered by Penny Nom.
Probability, counting and cards 2010-01-15
From Julie:
Hello,
  1. Select a card from the deck(card deck of 52 cards). Place the card back in the deck. What is the probability that both cards will be black? Is it 25/104, 24%?

  2. Select a card from the deck. Do not replace the card. Select another card. What is the probability that a red and a face card are the two cards you selected? Is it 2/221, .9%?

  3. Remove the red cards from the deck and assume that the remaining cards have been shuffled: select a card from the remaining deck. Place the card back in the deck. Select another card. What is the probability that both cards will be black? Is it 26/26 x 26/26= 1 x 1 = 1 , 100%?
    Select a card from the remaining deck. Do not replace the card. Select another card. What is the probability that a red 3 and a face card are the 2 cards you selected? Is it 0/26 x 6/26 = 0, 0%?

  4. Seperate 12 face cards from the rest of the deck. Assume that the face cards have been shsuffled. Select 3 cards from the pile of face cards. How many different arrangements can be made by selecting 1 of each card (jack, king, queen) from the pile? is it 1/4 x 1/4 x 1/4 x 1/4 = 1/64?

  5. How many different ways are there of selecting the queen of clubs, then the king of diamonds, and then the jack of hearts from the pile? Is it 1/12 x 1/12 x 1/12 = 1/1728?

Answered by Robert Dawson.
A schedule for a pool tournament 2010-01-14
From john:
I need to know how to set up a pool tournament 12-14 players playing once a week, either 10-12 weeks whichever works out the best, what players would play what players week one through week ten or twelve? I hope you can help, I cant figure out the formula.

thanks

John

Answered by Victoria West.
A circular photo in an octagonal frame 2010-01-13
From Mike:
I have a circle photo 18 and one quarter inches round. I want to cut frame for it in a octagon shape. What would the angle and the length of cut be?
Mike

Answered by Robert Dawson.
A triangular prism 2010-01-13
From Breon:
what is the formula for a triangular prism?
Answered by Robert Dawson.
Trigonometry and picture hanging 2010-01-13
From george:
The top of a picture 1m high 0.8m from the ceiling. At a point on the ceiling directly in front of the picture, we wish to install a light so that the angle subtended by the picture equals to the angle of depression of the top of the picture. How far out from the wall should the light be installed?
Answered by Penny Nom.
Related Rates Problem 2010-01-12
From Neven:
A woman raises a bucket of cement to a platform 40 ft above her head by means of a rope 80 ft long that passes over a pulley on the platform. If she holds her end of the rope firmly at head level and walks away at 5ft/s, how fast is the bucket rising when she is 30 ft away from the spot directly below the pulley? (G. F. Simmons, Calculus with Analytic Geometry, pg.142)
Answered by Penny Nom.
A proof by induction 2010-01-12
From Bhavya:
Prove by induction that if Xi >= 0 for all i, then

(Summation Xi from 1 to n)^2 >= Summation Xi^2 from 1 to n

Answered by Penny Nom.
Motorcycle markup 2010-01-12
From Harold:
I went to a motorcycle dealership and saw a motorcycle costing $11,000. Dealerships have a way of marking up a product about 300%. What is the original cost of the motorcycle before the 300% markup?
Answered by Robert Dawson.
The triangle inequality 2010-01-11
From Nazrul:
If A and B are any two vectors, then how can I prove that magnitude of (A+B)<=magnitude of A+magnitude of B. Please show me the process in details. Thank you.
Answered by Robert Dawson.
A proof involving real numbers 2010-01-11
From Amper:
Let a,b is an element of real numbers, and suppose that for every x>0 we have a is lesser than or equal to b+x.
(a) Show that a is lesser than or equal to b.
(b) Show that it does not follow that a is lesser than b.
i'm feeling bad of having no idea with this, hope i you can help me. GRACIAS!!

Answered by Penny Nom.
Gail and Bill drove to the beach 2010-01-10
From Nicole:
Gail and Bill drove to the beach at an average speed of 50mph. They return home over the same road at an average speed of 55mph. The tip took 30 min less time. How far is the beach from their house?
Answered by Stephen La Rocque.
The intervals where the function is positive and negative 2010-01-10
From Ron:
Hello

I'm trying to find out the intervals where the function is positive and negative. It's for a polynomial function y= -(x+2)^2 (x-2) and y= (x+1)(x+4)(x-3)

I have tried the right and left side of each x-intercepts, but I still don't understand the results

thank you for your help

Answered by Penny Nom.
(x+3) is a factor of 2x^3-3x^2+px+30 2010-01-09
From Ruby:
(x+3) is a factor of 2x^3-3x^2+px+30.
Determine the value of p

Answered by Penny Nom.
A pair of simultaneous equations 2010-01-09
From Yumiko:
Solve the following pair of simultaneous equations.

x^2 -4x = y^2-4
3y=2x - 3

Answered by Penny Nom.
The domain, range and inverse of a relation 2010-01-08
From Nazrul:
Given that A={-2,-1,0,1,2}, S={(x,y): x belongs to A, y belongs to A and y^2=x}. How can I find the domain and range of the relation S. What is the inverse relation of S. Please show me the process in details. Thank you.
Answered by Penny Nom.
A sports schedule for the ymca 2010-01-08
From Erich:
i have to make a sports schedule for the ymca. i have 7 teams. they need to play each other 2 times. we have 3 time slots per night. i would like them to have an even number of time slots thru the season. one team must have abye each week. can you chow mw the schedul. example time 1 team 1 vs. team 2. time 2 team 3 vs. team 4 time 3 team 5 vs. team 6. team 7 bye week 1.
Answered by Victoria West.
A quilt square 2010-01-07
From Kimi:
A quilt square is stitched along each diagonal to make 4 right triangles. Each diagonal is 12 inches long. ( 1 foot)

How many quilt squares can be cut from a piece of fabric that is 8 feet long and 2 feet wide??

Answered by Penny Nom.
A networking event 2010-01-07
From Tricia:
I have 70 people to go around to 10 tables with 7 at each table. only 6 from each table will rotate each time. There is 1 table monitor that always stays at same table. How do I make the rotation so everyone moves and gets to see all 70 people at the networking event?
Answered by Robert Dawson.
The area of a sector 2010-01-07
From angelkem:
how do you get the area of the circle, the sector, the shaded sector, non shaded sector and its total area if the diameter is 20 cm and each area of the circle is 45 degrees?
Answered by Penny Nom.
Cows, pigs and ducks 2010-01-06
From Eileen:
There are 239 cows and pigs in a farm. The total number of cows and ducks in the farm is 767. There are 5 times as many ducks as pigs. If the farmer can sell each duck for $7.50, how much can he collect from the sale of all his ducks?
Answered by Robert Dawson.
Dividing seashells among girls 2010-01-06
From Eileen:
A group of girls collected some seashells from the beach. They tried to divided these seashells equally among them. If each girl received 8 seashells, they would need 5 more seashells. If each girl received 7 seashells, they would have 3 seashells extra. How many seashells did they collect from the beach altogether?
Answered by Robert Dawson and Penny Nom.
A triangular pyramid 2010-01-05
From ally:
how many vertices, edges, faces in a triangular pyramid?
Answered by Robert Dawson.
Cutting a square into four shapes 2010-01-05
From Tracy:
A square with a side of 4 ft needs to be cut into four shapes with the same area. can you find six different was to cut the square?
Answered by Robert Dawson.
Holiday for 7 golfers 2010-01-05
From Andy:
I am having difficulty scheduling pairings for a forthcoming Golf holiday. There are 7 players and we will be playing 7 rounds - each round will be a 3 ball and a 4 ball. I am trying to work out a schedule that means everyone plays with each other roughly the same number of times (with a minimum of, say, 3 times if possible). Also, we are looking to ensure everyone plays in roughly the same number of 3 and 4 balls. Any suggestions would be much appreciated (we leave in 10 days time). Thank you.
Answered by Victoria West.
How many 4x4 tiles would I need for 10 sq ft? 2010-01-04
From Chris:
how many 4x4 tiles would I need for 10 sq ft
Answered by Robert Dawson.
The maximum area of a rectangle 2010-01-03
From Mohammad:
determine the maximum area of a rectangle with each perimeter to one decimal place?
a)100 cm b)72 m c)169 km d)143 mm

Answered by Penny Nom.
Write a function for the area in terms of x 2010-01-01
From Almarie:
Lauren has 40 feet of metal fencing material to fence three sides of a rectangular garden. A tall wooden fence serves as her fourth side. Write a function for the area of the garden A in terms of x, the width in feet.
Answered by Penny Nom.
The volume of an ellipsoid 2010-01-01
From Barbara:
I need to calculate the volume of an ellipsoid with the dimensions:

102.5 mm x 57 mm x 57 mm = L x W x H

I am using the formula V = 4/3 pi (.5L)(.5W)(.5H), but it isn't giving me a reasonable number.

Answered by Penny Nom.
A question from a boat builder 2010-01-01
From Grant:
I am a boat builder, trying to lay out shape of side's elevation. My question is, how do I define the length of a circle's radius, if I know the chord length (20 ft) and the segment of the radius between the chord and the circle is known (7 inches)?
Answered by Penny Nom.
The minimum point of a quadratic 2009-12-31
From rachel:
y=0.0008x^2-0.384x
What is the minimum point of this equation?

Answered by Penny Nom.
A linear equation with fractions 2009-12-31
From Michelle:
How would I solve: (-2)(-1/2x + 5) = (-2)(8)?
Answered by Penny Nom.
Chord length given the length & radius of an arc 2009-12-31
From Wayne:
Given the length & radius of an arc, is there a formula that will accurately calculate the chord length? I'm an architectural designer, and would need it explained in layman's terms. Thank you.
Wayne

Answered by Penny Nom.
The volume of a frustum 2009-12-29
From dave:
I have a frustum top 1.7r bottom .55r and 2.14 h I have to calculate cement in a silo every week. I am not very good at maths but i have been adding the top and bottom to get an average so as to turn it into a cylinder and i come up with 8.5m3 I know that the correct volume is 9.24 m3. Can you tell me why getting an average width on the top and bottom of a frustum doesn't work.
Thank you.

Answered by Chris Fisher.
Linear programming using the Simplex Method 2009-12-28
From William:
A gold processor has two sources of gold ore, source A and source B. In order to keep his plant running, at least three tons of ore must be processed each day. Ore from source A costs $20 per ton to process, and ore from source B costs $10 per ton to process. Costs must be kept to less than $80 per day. Moreover, Federal Regulations require that the amount of ore from source B cannot exceed twice the amount of ore from source A. If ore from source A yields 2 oz. of gold per ton, and ore from source B yields 3 oz. of gold per ton, how many tons of ore from both sources must be processed each day to maximize the amount of gold extracted subject to the above constraints? I need a linear programming solution or algorithm of the simplex method solution. Not a graphical solution. Thanks.
Answered by Janice Cotcher.
Stefan-Boltzmann Law 2009-12-28
From florence:
Hi- I have a college tutor and we are stumped with this question. Assuming a filament in a 100 W light bulb acts like a perfect blackbody, what is the temperature of the hottest portion of the filament if it has a surface area of 6.3 X 10^-5 m^2? The Stefan-Boltzmann constant is 5.67 X 10-4 W/(m^2 X K^2.\ Thank you for your help. If you can only answer one question per day please answer this one and guide me through a solution.
Answered by Janice Cotcher.
The Snail Problem 2009-12-28
From kishore:
A snail starts crawling from one end along a uniformly stretched elastic band. It crawls at a rate of 1 foot per minute. The band is initially 10 feet long and is instantaneously and uniformly stretched an additional 10 feet at the end of each minute. The snail maintains his grip on the band during the instant of each stretch. At what points in time is the snail (a) closest to the far end of the band, and (b) farthest from the far end of the band?
Answered by Janice Cotcher.
Related Rates of a Cylinderical Trough with a Horizontal Axis 2009-12-26
From Emily:
A cylinder is lying on it's side and being filled with water at a constant rate. Let the current height of water be t=0. When t=4, the cylinder is half full. When t=12, the cylinder is completely full. When is the rate of the height change increasing?
Answered by Janice Cotcher.
Finding Density Given Volumetric Thermal Expansion Coefficient 2009-12-26
From florence:
Hi- Please help me to apply the formula for this problem. The coefficient of volumetric for gold is 4.20 X 10^-5 C degrees. The density of gold is 19,300 kg/m^3 at 0.9 C degrees. What is the density of gold at 1050 degrees C. Could you please explain how to get the solution of 18,500 kg/m^3 Thank you for your help Florence
Answered by Janice Cotcher.
Triangles on a base of 2.4 meters 2009-12-26
From Allan:
Please,How do I calculate the height of a triangle when I only know the width of the base line,It is 2.4 mtrs.
Thankyou very much.

Answered by Penny Nom.
Truncated Octagonal Prism 2009-12-26
From freitas:
the top plane of the truncated octagonal right prism is 45 degree with respect to horizontal. find the volume
Answered by Janice Cotcher.
Finding Specific Heat of a Substance 2009-12-26
From erin:
Hi- 2 kg metal requires 1.00 X 10^4 J of heat to raise its temperature from 20 degrees C to 40 degrees C. What is the specific heat capacity of the metal?
Answered by Janice Cotcher.
15 golfers 3 rounds fivesomes 2009-12-23
From Dewey:
I have 15 golfers playing 3 rounds of golf. We will be playing in fivesomes. Any formula that has everyone playing with a different person each round?

Thank you. Dewey

Answered by Robert Dawson.
Perimeter 2009-12-23
From Jalyn:
I'm doing this math project comparing measurements and I want to know the perimeter of the Upper Peninsula of Michigan. Please Help me!

-Jalyn

Answered by Robert Dawson and Walter Whiteley.
A Lagoon, free form, inground swimming pool 2009-12-22
From donna:
What is the linear footage of a 14 x 23, Lagoon, free form, inground swimming pool?
Answered by Robert Dawson.
The tens digit of 0! + 1! + 2! + 3! + ... + 2000! 2009-12-21
From Alicia:
What is the tens digit of 0! + 1! + 2! + 3! + ... + 2000!?
Answered by Robert Dawson and Penny Nom.
Figure out the square root of something 2009-12-20
From eric:
is there an ez equation to figure out the square root of something?
Answered by Harley Weston.
A triangle with two equal medians 2009-12-19
From Nazrul:
If two medians of a triangle are equal , how can I prove that the triangle is isosceles.
Answered by Chris Fisher.
How much will i have saved in that year? 2009-12-18
From Loretto:
If i save a penny and it doubles every day for one year how much will i have saved in that year
Answered by Robert Dawson.
Houses on a triangular field 2009-12-18
From Beth:
Hi, this is the question i have to answer. Planning permission has been granted to build houses on the triangular field in the diagram. The housing density permitted is 1 house for each 200m². Calculate how many houses can be built on this

Beth

Answered by Robert Dawson.
Polygons, diagonals and the sum of the measures of the angles 2009-12-18
From jason:
find the set of polygons in which the number of diagonals is greater than the sum of the measures of the angles
Answered by Robert Dawson.
A ball is thrown upwards 2009-12-17
From Reuben:
A ball is h meters above the ground t seconds after it is thrown where h(t) = 40t – 5t ^2 . Find the time at which the ball first reaches a height of

(a) 10 meters
(b) 40 meters
(c) 100 meters

Answered by Harley Weston.
The acute angle formed by intersecting lines 2009-12-17
From Katie:
To the nearest tenth of a degree, find the size of the acute angle formed by the intersecting lines 3x + 2y = 12 and x - 2y = -2. Was is necessary to find the intersection point?
Answered by Robert Dawson.
A pile of sand 2009-12-16
From Malik:
Sand is leaking out of a hole at the bottom of a container at a rate of 90cm3/min. As it leaks out, it forms a pile in the shape of a right circular cone whose base is 30cm below the bottom of the container. The base radius is increasing at a rate of 6mm/min. If, at the instant that 600cm3 have leaked out, the radius is 12cm, find the amount of leakage when the pile touches the bottom of the container.
Answered by Harley Weston.
A trig question 2009-12-15
From A trig question:
Hey, my name is Candle
I'm in academic math10 and am stuck on my trig... one question I thought I had right because i used the cosine law I got wrong and can't figure out why... here's a copy of the question. (i guessed it was D... but my teacher said it's B)
Thanks
Candle

Answered by Robert Dawson.
Energy in calories 2009-12-15
From Josephine:
A soft drink manufacturer claims that a new diet soft drink is now "low Joule". The label indicates that the available energy per serving is 6300 J. What is the equivalent of this energy in calories? (1 Calorie=1000 cal)
Answered by Robert Dawson.
A 4 digit number 2009-12-15
From Lisa:
Billy Club was assigned the task of putting numbers on all the playground balls used during daily recess. Billy will number the balls using the following rules:

1. It will be a 4-digit number.

2. The digit in the thousands place will be a 1 or a 2.

3. The digit in the hundreds place will be a 2, 4, or 6.

4. The digit in the tens place will be an odd number.

5. The digit in the ones place will be greater than six.

How many balls can Billy number if he follows these rules?

Answered by Robert Dawson.
A circle problem 2009-12-14
From Fawad:
AP is a tangent at P to a circle centre O, where AP=6cm. The straight line AQC is such that QC= 9cm. Find the length, in cm of AQ.
Answered by Chris Fisher.
A telephone pole on a slope 2009-12-14
From Marissa:
A 10 meter telephone pole casts a 17 meter shadow directly down a slope when the angle of elevation of the sun is 42 degrees. Find the angle of elevation of the ground. Its a law of sines problem.
Answered by Penny Nom.
Bases larger than ten 2009-12-14
From Nick:
For numbers in bases larger than ten, what are the digits by mathematicians' conventions?

I assume the first 10 are always 0-9. I know two computer science conventions (for hexadecimal (A-F case-insensitively) and for base 85 (case-sensitivity plus 23 other characters)) and as a school student I learned that base 12 uses either T and E (which confused me when I thought they stood for Eleven and Twelve) or A and B, the latter as capitals only. Clearly, these conventions are in conflict.

I saw your reply in http://mathcentral.uregina.ca/QQ/database/QQ.09.99/magin1.html as accessed a few minutes ago but hope that in the 10 years since then mathematicians have arrived at a convention. Have they?
Thank you.

--
Nick

Answered by Claude Tardif.
A regular pentagon 2009-12-14
From Jamie:
A regular pentagon is inscribed in a circle of radius 4.5 cm. Determine its perimeter and area to one decimal place!

Thank YOU ! :)

Answered by Penny Nom.
Decimal equivalent 2009-12-13
From Asia:
In a forest,it rained 36 out of 90 days. What is the decimal equivalent of 30 out of 90 days
Answered by Penny Nom.
4x +3/4 = 1/4 2009-12-12
From Lynda:
4x +3/4 = 1/4

I know the answer is -1/8 but I don't know how to show the process of getting the answer

Answered by Penny Nom.
f(x)=x+2sinx 2009-12-12
From amroziz:
for which values of x does the graph of f(x)=x+2sinx have horizontal tangent
Answered by Harley Weston.
The volume of a compost pile 2009-12-11
From Tim:
I have a pile of compost that is78 feet long by 18 feet wide at base by 9 feet tall to a point. At the width it looks like a triangle I'm looking for the square yards Thanks
Answered by Penny Nom.
Dirt around a cylinder 2009-12-10
From curtis:
If I was to pile 3.5 cu. yds of dirt around a 5ft 6in. diameter cylinder within 24in. of the outside, how high will the dirt be?
Answered by Penny Nom.
The 3-ball lottery 2009-12-09
From Lee:
A friend and I are trying to calculate the odds of choosing the winning numbers in the 3-ball lottery (3-digits 0-9). We realize that there are 720 Permutations and 120 possible Combinations of said lottery, but we're having trouble figuring how to determine the "odds" or probability of winning (1) when order matters, and (2) when it doesn't. Please help!
Answered by Penny Nom.
e = m c^2 2009-12-09
From Jim:
Please explain the following formula: E=MC2 (squared)
Answered by Robert Dawson.
How fast is the distance between the two cars decreasing? 2009-12-08
From Jenny:
Two cares are on a collision course toward point P. The paths of the two cars make a 30 degree angle with each other. The first car is 40 km from P, and traveling toward P at 16 km/hour. The second car is 50 km from P, traveling at 20 km/hour. How fast is the (straight line) distance between the two cars decreasing. (Hint: Law of Cosines)
Answered by Harley Weston.
Simplifying a quadratic expression 2009-12-07
From Sabbie:
Hi, I need help solving this equation for my physics homework,so I can solve the quadratic equations for x and y. I've derived the following equations from the information provided in the question.

2a+3b = 4 -- 1
2a^2 + 3b^2 = 62 -- 2

a=2-(3/2)b is substituted into equation 2.

so, i get
2 (2-(3/2)b)^2 + 3b^2 = 62

this is where the problem begins. i cannot work through this to get to the correct quadratic equation

5b^2-8b-36=0

I can't seem to be able to figure out where I keep going wrong. I would really appreciate it if you could give me a step-by-step breakdown of the workout.

thanks!

Answered by Penny Nom.
An equilateral triangle is inscribed in a circle 2009-12-06
From anna:
An equilateral triangle is inscribed in a circle of radius 6. Find x and the length of one side of the equilateral triangle. The picture is a triangle where the corners touch the sides of a circle and there is a line drawn down the middle of the triangle. A point labeled D which is in the triangle but im pretty sure that its marking the radius of the circle. From that point D is a line going from that point to the bottom left corner of the triangle. So this line shall make another mini triangle. The bottom of the big triangle is then split into 2 segments and the left segment is labeled x. Please help for I am stuck!
Answered by Penny Nom.
The height of an isosceles triangle 2009-12-06
From Carl:
What is the height of an isosceles triangle if its base is 12cm, and its base angle is 72degrees?
Answered by Penny Nom.
A magic/math trick 2009-12-04
From Valentin:
What is the explanation for the following math trick: you think of any four digit number add those digits take that sum and subtract it from the first number then you say three of those new numbers in any order and the other person guesses the last digit. How does he do it?
Answered by Claude Tardif.
Two adjacent pig pens 2009-12-04
From alysia:
A pig farmer wants to build two adjacent pens using 96 feet of fencing. He wants rectangular pens that share the short side and the overall length is to be six more than twice the width. What should be the dimensions of the overall pen?
Answered by Penny Nom.
(9 - x^2)/(x - 3) 2009-12-04
From Sandy:
9-x^2/x-3
I need to know how to solve this.
Thanks

Answered by Penny Nom.
Airline routes 2009-12-01
From Kapilan:
Please answer the following question: From A to B there are four possible air routes. From B to C there are five possible air routes. From C to D there are three possible air routes. How many different trips can be taken from A to D and back without taking the same route on any section of the return trip?
Answered by Penny Nom.
A line through the y-intercepts 2009-12-01
From Rogerson:
Find the equation of the line through y-intercept of the lines 3x + 4y = 12 and 3x - y = 9.
Answered by Penny Nom.
Four fours 2009-11-29
From linda:
Four Fours order of operations challenge i need problems using 4's only that have an answer of 13, 18, and 19 than you
Answered by Penny Nom.
Milliliter conversion 2009-11-29
From Jan:
I am trying out a new bath salt recipe for one of my clients that has little children. Here then is the question, the recipe for color changing bubble bath calls for 0.15ml dry FD&C color. Since the recipe came from Australia, I have not a clue of how to change this into USA measurments
Answered by Penny Nom.
A place value problem 2009-11-29
From Ivy:
My tens digit is 4 less than my thousands digit. My ones digit is twice my tens digit. My ten thousands digit is one less than my ones digit. My hundreds digit is the sum of my tens and thousands digit. My thousands digit is 6.
Answered by Penny Nom.
The dimensions of a rectangular room 2009-11-26
From crystal:
. The length of a rectangular room is 7 feet longer than its width. The area of the room is 60 feet. What are the dimensions of the room?
Answered by Penny Nom.
Six-letter words 2009-11-24
From christine:
How many six-letter words (not necessarily an English words) are there in which exactly three of the letters are z's?
Answered by Claude Tardif.
Solving two equations, one with a square root 2009-11-23
From kacie:
y = square root of x+3
x-4y = -7

im having trouble with this problem...i have to find where they intersect.

Answered by Harley Weston.
The triangle formed by the tangent and the coordinates axes 2009-11-23
From Nirmala:
Given that y=1/x, x is not equal to zero. Prove that the area of the triangle formed by the tangent and the coordinates axes is 2.
Answered by Harley Weston.
A 24" circumference pipe 2009-11-21
From Cliff:
How long does a pipe with a 24" circumference need to be to hold 2 cubic feet?
Answered by Penny Nom.
A simplification 2009-11-21
From Lou:
In the following problem, [(7)/(x+2)]-[(x+8)/(4-x^2)]+[(3x-2)/4-4x+x^2)] what steps do you use to convert the middle portion to become +[(x+8)/(-4+x^2)]? The answer to the problem is supposed to be (11x^2-18x+8)/(x+2)(x-2)^2
Answered by Penny Nom.
Two overlapping circles 2009-11-19
From Raraa:
There are two identical circles . The edge of one circle is at the middle point of the other circle. There were overlapped . The area of the overlapped surface is 20000 square centimetres . How do I find the radius of the circle rounded to the nearest whole centimetre ?
Answered by Penny Nom.
Factoring 2009-11-19
From danyelle:
81x^2+36x+4

factoring i dont understand it at all

Answered by Robert Dawson.
The point of intersection of the pair of straight lines 2009-11-19
From Stephanie:
Find the point of intersection of the pair of straight lines:
y=-3x-9
-y=4x+2
x = _______ and y = ________

I am totally confused. Please help

Answered by Robert Dawson.
Angular speed 2009-11-19
From Atalie:
The engine of a sports car rotates at 5000 revolution per minute. Calculate the angular speed of the engine in radians per second, given that 2pi radians = 1 revolution
Answered by Robert Dawson.
A baseball diamond 2009-11-18
From maelee:
the official distance between home plate and second base in baseball diamond is 120ft. Find the area of the official ball diamond & the distance between the bases.
Answered by Robert Dawson, Chris Fisher and Penny Nom.
The three medians of a triangle 2009-11-18
From Nazrul:
If the three medians of a triangle are equal, how can I prove (without using the property that the three medians of a triangle cut one another at a point of trisection) that the triangle is equilateral.
Answered by Chris Fisher.
Matt has 40 coins 2009-11-18
From Amy:
Matt has 40 coins(nickels, dimes, and quarters). The total amount is $4.05. He has 7 more nickels than dimes. How many quarters does he have?
Answered by Penny Nom.
Two movies in one day 2009-11-18
From jazzy:
ann and tim saw two movies in one day.The two movies lasted 4hours long .One movie was an half hour longer than the other .How many hours did the shorter movie run
Answered by Penny Nom.
How many girls are there in the class? 2009-11-18
From Anisa:
There are 30 students in a class. A quarter of the boys ans half of the girls study music, making a total of 11 music students. How many girls are there in the class?
Answered by Penny Nom.
The base and height of a triangle are in the ratio of 3:2 2009-11-18
From Mike:
Find the base and height of a triangle using the ratio 3:2 and an area of 108 inches square. Is there a formula we need to use?

Thanks

Mike

Answered by Penny Nom.
Coefficients 2009-11-17
From Da'mon:
The Coefficients of the expression 8 - 5x - 4 + 3x are 5 and 3 Whether the statement is true or false.
Answered by Penny Nom.
One-eighth of one million dollars 2009-11-16
From Jarrett:
what is 1/8 equal to out of a million dollars.
Answered by Penny Nom.
Three angles and one side of a triangle 2009-11-16
From Esther:
How do i find the sides of an acute triangle if i know the angels are 60,45,75 and i only know one side which is 10? Thanks!
Answered by Penny Nom.
Two similar pentagons 2009-11-16
From louis:
The lengths of five sides of a pentagon are 1,3,5,7,and 12 units. If the length of the longest side of a similar polygon is 18 units, find the perimeter of the larger pentagon. I think I figured it out Not sure though.
Answered by Penny Nom.
The line through D(-4, 0) and E(2, 6) 2009-11-16
From Rogerson:
The point F is on the line through D(-4, 0) and E(2, 6) so that DF=4DE. Find the coordinates of F.
Answered by Penny Nom.
Excluded values 2009-11-14
From Janie:
I have to State the excluded values for this equation and then solve, but not sure how to do this. Here is the problem
(x+6)/x+3=(3)/(x+3)+2

Answered by Harley Weston.
Cables in a pipe 2009-11-13
From john:
how many 0.28 mm diameter cables will fit into a 19.05 mm diameter pipe
Answered by Stephen La Rocque.
A triangular shaped ditch 2009-11-12
From Tim:
I am planning on constructing a triangular shaped ditch the measures 550 meters in length, 6 meters wide at the surface, .3048 meters at the invert of the ditch and 2 meters deep.

How many cubic meters of material will have to be excavated to construct this ditch

Answered by Robert Dawson.
An isosceles trapezoid 2009-11-12
From lyjah:
what is parallel sides of an isosceles trapezoid measure 5cm and 11cm long.and oneof the other sides also measures s5cn long what of the isosceles trapeziod
Answered by Penny Nom.
Pennies and dimes 2009-11-11
From Rachel:
Kesha had only pennies and dimes in her change purse. When she counted out her change she had 64 cents/ of she had 6 times as many pennies as she did dimes, how many pennies did she have? Write equation and solve it.

Can someone help me with this?

Answered by Penny Nom.
A truncated cone 2009-11-11
From Lucian:
I need to calculate the bottom inside diameter of a truncated cone.
The top insdie diameter is 1450mm.
The material is 6mm thick
The cone angle is 20 degrees
The slant length is 152mm
I would like a formula so that I can build a spread sheet

Answered by Penny Nom.
The fourth vertex of a rectangle 2009-11-11
From Ellena:
The points A(2,-1), B(5,-3), and C(7,0) are the three vertices of a rectangle. Determine he coordinates of the fourth vertex.
Answered by Penny Nom.
A sequence of letters and numbers 2009-11-10
From Maria:
What is the last ten letters and numbers in the following series and how do I work it out so I can explain it to an eleven year old. J1F8M1A0M1J0J1
Answered by Robert Dawson, Claude Tardif and Harley Weston.
y = sin(x) + 3 2009-11-10
From Kapilan:
When graphing trigonometric functions like SIN and COSINE does f(x)=x squared equal to SIN or COSINE

Also what is the range, domain, period and ampliltude for y=sin@ + 3

Answered by Harley Weston.
f(x)= (e^x) / [(e^x)+(ex^2)] 2009-11-10
From natalie:
I'm trying to graph the function, f(x)= (e^x) / (e^x)+(ex^2) [e to the x divided by e to the x plus e times x squared] I know that there aren't any vertical asymptotes, but is there a horizontal asymptote? and also, I'm stuck on finding the concavity for this graph. I tried to find f "(x), but it came out to be really long and I am not sure how to find the x values for f "(x) without using a graphic calculator. thanks, natalie
Answered by Chris Fisher and Harley Weston.
A rate of 67 per 1000 2009-11-10
From ANTHNONY:
if the rate is 67 per 1000 the fraction is 67/1000 how do I present the expanded form
Answered by Penny Nom.
A drawing of a baseball diamond 2009-11-09
From Beth:
My husband has a drawn diagram on a piece of paper of a baseball diamond. He knows that on the paper 1 1/16" (1.6875") = 90 actual feet. How do we calculate how many feet 5 7/8" on the paper would be?
Answered by Penny Nom.
The cardinality of the prime numbers 2009-11-07
From Justin:
Hello there, I was just wondering since the number of primes is infinite, are they equal to infinity or Aleph-null? Justin
Answered by Robert Dawson and Victoria West.
sinA + cosA = 1 2009-11-06
From Nazrul:
What is the solution of sinA+cosA=1 where A is an acute angle? Will A=0 degree be included in the solution? Please explain.
Answered by Harley Weston.
Find the original price 2009-11-06
From Cyndy:
Find the original price of a pair of shoes if the sale price is $98.00 after a 30% discount.
Answered by Penny Nom.
Tetrahedral numbers 2009-11-06
From rena:
consider the tetrahedral numbers 1,4,10,20, etc. Can these numbers be described as a polynomial? If so, what is the degree of that polynomial?
Answered by Stephen La Rocque.
A minute hand 2009-11-05
From Pardha:
A minute hand of table clock is 3cms long. How far its tip move in 20 minutes
Answered by Penny Nom.
3x^3-3x-3 and 3x+6 2009-11-04
From spencer:
My question is 3x^3-3x-3 devided by 3x+6
Answered by Penny Nom.
Proportionality 2009-11-03
From Nazrul:
If x is proportional to y , how can I prove that (x^2-y^2) is proportional to xy. Please explain the process.
Answered by Penny Nom.
The roots of a polynomial 2009-11-03
From Jess:
Find the value of a positive integer k such that f(x)=x^2 - 2kx + 55 has roots at k +3 and k - 3.
Answered by Penny Nom.
A single die is rolled twice 2009-11-03
From Michelle:
A single die is rolled twice. Find the probability of getting two numbers whose sum exceeds 5. I can't seem to come up with any of these answers, Help. A. 0.72 B. 0.89 C. 0.68 D. 0.58
Answered by Penny Nom.
A number puzzle 2009-11-03
From Jacqui:
Write a number sentence. Use every digit once, 7, 4, 3, 6, 5,10. Insert math symbols +, - , *,/ and end up with the number 3. Use parentheses if necessary.

Jacqui

Answered by Claude Tardif and Penny Nom.
The extended real numbers 2009-11-02
From Justin:
Hello there, I was wondering is the set of extended real numbers a closed set or an open set?

Justin

Answered by Robert Dawson.
A lottery question 2009-11-01
From munirah:
assume the chances of winning the lottery are 1 in 1000.how many times would you have to play to attain 0.5 probability of winning at least once?
Answered by Robert Dawson and Claude Tardif.
At what rate are the people moving apart? 2009-11-01
From saira:
A man starts walking north at 4 ft/s from a point P. 5 minutes later a woman starts walking south at 5 ft/s from a point 500 ft due east of P. At what rate are the people moving apart 15 minute after the woman starts walking ?
Answered by Harley Weston.
Sarah's age 2009-10-31
From Kenneth:
2/3 of Mary’s age equals Sarah’s age and 3/4 of Ruth’s age equals Sarah’s age. If the total ages equal 46, how old is each girl?
Answered by Claude Tardif.
A path around a pond 2009-10-31
From adeniji:
find the area of a concrete path 2m wide surrounding a circular pond 12m in diameter
Answered by Penny Nom.
Where is the Father? 2009-10-30
From Jeffrey:
A mother is 21 years older than her kid and within 6 years the child will be 5 times younger than the mother. Where is the Father? Jeffrey
Answered by Robert Dawson and Claude Tardif.
Rationals 2009-10-30
From Jawsh:
I don't quite understand the whole idea of rationals. If you have 3Squareroot 49. Is that equal to 3Squareroot 7^2? Can you break it down with an example for me please.
Answered by Robert Dawson.
The intersection of two graphs 2009-10-30
From asma:
by using the suitable scales,draw both the graph of x/2 + y/4 =3 and y=2 on the same graph paper.Find the points of intersection of the two graph.
Answered by Penny Nom.
Painting a dome 2009-10-30
From Jessica:
A hemispherical dome with a radius of 50 ft will be given a coat of paint .01 inch thick. The Contractor for the job wants to estimate the number of gallons of paint needed. Use a differential to obtain an estimate (231 cubic inches/gallon) HINT: Approximate the change in volume of hemisphere corresponding to increase of .01 inch in the radius.
Answered by Robert Dawson.
Acreage calculation 2009-10-29
From Angela:
I am appraising a small portion of property for a right of way for a new sewer line. I have appraised many similar projects in the past; however the engineer always calculated the exact acreage and/or dimensions of the parcel being taken. My survey states the following: "141 linear ft. of 25' permanent R/W. I divided the linear feet x 12 and mult. by 25 to get the square footage, but it just doesn't seem right. Any suggestions? Am I missing something?
Answered by Robert Dawson.
An irregular quadrilateral 2009-10-29
From riya:
what is irregular quadrilateral?
Answered by Penny Nom.
Is it a rectangle? 2009-10-29
From sefora:
The coordinates of the vertices of quadrilaterals are given. Draw each quadrilateral on a grid. Determine whether it is a rectangle (show your work)

P(5,1), Q(-4,4), R(-6,-2), S(3,-5)

Answered by Penny Nom.
A triangle problem 2009-10-28
From Nazrul:
ABC is a triangle , D is the mid point of AB and DE is parallel to BC where E lies on AC. How can I prove that E is the mid point of AC.
Answered by Penny Nom.
Sarim has $1 in coins. 2009-10-27
From Elizabeth:
Here is the question asked: Sarim has $1 in coins. One-fifth of the coins are dimes, two-fifteenths are nickels, and two-thirds are pennies. Tell how many of each coin he has. I am not sure how to start this problem.
Answered by Penny Nom.
Two non zero vectors 2009-10-27
From Nazrul:
Given that a and b are two non zero vectors and ma+nb=0 where m and n are two scalars. How can I prove that m=n=0.
Answered by Robert Dawson.
Density of dirt and gravel 2009-10-26
From ken:
if digging a load of dirt and gravel in mining how many yards of material to the ton thanks
Answered by Penny Nom.
f(x) = |3x - 6| + 1 2009-10-26
From kelsey:
find the coordinates of the vertex f(x)= l3x-6l + 1
Answered by Penny Nom.
Graphing y=(4-x^2)^5 2009-10-25
From natalie:
I want to graph the curve of y=(4-x^2)^5 without using a graphing calculator. To do this, I'm suppose to find: domain, y and x intercepts, asymptotes, intervals of increase/decrease, local max/min, concavity and points of inflection. I got all the way to the step where I'm solving the concavity and I'm stuck. I found the f"(x) and it came out to be really large polynomial. I want to know how I can solve for the x of f"(x) without the use of a graphing calculator, when the polynomial has x^6 and x^8. Thank you so much, natalie
Answered by Harley Weston.
Books on a shelf 2009-10-25
From Julie:
a bookshelf is 36 inches long and contains (x) books each (n) inches thick. If each book half an inch thinner, the shelf would hold six more of the same book. What is x/n
Answered by Penny Nom.
Cutting a pipe at an arbitrary angle 2009-10-24
From Carol:
I have read your article on cutting pipe,etc. at a 45 degree angle. I need to develop an equation and pattern for cutting any size pipe (3" to 7") to any degree. I don't understand how to transfer the wave pattern to graph paper.
Thanks.

Answered by Harley Weston.
A word problem 2009-10-24
From Ryan:
Two times one number plus 7 times another number is equal to 17. the first number plus the second number is equal to 25. What are the two numbers? Please Help!
Answered by Penny Nom.
What jobs use conics? 2009-10-22
From denise:
i have to type a paper on what jobs use conics and i can not find anything can you help

thanks
Denise

Answered by Penny Nom.
A linear equation 2009-10-22
From Ryan:
How do you convert the equation 2x-6y=7? This is on my homework and I am really stuck!!!
Answered by Penny Nom.
e^pi and pi^e 2009-10-22
From Natalie:
Question from natalie, a student:

Without using your calculator, prove which is bigger: e^pi or pi^e

we talked about ad absurdum in class, so I'm assuming that is how I approach this question, but since neither of them have a variable( like the examples we actually solved in class), I'm not quite sure how to solve it....

Answered by Harley Weston.
Forming a square 2009-10-21
From christianah:
Two parallel lines have the equations y=x+4 and y=x-2.Two additional lines are drawn to form a square.What is the side length of the square?Write possible equations for the additional lines.Explain and justify your reasoning.
Answered by Penny Nom.
A linear system 2009-10-20
From marissa:
Solve this linear system
2x-y=5
3x+y=-9

Answered by Penny Nom.
A line and a circle 2009-10-19
From Renson:
Determine whether the line x-2y=0 cuts,touches or fail to meet the circle x^2+y^2-8x+6y-15=0.If it touches or cuts ,find the coordinates of the point(s) of contact
Answered by Harley Weston.
The hypotenuse of a right triangle 2009-10-18
From steven:
the perimeter of a right triangle is 20 cm. its area is 15 sq cm. find its hypotenuse.
Answered by Penny Nom.
Proof that the root of 27 is irrational 2009-10-18
From Scarlet:
How do you prove that the square root of 27 is irrational?
Answered by Victoria West.
The equation of a circle 2009-10-17
From Renson:
A circle passes through (4,2) and (11,-5) with center on the line 3y-5x+41=0.Determine its equation in standard form.
Answered by Penny Nom.
541.39(1 + i)^15 = 784.09 2009-10-14
From Fitore:
Hi, I noticed that this question was already posted up, however I was hoping I could solve it without having to use logs. Can you please help me? The equation is: 541.39(1 + i)^15 = 784.09
Answered by Penny Nom.
Concentric circles 2009-10-14
From maddy:
Find the exact area of the region bounded by two concentric circles with radii 10 inches and 6 inches.
Answered by Penny Nom.
The three angles of a triangle 2009-10-13
From Michelle:
The second angle of a triangular garden is four times as large as the first. The third angle is 45 less than the sum of the other two angles. Find the measure of the other two angles?
Answered by Penny Nom.
What fraction of my goal has been raised? 2009-10-13
From sally:
If my goal is to raisel $300 and I have only raised $240 what fraction of my goal has been raised
Answered by Penny Nom.
The radius of an arc 2009-10-13
From AYMAN:
Question from ayman:

i would like to know the radius of an arc , i do not know the angle , can u please explian it with an example ,

all i know , or i think i know is a draw line , the reason i asked yu guys this Q , is i am a boilermaker apprentice , sometimes i do deal with a bended pipes or flate plates of metal .

when i try to do the same bend as these pipes , it is quite hard so for me to find out the radius of the pipe already bended & find out the angle i will be able to know the full length of the material thats all , thank you

Answered by Robert Dawson.
How many gallons does it contain when it is 1/6 full? 2009-10-13
From Bren-Nesha:
A tank contains 200 gallons when it is 4/5 full. How many gallons does it contain when it is 1/6 full? I already know the answer but i would like to know how to arrive at the answer.
Answered by Penny Nom.
The cube of a number plus five times the number plus 1 2009-10-12
From JP:
Find a counterexample to the statement: The cube of a number added to the sum of five times the number and 1 is a prime number.The cube of a number added to the sum of five times the number and 1 is a prime number
Answered by Penny Nom.
Integration of cos (x^2) dx 2009-10-12
From Venkat:
How do you evaluate the integration of cos (x^2) dx ?
Answered by Harley Weston.
A max/min problem 2009-10-12
From avien:
a rectangle has a line of fixed length Lreaching from the vertex to the midpoint of one of the far sides. what is the maximum possible area of such a rectangle? SHOW SOLUTION USING CALCULUS
Answered by Penny Nom.
A right triangle 2009-10-12
From Josseph:
A right triangle has an area of 84ft sq. and a hypotenuse 25ft long. What are the lengths of the other two sides?
Answered by Penny Nom.
The area of a rectangle 2009-10-10
From Ann:
given a rectangle with sides a and 2a and diagonal d, find the area of the rectangle as a function of the diagonal,d
Answered by Penny Nom.
$6.25 in nickels quaters and dimes 2009-10-10
From miriam:
a piggy bank contains $ 6.25 in nickles quaters and dimes. there are 65 coins all together and there are four times as many nickles as quaters find the number of coins of each kind
Answered by Penny Nom.
Find the intersection of y=x-1 and y=sin(x) 2009-10-10
From Usama:
There are many modern calculators and softwares that can plot the functions. I have two very simple equations y=x-1 and y=sin(x), if i draw them manually on paper i can not get the accurate value. But i think a computer can easily draw perfect straight and curve lines so it should be easy to find the intersection point of two lines. Is there any way or software that can tell the exact value of intersection point of two lines?
Answered by Robert Dawson and Harley Weston.
Golf for 8 2009-10-10
From Chris:
I have a group of 8 golfers who will be playing three rounds, What is the best formula so we can all try to play with different people during the 3 rounds .
Answered by Victoria West.
Two rectangles 2009-10-08
From Lillian:
A rectangle is 5cm longer than twice its width. The width of another rectangle is 3cm less than the width of the first rectangle and its length is 6cm more than 3 times its width. If the perimeters are equal, find the dimensions of both rectangles
Answered by Penny Nom.
Elimination method 2009-10-08
From Kenty:
How do I solve this problem using the elimination method?
3x-7y=0
6x+4y=0
I am not sure how so if someone can show me a similar problem (instead of solving this one for me) that would be fantastic.

Answered by Penny Nom.
A number between 48 and 59 2009-10-07
From nakia:
I am a number less 59 but greater than 48. my ones digit is 2 more than my tens digit.What's the number?
Answered by Penny Nom.
Two jets are traveling towards each other 2009-10-05
From jake:
two jets are traveling towards each other and are 4000 km apart. the rate of one jet is 100 km/h faster than the rate of the other. if the jets pass each other after 2.5 hrs what is the rate of speed of the faster jet?
Answered by Penny Nom.
The maximum number of right angles in a polygon 2009-10-05
From Bruce:
Is there way other than by trial and error drawing to determine the maximum number of right angles in a polygon? Secondary question would be maximum number of right angles in a CONVEX polygon. Is there a mathematical way to look at this for both convex and concave polygons? Or are we limited to trial and error drawing?
Answered by Chris Fisher.
How high is the ledge? 2009-10-03
From gabby:
Standing on a ledge, there is a boat 25 degrees below you. The boat has a 1,000ft. tower. If the angle of elevation to the top of the tower is 15 degrees, how high is the ledge?
Answered by Stephen La Rocque.
A parabolic bridge 2009-10-03
From SANDRA:
a bridge is constructed across the river that is 200 feet wide. the arch is parabolic so that the focus is on the water. A sheep 50 ft wide and 30 ft high passes safely through the arch
a) find equation of the arch
*y-k=(-1/4)(x-h)^2
b) find the highest point of the arch

Answered by Stephen La Rocque.
24 golfers, 6 days 2009-10-03
From patrick:
i need a pairing schedule for 24 golfers for six days playing in foursomes. Please
Answered by Victoria West.
Fill required for a driveway 2009-10-03
From Rose:
We are going to be pouring a driveway and it is needing a lot of fill. I can't seem to figure this out. Measurements are 30 ft x 30 ft and it goes from 2' deep to 4" (which is what I need overall as it will be 4" of concrete for the driveway. Please HELP Rose in GA.
Answered by Penny Nom.
Completing the square 2009-10-02
From christopher:
how do i find the factors of x^2+3x+2 by completing the square ?
Answered by Robert Dawson.
Three circles 2009-10-02
From Brandon:
There is a quarter circle with a radius of 1. along one eged of it, there is a semi-circl with a diameter of 1, and its center is on the drawn line. there is another semi-circle again with the center on the other drawn line, and this one has an unknown diameter of X. both circles are internally tangent, and are tangent to each other. Find X.
Answered by Robert Dawson and Chris Fisher.
A right-angled triangle 2009-10-02
From Hunter:
The three sides of a right-angled triangle measure x-1, x+6, and 2x+1 in length. What are the possible lengths of the hypotenuse?
Answered by Penny Nom and Melanie Tyrer.
A line tangent to a parabola 2009-10-01
From kanchan:
for what value of c a line y=mx+c touches a parabola y^2=4a(x-a)
Answered by Penny Nom.
24 golfers 2009-10-01
From Peter:
I am working on a 19 week golf schedule with 24 golfers, and I would like to know how to make up six foursomes each week without having a player paired with the same golfers.
Answered by Victoria West.
In what order are the following digits? 2009-09-30
From Jewell:
In what order are the following digits? 0,2,3,6,7,1,9,4,5,8
Answered by Penny Nom and Claude Tardif.
Differentiating y= square root(x-1) 2009-09-29
From edith:
describe the x-values at which f is differentiable.
y= square root(x-1)

Answered by Penny Nom.
The volume of a levee 2009-09-29
From Wes:
I need a levee with a 2:1 slope on both sides (front and back), the length of the levee is 290 feet, the height of the levee is 15 feet, and the width of the levee, at the top, is 10 feet. How do I figure out how many CUBIC YARDS of dirt I need to make the levee? And what formula do I use in future to figure this out easily?
Answered by Penny Nom.
Evaluating -x^2 + (yz - 3)^2 2009-09-28
From Kathy:
My daughter and I are having trouble solving this equation:

-x²+ (yz – 3)²

if x=-4,y=2 and z=0 We are having difficulty with the -x² part of the equation. Thanks

Answered by Penny Nom.
A solution of 45% acetic acid 2009-09-28
From Autumn:
if a chemist has a 4 liter solution that consists of 90% acetic acid, how much water does he need to produce a solution of 45% acetic acid?
Answered by Penny Nom.
How fast am i going? 2009-09-28
From ron:
how fast am i going if i drive 40 miles in 20 minutes...what is the formula for figuring this out.
Answered by Leeanne Boehm.
A dog tethered to a round building 2009-09-28
From mido:
One dog was linked to the outer wall of a building round of 20 meters in diameter. If the length of chain linking the dog sufficient turnover of half the distance around the building, What area can guard dog?
Answered by Harley Weston.
The volume of a garbage bag 2009-09-28
From Steve:
I have often wondered how a garbage bags is determined in gallons - many different manufactures of these type bags often have different sizes stating they are a (for example) a 55 gallon bag - also I see some that show a range (20-30 gallon) yet is a specific size (16 x 14 x39 or a 30 x 36) I have searched and search but have been unable to find a formula
Answered by Robert Dawson.
Constructing a scale model 2009-09-27
From Colleen:
What is the formula i need to use to find out how to state a reasonable scale in the question below; Suppose you were constructing a model that must sit on a piece of cardboard that is 22cm by 28cm. STATE THE REASONABLE SCALE that you can use to build a model to represent the following items. Determine the dimensions of the model.

a) a rectangular prism with a length of 5m and a width of 2m?

Answered by Penny Nom.
Probability 2009-09-27
From Ed:
My mother died 3 years to the day after her daughter died. what are the odds of that happening by chance? thanks
Answered by Chris Fisher.
A 15 acre lot 2009-09-26
From Ronald:
The front and back are equal and the two sides are equal what would what would be the dimensions of a 15 acre lot?
Answered by Harley Weston.
The zeros of a polynomial 2009-09-25
From jordan:
find a polynomial of lowest degree with only real coefficients and having the given zeros. 1+2i, 2 (multiplicity 2)
Answered by Penny Nom.
Cubic feet of concrete 2009-09-24
From aLAN:
How many CF of concrete does it take to pour a 1000 SF area with concrete at 4" thick? I would like to see the steps and how you approach this too...thanks
Answered by Penny Nom.
Sandra's age 2009-09-24
From cindy:
Sandra is more than 20 years old and less than 60 years old. You can count by 7's to reach Sandra's age. Next year you will be able to count by 5's to reach Sandra's age. How old is Sandra?
Answered by Penny Nom.
5 x 8 + 6 divided 6 - 12 x 2 2009-09-24
From Susan:
5 x 8 + 6 divided 6 - 12 x 2. I am not sure of the rules of operation for this type of question
Answered by Penny Nom.
The worth of a word 2009-09-23
From Kaliegh:
If each letter of the alphabet is worth one cent more than the previous letter and the letter "a" is worth one cent. What words would have a worth equal to 1 cent,10 cents,50 cents,87 cents 64 cents and $1.00?
Answered by Robert Dawson.
Pennies, nickels and dimes 2009-09-22
From Kiadadh:
Show 12 ways to come up with 28 cents using only pennies, nickles and dimes
Answered by Robert Dawson.
Toss a coin and roll a die 2009-09-21
From Celeste:
Can you please tell me how to set up this problem to find the answer. I know they are independent from there I don't know. a coin and a regular six-sided die are tossed together once. What is the probability that the coin shows a head or the die has a 5 on the upward face?
Answered by Penny Nom.
24 golfers: 4 days 2009-09-21
From Steven:
We have 24 golfers (6 A's, 6 B's, 6 C's & 6 D's) playing 4 days. Each day a foursome consist of an A, a B, a C & a D player. Can you give me a pairing list so that no two golfers will play on the same foursome for the 4 days?
Answered by Chris Fisher and Victoria West.
Probability: selecting students in a particular order 2009-09-21
From Rajshri:
A class has 100 students with roll numbers 1 to 100. Six students are selected, one after another, at random to form a team. Find the probability that the roll numbers of first three students selected are in ascending order and the roll numbers of the last four students selected are in descending order.
Answered by Robert Dawson.
Hexadecagon 2009-09-20
From Rick:
Is there an easy way to figure the even side lengths of a Hexadecagon in layman's terms, so I know how long to cut the exterior support boards for my pool deck. The pool is a 16' diameter Hexadecagon and my Wife wants a 4' wide splash deck all the way around which would make the outside 24' in diameter.
Answered by Chris Fisher and Harley Weston.
Extraneous solutions 2009-09-20
From iyana:
what is an extraneous solution? what must you do to determine whether a extraneous solution?
Answered by Stephen La Rocque and Harley Weston.
Seven circles 2009-09-20
From Bobbi:
try to put number 1 to 7 in seven circles (one in the middle, 3 on top, 3 below) so the numbers in each row of three circles--vertical, horizontal, and diagonal -- add up to 12. Each number can be used only once.
Answered by Stephen La Rocque.
Segments of a ring gasket 2009-09-20
From Robert:
I am in the process of making an Excel spreadsheet in which our sales team just needs to enter the outside diameter, inside diameter, and number of segments to price ring gaskets that are too big to fit on a sheet of material and need to be cut into segments. With your help I was able to create a spread sheet that can calculate the Chord lengths, and Segment height on a single gasket segment. I am now stuck trying to come up with a formula to figure out the height of the second segment when it is stacked on the first segment, then use it to add more depending on the quantity of segments needed. I have an illustration below showing 2 segments (of a gasket that was segmented into 4 pieces) stacked together. I need to find a formula to get the dimension from "A" to "B".
Answered by Harley Weston.
Ordering numbers 2009-09-19
From tattiana:
how do you order these numbers from greatest to least .625,.75,.20, and .5
Answered by Penny Nom.
What is my number? 2009-09-18
From Hanna:
What is my number?
My number is a perfect square.
The only number in its prime factorization is 2.
My number is a factor of 32.
The sum of its digits is odd.

Answered by Penny Nom.
Repeating decimal to fraction 2009-09-18
From joan:
how can i answer it easily to convert terminating decimal to fraction? example of this is that, convert ...143143143... to fractional form
Answered by Robert Dawson.
How long would it take the students to arrive at their campsite 2009-09-18
From amanda:
..."A group of 5th grade student in New Zealand are going camping. They will hike from Wellingto to Ruapehu. Then they will follow a trail for another 1/2 mile to their campsite." Ok, 1 mile equals 5,280 feet. The class decided one step length is equal to 15 inches and there are 85 steps in 1 minute. Now for the question: About how long would it take the students to arrive at their campsite, if they don't make any stops???
Answered by Robert Dawson.
Dart teams 2009-09-18
From Sally:
I have 14 dart teams. Each team has a home bar.Usually it is one week home, the next week away. I have 3 locations that have one board only, yet have 2 teams at each location. I have one location that has 2 dart boards, and 3 teams. The last location have 3 dart boards and 4 teams.Can you make me a schedule that will be home and away, and every teams plays each other twice. In case you are wondering, the bars sponsor the teams.Thank you. Sally
Answered by Robert Dawson.
solve integral of ( x^2+x+1)^5 2009-09-18
From jaka:
solve integral of ( x^2+x+1)^5
Answered by Robert Dawson.
Exponential form 2009-09-18
From karen:
Q: Write the following in exponent from: a * a * a * a * * * a, where there are n -1 a's

note: *asterisk means times.

Answered by Penny Nom.
Two equations in two unknowns 2009-09-18
From Citizen:
x+-3y=7
-x+4y=7

Answered by Penny Nom.
Elliptic space 2009-09-18
From Htet:
What is elliptic space and geometry?
Answered by Robert Dawson and Chris Fisher.
how many ones are in 1 million 2009-09-17
From djuan:
how many ones are in 1 million
Answered by Harley Weston.
The method of substitution 2009-09-17
From laura:
ok! i really need help with this question plz help!!

x= - 4y +5
x+2y = 7

Answered by Penny Nom.
at+b=ar-c, solve for a 2009-09-16
From kimberly:
at+b=ar-c, solve for a
Answered by Penny Nom.
10x-8 /3 =6x+24 /5 2009-09-16
From Heidi:
10x-8 /3 =6x+24 /5.....I know the answer is 7/2 but how do I get there?
Answered by Penny Nom.
Common multiples of 36 and 48 2009-09-15
From Kamaldeep:
Find the first 2 common multiples of 36 and 48.
Answered by Penny Nom.
Ordering fractions 2009-09-15
From stacy:
I need help with listing fractions from the least to greatest. last problem of math is 5/6,7/9,23/27,13/18?
Answered by Penny Nom.
Order from least to greatest 2009-09-15
From myra:
which list of number is ordered from least to greatest? A.) 0.035,0.35,2/5,2.5 B) 2.5,2/5,0.35,0.035 c)2/5,0.35,2.5,0.035 D) 0.035,2.5,0.35,2/5
Answered by Penny Nom.
Digging a trench 2009-09-15
From Darrell:
In digging a pipe trench that measures 29 feet long, 2 feet 9 inches wide, and 3 feet 6 inches deep, how many cubic yards of earth were removed from the site?
Answered by Penny Nom.
A trillion grains of rice 2009-09-14
From akaila:
can 1trillion rice grains fit in a classroom with the area of 144.4m3
Answered by Penny Nom.
Friends in a class 2009-09-14
From pran:
it is known that among any group of three students in a class two of them are friends. the total number of students is 25. prove that there is a student who has at least 12 friends
Answered by Claude Tardif.
Domain and range 2009-09-11
From Aubrey:
I'm supposed to "state the domain/range of each of the following relations" and it's totally not making sense...

Examples:

{(x,y) : x = square root of y}

{(x,y) : y = x + 1, x is some even integer}

Thanks in advance!!

Aubrey

Answered by Robert Dawson.
The slope-intercept form 2009-09-11
From Shelby:
Write an equation in slope-intercept form of the line that is parallel to the graph of 3y-4x=1 and passes through (0,6)
Answered by Penny Nom.
Jogging and swimming 2009-09-11
From Brandon:
Trevor swims jogs 3 days and swims every 4 days. How often does he jog and swim on the same day
Answered by Leeanne Boehm.
Sagitta 2009-09-10
From Robert:
Can you please tell me if there is a formula to figure out the Sagitta of an arc when you know the radius, chord length, and arc length?
Answered by Chris Fisher and Harley Weston.
The volume of a closed 3D mesh object 2009-09-10
From Ozen:
I want to calculate the volume of a closed 3D mesh object having a surface made up triangles.
Answered by Robert Dawson.
Exponential form 2009-09-08
From hanna:
hi, well i am really bad in math and i have a math paper and i was just wondering what 3x3x3x3x3 was in exponential form and what is exponential in a way a person like me would understand?
Answered by Penny Nom.
A ladder leaning against a wall 2009-09-08
From Mackenzie:
A 7 foot long ladder is leaning against the building. The foot of the ladder is 2 feet from the base of the building. How far up the wall is the top of the ladder?
Answered by Stephen La Rocque.
A circular border around a pool 2009-09-08
From Calvin:
A pool in the shape of a circle measures 10 feet across. One cubic yard of concrete is to be used to create a circular border of uniform width around the pool. If the border is to have a depth of 3 inches, how wide will the border be? ( 1 cubic yard=27 cubic feet )
Answered by Stephen La Rocque.
Helen has twice as many dimes as nickles. 2009-09-08
From Dan:
Helen has twice as many dimes as nickles. She has 5 more quarters than nickles. She has $4.75. How many nickles does she have? I know she has 14 dimes, 7 nickles and 12 quarters. How do you put this into an algebra equation?
Answered by Stephen La Rocque.
A triangle on a sphere 2009-09-07
From Rohit:
How do I find the angles of a triangle drawn on a sphere (spherical triangle)?
Answered by Chris Fisher.
f(x + h)-f(x) 2009-09-07
From Marie:
For each function defined as follows, find a.) f (x+h) b. f(x+h)-f(x) c. [f(x+h)- f(x)]/h.

Problem: f(x)= -1/ x^2.

Answered by Harley Weston.
10 golfers 2009-09-06
From Bill:
We have 10 golfers scheduled to play over five days on vacation in November. Each day we will play in three groups of three and one foursome. Is there an optimum solution that would assure that every golfer plays with every other golfer at least once while no golfer plays an inordinate number of times with any other golfer or too often in a foursome?
Answered by Victoria West.
Divide and reduce to lowest terms. 2009-09-06
From Karen:
Divide and reduce to lowest terms. Use the cancellation technique as needed 4 2/3 divided by 12 14/3 divided by 12/1 not sure wheer to go from here with the cancellation technique
Answered by Penny Nom.
4 couples seating themselves at a round table 2009-09-04
From Hui:
Q: 4 couples seating themselves at a round table. Men must seat together and women seat together. How many ways are there? Answer in the printed text is 576. I got 144. Pls advice.
Answered by Claude Tardif.
A four-digit positive integer 2009-09-04
From TRACEY:
I AM A FOUR-DIGIT POSITIVE INTEGER SUCH THAT THE SUM OF MY DIGITS IS 18 AND MY DIGITS REVERSED ARE EXACTLY FOUR TIMES GREATER THAN MYSELF. WHAT NUMBER AM I?
Answered by Robert Dawson.
Simplify 2009-09-04
From Sarah:
How do you simplify: (x-4)^3?
Answered by Robert Dawson.
Airline overbooking 2009-09-03
From Nikita:
An airline company knows that 8% of it's passengers will not show up for their scheduled flights. A plane has 175 seats.

a) What is the probability that 10 passengers or fewer will not show up?

b) What is the probability that 10 to15 passengers will not show up?

c)What is the probability that exactly 10 passengers will not show up?

d) What is the probability that more than 19 passengers will not show up?

Answered by Robert Dawson.
Nickels and dimes 2009-09-03
From MATTHEW:
mike has 3 times as many nickels as Larry has dimes. Mike has 45 cents more than Larry. How much money does Mike have?
Answered by Penny Nom.
Show your work 2009-09-02
From Gerald:
We have a 6th grade student who can solve math problem successfully without showing her work. The teacher thinks it is not fair that she doesn't show her work and the other have to and do. What sort of classroom accommodation(s) would you recommend for this type of student. It would seem to be a popular problem since there are many student who think more global than sequential.
Answered by Victoria West, Penny Nom and Robert Dawson.
Normal Distribution 2009-09-02
From Nikita:
Scores on a college exam are known to be normally distributed with a standard deviation of 20. If the top 3% have scores in excess of 200, what is the mean score?
Answered by Robert Dawson.
Is -100 degree angle an acute angle? 2009-09-02
From Nazrul:
Is -100 degree angle an acute angle? Please explain. Here -100<90.
Answered by Robert Dawson.
Nickels, dimes and quarters 2009-09-02
From danielle:
In changing a $5 bill, Sarah received nine more dimes than nickles and seven fewer quarters than dimes. How many coins of each type did she receive?
Answered by Penny Nom.
Zero Angle 2009-09-01
From Nazrul:
Is zero degree an acute angle? Please explain.
Answered by Janice Cotcher.
What is 8 tons in exponential form? 2009-09-01
From Tyler:
what is 8 tons in exponential form
Answered by Penny Nom.
Cardinality of infinite sets 2009-09-01
From Brian:
I was reading an answer to a question on your site regarding infinite sets (http://mathcentral.uregina.ca/QQ/database/QQ.09.01/carlos1.html), and I think they may have got the answer wrong.

I his example, he claims that the set of real numbers BETWEEN 0 AND 1 is larger than the set of positive integers.

Please correct me if I am wrong, but I believe those two sets are -- pardon the expression -- equally infinite. For any integer, there is a corresponding real number between 0 and 1, and vice versa.

For instance, using the decimal as a "mirror", you can create a mirror image of any real number between 0 and 1 as an integer (i.e. 0.1234 gets mirrored as the integer 4321 -- I could write it out algebraically, if you want, but you get my point)

Am I wrong?

Thanks, Brian

Answered by Victoria West.
I have 9 players 4 rounds of 3 2009-08-31
From Larry:
I have 9 players 4 rounds of 3. I need each player to play one timw with everyone in the group
Answered by Victoria West.
Exponential form 2009-08-31
From cecil:
what is the exponent form 564000?
Answered by Stephen La Rocque and Harley Weston.
The equation of a circle 2009-08-30
From tagbo:
find the eqn of circle passing through (-1 1) and (-1 -4)and whose radiusis 5/2
Answered by Stephen La Rocque.
An area problem 2009-08-30
From Amber:
Area of hexagon ABCDEF with the vertices (3,0) (7,0) (9,2) (7,4) (3,4) (1,0)
Answered by Harley Weston.
Investing in four accounts 2009-08-29
From Kenneth:
Hello: An investor wants to divide $1000.00 into four amounts or accounts -- one account earning 2%, the second account earning 5%, the third earning 9%, and the fourth earning 12%. He wants the overall total return to equal 5%. What are some of the monetary amounts that can be invested into these four accounts to earn him the 5% total return, 5% of $1,000.00?
Answered by Penny Nom.
Simultaneous equations 2009-08-28
From onias:
solve 3/a - 2/b = 1/2 , 5/a + 3/b = 29/12
Answered by Robert Dawson.
y=2x+1 and y=2x-1 2009-08-28
From MARICELA:
Need help of how to work this problems

y=2x+1 and y=2x-1

First of all what is the difference?

Second I think is easy just to work with the X y with the line in the middle

but how do you get the numbers for each side.

Answered by Penny Nom.
Chickens and pigs 2009-08-28
From David:
A farmer has a number of chickens and pigs. There are 24 heads and 80 ft. How many pigs and how many chickens are there?
Answered by Robert Dawson.
The distance traveled on a bike 2009-08-28
From Dennis:
I would like to calculate the distance traveled on a bike based on the size of the tire and on the number of revolutions per minute. This is based on the rider traveling at a constant speed.
Answered by Stephen La Rocque.
A number puzzle 2009-08-28
From Angela:
Taking 1 through 9 and using them only once subtract them to equal 33333
Answered by Chris Fisher.
2(x-4)>-4+2x 2009-08-27
From jorna:
2(x-4)>-4+2x
Answered by Penny Nom.
The game of 24 2009-08-26
From melvi:
how can you get the numbner 24 but only useing the numbers 7,5,4,5
Answered by Penny Nom.
How do you convert cents/mile to dollars / hour? 2009-08-26
From seanna:
how do you convert cents/mile to dollars / hour
Answered by Penny Nom.
The taxable part of an invoice 2009-08-25
From Carol:
Question from CAROL: THE INVOICE TOTAL IS 2731.90 BUT IT DOES NOT SHOW WHAT THE NET AMOUNT IS THAT THE STATED GST OF 121.90 IS. HOW DO I CALCULATE THE NET AMOUNT THAT THIS GST AMOUNT REFERS TO?
Answered by Harley Weston.
How deep is the hole? 2009-08-25
From Scott:
If I have a hole three foot in circumference that runs at a 15degree angle from the surface and continues for a length of 100 feet, how deep would the hole be at 100 feet?
Answered by Penny Nom.
Fact families 2009-08-24
From Camasha:
I am trying to help my daughter with fact families. I understand the premise from your previous explanation on the site . Generally when applying addition and subtraction their are two add and two subtraction utilizing all three numbers. However what do you do when you have a series like like 9, 9, 18
Answered by Penny Nom.
Linear equations 2009-08-23
From shwetha:
The length of a rectangle is greater than the breadth by 18 cm. If both length and breadth are increased by 6 cm then area increases by 168 cm square. Find the length and breadth of the rectangle.
Answered by Stephen La Rocque.
The area of a rectangle 2009-08-21
From roxanne:
the dimensions of the rectangle: 3x length and 4x width. What is the area of the rectangle in square units?
Answered by Penny Nom.
Unlocking my suitcase 2009-08-21
From Jamboree:
I need to have all the possible combination of three digit between 0-9, i have seen the answer that there are 720 combination are there. Could you please show me all those combination numbers so that i could try it out to unlock my suitcase, will you please write it down all those combination and send it to me
Answered by Victoria West.
Graph 2x-3y+15=0 2009-08-20
From Diem:
Graph 2x-3y+15=0
Answered by Leeanne Boehm.
Fourth and fifth degree polynomials 2009-08-20
From Evin:
hello.i am a student . ax^4+bx^3+cx^2+dx+e=0 x=? i want to learn the solution or formula of equations of the fourth and fifth degree...PLEASE
Answered by Robert Dawson.
A paper towel roll 2009-08-19
From Jeff:
I am making a spiral tube with paper that is 2" in dia. and 102" long I will be using paper that is slit 3" wide how many lineal feet of paper will I need to to cover the 102" I will be using 3 rolls of paper that will over lap the other by half to make a hard tube (paper core) in a roll of paper towels Thanks Jeff
Answered by Penny Nom.
Golf for 15 2009-08-18
From Judy:
We have a group of 15 golfers that play a scramble with groups of 3. The problem is we play for 14 weeks. We want every player to play at least once with everyone and no more that 3 times. I am working on this and have just about accomplished the schedule, but with a few playing more than 3 times. I'm sure there must be some mathematical calculation to do this but I have yet to figure it out.
Answered by Victoria West.
Find the central angle 2009-08-18
From Larissa:
In a circle, the length of a chord AB is 4 cm and the length of the arc AB is 5 cm. Find the central angle theta, in radians, correct to four decimal places. Then give the answer to the nearest degree. I think I'm supposed to use Newton's method, but am not sure how to start with this problem.
Answered by Harley Weston.
The layout of an arch 2009-08-18
From Steven:
I am trying to layout a large radius between 2 points in a building and need a formula to figure different senarios for example: radius is 187'6" distance between 2 points is 34'8" need points 16" apart along the line between the 2 points to create the radius please help
Answered by Stephen La Rocque.
Sequence problem 2009-08-18
From James:
I have the equation: tn = (n)(n+1)/2 Which gives the sequence: 1, 3, 6, 10, 15 etc. However, I'd like to be able to solve for n. So that a t of 6 would give me the answer of 3, and t of 10 would give me the answer 4 etc.
Answered by Stephen La Rocque and Victoria West.
Big and small widgets 2009-08-17
From Leanne:
A machine makes big widgets. Another machine makes small widgets. 3 small widgets have the same mass as 2 big widgets. It takes the same amount of time to make 3 big widgets as it does to make 5 small widgets. The machines start at the same time and make widgets until the total mass of all the widgets made is equal to the mass of 380 small widgets. What is the total number of widgets made? (Please include workings).
Answered by Penny Nom.
A football schedule 2009-08-14
From Bill:
I am having trouble with a football schedule that has two divisions of five teams each, 1-5 and 6-10. 1-5 will play against each other in the first 4 weeks and also the the last 4 weeks with teams 6-10 completing the middle of the schedule. The schedule is 13 weeks. Thank you for your time
Answered by Chris Fisher and Victoria West.
An antiderivative problem 2009-08-13
From Indrajit:
∫4e^x + 6e^-x/(9e^x + 4e^-x)dx = Ax + Bloge(9e2x - 4) + C

then A=?......B=?.....C=?

plz solve it...."^" stands for "to the power of"....

Answered by Harley Weston.
A rectangular pen 2009-08-13
From Kari:
A rectangular pen is to be built using a total of 800 ft of fencing. Part of this fencing will be used to build a fence across the middle of the rectangle (the rectangle is 2 squares fused together so if you can please picture it). Find the length and width that will give a rectangle with maximum total area.
Answered by Stephen La Rocque.
Fitting the curve y=a*exp(b*x)+c 2009-08-12
From aika:
Could one show me the complete algorithm and formula for finding the coefficients (a,b, and c) in exponential regression model y=a*exp(b*x)+c
Answered by Robert Dawson.
Solve for s 2009-08-12
From Michelle:
My daughter and I are having trouble with this problem...All it says i solve thr formula for the indicated variable.
Height of an Equilateral Triangle.

Solve for s:

H= square root of 3/2 * S

Answered by Penny Nom.
The volume of a telephone pole 2009-08-11
From robert:
12.5"@ base x 7" @ top and 40' height. How would I find the volume in cubic feet for a telephone pole with these dimensions?
Answered by Harley Weston.
The volume of a tapered pole 2009-08-09
From robert:
what is the formula to use to come up with cubic feet in a tapered pole
Answered by Harley Weston.
Gravel in an aquarium 2009-08-09
From Bev:
Hi...I worked this part out, but want to know how many kilograms of gravel I will need...thanks Bev

Cylinder shape.

A cylindrical shape that has a radius of 82 centimeters and a depth of 7 centimeters has a volume of: Volume

= 0.14787 cubic meters
= 147870 cubic centimeters
= 0.1934 cubic yards
= 5.2219 cubic feet
= 9023.5 cubic inches
= 39.063 U.S. gallons
= 147.87 liters
* calculations accurate to 5 decimal places of precision

Answered by Harley Weston.
The leaning tower of Pisa 2009-08-09
From MF:
Would you have any idea how the 'latitude of 44 degrees N" has anything to do with this question and how I would apply it?

The leaning tower of Pisa leans toward the south at an angle of 5.5 degrees. One day near noon its shadow was measured to be 84.02 m long and the angle of elevation from the tip of the shadow to the top of the tower was measured as 32.0 degrees. To answer the question, assume that the tower is like a pole stuck in the ground, it has negligible width. Also, it is important to know that Pisa Italy is at a latitude of approx 44 degrees North because this affects the direction of the shadow.)

Answered by Stephen La Rocque.
An isosceles triangle 2009-08-09
From Megan:
Find the perimeter of an isosceles triangle with a verticle angle of 100 degrees and a base of 25 cm.

I think I could answer this if I knew what a verticle angle was.
Thanks

Answered by Stephen La Rocque.
Eye Height Level Found from Visual Angle 2009-08-07
From Jolie:
Hello, I am trying to figure out what height from the ground a person's eyes are if they have a 12 degree visual angle to a screen that is 58cm away from their eyes? Thank you.
Answered by Janice Cotcher.
The length of tape on a reel 2009-08-06
From Philiip:
What diameter reel would be required to roll up 1000 meters of tape that is 0.01 mm thick such that the final role left 10 mm empty space between the top of the roll and the outside of the reel, the hup that the tape was wound on was 2 cm?
Answered by Stephen La Rocque.
Number of Concrete Blocks 2009-08-06
From Simone:
Hi there, I have a quote to complete and I need to figure out the formula that will tell me how many concrete blocks I need to cover a certain area. Blocks can either be 8in or 12in. The area that I am looking to cover with blocks is 30182 sq ft. Thank you for your assistance.
Answered by Janice Cotcher and Stephen La Rocque.
Exponential form 2009-08-05
From motaz:
Let x = log_2 1/8 Write the exponential form of the equation and solve the equation for x
Answered by Stephen La Rocque.
Angular speed 2009-08-04
From Mary:
The engine of a sports car rotates at 5000 revolutions per minute (rpm). Calculate the angular speed of the engine in radians per second. Use 2 radians = 1 revolution.
Answered by Penny Nom.
Tossing a penny 26 times 2009-08-04
From Basma:
Jack is going ot toss a penny 26 times.
How many times should it land on heads?

Answered by Robert Dawson.
A spherical dome 2009-08-03
From Eric:
I have recently been asked to resurface a dome sculpture for my local council but i'm having problems working out the area. Here are the dimensions. The height of the dome is 3m from the ground to the top of the arc. The arc itself from the ground rising up to the 3m point and back down, is 10m. The dome is 7m wide from one side to the other through the centre, at ground level.
I hope there is enough detail here. It's been a long time since i was in a maths class. Thank you, hope to here from you soon!

Answered by Stephen La Rocque.
Math in everyday life 2009-08-03
From Naveen:
Dear sir, We are advised to do a project on "Mathematical modeling to solve various problems of our everyday life/environmental related problems...... Can u plz help us by mailng some ideas, suggestion,reference to make my project successful.... Thanking you...... Waiting for your favourable reply......
Answered by Penny Nom.
Two circles 2009-08-03
From Karan:
We are given 2 circles with radii 12cm and 3 cm. We have to find AB
Answered by Penny Nom.
A walkway around a flower garden 2009-08-01
From Barbara:
A rectangular flower bed measuring =C2=A0 10 feet by 15 feet =C2=A0 has a surrounding walkway on all sides that measure 186 square feet. How wide is the walkway?
Answered by Harley Weston.
root 2 is irrational 2009-07-31
From prateet:
prove that root2 is an irrational number.
Answered by Robert Dawson.
Two word problems 2009-07-31
From Lee:
1) a student saved $50.25 in one month. at that rate how long will it take the student to save $700?

2) for summer job, martin shampooed rugs. he could finish a 4mx 4m rug in 30 min. his supervisor could finish a similar:smaller rug in 20 min. how long would it take for them to finish the rug if they worked together?

Answered by Robert Dawson.
Two equations in two unknowns 2009-07-30
From Alicia:
I need help with the following equation please show me the steps to complete so that I will know how to do this my self.
2x-y=2 5x+y=5 Thanks for the help

Answered by Penny Nom.
A pentagon inscribed in a circle 2009-07-29
From Faisal:
O is the center of the circle. Sides AB and AE are equal. Angle B = 95, angle C = 130, angle D = 138. Find angles A and E. Diagram is attached. Best, Faisal
Answered by Chris Fisher.
Dimensions 2009-07-29
From Deborah:
The blueprint for landscaping a yard has a scale of 1/2" to 1 foot. If the blueprint is a rectangle 18 inches by 22 inches, what are the dimensions of the yard?
Answered by Robert Dawson.
Things equal to the same thing 2009-07-29
From nazrul:
If p^2-pa+q=0 and m^2-ma+n=0 , can we write p^2-pa+q= m^2-ma+n? Please explain.
Answered by Robert Dawson.
Torricelli's trumpet 2009-07-29
From Gary:
I was reading about torricelli's trumpet which is described by the equation1/x which is then rotated around the x axis which results in a figure which looks like a trumpet. Now in order to find the volume the integral 1/x^2 dx is used which diverges when integrated so the volume is finite.However if you integrate 1/x dx which is the formula on the plane the answer diverges. Now if you took an infinite area then rotated it around the x axis shouldn't you get an infinite volume? Notice the area I am talking about is under the line 1/x not the surface area of the trumpet which is what the painters paradox is about What am I missing? Thanks
Answered by Robert Dawson.
Constructing an isosceles triangle 2009-07-27
From Sanjay:
How to construct an isosceles triangle ABC with a base BC = 5.4 centimetre and altitude AD = 4.7 centimetre? Please help.
Answered by Janice Cotcher.
How can I trisect an angle? 2009-07-27
From Nazrul:
How can I trisect an angle?
Answered by Robert Dawson.
The inverse of F(x)=x^2-6x+13, for x>3 2009-07-26
From Juan:
The function F is given by F(x)=x^2-6x+13, for x>3.

A. Write F(x) in the form (x-a)^2+b

B.Find the inverse function F^-1

C.State the domain of F^-1

Answered by Penny Nom.
Inequalities Proof 2009-07-24
From ABOU:
good morning.......a b c are real positive no zero......proof that sq root(2a/(a+b))+sq root(2b/(b+c))+sq root(2c/(c+a))inferior or equal 3 thank you
Answered by Janice Cotcher.
Properties of Natural Numbers 2009-07-24
From nazrul:
If m,n,k are natural number how can I prove that (m+n)k=mk+nk. In the proof the properties of natural number should be used.
Answered by Janice Cotcher.
Proof of a Unique Solution 2009-07-24
From muele:
Find matrix A such that A is not invertible, and b such that Ax=b has a unique solution
Answered by Robert J. Dawson.
Finding Supplementary Angles 2009-07-24
From emir:

Answered by Robert J. Dawson.
Circular Measures 2009-07-24
From onyeka:
find the equation of a circle with point [3,7][5,5] and wich center lies on the line x-4y=1
Answered by Robert J. Dawson.
A degreaser for a fish tank 2009-07-23
From charlie:
I have a tank that is 10 feet in diameter that I want to put floating degreaser in.The rate for the degreaser is 1/2 gallon per square foot of surface water.How do I figure out how many gallons I need?
Answered by Penny Nom.
A father and son problem 2009-07-23
From yomi:
five years ago, a father was 3 times as old as is son, now their combines age's amount 110 years. what is the present age of the father?
Answered by Penny Nom.
Golfing with an unpopular golfer 2009-07-23
From Ian:
"We are a group of 8 golfers, one of whom is unpopular. How can we construct a schedule, of two foursomes, so that each person is scheduled to play with him the same number of times. What does the week by week schedule look like?"
Answered by Robert Dawson and Victoria West.
Units digit 2009-07-23
From Jyll:
How do solve for the units digit? And that, what is the units digit of the first 1000 prime numbers?
Answered by Robert Dawson.
Inequalities 2009-07-22
From Sindhu:
prove that 4^79 < ( (2^100) + (3^100) )< 4^80 ?
Answered by Robert Dawson.
Buying pens and staplers 2009-07-22
From Ellen:
one more math question,
six pens and four staplers cost $7.10. four pens and seven staplers cost $9.50 find the cost of one pen and one stapler.

Answered by Penny Nom.
Selling fish 2009-07-22
From Hemantee:
A fishmonger bought 150kg of fish. he sold 70%of it at the price of Rs90 per kg and the rest at Rs 75 per kg. he made a profit of 42.5 per cent. how much did he pay for the fish.
Answered by Penny Nom.
A sundial on an elliptical cylinder 2009-07-22
From Leo:
I want to build a sundial where the shadow falls on an elliptical cylinder. I can calculate the coordinates of the points on the cylinder that I want to mark. My problem is that I will have to make the surface as a flat sheet and bend it into an elliptical shape. However, I cannot work out a scheme to transfer the coordinates I know into a distance that will work when I bend the shape. Help!
Answered by Robert Dawson.
Area under curve 2009-07-21
From Morag:
What is the area bounded by the curve y=X^2(2-X) and the x axis between x=0 and x=2?
Answered by Janice Cotcher.
The girl guides bake sale 2009-07-20
From Barbra:
here is two questions i need help with,
1.)A toastmasters tea was attended by 550 people. Students paid 75 cents for their tickets, and the adults paid $1.20. if the recipts totaled $525, how many adults and how many students attended?
2.) One cookie and six bars cost $2.00 at the girl guides bake sale. At the same sale, three bars and six cookies cost $2.10. Find the cost of one cookie and the cost of one bar.

Answered by Penny Nom.
Annular sector 2009-07-20
From Ed:
What is the name for the section of a flat ring (annulus)? Similar to a section on a radar screen, a quadrilateral but the top and bottom are curved.
Answered by Robert Dawson.
A golf league with 12 players 2009-07-17
From Jane:
My weekly golf league has 12 players in 4 threesomes. How many weeks would it take to play everyone once and not have too many duplications? What would be the schedule each week? Thanks
Answered by Victoria West.
Portion of Work 2009-07-17
From Ellen:
A girl can shampoo the dog, clean his ears, and clip his nails in 45 minutes. Her sister can do it in 60 minutes. How long will the job take it they work together?
Answered by Stephen La Rocque.
Percentage of Acid 2009-07-17
From Leeanne:
here is my question: a chemist has 200 liters of solution that is 5% acid. how many liters of water (0% acid) must he add to make a solution that is 4% acid?
Answered by Stephen La Rocque.
Pre-Algebra Resources 2009-07-16
From M:
What is the best pre algebra book that is inexpensive and will really help me to understand pre algebra?
Answered by Janice Cotcher.
Finding the Line Joining Perpendicular Feet 2009-07-16
From mukulu:
Please help I've been tryng to search in your data but i failed to get the solution Find the equation of a straight line joining the feet of the perpendiculars drawn from the point A(1,1) to the line 3x-3y-4= 0 and 3x+y-6=0.
Answered by Janice Cotcher.
Flying with the sun in view 2009-07-15
From Randee:
If you wanted to fly in a plane, with a constant "just setting" sun in view, where would you start, what time of day, and what would the altitude and air speed need to be?
Answered by Robert Dawson and Stephen La Rocque.
The lateral area of a cone 2009-07-15
From ashley:
What is the radius of a cone with the lateral area being 443.3 mm^2 and the slant height being 14.7 mm.
Answered by Penny Nom.
Filling a hole 2009-07-14
From BARRY:
I HAVE AN AREA THAT i NEED TO FILL WITH DIRT AND AM TRYING TO FIGURE OUT THE CU. YDS NEEDED. tHEY CIRCUMFRANCE OF THE HOLE IS 28' ROUND AND SLOPES FROM 2.5 FEET AT ONE END TO 0 ' AT THE OTHER. OVERALL THE HOLE IS 30' LONG BY 30' WIDE . THE SLOPE RANGES FROM 2.5 ' TO APPROX 4"
Answered by Harley Weston.
A tangent to a circle 2009-07-14
From Eric:
Hi I am trying to complete a packet that has a list of questions to brush up on pre-calculus skills. The question asks "For the circle x^2 +y^2 + 6x - 4y + 3 = 0 find : the equation of the tangent at (-2,5). I have already found the equation for the circle and standard form and the center and radius. However, i do not know how to find the slope or y-intercept of the tangent line. Please help. Thanks.
Answered by Stephen La Rocque.
The surface area of a rectangular pyramid 2009-07-13
From Shivani:
What is the surface area of a rectangular pyramid with a length of 6 inches, a width of 7 inches and a SLANT height of 4 inches.
Answered by Penny Nom.
The product of two consecutive integers 2009-07-13
From kelly:
the product of two consecutive integers is 9 less than the square of the second integer
Answered by Penny Nom.
Two chords 2009-07-11
From Sarah:
Two parallel chords in the same circle have lengths of 30 cm and 48 cm. The circle has a radius of 25 cm. How far apart are the chords?
Answered by Penny Nom.
Arc length 2009-07-10
From farhad:
hi i need to measure length of arc by having only two measurements, first the length of the chord and the height of the curve. if i have a simple calculator that haven't sin cos tang (i want to calculate it in my mind)
Answered by Stephen La Rocque.
A circle and a chord 2009-07-10
From Paul:
I have three known points. X1,Y1 = 0,0 X2,Y2 = 3,1 and X3,Y3 = 10,0. Given these three points, how do I find the height from the center of the 10' chord to the circle's circumference above?
Answered by Stephen La Rocque and Harley Weston.
A base 7 number and a base 9 number 2009-07-09
From koteawarao:
Find a base 7 three-digit number which has its digits reversed when expressed in base 9.

ans: (281) base 7 and (182) base 9

Answered by Harley Weston.
Multiplying Large Numbers 2009-07-09
From Jessica:
What is 12 billion X 6 trillion?
Answered by Janice Cotcher.
Multiplying Large Numbers 2009-07-09
From Jessica:
What is 12 billion X 6 trillion?
Answered by Janice Cotcher.
Log_7 (4X) = Log_14 X 2009-07-08
From Mukulu:
Help me with this question because i tried my level best to get the right answer but i failed Log_7 (4X) = Log_14X
Answered by Stephen La Rocque.
x^3+1/x^3=18*sqrt(3) 2009-07-08
From Nazrul:
If x^3+1/x^3=18*sqrt(3) then how can I evaluate the value of x+1/x? Thank you for your help.
Answered by Robert Dawson.
The sides of a triangle 2009-07-08
From Susan:
The second side of a triangle is 5ft more than the length of the first side. The third side of the triangle is 10ft less than the length of the first side. The perimeter of the triangle is 61ft. Find the lengths of each side of the triangle.
Thank you so much!

Answered by Penny Nom.
Sand for sandbags 2009-07-07
From Jerry:
How much sand do I need to fill 750 sandbags that are 26 inches by 14 inches if I fill them at 60% of their full capacity?nad
Answered by Janice Cotcher.
Simultaneous Equations 2009-07-06
From Mukulu:
Solve the equation simultaneously X/5=(Y+2)/2= (Z-1)/4 ……………….eqt 1 3X+4Y+2Z-25=0 ………………eqt 2
Answered by Janice Cotcher.
Using the Remainder Theorem to Find an Unknown Coefficient. 2009-07-06
From Mukulu:
What is the value of a if 2x2-x-6 ,3x2-8x+4 and ax3-10x-4 have a common factor.
Answered by Janice Cotcher.
Number of Farm Animals 2009-07-06
From paul:
In the farm there are 50 heads of chicken and pigs and 100 feet. how many chickens and pigs are there in the farm
Answered by Robert J. Dawson.
The retail price for an ice cream scoop 2009-07-05
From Patricia:
Hi - I am trying to figure out the fairest retail price for an ice cream scoop. We have an ice cream parlor and the cost of the ice cream we buy by the bulk container has recently been increased so now we must charge more for our scoops. We pay a wholesale price of $25.40 for 242 ounces of ice cream. The cost per ounce is $0.105. What is the formula for figuring out how much to charge retail per scoop? This is probably so simple but confusing to me. Please help. Thank you!
Answered by Harley Weston.
A roll of paper 2009-07-05
From mark:
is there a simple way of finding out how much is left on a roll of paper. i have read similiar questions and answers on here but all seem very complicated and not being very good at maths does not help me much. some are in inches and all have different figures to mine so if i give my figures hopefully i wil understand it better. the radius of the cardboard core is 52.25mm, the radius of the paper at 2000 linear meters is 158.625mm. the thickness is 0.17mm. when nearing the end of the paper how do i work out how much is left in a simple way. thank you in advance for any help you can give me
Answered by Harley Weston.
The area of a triangle 2009-07-02
From Rory:
I'm trying to find the area of a triangle with points -2, 1; 4,1 and 7,4
Answered by Penny Nom.
A diagonal of a rectangle 2009-07-01
From Ruben:
Please assist me with the following problem: A rectangle has a diagonal that is 3.6 feet longer than the length and 7.1 feet longer than the width. What are the dimensions of the rectangle? Thanks
Answered by Harley Weston.
Two circles on a dome 2009-07-01
From Beth:
My question is related to a dome I would like to construct, for which I know the circumference of the base: 120ft. I now need to figure out the diameters of two smaller circles, one at 20ft along the arc of the dome form the ground, and the other at 30ft along the arc. Assuming a true hemisphere, or 180 degrees total arc, how can I calculate these two circumferences?

Beth

Answered by Stephen La Rocque.
Parallel lines 2009-07-01
From chris:
Write the slope of a line that is parallel to each line.
1.y=2x-5
2.y=-x+2
3.3x+y=10
4.5x-y=11
5.x+2y=6
6.2x-3y=9
7.4x+y=3
8.x+2y=14

Answered by Brennan Yaremko.
liquid in a 3/4 inch pipe 2009-06-30
From junior:
We are in dilemma at my job. We need to figure out the formula for how much water can our pipe hold. It's inside diameter is 13/16" and is 50ft. long?
Answered by Robert Dawson.
The extended real number system 2009-06-30
From Justin:
Hi again, thanks a lot for answering my previous question! I was also just wondering again if the extended real number system has a potential or actual infinity because it includes positive infinity as a point that exists at the end of it?

All the Best,

Justin

Answered by Robert Dawson.
The surface area of a fire pit 2009-06-30
From eric:
whatw is the square footage of the inside wall of a fire pit 2 feet deep and five feet in diameter? I need to estimate both the floor and wall square footage to order bricks.
Answered by Robert Dawson and Harley Weston.
Finding an Acute Angle using Trigonometric Identities 2009-06-29
From Nazrul:
How can I find the value of A if sinA-cosA=1 , where A is an acute angle.
Answered by Stephen La Rocque.
Potential infinity and actual infinity 2009-06-29
From Justin:
Hi there, I was just wondering what is the difference between the potential infinity and actual infinity in math? Thanks a lot for your help with this question!

All the Best,

Justin

Answered by Robert Dawson.
Coefficient thermal expansion of steel 2009-06-29
From roshni:
Coefficient thermal expansion of steel is 0.00000645/in/in/deg F if F was C(celcius) then what is the answer
Answered by Robert Dawson.
Proportionality 2009-06-29
From Nazrul:
If x is proportional to y and y is proportional to z , how can I prove that (x^2+y^2)^3/2 is proportional to z^3
Answered by Harley Weston.
Integers and decimals in day to day life 2009-06-29
From mitichie:
can you tell use of integers and decimals in our day to day life.
Answered by Penny Nom.
Designing a shelf 2009-06-28
From Bjorn:
I'm putting up a shelf and thought I'd be nice and cut the sharp and pointy corners off. I want to the cut to be at a 45 degree angle, but I also want the exposed edge to be the same length as from the wall to the beginning of the cut -- so the end of shelf will look like the first two sides of an octagon. I've managed to construct the solution, but I haven't been able to calculate it...
Please help!

Answered by Stephen La Rocque.
Eight golfers 2009-06-26
From E:
Hello,
We have a golf tournament. 8 players, 3 days, 2 fourballs per day. How can each player play with each other no more than twice, and at least once? Would like a speedy answer as we're going tomorrow!
Thank you

Answered by Laura Morrison.
Height of a Flag Pole 2009-06-26
From Jake:
Can anyone answer this I am very stumped. The flagpole on the roof of a Chocolate Factory is 13 cm in diameter and a perfect cylinder. If one end of a 2300-cm-long string is attached to the top of the flagpole, and wraps around the flagpole exactly 16 times before ending at the bottom of the flagpole, how tall is the flagpole? Need answer by Wednesday
Answered by Penny Nom.
Twelve golfers 2009-06-26
From Fred:
Hello- There are twelve golfers in 3 foursomes for three days. Is there a combination of four man foursomes that allow for no duplicates? In other words, is there a formula to insure the greatest amount of variety in the foursomes. Looking at it math like you have 1,2,3,4,5,6,7,8,9,10,11,12 grouped into four numbers each for three days. So if your have the first day 1,2,3,4 in one group, 5,6,7,8 in the second and 9,10,11,12 in the third what can you do for the next two days to have the fewest duplicates or have the greatest variety in the foursomes? Thanks
Answered by Karen Meagher and Victoria West.
Investing in multiple accounts 2009-06-26
From Kenneth:
Hello:

If an investor has $1000.00 to invest in multiple accounts, and he wants a total return of 4%, is there one calculation that can be used to determine what these amounts could be even though there may be numerous amounts used as answers for most of the following examples?

For example,
Invest $1000.00 @ 2% and 5% for total return of 4%.
Invest $1000.00 @ 2%, 3% and 5% for total return of 4%.
Invest $1000.00 @ 2%, 3%, and 5% for total return of 4%.
Invest $1000.00 @ 2%, 3%, 4% and 5% for total return of 4%.
etc.

Answered by Robert Dawson.
Infinite-Dimensional Spaces 2009-06-26
From Justin:
Hello again, I was also just wondering (in Hilbert Space and Function Space) are there infinite-dimensional spaces larger than each other in terms of cardinality? Thanks a lot for your help again! All the Best, Justin
Answered by Victoria West.
The surface area of a tank 2009-06-25
From Charles:
I need to calculate the external wall and top sqft surface area of a tank 54ft od x 15ft high with a dome top 54ft od x 8ft high.
Answered by Penny Nom.
(570 divided by 15) multiplied by (5 plus 26) 2009-06-24
From Sherrie:
What is the answer to this math question:
(570 divided by 15) multiplied by (5 plus 26)..........
Thank you,
Sherrie:-)

Answered by Penny Nom.
The Pythagorean theorem 2009-06-24
From supreet:
What are some real-world applications of the Pythagorean theorem?
and
How are the Pythagorean theorem and the distance formula related?

Answered by Harley Weston.
Is one Infinity larger than another in the extended real number system? 2009-06-24
From Justin:
Hello there, I was wondering if one infinity is larger than another in the extended real number system (just like in the transfinite ordinals and cardinals with respect to infinite sets) or are all infinities the same size in the extended real number system? Thanks sooo much for answering my question! I greatly appreciate it!

All the Best,

Justin

Answered by Robert Dawson.
Cubic meters of gravel in a 7 ton lorry 2009-06-21
From Jackie:
I have to transport gravel in a 7 ton lorry and I know how many cubic meters of gravel I need, but I need to know how many cubic meters of gravel a 7 ton lorry can hold.
Thanks, Jackie

Answered by Penny Nom.
What is 3 trillion divided by 3 billion? 2009-06-18
From Dianna:
What is 3 trillion divided by 3 billion?
Answered by Penny Nom.
Maximum Volume of a Cylinder Inscribed in a Sphere 2009-06-18
From Jim:
Hello I have a hard time finishing this question: A right circular cylinder has to be designed to sit inside a sphere of radius 6 meters so that each top and bottom of the cylinder touches the sphere along its complete circular edge. What are the dimensions of the cylinder of max volume and what is the volume?
Answered by Janice Cotcher.
Mowing 17 lawns 2009-06-18
From kevin:
I need help setting up a equation to solve the following question: If a gardener can mow 3 lawns in 7 hours, how long should it take him to mow 17 lawns. I can solve the problem, but I don't know how to set up the equation.
Answered by Penny Nom.
Two questions from math class 2009-06-18
From Con:
Hello,

My name is Con and my son is required to answer the following questions for his maths class.

He has attempted Q1 through trial and error and has found the answer to 72453. Is this correct?

He has attempted to draw the triangles described in Q2 in a number of ways and has found that BE can not equal ED and is dependent of angle BAC. Therefore, he claims that the triangle can not be drawn/practical. Is this correct or is there a slolution?

Q1.
Digits 2, 3, 4, 5 and 7 are each used once to compose a 5-digit number abcde such that 4 divides a 3-digit number abc, 5 divides a 3-digit number bcd and 3 divides a 3-digit number cde. Find the 5-digit number abcde.

Q2.
Let ABC be a triangle with AB=AC. D is a point on AC such that BC=BD. E is a point on AB such that BE = ED = AD. Find the size of the angle EAD. Con

Answered by Chris Fisher.
The integral of x^x 2009-06-18
From ANGIKAR:
what would be the integration of (X^Xdx)?

give answer in details.

Answered by Robert Dawson and Harley Weston.
Thermal Expansion of Steel 2009-06-17
From Ken:
Hi there, We are rollforming steel roofsheeting in 65M lengths and the = question of linear expansion has cropped up.I would like to know what = the expansion rate of this sheet would be over a temperature rise of say = 40degree F.in mm per Meter or whatever the norm is. The sheet is 0.53mm = thick and is 700mm in width,I hope this is sufficient info to enable you = to do your calculation.Many thanks, in anticipation. Ken
Answered by Janice Cotcher.
Divisibility 2009-06-17
From Sophia:
Hello
Please help my son with the solutions to the following:

a) Determine the remainder when 2^2009 + 1 is divided by 17;
b) Prove that 30^99 + 61^100 is divisible by 31;
c) It is known that numbers p and 8p^2+1 are primes. Find p.

Again, your assistance is greatly appreciated.
Thanks
Sophia

Answered by Robert Dawson.
Completing the square 2009-06-16
From rebecca:
Find the roots of the equation by Completing the square. Express your answer in exact form and in decimal form to 2 decimal places.
a) x(squared)+10x+23
b)3y(squared)+12y+3

Answered by Stephen La Rocque.
Triangular Numbers 2009-06-16
From Chinonyerem:
Question from Chinonyerem, a student:

Each of the numbers
1 = 1, 3 = 1+2, 6 = 1+2+3, 10 = 1+2+3+4 ,...
represents the number of dots that can be arranged evenly in an equilateral
triangle:
                            .
                  .        . .
        .         .       . . .             ...
.      . .      . . .    . . . .
This led the ancient Greeks to call a number TRIANGULAR if it is the
sum of consecutive integers, beginning with 1. Prove the following facts
concerning triangular numbers:
(a) A number is triangular if and only if it is of the form n(n+1)/2 for some n >= 1
(b) The integer n is a triangular number if and only if 8n+1 is a perfect square
(c) The sum of any two consecutive triangular numbers is a perfect square
(d) If n is a triangular number, then so are 9n+1, 25n+3, and 49n+6
Answered by Penny Nom.

You have the numbers 1 through 8, ... 2009-06-15
From Maryland:
You have the numbers 1 through 8, you can only use each number one time. Two of the numbers are multiplied together, two are subtracted, two more are subtracted and the last two are subtracted. All the answers are the same.
Can't get it, I have racked my brain trying to help.

Answered by Claude Tardif.
nC0 + nC1 + nC2 + ... + nCn = 2^n 2009-06-15
From Chinonyerem:
For n >= 1, derive the identity
nC0 + nC1 + nC2 + ... + nCn = 2^n
[Hint: Let a = b = 1 in the binomial theorem]

Answered by Penny Nom.
11 golfers playing 4 rounds 2009-06-14
From Brian:
I have 11 golfers playing 4 rounds of golf. It would be great if we could play at least once with everybody. I realize we will have 2 foursomes and 1 threesome each round...can you help?
Answered by Victoria West.
Number & Operation Puzzle 2009-06-12
From Francine:
How can I get the number 26 using 1,2,3,4,&5 and using addition, subtraction, multiplication and division. Each operator and digit must be used exactly once. My son presented me this question and I can't determine if it's even possible.
Answered by Janice Cotcher.
Volume of a Can 2009-06-12
From Zach:
A can of soda has a volume of 355 mL. The area of a circle is given as A=pieR2, where r= radius of the circle. So, the volume of the can is given by V=(pier2)h, where h is the height of the can. If a particular can has a height of 12.2 cm what is the radius of the can? I know the answer is 3.04 cm. I need help figuring out how to arrive at this answer- what were the mathmatical steps? Thanks, Zach
Answered by Stephen La Rocque.
The last non cero digit of a factorial 2009-06-12
From Wilson:
How can I find the last non cero digit from a factorial calculation of a big number. For example 10! = 3628800, the last non cero digit is 8. What is the last non cero digit of 10! ??
Answered by Robert Dawson.
The product of gradients between 2 perpendiculars lines 2009-06-11
From Alister:
how do i prove that the product of gradients between 2 perpendiculars lines equal to -1....
Answered by Penny Nom.
A rotating schedule 2009-06-10
From Doreen:
We have seven people - we want to create a rotating schedule for two people at a time to attend one day a week with each person working the same amount of days in the year.
Answered by Victoria West.
A number puzzle 2009-06-10
From Ann:
Hey,
    I have a math problem that I can't solve which just happens to be a "must-answer". The answer to the first problem is 25,452 and the second is 21,232. I know how to add but my teacher said that there is a pattern wherein you just have to look at the set of numbers and you already know the answer without really adding it first.  Here are the condition of the problem:
   1) the first, second and fourth numbers were provided by me. ( I have no idea how this works)
   2) the third and the fifth numbers were given by the teacher. ( She's the one who knows the technique)

# She says that there is a "pattern."
 Help me please as soon as you can. Thanks!!! Here it is:
         
         5454                                   1234
   +    3636                                   5678
         6363                                    4321
         2323                                    8765
         7676                                    1234
       _______                           _________
 
 
Please answer ASAP. Thank you!!!

Answered by Penny Nom.
Real World Applications of Mathematical Skills 2009-06-08
From Kathy:
I am teaching a student who is on the life skills program and is at the stage 2 level for maths but is in year 9 (stage 3). I am looking for maths lessons that will help her in life. Like maths in shopping, maths in fashion, maths in the home etc. Your help in finding lesson plans is urgently needed.
Answered by Janice Cotcher.
Vectors and the Law of Cosine 2009-06-08
From lauren:
once force of 20 pounds and one force of 15 pounds act on a body at the same point so that the resultant force is 19 pounds. Find, to the nearest degree, the angle between the two original forces
Answered by Janice Cotcher.
Cubic yards in an Irregular Shape 2009-06-08
From ron:
hi i have a berm 100'w x200'L at the base and 9'tall the top is 85'L x 20'w i would to find out how many cubic yards are in the pile and the formula
Answered by Janice Cotcher.
Initial Velocity of a Car Plunging Off a Cliff 2009-06-07
From Mimi:
A car fails to take a sharp turn on a flat road and plunges off a cliff. The car lands 65m horizontally and 43m vertically from where it left the edge of the cliff. How fast was the car travelling?
Answered by Stephen La Rocque.
Probability of Two Independent Events 2009-06-07
From Sharon:
A basketball player is given 2 freeshots, if the probability of making 1 of the 2 shots is 3/4 what is the probability of making 2 of the 2 shots or both shots?
Answered by Janice Cotcher.
Possible Values of a Base of a Logarithm 2009-06-05
From nazrul:
If log base x of 25=2 then what is the value of x? I am confused if the result be 5 or -5. Please explain. Thank you.
Answered by Robert J. Dawson.
Properties of Inequality 2009-06-05
From Nazrul:
We know that if a>b then (1/a)<(1/b) But why 5> -2 implies (1/5)>(1/-2) . Thank you in advance for your help.
Answered by Robert J. Dawson.
Linear feet into Square feet 2009-06-04
From Vickie:
Hoe do I change 10 linear feet into square feet?
Answered by Janice Cotcher.
Solving an Algebraeic Equation with Fractions 2009-06-04
From olivia:
solve for x (3x-1)/4 + (x+3)/6 = 3
Answered by Janice Cotcher.
At what speed does an automobile start to hydroplane? 2009-06-03
From Don:
How do you figure at what speed does an automobile start to hydroplane when driving on a wet surface.
Answered by Robert Dawson.
Omega 1 2009-06-03
From Justin:
Hello there, I was just wondering if the infinity in the extended real number system is the same as w1 (or Omega 1, the order structure of the real numbers) in the transfinite ordinals? Thanks so much for your help with this question, I really appreciate it!

Sincerely,

Justin

Answered by Robert Dawson and Harley Weston.
An aeroplane flying in a wind 2009-06-02
From Maria:
The aeroplane flies in a wind of speed 50 kph from the direction S80◦W. However, the direction in which it is pointed, and its speed in still air, are such that its resultant speed and direction are 800 kph, and direction N71.6◦E

I need to find the size of the angle between v and vw, in degrees to one decimal place, and find the speed at which the aeroplane would fly if the air were still, to the nearest kph.

It would help me greatly if I could see a diagram illustrating the velocity va that the aeroplane would have if the air were still, the velocity vw of the wind, and the resultant velocity v of the aeroplane (diagram should be in the form of a triangle illustrating how one velocity vector is the sum of the other two)

Thanks

Answered by Harley Weston.
Extraneous solutions 2009-06-02
From Ayana:
solve and check for extraneous solutions.

3x+6/ x²-4 = x+1/ x-2

x can not = {-2,2}

Answered by Penny Nom.
Stones around a pool 2009-06-01
From Jessica:
I am putting a yard pond in and I am trying to find out how many stones I would need for the perimeter.

The pond is 4ft (W) by ft (L) by 1.5ft (D)

Thank You,

Jessica

Answered by Penny Nom.
Conditional probability 2009-06-01
From Tanja:
A biased coin where P(heads) =3/5 is flipped 4 times. What is the probability of getting at least 3 heads given at least 1 heads is flipped? I can calculate this using P(A|B) =P(AandB)/P(B) but I've had students asking me how you would work this question out the "long way" without the formula. I'm am unsure about how you do this the long way.
Answered by Robert Dawson.
Two equations 2009-05-31
From tony:
y = 1.2x - 32.7,

y = -0.7x + 46.15

can you please solve this for me? i am not able to figure it out... thank you tony

Answered by Penny Nom.
The dimensions of a rectangle 2009-05-30
From Linda:
A rectangle has a perimeter of 38 feet & an area of 48 square feet. What are the dimensions of the rectangle?
Answered by Stephen La Rocque.
A border around a pool 2009-05-29
From PATTY:
POOL RADIUS IS 13 FEET ON THE OUTER EDGE, I WANT TO MAKE AN 18 INCH OUTSIDE BORDER AROUND THE POOL. WHAT IS THE VOLUME AROUND THE POOL. (THIS IS GOING FROM A 13 FT. RADIUS TO A 14.5 RADISU.
Answered by Penny Nom.
Solve 18(C-3)=162 for C 2009-05-29
From Alicia:
18(C-3)=162

adult student returning to school after 20 years and then never had it in school. Just doesn't click yet

Answered by Penny Nom.
4.6 quadrillion divided by 65,000,000 2009-05-28
From david:
4.6 quadrillion divided by 65,000,000
Answered by Penny Nom.
How many times does the graph of y=2xsquared -2x + 3 intersect the x-axis? 2009-05-27
From Henry:
how many times does the graph of y=2xsquared -2x + 3 intersect the x-axis?
Answered by Penny Nom.
Two ships and a lighthouse 2009-05-27
From Chelsey:
I have a question in regards to how do I know when to use tangent or cosine when determining angles. The question is: Looking north from the observation deck of a lighthouse 60 m above the sea, a lighthouse keeper sees two ships. The angles of depression to the ships is 5 degrees and 10 degrees. How far apart are the ships?

I don't understand which one to use when solving the equation.

Answered by Harley Weston.
Factor 6x^2+19x+3 2009-05-26
From michele:
how do you factor this trinomial. 6x^2+19x+3
Answered by Penny Nom.
Find out the length of a cup when its volume is halved 2009-05-25
From Thomas:
I'm having trouble with a question. What kind of formula would i use to find out the length of a cup when its volume is halved?
Answered by Stephen La Rocque.
Nickels, dimes and quarters 2009-05-25
From Amy:
Coming from South Africa I find working with nickels, dimes and quarters confusing. How many dimes are in a nickel and how many quarters are in a dime?
Answered by Penny Nom.
The Smythe family lot 2009-05-24
From Shrima:
We must determine the size of the lot. The Smythe family lot is 235ft across the front and 212ft down the side and 254ft down the other side. The sides of the lot are parallel to each other and perpendicular to the front. The size of the lot is two dimensional measure therefore you need to find the areaenclosed by the lot line.
Question:

What is the size of the Smythe lot including the unit?

Answered by Penny Nom.
I'm building a fence at my house. 2009-05-24
From dylan:
I'm building a fence at my house. If my hypotenuse is 50ft, what are the lengths of my other two legs?
Answered by Penny Nom.
The position of the fulcrum 2009-05-23
From jim:
I think I need a formula. I need to know how far an object will be lifted. A beam is 246 inches long on one side of the fulcrum, and 41 inches on the other side, if I push down 36 inches on the long side of the beam, how much will the short side move up?
Answered by Stephen La Rocque.
The dimensions of a larger cup 2009-05-23
From Elizabeth:
Hey If i have a cup that holds a volume of 477mL and the bottom radius is 2.8cm and the top radius is 4.9cm and the height is 10cm. If i increase the volume by one and a half times what is the new measurements if the cup is directly proportional to the first one. Thank you
Answered by Stephen La Rocque and Penny Nom.
Equivalent fractions 2009-05-22
From Jocelyn:
give two other ratios that are equal to each

example: 6/9

Answered by Penny Nom.
(x + 1) / 3 + (x + 2) / 7 = 2 2009-05-22
From MzDonna:
Question from MzDonna, a parent:

We can't seem to get an answer for this problem:
X + 1      X + 2
_____ + _____   = 2

   3              7

The book gives the answer as 10 ; But, we struck. Please help us.
                                               ___
                                                21       

Thanks in advance for your assistance.
Answered by Stephen La Rocque.

Application of Derivatives of Trig Functions 2009-05-21
From Alannah:
I have a word problem from my Calculus textbook that I can't figure out. Triangle ABC is inscribed in a semicircle with diameter BC=10cm. Find the value of angle B that produces the triangle of maximum area. I am supposed to set up an equation for the area of the triangle A=b x h/2 using Trig functions based on angle B to represent the base and height but I'm not sure how to do this when the side length given is not the hypotenuse.
Answered by Janice Cotcher.
A box contains two white socks and two blue socks 2009-05-20
From Phu:
a box contains two white socks and two blue socks. Two socks are drawn at random. What's the probability that they are match the same color
Answered by Penny Nom.
The width of a rectangle 2009-05-19
From Stacey:
If the length of rectangle is fixed at 29 cm, what widths will make the perimeter greater than 82 cm?
Answered by Penny Nom.
28 teams 14 lanes , 9 weeks no repeats 2009-05-19
From paul:
28 teams 14 lanes , 9 weeks no repeats
Answered by Karen Meagher.
(logx)squared + log (xcubed ) +2 =0 2009-05-19
From michael:
Solve for x: (logx)squared + log (xcubed ) +2 =0
Answered by Penny Nom.
Two trains 2009-05-18
From chris:
Train A passes a station at 3:15am going 40mph. Train B passes the same station at 3:27am going 50mph. At what time will Train B catch up with Train A?
Answered by Stephen La Rocque.
Two equations 2009-05-18
From Bob:
solve the system:

x/2 - 2y= -8
3x + 2/3y= -10

Answered by Stephen La Rocque.
The sum of the angles of a triangle 2009-05-18
From mary:
prove that the sum of the three angles inside any triangle always add up to 180 degrees?
Answered by Harley Weston.
4 tan(360/(2n)) 2009-05-18
From molly:
in the formula to find the area of any regular polygon how do you figure out 4tan part of the formula
Answered by Penny Nom.
43200 cubic inches in gallons 2009-05-17
From Denise:
how many gallons are in a tank that has a volume of 43,200 cubic inches?
Answered by Penny Nom.
A number puzzle 2009-05-17
From Gita:
Ok, we really need help for a homework problem due on Monday morning
I've been trying for 2 hours and can't figure it out---if I can't I'm not sure
how my son can....We've been given a puzzle with the following rules:
--The sum of each side must equal 15
--We can only use the numbers 1,2,3,4,5,6,7, 8
--A number can be used only once
(Hint: Think of all combinations of 3 numbers = 15)
--The puzzle looks like this:

_____ _______ ______

_____               ______

_____ _______ ______


Answered by Penny Nom.
Angles in a triangle 2009-05-16
From Robert:
The second angle of a triangle is twice the sizee of the first angle, the third angle is 48 degrees less than the sum of the other two angles. What are the measurements of all 3 angles? PS: please help
Answered by Stephen La Rocque.
Two Boats 2009-05-16
From Simon:
Two boats, one travelling towards the east and the other travelling towards the north, are 10 km apart afetr an hour. If the speed of one boat is 2 more than the speed of the other boat, what is the speed of the faster boat?
Answered by Stephen La Rocque.
|1/(x+1)| <1 2009-05-16
From Nazrul:
What is the solution of absolute value of (1/(x+1))<1? Please explain.
Answered by Stephen La Rocque.
An afternoon at the races 2009-05-15
From garths:
A professional gambler was betting at a horse racing afternoon.
At the end of the first race he had doubled his money.
He bet R30 on the second race and tripled the money he came with.
He bet 54 on the third race and quadrupled (4 times) the money he came with.
He bet R72 on the fourth race and lost it all. But he still had R48 left.
How much money did he take to the race track?

Answered by Harley Weston.
5 spades 2009-05-15
From Dee:
From a standard deck of 52 cards, how many cards would you have to draw, without looking at them , to be absolutely certain (a probability of 1) that you had 5 spades?
Answered by Robert Dawson.
Finding Height of a Triangle Given Area and Base 2009-05-14
From theodore:
area of a triangle is 40 in ^2 and base is 4 in what is the height
Answered by Robert Dawson.
Formula for Volume and Weight of Water in Pipe 2009-05-14
From Jonathan:
Do you have a formula for the amount of water that would be in a 12" schedule forty pipe in gal at any one time. The piping system will be under pressure but I need to know the overall weight of a single 20' stick of pipe so that I can determine my spacing for my concrete support inserts. Thanks.
Answered by Robert Dawson.
Ladder Leaning on a Wall 2009-05-14
From clinton:
a ladder rests against a wall 24m high.the foot of the ladder is 7m from the foot of the wall. How do you calculate the length of the ladder
Answered by Robert Dawson.
Cubic Feet for Around a Pool 2009-05-14
From Traci:
I have a pool that is 24 ft round. I want to put a trim of 2 inch deep and 1 foot wide white rock around the outside. My problem is that here you can only purchase the rock in bags of either 0.5 cu ft or 1 cu ft. How do I figure out how many bags i will need to do this right?
Answered by Robert Dawson.
Fraction Word Problem 2009-05-13
From Sonya:
Debbie wants to eat 1/6 of her 12 pieces of candy. How many pieces did she eat ?
Answered by Janice Cotcher.
A gravel pile in my driveway 2009-05-13
From Dean:
The only place I have to deliver gravel is in my driveway. The driveway is 16 ft wide by 18 ft long. How much gravel can I order as to not exceed the boundaries of my driveway when the truck dumps the gravel onto my driveway? I am assuming the angle of repose comes into play when calculating this elliptical pile? The maximum amount of gravel I need is 12 cubic yards.
Answered by Robert Dawson.
A reward and a fine 2009-05-12
From garth:
A maths teacher promised a learner that she would pay him R8 for each problem solved correctly, but she would fine him R5 every incorrect solution. At the end of 26 problems neither owed any money to the other. How many problems did the learner solve correctly?
Answered by Robert Dawson and Penny Nom.
Finding an Array to an Integer Linear Programming Problem 2009-05-12
From Debbie:
how to make an array for 3 box of juice for 25 students and 2 teacher. 1 pack of juices hols 3boxes the answer is 9 packages but how do you make array for this
Answered by Janice Cotcher.
The volume of an egg 2009-05-12
From Dylan:
What formula do you use to calculate the volume of an egg?
Answered by Stephen La Rocque and Penny Nom.
16 teams, 15 weeks, would like to pair up with no repeats. 2009-05-12
From Kimberly:
16 teams, 15 weeks, would like to pair up with no repeats. Thank you so very much.
Answered by Robert Dawson.
Solve -1<5-2x<10 for x 2009-05-12
From Helen:
solve the following inequality for x
-1<5-2x<10 Te less then sign by the one, should be less then or equal to.

Answered by Stephen La Rocque and Penny Nom.
differentiate y sin[x^2]=x sin[y^2] 2009-05-11
From mamiriri:
derivate y sin[x^2]=x sin[y^2]
Answered by Harley Weston.
The hypotenuse of a right angled triangle 2009-05-11
From Deb:
Find the length of the hypotenuse of a right angled triangle with one leg 7 cm longer than the other and the hypotenuse 2 cm longer than the longer leg. I've ended up with the hypotenuse = x+9, another side = x and the other side = x+7. what do i do next?
Answered by Penny Nom.
Six teams, two sites, and four days 2009-05-08
From John:
I have six teams, two sites, and four days. We have decided that we want to have tri-meets, where there would be three teams competing against each other at the same site each day. Here is the problem, I need to make sure that each team sees each other team at least once and each team goes to each site at least once.
Answered by Victoria West.
The dimensions of a toy chest 2009-05-07
From charlotta:
My son needs to find the 3 dimensions of a toy chest that has to have a volume of 24 cubic feet can you show how to get the answer
Answered by Penny Nom.
Perpendicular lines 2009-05-07
From shabnam:
the line presented by y= 3x-2 and a line perpendicular to it intersect at R(1,1). Determine the equation of the perpendicular line
Answered by Stephen La Rocque.
How many eighths are there in 1.25? 2009-05-06
From Tamara:
How many eighths are there in 1.25?
Answered by Penny Nom.
A car loan 2009-05-06
From Cindy:
A person takes out a car loan in the amount of $11,170.00 @ 4.5% interest. The person makes 17 payments of $500.00, then for the 18th payment (this person 'finds' some $) and pays off the whole thing. How much was their last (18th) payment? How much overall did they pay in principle? and interest?

The person who made the initial loan then takes $11,156.25 and places it into a High Interest Savings Account @ 2.25% annually on Nov 18, 2003. How much is now in the High Interest Savings Account if no futher funds have been added to the account and no withdrawls have been made?

Thanks for your help with this. I have been trying on and off for 3 years to figure this out.

C

Answered by Robert Dawson.
A quadratic equation with roots (4-i) and (4+i) 2009-05-05
From Kelly:
find a quadratic equation with roots (4-i) and (4+i)
Answered by Harley Weston.
Rolling a six on the sixth roll 2009-05-04
From Scott:
What are the odds of rolling a die 6 times in a row and rolling a 6 only on the 6th try, but not any other time? And how is this calculated?
Answered by Harley Weston.
Practical trigonometry 2009-05-04
From Lori:
I am an ex-math major turned home-schooling parent. I would desperately like to find a word-problem based trigonometry book for my 17-year old son. I don't want graphing or other gobbledy-gook that he'll never use. Does such a thing exist?
Answered by Robert Dawson.
A hexagon inside a triangle 2009-05-03
From Brian:
How do you figure out the area of an equilateral triangle with only the clue of a regular hexagon perfectly inside it (no boarders) and the only thing you know about the hexagon is it has an area of 12cm^2?
Answered by Stephen La Rocque.
A 6 team social softball league 2009-05-02
From Don:
Hi We have a 6 team social softball league with only two fields. Each team plays two games each Saturday. We have three time slots 10:00, 12:00 and 2:00 We play for 11 Saturdays and than have a 2 weekend playoff. All teams prefer double headers vs the split at 10:00and 2:00. How do we schedule as balanced a schedule as possible and minmize the splits ? Thanks Don
Answered by Laura Morrison and Victoria West.
Stairway construction 2009-05-02
From Olivia:
Hi, I'm Olivia and I'm in 8th grade studying algebra 1. I'm having some trouble with my math project. It has to do with rules of stairway construction. It says that there are two generally accepted rules to building stairs. Rule 1: x+y=43 and x+y=46 (cm) Rule 2: 2y+x=61 and 2y+x=64 (y is the riser and x is the tread length) I cant figure out how to graph them...it says they're supposed to intersect to make a square which is like the safety zone for building stairs (the stairs' rise and tread lengths are plotted as points) any points outside of that square are considered unsafe. Did i explain it ok? Thanks for your help. --olivia
Answered by Penny Nom.
Rectangular prisms 2009-05-01
From deborah:
Could you please tell me some examples of different objects in the real world of rectangular prisms?
Answered by Harley Weston.
The volume of 3-D figures 2009-04-29
From Serena:
What is the formula for finding the volume of 3-D figures
Answered by Robert Dawson.
Eight golfers, four days 2009-04-28
From Billy:
A golf Pairing question:

8 Golfers are Playing golf - one round per day
Four days of golf
(four twosomes) are paired each day to play
***No one can play with the same partner more than once
***Everyone must play with every other golfer in his foursome at least once

Is this possible?

Answered by Robert Dawson.
The integral of a to power x squared 2009-04-28
From JIM:
WHEN I ATTENDED U.OF T. (TORONTO ) MANY YEARS AGO WE WERE TOLD THE FOLLOWING INTEGRAL COULD NOT BE SOLVED : a to power x squared . is this still true ?

CURIOUS , JIM

Answered by Robert Dawson.
A two goat problem 2009-04-27
From Michael:
if you have a goat tied to a pole at one corner of a square paddock with one length of a side being 24m. what length must the rope be for the goat tied up to graze half the paddock? and if another goat is placed on the opposite corner and same length what is the amount of area they share grazing ?
Answered by Stephen La Rocque and Penny Nom.
Completing the square 2009-04-27
From Daniel:
I tried following an example of yours but I still couldn't figure it out. Here is my question,

2x^2 - 6x + 1 = 0.

Any help would be greatly appreciated.

Answered by Stephen La Rocque.
1 cubic yard of dirt = how many tons 2009-04-26
From jesse:
1 cubic yard of dirt = how many tons give the formula
Answered by Harley Weston.
The callaway system 2009-04-26
From Earl:
Do you have a formula to enter players scores using the callaway system that calculate each player score after their score is entered?
Answered by Victoria West.
An odd shaped lot with a curve to it 2009-04-25
From Chris:
Hi - Im trying out to figure out how to calculate the size of an odd shaped lot with a curve to it.

Chris

Answered by Harley Weston.
12% of 4 trillion 2009-04-25
From ROBERT:
WHAT IS 12% OF FOUR TRILLION
Answered by Penny Nom.
A word problem involving y = mx + b 2009-04-24
From Devon:
I need to put the following question in to y=mx+b form

I rent a gym for $150.00 for 30 students. Another time I rent the gym for @270.00 for 70 students. I need to also find a fixed rate.

Answered by Stephen La Rocque.
Negative nine minus 7x = negative 5 2009-04-23
From gigi:
what is negative nine minus 7x = negative 5
Answered by Robert Dawson.
Common multiples of 2 and 5 2009-04-23
From pat:
what are the common multiples of 2 and 5, through 30, because i been working on it for hours
Answered by Robert Dawson and Stephen La Rocque.
Two airplanes are 300 miles apart 2009-04-22
From afzalsultan:
Two airplanes are 300 miles apart and flying directly towards each other.One is flying at 200 miles per hour, and the other at 160 miles per hour.How long will it take for the two planes to meet
Answered by Robert Dawson.
An equivalence relation 2009-04-22
From syed:
D is the relation defined on Z as follows:

Z is an integer

For all m,n E Z, m D n if and only if 3 l (m^2 - n^2).

Find out whether the above relation satisfies to reflexive , Symmetric & transitive? Is it an equivalence?

Answered by Robert Dawson.
8 golfers in 3 rounds 2009-04-22
From patrick:
We have eight people, playing three rounds of golf. Would like to know the optimal pairings so that the most people can play with everyone. Thanks.
Answered by Robert Dawson.
Miles per hour and feet per second 2009-04-22
From mary:
A car traveling at 60 miles per hour, how many feet per second has it traveled?
Answered by Robert Dawson.
15 pigs in 4 pens 2009-04-22
From Melissa:
A farmer has 15 pigs. He wants to put them all into 4 pens, have an odd number of pigs in each pen, and have no pigs left over. Also, he isn't thinking about having any for dinner, as they are his pets.
How does he do it?

Answered by Robert Dawson.
A vertical radio tower is located on the top of a hill 2009-04-21
From Rafael:
A vertical radio tower is located on the top of a hill that has an angle of elevation of 10 degrees. A 70-foot guy wire is attached to the tower 45 feet above the hill.

a. Make a drawing to illustrate the situation
b. What angle does the guy wire make with the side of the hill?
c. How far from the base of the tower is the guy wire anchored to the hill?

What confuses me about this problem is the visual situation. Isn't the angle of the guy wire with the side of the hill the same as the angle of elevation? And if not, then how is one supposed to find the other angles without any more information?

Answered by Harley Weston.
A golf schedule for 18 teams 2009-04-21
From laura:
Could you help me form a golf schedule for 18 teams, playing a total of 8 weeks. Rotating the schedule each week so every team plays with a different team each week? I am using a shotgun format with 9 holes every week. Thank you for your input.
Answered by Victoria West.
The center of an ellipse 2009-04-21
From Nae:
what is the ellipse center of 5x^2+3y^2=15
Answered by Stephen La Rocque.
Two regular hexagons 2009-04-21
From niko:
Two regular hexagons are ____________ similar.
Answered by Stephen La Rocque.
Volume of a prism 2009-04-21
From David:
The base of a rectangular prism has an area of 15.3 square inches and a volume of 185.13 cubic inches. Write an equation that can be used to find the height "h" of the prism
Answered by Robert Dawson and Stephen La Rocque.
A perpendicular line in standard form 2009-04-21
From Kristy:
Can you help me with this equation? Find the equation, in standard form of the line perpendicular to 2x-3y=-5 and passing through (3,-2) With the equation in standard form with all integer coefficient.
Answered by Stephen La Rocque.
Two dice and the difference of squares 2009-04-21
From dave:
I dont understand how to do this: Two fair dice are rolled. What is the probability that the difference of the squares of the numbers is divisible by three? the answer is 5/9
Answered by Harley Weston.
The derivative using limits 2009-04-21
From Kirstin:
I am trying to take the limit of f(x) = [f(x+h)-f(x)] / h If you try taking the limit by substituting the limiting value h=0, you get 0/0, which of course is not the right answer. You rewrite f(x+h)-f(x) so it has a factor of h in it, which you cancel with the h in the denominator before you substitute h=0. But I am not sure how to do this. Thanks.
Answered by Robert Dawson.
A max-min problem 2009-04-20
From Charlene:
A fixed circle lies in the plane. A triangle is drawn inside the circle with all three vertices on the circle and two of the vertices at the ends of a diameter. Where should the third vertex lie to maximize the perimeter of the triangle?
Answered by Penny Nom.
The interior angles of a pentagon 2009-04-20
From Mary:
If four interior angles of a five-sided figure pentagon measure 100 degrees each, what will the fifth angle measure?
Answered by Harley Weston.
Two similar rectangles 2009-04-19
From Alyssa:
The ratio of the lengths of corresponding sides of two similar rectangles is 3:5. the small rectangle has an area of 36 square centimeters. What is the are of the large rectangle?
Answered by Penny Nom.
28 golfers 2009-04-18
From DON:
HI, I HAVE 28 GOLFERS AND I NEED A SCHEDULE FOR 24 DIFFERENT DAYS THERE WILL BE 7 GROUPS OF 4 PLAYERS EACH DAY. ALL PLAYERS WOULD LIKE TO PLAY WITH ONE ANOTHER AS EVEN AS POSSIBLE (SAY 2 TO 4 TIMES EACH OVER THE 24 DAYS)...THANKS DON
Answered by Victoria West.
The height of a right triangle 2009-04-17
From drakkar:
how do i find the height of a right triangle that in 176 feet in area
Answered by Penny Nom.
Nickles and dimes 2009-04-17
From Emily:
a vending machine contains nickels and dime coins which totals an amount of $14.50. There are 95 more nickels than the number of dime coins. How many of each coin does exist?
Answered by Penny Nom.
Make x the subject in this equation 2009-04-16
From victoria:
hi,
i need help trying to make x the subject in this equation:
7y + 2x = 15
i'm really stuck so any help will be good,

thanks

Answered by Penny Nom.
Exponential form 2009-04-16
From Pete:
Hi, How do you express ³√h^-4 in exponential form. I am having a lot of trouble with this one.
thanks
Pete

Answered by Stephen La Rocque.
Passing three cars 2009-04-16
From Larry:
If 3 cars are going 50 mph, and a fourth car pass the first 3 cars in 200 feet, how fast is the fourth car going?
Answered by Stephen La Rocque.
An octagonal shaped gravel pit 2009-04-16
From Melissa:
What is the square footage of a 7'3" square ocatogon with a height of 5 inches? I need to know how much pea gravel to buy for my sons gravel pit.
Answered by Stephen La Rocque.
What is the original number? 2009-04-16
From Yueh:
A five digit number is written on a blackboard. Ron erases one of the digits and adds a newly constructed number to the original one. The result is 41751. What is the original number?
Answered by Claude Tardif.
10 yards of rubber mulch 2009-04-15
From Donna:
If I want to buy 10 yards of rubber mulch (bulk), but it only comes in 20 pound bags, how many bags do I need to buy?
Answered by Harley Weston.
A number puzzle 2009-04-15
From Julie:
Hi, please help me!!! I have a subtraction puzzle... 5 digits minus 4 digits = 33333 Only using numbers 1-9 once each. I can get 33323 but then thats it!!! Please how do I work it out? There must be a simple way?? Thankyou
Answered by Claude Tardif.
The volume of a box 2009-04-15
From Karen:
Will 200 cubic centimeters of soil fit into the box? 8 CM by 6 CM and 4 CM deep
Answered by Penny Nom.
Infinity and Aleph-Null 2009-04-14
From Justin:
Yes, I am reading the Paul Halmos book on Set theory, thanks for telling me how to get it! I was just wondering from your last answer though if the positive real infinity of calculus then corresponds to Aleph-null? I am sorry if this is a similar question to the one I asked before but I was just wondering about this!

All the Best,

Justin

Answered by Robert Dawson.
Choices at a restaurant 2009-04-13
From Rob:
There is a restaurant you get:

Rice/Noodles (1) Main Ingredient (any) Sauce (1)
1 1 1
2 2 2
3 3 3
4 4 4
5 5 5
6 6 6
7 7  
  8  
  9  
  10  
  11  
  12  

So the question is how many different combinations are there. You can only have 1 rice/noodles in a selection and only 1 sauce in a selection but you can have between 1 and all twelve mains in a selection. there are 7 rice/noodles , 12 mains and 6 sauces. How many possibilies. I did it mentally in the restuarant, no pen, paper or calculater and i got 3276..i think thats wrong. please help

Rob
Answered by Robert Dawson, Stephen La Rocque and Claude Tardif.

A four-sided lot 2009-04-12
From Robert:
back of property is 137' across,with an angle of 122deg.on the left side and an angle of 140 deg. on the right side,the left side is 123' and the right side is 93' long,the left front is 130 deg. and the right front side is 150 deg. and the front of the property measures 78'
Answered by Harley Weston.
Expected value 2009-04-11
From jp:
you pay $.50 and pick a four-digit number. The state chooses a four-digit number at random and pays you $2500.00 if your number i chosen. What are the expected winnings from a $.50 Pick 4 wager?
Answered by Stephen La Rocque and Harley Weston.
2sinB=3tanA 2009-04-10
From Xanathax:
ABC is a right-angled triangle. 2sinB=3tanA. Calculate the measure of angle A.
Answered by Penny Nom.
Winding paper after a break 2009-04-10
From Olen:
Question from Olen:

I work in a paper mill and have been handed the task to search for a formula to determine how much paper needs to be added to a parent roll to make up the difference at the winder. (Ex. The spool diameter at the reel is 18.25" we measure roughly 33.5" to make two 58" rolls in the winder. If the is a paper break and the roll diameter in the winder is 30" how much do I add to a single parent roll (22" roughly) to make one 58 " and the 28" needed at the winder. I would appreciate any help to complete this task. I would like to be able to build a chart that operators can refer to based on what is needed. Thank you.

Answered by Harley Weston.
The axiom of choice and constructibe sets 2009-04-10
From sydney:
The axiom of choice asserts the existence of certain sets, but does not construct the set. What does "construct" mean here? For example, does it require showing the existence and uniqueness of some function yielding the set? In general, what does it mean to require the existence of a mathematical object be tied to a construction of it?
Answered by Claude Tardif.
Positive real infinity and Aleph-null 2009-04-09
From Justin:
Hello, I was just wondering why does the positive real infinity correspond to Aleph-null? Thanks a lot for answering my question!

Justin

Answered by Ami and Robert Dawson.
Parallel and perpendicular lines 2009-04-08
From Blake:
I need to know what the formula for working a problem. I need to know how to find if i line is parallel or perpendicular by only looking at the equation. i know you must know the slope but how do i find slope?
Answered by Stephen La Rocque.
A ladder against a wall 2009-04-08
From Jessica:
The angle of elevation of a 15 ft. ladder is 70 degrees, find out how far the base of the ladder if from the wall.
Answered by Penny Nom.
The integral of the square root of the sine function 2009-04-07
From Indrajit:
how to integrate this derivative???

∫√sinx

Answered by Harley Weston.
An arched or round top window 2009-04-07
From Dale:
I need a formula to figure the lineal footage of trim require to trim an arched or round top window. The variables that I have consist of the width of the window the height of the arc and the radius.
Answered by Harley Weston.
Multiplication of polynomials 2009-04-07
From Carla:
I am struggling to understand Multiplication of Polynomials. No matter how hard I try to understand Multiplication of Polynomials, I just can't get it!

The problem that I am trying to solve is this :

-3x^3y(-y + 2 -x^2 + x)

Answered by Robert Dawson.
The central angle of a chord 2009-04-06
From Dale:
How do I find the central angle if I only have the cord length and radius.
Answered by Harley Weston.
A golf trip for eight 2009-04-05
From Stuart:
8 of us are off on a golf trip later this month. We are playing 4 rounds in fourball format. Can we put together groups of four so that we play with each other at least twice? I have spent ages on this but can't get it to fit! Your help would be appreciated
Answered by Chris Fisher.
The length of an arc 2009-04-04
From Dale:
If I have the length of the chord and the height of the arc can I find the arc length with these dimensions?
Answered by Harley Weston.
A 5 digit push combination lock 2009-04-04
From Andrew:
I have a 5 digit push combo lock that i forgot the combo for. it has numbers 1-4 and each number can be pressed more then once ie. 11111, 36356 is there an algorithm out there that will list out the possible combinations for me? or maybe a way i could write them out myself.
Answered by Penny Nom.
How many miles per hour was it traveling? 2009-04-04
From Regina:
a car traveled 16 miles in 30 minutes. How many miles per hour was it traveling?
Answered by Penny Nom.
A toy pyramid 2009-04-03
From Darah:
Suppose you want to build a toy pyramid with a square base and a volume of 9000 cubic centimeters. What are the dimensions of a pyramid you might design? How many such pyramid designs are there?
Answered by Stephen La Rocque.
A large, hollow, ice cream cone 2009-04-03
From Darah:
A manufacturer is making a large, hollow, ice cream cone to serve as an ad for a local Baskin-Robbins. The ice cream cone is made up of a cone with height 8 feet, topped by a hemi-sphere with radius 6 feet. How much ice cream could the hollow object hold? If a gallon is 0.13368 cubic feet, how many gallons does it hold? If 3 gallons of Baskin-Robbins heavy cream chocolate blend weighs 24 pounds, how much would the ice cream cone weigh, excluding the weight of the construction material?
Answered by Stephen La Rocque.
Three consecutive odd integers 2009-04-02
From Brian:
Find the greatest of three consecutive odd integers if the product of the second and third is 8 more than 13 times the first.
Answered by Robert Dawson.
A fraction in its simplest form 2009-04-02
From Michael:
I'm in 4th grade and need to express decimals as a fraction in its simplest form. Is there a step by step method to figure out?

ex 0.64 = 64/100 = ?

Answered by Robert Dawson.
Sand falls from a conveyor belt 2009-04-01
From Tracy:
Sand falls from a conveyor belt at the rate of 10 cubic feet per minute onto a conical pile. The radius of the base is always equal to half the pile's height. How fast is the height growing when the pile is 5ft high?
Answered by Stephen La Rocque.
An octagonal deck 2009-04-01
From Terry:
area of octagon deck? perimeter 128ft, distance from center to any side 19ft
Answered by Stephen La Rocque.
Fractional part 2009-04-01
From Galina:
What fractional part of eight is eight hundredths
Answered by Stephen La Rocque.
A spherical Tootsie Roll Pop 2009-04-01
From Tracy:
A spherical Tootsie Roll Pop you are sucking on is giving up volume at a steady rate of .8 ml/min. How fast will the radius be decreasing when the Tootsie Roll Pop is 20 mm across?
Answered by Harley Weston.
GCD (a + b, a - b). 2009-04-01
From Tomas:
Let a and b integer and relatively prime. Prove that:
GCD (a + b , a - b) = 1 or 2

Answered by Stephen La Rocque.
A five game average of 148 2009-04-01
From Shelly:
(128 + 145 + 139 + 157 + x) divided by 5 = 148 matches the situation stated below:

A person bowled four games and scored 128, 145, 139, and 157. Find x, the score the person would need to have in the fifth game to average 148 for all five games.

How do I solve this and how am I supposed to add x like it says in the equation?

Answered by Penny Nom.
Why learn math? 2009-04-01
From Uno:
I got myself in a lot of trouble today. I got into an argument in school with my math teacher because while learning geometry I said that this was useless.
I don't understand why I need to learn algebra, geometry & trigonometry. I don't see how we use this in real life and it is almost like my teachers don't know either. They say I have to learn it because... I don't think that is a good enough answer. The only way I don't get suspended is if I can come up with real world applications of why we learn math.

I need help... I am already in deep trouble with my parents. Any resources on how learning a proof is used in real life?

Answered by Claude Tardif and Harley Weston.
Leading terms and leading coefficients 2009-03-31
From Susan:
Write the polynomial P(x)=x^3-3x^4+17x+11-4/3x^2 in descending order. Identify the leading term and the leading coefficient.
Answered by Penny Nom.
The length of a ladder 2009-03-30
From Susan:
The foot of an extension ladder is 7 ft. from a wall. The height that the ladder reaches on the wall and the length of the ladder are consecutive integers. How long is the ladder?
Answered by Penny Nom.
Four identical lots 2009-03-30
From Marina:
I really want to know the answer to this problem for my 6th grade son. I've already sent this question with the drawing but I couldn't send it correctly. I hope this one will pass through. Q: Divide evenly and identical the figure representing a lot, into four for the 4 siblings. I've sent a figure drawing as attachment. I will describe this in case it will not reach you: its a square divide into 4 triangle and 1 triangle is taken out living a letter M figure and this letter M figure is the one that will be divided.into 4 even and identical parts.

Marina

Answered by Claude Tardif.
More on the square root of 0.75 2009-03-30
From Blaine:
I read your response to How is the square root of 3/4 is greater than 3/4?

What I'm hoping for is a way for my students to use their own experience and number intuition to be able to make sense of the issue. As soon as my kids see "if y is this and x is this then..." their little eyes glaze over. Unfortunately, I can't come up with a way myself. Thank you for your help.
Answered by Penny Nom and stephen La Rocque.

Grain of rice on a chess board 2009-03-30
From andy:
Is there an easy way for my students to find out 8 to the power of 21 to solve the grains of rice doubling investigation on a chessboard? Any suggestions welcome. Thanks very much
Answered by Robert Dawson and Claude Tardif.
A circle tangent to the X-axis 2009-03-29
From Kiera:
Find an equation of the circle that satisfies the stated conditions.

Center C(4,-1), tangent to the x-axis

Answered by Penny Nom.
Payroll deductions 2009-03-29
From Jim:
I want to set up a spread sheet where i can enter my hourly pay or gross income, and calculate my tax deductions. I have my old pay stubs but I am stuck when I take the amount of the deduction and try to figure out the % used to calculate the amount

for example gross pay $1359, EI= $64, Fed Tax=$129, Prov tax = $35

these numbers I have provided are random but I think you can get the idea

Answered by Penny Nom.
A rolling wheel 2009-03-29
From Jules:
How far does a wheel of radius 2 feet roll along level ground in making 300 revolutions?
Answered by Penny Nom.
A parabolic arch 2009-03-28
From Jeni:
A doorway is in the shape of a parabolic arch.
Find the width of the doorway 1m above the floor.
Given: the height and the width of the doorway is 4m and 3m respectively.

Answered by Penny Nom.
The speed of a tire in revolutions per minute 2009-03-28
From Jules:
Suppose that a tire on a car has an outer diameter of 2,5 feet. How many revolutions per minute does that tire make when the car is travelling 60 miles per hour? (hint:first find the angular velocity in radians per minute.)
Answered by Harley Weston.
Subdividing a four-sided lot 2009-03-27
From kannan:
i have a 1.5 acre land having four sides
A-B=64.8 meters
B-C=94.2 meters
C-D=54.4 meters
D-A=127.2 meters
angle at D is 90
the side AB (frontage)is road facing i want to split the plot in to two halves having same almost area and almost equal frontage, kindly help me, if possible explain with drawing.

Regards
Kannan

Answered by Harley Weston.
My son has locked his ipod and can't remember the combo. 2009-03-27
From peggy:
my son has locked his ipod and can't remember the combo. He knows it has the numbers 2-5-8 in it but not the fourth number or the order. could you help us?
Answered by Robert Dawson.
A 5 ounce object traveling at 60 mph 2009-03-27
From James:
if a 5 ounce object traveling at 60 mph at 100 feet above the ground, how far will it travel forward before hitting the ground.
Answered by Robert Dawson.
An isosceles triangle 2009-03-26
From sela:
An isosceles triangle has two equal sides of length 10 cm. Theta is the angle between two equal sides.
a) Express area of a triangle as a function of theta
b) If theta is increasing at a rate of 10 degrees/minute, how fast is area changing at the instant theta=pi/3?
c) at what value of theta will the triangle have the maximum area?

Answered by Penny Nom.
A cord length and angle measurement 2009-03-26
From larry:
i have a radius of 185". and an arc length of 186.5". how do i find the arc degrees or cord length?
Answered by Harley Weston.
Depth to height ratio 2009-03-26
From Janet:
Is there a formula to determine how deep something (a cabinet) should be based on how tall it is?
Answered by Robert Dawson.
The base of a triangle 2009-03-25
From yinna:
how do I find the base of a triangle who's height is 2 times the base and the area is 9?
Answered by Penny Nom.
Octagonal panels for a horse pen 2009-03-25
From Tony:
I am building a pen for my horse. I am going to use 12' panels in the shape of an octagon. How many feet will he have from side to side using 12' panels.
Thanks,
Tony

Answered by Robert Dawson and Penny Nom.
More on mills 2009-03-25
From Kenneth:
If I want to express 13 mills as a decimal part of $1.00, the answer is $0.013. This amount is found by dividing 13 mills by 1000 mills/$1.00.

Can 13 mills be expressed as a decimal part of another amount instead of $1.00? For example, express 13 mills as a decimal part of $1.50 or express 13 mills as a decimal part of $0.10. What would these amounts be?

Answered by Robert Dawson.
The rate of change of the volume of a sphere 2009-03-25
From Kaylin:
why the rate of change of volume of a sphere is not constant even though dr/dt is constant?
Answered by Walter Whiteley.
x^2/5 - 4/5 = -3/5x 2009-03-25
From Lorrie:
x^2/5 - 4/5 = -3/5x
Answered by Penny Nom.
Z-score 2009-03-25
From Barb:
I am having trouble finding information on a z score and the conversion to the number of standard deviations a z value can be away from the mean. What exactly does that mean and what am I looking for? Help Please.
Answered by Harley Weston.
A max-min problem 2009-03-24
From Jay:
Determine the area of the largest rectangle that can be inscribed between the x-axis and the curve defined by y = 26 - x^2.
Answered by Harley Weston.
Choosing a 4 digit number at random 2009-03-24
From shabkhal:
If a four-digit number is chosen at random, what is the probability that the product of the digits is 12.
Answered by Harley Weston.
The diameter of a roll of plastic 2009-03-24
From truong:
hi. i have trouble to calculate the diameter of the plastic roll. the sheet is 765 m long and 0.8 mm to wrap around the core 400 mm in dia. please help me with the formula to calculate the dia of plastic roll, thanks in advance
Answered by Harley Weston.
An infinite number of solutions 2009-03-24
From Sean:
this is a linear equations problem;

first:
3535.5 + Fbd (.866) + Fbc (.5) - Fab (.5) = 0
and
-3535.5 - Fab (.866) - Fbc (.5) - Fbd (.5) = 0

Answered by Harley Weston.
The height of a mountain peak 2009-03-23
From james:
Can you please help us (my son Jim geometry) out with this problem, we cannot seem to find the correct method of solving, not sure how to solve the scalene triangle with what is given...if that's the correct method?? Can you please explain how this is solved!!

Thank you so much,

Jim and son !

Answered by Harley Weston.
Conversion factor 2009-03-23
From kab:
Would like to photocopy a Landscape Design. What is the Conversion Ratio or magnification required to convert 1" = 10 feet to 1/4" = 1 foot?
Answered by Harley Weston.
A 1.5:1 rectangle 2009-03-23
From melody:
I have an origami model that says to use a 1.5:1 rectangle. I don't understand how to calculate the size of paper I need to use. Surely there is a formula to calculate this ratio? Thanks for your help.
Answered by Penny Nom.
The base and height of an isosceles triangle 2009-03-23
From Chris:
How do you find the base and height of an isosceles triangle that has 2cm legs?
Answered by Harley Weston.
Find a polynomial function with the indicated zeros and satisfying the given conditions 2009-03-23
From Kristen:
Find a polynomial function with the indicated zeros and satisfying the given conditions. Simplofy your answer (no imaginary numbers or parentheses in the answer) Zeros: 1+2i, 1-2i, 5 ; f(-2)=1
Answered by Harley Weston.
The radius of a circle 2009-03-22
From Justin:
Find the radius of a circle with a circumference of 9.43cm.
Answered by Penny Nom.
What is the probability of accepting the box? 2009-03-22
From x:
a box consists of 10 light bulbs inspected by the following procedure: 2 bulbs are chosen at random without replacement. the box is only accepted if both bulbs work. suppose that the box contains 1 defective bulb. what is the probability of accepting the box?
Answered by Penny Nom.
A normal distribution problem 2009-03-21
From EDGAR:
to qualify for security officers training recruits are tested for stress tolerance. The scores are normally distributed with mean of 62 and a standard deviation of 8.
a.) If only the top 15% of recruits are selected, find the cutoff score
b.) If a candidate is rendomly selected, what is the probability that his or her socre is at least 55?

Answered by Harley Weston.
An equation for a line with a changing slope 2009-03-21
From Ben:
Is there anyway to write an equation for a line with a changing slope. e.g. A cell [hone company has a initial fee of 50$ paying for the first 100 minutes, the rate then increases to 10 cents per minute, then at 500 minutes the rate is 5 cents per minute.

If so please tell how.
Thanks

Answered by Penny Nom.
Percentiles 2009-03-21
From Shawn:
For a normal distribution of u=654.00 and o=138.00.
What is the percentile rank for X=426?

Answered by Harley Weston.
Problem solving 2009-03-21
From susie:
the product of a number and 9 increased by 15 is the same as one half of the difference of eight and four times the number. Find the number.
Answered by Stephen La Rocque.
A rectangular open box 2009-03-21
From tina:
An open box is to be made from a rectangular piece of tin by cutting two inches squares out of the corners and folding up the sides. The volume of the box will be 100 cubic inches. Find the dimensions of the rectangular piece of tin.
Answered by Stephen La Rocque.
A dead fly is stuck to a belt that passes over two pulleys 2009-03-21
From Jules:
A dead fly is stuck to a belt that passes over two pulleys 6 inches and 8 inches in radius. Assuming no slippage,how fast is the fly moving when the large pulley (8 inches) makes 21 revolutions per second ? How many revolutions per second does the small pulley make ? How long will it take the dead fly to travel 1 mile ?
Answered by Penny Nom.
The length of a chord 2009-03-20
From shirley:
show that the chord is equal to two times the radius time the sin of theta/2. The chord is connected to the two radii and theta is a central angle between the two radii
Answered by Penny Nom.
The height of a triangle 2009-03-20
From jon:
hi,
my daughter has to find the height of a triangle her side are 174ft and 184ft and the area is11,500 ft squared.
thanks for the help
jon

Answered by Penny Nom.
Milliliters of oxygen in your lungs 2009-03-20
From Leandrea:
if you have 3 liters of air in your lungs and 2/10 of that is oxygen, how many milliliters of oxygen are in your lungs?
Answered by Penny Nom.
The volume of a cylinder 2009-03-19
From zahir:
i want to ask one question, need i get the cross section of cylinder to calculate volume of cylinder
Answered by Harley Weston.
Find the retail price 2009-03-18
From Allie:
If you have food that costs $0.75 per portion and your food cost is 16.5% how would you find the retail price?
Answered by Penny Nom.
The ratio of the volumes of two cylinders 2009-03-18
From jdkaj:
cylinder a has radius 1m and height 4m. cylinder b has radius 2m and height 4m. find the ratio of the volue of cylinder a to the volume of b
Answered by Penny Nom.
The perimeter of a rectangle 2009-03-18
From marylyne:
the perimeter of the rectangular is 70 meters. if the width is 40% of the length, find the dimensions?
Answered by Penny Nom.
The units digit of 2^95 2009-03-18
From Sharvon:
what is the units digit of 2^95
Answered by Robert Dawson.
The volume of water in a cone 2009-03-17
From Freddie:
A ball of diameter 20cm rests in a conical container whose angle with the slant height and the vertical axis is 25degrees. if water is poured into the container just enough to touch the bottom of the ball, find the quantity of water in the container.
Answered by Penny Nom.
The sides of a parallelogram 2009-03-17
From Sami:
If ABCD is a parallelogram, prove that line AB is congruent to line CD. Clearly state your reasons and conjectures.
Answered by Penny Nom.
The midpoints of two sides of a triangle 2009-03-17
From Manis:
Prove that the line joining the midpoint of two sides of a triangle is parallel to the third and half of it.
Answered by Robert Dawson.
A plane cuts a line segment 2009-03-17
From Manis:
Find the ratio in which the line joining (2,4,16) and (3,5,-4) is divided by the plane 2x-3y+z+6=0.
Answered by Robert Dawson.
How many bags do I need? 2009-03-16
From Dawn:
I need to purchase an item that is .4 cubic feet in size and the amount I need is 5.625 cubic feet. How many bags do I need?
Answered by Penny Nom.
A slopped table top 2009-03-16
From anthony:
A perfect circle with radius 100m is supported by four legs, 50m long. The circle is flat when all four legs are 50m long, and tilted when the legs are different in length. The four legs are spread evenly 90 degrees apart at 0, 90, 180, and 270. Find the high and low points in degree and height when the legs are 55m at 0 degree, 70m at 90 degree, 45m at 180 degree, and 30m at 270 degree.

Not sure if they can be calculated using only 2 or 3 legs.

I have tried by picking the two adjacent highest legs (55m and 70m) so I know the high point falls into the first quadrant. Then I tried using ratio to determine where the high point is, which I think high point should be closer to the 70m leg, I am not sure. If I can find the high point, the low point should be opposite to it.

Answered by Chris Fisher and Harley Weston.
The odds against drawing a red queen 2009-03-15
From tabyia:
7. The common deck of 52 cards has 26 black cards (13 spades and 13 clubs) and 26 red cards (13 hearts and 13 diamonds). Each suit consists of ace, king, queen, jack, ten, nine, eight, seven, six, five, four, three and deuce. 13in all.

A single card is chosen at random. Find the odds against its being a red queen?

How many cards will I have to draw to be absolutely certain that I have drawn three black cards

Answered by Harley Weston.
How do I divide with a decimal is the divisor? 2009-03-15
From Shelly:
How do I divide with a decimal is the DIVISOR ( NOT divsor AND dividend.) ? Also, How do I know when to stop dividing?

P.s. please use long division

Answered by Penny Nom.
The volume of a pipe 2009-03-15
From kalen:
Okay I am doing a problem on a practice sheet to study with and I need help. The question states: How many gallons will 500 feet of six-inch water pipe hold? Do you have any suggestions/breakdowns on how to work out this problem so that in the future I will know how to do it?
Answered by Penny Nom.
An impossible trig problem 2009-03-14
From Alisha:
If csc theta = 3 and sec theta = square root of 3, what are the values of tan theta and cos^2 theta?
Answered by Harley Weston.
How many ball bearings can you make from a cone? 2009-03-14
From Vatsal:
there is a sum in my maths book which states the following: how many ball bearings of diameter 2.5 cm. can you get by melting a solid metalic cone of radi. 20cm and a Hight of 5 meters? how do i do this please tell me even the minute things like cancellatoins and where to use = , ; or any such thing because Im a bit weak in maths
Answered by Harley Weston.
Related rates 2009-03-14
From Jeevitha:
The side of an equilateral triangle decreases at the rate of 2 cm/s. At what rate is the area decreasing when the area is 100cm^2?
Answered by Stephen La Rocque.
License plates and poker hands 2009-03-14
From CC:
You have a license plate it can have 6 numbers/letters you can use the numerals 0-9 and the letters A-Z how many combos are possible and how did you figure it out?

Question 2,

Your dealt an Omaha hand You have KKKQQ, how many different hands can consist of the same cards.

Answered by Harley Weston.
Express 13 mills as a decimal part of $1.00. 2009-03-13
From Kenneth:
Hello:

I read the following in an old business mathematics textbook: "Express 13 mills as a decimal part of $1.00."

I can determine the answer as follows: There are 1000 mills in or per $1.00. 13 mills divided by 1000 mills/$1.00 equals $0.013, the answer.

My question is as follows: What does "...as a decimal part of $1.00" indicate or represent from "Express 13 mills as a decimal part of $1.00."?

Is (a decimal part of $1.00) any decimal less than $1.00 with the decimal having a dollar sign before it.

I thank you for your reply.

Answered by Harley Weston.
The scale for a model train 2009-03-13
From Shelly:
Lindy is planning to build a model of a train using a scale where 2 inches represents 25ft. If the train is 60 feet long, what is the length in inches that Lindy should build the model of the train?
Answered by Stephen La Rocque and Penny Nom.
Solving an equation for b 2009-03-13
From ernest:
If 2b=96+b+96+1/2b solve for b?
Answered by Penny Nom.
A linear system 2009-03-13
From Rasanga:
Use the row echelon form of the augmented matrix to solve the following linear system.
X1+2X2+X4=6
X3+6X4=7
X5=1

Answered by Harley Weston.
A normal distribution problem 2009-03-13
From jude:
Regarding the time it takes for an oil change has a normal distribution with a mean of 17.8 minutes and std. deviation of 5.2 minutes. A free oil change will be given to any customer that must wait beyond the guaranteed time. If they don't want to give more than 1% of its customers free oil changes how long should the guarantee be (to the nearest minute). Thank you.
Answered by Robert Dawson.
Probability 2009-03-12
From jude:
) Assuming that the heights of boys in high school basketball are normally distributed with a mean of 70 inches and a std dev. of 2.5 inches, how many boys in a group of 100 are expected to be 75 inches tall.

2) Past records from a bank show that the probability of being approved in the written application for hire is 0.63. Then the probability of being approved by the interview committee is 0.85, given that the candidate has been approved on the written application. What is the probability that a person will be approved on both the written application and the interview?

Answered by Robert Dawson.
Stacking eggs 2009-03-12
From Prashanth:
if you stack two eggs eah in row you must get 1 balance
if you stack three eggs eah in row you must get 2 balance
if you stack four eggs eah in row you must get 3 balance
if you stack five eggs eah in row you must get 4 balance
it goes on like 6 , 7 , 8 and finally
if you stack nine eggs eah in row you must get 8 balance what would be the answer and how to reach it

Answered by Robert Dawson.
A regular decagon is inscribed in a circle 2009-03-12
From Renata:
A regular decagon is inscribed in a circle of diameter 36 feet. Approximate the perimeter and area of the decagon.
Answered by Robert Dawson.
Three consecutive odd integers 2009-03-11
From Susan:
Find three consecutive odd integers such that the sum of the first, twice the second, and three times the third is 70
Answered by Penny Nom.
Water drains from a conical tank 2009-03-11
From Tyler:
Water drains from a conical tank at the rate of 5ft/min^3. If the initial radius of the tank is 4' and the initial height is 10'.
a) What is the relation between the variables h and r? (height and radius)
b) How fast is the water level dropping when h=6'?
Thanks for the help, i'm stumped.

Answered by Penny Nom.
The equation of a line 2009-03-11
From Teresa:
Write an equation in slope-intercept form that has a horizontal intercept of -7/3 and a vertical intercept of -7/9
Answered by Harley Weston.
A cubic polynomial 2009-03-11
From Pedro:
I am having trouble figuring out the soultions for the polynomial equation

6x^3-17x^2+4x+12=0

Answered by Robert Dawson.
The angles of a triangle 2009-03-11
From Marissa:
The angles in a triangle measure 7x-1, 18x+2, and 5x+10. Determine whether the triangle is acute, obtuse, or right. State your reasons clearly.
Answered by Robert Dawson.
Marcia's summer earnings 2009-03-11
From Yasmine:
Marcia saved money by working over the summer. Before school started, she spent 25%of the money of jewelry and then 2/3 of the remaining money on a digital camera. She still had $213 left. How much money did she have before she started shopping?
Answered by Robert Dawson and Penny Nom.
The weight of water in a swimming pool 2009-03-10
From ely:
How much does the water in a swimming pool 20 ft long, 10 ft wide, and 6 ft deep weigh?
Answered by Penny Nom.
66 2/3% 2009-03-10
From Traci:
Please convert 66 2/3% into a decimal and a fraction
Answered by Penny Nom.
A polyhedron 2009-03-10
From Mollie:
Hi, my name is Mollie and I'm in 5th grade. Here's the math problem I have for homework tonight:

"Larissa made a model of a polyhedron using 8 pieces of clay for the vertices and 18 straws for the edges. What type of polyhedron did Larissa make?" Thanks.

Answered by Penny Nom.
A right angle 2009-03-09
From bill:
I am a cotractor & i am laying out some piers in an L shape i need to know how to tell if they are square one side of my L is 18 feet the other is 11 feet 6 inches what is the formula to find out what the distance from point to point should be for square . I appreciate any help u can give me thank you
Answered by Penny Nom.
The slope of a line 2009-03-09
From Shannon:
Find x such that a straight line passing through (2,x) and (x+1, 3) has slope 2x+1/x-1 Please help! Missed this class, so I have no notes and the book doesn't have an example like this. Thank you!
Answered by Penny Nom.
Related rates 2009-03-09
From Megan:
A plane flying with a constant speed of 330 km/h passes over a ground radar station at an altitude of 3 km and climbs at an angle of 30°. At what rate is the distance from the plane to the radar station increasing a minute later?
Answered by Harley Weston.
2 types of season passes 2009-03-09
From Jaquanta:
A local water park has 2 types of season passes. Plan A costs a one - time fee of $142 for admission plus $8 for parking every trip. Plan B costs a one - time fee of $56 for parking plus $25 for admission every trip. How many visits must a person make for plan A and plan B to be equal in value?
Answered by Penny Nom.
The height of a right triangle 2009-03-08
From Anah:
Could you please tell me the formula for me to calculate the height of a triangle. I have the angles and side lengths. I am trying to calculate the height of an right triangle, the dimensions are as follows:

The Base/Hypotenuse is: 8x

Side A is 2x. Side B is 2x.

Answered by Harley Weston.
A property that decreases in value every year by 10 percent 2009-03-07
From PEter:
there is a competition problem where a cottage is worth 25000 years, but it decreases in value every year by 10 percent. Without a calculator, is there a way to find after how many years, the cottage will be worth 2500? (besides multiplying it out and besides logs)
Answered by Stephen La Rocque.
Cubic feet 2009-03-07
From A student:
What is cubic feet and how do I use it in this question? (picture below)
Answered by Stephen La Rocque.
A walkway through a park 2009-03-07
From felicia:
a diagonal walkway through a park is 18 meters long. If the park is a square, how long is one of its sides to the nearest tenth of a meter?
Answered by Penny Nom.
The sides of a triangle 2009-03-07
From felicia:
ona map, tannersville, chadwick, and barkersvile for a triangle. Chadwick is 70 miles from Tannersville and Barkersville is 90 miles from Tannersville. Which is a possible distance between Chadwick and Barkersville?
a. 5miles
b. 10 miles
c. 150miles
d. 200 miles

Answered by Harley Weston.
A 14 team league that is divided into 2 divisions 2009-03-07
From Ronnie:
I have a 14 team league that is divided into 2 divisions. We have a total of 18 weeks to play. I would like for each team to play the team in their division twice. The remaining 6 weeks would be played with the other division. Any help would be appreciated.
Answered by Chris Fisher and Victoria West.
An inequality with logs 2009-03-06
From Indrajit:
(x/10)^(logx - 2)<100..........how to solve this one??
Answered by Stephen La Rocque.
Infinite sets and infinite limits 2009-03-06
From Justin:
Hello, I know I have asked a similar question before but I was just wondering if set theory applies to the lim x->0, y=1/x=infinity and if so, what type of infinity would it be? Thanks a lot for your help with this question!

Regards,

Justin

Answered by Robert Dawson and Harley Weston.
Dependent & Independent Events 2009-03-06
From maci:
what is the probablility of drawing at random 2 blue marbles in a row if there is 3 red,4 green,2 yellow,and 5 blue marbles?
Answered by Janice Cotcher.
The amount of material remaining on a reel 2009-03-04
From James:
Question from james:

I am after a standard calculation so that after each usage of the Reel I can get the Remaining Quantity in kilos left on the roll

below is an example of roll

THICKNESS: 30 MICRON
REEL WIDTH: 7.5 CM
REEL DIAMETER: 76 CM
REEL WEIGHT: 7.13 KILOS

The core centre has a 9cm diameter, the weight of the roll is excluding the core , and after each use the diameter of the role is measured.

I am a factory supervisor working on project and need this calculation info for template.. Thanks James

Answered by Harley Weston.
Direct, inverse and indirect proportionality 2009-03-04
From Brian:
My 14 year old is confused about indirectly proportional and inversely proportional. On searching on the internet we were directed to: http://mathcentral.uregina.ca/QQ/database/QQ.09.00/ellis1.html Here leanne starts explaining indirectly proportional and ends up describing inversely proportional. Has Leanne got it wrong ?
What is indirectly proportional.

Thanks
Brian

Answered by Chris Fisher.
A shape with 10 sides 2009-03-03
From Sandra:
One of my son's math questions is as follows: Suppose I have a shape with 10 sides. I choose a vertex then draw lines to the other vertices that dont share sides with the first vertex. How many vertices will that be and why?
Answered by Penny Nom.
The angles in a parallelogram 2009-03-03
From dolly:
having a time-with equations inequaltites and problem solving= the sum of the measure of the angles of a parallelogram is 360.anglesa a and d , have the same measure as angle c and d,c is twice the measure of angle a, what is the angle for each.
Answered by Penny Nom.
A taxi charges .20 every 1/9th of a mile 2009-03-03
From ronald:
i need to know if a taxi charges .20 every 1/9th of a mile and i have to go 6.55miles how much would that be?
Answered by Robert Dawson.
A common tangent to two curves 2009-03-02
From Jay:
For what values of a and b will the parabola y = x^2 + ax + b be tangent to the curve y = x^3 at (1,1)?
Answered by Penny Nom.
The area of a 5-sided lot 2009-03-02
From Linda:
I have a problem I'm hoping you can provide some help with. When I check the property information for our home on our District of North Vancouver website, it shows that our property is supposed to be 1146.99 Square Metres. Yet another property which has identical measurements is listed as having 1173.7 Square Metres. I would like to figure out who is correct.
Answered by Harley Weston.
If 5 to the power of y = 625, what is y? 2009-03-02
From Charlene:
If 5 to the power of y = 625, what is y?
Answered by Penny Nom.
The sum is 43 and the difference is 9 2009-03-02
From Teresa:
Hi, My 10yo son came home with this two problems and we can't figure out how to solve them.

1. Find two numbers whose sum is 43 and whose difference is 9 and
2. Find two numbers whose sum is 25 and whose difference is 11..
Many thanks
Teresa

Answered by Robert Dawson, Stephen La Rocque and Harley Weston.
Is ACB a right angle? 2009-03-02
From racheal:
how to i prove angle ACB is not a rt. angle when given AT=BT=5;CT=4 in a indirect proof?
Answered by Robert Dawson.
Symmetries of a polyhedron 2009-03-02
From Vincent:
I want to ask is there a formula to find out the number of axes of rotation & plane of reflection of a 3D figure, like pentagon, pyramid?
Answered by Chris Fisher.
Comparing 2^3^4^5 ; 2^5^4^3 ; 5^4^3^2 etc. 2009-03-01
From JIM:
WHICH WOULD BE LARGEST USING THE NUMBERS 2,3,4,5 : 2^3^4^5 ; 2^5^4^3 ; 5^4^3^2 : ETC. OF ALL THE POSSIBLE COMBINATIONS ? IS THERE A GENERAL FORMULA ? I AM UNABLE TO FIND A COMPUTER THAT CAN HANDLE SUCH LARGE CALCULATIONS . I ALSO DO NOT THINK IT IS AS STRAIGHT- FORWARD AS FIRST APPEARS . THANK-YOU , JIM
Answered by Stephen La Rocque, Robert Dawson and Harley Weston.
Implicit differentiation 2009-03-01
From Emily:
determine the derivative y' at the point (1,0)
y= ln(x^2+y^2)

y'(1)= ??

Answered by Stephen La Rocque.
An octagonal landscaping frame 2009-03-01
From Richard:
Hi I am trying to put landscape timbers down in octagon shape that measures 6 feet across and outside measures 360 degrees.. The timbers are 4 inches by 4 inches. I need to know at what angle to cut boards and at what length i need to complete octagon.
Thanking you in advance for your kind assistance.
Richard :)

Answered by Harley Weston.
An annuity 2009-02-27
From Jules:
R 20 000 is deposited in the bank,at the end of each year R 5 000 is withdrawn from it, the interest is 13 percent compounded monthly. Calculate how many years that person will be able to use his/her money ?
Answered by Stephen La Rocque.
Simplify the square root of 125 x^2 2009-02-27
From Ashley:
√125x²

How do you simplify this equation assuming that variables can represent any real number?

Answered by Robert Dawson.
Where do X, Y and Z go? 2009-02-27
From Aj:
A E F H I K L M N T V W
B C D G J O P Q R S U

where do X, Y and Z go?

Answered by Robert Dawson.
36cm to the second power divide 10cm 2009-02-26
From val:
36cm to the second power divide 10cn
Answered by Penny Nom.
A triangular pyramid 2009-02-26
From Craig:
Can you help with this question:
Write the names of the faces and the number of each kind of face on a triangular pyramid?

Answered by Penny Nom.
Multiplying in different bases 2009-02-25
From Susan:
11 base 2 X 22 base 3 + 33 base 4 = _________ base 5
Answered by Robert Dawson.
A rectangular prism 2009-02-25
From Christina:
I am trying to find the area of the base of a rectangular prism where one cube=_1 cu in. The Answer is given in square inches. I also need the volume of the prism in cu in. For example length=3, w=2, h=5
Answered by Penny Nom.
Collecting an army 2009-02-25
From bevaz:
Question from bevaz, a student: A ruler orders his chamberlain to collect an army from 30 houses. The servant goes to the first house alone and collects one man. At each house after that he takes the same number of men as he has already collected, so at the second house he goes with one other and so on. How many men did he collect in all?
Answered by Penny Nom.
Some 9 letter permutations 2009-02-24
From Cecilia:
If the letters G, H, S are used exactly 3 times how many 9 letter combinations can be generated?
Answered by Penny Nom.
Triangles within triangles 2009-02-24
From Mari:
a large shaded triangle changes each day with a white triangle appearing in the center of each shaded one. If this pattern continues, how many white triangles will be there on the 6th day? On the 6th day, what fraction of the large outer triangle will be white?THANKS!
Answered by Robert Dawson.
Trig functions without geometric data 2009-02-24
From bob:
I do not understand how it is possible to find the sine, cosine, or tangent of an angle if there is no hypotenuse, opposite or adjacent side?!
Answered by Robert Dawson.
mutiplying decimals 2009-02-24
From telly:
how to do this sum0.24*0.398
Answered by Robert Dawson and Penny Nom.
Two pages are attendants of a king 2009-02-24
From Lauren:
Two pages are attendants of a king. For their service, one gets 13/6 dinaras a day and the other gets 3/2 dinaras. The first page owes the second 10 dinaras. Calculate when they will have the same amount.
Answered by Penny Nom.
More on the gst 2009-02-23
From melissa:
I need to know the amount of gst that was in the amount of 630.67? How do you figure this out? I don't need the amount of gst after, but the gst included?
Answered by Penny Nom.
An octagonal poker table 2009-02-23
From Corey:
I'd like to build an octagonal poker table using 4'x8' sheets of plywood. I would like each side edge to be approx. 2' wide. I am sure that due to the angles I can do this with one sheet of plywood, but i don't know how to measure for this. Can you help?
Answered by Robert Dawson and Harley Weston.
Amount of Water 2009-02-23
From Jeanette:
I received a water bill for 660,000 gal. of water. I don't think it is possible to pass this much water through a 1 1/2 inch pipe in 30 days. Is it possible to send that much water? I am assuming that there is a water pressure of 50 gpm, but I am not sure of that. It is a rural water system.
Answered by Janice Cotcher & Robert Dawson.
Buying 13,983,816 tickets for the 6/49 lotto 2009-02-23
From luis:
if 6/49 lotto combinations is 13,983,816 combinations at 10.pesos per combinationthe price amount is 139,838.160.00 is it wise for me to get all the combinations since the the pot money is more than 347 million pesos. Am I right?
Answered by Robert Dawson and Penny Nom.
Pressure at Given Depth 2009-02-22
From Tamara:
A sea is 3km deep. The average density of sea water there is 1020 kg/m^3. What is the water pressure at the bottom?
Answered by Janice Cotcher.
Time in a Swing 2009-02-22
From Barb:
I looked at the questions concerning a pendulum as I know I need to use this formula but I am stiil not able to figure this problem out. Can you help? If a child is on a swing with a 10 foot chain, then how long des it take to complete one cycle of the swing? I know I am suppose to use the formula 8T^2 = pi^2 L but I do not understand how to do this. Thanks
Answered by Janice Cotcher.
Percentage difference in pipe volumes 2009-02-22
From mike:
what is the % difference between a 3 inch diameter pipe compared to a 4 inch diameter pipe? and how do i find the answer?
Answered by Penny Nom.
16 golfers playing one round of golf on each of four days 2009-02-22
From Jim:
I have seen solutions for other combinations of setting up a golf trip but I haven't seen one for my group. We have 16 golfers playing one round of golf on each of four days. Is there a way for all of them to play in foursomes so that everyone gets to play with everyone else? Thanks for your consideration.
Answered by Chris Fisher.
Express the HCF of 1232 and 573 as 1232x + 573y = 1 2009-02-22
From Anonymous:
Express the HCF of 1232and 573 as 1232x + 573y = 1.
Answered by Victoria West.
A ¾ inch copper pipe 2009-02-22
From Ray:
Hi Do you have a formula I can use for this problem? And how to use it. Thank You How many feet of ¾ inch copper pipe will it take to hold 3 gallons of water?
Answered by Harley Weston.
Find the resultant of this displacement pair 2009-02-22
From katydidit:
Find the resultant of this displacement pair:
500 miles at 75 degrees east of north and 1500 miles at 20 degrees west of south.
How do I graph this and how do I solve this problem?

Answered by Penny Nom.
Surface area and scale factors 2009-02-21
From tina:
the surface area of a sphere is about 86 ft (2) find the surface area of a larger sphere that has a scale factor of 4
Answered by Penny Nom.
6 golfers playing 8 rounds 2009-02-21
From evan:
I have a group of 6 golfers playing 8 rounds. we would like to rotate the 3 somes so each person golfs with different people as many times as possible
Answered by Chris Fisher.
Three people working in pairs 2009-02-21
From bevaz:
A and B can together do a piece of work in 6 days, B and C together in 20 days and C and A in 7.5 days. how long will they require separately for the work?
Answered by Penny Nom.
The graph of y = -6 2009-02-21
From Barb:
I was given the problem that says "graph the line y = -6" how do I find all the set points that have a y coordinate of -6? I am so lost lost and have no clue how to start. Can you help me please? thanks.
Answered by Penny Nom.
Find an expression for the nth term 2009-02-20
From Dawn:
Find an expression for the nth term:

x= 1 2 3 4...n
y=11 8 5 2...?

I think it's a linear relation but don't know the equation that goes with it.

Answered by Stephen La Rocque.
Washing the dishes 2009-02-20
From rm:
a woman washing a large number of dishes after a meal was asked how many people attended the meal. she did not know but 'every two used a dish of rice between them, every three a dish for soup and every four a dish for meat, and there were 65 dishes in all.' how many people were there?
Answered by Robert Dawson.
Linear systems 2009-02-20
From Rose:
I have been having trouble trying to figure out these three math problems , I need help breaking them down so I could understand them better please help.
1. x = 7 - x
2 x - y + 8

2. 8 x + 5 y = 1 8 4
x - y = 3

3. y + 2 x = 3
y + 2 x = 4

I can't figure out how to break them down in the right order.

Answered by Penny Nom.
An octagonal pyramid 2009-02-20
From JUDY:
THE BASE OF A PYRAMID IS A REGULAR OCTAGON. HOW MANY CONGRUENT TRIANGULAR FACES DOES THE PYRAMID HAVE?
Answered by Penny Nom.
The focal point of a parabolic surface 2009-02-20
From kishore:
how to find out the focal point of a parabolic surfaces
Answered by Chris Langdon.
Gallons of water in a 3 mile pipe 2009-02-19
From tim:
I need to know how many gallons of water a 16" pipe that is 3 miles long holds?
Answered by Penny Nom.
A ten player schedule 2009-02-19
From john:
I HAVE 10 PLAYERS ONE GAME PER NITE FOR 10 WEEKS WHO PLAYS WHO FOR WEEK 1 THRU 10 THANK YOU
Answered by Victoria West.
The volume of a cylinder 2009-02-18
From jim:
given the volume of a cylinder at 565m^3 and the height of 20m how do i calculate the cross section area in order to determine the compression strength of the cylinder
Answered by Harley Weston.
Implicit differentiation 2009-02-18
From Sunny:
Find slope of the tangent line to the curve 2(x^2+y^2)2=25(x^2–y^2) at (3,-1)
Answered by Robert Dawson and Harley Weston.
The substitution method 2009-02-18
From Nicolette:
I am working on linear programming problems that I need to solve pairs of linear equations for. I am confused and am not sure how to solve these problems!
5x+3y=7      y=x-3 How can i solve this using the substitution method?

Answered by Penny Nom.
A line parallel to a given line 2009-02-18
From Lisa:
I'm having a hard time understanding the concept of how to write an equation of the line containing the given points and parallel to the given line.
Express the answer in the form of y=mx+b. (7,9); 2x+5y=4. i would appreciate it alot if someone can show ne how to do this. Thank you, Lisa

Answered by Penny Nom.
A dinner party 2009-02-18
From will:
You are serving 3 different entrees at a party. If 100 guests are having beef, 175 guests are having pasta, and 45 percent of the guests are having chicken how many guests are expected at this party?
Answered by Penny Nom.
More on cardinal numbers 2009-02-18
From Justin:
Hello again, I was just wondering that since the rules of Cantor's cardinal numbers in set theory do not apply to the infinity obtained by limits in calculus (ex. x->0, y=1/x=infinity), does that mean that this infinity is the largest quantity in both calculus and mathematics?

Justin

Answered by Robert Dawson.
The area between the x-axis and a curve 2009-02-18
From Lauren:
This is from a homework question I can't figure out.

Let R be the region in the fourth quadrant enclosed by the x-axis and the curve y= x^2 - 2kx, where k > 0. If the area of the region R is 36 then what is the value of k?

Answered by Robert Dawson.
Angle of depression 2009-02-18
From Meeka:
An aircraft flying at an altitude of 2000m is approaching an airport. If the angle of depression of the airport is 5 degrees, what is the distance from the plane to the airport measured along the ground? Round your answer to the nearest tenth of a kilometer.
Answered by Robert Dawson.
Broken vases 2009-02-17
From JC:
Hello, I need help with a question. It is said that a company delivers 1000 vases to a man. It charged $0.40 for every vase safely delivered. But the company had to pay the man $5.10 for each broken glass. If the man paid a total of $383.50 for the delivery, how many vases were broken?

Regs. ,
JC

Answered by Penny Nom.
A conical sleeve 2009-02-17
From Jonathan:
I'm having a hard time making a design pattern for a cone sleeve, the thing is the cone sleeve is 22 degrees, how can i know the angle of this when it is flat on paper, based on my calculations, it should be around 66 - 69, but i want it to be exact can anybody help?
Answered by Penny Nom.
Miles per hour to kilometers per second 2009-02-17
From Tamara:
If a car goes 60 miles in an hour how many kilometres will it go in a second?
Answered by Penny Nom.
Infinity and Set Theory 2009-02-17
From Justin:
I was just wondering is the limit x->0, y=1/x=infinity, the biggest uncountable infinity according to Cantor's cardinal numbers in set theory?

Justin

Answered by Robert Dawson.
The second derivative of h(x)=f(g(x)) 2009-02-16
From Kristina:
If h(x)=f(g(x)), and is differentiable, then find h"(x).
Answered by Robert Dawson.
Diagonals of a polygon 2009-02-16
From Julie:
How many diagonals can be drawn from one vertex in a 12-sided polygon?
Answered by Leeanne Boehm.
Frogs and kangaroos 2009-02-16
From Jenny:
Albert, Bernard, Charles, Daniel and Ellie play a game in which each is a frog or kangaroo. Frogs' statements are always false while Kangaroos' statements are alwas true. 1. Albert says that bernard is a kangaroo. 2. Charles says that Daniel is a frog. 3. Ellie says that Alvert is not a frog. 4. Bernard says that Charles is not a kangaroo. 5. Daniel says that Elie and Albert are different kinds of animals. How many frogs are there?
Answered by Penny Nom.
The surface area of a cube and rectangular prism 2009-02-16
From Mhiko:
how to derived the total surface area of a cube and rectangular prism using geoboard??
Answered by Robert Dawson.
Cantor's cardinality 2009-02-16
From Justin:
Hello, I was just wondering why the infinity from real numbers is smaller than Beth Two in the context of Cantor's cardinality set theory?

Justin

Answered by Robert Dawson.
Nullity 2009-02-16
From Justin:
What exactly does the term nullity mean in the context of transreal numbers invented by Dr. James Anderson?

Thanks for your help in answering this question!

All the Best,

Justin

Answered by Robert Dawson.
How many gallons does my soup kettle hold? 2009-02-15
From Old:
I have a round kettle I use to make soup for the hungry. I was curious on how many gallons it holds. The radius is 16 inches and the height is 18 inches.
Answered by Penny Nom.
A and B shared some beads. 2009-02-15
From Nicholas:
A and B shared some beads. A gave 1/3 to B, B will have 70 more than A. If A gave 1/5 to B, B would have 10 more than A. A have how many at first ?
Answered by Penny Nom.
The product of two integers their LCM and their HCF 2009-02-15
From Anonymous:
Two numbers have LCM = 60. If their product is 180, what is their HCF?
Answered by Harley Weston.
0/0 2009-02-15
From Justin:
Hello, I was just wondering, what is the difference between 0/0 being represented as nullity or as an indeterminate form?

Justin

Answered by Harley Weston.
The height of a triangle 2009-02-14
From GARIMA:
IF THE SIDES OF TRIANGLE ARE GIVEN . HOW WE WILL CALCULATE THE HEIGHT OF TRIANGLE.
Answered by Penny Nom.
Tearing down a wooden privacy fence 2009-02-14
From Dezra:
we are tearing down a wooden privacy fence, it is 180' in length and the boards are 4"w x 6' H=A0x 1" thick i have a truck that holds 15 cubic yards. I would like to=A0know how many loads it's going to take to haul it off.

thank you for your help
dezra

Answered by Harley Weston.
The floor area in a spherical space station 2009-02-14
From Ed:
I am writing a science fiction novel that involves a spherical space station with a radius of 800 meters. Inside, artificial gravity allows parallel floors set 4 meters apart. If you count the floor that has a radius of 800 meters as Floor 0, then the next floor up (Floor +1) would by 4 meters above the surface of Floor 0. There would then be Floor -1 4 meters down from Floor 0. This would continue until you reach the top or bottom floor, where there is at least 4 meters but less than 8 meters to the top or bottom of the sphere. Obviously the top and bottom floors would have the (same) smallest area, while Floor 0 would have about 2 million square feet.

My problem is figuring out the total area of all of the floors, or for that matter, any particular floors or the total number of floors (the total of all the + floors, the - floors (these numbers will be the same) plus Floor 0.

Ed

Answered by Penny Nom.
The sides of a polygonal building 2009-02-13
From mark:
length of sides of an 18 ft.diameter bldg

i need to find out if i should build an octagon or decagon to keep the sides of the bldg at 8ft or less. how do i figure the length of the sides of each?

Answered by Penny Nom.
8 golfers, 5 rounds 2009-02-12
From John:
8 of us are going on a golf trip next month. We are golfing 5 rounds. Is there any way that everyone can play with each other at least twice? I have been trying to figure it out but it always seems that someone plays with another golfer only once.
Answered by Chris Fisher.
Large exponents 2009-02-12
From Dan:
I would like to know how to calculate a whole number as the base that has a large exponent;Such as 7 to the 23rd.power=. Otherwise if I use a calculator it goes off the scale.
Answered by Stephen La Rocque.
The column space of a matrix 2009-02-12
From John:
Question from John, a student:

A=

1 1 0 0
1 1 0 0
0 0 1 0
0 0 1 1

B=

1 5 6
1 5 6
0 7 2
0 0 9

Is the space spanned by the columns of A the same as the space spanned by the columns of B?
Answered by Harley Weston.

9 golfers 2009-02-12
From Beth:
I am organizing a golf trip for 9 women. We are playing in threesomes and are playing four days. I would love to mix it up so that everyone gets to play with each other at least once. Any mathematical suggestions as to how I can make this happen??
Answered by Robert Dawson.
A four digit number 2009-02-12
From alex:
the number of tens plus the number of hundreds equals the number of thousands. the number of ones is half the number of tens and one more than the number of hundreds the hundreds digit is 2. what is this four digit number
Answered by Penny Nom.
6^x = y 2009-02-12
From Jamie:
Find x: 6^x=y
Answered by Harley Weston.
Forming an arc with 2 inch steel 2009-02-11
From Craige:
I need to calculate the bending radius of 2" wide steel to achieve given inside and outside arc lengths
Answered by Harley Weston.
A semi circular room 2009-02-11
From Taylor:
being 450pi (the area of a semi circle)

how much money would it cost to lay carpet at $2.57 per foot???

if the area of a semi circular room is 450 pi ft then what is the volume being that the diameter is 60 feet and the ceiling is 16 feet tall

Answered by Robert Dawson and Penny Nom.
Cubic feet gas to grams of gas 2009-02-11
From tim:
I was trying to convert cubic feet gas to grams of gas. Is there a formula to complete this task?
Answered by Robert Dawson.
Solid figures 2009-02-10
From Rebecca:
I need to know the names of solid figures. I have several pictures that I need to answer what type they are. Thanks, Rebecca 3rd grade
Answered by Penny Nom.
Evaluating numeric expressions 2009-02-10
From Jennifer:
-521-(-350)-164-(-299)
Answered by Penny Nom.
Solving an equality 2009-02-10
From Deborah:
2(v+10)<18
Answered by Penny Nom.
A goat in a square paddock 2009-02-10
From lachlan:
A farmer uses a rope to tether a goat to a pole at the corner of a square paddock. The length of one side of the square is 24m.

a) If the length of the rope is 12m,what % of the paddock can the goat reach? is the answer 19.62 %?

b) If the farmer wants the goat to be able to graze exactly half the area of the paddock, what length must the rope be?

Answered by Penny Nom.
The variance 2009-02-09
From Ashley:
what is the varinace of these scoress 53,67,43,54,58,54,47,35,45,56,45,47,54
Answered by Harley Weston.
Dividing the digits 1 - 9 into two groups 2009-02-09
From donna:
using the numbers 1 to 9 can you arrange these into 2 groups, both of which have the same total, and no you cant turn the 6 upside down
Answered by Penny Nom.
A definite integral 2009-02-09
From Mathata:
Evaluate: integral from 0 to 1, x^2 e^x^3dx
Answered by Harley Weston.
Seating around a circular table 2009-02-09
From Jenny:
A committee of 15 -- 9 women and 6 men -- is to be seated at a circular table (with 15 seats). In how many ways can the seats be assigned so that no two men are seated next to each other?
Answered by Victoria West.
Are these lines parallel? 2009-02-06
From Conor:
are these lines parallel?

y = 3x -2 and 6y = 3x +2

Answered by Stephen La Rocque.
A triangular pyramid 2009-02-06
From kamran:
what would be an example of a triangular pyramid
Answered by Stephen La Rocque.
A rectangular park 2009-02-06
From Debbie:
A park named Writer's Rectangle opened in town. When asked about the dimensions of the rectangle, the city planner, responded with these clues: ---The diagonals of the rectangular park plus its longer sides together measure seven times one of the shorter sides. ---The length of one diagonal is 250 m longer than one of the shorter sides. Use this information to find the area of the park.
Answered by Stephen La Rocque.
The height of a triangle 2009-02-06
From lisa:
find the height of a triangle, which can be split into two right triangles, but the base (4') is not split equally in half. No interior angles are given.
side a=3', side b=5'

Answered by Penny Nom.
A trig limit 2009-02-05
From Samantha:
lim x-> 0 ( ( r*cos(wt +h) + r*cos(wt) )/ h )

Where r & w are constants.

Answered by Harley Weston.
Congruent line segments 2009-02-05
From casie:
marica drew one-line segment on a graph with endpoint of (0,9) and (0,4). she drew another line segment with endpoints (1,1) and (6,1). are the line segment congruent?explain
Answered by Penny Nom.
A trig limit 2009-02-05
From Kathy:
Hi! I was just wondering how to do this question:

lim 1-cos2x/xsinx as x approaches 0

Thanks,

Kathy

Answered by Penny Nom.
Fill behind a retaining wall 2009-02-05
From Fausto:
i'm trying to figure out how many cubic yards i will need. I built a retaining wall that it's 5 feet high, running 90 feet long. There is a slope running to the wall of 24 degrees. The distance from the top of the slope to the wall is 12 feet. I don't know how to calculate how many cubic yards of fill dirt i need in there. Please help. Thanks!
Answered by Harley Weston.
The height of a triangle 2009-02-05
From Theresa:
Find the area of a triangle by first finding the height. base30ft hypotenuse50ft
Answered by Penny Nom.
Faces, edges and vertices 2009-02-04
From sheila:
what is the relationship between the number of faces and the number of edges of a triangular pyramid ?
Answered by Penny Nom.
A point on -8x^2+5xy+y^3=-149 2009-02-04
From Vivian:
Consider the curve defined by -8x2+5xy+y3=-149
a) find dy/dx
b) Write an equation for the line tangent to the curve at the point (4,-1)
c) There is a number k so that the point (4.2,k) is on the curve. Using the tangent line found in part b), approximate the value of k.
d) write an equation that can be solved to find the actual value of k so that the point (4.2,k) is on the curve
e) Solve the equation found in part d) for the value of k

Answered by Harley Weston.
12 golfers 2009-02-04
From david:
12 golfers playing in 3 four balls over five rounds. can everyone play with everyone else at least once.
Answered by Victoria West.
A 30-60-90 triangle 2009-02-03
From Inez:
If you have a 30-60-90 triangle and the only side you get is 73 and a 90 degree box, how do you find the area?
Answered by Penny Nom.
Randy' age and Mandy's age 2009-02-03
From carolina:
Randy is 4 times as old as Mandy, in 12 years, Mandies age is half of Randy's . how old is each now?
Answered by Penny Nom.
Fertilizer in a bin 2009-02-03
From Todd:
Hello I am looking for a formula to figure out the fertilizer volume in a hopper bottom bin not only when it is full but part full as well. When you are filling it is heaped up in the middle to make a cone and when you are emptying the bin the cone is inverted so it would be nice to be able to quickly figure out the tonnes partly filled and when full.
Lets say the bin is 32 feet high from top of bin where you fill to the bottom where the product goes out and it is 16 feet in diameter. I know how to calculate the cylinder it is the cones on the top and bottom of the bin I have the main question on.

Answered by Harley Weston.
A piecewise function for monthly salary 2009-02-03
From Josh:
A sales rep for a drug company is paid on the following scale. His base salary is $1600 per month. He is also paid a commission of 2% of sales over $20,000. When his gross sales reach $30,000 he is then paid 4% on the amount above that until his sales reach $50,000 after which point his commission becomes 6%. His commission reaches 8% when his sales total goes over $100,000 for the month. Set up a piecewise function for the information above.
Answered by Harley Weston.
Calculating markup 2009-02-03
From Tonya:
I'm trying to figure out the formula for the following, I have a product that I want to receive a certain profit, but with this product I also have to pay a percentage of commission to someone. I need to know how to calculate my new selling price taking into account the markup percentage, but I don't want my cut to be lowered. I have calculated the markup percentage to get my new selling price with the commission, but it lowers my cut once I pay the commission and I don't want it to do that. Thanks
Answered by Penny Nom.
What do I need to pass the course? 2009-02-02
From Lynnette:
If I received 100% on my assignment which is worth 20% of my total mark and 87% on my midterm which is worth 40% of my total mark what would I need to get on the final exam which is worth 40% of my mark. To pass the course I must have 80% or above. Thanks.
Answered by Penny Nom.
4 times as many or 4 times more? 2009-02-02
From Jackie:
Given : Here are 3 squares and 4 sets of 3 circles.

I wonder it is right to write in the below manner to represent the following Conclusion that
can be made from the above given information:

  1. There are 4 times as many circles as there are squares,

  2. There are 4 times fewer square than circles;

  3. There are 4 times more circles than squares.

  4. ...
Jackie

Answered by Robert Dawson.
Simultaneous equations with an xy term 2009-02-01
From angelee:
xy+5x-2y-10=0 2x+y=1
Answered by Penny Nom.
A falling object 2009-02-01
From Tamara:
We drop an object from rest (i.e. its initial spped is zero. It is falling freely toward the ground. What will be its speed after 5 seconds of falling?
Answered by Penny Nom.
What will be its speed after 10s? 2009-02-01
From Tamara:
An object has an initial speed of 5 m/s and is accelerating with 2 m/s2 (m/s squared) acceleration. What will be its speed after 10s?
Answered by Penny Nom.
The sequence 2n! - 1 2009-02-01
From Penny:
I am trying to help my son with this problem. Find the first five terms of the sequence that can be written from the formula A= 2n !-1.
Answered by Penny Nom.
How many gallons = (1 acre * .25 inch) 2009-01-31
From Thom:
How many gallons = (1 acre * .25 inch) Thanks Thom
Answered by Penny Nom.
One acre of land 2009-01-31
From michelle:
Can you tell me how many feet in length would it be for a acre if the land is 155 feet wide. Thank You! michelle
Answered by Penny Nom.
The resulting number is the square of an integer 2009-01-30
From Randy:
A positive integer n is chosen. Then the product n ( n + 1 ) is computed and two digits are appended to the end of the product. The resulting number is the square of an integer.

Show that it is always possible to complete the process above or give a positive integer n for which the process cannot be completed.

Answered by Chris Fisher and Claude Tardif.
7 golfers playing 4 days of golf 2009-01-30
From Trish:
I have 7 golfers and we are playing 4 days of golf. I would like to do pairings so that everyone gets to play with each other at least one time. Can you help with this? Not a math whiz here and looking for help. Thank you.
Answered by Victoria West.
(12+8)/2x42= 2009-01-30
From Susan:
(12+8)/2x42=
Answered by Harley Weston.
limit sinx/x 2009-01-30
From Jackie:
how to evaluate limit sinx/x as x tends to zero if x is in degrees
Answered by Stephen La Rocque and Harley Weston.
The intersection of two lines 2009-01-30
From kaylee:
how do you find and graph the intersect of these two equations: y=2x-4 (and) 2y=x thanks, kaylee
Answered by Stephen La Rocque.
Whitney's average test score was 79 2009-01-30
From dawn:
After 5 tests, Whitney's average test score was 79. After she scored 85 on the 6th test. What was her new average?
Answered by Penny Nom.
The amount of concrete in a football stand 2009-01-30
From jacques:
A football stand 150 yd long has 20 tiers of seats; each tier has a rise of 2ft and tread of 3ft.Its construction of reinforced concrete with a cross section as shown. Find the amount of the material used in its construction.
Answered by Penny Nom.
A railway cut 2009-01-30
From Jean:
A railway cut 200ft. long and 30ft. wide at the bottom is made with sloping sides, which are 80ft. and 60ft. in length. The 80-ft. side is inclined 45 degrees and the other side is 35 degrees to the horizontal. Find the cost of removing the earth at $2 per load, if the trucks have capacity of 4cu. yd.
Answered by Penny Nom.
A die is loaded so that... 2009-01-30
From Larry:
Hi. I'm having trouble solving this problem. Would you be kind enough to point me in the right direction?
A die is loaded so that the probability of a particular face rolling up is proportional to the number of dots on the face. In other words, getting a six is three times as probable as getting a two. what is the probability of getting an even number in one toss?

Answered by Victoria West.
Shapes 2009-01-29
From Rana:
I want to know about Circumference / Area / Volume of different shapes ie Circle , Oval , An Arc Trapezide etc
Answered by Robert Dawson and Harley Weston.
The height of an isosceles triangle 2009-01-29
From Mariah:
An isosceles triangle has sides 10cm, 10 cm and the base 4cm. How do you find out the height of the triangle?
Answered by Penny Nom.
Will Juanita be late for class? 2009-01-28
From safdar:
juanita bumps into tory in the hallway between period 1 and 2,there is a 45% chance that she will be late for class.if she does not bump into tory, she will make it to class on time. there is a 25% chance that juanita will bump into tory. what is the probability that she will be late on any given day. be sure to introduce your events for this question and show the appropriate formula given the events.
Answered by Penny Nom.
y = |x| - 4 2009-01-28
From tiana:
how do you find where to plot Y in a problem that states y=|x| -4?
Answered by Robert Dawson and Penny Nom.
Ambitious 7th Grader 2009-01-28
From John:
I have been asked to tutor a 7th grader who is very good in math. He is taking the most advanced course that his school offers to 7th graders. This course will cover Algebra 1 over a period of two years, using McDougal Littell's Algebra, Structure and Method, Book 1, a book I like a lot.

I have been hired not to help him with his schoolwork, which is too easy for him, but to guide him through something in math beyond his schoolwork. This was his idea, an idea which surprised his parents, but one which they support.

Do you have a suggestion for me? If so, I would be very grateful to hear from you. I think this could be a lot of fun for me and my new student, if I pick the right path.

Thanks and best regards, John John

Answered by Robert Dawson, Victoria West and Walter Whiteley.
Graphing a parabola 2009-01-27
From Kimberly:
I need help with this parabola: graph y=5x^2-5x-6. I am not really understanding how to graph parabolas in general. Can you help me?
Answered by Penny Nom.
Mixing candy 2009-01-27
From adriana:
The question says:
that a merchant wishes to mix candy worth 45 cents per pound, with some candy that costs 80 cents per pound. this will create a mixture of candy that sells for 65 cents per pound. there is a total amount of 350 pounds of candy.

What i need to know is:
The amount of pounds of candy at 65 cents per pound? and
The amount of pounds of candy at 80 cents per pounds?

Answered by Harley Weston.
25,000 pounds of sheet metal 2009-01-27
From JESSICA:
I need to convert 25,000 lbs. of 32x48 sheet metal into total sq. feet. What would be my formula?
Answered by Robert Dawson.
Find area of the rectangle in terms of L. 2009-01-27
From Arlen:
The length of a rectangle is L. The width is 8 less than the length. Find area of the rectangle in terms of L.
Answered by Penny Nom.
The vertical line test 2009-01-26
From bob:
what is a Vertical line test
Answered by Stephen La Rocque.
A tennis schedule 2009-01-26
From Lorraine:
I need to create a tennis schedule. We have 8 players, but only 4 play at one time. I need to create a schedule for 34 matches, but of the 34 each player plays 17 times. Each player must also play with each of the other 7 the same number of times. For instance player 1 will play 17 times, and will play with the other players the same number of times as player 2 does. Is this even possible?
Answered by Robert Dawson.
Grade of a hill 2009-01-26
From rachel:
if the grade is 18%, how many feet would it fall vertically over a horizontal distance of 250 yards.
Answered by Robert Dawson.
Solve (3x-2)/4 - 3x = 12 2009-01-26
From michelle:
3x-2/4 - 3x = 12
Answered by Penny Nom.
Linear speed 2009-01-26
From Hom:
A semi is driving cross country at 70mph. The tires on the semi have a 3 foot diameter. What is the linear speed in feet per minute?
Answered by Robert Dawson.
Using y = mx + b 2009-01-26
From Caroline:
How do you write a symbolic representation using the y=mx+b method?
Answered by Robert Dawson.
25 names in a hat 2009-01-26
From Irfan:
If there are 25 names in a hat, 15 girl's names and 10 boy's names, what is the probability that a name drawn at random will be
I. a girl's name
II. a boy's name
III. not a girl's name
IV. not a boy's name.

Answered by Robert Dawson.
How many color combinations are possible for the four rooms. 2009-01-26
From Irfan:
John wants to paint 4 rooms in his house, each a different color and he may choose from 7 different colors of paint. How many color combination are possible for the four rooms.
Answered by Robert Dawson.
A regular hexagon is inscribed in a circle. 2009-01-26
From Thejas:
A regular hexagon is inscribed in a circle. If the perimeter of the hexagon is 42 inches, how many inches are in the circumference of the circle? How do you express this in the terms of pi?
Answered by Robert Dawson and Penny Nom.
How many students were there 2009-01-25
From sooos:
After a math lecture in university, the students go to the cafeteria. EACH ONE of them buys a cheese sandwich and a cup of tea. They ALL pay $18.49. How many students were they?
Answered by Chris Fisher.
A cubical tank that is 8 m on a side 2009-01-25
From gina:
Suppose one edge of a cubic tank is 8m; Find the volume of the tank in cubic meters
Answered by Penny Nom.
The area of a triangle 2009-01-25
From black:
Explain how the formula for the area of a triangle can be determined by using the formula for the area of a parallelogram.
Answered by Penny Nom.
An octagonal carpet 2009-01-24
From Larry:
I am cutting a piece of carpet that is 9 feet x 9 feet and need to cut it into an octagon. How far do I cut from the corner of the square. thank you for any help.
Answered by Harley Weston.
How fast is the visible surface of the earth decreasing? 2009-01-24
From Ray:
A dive bomber loss altitude at a rate of 400 mph. How fast is the visible surface of the earth decreasing when the bomber is one mile high?
Answered by Harley Weston.
Perimeters and scale factors 2009-01-24
From dennis:
I have this work that says how are the value of the ratios for the perimeters related to the scale factor? PLEASE HELP ME OUT
Answered by Harley Weston.
An integral from 1 to infinity 2009-01-24
From Ray:
Determine the area bounded by the x-axis and the curve y=1/(x^2) from x=1 to x=infinity.
A. 1.00
B. infinity
C. indeterminate
D. 2.00

Answered by Harley Weston.
20 of us golf together in groups of 4 2009-01-24
From D.:
Every Sunday, 20 of us golf together in groups of 4. I am looking for a way that each of us play with 3 other people each week and ultimately get to play in groups that are unique. For instance if week 1, I play with 2, 3, 4 and then the next week I play with 5, 6, 7, and the 3rd week I play with 8, 9, 10 and so forth until I have played with everyone. Everbody else should be doing the same thing. Can you give me a schedule for this and how many weeks would it take for all of us to accomplish this where we all play with different combinations of people. (We should not play with the same person very often or even the same pairs of people but everyone should play with everybody else) I hope this makes sense........whew and thanks!
Answered by Victoria West.
Water flowing from a cone to a cylinder 2009-01-23
From Ray:
Water is passing through a conical filter 24 cm deep and 16 cm across the top into a cylindrical container of radius 6 cm. At what rate is the level of water in the cylinder rising when the depth of the water in the filter is 12 cm its level and is falling at the rate of 1 cm/min?
Answered by Harley Weston.
Two tangent circles 2009-01-23
From Murtaza:
Two circles touch externally at T. A chord of the first circle XY is produced and touches the other at Z. The chord ZT of the second circle, when produced, cuts the first circle at W. Prove that angle XTW = angle YTZ.
Answered by Robert Dawson and Chris Fisher.
A cyclic quadrilateral 2009-01-23
From Murtaza:
Line ATB touches a circle at T and TC is a diameter. AC and BC cut the circle at D and E respectively.Prove that the quadrilateral ADEB is cyclic.
Answered by Robert Dawson and Chris Fisher.
The tens digit of 0! + 1! + 2! + 3! + 4! .............+9999! + 10000! 2009-01-23
From Kaylyn:
determine the tens digit for the expression:
0! + 1! + 2! + 3! + 4! .............+9999! + 10000!

Answered by Robert Dawson and Penny Nom.
Archimedes' formula for parabolic arches 2009-01-23
From La:
Use calculus to verify Archimedes' formula for y=9-x^2. Prove Archimedes' formula for a general parabolic arch.
Answered by Harley Weston.
In the shadow of a flagpole 2009-01-22
From La:
How fast is the length of the shadow of an 18 foot flagpole growing when the angle of elevation of the sun is 45 degrees and is decreasing at a rate of 10 degrees per hour?
Answered by Harley Weston.
Probability and birthdays 2009-01-22
From La:
Neglecting the effect of leap years, what is the chance that, of 6 people selected at random, 2 will have the same birthday?
Answered by Penny Nom.
1 foot drop every 25 feet 2009-01-22
From jerry:
1 foot drop every 25 feet what is the angle of the degree
Answered by Penny Nom.
The parabola with vertex (7,-2) and directrix y = -3 2009-01-21
From Deann:
Find an equation of the parabola with vetrex (7,-2) and directrix y =(-3)
Answered by Penny Nom.
The height of a triangle 2009-01-20
From ELEY:
WHAT IS THE HEIGHT OF A TRIANGLE IF THE AREA IS 32.4 AND A BASE OF 6.75?
Answered by Robert Dawson and Penny Nom.
PST and GST 2009-01-20
From janet:
Hello I need to figure what one would pay pst 8% gst 5% on a total of $1700.00 I have a small business some pay with cash I need to calculate the taxes in reverse Can you help with formula that the government might like Thank you Janet
Answered by Harley Weston.
The Golden Ratio 2009-01-20
From Vincent:
hello, my name is Vince,

I am wondering how the Golden ratio was used by early mathematicians. What formula did they use to find it? open for anything... thank You!

Answered by Robert Dawson.
A triangle with sides of length 6 and 7 2009-01-20
From bill:
If I have a triangle with sides 6 and 7, which of the following is a possible area? The answers are 13 and 21, but not 24. I know this is because of a possible range of heights, but how do I calculate the range of heights? This is an SAT question. Thanks
Answered by Chris Fisher.
The number of gallons in a tank 2009-01-19
From Jim:
Hi, i need to know how many gallons of water can fit into our tank, and how to do the calculations my self. The tank is 72" round by 72" long, i would like some help. Thanks Jim.
Answered by Harley Weston.
The area of an isosceles triangle 2009-01-19
From faris:
how to calculate area of isosceles triangle with two angles of 45 degree and base of 6
Answered by Penny Nom.
The volume of a prism 2009-01-19
From Mady:
What is the general equation used to find the volume of a triangular prism?
Answered by Robert Dawson and Harley Weston.
The area of a quadrilateral shape 2009-01-19
From craig:
how do u work out the area of a quadrilateral shape
Answered by Robert Dawson.
Two chords in a circle 2009-01-19
From azlaan:
prove that line joining the midpoint of 2 equal chords substain equal angle with the chord?
Answered by Robert Dawson.
The ratio of the measures of the three angles of a triangle 2009-01-18
From mary:
The ratio of the measures of the three angles of a triangle is 2:5:8. What is the measure of each angle of the triangle?
Answered by Robert Dawson and Harley Weston.
20 miles of gasoline 2009-01-18
From Roger:
I used the volume formula and found that a cylinder with a 3" radius X 8.75" height yields just over 231 cu. in. (one gallon)- doesn't seem right, but that's what I got. Then found the percent that 8.175" is to one foot and multiplied that into the number of feet in 20 miles. I'm not good at math, so I'm starting to lose myself here. The short story is I get a cylinder that is 20 miles long and only .0000387" in diameter. Is this even close to the mark?
Answered by Harley Weston.
The point of intersection of the straight lines 2009-01-18
From Kieran:
Find the point of intersection of the straight lines

y=2x+4, and y+2x-16=0

Answered by Robert Dawson and Penny Nom.
The dimensions of a pool 2009-01-18
From denise:
a rectangular swimming pool is three times as long as it is wide. if the perimeter of the pool is 320 feet, what are its dimensions
Answered by Penny Nom.
Partial derivatives 2009-01-17
From Meghan:
I have a question I've been working at for a while with maxima/minima of partial derivatives.

"Postal rules require that the length + girth of a package (dimensions x, y, l) cannot exceed 84 inches in order to be mailed. Find the dimensions of the rectangular package of greatest volume that can be mailed. (84 = length + girth = l + 2x + 2y)"

Answered by Harley Weston.
How far is the airplane from the control tower? 2009-01-17
From Murtaza:
An aircraft is vertically above a point which is 10 km west and 15 km north of a control tower. The aircraft is 4000 m above the ground, how far is it from the control tower?
Answered by Penny Nom.
The whole number that precedes a number w+3 2009-01-17
From Morgan:
write a variable expression for "The whole number that precedes a number w+3"
Answered by Penny Nom.
A 28 week golf schedule 2009-01-17
From Don:
You have a 12 man golf schedule playing 8 weeks. Can you go further and make it a 28 week schedule. 3 foursomes. any help would be greatly appreciated.
Answered by Victoria West.
Fuel in a tank 2009-01-16
From Steve:
I am not a math expert and want help seeing how much fuel is in a tank at one given time the tank that i have is 27 inches round. It is 69 inches long. laying length ways. it holds 140 gallons of fuel at fill up. I do know that it would hold less at the bottom of the tank an more in the center. Is there anybody that could just give me the figures by the inch without a formula. thanks for your time
Answered by Stephen La Rocque.
Problem solving with fractions 2009-01-16
From Linda:
Sam and Mary each owned one-half stock in a printing company. Sam sold 2/5 of his stock to Mary. What fractional part of the printing business does Mary now own?
Answered by Stephen La Rocque.
A golf trip with 12 golfers 2009-01-16
From Brad:
I am organizing a golf trip with 12 golfers. We will play 5 rounds (3 foursomes each round) total and play two 9 holes matches per round (18holes). So two matches are created within each foursome. What are the possible combinations of foursomes so that everyone plays each other at least once with the least number of repeat matches? For example: Here are the foursomes for round 1:(1,2,3,4) (5,6,7,8) (9,10,11,12) match 1 round 1: 1vs2, 3vs4, 5vs6, 7vs8, 9vs10,11vs12 we can not split foursomes up for the second match (9 hole matches): So match 2 round 1: 1vs3, 2vs4,5vs7,6vs8,9vs11,10vs12
Answered by Victoria West.
Solve for x 2009-01-16
From maritza:
5=3(x-2)=
Answered by Penny Nom.
0.0708 lbs per linear yard 2009-01-16
From Kevin:
If I know that my product is .0708lbs per linear yard how do I calculate how many Linear yards are in a lb?
Answered by Penny Nom.
An expression for q quarters and n nickles 2009-01-16
From Morgan:
a variable expression for:

"The value in cents of q quarters and n nickels"

Answered by Penny Nom.
A line is perpendicular to another line 2009-01-16
From Greg:
A line is perpendicular to the line y=2x+3 and has the same x-intercept as x+3y+10. Find the equation of this line. Express your answer in the form of y=mx+b. Justify your answer.
Answered by Robert Dawson.
Saving for his daughter's education 2009-01-15
From George:
To save for his daughter’s university education at the end of each month Mr McLean sets aside £50 in a savings scheme paying 10% p.a. compounded monthly. He begins saving when his daughter is 3 years old. How much will he have saved by the time his daughter is 18?
Answered by Harley Weston.
Multiplying negative numbers 2009-01-15
From stephanie:
hi, I'm currently working in a grade 8 class where the students are learning about integers. the students were asked to find examples of how integers are applied in real life. they were able to find aplications for: adding and subtracting both positive and negative integers. however, we could not find an example of where two negative integers would be either divided or multiplied together in real life. For example: (-2)(-4)= +8. please help.
Answered by Harley Weston.
Does probability apply here? 2009-01-15
From Paul:
Suppose I have a dice in my hand that I am about to roll. The probability that I roll a six is, all things being equal, 1/6. I accept that.

Suppose now the I roll the dice and immediately cup my hand over the result. What is the probability that I have rolled a six? People seem to want to say it is still 1/6. But it can't be can it!? It is surely either 1 or 0, depending on whether I have in fact rolled a six?

Answered by Robert Dawson.
Concrete around a pipe 2009-01-14
From Doug:
How much concrete will i need for a hole that is 20 feet deep 20 inches in diameter with a 8-inch pipe in it. I need to know how much concrete on the outside of the 8-inch pipe.
Answered by Penny Nom.
The circumference of a table 2009-01-14
From Maggie:
My dinning room table is 48 inches. I need to know what its Circumference is? I do not know algebra so a formula will not help me.
Answered by Penny Nom.
A applied math trig problem 2009-01-13
From Simon:
I wish to find all the answers for the following equation over the interval (0,1):
cos^2(pi * n^x) + cos^2(pi * n^(1-x)) - 2 = 0 where n is any integer > 0

Answered by Robert Dawson and Harley Weston.
A rectangular prism 2009-01-13
From nika:
How many bases does a rectangular prism have? What are they?
Answered by Harley Weston.
For which integers a, b, c does a - b - c = a - (b - c)? 2009-01-13
From Johanna:
For which integers a, b, c does a - b - c = a - (b - c)?
Answered by Penny Nom.
Two methods of payment 2009-01-13
From adjoa:
owen was offered $225.00per week plus 3%on sales, or 15%of sales with no salary. How much must he sell before the second method of payment gives him the highest pay? Express your answer to the nearest dollar.
Answered by Penny Nom.
The volume of a cone 2009-01-13
From ab:
the area of the base of the cone shown is 314 cm and it's height is 12 cam what is the volume?
Answered by Penny Nom.
Lita and Rose start jogging on a 110 m circular track 2009-01-13
From Narcisa:
Lita and Rose start jogging on a 110 m circular track. They begin at the same point, but jog in opposite directions, lita at 8/3 meters per second and rose at 7/3 m per second. Find the number of times they they will pass each other during the first 15 minutes of jogging?
Answered by Robert Dawson.
Negative rate of change 2009-01-12
From hemanshu:
when i have to find rate of change of decrease in any value my ans comes in negative why??????????
Answered by Penny Nom.
A seven card poker hand 2009-01-12
From Sean:
from a standard card deck, seven cards are chosen at random. find the probability that the set chosen contains exactly three of each of two different face values.

the second part of the problem asks the probability that the card hand contains exactly three 2's and exactly three 5's.

Answered by Robert Dawson.
A 10% salt solution 2009-01-12
From Joanna:
How many liters of water must evaporate from 40 liters of a 10% salt solution to make a 25% salt solution?
Answered by Robert Dawson and Penny Nom.
16 golfers 2009-01-11
From Bill:
I have read many of your answers to similar questions but still need your assistance. We have 16 golfers that will be playing in 4 groups of 4 each day on an upcoming golf vacation. Can you suggest the best way to scheudle the individuals assigned to each daily foursome so that we have the most diversity in foursomes throughout the vacation. I am trying to have evryone play with as many members of the group over the week with the least amount of repetition. If you would please provide it for both 6 and 7 days, since we are not sure yet how many days we will be playing.
16 golfers-playing in groups of 4 each day-6 days
16 golfers-playing in groups of 4 each day-7 days

Answered by Victoria West.
The slope intercept form 2009-01-11
From Norma:
write equation in slope intercept form and then find the slope and y-intercpet of the corresponding line.

please show step by step

1). 2x-3y-9=0 and 2). 3x-4y+8=0

Answered by Penny Nom.
A stack of rectangles 2009-01-11
From ashwani:
Could you please advise the area of a rectangles pyramid. There are 5 rectangles placed one above the other. The top most rectangle has length 2 cm and height 2 cm. The subsequent rectangles length is increased by 1 cm on both the sides, while the height of the 2nd and 3rd rectangle is 2cm each, while the 4th & 5th has a height of 3 cm each. Could you please let me know the the area of the figure and the formula to calculate
Answered by Penny Nom.
A T-shirt fundraiser 2009-01-11
From Andre:
Lou's class is selling T-shirts for a fundraiser. The supplier charges 750$ for the initial design and the set-up plus 5$ for each imprinted shirt. The students sell the shirts for 15$ each.
a)How many T-shirts do the students need to sell to break even?
b)How much profit will the students make if they sell 150 shirts?

Answered by Harley Weston.
I lost the combo to my lock box 2009-01-10
From Rob:
I lost the combo to my lock box, I remember 2 of the #'s in the combo are 6, but dont remember which ones. What are the possible scenarios knowing the above? Thanks!
Answered by Stephen La Rocque.
Average and total 2009-01-10
From cecil:
the average of six number is 84. what is the total
Answered by Stephen La Rocque and Penny Nom.
A flat-topped pyramid 2009-01-10
From Tom:
I am planning to build an open ended, flat-topped pyramid with a rectangular base 28 x 36 in.; a square top 6 x6 in.; and a vertical height of 16 in.

I know how to calculate the dimensions of each of the pyramid sides but I don't know how to calculate the interior angles when I bend the sheet metal. I will cut the four sides separately and add a one inch flange to each "vertical" edge of the two larger sides to allow for a bonding surface for the assembly. How do I calculate that angle? I know it will be greater than 90 degrees but by how much?

Answered by Harley Weston.
Triangular prisms and pyramids 2009-01-10
From KJ:
What are some items found in a home or school that could be considered a triangular prism or triangular pyramid?
Answered by Robert Dawson and Penny Nom.
18 golfers and 5 rounds 2009-01-10
From steve:
We have 18 golfers 5 rounds . Would like 3 foursomes and 2threesomes each day Different groups each day.
Answered by Victoria West.
What is the maximum revenue? 2009-01-09
From Kristy:
A skating rink manager finds that revenue R based on an hourly fee x for skating is represented by the function R(x) = -200x^2 + 1500x

What is the maximum revenue and what hourly fee will produce maximum revenues?

Answered by Harley Weston.
A wooden deck around a pool 2009-01-09
From lupio:
a 20' pool is surrounded by a 3' wood deck, find the amount of material needed for the project
Answered by Robert Dawson and Penny Nom.
A max/min problem 2009-01-09
From Angelica:
have 400 feet of fence. Want to make a rectangular play area. What dimensions should I use to enclose the maximum possible area?
Answered by Robert Dawson.
Who am I? 2009-01-09
From daphne:
im a 2-digit number less than 85.the sum of my digits is 9.the number in my ones place is twice my tens digit. who am i?
Answered by Robert Dawson and Harley Weston.
What day of the week will it be? 2009-01-09
From dan:
suppose 3 days ago was a wednesday. what day of the week will 365 days from today be?
Answered by Robert Dawson.
Brothers and sisters 2009-01-09
From Jade:
Pat, Joe, Dave,Yoly, Dolor and Alma are three brothers - and sister - pairs who play double tennis. Brother and sister ca not play in the same pair. Pat and Yoly play Dave and Dolor on one day. Dave and Yoly play Joe and Alma on another day. Name each brother - and sister pair, in any order.
Answered by Robert Dawson.
A one cubic mile lake 2009-01-09
From dye:
how many liters are in a one cubic mile lake
Answered by Robert Dawson.
When is Sean's birthday? 2009-01-09
From Robin:
Sean is 4 months older than Tony. Mia is 6 months younger than Tony. If Mia's birthday is in April when is Sean's?
Answered by Robert Dawson.
The table of values for y=3x-10 2009-01-09
From Jade:
How can I know the value of x for y=3x-10 in this table of values pls. include the deatails

if x=-2 y=-16
if x=3 y=-1
but how if y=2 what is x

Answered by Penny Nom.
Two angles are supplementary 2009-01-08
From Stephanie:
two angles are supplementary, one of the angles is 30 degrees more than double the other angle. find the first angle, the second angle the complement of the given angle.
Answered by Robert Dawson.
The top half of a cone vs. the bottom half of the same cone 2009-01-08
From Tammy:
What is the ratio of the volume of the top half of a cone vs. the bottom half of the same cone? Is it seven times larger, or four times larger? and is the ratio consistent regardless of the size of the cone opening?
Answered by Penny Nom.
660 yards = ?? miles 2009-01-08
From Ashley:
How do I convert yards to miles, specifically, 660 yards = x miles. Please show work so I can figure other problems out.
Answered by Penny Nom.
50 coins with a value of $1.00 2009-01-08
From Ana:
you have 50 coins which have a total value of 1.00. What are the coins and how many of each do you have?
Answered by Robert Dawson.
Grade 4 Foundation Skills 2009-01-07
From Susan:
Hi, I'm taking over tutoring my 15 yr old niece, who has dyslexia. She has been being tutored by someone else,but my sister had problems with her. My niece needs help in math and english and spelling. Is there a test that I can do with her to see where she stands at this point? I do believe the math is at a 3rd, or 4th grade level. I'm not sure about the other two. Do you have any suggestions? All help is appreciated.
Answered by Victoria West.
Numbers or Percentage or probability or what? 2009-01-07
From Chew:
Teacher A teaches 50 students and 45 of them obtained straight As. Teacher B teaches 5 students and 5 of them obtained straight As. Which teacher would you send you child to and why?
Answered by Robert Dawson and Harley Weston.
The area of a region bounded by two curves 2009-01-07
From Rogerson:
Find the area, S, enclosed by the given curve(s) and the given line.
y = x^2 - x - 1, y = x+2

Answered by Harley Weston.
The volume and surface area of a sphere 2009-01-07
From destini:
the volume of a sphere is equal to its surface area. what is its radius
Answered by Chris Fisher.
A kennel with 3 individual pens 2009-01-06
From Jean:
An animal clinic wants to construct a kennel with 3 individual pens, each with a gate 4 feet wide and an area of 90 square feet. The fencing does not include the gates. Write a function to express the fencing as a function of x. Find the dimensions for each pen, to the nearest tenth of a foot that would produce the required area of 90 square feet but would use the least fencing. What is the minimum fencing to the nearest tenth?
Answered by Harley Weston.
Filling a round hole 2009-01-06
From Kathy:
I need to fill a hole that is 24 feet round x 12 inches deep. How much dirt will I need?
Answered by Robert Dawson.
y varies jointly as x and the square of z and... 2009-01-05
From peter:
I don't really get the terms in this problem. Could you explain it: If y varies jointly as x and the square of z and inversely as the cube of w, by what factor is y increased when z is doubled and w is divided by 3 and x remains the same?
Answered by Harley Weston.
A 4-digit number 2009-01-05
From Anya:
It is a 4-digit number

The tens are double the ones.

The thousands are double the tens.

The sum of the digits is 19.

What number is it?

Answered by Penny Nom.
The area enclosed by a curve and the x-axis 2009-01-04
From Rogerson:
Find the area, S, enclosed by the curve y = -x^2 + 6x - 5 and the x-axis in the interval 0≤x≤4.
Answered by Harley Weston.
Determine y'' by implicitly differentiating twice 2009-01-04
From Walter:
Given x^3 - 3xy + y^3 = 1 , determine y'' by implicitly differentiating twice. I cannot solve this. Would you be kind enough to perform the mathematics and show the steps involved in obtaining the solution?
Answered by Harley Weston.
Converting grams to milliliters 2009-01-04
From Faiza:
I was looking for how to convert grams into mL and I read on this site to convert one to the other you need to know what substance you are measuring. so if I am measuring sugar or any of the baking ingredient like flour how do I convert them from grams to mL is there a simple method I can use to do that?
Answered by Penny Nom.
The area of a region in the plane 2009-01-03
From Rogerson:
Find the area, S, of the shaded region enclosed by the given cureve, the given line and the x-axis.

y = -x^2 + 1
line x = 2

Answered by Harley Weston.
The integral of f(x)dx and the integral of f(x-4)dx 2009-01-02
From Katie:
Calculus: If the definite integral from -2 to 6 of f(x)dx=10 and the definite integral from 2 to 6 of f(x)dx=3, then the definite integral from 2 to 6 of f(x-4)dx= ?

I don't understand how to solve definite integrals when the function has something more than just x inside the parenthesis such as f(4-x).

Answered by Robert Dawson.
The radius of a cone 2009-01-02
From kalpaj:
A conical funnel holds 100ml. If the height of the funnel is 10 cm, determine its radius, to the nearest tenth of a centimeter.
Answered by Penny Nom.
Solving an inequality 2009-01-02
From jarred:
3x+8>2
Answered by Penny Nom.
A chord 2009-01-01
From Deepa:
Why in a circle,a chord is called as the longest diameter? (A chord does not passes through the centre of the circle but a diameter passes through the centre)
Answered by Penny Nom.
Eric bought 3 sweaters and 1 pair of pants 2009-01-01
From maya:
eric bought 3 sweaters and 1 pair of pants. Each sweater cost d dollars and the pants cost 20 dollars. Eric spent a total of 95 dollars. The equation below can be used to determine the cost of each sweater. what is the answer?
Answered by Penny Nom.
The volume of a pipe elbow 2008-12-31
From mhd:
how i can find volume of the pipe elbow of 4inches?
Answered by Stephen La Rocque.
The square root of 164 2008-12-31
From Melissa:
How do you simplify the square root of 164?
Answered by Stephen La Rocque.
What is i^i? 2008-12-27
From randomness:
i have learnt that 'i' is square root of -1. What is then i^i ? It baffles my maths teacher...
Answered by Robert Dawson and Penny Nom.
Two rectangular plots have the same area 2008-12-23
From Jason:
Two rectangular plots have the same area. The first has a perimeter of 40m while the second has a length 2m less than the first and a width 1m greater. Find the length and breadth of the first rectangle
Answered by Stephen La Rocque.
The area of a triangle 2008-12-23
From ashiya:
Hi,

I have attached figure of the triangle, Can you please help to find the area of Triangle ABC.

Thanks.

Ashiya

Answered by Stephen La Rocque.
24 golfers playing golf for 9 rounds 2008-12-23
From Duane:
Got 24 golfers playing golf for 9 rounds. Any formula where everyone can play with everyone else at least once. We are playing 4somes only. Thanks, Duane
Answered by Victoria West.
The base of a triangular pyramid 2008-12-20
From joe:
triangular pyramid
base height=
base width=8ft
height=6ft
volume=88ftcu.

Please explain base height & how to solve.

Answered by Penny Nom.
Pouring angles for a crucible 2008-12-20
From Richard:
I am trying to work at pouring angles and volume left in during pouring a crucible, The crucible is cylindrical and flat bottomed.

I know the diameter, radius and volume of the crucibles. and the volume of liquid going into it.

So lets say the crucible is only half full firstly I need to work out the angle just before its going to pour. ( I can work this out as long as there is a certain volume of liquid if its not enough I cant do it)

Now the problem I also need to work out how much I should tilt the crucible to allow a certain amount out and be able to do this untill the volume reaches 0 at 90' turn. This is where I am stuck.

The reason for needing to be able to work this out is so i can develop a constant flow for example 10Kg of metal per second. Thank you very much for you time

Answered by Harley Weston.
A spill of oil on a flat surface 2008-12-19
From Phillip:
Hello, my question is: If I spilled 25 gallons of diesel fuel on a flat surface how much area would the spill cover? How would I mathematically figure it out? Thanks!
Answered by Robert Dawson and Harley Weston.
A standard bill on a highway 2008-12-19
From Stephanie:
A standard bill on a highway has a perimeter of 14 feet. the length of the bill board is 6 feet greater than triple the width. What is the length? what is the width?
Answered by Robert Dawson and Penny Nom.
Three circular rings 2008-12-18
From seema:
three circular rings of equal radii of 1cm each are touching each other. a string runs all around the set of rings very tightly. what is the minimum length of string required to bind all the three rings ?
Answered by Robert Dawson.
Three circles inscribed in a circle 2008-12-18
From seema:
three equal circles each of radius 1 cm are circumscribed by a larger circle.find the perimeter of circumscribing circle?
Answered by Robert Dawson.
Solving an equation for D 2008-12-18
From Darrin:
In his finance class, my son is being asked to take the following equation and solve for "D":

1 / [1 - (1 + D/12)^T] = (1 / D) * [((12*B) / (P*T)) + Y + ((12N * Z) / (360T))] - [(12Z) / (360T)] * F

Answered by Robert Dawson.
The volume of a feed hopper 2008-12-18
From John:
I need to calculate the volume of a feed hopper, and I'm not sure how to break it down. The top of the hopper is 36" x 36", it is 30" deep, and ends at a 6" x 6" plate. One side of the hopper is straight top to bottom, of course tapering on two sides to meet at the plate. The other three sides angle down at about 75 degrees. I need to determine the cubic foot volume of this hopper (it is used for ground coffee) so I can configure a vibrator to knock down residual grounds. Thanks.
Answered by Robert Dawson.
Integral of cos^2 X between pi/2 and 0 2008-12-18
From Wanda:
Integral or Area of cos^2 X between pi/2 and 0.

The answer that I got is -pi/4. Is this correct? If not, how did you come up with your answer?

Answered by Robert Dawson.
The 2 rightmost digits 2008-12-18
From Peter:
Is there a pattern for the 2 rightmost digits of a power? For example, one problem for a math competition was what are the 2 rightmost digits of 3^1993?
Answered by Robert Dawson and Victoria West.
The angle between two lines 2008-12-17
From abhi:
how to calculate the angle between two lines, given the length of the lines.. angle should vary from 0 - 360 in the counterclockwise direction
Answered by Robert Dawson and Harley Weston.
An equilateral triangle 2008-12-17
From lorraine:
an equilateral triangle has side lenghts of10.the length of its altitude is?
Answered by Penny.
Solve for x 2008-12-16
From Melissa:
I have a test tomorrow and I'm hoping you can help me before then. I can never seem to solve the "RESOLVE X" problems, or in french resous pour x/
They look like this. 3x+2\6=2x-5\3. I only understand NOTHING from that. Another equation is 2(x+1)=3(x+2).

Answered by Robert Dawson and Penny Nom.
Two numbers 2008-12-15
From HERB:
I am thinking of two whole numbers. When I add them , their sum is 123. When I subtract the lesser number from the greater number their difference is 45. What are my numbers?
Answered by Penny Nom.
Why is the difference between british and american counting? 2008-12-15
From Muhammad:
Why is the difference between british and american counting?
Example 1 Billion american = 1,000,000,000 but,
1 Billion British = 1,000,000,000,000

Answered by Robert Dawson and Harley Weston.
The middle term of an arithmetic sequence 2008-12-15
From Leigh:
Find the sum of the first fifteen terms of an arithmetic series if the middle term is 92
Answered by Penny Nom.
Surface area of an irregular shape 2008-12-15
From Patrick:
An irregular shaped object (lets say a gold nugget, not smooth with pockets) can have its volume determined by comparing its mass in water.

Is there any method or means or anything that could be used to determine the surface area of this shape? Whether that be theoretical mathematical formula to using a special infrared technique,etc...

The problem I foresee is that the component parts cannot be divided into smaller geometric shapes. I would propose an answer although I don't know if it is a good one: A liquid material that dries super-thin, but has a very specific and easily determined volume/mass is coated over the object. Measure the mass difference between the beginning sample of fluid and the mass after the object has been coated. Then determine the surface area of the same mass of fluid in a geometric shape. Is this feasible?

Answered by Robert Dawson.
A rectangle with diagonal 8.5 inches 2008-12-14
From nekasia:
what is the area of a rectangle with a diagonal of 8.5 inches PLEASE SHOW WORK
Answered by Penny Nom.
A sphere in a can of water 2008-12-12
From Meghan:
A cylindrical can open at the top has (inside) base radius equal to 1. The height of the can is greater than 2. Imagine placing a steel sphere of radius less than 1 into the can, then pouring water into the can until the top of the sphere is just covered.

What should be the radius of the sphere so the volume of water used is as large as possible?

Answered by Harley Weston.
650g+2kg+195g=how many kg 2008-12-11
From christian:
650g+2kg+195g=how many kg
Answered by Penny.
Wall covering 2008-12-10
From Jeanie:
I need to convert 14 LY of wallcovering (54" wide) to square feet. Can you please help me?
Answered by Penny Nom.
The volume of a pipe 2008-12-10
From Walter:
What is the volume of a pipe 7.92 inches inside diameter and 22 km. long in cubic feet of water
Answered by Robert Dawson and Harley Weston.
A quadrilateral with two pairs of congruent sides 2008-12-10
From nicole:
what is the name of a figure that has 2 sides that measure 25 feet each and 2 sides that measure 10 feet each?
Answered by Robert Dawson.
The third vertex of a right triangle 2008-12-10
From prashant:
how to calculate coordinates of third vertex in a right angle triangle given the coordinates of hypotenuse as (1,3) and (-4,1)
Answered by Robert Dawson.
A wine decanter 2008-12-10
From Suzanne:
I am trying to buy a wine decanter for my dad for Christmas. One big problem I am having shopping online is finding the right size. Wine is sold by metric ml, with a standard bottle holding 750ml of wine. Decanters are measured by oz, and the sizes are all over the place, with one site alone having sizes 25oz, 26 1/2oz, 50oz, 59 1/8oz, and 68oz.

Can you help me with metric conversions so I can figure out how much wine these decanters hold? Like everybody else, I need this as soon as possible so I can get it shipped in time for Christmas. (I'm disabled & don't drive, so online shopping is my only way to get gifts.)

Thanks for your help!

Suzanne

Answered by Chris Fisher and Robert Dawson.
A quadrilateral with exactly 1 pair of parallel sides and no congruent sides 2008-12-10
From alie:
a quadrilateral with exactly 1 pair of parallel sides and no congruent sides is what?
Answered by Robert Dawson.
Student who is inverting digits 2008-12-09
From Tammie:
Do you have any strategies for addressing inversion of digits in two digit numbers. For example - The student the teacher is working with will write "14" and the number is 41. She has tried highlighting, practice writing numbers, and hundreds charts and finding patterns. Help! Thank you!
Answered by Robert Dawson and Victoria West.
What is one trillion divided by 10? 2008-12-09
From mark:
can you please tell me what one trillion divided by 10 is
Answered by Robert Dawson and Penny Nom.
The centroid and circumcircle of a triangle 2008-12-09
From prateet:
in an equilateral triangle prove that the centroid and centre of the circumcircle coincide

here i am not clear about the concept of centroid and circumcircle i cant understand how AGis 2/3 AD. please help in details about the topic mentioned.

Answered by Harley Weston.
Graphing 2008-12-09
From Natasha:
2x+y=5
Answered by Robert Dawson.
Electrical energy 2008-12-09
From ROHAN:
A lamp consumes 1000J of electrical energy in 10s. what is its power
Answered by Stephen La Rocque.
A line in slope intercept form 2008-12-08
From laurie:
the question: Write and equation of the line that passes through the point and has the give slope. Then rewrite the equation in slope intercept form. (2, 5 ) m= 3
Answered by Penny Nom.
The cost of a foundation 2008-12-08
From carl:
Hi, I need to work out the cost of a foundation.

The foundation is 1m deep x 500mm wide and is 6.3 meters long.

The cost per cubic meter for concrete is 1000 rand escuvation of earth will be 100 rand per cubic meter and back filling wil also be 100 rand per cubic meter.

What will the cost be

Answered by Harley Weston.
The sides of a rectangle 2008-12-08
From Mitchell:
The perimeter of rectangle ABCD (represented on a Cartesian plane) is 52cm. The equation associated with segment AD is 3x+4y-60=0 Point C is at (18, 14). What is the length of segment AD?
Answered by Penny Nom.
Square yards of dirt cover 2008-12-08
From Phillip:
How many square yards will 600 cubic yards of dirt cover, at a depth of 4 inches?
Answered by Harley Weston.
Golf - 24 guys 4-somes 6 rounds 2008-12-07
From Duane:
Thank you. What about 24 guys 4-somes 6 rounds everyone playing with each other at least once.

Thanks again.
Duane

Answered by Victoria West.
Ratio of Volumes of a Cylinder and a Cone 2008-12-06
From rohan:
A CYLINDER IS WITHIN THE CUBE TOUCHING ALL THE VERTICALS FACES . A CONE IS INSIDE THE CYLINDER.IF THEIR HEIGHT ARE SAME WITH SAME BASE ,FIND THE RATIO OF THEIR VOLUMES.
Answered by Janice Cotcher.
Gravity's Effect at a Distance 2008-12-06
From ROHAN:
AT WHAT HEIGHT FROM THE SURFACE OF EARTH,THE WEIGHT OF THE BODY BECOMES HALF? (RADIUS OF THE EARTH = 6400)
Answered by Stephen La Rocque.
Determining the Unknown Numbers 2008-12-06
From jacob:
the sum of two numbers is 45 and their difference is 15. what are the two numbers?
Answered by Janice Cotcher.
Limit of a Trig Function 2008-12-06
From Berta:
Evaluate limit xcsc2x/cos 5x as x goes to 0 ans is 2 but I get 1/2 x/sin2xcos5x = 2x/2sin2xcos5x= 1/2cos5x
Answered by Penny Nom.
Graphing Rational Functions 2008-12-05
From Bahra:
The question in my homework is: Find all vertical, horizontal, and slant asymptotes, x- and y-intercepts, and symetries, and then graph each function. Check your work with a graphing calculator. y=3/x+3 meanng: ( y= 3 over x+3)
Answered by Janice Cotcher.
What is the pre GST price? 2008-12-05
From Chris:
I've been looking at a few answers to the question of before and after GST price calculation. the suggestion have been :: multiply the pre GST price by 1.10 to get the price and reverse to get a pre GST price.

But how do you calculate the pre GST price from an existing retail price?

For example if you take an item that is $150.00 and multiply it by 0.10 you get $15.00 as the GST add on. Apparently giving you $135.00 as the pre GST price. However if you then multiply $135.00 by 1.10 you get the $148.50 as the total.

So the question is how does one calculate the real pre GST price from an existing retail price?

Regards,

Chris

Answered by Penny.
Combinations of Golf Groups 2008-12-05
From Duane:
We have a golf outing coming up with 25 golfers playing 5 rounds in 5-somes. Is there a formula to figure out where everyone can play with different people each round? Thanks, Duane
Answered by Victoria West.
The volume of a yogurt pot 2008-12-04
From Robyn:
Okay, I know you have some answers on converting this already, but I can not convert the yoghurt pot I need yet into ml. Is there some way to explain this using the yoghurt as an example? The pot's 150g of lowfat strawberry yoghurt... would the 'low-fat' have an effect on this? I'm sorry if this is a bother for you to answer, I just don't seem to be able to get it... Have a nice day. - Robyn
Answered by Janice Cotcher and Penny Nom.
6 golfers play threeball 2008-12-04
From Ian:
I have a group of 6 golfers wanting to play 3 rounds as 2x threeballs, but with different players each day. Is this possible? Can you provide some threeball combinations for this please?
Answered by Victoria West.
Decimals and fractions used in our daily lives 2008-12-04
From josh:
i have a projecct due friday and i need to know, how are decimals and fractions used in our daily lives. i am having trouble coming up with ideas. i need seven more that dont involve money or recipes. please help me.
Answered by Harley Weston.
I am a whole number less than 100 2008-12-03
From Cecelia:
If I am a whole number less than 100
and the sum of my digits is 4
and half of me is an odd number
What number am I?
Also am I prime or composite?

Answered by Penny.
The volume of a cone 2008-12-03
From Tamriko:
Hi! Help me, please to solve the following problem: The diameter of an ice-cream cone is 6 cm and the slant height is 10 cm. What volume of ice-cream would fit inside the cone? Thanks!
Answered by Penny.
How many four-legged animals are in the field? 2008-12-02
From evelyn:
Some chickens, cows and sheep are in the field. Their number of legs is 12 more than twice their number of heads. How many four-legged animals are in the field?
Answered by Harley Weston.
How high is the flagpole? 2008-12-02
From michael:
100 m from the base of a flagpole the angle of elevation of the top of the flagpole is 7degree 16'12".how high is the flagpole
Answered by Harley Weston.
The area of an isosceles triangle 2008-12-02
From prateet:
the area of an isosceles triangle is 60 sq cm and one of its equal sides is 13 cm. Find the base of the triangle.
Answered by Penny.
Topsoil on a soccer field 2008-12-02
From rita:
a park foreman decides that a soccer field needs 4 inches of topsoil. soccer field is 100 feet by 250 feet. 1 truckoad is 5 cubic yards. how many truckloads do we need to order. i keep confusing myself with formulas. please explain the steps to me thank you
Answered by Penny.
Fact families with 4 facts 2008-12-01
From Shazia:
Question from Shazia, a parent:

When does a fact family not have 4 related facts? Give an example.

Answered by Harley Weston.
3 equidistant points 2008-12-01
From Damien:
How do you find 3 equidistant points (C,D,E) on a line between point A(Xa, Ya) and point B(Xb, Yb) so that AC, CD, DE and EB are all equal?
Answered by Chris Fisher.
Two tangent circles and common tangents 2008-12-01
From Alan:
Radius of big circle 30cm, radius of small circle 10 cm. From the diagram, the radius from the tangent do not form a semicircle but at an angle. Find the perimeter of the band around both the circle. May need to use trigonometry to find reflex angle AOB, CMD and get the major arc length AB and minor arc length CD
Answered by Penny.
What is so important about quadratics? 2008-11-29
From zoe:
what is so important about quadratics?
Answered by Harley Weston.
Related rates 2008-11-26
From Lyudmyla:
How fast is the volume of a cone increasing when the radius of its base is 2 cm and growing at a rate of 0.4 cm/s, and its height is 5 cm and growing at a rate of 0.1 cm/s?
Answered by Harley Weston.
Dividing Large Numbers 2008-11-26
From Mark:
what is 7.9 trillion divided by 301 million?
Answered by Janice Cotcher.
A log base 2 problem 2008-11-26
From dave:
solve for x

(log base 2 of x) - (log base 2 of (x-2)) = 3

Answered by Penny Nom.
Two tangents to a circle 2008-11-26
From rogerson:
The length of the tangent to a circle is 15 cm. If the angle between the two tangent lines to the circle is 28 degrees, what is the radius of the circle?
Answered by Penny Nom.
Six golfers 2008-11-25
From Joe:
I have a group of 6 golfers wanting to play 3 or 4 rounds but with different players each day if possible. We know we cannot make it happen without some playing with the same person. Can you provide some pairings for this?
Answered by Victoria West.
Primes and polynomials 2008-11-24
From Andrew:
How effective are the two
pairs of polynomials, taken together, at "excluding" any prime p.

1. (a^2 - a.b + b^2) and (a^2 + a.b + b^2) Where b > a > p and (a, b, p) = 1

2. (c^2 - c.a + a^2) and (c^2 - c.b + b^2) Where c > b > a > p and (a, b, c, p) = 1
Answered by Victoria West.

A cereal box that has a volume of 12000cm^3 2008-11-24
From William:
Hi, I have this math question that says construct a ceral box that has a volume of 12000cm3 and the surface area of the box has to be between 3200cm2 and 400cm2, I found some dimensions that are 20 by 20 by 30 and it works, but when I attempt to make the box, or make a net I would need this hugh piece of paper/cardboard, which I don't have, is there an easier way to find different dimenshions so I dont need a hugh piece of paper to create a net easily? Because i've tried trial and error but it has taken to long and I cant find anything,else, thats resonable.
Answered by Harley Weston.
Symmetries of a parallelogram 2008-11-24
From anisha:
how many lines of symmetry does a parallelogram have. A square and a rect are a parallelogram too. so it is confusing?? pls help
Answered by Walter Whiteley.
Quadratic equations arising from practical problems 2008-11-24
From Lenval:
Why do quadratic equations arising from practical problems often give physically impossible solutions such as negative distances, negative times or, say, a polygon with a negative number of sides? For example: "A river in flood carries a tree t-squared - 23t metres downstream in t seconds. How long does it take for the tree to be carried 50 metres downstream?" This gives t=25 and t=-2. Please explain the second 'solution', which is not physically possible.
Answered by Chris Fisher and Victoria West.
How fast is the length of his shadow changing? 2008-11-22
From Desiree:
A spotlight on the ground shines on a wall 12 m away. If a man 2 m tall walks from the spotlight toward the building at a speed of 2.3 m/s, how fast is the length of his shadow on the building decreasing when he is 4 m from the building?
Answered by Harley Weston.
Cubic yards of fill dirt 2008-11-22
From Donnie:
I need to know the formula to figure cubic yds of fill dirt to fill an area from 4ft. deep to 18 inches deep this lot is on a slope and I am wanting to level 176 ft long by 4ft. deep the sides is 115 ft. wide the upper side is 18 inches deep.
Answered by Harley Weston.
Solve 3x +5= 4x-10 2008-11-22
From Nikko:
HELP please how do I solve these type of questions? 3x +5= 4x-10 or 4x +10= 6x-8
Answered by Penny Nom.
There are 30 marbles in a bag 2008-11-21
From Cori:
There are 30 marbles in a bag. Twice as many red, as blue, and 1 more green than there are red. What is the probability that when one is pulled out, it will be red?
Answered by Penny Nom.
Factor 9x^2 + 6x + 4 2008-11-21
From Jonah:
how can i solve this by factoring: 9x^2 + 6x + 4
Answered by Harley Weston.
The distribution of sample sums 2008-11-21
From Mark:
For large samples, the sample sum (Σ x) has an approximately normal distribution.
The mean of the sample sum is n*μ and standard deviation is (σ*√n). The distribution of savings per account for savings and loan institution has a mean equal to $750 and a standard deviation equal to $25. For a sample of 50 such accounts, find the probability that the sum in the 50 accounts exceeds $38,000.

Answered by Penny Nom.
Evaporation of waste water 2008-11-21
From Alexandra:
Engineers know that the amount of water lost due to evaporation is directly proportional to the surface area of the holding tanks and that the local climate causes water to evaporate at the rate of about gallon of water per hour for each square foot of water exposed to the air. About how many total gallons of waste water may enter the three tanks each day to maintain full capacity?
I know that the answer is 50x10^3 but i don't know why.

Answered by Penny Nom.
15 cubic yards of dirt 2008-11-20
From Phillip:
How far, in feet, will 15 cubic yards of dirt cover, if I'm laying it 12 feet wide and 4 inches deep? Please provide formula.
Answered by Penny Nom.
Four circles in a square 2008-11-19
From Anthony:
I have a square where one side measures 10cm and within that square There are four equal quarter circles. Each quarter circles starts in a different corner of the square and I am trying to find area inside the overlap on the quarter circles.
Answered by Janice Cotcher, Chris Fisher and Penny Nom.
The path of a small sailboat 2008-11-19
From jane:
a sailor in a small sailboat encounters shifting winds. she sails 2.00 km East then 3.40 km North East, then an additional distance in an unknown direction. Her final position is 6.68 km directly east of the starting point. find the magnitude & direction of the third leg of the voyage.
Answered by Harley Weston.
Jogging around a circular track 2008-11-19
From kim:
Joan wants to jog 10 miles on a circular track 1/4 miles in diameter. How many times must she circle the track?
Answered by Penny Nom.
evaluate (24 + n) - 6 if n = 4 2008-11-19
From A student:
evaluate (24=n)-6 if n=4
Answered by Penny Nom.
Sigma in a normal distribution 2008-11-18
From Justin:
Suppose the random variable Y can be described by a normal curve with Mu=40. For what value of the standard deviation is P(20 less than or equal to Y less than or equal to 60) = 0.50

-Justin

Answered by Harley Weston.
A normal distribution problem 2008-11-18
From Mark:
Final Averages are typically approximately normally distributed with a mean of 72 and a standard deviation of 12.5. your professor says that the top 8% of the class will receieve an A, the next 20%, a B, the next 42%, a C and the bottom 12%, an F.

a. What average must you exceed to obtain an A?
b. What Average must you exceed to receieve a grade better than a C?
c. What average must you obtain to pass the course? (you'll need a D or better)

Answered by Harley Weston.
An algebraic exercise with fractions and radicals 2008-11-18
From Rubén:
Question from Rubén, a teacher:

I have the ecuation:

U = [yb / (ab+a^2)]^1/2 + [ya / (ab+b^2)]^1/2

I know for sure this reduces to

U = [y (1/a + 1/b)]^1/2

but I cannot find a way to get into that result!

Thanks

Answered by Rubén Osuna.
Juanita and Jim each think of a number. 2008-11-18
From Maria:
Juanita and Jim each think of a number. Juanita's number is 8 more than Jim's number. The product of the two numbers is 65. What is Jim's number?
Answered by Penny Nom.
Insulation in an attic 2008-11-18
From Scott:
A homeowner wishes to insulate her attic with fiberglass insulation to conserve energy. The insulation comes in 40-cm-wide rolls that are cut to fit between the rafters in the attic. If the roof is 6 m from peak to eave and the attic space is 2 m high at the peak, how long does each of the pieces of insulation need to be? Round to the nearest tenth.
Answered by Harley Weston.
How do I compare fractions and decimals? 2008-11-18
From Alexis:
How do I compare fractions and decimals ?
Answered by Penny Nom.
A circle and a chord 2008-11-17
From Lydz:
The circle has a radius of 8cm The chord is 13cm long. Find the distance from the centre of the circle, to the chord.
Answered by Penny Nom.
How do you solve " (x-4)^3/2 = -6 "? 2008-11-17
From Jim:
How do you solve " (x-4)^3/2 = -6 " ?
Answered by Penny Nom.
The volume is decreased by 25% 2008-11-17
From Wanda:
If the volume of a sphere is decreased by 25%, what is the new equation?
Answered by Penny Nom.
The height, base and perimeter of a triangle 2008-11-17
From Zack:
How would i calculate the height of a triangle if the base is 10cm and the area is 50cm²? If possible how would i calculate the length around the the triangle.
Answered by Penny Nom.
How do you solve "logx^4=4? 2008-11-17
From Scott:
How do you solve "logx^4=4"?
Answered by Penny Nom.
License plates 2008-11-17
From Clayton:
Okay, so in this situation a license plate must have four letters (A-Z) and four numbers (0-9), ex. ABCD-1234, where repeating a letter or number is allowed ex. AAAA-1234, or ABCD-1111, or AAAA-1111. The order of letters first numbers second, or number first, letters second is allowable, and each state has its own plates, so ABCD-1234 from New York, and ABCD-1234 from Minnesota are considered different combinations, how many different license plates could there be?
Answered by Penny Nom.
A triangle inscribed in a circle 2008-11-17
From Wanda:
I have the same question that you guys answered 2007-03-02. I need more clarification. I UNDERSTAND how to get the radius=3, I get that it is an equilateral triangle so each vertex is 60 degrees, I get that area of triangle is 1/2 bh. I DO NOT understand why we multiply area X 3 , or how to calculate the values of base and height. Please explain a little further. Thanks.
Wanda

Answered by Penny Nom.
A question about primes and polynomials 2008-11-16
From Andrew:
Is it possible for (a^2 - a.b + b^2) to be divisible by a prime p where b > a > p and a, b, and p are relatively prime? I let a = n.p + x and b = m.p + y (where both x and y are < p) and after dividing throughout by p one gets to decide whether (x^2 - x.y + y^2) ==0 mod(p) ?! It seems unlikely unless p = (x^2 - x.y + y^2)? (Note:- All letters represent positive integers.)
Answered by Chris Fisher and Victoria West.
A word problem about a regular pentagon 2008-11-16
From Sandra:
If the area of a regular pentagon is A=1.720a^2, in which a is one of the sides. Find the area of a regular pentagon with a side that measures 50cm.
Answered by Chris Fisher and Penny Nom.
A game for 5 players 2008-11-16
From safdar:
five players agreed that after every game, the loser doubles every body's money. they played 5 games and everybody lost once. after the 5 games, everybody had 4128. How much money did each of the players have before the game? show your work.
Answered by Janice Cotcher and Harley Weston.
Probability and a pill bottle 2008-11-15
From Dana:
If a pill bottle contains either a clonazepam or clonidine pill and then a clonidine pill is added & the botttle is shaken up and a clonidine pill is "dumped" out, what are the chances a clonazepam pill remains in the bottle?
Answered by Stephen La Rocque.
Two cubic yards, 4 inches deep 2008-11-15
From Donna:
How many square feet will 2 cubic yards cover if the depth is 4 inches?
Answered by Stephen La Rocque.
The midpoint o a line segment 2008-11-15
From Jane:
The vertices of a triangle are at (1,7), (6,-1) and (0,3). Find the coordinates of the midpoints of the sides.
Answered by Penny Nom.
The diagonals of an isosceles trapezoid 2008-11-14
From hazel:
how to solve the diagonals of an isosceles trapezoid? what is the formula?
Answered by Harley Weston.
A ratio to a decimal 2008-11-14
From manny:
Express 22 : 5 as a decimal. Round to the nearest hundredth
Answered by Penny Nom.
Choosing light bulbs from a box 2008-11-14
From Annie:
A box of light bulbs contains 3 bulbs, two bulbs are defective.

What is the probability of choosing a non-defective bulb and a defective bulb in 2 draws?
What is the probability of choosing 3 defective bulbs followed by a non defective bulb?
What is the probability of choosing first 2 non defective bulbs followed by a defective bulb?

Answered by Penny Nom.
Multiplication 2008-11-14
From natalie:
what is 8 times 30
Answered by Penny Nom.
How far are the boats apart? 2008-11-14
From dom:
Two boats leave port at the same time. They leave at 150 degree angle. One boat travels at 10mph and the other at 20mph. After two hours how far are the boats apart?
Answered by Penny Nom.
Confusion in a multiple choice question 2008-11-14
From BJ:
My son got this math problem which he could mostly solve. Here it is:

The highest location in a certain country is 4525 m above sea level. The lowest point in the same country is 192 m below sea level.

a) Find the difference of the two elevations. His answer: a-(-b)= a+b or 4525-(-192)= 4525+192=4717 m. No problem.

b) A city is 2221 m above sea level. Is this elevation closer to the highest point or the lowest point? His answer: highest point because: 4525-2221= 2304 m (closest to the highest point) and 2221-(-192)=2221+192= 2413 m. (farthest to the lowest point). OK so far.

But then he was given 4 choices for this question with no other information:

a) 4717 m; lowest b) 4333 m; lowest c) 4333 m; highest d) 4717 m; highest

What does it mean? What's the connection with the rest of the problem?

Answered by Harley Weston.
A cross-sectional area 2008-11-13
From David:
I am doing a science project examining the tensile strength of seaweed fronds. I need to calculate the cross-sectional area from the major and minor diameters of the elliptical central axis of the seaweed frond. What formula would I use to get this. The strength of the segments is expressed in terms of cross-sectional area of the frond at the test site.
Answered by Penny Nom.
What is -5x+3y=24? 2008-11-13
From Robert:
What is -5x+3y=24???????
Answered by Stephen La Rocque.
The angles and sides of a triangle 2008-11-13
From JAMIE:
a triangle with a side(b)37m an angle(C)70degrees and (a)79m find values of angles A and B and length of side c
Answered by Stephen La Rocque.
A circle and a chord 2008-11-13
From jane:
the center of a circle is at (-3,2) and its radius is 7. find the length of the chord, which is bisected at (3,1).
Answered by Penny Nom.
This question needs to be thrown away 2008-11-13
From ang:
I'm a friend trying to help a friend in lower algebra. I feel like this question needs to be thrown away. It's so ill written.
Find an mathematical expression/equation: Half of the quotient of the total of a number and negative six and a number less negative six is the ratio of a number to a number more than negative seventeen

Answered by Stephen La Rocque and Harley Weston.
An odd shaped lot 2008-11-13
From Janice:
Hi I am attaching a survey of an odd shaped real estate lot. I am hoping you can help me determine the acreage.

Janice

Answered by Harley Weston.
How do you work out 2/17 as a decimal? 2008-11-13
From p:
how do you work out 2/17 as a decimal?
Answered by Penny Nom.
The volume of a cyclone 2008-11-13
From Dianna:
I need to figure the cubic yards of a cyclone. 144" down to 48" 60" high
Answered by Penny Nom.
An equation with three radicals 2008-11-13
From Crystal:
I need to solve this radical equation but i am confused on how to solve it with three square roots in it. sqrt(3x)+3-sqrt(2x-3)=sqrt(3x-2)
Answered by Harley Weston.
Real and imaginary zeros 2008-11-12
From David:
Find all the real and imaginary zeros for each polynomial. Factor each polynomial. Leave factors with imaginary zeros in quadratic form.

h(x)= x^5 +2x^4 - 10x^3 -20x^2 +9x + 18

Answered by Harley Weston.
A conical funnel 2008-11-12
From Rachael:
Hello, I am a 10th grader in AP Calc, and can not figure out this question: Water is running out of a conical funnel at the rate of 1 inch^3/sec. If the radius of the base of the funnel is 4 in. and the altitude is 8 in., find the rate at which the water level is dropping when it is 2 in. from the top.
Answered by Harley Weston.
Equations with fractional powers and roots 2008-11-12
From David:
Find all real solution to each equation.

x^-2/3 = 9 how do you do this if its to the power

x + 1= Square root (3x +13) can you explain how to get an solution with square root?

Answered by Penny Nom.
Mary and Jane on a halfmile track 2008-11-10
From alan:
mary is walking 15mph on a half mile track. jane is walking 12mph on a halfmile track. how much time will it take mary to pass jane.? HOW MANY TIMES WILL MARY PASS JANE IN 1 HOUR?
Answered by Chris Fisher and Penny Nom.
Supplementary angles 2008-11-10
From Anna:
<A and <B are supplementary angles. Twice the measure of <B is one-fourth the measure of <A. Find the measure of both angles.
Answered by Penny Nom.
Liquid in a drum 2008-11-10
From RAY:
How can I determine the exact amount of liquid in an overturned drum?
Answered by Penny Nom and Victoria West.
How fast is the distance between the airplanes decreasing? 2008-11-10
From Crystal:
At a certain instant, airplane A is flying a level course at 500 mph. At the same time, airplane B is straight above airplane A and flying at the rate of 700 mph. On a course that intercepts A's course at a point C that is 4 miles from B and 2 miles from A. At the instant in question, how fast is the distance between the airplanes decreasing?
Answered by Harley Weston.
The reciprocal of a mixed number 2008-11-08
From Trini:
Can the reciprocal of a mixed number be another mixed number? Explain, and give an example.
Answered by Penny Nom.
A flagpole and a yardstick 2008-11-07
From Wanda:
One boy holds a yardstick vertically at a point 40 feet from the base of the flagpole. The other boy backs away from the pole to a point where he sights the top of the pole over the top of the yardstick. If his position is 1 ft 9 in from the yardstick and his eye level is 2 ft above the ground, find the height of the flagpole.
Answered by Penny Nom.
A repeating decimal 2008-11-07
From mike:
what is 0.0028282828 recurring as a fraction?
Answered by Penny Nom.
Algebraic equations with fractions 2008-11-07
From John:
Solve each equation. Check each solution.
1/8+5x/x+2=5/2
10/2y+8 - 7y+8/y(squared)-16 = -8/2y-8

Answered by Harley Weston.
Solve the equation 2x + y = 6 for y. 2008-11-07
From fatiuma:
Solve the equation 2x + y = 6 for y.
Answered by Penny Nom.
The dimensions of a rectangle 2008-11-07
From laela:
A rectangle is 8 feet long and 6 feet wide.If the dimension is increased by the same number of feet,the area of the new rectangle formed is 32 square feet more than the area of the original rectangle.By how many feet was the each dimension increased?
Answered by Penny Nom.
Purchasing pens and pencils 2008-11-07
From Maggie:
A mechanical pencil costs 50 cents and a pen costs 25 cents. If I spend 5.00 on 17 pens and pencils, how many of each does that buy?
Answered by Penny Nom.
Compatible numbers 2008-11-07
From Erika:
When working with compatible numbers, when do you choose to round up or down? For example: In the problem: 373 divided by 4, one could round 373 to 400 and keep the 4 the same, and the answer would be 100; OR, one could round 373 down to 360 and keep the 4 the same, and the answer would be 90. When given the problem as an assignment, which way dictates how one would round, up or down?
Answered by Penny Nom.
A 4 digit number 2008-11-06
From Chris:
numbers 0 - 9 form a 4 digit number with no repetition.

It would round to 3000 if rounded to the nearest thousand

The digit in the tens place is twice the value of the digit in the thousands place.

It is an odd number.

It contains the largest digit.

It contains one digit less than 2.

The digit in the hundreds place is odd.

Can you find the 2 numbers?

Answered by Penny Nom.
The height of an equilateral triangle 2008-11-06
From touqeer:
My question is that how can we find the height of an equilateral triangle without using pythagoras theorem?
Answered by Penny Nom.
The cost of a steel plate 2008-11-05
From Tammie:
What is the formula to calculate the following:

I have a plate of steel that is 1/4" x 48" x 192" at a price of $15.23 per square foot What is the formula to calculate the cost if I only use 1/4" x 47-1/4" x 68-1/8" of that steel plate?

Answered by Penny Nom.
A trig limit 2008-11-04
From Teri:
Although I have this problem completely worked out in front of me I still cannot understand how it was done. The problem is:
Find the limit.
lim x->0 sin2x/tan7x.

Answered by Harley Weston.
A rectangle is twice as long as it is wide. 2008-11-04
From Nawal:
A rectangle is twice as long as it is wide.
If both its dimensions are increased by 4 m, its area is increased by 88 m ^ 2.
Find the dimensions of the original rectangle.

Answered by Penny Nom.
Separating variables 2008-11-04
From Terry:
by separating variables solve the initial value problem

(x+1)y' + y = 0 y(0) = 1

Answered by Harley Weston.
Absolute value 2008-11-04
From Shane:
What is the absolute value of -4
Answered by Penny Nom.
The volume of a pond 2008-11-04
From Malcolm:
I am looking for a formula to find the volume of my pond. I know the average surface diameter and the average bottom arc length.(The arc from one bank through a point in the bottom and on to the surface on the other bank. I am not able to get a good measurement of the depth at this time. Can the volume be figured with the known measurements (the cord and the arc)? Thanks: MS
Answered by Harley Weston.
Partial variation 2008-11-03
From Diamond:
what are the 2 parts of partial variation?
Answered by Penny Nom.
A belt drives a pulley 2008-11-03
From Rebecca:
A point on a belt is moving at the rate of 30 ft/sec and the belt drives a pulley at the rate of 200 rpm. What is the radius of the pulley?
Answered by Penny Nom.
An arch is in the form of a semi ellipse 2008-11-03
From jessica:
An arch in the form of a semi ellipse has a span of 10 meters and a central height of 4 m. Find the heights of the arch at a point of 3 meters from the semi minor axis.
Answered by Penny Nom.
Three consecutive odd integers 2008-11-03
From kim:
Determine three consecutive odd integers such that twice the sum of the second and third is 29 less than 5 times the first.
Answered by Penny Nom.
The part before the repetend 2008-11-02
From Puerto:
Hello, I teach mathematics in a bilingual programme using English. I have learned that the group of digits that repeat in a repeating decimal is called repetend but i need to know how are called the previous ones when the decimal is a delayed-repeating one. I mean, in 0.34555555 5 is the repetend, the period is one and...how are called 34? Thanks in advance.
Answered by Stephen La Rocque and Penny Nom.
The prime factorization of one billion 2008-11-02
From Alta:
The prime factorization of 1000 is 2 cubed times 5 cubed. How do you write the prime factorization of one billion using exponents?
Answered by Penny Nom.
A circle tangent to a line and with its centre on another line 2008-11-01
From liza:
Find the equation of the circle of radius squareroot 26 tangent to the line 5x+y=13 and having its center on the line 3x+y+7=0.
Answered by Chris Fisher.
The diameter of a circle 2008-11-01
From lupito:
a circle has a 30`sector,33pi.find the diameter of the circle.How do I set the equation up.
Answered by Victoria West.
How far does the ball travel? 2008-11-01
From Betty:
This is a question that is perplexing me. I tried to solve it with the Pythagorean Theorem but have not been able to get the right answer. A ball attached to the moving end of the 5-meter arm of a pendulum. The pendulum swings through a 90 degree arc once. Approximately how far, in meters, does the ball travel?
Answered by Victoria West.
The angles of a triangle 2008-10-31
From kyla:
an isosceles triangle has two 50 degrees angles.what is the measure of the third angle? explain how you found your answer.
Answered by Penny Nom.
Compatible numbers 2008-10-30
From Paul:
Use compatible numbers to estimate the quotient of 23.52 and 11.04
Answered by Penny Nom.
A bay window 2008-10-30
From Scott:
Given the length of an arc, the rise of an arc, and the number of segments that I need to divide the arc into - how do I determine the length of each segment? Imagine a bay window. It could have 5 side or 7 sides, just as an example. How do I determine the width of each window given that the unit will be mounted into a frame with a 96 inch opening. The rise of the unit will extend out 18 inches. Lets say that the number of single windows unit within the unit is 5. How wide should each window be? Do you understand?
Answered by Harley Weston.
What are the equivalent to 0.68? 2008-10-28
From clay:
what are the equivalent to 0.68 ?
Answered by Penny Nom.
Equations with the variable in the exponent 2008-10-28
From fari:
Solve for x

5 * 4^x-3=40
5^x=25^(x+8)

Answered by Penny Nom.
A homework problem 2008-10-28
From Shawn:
I'm checking my son's homework and we disagree on the solution. The problem reads: 135mn^4 (n to the 4th power) over 50n^2 (n squared) I think the answer is: 27mn^2 (squared) over 10 Can you help?
Answered by Penny Nom.
A quadratic 2008-10-27
From Giselle:
3x squared minus 2x equals -5
Answered by Penny Nom.
Nickels, dimes and quarters 2008-10-27
From val:
I would like help working out a math word problem. A collection of 30 coins worth $5.50 consists of nickels, dimes, and quarters. There are twice as many dimes as nickels. How many quarters. Could you teach me how to find this formula?
Answered by Penny Nom.
Three congruent rectangles 2008-10-27
From Meagan:
Here is my problem: Three congruent (non-square) rectangles are placed to form a larger rectangle. [Two are oriented the same way and the "stacked" while the third is rotated 90 degrees and placed next to the other two.] The total area is 1350 square cetimeters. Square ABCD is then created that has the same perimeter as the large rectangle that was created. E is the midpoint of line CD and F is the midpoint of BC. Find the area of triangle AEF.
Answered by Penny Nom.
Tangent line 2008-10-27
From Maddie:
Find a general equation for a line that touches, but does not pass through the function Y=X2(X squared) + BX+C.
Answered by Stephen La Rocque and Victoria West.
The length of a rectangle is 6 inches less than 3 times its width. 2008-10-26
From Angela:
The length of a rectangle is 6 inches less than 3 times its width. Find the dimensions of the rectangle if its area is 45 square inches.
Answered by Penny Nom.
Two tangent lines to a parabola 2008-10-26
From Marcus:
Show that the tangent lines to the parabola y = ax^2 + bx + c at any two points with x-coordinates p and q must intersect at a point whose x-coordinate is halfway between p and q.
Answered by Penny Nom.
The height of a tank 2008-10-26
From Kyle:
If a water tank holds 750 cubic meters of water and it is 15 metres long and 10 metres wide how many meters tall is it ?
Answered by Penny Nom.
The amount of diesel fuel in a tank 2008-10-26
From Mike:
I have an cylinder shaped oil tank that holds approx. 283 gals of diesel fuel. The tank is laying on on its side, (like taking a garbage can and laying it down on its side). What is the formula to calculate how much oil is in the tank at any given time. If the tank was sitting on end it would be easy to figure out, but it is not.
Answered by Stephen La Rocque.
An irregular, truncated pyramid 2008-10-24
From phillip:
Hi, I have a rectangular base to square top, pyramid shape to fabricate out of steel plate (4 sections). The base is 3000mm by 800mm, the height is 1100mm and the top is 350mm square. What I need to know is the lengths of the sides of the plate(were they meet vertically). Hope ive made it clear enough. Thanks Phil.
Answered by Stephen La Rocque.
Water is leaking from a conical tank 2008-10-24
From Kimberly:
Water is leaking out of an inverted conical tank at a rate of 12000 cm3/min at the same time that water is being pumped into the tank at a constant rate. The tank has height 6 m and the diameter at the top is 4 m. If the water level is rising at a rate of 20 cm/min when the height of the water is 2 m, find the rate at which water is being pumped into the tank.
Answered by Stephen La Rocque.
Put in lowest terms 6a^2c/8ab 2008-10-24
From hana:
put in lowest terms 6a^2c/8ab
Answered by Stephen La Rocque.
Polygons with two names 2008-10-24
From Connor:
Why do some polygons have two names and what are they?. Thanks
Answered by Harley Weston.
Taxes in Taxylvania 2008-10-22
From April:
Taxylvania has a tax code that rewards charitable giving. If a person gives p% of his income to charity, that person pays (35-1.8p)% tax on the remaining money. For example, if a person gives 10% of his income to charity, he pays 17 % tax on the remaining money. If a person gives 19.44% of his income to charity, he pays no tax on the remaining money. A person does not receive a tax refund if he gives more than 19.44% of his income to charity. Count Taxula earns $27,000. What percentage of his income should he give to charity to maximize the money he has after taxes and charitable giving?
Answered by Harley Weston.
Antiderivative of 1/(x(1 - x)) 2008-10-22
From Matt:
derivative of dx/(x(1-x))

From what I've seen I should break apart the equation as such derivative of dx/x - dx/(1-x) and then get the 2 corresponding log functions.

If that is correct why does this factoring work, if that is incorrect what is the proper way to find the derivative.

Answered by Harley Weston.
Trinomial Distribution 2008-10-22
From Darya:
show that (x^2-x+3)(2x^2-3x-9) = Ax^4+Bx^3+C where A,B and C are constants to be found.
Answered by Janice Cotcher.
A linear system 2008-10-22
From Tiffany:
solve:

x + y + z = 4
2x - 3y - z = 1
4x + y - 2z = 16

Answered by Penny Nom.
nth term 2008-10-22
From jen:
what is a nth term and how do i find the nth term of any sequence
Answered by Penny Nom.
Area of a triangle 2008-10-22
From Jia:
I need to find 3 ways of finding the area of a triangle. I've already had one method down which is b x h and divided by 2. What other 2 ways can i find the area of a triangle?
Answered by Chris Fisher.
The chairs on a ski lift 2008-10-21
From Travis:
If you are sitting on chair 33 of a ski lift, which is a loop. Chairs are numbered consecutively, starting with 1. Chair 97 passes you halfway up the ski slope. How many chairs are on the lift?
Answered by Penny Nom.
The sum of the reciprocals 2008-10-21
From garry:
the sum of two different positive numbers is 75 while the product of the same two numbers is 25. Fine the sum of the reciprocals of these two numbers
Answered by Penny Nom.
Exponential form 2008-10-21
From layla:
how we wite 256 in exponential form
Answered by Harley Weston.
What number has 5 hundreds and 12 tens? 2008-10-21
From kellee:
What number has 5 hundreds and 12 tens?

I would say the number is 620. But, my daughter thinks that is wrong.

Answered by Harley Weston.
Scaling a logo 2008-10-20
From Carl:
Hello! I hope that you can help me out here. I've been trying to figure this out for awhile now and can't come up with the answer logically. I am a graphic designer by trade and need to figure out how large my client's logo should be. The logo will need to be sized proportionally with the surface area of the page. The logo will always be sized at 1.2938(w) x .75(h) on a letter size page (8.5 x 11).
My question is...
When you increase the size of the page (not always proportionally sized - dimensionally) how do figure out the NEW size of the logo. I could have a page 20" x 40" or 30' x 4'... or ANY size really. I believe I have the new surface area of the logo but can't find the new dimensions.

Answered by Harley Weston.
Melting ice on a hemisphere 2008-10-20
From heather:
The top of a silo is the shape of a hemishere of diameter 20 ft. if it is coated uniformly with a layer of ice, and if the thickness is decreasing at a rate of 1/4 in/hr, how fast is the volume of ice changing when the ice is 2 inches thick?
Answered by Penny Nom.
The ratio of two chemicals 2008-10-20
From Nathan:
if there is a chemical mixture of 5:2 and the total amount of the entire solution is 1400 gallons, how many gallons make up the stronger side of the solution? I believe the answer is 1,000 but I figured it out in a non-traditional way so I wanted to understand the basic formula used to determine these types of questions. thank you
Answered by Penny Nom.
I have 6 vertices and 10 edges 2008-10-20
From Jacquelin:
I have 6 vertices and 10 edges. One of my faces is a regular polygon.
Answered by Chris Fisher.
How much does Luc pay for his tools?? 2008-10-20
From imran:
Luc makes $1 profit on each tool he sells. Marc makes a $2 profit while paying $5 less for his tools than Luc. Marc thus makes $6 more profit for every $100 invested. How much does Luc pay for his tools??
Answered by Penny Nom.
A canal with cross section a semicircle 2008-10-19
From Connor:
a canal with cross section a semicircle is 10m deep at the centre. Find an equation for the semicircle and use it to find the depth 4m from the edge
Answered by Penny Nom.
A falling block 2008-10-18
From KOBINA:
A 50 kg block being held at rest 20m above the ground is released. the block falls (no friction). how fast is the block traveling (in m/sec) when it has lost 40% of its original potential energy.
Answered by Stephen La Rocque.
The slope of a tangent line 2008-10-18
From Amanda:
If f(x)=square root of (x+4), and the slope of the tangent line at x=21 was 1/n for some integer n, then what would you expect n to be?
Answered by Stephen La Rocque.
Two equations in two unknowns 2008-10-17
From Dushayne:
Please help me in solving this problem:
a. 3x-4y=32
5x+2y=10

b. 2x+3y=11
4x+3y=10

Answered by Penny Nom.
A geometric construction 2008-10-17
From M:
Given any 3 parallel lines on a plane, how to construct an equilateral triangle with each vertex on each line?
Answered by Chris Fisher.
Using the augmented matrix 2008-10-17
From Karlena:
I am supposed to write the augmented matrix of the system and use the matrix method to solve the system. I must show my work algebraically

x+y+2z=30
2x+3y+2z=53
x+2y+3z=47

Answered by Harley Weston.
A football field covered in garbage 2008-10-17
From AMie:
a small city produces 500,000 cubic ft of garbage per week. If all of this garbage were stacked neatly (in a nice vertical pile) on a 100-yard by 60-yard football field, how high would the pile be (in feet)?
Answered by Penny Nom.
A tangent line to f(x) = 1/(x - 1) 2008-10-16
From Amanda:
If f(x)=1/(x-1) then what is the slope of the tangent line at x=-2?
Answered by Harley Weston.
The area of a rug 2008-10-16
From Natcha:
How can I convert the Carpet Roll Size to Square Meters. I have one Roll of carpet, (Quality is 2300 g/m2; 7 pitch/in, 9 rows/in; width: 2m) with dimension 0.5 m width and 4 m length. I needed to know that quantity of that Roll in Square Meters.
Answered by Penny Nom.
A roll of film 2008-10-16
From John:
I need to know how to calculate the build up on a roll of film is calculated. Example: I start with a 6" diameter core, and I start winding .005" thick film on the core, so I am adding a total of .010" to the diameter each wrap. If I continue to do this for a total length of film of 3000 feet, what will the roll diameter be? So what I need is the formula to perform this type of calculation. Can you help me? I want to be able to plug the formula in a spread sheet and to be able to input a core diameter, a film thickness and a total length and get a roll diameter.
Answered by Penny Nom.
Related rates 2008-10-16
From Gisela:
As sand leaks out of a hole in a container, it forms a conical pile whose altitude is always the same as its radius. If the height of the pile is increasing at a rate of 6 in/min, find the rate at which the sand is leaking out when the altitude is 10in.
Answered by Penny Nom.
The dimensions of a rectangle 2008-10-16
From cristine:
the area of a rectangle is 675sq.cm. if the width is 1/3 of the length. find the dimensions of the rectangle....
Answered by Penny Nom.
Perpendicular lines 2008-10-16
From Hailey:
In the xy-plane, the line with equation 2x + y = 3 is perpendicular to the line with equation y = mx + b, where m and b are constants. What is the value of m?
Answered by Penny Nom.
The volume of a slice from a wedge 2008-10-16
From Jeff:
If I have a wedge shaped object (ie, a triangle of non-specific type extruded along a length) and it is sliced from the two-corner base at one end to the one-point tip at the other, what is the volume of the upper (smaller) part as a percentage of the whole? I hope I have explained it clearly enough. Thanks.
Answered by Harley Weston.
The rate of change of the volume of a cone 2008-10-15
From Barbara:
Suppose that both the radius r and height h of a circular cone change at a rate of 2 cm/s. How fast is the volume of the cone increasing when r = 10 and h = 20?
Answered by Harley Weston.
Percentage of an hour 2008-10-15
From JESSICA:
HI I AM IN GRADE 7 AND FOR MATH HOMEWORK MY TEACHER ASKED US TO MAKE 8 DIFFERENT TIMES(clocks) AND TELL HIM WHAT THE PERCENTAGE FOR THE TIME IS. BUT I DON'T KNOW HOW TO CONVERT TIME TO A PERCENT. PLEASE HELP ME??!!??

JESSICA SEVENTH GRADE

Answered by Penny Nom.
Concavity and the second derivative 2008-10-15
From Christina:
I'm having trouble solving for a second derivative for the following graphing question.

f(x) = (X^2+2x+4)/2x

using the quotient rule, I found:
f'(x) = (x^2-4)/(2x^2)

however, using the quotient rule again I can't seem to solve it (concavity):
f'''(x)=[(2x)(2x^2)-(x^2-4)(4x)]/[(2x^2)^2]
f''(x)=[(4x^3-(4x^3 -16x)]/4x^4
f''(x)=16x/4x^4
f''(x)=4/x^3

and making the equation equal to zero result in 0=4 which doesn't seem to make sense...

Answered by Penny Nom.
Millimeters to inches 2008-10-15
From diane:
can you convert milliliters to inches?
Answered by Penny Nom.
The sides of an octagon 2008-10-12
From side lengthsGarry:
I need the length of the sides and the degrees of the angles for a 30 inch diameter octagon
Answered by Janice Cotcher.
A normal to a curve 2008-10-11
From sundar:
How do I find a normal to a curve defined by equation y = a*x^3+b*x^2+c*x+d
Answered by Penny Nom.
The volume of fluid in a semi circular trough 2008-10-10
From Kerry:
How do I find the volume of fluid in a semi circular trough?
Answered by Penny Nom.
A cone shaped pile 2008-10-10
From Nadine:
I have a pile of wheat in the shape of a cone. I would like to know how much wheat I have. I have found the equation "V=1/3 pie r squared h" , but it dosn't work! The pile is 7 feet high, diameter is 50 feet, circumference is 185 feet. I also measured the slope 19feet. (Probably not needed) You need to know that wheat weighs 60 pounds per bushel, and I would like to know how many bushels I have. Even if I could receive the # of Volume Bushels, I could convert that. Can you help me??
Answered by Stephen La Rocque.
Linear and Angular Velocity 2008-10-10
From Matthew:
The pedal and gear relationship of a bicycle is shown. The radii of the gears are r(sub1) = 5 cm and r(sub2)= 12 cm. The radius of the wheel is r(sub3) = 30 cm. How many rotations per minute of the pedal gear will produce a racing cyclist's speed of 60 km per hr. For a diagram: http://bikepedalsgears.weebly.com/
Answered by Stephen La Rocque.
Two inequalities 2008-10-08
From s:
3y > 4x
2x-3y>-6

Answered by Stephen La Rocque.
Concrete around a pipe 2008-10-08
From mick:
i am trying to find the area of concrete to be placed around a concrete pipe! Dimensions can be 12m wide by 5m wide by 7m deep and pipe been 8m by 600mm radius...
Answered by Stephen La Rocque.
Two modular equations 2008-10-08
From Mhiko:
please solve this Chinese remainder problem..and give me a solution or rule in order to solve this problem/

x=2mod15
x=1mod25

Answered by Stephen La Rocque.
Maximize revenue 2008-10-08
From Donna:
A university is trying to determine what price to charge for football tickets. At a price of 6.oo/ticket it averages 70000 people per game. For every 1.oo increase in price, it loses 10000 people from the average attendance. Each person on average spends 1.5o on concessions. What ticket price should be charged in order to maximize revenue. price = 6+x, x is the number of increases.
ticket sales = 70000- 10000x
concession revenue 1.5(70000 - 10000x)
I just do not know what to do with the concession part of this equation (6+x) x (70000 - 10000x) I can understand but not the concession part please help. thx.

Answered by Penny Nom.
The dimensions of a pool 2008-10-08
From Justine:
This may be a really silly question but I am trying to work out what the possible dimension a pool may be if we have 20m length of wall of an above ground pool (laid out flat on ground)

We have been given a second hand above ground pool and we know it was in an oval configuration and we are just trying to get a guesstimate of the potential size it may be as the people couldn't tell us the size

Answered by Penny Nom.
A linear equation with fractions 2008-10-07
From Christopher:
1/6(x-18)+1/8(x+8)=x-9
Answered by Penny Nom.
The rise over the run 2008-10-07
From Mak:
what is the ratio of the rise to run?
Answered by Penny Nom.
A double discount 2008-10-06
From Joanna:
An instrument store gives a 10% discount to all students off the original cost of an instrument. During a back to school sale an additional 15% is taken off the discounted price. Julie, a student at the local high school, purchases a flute for $306. How much did it originally cost?
Answered by Penny Nom.
Area of a semicircle 2008-10-06
From Benjamin:
How do you find the area of a semi-circle if the diameter is a variable.
Answered by Penny Nom.
Octagon angles 2008-10-06
From John:
My daughter wants to build an octagon using PVC pipe fittings. There are several pipe "elbows" available, including 45 degree. I'm trying to determine if if an octagon can be constructed by using 8 pairs of 45 degree elbows, connected to each other by 8 sections of straight pipe of some equal legnth. This makes sense to me intiutively, but I do not know how this can be properly expressed in geometric terms.
Answered by Chris Fisher, Victoria West and Harley Weston.
The average rate of change of gasoline used 2008-10-06
From JHulie:
What is the average rate of change of gasoline used, measured in miles per gallons if you travel 212 miles, then you fill your gas tank up again and it takes 10.8 gallons. If you designate your change in distance as 212 miles and your change in gallons as 10.8?
Answered by Penny Nom.
equivalent fractions 2008-10-06
From marcella:
a fraction equivalent to 16/20
Answered by Harley Weston.
Logs and exponentials 2008-10-06
From Charles:
I seem to remember you could solve exponential problems using natural logs quite simply without calculators, such as the fifth root of 400 or 33 to the 4th power. All you did was convert the number to its natural log and then just divide by 5 or multiply by 4 and then convert it back. I just remember how to do this. Thanks.
Answered by Harley Weston.
The height of a right triangle 2008-10-06
From David:
Could you please tell me the height of a right angle triangle when the base measurement is 4930mm and the base/hypotenuse angle is 10 degrees. this will help a lot.
Answered by Harley Weston.
Algebraic fractions 2008-10-06
From Kayla:
(x^2-9)/x times (x^3-4x)/(x^2+5x+6)
Answered by Harley Weston.
An arc and an angle 2008-10-05
From Cory:
I have a chord length of 150'. From the left starting point, I know that 30' right of starting point is 9' in height. This would be the top of the arc.

What is the arc in degrees and can anyone display an image to help me understand?

Answered by Penny Nom.
The break even point 2008-10-03
From dina:
virgin record company has just signed a new artist to its label. They have pais the new group 25000 dollars to record their album and paid each band member 10000 dollars each. The band is not given a commission on each cd sold or downloaded, therefore, each cd sold is profit earned by virgin. How many cd's would they have to sell to break even if this new band has 5 members? Each cd is sold at the price of 20$.
Answered by Penny Nom.
A different approach to a word problem 2008-10-03
From Kenneth:
Sarah's age is 2/3 of Mary's age and 3/4 of Ruth's age. The sum of their ages is 46 years. How old is each?
Answered by Penny Nom.
Finding the Distance Between Two Latitudes 2008-10-02
From Samua:
Assuming that the Earth is a sphere of radius 4000 miles and that the cities are on the same longitude (one city is due north to the other). Find the distance between the cities with the latitudes of 37 degrees 47'36'' and another city with 47 degrees 37'18''. Heeeeeeeeeeelp!
Answered by Janice Cotcher.
Compatible numbers 2008-10-02
From jeff:
what exactly are compatible numbers, math problem is:
Estimate the total weight of 2 boxes that weigh 9.4 pounds and 62,6 pounds using rounding and compatible numbers. Which estimate is closer to the actual weight and why?

Answered by Harley Weston.
4^x = 2^x + 12 solve for x 2008-10-02
From Dave:
4^x = 2^x + 12 solve for x.
Answered by Harley Weston.
Find an All Odd 4 - Digit Number 2008-10-02
From Denise:
a number has 4 different odd digits. the difference between the greatest digit and the least digit is 6, the number is greater than 2,000 and less than 3,160. what is the number?
Answered by Janice Cotcher.
Does an oval have sides? 2008-10-02
From reid:
My 6 yo neice came home with her math homework and she was supposed to identify which objects had sides.One of the objects was an oval.I don't believe it has sides because it is curved and I don't think that would make it an object with sides.What would be the correct answer?Thanks,Reid
Answered by Janice Cotcher.
The length of a tank 2008-10-01
From alex:
I have a tank which I can only partly see. It is 70cms wide, 45 cm deep and hold 500litres. How long is it?
Answered by Harley Weston.
Working together but one person starts first 2008-09-30
From David:
Della can scrape the barnacles from a 70 ft. yacht in 10 hours using an electric barnacle scraper. Don can do the same job in 15 hours using a manual scraper. If Della started helping Don three hours after Don started scraping, how long will it take to scrape the entire boat?
Answered by Penny Nom.
The vertex of a parabola 2008-09-30
From Anne:
How do you find the vertex of the function f(x)=-x squared+7x-6
Answered by Penny Nom.
Lines that pass through a point 2008-09-30
From Abigail:
what is the name for a point that a group of lines pass through that contains the letters c e n p i l?
Answered by Penny Nom.
z(z+1)-x(x+1) / z-x 2008-09-30
From sylvia:
z(z+1)-x(x+1) / z-x

HOW DO I SIMPLIFY THIS

Answered by Penny Nom.
The perimeter of a triangle 2008-09-29
From Debbie:
How do I calculate the perimeter of a triangle if the lengths of two sides are equal and all the angles are known?
Answered by Harley Weston.
Half life 2008-09-29
From aisyah:
if dA/dt = kA and A(0) = A as the model for the decay of a radioactive substance, how can show that in general, the half life T of substance is T = -(In 2)/k
Answered by Harley Weston.
How tall is the wall? 2008-09-29
From ash:
you and bob are separated by a tall wall you stand 10 feet further from the wall than bob your angle of elevation is 37 degrees and his 44 degrees how tall is the wall?
Answered by Penny Nom.
How many parallel tangents may a circle have? 2008-09-29
From Manish:
how many parallel tangents may a circle have? the text book shows two.but a circle can have infinite tangents.then why not parallel tangents coz theoretically each tangent have a parallel tangnts then no. of parallel tangent a circle may have is equals to half of the infinity i.e. infinity..
Answered by Walter Whiteley.
3,6,10,15,and 21 2008-09-28
From jarred:
i am currently stumped on a math project that requires me to find out the recursive formula for a sequence of numbers. the numbers in the sequence are 3,6,10,15,and 21. Thank you for your time.
Answered by Walter Whiteley.
The area of an arched transom 2008-09-28
From Ivan:
What is the formula for figuring out the square footage of arched transom windows & doors?
Answered by Harley Weston.
Increase pricing by 5% 2008-09-28
From Adrian:
I am working to increase pricing by 5%. If the price is 100, I would typically use the formulas 100 * 1.05 = 105, which is a $5 increase. An associate suggests I divide to get the desired increase. For example, using $100 with a 5 percent increase. I would use the forumula 100/.95 = 105.26. Thas is a 5.26 increase. Can you explain this method to me an why the different increases? Which one is correct.
Answered by Penny Nom.
What is the perimeter of the rectangle? 2008-09-27
From Pauline:
A rectangle has an area of 27 squared inches. the height is 3 times the base What is the perimeter of the rectangle?
Answered by Penny Nom.
The vertices of a triangle 2008-09-27
From T.:
how do you find the coordinates of the vertices of the triangle with the sides determined by the graphs of the following equations:4x+3y+1=0,4x-3y-17=0,4x-9y+13=0?
Answered by Harley Weston.
Simplify 6(a+2b)+8a-16b 2008-09-27
From madison:
6(a+2b)+8a-16b
Answered by Penny Nom.
What is the exact value of pi? 2008-09-27
From mahesh:
what is the exact value of pi.
22/7 is accurate value of pi, if not then how can i find out the exact pi value.

Answered by Harley Weston.
The base of a triangle is twice its height 2008-09-26
From carolyn:
The length of the base of a triangle is twice its height. If the area od the triangle is 100 square kilometers, find the height.
Answered by Penny Nom.
The standard form Ax+By=C 2008-09-26
From Susan:
I am having difficulty changing the point-slope form of the equation of the line y-y1=m(x-x1) to the standard form Ax+By=C. You must express A,B,C in terms of the constants m,x1,y1. I have been able to insert numbers and switch easily from the general to the point slope equation, but I am stumped on how to convert it this way.
Answered by Penny Nom.
5 units from (1,2) and 5 units from y-axis 2008-09-26
From Shaun:
Find the points (x,y) in the plane that are 5 units from (1,2) and 5 units from y-axis. I am more interested in the approach, in general terms, than the numerical values.
Answered by Penny Nom.
Compatible numbers 2008-09-25
From Katie:
I am having trouble remembering how to find compatible numbers used to find the estimate in the foloowing problem. Can you please help me solve it and explain it to me? My son is in 5th grade and I need help. Thank you. Here is the problem: 2,752 / 28 estimate: 90
Answered by Penny Nom.
Extraneous solutions 2008-09-25
From crystal:
/6x+7/=5x+2
Answered by Penny Nom.
A right triangle 2008-09-25
From john:
a right triangle has hypotenuse wich measures 20cm and a perimeter 47cm. find the measure of the remaining two sides
Answered by Harley Weston.
Exponential form 2008-09-25
From Pat:
The African bush elephant is the largest land animal and weighs about 8 tons. Write this amount in exponential form.
Answered by Penny Nom.
Some number words 2008-09-25
From jodie:
My son would like to know the names of digits. When counting it starts ones, tens, hundred, thousand, millions, billions, trillion...he would like to know the order of what comes next and how far we have named them..
Answered by Penny Nom.
The region between two circles 2008-09-24
From Carol:
Good day! Here is a picture of the problem that we need to solve. (I send the picture through e-mail.) A small circle is inside a larger circle, the only given in the problem is the chord of the larger circle tangent to the smaller circle which measures 16cm. The question is, what is the area of the shaded region? Can you answer this question? Thanks! :)
Answered by Harley Weston.
Consecutive Even Integer Sides of a Triangle 2008-09-24
From Brad:
I am having a problem coming up with a formula for my son's eighth grade math problem. We have found the answer by guess and check but have a mental block on the equation. Any help would be appreciated. The problem is: A right triangle has sides whose lengths in feet are consecutive even intergers. Determine the length of each side. Thanks
Answered by Janice Cotcher.
Parallel Tangents 2008-09-24
From manish:
how many parallel tangents may a circle have? the text book shows two.but a crcle can have infinite tangents.
Answered by Janice Cotcher.
0.99999.... 2008-09-23
From Eve:
Hi, i had a problem with change 0.99999... this recurring decimal to a fraction. I know the method, but the answer I got is 1 as you can see below.

Where have i done wrong?

Answered by Harley Weston.
A three 0-9 tumbler lock 2008-09-22
From Louis:
I recently gave my daughter an old briefcase which uses a three 0-9 tumbler lock. She set the lock and now she can’t open one side. Q. Is there a 3 digit combination number generator available to give me the 1000 possibilities?

Thanks,

Lou

PS In this type of lock you can repeat the same digit three times.

Answered by Penny Nom.
Probability: Marbles in a jar 2008-09-22
From Andrea:
Suppose you have a jar containing 100 red marbles and 100 white marbles.
A) If you draw 5 marbles in a row, throwing each marble across as you draw it, what is the probability that at least one of them was red?
B) If you draw 101 marbles in a row, throwing each one across the room as you draw it, now what is the probability that at least one of them was red?

Answered by Harley Weston.
Reciprocals 2008-09-22
From Josina:
What is the reciprocal of 1.8 and also send me the working of the answer
Answered by Penny Nom.
Sample variance: inches to centimeters 2008-09-22
From Anita:
The average height of a sample of basketball players is 6 feet, 2 inches or 74 inches. The standard deviation of this sample of players is 4 inches. If each person's height were to be multiplied by 2.54, what would be the value of the resulting sample variance?
Answered by Harley Weston.
Working together 2008-09-22
From Wade:
My daughter and I are having problems with this question: Maggie can mow the lawn in 6 hrs. If her sister, Julie, helps her, it only take 2 hrs. How long does it take Julie to mow the lawn alone.
Answered by Penny Nom.
The minimum value of f(x)=maximum{x,x+1,2-x} 2008-09-21
From Saurabh:
The minimum value of the function defined by f(x)=maximum{x,x+1,2-x} ?
Answered by Penny Nom.
The sides of an octagon 2008-09-21
From mark:
I need to know the measurements of an octagon with a inside diameter of 34.5 inches from side to side
Answered by Penny Nom.
The area of a garden 2008-09-20
From Hannah:
How do you find the area of a semi circle when no measurement is given? The question is find the area of the garden which is semi circular. The measurements are 24m and 26m and the semi circle's height is 10m!
Answered by Penny Nom.
Geometry with A Ladder Using Trig Functions 2008-09-20
From Please:
A ladder makes an angle of 50 degrees with the ground. if the base of the ladder is 10 feet from the building, how high up the building does the ladder reach? i saw that someone else had a similar question, but we arn't using the pythagorean therom. we are using trig functions. Most of the problems we are doing is with the tangent. so this one probably is too.
Answered by Penny Nom.
Cutting a pipe at an arbitrary angle 2008-09-20
From John:
from the original question Al asked about cutting a 200 diameter pipe in 45 degrees. can someone explain the math steps required to creating the graph. I am trying to do the same thing only using a 150 degree cut.
Answered by Harley Weston.
A father and daughter problem 2008-09-19
From Lamarr:
A father is four times as old as his daughter is now. In 20 years he will be only twice as old as his daughter. How old are the father and daughter now?
Answered by Penny Nom.
Simplify [[1/(x+3)]-1/3]/x 2008-09-19
From diana:
How does [[1/(x+3)]-1/3]/x simplify
Answered by Harley Weston.
Oils and fats 2008-09-18
From Barry:
I'm doing a nutrition assignment and I'm adding up my Oils and Fats category. I have 10ml of butter to add to 31g of fat so I'm trying to figure out what to do or I should say how to figure out the answer. Barry
Answered by Penny Nom.
The weight of a concrete cylinder 2008-09-18
From Curtis:
I am trying to calculate the weight of a concrete cylinder. The diameter is 56" and the thickness or depth is 24". Can anyone help me? If possible I would like to see the formula as to how the answer was derived. Thank you
Answered by Harley Weston.
Simplifying equations 2008-09-18
From diana:
I do not know how to simplify equations. How does 5x^2-7X+2 become (x-1) (5x-2)?
Answered by Penny Nom.
Concert tickets 2008-09-18
From Keisha:
Gross Receipts. Six thousand tickets are to be sold for a concert, some for $9 and the rest for $15each. If x is the number of $9 tickets sold , write an algebraic expression that represents the gross receipts from ticket sales, assuming all tickets are sold. Simplify the expression.
Answered by Penny Nom.
An Euler diagram and a logic argument 2008-09-18
From Regina:
Use a Euler diagram to determine whether the following argument is valid or invalid.

No wizard can yodel
All lizards can yodel

No wizard is a lizard

Answered by Penny Nom.
Adding fractions 2008-09-17
From amanpreet:
please can you help me to simplify:

2(c-3) / 7 + 3(4c-2)/2

Answered by Penny Nom.
The volume of a cylinder in gallons 2008-09-17
From subash:
i want to calculate volume of cylinder tank whose height is 1700mm and diameter is 1900mm and to convert volume into gallon ?
Answered by Harley Weston.
Art and Integers 2008-09-17
From pamela:
how do artists use integers?
Answered by Janice Cotcher.
Two Step Equation 2008-09-17
From calea:
i really dont now what two-step equation means?
Answered by Janice Cotcher.
Finding Traveling Time of a Projectile Given a Formula 2008-09-17
From David:
A juggler tosses a ballinto the air with a velocity of 40ft/sec from a height of 4 ft. Use s= -6t^2 + vot+so to find how long it takes for the ball to return to the height of 4 ft. I was wondering how should i start this out, and can you give me a brief explanation on the formula they ask you to use? thx
Answered by Janice Cotcher.
The volume of a box 2008-09-16
From Andre:
volume of the box cubic centimeters

____ X _____ X ______ X =_______

The length is 8.5 Width is 3.4

Answered by Penny Nom.
Exponential form 2008-09-16
From shawn:
Write the product in exponential form: 5.5.5.2
Answered by Penny Nom.
What is the derivative of (2^sinx)/(logbase4(2x+1))? 2008-09-16
From Jesse:
What is the derivative of (2^sinx)/(logbase4(2x+1))
Answered by Harley Weston.
Cubic yards and quarts 2008-09-16
From Mary:
I need 2 yards of soil, how many quarts is this?
Answered by Penny Nom.
The Intermediate Value Theorem 2008-09-16
From A.:
When dealing with the intermediate value theorem you have the function x^2. It bounces on the axis so you can't tell if lies on the interval [a,b]. So is the ivt proven false or does the ivt not tell you all the roots for sure.
Answered by Harley Weston.
Limit as it Approaches a Constant 2008-09-16
From Brooks:
what is the limit as x approaches 10 for f(x) = sqrt(x-1) = 3 pleeeeeaase help me out here
Answered by Janice Cotcher.
Solving Exponential Equations Using Logarithms 2008-09-15
From Todd:
7(6^x)=6e^-2x
Answered by Janice Cotcher.
The Width of a Mown Strip of Lawn 2008-09-15
From Rimi:
USE FACTORING, QUAD FORMULA, AND COMPLETE THE SQUARE! Chris cuts half a rectangular lawn, 40m by 30m, by mowing strips of equal width around the perimeter. Jonah cuts the small rectangle left. How wide a strip does Chris cut so that they share the work equally?
Answered by Harley J Weston.
A multiple choice exam 2008-09-13
From Phalange:
A multiple choice exam consists of 12 questions, each having 5 possible answers. To pass, you must answer at least 9 out of 12 questions correctly. What is the probability of passing if:
a. You go into the exam without knowing a thing, and have to resort to pure guessing?
b. You have studied enough so that on each question, 3 choices can be eliminated. But then you have to make a pure guess between the remaining 2 choices.
c. You have studied enough so that you know for sure the correct answer on 2 questions. For the remaining 10 questions you have to resort to pure guessing.

Answered by Harley Weston.
Three addends that sum to ten 2008-09-13
From Alfred:
How many probabilities are there to have 3 addends to get a sum of 10? What is the formula?
Answered by Harley Weston.
The height and base of a triangle 2008-09-12
From Bailey:
The length of the base of a triangle is 1cm less than 5 times the height of the triangle. The area of the triangle is 21 square cm. What is the the height of the triangle and the length of the base of the triangle?
Answered by Stephen La Rocque.
Cutting a pipe on a 45 degree angle 2008-09-12
From Bakshani:
how do you a mark a pipe 5 inch diameter and cut it to form a 45 degree angel
Answered by Stephen La Rocque.
A garden in the shape of an isosceles triangle 2008-09-12
From Rita:
Gregory wants to build a garden in the shape of an isosceles triangle with one of the congruent sides equal to 12 yards. If the area of his garden will be 55 square yards, find, to the nearest tenth of a degree, the three angles of the triangle.
Answered by Penny Nom.
The area of a quadrilateral 2008-09-11
From Mike:
How do I find the area of a quadrilateral that has one sloped side?
Answered by Penny Nom.
The perimeter of an equilateral triangle 2008-09-11
From Gerry:
How can I find the perimeter (length of side) of an equilateral triangle if the only information I have is the altitude?
Answered by Penny Nom.
Supplementary angles 2008-09-11
From Shanaz:
Find an angle such that 3 times its supplement equals 450.
Answered by Penny Nom.
A line through two points 2008-09-10
From Conor:
Find the equation of the straight line which passes through the points (-2,14) and (8,-1)
Answered by Penny Nom.
An ice cream cone 2008-09-09
From olivia:
Judy has a sugar cone and wants to know how many cubic inches of ice cream it will hold if it is filled completely to the top of the cone and no more. The cone has a height of 4.5 inches and a radius of 1.5 inches.
Answered by Penny Nom.
Balls and cubes in an urn 2008-09-09
From Dave:
In an urn, there are 80 objects of two kinds: cubes (C) and balls (B). An object can be either red (R) or green (G). Note that all the four combinations are possible and that the number of cubes is not necessarily equal to the number of balls. Similarly, the number of red objects is not necessarily equal to the number of green objects. Someone tells us that in the urn there are 20 red cubes, 50 balls, and 30 red objects. An object is randomly selected from the urn.
(a). What is the probability that a green ball is selected?
(b). If we know that a cube has been selected, what is the probability that it's red?
(c). If we know that a red object has been selected, what is the probability that it's a cube?

Answered by Harley Weston.
Angle of elevation 2008-09-09
From kristy:
A man on the tenth floor of a building shouts down to a person on the street. If the angle of elevation from the street to the man in the building is 35° and the man in the building is 40 feet up, about how far away from the building is the person on the street?
Answered by Penny Nom.
(5x - 4) / 6 = -9 2008-09-09
From Conor:
Solve to find x:

(5x - 4) / 6 = -9

Answered by Penny Nom.
Repeating fractions 2008-09-09
From Juli:
My teacher recently put my math class to the test... We were deiscussing repeating fractions and she asked us to find out what the bar over a repeating decimal is called. I found out it was called the vinculum. But she also said to find out what the number under the vinculum was called. I can't seem to find it anywhere.
Answered by Penny Nom.
The height of a tree 2008-09-09
From danice:
At a certain time of day, the angle of elevation of the sun is 30°. A tree has a shadow that is 25 feet long. Find the height of the tree to the nearest foot.
Answered by Penny Nom.
How would put .12 with 2 repeating into a fraction? 2008-09-09
From Savanna:
How would put .12 with 2 repeating in fraction?

Savanna!

Answered by Penny Nom.
How many ml are in 40 grams? 2008-09-09
From Nancy:
I need to know how many ml are in 40 grams. So what is the base 1ml= how may grams
Answered by Harley Weston.
The perimeter of a right triangle 2008-09-09
From Shanon:
I know that A=1/2bh, but for the life of me cannot figure out my 6th graders math problem.

What is the perimeter of a right triangle whose area = 600 sq. ft.

Answered by Victoria West.
Perpendicular 2008-09-08
From amanda:
hi my name is amanda medjuck i am in grade 7. we are learning basic geometry terms and have to define words BUT I DONT KNOW WHAT A PERPENDICULAR LINE IS PLEASE HELP ME!!!
Answered by Penny Nom.
0.151515...=15/99 2008-09-08
From Emma:
This week, my Algebra teacher told us about the pattern between infinitely repeating decimals and their corresponding fractions. (ex. .2222222...= 2/9, .151515...=15/99, 456456456...=456/999, etc.) I was just wondering the reason why this pattern occurs. Is there a certain element that causes this pattern to occur?
Thanks
-Emma

Answered by Penny Nom.
Collinear 2008-09-08
From Patrick:
On page 17 of my son's 9th grade math book Geometry by Jurgensen, Brown, and Jurgensen (copyright 2004), the authors state from the diagram that you can conclude the points A, B, and C are collinear. In the diagram, the points A, B, and C appear to be situated on a straight line; however, nothing in the diagram states that it is a straight line. I told him that you can't conclude they are collinear because angle ABC could be 179.9 degrees instead of 180 degrees since nothing in the diagram states otherwise. His teacher marked his answer incorrect on a test but I still believe that I am correct. How do conclude points are collinear if nothing tells you that they are situated on a line?
Answered by Harley Weston.
The biggest right circular cone that can be inscribed in a sphere 2008-09-08
From astrogirl:
find the volume of the biggest right circular cone that can be inscribed in a sphere of radius a=3
Answered by Harley Weston.
Finding the Speed of a Truck 2008-09-08
From Rita:
A state trooper is hidden 30 feet from a highway. One second after a truck passes, the angle theta between the highway and the line of observation from the patrol car to the truck is measured. (a) If the angle measures 15 degrees, how fast is the truck traveling? Express the answer in feet per second and miles per hour.
Answered by Janice Cotcher.
Pages in a Book 2008-09-08
From Pandora:
Is there a easier way to explain to my child regarding "pages" problem - like when to add one? when to sub tract one? John read 46 pages in his new book. If he began reading at the top of page 33, on what page did he finish? and John needs to do 45 pages of problems in his workbook. He finished this assignment at the the bottom of page 99. On what gate did John begain? Thank you.
Answered by Walter Whiteley.
An exclusion zone around a triangle 2008-09-07
From Awrongo:
A long time ago Mr Gibson found an island shaped as a triangle with three straight shores of length 3 km,4 km and 5 km. He declared an 'exclusion zone' around his island and forbade anyone to come within 1 km of his shore. What was the area of his exclusion zone?
Answered by Stephen La Rocque and Penny Nom.
1/3(2x-1)=1 2008-09-07
From Jen:
1/3(2x-1)=1
Answered by Penny Nom.
A security camera 2008-09-07
From Rita:
A security camera in a neighborhhod carnival is mounted on a wall 9 feet above the floor inside a video gallery. What angle of depression should be used if the camera is to be directed to a spot 6 feet above the floor and 12 feet from the wall?
Answered by Stephen La Rocque and Harley Weston.
Mixing watercolour paints 2008-09-07
From bob:
So, I'm making a chart to help me mix watercolor paints. There are 25 different paints. Each paint will mix with another paint just 1 time. There is no need to mix red with blue, and then blue with red. Since the result is the same, we only need 1 combination of the 2 paints. And there is no need to mix red with red, so we can eliminate 25 combinations of the same paint.
Answered by Penny Nom.
2^x = 1,000,000 2008-09-06
From Peter:
How do I solve for "x" in the following equation: "2 to the power of x = 1,000,000" ?
Answered by Penny Nom.
A tangent to a circle 2008-09-06
From Jake:
Find an equation of the line that is tangent to the circle x^2 + y^2 = 3 at the point (1,√2)
Answered by Penny Nom.
An array of tulip bulbs 2008-09-06
From Mary:
My daughter had this problem on one of her tests. She forgot to ask her teacher what she did wrong. I understand she did not answer the question in full, but what she did answer I feel is right.

Please explain to me the correct answer. Thank you. Mary

Answered by Harley Weston.
Parallel and perpendicular lines 2008-09-06
From C:
Can you help me determine whether these lines are parallel, perpendicular or neither and why?

Problem 1: y=3x+2
y=1/3x - 4

Problem 2: y=1/3x + 1/2
y=1/3 x -2

Answered by Penny Nom.
Simplify ((5/7)/(5/14)) + (3/4) 2008-09-06
From Emal:
How do you simplify completely: ((5/7)/(5/14)) + (3/4)?
Answered by Penny Nom.
Mowing a Lawn in Concentric Bands 2008-09-05
From Mari:
two kids are mowing half of their backyard which is rectangular with dimensions 75 ft by 90 ft. The first kid starts mowing at a corner gradually working his way toward the middle by mowing concentric bands around the outside edges.If the mower cuts 3-ft wide path, at what point should the first kid stop and the second kid start mowing? THANKS!
Answered by Janice Cotcher.
The percentage of sand in a soil mixture 2008-09-05
From kristi:
2 m 3 of soil containing 35% sand was mixed into 6 m 4 of soil containing 15% sand. What is the sand content of the mixture?
Answered by Penny Nom.
Mathematical induction 2008-09-05
From James:
I need to prove a problem by induction regarding the Triangle Inequality. The problem is

abs(a1 + a2 +...+an) <= abs(a1) + abs(a2) +...+ abs(an).

Answered by Victoria West.
3 miles in 6 seconds 2008-09-05
From Grant:
if a rocket sled is traveling 3 miles in 6 seconds, how fast is it going in miles per hour?
Answered by Harley Weston.
How do you write 92 tenths as a decimal? 2008-09-05
From Miriam:
How do you write 92 tenths as a decimal?
Answered by Penny Nom.
Hundreds, tens, and units digits 2008-09-05
From Cynthia:
write an expression for the value of a three-digit number whos hundreds, tens, and units digits are x, y, and z, respectively
Answered by Penny Nom.
The selling price 2008-09-04
From Robert:
Client wants to sell his property providing he is left with 100000.00$ net. How do I calculate selling price if commission is 6% and taxes on the commission 6% Fed and 7.5%prov. I'm taking the real estate class and this one really got me.
Answered by Penny Nom.
Microwave ovens and probability 2008-09-04
From Vi:
A microwave oven manufacturer claims that only 10% of the ovens it makes will need repair in the first year. Suppose three recent customers are independently chosen.
a. If the manufacturer is correct, what is the probability that at least two of the three ovens will need repair in the first year?
b. If at least two of the three customers' ovens need repair in the first year, what inference may be made about the manufacturer's claim?

Answered by Harley Weston.
Half a circle 2008-09-04
From brandon:
what are the area and perimeter and area of half a circle??
Answered by Penny Nom.
A surveillance satellite 2008-09-04
From Rita:
A surveillance satellite circles Earth at a height of h miles above the surface. Suppose that d is the distance, in miles, on the surface of Earth that can be observed from the satellite.

(a) Find an equation that relates the central angle theta to the height h.

(b) Find an equation that relates the observable distance d and theta.

(c) Find an equation that relates d and h.

Answered by Penny Nom.
Aboriginal number groupings 2008-09-04
From Barbara:
Our current place value system is based on the notion of 10s. Did Aborginal people use a similar system or did they group numbers differently?
Answered by Harley Weston.
The length of an arc 2008-09-04
From Angie:
Segment PR is a diameter of circle S. If angle P =3D 25, find minor arc QR. This circle has an isosceles triangle in it, it is connected to the diameter,
Answered by Harley Weston.
A line parallel to y= 1/2x + 5 2008-09-03
From Michelle:
Find the equation of a line parallel to y= 1/2x + 5 if the required line passes through ( 3,4)
Answered by Penny Nom.
Largest Inscribed Rectangle 2008-09-03
From astrogirl:
find the shape and area of the largest rectangle that can be inscribed in a circle of a diameter a=2
Answered by Janice Cotcher.
How accurate is the following calculation? 2008-09-03
From Craig:
Given that these numbers are only accurate to 2 decimal places, how accurate is the following calculation?

1.73 - 2.16 + 0.08 + 1.00 - 2.23 - 0.97 + 3.02 = 0.47

How do I work out the accuracy?

Answered by Penny Nom.
20 golfers playing 5 rounds in 4-somes 2008-09-03
From joe:
20 golfers playing 5 rounds in 4-somes. 5 players in group A play with all 15 golfers in groups B,C,D but not with any in their own group (A). 5 players in group B play with all 15 players in groups A,C,D but not with any in their own group (B) and so on. So each 4-some has an A,B,C,&D player each of the 5 rounds.
Answered by Chris Fisher.
1,679,616 as a power of 6 2008-09-03
From Denise:
express 1,679,616 in exponential form as a power of 6.
Answered by Penny Nom.
A circular swimming pool 2008-09-03
From David:
A circular swimming pool with a diameter of 30ft and a horizontal bottom contains 22,000 gal of water. What is the depth of the water in the pool? (one cubic foot contains approximately 7.5 gal of water.)

I'm having a great deal of problem dealing with this, i dont know if the equation I'm using is correct or am i plugging it in the wrong place. V=TTr^2h . if you can could you please explain to me step by step on how to set up this problem? the horizontal bottom thrown me off the most

Answered by Penny Nom.
A word problem with three numbers 2008-09-02
From Wanda:
"Jabari is thinking of three numbers. The greatest number is twice as big as the least number. The middle number is three more than the least number. The sum of the three numbers is 75. Find the numbers."
Answered by Victoria West.
A rectangular tank of water 2008-09-02
From Angela:
A rectangular tank is 80 ft long, 30 ft wide, and 20 ft deep. If 100,000 gallons of water were pumped into the tank, what would be the water rise?
Answered by Penny Nom.
Cows, chickens, heads and feet 2008-09-01
From marina:
Iam helping my 6th grader son solve this problem. I found similar problem in the Q&A.I understood the simple answer without using an equation. I want to solve this using the equation.This is the question about chicken and cows. There are cows and chickens in the farm. There are 65 heads and 226 legs. How many chicken are there?
Answered by Harley Weston.
A triangle and an inscribed circle 2008-09-01
From Nancy:
I'm a computer programming student, and I'm supposed to figure out how to find the area of a circle inside a triangle if someone types in the length of each side of the triangle.

So, a user can type in any three numbers they want into the three "side length" boxes, and I have to find the area of the circle that would fit inside the triangle they create from those values.

So the circle can be any size, depending on the size and shape of the triangle the user creates. The circle has to touch all three sides of the triangle somewhere. Then, my program calculates the area of the triangle and thus the area of the circle. I just need to know how the circle would change depending on the length of each side of the triangle that the user puts in. Is there a way to find out how the circle's area is related to the triangle around it?

Answered by Chris Fisher.
How many litres of rain is this? 2008-09-01
From Todd:
If i have 10mls of rain over one square metre how many litres would I and how do I do the maths for this?
Answered by Penny Nom.
The saddle height on a bicycle 2008-08-31
From David:
The saddle height on a bicycle should be 109% of the inside leg measurement of the rider (http://www.harriscyclery.com). If the saddle height is 37 in., then what is the inside leg measurement?

I don't know how to set up the equation if you can teach me how to set it up it would be great thanks

Answered by Harley Weston.
Divide $100.00 between two friends 2008-08-31
From anna:
divide $100.00 between two friends, so that one has $5.50 more than the other
Answered by Penny Nom.
A roof angle 2008-08-30
From carla:
Use vectors to find the angle in the attached diagram
Answered by Chris Fisher and Stephen La Rocque.
v-4(4-v) = -2(2v-1) 2008-08-29
From c:
Please explain all the steps to solving this

v-4(4-v) = -2(2v-1)

Answered by Penny Nom.
Jean-Charles de Borda 2008-08-29
From Joanne:
I am researching Jean-Charles de Borda as a mathmatician. What was his work focused on and where can I find more information?
Answered by Penny Nom.
Number of angles formed by rays 2008-08-29
From Shon:
what is the formula for finding the number of angels that can be named by a given number of rays with the same endpoint?
Answered by Stephen La Rocque.
The lengths of the sides of a rectangle are in ratio 5:3 2008-08-29
From todd:
the lengths of the sides of a rectangle are in ratio 5:3 the perimeter of the rectangle is 32 cm find the length of each side
Answered by Stephen La Rocque.
The flight of an airplane 2008-08-29
From kobina:
You are traveling on an airplane that is moving at 500 mph in a direction 40deg north of west.Therefore , You are travelling (due to the plane ) at how many mph toward the west.
Answered by Stephen La Rocque.
An algebraic expression 2008-08-29
From Antonio:
write a variable expression for 9 less than k
Answered by Penny Nom.
A fraction word problem 2008-08-29
From Sylvia:
The denominator of a fraction is 1 less than 4 times the numerator. If the numerator is doubled and the denominator is increased by 6, the value of the resulting fraction is 2/5. Find the original fraction.
Answered by Penny Nom.
How old are Brett and Jenny ? 2008-08-28
From Todd:
Brett and Jenny are each 10 years old. The sum of their ages is 11. Brett is 5 years older than Jenny. How old are they?
Answered by Janice Cotcher and Victiria West.
Long Division with Decimals 2008-08-28
From Ghita:
I need help with these 2 questions: Find the quotients: 52.576 divided by 21 71.331 divided by 93 Ralph spent 5/9 of an hour mixing paints and 7/9 of an hour painting.How much time in all did Ralph spend on his project? Thank you,Please answer soon
Answered by Janice Cotcher.
Abby has 45 coins 2008-08-28
From Sonia:
Use an algebraic approach to solve the problem. Abby has 45 coins, consisting only of dimes and quarters, worth $6.6. How many dimes and how many quarters does she have?
Answered by Penny Nom and Victoria West.
How many miles of track? 2008-08-27
From Stephanie:
I have a 50 ac tract of Railroad I need to know approx how many miles of tr= ack would be included in 50 ac
Answered by Penny Nom.
An equivalence relation 2008-08-27
From Francesca:
In this problem we are told to find out if the given set is an equivalence relation or not. {(x,y) such that 4 divides x-y}. What I am confused about is whether or not for instance if I picked (1,5) as a part of the set whether I could put in (5,1) as well or whether that would be wrong. Thanks!
Answered by Victoria West.
Angles in an octagon 2008-08-26
From Arvie:
I have seen your answer to Kay on how she can find the length for the sides of an octagon for a 4 foot window, but what is the angle that will need to be cut to get to 4 feet
Answered by Penny Nom.
Four weights 2008-08-25
From Darla:
A balance had known weights of 1/2 lb, 1/4 lb, 1/8 lb, and 1 oz. Edna's object weighted 2 oz. more than Paul's. Each used 3 known weights to weight his or her object. which know weights did each use?
Answered by Penny Nom.
Convert Repeating Decimal into a Fraction 2008-08-25
From Craig:
Can you help me to convert: 0.444444 (repeating decimal - base 10) to base 9?
Answered by Janice Cotcher.
Solve e^x + e^(x+1) = 250 2008-08-25
From Laura:
"solve this equation correct to three decimals"
e^x + e^x+1= 250

Answered by Penny Nom.
GST and PST 2008-08-25
From Monique:
okay i live in ontario and i want to find out what my grand total will be so i know how much i will have to pay...
so if i have a before tax total of $48.40... what will my grand total be? with GST and PST i know that GST=5% and PST=8%

Answered by Penny Nom.
The length of a pipe 2008-08-25
From John:
How many feet of one inch pipe would it take to hold 30 and 40 gallons of water?

Imperial gallons please.

Answered by Penny Nom.
The area of a triangle 2008-08-25
From Beverly:
What is the area of a triangle on a graph with the following points:
18,110
6, 110
12, 100

Answered by Penny Nom.
The square root of a fraction 2008-08-24
From Lauren:
How do you solve square roots of fractions? Does the format change if it is either a proper or an improper fraction?

Ex. the square root of 1/4 or the square roots of 80/25

Answered by Penny Nom.
A linear system 2008-08-23
From dipesh:
consider the system of equations
x-6y+2z=5
2x-9y-z=14
4x+12y-3z=19
by reducing the augm matrix to row echelon form, find the solution to the equation.

leaving the first two equations the third is changed to -3x+6y+14z=-31 show the equations have infinitely many solutions and give a general formula for them

Answered by Penny Nom.
Radii and Chords Create a Non-Right Triangle 2008-08-22
From Beary:
AOC is a diameter of circle O. Line AB is 12, and lines OA and OC (the radii) are 10. Find the length of line BO and chord BC.
Answered by Janice Cotcher.
Simplifying Algebraic Expressions 2008-08-22
From Jacky:
x^2-y^2+4x+4y
Answered by Penny Nom.
Two tangent circles 2008-08-22
From Michele:
A circle of radius 2 is externally tangent to a circle of radius 8, How do you find the length of their common tangent.
Answered by Penny Nom.
Function or Not? 2008-08-22
From Katie:
Determine whether y is a function of x: [(x^2)y]-(x^2)+(4y)=0 ....thx
Answered by Janice Cotcher.
Trough Filling with Water 2008-08-21
From lanny:
a triangular trough is 10 feet long, 6 feet across the top, and 3 feet deep. if water flows at the rate of 12 cubic inches per minute, find how fast the surface is rising when the water is 6 inches deep.
Answered by Janice Cotcher.
Irregular polygon and Circle that Intersects All Sides 2008-08-20
From Xetro:
Hi, Suppose you have an irregular polygon(convex or concave) with n > 3 sides. The question is - Find some circle that will cut(in limiting case - touch) all the sides of that polygon. It doesnt matter how many times it cuts the side(1 or 2), it just have to cut or touch it. How to find such a circle? or how to decide if such circle even exists? What if those segments do not form a polygon but are some arbitrary segments ? Really want to know how to do it................ Thanks a lot.. Regards, Xetro
Answered by Janice Cotcher.
Eccentricity of an Elliptical Orbit 2008-08-20
From Gina:
A satellite has an elliptical orbit around the earth with one focus at the earth’s center, E. The earth’s radius is 4,000 miles. The highest point that the satellite is from the surface of the earth is 800 miles, and the lowest is 200 miles. What is the eccentricity of the satellite’s orbit?
Answered by Janice Cotcher.
A square with diagonal 200 metres 2008-08-20
From kelvin:
find the are and perimeter of a square with diagonal of 200 meters?
Answered by Penny Nom.
Angular & Linear Speed from a Sine Graph 2008-08-19
From Kim:
Kim, a student: I am given a graph with a wave. The amplitude is 5cm and the period is 4cm. I am suppose to find the angular speed. What I need to know is the formula for angular speed and how do I use these numbers to get the correct answer.
Answered by Janice Cotcher.
Factoring x^2 + 729 2008-08-19
From peter:
hello I,am having trouble factorising a polynomial into polynomial factors with real coefficients please can you help the polynomial is x^2+729
Answered by Harley Weston.
Subdividing a chord 2008-08-18
From austin:
Here is my question. Imagine I have a circle of known radius 25 feet, and a chord with a mid point height of 6 inches from a central point on the chord to the circumference of the circle. I wish to divide this chord into a number of equal divisions. How can I calculate the measurement of the perpendicular line at each division of the chord to the circumference and at a 90 degrees at each division
Answered by Penny Nom.
A string around the world 2008-08-18
From Terry:
I heard this on T.V. and was wondering if it was true. Place a string around the world, you would need 25,000 miles of string. If you raised the string off of the ground by 1 foot all the way around. How much more string would you need?
Answered by Penny Nom.
Joining vertices in a polygon 2008-08-18
From Megan:
I'm trying to develop a relationship between the number of points of a regular polygon and the
(a) number of lines you could draw between those points,
(b) number of triangles you could draw,
(c) number of quadrilaterals,
etc.

Answered by Penny Nom.
Metres and miles 2008-08-18
From Bill:
I enter a race that is 10,000 meters long,how many miles do i have to run?
Answered by Victoria West.
Solving for Shared Height of Two Right Triangles 2008-08-17
From Heidi:
find the height of a triangle, which can be split into two right triangles, but the base (50m) is not split equally in half. one end of the base is 40 degrees, while the other is 30 degrees.
Answered by Janice Cotcher.
TRIG IDENTITIES 2008-08-17
From VINCE:
HOW DO YOU SIMPLIFY THE FOLLOWING:- 2sin3(theta)cos3(theta)? Regards
Answered by Harley Weston.
Similar Triangles Given a Median 2008-08-17
From RAM:
Sides AB , AC and median AD of a triangle ABC are respectively proportional to PQ PR and median PM of another triangle PQR. Show that ABC is similar to PQR
Answered by Janice Cotcher.
A triangular field 2008-08-15
From joneka:
Barb walked five times aroun a triangle field all together she walked 435 feet the first side of the triangle fiel is 23 feet the length of the other two sides are of the same measure. What are the dimensions of the triangle field?
Answered by Penny Nom.
Three letter combinations 2008-08-15
From Shelbie:
How many 3 letter combinations are possible, if you can repeat each letter?
I learned an easy way to do this in school, but fail to remeber what that was...

Answered by Penny Nom.
How do you solve for x when (x^4)-x is greater than or equal to 0? 2008-08-14
From Katie:
How do you solve for x when (x^4)-x is greater than or equal to 0?
Answered by Penny Nom.
Coefficients, Like Terms, and Constants 2008-08-14
From Maddie:
How Would I Answer This Problem?

  Coefficients Like Terms Constants
3x+7      
4m+(-3n)+n      
6kp+9k+kp-14      
-8y+6ab+7-3ba      
c+2c+c-5c+1      


I Would Really Appreciate It If You Could Help Me
Sincerely,
Maddie

Answered by Penny Nom.
[f(x)-f(1)]/(x-1) 2008-08-14
From katie:
Evaluate (if possible) the function of the given value of the independent variable:

f(x)=(x^3)-x:

[f(x)-f(1)]/(x-1)

Answered by Penny Nom.
How many city blocks in a mile? 2008-08-14
From DARLENE:
HOW MANY CITY BLOCKS EQUAL 1 MILE IN BIRMINGHAM, AL?
Answered by Penny Nom.
Similar triangles 2008-08-12
From ramarao:
D is a point on side BC of a triangle ABC such that BD/CD=AB/AC. prove that AD is the bisector AD is the bisector of BAC
Answered by Chris Fisher.
The volume of a ditch 2008-08-12
From John:
Question from John: I am trying to figure out how much flowfill I will need for a ditch that we are digging out. The ditch is 300' long and is 4' deep on one end and 2' deep on the other end. We are going to dig this ditch out and replace it with a flowfill concrete material. How much flowfill material will we need or what is going to be the volume of this ditch?
Answered by Penny Nom.
Algebra 2008-08-10
From Reasat:
How to understand algebra?
Answered by Stephen La Rocque.
The base of a triangle 2008-08-10
From Mark:
What is the formula I need to use to find the base of a triangle sail that is 16 feet high and has an area of 64 square feet.
Answered by Stephen La Rocque and Penny Nom.
Two problems involving reciprocals 2008-08-10
From Dee:
I have a few questions that im have trouble working out...

1. One number is three times another number; if each is increased by 1 the sum of the reciprocals is 10/21. Find the numbers

2.A tank can be filled by 2 pipes together in 6 hours; if the larger pipe alone takes 5 hours less than the smaller to fill the tank, find the time in which each pipe alone would fill the tank

Answered by Penny Nom.
The volume of a swimming pool 2008-08-10
From Ron:
What is the volume of a swimming pool when its length is 40 ft, width 20ft, the deep end is 10 ft and the shallow end is 3 ft.?
Answered by Penny Nom.
Solve for y in terms of x 2008-08-10
From Rosie:
Solve equation for y

-4x=12+3y

Answered by Penny Nom.
A linear function 2008-08-07
From Warren:
What are the conditions of the numerical coefficients of the standard form Ax+By=C in Linear Function? How will you transform y=square root of 3 x + 1 in standard form Ax+By=C?
Answered by Penny Nom.
Angles in a regular tetrahedron 2008-08-07
From Carla:
Hi guys, A regular tetrahedron has all its edges 8cm in length. Find the angles which an edge makes with the base. Thanks. Carla
Answered by Penny Nom.
1/X-2 + 3/X+3 = 4/X^2+X-6 2008-08-06
From Molly:
Hi, I'm beginning pre-calc this semester, and for practice over the summer, the teacher has given us a review packet with algebra 2 stuff. Well, I've forgotten most of the fraction stuff on it. Would you please help? Here's the question:

1/X-2 + 3/X+3 = 4/X^2+X-6

Answered by Penny Nom.
Square miles 2008-08-06
From Koni:
I'm trying to figure out the distance our buses travel throughout our district to pick up students for school. How do you figure square miles?

If our district measures 6 1/2 miles wide and 2 3/4 miles deep, what is our total square miles?

Answered by Penny Nom.
Arc-length and sector-angle 2008-08-06
From Benson:
If chord length, radius are given, How to find the sector angle and arc-length
Answered by Janice Cotcher.
Cost of pencil is rising every year since 7 years ago 2008-08-06
From Mari:
Cost of pencil is rising every year since 7 years ago. If the cost of pencil last year was 60 cents which is the sum of the cost from the last two previous years, what is the cost of the pencil now, the cost seven years ago.
Answered by Penny Nom.
The angle between two faces of a pyramid 2008-08-06
From Carla:
A pyramid has its vertex directly above the centre of its square base. The edges of the base are each 6cm, and the vertical height is 8cm. Find the angle between 2 adjacent slant faces.
Answered by Penny Nom.
A cone on the end of a cylider 2008-08-05
From Jerry:
Hi. I need to make a plastic cone to fit on the end of a cylinder. The only dimension i have is the diameter of the cylinder. This 553 mm. I need to find the formulae required and how much material will be required.
Answered by Penny Nom.
Choosing 5 numbers from 39 2008-08-04
From robert:
Hi,

I am sending you the web page where I found the question of 39 numbers and = 5 choices, please see below.

How does they comes up with the 120 combinations?

How can I comes up with the 120 combinations in a printout form?

Please read all the way to the bottom of there example and see where they talks about the 120 combinations.

Answered by Victoria West and Harley Weston.
Some angles in a pyramid 2008-08-04
From Carla:
A symmetrical pyramid stands on a square base of side 8cm. The slant height of the pyramidis 20cm. Find the angle between the slant edge and the base, and the angle between a slant face and the base.
Answered by Penny Nom.
Find the product of 2^35 and 5^38 in sci. notation. 2008-08-03
From Peter:
I am preparing for a competition and a lot of the non-calculator problems are like find the product of 2^35 and 5^38 in sci. notation. How would you do that?
Answered by Penny Nom.
A cubic mile of water 2008-08-03
From Fred:
[QQ] HOW MUCH WATER IS THERE IN A CUBIC MILE?
Answered by Stephen La Rocque.
Non-terminating, non-repeating decimals 2008-08-03
From Peter:
How do you take a random, non-terminating, non-repeating decimal into a fraction?
Answered by Stephen La Rocque.
The volume of a fuel tank 2008-08-01
From G.:
I have a fuel tank that is 70 inches by 48 inches and 24 inches deep, how many imperial gallons does it hold?
Answered by Victoria West.
Two equations in two unknowns 2008-08-01
From James:
Question from james, a student:

3x+2y=9;24x-2y=18

My math teacher says that it is (2,-4)

Can someone please explain how to figure this out by graphing?

Answered by Victoria West.
Area of triangle formed by three tangent circles 2008-07-31
From brian:
Three circles with radii 3,4 and 5 touch each other. The circles are tangent to each other. What is the area of the triangle formed by the centers of the circles?
Answered by Stephen La Rocque.
algebra 2008-07-31
From Eric:
Would you please solve & explain this equation to me: x^2+2x=x(x+2)? Thank you
Answered by Penny Nom & Stephen La Rocque.
Finding Angles in a Pyramid 2008-07-30
From Carla:
A pyramid has its vertex directly above the centre of its square base. The edges of the base are each 8cm, and the vertical height is 10cm. Find the angle between the slant face and the base, and the angle between the slant edge and the base.
Answered by Janice Cotcher.
Inscribed Rectangle 2008-07-30
From Felicia:
A rectangle whose base is twice its altitude is inscribed in a circle whose radius is 5 mm. Find the area of the rectangle.
Answered by Penny Nom.
Lot size of a four sided lot 2008-07-30
From JAMES:
I'm trying to find out this lot size in acres. I looked at a bunch of your suggestions, but I guess I'm not smart enough to figure it out.. I have attached an image of the lot, hopefully with enough info. for you..

Thanks,
Jim

Answered by Penny Nom.
Right Sided Limit of an Exponential Function 2008-07-29
From joseph:
I am trying to find the limit as X approaches 0 from the positive side of x to the sin of x which look like this lim x^sinx x->0+
Answered by Harley Weston.
Limit as it Approaches Infinity 2008-07-29
From mary:
i was trying to find the limit of this problem the limit as x approches infinity of x minus cosx divided by x lim x-cosx/x x->oo
Answered by Harley Weston.
Integral of X^2 2008-07-28
From Hemanshu:
Integral of X^2
Answered by Janice Cotcher.
Solving with the Natural Log 2008-07-28
From CB:
I am trying to solve lnx +3ln2 = ln 2/x. Every way I've tried when I go to plug in the solution it doesn't come out right.
Answered by Penny Nom.
The product of the digits of a four digit number 2008-07-26
From Pete:
I am a student preparing for a competition and this was one of the prep problems: The product of the digits of a four digit number is 6x5x4x3x2x1. how many such numbers are there with this property?
Answered by Stephen La Rocque.
Solving for Two Variables 2008-07-24
From Mary:
can you help, 2/35= 1/x +1/y ; x
Answered by Janice Cotcher, Victoria West and Harley Weston.
Sum of Digits 2008-07-24
From Warren:
If 100^25-25 is written in decimal notation, what is the sum of digits?
Answered by Penny Nom.
Volume of a Shed 2008-07-24
From lou:
I need to know the interior volume in cubic feet of a shed with the dimensions 8' x 10' x 7' with a roof peak of 9'. Thanks
Answered by Janice Cotcher.
Constructing An Open Box 2008-07-23
From Rita:
An open box with a square base is needed to have a volume of 10 cubic feet. (a) Express the amount A of material used to make such a box as a function of the length x of a side of the square base. (b) How much material is needed for a base 1 foot by 1 foot?
Answered by Janice Cotcher.
Alpha level 2008-07-23
From anonymous:
What is the advantage of using an alpha level of .01 versus a level of .05, What is the disadvantage of using a smaller alpha level?
Answered by Janice Cotcher.
Simultaneous equations 2008-07-23
From Franco:
Solve

3 D + E - F = -10
-2 D - F = -4
-3 D - 4 E - F = -25

Franco

Answered by Penny Nom.
How many gallons of fuel still in the barrel? 2008-07-22
From Charles:
I have barrel 6 feet long and 3 feet diameter that is laying on it's side with 5 inches of fuel, how many gallons of fuel still in barrel
Answered by Penny Nom.
3-D Vectors 2008-07-22
From jenina:
1. The summit of a mountain, 2450m above base camp, is measured on a map to be 4580m horizontally from the camp in a direction 32.4º west of north. What are the components of the displacement vector from camp to summit? What is its magnitude? Choose the x axis east, y axis north and z axis up.
Answered by Janice Cotcher.
The midpoints of the sides of a quadrilateral 2008-07-22
From JOEL:
A QUADRILATERAL A,B,C,D HAS THE CO -ORDINATE [2,5],[8,7],[10,3]&[0,1] RESPECTIVLY [B,E,G,H] ARE THE MID POINTS OF THE SIDES AB,BC,CD&DA RESPECTIVLY FIND THE MID POINTS OF [FG]&[FH]
Answered by Penny Nom.
The maximum range of a projectile 2008-07-22
From kwame:
the range R of projectile fired with an initial velocity Vo ,at an angle of elevation (@ )theta from the horizontal is given by the equation R = (Vo(squared) sin2theta)/g. where g is the accelation due to gravity . Find the angle theta such that the projectile has maximum range .
Answered by Harley Weston.
Time with Non-Uniform Motion 2008-07-21
From jenina:
An unmarked police car, travelling a constant 95km/h, is passed by a speeder travelling 145km/h. precisely 1.00s after the speed passes, the policeman steps on the accelerator. If the police car’s acceleration is 2.00m/s², how much time elapses after the police car is passed until it overtakes the speeder (assumed moving at constant speed)?
Answered by Janice Cotcher.
The weight of a cubic yard of dirt 2008-07-21
From Robert:
I need to know how many pounds of dirt is in a cubic yard
Answered by Penny Nom.
Taking two math classes 2008-07-21
From Bailey:
i am going into the 7th grade, but i am going to be taking 9 th grade math. i was wondering if anyone could help survive doing two different math classes at the same time.
Answered by Harley Weston.
An equilateral triangle 2008-07-21
From Meagan:
An equilateral triangle has vertices at (0,0) and (6,0) in a coordinate plane. What are the coordinates of the third vertex? You may want to sketch it out. Note: The sides of an equilateral triangle are identical in length.
Answered by Harley Weston.
The number of digits in a really big exponent 2008-07-21
From Pete:
how would you find the number of digits in a really big exponent without a calculator?
Answered by Penny Nom.
A hexagon has 3 parallelograms 2008-07-21
From Stephanie:
A hexagon has 3 parallelograms but I am unsure of how to cut a hexagon into 3 parallelograms. Can you show me what this would look like?
Answered by Harley Weston.
Factoring a quadratic trinomial 2008-07-21
From Jeff:
c^2+4c+4

x^2-7x-18

Answered by Penny Nom.
Three consecutive positive integers 2008-07-20
From AC:
find three consecutive positive integers such that the product of the first and third, minus the second, is 1 more than 4 times the third
Answered by Penny Nom.
Four Positive Integers 2008-07-20
From william:
let a, b, c and n be positive integers. If a+b+c=(19)(97) and a+n=b-n=c/n, compute the value of a.
Answered by Janice Cotcher.
???? x ? = ???? 2008-07-20
From waiyan:
????
x ?
????
using 1 2 3 4 5 6 7 8 9

Answered by Janice Cotcher.
A square and a circle 2008-07-20
From kobina:
4 ft of a wire is to be used to form a square and a circle. how much of the wire is to be used for the square and how much should be used for the square in order to enclose the maximum total area
Answered by Harley Weston.
A bijection from (0,1)x(0,1) to (0,1) 2008-07-20
From Adam:
I'm trying to prove that the function that takes the open square (0,1)x(0,1) to (0,1) is a bijection (and hence a continuum).

If we take an element (x,y) of (0,1)x(0,1) and represent (x,y) as (0.x1 x2 x3 x4..., 0.y1 y2 y3 y4...) aka x1 represents the tenths digit of x, x2 represents the hundredths, etc. Then we can define a function f((x,y)) = 0.x1 y1 x2 y2 x3 y3... However, this is not a bijection. I hypothesize this is because you'd be unable to create the number 0.1 as x=0.1 and would have to be y=0, which contradicts the open interval (0,1) defined for y. We have been told though, if we create the same function, except that we "group" 9's with their next digit into a "block" we can create a bijection. For example, if x=0.786923 and y=0.699213, then we define x1 to x3 as normal, but x4= 92, and x5=3. For y, we define y1 as normal, but y2=992, and y3 to y4 as normal. hence f((x,y)) = 0.7 6 8 992 6 1 92 3 3.

My questions are a) is my hypothesis on why the original function is not a bijection correct? b) why does the special blocking in the new function make a bijection?

Answered by Victoria West.
Choosing 5 numbers out of 39 numbers 2008-07-18
From robert:
Please list the way I can choose 5 numbers out of 39 numbers, without repeating them, starting at number 1.

Please send me a list of all 120 combination.

Answered by Harley Weston.
The height of a triangle 2008-07-18
From Becc:
I have to find the height of a triangle.
The base is 3.1cm say ab
One of the top sides is 4cm say ca
The other side is 2cmsay cb

Answered by Penny Nom.
Graphing Using Double Angle Identities 2008-07-16
From Hodan:
the Question is: Describe how you could use your knowledge of Double angle formulas to sketch the graph of each function. Include a sketch with your description.A) F(x)=sin x cos x B)F(x)=2 cos(squared)x C) F(x)= tan(x) (divided) by 1-tan(squared) x
Answered by Janice Cotcher.
Rational numbers 2008-07-16
From hinal:
list 12 rational numbers which lie between
a) -1 and 0
b) -3 and -3

Answered by Penny Nom.
An isosceles triangle inscribed in a circle 2008-07-15
From Anne:
Here is the math problem quoted from book:
"An isosceles triangle is inscribed in a circle of radius R, where R is a constant. Express the area within the circle but outside the triangle as a function of h, where h denotes the height of the triangle."

Answered by Penny Nom.
multiply = 3a - 3b by a+b 2008-07-15
From chaleen:
multiply = 3a - 3b by a+b
Answered by Penny Nom.
How much sand do I order? 2008-07-15
From jackson:
We would like to put 2 inches of sand down. The pool is 24 feet. How much sand will we need? The person we spoke to yesterday was talking in tons, but on these sites it is talking cubic yards or feet. Please help. We would like to get the sand ordered so we can get the pool up!
Answered by Harley Weston.
The length of an arc 2008-07-14
From Chris:
trying to find the length of an arc or segment of a circle when the radius or circumference is unknown take a circle, put a line across the circle label it A /B /C A and C are the end points, B is the middle line length is 86. From B to the side of circle is 16. Label that point D need to find the length of A D C
Answered by Penny Nom.
n choose r equals n-1 choose r plus n-1 choose r - 1 2008-07-14
From fae:
Prove that

( n ) = ( n – 1) + ( n - 1 )
( r )         ( r )        (r-1)

NOTE: the ( ) should be one for n taken r and so on. but there is no one big ( ) that will cater two lines

Answered by Janice Cotcher.
A reflected line segment 2008-07-14
From Peter:
Segment AB has endpoints A(-1,2) and (3,1). Segment AB is reflected over the y-axis such that A becomes A' and B becomes B'. What is the positive difference between the lengths of segment AA' and segment BB'?
Answered by Janice Cotcher.
A silo with a flat side 2008-07-14
From Amy:
Without using the Pythagorean Theorem, determine the capacity of a silo in cubic feet of grain if: the cylinder-shaped silo has one flat, rectangular face that rests against the side of the barn; the height of the silo is 30 feet and the face resting against the barn is 10 feet wide; the barn is approximately 5 feet from the center of the silo.
Answered by Harley Weston.
Like terms 2008-07-14
From Jana:
Hello, i am a student in year 8 doing Algebra. My Teacher is doing a sheet with us and it asks us to find Like Terms.
The Questions have sentences and you have to find the Like Term. I was wondering, does it matter when there is numbers that are different? are they still like terms?
I'm really confused, i'll write out an example of the questions, so you know what i mean.

It says to find the like terms:
(1.) c- 3c + 7c
(2.) 4x - 7x + 5x
(3.) 3xy + 4 xy

Answered by Penny Nom.
The area of a triangle - no height given 2008-07-12
From phoenix:
given triangle with angles 30,105 45 degrees. base = 2radical 2, no height given
Answered by Penny Nom.
Chords and arcs 2008-07-11
From Ronnie:
We are trying to build a semi life size ark decoration , and we are trying to cut the sides out . The curved sides and we can't figure our radius , all we know is that our chord length is 24ft. any suggestions on how to find the radius or maybe even the arc length or circumference or diameter?
Answered by Harley Weston.
A Falling Rock 2008-07-11
From Rita:
This question has 3 parts. If a rock falls from a height of 20 meters on the planet Jupiter, its height H (in meters) after x seconds is about H(x) = 20 - 13x^2 (a) What is the height of the rock when x = 1.2 seconds? (b) When is the height of the rock 15 meters? (c) When does the rock strike the ground? Thank you
Answered by Janice Cotcher.
Mathematical induction 2008-07-11
From lyn:
can you give me a basic example of a mathematical induction
Answered by Harley Weston.
A difference quotient 2008-07-10
From Rita:
Find the difference quotient of f, that is, find [f (x + h) - f (x)]/h, where h does not = 0 for the given function. Be sure to simplify.

f(x) = 1/(x + 3)

Answered by Janice Cotcher.
Overbooking flights 2008-07-10
From DON:
Overbooking by Airlines This is a simplified version of calculations used by airlines when they overbook flights. They realize that a certain percentage of ticketed passengers will cancel at the last minute. Therefore, to avoid empty seats, they sell more tickets than there are seats, hoping that just about the right number of passengers show up. We will assume that the no-show rate is five percent. For a flight with 220 seats, the airline wants to find how sensitive various probabilities are to the number of tickets it issues. In particular, it wants to calculate
a) the probability that more than 225 passengers show up
b) the probability that more than 220 passengers show up
c) the probability that at least 215 seats will be filled
d) the probability that at least 210 seats will be filled.
To assess the benefits and drawbacks of issuing various numbers of tickets on an airline flight with 220 seats, create a table showing as many different scenarios as possible (table only on one page when printed) and use a second page for your analysis and recommendation to the airline. Which are the good cases, which are the bad cases for the airline?

Answered by Janice Cotcher.
Two numbers whose sum is 25 and whose product is 120 2008-07-10
From Michelle:
Solve the following problem in at least two different ways.
Find two numbers whose sum is 25 and whose product is 120.

I know how to solve this problem by creating two equations x+y=25 and xy=120 and then use substitution.

Are there any different ways of solving this problem?

Answered by Janice Cotcher and Penny Nom.
How far is the ladder from the wall? 2008-07-10
From Al:
a ladder is leaning against a wall at a 70 degree angle and the ladder is 20 feet tall...how far away is the ladder from the wall
Answered by Penny Nom.
What is the account value after 3 years? 2008-07-10
From Carlos:
Question Suppose that you deposited $3,000 at the beginning of each year for three years in a savings account earning 4%. What is the account value after 3 years (approx)?
a. 9,120
b. 9,211
c. 9,739
d. 9,989

Answered by Penny Nom.
A convex quadrilateral in spherical geometry 2008-07-09
From Joan:
What is the min and max number of obtuse angles possible ia a convex quadrilateral in Spherical Geometry? I know that the Saccheri has 2 obtuse angles and the Lambert has one, but are there other possibilities? Thanks for your help.
Answered by Chris Fisher.
A tangent line to a circle 2008-07-09
From Rita:
The tangent to a circle may be defined as the line that intersects the circle in a single point, called the point of tangency. If the equation of the circle is x^2 + y^2 = r^2 and the equation of the tangent line is y = mx + b, show

r^2(1 + m^2) = b^2

Answered by Harley Weston.
A four sided lot 2008-07-09
From michelle:
the front is 65.29
left side 162.26
the back 155.00
right side 101.01

right angle=50.00
left angle= 153.30

Answered by Harley Weston.
Hexadecagon 2008-07-09
From Dave:
I would like to know at what length and angle I would cut wood to make a Hexadecagon.
Answered by Penny Nom.
A dog tied to a round building 2008-07-08
From maitham:
i have this question which i don't know how to solve it :

One dog was linked to the outer wall of a building round of 20 meters in diameter. If the length of chain linking the dog sufficient turnover of half the distance around the building, What area can guard dog?

they said that we can solve it by integral .. can you solve it for me?

Answered by Harley Weston.
Zip codes, postal codes and 5 digit codes 2008-07-08
From Mike:
Sorry if you have answered this before but I don't understand the explanations I have seen. How many possible combinations are in a 5 digit code (like a Zip Code). Is there an equation that can be used for this type of question?
Answered by Penny Nom.
Two trains 2008-07-07
From Elisha:
Trains at each end of the 50. km long Eurotunnel under the English Channel start at the same time into the tunnel. Find their speeds if the train from France travels 8. km/h faster than the train from England and they pass in 17 minutes.
Answered by Janice Cotcher and Penny Nom.
Probability and two digit numbers 2008-07-07
From Peter:
What is the probability that if you multiply two randomly selected two digit whole numbers, the result is greater that 100 and less than 200. Express your answer as a common fraction.
Answered by Janice Cotcher.
CIRCLES 2008-07-07
From daryl:
Find the equation of the smaller circle that is tangent to the axes and the circle x(squared)+y(squared)=2x+2y-1?
Answered by Penny.
Cubic centimeters and milliliters 2008-07-07
From Robert:
how many milliliters are in 750 cubic centimeters?
Answered by Harley Weston.
Choosing a number from 1 - 400 2008-07-04
From Peter:
The numbers 1-400, inclusive, are put into a hat. What is the probability that the first number chosen at random is a multiple of 4 or 17? Express your answer as a common fraction. How do you do this the question without a calculator?
Answered by Leeanne Boehm.
Divisibility by 11 2008-07-04
From Peter:
For what single digit value of n is the number n53nn672 divisible by 11?
Answered by Leeanne Boehm.
Probability of Matching Socks 2008-07-04
From Peter:
Rick has 6 different pairs of socks. Whats the probability that two randomly selected socks will be from a matching pair?
Answered by Leeanne.
Two triangles and a circle 2008-07-03
From Anita:
An equilateral triangle with side of length 1 cm is inscribed in a circle. A second equilateral triangle is circumscribed about the circle with all sides tangent to the circle. Find the length of a side of the second triangle.
Answered by Harley Weston.
Exponential form 2008-07-03
From Kent:
How to write 8 365 005 106 in expanded and exponential form? May I request you also to explane how does it solve.
Answered by Stephen La Rocque.
A series solution of y' = xy 2008-07-03
From sasha:
I've to find the power series solution of the differential equation: y' = xy.
I don't know how to find the recursive equation. Can you please help me. Thanks

Answered by Harley Weston.
Polynomial division and remainders 2008-07-02
From RAM:
If the polynomial x4-6x3+16x2-25x+10 is divided by another polynomial x2-2x+k, the remainder comes out to be x+a. Find k and a.
Answered by Janet Cotcher and Harley Weston.
z-score 2008-07-02
From Candace:
If a normal-shaped distribution has a mean of 80 and a standard deviation of 15 what is the z-score for M=84 for a sample of n=25 scores. Is this sample mean in the middle 95% of the distribution?
Answered by Harley Weston.
Units digit 2008-07-02
From seemachauhan:
How to find unit digit of various numbers and what are unit digits?
Answered by Penny Nom.
The distribution of sample means 2008-07-02
From crystal:
A population forms a normal distribution with a mean of 75 and a standard deviation of 20. How do I sketch the distribution of sample means for samples of n= 100?
Answered by Harley Weston.
Fit 7 different rectangles on grid paper 2008-07-02
From anonomous:
I need to fit 7 different rectangles on grid paper of 1cm squared. Each squared centimetre equals 4 squared metres, and it needs to end up as area 64 square metres.
Answered by Janice Cotcher.
GST and PST 2008-07-02
From Sharon:
The question was asked before but it was based on a G.S.T of 7% and I am now interested in the answer for 5% G.S.T. It usually comes up when I husband bring homes a gas bill that doesn't break down the gst and pst, but rather just gives him the total cost I have to know how much GST he paid. If it is Diesel the PST isn't included but if it is regular gas he pays both. PST is 7%.
Answered by Penny Nom.
Miles per hour 2008-07-01
From Nedra:
If I walk 1 mile and 2/10ths in 20 minutes how fast am I going? AND how many calories have I burned off?
Answered by Penny Nom.
Some phone numbers 2008-06-30
From star:
What do you think the probability of finding the last 3 digits of a telephone number if the 1st digit begins with 7?
Answered by Janice Cotcher and Harley Weston.
A rectangular room 2008-06-30
From Peter:
A rectangular room has a perimeter of 42 feet and an area of 108 square feet. What is the length, in feet, of the shorter side?
Answered by Penny Nom.
Remainders 2008-06-30
From vivek:
what is the remainder when 2050*2071*2095 is divided by 23 ? this question needs to be done in as less time as possible.
Answered by Penny Nom.
A five-sided coin 2008-06-29
From carla:
A new five-sided coin is to be made in the shape of figure 8.6 The point A on the circumference of the coin is the centre of arc CD, which has a radius of 2cm Similarly B is the centre of arc DE, and so on. Find the area of one face of the coin.
Answered by Harley Weston.
The size of a pipe 2008-06-29
From payal:
hi , i want to know the area of a pipe with 63.5 mm diameter and 3718 mm height . can u pls help me
Answered by Harley Weston.
A concrete spectator area 2008-06-29
From Paige:
an area of the school courts is 58.5m x 60.4m, The spectator area is to be 2m wide and the concrete has to be 10cm deep Please can you calculate the volume of concrete needs for the spectator area?
Answered by Penny Nom.
4 by 4 determinants 2008-06-27
From rav:
How to solve problems of determinants which has four rows and four columns& please give me easy tips to solve permutations and combinations problems.
Answered by Harley Weston.
Write log_2 (1/8) = -3 in exponential form 2008-06-27
From Jaime:
how do you write log2 (1/8) = -3 in exponential form?

the 1/8 is a fraction and the 2 is a base of 2

Answered by Penny Nom.
How many handshakes take place? 2008-06-26
From Peter:
Eight people of different heights are at a party. Each person decides to only shake hands with people shorter than him/herself. How many handshakes take place? My math book says it is 0. Is it wrong? If so, what is the answer?
Answered by Harley Weston.
What is the number of miles traveled by each tire? 2008-06-26
From Peter:
A car is driven 40,000 miles using four tires and a spare tire. The tires are rotated so that each tire travels the same number of miles. What is the number of miles traveled by each tire?
Answered by Penny Nom.
Kilometers per liters to miles per gallon 2008-06-26
From margaret:
we travelled 130 kilometers on 10 liters of gas. How many miles to the gallon did I get
Answered by Penny Nom.
The exterior angles of a polygon 2008-06-26
From evelina:
how many sides has a regular polygon if the measures of the exterior angle is given
Answered by Walter Whiteley.
The smallest number divisible by 1 to 9 2008-06-26
From Peggy:
What is the smallest number divisible by each of the first nine counting numbers?
Answered by Penny Nom.
A rectangle 2008-06-26
From SEBASTIAN:
How to arrange quadrilateral in the form of a rectangle?
Answered by Penny Nom.
Four fours 2008-06-25
From jen:
Use four 4's and some of the symbols: +,-,X, / and (), to give expressions for the whole numbers from 0 through9. Example : (4X4+4)/4=5
Answered by Janice Cotcher.
A five card hand of playing cards with all 4 tens 2008-06-25
From shari:
of the 2598960 different five card hands from a deck of 52 playing cards. how many would contain all ten's?
Answered by Janice Cotcher.
Acres and square miles 2008-06-25
From Jill:
how many miles are in 417 acres
Answered by Penny Nom.
A six sided lot 2008-06-25
From Zack:
Please help me find the area of my lot. I am sending a fax of the drawing and dimensions. Zack
Answered by Harley Weston.
A golf tournament 2008-06-24
From Roland:
We have a golf tournament, 12 players, three rounds. We want to pair so that we have pairings with as few duplications as possible.
Answered by Janice Cotcher.
Slope and rate of change 2008-06-23
From Lee:
What is the difference between a slope and a rate of change?
Answered by Stephen La Rocque.
An 18,000 mile automobile trip 2008-06-23
From rachel:
two men were about to start on an 18,000 mile automobile trip. The tires on their car were each good for 12,000 miles but not a foot more. how many spares will they need to go 18,000 miles
Answered by Stephen La Rocque.
A number that ends with 2 2008-06-23
From Jim:
A number ends in 2. If you were to place that digit at the beginning of the number, the resulting number will be twice the original. Find the smallest such number.
Answered by Stephen La Rocque.
Watts and kilowatts 2008-06-23
From Kenneth:
Hello: Here is my question: Find the kilowatt-hours of electricity used by a light bulb with a rating of 60 watts if the light is used for 10 hours.
Answered by Stephen La Rocque and Harley Weston.
The sum of the digits of a number 2008-06-23
From Ben:
Question: Using mathematical induction, prove that if the sum of the digits of a number is divisible by three, then the number itself is also divisible by 3.
Answered by Penny Nom.
I will be x years old in the year x squared 2008-06-22
From rachel:
you probably know someone who will x years old in the year x squared. When was that person born?
Answered by ha.
How many gallons in an oval shaped tank? 2008-06-22
From dale:
how many gallons in an oval shaped tank, 72in long x46.5in wide x23in deep
Answered by Penny Nom.
The height of an isosceles triangle 2008-06-22
From Evelyn:
Hi! I am facing problem finding the height of the isosceles triangle.
Answered by Penny Nom.
Two forces acting on a body 2008-06-21
From Rita:
Before answering the question below, what exactly is, in basic words, resultant force? QUESTION: Two forces of 50 and 68 pounds act on a body to produce a resultant force of 70 pounds. Find, to the nearest 10 minutes or nearest tenth of a degree, the angle formed between the resultant force and the smaller force.
Answered by Harley Weston.
sin(2x)/sin(3x) 2008-06-19
From matt:
how does sin2x break down (not with identities) and how would sin3x be created. My prob. is sin 2x/ sin 3x and I want to know how the double(or triple angle) would break down. I want to be able to cancel out sins. Thanks!
Answered by Harley Weston.
A circle inscribed in a square acre 2008-06-19
From Scott:
Q'm trying to find out the sq footage of the corners of an acre. If an acre is 43,560 sq and if I have this right the surface area of the circumference is 1040 ft. what is the combined square footage of the 4 corners? Or the percentage the original acre?
Answered by Harley Weston.
9(d+5)=3(d-3)+6d 2008-06-19
From davontay:
9(d+5)=3(d-3)+6d
Answered by Harley Weston.
A jar of pennies 2008-06-18
From Ryan:
if you are given a jar of pennies and you had to find the weight of one penny without opening the jar.when you graph it, You have to y= the mass of the container and x= number of coins you use the slope to find the weight of the penny how would u go about doing that?
Answered by Stephen La Rocque.
Interest compounded daily for 6 months 2008-06-18
From Cheryl:
Compounded interest daily on cd.....$1,000.00 at 2.53% for 6 months. How much will this CD earn?
Answered by Penny Nom.
Concrete blocks 2008-06-18
From marvin:
How many cement blocks do i need to build a 8 ft. tall wall x 1,300 ft. long?
Answered by Harley Weston.
A line which is perpendicular to another line 2008-06-18
From Emily:
How do you solve perpendicular bisectors with a slope and a point?
Answered by Penny Nom.
What is a 1% slope? 2008-06-17
From randy:
trying to figure out percentage of slope lets say i have to run a pipe 80' at a 1%slope how much pitch is that?
Answered by Harley Weston.
A space camera circles the Earth 2008-06-16
From Rita:
A space camera circles the Earth at a height of h miles above the surface. Suppose that d distance, IN MILES, on the surface of the Earth can be seen from the camera.

(a) Find an equation that relates the central angle theta to the height h.

(b) Find an equation that relates the observable distance d and theta.

(c) Find an equation that relates d and h.

(d) If d is to be 3500 miles, how high must the camera orbit above Earth?

(e) If the camera orbits at a height of 400 miles, what distance d on the surface can be seen?

Answered by Penny Nom.
Cows and chickens 2008-06-16
From damebochie:
I got this problem and want to write a system of equation for it:
Farmer Joe has cows and chickens on his farm. One day he count 76 legs and 24 heads. How many cows and how many chickens are on the farm? write a system of equations and solve.
I don't have any problem to solve it, but I just can;t see how I am going to write this system of equation. Thank you so much for helping?

Answered by Harley Weston.
Percent difference? 2008-06-14
From Cathy:
I am trying to verify a "percent of difference" as stated on a tax form. I cannot seem to come up with the equation to do so.
Here are a few examples on the form:
Last year $62.91 this year $66.05 tax difference $3.14 percent difference -5.64%
Last year $ 2014.97 this year $1982.93 tax difference $-32.04 percent difference 57.56%
Last year $97.66 this year $79.58 tax difference $-18.08 percent difference 32.48%
I have tried several different ways and per some of Q&A's already on your site I still have a problem and would appreciate some assistance.
Thanks-Cathy

Answered by Penny Nom.
Escort cards on a rectangular table 2008-06-13
From Susan:
What size rectangular table will we need to put 230- 2" X 3.5" escort cards on for my daughter's wedding reception with a spacing of .5 inch between them on all sides? Thanks so much for your help
Answered by Penny Nom.
A right prism with a square base 2008-06-13
From Malik:
Question from Malik: A company wants to alter the dimension of container. The new container is in the shape of right prism equivalent to the current one. The current container has height of 12cm and square base. The new container has also square base, but in the dimension base is increased by 3cm and height is decreased by 5cm. if the cardboard is used to manufacture the new container, what area of cardboard is used to make new container?
Answered by Penny Nom.
How many yards of concrete will I need? 2008-06-12
From Ron:
I am putting up a flag pole, and I am trying to fiqure out how much concrete I need for the job. I have a round cardboard cement tube which is 18" across and 24" tall. The bottom of the tube will be filled with 3" of gravel and 3" of sand and the rest will be filled with concrete. Could you tell me how many yards of concrete I need.
Answered by Harley Weston.
-2(x - 1)^2 + 8 2008-06-12
From kasondra:
y=-2(x-1)2[thats a square] +8 that need to be put in standard form
the answer is y= -2x2 -4x + 6
i got y= -2x2 -4x -4
i just don't get how my teacher got that answer

Answered by Penny Nom.
If the arc is 75mm, what is the radius? 2008-06-12
From malcolm:
If the are is 75mm, what is the radius?
Answered by Janice Cotcher and Harley Weston.
The current in a river 2008-06-12
From Joi:
To approximate the speed of the current of a river, a circular paddle wheel with radius 4 feet is lowered into the water. If the current causes the wheel to rotate at a speed of 10 revolutions per minute, what is the speed of the current? Express your answer in miles per hour.
Answered by Harley Weston.
The rate of change in the depth of the water 2008-06-12
From Liz:
A rectangular pool 50ft long and 30ft. wide has a depth of 8 ft. for the first 20 ft. for its length and a depth of 3 ft. on the last 20ft. of its length and tapers linearly for the 10 ft in the middle of its length. the pool is being filled with water at the rate of 3ftcubed/ min at what rate is the depth of the water in the pool increasing after 15 hours?
Answered by Harley Weston.
The length and width of a rectangle 2008-06-11
From gregg:
the perimeter of a rectangle is 70 feet. the ratio of length to width is 4:3. how would i calculate the length of the height and width?
Answered by Penny Nom.
x=(-14x+16)/x-8 2008-06-11
From Kory:
Solve the equation.
x=(-14x+16)/x-8

Answered by Penny Nom.
Rectangular cards on a round table 2008-06-11
From Susan:
How many 4" X 3.5" wide escort cards will fit on a 48" diameter table. These are for my daughters wedding. I have already added the extra spacing between the cards into the size of the cards.Thanks for your help
Answered by Janice Cotcher and Harley Weston.
The angles of a parallelogram 2008-06-11
From light:
ABCD is a parallelogram. If angle A. = (4x=17)degrees and angle B. = (3x-5)degrees , what is angle c?
Answered by Penny Nom.
The radius of a sphere if you know the volume 2008-06-11
From Cey:
how to i find the radius of a sphere with a volume of 1000cm cubed using the formula v=4/3 pi r squared???????/
Answered by Penny Nom.
The surface area of a lemon 2008-06-11
From Susan:
How would I find the surface area of a lemon
Answered by Harley Weston.
Double one penny for 30 days 2008-06-11
From kerwin:
one penny doubled for 30 days what is the answer?
Answered by Penny Nom.
What is the GST component? 2008-06-11
From Jilly:
I need to find a quick and easy formula for finding the gst (10%) of the total amount in $ terms, eg: the total is $500 what is the gst component?
Answered by Penny Nom.
What are the dimensions of the rectangle? 2008-06-11
From Denise:
A rectangle has a diagonal that is 3.6 feet longer than the length and 7.1 feet longer than the width. What are the dimensions of the rectangle?
Answered by Claude Tardif and Victoria West.
Packing a truck 2008-06-10
From Ann:
I am renting a 24ft box truck and have been told it is 1401 cubic feet. I want to know if I have 23 display cases to move and each is 6ft X 2ft X 3ft will the all fit? Or do I need additional info in order to figure this out?
Answered by Penny Nom.
Two circles 2008-06-10
From cey:
the diameter of the larger circle is 20cm, and the smaller 10cm. what is the shaded area??
Answered by Janice Cotcher.
How do i find the area of an irregular figure? 2008-06-10
From DAVE:
How do i find the area of an irregular figure?
Answered by Penny Nom.
.9 repeating plus .2 repeating 2008-06-10
From megan:
How do you express the addition of .9 repeating plus .2 repeating?
Answered by Penny Nom.
An ellipse and circle with the same area 2008-06-09
From Michael:
The area of the ellipse if equal to the area of a circle with radius=40 ft. Find the values of a and b using appropriate algebraic techniques, not basic math computations. A=pi ab and satisfy the constraint a+b=100????
Answered by Penny Nom.
How many different ways can cards fall when shuffling? 2008-06-09
From Twyla:
How many different ways can cards fall when shuffling? In other words, if you have 52 numbers, each one unique, how many different ways can they be arranged, using each number only once?
Answered by Penny Nom.
All wiffs are miffs and all miffs are kiffs. 2008-06-08
From Allison:
All wiffs are miffs. all miffs are kiffs. There are 25 wiffs. There are 76 kiffs. 33 kiffs are not miffs. How many miffs are not wiffs?
Answered by Penny Nom.
Arclength 2008-06-07
From prem:
i am engineer, i need the formula for calculate the arclength by using only chord length and radius only
Answered by Penny Nom.
g(m-1,2n)+n 2008-06-07
From Florence:
What does g(m-1,2n)+n for m>0, n>=0 mean.
Answered by Penny Nom.
90% of gross leaseable footage 2008-06-07
From Rosalind:
90% of gross leaseable footage in the shopping center, being 394,577, excluding that square footage leased by major anchor tenants, being 284,747. Would you deduct the 284,747 from the 394,577, then multiply by the 90%. Or would you multiply the 394,577 by the 90%, then deduct the 284,747?
Answered by Penny Nom.
How many gallons of oil will be needed to raise a 4" plunger 25 feet? 2008-06-06
From cory:
How many gallons of oil will be needed to raise a 4" plunger 25 feet?
Answered by Penny Nom.
The positive real zeros of f(x) = x^4 + x^3 - 7x - 1 2008-06-06
From CiCi:
How many positive real zeros does the function f(x) = x4 + x3 - 7x - 1 have?
Answered by Janice Cotcher and Harley Weston.
Filling a hole with sand 2008-06-05
From greg:
I have a 18 ft. diameter circle and need to put 2 in. of sand in it i need to know how much sand in cubic yards and tons
Answered by Harley Weston.
The mean and variance 2008-06-05
From Donny:
An investment will be worth $1,000, $2,000, or $5,000 at the end of the year. The probabilities of these values are .25, .60, and .15, respectively. Determine the mean and variance of the worth of the investment.
Answered by Harley Weston.
The volume of a hexagonal pyramid 2008-06-05
From Angie:
My son is working on a hexagonal pyramid. The base is 4 and the slant height is 6. We know that the answer is 48 times the sqrt of 2. We don't know how they got it. We attached the drawing of it.
Answered by Penny Nom.
A recursive formula for 9, -18, 36, -27, ... 2008-06-05
From Rita:
I am having trouble understanding how to write formulas (I should say create formulas) from a given sequence. It does not matter what sort of sequence it is. The confusion remains. I have not found a textbook or easy math book that explains this process for the average Joe to grasp. Here are the two questions:

(1) Write a recursive formula for the sequence
9, -18, 36, -72, ...

(2) Write a recursive formula for the sequence
3, 3(sqrt{3}), 9, 9(sqrt{3})

Answered by Penny Nom.
The two acute angles in a right triangle 2008-06-05
From Crystal:
In the right triangle the ratio of the measure of the two acute angles is 4:1. What is the measure in degrees of the larger acute angle?
Answered by Penny Nom.
How do you add decimals? 2008-06-05
From nyheashiacurry:
how do you add up decimals?
Answered by Janice Cotcher.
The path of a baseball 2008-06-04
From Tom:
How high would a batter have to hit a ball to clear the stadium roof in right field if the distance to the right field fence is 320 feet from home plate? Is this solvable?
Answered by Janice Cotcher.
The interior angles of an 18-sided polygon 2008-06-04
From Alfred:
How do I find the sum of the interior angles of an 18-sided polygon?
Answered by Janice Cotcher.
The volume of a solid 2008-06-04
From Tom:
The base of a certain solid is a circle of radius 2 while cross sections perpendicular to the base are isosceles triangles of height 1. What is the volume of the solid?
Answered by Harley Weston.
Three mutually tangent circles 2008-06-04
From Jacob:
If three circles are mutually tangent, does that mean that the two tangent lines are perpendicular?
Answered by Chris Fisher.
Calculating a price based on markup 2008-06-03
From Sheldon:
I know the most common form for retail businesses of calculating a price based on markup is: Cost/(1 - Markup), but how is this supposed to work for a business that desires a markup of over 100%.
Answered by Harley Weston.
A 4 sided lot 2008-06-03
From Pat:
If a lot of property is: front 127.87, back 156.17, one side is 129.73, the other side is 125.00, then how much total acreage is this.
Answered by Harley Weston.
The number of hours of daylight 2008-06-03
From Marilyn:
Hi, could you please help me with this question? In a city (in the Northern Hemisphere) the minimum number of hours of daylight is 9.6 and the maximum number is 14.4. If the 80th day of the year (March 21) has 12 hours of daylight, determine a sine function which gives the number of hours of daylight for any given day of the year. (Jan 1 = 1, Jan 2 = 2, etc). Thank you!
Answered by Harley Weston.
A quadratic equation 2008-06-03
From Drew:
A solution of x^2-8x=-17 is

-4 or -4+I or 4 or 4+i

Answered by Janice Cotcher.
The area of a regular pentagon 2008-06-02
From Jenna:
I would like to know if there is a formula to find the area of a regular Pentagon that is not inscribed in a circle only apothem and side length are given.
Answered by Penny Nom.
Substitution into a formula 2008-06-02
From beth:
Given q(x) = 7xsquared + 9, find q(-7)

These were the choices I had for the answers to this problem
1.) 343
2.) -40
3.) 334
4.) -89
5.) 352

Answered by Penny Nom.
A bridge is built in the shape of a parabolic arch 2008-06-02
From megan:
A bridge is built in the shape of a parbolic arch. The bridge has a span of 192 feet and a maximum height of 30 feet. Find the height of the arch at 20 feet from its center. I need the equation and what to fill into the equation...please and thankyou!
Answered by Penny Nom.
Lowest common denominator 2008-05-31
From marlene:
cant get the common lowest denominator of 10,46,64
Answered by Janice Cotcher.
Substitution method 2008-05-30
From taquicia:
2y-x=3
x=5+y

Answered by Janice Cotcher.
One chord is twice as long as another 2008-05-30
From Stacey:
In a circle, If one chord is twice as long as another, is the longer chord's arc also twice the size of the smaller chord? True or False
Answered by Chris Fisher.
x/4 = 3 1/2 2008-05-30
From Kelsey:
How do you solve for "X" in the problem below?

X
--- = 3 1/2
4

--Kelsey

Answered by Victoria West.
4x^3 - 6x + 2 = 0 2008-05-28
From paul:
4x^3 - 6x + 2 = 0
Answered by Janice Cotcher.
The surface area of an "egg shaped" object 2008-05-28
From Tom:
How do you calculated the surface area of an "egg shaped" object (ovoid) ?
Answered by Harley Weston.
Scale factor 2008-05-28
From Riley:
I have a math problem, and a big test coming up on Friday or Thursday (my teacher hasn't decided yet.) I need help understanding how I would do this problem. Here it is: they wanted me to find the perimeter and area of a rectangle when they only give me the scale factor. I've delt with scale factors before, but this is quite confusing. I have done the scale factor way by having a rectangle that maybe has a width of 10 and a height of 20, and I needed to find what the scale factor would be for a rectangle with the width of 5 and a height of 10. I know that they scale factor would be 2, right? But this just gives me the scale factor, and I have to FIND the perimeter and area. Hopefully you can get back to me tonight.
Answered by Harley Weston.
The area of a rectangle 2008-05-27
From Becky:
the area of a rectangular lot is represented by 6t^2+13t-5. What are the length and the width of the rectangular lot?
Answered by Harley Weston.
The length of a shadow 2008-05-27
From Simon:
A figure skater is directly beneath a spotlight 10 m above the ice. IF she skates away from the light at a rate of 6m/s and the spot follows her, how fast is her shadow's head moving when she is 8m from her starting point? The skater is (almost) 1.6m tall with her skates on.
Answered by Stephen La Rocque and Harley Weston.
A cat is walking on the tread of a tank 2008-05-26
From Kathleen:
If a cat is walking on the tread of a tank travelling at 28kmh how fast does the cat have to walk to avoid falling under the tracks?
Answered by Janice Cotcher.
Convert cubic inches to gallons 2008-05-26
From Rosemarie:
How do you convert cubic inches to gallons? Thanks for your help.
Answered by Penny Nom.
I want to fill my backyard slope with fill dirt. 2008-05-25
From Piero:
I want to fill my backyard slope with fill dirt. The slope is 50 feet wide and the distance from the top of the top of the slope to the bottom is about 15 feet deep and the slope angle is at a 45. I want to know how many yards of dirt do I need to fill a space that is 20 feet out, 15 feet deep at a 45 degree angle.
Answered by Stephen La Rocque.
Find a way to represent the length and width of the box. 2008-05-25
From Angela:
The volume of a box that is two inches high can be represented by V=2X^2-20x+32. Find a way to represent the length and width of the box. (Hint: you will need to use factoring).
Answered by Stephen La Rocque.
Liquid is being pored into the top of a funnel 2008-05-25
From Stella:
Liquid is being pored into the top of a funnel at a steady rate of 200cm^3/s. The funnel is in the shape of an inverted right circular cone with a radius equal to its height. It has a small hole in the bottom where the liquid is flowing out at a rate of 20cm^3/s. How fast is the height of the liquid changing when the liquid in the funnel is 15cm deep?
At the instance when the height of the liquid is 25cm, the funnel becomes clogged at the bottom and no mo re liquid flows out. How fast does the height of the liquid change just after this occurs?

Answered by Stephen La Rocque.
Ten acres 2008-05-25
From RICHARD:
hi, i am getting 10 acres of ground from a aunt which is going to be taken out of a 50 acre tract.both the north and south lines are 1320 feet long.i need to know how long the east and west lines will be to make the 10 acres and what the square footage will be for the 10 acres
Answered by Harley Weston.
Gasoline in a cylindrical tank 2008-05-25
From Simon:
A cylindrical tank with height 15m and diameter 2m is being filled with gasoline at a rate of 500L/min. At what rate is the fluid level in the tank rising?(1L = 1000cm^3). About how long will it take to fill the tank?
Answered by Penny Nom.
A triangle and a square 2008-05-24
From s:
how to construct a triangle whose area is equal to the area of a given square?
Answered by Penny Nom.
Applications of trigonometry 2008-05-24
From Mohita:
I have got a project in the school and i am not getting anything about the topic. The topic is that we need to find the application of trigonometry on any one of the real life situations using 3-dimensional figures. I mean how can trigonometry can be used in real life situations like navigation, architecture, survey, astronomy etc.
Answered by Penny Nom.
3 radical 48 +11 radical 75 2008-05-24
From Maria:
Add 3 radical 48 +11 radical 75. Write answer in simpliest form.
Answered by Penny Nom.
What is the length of the 3rd side? 2008-05-23
From lee:
If you have a triangle 12ft across the bottom then on the right is a 90 degree angle and that side is 18ft tall. What is the length of the 3rd side?
Answered by Penny Nom.
Interest compounded daily 2008-05-23
From Jeanmarie:
WHAT IS THE INTEREST IN $ ON 107,495.10 AT AN INTEREST OF 5.13% FOR COMPOUNDED DAILY FOR 16 DAYS?
Answered by Penny Nom.
How many bricks I can place around a 26-inch circle? 2008-05-22
From Jon:
I want to know how many bricks I can place around a 26-inch circle? There must be a formula other than trial and error. The length of the bricks is 6-inches. [How many 6-inch tangents can be in a 26-inch circle? Thank you very much. Jon
Answered by Harley Weston.
limit x->0 (logx-log(x-1)) 2008-05-22
From kritica:
lt x->0 (logx-log(x-1))
Answered by Harley Weston.
Inverse variation 2008-05-22
From leria:
In Boyle's law, the pressure of gas varies inversely with the volume of the gas. A volume of 11.2 liters of air exerts a pressure of 2.0 atmospheres.

determine the constant of variation and write an equation the models the relationship between the pressure of the gas P and its volume, V. Use the equation to predict the volume of air necessary to exert a presure of 0.25 atmospheres.

Answered by Janice Cotcher.
LCM and GCD 2008-05-22
From kasirao:
Please help me in knowing the proof for the equality, "The product of two natural numbers is equal to their LCM and GCD"
Answered by Penny Nom.
Double the the dimensions of a rectangle 2008-05-22
From Destiny:
Double the the dimensions of a rectangle that is 3 by 4 in. What is the area?
Answered by Penny Nom.
A 9 team flag football tournament 2008-05-22
From James:
I like to run a 9 team flag football tournament where each team is guaranteed 3 games prior to playoffs. All games must have playoff implications and must not be a consolation game.
Answered by Victoria West.
A season schedule for 9 teams 2008-05-22
From James:
would like to put together a season schedule for 9 teams with each team playing three or 4 games leading to a 1 to 9 ranking.
Answered by Victoria West.
Volume/weight of a salt shedweight 2008-05-21
From James:
How do I find the volume/weight of a salt shed. I have a 3000 ton barn, I need a better way of finding how much salt to order as it gets used. I know how to calculate volume, how do i get tonnage from there??
Answered by Harley Weston.
Covering the bottom of a pool 2008-05-21
From Chris:
I need to know how many sheets of 4 foot by 8 foot by 1 inch foam I will need to cover the bottom of a 27 foot diameter pool? Thanks!
Answered by JaniceCotcher.
Nine digit numbers 2008-05-21
From Alex:
List of Nine digit numbers, that can be divided by nine?
Answered by Janice Cotcher.
1+1/1+1/x 2008-05-21
From nadean:
1+1/1+1/x
Answered by Penny Nom.
Throwing a ball on the moon 2008-05-21
From leria:
equation for motion of moon is h=2.67t^2 + vt + s equation for earth h=-16t + vt + s

Suppose you are on an outpost on the moon and Jan is back on Earth. Both people toss a ball from a height of 96 feet with an initial velocity of 16 feet per second. How much longer will your ball stay in motion on the moon than Jan's on the earth?

Math Central,

I'm having a difficult time understanding which numbers to plug into the formula.

Answered by Stephen La Rocque.
$16000 at 7% per annum for six months 2008-05-21
From Delbert:
If my principal balance if $16000.00 and I paid 7% interest per annum but only kept the loan for 6 months How much interest would I owe after 6 months.
Answered by Harley Weston.
A weird shaped lot 2008-05-21
From James:
Need to find square foot of a residential lot on a cul-de-sac. Weird triangular shape. Measurements : 126.18 170.08 138.2 48.4
Answered by Harley Weston.
The angles of an irregular pentagon 2008-05-20
From victoria:
The sum of the measure of two angles is 240. If the remaining angles are congruent, what is the measure of each angle?
Answered by Penny Nom.
The cube of -2y in exponential form 2008-05-19
From Xiu:
I dont get this question.. Write "the cube of -2y" in exponential form. I know the answer is -(2y)^2 but i really don't understand why. Can you explain how to do this whole problem for me?
Answered by Leeanne Boehm.
Converting a decimal to a fraction 2008-05-19
From judy:
please explain how to get from the decimal number .3437 to the fraction 11/32
Answered by Leeanne Boehm.
Guy wires for a tower 2008-05-19
From larissa:
a radio tower 500 feet high is located on the side of a hill ( the hill has an inclination to the horizontal of 5 degrees.) How long should two guy wires be if they are connected to the top of the tower and are secured at two points 100 feet directly above ( up the hill ) and directly below the base of the tower?
Answered by Penny Nom.
A triangle inscribed in a semicircle 2008-05-19
From Larissa:
Find the area of the shaded region outside of a triangle inscribed (meaning the all three points of the triangle are on the circle ) in a half circle of diameter 10 inches, if one side of the triangle is the diameter and the other side is 8 inches long. (A triangle that is inscribed in a triangle is a right triangle by definition.)
Answered by Penny Nom.
Elevation gain 2008-05-18
From Mark:
How do you calculate elevation gain on a treadmill? e.g. 5.0 miles (26,400ft) @ 15% incline. I have been estimating it as 26,400 x .15 = 3,960 ft of elevation gain. Hopeful Mount Rainier Climber
Answered by Victoria West.
A hemisphere and a cylinder 2008-05-18
From Libby:
A giant scoop, operated by a crane, is in the shape of a hemisphere of radius 24 inches. The scoop is filled with molten steel. The steel is then poured into a cylindrical storage tank with diameter 32 inches. How high will the molten steel rise?

I got the volume of the hemisphere and the volume of the cylinder, but I'm not exactly sure what to do next.

Answered by Penny Nom.
Which is larger, 727464^512693 or 624610^518548? 2008-05-18
From Yahweh:
Lets say I have two very large numbers in the form of a^b: 727464^512693 and 624610^518548

I want to calculate which number is larger, but its unreasonable to type these numbers into a spreadsheet to get an answer right away. How could I determine the larger or the two numbers efficiently?

Answered by Stephen La Rocque.
The wedges in a circle graph 2008-05-18
From Libby:
I don't understand how to do this word problem: The cafeteria workers distributed a survey to the student body asking students to pick 1 from a list of 5 choices for their favorite lunch. The circle graph below gives the results of the survey for the students who responded. What is the measure, in degrees, of the central angle for each choice?

And there is a pie chart that has food choices with these percentages: 25%, 20%, 15%, 10%, and 30%

Answered by Leeanne Boehm and Penny Nom.
A right angled triangle 2008-05-16
From bryan:
The sides of a right angled triangle are x, x+1, x+2 cm. Find the perimeter.
Answered by Penny Nom.
Girls, cats and kittens 2008-05-16
From leria:
There are 7 girls in a bus. Each girl has 7 backpacks In each backpack, there are 7 big cats For every big cat, there are 7 little cats.

How many legs are there in the bus? One hint? it's a 6 digit number

Answered by Penny Nom.
The length of an arc 2008-05-16
From Don:
I am trying to determine the length of an arch with a two foot backset from 32 foot frontage.
Answered by Harley Weston.
The sides of a rectangle 2008-05-16
From bill:
The area of the rectangle is 170 sq.ft. the width is unknown and the length is the width plus 7 ft. I set it up like this w(w+7)=170 and this is where I get lost. Thanks, Bill
Answered by Penny Nom.
Fencing around a playground 2008-05-15
From Vanessa:
If the playground is a rectangle with it's width 10ft shorter than it's length, and it can be enclosed by exactly 140ft of fence, what is the length and width of the playground?
Answered by Penny Nom.
Tyres in a shipping container 2008-05-14
From Carlene:
I need to work out how many tyres will go in a shipping container. The shipping container is 10 foot long, 8 foot wide and 8.6 foot high. The tyre is 175 millimeters across and it is 14 inches in tyre thickness. How many tyres will fit in my container?
Answered by Stephen La Rocque.
The fourth side of a property 2008-05-14
From Penny:
Property has 3 measurements and 2 angles and I want the altitude at a particular point. Corners are sw, nw, ne and se. 1 want the 4th saide measurement confirmed (we think 146.07') and the altitude measurement at 25' from sw (on the sw-se side) The angles provided are interior at nw 67.38 degrees and the opposing corner se interior appears to be 68.58 degrees. the 3 sides with measurements are: sw to nw=54.07'. sw to se 146.07' and se to ne 53.57'. If you would help, we would really appreciate it. Thanks! Penny
Answered by Harley Weston.
The length of an arc 2008-05-12
From patricia:
Find the length of the arc on a circle of radius r = 16 inches intercepted by a central angle [theta] = 60 degrees
Answered by Penny Nom.
The weight of a concrete column 2008-05-11
From russell:
a cylindrical form is filled with a slow curing concrete. The base of the form is 10 ft in radius, and height is 25 ft. while the concrete hardens, gravity causes the density to vary from a density of 90 lbs/ft^3 at the bottom to a density of 50 lb/ft^3 at the top. Assume that the density varies linearly from the top to the bottom, and compute the total weight of the resulting concrete column
Answered by Harley Weston.
How old are the sisters? 2008-05-11
From George:
Theresa is four years older than her younger sister Joni. Twelve years ago, she was twice as old as her sister. Find the present age of the two siblings.
Answered by Penny Nom.
Selling price 2008-05-10
From Roxane:
I'm trying to figure out the formula for the following, I have a product that I want to receive a certain profit margin, but with this product I also have to pay a percentage of commission to someone. I need to know how to calculate my new selling price taking into account the markup percentage, but I don't want my gross margin to be lowered. I have calculated the markup percentage to get my new selling price with the commission, but it lowers my profit margin once I pay the commission and I don't want it to do that.
Answered by Penny Nom.
The area of a regular hexagonal pyramid 2008-05-10
From Kim:
i need help finding the area of a regular hexagonal pyramid.
Answered by Penny Nom.
The area of a sector of a circle 2008-05-10
From lupio:
a problem asks for the area of a sector in a circle,no central angle,in circle is given only 5cm radius, three answers 180-90-35
can't find a way to start.Help

Answered by Harley Weston.
A crate is launched from an airplane 2008-05-10
From Michelle:
A crate is launched from an airplane, 75 feet above the ocean, with an initial vertical velocity of 60 feet per second. How many seconds will it take for the crate to hit the water? Round to the nearest hundredth.
Answered by Harley Weston.
Options 1 2 4 8 16 32 64 128 etc. 2008-05-09
From Abraham:
I have seen programs with options and I wish to create a similar program.

I have options 1 2 4 8 16 32 64 128 etc.
If I receive option 3 means option 1 and 2 are included. Option 20 means option 16 and 4 are included. that because of the sum of those numbers is the olny way to get the given number.

I want to know if is there a way
to know which number from options are included.
How I know:
1 is Included
2 is Included
4 is Included

Answered by Stephen La Rocque.
A linear model 2008-05-09
From Shelby:
How would you write a linear model to represent the population of a city that has a population of 547,725 and a growth rate of -25,195 per year with t represents the number of years since 1994?
Answered by Penny Nom.
Excluded values 2008-05-09
From Andrew:
Finding Excluded Values

j+1
----
2j-6

Answered by Penny Nom.
The volume of a ditch 2008-05-09
From Dixie:
I need to find the cubic feet of a ditch 36 inches wide by 28 inches high, by 1561 feet long. then I need to convert it to cubic yards then to tons. of rock
Answered by Penny Nom.
A quadrilateral 2008-05-08
From reena:
two exterior angles quadrilateral are 100 degrees 110 degrees each. two remaing interior one angle twice size other find size of each angle
Answered by Penny Nom.
How can the distance between two points be irrational? 2008-05-07
From Dawn:
Yesterday my advanced 7th grade pre-algebra students were finding the distance between two points on a coordinate grid using the distance formula. A student asked me how come we or math I guess is able to represent a finite distance with an irrational number. I found this to be a very interesting question. I and the 7 other math teachers on my staff had discussed this and we have a variety of thing to say to the kid, but nothing that amounts to an actual answer. The students have studied what it means for a number to be irrational, and they cannot understand how that can "measure" a line that obviously stops. Got any suggestions?
Answered by Stephen La Rocque, Penny Nom, Claude Tardif and Victoria West.
seven + eight = twelve 2008-05-07
From evelyn:
My son's teacher has given the children this question to answer and I do not have an anser.

seven + eight = twelve

Answered by Stephen La Rocque.
An isosceles triangle 2008-05-07
From Jay:
Given: An Isosceles Triangle with the area=16m squared. What would be the length of each leg.
Answered by Stephen La Rocque.
The sides of an octagon 2008-05-07
From Frank:
Question from Frank, a parent: Hi, Need to lay out a 10 ft. diameter octagon on the ground. What is the length of the sides ? And the angles ?
Answered by Harley Weston.
The distance from the chord to the centre 2008-05-04
From sle:
What is the method for figuring this type of question out? in a circle whose radius is 25, a chord has a length of 40. find the distance of this chord from the center of the circle
Answered by Penny Nom.
A root of a polynomial 2008-05-03
From john:
here is a word problem that my teacher gave us to do can you help me understand it

Give possible formulas (in factored form) and sketch the graphs for two different polynomials that have roots -3, -1, and 2, but cross the x-axis only at x=2 and have y-intercept 4.5. What is the lowest degree such a polynomial can have? Can it have an even degree? An odd degree? Explain why.

her term "root" has completely thrown me off, i've reviewed my book and cant find what she is referring to. can anyone help?

Answered by Penny Nom.
The inverse of a function 2008-05-03
From keith:
please help me find the inverse of this function:

h(t) = 2 + 4 ln(1-5t)

Answered by Stephen La Rocque.
(1-i)ln(1+i) 2008-05-02
From Kim:
I am stuck on the expansion of (1-i)ln(1+i)=(1-i)[ln(square root of 2)+i(3.14/4 = 2n3.14)]
Answered by Harley Weston.
How many gallons are in 1,500 liters? 2008-05-01
From bambi:
[How many gallons are in 1,500 liters?
Answered by Penny Nom.
Division by zero 2008-05-01
From Bill:
Why does multiplication have a favored status so as to make division by zero undefined?
Answered by Harley Weston.
1/3-5/6=1/x, solve for x 2008-05-01
From Lavada:
1/3-5/6=1/x How do I solve this problem.
Answered by Penny Nom.
How do you reduce an equation with multiple variables? 2008-04-30
From Jonathon:
How do you reduce an equation with multiple variables?

For example, if 3x + y = k(x-3), what would x be equal to?

Answered by Penny Nom.
The standard form of an ellipse 2008-04-30
From Rebecca:
I have to write the following equation into standard form of an ellipse:

9(X-1)^2 + (Y+1)^2 = 1

Answered by Stephen La Rocque and Harley Weston.
(9x^2-4)/(3x^2-5x+2) * (9x^4 - 6x^3 +4x^2)/(27x^4+8x) 2008-04-29
From Jonathon:
I don't remember how to solve an equation in the form of the following:

(9x^2-4)/(3x^2-5x+2) * (9x^4 - 6x^3 +4x^2)/(27x^4+8x) =

Answered by Stephen La Rocque.
Fencing a triangular field 2008-04-29
From Julieta:
The area of a field shaped like a triangle is 750yards squared. One leg of the field is fenced with wood costing $5/yd. The remainder of the perimeter of the field is fenced with steel costing $10/yd. The perimeter of the field is 150yards. the total cost of the fencing is $1200.
a. What is the length of the leg fenced with wood?
b. what is the length of the leg fenced with steel?

Answered by Penny Nom.
The ratio of their perimeters to two regular hexagons 2008-04-29
From td:
Two regular hexagons have areas of 54 square root of 3 and 150 square root of 3, what is the ratio of their perimeters?
Answered by Stephen La Rocque.
Adding rational functions 2008-04-29
From Jonathon:
1/(x+3) + 1/(x^2+5x+6) =
Answered by Penny Nom.
The Pythagorean theorem with triangles rather than squares 2008-04-29
From Zachary:
I need to figure out how to prove the pythagorean theoorem using equilateral triangles instead of using square. I know that A^2+B^2=C^2, but how do you get that by using equilateral triangles. I know the area of a triangle is BH1/2=Area. So what i need to know is how to derieve the formula of a triangle to get the pythagorean theorem
Answered by Penny Nom.
A bowl is the shape of a hemisphere 2008-04-28
From josh:
a bowl is the shape of a hemisphere with diameter 30 cm and water is poured into the bowl to a height h cm. how do i find the volume of the water in the bowl
Answered by Harley Weston.
The diameter of a circle 2008-04-28
From Mary:
What is the diameter of a circle if the area is 132.7 square meters? I have tried the formula C=2 times pi radius and came up up 21.1. I also tried C = pi times d and came up with 42.26. The answer in the book says 13. How did they get this answer? Thank you for your help.
Answered by Penny Nom.
A lidless box with square ends 2008-04-28
From Chris:
A lidless box with square ends is to be made from a thin sheet of metal. Determine the least area of the metal for which the volume of the box is 3.5m^3. I did this question and my answer is 11.08m^2 is this correct? If no can you show how you got the correct answer.
Answered by Stephen La Rocque and Harley Weston.
A rectangle inscribed in a circle 2008-04-27
From sridhar:
A rectangle with perimeter 28 cm inscribed in a circle of radius 5 cm find the area ?
Answered by Penny Nom.
An octagonal prism 2008-04-27
From Melanie:
My son is identifying geometric shapes in the real world? We are stuck on octagonal prism, rectangular prism and square prism. Can you help me out with some examples. Thanks
Answered by Penny Nom.
A circle inscribed in a triangle 2008-04-26
From Amar:
I have been given a circle inscribed in a triangle and have been told to prove that the ratio of the perimeter of the triangle to the circumference of the circle is the same as the ratio of the area of the triangle to the area of the circle. How would this be done?
Answered by Stephen La Rocque and Walter Whiteley.
Two overlapping circles 2008-04-26
From Michelle:
Two overlapping circles O and Q have the common chord AB (vertical line between the overlapping circles). If AB is 6 and circle O has a radius of length 4 (horizontal line going through the overlapping circles and touching the side of the circle) and circle Q has a radius of length 6, how long is OQ.
Answered by Penny Nom and Walter Whiteley.
At what value of t is the maximum acceleration? 2008-04-25
From Mary:
Velocity of a function (which is the first derivative of its position) is defined over the interval 0 to 12 using the following piecewise function: v(t)=-1 from 0 to 4, v(t)=x-5 from (4 to 8 and v(t)=-x+11 from (8 to 12. At what value of t is the maximum acceleration?
Answered by Stephen La Rocque.
1?2?3?4 = 7 2008-04-25
From john:
1 2 3 4 = 7

The numbers have to stay in order of 1234 and when added,subtracted,multiplied or divided they have to equal 7.

Answered by Stephen La Rocque.
Related rates 2008-04-25
From Mary:
A rectangular box is 10 inches high. It's length increases at a rate of 2 inches per second and it's width decreases at the rate of 4 inches per second. When the length is 8 inches and the width is 6 inches, what is the rate of change of the volume?
Answered by Stephen La Rocque.
-5x < 25+10 2008-04-25
From Courtney:
What is the answer to -5x<25+10
Answered by Stephen La Rocque.
The radius of a circle 2008-04-25
From kathy:
How do you find the radius of a circle if the area is 803.84 and using 3.14 for pi.
Answered by Penny Nom.
For what values of a does x^3-13x+a have 3 distinct roots? 2008-04-25
From Jyotishman:
What can be the value of a for the equation to have 3 roots?
x^3-13x+a=0

Answered by Harley Weston.
An irregular tetrahedron 2008-04-24
From RAUL:
I am looking an expression for an edge length as function of the other five edge lengths of irregular tetrahedron.
Answered by Walter Whiteley.
15 + 0.10n = 0.25n, solve for n 2008-04-24
From monica:
15 + 0.10n = 0.25n

Solve for n.

Answered by Penny Nom.
Find the length and the width of the rectangle 2008-04-24
From Vickie:
A rectangle is 2 times longer than it is wide. It has a diagonal length of 50 centimeters. Find the length and the width of the rectangle. Round your answers to the nearest tenth. Show your work.
Answered by Penny Nom.
Naming a figure 2008-04-24
From P.:
What is the name of a figure that has 2 sides that measure 25 feet each and 2 sides that measure 10 feet each?
Answered by Penny Nom.
A volume of revolution 2008-04-24
From Sabahat:
Hi, i have a region enclosed by both axes, the line x=2 and the curve y=1/8 x2 + 2 is rotated about the y-axis to form a solid . How can i find the volume of this solid?. (Please note that y equation is read as y =1 over 8 times x square plus 2.) I will be really grateful if you answer this question. :)
Answered by Harley Weston.
How many candles are on dave's birthhday cake? 2008-04-23
From karen:
on daves birthday, his brother henry is 17 years younger than 3 times dave's age,. the boys father ,tim is 17 years older than twice henry's age if dave is 7 years younger than his brother. how many candles are on dave's birthday cake?
Answered by Chris Langdon and Stephen La Rocque.
An open box 2008-04-23
From Le:
Metal Fabrication; If an open box is made from a tin sheet 8 in square by cutting out identical squares from each corner and bending up the resulting flaps, determine the dimensions of the largest box that can be made.
Answered by Harley Weston.
A 6 digit number 2008-04-23
From rajiv:
write the smallest 6-digit number using three different digits with 6 in the ten thousands place and 1 in the hundreds place
Answered by Penny Nom.
What is the altitude of the plane? 2008-04-23
From tiana:
If a plane is taking off and its climbing at 380 feet per minute at 205 MPH, what is the altitude of the plane after it traveled 115 miles?
Answered by Penny Nom.
The weight of a bar of steel 2008-04-22
From Himmler:
if a steel weighs 0.30 pounds per cubic inch, then the weight of a 2 inch square steel bar 90 inches long is------------pounds.
Answered by Penny Nom.
How is the square root of 3/4 is greater than 3/4? 2008-04-22
From Serena:
How is the square root of 3/4 is greater than 3/4?
Answered by Penny Nom.
What is the integral of 13sin^3(x)*cos^7(x)dx? 2008-04-22
From Cathrine:
I am having trouble integrating this problem. It says to evaluate the integral but I don't know what to do or how to do it.

It is the integral of
13sin^3(x)*cos^7(x)d

Answered by Harley Weston.
Find the quadratic equation with roots at (3-i) and its complex conjugate 2008-04-22
From Tiffany:
Find the quadratic equation with roots at (3-i) and its complex conjugate.
Answered by Penny Nom.
The isoperimetric inequality 2008-04-21
From Dutch:
I'm searching for the proof that the sphere has the smallest volume of any figure and maximizes the volume of any figure.
Answered by Chris Fisher.
f(x)=sin^3(3x^2) find f ' (x) 2008-04-21
From Michael:
f(x)=sin^3(3x^2) find f ' (x)
Answered by Harley Weston.
The perimeter of a half acre square 2008-04-20
From s:
Measured in feet. What would be the perimeter of a square area of land measuring 1/2 acre?
Answered by Penny Nom.
The sum of the ages of Petra and her mother is 45 2008-04-20
From Wendy:
The sum of the ages of Petra and her mother is 45. Her mother is 12 years more than twice as old as Petra. How old are Petra and her mother?
Answered by Leeanne Boehm.
x/3 -5 = 3(x-2) 2008-04-20
From Mrs:
I am having trouble helping my son solve the following equation:

x/3 -5 = 3(x-2)

Answered by Penny Nom.
A water alcohol mixture 2008-04-20
From Mary:
How many oz. of solution that is 90% alcohol ( and 10% water) is needed to be mixed with 5 ounces of a solution that is 50% alcohol in order to obtain a solution that is 80% alcohol?
Answered by Penny Nom.
The dimensions of a rectangle 2008-04-20
From Brandy:
Find the dimensions of a rectangle with a primeter of 54 cm if the length is 5 less than three times the width.
Answered by Penny Nom.
Simplify 5x^5/8 / 15x^2/12 2008-04-18
From Kyler:
5x^5/8 / 15x^2/12 Divide the rational expressions. Simplify the results.
Answered by Penny Nom.
A polynomial equation 2008-04-18
From Joan:
Find all polynomials P(x) satisfying x^2P(x) + P(1-x) = 2x-x^4
Answered by Harley Weston.
A solid figure with 3 rectangular faces and 2 triangular faces 2008-04-17
From Emily:
I am in the 3rd grade and my teacher gave us a question. What is a solid figure with 3 rectangular faces and 2 triangular faces?
Answered by Penny Nom.
Two intersecting circles 2008-04-17
From Muhammad:
Hi, how can we find the perimeter and area of the region common to two intersecting circles of radii 6 cm and 4 cm with centers 7 cm apart.
Answered by Harley Weston.
Integrate 2x(x^2-1)^5 dx 2008-04-17
From joey:
pls help me to integrate
2x(x^2-1)^5 dx using the substitution u=x^2-1

Answered by Harley Weston.
The height of a triangle 2008-04-17
From Trent:
I need to know the area of a triangle. One side is 5 the base is 12 and the angle between them is 52 degrees. No height is specified. How do I find the height to get the area?
Answered by Penny Nom.
The volume of a stone 2008-04-17
From sara:
a rectangular tank 30 cm long and 20 cm wide is filled with water to a depth of 8 cm. When a stone was put in, the water level rose to 11 cm. Find the volume of the stone. (Assume that the stone is completely under water.)
Answered by Penny Nom.
A 30-60-90 triangle 2008-04-16
From Ron:
30-60-90 triangle. Base is 2.75 how do I find the length of the other two sides?
Answered by Penny Nom.
Excluded values 2008-04-16
From Jenny:
Give the excluded values for the fraction m+5/mn+3m

m=
n=

Answered by Penny Nom.
The three sides of a triangle 2008-04-16
From Bridgett:
The sum of the lengths of any 2 sides of a triangle must be greater than the third side. If the triangle has one side that is 11 CM, and the second side of the triangle is 4 cm less than twice the third side, what lengths do the 2nd and 3rd side have to be?

between 0000-00-00 and 9999-99-99


Answered by Stephen La Rocque.
The area of an asphalt parking lot 2008-04-16
From Lidia:
An asphalt parking lot measures 100m x 500m. Calculate the area in square meters.
Answered by Penny Nom.
Chord, radius, arc length and central angle 2008-04-15
From Cindy:
There is a railroad curve with a chord length of 2000 ft. and a central angle of 35 degrees. What is the radius and arc length of the circular arc?
Answered by Stephen La Rocque.
Escalator challenge 2008-04-13
From Corey:
Walking up a broken escalator (escalator not working) takes 75 seconds. Riding up the same escalator (not walking) when it is working takes 45 seconds. How long would it take a person to walk up the escalator when it is working ?
Answered by Stephen La Rocque.
The area bounded by 3 curves 2008-04-13
From Sabahat:
Hi, I have enclosed a diagram.
The diagram shows the curve y=(2x-5)4. The point P has co-ordinates (4,81) and the tangent to the curve at P meets the x-axis at Q.

Find the area of the region (shaded in the diagram) enclosed between the curve, PQ and the x-axis . (Please note that the equation y is read as y=2x -5 whole raise to power 4.)

Answered by Stephen La Rocque.
A tangent to two circles 2008-04-13
From erson:
find the length of the tangent segment AB to two circles whose radii are a and b respectively, when the circles touch each other.

the illustration looks like this...hope you'll understand... there are 2 circles - one is big one is small. they touch each other. and there is this irregular 4 sided polygon that connects them...there is a line that connects them from their center point and another from the tip of the circles...and that's it...i cannot explain very well please bear with me

Answered by Stephen La Rocque.
A tangent to a circle 2008-04-13
From rogerson:
Line t from point P is tangent to circle O at T, the point of tangency. Find the length of PT when the radius of the circle is 5cm and the distance between points P and O is 8cm.
Answered by Stephen La Rocque.
A coin is tossed 30 times 2008-04-13
From seulki:
coin tossed 30 times, what is the probability that the heads show up fewer than 17 times?
Answered by Harley Weston.
f(x) =ax^blnx 2008-04-13
From charles:
supposef(x) =ax^blnx is a real- valued function. Determine exact values(not decimal approximations) fro nonzero constants a and b so that the function f has a critical point at x=e^3 and a maximum value of 1/2e
Answered by Harley Weston.
A parallel line to a given line 2008-04-12
From john:
A parallel line to a given line through a given point not on the a given line
Answered by Harley Weston.
The maximum area of a pizza slice 2008-04-12
From charles:
A slice of pizza in the form of a sector of a circle has a perimeter of 24 inches. what value for the radius of the pizza makes the slice largest[when o is the central angle in radians, the area of the sector is given by A= r^20/2and the length on the circle is given by s=r0
Answered by Harley Weston.
log2x+log(x+4)=1 2008-04-12
From ryan:
solve the equation for x, log2x+log(x+4)=1
Answered by Penny Nom.
The capacity of a silo 2008-04-12
From Candonn:
I have a cement silo that is 30 ft. high and has a diameter of 12 ft. I was wondering what the capacity of the silo is in tons of dry material. The silo is flat on the bottom also.
Answered by Penny Nom.
Jill and Matt run on a 400 meter long oval track 2008-04-11
From jamie:
Jill and Matt run on a 400 meter long oval track. Jill and Matt each run 10 laps daily and both run in the same lane. One day Jill and Matt began at the same point, but ran around the track in opposite directions. Jill ran at a constant speed that was 2 meters per second faster than matts constant speed. Jill passed Matt for the first time in 40 seconds. Jill ran at a constant rate of how many meters per second?
Answered by Penny Nom.
The slope intercept form 2008-04-11
From Dave:
Hi, why is there a y-intercept in the slope intercept form?What does the y-intecept have to do with the equation?
Answered by Penny Nom.
A truncated cone 2008-04-11
From jason:
i need to know how to figure the cubic feet of a cone that the top is 72" wide the bottom is 25" wide and it is 48" tall. is there a specific formula for this?
Answered by Penny Nom.
Some 4 sided polygons 2008-04-11
From kathy:
I am a 4th grade student and need to complete this question:

I need to draw 2 polygons with 4 sides and 2 right angles. Please help. Thanks

Answered by Penny Nom.
A rectangle word problem 2008-04-10
From Nick:
A rectangle is twice as long as it is wide. If the length and width are both increased by 5cm, the resulting rectangle has an area of 50cm^2. Find the dimensions of the original.
Answered by Penny Nom.
The volume of a triangular shaped pond 2008-04-10
From Bret:
we have a pond that is triangular shape : 130ftx130ftx150ft and is 4ft deep. How can I figure out as to how many gallons of water it will take to fill it up? Thanks for your help :)
Answered by Penny Nom.
A chain letter 2008-04-10
From Lesley:
In the mail you receive a chain letter with a list of ten names on it. The instructions tell you to send $2 to the person named on the top of the list, cross his name out, and place your name on the bottom. YOu then have to send ten copies of this letter to friends with the same instructions. If no one breaks the chain how much money should you receive?
Answered by Stephen La Rocque and Harley Weston.
The chain rule 2008-04-10
From joey:
pls help me to Differentiate
y=(3x^2-4x)^8

Answered by Harley Weston.
Cutting a 200 diameter pipe at 45 degree angle 2008-04-10
From Al:
i want to cut a 200 diameter pipe in 45degrees. Can you demonstrate how to develop a flat rap around please
Answered by Harley Weston.
Complete the square 2008-04-09
From jessica:
can you check my answer please thanks

f(x)=2x^2-4x+5

I got 2(x+2)^2-3 for when it asked to simplfy it to standard form of a quadratic equation

Answered by Stephen La Rocque.
An isosceles right triangl 2008-04-09
From Stephanie:
An isosceles right triangle has an area of 35 sq cm. What are the lengths of the sides?
Answered by Walter Whiteley.
A regular polygon 2008-04-09
From chase:
name the regular polygon if the sum of the measures of the polygon is 1,440 and the measure of one angle is 144
Answered by Stephen La Rocque.
2500mL of Phosphoric acid 2008-04-09
From Trina:
If I have 2500mL of Phosphoric acid that has a fluoride concentration of 0.91% and need to increase it to 1.40% how much 49% HF do I need to add?
Answered by Harley Weston.
Complete the square 2008-04-09
From jessica:
Use the technique of completing the square to put f(x)= 2x^2-4x+5 in the form f F(x)= a(x-h)^2+k were h,k=vertex of the parabola. identify the coordinate (h,k)of the maximum or minimum point of the associated parabola
Answered by Penny Nom.
Side lengths and angles in a regular octagon 2008-04-09
From Lori:
I am a builder and need to find the length of the sides of an octagon. I know the length between the parallel sides (26 feet). What is the length of each side? What is the angle measurement?
Answered by Harley Weston.
Matrix multiplication 2008-04-08
From manashi:
i. why matrix division is not possible?
ii.when we add or subtract two matrix , getting the result by addind or subtracting correspondind elements....but in case of multiplication it is not but why?

Answered by Harley Weston.
A square garden of tulips 2008-04-08
From Vincent:
Each year Brandon plants tulip bulbs in a square flower bed. This year Brandon's flower bed has 29 more bulbs than it had last year. If the flower bed is still square how many tulip bulbs are in it this year?
Answered by Penny Nom.
The weight of 1 litre of milk 2008-04-07
From IMRAN:
WHAT WEIGHT IN KILOGRAMS DOES 1 LITRE OF MILK HAVE?
Answered by Penny Nom.
How many bags of mulch? 2008-04-07
From MJ:
I have an area that is 35' X 50' and I want to cover it with 2" of mulch.

Do I take length X Width X depth? Bags of mulch are sold in 2 or 3 cubic feet per bag.....I just want to know how to do the math. can you please send me the equation?

Answered by Penny Nom.
The amount of a washing machine 2008-04-07
From Eddy:
we have a washing machine with a specific cylinder size, which we could calculate the volume as a regular cylinder formulas. The question is, if we fill the washer cylinder with water at a specific height on water , example 15 cm height, how can we find the volume of water at that specific height?
Answered by Penny Nom.
What fraction of 45 is 10? 2008-04-06
From Scott:
45 x fraction? = 10
Answered by Penny Nom.
How much do the pumpkins weigh? 2008-04-05
From Jose:
Mr. Appleberry brought in pumpkins for science class, one for each group. The fist task wa to weigh the pumpkins, but the scales they had wouldn't register any weight under 7 pounds, and it seemed that two ot three pumpkins were under that weight. The class decided that they could weigh the pumpkins two at a time, so they weighed each pumpkin with each of other pumpkins.

The weights registered on the scale were:

7 pounds
8 pounds
9 pounds
10 pounds
11 pounds
12 pounds

How many pumpkins were there, and how much did each weigh?

Answered by Penny Nom.
1-(1-P)^75=0.05 2008-04-05
From Amy:
1-(1-P)^75=0.05
Answered by Stephen La Rocque and Penny Nom.
A volume of revolution 2008-04-04
From ted:
Consider the region bounded by y=x^2 + 1, y=5-3x and y=5. Sketch and shade the given region; then set up but dont evaluate teh integrals to find the following:

a) The volume of the solid generated by rotating the region about the line y=5

b) the volume of the solid generated by rotating the region about the y-axis

Answered by Penny Nom.
Compatible numbers 2008-04-04
From dawn:
Need help. Estimate the quotient. Tell what compatible numbers you used.

817divided by 4 =

Answered by Penny Nom.
The angle between the hands on a clock 2008-04-04
From Kristin:
The question I need help with is : Bobo looked at the clock at 1:25. What was the precise measure of the angle formed by the hour and minute hands of the clock? Assume that each of the clock hands moves at a constant rate continuously around the face of the clock. Thus, in any given fractional part of an hour, each hand will move that same fractional part of an hour's worth of movement.
Answered by Stephen La Rocque.
lim as x approaches infinite of 5x + 2/x-1 2008-04-04
From Jordan:
how to solve this.

lim as x approaches infinite of 5x + 2/x-1

Answered by Stephen La Rocque and Harley Weston.
Finding the radius when only given chord length 2008-04-03
From Lorraine:
There are two chords in a circle, an 8 inch chord and a 10 inch chord. The 8 inch chord is twice the distance from the center as the 10 inch chord. What is the radius?
Answered by Stephen La Rocque.
The shortest distance from a point to a line 2008-04-03
From Katherine:
How do I find the shortest distance from the point (3,5) to the line x = -2
Answered by Stephen La Rocque.
The area of a circle 2008-04-03
From Norm:
how to compute the square footage of a circle
Answered by Penny Nom.
A shape 2008-04-03
From Joshua:
what shape has 4 faces, 4 corners and 9 edges.
Answered by Penny Nom.
Perimeter, area and diagonals of a rectangle 2008-04-03
From jenea:
I have to find the perimeter as well as the area of a rectangle when given the diagonals are 12 units long and they intersect at a 60 degree angle. how do i find both the area and perimeter; do i use the 30 60 90 triangle rule?
Answered by Stephen La Rocque and Penny Nom.
A 100 acre square pasture 2008-04-03
From Richard:
I need to know how many lineal feet there is around a 100 acre pasture that is square in shape. I am no longer able to get out and measure it myself, and cannot remember the math to compute it.
Answered by Penny Nom.
1 mile per minute 2008-04-01
From jennifer:
If you are traveling at 1 mile per minute how fast would you need to be going
Answered by Stephen La Rocque.
Two consecutive odd numbers 2008-04-01
From prinsilla:
of all pairs of consecutive odd integers whose sum is greater than 75. find the pair whose sum is least?
Answered by Stephen La Rocque.
810 slabs of concrete 2008-04-01
From Wesley:
I'm making a 4" x 4" x 2" (Inches) concrete slab (approximately 750 of them, I'm going to plan for 810 for extra material). I'm trying to find out how many bags of cement I need. Each 60 lbs bag makes approximately .5 cu/ft.
Answered by Harley Weston.
How long will it take to pump the water out of the basement? 2008-04-01
From Shiva:
I need to pump water out of a flooded basement, using two 50 (gpm) pumps. The basement has the dimensions shown and is flooded to a depth of 16 inches. How long will it take to pump the water out of the basement?
Answered by Harley Weston.
The rectangle with smallest perimeter 2008-04-01
From nicole:
when given an area,squares are the rectangles with the smallest perimeter write the length and width of the rectangle with the smallest perimeter
Answered by Penny Nom.
Common fraction to decimal fraction 2008-04-01
From chadwick:
how do I turn 3/32 into a decimal
Answered by Penny Nom.
-2 ^ 2 + (6 - 9) / (-3) + 4 * (-2) 2008-04-01
From Jeth:
How do I simplify: -2 ^ 2 + (6 - 9) / (-3) + 4 * (-2)
Answered by Penny Nom.
Packing a trailer 2008-04-01
From Shellie:
I have a trailer that is 8 ft x 5 ft. I have a hutch base that is 32 in x 52 in, hutch top is 47 in x 52 in, table 56 in x 44 in, 6 chairs that are 48 in x 20 in. I know that the cubic feet of the trailer is l x w x h=3D 5 x 8 x 5=3D 200 cubic feet I know that the area of the items are approximately 1026 feet.

How do I know if these things will fit? I would like to know the math behind it?

Answered by Penny Nom.
The integral of dx / (4x^2 - 25)^3/2 2008-04-01
From Meghan:
I have a question from the trigonometric substitution of my calculus course.

integral of dx / (4x^2 - 25)^3/2

Answered by Harley Weston.
A rectangular prism 2008-03-31
From Chanlie:
i need need to find a rectangular prism with a total surface area of 210 units.
Answered by Penny Nom.
I have to fill a hole with clay 2008-03-31
From Tim:
Dear Sirs I have to fill a hole with clay and then 8 inches of top soil. The area is a 24 foot circular hole left by a now removed swimming pool. The area in question pithces from the house from 48" to ground level within that 24 feet. I was wondering how many yards o the top soil has to be 5-8 inches deep thank you f fill(clay) I would need and how many Yards of top soil to cover that hole
Answered by Harley Weston.
How many bags of cement do I need 2008-03-30
From Joe:
How many bags of cement do I need if I want to extend my carport 6 inches all around? The carport is 15ft. x 15ft. and the existing slab is 4 inches thick.
Answered by Harley Weston.
How far is she from her starting point? 2008-03-30
From Nancy:
A child runs due east for 23.5m and then she runs 10.5m due south. How far is she from her starting point?
Answered by Stephen La Rocque.
2 times 0 times 0 times 4 2008-03-30
From ben:
what is 2 times 0 times 0 times 4?
Answered by Penny Nom.
The amount of water in a pool 2008-03-30
From anurag:
what will the weight of water in a swimming pool having dimensions of 18ft *10ft*5 ft? how much water will be needed for filling it up?
Answered by Penny Nom.
The distance between the sides of a parallelogram 2008-03-30
From jhon:
Calculate the perpendicular distance between AB and DC of a parallelogram. Note: AB and DC are the sides of the parallelogram.
Answered by Penny Nom.
The exponential form of x radical y to the third power 2008-03-30
From Jennifer:
what is the exponential for of x radical y to the third power
Answered by Penny Nom.
A normal distribution problem 2008-03-29
From Lorie:
3. The amount of annual snowfall in a certain mountain range is normally distributed with a mean of 109 inches, and a standard deviation of 10 inches.

a. What is the probability that the mean annual snowfall during 40 randomly picked years will exceed 111.8 inches?

Answered by Harley Weston.
A fish tank in the shape of an irregular pentagon 2008-03-29
From richie:
i am building a fish tank. it is going to be an irregular pentagon. the sides are going to be
24"
24"
8"
8"
32"(approximately)

there will 3 right angles A, B, E

my question is how to figure out the degree of the angles that are not right angles (C,D)?

Answered by Chris Fisher.
The area of an equilateral triangle 2008-03-28
From Elizabeth-Morgan:
I have a problem that reads: Find the area of a triangle with sides of 4 cm. Find the height first.

This means that I have an equilateral triangle, and I need a formula to calculate the height using the base, and the sides.

Answered by Stephen La Rocque.
Probability and Merta trains 2008-03-27
From Lorie:
4. Merta claims that 74% of its trains are on time.

a. Find the probability that among the 60 trains, 38 or fewer arrived on time.

Answered by Harley Weston.
A max-min problem 2008-03-27
From LSL:
show that of all rectangle with a given area, the square has the smallest perimeter.
Answered by Penny Nom.
A car tire full of concrete 2008-03-27
From robert:
I want to build a volleyball net support. I am using a car tire 24"odx16"id filled with concrete. how much will this weigh? thanks
Answered by Penny Nom.
Similar triangles 2008-03-26
From Nisha:
In triangle ABC, LM is parallel to AB. If AL=x-3,AC=2x,BM=x-2,BC=2x=3. Find x
Answered by Penny Nom.
A complex cubic polynomial 2008-03-26
From wael:
how do we solve in C the following equation:
z^3 + (5i-6)z^2 + (9-24i)z + 13i + 18 = 0 if it admits a pure imaginary root?

Answered by Harley Weston.
Painting a mural 2008-03-25
From JUAN:
THERE IS AN AREA TO BE PAINTED WHICH IS 6 FEET LONG AND 3 FEET WIDE IT HAD BEEN DIVIDED INTO 1 FT SQUARES. Two students will paint each square how many students will work on the mural?
Answered by Penny Nom.
The area of a rectangle 2008-03-25
From Raphael:
What is the area of rectangle that measures 7 yards by 23 yards
Answered by Penny Nom.
A function satisfying f(x) + 2f(1/x) = x 2008-03-25
From Joan:
Let f be a function satisfying f(x) + 2f(1/x) =x for all real numbers (x does not equal 0)
a) Find f(1) justify your answer

b) Find f(2) justify your answer

Answered by Penny Nom.
The length of the ramp on a motorcycle trailer 2008-03-25
From Joshua:
I am currently building a motorcycle trailer. I am trying to figure out the length of the ramp so that the bikes don't scrap the trailer/ramp as they are loaded. This is the info I have: the motorcycle is 6" off the ground in the center, the point where the tires touch the ground are 80" apart, the trailer deck is 20" high. How do I figure the length of the ramp? Please show equation so I have for future reference with different measurements.
Answered by Stephen La Rocque and Harley Weston.
Light years to miles 2008-03-24
From Robert:
HOW MANY MILES ARE THERE IN 7.5 BILLION LIGHT YEARS.
Answered by Penny Nom.
The decimal equivalent of a fraction 2008-03-24
From Mario:
What is the decimal equivalent of the fractions 5/8?
Answered by Penny Nom.
A word problem about numbers 2008-03-24
From kathy:
five times a number minus a second number is 6. The sum of the numbers is 12. find the numbers
Answered by Penny Nom.
A hydraulic cylinder 2008-03-24
From james:
I am trying to adjust the placement of a hydraulic cylinder that raises a dump bed up from the frame of a truck. How long would the cylinder (height of a triangle) have to be to raise the bed to a 70 degree angle? The base from pivot to cylinder is 132.5 inches.
Answered by Stephen La Rocque.
The volume of a footing 2008-03-23
From Brett:
You need to pour an 8"x16" footing that is 100ft. long. At a cost of $65 per cubic yard, how much will this cost?

(A) $205 or less
(B) more than $205, but less than $220
(C) $220 to $235
(D) more than $235

Answered by Penny Nom.
The volume of a dam 2008-03-23
From JD:
How many cubic yards of soil are required to build a small Pond Dam:
100ft wide bottom x 300ft long x 15ft High with a 14ft wide Flat top

Answered by Harley Weston.
A word problem about a triangle 2008-03-23
From kathy:
The measure of the largest angle of a triangle is twice the measure of the second largest. The measure of the second largest is 20 degrees more than the measure of the smallest. Find the measure of each angle.
Answered by Melanie Tyrer.
Circles and semicircles 2008-03-23
From sally:
Hi i am sally ,

i need to know what is the formula of :

Find perimeter and area of a semicircle ?

Find the area of a circle whose circumference ?

Answered by Penny Nom.
The area of the triangle with vertices (-1,-8), (-1,1), and (5,1) 2008-03-23
From jimmy:
find the area of a triangle whose vertices are (-1,-8), (-1,1), and (5,1)
Answered by Penny Nom.
Angles subtended by the same arc 2008-03-23
From Reid:
Prove that two inscribed angles subtended by the same arc are equal.
Answered by Stephen La Rocque.
A probability distribution 2008-03-23
From Lorie:
The probabilities that a batch of 4 computers will contain 1, 2, 3, and 4 defective computers are 0.6274, 0.3102, 0.0575, and 0.001 respectively.

a. Set up a probability distribution to describe this situation.

Answered by Penny Nom.
A number puzzle 2008-03-22
From Cassidy:
1. Number has three digits
2. The second digit is an odd number
3. The first and third digits are even numbers
4. The number can be divided by five, six, and seven with a remainder of four.

How do you even start this type of problem?

Answered by Victoria West.
Is this an irrational number? 2008-03-22
From Tom:
Is it possible to build an irrational number with numbers that are just 1's and 2's where only finitely many 2's appear? If so, how do we build it and how do we know it is irrational?
Answered by Penny Nom.
5 9 1 6 10 12 2, where does 3 fit in and why? 2008-03-22
From Cassidy:
number sequence

5 9 1 6 10 12 2, where does 3 fit in and why?

Answered by Claude Tardif.
The radius of a circle 2008-03-22
From danny:
waht is the radius of a circle, if the circumference is 800?
Answered by Penny Nom.
Converting ounces to milliliters 2008-03-21
From kelly:
As I have to drink 12 - 8 ounce glasses of water a day and my water bottle holds 591 ml, how many bottles of water must I drink?
Answered by Penny Nom.
How would I graph: x - 2 divided by x^2 - 4? 2008-03-19
From Sean:
How would I graph: x - 2 divided by x^2 - 4 using a method in extreme values of functions or completing the square

x - 2/ x^2-4

Answered by Stephen La Rocque.
Expected value and standard deviation 2008-03-19
From Patrick:
Heres a question i cant figure out:
a small airline company has only three flights per day. The number of delayed flights per day is regarded as a random variable, and I'm supposed to calculate the expected value and standard deviation of the number of delays. the probability distribution looks like:
No. of Delays: 0 1 2 3
Prob. of delay: 05. 0.3 .1 .1

Answered by Harley Weston.
A pentagon inscribed in a circle 2008-03-19
From Elaine:
My question as written on my homework is: Given a pentagon inscribed in a circle of radius r, determine a) the angle between any two sides of the pentagaon b) the perimeter of the pentagaon c) the area of the pentagon. I know this kind of counts as three questions, so if you can only answer one, that's okay. Any help will be much appreciated. Thanks!
Answered by Stephen La Rocque.
The area of a right triangle 2008-03-19
From Tate:
I need to find the area of a right triangle. I already know that the formula would be a squared + b squared = c squared. However, I only have the length of the hypotenuse and the base.
Answered by Victoria West.
Equivalence relation 2008-03-19
From Hmod:
State with reasons, whether the relation R on N X N given by mRn if m and n have difefrent remainders on division by 2 is an Equivalence relation
Answered by Victoria West.
A pen with 40 post on each side 2008-03-18
From Manpreet:
how many post do you need to make a pen with 40 post on each side
Answered by Penny Nom.
Any regular polygon inscribed in a circle 2008-03-17
From lindsay:
how do find the perimeter of a regular octagon inscribed in a circle with a radius of 5 units
Answered by Stephen La Rocque.
Twenty golfers 2008-03-16
From john:
I have a group of 20 golfers. We plan on playing 6 rounds of golf in 4 days. I would like to have the most possible combinations of players without each player playing with anyone if possible, in the 4 days of golf
Answered by Victoria West.
Determine the capacity of Farmer John's silo 2008-03-16
From paula:
Farmer John stores grain in a large silo located at the edge of his farm. The cylinder-shaped silo has one flat, rectangular face that rests against the side of his barn. The height of the silo is 30 feet and the face resting against the barn is 10 feet wide. If the barn is approximately 5 feet from the center of the silo, determine the capacity of Farmer John’s silo in cubic feet of grain.
Answered by Penny Nom.
The area of an equilateral triangle 2008-03-16
From michael:
my daughter has been given a task and I'm stuck too. if an area of an equilateral triangle = 85cm squared there are no other dimensions provided. how do I calculate the length of each side ? please
Answered by Penny Nom.
The surface area of a cube and a cylinder 2008-03-16
From sunday:
I'm doing a unit assessment, and there is something I don't understand. Now in my textbook, it says that surface area for a cube is different from a cylinder. Surface area for a cylinder has two numbers, but surface area for a cube has 3 or more numbers. And plus, I can't figure out the volume for a cube because it only shows volume for a cylinder. So how do I get this right?
Answered by Harley Weston.
A train and a boat 2008-03-15
From Sabrina:
A railroad bridge is 20m above, and at right angles to, a river. A person in a train travelling at 60 km/h passes over the centre of the bridge at the same instant that a person in a motorboat travelling at 20km/h passes under the centre of the bridge. How fast are the two people separating 10s later?
Answered by Harley Weston.
Mathematics in cooking 2008-03-15
From Emily:
What forms of Mathematics are used in Cooking?
Answered by Harley Weston.
The limit of (sin4x)/(tan9x) as x approaches zero 2008-03-14
From stev:
as x->0. How do you find the limit of f(x)=(sin4x)/(tan9X)
Answered by Harley Weston.
The area of a triangle 2008-03-14
From Tara:
Area of triangle is 4,018 square yards. Its base is 82 yards. What is its height? Thanks
Answered by Stephen La Rocque.
The length of a side of a square 2008-03-12
From laura:
if i had a square of 2.3 square meters what would its radius be?(in centimeters please)
Answered by Penny Nom.
Moving grain across the river 2008-03-12
From Janice:
Wesley needs to move 820 kg of grain across the river in a canoe that can carry no more than 120 kg. Wesley has a mass of 70 kg. What is the fewest number of trips he will have to make? A. 17 trips; B. 18 trips; C. 32 trips; D. 33 trips.
Answered by Penny Nom.
A dilation project 2008-03-12
From john:
I am working on a dilation project. How do I dilate an irregular octagon on a scale of 3.5
Answered by Stephen La Rocque.
A circle and a square 2008-03-12
From mae:
what are the formulas on getting the area of the square outside an inscribed circular region?
Answered by Penny Nom.
The centre and radius of a circle 2008-03-12
From Ryan:
hello and thank you for such a wonderful service. This problem I think needs to be checked could you take a gander at it and tell me if i get it correct thanks find the center and the radius of this circle x^2+y^2=8x-2y+15=0 I cam up with center -2, 1/2 and a radius of 11 3/4
Answered by Harley Weston.
Find the present age of the two siblings 2008-03-11
From jessica:
Theresa is four years older than her younger sister Joni. Twelve years ago, she was twice as old as her sister . Find the present age of the two siblings.
Answered by Stephen La Rocque.
A width-to-length ratio of 2 : 3 2008-03-11
From Stephanie:
The national flag of France has a width-to-length ratio of 2 : 3. What is the width of an 18-inch long French flag?
Answered by Penny Nom.
Completing the square 2008-03-11
From Ryan:
solve for x by completing the squares, x^2+4x-8=0
Answered by Stephen La Rocque.
The diagonals of a hendecagon 2008-03-10
From karan:
How many diagonals does a hendecagon have?
Answered by Penny Nom.
A linear system 2008-03-10
From sandhiya:
one group of people purchased 10 hotdogs and 5 soft drinks at a cost of $12.50. a second group bought 7 hotdogs and 4 soft drinks a ta cost of $9.00 find the cost of a single hot and a single soft drink.(using the row echelon form to solve)
Answered by Penny Nom.
Repeating decimals 2008-03-10
From Blaine:
Is it possible to put a repeating decimal number into a calculator to solve a problem?

EX: Write 39.3939... as a fraction.

Answered by Penny Nom.
A common chord to two circles 2008-03-09
From shubha:
please help me out with this problem. find the length of the common chord of the intersecting circles x2+y2-4x-5=0 and x2+y2-2x+8y+9=0
Answered by Stephen La Rocque.
Average speed up and down a hill 2008-03-09
From Judith:
An old broken-down ar has to travel a two miles route, up and down a hill. Because it is so old, the car travels the first mile - the ascent - at an average speed of fifteen miles per hour. How fast must it cover the second mile - the decent - to achieve an average speed of thirty miles per hours? (There is a missing variable, but what is it?)
Answered by Penny Nom.
Saving to buy a coat 2008-03-09
From alayna:
Jackie is saving her money to buy a coat that costs $121. If she already has $61 and saves $15 each week, in how many weeks can she buy the coat ? Make a function table to answer the question.
Answered by Penny Nom.
Side lengths and area of a triangle 2008-03-08
From angela:
In the diagram of ABC the angle A is 65 the angle B is 70 and the side opposite vertex B is 7. Find the length of the side opposite vertex A and find the area of ABC
Answered by Penny Nom.
The volume of a countertop 2008-03-08
From Matthew:
I need to calculate volume down to cubic feet. I pour concrete countertops and cant figure out the formula to convert inches and feet. If I have a 2.5 inch by 2 foot by 16 foot concrete countertop how do I convert it to cubic feet and hence know how many bags of concrete at .6 cubic feet per bag I will need. I am trying to input the formula into a spreadsheet. thank you so much Matt
Answered by Penny Nom.
Probability and hair 2008-03-08
From CELESTE:
MY BOSS POSED THIS QUESTION TO US. THE FIRST WITH THE CORRECT ANSWER WINS. tHANKS! In a city of 200,000 people, no one has more than 100,000 hairs on his or her head, and no one will have fewer than 1. What are the chances that two inhabitants have exactly the same number of hairs? (Nose and ear hairs don't count.)
Answered by Penny Nom.
Two circles and a triangle 2008-03-07
From Adrian:
The vertices of a right-angled triangle are on a circle of radius R and the sides of the triangle are tangent to another circle of radius r. If the lengths of the sides about the right angle are 16 and 30, determine the value of R+r
Answered by Penny Nom.
Separate three different sized marbles 2008-03-07
From Jana:
hello, I love this site, it explains everything so well. I have been asked a question in science, it asks for me to make a devise which separated three different sized marbles. I have thought this through alot but it still never makes sense to me. Everything i seem to think of never works out or has something that will make it not work. I'm really stuck on this one...please help me..! Thankyou so much Jana
Answered by Stephen La Rocque.
Two perpendicular lines 2008-03-07
From Daphne:
Does anyone know how to find a line perpendicular to -3x=4y=20 and passes through the point (-12,0)?
Answered by Stephen La Rocque and Penny Nom.
A fence around a rectangular region 2008-03-06
From sharon:
a farmer want to make a rectangular fence he has 750 feet of material . if the width is 95 feet what is the area of the rectangle
Answered by Penny Nom.
Cubic feet to liters 2008-03-06
From chase:
if 1cu.ft=6.229 imp. gal and1 imp.gal=4.546 liters how many liters does of 128cuft. hold
Answered by Penny Nom.
The dimensions of a rectangle 2008-03-06
From Julie:
Find the dimensions of a rectangle with an area of 400. Width x ft. Length (2x-7)ft.
Answered by Stephen La Rocque and Penny Nom.
Two points moving on a straight line 2008-03-06
From laura:
Particle A travels at constant speed of 1.0ms-1 in straight line, passing particle B, at which time particle B beings traveling with constant acceleration of 0.2ms-2 until reaching Particle A.
What equations represent the displacement of particle A and particle B measure from the point they first pass?
How long does it take for particle B to catch up with particle A?
What is the speed of particle B when reaching particle A?
What distance have both particles traveled from where they first pass to where they meet again?
Calculate the displacement of both particle A and B at 2 second intervals between t=0s and t=12s?

Answered by Stephen La Rocque and Penny Nom.
Polar coordinates 2008-03-05
From kumi:
2.Sketch the graph of r=(2)/(1-sinθ). Make sure you show a complete chart demonstrating how you arrive at your sketch, and sketch the graph neatly and clearly with proper labeling.
b. change the equation above to cartesian coordinates and identify the type of graph.
c.explain which coordinate system gives an easier function to graph and why
d.find a function of the form r=f(θ) for a circle centered at (8,π/2) and with the point (0,0) lying on the circle

Answered by Penny Nom.
Parametric equations 2008-03-05
From kumi:
I have made a picture of a football pass 45 degrees with 30yds and height of 7ft and 4ft. But I am just stuck there. I have stared at the worksheet for soo long. I have no idea how to solve this, and I don't even know where this is going (I don't understand how parametric equations work). Can you help me with this problem?

1.The quarterback of a football team releases a pass at a height, h, of 7ft above the playing field, and the football is caught by a reciever at a height of 4ft, 30yds directly downfield. The pass is released at an angle of 45 degrees with the horizontal with an initial velocity of vo. The parametric equations for the position of the football at time t are given, in general, by x(t)=(vo cosθ)t and y=h+(vo sinθ)t-16t^2.

a. find the initial velocity of the football when it is released
b. write the specific set of parametric equations for the path of the football
c.use a graphing calculator to graph the path of the football and approximate its maximum height
d. find the time the reciever has to position himself after the quarterback releases the football

Answered by Harley Weston.
An arithmetic expression 2008-03-05
From Kasani:
evaluate the expression: -2^-1[-14 - 4(-6)] - (7 - 3)
Answered by Penny Nom.
One number is four less than five times another number 2008-03-05
From alice:
One number is four less than, five times another number. The total of the two numbers is 50. What are the two numbers?
Answered by Penny Nom.
A roll of paper 2008-03-05
From Sheik:
How I Convert My Roll Weight Into Meters?
Roll Width : 241mm
Thickness : 56gsm
Weight :81 Kg

Answered by Harley Weston.
Is every rectangle a square? 2008-03-04
From Odette:
if every square is a rectangle, is every rectangle aslo a square?
Answered by Penny Nom.
I need to level a 18ft x 21ft area that is very uneven. 2008-03-04
From Shelia:
I need to level a 18ft x 21ft area that is very uneven. I have used a line level to determine the number of inches at various places in the area. This varies from 1 inch to 15 inches in order to bring the garden area up level. How do I calculate cubic yards so I can order topsoil for the area. If needed, I have the number of inches needed for each square yard area.
Answered by Penny Nom.
A 6 pointed star 2008-03-04
From Siddharth:
When 2 congruent equilateral triangles share a common center, their union can be a star If their overlap is a regular hexagon with an area of 60, what is the area of one of the original equilateral triangles?

a) 60 b) 70 c) 80 d)90 e)100

Answered by Stephen La Rocque.
A three dimensional object 2008-03-04
From Briana:
what is the 3d term for pentagon? like a circle is a shhere?
Answered by Penny Nom.
What was the ostrich's speed in miles per hour? 2008-03-03
From jacob:
If it took an ostrich 2.5 seconds to travel 110 feet and the average speed is 44 feet/ seconds, what was the ostrich's speed in miles per hour?
Answered by Penny Nom.
Triangular prisms and pyramids 2008-03-03
From LORENA:
hi i am in yr 6 and i need to know how many faces,vertices and edges these shapes have..
. TRIANGULAR PYRAMID
. TRIANGULAR PRISM

Answered by Penny Nom.
The diameter of a circle 2008-03-03
From Kathie:
I know the length of the curved portion of a semi-circle is 200 ft. but I need to find the diameter to get a total perimeter and area. How do I find the diameter to get the perimeter?
Answered by Penny Nom.
What point on the graph y = e^x is closest to the origin? 2008-03-03
From elvina:
What point on the graph y = e^x is closest to the origin? Justify your answer.
Answered by Stephen La Rocque.
A puzzle about hats 2008-03-03
From Sylvia:
There were 4 people sitting in a row, truing to win $50.00 from a riddle.

The one in front was wearing a red hat. The second from the front was wearing a blue hat. The third from the front was wearing a red hat. The last was wearing a blue hat and is blind-folded.

Each of them were informed that they are all wearing a hat, the last person is blind-folded and that there are 2 red and 2 blue hats. If they can guess the colour of their hat, then they win the prize.

There was a long silence, then one of them guessed the color of his hat correctly. Who was it? and explain why.

(note that they can only see the person in front of them and are not allowed to turn around and see behind them.)

Answered by Penny Nom.
The volume of a cereal box 2008-03-03
From Lawson:
I have calculated the volume of a box of cereal whose dimensions are: Length - 19 cm
Width - 3 cm
Height - 27 cm

I need help with the follow question: If 2cm3 = 30 pieces of your cereal, how many pieces of cereal will fit in your box? What do I do the calculate?

Answered by Penny Nom.
The volume of a truncated prism 2008-03-03
From jhey:
The volume of a truncated prism is 6240 cu cm. The base is a right section in the form of an equilateral triangle. The edges perpendicular to the base are 15 cm, 18 cm, and 15 cm long. Find the length of one side of the base.
Answered by Harley Weston.
Choosing 5 numbers from 56 numbers 2008-03-02
From alexavier:
need all of the 5 digit combinations using the numbers 1-56.
Answered by Harley Weston.
Time and a half 2008-03-01
From Travis:
Bill receives time and a half for working overtime. If Bill is paid $55 for each hour of overtime, how much is his regular hourly wage?
Answered by Stephen La Rocque.
How much did the investor gain from selling his stock? 2008-03-01
From Julie:
An investor sold 35 shares of stock at 21 and 3/4 per share. The investor had purchased the stock for 18 and 1/2 per share. How much did the investor gain from selling his stock? Thank You in advance for your help
Answered by Stephen La Rocque.
The angles of a triangle 2008-03-01
From Allen:
How to find the other two angles of a scalene triangle if one angle is given
Answered by Steve La Rocque and Penny Nom.
Simultaneous equations 2008-02-29
From CONOR:
I was wondering if you could help me with this problem

7x - 5y = -1
3y = 4x

Answered by Penny Nom.
A package of supplies is dropped from a plane 2008-02-29
From beth:
An airplane flies west at a speed of 200m/s. A package of supplies is dropped from the plane to some campers. if the plane is at an altitude of 1000meters, how far from the campers should the package be dropped to land near them? ignore wind resistance.
Answered by Stephen La Rocque.
Is a square a rectangle? 2008-02-29
From Jaime:
Is a square a rectangle?
Answered by Penny Nom.
The perimeter of a sector of a circle 2008-02-28
From erica:
What is the perimeter of a sector if the radius is 18 and the middle arc is 150?
Answered by Penny Nom.
TV watchers on Thursday and Saturday 2008-02-28
From flor:
Thursday is the most watched night on television with 20 million more viewers than Saturday. the total for both nights is 102 million. how many viewers watch on each of these nights?
Answered by Penny Nom.
Some solid figures 2008-02-28
From tiffany:
What solid figure could have? It has 4 more edges than vertices,
Answered by Penny Nom.
The radius of a circle 2008-02-28
From SteVonee:
Estimate the radius of a circle with the given circumference that is 192ft
Answered by Penny Nom.
3 consecutive odd integers 2008-02-28
From Pat:
For 3 consecutive odd integers, twice the first integer, plus 5 more than the second integer, plus three less than four times the third integer equals -85. Find the 3 consecutive odd integers.
Answered by Penny Nom.
A triangle problem 2008-02-28
From Jessica:
Triangle ABC Angle C is 3 times angle A angle B is 30 degrees less than angle C what are the measurements of the angles I have the measurements and I have the Equation but I can't figure out quite how to solve it the proper way
Answered by Stephen La Rocque.
Rolling a 5 before an 11 2008-02-27
From Jeremy:
If rolling two six-sided dice, what is the probability that a sum of 5 will be rolled before a sum of 11?
Answered by Stephen La Rocque and Harley Weston.
A proof in geometry 2008-02-27
From Kimberly:
I'm trying to write a proof for the following: If all altitudes are equal in an equilateral triangle then all sides are equal.
Answered by Stephen La Rocque and Penny Nom.
Multiple guess 2008-02-27
From debbie:
What is the probability of getting 6 out of 10 (a,b,c,d,e) questions correct, by random selection?
Answered by Stephen La Rocque.
Two regular pentagons 2008-02-27
From Shirley:
Regular pentagon QRSTU has a side length of 12 centimeters and an area of 248 square centimeters. Regular pentagon VWXYZ has a perimeter of 140 centimeters.

I need to find its area.

Answered by Stephen La Rocque.
A line drawn between 2 vertices 2008-02-26
From Arshad:
what is a line drawn between 2 vertices which are not next to each other called.
Answered by Penny Nom.
I cut the cylinder at a 45 degree angle 2008-02-26
From Shannon:
I have a cylinder with a radius of 2' 1 5/8". How do I calculate the radius increase when I cut the cylinder at a 45 degree angle?
Answered by Harley Weston.
A right triangle 2008-02-25
From Ali:
The length of a hypotenuse of a right triangle is 2 metre more than the longer leg. The length of the longer leg is 7 metre more than the length of the shorter leg. Find the number of metre in length of each side of the right triangle.
Answered by Stephen La Rocque.
A Norman window 2008-02-25
From Jason:
If the perimeter of a Norman window is 20 feet, what is the maximum area of the window?
Answered by Stephen La Rocque.
All possible arrangements of the word "PROBLEM" 2008-02-25
From student:
Consider all possible arrangements of the word "PROBLEM". If a word is picked at random, find the probability that

(a) the word starts with a vowel

(b) the word ends with a consonant.

Answered by Harley Weston.
The surface area of an ellipse 2008-02-25
From paritosh:
how is the surface area of a elliptical dome be calculated when the two diameters are 55 metres and 35 metres and the height of the structure is 13.4 mts?
Answered by Harley Weston.
The base of an isosceles triangle 2008-02-24
From tahrima:
find the base of an isosceles triangle whose area is 12sq.cm and the length of 1 of the equal sides is 5 cm.
Answered by Stephen La Rocque.
Pea gravel 2008-02-24
From Linda:
I have a space of 16 feet by 8 feet. I want to fill it with pea gravel 6 inches deep. If the bags of pea gravel are .53 cubic foot. How many bags would I need. Thanks so much.
Answered by Stephen La Rocque.
lim sin 4x/sin 6x as x approaches zero 2008-02-22
From josh:
lim sin 4x/sin 6x
x approachs 0

Answered by Stephen La Rocque and Harley Weston.
Fact family 2008-02-21
From Anand:
Can you think of a fact family that has only two facts? Write the fact family
Answered by Penny Nom.
Actural cost, selling price and commission 2008-02-21
From LiSa:
I have a question about a few things if anyone can answer... ex. actual cost (AC) = $1000 selling price (SP) = $ amount earned for company (C) = 15% of... selling price? commission 1 (C1) = 2.5% commission 2 (C2) = 2.5% How would you calculate how much the company's commission amount would be as well as the amounts for the other 2 that get 2.5%? And what would the selling price be? (the 15% $ + 2.5% $ + 2.5% $ ?) Is the 2.5% taken from the original amount? I am soooo confused! If anyone could help, that would be great!
Answered by Harley Weston.
The diagonal of a rectangle 2008-02-21
From tim:
is there a formula for finding the diagonal of a rectangle?
Answered by Penny Nom.
Algebraic fractions 2008-02-21
From sergio:
how to simplify
(x^2 +5x+6)/(x^2 - 4) x (x^2 -5x+6)/(x^2-9)

Answered by Stephen La Rocque.
Row echelon form 2008-02-21
From cOCO:
USE ELEMENTARY ROW OPERATIONS TO WRITE THE MATRIX BELOW IN ROW ECHELON FORM.
1 1 8 - 2
2 -1 1 11
2 1 14 -11

Answered by Penny Nom.
8% simple interest for 6 years 2008-02-20
From Tiffany:
cindy puts 5,025 in a bank account that earns a simple interest rate of 8%. How much money will be in the account at the end of 6 years?
Answered by Stephen La Rocque and Penny Nom.
1/(1x2)+1/(2x3)+1/(3x4)...+1/(n(n+1)) 2008-02-20
From hossun:
Find a formula for 1/(1x2)+1/(2x3)+1/(3x4)...+1/(n(n+1)) by examining the values of this expression for small values of n. Use mathematical induction to prove your result.
Answered by Stephen La Rocque.
A fractional inequality 2008-02-20
From ed:
solve the inequality
(x+3)^2 (2-x)
___________ < 0
(x+4) (x^2 - 4)

Answered by Penny Nom.
18x=6(x*x) 2008-02-20
From peyton:
18x=6(x*x)
Answered by Stephen La Rocque.
Factoring a trinomial 2008-02-20
From Adrienne:
a field an rea of (x^2+x-6) square metres. State expressions for its length and width.
Answered by Penny Nom.
Peg has 12 more cases to unload than Mike 2008-02-20
From brianda:
Peg has 12 more cases to unload than Mike. They have a total of 150 cases to unload. How my cases does each have to unload?
Answered by Penny Nom.
Two right triangles 2008-02-18
From Amy:
Hi, I have a problem that has me stumped.
There are two right triangles that share the same adjacent side (AB) which is 240'. The right angles of each triangle are at the opposite ends of the shared side.
The Opposite side (AC) of Triangle ABC is 180'
and the Hypotenuse (CB) is 300'.
The Opposite side (BD) of Triangle ABD is 100'
and the Hypotenuse (AD) is 260'.
How do I figure out where the hypotenuses intersect?

Answered by Stephen La Rocque and Harley Weston.
A complex quadratic 2008-02-18
From Ash:
z^2-(6+2i)z+(8+6i)=0

Solve for Z

Answered by Steve La Rocque and Penny Nom.
The area of a right triangle 2008-02-18
From ASHLEY:
WHAT IS THE AREA OF A RIGHT TRIANGLE? WITH THE SHORT SIDE BEING 16 AND THE HYPOTENUSE 20?
Answered by Penny Nom.
Subtracting two algebraic fractions 2008-02-17
From Willie:
Subtract and express the answer in simplest form. 5x-3/6-x+3/6
Answered by Penny Nom.
The equation of a line 2008-02-17
From Jennifer:
Given the point A(3,2)B(5,8) find the equation of the line AB ( in standard form )
Answered by Penny Nom.
The slope intercept form of a line 2008-02-17
From chris:
find the slope and y intercept of the line (2x+4)-3y=11(x-1) is this equation in slope point form.
Answered by Penny Nom.
Drawing marbles from a bag 2008-02-17
From jan:
9 marbles are placed in a bag. 4 are blue, 3 are green, 2 are red. what is the probability of the following: a)p(g) (b) p(b) (c) p(r or g)
Answered by Chris Langdon.
Exponential form 2008-02-16
From annna:
i already got the answer for what is the prime factorization of 648 which is 2x2x2x3x3x3x3 but how do i write that answer in exponential form?
Answered by Penny Nom.
The length of the third side of a triangle 2008-02-16
From mary:
I have an angle of 72 degrees and each of the sides are 5' long. What is the distance from each of the ends of the 5 feet to form a triangle.
Answered by Stephen La Rocque.
A ball bearing is placed on an inclined plane 2008-02-15
From Leah:
A ball bearing is placed on an inclined plane and begins to roll. The angle of elevation of the plane is x. The distance (in meters) that the ball bearing rolls in t seconds is s(t) = 4.9(sin x)t^2. What is the speed of the ball bearing, and what value of x will produce the maximum speed at a particular time?
Answered by Penny Nom.
A parabolic arch 2008-02-14
From Angela:
A parabolic arch has an equation of x^2 + 20y - 400 = 0 where x is measured in feet. How do I find the maximum height of the arch?
Answered by Penny Nom.
Two regions with equal area 2008-02-13
From James:
There is a line through the origin that divides the region bounded by the parabola y=3x-5x^2 and the x-axis into two regions with equal area. What is the slope of that line?
Answered by Harley Weston.
Growth factor and growth rate 2008-02-13
From william:
what is the difference between growth factor and growth rate?
Answered by Stephen La Rocque and Harley Weston.
Scale factors 2008-02-13
From Dayne:
My teacher doesn't now how to teach us scale factors and i have a problem to make it easier to understand. The problem is: A map of Levi's property is being made with a scale of 2 cm : 3 meters. What is the scale factor?
Answered by Stephen La Rocque.
I am a 4-digit number 2008-02-12
From Nickie:
I am a 4-digit number with no repeating digits. I am divisible by 5, my first two digits (left to right) make a number divisible by 3, and my first three digits make a number divisible by 4. Also, my digits have a sum of 19 and I have the digit 7 in the thousands place. Who am I?
Answered by Penny Nom.
Women, cats and kittens 2008-02-12
From Paige:
There are 7 women on a bus. Each has a backpack. In each backpack there are seven cats. Each cat has seven kittens. How many legs are there total on the bus?
Answered by Penny Nom.
Prove (p & ~q) v p = p 2008-02-12
From Tom:
Prove (p & ~q) v p = p

Do I need to use the absorption law p v (p & q)=p? If yes,how do I cope with the NOT(~) in q?Thank you.

Answered by Harley Weston.
How do I graph y = -2x^2? 2008-02-12
From Max:
How do I graph y=-2x^2?
Answered by Penny Nom.
A real life example of a decagon 2008-02-12
From Htet:
I have a math dictionary to complete by February 13, 2008, Wednesday, and I need to know what a real life example of a decagon can be. I need help on this!
Answered by Penny Nom.
The area of a rectangle 2008-02-12
From mark:
The diagonal of a rectangle is 15cm, and the perimeter is 38cm. What is the area? It is possible to find the answer without finding the dimensions of the rectangle. Please help me and show as much of the problem as possible. Thank You
Answered by Penny Nom.
Drawing blocks from a box 2008-02-11
From Inez:
A box contains 3 red blocks and 1 green block. A block is chosen without looking and not put back. Then another block is chosen. What are the possible outcomes?
Answered by Penny Nom.
A triangle and its incenter 2008-02-11
From Elle:
In the figure, P is the incenter of triangle ABC, the radius of the inscribed circle is 6cm, and the perimeter of triange ABC is 38cm. What is the area of triangle ABC?
Answered by Penny Nom.
A briefcase lock 2008-02-11
From Alfred:
A briefcase lock has 3 columns of numbers each 0-9. It opens with the correct combination of three numbers. How many combinations are possible to open the lock.
Answered by Penny Nom.
The angles in a parallelogram 2008-02-11
From lauren:
if the sum of the measures are 360 in a parallelogram. a and d are same measures, c and b are same too. If c is twice the measure of a what are all the angles?
Answered by Penny Nom.
5x^2 - 45 2008-02-11
From Tiana:
factor: 5x^2 - 45
Answered by Stephen La Rocque.
A 20 foot board is cut into two pieces 2008-02-11
From Sherry:
A 20 foot board is cut into two pieces. If we let x= the length of the longer piece, what is the length of the shorter piece?
Answered by Stephen La Rocque.
A square and an equilateral triangle 2008-02-10
From Emily:
A square pen and a pen shaped like an equilateral triangle have equal perimeters. Find the length of the sides of each pen if the sides of the triangular pen are fifteen less than twice a side of the square pen.
Answered by Stephen La Rocque.
The circumference and radius of a circle 2008-02-10
From Ray:
How do you find the circumference or radius of an area presuming it is a circle. Or in other words how do you find the c or r given only the area is 50 sq metres
Answered by Penny Nom.
The diagonal of a cube 2008-02-10
From Praveen:
Given a Cube's diagonal, how does one find the edge of the cube.
Answered by Stephen La Rocque.
A flagpole and a statue 2008-02-09
From Krista:
A flagpole casts a ten meter (m) shadow at the same time as a six metre (m) statute beside it casts a two metre shadow. What is the height of the flagpole??
Answered by Steve La Rocque and Penny Nom.
The width of a rectangle is 4 less than twice its length. 2008-02-09
From Shawn:
The width of a rectangle is 4 less than twice its length. If the area of the rectangle is 72 cm, what is the length of the diagonal
Answered by Stephen La Rocque.
Three hens 2008-02-09
From joan:
A farmer has 3 hens. It is determined that each hen lays 6 eggs in one week. a)find the probability that a hen lays an egg in a randomly chosen day b) find the probability that each hen lays an egg in a randomly chosen day. c) find the probability that no hen lays an egg in a randomly chosen day. d) draw a tree diagram to illustrate the number of hens laying an egg in a randomly chosen day..
Answered by Stephen La Rocque.
A three sided shape 2008-02-09
From Tim:
Question from Tim:

How would I calculate the area in square feet of a "triangle" with sides:

Base is 36 feet; height is 70 feet; and the thirs side is an arch (curve) of 77 feet.

Answered by Harley Weston.
Radians 2008-02-08
From Brian:
I'm teaching Precalculus right now and will be covering the formula to convert angular speed into linear speed. My question is about the units involved. Using v=rw, if r is mearsured in feet and w is measured in radians/second then v is measured in ft/sec. What happened to the units of radians? I can't find a good way to explain why they are not part of units for velocity. Do you have a good explanation? Thanks,
Answered by Stephen La Rocque.
Classifying a triangle 2008-02-07
From kevin:
scalene triangle 8 ft base right side 9.5 left side 12 ft what is the angles
Answered by Penny Nom.
Find two numbers with HCF of 3 and LCM of 180 2008-02-07
From matthew:
Hi, please help me with this, Find two numbers with HCF of 3 and LCM of 180
Answered by Stephen La Rocque and Penny Nom.
A chord of a circle 2008-02-07
From Mahesh:
find the length of the cord intercepted by a circle with equation X2 +Y2 -6X+4Y-12=0 with a line of equation 4X-3Y+2=0 Pl help me solve this problem thanks MNK
Answered by Stephen La Rocque.
Three balls packed in a box 2008-02-07
From jasmin:
Four spherical balls having diameter of 4 cm are placed in a square box whose inside base dimensions are 8 cm. In the space between the first four spherical balls, a fifth spherical ball of the same diameter is placed. How deep must the box be in order that the top will touch the fifth ball?
Answered by Stephen La Rocque.
The distance around a building 2008-02-06
From olivia:
If you walk around a building that is 102,869 sq ft, how many miles is that?
Answered by Penny Nom.
Bales of hay 2008-02-06
From mark:
5 bales of hay are weighed 2 at a time in all possible ways. The weights are
110
112
113
114
115
116
117
118
120
121
How much does each bale weigh

Answered by Stephen La Rocque and Penny Nom.
A pre-employment evaluation 2008-02-06
From lisa:
An employer gives a pre-employment evaluation to a large group of applicants. The scores are normally distributed with a mean of 154 and a standard deviation of 21. The employer wants to interview only those applicants who score in the top 15%. What should the cut off score be for the interviews?
Answered by Harley Weston.
The equation of a circle 2008-02-05
From aime:
Find the equation of the circle tangent to 3x+4y-15=0 at P1(1,3) and passing through P2(6,3) and P3(0,5)?
Answered by Stephen La Rocque.
Multiplying decimal numbers 2008-02-05
From alwyn:
Why should when you Multiplying Decimal numbers is value becoming less and less? don't you think even decimal number is a quantity and in no chance when it multiplies its should become less or nil !!!

In fact all Multiplying and or adding the value will go up and only when you subtract and divide it should become less !!

Answered by Stephen La Rocque and Penny Nom.
Two spinners 2008-02-05
From Kim:
Zack has two spinners with numbers on them. the probability of spinning a 7 on both spinners is 3/20. if the probability of spinning a 7 on the first spinner alone is 1/4 what is the probability of getting a 7 on the second spinner alone?
Answered by Penny Nom.
The radius of a circular arc 2008-02-04
From Bill:
Hi,the Central Angle of a sector of a circle is 40 degrees. The circular arc of the curve has a chord length of 3000 ft. Find the radius(r) of the circular arc.
Answered by Penny Nom.
The volume of a pyramid 2008-02-03
From Britni:
The base of this pyramid is an equilateral triangle with sides 8 units. The altitude of the pyramid is square root of 3. What is its volume?
Answered by Penny Nom.
Total profit 2008-02-03
From Rhonda:
I help to manage a concession stand. We sell a variety of items. Some have a profit of 60%, some 80%, etc. We have done nothing scientific to choose our prices. We just try to stay in the ballpark of schools around us. Our administrator wants to pay those that run the concession stand a percent of what is in the cash box after start-up money is taken out. We have some gathered some data (not much) on how many of each item we sell and can certainly calculate percent profit. Is the profit for the entire concession stand a weighed average? I'm thinking (% profit on candy)(number candy sold) + (percent profit nachos)(number nachos sold) + (percent profit pop)(number pop sold), etc. Will this work? Does the AMOUNT of profit for these different items need to be figured-in? Hmmm. . .
Answered by Victoria West and Harley Weston.
A inear system 2008-02-03
From Kelly:
is (-1,5) a solution of each system? y=-x+4 y=-1/5x
Answered by Penny Nom.
Numbers and Operations 2008-02-02
From Charlotte:
On the TABE (Test for Adult Basic Education) test, one of the categories is number operation. I would like to know what consist of number operation.
Answered by Penny Nom.
The size of a lot 2008-02-02
From linda:
Hi, could you please tell me how close to an acre of land my lot is. The lot size is 140 ft by 200ft. I am trying to see how close to an acre it is.
Answered by Penny Nom.
The perimeter of a semicircle 2008-02-02
From Lisa:
How do you find the perimeter of a semi circle when you are only given the diameter, say 24 units?
Answered by Penny Nom.
Two equations in x and y 2008-02-01
From leslie:
6x+8y=40 4x+5y=25
Answered by Penny Nom.
The dimensions of a rectangle 2008-02-01
From Evan:
you are given that the area of a rectangle is 21.6in sq. find the base and the height of the rectangle.
Answered by Penny Nom.
Tiles on a basement floor 2008-01-31
From ORIETTA:
I have a basement 1000 sq ft I want to purchase tile that are 18x18inches and they are charging 4.15 sq ft how much would it cost since I will need less tiles to cover the area since it's not 12X12
Answered by Penny Nom.
A 24 hour clock 2008-01-31
From CARLA:
at a certain time i looked at a 24-hour digital clock and noticed some things about the time. (1 The number in the hours section plus the number in the minutes section makes 60.(2 One of the digits in the minutes section is the square root of the other digit in the minutes section.(3 The sum of the digits in the minutes section is the number in the hours section reversed. what is the time?
Answered by Penny Nom.
A dependent variable 2008-01-30
From lavada:
i need the definition to dependent variable
Answered by Stephen La Rocque.
The standard form of a line 2008-01-30
From Thomas:
I'm attempting to get this in standard form

slope=3;(1,5)

Answered by Penny Nom.
Sum and difference of cubes 2008-01-30
From Amanda:
It has been a really long time since I was in Algebra and I can't remember how to factor cubes such as x^3 +81 or subtracting/adding fractions with variables such as [1/(x+h)+2]-[1/x+2]. Please help!!!
Answered by Penny Nom.
The acre measurement of a property 2008-01-30
From Kim:
I would like to know the acre measurement of a property that is 673.68 feet by 820.76 feet.
Answered by Penny Nom.
What solid figure has 1 flat surface and 0 vertices? 2008-01-30
From Cheronda:
What solid figure has 1 flat surface and 0 vertices?
Answered by Stephen La Rocque and Harley Weston.
Integration by parts 2008-01-30
From seth:
hi i really dont understand integr ation by parts. for example, the integral(t^2sintdt. i have u=t^2 and v'=sint also u'=t^/3 v=-cost for the formula i have uv-integralvu' dx this is all well and good but i cant get it right.
Answered by Harley Weston.
The interior angles of a parallelogram 2008-01-29
From amber:
the measure of one interior angle of a parallelogram is 42 degrees more than twice the measure of another angle. Find the measure of each angle.
Answered by Penny Nom.
Digging a pool 2008-01-29
From tom:
digging a pond 100ft by 70ft by12ft with a 2:1 side slope. how many cubic yards will be removed
Answered by Harley Weston.
Volume of a log 2008-01-29
From Reji:
i purchased a teak wood length 20feet and perimeter 60inch. How much cubic feet will be the wood.
Answered by Stephen La Rocque.
Repeating decimals 2008-01-29
From Malise:
Write each repeating decimal using bar notation. 0.428571428...
Answered by Penny Nom.
Angle of Elevation 2008-01-29
From Rita:
Uluru or Ayers Rock is a sacred place for Aborigines of the western desert of Australia. Chun-Wei uses a surveying device to measure the angle of elevation to the top of the rock to be 11.5 degrees. He walks half a mile closer and measures the angle of elevation to be 23.9 degrees. How high is Ayers Rock in feet?
Answered by Stephen La Rocque.
Belled-out pier 2008-01-28
From Gina:
I need to know how to find the total yards needed to fill a concrete pier that is 54"/ 108" and 26' deep. That is...54" @ the top of the pier belled to 108" @ the bottom...26' deep.
Answered by Stephen La Rocque.
The interior angles of a parallelogram 2008-01-28
From steffie:
How do you calculate the interior angle sof a parallelogram?
Answered by Penny Nom.
Inscribed square in a triangle 2008-01-27
From Adrian:
Consider a right-angled triangle PQR, where QR is the base and PQ is the height. QR=4cm and PQ=3cm. A square is inscribed in this triangle.Determine the length of one side of the square.
Answered by Stephen La Rocque.
How far from the corner will the police catch the car? 2008-01-27
From Joel:
A car is going down the road at a speed of 90km/h in a 50km/h zone. As the car passes a corner, a police cruiser sitting there notices it. Five seconds later the cruiser begins accelerating at 5.0m/s squared. Assuming that he is able to maintain this acceleration, how far from the corner will the police catch the car? How fast will the cruiser be going?
Answered by Stephen La Rocque.
Cross-sectional area of a fire hose 2008-01-27
From Tom:
My question is this I know the equations for the final answer but the book skips how to get to one section of the equation. Q=(a)(v) A= The cross-sectional area of the conduit in square feet (ft2). So if you have a 5 inch hose how do they get A=0.136 ft2 ? And I know the velocity already v= velocity in ft per second.
Answered by Stephen La Rocque.
A lottery 2008-01-27
From Stephen:
formula for calculating five numbers dropping out of ninety numbers.eg: 37-16-29-10-45 dropped out of ninety numbers.What formula can be used to predict the next five numbers out of the ninety?
Answered by Stephen La Rocque.
The volume of a tank 2008-01-27
From Fred:
A cubic tank holds 1,000 kilograms of water what are the dimensions of the tank in meters?
Answered by Harley Weston.
The Shaw family wants a new swimming pool 2008-01-26
From Mable:
The Shaw family wants a new swimming pool in their backyard. The pool company stated that the first job is to remove dirt from an area 30ft x 60ft. Next, a hole must be dug 25ft x 55ft x an average of 6ft deep. The finished pool would measure 20ft x 50ft x 5ft deep (average). How many square yards of dirt must be removed from the lawn?How much would it cost the Shaw family for dirt removal at $46 a cubic yard (nearest cubic yard)?
Answered by Stephen La Rocque.
Cantor's diagonal argument 2008-01-26
From David:
Cantor's theory using a diagonal across a list of real numbers to proven uncoutability has always puzzled me.First in base ten, it feels like hocus pocus so I began thinking of the Boolean numbers as truer representations of place value (on,off). Secondly his list was always arbitrary or so I recollect. Therefore, I suggested using a seriesA=.10000....,B=.01000. C=.11000, etc. Any diagonal is already located among the numbers listed. My only alteration is that since the final digit is always unrepresentably either one or zero, but it must be one or the other, I make an assumption that if x= .abc...1 and y= .abc...2 the only two possibilities and I choose to count F=x+y then then the numbers are countable= Z=sumFi,where I=2+2^2+2^3...

I hope this sketch is enough description, I asked Rudy Rucker more formally but got no mathematical response, someone else gave me some tale about slippery epsilon. What do tyou think of recasting his proofs in more rigorous form? David French

Answered by Claude Tardif and Walter Whiteley.
Sum of factorials 2008-01-26
From Emily:
What is the tens digit in the sum 7! + 8! + 9! ... + 2006!
Answered by Stephen La Rocque.
A car goes 15 miles on a gallon of gas 2008-01-26
From Raven:
A car goes 15 miles on a gallon of gas when it goes 50mph. When the car is driven 60mph it only goes 80% as far. How far will it travel on a gallon of gas at 60mph? I solved this two ways: 15 * .80 = 12 It goes only 12 miles or x/60 = 15/50 which is x = 12

This answer in the book is 4/5 * 15 = x. Then x = 12 How did they come up with 4/5?

Answered by Penny Nom.
Inflection points 2008-01-25
From Armando:
Hi, Im trying to write a program that takes an equation ( f(x) = 0 ) and returns a list of the inflexion points in a given interval. there must be (I think) a mathematical method or algorithm to do this, probably involving the (second) derivate of the function. However I have not found such a method yet. Any help on this will be much appreciated.
Answered by Stephen La Rocque and Harley Weston.
Linear feet to metres 2008-01-25
From Jillian:
i have a measurement of something in linear feet. i dont know what a linear foot is so i want to convert it to meters to have an idea. how do i convert.
Answered by Penny Nom.
Tossing a coin 2008-01-25
From Christopher:
Jack tosses a coin 30 times. How many times would you expect the coin to land tails side up?
Answered by Harley Weston.
The standard form of a line 2008-01-24
From James:
I need to put it in Standard from ax+by=c the problem is (-4,0); parallel to y=-2+1 how can I put this in Standard from
Answered by Stephen La Rocque.
Finding the area of an isosceles triangle given one angle and the inradius 2008-01-24
From Saurabh:
Given an isosceles Triangle, whose one angle is 120 and inradius is √3. So area of triangle is?
Answered by Stephen La Rocque.
A relationship between numbers 2008-01-24
From Robyn:
Study these problems: ? 2=4 ? 5=25 ? 9=81

1. Explain what ? means (hint there are two answers).
2. What number is equal to ? 6
3. What number does ? 12 represent?

Answered by Stephen La Rocque and Penny Nom.
An integral 2008-01-24
From Meghan:
Evaluate the indefinite integral of (x^3 +1)^(1/3) * x^5 * dx.
Answered by Brennan Yaremko.
Three consecutive odd integers 2008-01-24
From daniel:
find three consecutive odd integers such that three times the second minus the third is 11 more than the first. help! i have x+1, x+3, x+5 as my 3 odd integers is this correct its been a while.
Answered by Penny Nom.
Absolute value 2008-01-24
From Cara:
Give the absolute value of this equation. {-3 + 1} - {5 - 7} note: the brackets are the absolute value lines.
Answered by Penny Nom.
A 3 digit number 2008-01-23
From Adrian:
Consider a 3-digit number,abc,where a is not equal to c.Switch it so that it becomes cba.Subtract the lesser from the greater to obtain rst.Switch this so that it is tsr.Prove that rst+tsr=1089
Answered by Stephen La Rocque.
An integral 2008-01-23
From Russell:
I checked out the integration site, but would like an explanation for integrating this integral
Tan^3[x] * _dx__
Cos^2[x]
Hope this input is correct! the part that throws me off is Tan^3[x], if it was squared rather than cubed I would have a easier time dealing with this one.

Answered by Harley Weston.
Permuting the letters in ELEVEN 2008-01-23
From Beth:
(i) Find the number of distinct permutations that can be formed from all of the letters of the word ELEVEN.
(ii) How many of them begin and end with E?
(iii) How many of them have the 3 E's together?
(iv) How many begin with E and end with N?

Answered by Stephen La Rocque.
Find the slope of the line with equation 7-3y=2x 2008-01-23
From nell:
Find the slope of the line with equation 7-3y=2x
Answered by Stephen La Rocque.
Change a mixed fraction into a decimal 2008-01-23
From Qiana:
I need to know how to change a mixed fraction into a decimal. the mixed fraction is 9 1/2
Answered by Penny Nom.
Out of school applications of Pythagoras Theorem 2008-01-23
From Laura:
Hi, I am currently working on a math summative in which I have to choose a real life subject and relate it back to the material in my grade 12 math class. I find the history and discovery behind the Pythagorean Theorem and Identity very interesting, but I have yet to find a real-life application of the equations. Yes, I know they are used for finding distances, heights etc., but realistically, how many people actually use it in those situations? Very few. I was hoping for a new application. Is the pythagorean theorem (sin^2x + cos^2x = 1) even applicable? Thank you, Laura
Answered by Harley Weston.
The smallest possible perimeter 2008-01-23
From RS:
If two points of a triangle are fixed, then how can the third point be placed in order to get the smallest possible perimeter of the triangle.
Answered by Chris Fisher and Penny Nom.
A parallelogram and a rhombus 2008-01-22
From miguel:
i have a problem proving a parallelogram a rhombus.. if a diagonal of a parallelogram bisects an angle of the parallelogram , then its a rhombus prove
Answered by Stephen La Rocque and Walter Whiteley.
A linear system 2008-01-22
From Lisa:
I am supposed to use augmented matrices to solve this system of equations. x - 3y = 5; 4x - 12y = 13. I think it is unsolvable but I want to be sure I am not just giving up. Thanks in advance for your help! Lisa.
Answered by Stephen La Rocque and Brennan Yaremko.
Number of points with integer coordinates which lie inside the triangle 2008-01-22
From Saurabh:
Number of points with integer coordinates which lie inside the triange whose verticies are (0,0) , (0,21) and (21,0) ???
Answered by Stephen La Rocque.
A toad climbs the stairs 2008-01-22
From Kate:
The question is: A small toad is sitting at the bottom of the stairs. He wants to get to the 10th step, but is having some difficulty. He jumps up 2 steps and then back 1 each time. How many jumps will he take to reach the 10th step?
Answered by Stephen La Rocque and Penny Nom.
How many combinations of 8614 are divisible by 7? 2008-01-22
From Rebecca:
How many combinations of 8614 are divisible by 7 equally (with no remainder)?
Answered by Penny Nom.
A regular polygon 2008-01-22
From Andy:
Hi. i am looking for an equation to find the radius of any given regular polygon, knowing the length of one of it's sides and how many sides it has.. for instance. A pentagon with 10 unit side , a hexagon with a 10 unit side, a septagon with a 10 unit side or tougher still a triangle wit a 10 unit side. I am mathematically challenged so please try to keep it in layman's terms. Basically im looking to array rectangles, inside corner to inside corner around a common center. The rectangle's widths change but are always equal/ uniform among one array. Im not looking for the answer but am looking for the formula. Thanx in advance, Andy
Answered by Stephen La Rocque.
The three angles of a triangle 2008-01-21
From val:
two angle in a triangle equal 120 degree. what is the measure of the third angle?
Answered by Penny Nom.
The equation of a line 2008-01-20
From Sara:
The question is:

Write the equation of a line that is parallel to y=3x-2 through the point (-5,9). I think the correct equation for this is y=3x+24. But I have to write it in standard form, and I can't seem to figure out how to do that. Please answer soon! Thanks!

Answered by Stephen La Rocque and Penny Nom.
The line through (-22 ;-1) and (-23, -2) 2008-01-20
From Zachery:
i have a slope that equals 1/1 and my coordinates are (-22;-1) and (-23;-2) and i am trying to change it into the equation y=mx+b and the farthest that i've gotten is y=1x_ (i can not find out how to find b.) So i was wondering if you could help me find out what b is because i have got no clue!!!!!
Answered by Penny Nom.
Choosing 10 numbers from 40 numbers 2008-01-18
From Antwan:
I have 40 numbers........number 1-40. I want to know how many times i can chose 10 of those numbers without picking the same exact sequence twice if its even possible?
Answered by Stephen La Rocque.
Two circles in a rectangle 2008-01-18
From Alex:
One side of a rectangle is 10 and the other is x.

Two circles with equal radii are inscribed in the rectangle and like a Venn Diagram, the circles overlap.

Each circle touches the top and bottom of the rectangle and the left circle touches the left side of the rectangle and the right circle touches the right side of the triangle.

The distance between the centre of each circle is 2x/3.

Find x

Answered by Penny Nom.
Maximize income 2008-01-18
From Chris:
Lemon Motors have been selling an average of 60 new cars per month at $800 over the factory price. They are considering an increase in this markup. A marketing survey indicates that for every $20 increase, they will sell 1 less car per month. What should their new markup be in order to maximize income?
Answered by Stephen La Rocque and Harley Weston.
A word problem 2008-01-18
From Marouf:
How can you make a word problem for 6 2/3 and solve the word problem? I'm having trouble. Please help
Answered by Stephen La Rocque.
Two bikers race on a circular track 2008-01-17
From Abby:
Two bikers race on a circular track. Biker A can circle the track in 8 minutes, and Biker B can circle the track in 6 minutes. From the start of the race, how many minutes will it be before Biker B overtakes Biker A?
Answered by Stephen La Rocque and Penny Nom.
The angles of a triangle given the three sides 2008-01-17
From Lucy:
Is there a way to find the angles of a triangle just by knowing the lengths of it's sides? It seems like the would be a relationship between the two, but I'm not sure.
Answered by Stephen La Rocque and Harley Weston.
GCF and LCD 2008-01-17
From Peter:
I need to know the differences between GCF and LCD.
Answered by Penny Nom.
How many meters/kilometers/millimeters would that be? 2008-01-17
From chantal:
if a ocean/lake/river is 250meters from where you are, how long would it take to get to shore? how many meters/kilometers/millimeters would that be?
Answered by Penny Nom.
8/9 + x = 9/5 2008-01-17
From nas:
8/9+x=9/5
Answered by Penny Nom.
What is the relationship between 0/6, 2/4, 4/2, 6/0? 2008-01-17
From Olugbenga:
What is the relationship between 0/6, 2/4, 4/2, 6/0?
Answered by Penny Nom.
A sector of a circle 2008-01-17
From Amandsa:
What is the formula for finding the sector of a circle?
Answered by Penny Nom.
A large concrete shape 2008-01-16
From Keith:
what is the cubic yards of an area that is not a perfect 1/4 circle? The dimensions are 100ft. x 60ft. x 125ft. curcumferal arch x 3ft. depth?
Answered by Stephen La Rocque.
Fact families with multiplication and division 2008-01-16
From Donna:
I am trying to help my granddaughter with some math homework. I looked at what you have on fact families. Am I correct to assume that the same process applies to multiplication and division as well? Her question asks for fact families for 2, 8, & 16.
Answered by Penny Nom.
Graphing a parabola 2008-01-16
From Sean:
How do I find the roots and describe the roots when graphing a parabola?

y = - x^2 - 4x -3

Answered by Stephen La Rocque.
Explaining the factoring for the difference of cubes 2008-01-16
From Bill:
A student asked me where did the "difference of cubes" and "sum of cubes" come from. I did not have an answer for her. She is very bright and understands how they work but wanted to know where they derived from. Any help you can offer would be great. Thanks
Answered by Stephen La Rocque.
A 16 sided lot 2008-01-16
From R:
I am having a land of 16 irregular sides i know all the length of the sides how to calculate the area of the land
Answered by Harley Weston.
How many rolls of stock do I need to buy? 2008-01-16
From Dan:
How many rolls of stock do I need to buy?
Given:
The rolls are 40" wide, 20" roll o.d., 4" core o.d., gauge is 0.008" I need to cut these master rolls in 6" strips (4" of waste per master roll) and I need 1,270,500 linear feet of this strip.

Answered by Stephen La Rocque.
8 workers dug 3/8 of a tunnel in 10 days 2008-01-15
From Marc:
8 workers dug 3/8 of a tunnel in 10 days. If the need to finish the remaining 5/8 of the tunnel in 3 1/3 days, how many more workers must they hire?
Answered by Penny Nom.
Balancing on a fulcrum (net torque and equilbrium) 2008-01-15
From Eric:
If I have a 4" bar that has no weight value with a 275lb. weight on one side and a 125lb. weight on the other side what would be the folcrum point? Please keep the answer very simple. Also is there a fixed equation to figure folcrum points? Is there an equation for if the 4" bar had a weight value?
Answered by Stephen La Rocque.
Maximum volume of a box 2008-01-15
From Rajesh:
A square piece of a cardboard of sides ten inches has four equal peices are removed at the corners, then the sides are turned up to form an open box. What is the maximum volume such a box can have?
Answered by Stephen La Rocque.
Two rectangles 2008-01-15
From Paul:
If the smaller dimension of a rectangle is increased by 3 feet and the larger dimension by 5 feet, one dimension becomes 3/5 of the other, and the area is increased by 135 square feet. find the original dimensions
Answered by Penny Nom.
Constructing a pool 2008-01-15
From Scott:
what it the cubic yards needed for this pool. It is 14ft. X 28ft. and 6 inches thick. It is 3.5ft. deep on the shallow end and 6ft. deep at the deep end.
Answered by Stephen La Rocque.
How many bags of cement will I need? 2008-01-15
From James:
I want to extend my front porshe more to the left atleast 14' 1" length wise and 5' 1" width wise with 4" thickness. Now i already decimalized in feet those inches, resulting with 14.08 length and 5.08width and 0.33 of thickness. I then Get my square feet of 71.53. And go on to cubic feet getting a 23.60 in cubic feet. Now i want to convert to cubic yards so i then divide 23.60 into 27 and get 0.87. Now my question is how would i use this to go and buy the necessary bags of cement knowing that a 94 lb. bag of cement is 1 cubic foot of bulk material. To fill 4 inch of thickness and 14' of length and 5 of width?
Answered by Stephen La Rocque.
2 divided by x plus 1 is equal to 3 2008-01-14
From chilly:
with x being the subject of the formula, what is the answer of 2 divided by x plus 1 is equal to 3?
Answered by Penny Nom.
Two solutions using the law of sines 2008-01-14
From Kate:
I am working on the Law of Sines and I have a problem that says: Find a value for b so that the triangle has 2 solutions.

I am given that A = 36 degrees and a = 5. Now, I learned that for a triangle to have 2 solutions, h < a < b. BUT...my answer key says the answer is: 5 < b < 5/sin 36. I can't figure out how to make this fit with h < a < b.

Answered by Harley Weston.
Conditional probability 2008-01-14
From Sara:
Hello! Ian has a photocopier in his office. If the photocopier is working on a certain day then the probability that it will be working the next day is 0.9. If the photocopier is not working on a certain day then the probability that it will not be working the next day is 0.3.
The photocopier is not working on Monday.
(a) Calculate the probability that it will be working on Tuesday and Wednesday.
(b) Calculate the probability that it will not be working on Wednesday.
Could you please draw a tree diagram for this problem.

Answered by Penny Nom.
The sum of two numbers is 36 2008-01-14
From May:
The sum of two numbers is 36.
Their product is 320.
What are the two numbers?

Answered by Penny Nom.
How tall is the pole? 2008-01-13
From Michael:
If a 25 foot cable is attached to the top of a pole, and that cable attaches to the ground ten feet away from that pole, how tall is the pole? The 25 foot cable is the hypotenuse of a right triangle with a base of ten feet.
Answered by Leeanne Boehm and Penny Nom.
Oil in a tank 2008-01-13
From tanakura:
an oil tank contain 208.3 gallons of oil .whenever the amount of oil drops below 70 gallons,an alarm sound .if 153.4 gallons are pumped into a delivery truck ,how many gallons must be pumped back into the tank in order to shut of the alarm?
Answered by Penny Nom.
1.2 as a fraction 2008-01-12
From Confused:
i have confused my brain and its driving me crazy i am trying to write 1.2 as a fraction but each time i do it i come out with a different number please help
Answered by Penny Nom.
The area of a sector of a circle 2008-01-11
From Serenity:
A denotes the area of a sector of a circle of radius r formed by the central angle theta. Find the area A if r=10 meters and theta = 1/2 radians.
Answered by Penny Nom.
A circle of diameter 12 cm 2008-01-11
From Marcy:
The diameter of the circle is 12cm. I need to convert the diameter to m2.
Answered by Penny Nom.
Linear systems 2008-01-10
From Sandra:
I have two questions. Please solve both. Thank you.

1.) In your chemistry class, you have a bottle of 5% boric acid solution and a bottle of 2% boric acid solution. You need 60 milliliters of a 3% boric acid for an experiment. How much of each solution do you need to mix them together?

2.) You plant a 14-inch hemlock tree in your backyard that grows at a rate of 4 inches per year and an 8 inch blue spruce that grows at a rate of 6 inches per year. In how many years after you plant the trees will the trees be the same height? How tall will each tree be? Thank you. I did not understand neither problem, so I could not show any work. Please help.

Answered by Stephen La Rocque and Harley Weston.
Average Yearly Percentage Change 2008-01-10
From Ray:
It's about average test scores over a three year span. If the scores were:

2005- 52.6%
2006- 42.5%
2007- 45.5%


What is the average amount they changed each year in three years?

Answered by Stephen La Rocque and Harley Weston.
Forces on an inclined plane 2008-01-10
From Ron:
A body that weighs 540lbs is caused to slide up an inclined plane with a uniform velocity by a force that acts parallel to the plane. For each foot of horizontal distance, there is a vertical rise of 2in. If the coefficient of sliding friction is 0.16, what force is required to move the body?
Answered by Stephen La Rocque.
Base salary plus commission 2008-01-10
From mike:
i need some help with this word problem. in november 2001, hall kinion advertised an account executive position in savannah georgia for a salesperson with experience selling internet services. the job advertised a 3500 dollar base salary and 24000 dollar to 48000 dollar in commissions in the first year. Assuming a 20 percent commission rate, how many dollars in sales would u have to generate to earn 64000 dollars annually.
Answered by Penny Nom.
Seven circles in a circle 2008-01-10
From fae:
what is the area of the remaining portion of a large circle with radius 12cm and the seven smaller and equal circles just fit inside?
Answered by Penny Nom.
Sqrt(x^2 + y^2) 2008-01-10
From Darcie:
Can you simplify this problem?: square root of (x squared plus y squared). Note: x squared plus y squared is under the radical.
Answered by Penny Nom.
A plywood sheet in a doorway 2008-01-10
From albetel:
adoor is 7 feet tall and 36 inches wide. what is the widest sheet of plywood that can fit through the door?
Answered by Penny Nom.
2(x - 5) + 45 = 4 - (3 - 2x) 2008-01-09
From Kieran:
Solve
2(x-5)+45=4-(3-2x)

Answered by Penny Nom.
How do I write the following in exponential form? 2008-01-09
From Mandy:
How do I write the following in exponential form?

( square root of a^-2 b)^5

Answered by Penny Nom.
A cylinder of jelly beans 2008-01-08
From Kelly:
We are trying to figure out how many jelly beans it will take to fill a cylinder that is 10 feet tall and has a diameter of 6"?

also, if it can be coverted to ounces and then pounds so we know how much to order.

Answered by Stephen La Rocque.
Exponential form 2008-01-08
From liam:
i was wondering if anyone could help me with this question and tell me how and what the did; exponetial form of 4 326 180 501
Answered by Stephen La Rocque.
Donuts and bagels for the first period class 2008-01-08
From Karen:
Mr. Pace bought breakfast for his first period class. Ten donuts and fifteen bagels cost a total of $15.75. Mrs. Pace bought breakfast for her class. Five donuts and twenty bagels cost a total of $17.00. Set up and solve a system of linear equations to determine the cost of one donut and one bagel.
Answered by Penny Nom.
A binomial probability problem 2008-01-08
From rougi:
According to statistics of the company about 4% of the people who buy tickets do not arrive to the flight . The company sells 205 tickets to plane that has 200 places . Use the binomial distribution to calculate what is the probability that all the people who come to the flight get a place.
Answered by Harley Weston.
Multiplying numbers in bases other than 10 2008-01-08
From Imaiya:
In class we've been learning about numbers with a base other than ten. For example, base 7, base 5, and base 6. What's really stumping me is how to multiply these numbers, when they aren't a base of 10. I've had it explained to me once, but don't seem to understand it. Here are the equations, and I've listed the first few numbers for that particular base.

Base 5: 0,1,2,3,4,10,11,12,13,14,20 etc... The Equation: 34 x 42 Base 9: 1,2,3,4,5,6,7,8,10,11,12,13,14, etc... The Equation: 56 x 78

If you could explain to me how to multiply numbers with any type of base, I'd really appreaciate it. Also if you could show me how to do one of the questions that I listed, that would really help me =) Thanks so much, and once again, thank you for your great site!

Answered by Stephen La Rocque.
How far is the jet from the lighthouse? 2008-01-07
From Natalie:
Question: A ship spots a lighthouse that is 53m high, at an angle of elevation of 7 degrees that is directly north of the ship. The same ship spots a jet travelling N62E at an altitude of 1500m with an angle of elevation of 15 degrees. How far is the jet from the lighthouse?

Natalie

Answered by Harley Weston.
What year were the girls born in? 2008-01-06
From Nyra:
Christmas 2004 was over and three teenagers were looking forward to their birthdays. Eric's was in February and Lisa's and Tammy's in January. Tammy is 8 weeks older than Eric, but 3 days younger than Lisa. What year were the girls born in?
Answered by Claude Tardif.
The radius of a planet 2008-01-06
From Ben:
Two people who are both h feet tall are standing on a spherical planet. One person walks a distance d in feet away from the the other person. At this point, the person walking turns around and can no longer see the top of the other persons head. What is the radius of the planet?
Answered by Stephen La Rocque and Harley Weston.
Partial fractions 2008-01-06
From taiwo:
i copied the question below from john bird's higher engineering mathematics fourth edition (pg 19) after solving all the partial fraction questions from the book except this:

x^3 + 4^2 + 20x - 7 over (x-1)^2 (x^+8). i will appreciate it if you help me out so i can know whether their answer is correct. thanks alot

Answered by Harley Weston.
The area of a piece of property 2008-01-05
From Greg:
I need to know the area of a six sided piece of property. I sent a map by regular mail.
Answered by Harley Weston.
One trillion divided by 2,600,000 2008-01-05
From Carlton:
If you divide one trillion by 2,600,000 what is the answer?
Answered by Penny Nom.
What is the speed of the plane in still air? 2008-01-05
From oscar:
A plane flies 400 miles with the wind and 340 miles against the wind in the same length of time. If the speed of the wind is 24mph, what is the speed of the plane in still air?
Answered by Penny Nom.
The height of a triangle 2008-01-04
From Dara:
The area of a triangle is 54m2. If its base is 5m, find its height.
Answered by Penny Nom.
The shortest distance to a line 2008-01-04
From nana:
3y+4x-12=0 is the equation of the line AB on a graph......the line AB joins both y and x axises together.....point A is on y axis and its value is 3....point B is on x axis and its value is 4.....Question : from point O(0,0).find the point ont the line AB
Answered by Stephen La Rocque.
A tangent to a circle 2008-01-04
From adam:
Find a>0 so that the line y=x+a is a tangent to the circle x^2 +y^2=2.
Answered by Stephen La Rocque and Harley Weston.
Surds 2008-01-04
From INDRAJIT:
SUBJECT: SURD How do I simplify this expression with surds in the denominator? INDRAJIT INDIA
Answered by Stephen La Rocque.
What length of hose will I need? 2008-01-03
From Justin:
I have one cubic yard of liquid that I want to convert to a cylindrical volume with the inside dimensions being 3/4 of an inch. I am trying determine what the height/length would be to set these equal. i.e. What length of hose will I need (with an inside diameter of 3/4") in order to accommodate exactly one cubic yard of liquid?
Answered by Penny Nom.
Retail hiring in Britain 2008-01-03
From ROGI:
major hiring in retail in BRITAIN,ACCORDING TO MORE IN STORE ARE ASFOLLOWS:9,000 AT SAFEWAY; 5000 AT BOOTS; 3400 AT DEBENHAM; AND 1700 AT MARKS AND SPENCER. WHAT IS PROBABILITY THAT THE A RANDOMLY SELECTED NEW HIRE FROM THESE WAS HIRED BY MARKS AND SPENCER?
Answered by Penny Nom.
A 5 button lock 2008-01-02
From mandy:
i have a problem. I have a 5 button lock.and i seem to have forgotten the combination. You can only push each button once but you can push more than one button at a time and you don't have to use each button but i am pretty sure i used all five. for example it can be 1,2,3,4,5 or 1,2&3,4,5. I was wondering if anyone can get me a list of all the possibilities because i really need to get the box open.
Answered by Penny Nom.
Multiplying exponentials 2008-01-02
From jessie:
Can you please explain how to put x^3 y^2 times x^1 y^3 in simplest exponential form?
Answered by Penny Nom.
Radicals in exponential form 2008-01-02
From Andre:
The title of the section in my textbook is to write each of the following radicals in exponential form.

My question is how do u write the squareroot of 10 in exponential form?

Answered by Penny Nom.
Practical applications of sequences 2008-01-01
From carl:
can you give me examples of different kinds of practical applications of sequences?
Answered by Penny Nom.
Lining up coins visually using geometry and trigonometry 2007-12-31
From Jessica:
a) In what order would you arrange a penny, a nickel, a dime, a quarter, and a half-dollar so that they all have the same apparent size? The diameters of the coins, in thousandths of inches, are as follows: penny, 750; nickel, 835; dime, 705; quarter, 955; half-dollar, 1205.
b) How should the coins be placed, if the distance between the dime and the half-dollar is 100 units? How far from thw dime should your eye be to see that all the coins have the same apparent size?
c) What angle do the coins subtend when they have the same apparent size?

Answered by Stephen La Rocque.
A spherical bubble gum bubble 2007-12-31
From Houston:
Bazooka Joe is blowing a spherical bubble gum bubble. Let V be the volume in the bubble, R the inside of the bubble, and T the thickness of the bubble. V, T, and R are functions of time t.

(a) Write a formula for V in terms of T and R. Hint: Draw a picture
(b) Assume that the amount of bubble gum in the bubble is not changing. What is V'(t)?
(c) After 5 minutes of blowing a bubble gum bubble, the bubble is 3ft in diameter and .01 feet thick. If the inside radius of the bubble is expanding at a rate of .5 feet per minute, how fast is the thickness changing? Hint: Remember that the volume of gum in the bubble does not change over time.

Answered by Harley Weston.
A rectangle is three times as long as it is wide. 2007-12-31
From Robert:
A rectangle is three times as long as it is wide. If the length is decreased by 2 cm and the width is increased by 3 cm, the new rectangle has an area of 42 cm. What are the dimensions of the original rectangle?
Answered by Penny Nom.
The surface area of the rectangular solid 2007-12-30
From Sara:
For my math homework i found the volume of a rectangular solid whose deminsions were in feet and the numbers were 10 by 20 by 30
10 = hight
20 = short side base
30 = long side base
and now i have to find the surface area of the rectangular solid....
i know how to do this but i can't get the right answer it should be 2,200 ft squared but i keep getting 13,00 feet squared

Answered by Penny Nom.
The integral of 1/ (x(x+1)^0.5) 2007-12-29
From Nooruddin:
Integral of
dx / x(x+1)^0.5
(boundaries are 5 and 3)

Answered by Harley Weston.
The standard normal distribution 2007-12-29
From GEORGE:
Statistics texts state that in a normal distribution, 1 standard deviation covers 68%, 2SD 95% 3SD 99.7%. However, on looking at the tables for normal curve areas, the percentage for a z value of 1 is 84.13. 68 % of values are covered at a z value of 0.47. The discrepancies exist even for other values but are smaller. So why is the rule of thumb so different from the Table for Normal Curve areas?
Answered by Harley Weston.
How would one find the radius? 2007-12-29
From Ned:
Given an arc with length of 192 inches (don't know chord length), and arc height of 6 inches, how would one find the radius?
Answered by Stephen La Rocque and Harley Weston.
The centroid of a polygon 2007-12-29
From David:
I am playing around with an idea. I've read previous answers to questions (using surveyor calculation methods) related to finding the area of an irregular shape parcel of land (or irregular polygon). Is there a formula or method for calculating the centroid (mid-point) for this type of polygon. I've seen plenty of methods for calculating the mid-point for a triangle or regular shape poloygon but few for an irregular shape polygon such as a parcel of land. Thanks...
Answered by Harley Weston.
Sally, Charlie and Snoopy in a race 2007-12-29
From Marie:
Sally and Charlie are having a race. Charlie gets a 900 foot lead and runs 8 inches per second. Sally begins at the starting line and runs at a rate of 5 ft per second. Charlie's dog is also in the race. Snoopy starts 1100 feet ahead of Sally and runs toward the starting line at a rate of 1 foot per second.

When will Charlie and Snoopy past each other and how far will they be from the starting line?

Answered by Penny Nom.
Calculate the two possible lengths of the third side. 2007-12-28
From Lisa:
An isosceles triangle has an area of 25.6 sq cm. The two equal sides are 8.4 cm long. Calculate the two possible lengths of the third side.
Answered by Penny Nom.
How many complete revolutions does each wheel make? 2007-12-28
From varoon:
The wheels of a car are of diameter 80cm each. How many complete revolutions does each wheel make in 10 minutes when the car is travelling at a speed of 66kms per hour?
Answered by Penny Nom.
Differentiate 2007-12-28
From taiwo:
i am finding it difficult to use first principle to differentiate this question: y=xcos2x. can u help me.
Answered by Penny Nom.
Finding the equation of a line from its graph 2007-12-25
From ken:
I have a graph and it is asking me to find the slope of each line, and then write its equation. the graph show a negative slope. Would appreciate the answer if you could show me how to do this question.
Kind regards: Ken

Answered by Stephen La Rocque.
The case of the forgotten umbrella 2007-12-25
From rougi:
a lecturer goes to a bar because it looks like raning.he takes an umbrella with him. on the way to bar he forgets the umbrella in the bus with probability 0,1,in the bar with probability 0.3 and on his way home in the taxi with probability 0,6
a)what is probability that he forgets the umbrella on his tripe?
b)when he gets home he realizes that he has forgotten his umbrella,what is probability that he has forgotten it in the taxi?

Answered by Stephen La Rocque and Harley Weston.
An irregular octagon 2007-12-23
From Sheldon:
I am attempting to construct an irregular octagon picture frame out of bamboo. The bamboo is 1" in diameter and the opening should be 20" H X 16"W.
What measurements should be used?

Answered by Penny Nom.
The dimensions of a rectangle 2007-12-22
From Justin:
I'm trying to find the width of a rectangle. I have its area and its height. The area is 32 inches square and the width is 4 feet (48 inches). Can someone help explain this to me?
Answered by Penny Nom.
How many triangles can be formed? 2007-12-21
From pankaj:
Q.There is 2 lines parallel to each other. If 1st has 5 points in it and 2nd 3 points in it,how many triangles can be formed?
Answered by Penny Nom.
The probability of the second win on the third test 2007-12-21
From pankaj:
Q.The probability of india winning a test match against india is 1/2. Assuming independence from match to match. what is the probability that india's second win occurs at 3rd test in a 5 match series?
Answered by Penny Nom.
Volume in cubic feet 2007-12-20
From kim:
if i have the inches and need to convert them to cubic feet, how do i do that? i thought it was height X width X length.
Answered by Penny Nom.
Find the lengths, and explain in a 'let statement' 2007-12-20
From Cordilla:
the length of the sides of a triangle are 3 consecutive whole numbers. The perimeter of the triangle is 102 meters. Find the lengths, and explain in a' let statement'.
Answered by Stephen La Rocque and Harley Weston.
Smallest cone containing a 4cm radius inscribed sphere 2007-12-19
From Eva:
A sphere with a radius of 4cm is inscribed into a cone. Find the minimum volume of the cone.
Answered by Stephen La Rocque.
Simplifying square roots 2007-12-19
From Ciara:
How would you calculate 2 to the square root of 8 plus 4 to the square root of 2 minus 5 to the square root of 2?
Answered by Stephen La Rocque.
Partitioning a clock face 2007-12-19
From Kim:
Using a clock with a regular circular faced dial, draw two straight lines such that the sum of the numbers in each of the three areas is equal.
Answered by The team at Math Central.
Linearize a graph 2007-12-19
From Justene:
My daughter has a physics lab where she must linearize a graph of acc vs. mass. I have no idea how to help her.
Answered by Stephen La Rocque.
Which expression gives the greatest value? 2007-12-18
From Jordan:
'Look at these expressions'

n-2, 2n, n(squared), n over 2, and 2 over n (sorry I can't type the expressions out)

A) Which expression gives the greatest value when n is between 1 and 2?
B) Which expression gives the greatest value when n is between 0 and 1?
c) Which expression gives the greatest value when n is negative?

As you can see, this IS ONE question split in to three parts lol!

Can you tell me how I go about tackling this question? I lost marks on the test because of not understanding the question.

Answered by Stephen La Rocque.
Three workers working together 2007-12-18
From Rajesh:
Worker A can do a piece of work in 15 days, worker B in 12 days, and worker C in 10 days. Worker A works 2 days, worker B 3 days, and worker C 3 days. In what time can worker A and worker B finish the job by working together?
Answered by Penny Nom.
A bronze statue is made in two sizes. 2007-12-17
From Sara:
Hello! A bronze statue is made in two sizes. The taller statue is 15 cm high and the shorter one is 9 cm high. The taller statue weighs 3.75 kg. What is the weight of the shorter statue?
Answered by Penny Nom.
Circle area 2007-12-17
From Marlena:
Ok, i do not see how to get the area of circles! they give you the circumfrance which is 43.96cm and then u have to find the radius and area. I got the area right which was 7cm but then i got 21.98 for a area and the real answer was 150.39cm What did i do wrong?
Answered by Penny Nom.
A 300 ton train locomotive is built at exact half scale in all dimensions 2007-12-16
From Normand:
When a 300 ton train locomotive is built at exact half scale in all dimensions we assumed the weight would be 150 tons. Like a bar of solid steel cut in half would be. Why does this not work out? What are we missing here?
Answered by Penny Nom.
What length scale factor should he use? 2007-12-16
From Lisa:
Hi! Joe wants to enlarge a picture so that its area is doubled. What length scale factor should he use?
Answered by Penny Nom.
lim sinx/(x +tanx) 2007-12-16
From shimelis:
i have problem how do you solve this equation lim sinx/(x +tanx)
Answered by Harley Weston.
The Principle of Mathematical Induction 2007-12-15
From iris:
we have some confusion in our problem. Please help us. We would like to know "the principle of mathematical induction"
(i) for n=1, p(1) is true.
(ii) assume that for n=k>=1, p(k) is true we have to prove p(k+1) is true. Here (Is n=k>=1 true? or Is n=k.1 true?)
Thanks.

Answered by Penny Nom and Victoria West.
The perimeter of circle 2007-12-15
From shaquilla:
what is the perimeter of a circle.
Answered by Penny Nom.
Negative numbers 2007-12-14
From mannal:
what negative integers do we use in the real world? like -30 temperature?
Answered by Penny Nom.
A triangle and a rectangle 2007-12-14
From Someone:
A certain triangle has sides that are, respectively, 6inches, 8inches, and 10 inches. A rectangle with a n area equal to that of the triangle has a width of 3 inches. What is the perimeter of the rectangle, in inches.
Answered by Penny Nom.
Inflation rate 2007-12-14
From cassie:
In 1972, Bob could fill his car with gasoline at a cost of $3.00. In 2007 it cost him $45.00 to fill his car. From 1972 to 2007, what is the annual inflation rate for gasoline? How does this inflation rate change when you change the cost to $0.25 per gallon in 1972 to $3.20 per gallon in 2007?
Answered by Stephen La Rocque.
Finding all the angles 2007-12-13
From Rajesh:
An eqilateral triangle is drawn in a square with one of the side as its base and draw the lines from the other angular sides such that there are four triangles formed inside the square which includes the equilateral triangle.I want to know all the angles of all the triangles formed inside the square.
Answered by Stephen La Rocque and Penny Nom.
A palindrome 2007-12-13
From pradip:
I have discovered that for any n-digit number, when it's terminal digits are reversed,e.g.,1729 & 9721 & the resulting combinations are subtracted, the resulting solution is always a multiple of 9 & when this multiple of 9 is divided by 9, we will always get a palindrome, Example- 1234 & 4231, 4231-1234=2997/9=333., please I am not sure about the above fact, would please help me in confirming my discovery.
Answered by Harley Weston.
A 3 digit number 2007-12-13
From Kim:
Find the number from the following clues:

A) is is a 3 digit even whole # divisible by 5
B} Each digit is different
C) Its hundreds digit is greater than its tens digit
D) It is less that 400, divisible by 3, and has only 1 odd digit.

Answered by Penny Nom.
A 45-45-90 triangle 2007-12-13
From Aaditya:
explain to me please how to do the 45-45-90 theorem when one of the legs(not the hypotenuse) is 3. How do you find the remaining two sides? please help me out.
Answered by Leeanne Boehm.
The dimensions of a rectangle 2007-12-12
From tara:
The length if a rectangle is 5 feet less than twice its width. The perimeter of the rectangle is 38 feet. let w represent the width. write an equation for the perimeter of the rectangle in terms of w. Then solve the equation to find the length and width o the rectangle.
Answered by Penny Nom.
How many houses must they paint? 2007-12-12
From Natalie:
The question is: Four boys work together,painting houses for the summer.For each house they paint they get 256 pounds. If they work for four months of the summer and their expenses are 152 pounds per a month, how many houses must they paint for each of them to have one thousand pounds at the end of the summer?
Answered by Penny Nom.
Find the perimeter of the original rectangle. 2007-12-11
From Hannah:
The area of a rectangle is 360 square meters. If the rectangle's length is increased by 10 meters and the width is decreased by 6 meters, its area does not change. Find the perimeter of the original rectangle.
Answered by Melanie Tyrer.
Shooting an arrow over a wall 2007-12-11
From Amy:
The path of a large arrow fired from a non torsion catapult can be modeled by y=-0.0044x^2 + 1.68x, where x is the distance the arrow travelled (in yards) and y is the height of the arrow (in yards). Given the height of a castle wall, find the safest distance from the wall to launch an arrow over a 120 yard-high wall.
Answered by Victoria West and Stephen La Rocque.
The tangent to a curve 2007-12-10
From Christy:
I know this question is simple but I can't figure out what I'm doing wrong.
Find the equation of the tangent line to the curve 2x^2 - y^4= 1 at the point (1,1).

Answered by Penny Nom.
A math trick 2007-12-10
From Megan:
I need to write a mathematical explanation of why this works!

Start with a four digit number. (a positive integer, and all digits can NOT be the same. At least one must be different)
Rearrange that four digit number.
Subtract the smaller 4-digit number from the larger.
Now circel one digit. (canNOT be zero, because that is already a circle)
Now re-write that number excluding the circled digit.
Compute the sum of the digits.
Now write down the next multiple of 9 that is larger than the sum.
Subtract the Sum from the multiple. (multiple - sum of digits)
Report Difference = to number circled.

The resulting number should be the number that originally circled.

Answered by Penny Nom.
Animals for sale 2007-12-09
From Marion:
This question has come up and I know the answer and the equation, but how do you solve this?
Please share the proof. cows=$10, pigs=$3, chickens=$.50
In any combination buy 100 animals with $100.

Answered by Leeanne Boehm.
The rational root theorem 2007-12-09
From Josh:
Prove the rational root theorem (where, in a polynomial equation, all factors of the constant divided by all factors of the leading coefficient are possible rational roots of the equation).
Answered by Penny Nom.
Imaginary roots 2007-12-09
From Josh:
What is the correlation between imaginary roots (of a quadratic or other polynomial equation) and the graph of the equation? As in, how can one represent imaginary solutions graphically (and why does that work)?
Answered by Harley Weston.
Polygons 2007-12-09
From Nicholle:
a polygon with all sides the same length and all angles the same measure is called a ---- polygon.

What word goes in the blank.

Answered by Penny Nom.
Related rates - tree growth 2007-12-09
From Christy:
How do I go about answering this question, I know I have to find dv/dt, but I'm not sure how to start.
The volume of a certain tree is given by V= 1/12pie C^2h where C is the circumference of the tree at the ground level and h is the height of the tree. If C=5feet and growing at the rate of 0.2feet per yer, and if h=22feet and is growing at 4 feet per year, find the rate of growth of the volume, V.

Answered by Stephen La Rocque and Harley Weston.
LCM over 100 2007-12-09
From Aris:
A teacher has more than 100 sweets. She thinks that if she give 6, 8 or 9 sweets to her students she will have no remaining in the case. What is the smallest number of sweets in the bag?
Answered by Stephen La Rocque.
ln(x)/x 2007-12-07
From Nooruddin:
How can I calculate the absolute minimum of (ln x)/x?
Answered by Stephen La Rocque.
Three students, four bars 2007-12-07
From rougi:
in a certain town, there are 4 different bars and 3 student (independently)decided to go bars :
a)what is probability that these three student go to the same bar (at certain night)?
b) what is probability that these three go to different bars?

Answered by Penny Nom.
A portion of a circle 2007-12-07
From Richard:
please could you tell me the formula and answer to the following. I have a circle portion with a radius of 150mm and an angle of 120 deg. If an arc is drawn from one point to the other as the radii touch the circle circumference, i need to know the area of the segment and the formula to carry out the answer. Many Thanks
Answered by Penny Nom.
Two logic problems 2007-12-07
From Grace:
1. You are an archaeologist that has just unearthed a long-sought triplet of ancient treasure chests. One chest is plated with silver, one with gold, and one with bronze. According to legend, one of the three chests is filled with great treasure, whereas the other two chests house a man-eating python that can rip your head off. Faced with a dilemma, you then notice that there are inscriptions on the chests:

Silver Chest - Treasure is in this Chest.
Gold Chest - Treasure is not in this Chest.
Bronze Chest - Treasure is not in the Gold Chest

. You know that at least one of the inscriptions is true, and at least one of the inscriptions is false. Which chest do you open?
(a) Silver (b) Gold (c) Bronze

Answered by Penny Nom.
Bags of concrete 2007-12-06
From Chris:
I need 1 and one half yards of concrete for a project. If one bag is equal to 0.38 cubic feet, how many bags will I need?
Answered by Penny Nom.
A right triangle 2007-12-06
From Shubhomoy:
The co-ordinates of a hypotenuse are (1,3) and (-4,1). Find the equations of the perpendicular sides.
Answered by Harley Weston.
Plotting a function 2007-12-06
From Paula:
My son has a problem that says plot the point: 2x-3y=9
I have explained to him that there must be a function for y in order to plot that point, but he doesn't think so. The question above that one says find the function for y, and I assume we should use that function. Am I wrong? Can we plot a point without a y function.

Answered by Stephen La Rocque.
System of equations 2007-12-06
From Jenn:
change the equation,x-y=4 to form y=mx+b the solution to the system of equations y=2x and y=-x+3 is
Answered by Stephen La Rocque.
A circle inscribed in a triangle 2007-12-06
From Linnea:
I have a tringle with a circle inscribed in it. My teacher wants me to find the radius of the circle. This is what she gave me to work with. The triangle is ABC, AB = AC = 6, and BC = 4. She also told us to use A(squared) + B(squared) = C(squared). and that there are altitudes and and incenter. I have no idea how to do this.
Answered by Harley Weston.
Chicken and goat feet 2007-12-05
From Kim:
Old McDonald raises goats and chickens. The animals have a total of 100 heads adn 360 feet. How many goats and how many chickens does Mr. McDonald have?
Answered by Stephen La Rocque and Penny Nom.
Third side of a triangle 2007-12-05
From Sudhir:
How can we find the all positive integral values for the third side of a triangle, whose other two sides measure 15 and 20 respectively. (No other information viz. the type of triangle, area, perimeter or angles etc. is given.)
Answered by Stephen La Rocque.
A radius and a tangent to a circle 2007-12-04
From elizabeth:
show that the radius of a circle meets a tangent line to the circle in a 90 degree angle. hint: start by assuming they are not perpendicular and at a contradiction.
Answered by Penny Nom.
How many gallons are in the tank? 2007-12-03
From bobby:
my tank holds 740 gallons and the dimensions are 8 ft tall and 4 ft diameter the wine is 3 ft 7.5 inches from the top can you figure out how many gallons are in the tank ?
Answered by Penny Nom.
Proving a quadrilateral is a rhombus 2007-12-03
From Jeanie:
How do you prove that a quadrilateral is a rhombus because the diagonals of the quadrilateral are perpendicular and bisect each other using the 2-column proof method?
Answered by Stephen La Rocque.
Area = 1/2 ab SinC 2007-12-03
From Eileen:
Given: Acute triangle ABC, with a, b, c, being the respective opposite sides to angle A, angle B, angle C, and altitude, h, drawn from angle B to b. Prove: The area of trianlge ABC=1/2abSin C
Answered by Stephen La Rocque.
Ball bearings in an oil can 2007-12-03
From Lisa:
One hundred ball bearings with radius 5 mm are dropped into a cylindrical can, which is half full of oil. The height of the cylinder is 20 cm and the radius is 8 cm. By how much does the level of the oil rise?
Answered by Stephen La Rocque.
Delivering microwave ovens 2007-12-02
From Barbara:
Fernando and Kevin are delivering microwave ovens to a local appliance store, whose freight elevator can hold no more than 900 pounds. If Fernando weighs 180 pounds, Kevin weighs 240 pounds, and the microwave ovens weigh 30 pounds each, how many microwaves can Fernando and Kevin deliver at one time if they must ride in the elevator as well?
Answered by Stephen La Rocque.
Repeating decimals 2007-12-02
From Jack:
I've been working on an equation to support my theory of repeating decimals. Specifically the correlation between the 9 and 11 denominators. I wanted to know if there was already an equation to describe this correlation? Mine is x/11=.b repeating and b=9x, so if you use any single digit number for x (say 4) if 9 x 4 = 36 then 4/11 = .36363636... I just want to know if this theory already exists and if so, what is it called?
Answered by Victoria West.
6 nickels, 4 dimes, and 2 quarters 2007-12-02
From manny:
Suppose you have 6 nickels, 4 dimes, and 2 quarters in your pocket. If you draw a coin randomly from your pocket, what is the probability that

a. You will draw a dime?

Answered by Penny Nom.
Converting grams of hair spray into milliliters 2007-12-02
From komal:
I am confused that i am converting 75grams of hair spray ( liquid) into ML? but i couldnt get the answer what is perfect answer for that please help me for this thing.
Answered by Penny Nom.
Blood type probabilities 2007-12-01
From Noelle:
Question from Noelle, a student:

Blood Type % Population
O+               37.4%
O-                 6.6%
A+               35.8%
A-                 6.3%
B+                8.5%
B-                 1.5%
AB+              3.4%
AB-                 .6%

I need to make another table showing the probablility of meeting someone in each of the eight blood groups. I'm not sure how to find the probability with percentages. Thanks for your help.
Answered by Penny Nom.

What angle should he turn? 2007-12-01
From Jasmine:
Bob is traveling due north he then turns left 45 degrees followed by four 20 degree turns to the right

What is his new heading if he needed to turn so his heading was SSW what direction and what angle should he turn?

Answered by Walter Whiteley.
The diameter of a gasket 2007-12-01
From Jorge:
I have an application where I am using a circular silicone gasket. I have the total length but would like to calculate the diameter with the given length. What formula should be used? We receive these gaskets in a roll and cut them about 21 inches and glue the ends together to make it one piece. I would prefer to purchase these already cut and glued but require a diameter. If I had the overall diameter from a 21 inch length gasket, I can look on-line for a part that comes close to the diameter.

Jorge

Answered by Harley Weston.
The area of a triangle 2007-11-30
From Kevin:
if the base of the triangle is 13ft and the altitude is 6ft then what is the area, and how do we calculate this?
Answered by Penny Nom.
Gross sales 2007-11-30
From tom:
here is my question: trying to determine what gross sales were.

I have.....Gross sales - (gross X .0675) = 2,245,009

Answered by Penny Nom.
The square footage of a property 2007-11-30
From Warren:
The west property line is 240' long with a 90-degree angle to the south property line.
The south property line is 493' long with a 90-degree angle to the east property line.
The east boundary is 165' long.
The length of the north property line is not known but it is a straight line from the north end of the east property line to the north end of the west property line.

Could you please tell me the total square feet and how many acres?
And how you calculated the answer

Answered by Penny Nom.
Exponential form 2007-11-30
From Sasha:
How do I write the number 127398 in exponential for?
Answered by Penny Nom.
The scale of a picture 2007-11-30
From Sylvia:
1. A artist draws a picture of a house with a rose brush in front. In his picture the rose brush is 1.5 cm high and the house is 7.5 cm high. In reality the rose brush is 0.75 m high. How tall is the house in reality (in meters)? Explain

2. There are six short pieces of link chain. Each having four links. It takes 10 sec to cut a link and 25 sec to weld it back together. What is the shortest possible time to make the longest chain? Explain

Answered by Penny Nom.
Count the legs 2007-11-29
From John:
There is a bus with 7 children inside of it
Each child has 7 rucksacks
In each rucksack there are 7 big cats
QUESTION: How many legs are in the bus?

Answered by Stephen La Rocque..
An augmented matrix 2007-11-29
From Beth:
I am having troubles fiinding this augnmented matrix. I know how to do it but everytime i do it it get a different answer and its not the right answer. Could you help me to make sure I do it right?

Here is the problem:
4x - 3y = 5
2x +9y = 6

Answered by Harley Weston.
More on completing the square 2007-11-29
From June:
x^2 - 6x = 16 I found you already had this question but the answer you gave did not help me to explain how to answer the problem for my daughter. I do not understand how to complete the square for this paticular question and the example you showed did not help. I would appreciate any further help you could give me.
Answered by Penny Nom.
math puzzle 2007-11-29
From Will:
This is a math puzzle called "What's my number"
1. It is a three-digit whole number.
2. It is a square number.
3. It is divisible by 5.
4. it is not an even number.
5.Each of it digit is not different.
6. It is divisible by 9.
7.It is less than 500.
8. Its ones digit is 5.
9.It has only one odd digit.
10. Its tens digit is less than its ones digit.

Answered by Leeanne Boehm.
(32-2 x 5) divided by 2 + 8 2007-11-28
From Kim:
Solve

(32-2 x 5) divided by 2 + 8

Answered by Leeanne Boehm.
An arched entry 2007-11-28
From amber:
i am working with an arched entry. i know that the radius is 25' and the height is 20'-11". i need to know the length of the arch and degree of bend of the arch. how do i find these?
Answered by Stephen La Rocque.
Percent change between two value ranges 2007-11-28
From Joe:
How do you calculate a percent change between tow value ranges - for instance if I project a range for 2007 to be between 100 and 120 and a range for 2008 to be between 120 and 140, how do I calculate the estimated increase between the range? Is it 0% to 40% (taking the two inside values rto calculate the minimum and the two outside values rto calculate the maximum?)
Answered by Harley Weston.
Multiplying an inequality by a negative number 2007-11-27
From Lisa:
Why is it that you have to switch the sign of the inequality when you divide or multiply by an negative number to solve for a variable?
Answered by Penny Nom and Walter Whiteley.
The probability of an odd number 2007-11-27
From Malonda:
Suppose you select a 3-digit number at random from the set of all positive 3-digit numbers. Find each probability. P(odd numbers)
Answered by Stephen La Rocque.
A math riddle 2007-11-27
From Kathleen:
My 4th grade daughter was given a math riddle which she is expected to answer using a problem solving strategy. The question is:

The number of my hundreds and the number of my thousands is 3. The number of my tens is 7 times the number of my thousands. All my digits are different. What am I?

I have tried to come up with an answer, but am beginning to think it is less complicated than what I have come up with.

Answered by Stephen La Rocque.
Will the container hold all the liquid? 2007-11-26
From john:
a bottle contains 750 ml of oil. the oil is poured into a container 12 cm high and with a square base of side 10cm.
(a) will the container hold all the liquid
(b) if the answer to (a) is 'no', how much liquid will be spilt?
(c) if the answer to (a) is 'yes', how much more liquid will the container hold?

Answered by Leeanne Boehm.
Rotating letters 2007-11-26
From sherry:
Ms. Thomas said to her class, when my older was little his nickname was Zed. The nickname my brother gave me was like his but with the first letter turned 90 degrees the second letter changed to the first letter of the alphabet and the third letter turned a half rotation. Does anyone know what my nickname was. My brother gave me the nickname because I slept too much. Could you please explain this to me so I can help my child solve it. Thanks
Answered by Leeanne Boehm.
A mystery shape 2007-11-26
From Daniel:
I am in year 3 and have been given some homework to do with my mum. Question is: my shape has two circular faces and one curved face. It has two edges and no vertices. Can you help me identify the shape please.
Answered by Penny Nom.
Straight lines 2007-11-26
From Divyansh:
hello i am in eleventh class and am, preparing a project on straight lines i cant really find uses of straight lines and its equations in daily life i am also thankful to you in advance and am waiting for your answer eagerly because i need to submit my project only this week thanking you
Answered by Penny Nom.
Find the centroid 2007-11-26
From josh:
vertices X(-2,6) Y(4,10) Z(14,6) find the coordinates of the centroid
Answered by Stephen La Rocque.
An isosceles triangle 2007-11-26
From Nancy:
I needed to help my 9th grade daughter regarding a geometry problem. After a while I realized I am not getting anywhere. I saw that in 2005 someone asked the same question and you gave them a hint. Unfortunately it still did not help. Because I had figured that much!Can you kindly help me proof this problem?
PX and QY are attitudes of acute triangle PQR, and Z is the midpoint of PQ. Can you write a proof that triangle XYZ is isosceles?
I am sure it is something simple I am missing, but I just can not seem to be able to see it. Thank you much.

Answered by Harley Weston.
Completing the square 2007-11-26
From Julie:
x^2 - 6x = 16 Please, Help I am stuck on this math problem. I need to solve by completing the square. I need the steps of solving the problem and the answer, Please.
Answered by Penny Nom.
Maximize the product 2007-11-25
From David:
Hi i have this site call calcchat.com, but i dont understand how they explained this can you take a look? The question is: Direction: Find two positive numbers that satisfy the given requirements. The sum is S and the product is a maximum this is what they did 1) Let x and y be two positive numbers such that x + y = S 2)P = xy 3) = x (S - x) 4) =Sx - x^2 5)...etc. the thing i dont get is how did they go from step 2 to step 3 and also i know this sound dumb but how did they get step 2? =)
Answered by Harley Weston.
The volume of a soccer ball 2007-11-25
From Ria:
A student of me is working on a paper about soccer and math/physics. He wants to calculate the volume of the ball, which is composed of 5- and 6-sided pyramids. We are looking after a formula to express the volume of these pyramids in function of the radius of the ball (and if possible how we can derive this formula). Afterwards we'll try to calculate the difference with the volume of a sphere with radius r.
Answered by Stephen La Rocque.
A rectangular plot of farmland 2007-11-25
From Christy:
A rectangular plot of farmland will be bounded on one side by a river and on the other three sides by a single-strand electric fence. With 800m of wire at your disposal, what is the largest area you can enclose, and what are its dimensions?
Answered by Harley Weston.
A percent as a fraction 2007-11-24
From Abby:
write 43.75% as a fraction or mixed number in lowest terms.
Answered by Penny Nom.
The maximum area of a rectangle 2007-11-23
From Christy:
Question from Christy, a student:

Show that among all rectangles with an 8m perimeter, the one with the largest area is a square.

I know this is simple but I'm not sure if I'm doing it correctly. Here is what I did.

1. A = xy
2. 8 = 2x+2y
3. y = 4-x
4. A = x(4-x) = 4x-x^2


Not sure what to do from this point because I don't know if its right.

Answered by Harley Weston.
Dividing a circle 2007-11-23
From matt:
hi. can you please send me a diagram of how to draw 3 lines in a circle to get 8 sections.
Answered by Stephen La Rocque.
A curve sketch 2007-11-22
From Ahson:
Find critical points, determine the monotonicity and concavity and sketch a graph of f(x) with any local maximum, local minimum and inflection points labeled:

1. f(x) = x^4 - x^3 - 3x^2 + 1

Answered by Harley Weston.
Area of a 17-sided lot 2007-11-21
From Lynda:
My uncle is wanting to buy this piece of land [a 17-sided polygon] but we are questioning the acerage total. the measurements are [on the attached diagram].
Answered by Stephen La Rocque.
What is the sum of the reciprocals? 2007-11-21
From anya:
the sum of two numbers is 32, and the product of these 2 numbers is 48. what i sthe sum of the reciprocals of the 2 numbers? Express answer as a common fraction
Answered by Penny Nom.
A 5-digit whole number 2007-11-19
From sunjoo:
a friend tells you to think of a 5-digit whole number. then do the following in order : multiply the number by 2. add 200 to the result. divide the result by 2. subtract 100 from the result. what is the greatest number you can get as an answer?
Answered by Stephen La Rocque.
The derivative 2007-11-19
From ralf:
Find the derivative of the function
1-. y=1+2x8
2-. y=(1+2x )8

Answered by Harley Weston.
I know it's heavy, but how heavy, per linear foot? 2007-11-19
From Gary:
I am trying to find out the weight of a linear foot of a steel bar with the following measurements.
Thickness - 1.125"
Length - 120"
Height - 18"

Density of carbon steel - .2837
I know it's heavy, but how heavy, per linear foot?

Answered by Stephen La Rocque.
Confidence level 2007-11-19
From Fara:
It is common for public opinion polls to have a " confidence level" of 95%, meaning that there is a 0.95 probability that the poll results are accurate within the claimed margins of error. If six different organizations conduct independent polls, what is the probability that all six of them are accurate within the claimed margins of error? Does the results suggest that with a confidence level of 95%, we can expect that almost all polls will be within the claimed margin of error?
Answered by Harley Weston.
Cubic feet and gallons 2007-11-19
From Leonard:
Every conversion I find to convert a cubic foot to a gallon is in liquid. Question, how would 1 cubic foot convert to gallons in dry measurement? Example say you have 1 Gallon of packing peanuts how many cubic feet would that be? I can do this and have done it for the heck of it, I took a box 12x12x12 inches, which is a cubic foot, filled it with packing peanuts the tried to fill a 1 Gallon container, that amount almost fills a 1 gallon bucket twice. I even ask a math teacher and was told that 1 cubic foot equaled 7.481 gallons, then I said that is liquid, teacher ansered yes, then i ask what about dry and got a response of I do not know.
Answered by Harley Weston.
Elimination of mayan prisoners 2007-11-19
From Jim:
An evil Mayan emperor decides to make an example of 1,000 prisoners. He stands them in a circle with numbers one to a thousand marked on their shirts. He then starts counting: "one in, two, in three out" The third man is immediately executed. This continues round and round and round the circle. While the number in the circle shrinks, every third prisoner is pushed out and executed. And it continues even when there are only two prisoners left alive. What number is on the last prisoner's shirt?
Answered by Victoria West.
Exponential Growth 2007-11-19
From lisa:
Hi i am a 8th grader who is learning about algebra and we are having test on exponent and i don't really get it i asked my teacher and she told me i should pay more attention so i want to know what exponential Growth means
Answered by Penny Nom.
Compatible numbers 2007-11-19
From Brendan:
Let's say you have 291/2 and its between 281/2=14 and 301/2=15. My question is where do the 14 and the 15 come from or in other words how do you get those?
Answered by Penny Nom.
Lottery Problem 2007-11-18
From A lottery problem:
There are 50 numbers in a Lottery Draw and 5 numbers are drawn out. The player chooses 5 numbers can you tell what is the lowest number of entries required to guarantee that 3 of the 5 numbers entered match 3 of the 5 numbers drawn?
Answered by Victoria West.
A rectangle in an ellipse 2007-11-18
From David:
I need to find the max area of a rectangle inscribed in an ellipse with the equation x^2+4y^2=4.. What I have so far is f(x,y)=4xy g(x,y)=x^2+4y^2-4=0,

y=sqrtx^2-4/4 f'(x)=2x^2/sqrt-4x^2+2(sqrt-4+x^2).

What I need to know is how to finish the problem and find the actual mas area of the rectangle. David

Answered by Penny Nom.
ln (x) - x = ln (2) 2007-11-18
From James:
How do you make x the subject of

ln (x) - x = ln (2)

Answered by Stephen La Rocque.
Red balls and blue balls 2007-11-18
From Md:
In a group of 15, 7 have Red Balls, 8 have Blue Balls and 3 have neither. What fraction of the group has both Red Balls and Blue Balls?
Answered by Stephen La Rocque.
Find the radius of a circle given the center and a point on the circle 2007-11-18
From Raymund:
Find the radius if the center is at (0, -5) and one point on the circle is (2,3)
Answered by Stephen La Rocque.
Two lines 2007-11-18
From Kate:
represent qraphically the equations of these lines and their intersecting points

x-y=2 3x+y=11

Answered by Penny Nom.
Red hens and white hens 2007-11-18
From Rapin:
In 15 days, 4 white hens and 3 red hens lay eggs equal to 3 white hens and 5 red hens lay eggs in 12 days , how many days that white hens can lay eggs the same amount as red hens lay eggs in 20 days ?
Answered by Steve La Rocque and Penny Nom.
Exponential form 2007-11-16
From ashley:
write a sentence for exponential form.
Answered by Penny Nom.
lim [x + squareroot(x^2 + 3)] as x->-inf 2007-11-16
From David:
Find the limit. (Hint: treat the expression as a fraction whose denominator is 1, and rationalize the numerator.)

lim [x + squareroot(x^2 + 3)] as x->-inf

i got to

lim -3/(x - squareroot(x^2 + 3)) as x->-inf

but i'm having trouble understanding why the answer is 0 plz explain thx

Answered by Harley Weston.
A rectangular prism 2007-11-16
From nicholas:
What is the height the surface area to a rectangular prism with a volume of 350 milliliters?
Answered by Stephen La Rocque.
How many yards of fencing do I need? 2007-11-16
From Terrie:
I have to order fencing for a 25-acre, rectangular field.One side of the field is 1/4 mile. How many yards of fencing do I need to enclose the field?
Answered by Stephen La Rocque.
A cylindrical tank 2007-11-16
From Mario:
I want to determine how many gallons i have inside a cylinder (tank) that is resting on its side (the Height), NOT standing up. I know V=pi x r2 x H. And 1 cubic foot = 7.48 gallons. Here are the dimensions r=2' H=20'. Now my question is how do I determine how much liquid i have inside, if the level of the liquid is about 1/2 of the way of its Diameter (in other words 2'). Remember this cylinder is lying on its side.
Answered by Penny Nom.
How many yards of dirt? 2007-11-16
From wade:
i own 4acres of land and would like to fill it with 10inches of dirt can you tell me how many yard of dirt it will take to do so
Answered by Penny Nom.
Name the polygon 2007-11-15
From Austin:
My son has a math question that I'm trying to help him with. The question states name the polygon. Then name the vertices. I believe its a trapezoid. Each vertex has a letter and I'm not sure what the teacher is really asking for?
Answered by Penny Nom.
The best price for topsoil 2007-11-15
From Dawn:
What is a better deal for topsoil, $150.00 for 10 cubic yards, $170.00 for 22 tons, or $200.00 for 11.5 yards?
Answered by Penny Nom.
Why do they use the letter b in y=mx+b? 2007-11-15
From virginia:
why do they use the letter b in y=mx+b? im asking this because my math teacher said he will give extra points to anyone who finds this out first and i was hoping you guys would be the ones who could help.
Answered by Penny Nom.
Three colouring the USA map 2007-11-15
From Hammed:
Prove that if each of the 50 states in the USA is assigned one of the colors red, white, or blue, then some pair of adjacent states must have the same color. (If you think that Arizona and Colorado are adjacent, then you may not consider New Mexico and Utah to be adjacent, too. You can't have it both ways!)
Answered by Victoria West.
How do you convert eleven fifteenths into a decimal? 2007-11-14
From Lauren:
How do you convert eleven fifteenths into a decimal?
Answered by Penny Nom and Stephen La Rocque.
x^2 + x^3 = n^2 2007-11-14
From Rapin:
x and n are the whole number and less than 100 , x^2 + x^3 = n^2, please help to solve this equation.
Answered by Penny Nom.
A limit of the form 0 times infinity 2007-11-13
From Russell:
find the following limit

lim x((e^1/x) -1) as x --> infinity

Answered by Penny Nom.
Radial motion 2007-11-13
From abdulai:
a circular saw with diameter 18.4cm rotates at 2400rpm. What is the angular speed of a point on the edge of the saw blade in radians per second?
Answered by Penny Nom.
Our company charges a 4% margin 2007-11-13
From Nadja:
Our company charges a 4% margin on top of a pay rate to obtain the total charge rate. A client is denying that the total charge rate is calculated in the following way:

Pay rate/0.96 = Charge rate

Please could you provide me with an explanation which I can pass on as to why it is calculated in this way?

Answered by Penny Nom.
The areas of a rectangle and a triangle 2007-11-13
From karen:
hoe to calculate the shaded area of a triangle in a rectangle
Answered by Penny Nom.
EFGH x 4 = HGFE 2007-11-13
From ROBIN:
FIND THE NUMBERS THAT EACH LETTER STANDS FOR IN THE PROBLEM BELOW.

EFGH x 4 = HGFE

Answered by Stephen La Rocque.
Local maxima, minima and inflection points 2007-11-13
From Russell:
let f(x) = x^3 - 3a^2^ x +2a^4 with a parameter a > 1.

Find the coordinates of local minimum and local maximum

Find the coordinates of the inflection points

Answered by Harley Weston.
p-values and t-distributions 2007-11-13
From Don:
I have a t-test statistic of 1.28 and a degree of freedom of 6. I know the p-value is .248 I cannot figure out how to calculate that value. I have used software to get it but I want to know how to calculate it using the t-distribution table.
Answered by Harley Weston.
Whole numbers and improper fractions 2007-11-13
From Jennifer:
I would like to know how you convert whole numbers into improper fractions
Answered by Melanie Tyrer.
A particle moves in the xy-plane 2007-11-12
From Russell:
A particle moves in the xy-plane with

X = 2t^3 - 12t^2 + 18t

Y = 3t^4 - 28t^3 + 72t^2

find an equation of the line tangent to the given curve at t_0_ = 1

note: t_0_ is t subscript 0

Answered by Harley Weston.
The third side of a triange 2007-11-12
From Christine:
I need the inequality to describe the third side of a triange if two sides are known. First side is 23 and second side is 15. What is the third side?
Answered by Stephen La Rocque and Victoria West.
Maximize his profit 2007-11-12
From apoorva:
During the summer months Terry makes and sells necklaces on the beach. Last summer he sold the necklaces for $10 each and his sales averaged 20 per day. When he increased the price by $1, he found that he lost two sales per day.

a. Find the demand function, assuming it is linear.
b. If the material for each necklace costs Terry $6, what should the selling price be to maximize his profit?

Answered by Penny Nom.
Family of functions 2007-11-12
From Russell:
Consider the family of functions
f(t)= Asin3t + Acos3t +Bsin8t + Bcos8t

find exact values of parameters A and B so that f(0) = 2 and f ' (0) = 1

Answered by Stephen La Rocque.
A quadratic function word problem 2007-11-12
From liz:
A kernal of popcorn contains water that expands when the kernal is heated, causing it to pop. The equations below give the "popping volume"y(in cubic centimeters per gram) of popcorn with moisture content x(as a percent of the popcorn's weight).

hot air popping: y = -0.761x^2 + 21.4x-94.8
hot oil popping: y = -0.652x^2 + 17.7x -76.0

A) for hot air popping, what moisture content maximizes popping volume? What is the maximum volume?
B) For hot oil popping, what mositure content maximizes popping volume? What is the maximum volume?
C) the moisture content of popcorn tyipcally ranges from 8% to 18%. Graph the equations for hot air and hot oil popping on the interval 8 less then or equal to x and less then or equal to 18.
D) Based on the graphs from part(c), what general statement can you make about the volume of popcorn produced from hot air popping verus hot oil popping for any moisture content in the interval8 less then or equal to x and less then or equal to 18.

Answered by Penny Nom.
6 consecutive multiples of 6 2007-11-11
From jeff:
find 6 consecutive multiples of 6 whose sum is the least common multiple of 13 and 18
Answered by Penny Nom.
How do i convert milliliters to grams for water and also for copper sulfate (CuSO4)? 2007-11-11
From jenn:
how do i convert milliliters to grams for water and also for copper sulfate (CuSO4)?
Answered by Harley Weston.
A normal distribution problem 2007-11-11
From Jenny:
I am a part-time student so that i have no time to ask the lecturer. moreover the book which i borrowed from state library don't have any answer. but i have already done with most of the question. but these three question which i attached is really confusing me. i am very glad that you help me.
Answered by Harley Weston.
The radius of an arch 2007-11-10
From Mark:
How do you determine the raduis or diameter of a circle based on the folowing information:
1. The distance along the circle between two points is 35'-2". This creates an arch.
2. The (chord) distance between the two points is 30'-8".
3. The distance from the center of the chord (on a 90 degree) to the arch is 6'-10 3/4".

Answered by Harley Weston.
Binomial probability 2007-11-10
From areej:
In any table of binomial probability with n trials, why is it that the probabilities for k=n are not tabulated???
Answered by Harley Weston.
Related Rates (streetlamp and shadow) 2007-11-09
From Casey:
A street light is mounted at the top of a 15ft pole. A man 6ft tall walks away from the pole at a rate of 5ft per second. How fast is the tip of his shadow moving when he is 40ft from the pole?
Answered by Stephen La Rocque and Penny Nom.
I have 23.50 in quarters and dimes 2007-11-09
From hannah:
i have 23.50 in quarters and dimes. if i have 15 more dimes than two times the amount of quarters i have, how many coins do i have total?
Answered by Penny Nom.
Two integrals 2007-11-09
From Akilan:
how to integrate these (tan x)^6(sec x)^4 and sinh(x)(cosh(x))^2.

Please send me how to do this question. Having exams on Monday. Please help.

Answered by Harley Weston.
Increasing and decreasing for functions 2007-11-09
From David:
Direction: Identify the open intervals on which the function is increasing or decreasing.

f(x)=1/(x^2)
f'(x)= -2/(x^3)

i understand how to get up until there, and the undf. is x=0, but now i'm having problem setting up the number table chart. i cant remember how, and where to place the increase and decrease + - the chart, for example <---------0----------> where would the increase and the decrease be place?

Answered by Harley Weston.
Meters per second to miles per hour 2007-11-08
From Niecey:
If Greg was timed at 0.91 seconds for 10 meters running the 100 meter dash, at that speed, could he pass a car traveling 15 miles per hour in a school zone?
Answered by Penny Nom.
A rectangular-shaped bedroom 2007-11-08
From Svitlana:
Professor Reissig is doing renovations on one of the rectangular-shaped bedrooms in his house. He is putting up a border around the room. The room’s length is seven feet more than two-thirds of its width. The area of this bedroom is one hundred eighty square feet. If the border costs $1.25 per six inches, what is the minimum cost for the border?
Answered by Penny Nom.
Changing units and sizes of things 2007-11-08
From Carol:
volume of cube = 216 cubic inches how do I calculate cubic feet and calculate its surface area in square feet. a rectangular box has dimensions of 3 x 6 x 15 how do I calculate surface area in square yards and calculate volume in cubic inches. if I double the dimensions how does it affect the volume
Answered by Stephen La Rocque.
Simple interest 2007-11-08
From Lee:
If a bank pays 3% simple interest anually on savings, and you did not take any money out of your account, how much money would you have deposited to earn $45 in interest
Answered by Stephen La Rocque.
Two triangles 2007-11-08
From Barbara:
If the base of a larger triangle is 34 inches long, what is the length of side A of the smaller triangle?

(the small triangle has on top the letter a on the right side of the triangle it has the letter b, and at the bottom of the small triangle it has the number 17)

Answered by Penny Nom.
Related Rates (a water trough) 2007-11-07
From Christina:
A rectangular trough is 3ft long , 2ft across the top and 4 ft deep. If water flows in at the rate of 2ft^3/min, how fast is the surface rising when the water is 1 ft deep ?
Answered by Stephen La Rocque.
Equivalent Fractions 2007-11-07
From Marlene:
find equivalent fractions with a denominator of 8 1/2
Answered by Stephen La Rocque.
The width of a rectangle 2007-11-07
From BJ:
The perimeter of a rectangular field is yards. If the length of the field is yards, what is its width?
Answered by Penny Nom.
What is a rational number between -5.9 and -6? 2007-11-06
From cathy:
What is the rational number between -5.9 and -6?
Answered by Penny Nom.
A 3 digit lock 2007-11-06
From Michael:
We have a 3 digit lock that only allows for 5 possible numbers to be used. (The numbers can repeat) What are the total number of combinations that can be created.
Answered by Penny Nom.
A rectangular hyperbola 2007-11-06
From Gareth:
2. Consider the rectangular hyperbola xy = c2 with parametrization (x, y) = (ct, c/t), and t 6= 0.
i. Derive (i.e. do not just quote) the equations of the tangent and normal at the point P with (parametric) coordinates (cp, c/p).
ii. Consider the points P : (cp, c/p) and Q : (cq, c/q) on the hyperbola. Find the equation of the straight line that joins the points P and Q.
iii. Consider a point R : (cr, c/r) also on the hyperbola, and suppose that in the triangle PRQ, \PRQ = 90◦. Prove that the normal (to the hyperbola) at R is parallel to the line PQ.

Answered by Harley Weston.
Constructing the centre of a circle 2007-11-06
From Carolyn:
I have a line segment AB. I need to divide a segment into 3 parts that are congruent. Please help how to divide it. The answer that I was given previous was not the way the teacher wanted it.

I also need information on how to find the center of a circle given an arc.

Answered by Penny Nom.
The volume of a vial 2007-11-06
From Derrick:
An empty vial has a mass of 65.92 g. If the vial has a mass of 202.91 g when completely filled with liquid carbon terachloride (d = 1.59 g/cm3), what is the volume of the vial? I have tried several solutions but none i can come up with seem to be correct.
Answered by Penny Nom.
The hourly rate plus time and a half 2007-11-05
From Barbara:
simon arrive to work at 8:15am - and left at 10:30pm if simon gets paid by the hour at a rate of $10.00 and time 1/2 for any hours worked over 8 in a day how much did he get paid?

a.120.25
b. 160.75
c. 173.75
d. 180
e. 182.50

Answered by Penny Nom.
The height and area of a triangle 2007-11-05
From Daniella:
I need the exact formulas to find the height, the area,the lenght of triangles...
Answered by Penny Nom.
Great circles 2007-11-05
From Lindsay:
Does a sphere have only ONE great circle? Explain?
Answered by Stephen La Rocque.
The height of a triangle 2007-11-03
From William:
I need to find the height of a triangle if the base is 15, one side is 14 and the other side is 13.
Answered by Stephen La Rocque and Penny Nom.
Chris has twice as many $1 bills as pennies 2007-11-02
From vicky:
Chris has twice as many $1 bills as pennies, twice as many dimes as he has $10 bills, and twice as many pennies as he has dimes. How much money does he have?
Answered by Penny Nom.
Stem & Leaf Plots 2007-11-02
From Jen:
Trying to help my Gr.4 son with his math and i am a little lost here are the stem and leaf plots..
Stem Leaves
4          02
3          1226699
2          011255789
1          33357899

Stem Leaves
2          00011111223334455
1          range566788899

Question - How are the ranges for the stem & leaf plots different ?

Answered by Penny Nom.
I have a piece of material 44 inches by 78 inches 2007-11-02
From Marianne:
I have a piece of material 44 inches by 78 inches. How many linear yards can I get from it?
Answered by Penny Nom.
Convert a diameter of a pencil into centimeters 2007-11-02
From Kaitlyn:
I need to convert a diameter of a pencil into centimeters, but how many diameters are in a centimetres? Thanks =)
Answered by Penny Nom.
A right angle triangle 2007-11-02
From PRAIMNAUTH:
In a right angle triangle , knowing one angle (90 degree ) and the length of the base, how can you calculate the hight and the hypotenuse ?
Answered by Penny Nom.
Completing the square 2007-11-01
From Mark:
An architect is designing a museum entranceway in the shape of a parabolic arch represented by the equation y = -x2 + 20x, where 0 x 20 and all dimensions are expressed in feet. Determine the maximum height, in feet, of the arch.
Answered by Stephen La Rocque.
f(x+y) = f(x) + f(y) + 2xy 2007-11-01
From Marcia:
For all real numbers x and y, let f be a function such that f(x+y) = f(x) + f(y) + 2xy and such that the limit as h -> 0 of f(h) / h = 7, find: f(0), use the definition of the derivative to find f'(x), and find f(x).
Answered by Penny Nom.
A decimal to a fraction 2007-11-01
From Lisa:
i am trying to turn a decimal into a fraction. my example is to turn 10.5 into a fraction at its lowest form
Answered by Penny Nom.
A number puzzle 2007-10-31
From matthew:
please help.make 200.take 4 numbers from the numbers 1-9,place the numbers into a square containing 4 equal squares 1 number to each square.example 13
57
you now have 4 sets of numbers 13 ,57 ,reading across and 15 , 37. reading down i have to find 2 or more ways of making i have to find the numbers that when added together using this method make 200.you can only use each number once.

Answered by Penny Nom.
A wiffleball field 2007-10-31
From Svitlana:
The Adam's family has set up a wiffleball field in their backyard. The bases are arranged like a typical baseball diamond, where the distance between consecutive bases is the same. First base is opposite of third base, and second base is opposite of home plate. The distance between consecutive bases is 50 feet. Now, the pitcher stands 25 feet from home plate and lies on the line between home plate and second base. How far is the pitcher from first base? Round your answer down to the nearest inch.
Answered by Penny Nom.
The base and height of a triangle triangle 2007-10-31
From mike:
How do you find the length of the base or the height of a triangle if the area is 30 square feet?
Answered by Penny Nom.
A circular table is pushed into the corner 2007-10-31
From Will:
A circular table is pushed into the corner of a square room so that one point is 8 inches from one wall and 9 inches from the other. What is the radius of the table?
Answered by Penny Nom.
The rate of change of the concentration of a solution 2007-10-30
From Nicholas:
A barrel initially has two kg of salt dissolved in twenty liters of water. If water flows in the rate of 0.4 liters per minute and the well-mixed salt water solution flow out at the same rate, how much salt is present after 8 minutes? I tried working backwards given the answer but I can't seen to get their answer of ~1.7kg. Any help would be great! Thanks
Answered by Harley Weston.
Compatible numbers 2007-10-30
From Gary:
How do I estimate the quotient. and then tell what compatible numbers were used. 703/7
Answered by Penny Nom.
An irrational number 2007-10-29
From Clara:
Find an irrational number between 0.53 (with 53 as repeating) and 0.54 (with 54 repeating)

I changed each to 53/99 and 54/99 with 1/99 being the difference.

Please help me.

Answered by Stephen La Rocque, Penny Nom and Harley Weston.
Constantly accelerated motion 2007-10-29
From Priyanka:
A, B and C are three points lying in that order on a straight line. A body is projected from B towards A with speed 3 m/s. The body experiences an acceleration of 1 m/s^2 towards C. If BC =20 m, find the time taken to reach C and the distance travelled by the body from the moment of projection until it reaches C.
Answered by Stephen La Rocque.
The height of an isoceles triangle 2007-10-29
From Maura:
I am trying to calculate the height of an isoceles triangle but in this case AB is not equal to AC. How do I calculate this? Thank you, Maura age 13
Answered by Stephen La Rocque.
The length of the side of a triangle 2007-10-28
From devon:
hay ok i have the height of a triangle but how do i find the length od the side if all of the sides are the same length
Answered by Penny Nom.
A line between 2 vertices 2007-10-28
From robert:
what do we call a line between 2 vertices which are not next to each other
Answered by Walter Whiteley.
For which values of k will k/240 be a terminating decimal? 2007-10-28
From Clara:
For which values of k will k/240 be a terminating decimal?
Answered by Stephen La Rocque.
l'Hospital's rule 2007-10-28
From SWAPNA:
Evaluate the following limit using L'Hospitals Rule
lim (sin x/x) ^ 1/(x^2)
x-->0

Answered by Harley Weston.
A circle is inscribed in a square 2007-10-28
From Carolyn:
A circle is inscribed in a square. What percentage of the are of the square is inside the circle.
Answered by Victoria West.
An equalateral triangle is 30 acres 2007-10-28
From mike:
If the area of an equilateral triangle is 30 acres, what is the length of each side in feet or miles?
Answered by Penny Nom and Victoria West.
Complex numbers 2007-10-27
From Dylan:
My problem is to prove:

|z|^2 = zz* Where z is the complex number x + iy and z* is it's complex conjugate x - iy.

If the absolute value of i is 1, then it looks like: |z|^2 = |x+y| |x+y| = x^2 + 2xy + y^2

And zz* = x^2 + y^2. for these to be equal, 2xy = 0. This seems wrong to me. What am I doing wrong?

Answered by Penny Nom.
The realtionship between lcm and gcd 2007-10-27
From Brenda:
why does multiplying lcm of (n,s) by gcd (n,s) give n times s. when (n,s) are any two numbers?
Answered by Penny Nom.
How to solve related rates problems 2007-10-27
From David:
Can you plz explain how and where you come up with an equation to solve this?
Find the rate of change of the distance between the origin and a moving point on the graph of y = sin x if dx/dt = 2 centimeters per second.

Answered by Stephen La Rocque.
Place value 2007-10-27
From eric:
True or False: In the number 378, there are 3 hundreds, 7 tens, and 8 ones.
Answered by Penny Nom.
Is there a practical use for radian measure? 2007-10-26
From Paula:
Is there a practical use for radian measure in any profession? Which professions might us radian as opposed to degree measure?
Answered by Harley Weston.
The largest number you can add ... 2007-10-26
From Dryden:
What is meant by asking what is the greatest number you can add to that number without having to regroup in any place?
Answered by Penny Nom.
collinear points 2007-10-26
From Carl:
the points H,S,D,K,L, and B are collinear.
KS+SB=KB
DH+HS=DS
DH+HK=DK

point L and B are not between any other labeled points on the line.
<------------------------------------------------------------------------->

Answered by Penny Nom.
I need to rebuild a wagon wheel 2007-10-26
From Pat:
I need to rebuild a wagon wheel, the metal wheel rim is 41" diameter, inside the rim is a one and a half inch by one and a half inch wood wheel. I thought I would glue up a hexagon from a 2x6 or 2x8 piece of wood and then draw and cut out the 41" diameter wood circle. ? what would work better the 2x6 or the 2x8 and what is the lenghth of the cuts needed in order to give me the 41' diameter I need.
Answered by Harley Weston.
Area of a triangle formed by three points on a graph 2007-10-26
From Betty:
My question is 'find the are of a triangle whose vertices have coordinates (3,5),(6,-5), and (-4,10)
Answered by Stephen La Rocque.
Hexagon inscribed in a circle 2007-10-26
From VIVEK:
what are the properties of a regular hexagon inscribed in a circle. If the radius of the circle is given then how to find the side of the regular hexagon
Answered by Stephen La Rocque.
Related rates 2007-10-26
From David:
A trough is 12 feet long and 3 feet across the top.(look like an upsidedown triangle square). Its ends are isosceles triangles with altitudes of 3 feet.

a) If water is being pumped into the trough at 2 cubic feet per minute, how fast is the water level rising when h is 1 foot deep?

b) If the water is rising at a rate of 3/8 inch per minute when h=2, determine the rate at which water is being pumped into the trough. thank you so much for helping me out

Answered by Stephen La Rocque.
The length of a rectangle is twice the width 2007-10-26
From Tami:
The length of a rectangle is twice the width. The number of square units in it's arear is four times the number of units in it's perimeter. What are the dimensions of the rectangle?
Answered by Penny Nom.
Magic triangles 2007-10-25
From joshine:
i confuse about a magic triangle.but my teacher ask me to make examples of it in 4 different magic triangle and also different pattern can you help me about it?
Answered by Penny Nom.
Two mirrors 2007-10-24
From Peter:
The reflecting surfaces of two intersecting flat mirrors are at an angle θ (0° < θ < 90°). For a light ray that strikes the horizontal mirror, show that the emerging ray will intersect the incident ray at an angle β = 180° – 2θ.
Answered by Stephen La Rocque.
parabolic arch 2007-10-24
From ABHILASH:
How find parabolic arch perimeter.
Answered by Harley Weston.
Given a six by six square, how rectangles are there in the square? 2007-10-24
From Maria:
Given a six by six square, how rectangles are there in the square?
Answered by Stephen La Rocque.
A sixfold increase 2007-10-24
From Fred:
If I have $500 and it grew to $3,000, what is the correct description of the increase? Is it a sixfold increase (sextupled), or a fivefold increase (quintupled)? When I divide $3,000 by $500, the result is six, therefore, a sixfold increase?
Answered by Stephen La Rocque.
6(x-1)-3(x+1)-12=56+5x 2007-10-24
From Megan:
6(x-1)-3(x+1)-12=56+5x
Answered by Penny Nom.
The length of a rectangle 2007-10-24
From Kharez:
What is the length of a rectangle with an area of 24 square feet and a width of 4 feet?
Answered by Penny Nom.
Arc length and height in a circle 2007-10-23
From Bruce:
I have been trying to find a formula that relates height of a segment from the bottommost point of a circle toward the center of the circle to the corresponding distance along the circumference of the circle (i.e. at the point on the circumference intersected by a line perpendicular to that segment). The unknown variable is the height of the segment; the known variables are the radius of the circle and the distance along the circumference.
Answered by Stephen La Rocque.
An equilateral triangle with height 2 inches 2007-10-23
From Christine:
I'm doing an project in school about tessellations. But first, I have to construct an equilateral triangle with an altitude of exactly 2 inches. I know how to draw an equilateral triangle...but I don't know what I have to do in order for the triangle to have an altitude of 2 inches.
Answered by Penny Nom.
The distance of the Andromeda Galaxy 2007-10-22
From ROLY:
What is 2.5 million miles x 6 trillion miles (light year)
This is the distance of the ANDROMEDA Galaxy.

Answered by Stephen La Rocque.
An algebra exercise 2007-10-22
From Math:
The number of moles of an ideal gas can be found from the formula:

PV/RT=N

solve the formula for "R", the ideal gas constant.

Answered by Penny Nom.
The rate of change of the area of a triangle 2007-10-22
From Ahlee:
So my question is: The included angle of the two sides of a constant equal length s of an isosceles triangle is ϑ.
(a) Show that the area of the triangle is given by A=1/2s^2 sinϑ
(b) If ϑ is increasing at the rate of 1/2 radian per minute, find the rate of change of the area when ϑ=pi/6 and ϑ=pi/3.
(c) Explain why the rate of change of the area of a triangle is not constant even though dϑ/dt is constant

Answered by Penny Nom.
sin 43= y/20 2007-10-21
From stefany:
if sin 43= y/20, what is the value of y to the nearest tenth?
Answered by Penny Nom.
Two probability density functions 2007-10-21
From ravinder:
A program consists of two modules executed sequentially. Let f1(t) and f2(t) respectively denote the probability density functions of time taken to execute the two modules. The probability density function of the overall time taken to execute the program is given by

(1) f1(t) + f2(t)

(2) INTEGRAL OF(f1(x)f2(x)dx) from 0 to t

(3) INTEGRAL OF(f1(x) f2(t-x) dx) from 0 to t

(4) max{f1(t), f2(t)}

Answered by Andrei Volodin.
A multiple choice exam 2007-10-21
From jon:
a student is taking a multiple choice exam in which each question has four choices. assuming that she has no knowledge of the correct answers to any of the questions, she has decided on a strategy in which she will place four balls (marked A, B, C, D) into a box. she randomly selects one ball for each question and replaced the ball in the box. the marking will determine her answer to the question. there are five multiple choice questions on the exam. what's the probability that she will ...
Answered by Stephen La Rocque and Harley Weston.
A line parallel to a given line 2007-10-20
From Samaira:
Given Y=2/3 + 3/4x
Give the equation that is parallel to the given one and which touches the x-axis in x=3

Answered by Leeanne Boehm and Stephen La Rocque.
The volume of a regular octagonal prism 2007-10-20
From Chris:
I am trying to figure out the volume of a regular octagonal prism, with side lengths of 4 inches. How do I do this?
Answered by Penny Nom.
Stanley Cup costume (truncated cone pattern) 2007-10-19
From Janet:
You hockey fans will love this question. I am making a Stanley Cup halloween costume and need a flat pattern for the bowl portion. I believe a truncated cone will work nicely. Base circumference needs to be 32" (10.19 diameter). Top circumference needs to be 44" (14 diameter). It needs to be 8" high. Thanks for your help.
Answered by Stephen La Rocque.
An octagonal table 2007-10-18
From Lorne:
I am 77 years old and want to build a table top with each side measuring 23 inches. I believe the diameter would be 55.5 inches. Is this correct and what is the angle of the cuts I have to make? Thank you for your patience.
Answered by Penny Nom.
A rectangular trough 2007-10-18
From David:
A rectangular trough is 2 meter long, 0.5 meter across the top and 1 meter deep. At what rate must water be poured into the trough such that the depth of the water is increasing at 1m/min. when the depth of the water is 0.7m. I know this involves implicit differentiation somehow, but the 3 variables, since V=l*w*h for a rectangle is confusing me. I'm not sure whether one of the variables should be fixed or not, since I'm not getting anywhere with this right now. Any help would be great.
Answered by Stephen La Rocque and Penny Nom.
4x+8y=32 2007-10-18
From Sara:
Rewrite the following equation:
4x+8y=32
in function form ("solve for y").

Answered by Penny Nom.
A conical cup 2007-10-18
From Nicholas:
Water is leaking out of a small hole at the tip of a conical paper cup at the rate of 1cm^3/min. The cup has height 8cm and radius 6cm, and is initially full up to the top. Find the rate of change of the height of water in the cup when the cup just begins to leak. Since V= (pi/3)r^2h, how do I eliminate a variable or change the equation so I that I can answer the question? Thanks.
Answered by Penny Nom.
0 = (1/t) - [ln(1+r) ln(t)] 2007-10-18
From Brandi:
Objective: solve for t, where r = 0.05

0 = (1/t) - [ln(1+r) ln(t)]

Answered by Penny Nom.
A triangular lot 2007-10-17
From Brian:
Seeking the square footage of a triangular property lot, dimensions are: 620 X 620X 720.
Answered by Penny Nom and Melanie Tyrer.
Paying with silver - Part 2 2007-10-17
From Shanna:
The paying with silver problem. I understand how to do the problem, but could you please explain how you would use base 2 arithmetic to solve it.
Answered by Penny Nom.
Calculate the dimensions of the rectangle 2007-10-16
From Sara:
The perimeter of a rectangle is 62cm. The length of the diagonal is 25 cm. Calculate the dimensions of the rectangle.
Answered by Stephen La Rocque.
Surface area of an open-ended cone 2007-10-16
From Lorne:
What is the surface area of an open ended cone? Measured at 10' high, 16' diameter on the bottom and 2' diameter at the top.
Answered by Stephen La Rocque.
5x + 9 = 3x + 7 2007-10-16
From kristina:
hi im kristina and i cannot figure out how 2 do these multi-step equations

5x + 9 = 3x + 7 i can on getting the wrong answer according 2 the book

Answered by Penny Nom.
How many cubic yards of dirt do I need? 2007-10-16
From BELINDA:
I need to fill an area of 30,000 square feet with dirt that is 18" in height. How many cubic yards of dirt do I need to do this?
Answered by Penny Nom.
lim x->1 (root x - x^2)/{1 - root x) 2007-10-16
From Meghan:
Hi! I have a question from my Calculus textbook that I've been picking at for a while and I'm stuck.

lim x->1 (root x - x^2)/{1 - root x).

Answered by Stephen La Rocque and Penny Nom.
Related rates 2007-10-15
From Alexis:
Example 1. An observer is tracking a small plane flying at an altitude of 5000 ft. The plane flies directly over the observer on a horizontal path at the fixed rate of 1000 ft/min. Find the rate of change of the distance from the plane to the observer when the plane has flown 12,000 feet after passing directly over the observer.
Answered by Stephen La Rocque.
How many tons of asphalt are in 12 cubic yards? 2007-10-15
From Matthew:
I want to know how many tons of asphalt are in 12 cubic yards?
Answered by Penny Nom.
How do you find the radius of a circle if you only know its area 2007-10-15
From s:
how do you find the radius of a circle if you only know the area of the circle. Do you somehow reverse the Pi formula.
Answered by Penny Nom.
2(y minus 3) = 2007-10-15
From Alissa:
i have more then one question: 1. 2(y minus 3) = 2. 3(4 plus x) = 3. negative 6(2a plus 3b) = 4. negative 2(5x plus 2y) = 5. negative 3(x minus y) = 6. negative 3(3x minus 4w) =
Answered by Penny Nom.
4 odd numbers that equal 21 2007-10-15
From gary:
4 odd numbers that equal 21
Answered by Penny Nom.
Four triangles in a square 2007-10-15
From Kristina:
A square with side lengths of 6 cm is divided into 3 right triangles and a larger isosceles triangle. If the three right triangles have equal area, find the exact area of the isosceles triangle.
Answered by Stephen La Rocque.
lim (1- 2 cosx) / (sin(x- pi/3)) 2007-10-15
From hanan:
lim (1- 2 cosx) / (sin(x- л/3))
Answered by Harley Weston.
The dimensions of a rectangle 2007-10-14
From Elizabeth:
The perimeter of a rectangle is 20m. The area is 20sqm. Calculate the dimensions of the rectangle. Give your answers correct to two decimal places.
Answered by Penny Nom.
Nickels, dimes and pennies 2007-10-14
From tom:
you have $1.00 and 1/5 of it is dimes, 2/15 is nickels, and 2/3 is pennies. How many coins of each are there. how do i set it up to solve this problem. thanks, tom
Answered by Penny Nom.
How much can Daisy spend? 2007-10-14
From Math:
Daisy has $25.00 to spend on dinner. Her sister tells her to leave 18% of the bill as a tip and the sales tax is 8% of the bill. What is the maximum amount she can spend on dinner in order to cover the meal, sales tax, and the tip?
Answered by Penny Nom.
Working with x 2007-10-12
From Robert:
The question: The measure of an exterior angle of a regular polygon is given. Find the measure of an interior angle, and find the sides.
41. 36
42. 18
44. 'X'

The attachment has what she has done for 41 and 42. Need help with 44 (lots of help) Thank you in advance for your site and your help. Robert

Answered by Penny Nom.
How much water is in a hose? 2007-10-12
From Bryan:
I need to know how much water (gal) is in a hose thats 100' long by 5" in dia. Thanks
Answered by Stephen La Rocque.
Domain and range 2007-10-12
From Dawn:
Hi, I've been out of school for 8 years and recently picked up a math correspondence course. I'm having trouble trying to figure out the range and domain of a linear function. I've read everything I can find in my text, lessons and on your site and I still can't figure out what I'm suppose to be doing. The function is y=2x+1. Please help.
Answered by Penny Nom.
13 year and 17 year locusts 2007-10-12
From stefan:
how many years pass between the years when both 13 year and 17 year locusts are out at the same time?
Answered by Penny Nom.
A trig limit 2007-10-12
From Amanda:
What is the limit, as x tends to zero, of: (1-cos(4x))/(xsin(x))? Thank you! ~Amanda
Answered by Harley Weston.
A stem leaf and plot with decimals 2007-10-12
From Brandi:
My child came home with a worksheet regarding stem and leaf plot. I figured out how to work it with whole number but not with decimals. Can you tell me how to do this with stem and leaf graph? They are even asking for a conclusion on the information. This makes no sense to me. Please explain and send an example problem also. Thank you!
Answered by Harley Weston.
What is a friendly Factor? 2007-10-12
From Christine:
Hi mi niece and I are trying to figure out what friendly factors are. An example of one of the questions: Use friendly factors to multiply the following. 8x25=. Can you please help? thank you
Answered by Penny Nom.
A dinner club schedule 2007-10-12
From irv:
The problem is that I have 12 couples, 1 through 12, in a dinner club, and each couple is host once with 3 of the other couples. he dinners will take place over 4 months. I am trying to not repeat any of the couples. We had the formula for 16 and 20 couples but not for 12. Are you able to help with the groupings for 12. Thanks very much. Irv
Answered by Victoria West.
A 9 team league 2007-10-11
From Dary:
I have a 9 team pool league. Each team must play each other once only. But 2 teams cannot play at the same place location each week. I can't seem to match this up and I get stuck.
Answered by Victoria West.
How many students attended the get-together? 2007-10-11
From Diane:
Some students planned for a get-together. The budget for food was $500. Five of the students failed to come because of the distance and therefore the cost of food for each member increased by $5. How many students attended the get-together?
Answered by Stephen La Rocque.
The angles in a polygon 2007-10-11
From Farzan:
Prove with induction that in a polygon( that may not be convex ) with n sides, the sum of the amounts of the angles become 180(n-2). If there is any easier methods to prove the problem, please write as well.
Answered by Stephen La Rocque.
The average rate of change of a function 2007-10-11
From vern:
Find the average rate of change of the function over the given interval. Compare this average rate of change with the instantaneous rates of change at the endpoints of the interval. f(X)=sinX for the inverval [0,pi/6]?
Answered by Harley Weston.
Total payments for a loan 2007-10-11
From Robin:
A person is going to arrange a loan at 3 mill. he will pay 150000 back each year , so it take 20 yrs. to finish. In addition he pays 5% interest of what is leaft each year. How do I find the sum-function which shows the total sum he have payed?
Answered by Stephen La Rocque.
Substitution method 2007-10-11
From Kevin:
3xx+2y=-36-y=11
Answered by Stephen La Rocque.
Area of a quadrilateral 2007-10-10
From Courtney:
how would i find the area of a quadrilateral..
the sides are a (/) is 6cm, b (—) is 9 cm, and c (\) is 7 cm..
the angle between a and b is 140 degrees and b and c is 115 degrees..

Answered by Stephen La Rocque.
Given the arc length and chord length, what is the radius? 2007-10-10
From Wayne:
I have the actual length of an arc plus the length of the cord. How do I determine the radius of the arc.
Answered by Harley Weston.
Parabolic suspension bridge 2007-10-09
From Jessica:
A suspesion bridge with weight uniformly distributed along its length has twin towers that extend 75 meters abouve the road surfce and are 400 meters apart.The cables are parabolic in shape and are suspended from the tops of the towers. The cables touch the road surface at the center of the bridge. Find the height of the cables at a point 100 meters from the center. (Assume that the road is level.)
Answered by Stephen La Rocque.
Simplifying algebraic expressions 2007-10-09
From Sakeena:
(2^2*3)^x+1/2^2x*3x
Answered by Stephen La Rocque.
Parabolic arch 2007-10-09
From Nisa:
A parabolic arch has a span of 120 feet and a maximum height of 25 feet. Choose suitable rectangular coordinate axes and find the equation of the parabola. Then calculate the height of the arch at points 10 feet,20feet,and 40 feet from the center.
Answered by Stephen La Rocque.
Simplifying rational expressions 2007-10-09
From Bama:
x(x-2)+1 divided by x^2-3x+2
Answered by Stephen La Rocque.
Subtracting rational expressions 2007-10-09
From Pamela:
Subtract:
6p – w     __     4c + p
 w – 4             w – 4

Answered by Stephen La Rocque.
Mixing cement 2007-10-09
From Sandra:
How many bags of cement do I need for a floor for a house that is 8meters long, 8meters wide and 50cm deep.
Answered by Stephen La Rocque.
calculating percentages for my class 2007-10-09
From Cori:
I teach a class in which there are seven projects/quizzes worth 10% each, and the midterm and the final are each worth 15%. No problem figuring out their final grade, but I have to give a mid-term average, having only FOUR of the seven 10% things, and the midterm test. The closest I've come to an accurate "average" is to just add them together (as equals) and average them.
Answered by Victoria West & Harley Weston.
Area and side length of a hexagonal tent 2007-10-09
From Mark:
Trying to buy a tent with 6 sides, in the catalog they give me sq. ft., what is the distance from side to side of a hexagonal tent with an area of 90 sq. ft.
Answered by Stephen La Rocque.
Coin jar 2007-10-07
From a student:
Sally empties his jar of coins. It contains $3.75 in nickels, dimes, and quarters. The number of dimes is twice the number of nickels and the number of quarters is three less than the number of nickels. Determine how many nickels, dimes, and quarters were in the in the jar.
Answered by Stephen La Rocque.
Doubling every day 2007-10-07
From Jerrie:
Patti saved pennies for 10 days. The first day she saved 1 cent. Every day after that she saved twice as much as the day before. How much money did Patti save altogether?
Answered by Stephen La Rocque.
Solving four simultaneous equations (system of four linear equations) 2007-10-07
From Johan:
I need some help in solving this question
x + 2y - 3z + 4w = 12
2x + 2y - 2z + 3w = 10
0 + y + z + 0 = -1
x - y + z - 2w = -4

Answered by Stephen La Rocque.
Problems solved using quadratic equations 2007-10-07
From Lisa:
a. Two positive numbers differ by 5. Their product is 234. What are the two numbers? b. The sum of two numbers is 12. Their product is 30. What are the two numbers? c.Two numbers differ by 5. Their squares differ by 55. What are the two numbers? d. The sum of two numbers is 15. Their product is 56. By forming an equation, find the two numbers.
Answered by Penny Nom.
Roof trusses 2007-10-05
From Mary:
I need to know how algebra is used in making or designing of roof trusses for a house. I have to present a poster in class showing how algebra is used in daily jobs. My dad works at a truss company.
Answered by Chris Langdon.
Number of real solutions 2007-10-05
From Sujith:
The no of real solutions of the equations 1+x+x^2+x^3 = x^4+x^5 is ?
Answered by Harley Weston.
Solve for K 2007-10-05
From Ben:
Solve the formula for K C=30d+0.10k
Answered by Stephen La Rocque.
Triangle perimeter and area 2007-10-05
From divakar:
The sides of the triangle are integers. The perimeter is 8. What is the area?
Answered by Stephen La Rocque.
Find the equation of a line given two points 2007-10-05
From Tiffany:
The equation for the line through the points (11,2) and (18,12) can be written in the form Ax+By=C. Find A and B.
Answered by Victoria West.
Maximizing profits II 2007-10-05
From a student:
Suppose there are three firms with the same demand function. The function is Q=1000-40P. Each firm also a a cost function.
Firm 1: 4000+5Q, Firm 2: 3000+5Q, Firm 3: 3000+7Q.
What price should each firm charge if it wants to maximize profits.

Answered by Harley Weston.
Show that there is no five-digit number which 2007-10-04
From Greg:
show that there is no five-digit number which uses each of the digits 1,2,3,4,5 such that the numbers formed by the first digit is divisible by 1, by the first two digits is divisible by 2 by the first three digits is divisible by 3, by the first four digits is divisible by 4, by the first five digits is divisible by 5
Answered by Stephen La Rocque, Claude Tradif and Victoria West.
Equation of a tangent line to a function 2007-10-04
From Princess:
If f ' (x) = 3x^2 +1, find the equation of the tangent line to f(x) = x^3 + x at x= -1.
Answered by Stephen La Rocque.
Finding radius given chord length and distance to center 2007-10-04
From Venus:
a chord of 48mm long is 7mm from the center of the circle. What is the radius of the circle?
Answered by Stephen La Rocque.
Arc lengths, central angles and radii 2007-10-04
From Ashutosh:
Jose can remember that the length of an arc is 440cm, but he cannot remember the radius of the arc or the angle at the center. He does know that the angle was a whole number of degrees and the radius was less than 100cm. Find three possible angles and write down the size of each of the possible radii.
Answered by Stephen La Rocque.
Finding the last non-zero digits of large factorials 2007-10-04
From Mukesh:
i have to find last five non zero digits of integer which can be very large ( upto 10^12) . i can find last non zero digit of of any factorial. Now my problem is that i have to find last five non zero digit of factorial and also i want to general method for last K non zero digits of factorial n. For example 10!=3628800 so last non zero digit is 8 ,last two non zero digit is 88 .....and last five non zero digit is 36288.
Answered by Victoria West.
Rolling a die repeatedly (you can't just add percentages) 2007-10-04
From Howard:
If my son has a 16.66% chance of rolling a particular number on a dice, and he rolls for it 6 times, he knows (from experience) that his odds of getting at least 1 right are not 100% (16.66% +16.66%+16.66% etc). He asked me if there was an equation that would give him the correct percentage chance of success when trying X times for a Y% chance of something.
Answered by Victoria West.
Gas and oil at a 50:1 ratio 2007-10-03
From Cindy:
How much gas and oil do you put in a 1.7 litre jug at a 50:1 ratio?
Answered by Stephen La Rocque.
What is the total surface area of a cylinder? 2007-10-03
From MATT:
what is the total surface area of a cylinder with outside diameter 40 cm. thickness 10cm. and length 2m. Work in cm^2
Answered by Stephen La Rocque.
A sheet of corrugated iron 2007-10-03
From Ashutosh:
A sheet of corrugated iron has corrugations based on the arc of a circle of radius of 3 cm and a central angle of 180 degrees. How many corrugations are formed when a flat sheet of steel 1 metre wide is bent to form a sheet of corrugated iron?
Answered by Stephen La Rocque.
Equivalent fractions 2007-10-03
From kiki:
I want to know how to this equivalent decimal for 6.0250.
Answered by Stephen la Rocque and Penny Nom.
Golf pairings 2007-10-02
From Mike:
Regarding arranging golf players so no person plays with anyone more than once. You have given examples for 16 and 24 players. If it can be done, i need a solution for 20 players, 4 players per team one round per day for 5 days
Answered by Victoria West.
The equation of a parabola 2007-10-02
From srujana:
Can we find the equation of the parabola when only two points on it are known and neither of them is the focus nor the vertex?
Answered by Harley Weston.
Find the area of the traingle 2007-10-02
From Indrajit:
The perimeter of a triangle is equal to the perimeter of a semicircle of radius 35 cm.The ratio of the triangles is 5:6:7. Find the area of the triangle? [pie = 22/7]
Answered by Stephen La Rocque.
The amount of dirt 5 feet deep over a 50 acres 2007-10-02
From Debbie:
Please tell me how to calculate the amount of dirt stripped 5 feet deep over a 50 acre site. How do you calculate the amount of dirt in an acre and then excavating it a certain depth. How does one calculate the amount of dirt excavated and moved and how to price to charge for the service?
Answered by Stephen La Rocque and Victoria West.
Equivalent mixed numbers 2007-10-02
From negra:
an average-sized person can burn about 6 1/2 calories a minute while ridinng a bike. Which of the following is equivalent to that amount?
a) 1 2/2

b) 5 6/2

c) 6 2/4

d)6 2/6

Answered by Stephen La Rocque and Penny Nom.
Motion along a line 2007-10-02
From Claudette:
If the position function of a particle is x(t) = sin^2(2t), find the distance that the particle travels from t = 0 to t = 2
Answered by Harley Weston.
How many ten thousands makes one million? 2007-10-02
From Payton:
how many ten thousands makes one million?
Answered by Penny Nom.
Irrational functions 2007-10-01
From alicia:
i have a question about irrationals functions.
i have been using them quite some time now, but i wonder where they can be found in daily life?
i hope you can help me,

Answered by Harley Weston.
Equation of a circle circumscribing a triangle with given vertices 2007-10-01
From Randy:
How do I determine the equation of a circle when it is circumscribed by a triangle whose vertices are (-1, 6), (3, -2), and (2, 5)?
Answered by Stephen La Rocque.
3 times x to the power -2 2007-10-01
From Jennifer:
The problem is 3 times x to the power -2.

I was just supposed to simplify, and I have no idea how.

Answered by Penny Nom.
The normal approximation to the binomial 2007-09-30
From m.j.:
Slot Machines The probability of winning on a slot machine is 5%. If a person plays the machine 500 times, find the probability of winning 30 times. Use the normal approximation to the binomial distribution.
Answered by Harley Weston.
y squared over 3 times 8 over y 2007-09-30
From John:
y squared over 3 times 8 over y
Answered by Stephen La Rocque.
Three circles 2007-09-29
From Kevin:
3 given circles of R80, R56 & R24 are all in contact. The 2 smaller ones are inscribed in the big one. Find by calculation or graphically (both if possible) the radiusof a 3rd circle which will be in contact with all 3 given circles.
Answered by Chris Fisher.
Oil in a pipe 2007-09-29
From Kristi:
How much oil is in a 40 foot section of pipe that its inside diameter is 36 inches?
Answered by Victoria West.
3x2/3 = 12/3 2007-09-28
From Shafan:
3x^2=12
Answered by Penny Nom.
How many barrels of liquid will it hold? 2007-09-28
From Jason:
A tank is 8' wide by 8' tall x 45' long. How many barrels of liquid will it hold??
Answered by Stephen La Rocque and Harley Weston.
I have a 6 digits 2007-09-28
From Kim:
I have a 6 digits. My hundred-thousands digit is 1 less that my ones digit, 8 more than my thousands digit, twice my tens digit, and 4 times my hundreds digit. My ten-thousands digit is 0. What number am I?
Answered by Stephen La Rocque.
The country with the most universities 2007-09-28
From j:
the country with the most universities is india followed by the united states. If india has 2649 more universities than united state and there combine total is 14,165, find the number of universities in india and the number in the united states.
Answered by Stephen La Rocque.
Size of a sphere fitting inside a cone 2007-09-27
From Juan:
I am supposed to find the largest sphere that will fit into a cone. I am assuming is a maximizing problem, but I am not sure of what relation (between a cone and a sphere) to use.
Answered by Penny Nom and Stephen La Rocque.
A normal distribution problem 2007-09-27
From m.j.:
Car Loan Rates The national average for a new car loan was 8.28%. If the rate is normally distributed with a standard deviation of 3.5%, find these probabilities. a. One can receive a rate less than 9%. b. One can receive a rate less than 8%.
Answered by Harley Weston.
400 square yards of sidewalk 2007-09-27
From Vanessa:
I am having a small problem being a subcontractor I'll having trouble calculating how much concrete it will take to pour a 400 square yard sidewalk 5' wide 4" depth can you show me the equation to figure out how many yards of concrete i will need. Can you help me please? Thank you very much and have a wonderful day.
Answered by Penny Nom.
How can that be? 2007-09-26
From Ken:
Al and I have 10 computers on a shelf 10 Team Members came passing by Each took a computer off the shelf, leaving 9 computers still on the shelf How can that be?
Answered by Stephen La Rocque.
x= (y/z)ln((1+a^2)/(1-a^2)) 2007-09-26
From John:
If x= (y/z)ln((1+a^2)/(1-a^2)) Please solve for 'a' Thanks in advance!
Answered by Penny Nom.
Exponents 2007-09-26
From Calvin:
Exponential form 5 3exponent x 5 4exponent divide 5 2exponent
Answered by Stephen La Rocque.
A tangent line to a circle 2007-09-26
From Randy:
Find the equation of the tangent line at coordinates (-1 , 4) on the circle x^2 + y^2 - 4x - 21 = 0

I would like to learn the fastest way to relate any coordinates of a circle to any possible point of tangency.

Answered by Stephen La Rocque.
T-totals 2007-09-26
From Asma:
I wanted to know what i should include in my T-totals maths coursework and also how i should lay out the coursework?
Answered by Penny Nom.
The length of an arc 2007-09-26
From swarna:
a wire of length 32 cm is bend to form a sector of circle of radius of 6 cm find the length of the arc of the sector
Answered by Penny Nom.
Can you tell me how many gallons it holds? 2007-09-25
From Jeff:
Hi my water tank is 10ft long and 45" around,can you tell me how many gallons it holds.thankyou jeff
Answered by Penny Nom.
A geodesic dome 2007-09-25
From Roger:
In my Science-Fiction series, I have a Dyson's Sphere tiled with regular hexagons. The number of hexagons is over 300,000 and the radius of the Sphere is roughly 80,000,000 miles. The actual size of the Sphere and hexagons have been left flexible until I can come up with a definite number of hexagons that would fit. My problem is the pattern of hexagons which would fit within the sphere without leaving gaps or overlapping.

My best guess has been to use four equilateral triangles composed of 78606 hexagons, (396 per edge) arranged around the sphere with six 'zippers' to connect them and four 'caps' at the points, for a total of 316804 hexagons. Given the fact that each Hex is the same size, does this seem plausible? Is there some pattern formula I can use to play with these figures? Simple divsion of areas will not work if the number derived will not fit into the pattern to leave a perfectly tiled surface. Thank you.

Answered by Chris Fisher.
1/2+X=5/8 2007-09-24
From Tasha:
What is the sum of X in the problem below and explain your answer please?

1/2+X=5/8

Answered by Chris Langdon.
A piece of wire is bent in the form of a circle 2007-09-24
From Renece:
a piece of wire is bent in the form of a circle and it encloses an area of 154cm a) calculate the radius of the circle b) the circumference of the circle use 22/7 The same piece of wire is then bent into a square d) calculate the area enclosed by the square.
Answered by Penny Nom.
Exponential regression 2007-09-24
From Anonymous:
I need help in finding this. Find a formula for exponential regression. Determine how to calculate a and b in the formula. Y = ba^x where y is as a function of x.
Answered by Penny Nom.
A convex quadrilateral 2007-09-24
From alex:
if a convex quadrilateral ABCD is set up so that angles b and c are fixed (for example angle b= 73 and angle c =150), and sides AB and CD are congruent; can angles a and b vary, or are they also fixed
Answered by Brent Michelson.
The weights of packages are normally distributed 2007-09-23
From alan:
the weight of a packet of sweets produced in a factory are normally distributed.
the mean weight is 100g
the standard deviation is 2g
all packets weighing less than 99g and more then 105g are rejected
what proportion are rejected

Answered by Penny Nom.
The diagonal of a square 2007-09-23
From Nasreen:
A(7,2) and C(1,4) are two vertices of a square ABCD find in the form ax+by=c the equation of the diagonal BD
Answered by Stephen La Rocque.
Sample variance and population variance 2007-09-22
From Willy:
I have a question in sample variance and population
my name is Willy
the question in the attached file...

Answered by Harley Weston.
Finding equations, intersection point of two lines at right angles 2007-09-22
From Yaz:
Find the equation of the line joining A(-1,-9) to B(6,120). Another line passes through C(7,-5) and meets AB at rigth angle of D. Find the euation of CD and calculate the co-ordinates of D.
Answered by Stephen La Rocque.
The incline of my yard as it slopes away from my house 2007-09-22
From Janet:
I need to calculate the degree (or is it percent?) of incline of my yard as it slopes away from my house. Do I visualize a right angle formed by a level line from the bottom of the slope to the side of the house, and then compare the line of the slope against the right angle to get the degree? Or is an incline calculated by the measurement of the rise in 100 feet? For instance, if the ground rises 3 feel in a distance of 100 feet, is that a 3% incline? And if it rises 4 feet in a distance of 25 feet, is that a 16% incline? Thanks so much.
Answered by Stephen La Rocque.
Two equations in two unknowns 2007-09-22
From Mary:
Having problems doing this problem, looking for a solution with the work. I would like to see how you got your answer, to see what I was doing wrong.

solve using the substitution method, is there "no solution" or "infinitely many solutions"

4x+y=4
2x+8y=0

Answered by Stephen La Rocque and Leeanne Boehm.
How much water to fill my bedroom? 2007-09-22
From hector:
how can i find out how much water i can fill my bedroom with when it measures 12 ft.x12 ft.x10 ft.
Answered by Steve La Rocque and Penny Nom.
The derivative of f(x)=1/(x-1) 2007-09-21
From Michelle:
im having trouble finding the derivative of f(x)=1/(x-1) using the f(x+h)-f(x)/h method.
Answered by Stephen La Rocque.
Working together 2007-09-21
From farah:
A can do a piece of work in 10 days while B can do the same work in 15 days . They work together for 5 days and the rest of the work is done by C in 2 days . If they get 400$ for the whole work , what should be A's share.
Answered by Harley Weston.
Water flowing into a tank 2007-09-21
From andrew:
Hi, I've been having real trouble visualizing this problem as apposed to a conical tank. It says the base of a pyramid-shaped tank is a square with sides of length 12 feet. The vertex of the pyramid is 10 feet above the base. The tank is filled to a depth of 4 feet, water is flowing into the tank at the rate of 2 cubic feet per minute. Find the rate of change of the depth of water in the tank.
Answered by Harley Weston.
Metres per minute to miles per hour 2007-09-20
From Angela:
If a person is traveling 150 meters per minute, what is their speed in miles per hour?
Answered by Stephen La Rocque and Harley Weston.
Points that are 15 units from the origin 2007-09-20
From Paula:
Find the coordinates of any point(s) 15 units away from the origin with an x-coordinate of 9.

I was given the answer: (9,12) and (9,-12), but I do not understand how these numbers were calculated. Thank you for your help!

Answered by Stephen La Rocque.
(5C squared + 14C + 11) divided by (5C + 4) 2007-09-20
From John:
(5C squared + 14C + 11) divided by (5C + 4)
Answered by Harley Weston.
Area of an irregular lot 2007-09-20
From Joanne:
I found your web site through ask.com & it's great. Really hoping you can help me. We are looking at purchasing an existing home with a lot size that is advertised at 2.04 acres, however, even with my limited math skills I know this is not correct.

Their calculation takes the deepest part of the lost (179 feet) multiplied by the widest part of the lost (497.26 feet) as if it was a perfect rectangle. As you can see from the faxed diagram it is far from rectangular in shape.

Can you advise as to the true lot size? As I mentioned we are looking at this property as a purchase so time is of the essence. I have also faxed this information (with the diagram) to the number listed on your web site. Thank you so much for your help.

Answered by Harley Weston.
Least Common Multiples 2007-09-20
From Aiyanna:
What is The Lcm Of 3,7,and 8 Because my Teacher gave me That and he didnt even know the answer.... I Worked and worked but I Couldnt Find It.....
Answered by Penny Nom and Victoria West.
What is the inverse function of y=x^3 +x? 2007-09-19
From saman:
what is the inverse function of y=x^3 +x
Answered by Harley Weston.
Filling a planting bed 2007-09-18
From Luke:
How many 40lbs bags will it take to fill area 21feet long 45inchs wide 9 inch deep?
Answered by Penny Nom.
The perimeter of a regular polygon 2007-09-18
From Ashwynn:
why does the area of regular polygons with a perimeter of 1000m increase as the number of sides increase?
Answered by Stephen La Rocque.
The range of a projectile 2007-09-18
From Claudette:
This is a maximum minimum problem that my textbook didn't even try to give an example of how to do it in the text itself. It just suddenly appears in the exercises. Problem: The range of a projectile is R = v^2 Sin 2x/g, where v is its initial velocity, g is the acceleration due to gravity and is a constant, and x is the firing angle. Find the angle that maximizes the projectile's range. The author gives no information other than the formula. I thought to find the derivative of the formula setting that to zero, but once I had done that, I still had nothing that addressed the author's question. Any help would be sincerely appreciated. Claudette
Answered by Stephen La Rocque.
Find my number using the following clues: 2007-09-18
From T:
Find my number using the following clues: my number will make an array 2 rows wide my number is greater than 5 my number is less than 30 my number will make a square array my number is________? I was not sure where to begin with this word problem - can you help?? Thank you, T
Answered by Penny Nom.
How do I determine how many tons, 15 yards of top soil is? 2007-09-18
From Terry:
One company is selling topsoil by the yard and another by the ton, how do I determine how many tons, 15 yards of top soil is?
Answered by Penny Nom.
Prove that any two consecutive integers are relativley prime. 2007-09-18
From Michael:
Im not very good at proofs and I was wandering if you would be able to help me with the following question: Prove that any two consecutive integers is relativley prime. Thanks a million.
Answered by Penny Nom.
A word problem 2007-09-17
From Martin:
One more than the difference between 18 and seven times a number is -9
Answered by Stephen La Rocque.
A fraction that cannot be simplified 2007-09-17
From Kevin:
Make a fraction that cannot be simplified and has 24 as its denominator?
Answered by Stephen La Rocque and Chris Langdon.
Two cars 2007-09-17
From Lanie:
Two cars enter the Florida Turnpike at Commercial Boulevard at 8:00am, each heading for Wildwood. One car's average speed is 10 miles per hour more than the other's. The faster car arrives at Wildwood at 11:00 am, 1/2 hour before the other car. What is the average speed of each car? How far did each travel?
Answered by Stephen La Rocque.
Taping the gym floor 2007-09-17
From Marina:
You are helping the coach tape lines on the gym floor to divide the length of the gym into 8 lanes of the same size. If it takes one minute to tape a line,how long will it take to do the job. State the strategy /strategies you used and why?
Answered by Penny Nom.
The complement of an angle 2007-09-16
From tom:
The measure of the complement is 12 more than twice the original angle. Find the measure of the angles.
Answered by Stephen La Rocque.
Finding the center of a circle that goes through three given points 2007-09-16
From Gary:
I am given three points represented by their Latitude and Longitude. How do I determine the Latitude and Longitude of the center of a circle through the three given points? The lat/lon of the three points are: A. N 43 degrees 30.251 min. W 96 degrees 45.695 min. B. N 43 degrees 30.006 min. W 96 degrees 45.082 min. C. N 43 degrees 30.719 min. W 96 degrees 45.410 min. Thanks.
Answered by Stephen La Rocque.
how do you simplify 3a + 13b = 37 2007-09-16
From brandon:
how do you simplify 3a + 13b = 37
Answered by Penny Nom.
Tens and ones 2007-09-16
From julia:
My son has come home with a math question "what i know about 10s and 1s" we're totally stumped any information you can provide would be greatly appreciated
Answered by Harley Weston.
Volume of a cylinder 2007-09-15
From mike:
I need to know the formula to measure conceret celinders
Answered by Penny Nom.
Volume, mass and density 2007-09-14
From Nathan:
I have a sample of material that is 10cmx10cmx5cm and has a mass of 29 grams. How many grams would a sample of the same material that was 96cmx188cmx5cm be?
Answered by Penny Nom.
How much stone will be needed to fill the trench? 2007-09-14
From Debbie:
A trench 100metres long 1ft wide*18inches deep with a 4inch pipe laid. How much stone will be needed to fill the trench.
How much weight in stone would I need to fill it?

Answered by Penny Nom and Victoria West.
Differentiate x^(1/3) using first principles 2007-09-14
From Sheila:
our teacher gave us this question as a challenge and even he couldnt figure it out: Differentiate x^(1/3) [aka the cube root of x] using first principles. i know the answer is 1/(3.x^2/3), but how is it possible using first principles?
Answered by Harley Weston.
Is it platinum or silver? 2007-09-14
From Jordan:
In Chemistry we are doing a lot of problems that are much more math related than anything. One of my questions reads: A certain medallion has a mass of 55.64 g. When placed in 75.2 mL of water in a graduated cylinder, the water level rises and then reads 77.8 mL. Is the medallion made of platinum (D=21.4 g/mL) or silver (D= 10.5 g/mL)?
Answered by Penny Nom.
The contents of a silo 2007-09-14
From Steven:
I need to know how to find the weight of the contents of a silo that is 4 feet across and is funnel shaped at 60 degrees
Answered by Penny Nom.
The area of half a circle 2007-09-14
From Heather:
We need the formula for area of half circle please explain where each number comes from.
Answered by Penny Nom.
An equilateral triangle 2007-09-13
From Bibi:
An equilateral triangle has a point P inside it. PA, PB, PC are the perpendiculars from point P to the three sides of the triangle. Help me show that PA+PB+PC is the same no matter where the point P is..?
Answered by Harley Weston.
Cutting a cake 2007-09-13
From Bibi:
how can one cut 8 pieces from a round cake with 3 straight slices of a knife, without moving any of the pieces? Help me explain this..
Answered by Stephen La Rocque.
Composite angles 2007-09-13
From Gilbert:
what is a composite angle? please help
Answered by Stephen La Rocque.
An irregular lot 2007-09-12
From Jay:
How do I calculate the area of the following Lot?
N 15 degrees 1 minute 22 seconds E for 182.2 feet
S 89 degrees 46 minutes 26 seconds E for 218.4 feet
N 51 degrees 16 minutes 21 seconds E for for 307.1 feet
L= 31.63 feet with a radius of 50 feet

There is a discrepancy between county records and what the landowner is telling me

Answered by Stephen la rocque and Harley Weston.
1-4/square root 3 2007-09-12
From Prudence:
How to solve this question?
1 - 4/√3

Answered by Penny Nom.
How many gallons per minute? 2007-09-12
From Diane:
HI I have a natural water spring and I am trying to determine how many gallons per minute will flow in an 8" pipe? I know one gallon is 231 cubic inches and V=nr2h - so if i had one foot of 8" pipe it would hold 2.6 gallons but I'm looking for the flow rate of how many gallons per minute? Thanks for your help.
Answered by Stephen la Rocque.
A place value chart 2007-09-12
From glenwood:
draw a million chart then place the following numbers in the proper place. one hundred and nine million, seven hundred and twenty-three
Answered by Penny Nom.
Exponential form 2007-09-12
From Ericha:
How do I express 1,679,616 in exponential form as a power of 6 using a calculator?
Answered by Stepehe La Rocque and Penny Nom.
How many miles per hour was I driving 2007-09-11
From Gary:
How many miles per hour was I driving to go 37 miles in 13 minutes
Answered by Victoria West.
The circumference of a circle 2007-09-11
From annette:
circumference of a 3.75 ft circle I have not done this in years and forgot how.
Answered by Stephen la Rocque and Victoria West.
How much space we are talking about when they say 500 cf? 2007-09-11
From Rhonda:
I was informed while planning a move that the moving company will allow 500 cubic feet of moving space. Anything over that is $5.99 per cf. How do I find out exactly how much space we are talking about when they say 500 cf ?? Thank-you in advance for the advice.
Answered by Stephen la Rocque.
The perimeter of a semi-circle 2007-09-11
From Confused:
What is the formula for the perimeter of a semi-circle? Can you please help me out!!! I need this answer by tomorrow.
Answered by Stephen la Rocque.
Find the area between (-4,0) (2,0) (2,6) 2007-09-11
From sabrina:
Find the area between (-4,0) (2,0) (2,6)
Answered by Penny Nom.
x = y^2 and x = -4 y^2 2007-09-11
From Jil:
My question is when dealing with parabolas, x=y^2, so that they are sifted on their side you could say, what will happen to the graph if you change it to x=-4y^2. I understand that the - flips the graph in the other direction but can you simply just plug in numbers and increase the stretch of the y?
Answered by Penny Nom.
What was the water level in the cylinder? 2007-09-11
From glen:
After a 5.63-g sample of Wood's metal has been added to the water in a 10-mL graduated cylinder, the new water level is 8.7mL. What was the water level in the cylinder before the sample was added?
Answered by Stephen la Rocque.
A rectangle inscribed in a circle 2007-09-11
From Sobeida:
A rectangle that is x feet wide is inscribed in a circle of radius 8 feet. What is the area of the rectangle as a function of x. Thanks!
Answered by Stephen la Rocque.
How many vertices are there? 2007-09-10
From Makayla:
I've just started the 4th grade and I am having trouble with a question on my homework. The questions is: Carrie is arranging sticks to make shapes. she has made a closed figure, with 10 sides How many vertices are there? I believe the answer is ten but I'm not sure.
Answered by Stephen La Rocque and Walter Whiteley.
Water in a conical tank 2007-09-10
From Greg:
Joe is conducting an experiment to study the rate of flow of water from a conical tank. The dimensions of the conical tank are:
Radius at the initial water level = 13.7 cm
Radius at the reference point = 12.8 cm
Initially the tank is full of water. There is a circular orifice at the bottom of the conical tank with a diameter of 0.635 cm. The water drains from the conical tank into an empty cylindrical tank lying on its side with a radius of 0.500 ft and a length L (ft).

Joe observed the water discharged with an average velocity of 1.50 m/s as the water level lowered from the initial height of 14.0 cm to 5.00 cm in the conical tank. Answer the following: 1. If the initial height of water in the conical tank is 14.0 cm (measured from the reference point, see Fig. 1), how long in seconds will it take for the water level to drain to a height of 5.00 cm?? NOTE: Height refers to the vertical height.

What formula would I use to find out how long in seconds it takes for the water level to drop?

Answered by Harley Weston.
Two inches of topsoil 2007-09-10
From Nick:
I have to cover a 5,300 square feet area with topsoil 2" deep. How many yards would I need?
Answered by Harley Weston.
The elevation of the sun 2007-09-10
From Elena:
The "angle of elevation" of an object about you is the angle between a horizontal line of sight between you and the object. (See figure) After the sun rises, its angle of elevation increases rapidly at first, then more slowly, reaching a maximum near noontime. Then the angle decreases until sunset. The next day the phenomenon repeats itself. Assume that when the sun is up, its angle of elevation (E) varies sinusoidally with the time of day. Let t be the number of hours that has elapsed since midnight last night. Assume that the amplitude of this sinsoid is 60 degrees, and the maximum angle of elevation occurs at 12:45 p.m.. Assume that at this time of year the sinusoidal axis is at E=-5 degrees. The period is, of course, 24 hours.

a. Sketch a graph of this function
b. What is the real-world significance of the t - intercepts?
c. What is the real world significance of the portion of the sinusoid, which is below the t-axis?
d. Predict the angle of elevation at 9:27 a.m., and at 2:30 p.m.
e. Predict the time of sunrise
f. As you know, the maximum angle of elevation increases and decreases with the change of the seasons. Also, the times of sunrise and sunset change with the seasons. What one change could you make to your mathematical model that would allow you to use it for predicting the angle of elevation of the sun at time on any day of the year.

Answered by Harley Weston.
Segments on a line 2007-09-10
From Cathy:
This question was on my daughter's geometry assignment. Write a general rule of formula for finding the number of segments that can be named by a given number of points on a line. For example, 2 points on a line = 1 line segment; 3 points on a line = 3 segments; 5 points on a line = 10 segments.
Answered by Penny Nom.
Divide a circle into 5 equal parts 2007-09-07
From Kathy:
How do you divide a circle into 5 equal parts using a protractor?
Answered by Penny Nom.
A parabola with vertex (-1,1) 2007-09-07
From Ronaldinho:
Hello. How do you find the equation of a parabola shown on a graph? i know the vertex is (-1,1) and that's it. Thanks!
Answered by Stephen La Rocque and Penny Nom.
Dividing a pizza 2007-09-07
From maryjane:
how do you divide a pizza into eight parts using only three cuts?
Answered by Chris Fisher and Stephen La Rocque.
The slope of a line 2007-09-06
From Danielle:
Hello. I need some help with this: i am supposed to find the equation of a line with a point (2/3,5) and (-5/6,-4) the answer in the book says y=6x+1. My question is how did they get for the slope?
Answered by Penny Nom.
Perpendicular lines 2007-09-06
From dinesh:
we know that for two perpendicular lines m1*m2=-1perpendicular but this is not true for x-axis and y-axis?
Answered by Harley Weston.
2x+1+x-4+4x+1 2007-09-06
From laurie:
2x+1+x-4+4x+1
Answered by Penny Nom.
0.24 1/2 2007-09-06
From Kenneth:
The fraction, 1/2, in 0.24 1/2 occupies the 1/100 place along with the 4.
If the fraction, 1/2, is changed to a decimal, as in 0.245, the last 5 in 0.245 occupies the 1/1000 place.
Why doesn't the 1/2 in the decimal 0.24 1/2 occupy the 1/1000 place (0.001) instead of the 1/100 place?

Answered by Stephen La Rocque.
How many two digit numbers contain at least one 7? 2007-09-06
From Janet:
How many two digit numbers contain at least one number seven?
Answered by Penny Nom.
13 = x/20 2007-09-06
From Melissa:
can u please help me with this question?
13 = x/20

Answered by Penny Nom.
Prime factors in exponential form 2007-09-06
From Michelle:
how would you write this: factor 48 as a product of primes written in exponential form
Answered by Stephen La Rocque.
Liquid in a pipe 2007-09-05
From christian:
how many cubic feet are in a pipe that is 3" (inside diameter) and 459 feet long?
Answered by Stephen la Rocque.
Exponential form 2007-09-05
From jeanette:
the African bush elephant is the largest animal and weighs 8 tons. write this amount in exponential form.
Answered by Stephen la Rocque and Penny Nom.
Compatible numbers 2007-09-05
From Beth:
Estimate using compatible numbers:

2 X 3978 =

102/25 =

Answered by Stephen la Rocque.
A right angles triangle 2007-09-05
From Daryl:
I have a right-angled triangle. two of its sides are 12.6cm. The third/last line is missing. How can I find the length of the last line?
Answered by Stephen La Rocque and Penny Nom.
The area of a circle knowing only the length of a chord 2007-09-05
From James:
I need some help in the right directions with a problem. I was presented with a problem where I need to find the area of a circle knowing only the length of a chord.

the is a circle in the center of a larger circle (which the size of either could change) the only thing that matter is that the chord is 100 ft long and rests on top of the smaller circle.

Answered by Stephen la Rocque and Brennan Yaremko.
Variable expressions - part 2 2007-09-04
From lisa:
how do I write a variable expression for word phrase example x less than = 2 or 2 less than p 4 fewer than job please help
Answered by Penny Nom.
Triangles, squares and pentagons? 2007-09-04
From Malea:
How do you find the nth term in a set of triangles, squares and pentagons?
Answered by Harley Weston.
Palindromes 2007-09-04
From ANJUM:
Are there palindromes in decimals and fractions?
Answered by Harley Weston.
The tangent to y = x^3 at x = 0 2007-09-04
From Amit:
consider the equation = x^3. The equation of tangent to this curve (which is smmetrical in Ist and IVth quadrant) at (0,0) is y=0, which is x-axis. but graphically one can visulize that x-axis intersects the curve, so how can it be the tangent to the curve. Please help.
Answered by Harley Weston.
How many stairs in the flight are stepped on exactly once? 2007-09-03
From Bill:
Four friends are racing together down a long flight of stairs. A goes 2 steps at a time, B 3 at a time, C 4 at atime and D 5 and at a time. The only steps on which all four tread are the top one and the bottom one. How many stairs in the flight are stepped on exactly once.
Answered by Harley Weston.
16x^2+4xy^2+8x divided by 4xy 2007-09-02
From Ricardo:
how can i solve that problem of polynomial division?

16x^2+4xy^2+8x divided by 4xy

Answered by Penny Nom.
The quotient of thirty and ten times a number. 2007-09-02
From Jenna:
I need help on How to translate a phrase into an algebraic expression: The quotient of thirty and ten times a number.
Answered by Penny Nom.
A rectangular sheet of paper is folded along the diagonal 2007-09-01
From Amit:
A rectangular sheet of a paper with dimensions a and b is folded along its diagonal. What is the area of the overlapped region?
Answered by Harley Weston.
Compatible numbers 2007-09-01
From andrea:
Estimate using compatible numbers. 249 /64
Answered by Harley Weston.
Standard deviation 2007-09-01
From Pat:
I have looked over your math question and mines is not there. for a binomiial distribution with parmeter n and p, the mean is np and the standard deviation is np(1-p). I understand how to get the np but not understanding how to get the (1-p). For example np=(40)(0.48)(1-0.48)=3.160. How can I get the (1-0.48) answer please
Answered by Harley Weston.
A word problem 2007-08-31
From Regian:
How can I write a word problem for the following numerical expression 3 (4+3)+2 And then how do I simplify the expression
Answered by Penny Nom.
How many cubic yards of concrete? 2007-08-31
From Teresa:
How many cubic yards of concrete are required for the structure with a bottom piece that is 8" high by 72" by 72", a side that is 8" by 72" by 60", another side that is 8" by 72" by 60 ", a side that is 8" by 72" by 60" with a circle cut out that has a radius of 18", and finally another side that is 8" by 72" by 60" that has a circle cut out that has a radius of 12".
Answered by Stephen La Rocque.
Is there a way I can calculate correct square footage for odd shapes? 2007-08-31
From Tom:
I manage a golf course, and we are about to over seed the course for the upcoming winter months. We will be over seeding all greens, fairways, and tee boxes. Noone of the area that we will be placing seed on is a regular shape. The closest shapes they would be is a circle, rectangle or square, but most of them are all odd CURVED shapes. I know what the equation is for Square ft., and I know how many Square ft. are in an acre, but I need to be precise with the amount of seed I order. Is there a way I can calculate correct square footage for odd shapes.
Answered by Harley Weston.
Equivalent fractions 2007-08-31
From Darlene:
What is the equivalent decimal for 0.3 and 6.53
Answered by Penny Nom.
Exponential form 2007-08-31
From Victoria:
The cost of each space shuttle is about $10,000,000. Write this amount in exponential form.
Answered by Stephen La Rocque.
Complements and supplements 2007-08-31
From Gretchen:
An angle is its own complement. Find the measure of a suppllement to this angle.
Answered by Stephen La Rocque.
A turtle tank 2007-08-30
From Natasha:
I have a tub that we are going to use for a turtle tank. it is 3 feet long 1foot 7in wide and 1foot 5 in deep. how many us gallons will this tub hold?
Answered by Harley Weston.
Exponential form 2007-08-29
From Emily:
Write the Exponential Form of 125 x 25 and 81 x 27.
Answered by Stephen La Rocque.
The digital lock for a cell phone 2007-08-29
From Hollie:
i have a four digit lock code on my cell phone and cannot remember what it is. i know that it does not contain a 2 or a 4 and none of the numbers are repeated. the numbers range from 0 to nine and the only way to figure out what the code is, is to try randomly. I was wondering if you could give me a list of possible combinations using the information i gave you to make it easier to crack the code.
Answered by Penny Nom.
Rotational energy needs 2007-08-29
From will:
how much energy in KW do i need to start and subsequently rotate a 4 ton fly wheel of 1 metre radius at 12 rpm?
Answered by Stephen La Rocque.
Similar triangles 2007-08-29
From James:
Question: In the triangle ABC, X is a point on AC. AX = 15 m and XC = 5 m. The angle AXB is 60 degrees and the angle ABC is 120 degrees. Find the length of BC.

I am sure to an extent that this has to do with similar triangles, but I am not certain.

Answered by Harley Weston.
A quadrilateral problem 2007-08-29
From James:
The sides BC and AD of a quadrilateral ABCD are parallel. X is the midpoint of AB. The area of ABCD is Y. Find the area of the triangle CXD in the terms of Y.
Answered by Harley Weston.
Question 2007-08-28
From Reneilwe:
Above is a plan of a house. The scale is 1 cm = 0,8 metres.
3.1 Describe briefly the rooms you come to as you go through the front door and down the passage. Say whether each room is on your left, right, ahead, etc
3.2 Find the actual dimensions of the bathroom (length and breadth)
3.3 Give the length of the passage in metres.
3.4 Give the area of a carpet needed to cover the passage
3.5 A child is in the lounge, at the door into the passage. He pushes a toy car, and it take 2 seconds to go from one end of the passage to the other. Calculate the speed at which the car travels, first in metres per second, then in kilometres per hour. Give the answer correct to 1 decimal place.

Answered by Penny Nom.
Finding a geometric mean and taking its antilog 2007-08-27
From Jack:
Would like to know the basic geometric mean calculation. Also how would I take the antilog from this number?
Answered by Stephen La Rocque.
Applications of sequences and series 2007-08-27
From Trish:
I'm a grade 12 learner working on a math project based on sequences and series. I'd like to know the different types of sequences and series such as fibonacci, fourier, farey, etc.

I've already used the Fibonacci Sequence and Harmonic Series and need two more. The simpler the sequence or series type the better.

I'd also like to know in which non-mathematical areas use sequences and series and how. Areas such as engineering or science.

Answered by Penny Nom.
What size is a lot? 2007-08-27
From GARY:
what size is a lot? how many lots equal to an acre?
Answered by Stephen La Rocque.
Another circle problem 2007-08-27
From Lindsay:
Hello. I'm trying to a math problem and have searched the internet for equations, but have come up empty handed. If you could help, that would be greatly appreciated! The question is stated thus: Find the equation of the following circle: the circle that passes through the origin and has intercepts equal to 1 and 2 on the x- and y-axes respectively.
Answered by Stephen La Rocque and Penny Nom.
A circle and a tangent 2007-08-27
From Lindsay:
Hello. I'm trying to do a math problem and have searched the internet for equations but have come up empty handed. If you could help, that would be greatly appreciated! The problem is stated thus: A circle is tangent to the y-axis at y=3 and has one x-intercept at x=1.
a. Determine the other x-intercept
b. deduce the equation of the circle.

Answered by Penny Nom.
Filling an old swimming pool 2007-08-27
From Russ:
I would like to know how much fill material I would need to fill an old swimming pool. The pool is 18' wide x 36' long and is 4' to 10' deep.
Answered by Penny Nom.
Reference angles 2007-08-25
From Jenny:
find the reference angles for the angles given below, find the quadrants in which the angles lie
1. 0=6n/7
2. 0=3.3

Answered by Stephen La Rocque.
Amy is playing a game 2007-08-25
From vanessa:
problem statement:
amy is playing a game.
in order to win this game, she has to draw a red-coloured card from a given box.
advise her on her chances of winning.

Answered by Penny Nom.
z+81 = 9z-7 2007-08-24
From Erin:
Question from erin, a student:

z+81=9z-7

Answered by Stephen la Rocque.
Two-column proof for a circle geometry problem 2007-08-24
From Kendra:
i have to prove that tangents to a circle at the endpoints of a diatmeter are parallel by stating whats given, whats to prove and a plane, then write a two column proof i dont understand this
Answered by Stephen La Rocque.
A frustum of a right pyramid 2007-08-24
From Andrew:
Find the volume of a frustum of a right pyramid whose lower base is a square with a side 5 in., whose upper base is a square with a side 3in., and whose altitude is 12 in. Round your answer to the nearest whole number.
A. 47cu in.     C. 226 cu in.
B. 196 cu in.     D. 1036 cu in.

Answered by Stephen la Rocque.
More on a percentage question 2007-08-23
From Dee:
In a previous question, you answered:

"What percentage of $34.51 is $10.71?

I translate this English statement into a mathematical statement (equation) using the following

"what" is the unknown, call it x
"percent" is per-one-hundred, that is over 100
"of" is multiplication
"is" is the verb, which in an equation is the equal sign
Thus the English statement above becomes


x/100 $34.51 = $10.71

Solving for x yields

x = 31.03

Hence the gross profit is 31% of your selling price."

How did you solve for 'x'

Answered by Penny Nom.
32,648 square footage = how much land? 2007-08-22
From Elizabeth:
1. What amount of Land = an acre?
2. 32,648 square footage = how much land?

Answered by Penny Nom.
Which is more, 1.25 grams or 4.7 ml? 2007-08-22
From jenni:
Which is more? 1.25 grams or 4.7 ml
Answered by Penny Nom.
65 grams =_____?mg 2007-08-22
From Quan:
65 grams = _____?mg
Answered by Penny Nom.
Converting a repeating decimal to a common fraction 2007-08-22
From isabelle:
how do you turn 6.333... into a fraction in simplest form?
Answered by Stephen La Rocque and Penny Nom.
Where do you use trigonometry? 2007-08-21
From jenny:
where do you use trigonometry besides architecture and engineering?
Answered by Stephen La Rocque.
Two coplanar lines 2007-08-21
From Robin:
I am going two show that the two lines are coplanar:

(x-5)/4=(y-7)/4=-(z+3)/5
(x-8)/7=y-4=(z-5)/3

I know I have to find a point that lie on both lines, but dont really get it.

Answered by Stephen La Rocque and Penny Nom.
Combining probabilities 2007-08-20
From Arul:
Xavier, Yvonne, and Zelda each try independently to solve a problem. If their individual probabilities for success are 1/4, 1/2 , and 5/8 , respectively, what is the probability that Xavier and Yvonne, but not Zelda, will solve the problem ?
Answered by Stephen La Rocque.
Markup 2007-08-20
From Drew:
I own a manufacturing business and I need to figure out the best way to mark up our products so that we can make a profit for this year. Say for instance we have a plow that we can make for $938.00. We need to mark it up 20% so that we have some income. Do we take the $938 x 20% = $1125.60 or $938 / 80% = $1172.50?
Answered by Stephen la rocque and Harley Weston.
A geometry problem 2007-08-20
From samhita:
ABC is a triangle. Let D be a point on side BC produced beyond B such that BD=BA. Let M be the mid-point of AC. The bisector of angle ABC meets DM at P. Prove that angle BAP=angle ACB.
Answered by Chris Fisher.
I want to fill the hole up with dirt 2007-08-19
From Roger:
I have an 18 foot round hole which is 16 inches deep. I want to fill the hole up with dirt. How much dirt do I need to fill the hole??
Answered by Penny Nom.
What feature on a graph shows the solution? 2007-08-19
From San:
To solve the equation 2sinx+square root 3=0, what feature of the graph, in the form y=asinx+b would show the solution? Draw a graph to show the solution. thanks.
Answered by Stephen La Rocque.
The area of a five sided lot 2007-08-19
From Karyn:
I have been looking at your site, but I am still confused as to how to figure out the exact square footage of my irregular lot (since it is a gradual fan lot from the front, and triangles out in the back).
Answered by Harley Weston.
Tangents to a circle 2007-08-18
From Laura:
I have tangents from point A and B that intersect at C. A third tangent XY lies in between the two lines that I have already drawn. I measured the perimeter and then I drew another line that was tangent to the circle and was inside the two lines again and measured the perimeter again. The perimeters were the same but I don't know how to prove why this happened and write a theorem for it.
Answered by Chris Fisher.
The weight of a hollow cylinder 2007-08-18
From chris:
how do you find how much a 4 foot diameter cylinder 4ft tall with a thicknees of 6 inches weigh
Answered by Stephen La Rocque.
Re-arranging the alphabet 2007-08-18
From samhita:
Find the number of ways of arranging 26 letters in the English alphabet in a row such that there are exactly 5 letters between x and y.
Answered by Stephen La Rocque and Penny Nom.
A chord of a parabola 2007-08-17
From Robin:
I am going to show that the chord PtPt' of the parabola has the equation 2x-y(t+t')+2att' A point on the parabola can be represented as Pt= (at2, 2at).
Answered by Penny Nom.
How much water is this? 2007-08-16
From Judy:
Have an area 731 sq. ft with 5" of water all the way around. How many gallons is this? Your help is appreciated.
Answered by Stephen La Rocque.
Relations and functions 2007-08-16
From virginia:
determine whether each relation is a function provide reasons for identifying relation
(3, 4) (5, 9) (9, 9) (2, 3)
(0, 0) (0, 1) (1, 4) (2, 4)
(2, 1) (4, 5) (8, 4) (1, 0)
(8, 3) (8, 0) (7, 7) (4, 7)

Answered by Steve La Rocque.
Point-slope form 2007-08-16
From jenny:
write an equation in the point-slope form and the intercept form slope=4,passing through (1,3) slope=8,passing through (4,-1) slope= -5,passing through (-4,-2) slope= -2,passing through(0,-3)
Answered by Leeanne Boehm.
What is 240mL of water in cups? 2007-08-16
From Gareth:
What is 240mL of water in cups?
Answered by Penny Nom.
How do I find out how much cubic meters will spread over meter squared area? 2007-08-16
From Derek:
How do I find out how much cubic meters will spread over meter squared area?
Answered by Penny Nom.
Area and perimeter 2007-08-16
From sheryl:
a 50*70 horse pen is 240 feet and 3500 square feet a 60*60 horse pen is 240 feet and 3600 square feet why is there a 100 square feet difference?
Answered by Penny Nom.
Angle of depression to a football 2007-08-15
From Tosin:
The angle of depression to a football from the top of building 10m high is 17 degrees. Find the distance of the ball from the foot of the building.
Answered by Stephen La Rocque.
(x^3)-(4x^2)+(4x)=0 2007-08-15
From Amanda:
solve.
(x^3)-(4x^2)+(4x)=0

Answered by Leeanne Boehm.
Two circles C1 and C2 meet at the points A and B 2007-08-15
From Jerry:
Two circles C1 and C2 meet at the points A and B. The tangent to C1 at A meets C2 at P. Point Q inside C1 lies on the circumference of C2. When produced, BQ meets C1 at S and PA produced at T. Prove that AS is parallel to PQ.
Answered by Chris Fisher.
The area and perimeter of a rectangle 2007-08-15
From Brooklyn:
Is there a way to find the perimeter of a rectangle if you have the area, or vice versa? If so, what is the equation?
Answered by Leeanne Boehm and Penny Nom.
Division by decimals 2007-08-15
From Brooklyn:
I am studying for an exam, and I will not be able to use a calculator. I can not figure out how to do division is the number on the outside of the box is a decimal. Ex: 2/6.3=X Can you please help me figure this out?
Answered by Penny Nom.
The length of the third side of a triangle 2007-08-15
From Brooklyn:
What is the equation to find the length of the third side of a triangle if you have the length of A, B, and the angles(s)?
Answered by Stephen La Rocque.
Circle Geometry 2007-08-14
From Robin:
In a triangle ABC, angle A=75 and B=60. A circle circumscribes the triangle. The tangents of the at points A and B meet in a point D outside the circle. Show that ABD is an isosceles triangle with a right angle at D. Diagram included.
Answered by Stephen La Rocque.
Cubic polynomials 2007-08-14
From Jalon:
The cubic function f(x) = (x+2)^3 touches the x axis only once at x = -2 (negative two). it could also be written as f(x) = (x+2)(x+2)(x+2)

Investigate the cubic functions below as well as the one above and comment clearlt and fully on where they touch/intersect the x axis, and how these points relate to the given function.

a) f(x) = (x – 3)(x +4)(x – 2)
b) f(x) = x(x + 1)^2 (^2 = that means it squared)

Your comments should reference graphs illustrating your conclusions and display another 2 trinomial graphs that demonstrate your conclusions. particular attention should be given to the number of times your function crosses or touches the X axis. i dont know how this works but could you send the answer to..... spongy_91@hotmail.com

Answered by Stephen La Rocque.
A nonlinear system 2007-08-14
From Marsia:
Explain how the solution of a nonlinear system could be just a point, or two points. Sketching these may make it easier
Answered by Penny Nom.
Adding algebraic fractions 2007-08-14
From John:
Ive completely forgot anything to do with the subject mentioned, so my question is straight to the point..

I need to know how to do the following problem (Preferably do not give me an answer though) (k/3k-8) - (4/k+2)

Answered by Penny Nom.
What are the two numbers? 2007-08-13
From Tiffany:
The sum of two number is 29.The difference is 1. What are the two numbers?
Answered by Stephen La Rocque, Melanie Tyrer and Penny Nom.
10r^2 - 35r = 0 2007-08-13
From Aranxa:
how do u solve this equation by factoring: 10r^2 - 35r = 0
Answered by Penny Nom.
Twenty dining tables 2007-08-13
From priya:
Utkarsh bought 20 dining tables for Rs 120000 and sold these at a profit equal to the S.P of 4 dining tables.Find the S.P of one dining table.(give the answer in statements)
Answered by Stephen La Rocque.
Can you solve this by factoring? 10r^2 - 35r = 10 2007-08-12
From Aranxa:
how do u solve this equation by factoring:
10r^2 - 35r = 10

Answered by Stephen La Rocque.
Water bill woes 2007-08-12
From Terry:
I know that a cubic foot of water=7.48051945 gals., but I keep getting my water bill and they go by thousands of a gal. per cubic foot. Is there a conversion that says that shows this? Water department=.763 x cubic foot used=thousands of gallons water used that month. Exampel: 72cu. ft meter reading x .763+ 58.--- galons of water I use in a month. Total 58=thousand = +gallons. This is wrong. There are only two people in the house.
Answered by Stephen La Rocque.
The equation of a circle 2007-08-12
From ranessa:
Write the equation of the circle that satisfies the given condition.

C(0,8) ; R = 5

Answered by Penny Nom.
What number am I? 2007-08-11
From Belinda:
What number am I? My millions digit is 4. My billions digit is two time = my millions digit. My hundred thousands digit is two more than my = millions digit and three more than my ones digit. My ten millions digit = is one less than my ones digit. My other digits are zero. Write my = short word form.
Answered by Penny Nom.
Slopes of a triangle's medians 2007-08-11
From Gina:
I am trying to help my daughter with this, but I am very weak in Math.Please help! Find slopes of the medians of a triangle whose vertices are P(1,3), Q(3,5), and R(6,2)
Answered by Stephen La Rocque.
A right triangle 2007-08-11
From Peter:
Ok, is it possible to find the height and base of a right triangle when all the information you are given is the length of the hypotenuse?
I also know that the angle between side b and the hypotenuse is 45 degrees. please help.

Answered by Walter Whiteley.
The swaying of a building in the wind 2007-08-11
From San:
During a strong wind, a tall builing, such as the CN Tower, can sway back and forth as much as 100cm, with a period of 10 seconds. Please help me to determine the equation for this function, in the form y=asinkx
Answered by Stephen La Rocque.
How many pennies will a half gallon bottle hold? 2007-08-11
From Harold:
How many pennies will a half gallon bottle hold
Answered by Penny Nom.
How many complete cycles does the piston make in 30minutes? 2007-08-11
From San:
A piston in a large factory engine moves up and down in a cylinder. The height, h centimetres, of the piston at t seconds is given by formula h=120sin(pi)t+200 How many complete cycles does the piston make in 30minutes?
Answered by Penny Nom.
GST and PST 2007-08-11
From Cynthia:
I paid $15.98 for an item. Both GST (.96) & PST ($1.12) were added for a total of $18.06. I know that .96 is GST (6%) and $1.12 is PST (7%).
Other than looking at the receipt, if all I have is the total amount paid, how do I calculate how much of the total are taxes?

Answered by Penny Nom.
Diagonals on a cylinder 2007-08-10
From John:
I have a cylinder that is 37.5" width X 25.25" circumference. I need to create a repeating diagonal lined pattern on the cylinder so that when it prints it joins perfectly at the circumference repeat. The design must follow these specs. Black diagonal lines need to be 1.250" max width X 1.5" max spacing between the black lines. Please Help
Answered by Stephen La Rocque.
Paying off a loan 2007-08-08
From Iona:
I borrowed $6000. 00 from a friend to pay school fee. He said I should not pay back any interest but pay $350.00 monthly after graduation. (1) Make an equation from the above (2) make a graph (3) what is the relationship between the months after graduation and the loan? (4) what is the horizontal and vertical intercept relate to the problem?
Answered by Stephen La Rocque.
An old bike lock 2007-08-08
From Jeremy:
I have an old bike lock i found in the garage of the house i just moved in to. It has four columns and the nubers are 1 through 6. I understand that there are 1236 (6x6x6x6) possibilities. Is there a graph of all the possible combinations cuz i looked all over the web and couldn't find one.
Answered by Penny Nom.
Chords 2007-08-08
From HIMANSHU:
we know that every straight line is the chord of a big circle. i have a circle with radius 3.5feet. I want to know what is the length of the chord, which would be a straight line.
Answered by Stephen La Rocque.
I want to learn math 2007-08-07
From Maria:
Hello. I am 16-years-old, and I would like to learn as much mathematics as I can and to really understand it. I would like to know what people mean when they say things like "thinking mathematically" or "math is equally beautiful and true". However, I'm not sure where to start. :-) What do you think of going back to the beginning and working my way through Euclid? Your help would be very much appreciated. Thank you.
Answered by Harley Weston.
The irrationality of PI 2007-08-07
From Matthew:
I have what I like to think of as a rather interesting question that I can't explain confidently for the life of me. If we take a circle with a radius of 1 and we calculate the circumference, we can use 2 pi R. Doing this calculation results in a circumference of 6.28318530717~ which goes on forever. However, if you were to take a that same circle in the real world, say with radius 1cm and wrap a string around it, and then measure the string, you don't get 6.28~, you get something like 6.2, a much more finite distance. The length of the string is not an irrational number, like the math claims it to be.
Answered by Claude Tardiff and Stephen La Rocque.
4,18,48,100, __ 2007-08-07
From Jeremy:
What is the next number in the following sequence?
4,18,48,100, __

Answered by Leeanne Boehm and Stephen La Rocque.
Find the dimensions of the rectangle that will contain the greatest area 2007-08-06
From Julirose:
The perimeter of a rectangle is 38 meters. Find the dimensions of the rectangle that will contain the greatest area.
Answered by Penny Nom.
Linear systems and inverses 2007-08-06
From Marsia:
explain how inverses are used to solve linear systems.
Answered by Penny Nom.
Angular speed 2007-08-05
From Virginia:
the engine of a sports car rotates at 5,000 revolutions per minute (rpm). calculate the angular speed of the engine in radians per second. use 2n radians = 1 revolution
Answered by Stephen La Rocque.
The volume of a hexagonal pyramid 2007-08-05
From monica:
what is the volume of a hexagon when the base of one triangle is 6cm and has the height of 5.1cm the height of the regular hexagonal pyramid is 18cm?
Answered by Penny Nom.
All five angles of a pentagon ABCDE are equal 2007-08-04
From Jerry:
All five angles of a pentagon ABCDE are equal. The diagonals AC, AD and BD have the same length. Prove that all five sides of the pentagon are equal in length.
Answered by Penny Nom.
Induction - divisibility 2007-08-04
From Jerry:
How would you prove that for any positive integer n, the value of the expression 3^(2n+2) - 8n -9 is divisible by 64.
Answered by Chris Fisher and Penny Nom.
Dirt in a sloped round hole 2007-08-03
From Ken:
How much dirt do I need to fill a ground hole 20 ft in dia. with a sloping floor that has a ht. of 5" on one side and 12" on the other side?
Answered by Stephen La Rocque.
Scalar triple product and volume of a general tetrahedron 2007-08-03
From Anurag:
how do you prove that volume of tetrahedron= 1/12 times scalar triple product of vectors a,b and c?
Answered by Stephen La Rocque.
Factorial fraction 2007-08-03
From Sekhoane:
Expand completely: (N-2)!/N!(9N-1)!
Answered by Stephen La Rocque.
Buying and selling apples 2007-08-03
From Priya:
Gorang bought 60 Kg of apples at Rs 48 per Kg. He sold 70% of the apples at Rs 60 per Kg and the remaining apples at Rs 35 per Kg.Find Gorang's gain or loss percent on the whole transaction.
Answered by Stephen La Rocque.
Radical equations 2007-08-03
From JC:
I have to solve this radical equation using the power rule; squaring twice.
radical 2x+3- radical x+1= 1.

Answered by Stephen La Rocque.
What fraction of the letters of the alphabet is each word? 2007-08-02
From Clayton:
How do I figure this out, the math question has the answers but I have no idea how to get it. my Mom can't figure it out either.

What fraction of the letters of the alphabet are each word
Man = 3/26
Glasses = 7/26
Integrity = 1/4
computer = 2/9

Answered by Paul Betts and Harley Weston.
A statistics example 2007-08-02
From Claudia:
A particular employee arrives to work some time between 8:00 am - 8:30 am. Based on past experience the Company has determined that the employee is equally likely to arrive at any time between 8:00 am - 8:30 am.

On average, what time does the employee arrive?

What is the standard deviation of the time at which the employee arrives?

Find the probability that the employee arrives exactly at 8:12 am?

Find the probability that the employee arrives between 8:20 am - 8:25 am?

Answered by Har.
The height of a pole 2007-08-02
From lalaine:
Hi, this is my problem.. From a point 50.2 m to the pole, a student measured the angle of elevation to the top of the pole to be 32°. Find the height of the pole if the student's height from his feet to his eyes is about 4 ft.
Answered by Penny Nom.
A gas tank 2007-08-01
From Randy:
I am trying to determine the size of the gas tank on a used truck I just purchased. There is no tag on it showing the size. It measures 50" wide by 12" high by 18" deep. How many gallons of gas does it hold?
Answered by Penny Nom.
An irrigation ditch 2007-07-31
From Kevin:
I would like to know how many cubic yards of concrete I would need to fill a section of irrigation ditch that measures 12 inches on the bottom, 40 inches on top, 16 inches high and 20 feet long with a 18 inch diameter, 20 foot long culvert sitting in the ditch (to remain open). I am trying to build a roadway across the irrigation ditch. Thanks very much.
Answered by Penny Nom.
Rolling a metal plate 2007-07-31
From k.a.suresh:
How to calculate the plate size for rolling (2010mm OD)plate thick is 6mm
Answered by Harley Weston.
Logs and exponentials 2007-07-31
From Marsia:
Explain how the functions of exponents and logarithms relate to each other.
Answered by Penny Nom.
Vehicle collision 2007-07-31
From Tania:
If a driver falls asleep at the wheel of a newer model car and slams into the rear of a parked classic mustang (2860-3280lb) and the parked car is thrown 100 ft across the street, what would be the minimum speed required by the newer car in order to move the mustang?
Answered by Penny Nom.
The graph of a log function 2007-07-30
From Marsia:
How would you describe to another student what the graph of a logarithmic function looks like?
Answered by Penny Nom.
An athletic track 2007-07-30
From tammy:
The inner boundary ABCDEF of an athletic track cinsists of two straight parts each 90m long and two semi circular ends as shown in the diagram.If the perimeter of the inner boundary is 400m, calculate the diameter AE of the inner semi circle.

Part 2. If the track is 3.5m wide an athlete starts the 400m run at X and remains in the outer lane . He finishes at Y(1) Calculate the outer diameter YZ
(2)The distance XY (take pie as 22/7)

Answered by Penny Nom.
Three points on the circumference of a circle 2007-07-30
From Bharathi:
given a circle with radius r and a point x,y on its circumference,output two other points x1,y1 and x2,y2 on the circle so thar all 3 points form a equilateral triangle.
Answered by Stephen La Rocque.
Using the chain rule to solve a derivative 2007-07-29
From Charles:
I need to find the derivative fo the following function.   _______________________
\/ ______________
  \/ (x - 1) / (x + 2) + 1

Answered by Stephen La Rocque.
Two chords in a circle 2007-07-29
From Jerry:
Points A and C lie on the circumference of a circle. B is a point inside the circle. When produced, AB and CB meet the circumference at points E and D respectively. Prove that AB = CB, then EB = BD.
Answered by Stephen La Rocque.
The odds of winning a contest 2007-07-29
From Kevin:
I have been asked by a customer in Missisauga to track down some odds of winning a contest. In this contest, the last 4 digits of the winner's driver's license number have to match the last 4 digits of a Harley VIN number. There are 200 people maximum allowed into the drawing, 10,000 total possible VIN combos. Would the odds be 1:50 or something other than that?
Answered by Harley Weston.
Building a garage 2007-07-29
From charles:
I want to build a garage that is 24 feet 4 inches wide by 50 feet long. can you please tell me what the length of one corner is to the other?
Answered by Penny Nom.
320 acre lots 2007-07-29
From melisa:
we are splitting a 320 acre parcel into five 35 acre lots with one 145 ace lot. I have the footage of each side there are eight: 5280 x 2640x2640then out1320x1320up x2640back inx1320upx1320this now would meet back to the 5280foot side i know there is a half of a square mile and 320 acres.what would the measurements be in feet for length to make a square or a rectangle to make a 35 acre lot?
Answered by Harley Weston.
A 20,000 gallon fuel bag 2007-07-28
From Kevin:
I have a 20,000 gallon fuel bag Length 32'x Width 29'x Height 4.2'. I need to build a containment around it. If the bag was to rupture the containment would have to hold the 20,000 gallons of fuel plus ten percent. What size would the containment have to be in order to hold the fuel.
Answered by Stephen La Rocque.
Equation of a perpendicular line 2007-07-28
From Marina:
Find the equation, is standard form, with all integers coefficients, of the line perpendicular to x+3y=6 and passing through (-3,5)
Answered by Stephen La Rocque.
A polynomial equation 2007-07-28
From claire:
Hi I sent this question a while ago but I think I put in the wrong email address. So here it goes again....

If 4x^3 - 6x^2 + 1 = (x-2)(x+1)Q(x) +ax+ b where Q(x) is a polynomial, find a and b.

Answered by Penny Nom.
Solve y'' + y = 0 2007-07-28
From Shih-ya:
How do you solve y’’ + y = 0
Answered by Stephen La Rocque and Harley Weston.
At what speed does Chuck travel? 2007-07-27
From Jessica:
Chuck and Dana agree to meet in Chicage for the weekend. Chuck travels 224 miles in the same time that Dana travels 204 miles. If Chuck's rate of travel is 5 mph more than Dana's, and they travel the same length of time, at what speed does Chuck travel?
Answered by Harley Weston.
More on the cardinality of sets 2007-07-27
From Mac:
Can you please help me to find and verify whether the following are finite, countably infinite and uncountable ?
Answered by Harley Weston.
A pile of soil that is 284' around and 12' high 2007-07-26
From bev:
I have a pile of soil that is 284' around and 12' high. I need to know how many yards are in that pile. And then how do I convert that to cubic yards
Answered by Penny Nom.
Multiplcation of two negative numbers 2007-07-26
From Brett:
Someone asked a question about multiplication and division of two negative numbers yielding a positive result here: http://mathcentral.uregina.ca/qq/database/QQ.09.99/butler1.html I was not fully happy with the explanation b/c I want to give me daughter a real-world example and I can't seem to find one.

The following illustrates why multiplying negative numbers has become difficult to explain:

2 X 2 = 4

----(-4)---(-2)---0---2---4
In this example we start with 2 and then want 2 more of them. When we move across the number line from 2 to our answer, which is four, we have moved only 2 units to the right.

-2 X -2 = 4

----(-4)---(-2)---0---2---4
In this example we start with -2 and then want -2 more of them. When we move across the number line from -2 to our answer, which is four, we have moved 6 units to the right.

How can the phenomenon of multiplying two negative numbers being more powerful than multiplying two positive numbers be explained? -Brett

Answered by Stephen La Rocque and Harley Weston.
Cubic metres of topsoil 2007-07-25
From Laurie:
How do I convert cubic meters to square meters?
Answered by Harley Weston.
Guessing multiple choice answers 2007-07-24
From carla:
You are taking a multiple choice quiz that consist in 3 questions, each question has 3 possible answers only one is correct. To complete the quiz you randomly guess the answer to each question . Find the probability of guessing exactly 2 answer correctly. b) at least to answer correctly . c)less than two answer correctly
Answered by Stephen La Rocque.
Countable and uncountable sets 2007-07-24
From Mac:
Hi, i tried to read few webpages related to the countably infinite and uncountable sets. Even i read few questions from this forum.

But i am not convinced with this explanation. If you have any good book that explains this in layman term, please redirect me to that.
1) Can you please explain what is the difference between these too ?
2) How could you say set of Natural number and set of even numbers are countably infinite ?
N={1,2,3,...} and Even= {2,4,6,...}
When an element in the even set is some 2n, we will map it to 'n'.So now we have a bigger number(2n) right ?
Sorry, i didn't understand that.
...

Can you please help me out to understand that ?

Answered by Harley Weston.
Area of a star in a regular pentagon with side length 10cm 2007-07-24
From Chetna:
A regular pentagon with side 10 cm has a star drawn within (the vertices match). What is the area of the star?
Answered by Stephen La Rocque.
(1 - i)^5 2007-07-24
From sofia:
Compute the given arithmetic expression and give the answer in the form a + bi where a,b element in R. 1. (1 - i)^5
Answered by Harley Weston.
A complex number in polar form 2007-07-23
From roland:
write the given complex number z in polar form lzl(p+qi) where lp + qil=1 for 3 - 4i.
Answered by Harley Weston.
Angle of depression 2007-07-23
From joyce:
hello, here is my problem......
As you stand on a bridge w/c is 100 ft. above the water you are looking @ an approaching barge. If the A of top of the front of the bridge is 29.04 degrees and the angle of depression of the rear is 17.36 degrees . Find the length of the barge?

Answered by Harley Weston.
Equality of sets 2007-07-23
From Mac:
Hi, I learnt set theory recently. My teacher and few of the weblink actually give different definition for basic set. Can you please solve this ?

My teacher says, {1,2,3} and {1,1,2,3} is also set.
But in this link http://library.thinkquest.org/C0126820/setsubset.html it says,
"A set has no duplicate elements. An element is either a member of a set or not. It cannot be in the set twice."
and "{1, 2, 3} is the same as the set {1, 3, 2, 3, 1}"

My question is,
1. Whether duplicates allowed in the set or not ?
2. Even if the duplicates are allowed, {1,2,3} and {1,1,2,2,3,3} are same or not ?

Answered by Penny Nom and Harley Weston.
What are the dimensions of the field? 2007-07-23
From Debourah:
a soccer field is 1080 square meters the width is 2 meters less than one-third the length. What are the dimensions of the field?
Answered by Penny Nom.
f(x) = (x^4) - 4x^3 2007-07-22
From Michael:
I'm a student who needs your help. I hope you'll be able to answer my question. Here it is: Given the function f(x)=(x^4)-4x^3, determine the intervals over which the function is increasing, decreasing or constant. Find all zeros of f(x) and indicate any relative minimum and maximum values of the function.
Any help would be appreciated. Thank you for your time.

Answered by Harley Weston.
A football stand 2007-07-19
From jean:
a football stand 150 yd. long has 20 tiers of seats; each has a rise of 2 ft. and tread of 3 ft. it constructed of reinforced concrete with a cross section of 36*54. find the amount of material used in its construction.
Answered by Harley Weston.
The volume of a cistern 2007-07-19
From sharon:
We have a cistern to hold spring water that measures 12 feet in depth by 5 feet in diameter. How much in liquid gallons will it hold?
Answered by Penny Nom.
A triangular trough 2007-07-19
From janine:
A trough is formed by nailing together, edge to edge, two boards 11 ft. in length, so that the right section is a right triangle. If a 15 gallon of water are poured into the trough and if the trough is held level so that a right section of water is an isosceles right triangle, how deep is the water? (231cu.in=1 gal.)
Answered by Penny Nom.
An oblique triangle 2007-07-19
From fhay:
In an oblique triangle, If side a=95, side b=102 and side c = 150 find the missing angles solved by right angles...Thank's a lot......
Answered by Penny Nom.
A matrix of polynomials 2007-07-18
From Mac:
can you please help me out to solve this ?

Let A be a n*n matrix, the elements of which are real (or complex) polynomial in x. If r rows of the determinant becomes identical when x=a, then the determinant
A) has a factor of order r
B) has a factor or order > r
C) has no factor
D) has a factor of order < r

Answered by Harley Weston.
How many diagonals does a polygon have? 2007-07-18
From Edna:
how do you find the numbers of diagonals of a polygons?
Answered by Stephen La Rocque.
If x = 2 then 4 + 5x / 4 + x = ? 2007-07-18
From Daniel:
If x = 2 then 4 + 5x / 4 + x = ?
Answered by Stephen La Rocque.
Fractions of fractions 2007-07-18
From Bridget:
Sam and Mary each owned one-half stock in a printing company. Sam sold 2/3 of his stock to Mary. What fractional part of the printing business does Mary now own?
Answered by Penny Nom.
What is the bearing from the port to the ship? 2007-07-18
From fhay:
A ship leaves a port and sails for 4 hours on a course of 78 degrees at 18 knots. Then the ship changes its course to 168 degrees and sails for 6 hours at 16 knots. After 10 hours(a) what is the distance of the ship from a port and (b) what is the bearing from the port to the ship?
Answered by Harley Weston.
Angle of depression (declination) between sailboat observations 2007-07-18
From Joyce:
From a cliff 150 ft above a lake, we see a boat sailing directly towards us. The angle of depression of the boat is seen to be 5 degrees and 7 inches and 11 degrees and 18 inches. Find the distance sailed between observations.
Answered by Stephen La Rocque.
Angle of inclination from the horizontal 2007-07-18
From Joyce:
In flying upward for 1260 yards along a straight inclined path airplane rises 156 yards. Find the climbing angle ( the angle of inclination from the horizontal) Thank you in advance
Answered by Stephen La Rocque.
Equivalence relations 2007-07-18
From jim:
Determine whether the given relation is an equivalence relation on the set. Describe the partition arising from each equivalence relations.
1. x is Related to y in Real number if lxl=lyl.
2. n is Related to m in Z+ if n and m have the same number of digits in the usual base ten notation.

Let n be a particular integer in Z+. Show that congruence modulo n is an equivalence relation on Z.

Answered by Harley Weston.
The area of a triangular pyramid 2007-07-18
From Jhezelle:
What is the formula of finding the surface area of a triangular pyramid?
Answered by Penny Nom.
Circle Geometry III 2007-07-17
From Sean:
Two rays are drawn from the same point A outside a circle, and intersect the circle as shown in the picture. Prove that the measure of angle A is one-half the difference between the measures of arcs BD and CE.
Answered by Stephen La Rocque.
The height of an arc at the center 2007-07-17
From Bob:
I have a circle with a 26" radius and a cord whose length is 20". How can I find the height of the arc at the center point of the cord?
Answered by Penny Nom.
Pythagoras theorem in daily life 2007-07-17
From sana:
i would like to what are the 5 practical uses of the Pythagoras theorem in daily life??? its for a math project thanx a lot sana
Answered by Penny Nom.
Circle Geometry II 2007-07-17
From Sean:
Let M be a point outside a circle, and let a line through M be tangent to the circle at point P. Let the line through M and the center of the circle intersect the circle in points Q, R.
Prove that │PM│2 = │MQ│ x │MR│

Answered by Stephen La Rocque.
The area of a property 2007-07-17
From Mary:
I currently have an opportunity to sell my property to a business and they would like me to send a proposal to them with details. I need to know the square footage of my property and have tried using your formula from similar questions. The dimensions of my property are as follows
front line facing street: 19.908 m
right side : 37.338 m
back line : 29.779 m
left side : 43.983m
The front line and the right side are at right angles to each other. Can you formulate and reply? Thank you Mary

Answered by Harley Weston.
Circle Geometry - Quadrilateral circumscribing a circle 2007-07-17
From Sean:
Four lines are tangent to a circle that form a quadrilateral. It appears that the quadrilateral is a trapeziod but this is not a given. Prove that the combined lengths of two opposing sides of the quadrilateral are equal to the combined lengths of the other two opposing sides of the quadrilateral.
Answered by Stephen La Rocque.
The domain and range of a quadratic function 2007-07-17
From Linda:
I have been trying to solve this problem but I'm unable to figure it out. How do I find the domain and range of y=-(x+1)^2-3? Please explain...thanks!
Answered by Stephen La Rocque and Penny Nom.
Sample size 2007-07-17
From Nurse:
If a population has standard deviation 20, what is = the minimum sample size to be 95% confident that the error should be = accurate to within 4?
Answered by Harley Weston.
A model for a building 2007-07-17
From Carl:
A model for a building is 50 centimeters long, 38 centimeters wide, and 225 centimeters tall on model 1 centimeter represents 2.5 meters. I need to find the volume of the actual building in cubic meters,and the surface area of the building including ceilings and floors. round to the nearest tenth of a square meter.
Answered by Harley Weston.
The height of a triangle 2007-07-17
From linda:
IF AREA EQUALS 12radical 30 of the triangle find the height B= 3 radical 3
Answered by Penny Nom.
Radical 96 plus radical 27 divide by radical 3 2007-07-17
From linda:
radical 96 plus radical 27 divide by radical 3
Answered by Penny Nom.
A normal to a curve 2007-07-16
From Samantha:
The function f is defined by f:x --> -0.5x^2 + 2x + 2.5
Let N be the normal to the curve at the point where the graph intercepts the y-axis. Show that the equation of N may be written as y = -0.5x + 2.5.
Let g:x--> -0.5x + 2.5
(i) find the solutions of f(x) = g(x)
(ii) hence find the coordinates of the other point of intersection of the normal and the curve

Answered by Penny Nom.
The isosceles triangle of largest area with perimeter 12cm 2007-07-16
From sharul:
find the dimension of isosceles triangle of largest area with perimeter 12cm
Answered by Harley Weston.
Another trig limit 2007-07-16
From Richter:
lim (π + x)/[cos(x/2)]
x-->-π

Answered by Harley Weston.
A trig limit 2007-07-16
From Richter:
what is the value of lim [cos (π/x)]/(x-2) as x-->2?
Answered by Harley Weston.
Height of a tower from two observations 2007-07-16
From joyce:
An observer wishes to determine the height of a tower. He takes sight @ the top of the tower from A & B w/c are 5oft. apart @ the elevation on a direct line w/ the tower. The vertical angle @ point A is 30 degrees & the point B is 40 degrees. What is the height of the tower? Find the value of x in angle tangent 40 degrees and 30 degrees? Show the solution of the value of x?
Answered by Stephen La Rocque.
Height of an antenna (angle of elevation) 2007-07-16
From Fhay:
An antenna stands on the edge of the top of a 52 story building from a point 320 ft. from the base of the building, the angle of elevation to the top of the antenna is 64 degrees in each story is 12 ft. high. Find the height of the antenna
Answered by Stephen La Rocque.
Finding the area of a circle from its circumference 2007-07-16
From Amanda:
what is the area of a circle with a circumference of 3000 metres?
Answered by Stephen La Rocque.
The two towers (angles of elevation trigonometry) 2007-07-14
From joyce:
The angle of elevation of tower B from the top of tower A is 28 degrees and the angle of elevation of the top of tower A from the base is 46 degrees Find the height of tower A if tower B is 120 m high?
Answered by Stephen La Rocque.
Proving a quadrilateral is a rectangle 2007-07-14
From Sonja:
I was having this discussion with another teacher and we need a third opinion. When you are trying to prove a quadrilateral is a rectangle which method should you use:
  1. Prove the shape is a parallelogram by doing slope 4 times by stating that parallel lines have equal slopes. Then proving a right angle by stating that perpendicular lines have negative reciprocal slopes.
  2. Doing the slope 4 times and stating that the shape is a rectangle because opposite sides are parallel because of equal slopes and it contains a right angle because of negative reciprocal slopes.
I guess the real question is do you have to first state that the shape is a parallelogram?

Answered by Stephen La Rocque.
Comparing flow rates of two pipes 2007-07-14
From Kenneth:
If two water pipes are 3 feet long, but one of them has a 1 foot diameter and the second has a 1 1/2 foot diameter, what simple mathematical method can be used to determine how much faster one pipe can drain water than the other pipe?
Answered by Stephen La Rocque.
Implicit Derivatives 2007-07-13
From Charles:
I need help computing y' by implicit differentiation the question is: y^2 + x/y + 4x^2 - 3
Answered by Stephen La Rocque.
Any regular polygon inscribed in a circle 2007-07-12
From DJ:
Circle with r=12" is inscribed in a regular octagon. What is the length of each octagon segment? Note: Our answer works for any regular polygon inscribed in any circle.
Answered by Stephen La Rocque.
Length of a circle 2007-07-11
From Debra:
i have a radius of 73 inches, i need to know the length of the circle please.
Answered by Stephen La Rocque.
Discount percentage 2007-07-11
From Greg:
List Price: $50.00 Discount: 33 1/3% What would the net price be?
Answered by Stephen La Rocque.
Compound interest 2007-07-11
From Michelle:
Present Value of $1500 to be paid in Year 7, with an interest rate of 12%
Answered by Stephen La Rocque.
Converting measurements 2007-07-11
From Nicole:
if you have 8 oz of water how many cc: will there be.
Answered by Stephen La Rocque.
Calculating sales taxes 2007-07-11
From Tonya:
Hi, Im having trouble calculating GST and PST I know that in this province, GST is 6% and PST is 7%, I have a total of $275, and GST is 0.06, and PST is 0.07, so do I add those two and multiply them by the total, $275?
Answered by Stephen La Rocque.
The probability of rolling double sixes twice in a row 2007-07-10
From Matt:
what is the probability of rolling double sixes twice in a row?
Answered by Stephen La Rocque.
Grams of vodka 2007-07-10
From Andrew:
milliliters to grams..vodka 80 proof?
Answered by Stephen La Rocque.
How many women and men math students are there? 2007-07-09
From Bing:
there are 38 math students. How many men and women strudents are there if the women exceeded one half of the men by 17?
Answered by Stephen La Rocque.
Derivative of a Function 2007-07-09
From Bob:
What is the derivative of the function a sub n = [n/(n+1)]^n ?
Answered by Stephen La Rocque.
loga b + logb a < 0 2007-07-08
From Claire:
Suppose a, b are real numbers taking all positive values except 1, determine (without the use of calculators) whether there exists values of a and b such that loga b + logb a < 0
Answered by Penny Nom.
Proof that any side of a triangle is less than half the perimeter. 2007-07-07
From Omkar:
Any side of a triangle is smaller than half of its perimeter, prove this in short ?
Answered by Stephen La Rocque.
Five button push lock 2007-07-06
From Jeffrey:
We have a 5 button push lock and we were wondering how many different codes were possible in this situation. In this lock each button can only be pressed once (ie it's not a numbered key pad) and order matters. How would we find out the # of possible codes?
Answered by Stephen La Rocque.
Can't find circumference of an ellipse 2007-07-06
From Michele:
I need to figure the circumference of an oval and I know the height and width.
Answered by Penny Nom.
Finding the radius of an inscribed circle 2007-07-05
From Maria:
I need to find the radius of a circle which is inscribed inside an obtuse triangle ABC. I know all the angles and all the lengths of the triangle.
Answered by Stephen La Rocque and Chris Fisher.
Using calculus to prove the formula for the area of a triangle 2007-07-04
From Apratim:
Using calculus how can one show that the area of any triangle is 1/2 times its base times its height?
Answered by Stephen La Rocque.
Tons of concrete 2007-07-03
From Trey:
I have 300,000 square feet of concrete and want to know how to convert this to tons?
Answered by Harley Weston.
An irregular hexagon 2007-07-03
From Rebecca:
I need to find the shape of an irregular hexagon, but only have the lengths four of the sides.

It actually looks like a rectangle with a trapezoid on top of it.

The bottom of the rectangle is 150'. The sides are 75'. The length of the top is 130'. I do not have the lengths of the two smaller sides.

Is there any way to find the area of this shape?

Answered by Penny Nom.
A statistical difference 2007-07-03
From a student:
I am writing a paper for a grad class in which I am comparing achievement levels of boys and girls. I have data that shows the average marks for girls and boys in each grade 12 subject for the entire province. My prof want me to show there is a statistical difference. What do I have to do to show this? I don't have access to the raw data, so don't know the SD. Is there anything I can do mathematically with the data that I have? (I can determine n for each set of data, it ranges between 2400 and about 13000 for each subject area.)
Answered by Chris Fisher and Harley Weston.
The volume of a pool 2007-07-03
From maria:
we have a pool that is 2 feet by 5 feet by 3 1/2 feet - how many gallons of water does it hold
Answered by Stephen La Rocque.
I need to provide 712 cubic feet to water storage capacity 2007-07-03
From William:
I need to provide 712 cubic feet to water storage capacity. If I use 15 inch diameter pipe, what would the length need to be to provide the capacity?
Answered by Stephen La Rocque.
A square and an equilateral triangle have the same perimeter. 2007-07-02
From eve:
I am trying to help my daughter with the following problem: A square and an equilateral triangle have the same perimeter. The sum of one side of the square and one side of the triangle equals 28 cm. Find the sides of the figures.
Answered by Penny Nom.
Algebraic EquationsAx+By=C 2007-07-02
From Juan:
The general "Standard" form of an equation is Ax+By=C. Re-Write this equation in slope-intercept (y=mx+b) form.
Answered by Harley Weston.
Alphanumeric addition 2007-07-01
From JYOTI:
the letters in the alphanumatic addition are all different. find the numbers indicated in codes. all letters are digits from 0 to 9. SEND+MORE=MONEY.
Answered by Stephen La Rocque.
The combintion of a bike lock 2007-07-01
From Tricia:
I have a bike lock that I know the numbers 3, 6, 8, & 9 are part of the combination, but I can't remember the order. What is the calculation to get all of the possible combinations.
Answered by Stephen La Rocque.
A sand base for a pool 2007-07-01
From Clark:
I am adding sand for a base for a pool. It is 20' round and I would like the sand to be 2" deep. Can you tell me how many cubic feet that would be? Thanks!
Answered by Harley Weston.
A cylindrical slab of concrete 2007-06-30
From Kenn:
I have a tank that needs a base of cement put in it. I dont know how to do this. The tank is 8 feet across and I want the cement to be 5 or 6 inches deep. Can you please tell me how much cement I need to do this.
Answered by Harley Weston.
The lock code for my phone 2007-06-30
From Amanda:
okay so i forgot my lock code for my phone its four digits but i know it doesn't have a 1 or a 0 in it so it has to be from numbers 2 to 9 i need to know all the four digit combinations asap cause i cant use my phone till can someone please help me Amanda
Answered by Penny Nom.
What is the spacing for spindles? 2007-06-30
From david:
what is the spacing for spindles? a 12ft railing 17 spindles at 1 5/8in ea
Answered by Harley Weston.
If A is 1 and Z is 26 2007-06-29
From Sue:
If A is 1 and Z is 26 is there a word that when you multiply all the numbers instead of letters adds up to a million?
Answered by Chris Fisher and Harley Weston.
A linear system 2007-06-29
From Kim:
solve for x,y,z

x-y+z=4 2x+2y-z=-2 x-2y+2z=7

Answered by Harley Weston.
log(0.81) 2007-06-28
From Ade:
If log 3 = 0.477 . The answer of log 0.81
Answered by Harley Weston.
The number of blocks on the face of a pyramid 2007-06-27
From Heather:
I was wondering if there was an equation in order to calculate the number of blocks on the face of a pyramid. For example:

X
X X
X X X

Is there a specific formula in order to calculate that there are 6 blocks here? Thank You, Heather

Answered by Harley Weston.
Log base 2 of log base 2 of x 2007-06-27
From alex:
y = log base 2 of lag base 2 of x The slope of the tangent to the given curve at its x-intercept is..?
Answered by Harley Weston.
Splitting the profits unequally 2007-06-27
From Maci:
We recently hosted a fundraiser. Not everyone will receive a whole share of the profits because they did not participate the entire time. Several will receive only 1/2 or 1/3 share. How do I divide the profits?
Answered by Stephen La Rocque.
Using Heron's Formula to help maximize the area of a triangle 2007-06-27
From Claire:
Given one side of a triangle is 4 cm and the ratio 1:3 for the other 2 sides. What is the largest area of the triangle?
Answered by Stephen La Rocque and Harley Weston.
How do I convert inches to mm in my head? 2007-06-27
From philip:
Hi, I would like to know how to convert inches to mm in my head, I use socket sets and want to know an easy way to work out what 3/16, 7/32 is in mm. Thanks Phil
Answered by Harley Weston.
The area of an irregular polygon 2007-06-27
From chris:
finding square footage of a polygon with unequal measurements
Answered by Stephen La Rocque and Harley Weston.
Edging around a pool 2007-06-26
From Celeste:
I have a 27ft. above ground pool. I want to put edging around it but do not know how many feet of edging I need. Can You help? The edging will be approximately 2 feet away from the pool.
Answered by Penny Nom.
sin|x| and cos|x| 2007-06-25
From Mac:
Can anyone tell me whether sin|x| and cos|x| is differentiable at x=0 ? As far as i know, cos(x) and sin(x) is differentiable at all x.
Answered by Penny Nom and Stephen La Rocque.
Limits as x approaches a constant 2007-06-25
From Mac:
can you please tell me what is the reason they say "denominator is a negative quantity" in the solution 11 and "denominator is a positive quantity" solution 10 ??
If i guess correctly, for solution 10, its because of x^2 in the denominator.

Answered by Penny Nom.
A cubic yard of gravel 2007-06-24
From Duncan:
I'm trying to figure out how much area 1 cubic yard of gravel would cover at a depth of 2 inches? I've asked around and have many different answers.
Answered by Stephen La Rocque.
Volume in a triangular water trough 2007-06-24
From David:
A water trough has sloping sides of length 500mm making it triangular in cross section, with vertical ends. The width at the top is 600mm and the length is 2.0 metres.
(i) Calculate the capacity of the trough, giving your answer accurate to the nearest litre.
(ii) Find out the depth of the water when the trough is half full.

Answered by Stephen La Rocque.
The volume of a cube 2007-06-23
From Leah:
I am having trouble finding the equation for this problem.: What is the volume of a cube with diagonals of length 6 sqrt(3) cm? answer: (BLANK) cm3
Answered by Penny Nom.
The substitution method 2007-06-23
From mary:
solve the system by the method of substitution: -5x+2y=-15 4x-y=9
Answered by Stephen La Rocque.
Exponential form: x^y 2007-06-22
From Kishor:
whats the easy way to calculate X raised to Y where y is much greater than x.
Answered by Stephen La Rocque.
Volume of a box 2007-06-22
From ashley:
A storage bin is 18 ft long, 6 ft wide and 3 ft high. What is its volume in cubic feet?
Answered by Penny Nom.
Simplify 2007-06-21
From A student:
what is the answer of 3y+5/2(4y+1)+5y/2
Answered by Penny Nom.
Rewrite 2x+4y+2=3y+5 in general form 2007-06-21
From dawn:
how do you rewrite 2x+4y+2=3y+5 in general form?
Answered by Stephen La Rocque.
(2x + 3y)^2 = 0 2007-06-21
From Jon:
how do i solve for x and y if (2x+3y)²=0?
Answered by Stephen La Rocque.
The sum of angles in a square is three hundred and sixty degrees 2007-06-21
From victor:
can you please send to me a formalize proof for the assertion that, the sum of angles in a square is three hundred and sixty degrees. i will be grateful to hear from you
Answered by Stephen La Rocque.
Simultaneous equations : the Elimination method 2007-06-21
From Patricia:
I need to find the value of X and Y using the Elimination method.

5/x + 3/y=4
25/x-2/y=3

Answered by Stephen La Rocque.
The area of part of a circle 2007-06-21
From Sarah:
Hi, I need help finding the area of a part of a circle that involves a chord. I need this answer/help very soon if at all possible. I have attached a drawing of what I am trying to solve. Thank you for your help! Sarah!
Answered by Penny Nom.
Simplifying complex denominators 2007-06-21
From Krys:
How do I simplify completely? ((4+i ) / (3+i )) - ((2-i ) / (5-i ))
Answered by Stephen La Rocque.
A polynomial with integer coefficients 2007-06-20
From Krys:
How do I find a polynomial P(x) with integer coefficient that has degree 4 and zeros 1-2i and 1 (with multiplicity of 2)?
Answered by Penny Nom.
Factor X^6 - 64 2007-06-20
From Krystal:
How do I Factor X^6 - 64 into linear factors?
Answered by Penny Nom.
Making a box with a particular volume 2007-06-20
From Kimberly:
How do you convert 487 cf into box?
Answered by Stephen La Rocque.
Finding the center of an ellipse 2007-06-20
From Sima:
find the center of the ellipse with the equation 3x^2 +4y^2+18x-32y-5=0
Answered by Penny Nom.
The perimeter of a triangle 2007-06-20
From chelsie:
how do i find the perimeter of a triangle given the area, base, and height?
Answered by Penny Nom and Walter Whiteley.
The volume of a pool 2007-06-20
From Mary:
How can I calculate the square footage of my 18 feet diameter, 4 feet deep swimming pool?
Answered by Penny Nom.
How many acres is 1.8 sq miles? 2007-06-19
From Jordanne:
How many acres is 1.8sq miles?
Answered by Penny Nom.
How many gallons of water 2007-06-19
From Marie:
A water treatment plant is built with three cylindrical tanks to contaim the water for a town. Each tank has a radius of 15 feet and a depth of 25 ft.

If there are about 7.5 gallons in a cubic foot of water, approximately how many gallons of water can be treated at the plant at any one time?

Answered by Stephen La Rocque.
A square and a rectangle 2007-06-19
From Clarence:
What is the difference between rectangle and a square
Answered by Penny Nom.
How many tons of sand? 2007-06-19
From Richard:
I need to know how many tons of sand it would take to fill a container 20 feet long, 20 feet wide, and 6 inches deep. I am not sure of the makeup of the sand as far as dry, loose, wet, etc. If you could give me calculations of all the variants it would be appreciated.
Answered by Stephen La Rocque and Penny Nom.
Simultaneous inequalities 2007-06-18
From Freddy:
Watson Electric has production facilities in Valley Mills, Marlin,and Hillsboro. Each one produces radios,stereos,and TV sets. There production capacities are

Valley Mills: 10 radios, 12 stereos, and 6 TV sets per hour Marlin: 7 radios, 10 stereos, and 8 TV sets per hour Hillsboro: 5 Radios, 4 Stereos, amid 13 TV sets per hour

QUESTION

How many hours should each plant be scheduled to fill an order of 1095 radios, 1230 stereos, and 1490 TV sets?

Answered by Stephen La Rocque.
Area of a circular garden 2007-06-18
From Cynthia:
Hi is this the correct formula for this problem? What is the approximate area of a circular garden that is 20 feet in diameters?
Answered by Stephen La Rocque.
Using the Pythagorean Theorem 2007-06-18
From cynthia:
Hi, If I have a question with a right triangle and it asks.... If ABC is say 400 miles. How much shorter will the miles be if I travel from BC? I don't exactly remember the question but, I would I solve a problem similiar to this one?
Answered by Stephen La Rocque.
The length of an arc 2007-06-18
From Lenny:
How do you find out the arc length of a sector 50 degrees wide??
Answered by Stephen La Rocque and Penny Nom.
Babylonian geometry 2007-06-17
From marleen:
The following problem and the solution were found on a Babylonian tablet dating from about 2600BC:

Problem:
60 is the Circumference, 2 is the perpendicular, find the chord.

Solution:

Thou double 2 and get 4
Take 4 from 20, thou gettest 16
Square 16, thou gettest 256
Take 256 from 400, thou gettest 144

Whence the square root of 144, 12 is the chord.

Such is the procedure. Modern day mathematicians have reasoned that the Babylonian Mathematician who solved this problem assumed that the value of Pi is 3. By explaining in detail how the Babylonian Mathematician must have solved this problem, justify the reasoning of the modern mathematicians.
Answered by Stephen La Rocque.

The Golden gate bridge 2007-06-17
From Khan:
I am doing a project on parabola and Bridges. I have chosen the Golden gate bridge as my bridge. Now The suspension cables are shaped like a parabola.

We have to derive an equation for this parabola. We have to assume the vertex is (0,0). Now I am having troubles writing the equation in standard form.

The information is this Height of tower above water: 746 ft = 227 m

Height of tower above roadway: 500 ft = 152 m

Length of one side span: 1,125 ft = 343 m

length of suspension span including main span and side spans: 1.2 miles = 6,450 ft

Now my question is this, i KNOW THE standard form for this parabola opening up would be (x-h)squared = 4a (y-k)

Since h,k are 0,0 the equation will now take the form of xsquared = 4ay My question is to find the equation I need to know A how do i get this a based on the above provided info. Thanks for your help and concern.

Answered by Penny Nom.
An inequality 2007-06-16
From jerry:
Two real numbers a and b satisfy the equation ab=1.
i) Prove that a^6 + 4b^6 >4
ii) Find out whether the inequality a^6 + 4b^6 >4 holds for all a and b such that ab = 1

Answered by Stephen La Rocque.
Solving a literal equation 2007-06-15
From Allen:
Solve each literal equation for the indicated variable. D= C-S/n (solve for s) and A=1/2h (B+b) (solve for b)
Answered by Penny Nom.
Compatible numbers 2007-06-15
From meryl:
what is a compatible number? ex.6,321/8
Answered by Penny Nom.
Algebra : 9x + 7 = 5x - 3 2007-06-15
From Allen:
I am having a problem solving for x. The problem is: 9x + 7 = 5x - 3
Answered by Stephen La Rocque.
Probability and blood type 2007-06-15
From Jim:
Blood Type Male Female Total
O 80 370 450
A 150 250 400
B 50 50 100
AB 20 30 50
Total 400 600 1000
What is the probability that the person selected is female if the person’s blood type is O?
What is the probability that the person selected has blood type AB if the person is male?
What is the probability that the person selected is a female with blood type O?

Answered by Penny Nom.
Simplifying a quartic rational expression using long division 2007-06-14
From Megan:
x+2/12x^4+17x^3+0x^2+8x-40=
Answered by Stephen La Rocque and Penny Nom.
What is the name of the shape that is 3D and irregular? 2007-06-14
From alan:
What is the name of the shape that is 3D and irregular?
Answered by Stephen La Rocque and Penny Nom.
Circumscribing a golden cuboid with a sphere: surface areas 2007-06-14
From Ainslie:
A golden cuboid is defined as a rectangular prism whose length, width and height are in the ratio of phi : 1 : 1/phi.
Prove that the ratio of the Surface Areas of the golden cuboid to that of the sphere that circumscribes it is Phi : Pi.

Answered by Stephen La Rocque.
A system of linear equations 2007-06-14
From Alfredo:
A student club sponsored a jazz concert and charged $3 admission for students, $5 for faculty, and $8 for the general public. The total ticket sales amounted to $2542. Three times as many students bought tickets as faculty. The general public bought twice as many tickets as did the students. Set up the equations that determine how many tickets were sold to each group and solve.
Answered by Stephen La Rocque and Penny Nom.
Angles of depression 2007-06-13
From Phonda:
The pilot of a small private plane can look forward and see the control tower for a small airstrip. Beyond that is a large factory that is 3 milies from the airstrip. The angles of depression are 12.5 degrees and 4.8 degrees respectively. Find the airplane's altitude, to the nearest ten feet.
Answered by Stephen La Rocque.
A probability problem 2007-06-13
From Lina:
A point P is selected at random from the interior of the pentagon with vertices A = (0,2), B = (4,0), C = (2 π+1,0), D = (2 π+1,4), and E = (0,4). What is the probability that
Answered by Stephen La Rocque and Penny Nom.
The size of a lot 2007-06-13
From Larry:
This is driving me crazy, I know i learned this in college but that was so many years ago haha. I am trying to determine how many acres my property is. Unfortunately everywhere i look is like another language to me when seeing these formulas. My property is square shaped so starting from one side and just working around the dimensions are: 114 feet x 95.88 feet x 100 feet x 91.91 feet.
Answered by Penny Nom.
What is the size of the circle it would make? 2007-06-13
From ed:
i have a piece of sheet metal 56' 6" long what is the size of the circle it would make
Answered by Penny Nom.
Standard Deviation 2007-06-13
From Adrian:
If you are told that the mean salary of a certain group of workers is $30,000 with a standard deviation of $4000, what proportion of workers earn over $38,000? What proportion of workers earn less than $18,000? Assume the distribution of wages is normal.
Answered by Stephen La Rocque.
Percentage increase and decrease 2007-06-13
From Anna:
I just want to if the following formula is used for the computation of a PERCENTAGE DECREASE:

Old Value - New Value/ Old Value x 100%

My immediate superior is under the impression that the said formula works ONLY for the computation of a percentage INCREASE. He believes that the formula for a percentage DECREASE:

Old Value - New Value/ NEW Value x 100%

On this score, I want to know the correct formula for the computation of percentage decrease. Thank you so much.


Answered by Penny Nom.
Express a certain variable as a function of another variable 2007-06-12
From Gilligan:
The volume V of a right circular cone is V = (1/3)pi(r^2)(h). If the height is TWICE the radius, express the volume V as a function of r.
I see the phrase "express a certain variable as a function of another variable" a lot in math books. What exactly does that mean? Is there another way to say the same thing?

Answered by Stephen La Rocque.
A ten- foot long board is cut into three pieces 2007-06-12
From Cynthia:
How do I solve this problem?

A ten- foot long board is cut into three pieces. The second piece is half as long as the first. The third piece is 4 1/2 longer than the second. How long is the first?

Answered by Penny Nom.
Projectile motion equations 2007-06-12
From Dillon:
Ron throws a ball straight up with an initial speed of 60 feet per second from a height of 5 feet. Find parametric equations that describe the motion of the ball as a function of time. How long is the ball in the air? When is the ball at its maximum height? What is the maximum height of the ball?
Answered by Stephen La Rocque.
Converting dollars to gallons 2007-06-12
From Cynthia:
How do I solve this problem? If a family uses .20 per 100 gallon of water. How many gallons would $10.00 cost, divided amoung 3 people? Thanks, Cynthia
Answered by Stephen La Rocque.
What is his regular hourly rate? 2007-06-12
From Gilligan:
John Ritter, who is paid time-and-a-half for hours greater than 40, had a gross weekly wage of $442 for 48 hours. What is his regular hourly rate?
Answered by Penny Nom.
How high does the ladder reach? 2007-06-11
From Madi:
A ladder 8 ft long resting on a house makes a 60 degree angle with the ground. how far up the house does it reach?
Answered by Stephen La Rocque.
A sequence of circles 2007-06-11
From Ann:
Please help with solving the following problem!!! A circle is inscribed in an equilateral triangle with a side of length 2. Three circles are drawn externally tangent to this circle and internally tangent to 2 sides of the triangle. 3 more circles are drawn externally tantgent to these circles and internally tangent to 2 sides of the triangle. if this process continued forever, what would be the sum of the areas of all the circle? the answer 1 parent came up with was Pie over 2, but we don't know how he did it. Can you please show the work or explain the answer to this problem? Thank you Ann p s my daughter is in 9th grade math.
Answered by Steve La Rocque, Chris Fisher and Penny Nom.
A cord of wood 2007-06-11
From Wendi:
A cord of wood is equal to a stack of wood that is 8 feet long, 4 feet wide, and 4 feet high. How many cords of wood are in 1 cubic foot?
Answered by Stephen La Rocque.
The area of a quadrilateral 2007-06-10
From Lucy:
Calculate the area of the quadrilateral ABCD. AB= 4.1cm, BC = 7.6cm, AD= 5.4 cm, CD= ? Angle ABC = 117, Angle ADC = 62. Give your answer correct to 3 significant figures.
Answered by Stephen La Rocque and Penny Nom.
Rational polynomial expressions 2007-06-09
From a student:
I have a question that continues to lead me to the answer x/(x - 1) but according to my math book, the answer should be 1/(x - 1). Who is right? Who is wrong? Here is the question: (x^2 - 6x - 27)/(x^2 - 11x + 18) DIVIDED by (x^3 + 2x^2 - 3x)/(x^2 - 2x)
Answered by Stephen La Rocque.
The law of sines 2007-06-09
From Felicia:
A parallelogram has one side that is 12.0 cm and one angle that is 65°. The shorter diagonal is 25.0 cm. To the nearest tenth of a centimetre, how long is the other side of the parallelogram? Use the sine law.
Answered by Penny Nom.
What is the total weight of the fish? 2007-06-08
From Mike:
The tail of a fish weighs 9 oz. The head weighs as much as the tail and half of the body. The body weighs as much as the head and tail combined. What is the total weight of the fish?
Answered by Stephen La Rocque.
A pipe full of water 2007-06-07
From paul:
What is the volume of water in a 100mm inside diameter pipe that is 12.5 meters long What would be the pressure at the bottom if the pipe was installed in the vertical plain
Answered by Steve La Rocque, Penny Nom and Gabriel Potter.
Percentage below the monthly quota 2007-06-07
From Greg:
Sales quantities for the month were 12,834 with a monthly quota of 14,796. What is the percentage below the monthly quota?
Answered by Penny Nom.
Two tangent lines to y=x^3 2007-06-07
From stephanie:
find the equations of two tangent lines to the y=x^3 function through the point (2,8)
Answered by Penny Nom.
Yards of top soil 2007-06-06
From Cristina:
How many yards of top soil do I need in a area of 2000 square feet with 18" depth.
Answered by Penny Nom.
Rate of change of distance between the clock hands 2007-06-05
From Jonathan:
A certain Clock has a minute hand with a length of 4 inches long and an hour hand with a length of 3 inches long. How fast is the distance between tips of these hands changing at 9:00?
Answered by Stephen La Rocque.
Solving a quadratic equation using the Quadratic Formula 2007-06-05
From Stuart:
Solve the following quadratic equation: 3xsquared-5x-4=0
Answered by Stephen La Rocque.
The area of equilateral triangle 2007-06-05
From solomon:
describe the area of equilateral triangle in terms of its side? solve in two ways.
Answered by Stephen La Rocque.
Probability tree - two switches failing 2007-06-05
From Maura:
draw a probability to show the outcomes of two new switches (a) of both switches being faulty (b)both switches are not faulty (c) that one switch is faulty. The failure rate is 1/10.
Answered by Stephen La Rocque.
What happens when you have zero's on both sides? 2007-06-05
From Lily:
On the substitution method what happens when you have zero's on both sides of the equation? Is that considered no solution or infinitely many?
Answered by Stephen La Rocque and Penny Nom.
Finding the area of a lot without knowing the angles 2007-06-05
From Cristin:
I have a land-locked piece of land. I need to know the square footage in order to determine the acreage. My deed only gives the lengths of its five sides.
Answered by Stephen La Rocque.
A circle and polygon with equal perimeters 2007-06-05
From matthew:
Why does a circle have the largest area of any polygon when the perimeter is always 1000m for every polygon?
Answered by Stephen La Rocque and Penny Nom.
What angle does the tire turn in 60s? 2007-06-04
From San:
An automobile, travelling at 80km/h, is towing a trailer. The wheels of the trailer are 40cm in diameter. Through what angle does the tire turn in 60second?
Answered by Stephen La Rocque.
A flagpole and a telescope 2007-06-04
From Fabiola:
A telescope is mounted on a tripod 5 ft above the ground and 20 ft from a flagpole. The telescope must be rotated 48° from horizontal to see the top of the flagpole. How tall is the flagpole?
Answered by Stephen La Rocque.
Algebra - find the number 2007-06-04
From Fabiola:
When 4 times a number is increased by 40, the answer is the same as when100 is decreased by a number. Find the number
Answered by Stephen La Rocque.
A regular octagon is made up of 8 identical isosceles triangles 2007-06-04
From Alex:
A regular octagon is made up of 8 identical isosceles triangles. If the perimeterof each isosceles triangle and the hexagon are 58 cm and 168 com respectively, find the length of the identical sides of the triangle.
Answered by Stephen La Rocque.
Volume of a brewer's sphere 2007-06-04
From Edward:
A Brewer's Sphere that has a 9' Dia. Holds how much Brew? How many 40oz bottles will be needed to hold his batch of Brew?
Answered by Stephen La Rocque.
How many faces does a triangle have? 2007-06-04
From Daniela:
how many faces does a triangle have?
Answered by Stephen La Rocque.
A species of beetle has a 3yr life span 2007-06-03
From Gilligan:
A species of beetle has a 3yr life span. Half the females in their first year will survive into the second year the rate from second to 3rd year is 1/3. Each female breeds in the 3rd year of life, producing 6 females and 1 male. No females survive beyond the 3rd year.

Let xsub1,xsub2,xsub3 denote the number of females in the 1st, 2nd and 3rd year.

(a) What will be the number of females in the 1st, 2nd and 3rd years of life next year?

(b)Show that the numbers in each group will return to the same values every 3 years?

(c) Given a total of 3600 beetles, over the 3 age groups, calculate the number in each group?

Answered by Penny Nom.
How fast was he going? 2007-06-03
From Mary:
if a motorcycle left a 63 foot skid mark before hitting a car how fast was he going ?
Answered by Penny Nom.
Equation of a line 2007-06-03
From Mahrukh:
Hi i would like to know how to find the equation of a line, when you are given a piont (3, -1) and the x intercept which is -1?
Answered by Penny Nom.
The difference between two dice 2007-06-02
From smiley:
Two standards dice are rolled. Determine the probability that the difference between the two numbers on the dice is 2
Answered by Penny Nom.
Converting units 2007-06-01
From SAMANTHA:
My hot tub is 61 square feet, how many gallons would it be?
Answered by Stephen La Rocque.
2x + 3x = 30 2007-06-01
From Jessica:
2x+3x=30
Answered by Stephen La Rocque.
Points in a plane 2007-06-01
From Gabrielle:
In general how many planes are there which contain two given points, three given points, and four given points?
Answered by Steve La Rocque and Walter Whiteley.
Area of an isosceles triangle 2007-06-01
From Josh:
In a previous question answered by Sue regarding the area of a regular polygon you gave a formula for the area of an isosceles. My question is how did you get this formula? Can you please explain to mean the process that you used to get that formula? Thanks
Answered by Stephen La Rocque.
The volume of a triangular prism 2007-06-01
From Hannah:
How do I find the area of the base of a triangular prism, so that I can find the volume by using the equation on the attachment. The formula is that volume equals one half times (base times height) times triangle height. Thank you for your help. Hannah
Answered by Penny Nom.
Karats 2007-06-01
From Lisa:
What is the karat count of gold in a bracelet that contains 15g of gold and 5 g of silver?
Answered by Penny Nom.
A circle inside a square 2007-05-31
From Mer:
there is a circle inside a square... there is a shaded rectangle that measures 5mm X 10mm and they want to know what the radius is. The edge of the shaded rectangle touches a point on the circle.
Answered by Penny Nom.
The lcm of polynomials 2007-05-31
From devin:
x^3-8 , x^3-4, x^2+4x+4
Answered by Penny Nom.
Repeating decimals and rational numbers 2007-05-31
From lil:
Why are repeating decimals considered rational numbers?
Answered by Penny Nom and Gabriel Potter.
A 5 digit key 2007-05-31
From Michael:
I have a 5 digit key pad lock on my truck. I want to print out all of the combinations to try. pad one=0 pad two =1 pad three=2 pad four=3 pad five =4. can you help me find and print all combinations?
Answered by Harley Weston.
The number of grams of sulfuric acid 2007-05-31
From cookie:
Hi,
Can you please let me know if I've done this correctly?

Calculate the number of grams of sulfuric acid in 0.500L of battery acid if the solution has a density of 1.28 g/mL and is 38.1% sulfuric acid by mass.

Answered by Stephen La Rocque.
The intersection of two planes 2007-05-31
From Sarim:
How to find a intersection of two planes?
Answered by Penny Nom.
What is the highest known numerical value? 2007-05-29
From Peg:
What is the highest known numerical value?
Answered by Gabriel Potter.
Water tank volume 2007-05-29
From Cecelia:
I HAVE A SEMI CIRCULAR TANK AGAINST A WALL MEASURING 6'10" ACROSS THE WIDEST PART AND 3' FROM THE WALL TO THE FRONT OF THE TANK. THE DEPTH IS 14". PLEASE HELP ME TO CALCULATE THE VOLUME OF WATER IN THE TANK.
Answered by Stephen La Rocque.
Factor Completely 2007-05-29
From Ema:
Question 1:
Factor completely. 3x(x – 4) + 5(x – 4)
Question 2:
Factor completely. x^2 + 2x + 3

Answered by Penny Nom.
Fractions, decimals and profit 2007-05-29
From fatima:
how can i change a fraction into a decimal and how do i change a decimal into a fraction................
anther question please what does making a profit have to do with percentage change!!!!!!

Answered by Penny Nom.
Volumes of cones and cylinders 2007-05-29
From George:
1. The volume of a cylinder is 1353cm3
A) What is the volume of a cone with the same radius as the cylinder but double the hieght of cylinder?
B) What is the volume of a cone with the same height as the cylinder but with three times its radius?

Answered by Steve La Rocque and Walter Whiteley.
y=mx+b 2007-05-29
From molly:
Im having a problem with the y=mx+b for our homework we have a table that goes from 0 to 6 , 1 to 7 and 2 to 8 and we have to make a equation of that. Please help.
Answered by Penny Nom.
x/2+3=2 2007-05-28
From Ethel:
what is the formula and the answer to x/2+3=2?
Answered by Penny Nom.
A scale factor 2007-05-28
From Chandler:
I am rather confused about finding the area of a triangle they give me the area of the larger triangle it is 25 units but I can't figure out how to find the area of the smaller triangle the scale is 3:5 please help me ASAP. Thanks so much I really appreciate it oh the triangles are similar
Answered by Stephen La Rocque and Penny Nom.
The attendance at football games 2007-05-28
From cookie:
The attendance at football games at a certain stadium is normally distributed, with a mean of 55,000 and a standard deviation of 4,500.
a) What is the probability of the attendance exceeding 60,000?
b) What is the probability of the attendance being between 45,000 and 50,000?

Answered by Penny Nom.
The distance between parallel lines 2007-05-28
From Kristie:
I can't figure out how to determine the distance between parallel lines I am taking linear algebra and vector geometry. I was given two lines in the form #x+#y =# told they were parallel and asked to find the distance between them?? Please help Thanks
Answered by Penny Nom.
The limit of a rational function 2007-05-28
From Imad:
     3 _______ 3 _______
lim \/ 1 + x -    \/ 1 - x
x->0 ---- ----------------------------
              x

Answered by Penny Nom.
A point on a semicircle 2007-05-28
From arun:
a semi circle is drawn with ab as diameter from p a point on ab a line perpendicular to ab is drawn meeting circumference of semi circle at c, ac = 2cm, cd = 6cm find area of the semi circle?
Answered by Penny Nom.
A circular blob of molasses 2007-05-28
From Julie:
A circular blob of molasses of uniform thickness has a volume of 1 m^3. The thickness of the molasses is decreasing at a rate of 0.1 cm/hour. At what rate is the radius of the molasses increasing when the radius is 8 m?

Thanks,
Julia

Answered by Penny Nom.
I want to divide my land 2007-05-27
From alfredo:
i want to divide my land. It is north is 11.86 ft, the S is 110.00 ft, the W is 644.34 ft the E is 644.58 ft. which =to1.606 acre. how many feet going from N to S would it be to make 1.5 acre
Answered by Penny Nom.
The surface area of a fire hose 2007-05-26
From Vanessa:
Why do we need to find the surface area of a fire hose (cylinder).
Answered by Stephen La Rocque.
More on quadrilateral shape names 2007-05-26
From Don:
If North Americans call a quadrilateral with no parallel sides a trapezium, is a kite merely a special type of trapezium? Can a rhombus be a kite?
Answered by Walter Whiteley and Penny Nom.
What are the odds that the selections were random? 2007-05-26
From alan:
An employer must select three people for layoff from three groups. Group 1 has 4 members and the oldest member is selected. Group 2 has 7 members and the oldest member is selected. Group 3 has 2 members and the younger is selected. Assuming equal qualifications, what are the odds that the selections were random vs. biased as to age? What is the formula to determine this?
Answered by Penny Nom.
The circumference of an ellipse 2007-05-25
From Graham:
I have an ellipse that I need to find the circumference of. It is 40ft at its longest point and 25ft at its shortest
Answered by Penny Nom.
Discovering the incircle of an irregular polygon 2007-05-25
From Joaquim:
I've been searching in some books and many websites, but I couldn't find a formula or algorithm for discovering the incircle of an irregular polygon, could you please help me?
Answered by Walter Whiteley.
Area of a hexagon (three ways to find it) 2007-05-24
From Josh:
I have to work out the area of a hexagon when I know that the perimeter of it is 1000m. I have tried many different ways but I get different answers.
Answered by Stephen La Rocque.
Volume of a chest 2007-05-24
From Jerry:
Heighth: 28" Width: 32" Depth: 20"
Answered by Stephen La Rocque.
Simplifying a square root 2007-05-24
From Leigh:
I am trying to help my son. Simplify sq root 81 x to the 2nd power y to the 12th power.
Answered by Stephen La Rocque.
System of equations 2007-05-24
From Chris:
Find all real solutions (x,y,z,w) of the system of equations:
2y= x + x/17, 2z= y + y/17, 2w = z + z/17, 2x= w + w/17

Answered by Penny Nom.
Normal distribution 2007-05-24
From Paula:
Consider a data set that is normally distributed. The mean of the data set is equal to 10,000.
a.) Suppose that, for this data set, 10,625 has a "z-value" = 2.5. Solve for the standard deviation of the data set.
b.) Solve for the "z-value" of 9,900.

Answered by Penny Nom.
Solve for I 2007-05-23
From Mark:
A=4.9(B)*C^(.00925(D-.542))* E^(-.015(D))*E^(-.013(F))* (G/B)^(-.00825(D-.225))* H^-.877*(I/B)^.511*(J/B)^.319
Answered by Stephen La Rocque.
Mulch price comparison (price per cubic yard versus price per cubic foot) 2007-05-23
From Dianna:
This is a 2-fold question. I am getting some mulch that is measured in 1 cubic yard for $25.00. Walmart has a 2 cubic bag for $3.00. I need to cover a 376 sq ft area. I need to know which is the best deal and how much I need to buy to cover the 376 sq ft.
Answered by Stephen La Rocque.
How many tons of dirt is this? 2007-05-23
From Deb:
I need to know how many tons of dirt(6 inches deep)would it take to cover 8 acres?
Answered by Stephen La Rocque.
Factoring a trinomial in two variables 2007-05-23
From Ema:
How do you factor this completely. x^5y^3-3x^4y^2-28x^3y
Answered by Stephen La Rocque.
Algebra (Solve for B) 2007-05-23
From Mark:
A = .250 * (B^2*SIN(C)) / (D*COS^2(E)). I need to solve for B.
Answered by Stephen La Rocque.
Probability that a restaurant will succeed 2007-05-23
From Cookie:
You wish to open a new restaurant and are considering locations in Toronto and Hamilton, but only one location will actually become available. If it is built in Toronto, the restaurant stands an 80% chance of successfully surviving its first year. However, if it is built in Hamilton, its chance of survival falls to 65%. It is estimated that the chance of Hamilton being available is 60% and Toronto being available is 40%. Find the probability that the restaurant will :
a) survive its first year
b) be built in Toronto and survive its first year
c) be built in Hamilton, given that it survived its first year
d) not survive its first year, given that it is built in Toronto Thanks

Answered by Penny Nom.
Positive rational numbers 2007-05-22
From Rebecca:
What is the definition of positive rationals/positive rational numbers?
Answered by Stephen La Rocque.
Using a stem and leaf plot 2007-05-22
From Maegan:
The students in Mr.Smiths PE class collected data on the number of sit ups the students were able to do in three minutes. Use the data to create a stem and leaf plot. What is the median for the class data? range? mode?
Answered by Penny Nom.
Relative sizes of circles 2007-05-22
From Griselda:
There are 4 small circles inscribed in a bigger circle. The larger circle has a radius of 10 cm. Find the radius of the largest circle which will fit in the middle.
Answered by Stephen La Rocque.
Lot acreage 2007-05-21
From Sherri:
I have a lot that is 2565 x 1056 x 2636 x 285 that is roughly in the shape of a triangle, with the small strip on the top of the triangle, that I would like to figure the sq. footage for. Could I draw a line from one corner on the top to the opposite corner on the bottom and get the area of those two triangles, then add that together to come up with the sq. ft.?
Answered by Stephen La Rocque.
Converting yogurt 2007-05-21
From Annie:
I'm trying to follow a diet plan. The diet says I should have a 125ml tub of natural low fat bio yogurt. The only problem is, the yogurt pot lists its weight as 150g (with a large 'e' after it). Can you tell me how much of the yogurt pot I'm suppost to have please, or give me a conversion chart to change from g to ml for natural bio yogurt?
Answered by Stephen La Rocque.
Converting mass of sand to volume (a sand ceremony) 2007-05-21
From Linda:
I'm getting married. We are having a sand ceremony. The empty bottle we are using holds 800ml. I have bags of colored sand. Each bag is 30g. There is 7 people participating in this ceremony. What I need to find out is how many grams of sand i need for 7 people to participate to fill this jar.
Answered by Penny Nom.
Set up two simultaneous equations 2007-05-21
From Admire:
The cost of producing windscreen wipers blades at a factory ais partly fixed (due to operating overheads) and partly dependent on the number of blades produced. It costs $300 to produce 1000, and $600 to produce 5000 blades. How much would it cost to produce 24000 blades?
Answered by Penny Nom.
A push button lock 2007-05-21
From Werner:
I have a push button lock (the numbers entered does not have a start and end i.e. 1234 = 4321 or 2341 = 3241 ...)

So if it is a 4 digit code and you have 10 buttons (0 to 9) there should be 5040 (?) possibilities and how would I list these unique combinations

Answered by Penny Nom.
Constant rate of sand falling in a cone 2007-05-20
From Nhi:
Sand is falling into a conical pile . After 5 min. the pile has radius 24 and height 26 . After 7 min. tell how far the point c. is from the top of the cone (A).
Answered by Stephen La Rocque.
Total of 100 with 1, 2, 3, 4, 5, 6, 7, 8, 9 2007-05-20
From Tyler:
use all the numbers shown, any of the following four operations +,-,x, and division and parentheses to make a total of 100. You may not change the order of the numbers, or use them more than once. 1 2 3 4 5 6 7 8 9 Please help asap I can't figure this out. Thank you Tyler
Answered by Stephen La Rocque.
Areas of polygons 2007-05-20
From kendra:
i have questions on how to find the area of pentagons, hexagons, and nonagons. i was asked to find the area of all of them and they gave me either the length of one side or the perimeter.
Answered by Stephen La Rocque.
Probability tree 2007-05-19
From Patrick:
My question involves a video game called Guild Wars. In the game you can have modifications(mods) on items. One such mod is that you have a 20% chance to halve the casting time of a spell. You can have 2 of these mods available to you. Now, it is to my understaning that combined, you would have a 32% chance to halve the casting time and a 4% chance to quarter the casting time. Is this correct?
Answered by Stephen La Rocque.
Base seven 2007-05-19
From Dan:
A number X is converted to base 7 and becomes a four digit number. Its leftmost digit is removed and written again as the rightmost digit. The number thus obtained is twice X. Find the decimal representations of all such numbers X.
Answered by Penny Nom.
Parabolas in the real world 2007-05-18
From Katherine:
Hi, my name is Katherine, and my mean old math teacher just assigned us a test in which we have to write two examples of how parabolas are used in the real world, each one page single spaced, size 12!! I know you have already answered some questions like this, but I still don't understand the whole baseball thing, and any other way parabolas are used. And how I can write a whole page on it. But that's my problem, not yours, I just need help with a little explanation on how parabolas are actually used today. I know this might be kind of confusing for you, but imagine how it is for me!!
Answered by Stephen La Rocque.
Using FOIL for (2x - 6)^2 2007-05-18
From TONI:
I am having a problem finding examples of how to solve (2x-6)^2.
Answered by Stephen La Rocque.
Divisibility 2007-05-18
From Ashish:
A number is divisible by 2^n if the last n digits of the number are divisible by 2^n. Why?
Answered by Penny Nom and Claude Tardif.
(3x-2(x-3)+2)(3x-2(x-3)+2) 2007-05-17
From Jeff:
(3x-2(x-3)+2)(3x-2(x-3)+2)
Answered by Penny Nom.
Finding square roots 2007-05-17
From Chandler:
I would like an easy way to find the square root of a number.
Answered by Gabriel Potter.
Lateral area of a right cone 2007-05-17
From Crystal:
In my homework the question says the lateral area of a right cone is 226.08 cm cubed. the slant hieght is 12 cm. Find the total surface area. How do I do that?
Answered by Stephen La Rocque.
Hexagonal pyramids 2007-05-17
From Tracey:
How many faces, edges, and vertices does a hexagonal pyramid have?
Answered by Stephen La Rocque.
Altitudes of scalene triangles 2007-05-17
From Chris:
Two of the altitudes of a scalene triangle ABC have length 4 and 12. If the length of the third altitude is also an integer, what is the biggest that it can be?
Answered by J. Chris Fisher.
Cost of replacing a driveway 2007-05-17
From Chris:
A homeowner needs to replace a driveay which is 12 feet wide and 90 feet long. The old driveway needs to be dug out. It is 9 inches deep. It costs $35 per cubic yard to remove the old driveway and $75 per cubic yard to lay the new one. What will be the cost of the complete job?
Answered by Penny Nom.
Parabolas 2007-05-16
From Andy:
How do you write an equation of a parabola that opens to the left with a vertex of (2,-6)?
Answered by Stephen La Rocque.
Comparing the areas of various shapes 2007-05-16
From Kathy:
If the perimeters are the same, which has the greater area, a circle, a square, an ellipse, or an octagon?
Answered by Stephen La Rocque.
All the possible 8 digit combinations of 0-9 2007-05-16
From Christofer:
What are all the possible 8 digit combinations of 0-9? List please I know that there are 43046721 combinations but I want to know what they are.
Answered by Stephen La Rocque.
Buying topsoil 2007-05-16
From Claudia:
I am trying to figure out if it is cheaper to buy topsoil by the bag (one 40lbs bag for $1.66) or having it delivered for $60 for 4 cubic yards.
Answered by Penny Nom.
4 digit lock 2007-05-16
From Kathy:
Hi, My name is Kathy and I'm a REALTOR, I have 2 different lockboxes that I cannot remember the combinations for, so what I need is all the different combinations for 4 digits that cannot be the same, i.e. 0123. Can you help me out? I would hate to throw them away as they cost about $30 each. Thank you, Kathy
Answered by Stephen La Rocque.
Volume of a wedge 2007-05-14
From Mark:
We need to fill in dirt along a 75 foot long fence, 3 feet out , statring at 6 inches agaisnt the fence, sloping to 2 inches at the grass. I am trying to calculate the volume to order enough cubic yards of dirt to fill this.
Answered by Stephen La Rocque.
Maximizing the volume of a cone given the slant length 2007-05-14
From Christina:
A coffee filter for a new coffee maker is to be designed using a conical filter. The filter is to be made from a circle of radius 10cm with a sector cut from it such that the volume of coffee held in the filter is maximised. Determine the dimensions of the filter such that the volume is maximised.
Answered by Stephen La Rocque and Kerstin Voigt.
Weight per area conversions 2007-05-13
From Dermot:
A lawn weedkiller to be spread at the rate of 35g per Sq meter. My new spreader is calibrated in lbs per 1000 sq ft !!!!
Answered by Penny Nom and Stephen La Rocque.
Surface area of a cylinder 2007-05-13
From Nina:
if a cylinder is 23 ft high with a 12 ft radius, how do I find the surface area?
Answered by Penny Nom.
Null hypothesis 2007-05-13
From Dani:
A poll of 1068 American voters indicated that 48% supported te Democratic candidate for presidency. At the 0.05 level of significance, test the claim that less than 50% of American voters preferred the Democratic candidate.
Answered by Penny Nom.
Find the sample size needed 2007-05-13
From Mini:
Find the sample size needed to be 98% confident thata marketing survey on the proportion of shoppers who use the internet for holiday shopping is accurate within a margin of error of 0.02. Assume that the conditions for a binomial distribution are met, and that a current estimate for a sample proportion does not exist.
Answered by Penny Nom.
Interval of the domain 2007-05-13
From Gale:
What does the term interval of the domain mean?
Answered by Penny Nom and Stephen La Rocque.
Find the equation of a line passing through two given points 2007-05-12
From Kenzie:
I am having problems solving an equation of the line passing through (-2,4) and (6,0)
Answered by Penny Nom and Stephen La Rocque.
Pattern for a truncated cone 2007-05-11
From Mike:
I have been trying to get this cone flat so I can build this column. Can you please help me so I can figure this out? Thanks for your help.
Answered by Stephen La Rocque.
Linear growth 2007-05-11
From Rebecca:
What is the definition of linear growth?
Answered by Penny Nom.
Converting mass and volume of water 2007-05-10
From Shian:
Hello please help, I just can't seem to get the convertion thing and my mom tried to help but I still don't get it. How many grams does 1 milliliter of water weight? I was told 1g. How many grams does 0.1 liter of water weight? I was told 0.1g. Can you tell me if the answers given are correct with an explaination that I can understand? Thank you.
Answered by Stephen La Rocque.
Algebra problem 2007-05-10
From Sam:
What is the proper answer for this equation?
3x - 2 ( 5y (4x +2))

Answered by Stephen La Rocque.
Angle of Elevation 2007-05-10
From Micky:
Two Buildings are on opposite sides of a street 40 feet wide. The taller of the two buildings is 580 feet tall. The angle of depression from the top of the tallest building to the shorter building across the street is 57 degrees. Find the height of the shorter building.
Answered by Stephen La Rocque.
Pre-algebra volume of a prism 2007-05-10
From Brenda:
Find the volume of a triangular prism with a base of 5 cm by 5 cm and a height of 12.5 cm. Round to the nearest tenth.
Answered by Stephen La Rocque.
Understanding log equations 2007-05-10
From Sean:
The number, N, of people who have heard a rumor spread by mass media at time, t, in days is modelled by N (t) = a / 1+ be^ -kt. If 50 people have heard the rumour initially and 300,000 people hear the rumour eventually, find a and b. If the rumour is initially spreading at the rate of 500 people per day, find k.
Answered by Penny Nom.
Interest Rates 2007-05-09
From Julia:
Using the formula I=PRT to figure out the missing variable.
P=$9530, T=7 months, A= $9919.14

Answered by Stephen La Rocque.
What number am I? 2007-05-09
From austin:
i am a positive integer if im squared -5 is added to me. -10 is subtracted from me, and the remainder is divided by 2 the result is 7. what integer am i
Answered by Stephen La Rocque.
Area of a pentagonal lot 2007-05-09
From Mimi:
I would like to know the square footage of a pentagon with the following measurements: Property is .43 of an Acre Front: 57.11' NW: 105.78' S: 120' SE: 145' E: 102.86' I don't want you to try to show me how to work thru the problem since I will never comprehend it. For some reason, I am unable to perform these kinds of equations. I will greatly appreciate your help.
Answered by Penny Nom.
Area of irregular surfaces 2007-05-09
From Dustan:
I am working on a way to compute very accurate areas for irregular surfaces by using the idea of a largest possible circle...
Answered by Chris Fisher.
Find the real solutions in this trig equation 2007-05-09
From tony:
list all real solutions of the equation that are in the interval [0,2π)
2cos(x) + tan(x) = sec(x)

Answered by Penny Nom and Stephen La Rocque.
Cubic feet of sand 2007-05-09
From Paul:
I have an above ground pool that is 585 sq feet. I want to cover the bottom in 3 inches of sand. The sand supplier says I need about 6 cubic yards, based on your prior answers on this subject. I come up with 1.8 yards. What is the correct amount?
Answered by Stephen La Rocque.
Combinations 2007-05-09
From Michael:
Show that 5 X C(n,5) = n X C(n-1,4)
Answered by Stephen La Rocque.
Probability of getting an A 2007-05-09
From Christine:
In a class of 15 people, exactly 3 got an A. If 2 people are randomly chosen from this class, what is the probability that at least one of these 2 got an A?
Answered by Paul Betts.
Area of an equilateral triangle 2007-05-09
From Diana:
how do i find the area of an equilateral triangle when they give us the base?
Answered by Penny Nom.
Is the point on a cone called a vertex? 2007-05-09
From Felicia:
Does a cone have a vertice? My teacher says that a vertice can only be made if two or more edges join up at an angle, so what do you call a point on a cone?
Answered by Walter Whiteley.
Permutations, probability and standard normals 2007-05-09
From Katrina:
I have a few problems that i seem to be stuck on or can not start. Can you please help me ?
1) There are 20 people on an event planning committee. How many differnt ways can a chairperson and assistant to the chairperson be selected?
2) An unprepared student makes random guesses for 10 true or false questions on a quiz. Find the probability that the student passes the quiz by guessing 7 of the questions correctly.
3) The heights of 18 year old men are normally distributed with a mean of 68 inches and a standard deviation of 3 inches. If a random sample of 25 18-year old men is selected what is the probability that the mean height is between 68.5 and 72 inches?

Answered by Penny Nom.
A committee of 4 2007-05-09
From Danielle:
I was wondering: What is the probability that both Barb and Bob are both on the same committee of 4 which is chosen fom their class of 10 people? Is it 2/5? Thanks a bunch!
Answered by Penny Nom.
Who was the mathematician that united algebra and geometry? 2007-05-08
From Victor:
who was the mathematician that united algebra and geometry
Answered by Penny Nom.
Uses of pi 2007-05-08
From Kari:
How is pi used in real life?
Answered by Penny Nom.
The substitution method 2007-05-08
From A student:
I need help on these two problems( substitution method)

-2x+7y=10
x-3y= -3
and
6x-8y= -12
-3x+4y=9

Answered by Penny Nom.
Direct variation 2007-05-07
From Jennifer:
assume that y varies directly as x. find y when x = 54 if y =1/4 when x=3/2
Answered by Stephen La Rocque.
Probability of H.I.V. tests 2007-05-07
From Danielle:
A medical test detects H.I.V. Among those who have H.I.V., the test will detect the disease with probability 0.95; among those who do NOT have H.I.V., the test will falsely claim that H.I.V. is present with probability 0.0125. Among those who take this test, 4% have H.I.V. The test is given to Lucille, and indicates that she has H.I.V. What is the probability that Lucille actually has H.I.V.?
Answered by Stephen La Rocque.
Probability 2007-05-07
From Danielle:
A box contains 3 defective lights and 5 non-defective lights. The lights are tested one at a time without replacement. What is the probability that the 3 defective bulbs will be found in the first 3 tests? is it 1/56 by chance?
Answered by Stephen La Rocque.
Area of region between circle and inscribed octagon 2007-05-07
From amy:
I have to find the area of the shaded region where there is an octagon inscribed in a circle The radius is 4 inches. The shaded region is everything besides the octagon inside the circle. How can I find the area of the shaded region? Thank you!
Answered by Stephen La Rocque.
How many sales does she need? 2007-05-07
From Jimi:
The new businessperson hopes to earn $50,000 per year. The profit percentage for this type of business is typically 10%. How much must sales be?
Answered by Stephen La Rocque.
Slicing a double-napped cone with a plane 2007-05-07
From Andrew:
I am writing a paper about creating parallel lines by slicing a double-napped cone with a plane. I have found out how it can be shown by algebra that the equations for parallel lines are generated from the degenerate case of a second degree polynomial in two variables, but I have yet to find a source with a visual representation of this case. Do you know if it exists?
Answered by Chris Fisher.
The radius of a cone 2007-05-07
From Braden:
i need to find the surface area but i only have the slant height and the height i need the radius how do i find it?
Answered by Penny Nom.
The dimensions of a rectangle 2007-05-07
From Stephanie:
The perimeter of a rectangle is 54yds , and the area of the rectangle is 180yds^2 . Find the dimensions of the rectangle.
Answered by Penny Nom.
Road incline grades 2007-05-07
From cecil:
convert grade percent to degrees
Answered by Stephen La Rocque.
The circumference of part of a circle 2007-05-06
From Wallis:
I have more then a quater of a circle, but less than half. What is the circumference if the two side are 7cm and the angle between them is 110 degrees?
Answered by Penny Nom.
Optimization - carrying a pipe 2007-05-05
From A student:
A steel pipe is taken to a 9ft wide corridor. At the end of the corridor there is a 90° turn, to a 6ft wide corridor. How long is the longest pipe than can be turned in this corner?
Answered by Stephen La Rocque.
The sum of the ages of petra and her mother is 51 2007-05-05
From Natakie:
the sum of the ages of petra and her mother is 51. Her mother is 12 years more than twice as old as petra. how old are petra and her mother.
Answered by Stephen La Rocque.
A speaker box 2007-05-05
From darell:
I am trying to build a speaker box, and i am trying to find out the measurements of a 1.5cubic feet box.
Answered by Penny Nom.
Lottery numbers 2007-05-04
From Trish:
What are all the six digit combinations between 1 & 36
Answered by Stephen La Rocque.
A point inside a circle 2007-05-04
From edgemitter:
There is a point inside a circle (but not in its center) where two perpendicular secants intersect, dividing the circle into four regions with different area. Calculate the area of the four regions
Answered by Walter Whiteley.
Square footage of a lot 2007-05-03
From Linda:
I have two lots which I am trying to calcuate the square footage of

one lot is 50 x 285.72 x 50 x 270

2nd lot is 50 x 285.72 x 50 x 277.61

Any help would be appreciated. The answers would be GREATLY appreciated.

Answered by Penny Nom.
Comparing two pay scales 2007-05-03
From san:
Mable is offered a job selling magazine subscriptions. She has the choice of two pay scales. Pay scale 1:
She can be paid $0.65 for each subscription she sells.
Pay scale 2:
She can be paid $0.10 for the first subscription, with the wage gong up $0.05 more for each subscription after the first.
For her first sale she would make $0.10, for her second sale she would make $0.15, for her third sale she would make $0.20, and so on.
Please compare and analyze the two scales. Which scale is better?

Answered by Penny Nom.
Edging surrounding a round pool 2007-05-03
From Carol:
Hello, I am new at this and very rusty on my math. I am getting a 24 Ft. round pool and would like to put 2ft width stone (small) edging around it. How much would I need to buy. I have to buy it by the yard (cubic yard) I am not looking for an exact, just an approx. even would be great. Thanks Carol
Answered by Stephen La Rocque.
Reverse Calculation of GST and PST in Quebec 2007-05-03
From Derek:
Could you please tell me how to figure out the PST and GST paid on an item. Example: I paid $10 total (including taxes) and the GST is 6% and the PST is 7.5%, HOWEVER, in Quebec, we pay the PST on the sub-total+GST. Thanks, Derek
Answered by Stephen La Rocque.
Continuity of y = |x| 2007-05-02
From moulipriya:
Is the curve y = | x | continuous everywhere?
Answered by Penny Nom.
The vertices of a triangle 2007-05-02
From natakie:
Find the coordinate of the vertices of a triangle whose sides lie on the following three lines. 2x+5y-16=0, 4x-3y-6=0 and 3x+y+2=0
Answered by Penny Nom.
The height of a triangle 2007-05-02
From Cassie:
If you have a scalene triangle with sides measuring 8 units, 6 units, and 10 units (and 10 is the base) how do i find the height perpendicular to the base of 10?
Answered by Penny Nom.
Two concentric circles form an annulus 2007-05-02
From A student:
In the diagram below, two concentric circles form an annulus. The vertical line is tangent to the inner circle, and forms the diameter of a third circle.

Explain why the areas of the annulus and third circle are the same.

Answered by Penny Nom.
Simplify -36x squared over 18x 2007-05-01
From Emily:
Simplify -36x squared over 18x
Answered by Penny Nom.
Angular velocity 2007-05-01
From Bruce:
a tachometer of a Ford Explorer reads 2,100 rpm at 60 mph. Find the equivalent angular velocity in degrees per minute and radians per minute
Answered by Stephen La Rocque.
A tugboat's speed 2007-04-30
From Amanda:
a tugboat must travel 24 miles against a 4 mile per hour current on the Potomac River and return. At what constant speed must the tugboat travel to make the trip in 12 hours. Round answer to the nearest tenth mph.
Answered by Stephen La Rocque.
What is 2x + 1? 2007-04-30
From Cynthia:
Hi, How do I solve this problem? If -1/3x + 7= 16, what is the value of 2x+1 Thanks, Cynthia
Answered by Stephen La Rocque.
Finding a parabola's equation by looking at its graph 2007-04-30
From Kenzie:
The graph shows an arrow going upward crossing at the -2 on the x line and crossing the 3 on the x line and the vertex on the -6 on the y line.
Answered by Stephen La Rocque.
LCD 2007-04-29
From Jackson:
I am having trouble getting the LCD for the following question. 1 3/8 x 8/9=
Answered by Penny Nom and Melanie Tyrer.
Simplifying a rational expression 2007-04-29
From Tamika:
X^2 + 10X + 24
______________
X + 6
SIMPLIFY EACH RATIONAL EXPRESSION (IF YOU CAN'T SIMPLIFY IT, WRITE ALREADY SIMPLIFIED)

Answered by Stephen La Rocque.
Two equations 2007-04-29
From lavon:
I'm having trouble figuring out a linear equation, can you help?

- 3x + 2y = 8
3x + 2y = 8

Answered by Penny Nom.
Finding out if two line equations are parallel or perpendicular 2007-04-28
From Vincent:
Can you determine if the two lines are parallel, perpendicular, or neither 1/2x+4y=3/5 y/1 + 10/5= 8x I think there perpendicular but how would you solve the equations
Answered by Stephen La Rocque.
The grade of a shoulder 2007-04-28
From Robert:
I am building a road with a 1.2 meter shoulder. The plan calls fo a 6% shoulder grade. When I use my metric calculator I come up with a different answer when I multiply 3/4 of an inch by 1.2 meters and when I multiply 1.2 meters by .06%?
Answered by Penny Nom.
A line parallel to a given line 2007-04-28
From vince:
write an equation of the line satisfying the given conditions Through (2,-3) parallel to 2x=3y+5 and graph both equations
Answered by Penny Nom.
Maximize the volume of a cone 2007-04-27
From ashley:
hello, I've been stumped for hours on this problem and can't quite figure it out. The question is: A tepee is a cone-shaped shelter with no bottom. Suppose you have 200 square feet of canvas (shaped however you like) to make a tepee. Use calculus to find the height and radius of such a tepee that encloses the biggest volume. Can you help??
Answered by Stephen La Rocque and Penny Nom.
A line parallel to a given line 2007-04-26
From vincent:
I forgot how to write the equation of the line satisfying the given conditions through (2,-3) parallel to 2x=3y+5

and how would you graph this

Answered by Penny Nom.
(1+i)^(2-i) 2007-04-26
From Eilis:
How do I solve (1+i) to the power of 2-i?

i.e. (1+i)^(2-i)

Answered by Penny Nom.
Complementary and supplementary angles 2007-04-26
From Tracey:
I have looked at your answers for students asking if groups of more than two angles can be considered either complementary or supplementary. Your answer is basically "no" because of historic definition. However, I present to you the following case to consider:

Segments AB, CD and EF intersect at point G creating 6 angles numbered 1-6 in a clockwise manner. If Angle 1=25 degrees, and angle 2 = 106 degrees, would the only way to calculate the measures of angles 3 and 6 not be to consider the definition of supplementary angles? And, if one was to be doing a proof of this, would not the reason be "definition of supplementary angles"?

This, then, creates a group of 3 angles that are supplementary.

Help me correct my logic if it is flawed.

Answered by Penny Nom.
The vertices of a hexagonal pyramid 2007-04-26
From Madeline:
How many vertices does a hexagonal pyramid have??
Answered by Penny Nom.
Two parallel lines 2007-04-25
From prasanna:
Let me know the Equation of a line which is parallel to some other line at a distance of Dmts.
Answered by Penny Nom.
A cylinder inside a sphere 2007-04-25
From Louise:
i need to find the maximum volume of a cylinder that can fit inside a sphere of diamter 16cm
Answered by Penny Nom.
A linear system 2007-04-25
From Tee:
2x -y =5
2x +=y =5

and also
3/7x + 5/9y =27 (fractions)

I have to find the addition method to both problems

Answered by Penny Nom.
How many tons of sand is needed 2007-04-25
From Jennifer:
How many tons of sand is needed to cover 350 cubic square feet?
Answered by Penny Nom.
What is the square root of 729 to the third power? 2007-04-25
From Tori:
What is the square root of 729 to the third power?
Answered by Penny Nom.
Linear programming 2007-04-24
From Sylvia:
What is graphing linear programming?
Answered by Penny Nom.
A triangle problem 2007-04-23
From Dimitri:
In the triangle ABC, M is the midpoint of AB and N is the midpoint of AC. Prove that CM and BN cannot bisect. (I can prove it by contradiction if it is a scalene triangle, but i cant seem to prove it for isosceles)
Answered by Chris Fisher.
How much water is in the pool 2007-04-23
From Kayla:
how much water is in a pool which is 10m x 5m and is 1.5m to 2.5m in depth?
Answered by Penny Nom.
A growing heap of sand: related rates 2007-04-23
From Charles:
Sand falls on to a horizontal ground at the rate of 9m ^ 3 per second and forms a heap in the shape of a right circular cone with vertical angle 60. Show that 10 seconds after the sand begins to fall, the rate at which the radius of the pile is increasing is 3 ^ (1/3) * (4/pi) ^ (1/3) m per minute.
Answered by Stephen La Rocque and Penny Nom.
How can I find out how many sides there are? 2007-04-23
From gn:
I know the angle measurements of a convex polygon. They are congruent angles. How can I find out how many sides there are?
Answered by Steve La Rocque, Penny Nom and Walter Whiteley.
The size of a freezer 2007-04-23
From Jacqui:
How many can packages can fit? I have a freezer that is 7cf. But it contains a little tray component. The freezer measurements without the tray is(bear with me, I'm not good with LxWxD) 23 across x 16 W x 25 deep (I'm assuming depth) and the tray is 8 across x 16 w x 15 D. We are trying to see how many 9 L x 6 W x 3.5 D packages can fit in the freezer. Please help me. I'm getting app. 60. I need 84 to be able to fit.
Answered by Penny Nom.
A bag of mulch 2007-04-22
From John:
I have a bag of mulch that contains two cubic feet of mulch. If I speard it out at a height of one inch, in a three foot wide flower bed - how feet can I cover?
Answered by Stephen La Rocque.
What is the speed of the boat? 2007-04-22
From Sonja:
A boat on a river travels downstream between two points, 20 miles apart, in one hour. The return trip against the current takes 2 1/2 hours. What is the speed of the boat? The answer in the book says 14 mi/hr, but I don't understand how to figure it out. It must be very simple, but I'm stuck. Thank you so much for helping me out. This is a FANTASTIC website! -Sonja
Answered by Penny Nom.
A square contains five circles with the same radius. 2007-04-21
From Jamie:
A square has a side length on 1 m. The square contains five circles with the same radius. The centre of one circle is at the centre of the square and it touches the other four circles. Each of the other four circles touches two sides of the square and the center circle. Find the radius.
Answered by Penny Nom.
A corny problem 2007-04-21
From Robby:
My name is Robby and I need help with a problem. I have found at least 4 correct answers, but I can't figure out how I got the answer or if there is a formula that I can use. I got my answers by trial and error. Problem: One hundred bushels of corn are to be divided among 100 men, women, and children. Men get 3 bushels each. Women get two bushels each. Children get 1/2 bushel each. How can the bushels be distributed with no leftovers? Is there more than one solution? So far, I have 4 answers: 8 men,20 women, and 72 children, 2 men, 30 women, and 68 children, 5 men, 25 women, and 70 children, and 11 men, l5 women, and 74 children. I need to know if there is a formula to use, or if it's just trial and error, or if a pattern is involved.
Answered by Stephen La Rocque and Penny Nom.
Constructing an octagonal deck around a circular pool 2007-04-20
From Cliff:
[I am building an] octagonal desk encompassing 17 foot diameter circle for pool. I have seen other octagonal calculations but none of these tell me how much allowance for a circle to fit within the octagon without losing the circle edge can anyone help thanks cliff
Answered by Stephen La Rocque.
Two lines intersect at a right angle 2007-04-20
From ellen:
The line with the equation 3x + by = 6 intersects with the line 6y + ax = c at right angles at the point (4,6). Determine the values of a, b & c.
Answered by Stephen La Rocque.
A normal distributiion question 2007-04-20
From Erika:
The amount of time required for a certain type of automobile transmission repair at a service garage is normally distributed with the mean = 45 minutes and the standard deviation =8.0 minutes. The service manager plans to have work begin on the transmission of a customer’s car 10 minutes after the car is dropped off, and he tells the customer that the car will be ready within one hour total time. What is the probability that he will be wrong? Illustrate the proportion of area under the normal curve which is relevant in this case.

What is the required working time allotment such that there is a 75 percent chance that the transmission repair will be completed withing that time? Illustrate the proportion of area that is relevant.

Answered by Penny Nom.
Relating volume to depth in a cylindrical container 2007-04-19
From Dan:
I have a cylindrical polyethylene rain barrel 23" in diameter and 35" high. If I've calculated correctly thus far, its volume is:

14, 547.49 cubic inches (or 8.418686 cubic feet or 0.31 cubic yards)

and could, ideally, contain 62.6 gallons of rain water whose weight would be 525.32 lbs. (62.4 lbs./cubic foot X 8.418686).

I'm having difficulty relating inch depth markings on the side of the barrel to volume capacity; i.e., how many inches in depth in a cylinder of the size given would = how many gallons (or quarts) of water. The barrel stands upright on its end and is, thus, 35" high when oriented in this fashion, much as a 55-gallon oil drum appears when similarly stowed. Dan Watley

Answered by Stephen La Rocque.
Two concentric circles 2007-04-19
From James:
Two concentric circles have a chord running through the outer one. The chord is the tangent of the inner circle and is 14 cm.The outer circle is shaded and the inner circle is not. Find the exact area of the shaded region without using a calculator.
Answered by Stephen La Rocque.
Four semi-circles are drawn inside a square 2007-04-19
From James:
Four semi-circles are drawn inside a square, with the diameter being the length of the square.The overlapping portion of the semicircles are shaded. What fraction is shaded?
Answered by Penny Nom.
Liquid is being poured into the top of a funnel 2007-04-19
From neroshan:
Liquid is being poured into the top of a funnel at a steady rate of 200cm^3/s. The funnel is in the shape of an inverted right circular cone with a radius equal to its height. It has a small hole at the bottom where the liquid is flowing out at a rate of 20 cm^3/s. How fast is the height of the liquid changing when the liquid in the funnel is 15 cm deep? At the instant when the height of the liquid is 25cm, the funnel becomes clogged at the bottom and no more liquid flows out. How fast does the height of the liquid change just after this occurs?
Answered by Penny Nom.
All samples of size 3 2007-04-19
From Liz:
Consider the population of the first seven integers: 1, 2, 3, 4, 5, 6, and 7; N=7. For this population, mean = 2 and standard deviation = 2.
a. How many samples of size three can be extracted from this population (sampling without replacement)?
b. Form the complete set of samples of size three and for each sample, compute the sample mean and median.

Answered by Penny Nom.
A scale factor 2007-04-18
From QueenB:
Lindy is Planning To Build a Model Train Using A Scale Where 2 inches represents 25 feet. If the train is 60 feet long, what is the length in inches that lindy should Build The Model Of the train?
Answered by Stephen La Rocque.
I am looking for a triangle joke 2007-04-18
From deborah:
Hi there, I just found your website. Wow!
I am presenting a triangle presentation on Friday at school and I am looking for a triangle joke to open up the presentation.
Do you have one to share with me? I'm discussing types of triangles if that helps.

Answered by Stephen La Rocque.
How many different ways can she purchase candy? 2007-04-18
From fathia:
Stephanie can buy lemon candies for 3 cents each and chocolate candies for 5 cents each. Stephanie has 64 cents to spend. How many different ways can she purchase candy if she wants to spend all her money?
Answered by Stephen La Rocque.
-12/(7 - i) 2007-04-18
From Diana:
Perform the operation. Write all answers in a + bi form.

-12
---------
7 - radical -1

Answered by Penny Nom.
Ax + By = C 2007-04-18
From Diana:
find an equation in general form Ax + By = C with the following properties passing through points (a,b) and (2a, 2b)
Answered by Penny Nom.
By how many feet was each dimension increased? 2007-04-18
From Becca:
A rectangle is 8 feet long and 6 feet wide. If each dimension is increased by the same number of feet, the area off the new rectangle formed is 32 square feet more than the area of the original rectangle. By how many feet was each dimension increased? I need a diagram and 5 or more sentences for an explanation. Thanks!
Answered by Stephen La Rocque.
Radical form 2007-04-18
From Cassia:
Hi Um, I had a question for algebra, I'm just curious of how to put x^3/8y^1/4 in radical form. Can you help me?
Answered by Stephen La Rocque.
A box-and-whisker plot with no whiskers 2007-04-18
From Paula:
Is it possible for a box-and-whisker plot to have no whiskers? 3 whiskers?
Answered by Penny Nom.
Minimum cost for a fixed volume 2007-04-18
From James:
My question goes: A silo is to be constructed and surmounted by a hemisphere. The material of the hemisphere cost twice as much as the walls of the silo. Determine the dimensions to be used of cost is to be kept to a minimum and the volume is fixed.
Answered by Penny Nom.
The area of a regular octagon 2007-04-17
From Adam:
I need to find the area of a regular octagon with side length of 12.5m
Answered by Penny Nom.
.07 = (1 + x)/(1+.12) - 1 2007-04-17
From Kathy:
How would you solve

.07 = (1 + x)/(1+.12) - 1

Answered by Stephen La Rocque.
Lowest terms 2007-04-17
From Dawn:
For some reason, I have a hard time with Ratio and/or rate problems. My problem is as follow: Write each rate or ratio as a fraction in lowest terms. 6 days to 39 hours
Answered by Stephen La Rocque.
Write each rate or ratio as a fraction in lowest terms 2007-04-17
From Dawn:
Write each rate or ratio as a fraction in lowest terms. $59.00 for 231 minutes
Answered by Stephen La Rocque.
Two tangents to a circle 2007-04-17
From Doug:
Two distinct, nonparallel lines are tangen to a circle. The measurement of the angle between the two lines is 54 degrees (angle QVP). Suppose the diameter of the circle is 2 cm. What is the distance VP? Suppose the distance VP is 3.93 cm. What is the diameter of the circle? Find a formula for d, the diameter of the circle, in terms of VP. Find a formula for VP in terms of d, the diameter of the circle.
Answered by Stephen La Rocque.
Exponential form 2007-04-16
From Cassia:
I was just wondering, how do you write the squared root of 7x(to the 5th) in exponential form? If you could help explain that I'd be grateful. Thanks
Answered by Brennan Yaremko.
Algebra 2007-04-16
From Eugene:
X over 2, -3 =0 ?
Answered by Haley Ess.
Fractions! Fractions! Fractions! 2007-04-16
From Maria:
Why are fractions important for:
1) The study of Mathematics
2) Real life

Answered by Haley Ess.
A gardener has 140 ft of fencing to fence 2007-04-16
From Johann:
A gardener has 140 ft of fencing to fence in a rectangular shaped vegetable garden. To plant all his vegetables, the gardener needs more than 825 sq ft of space. What are the possibles values for the length of the rectangle that would meet the space requirements? You may assume that the variable x represents the length of the rectangle.
Answered by Stephen La Rocque.
Simultaneous equations 2007-04-16
From kyrie:
simultaneous equation 4x + 3y = 21 2x * y = 8
Answered by Penny Nom.
Ounces and liters 2007-04-16
From Simone:
I want to know how much is 80 ounces of water in liters?
Answered by Penny Nom.
the sum form r = 0 to r = m of C(n+r,n)=? 2007-04-15
From Aastha:
FIND THE VALUE OF:- the sum form r = 0 to r = m of C(n+r,n)=?
Answered by Penny Nom.
Calculating interest 2007-04-15
From Gerry:
Calculate the amount of interest received on £120.00 desposited in an account paying 3% simple interest for three years.
Answered by Stephen La Rocque.
The second derivative 2007-04-14
From Gerry:
In mathematical context,what do you understand by the term "Second Derivative"
Answered by Penny Nom.
The area of a regular hexagon 2007-04-14
From Chantel:
Could you please explain to me why the formula for a regular hexagon is 3√3/2 multiplied by side²? Thank you very much.
Answered by Stephen La Rocque.
The area of an irregular pentagon 2007-04-14
From Steven:
I have been given a coursework in which i have to investigate various shapes to find the one that would give the largest area with 1000m perimeter. I'm stuck in a part where i have to calculate the area of a irregular pentagon. i don't know where to start from. I can make up the length of all the sides (5 sides), so long they add up to 1000, but i'm not quite sure what to do after that.
Answered by Penny Nom.
One number is 12 more than another. 2007-04-14
From Devon:
One number is 12 more than another. The sum of the smaller number and twice the larger number is 39. Find the larger number.
Answered by Penny Nom.
Division of polynomials 2007-04-14
From Rajendra:
Find the remainder when x3+3x+1 is divided by x+pi
Answered by Penny Nom.
The angle of depression 2007-04-14
From Mary-Beth:
Two towers are 30 m apart. From the 15th floor, 40 m up, find the angle of depression to the base of the taller tower? I think the answer is 53, but the answer book says 37.
Answered by Penny Nom.
Word problems 2007-04-13
From Perla:
1.A school district is proposing a 5% increase in the numbber of days in a school year. How long would a school year be with the proposed increase?

2. Keikio spent the day bird watching and counted 34 more birds in the morning than in the afternoon. If she counted a total of 76 birds, how many birds did she count in the afternoon?

Answered by Penny Nom.
Intersection of a line and a circle 2007-04-12
From gaby:
The sum of two numbers is 9. The sum of the squares of the two numbers is 41. Find the numbers.
Answered by Steve La Rocque and Melanie Tyrer.
A wooden hexagonal pyramid 2007-04-12
From David:
Im trying to form a hexagonal pyramid out of plywood and need to know the height to the center point of the pyramid. I want to use standard 4x8 sheets of plywood and split them diagonally. Then with the resulting triangles combine them to form one triangle. Then i want to accumulate 6 of these triangle total and put together to form a hexagonal pyramid.
Answered by Stephen La Rocque.
An arc shaped groove into a peice of metal 2007-04-12
From daniel:
hello i work at an engineering workshop the other night i was asked to machine an arc shaped groove into a piece of metal the cord length was 6 mm and the height from the middle of the cord to the arc was 1mm i was hoping to find the diameter of the cutter needed to do the job and also the formula to work out how to find the diameter. i believe it is 10mm dia thankyou for your time and knowledge
Answered by Stephen La Rocque.
Swimming pool water 2007-04-12
From tina:
How much water does a 24' round, 4' deep swimming pool hold?
Answered by Haley Ess.
Solving Quadratic-Liner Equations algebraically 2007-04-12
From Hector:
Solving Quadratic-Linear Equations algebraically.
y=x^2-4x+3
y=x-1

Answered by Stephen La Rocque.
6 purple and 4 orange socks 2007-04-12
From Reane:
^ purple and 4 orange socks. Choose one then choose a second without putting the 1st back. What is the probability of getting 2 purple socks. I can't reach back far enough in my memory to recall how to do this.
Answered by Gabriel Potter.
A 13 sided figure 2007-04-12
From Kiara:
I was wondering what you call a 13 sided figure. I'm in 6th grade and my teacher said i could get extra credit if I figure this out. Thanks a lot for your help.When I get extra credit I will be sure to mention you guys! Thanks!
Answered by Stephen La Rocque.
Reducing the over all size of a irregular pentagon 2007-04-11
From Gilbert:
I'm having trouble figuring out how to reduce the over all size of a irregular pentagon. The pentagon I'm trying to reduce has 2 right angles adjacent to each other. Basically the shape of a house. I have width, length of the legs and the height of the peak. But cannot figure out how to get the size of a reduced version that keeps the same proportion and angles. Thanks Gilbert
Answered by Penny Nom.
The volume of a cylinder 2007-04-11
From Gonzalez:
hi, I have a problem, i need the formula to find the volume, area, surface area. I have found that the are is pi X r^2, but i don't know how to find radius. if you could help me, I'd be very happy.
Answered by Penny Nom.
What is the limit of 3.x^(3/x) as x approaches +infinity? 2007-04-11
From Teodora:
What is the limit of 3.x^3/x as x approaches +infinity ?
Answered by Haley Ess.
I am filling a hole in the shape of a cube 2007-04-11
From Bethany:
If I am filling a hole in the shape of a cube with 64 cubic yards of dirt, how many feet deep will my hole be.
Answered by Stephen La Rocque.
Determine the probability that the triangle is acute. 2007-04-10
From Greg:
Points A1, A2, . . ., AN are equally spaced around the circumference of a circle and N >=3. Three of these points are selected at random and a triangle is formed using these points as its vertices. If N = 2k for some positive integer k >= 2, determine the probability that the triangle is acute.
Answered by Penny Nom.
Almost surely 2007-04-10
From Ben:
what does "almost surely" mean in probability? what's the diff between absolutely certain and "almost surely"? thx.
Answered by Penny Nom.
I have an isosceles triangle. 2007-04-10
From Stephanie:
I have an isosceles triangle. The two equal sides are given to me, but not the base. The equal sides are 12. I have to find the base of the triangle. Help!!
Answered by Stephen La Rocque.
Is zero a number or a place holder? 2007-04-09
From Chris:
Is zero a number or a place holder?
Answered by Penny Nom.
No solution to a linear system 2007-04-09
From Ryan:
When is it possible to get no solution when solving a linear system?
Answered by Leeanne Boehm, Steve La Rocque, Penny Nom and Melanie Tyrer.
Water is being pumped into a trough 2007-04-09
From Michael:
Water is being pumped into a trough that is 4.5m long and has a cross section in the shape of an equilateral triangle 1.5m on a side. If the rate of inflow is 2 cubic meters per minute how fast is the water level rising when the water is 0.5m deep?
Answered by Stephen La Rocque.
A hypothesis test 2007-04-09
From Katrina:
I have already tried to do this problem but im having a very had time with it. Can you please help me.

Glamour Magazine sponsored a survey of 2500 prospective brides and found that 60% of them spent less than $750 on their wedding gown. Use a 0.01 significance level to test the claim that less than 62% of brides spend less than $750 on their wedding gown. How are the results affected if it is learned that the responses were obtained from magazine readers who decided to respond to the survey through an Internet Web site?

Answered by Penny Nom.
Yards of sand in a ton 2007-04-09
From Teresa:
how many yards of sand are in a ton and how many yards of limestone does it take to make a ton
Answered by Penny Nom.
Word problems 2007-04-09
From Danielle:
I am teaching a student who cannot understand which operation to use when doing word problems. I have tried a number of different ways to show him...any suggestions?
Answered by Haley Ess.
A cabinet with an arched front 2007-04-09
From Joe:
I am building an arched front cabinet that is 71 inches wide, 12 inches deep at both ends and 16 inches deep at the center. To accurately build this cabinet I need to known the radius of the circle that would form that arch. Thanks, Joe
Answered by Penny Nom.
Logarithms 2007-04-09
From Emi:
Given log10 2=0.30 and log10 3=0.47 How to i express 6 x 10power 14 in logarithm answer. Please help me. Thank you. Emi
Answered by Stephen La Rocque.
Area of circles within a circle 2007-04-08
From Avaline:
Imagine that there are four small circles inscribed in a bigger circle. The 4 small circles are shaded. What is the ratio of the area of the shaded region to the area of the unshaded region?
Answered by Stephen La Rocque.
A volume of revolution 2007-04-08
From christina:
find the volume of the solid formed when region bounded by y=x/3, y=2 and the y-axis. it is revolved about the x-axis.

the assignment was to use both the washer method and the shell method but when i solved for the volume, i got different answers. i think my shell method is wrong because i know i'm having difficulties with using "dy" instead of "dx" here's my work so far:

Answered by Penny Nom.
An arched opening for a large doorway 2007-04-08
From Richard:
I am trying to build an arched opening for a large doorway...I know the vertical sides of the opening to be 8'-9" from the floor to the lowest point of the arch on each side...I know it is 15 1/2" from the center horizontal point to the top of the arc...I know the vertical sides are 11'-11" apart...what I need to know is the radius to create the proper arc. Can you help?
Answered by Stephen La Rocque.
Find the volume of the solid 2007-04-07
From tricia:
a solid is constructed so that it has a circular base of radius r centimeters and every plane section perpendicular to a certain diameter of the base is a square, with a side of the square being a chord of the circle. find the volume of the solid at first i thought the length of a side of the square would be r, but that isn't awlays be true- only when the chord is in the center. so how can i solve this without any values? i dont understand the relationship between the chord and radius, except that the radius intercepts the chord at the midpoint. i know i hav to take the integral to get the volume, but how do i even find the area of one of the squares? please help, thanks, tricia
Answered by Penny Nom.
How far apart are the cars? 2007-04-07
From ian:
2 cars set off in opposite directions and travel for 6 miles, they both take a left turn, and travel for a further 8 miles........... how far apart are the cars???

a) 10 miles
b) 22 miles
c) 20 miles
d) 18 miles

Answered by Penny Nom.
How long would it take to have $100,000? 2007-04-07
From Carolyn:
If I invested $1000.00 (one time ) at 1% compounded daily, how long would it take to have $100,000. ?
Answered by Stephen La Rocque.
3 divided by 3 to it's fifth root 2007-04-06
From Annie:
How do I transform the equation 3 divided by 3 to it's fifth root to simple radical form (getting the radical out of the denominator)?
Answered by Penny Nom.
When will the rocket hit the ground? 2007-04-05
From Megan:
A model rocket is launched at an upward velocity of 460feet per second off a platform that is 40 feet up from the ground. Use the formula h(t)= -16+vo+ho where vo is the initial velocity an ho is the initial height. A.) Find the maximum hight of the rocket. B.) How long does it take to reach the maximum height? C.) When will the rocket hit the ground?
Answered by Stephen La Rocque.
Radius of a circle in a square 2007-04-05
From Lori:
A circle is inscribed in a square. What is the radius of the circle? If there is a small rectangle with a 2 ft. top and a 1ft side at the left in the square touching the corner of the circle.
Answered by Stephen La Rocque and Penny Nom.
A right triangle 2007-04-05
From Lee:
What would be the length of the long side of a right angle triangle if one side was 47cm and the other was 56cm. Many Thanks
Answered by Steve La Rocque and Jaymi Peterson.
The sum of two rational numbers 2007-04-04
From Fathia:
Hi, Could please help me with this question? Prove that if a and b are rational numbers then a + b is a rational number. Thank you,
Answered by Stephen La Rocque.
What is the hypotenuse of a right traingle 2007-04-04
From debbie:
what is the hypotenuse of a right triangle with sides of 38 meters and 24.2 meters.
Answered by Stephen La Rocque.
Arithmetic means and geometric means 2007-04-04
From Dani:
Hi! I was just wondering why the arithmetic mean of sets of numbers is larger than the mean proportional of the same numbers? Thanks! Dani
Answered by Haley Ess.
Simplify (1/3 + 1/3x) / (1/x + x/3x) 2007-04-04
From Farina:
how do i simplify (1/3 + 1/3x) / (1/x + x/3x)
Answered by Penny Nom.
A set of points in space 2007-04-04
From Lenny:
What is a set of points in space the same given distance from its center point called?
Answered by Stephen La Rocque.
Boat trigonometry 2007-04-04
From Kimi:
Hi, I have been working on the attached math problem for my college trig class for over a week. Every avenue I've tried seems to lead either to a dead end, an unreasonable answer, or extremely complicated computations. I was able to calculate the speed of the boat to 3.51 mph, but cannot figure out the measure of angle beta. Once I can figure that out, I can do the rest of the problem. Thanks, Kimi
Answered by Stephen La Rocque.
Algebra 2007-04-03
From Emily:
9,365 and 2,242 added to the difference between 255 and half of a number is 11,784. What is the number?
Answered by Haley Ess.
An equation with fractions 2007-04-03
From deanna:
x      x
-- + -- =8
9       3
need help with showing work and answer

Answered by Penny Nom.
How old will they both be when ... 2007-04-03
From JOHN:
Lisa is 15 years old. Marcus is 49 years old. How old will they both be when Marcus is three times as old as Lisa?
Answered by Stephen La Rocque.
A pipe with 8 inch inside diameter and ten feet long 2007-04-03
From Frank:
How many cubit feet are there in a pipe with 8 inch inside diameter and ten feet long. Thank you
Answered by Stephen La Rocque.
-4x=1/5 2007-04-03
From deanna:
-4x=1/5 need help with showing work and answer
Answered by Stephen La Rocque.
The equation of a parabola 2007-04-03
From Suez:
Find a parabola that passes through the point (1,4) and whose tangent lines at x= -1 and x= -5 have slopes 6 and -2 respectively.
Answered by Stephen La Rocque.
The area of a parcel of land 2007-04-03
From Ricky:
I need the attached land parcel (#0436) converted to acreage. This would be of great help to me. Thanks, Ricky
Answered by Penny Nom.
Understanding fractions and their equivalents 2007-04-02
From Gail:
How can I get fourth grade students to understand fractions and their equivalents?
Answered by Diane Hanson.
Interior and exterior angles 2007-04-02
From Anuj:
If in a regular polygon, each exterior angle is twice the interior angle,find the number of sides?
Answered by Leeanne Boehm.
Find the expression that represents it perimeter. 2007-04-01
From Amy:
A rectangle has sides of 3x-4 and 7x+10. Find the expression that represents it perimeter.
Answered by Steve La Rocque and Haley Ess.
Tina swims 4 miles upstream at 1 mph 2007-03-31
From Michael:
If Tina swims 4 miles upstream at 1 mph and back downstream to the same point to the same point at 4 mph, what is her average speed?
Answered by Penny Nom.
What is the intensity 5m below the surface? 2007-03-31
From david:
I have this question which I am supposed to set it up and solve as a differential equation. I know how to solve the diffrential equation but I am having hard time understanding this question. Here is the question: The intensity of light in the ocean decreases the deeper you dive. In fact, the rate at which the intensity decreases is proportional to the current intensity. Setup the corresponding differential equation and solve for I(Y), the intensity I as a function of current intensity Y. If the light intensity 2m below the surface is 25% of the intensity at the surface, what is the intensity 5m below the surface. Can you please explain to me what does it mean by current intensity and how do I set this equation up. Thanks for the help.
Answered by Penny Nom.
An octagonal box 2007-03-31
From eddie:
I need to make one wood box in octagon type, that has 1125.00 square feet. I want to know how many feet in one side
Answered by Penny Nom.
How much gravel do I need 2007-03-31
From linda:
how much gravel do I need for 254 square feet to be 4 inches deep?
Answered by Penny Nom.
A "claw setting" for a gemstone 2007-03-30
From Stephanie:
I'm trying to make a cone out of a flat sheet of metal for a "claw setting" for a gemstone. The cone must be 8mm wide at the top and 11mm long tapering to a point. But because the prongs must be cut out of the top the cone should not start to taper for a length of 3mm from that top 8mm. The 3mm prong is then bent over the 8mm stone. That probably doesn't make enough sense. But I don't know how to explain it. If it helps a claw setting is the very common prong setting for engagement rings or earrings. Please help as soon as possible as this is a commissioned piece for someone and I'm running out of time. I don't remember any math really from high school so please make the instructions really easy to follow. Thank You!!
Answered by Penny Nom.
The size of a lot 2007-03-30
From Phyllis:
Could you please figure out the square footage and acreage of my lot? One side is 480ft One side is 410ft One side is 480ft The last side is 410ft Thank you.
Answered by Penny Nom.
Golf schedule 2007-03-30
From Jim:
I am organising a golf break for 12 golfers playing in 3 fourballs over 5 days. I would like to maximise the number of different golfers each player can play with. Any help would br greatly appreciated.
Answered by Penny Nom.
Expected value 2007-03-30
From Katrina:
A term life insurance policy will pay a beneficiary a certain sum of money upon the death of the policy holder. These policies have premiums that must be paid annually. Suppose a life insurance company sells a $250,000 one-year term life insurance policy to a 20-year-old male for $350. According to the National Vital Statistics Report, Vol. 47, No. 28, the probability the male will survive the year is 0.99865. Compute the expected value of this policy to the insurance company. Does the company expect to make money at this rate?
Answered by Penny Nom.
A case of wine 2007-03-30
From Carol:
Suppose 3 of 12 bottles in a case of wine are bad. If you randomly select 2 bottles what is the probability that one is good and one is bad
Answered by Penny Nom.
Colour blindness 2007-03-29
From Katrina:
9% of men and .25% of woman can't distinguish between the colors red and green. This is the type of color blindness that causes problems with traffic lights. If 6 men are randomly selected for a study of traffic signal perceptions, find the probability that exactly two of them cannot distinguish between red and green.
Answered by Stephen La Rocque.
How many tons it will take to fill in this area 2007-03-29
From Amy:
I have to fill an area with millings (crushed stone). The measurements are 14 foot long, 14 foot wide, and a depth of 6 inches. How do I find the square footage and how do I convert that into tons? I need to know how many tons it will take to fill in this area.
Answered by Penny Nom.
A parabolic arch 2007-03-29
From A student:
I am trying to figure out how to work this problem as it doesn't have many details.
The problem ask for an equation to satisfy a parabolic arch y = 16 - 0.25x^2 for y>=0.
Find the width w of the arch.

Answered by Stephen La Rocque.
6- team schedule for softball 2007-03-29
From Angie:
I have 6-teams that can only play 3 days a week. Each team must get 18 games in for the season. I have to split them being the home team and the away team sometimes.
Answered by Penny Nom.
Acceptance sampling 2007-03-29
From Katrina:
The Medassist Pharmaceutical Company recieves large shipments of asprin tablets and uses this acceptance sampling plan: Randomly select and test 24 tablets, then accept the whole batch if there is only one or none that doesnt meet the required specifications. If a particular shipment of thousands of asprin tablets actually has a 4% rate of defects, what is the probability that this whole shipment will be accepted?
Answered by Penny Nom.
How many yards of gravel do I need? 2007-03-28
From Joel:
I am constructing a French drain 100 feet long x 6 inches wide by 18 inches deep. Within this trench will be a 4" diameter pipe which will be surrounded by washed gravel. How many yards of gravel do I need? Thanks!
Answered by Stephen La Rocque.
A beam on a lighthouse 2007-03-28
From Lisa:
A beam on a lighthouse 2000 metres away from the nearest point P on a straight shoreline revolves at the rate of 10 pi radians per minute. How fast is the beam of the light moving along the shoreline when it is 500 metres from P?
Answered by Stephen La Rocque.
Find the measure of all 3 angles. 2007-03-28
From Brittney:
The smallest angle in a triangle is one-third of the largest angle. The third angle is 10 more than the smallest one. Find the measure of all 3 angles.
Answered by Stephen La Rocque.
Choosing a bicycle 2007-03-28
From Jackie:
A specific brand of bike comes in two frames, for males or females. Each frame comes in a choice of two colors, red and blue, and with a choice of three seats, soft, medium, and hard.
a) Use the counting principle to determine the number of different arrangements of bicycles that are possible.
b) Construct a tree diagram illustrating all the different arrangements of bicycles that are possible.
c) List the sample space.

Answered by Penny Nom.
The width of an arch 2007-03-28
From Brad:
A parabolic arch satisfies the equation y= 16 - 0.25x^2 for y >= 0. Find the width w of the arch.
Answered by Penny Nom.
The volume of a cylinder 2007-03-28
From bob:
how do you find the volume of a cylinder that is 12 centimeters wide and 16 centimeters tall?
Answered by Penny Nom.
Names of solid figures 2007-03-28
From Debbie:
What is the name of the solid figure: 1) base with 5 equal sides and faces that are 5 triangles 2) all 6 faces are squares and 3) 2 congruent pentagons for bases and 5 rectangular faces. Please help! Thanks
Answered by Penny Nom.
The angles in a right triangle 2007-03-28
From Golaan:
I need the to understand the formula for finding either of the acute angles of a right triangle given it's hieght length and base length. I want to find the degrees of either accute angle. So for this example I have a right triangle with a height of 410 meters and a base length of 1,700 meters. I don't understand cosine, sine, and tangent or the other ones at all. So if the solution includes those (which I believe it does) could b very verbose yet in a very elementary way?

The purpose of ths is that I have to define the general incline angles (or grade) of various areas of terrain. I know the distance by map and also the the altitude at either endpoint.

Answered by Penny Nom.
The foci of an ellipse 2007-03-27
From Brad:
I am trying to figure out how to find the foci of an ellipse x^2/7 + y^2/16 = 1. Since 16 is the largest denominator I know the major axis is going to be the y axis. Do I now take 7-c^2=16. c^2=16-7, c^2=9, c=3. So is my foci (0,+-3).
Answered by Penny Nom.
Slopes of perpendicular lines 2007-03-27
From Mykolyn:
How do you find the slope of a line that is perpendicular to the graph of: y= -1/2x+4?
Answered by Leeanne Boehm and Sara Ulmer.
Solve for x and y 2007-03-27
From anthony:
solve y=4x and x+y=5
Answered by Sara Ulmer.
Factoring a trinomial 2007-03-27
From Kim:
Hi, could you please help me solve this.

3x(squared) +20x - 7

Answered by Leeanne Boehm.
The volume of a trough 2007-03-27
From Tracy:
I need help trying to find the volume of a 9 ft long trough that has two equilateral triangles on the end. When the water of the trough is 2 feet deep and its depth keeps increasing by 1/2 ft/minute. I have to figure out at what rate the water is flowing into the trough.
Can you point me in the right direction for finding the equation for the volume of the trough?

Answered by Penny Nom.
Schedule for a 10 team league 2007-03-27
From Pete:
I don't know if this question involves a math solution, but I am running out of ideas. I have a 10 team league. Each team must play each other once only. I can't seem to match this up and get stuck at the end. Please help. Thanks, Pete
Answered by Penny Nom.
A triangular prism 2007-03-26
From Tom:
Hi, I need to find a container in the shape of a triangular prism that will fit four table tennis balls (this is for a math project!). These balls have a diameter of 4cm, so a radius of 2. I know that the formula for finding an incircle from a triangle is:
radius of incircle = 2area of triangle / perimeter of triangle
But I need to know the length of one side of the triangle from the incircle (the triangle needs to be equalateral). Can you help me find a formula for this? Thanks so much! Tom

Answered by Penny Nom.
The size of a parabolic transmitter/receiver 2007-03-26
From Evan:
I am making a parabola for my home wireless LAN. I feel pretty confident that I can make a parabolic trough that will work. But I am curious about size. Is there really any advantage to using a deep (more depth) parabolic shape over a shallow one as long as you use the correct focal point. And is bigger better? I know that my parabola has to be bigger than the waves it is getting which wont be a problem but if I make the diameter bigger does it get more effective or is there such a thing as "too much of a good thing"? I have looked everywhere for an answer and have come up short. Thanks!
Answered by Stephen La Rocque.
Simplifying an expression in probability 2007-03-26
From Natasha:
well it is p(4) = 5! ------ X (0.5) ^4 X (1- 0.5)^5-4 = 0.15625 4!(5-4)! Now the ^ is to the power of (I believe) Its written in the book as (1-0.5)4 (but the 4 is little and on the top right corner of the first bracket now the second ^5-4 is also little on the top right corner of the 2nd bracket. This is an equation from my text book but for the life of me I cannot figure out how to solve it. If you can give me step by step procedure I'd like that. The answer is also there I just don't know how to get to the answer. Thank you.
Answered by Stephen La Rocque.
The area of an irregular pentagon 2007-03-26
From grace:
hi I am doing graphics and need to find the area of the section given to us (it is just drawn). the sides measure 15m, 35.4m, 5m, 17.8m, 35.7m. the angle between the 15m side and the 35.4m side is 17degrees, the angle between the 35.4m side and the 5m side is 73degrees. the angle between the 5m side and the 17.8m side is 151degrees, the angle between the 17.8m side and the 35.7m side is 90degrees and the angle between the 35.7m side and the 15m side is 95degrees. if you could tell me the area i would be extremely grateful. thanks very much.
Answered by Penny Nom.
Inscribed square in inscribed circle 2007-03-25
From Confuzzled:
"The largest possible circle is drawn inside a square. Then the largest possible square is drawn inside this circle. What is the area of the inner square as a fraction of the area of the outer square?"
This is a question from the Grade 8 Gauss contest and I don't know how to work it out without drawing it. My teacher says it's possible but I still don't get it. Please help!!!

Answered by Stephen La Rocque.
Painting a pool 2007-03-25
From vincent:
Determine the total cost to put 2 coats of paint on the interior pool and amount of water needed to fill pool

facts:
a gallon of paint will cover 50 square feet and cost $18.00 the pool will be filled when water is leveled 6" from the top of pool there are approximately 7.48 gallons of water per cubic foot Rectangle shape pool 36.0 long and 20.0 wide from top

Answered by Stephen La Rocque.
The distance between two fire towers 2007-03-23
From tony:
Two fire towers are 30km apart, tower A is due west of tower B. A fire is spotted from the towers, and the bearing from A and B are N76degreesE and N56degreesW, respectively. Find the distance from the fire to the straight line connecting tower A to tower B.
Answered by Stephen La Rocque.
A linear system 2007-03-23
From Adam:
Solve the system by graphing:
4y>6x
-3x+2y<-6

Answered by Stephen La Rocque and Penny Nom.
Graphing a line 2007-03-23
From mitchell:
Graph each equation. y = 2x + 4
Answered by Jaymi Peterson and Haley Ess.
The volume of a hopper 2007-03-23
From amitesh:
Let me know how to calculate the tonnage of hopper / bins / chutes of different sizes and dimensions.
Answered by Penny Nom.
An inheritance problem 2007-03-23
From Carl:
A father of 3 boys, who owns 17 horses dies. He leaves his first son 1/2 of the horses. The sons hate each other. The father leaves the second son 1/3 of the horses. They are ready to kill each other. He leaves his youngest son 1/6 of the horses. Without cutting the horses into pieces how do they split the horses? I heard this puzzle when I was 12 years old. 1968 The preacher rides up on his horse and said "If I give you my horse will you quit fighting ? They agree to stop fighting?" Why?
Answered by Penny Nom.
The law of cosines 2007-03-23
From chetna:
Q 1) In triangle LMN, l=7, m=5 , n=4. find ANGLE N. After applying the rule and substituting values i'm getting Cos n= 58/40. Is there something wrong. The answer at the back of the book is 34 degrees.
Answered by Penny Nom.
The game of 24 2007-03-22
From Britney:
I have a math problem with the game 24. I am not sure how to solve it. You can use +, -, multiplication, division, ( ), square root, and exponents. Please help me. The numbers are 1, 6, 7 and 8. You have to use all the numbers and you can only use them once.
Answered by Penny Nom.
y = sin(2x) 2007-03-22
From bader:
sin(2x) find dx/dy
Answered by Penny Nom.
The velocity of an aircraft 2007-03-22
From William:
Hello, I don't understand how to work this problem. Thanks in advance for your help!

A jet moving at 500.0 km/h due east moves into a region where the wind is blowing at 120.0 km/h in a direction 30.0 degrees north of east. What is the new velocity and direction of the aircraft relative to the ground?

Answered by Penny Nom.
Which solid figure has 4 more edges than vertices? 2007-03-22
From dawn:
which solid figure has 4 more edges than vertices?
Answered by Penny Nom.
The height of a pole 2007-03-22
From clyde:
I am trying to determine the height of a pole by using 3 angles top 30.8 deg, center 17.3 deg, and bottom -5.8 deg. Can you point me in the right direction ?
Answered by Stephen La Rocque.
The Reuleaux Triangle 2007-03-22
From Dani:
Hi! My question is: Determine how the area of the Reuleaux Triangle of width h compares to the area of a circle of width h? Thanks! Dani
Answered by Penny Nom.
Adding and Subtracting Rational Expressions 2007-03-21
From Pricilla:
Pricilla

I do not know how to do this problem. Don't know where to start.

[(5)/(2x-8)]+[(3x^2+15)/(x^3-4x^2+5x-20)]

Answered by Stephen La Rocque.
Angles of depression 2007-03-21
From romaine:
a woman of height 1.4m standing on top of a building of height 34.6m veiws a tree some distance away. she observes that the angle of depression of the bottom of the tree is 35 degrees, and the angle of depression of the top of the tree is 29 degrees. assume that the building and the tree is on level ground.
(a) calculate the distance of the woman from the top of the tree measured along her line of sight.
(b) determine the height of the tree.

Answered by Stephen La Rocque.
What makes a relation a function? 2007-03-21
From san:
What makes a relation a function?
Answered by Haley Ess.
Examples of algebra used in life 2007-03-21
From Arianna:
i need to find about 5 different examples of algebra used in life, and im not sure where to start. does anyone have any suggestions as to topics that i should choose for my project? thank you so much for all of your input. Arianna Leigh
Answered by Penny Nom.
How many yards of soil is 55 quarts? 2007-03-20
From Claudette:
Costco sells bags of dirt in the quantity "55 quarts".. This makes no sense to me. How many yards of soil is 55 quarts? Thanks!
Answered by Stephen La Rocque.
Compound interest 2007-03-20
From cos:
how much would $2000 be at the end of 1 yr. at a daily compounded interest rate of 3% be? the 3% is not yearly but daily. thank you very much for your help!!
Answered by Penny Nom.
28 golfers playing golf over 7 days 2007-03-20
From ian:
I have 28 golfers playing golf over 7 days I would like everybody to play with each other at least once Is there a schedule? Thank you Ian
Answered by Penny Nom.
Write the interval in absolute value notation 2007-03-20
From Timothy:
1. Write interval in absolute value notation
i) xE[0,9]
ii) xE[-2,20]

Answered by Penny Nom.
A hexagonal prism 2007-03-20
From makiya:
i need help trying to find the surface area and the volume of a hexigon that has an edgde length of 6cm and a hieght of 12 cm i allready know how do do this by breaking it into triangles and finging the surface area and the volume that way i just need to know how to find a faster formuly that aplies to just hexigons thanks makiya
Answered by Penny Nom.
18x - 5 = 3(6x - 2) 2007-03-19
From Lordyne:
How do I work this math problem? 18x - 5 = 3(6x - 2)
Answered by Pam Fowler and Penny Nom.
How many hours will it take to fill completely? 2007-03-19
From Leslie:
If a rectangular prism measures 8" and measures 12" by 16", what is it's volume? If after 8 hours the prism is 6" full, how long will it take to fill and how many hours will it take to fill completely?
Answered by Stephen La Rocque.
A one pass calculation of standard deviation 2007-03-19
From Murtaza:
Hi I searched the listings but couldn't find what i am looking for. I have a dataset and i am allowed only 1 pass through it. At the end of this pass i must have the mean and standard deviation. Calculating the mean is easy as also demonstrated in your listings. I think the same can be done for standard deviation as well. But the doubt that i have is this :- Will the calculated standard deviation be sequence dependent ? i.e. if i change the order in which the data items enter the system, will the std deviation also change ?? Thanks in advance Murtaza
Answered by Penny Nom.
Three equations in three unknowns 2007-03-18
From Shawna:
You are told that you are working with three different numbers. When the first number is added to twice the other two numbers, the result s 64( i.e x+2y+2z= 64). When the second number is added to twice the other two numbers the result is 62 (y+2x+2z=6). Finally, when the third number is added to twice the other two numbers, the resut is 59 z+2x+2y=59) Determine the three numbers?
Answered by Penny Nom.
n-1/(n+1)! + n+1/n! 2007-03-18
From Cody:
How do you go about simplifying something like this; n-1/(n+1)! + n+1/n!?
Answered by Steve La Rocque and Claude Tardif.
A triangle 2007-03-18
From Farzana:
pointsA(1,0),B(8,0),C(3,4) are the vertices of a triangle.What is the of this triangle?
Answered by Stephen La Rocque.
Nickels, dimes and quarters 2007-03-18
From Alexandra:
Ellen has two more nickels than dimes and three more quarters than nickels. She has $3.35 in all. How many coins of each type does she have?
Answered by Penny Nom.
How may litres are in my fish pond 2007-03-18
From beryl:
I need to work how may litres in my fish pond. I have measures it in two sections because of it shape. One section is 1600w, 1100L, 540D. the other section is 1060W, 1800L, 380D. Thank you beryl
Answered by Penny Nom.
The volume of a tube 2007-03-17
From Tiffany:
If I have a 42" diameter tube with 3 1/2" walls and 108" in length. How may gallons of water will it hold? Thank you, Tiffany
Answered by Penny Nom.
y = 1/4(x+3)^2-4 2007-03-17
From Irene:
How the graph of a parabola f(x)=1/4(x+3)squared-4 can be obtained from the graph of y=xsquared, using Translations and Scalings.
Answered by Penny Nom.
A water treatment plant 2007-03-17
From Elvia:
A water treatment plant is built with three cylindrical tanks to contain the water for a town. Each tank has a radius of 15 feet and a depth of 25 feet. If there are 7.5 gallons in a cubic foot of water, approximately how many gallons of water can be treated at the plant at any one time?
Answered by Penny Nom.
A line that is perpendicular to another line 2007-03-16
From Jesse:
How do you write an equation of a line that is perpendicular to y+2x+3 and passes through (3,4); I don't understand the steps of how to do this????
Answered by Stephen La Rocque and Penny Nom.
Percentage markup 2007-03-16
From Debbie:
What formula or formulas do I use if I want to ultimately charge a customer $300 for a service, but need to factor in a 15% commission for a sales rep, $20 to cover the cost of our overhead, the cost of a $25 coupon that we will be sending to the customer to apply to future services. In other words, what amount do I initially quote the customer to be able to cover these costs?
Answered by Stephen La Rocque.
A river crossing 2007-03-16
From tara:
A river has a constant current of 4 kilometers per hour. At what angle to a boat dock should a motorboat, capable of maintaining a constant speed of 20 kilometers per hour, be headed on order to reach a point directly opposite the dock? If the river is 1/2 kilometer wide, how long with it take to cross?
Answered by Stephen La Rocque.
Equivalent decimals 2007-03-16
From megan:
Write an equivalent decimal for 13.48.
Answered by Stephen La Rocque and Penny Nom.
y-intercept form 2007-03-16
From Ken:
Simple way on how to convert to & from y-intercept form & Std. form. Eg: Convert into y-intercept form: 3x+4y+7=0, Eg.: Convert into Std. Form: y=-1/2x +3, y=50x +300. I would appreciate your help in this matter, as I find it hard to understand. Kind Regards: Ken
Answered by Penny Nom.
The question is whether the games is fair or unfair? 2007-03-15
From kk:
Players have two red chips each with an A side and a B side, and one blue chip with an A side and a B side. The players flip all three chips. Player one wins if both red chips show A, if the blue chip shows A, or if all chips show A. Otherwise player two wins. The Question is: Whether the games is fair or unfair and to draw a probability tree to show the answer. Hope you can help
Answered by Haley Ess.
An isosceles Triangle 2007-03-15
From Devon:
The length of one of the equal legs of an isosceles triangle is 8 cm less than 4 times the length of the base. If the perimeter is 29 cm, find the length of one of the equal legs.
Answered by Stephen La Rocque.
Triple angle tangent formula 2007-03-15
From sam:
Hi I am trying to derive a triple angle formulae for tan. I know i need to use compound and double angle formulae but am finding it difficult to "clean" up my fraction to get the triple angle formulae can you show me a worked derivation?! thanks
Answered by Penny Nom.
Circles and chords 2007-03-15
From Henry:
Where do you place a chord in a circle such that it divides the area of the circle 1/3 and 2/3? Also, what is the length of the chord?
Answered by Stephen La Rocque.
Two busses 2007-03-15
From Devon:
A bus leaves a station at 1 p.m., traveling west at an average rate of 44 mi/h. One hour later a second bus leaves the same station, traveling east at a rate of 48 mi/h. At what time will the two buses be 274 mi apart?
Answered by Steve La Rocque and Penny Nom.
The area of a property 2007-03-14
From Greg:
Hi my name is Greg. I have a piece of property I am trying to buy, but need to find out the square feet of it first. The four sides are unequal lengths and none at a right angle or parallel. Here are the measurement in feet. The base of the square is 29.12 feet in width.
The right side of the square is 44.33 feet high.
The top of the square is 28.80 feet in width.
The left side of the square is 46.20 feet high.
I have not done this math in ages and really need your help.

Answered by Harley Weston.
Cut a given sphere by a plane 2007-03-14
From Rory:
Cut a given sphere by a plane so that the volumes of the two segments formed shall be in a given ratio. Show that in modern notation, this leads to the cubic equation m(r+x)^2(2r-x)=n(r-x)^2(2r+x) where r is the radius of the sphere, x is the distance of the cutting plane from the centre of the sphere, and m/n < 1 is the given ratio. Can you help me show the solution to this question. Thank you.
Answered by Penny Nom.
The dimensions of a rectangle with least perimeter for given area 2007-03-14
From Karlianna:
find dimensions of a rectangle with least perimeter for given area
A) 30cm2
b) 50cm2

Answered by Penny Nom.
A stem and leaf plot 2007-03-14
From Hazel:
Hi, I wanna ask about my son's homework re: reading stem and leaf plots. It's really hard for me to understand about this..please show me how. Ex. Number of dogs competing in dog shows

Stem Leaf
1 62024
2 5295
3 166
4 05

1. What was the least number of dogs at a dog show?
2.What was the greatest number of dogs at a dog show?
3. How many dogs shows are reported in the stem and leaf plot? thanks...Hazel

Answered by Penny Nom.
Greatest common divisor 2007-03-14
From Marie:
How do I prove this? Let h be the greatest common divisor of the positive integers a and b. Show that there exist integers p and q (not necessarily positive) such that pa + qb = h. Thanks
Answered by Penny Nom.
How many marbles are Red and how many marbles are White? 2007-03-13
From Angie:
A jar has 16 marbles in it. Most of the marbles are Red and the rest are White. Two marbles are taken out of the jar at the same time. It is equally probable that two marbles are of the same colour as the two marbles of different colour (that is the probability that they are both red or both white is the same as they are different). How many marbles are Red and how many marbles are White? Could anyone help me with the above question? Best regards, Angie
Answered by Danny Dyer.
A regular quadrilateral 2007-03-13
From Jackie:
How can the measure of each angle of a regular quadrilateral be determined?
Answered by Haley Ess.
Angle of elevation 2007-03-13
From Joslyn:
A ship at sea sights a 12m high lighthouse on a cliff which is 80m above sea level. If the angle of elevation to the top of the lighthouse is 27 degrees, calculate the distance from the ship to the shore.
Answered by Haley Ess.
Kicking a field goal 2007-03-13
From san:
A football player attempts a field goal by kicking the football. The ball follows the path modelled by the equation h= -4.9^2+10t+3, where h is the height of the ball above the ground in metres, and t is the time since the ball was kicked in seconds. *the ball must clear the uprights for the field goal to count. The uprights are approximately 5 m high. How long does the ball stay above 5 m in height?
Answered by Penny Nom.
Sketch the graph of the hyperbola and its asymptotes 2007-03-13
From Katrina:
Given (y-2) ^2 – x^2/4 =1 do the following: Sketch the graph of the hyperbola and its asymptotes. Label the vertices on the graph I attached the graph thanks for your help!
Answered by Penny Nom.
Using complex numbers 2007-03-12
From Kara:
Do you use complex numbers in your job?
Answered by Stephen La Rocque and Penny Nom.
exponential decay of Carbon-14 2007-03-12
From Christine:
How do I set up the exponential decay formula for Carbon 14 with an initial amount of 1500 grams.
Answered by Stephen La Rocque.
A roadway over a river 2007-03-12
From Taranjeet:
My teacher has given us bridge with only one measurement. From the river to the roadway is 50 metres in length (vertically) The question he wants us to find out is. What is the distance between the vertical supports. He has said that: At a horizontal distance of 'x' metres from the foot of the arch the height of the arch, the height of the arch above the river 'h' metres is given by: h=-1/40(x squared) = 3x I don't understand how to find the distance between the support beams. Thank you
Answered by Penny Nom.
A trig limit 2007-03-11
From Lo:
tan(2*x)/sin(3*x)
Answered by Penny Nom.
The period of a simple pendulum 2007-03-10
From Melissa:
The period of a simple pendulum of length L feet is given by: T=2pi(sqrt(L/g))seconds. It is assumed that g, the acceleration due to gravity on the surface of the earth, is 32 feet per second per second. If the pendulum is a clock that keeps good time when L=4 feet, how much time will the clock gain in 24 hours if the length of the pendulum is decreased to 3.97 feet? (Use differentials and evaluate the necessary derivative at L=4 feet.) Answer is in seconds. Melissa
Answered by Penny Nom.
A truncated cone 2007-03-10
From Russell:
Hello, I have attempted to use two of your answers already given and had no real success. This young lady is making a cat food dispenser using a truncated cone. The top of the cone as a diameter of 5 inches with a height of 6 1/2 inches and diameter of 3 inches for the bottom.

Could you please map out a solution for the both of us to understand? Thank you so much for your time and for this wonderful service.

Russell

Answered by Penny Nom.
Can the triangle be called both an acute triangle and an equilateral triangle? 2007-03-10
From Jane:
If all sides of a triangle are the same length and all angles are 60 degrees, can the triangle be called both an acute triangle and an equilateral triangle?
Answered by Walter Whiteley.
How many barrels would this container hold per inch? 2007-03-10
From LaDonna:
My container is 50" across and 40.5' ft. long. I believe it would hold 140 barrels. If measuring, how many barrels would this container hold per inch?
Answered by Penny Nom.
The volume of a cylinder 2007-03-09
From Lakeshia:
2) The volume of a cylinder (think about the volume of a can) is given by V = πr2h where r is the radius of the cylinder and h is the height of the cylinder. Suppose the volume of the can is 100 cubic centimeters. a)Write h as a function of r. Keep "p" in the function's equation. Answer b) What is the measurement of the height if the radius of the cylinder is 2 centimeters? Round your answer to the nearest whole number. Answer
Answered by Penny Nom.
Statistics data 2007-03-08
From Parnini:
The following table shows the expenditure on different household items by a family. Draw a line graph for the following data:
Items : Food Rent Cloth Education Saving Others
Expenditure : 2000 2.500 1.500 1.200 4000 3.500

Answered by Stephen La Rocque.
A steel pipe pile 2007-03-08
From Sean:
Need to fill a 18" OD x 60 feet steel pipe pile with a wall thickness of 1/2" What is the amount of concrete I need in order to fill? Thank you
Answered by Stephen La Rocque.
An irregular pentagon 2007-03-08
From kelly:
If I have a pentagon with 2 sides of length 5 and one with a length of 2, what are the lengths of other 2 sides?
Answered by Stephen La Rocque.
A fountain as a parabola 2007-03-08
From Emily:
I have to do a math project proving that something in real life is a parabola. I really need some help here because i don't know where to start. I want to do it on a fountain and prove it's a parabola but how do i do that? I would really appreciate it if you could help Emily
Answered by Stephen La Rocque.
Chords and arc lengths 2007-03-08
From Angela:
my dad, who is a welder, asked me a question pertaining to chords and points on an arc to which I cannot for the life of me find an answer or an equation. if you could help, it would be much appreciated. I am sending an attachment of the problem.
Answered by Stephen La Rocque.
Find the length of the belt 2007-03-08
From Helen:
Find the length of the belt
Answered by Penny Nom.
A circle is stretched horizontally by a factor of 2 2007-03-07
From bob:
I was wondering if you could double check my work?

the question is as follows:
The circle X^2+y^2-2x-3= 0 is stretched horizontally by a factor of 2 to obtain an ellipse what is the equation of this ellipse in general form?

Answered by Penny Nom.
An excavation 2007-03-07
From Brandon:
you have and excavation area 140' long by 20' wide by 3' deep. What is the excavated volume?
Answered by Penny Nom and Brennan Yaremko.
The height of a hexagonal pyramid 2007-03-07
From Carol:
i have been set a question that gives a regular hexagon and each of the sloping sides is 6cm from a corner of a base to the apex(point). it also has 2cm sides it looks sort of like this:
/\6cm
2cm

Answered by Penny Nom.
How much soil do I get? 2007-03-07
From Robbi:
I have a garden area that is 19 feet by 11 feet and I would like it to be 2 feet deep. I need to know how much soil to get in feet and yards. Could you please give me the formulas to figure this out. Thanks Robbi
Answered by Penny Nom and Melanie Tyrer.
4th grade equivalent fractions 2007-03-07
From Raymond:
5/8=?/16 ?=?
Answered by Melanie Tyrer and Sara Ulmer.
How much fill I would need 2007-03-06
From Tracey:
Oh am I confused! I need to know how much fill I would need to cover an area that is 28 feet in diameter by 8 inches deep. Thank you ...
Answered by Penny Nom.
How many gallons of water in a mcf (million cubic feet)? 2007-03-06
From Willene:
How many gallons of water in a mcf (million cubic feet)?
Answered by Penny Nom.
Rolling two dice 2007-03-06
From Mary:
I have a test and I need to know how to figure out how to find the probability of rolling 2 dice and coming up with a 7 for both rolls. Could you please explain how to find the probability? Thank you. Mary
Answered by Penny Nom.
An irregular polygon 2007-03-06
From Dike:
How do i construct an irregular polygon
Answered by Penny Nom.
circles 2007-03-06
From chetna:
A large circle has a radius of 10cm.Given four congruent circles tangent to one another within the large circle what is the radius of the largest circle which will fit in the middle ?
Answered by Stephen La Rocque.
A metal shroud for a outdoor fireplace 2007-03-06
From Arnold:
I am making a metal shroud for a outdoor fireplace, it is basically a lampshade type pattern,like the bottom of a cone.The top has to be 6 inches to fit the 6 inch stovepipe,and the bottom will be a 24 inch circle. the sides will be 18 inches in length.With the cost of the sheet metal,I can only afford to cut this out once,can you help me with the pattern ?
Answered by Penny Nom.
Prove that the triangle ABC is isosceles 2007-03-05
From Lisa:
I am stuck on this problem... If AB = CD (parellel line), and
Answered by Stephen La Rocque and Penny Nom.
One acre of water, one foot deep 2007-03-05
From Jeff:
If you have one square acre of ground flooded with one foot of water and a pump that pumps 1000 gallons a minute how long would it take to drain the one acre of ground?
Answered by Penny Nom.
The net for a pentagonal pyramid 2007-03-05
From MaKayla:
How many bases does the net for a pentagonal pyramid have?
Answered by Penny Nom.
Finding the vertical height of a roof 2007-03-05
From Zainab:
The question is: If the vertical height if a triangle is half the width of the base and the slant length is 6 metres, find the exact vertical height of this part of the roof. I'm actually confused about finding out the height of an equilateral triangle if you're only given the length or slant height. Please help! O.o
Answered by Stephen La Rocque.
a^2 + b^2 = c^2 2007-03-04
From Colburn:
27 feet long base 14 feet height what is the diagonal
Answered by Stephen La Rocque.
The volume of a trailer 2007-03-04
From Ron:
I just can not seem to work this out, i think it is old age catching up with me, I am buying a trailer that is 25 feet long 7.5 foot wide and with 4 foot sides and i am trying to work out how many m3 it will hold can anyone help. Thank you in advance for any help. Regards Ron
Answered by Penny Nom.
A long narrow cone 2007-03-04
From Amy:
I am an art student making a piece of jewelry out of a flat metal sheet. I'm trying to make a long narrow cone that's roughly 3 1/4" long and just slightly under 1" wide at the base. Since I'm no math wiz, I'm having a really hard time. Please help.
Answered by Penny Nom.
The area of an inscribed triangle 2007-03-02
From Caitlin:
What is the area of an equalateral triangle inscribed in a circle whose circumference is 6 pie?? PLEASE HELP
Answered by Penny Nom.
Slope 2007-03-02
From Lacey:
My algebra 2 class is researching graphing and slope and we would like to know how we use graphing and slope in everyday life. So if you could get back to me with some real world examples i would really appreciate it. Thanks for your help!
Answered by Steve La Rocque, Penny Nom and Sara Ulmer.
Finding the x-intercept 2007-03-02
From Ash:
Y = -1/100X^2 =27/10X +0 = 0 THE Y INTERCEPT IS 50 HOW DO I FIND THE X INTERCEPT
Answered by Gabriel Potter.
Grams to millilitres 2007-03-02
From anton:
I am trying to convert 28 grams of weight loss powder into mils
Answered by Gabriel Potter.
Is c relatively prime to (n.p)? 2007-03-02
From Andrew:
Please could you help me with a small mathematics problem?! I've put = the problem into three parts but it isn't really three problems; just = one! Given:- {[(n.p)^p] / [c + (n.p)]} =3D "an integer" (where c > = (n.p); c and n are integers, n is even and p is an odd prime number) Can one prove that c cannot be relatively prime to (n.p)? { i.e. c = must have the factor (n.p) ?} If so, is the converse also true? {i.e. If c is relatively prime to = (n.p), then [c + (n.p)] cannot be a factor of (n.p)^p ?} Lastly but similarly, if c is relatively prime to a and c > a > = (n.p) ; (c - a) > (n.p) **; and c is not relatively prime to (n.p) = would it be true that (c - a) cannot be a factor (n.p)^p ? I hope my question makes sense?
Answered by Penny Nom.
Mutually exclusive events 2007-03-01
From kalyssa:
will you me an example of two events that are mutually exclusive and could you explain to me what mutually exclusive means?
Answered by Steve La Rocque, Pam Fowler and Penny Nom.
How many yards of concrete? 2007-02-28
From jeremy:
I need to figure how many yards of concrete to use. I work for an asphalt company, and we are expanding into concrete. Currently we measure everything in sq. ft. So we came up with 6,960 sq ft of concrete to remove and replace. Currently there is 6" of concrete, and 2" of bedding material. We would like to replace only 4.5" of concrete, the rest in bedding. Here is how I figured, am I correct. 6,960 Sq ft (to simplify I made the Sq ft 350 X 20 ) 350 ft X 20 ft X .33 (1/3 ft or 4") = 2,310 cubic feet 2,310 cb ft / 27 = 85.55 cubic yards Is this correct?
Answered by Penny Nom.
Simultaneous equations with fractions 2007-02-28
From Alyca:
Hello Math Central, I am a grade 10 student taking Academic math. Our unit right now is method of substitution and elimination. I'm stuck on this one question that I've been doing forever. Please help =)

*For this equation I have to do method of elimination, but it's so much harder with fractions...could some one please explain to me how to do it step by step?*
x y 2
-- - -- = - --
3 6 3

x y 1
-- - -- = 1---
12 4 2

Answered by Steve La Rocque and Ashley Mang.
sin(3a), cos(3a) and tan(3a) 2007-02-28
From mailene:
hi, i...indeed,to..need..your..help
how..cn..i..prove..this,formula???
sin3a=3sina-4sin^3a
cos3a=4cos^3-3cosa
tan3a=3tan-tan^3a /1-3tan^2a

the..symbol...^is..the..expOnent

Answered by Haley Ess and Penny Nom.
The intersection of two lines 2007-02-28
From Tamara:
I was told to make a graph to help solve this question. I was given two equations. One was x-2y=3 and the other was 2x+y=-4. We have to change it to the equation of the line format. Then we graph the line and find the point where they meet. I've been trying to do that, but when i check the answer, It turns out wrong. It's meeting in the wrong spot. I need help please.
Answered by Penny Nom.
Parallel lines 2007-02-27
From Jennifer:
How to you decide whether the graphs of these two equations are parallel linse: y=3x+2, y=1/3x+4
Answered by Stephen La Rocque and Penny Nom.
A logarithm of a product and the product of logarithms 2007-02-27
From Sharon:
Can you please explain Explain the difference between a logarithm of a product and the product of logarithms and give examples of each?
Answered by Stephen La Rocque and Penny Nom.
Social security numbers 2007-02-27
From Tish:
Social Security numbers are in the form ###-##-####, where each symbol represents a number 0-9, how many are possible with this format? Will we ever run out of Social Security numbers?

(This is one question with two parts, I saw some examples but I am still confused because if there are 1 million possibilities then how have we been able to issue out numbers to over 280 US citizens?) (For the second part a yes or no answer is sufficient)

Answered by Penny Nom.
Who won the backstroke? 2007-02-26
From Timothy:
Three girls held a series of swimming races among themselves. They decided to award a positive whole number of points for finishing first, second and third, where the number of points for first was more than the number for second, which was more than the number for third. The same number of points was awarded for the place in each race. There were no ties.

The first race was breaststroke. Altogether Sarah accumulated 20 points, Michelle 10 points and Catherine 9 points. If Sarah did not win the backstroke race, who did win the backstroke?

Answered by Penny Nom.
Solve for c 2007-02-26
From Steffane:
A over m = b over c sorry I can't put them over each other
Answered by Penny Nom.
How old are they now? 2007-02-26
From Ali:
Please help us. My daughter can't get this answer. The sum of Louise's age and Todd's age is 34. 5 years ago the sum of twice Louise's age and 3 times Todd's age was 61. How old are they now?
Answered by Haley Ess and Steve La Rocque.
How many cubic yards can my truck hold? 2007-02-25
From Mike:
If I have a dump truck that has a box that is 14' long sides that are 3.5' high and the width is 7.5' wide... how do I figure out how many cubic yards this truck can hold?
Answered by Steve La Rocque, Penny Nom and Melanie Tyrer.
A cylinder with a hemispheric cap 2007-02-25
From lee:
i have a diagram of cylinder with a height of 1.260m and radius 0.340m a hemisphere of radius0.400m is bonded to one end of the cylinder with a constant overlap of 0.06m between them a cylinder of radius 0.170m and length 1.280m is bored out of the composite body what is the volume remaining

also the composite body is to be painted i need to calculate the total area to be covered

could you please have a look thank you

Answered by Penny Nom.
Cutting the top off a circle 2007-02-25
From Daniel:
If a circle as a diameter of D I cut off a straight part at C from the top What is the formula to find the area of the left over part?
Answered by Penny Nom.
The grade 6 math curriculum 2007-02-24
From Ajay:
Do you have a list of topics a 6th grade students should learn in the 6th grade? I am "rusty" in math and would like to assist my son... Example: if an icetray has X cc of water and is split into 8 parts...what is the volume of each part...is this truly 6th grade level math?
Answered by Pam Fowler and Penny Nom.
Walk around a 14 acre perimeter 2007-02-24
From Maria:
How many times would you have to walk around a 14 acre perimeter in order to walk 1 mile?
Answered by Penny Nom.
At what rate is the area of the triangle changing? 2007-02-24
From mac:
two sticks 3.5 feet long are hinged together and are stood up to form an isosceles triangle with the floor. The sticks slide apart, and at the moment when the triangle is equilateral, the angle is increasing at the rate of 1/3 radian/sec. At what rate is the area of the triangle increasing or decreasing at that moment?
Mac

Answered by Penny Nom.
The volume of a cone 2007-02-24
From SAFDAR:
How to derive the formula for volume of cone?
Answered by Penny Nom.
The area of an L-shaped polygon 2007-02-23
From Louanne:
How do I find the surface area of a L shaped polygon?
Answered by Melanie Tyrer.
How long does she have to get out of the way? 2007-02-23
From Sharon:
How do I solve this? The height h (in feet) of an object that is dropped from the height of s feet is given by the formula h = s - 16t^2 , where t is the time the object has been falling. A 6 foot tall woman on a sidewalk looks directly overhead and sees a window washer drop a bottle from the 6 story. How long does she have to get out of the way? Round to the nearest tenth. (A story is 12 feet.)
Answered by Penny Nom.
The writing of numerals 2007-02-23
From Abbas:
What is the reason to write the integral such that one as 1 two as 2 similarly three as 3 and vice versa
Answered by Penny Nom.
The three sides of a right triangle 2007-02-23
From Sharon:
I know the hypotenuse of a right triangle is a^2 + b^2 = c^2 but when I replace this with the numbers given, I can't solve it.

Here is my question: The hypotenuse of a right triangle is 2.8 units long. The longer leg is 1.9 units longer than the shorter leg. Find the lengths of the sides of the triangle.

Answered by Pam Fowler and Penny Nom.
The three sides of a triangle 2007-02-23
From Christopher:
If two sides of a triangel have lengths 4 and 9, then the length of the third side may be any number:
a) Less than 13
b) Greater than 5 but less than 13
c) Greatehr than 4 but less than 9
d) Greater than 5

Answered by Haley Ess.
Circles 2007-02-22
From Erika:
I have a research paper due on real life uses of conic sections I've looked through all your conic topics and uses of them, but and i cant seem to find real life uses for circles. What are real life uses of circles?
Answered by Penny Nom.
A pentagon inscribed in a circle 2007-02-22
From Amanda:
Find the formula for calculating the length of the side of a pentagon given the radius of the circle that encloses it. Once you find the formula, find the length of the side of a pentagon which is enclosed in a circle 12 cm in diameter. So I need to know the formula, and the length of the side of the pentagon. Thank you!!
Answered by Penny Nom.
Similar triangles 2007-02-21
From Vivienne:
The lengths of the sides of a triangle are 8, 15, & 17. If the longest side of a simlar triangle is 51, what is the length of the shortest side?
Answered by Stephen La Rocque and Penny Nom.
Surface area of a triangular prism 2007-02-21
From Katrina:
What is the formula to finding the surface area of a triangular prism?
Answered by Gabriel Potter.
A limit 2007-02-21
From Amanda:
I have tried to work out this problem by using the properties of limits and the conjugate rule to simplify but I am not real sure how to approach the problem or what they are asking for any help would be greatly appreciated. Amanda
Answered by Penny Nom.
Diagonals of a rectangular grid 2007-02-21
From matthew:
in a rectangular grid 3600x288 (or any large rectangle with an even number of units) how can I determine the number of vertices a diagonal line will pass through?
Answered by Stephen La Rocque and Penny Nom.
A circle tangent problem 2007-02-21
From Jason:
Please help me to solve this problem. Thank you very much. Find the equation of the circle tangent to 3x+y+14=0 and x+3y+10=0 with radius squareroot of 10.
Answered by Stephen La Rocque and Penny Nom.
What jobs involve using conics? 2007-02-21
From Nicki:
What jobs involve using conics?
Answered by Penny Nom.
Acceleration 2007-02-20
From thomas:
Don't know if this service still exists. ACCELERATION CALCULATION initial vel 0km/hr final vel 24km/hr time 3s
Answered by Stephen La Rocque.
Finding the equation of the circle 2007-02-20
From ning:
Given the radius of a circle square root of 10 tangent to the line 3x+y+19 = 0 and passing through (0,-3), how can i solve the equation of circle? thank you...
Answered by Chris Fisher, Steve La Rocque and Penny Nom.
Find the area of the triangle 2007-02-20
From Christina:
Graph the function f(X)= x+1/x-1 and graph the tangent line to the function at the points A:(2,3) and B:(-1,0). The point of intersection of the two tangent lines is C. Find the area of the triangle ABC.
Answered by Stephen La Rocque.
A common tangent line 2007-02-20
From chris:
I have this problem I have been working on for days and cannot figure it out. it states: Find the two points on the curve y=x^4 - 2x^2 - x that have a common tangent line.

I know you use the first derivative to find the tangent line so if it is a common tangent line should you find two of the exact same tangent line equations at two coordinate points?

Answered by Chris Fisher.
Grams/milliliters 2007-02-19
From Keshia:
If there are 27 grams of sugar in 240 mililiters, How many grams are in 100 mililiters? I have been trying to figure this question out for about an hour and I cannot seem to get it. Thank you for your help.
Answered by Chris Fisher and Haley Ess.
The length of the side of a hexagon 2007-02-19
From Nick:
if the area of a regular hexagon is 259.807 how do i find the length of each side, and the apothem? thanks if you can help
Answered by Penny Nom.
A concrete slab in the shape of a regular hexagon 2007-02-19
From Mandy:
Hi, We are laying a concrete slab in the shape of a regular hexagon with each side being 2m long. The slab will be 10cm thick. How much concrete will we need? Thank you ever so much! Mandy
Answered by Penny Nom.
The surface area of a hexagonal prism 2007-02-19
From Cari:
How do you find the surface area of a hexagonal prism? Thank you.
Answered by Penny Nom.
Increasing speed and decreasing time 2007-02-18
From Nigel:
I am completely lost...can you help with this: The time, t, to travel a distance, d, at a constant speed, v, can be obtained from the formula: d = vt. If the speed is increased by x, find an expression to represent the decrease in time to travel the same distance.
Answered by Stephen La Rocque.
What is the remainder when 3^2007 is divided by seven? 2007-02-18
From Chris:
What is the remainder when 3^2007 is divided by seven??
Answered by Leeanne Boehm.
The area of a lot 2007-02-18
From Steve:
our land is four sided, but not the same length. We are doing a property line revision and the new measurements are, 479.46 by 715 by477.81 by 582. what will this give me in acreage.
Answered by Harley Weston.
How many socks must you take from the drawer? 2007-02-18
From Gary:
Suppose you have 6 striped socks and 4 red socks in a drawer. How many socks must you take from the drawer to be certain of having a pair of red socks?
Answered by Pam Fowler.
Weightng coins 2007-02-18
From Gary:
You are considering buying 27 silver coins that look alike, but you have been told that one of the coins is a lightweight counterfeit. Find the least number of weighings on a balance scale that you can use to be certain you have found the counterfeit coin.
Answered by Penny Nom.
Optical illusions 2007-02-18
From Jami:
Hi, I'm Jami and I'm in 10th grade.I'm doing a geometry research project on optical illusions and need to know how math is involved.I have an idea already of how our eyes percept 2 dimensional images and construct them into 3D images but, that isn't my question.There are many books that contain optical illusion pictures that have secret messages in them or have objects popping out. Is there a mathematical way in that optical illusion pictures are constructed?
Answered by Walter Whiteley and Harley Weston.
I have 350 ft. of 3/8 ID tubing. 2007-02-17
From Fred:
I have 350 ft. of 3/8 ID tubing. How many 64 oz. gallons of liquid will it hold?
Answered by Penny Nom.
*** x ** = **** 2007-02-16
From Dani:
Use the numbers 1-9 only once to make the following: A three digit number times a two digit number that gives a four digit number. That is *** x ** = **** (the numbers 1-9 can only be used once for each star). I came up with the answer 138 x 42 = 5796 but I want to know whether this is the only solution and if not, is there a formula to come up with the answer. Thanks! I appreciate your help!
Answered by Chris Fisher and Claude Tardif.
A concrete foundation 2007-02-15
From Jeremy:
BiltWEll Construction needs to create a concrete foundation. 5ft deep measuring 60ft by 32ft, outside dimensions, with walls 7in. How many cubic yards of concrete will they need?
Answered by Penny Nom.
A gymnast is swinging on a high bar. 2007-02-14
From William:
A gymnast is swinging on a high bar. The distance between his waist (center of mass) and the bar is 1.1 m. If his speed at the top of his swing is zero and his gain in speed is due entirely to his change in gravitational potential energy, find his speed at the bottom of his swing.
Answered by Stephen La Rocque.
Factoring polynomials 2007-02-14
From Joe:
I am in the eighth grade, and we are learning the equivalent of Algebra 2. I have no ides how to factor (x-2)(x^2-1)-6x-6 You help is most aprreciated. Thank you! Joe
Answered by Stephen La Rocque.
The volume of a prism 2007-02-14
From jasmine:
What is the formula to find the volume of a triangular prism
Answered by Penny Nom.
Things that are shaped like a parabola 2007-02-14
From carra:
i can't find other examples of things that are shaped like a parabola except for bridges.............. Pls. help it is due tomorrow. thank you very much:)
Answered by Steve La Rocque, Penny Nom and Walter Whiteley.
Centimeters to cubic meters and grams to kilograms 2007-02-14
From Colleen:
A box of balls measures 120cm by 20cm by 20cm and weighs 48 g.
One thousand boxes = x cubic meters and weighs x kilograms.
Also, one million boxes = x cubic metes and weighs x kilograms.
One billion boxes = x cubic meters and weighs x kilograms. Solve for x.

Answered by Stephen La Rocque.
What constitutes a face for a geometric solid? 2007-02-14
From Erica:
What constitutes a face for a geometric solid? We have been getting conflicting answers from different textbooks. Some teachers are saying that cylinders and cones do not have faces. I'm thinking that cylinders have 2 and cones have 1. Please enlighten me. Thanks!
Answered by Walter Whiteley.
A gallon of water 2007-02-14
From Ashley:
Is four 20 oz bottles of water a gallon? How many
Answered by Gabriel Potter.
Cubic inches 2007-02-13
From Renee:
a glass has a height of 3 inches and a radius of 2 inches. about how many cubic inches of fwater will the glass hold? and how do you figure the answer? thank you...................renee
Answered by Gabriel Potter and Melanie Tyrer.
Trig - Ferris wheel 2007-02-13
From Anthony:
A ferris wheel is 250 feet in diameter and revolves every 40 seconds when in motion. Your step up to seat on the wheel at the bottom 2 feet above the ground so you are sitting 4 feet above the ground to start. Derive the formula for the height of your seat at time (t). If I go three times around, how long is the ride in ditance traveled?
Answered by Stephen La Rocque and Penny Nom.
A person jumps from the roof of a house 2007-02-13
From Jessie:
A person jumps from the roof of a house 3.9m high. When he strikes the ground below, he bends his knees so that the torso decelerates over an approximate distance of 0.70m. If the mass of his torso (excluding legs) is 42kg, find a) his velocity just before his feet strike the ground , and b) the average force exerted on his torso by his legs during deceleration.

Please provide calculations and answers for comparison.

Answered by Stephen La Rocque.
Algebra 2007-02-13
From B.Wilson:
The teacher has asked to solve the following, but I have never seen this before: 6x/2 = 4x/5
Answered by Haley Ess.
Countable and uncountable sets 2007-02-13
From piyush:
we se that union of countably infinite no of sets having countably infinite number of elements is a countable set we can express p(n) (i.e power set of natural number) as a union of countable infinite number of sets i.e p(n)=s1Us2Us3..... where s1=null s2={1,2,3,4,5..........} s3={{1,1},{1,2},{1,3},..............{2,1},{2,2}........} using the same statement can we prove that power set of natural number is a infinit countable set
Answered by Penny Nom and Claude Tardif.
An augmented matrix 2007-02-13
From Mary:
I've been trying for quite some time now to figure this out. I have to solve this by using the Gauss-Jordan Method: 3x - y = 15 2x + 3y = 10 Can anyone help me?
Answered by Penny Nom and Gabriel Potter.
a specific 3-d shape 2007-02-12
From Robby:
What three dimensional shape has 6 vertices, 5 faces, and two congruent triangular bases? I can't find the answer to this question anywhere. Your help will be appreciated. Thanks, Robby Craig
Answered by Pam Fowler, Gabe Potter and Walter Whiteley.
Exponential form of complex numbers 2007-02-12
From Austin:
When dealing with imaginary numbers in engineering, I am having trouble getting things into the exponential form. The equation is -1+i now I do know that re^(theta)i = r*cos(theta) + r*i*sin(theta). Just not quite understanding the order of operations. Thanks
Answered by Penny Nom.
Square centimeters 2007-02-12
From Ivan:
how many square centimeters are in a rectangle that is 210 centimeters long by 39 centimeters wide.? thank you
Answered by Karen McIver and Melanie Tyrer.
Two concentric circles 2007-02-11
From maria:
i have a problem with this quadratic word problem which i am trying to solve it but couldn,t get it please help me to solve this question Two concentric circles are drawn, the radius of one being 2cm greater than that of the other. The area of the ring enclosed between the two circles is one quarter of the area of the smaller circle.Calculate the radii of the circles,correct to three significant figures. (Don,t substitute for pie)
Answered by Stephen La Rocque.
Volume of an inner tube 2007-02-10
From Bubba:
For a science project, I'm collecting methane gas in an inner tube. In addition to measuring psi of the inner tube, I'd like to calculate the volume of gas collected. What is the formula for or how would I calculate the volume of an inner tube? I appreciate any help you can give me.

Thanks so much. Bubba

Answered by Penny Nom.
Large exponents 2007-02-09
From Nick:
I am trying to figure out an extremely large number. It relates to the estimated number of bacterial divisions in 12775 generations of bacteria.
The problem I need to solve is:
2^12775 or 2 to the power of 12,775.

Answered by Stephen La Rocque and Penny Nom.
Alligation 2007-02-09
From Shanah:
Orders: 360 mL of 75% acetic acid Stock: 99% acetic acid and distilled water Find the volume of acetic acid and distilled water to get the needed concentration. Any help with this is greatly appreciated, Thanks
Answered by Stephen La Rocque.
Multiplication of polynomial 2007-02-09
From John:
hello.. i just want to ask how can i solve this?? (2x+3)(6x+8)
Answered by Steve La Rocque, Penny Nom and Brennan Yaremko.
The capacity of a tank 2007-02-08
From lorn:
what is the capacity of a tank height is 110 feet diameter is 24 feet
Answered by Brennan Yaremko.
Perimeter of an octagon inside a circle 2007-02-08
From Courtney:
a regular octagon is inscribed in a circle with a radius of 12 cm. find the perimeter of the octagon?
Answered by Stephen La Rocque.
Converting sq inches to sq millimeters 2007-02-08
From Michael:
If I have 1/32 sq inches, what does that equal in sq millimeters?
Answered by Stephen La Rocque.
Simultaneous equations with envelopes 2007-02-08
From Mick:
There were 17 envelopes bought, some were brown, some were white. The brown envelopes cost one cent more per envelope than the white ones. The total cost was 80 cents. How much of each type of envelope was bought? --Many thanks!
Answered by Stephen La Rocque.
Romeo is throwing pebbles 2007-02-08
From Jessie:
Romeo is chucking pebbles gently up to Juliet's window, and he wants the pebbles to hit the window with only a horizontal component of velocity. He is standing at the edge of a rose garden 4.5m below her window and 5.0m from the base of the wall. How fast are the pebbles going when they hit her window?
Answered by Stephen La Rocque.
BEDMAS solving for X 2007-02-07
From damian:
Hi, I am having a problem understanding why when using BEDMAS the text book example of the following question has used bedmas and reduced the calculation to 4/3.

Text book formula:
X/200,000 = (X - 90,000) / 150,000
X = 4/3 * (X - 90,000)
X = 360,000


My solution: I always thought that when you do something to one side you do the same to the other. Why would the equation bring the 200,000 / 150,000 down to 4 / 3 I would think it would look something like this.
X/200,000 = (X - 90,000) / 150,000
X = 200,000*((X - 90,000) / 150,000)

Answered by Stephen La Rocque.
Area of a rectangle 2007-02-07
From kyle:
paul has enough material to make a fence 22 feet long. he will use this material to fence of a rectangular section of his yard. the rectangles sides must all be whole numbers. i am having a hard time finding the greatest and the least area that he can enclose his fece
Answered by Haley Ess.
The interior angles of a parallelogram 2007-02-07
From jenniffeir:
Can a parallelogram have two 45 degree angles and two 75 degree angles?
Answered by Haley Ess.
A triangle inscribed in a semicircle 2007-02-06
From Benneth:
Consider a triangle inscribed in a semicircle with a radius of R. What are th possible perimeters for the triagle? And the areas?
Answered by Penny Nom.
Dividing money unequally 2007-02-06
From Linda:
I have a total of $13,826.48. It is to be divided by 15 kids - 2 of the kids only get half of what the other 13 do - how much do the 13 get and how much does the 2 get? Thanks
Answered by Haley Ess and Steve La Rocque.
Nickels, dimes and quarters 2007-02-05
From Avinash:
Mary has 48 coins made up of nickels, dimes and quarters with a total value of $5.10. She has 4 more dimes than nickels and quarters combined. How many coins of each kind does she have? Use matrix to solve the system
Answered by Stephen La Rocque.
Multiplying and dividing by zero 2007-02-05
From colin:
i would like to if u multiply 1,000,000 by 0 does it equal o or 1.

if you have a number eg 15689 and you divide by o what would the answer be is it 1

Answered by Penny Nom.
Angle of elevation 2007-02-05
From Zee:
A 55 ft. flagpole casts a 25 ft. shadow. Calculate the angle of elevation to the sun to the nearest degree.
Answered by Stephen La Rocque.
Driving on a banked roadway 2007-02-05
From William:
A race track is shaped like an inverted cone. Race cars drive around this track in a horizontal path. If a drivers speed is 34 m/s, at what distance from the tip of the cone should he drive his car if there is no friction?

THANKS IN ADVANCE FOR ANY ASSISTANCE YOU CAN PROVIDE!

Sincerely,
William Bush

Answered by Stephen La Rocque.
$380 Billion divided by 29.6 million 2007-02-05
From Bob:
$380 Billion divided by Iraq population of 29.6 million equals $_____ / Iraqi.
Answered by Penny Nom.
A rectangle inscribed in a circle 2007-02-04
From Benneth:
Consider a rectangle with radius R inscribed in a circle. What are the possible areas of the rectangle?
Answered by Steve La Rocque and Walter Whiteley.
How many diagonals does a hexagon have? 2007-02-03
From Marsha:
How many diagonals does a hexagon have?
Answered by Stephen La Rocque.
14 golfers--6 days 2007-02-02
From tom:
14 of us are going golfing-- over the course of 6 days is there a formula that would allow us to have the chance to play with as many different partners as possible--2 foursome, and two threesomes
Answered by Chris Fisher.
More on a man and his wife on an escalator 2007-02-02
From Patrick:
Hello, I was helping my son with a homework problem and came across a very similar on your site:

http://mathcentral.uregina.ca/QQ/database/QQ.09.01/monty1.html

We are trying to understand the answer shown. There seems to be leap, in the answer that isn't explained or we can't see it. Specifically in the answer it says:

"We know that the woman olny walks 21 steps to reach the top, and thus in the time she walks 7 more steps the escalator also goes up 7 steps. Hence the woman and the escalator are travelling at the same rate."

We understand that she still has 7 steps left, but how is it that you can conclude from that (or other factors) that during those 7 steps the escalator will also travel 7 steps.

Thanks in advance for any help you can offer. Pat

Answered by Stephen La Rocque.
Patrick's new job 2007-02-02
From kathy:
patrick has been offered jobs by two different automobile repair shops. his income from the first shop will be $1,200 per month plus 3% of the monthly sales. the second shop will pay patrick $1,500 per month plus 2% of the monthly sales. part a let s equal the amount of monthly sales and i equal patrick's income in dollars. write a system of two equations that could be used to determine the monthly income from either shop. part b solve the equation to determine what amount of monthly sales for each shop would result in patrick earning the same monthly income, i, from both shops
Answered by Penny Nom.
Yardage of dirt fill 2007-02-02
From Gene:
I am removing dirt then replacing it with good dirt. I need to know how many yards of fill dirt it would take to fill a diameter 165 feet with fill of 13 ft would take how many cubic yards of dirt.
Answered by Penny Nom.
Price of topsoil 2007-02-02
From Tyler:
I need to convert cubic yards to square feet. I have topsoil that is $34.30 a cubic yard and I need to know what the price is in square feet. The depth is 18".
Answered by Penny Nom.
Derivatives 2007-02-01
From Jacob:
Find the derivative of: y= pi^2+x^2+3xy+sin(y^2)
Answered by Penny Nom.
Fencing a field 2007-02-01
From Carol:
"The area of a rectangular field is 1200m^2. Two parallel sides are fenced with aluminum at $20/m. The remaining two sides are fenced with steel at $10/m. The total cost of the fencing is $2200." What is the length of each side fenced with aluminum and steel?
Answered by Stephen La Rocque.
Rectangles and squares 2007-01-31
From Shelby:
I am doing homework and I need to know why squares are rectangles but not all rectangles are squares?
Answered by Penny Nom.
Angle of clock hands 2007-01-31
From Sophia:
How many degrees are in the aute angle formed by the hands of a clock at 2:20 p.m.?
Answered by Stephen La Rocque.
The perimeter and area of a right angled triangle 2007-01-31
From Naveen:
I need to calculate the area of a right angled triangle. However the measurements of the 3 sides is missing. All we are told is that the perimeter of the triangle is 1000cm.
Answered by Penny Nom.
The volume of a cone 2007-01-31
From ajay:
WHY VOLUME OF CONE IS ONE THIRD OF THE VOLUME OF CYLINDER?
Answered by Penny Nom.
The substitution method 2007-01-31
From Victoria:
how do i solve this problem using the substitution method?
2x-5= -14
-7x+14y= -5

Answered by Stephen La Rocque.
The elimination method 2007-01-31
From Addrianna:
x-2y=2
3x-5y=7

Answered by Stephen La Rocque.
Water in a triangular trough 2007-01-30
From Trina:
the trough is 5 feet long and its vertical cross sections are inverted isosceles triangles with base 2 feet and height 3 feet. water is draining out of the trough at a rate of 2 cubic feet per minute. at any time t, let h be the depth and v be the volume of water in the trough.
a. find the volume of water in the trough when it is full
b. what is the rate of change in h at the instant when the trough is .25 full by volume?
c. what is the rate of change in the area of the surface of the water at the instant when the trough is .25 full by volume?

Answered by Penny Nom.
The density of an irregular wooden block 2007-01-30
From Carlos:
I'm trying to help my son but I don't really have too much knowledge about this topic. he is been asked to convert 10 milliliter to gram, I think that since 1 g = 1ml the answer will be 10 g also there is another question of how of the density of an irregular wooden block is affected if:
a) the center is hollow:
b) the block is not completely submerged when determining the volume.

Answered by Stephen La Rocque and Penny Nom.
Simplify 18e^2fg/32eg^2 2007-01-30
From Morgan:
how do you simplify 18e^2fg/32eg^2
Answered by Penny Nom.
A thin wire 20 centimeters long is formed into a rectangle 2007-01-29
From Trisha:
a thin wire 20 centimeters long is formed into a rectangle. if the width of this rectangle is 4 centimeters, what is the length?
Answered by Penny Nom.
Height of a right triangle? 2007-01-29
From Engelbert:
The base of the right triangle is 50 ft. At the angle (on the base of the triangle) across from the right angle is labeled portion (a triangle within a triangle), the base of this little triangle is 10 ft. and the height is 6 ft. What equation can i use to solve this?
Answered by Steve La Rocque and Haley Ess.
Concrete paving blocks 2007-01-29
From Engelbert:
Read-mixed concrete is sold by the cubic yard. A landscape contractor wants to form concrete paving blocks that are rectangular solids with dimensions 36 inches by 36 inches by 6 inches. Assuming there is no waste, how many such blocks can she expect to make from one cubic yard of concrete? (1 yard= 36 inches)
Answered by Stephen La Rocque.
11^n +22^n = 55^n 2007-01-29
From Ankit:
11^n +22^n = 55^n

find the value of n?

Answered by Penny Nom.
A right triangle 2007-01-29
From Tayler:
One leg of a right triangle is 8 cm. less than the hypotenuse while the other leg is 1 cm. less than the hypotenuse. Find the length of the hypotenuse.
Answered by Penny Nom.
How many gallons will it hold? 2007-01-27
From Taye:
I am building a aquarium for my daughter, It is 6 feet long 24 inches high and only 5 inches wide. How many gallons will it hold?
Answered by Stephen La Rocque and Penny Nom.
The height of an isosceles triangle 2007-01-27
From Brendan:
I need to find the height of an isosceles triangle whose angles are 52, 52 and 76 degrees. The base is 100, and the two equal sides are unknown. How would I go about this?
Answered by Stephen La Rocque.
converting square miles into square meters 2007-01-27
From Susan:
how do you convert square miles into square meters
Answered by Stephen La Rocque.
The centre and radius of a circle 2007-01-27
From A student:
x^2+y^2=121 is the equation of the Circle C

(1) Write down the center and the radius of C.

Answered by Stephen La Rocque.
A boat travels 60 miles downstream in the same time it takes to go 36 miles upstream 2007-01-27
From Liz:
A boat travels 60 miles downstream in the same time it takes to go 36 miles upstream. The speed of the boat in still water is 15 mi/h greater than the speed of the current. Find the speed of the current.
Answered by Stephen La Rocque.
A curve on a cylinder 2007-01-27
From John:
think of a tube,say 50mm in diameter made out of cardboard, project vertical lines at right angles from the base at say every 2mm right round the tube in pencil.No pretend you can put this cardboard tube in a saw and cut it at 45degrees. Get a pair of scissors and cut it at the lowest end and lay it out flat.It now looks like a graph,how do you work out each of these vertical lengths possibly chord lengths
Answered by Stephen La Rocque and Penny Nom.
The sum of the digits of a two-digit number is 13. 2007-01-27
From Liz:
The sum of the digits of a two-digit number is 13. If the digits are reversed, the new number is 9 more than the original number. Find the original number.
Answered by Penny Nom.
The interest is compounded daily 2007-01-26
From Kimberey:
If I put $2000 in a savings account with 5.05% APR and the interest is compounded daily, but credited monthly, how much would I have at the end of a year?
Answered by Stephen La Rocque.
Can you make a square out of a right triangle by using only one straight line? 2007-01-26
From Debra:
Can you make a square out of a right triangle by using only one straight line?
Answered by Stephen La Rocque and Penny Nom.
Upper Quartiles 2007-01-26
From Jamie:
I see you have a question about Q3 with even numbers but what about odd numbers? I have a problem with 19 numbers 36,45,49,53,55,56,59,61,62,65,67,70,75,81,82,86,89,94,99. Is there anyway the answer could be 81.5 because every time I do it I get 82 and my teacher tells me that is wrong. So in conclusion how do you do it?
Answered by Penny Nom.
Take $.01 and double it every day for 30 days. 2007-01-26
From Pat:
Take $.01 and double it every day for 30 days. How many dollars would you have?
Answered by Stephen La Rocque.
The altitude of regular n-side pyramid 2007-01-26
From Wendimu:
The altitude of regular n-side pyramid is half of it's base side.What is the angle between the lateral face and it's base?
Answered by Penny Nom.
Grams and mls 2007-01-26
From Larissa:
Is there a formula to convert grams to mls..?
Answered by Stephen La Rocque.
Concrete slab 2007-01-26
From Cindy:
How much concrete will it take to pour a slab 16ft.By 18ft. 3inches deep
Answered by Penny Nom.
The area of a ellipse 2007-01-25
From Ranjit:
I have a task in which i have to find the area of a ellipse. i find this difficult because i have only been provided with the perimeter, which is 1000m.
Answered by Chris Fisher.
A circle inscribed in an octagon 2007-01-25
From Anna:
If I know that the sides of my octagon are 8 units, how do I determine the radius of an inscribed circle?
Answered by Penny Nom.
Splitting a 5 acre piece of property 2007-01-25
From John:
I have a 5 acre piece of property that I wish to split into 5 individual 1 acre parcels. The 5 acre parcel 1s 660 feet in length and 330 feet in width. The first two parcels are 264'x165', but I am asking for help getting the perimeters of the last 3 lots, one of which is a irregular pentagon. All of the one acre parcels must have an area of 43,560 square feet each.
Answered by Penny Nom.
A regular pentagonal pyramid 2007-01-25
From Vickifazio:
a regular pentagonal pyramid has a base perimeter of 1,085 ft. How long is one side of the base?
Answered by Stephen La Rocque.
Freefalling objects 2007-01-25
From Patricia:
How long will it take a stone dropped from atop a 1000-meter tall building to reach the ground?
Answered by Stephen La Rocque.
The scale on a drawing 2007-01-24
From Sandra:
If I have a 1cm x 1cm square and enlarge it to a 2cm x 2 cm square, does that show a 1:2 scale or 1:4 scale? Do you use the length only or is area involved?
Answered by Penny Nom.
Percent elevation gain 2007-01-24
From Susan:
okay... so if I hike Gregory Canyon and it's a 1.1 mile hike with a 900 foot elevation gain, what's the percent elevation gain?
Answered by Stephen La Rocque.
An algebra exercise 2007-01-24
From Clara:
Given x + 2y = -2 and x - 2y = 18, find:
a) x^2 - 4y^2
b) x^2 + 4y^2

Answered by Stephen La Rocque and Penny Nom.
Interest compounded daily 2007-01-24
From Scott:
I would like to know what 4.88% interest compounded daily would amount to on $24,000.
Answered by Penny Nom.
What is the point of intersection of two sides of a polygon? 2007-01-23
From Tyler:
What is the point of intersection of two sides of a polygon?
Answered by Stephen La Rocque.
Maximizing profit 2007-01-23
From Denise:
Total Profit= Total Revenue-Total Cost P(x)=R(x)-C(x) Where x is the number of units sold. Find the maximum profit and the number of units that must be sold in order to get that profit. R(x)=5x C(x)=.001x^2+1.2x+60
Answered by Stephen La Rocque.
p=2l+2w find l 2007-01-23
From Jadesola:
p=2l+2w find l
Answered by Stephen La Rocque and Penny Nom.
The volume of a pool 2007-01-23
From Steve:
how do we measure the cubic feet of a pool. The pool is 50 feet long by 40 feet wide by 4 feet at the shallow end and 9 feet at the deep end.
Answered by Penny Nom.
how many milliliters does it take to fill a liter? 2007-01-22
From Heather:
how many milliliters does it take to fill a liter?
Answered by Penny Nom.
(3x+5) / 2 = (5x+3) / (3)+ 2 2007-01-22
From Jaclyn:
(3x+5) / 2 = (5x+3) / (3)+ 2
Answered by Penny Nom.
How many locations for the lampposts are possible? 2007-01-21
From Madeeha:
Maria's backyard has two trees that are 40 feet apart, as shown in the accompanying diagram. She wants to place lampposts so that the posts are 30 feet from both of the trees. Draw a sketch to show where the lampposts could be placed in relation to the trees. How many locations for the lampposts are possible?
Answered by Penny Nom.
The diagonals of a parallelogram are mutually bisecting 2007-01-21
From Greta:
I am having trouble proving that the diagonals of a parallelogram are mutually bisecting.
Answered by Penny Nom.
Simplify (2x)^4 (1/2x^3)^2 2007-01-21
From Sarah:
Simplify (2x)^4 (1/2x^3)^2
Answered by Penny Nom.
Line segments on dot paper 2007-01-21
From Khaori:
The three line segments below are drawn on centimeter dot paper.

a. Find the length of each segment to the nearest ten-thousandth of a centimeter.
b. Could these line segments be arranged to form a triangle? If no, explain why or why not. If yes, answer this question: could they form a right triangle? Explain why or why not.

Answered by Penny Nom.
The area of a scalene triangle 2007-01-20
From Kalsi:
I have a scalene triangle in which i have to find the area.
Answered by Penny Nom.
Calculating a person's volume 2007-01-20
From James:
Hi i am currently trying to calculate my volume. I have tried to do this using a tank of water and measuring the difference. i worked out that my volume is 112 litres. my question is that i want to make a box that would hold my volume and so i need to convert it into centimeters. i know that there are 100 cm^2 in a litre. if i divide this by 4 i would be 91x91x91ft. is this simply to do with density. if so how could i make this more realistic?
Answered by Stephen La Rocque.
A formula for figuring a tanks volume at various levels 2007-01-20
From Dave:
I know there has to be a general formula to figure out a tanks volume where as you know the total volume but you need to know what it will hold at various levels measured in 1/8th inches in depth and that you may be able to plug this formula into a program like say excel and thus create a chart unique to that tank. But for the lack of me I can not figure it out.
Answered by Stephen La Rocque.
Which lets you put equal groups together? 2007-01-20
From Kathy:
Which lets you put equal groups together? division, multiplication, subtraction, estimation
Answered by Stephen La Rocque.
Irregular multi sided polygon 2007-01-20
From Graham:
I have an irregular polygon. I know the length of all the sides and know approximately all the internal angles. Is there a formula or table that can calculate the area for me?
Answered by Steve La Rocque, Chris Fisher and Penny Nom.
Depth and volume in a cylindrical tank 2007-01-20
From Andy:
I need to fill a 10 foot diameter by 35 foot long cylinder 85% full. Is there a way to calculate at what depth equals 85%? ie, if I put a stick in the tank would it be at 8.5 feet?
Answered by Stephen La Rocque.
mls and grams 2007-01-19
From JJ:
If I have a 2000litre pool and need to put 2mg of chlorine per litre, how many mls do I need to add?
Answered by Penny Nom.
The area of an irregular pentagon 2007-01-18
From Ranjit:
I would like to find the area of an irregular pentagon where the total perimeter is 1000m. Is there a Heron's formula for this?
Answered by Stephen La Rocque.
x divided by 8+2= 3x divided by 4-3. 2007-01-18
From Fabiola:
x divided by 8+2= 3x divided by 4-3.
need to solve for x

Answered by Penny Nom.
Answer to skill testing question 2007-01-18
From Mike:
This question was on a ballot. Is the right answer 36 or 1? 5+2-1X6=
Answered by Stephen La Rocque.
Angular speed 2007-01-18
From Cristina:
A car is moving at a rate of 50 miles per hour, and the diameter of its wheels is 2.5 feet.
a) Find the number of revolutions per minute the wheels are rotating.
b) Find the angular speed of the wheels in radians per minute.

Answered by Stephen La Rocque.
Comparing two fractions 2007-01-18
From Kayla:
Why does eight over twelve compared to one half work when you use cross multiplication.
Answered by Penny Nom.
As many player combinations as possible 2007-01-18
From Ken:
I coach a 5th grade basketball team with 12 players. I'd like to come up with a way to get as many player combinations as possible with two teams of 6 so that the makeup of the teams is different each time. An example would be: Players 1-6 on one team and players 7-12 on the other. Then I could have 1,3,5,7,9,11 on one team and the evens on the other. Then 1,2,6,7,9,10 on one team and the rest on the other. Can you help set something like this up?
Answered by Chris Fisher.
Points of intersection 2007-01-18
From Mark:
Find the points of intersection of the given pair of curves. 2x - 3y = -8 and 3x - 5y = -13
Answered by Steve La Rocque and Haley Ess.
Find the probability of choosing a red or 5 ball. 2007-01-17
From Carl:
four red balls numbered 1 thru 4 and three blue balls numbered 3 thru 5 placed in a bag. a ball is drawn from the bag. find the probability of choosing a red or 5 ball.
Answered by Stephen La Rocque.
How do you write the equation y= 4x-2 in ax+by=c form? 2007-01-17
From Evan:
How do you write the equation y= 4x-2 in ax+by=c form?
Answered by Penny Nom.
I need to store 26600 gallons of water 2007-01-17
From Mauricio:
I have to figure out the "height" of a square tank that is 18' wide and 18' deep, so I can store 26600 gallons of water?
Answered by Stephen La Rocque and Penny Nom.
The height of a triangle 2007-01-17
From Sharon:
I am needing to determine the height of a triangle with a 50 ft base and one side is 40 ft the other side being 30 ft.
Answered by Penny Nom.
Two consecutive angles of a parallelogram 2007-01-17
From Veronica:
The measures of two consecutive angles of a parallelogram are in the ratio 9:11. Find the measure of each angle.
Answered by Penny Nom.
The volume of a frustum of a pyramid 2007-01-17
From Sam:
Find the volume of a frustum of a pyramid with square base of side b, square top of side a, and height h.
Answered by Penny Nom.
The volume of a sphere 2007-01-16
From Anshul:
How to prove or find the volume of sphere?
Answered by Harley Weston.
At what speeds were they travelling? 2007-01-16
From Maria:
Kim and Julie are joining Nicole at her parents cottage for the weekend. The cottage is 150km away from their neighbourhood. Kim can leave directly after school but Julie will be leaving after band practice, an hour and a quarter later. Kim took her time and drove slowly, averaging 20km/h slower than Julie. They both arrived at the same time.
At what speeds were they travelling?

Answered by Stephen La Rocque.
How much lemon oil do you need to make 10.15L of polish? 2007-01-16
From Sajad:
to make an environmentally safe furniture polish, you can combine lemon oil and mineral oil in the ratio 3:200, by volume. How much lemon oil do you need to make 10.15L of polish?
Answered by Penny Nom.
Show that 3k+2 and 5k+3 are relatively prime 2007-01-15
From Andrea:
show that if k is a positive integer, then 3k+2 and 5k+3 are relatively prime
Answered by Penny Nom.
Probability and Lisa's rings 2007-01-15
From Spencer:
lisa has 9 rings in her jewelry box. five are gold and 4 are silver. if she randomly select 3 rings to wear to a party, find each probability p(2 silver or 2 gold)
Answered by Stephen La Rocque.
A line through the point (4, 7) and parallel to the line y=2 2007-01-14
From Chasity:
i am suppose to write an equation of a line through the point (4, 7) and parallel to the line y=2. please help me
Answered by Penny Nom.
The height of a triangle 2007-01-14
From Shanelle:
I'm trying to find the height of a triangle but i only know the base and one of the sides as well as the angle between those two sides...but how can i find the height?
Answered by Penny Nom.
A line perpendicular to another line 2007-01-13
From Dusty:
Find an equation for the line perpendicular to: 3x - 4y + 1 = 0 passing through (4,7)
Answered by Leeanne Boehm and Penny Nom.
An octagonal bird house 2007-01-13
From Soren:
I'm in the process of building a birdhouse that is an octagon (based on previous questions, looks like that's a familiar tune). The essential elements are known, but I get stuck when trying to determine the angle for the cuts that would be made to the thickness of the wood so that they all fit together when assembled. Each octagonal section is 7 inches in width and the peak of the roof will be 2 inches higher than the sides. My sense is that the angle cuts that need to be made to the 'height' of each piece of wood. By height I mean the thinnest part of the wood that is neither the length nor the width to use colloquial terms. While it's clear that a slight angle is needed, it would seem that the angle would necessarily change as the distance from the top of any one side to the peak changes. Please advise if more clarification is needed. The 2 inches is random and can be changed if more convenient. Whew!
Answered by Harley Weston.
Throwing a stone downwards 2007-01-12
From Momo:
a stone is thrown vertically downward with an initial speed of 14m/s from a height of 65 m above the ground. A) how far does the stone travel in the first 2.0 s? B) what is the stone velocity just before it hits the ground?
Answered by Stephen La Rocque.
3x > -12 2007-01-11
From Jen:
In the following inequality, would one flip the inequality symbol?

3x > -12

Answered by Stephen La Rocque.
In the shadow of a building 2007-01-11
From Bill:
if a building b feet cast a shadow f feet long, then, at the same time of the day, a tree t feet high will cast a shadow, how many feet long?
Answered by Stephen La Rocque.
One leg of a right triangle 2007-01-11
From Eric:
I have a right triangle and the only known leg is one leg adjacent to the right angle. The question is how to find either of the other angles?
Answered by Penny Nom.
The volume of a box in cubic yards 2007-01-11
From Robert:
i have a box 44inch high 16foot long on top and 8 foot long on the bottom 5foot wide how do i find the cubic yards
Answered by Penny Nom.
Input-output table 2007-01-10
From Richard:
Question is make an input - output table for the function using the domain 0,1,2,3,4, Then state the range of the function.

y =x + 8 ?

y =0.4x +3 ?

Answered by Penny Nom.
Fact families with fractions 2007-01-10
From Joe:
I am trying to help my son with a math problem dealing with fact families and fractions. The question only gives 2 numbers 1/16 + 1/12 and 5/4 - 4/5. Can you help me understand how fact families work with fractions?
Answered by Penny Nom.
y = 5 x - 3 2007-01-10
From Richard:
Question is the function y = 5 x - 3 for the given value of x 7 , 1 or 4
Answered by Stephen La Rocque and Penny Nom.
A cylinder has how many flat surfaces, vertices, and edges? 2007-01-09
From Stacy:
A cylinder has how many flat surfaces, vertices, and edges?
Answered by Diane Hanson and Penny Nom.
Write the equation of a vertical line passing through (6,-2) 2007-01-09
From Alex:
Write the equation of a vertical line passing through (6,-2)
Answered by Penny Nom.
Products with symbols 2007-01-09
From Jennifer:
Find the product of each
(8x)(-4)
(3x)(5y)(7z)
(8x^3y)-5X^2)
(6a)(3a)(-b^2)

Answered by Penny Nom.
Integrate x^8 (x^8 + 2)^2 ((x^8 + 2)^3 + 1)^4 2007-01-09
From James:
How do you integrate x^8 (x^8 + 2)^2 ((x^8 + 2)^3 + 1)^4
Answered by Penny Nom.
How many gallons of water in 6600 feet of half inch pipe 2007-01-09
From David:
How many gallons of water in 6600 feet of half inch pipe
Answered by Stephen La Rocque.
Find the area of the rectangle 2007-01-09
From Mike:
The perimeter of a rectangle is 72 in. The base is 3 times the height. find the area of the rectangle?
Answered by Penny Nom.
y = x^2 - 2 2007-01-08
From Mike:
Graph the quadratic equation: y=Xsq-2
Answered by Penny Nom.
Angular speed 2007-01-06
From Robert:
A rear wheel and tire on a John Deere 8520 farm tractor has a 39-in. radius. Find the angle (in radians) through which a wheel rotates in 12 sec if the tractor is traveling at a speed of 22 mph.
Answered by Penny Nom.
x + 2y < 6 2007-01-06
From Mike:
I understand solving by graphing, but i am having trouble with the graphing of in equalities. i solved : x + 2y<6
Answered by Penny Nom.
A probability problem 2007-01-06
From Rose:
The radius of the a small circle is 1 and the radius of the large circle is 2. If two points are selected at random from the interior of the large circle, what is the probability that both points will be from the shaded region? the shaded area outside the small circle.
Answered by Penny Nom.
Are all rectangles trapezoids? 2007-01-05
From Sarah:
Are all rectangles trapezoids?
Answered by Chris Fisher.
How much money is that? 2007-01-05
From Stephanie:
The question is asking fill in the whole box. It is set of 15 nickels. It says circle 2/5 of the nickels? The next question ask How much money is that?
Answered by Penny Nom.
A word problem 2007-01-05
From Harold:
the sum of one number plus four less than five times another is 26.
Answered by Penny Nom.
What are the dimensions of the most economical container? 2007-01-04
From Ashely:
A cylindrical container costs $2.00 per square foot for the sides and $3.00 a square foot for the top and bottom. The container must hold 100 cubic feet of material. What are the dimensions of the most economical container.
Answered by Stephen La Rocque.
How do I determine the length of an ellipse if the width and area are known? 2007-01-04
From Tom:
How do I determine the length of an ellipse if the width and area are known?
Answered by Steve La Rocque and Karen McIver.
A linear system 2007-01-04
From Harold:
System A (4, 2/3)
2x-3y=6 and x-3y=2

I approached this problem using the addition method.

Could i have found the answer using the graphing method?

Answered by Penny Nom.
The slope-intercept form 2007-01-04
From Earnestine:
Given a line containing the points (1,4), (2,7), and (3,10) determine the slope-intercept form of the equation, provide one additional point on this line, and graph the function.
Answered by Penny Nom.
Annualized 2007-01-03
From Dan:
What would my rate of compound interest be per month? Invested $80,000 and in 150 days have increased that amount by $14,300.
Answered by Stephen La Rocque.
Tiles arranged in rows 2007-01-02
From Ann:
Some tiles are arranged in rows so that the number of tiles in each row is 8 more than the number of rows. The same number of tiles can be arranged in 3 more rows than the first pattern with 16 tiles in each row. Find the total number of tiles.
Answered by Stephen La Rocque.
Where do the medians of a triangle meet? 2007-01-02
From Brittany:
Where do the medians of a triangle meet?
1. Inside the triangle
v 2. On the traingle
3. Outside the traingle

Answered by Stephen La Rocque.
Can the trailer safely pass under the bridge? 2007-01-02
From Jim:
A truck hauling a double-wide trailer needs to pass under a parabolic-arched bridge en route or take a 50 mile detour. The trailer is 14 feet high and 15 feet wide. The arch supporting the bridge at this underpass is 18 feet high at the center and 40 feet wide at the base. Can the trailer safely pass under the bridge?
Answered by Stephen La Rocque.
How do the perimeters of the two pentagons compare? 2007-01-02
From Robert:
If the area of one pentagon is eighty-one times the area of another pentagon, how do the perimeters of the two pentagons compare?
Answered by Penny Nom.
Interest compounded daily 2007-01-01
From Kelly:
I have made an investment that offers one-half of 1% interest compounded daily. What would that interest calculate to with a $100,000.00 investment for one year ? I have not found a calculative formula that will calculate these figures ... Please HELP !!!
Answered by Penny Nom.
Apples, oranges, bananas and pears 2006-12-30
From Richard:
If 6 oranges weights as much as 4 apples, 5 bananas weights as much as 3 pears, and ten bananas weights as much as 8 oranges. How many pears and apples would you need to have their weights balanced.
Answered by Penny Nom.
Calculating rainwater volume 2006-12-30
From Mark:
Trying to calculate 1" rainfall on a 1000 sq' roof=how many gallons of water would collect in our cistern.
Answered by Stephen La Rocque.
An octagonal birdhouse 2006-12-30
From Verner:
I am building a octagon birdhouse,what degree would I cut each side of each piece of wood to assemble the birdhouse?
Answered by Penny Nom.
The area of a triangle 2006-12-29
From Paula:
The area of a triangle is 24cm². Its height is 2cm more than its base. Determine the base and the height.
Answered by Penny Nom.
Who invented the division symbol? 2006-12-29
From Alex:
Who invented the division symbol?
Answered by Penny Nom.
How many gallons of water does an 8 inch pipe that is 50 foot long hold? 2006-12-29
From Bryan:
How many gallons of water does an 8 inch pipe that is 50 foot long hold?
Answered by Penny Nom.
An isosceles triangle 2006-12-29
From Katrina:
Is there any way to calculate the height or the length of the equal sides of an isosceles triangle given only the base length and the angles?
Answered by Penny Nom.
What size pulley would I need? 2006-12-29
From Chris:
If I have a motor that's spinning at 950 RPM's with a pulley that's 6in diameter with a belt running to a generator, What size pulley would I need on the generator to make it spin at 3600 RPM
Answered by Penny Nom.
A quuadratic equation 2006-12-22
From Harold:
L=3w-4
A=96Square
I used the quadratic equation to solve and my answer is still wrong

I convert the square of 1168 to decimal.

here is where i start
(3w-4) x w=96

could you give me an example

Answered by Stephen La Rocque.
6/x-2y - 15/x+y = 0.5 2006-12-21
From Emeka:
6/x-2y - 15/x+y = 0.5
Answered by Stephen La Rocque.
Prove that square root of 3 is irrational. 2006-12-21
From Humera:
Prove that square root of 3 is irrational.
Answered by Stephen La Rocque.
Skill Testing Question 2006-12-20
From Tonia:
Skill Testing Question:

Multiply five(5) x four (4)
Add twenty two (22)
Divide by two (2)
Subtract six (6)

Answered by Penny Nom.
Splitting A Circle Evenly 2006-12-20
From Joe:
I'm trying to make a game board and instead of having it square, I would like to give it a curve (the game is Parcheesi). The attached diagram is pretty much completed (done in AutoCAD). What I would like to know is how to manually find the points that intersect the red line. In other words, evenly split the semi-circle into 8 pieces.
Answered by Penny Nom.
A regular polygon inscribed in a circle 2006-12-19
From Katy:
If a regular hexagon is inscribed in a circle of radius 6.72 centimeters, find the length of one side of the pentagon. How would I got about explaining this?
Answered by Penny Nom.
Miles, square feet and acres 2006-12-18
From Emily:
How do you convert miles to square feet? I have to convert 2.86 miles to square feet and then convert 72.5 acres to square feet and add them together. I looked on your website and what does the word hence mean?
Answered by Penny Nom.
The slope-intercept form 2006-12-17
From Anthony:
I'm really confused with this, tried to figure it out on my own but i can't seem to grasp it here's the problem:
Two points are given
a. Write an equation in slope-intercept form for the line containing the two points.
b. Graph the line from Part a and check that both points are on the line.

Answered by Penny Nom.
The product of of two rotations 2006-12-17
From Katie:
Is the product of of two rotations over a different center point always a translation?
Answered by Walter Whiteley.
Can you characterize ALL polygons by the number of diagonals it has? 2006-12-17
From Katie:
Can you characterize ALL polygons by the number of diagonals it has?
Answered by Penny Nom.
How many gallons of water will it hold? 2006-12-17
From Lois:
I have a tank that is 48"long 26" wide and 21 " deep. How many gallons of water will it hold?
Answered by Penny Nom.
Th pitch of a roof 2006-12-15
From Rob:
I am trying to determine how much to add for roof pitch. I guess the best way would be to use a right triangle as an example.

The base of the triangle is always 12 inches. The leg pointing straight up is a variable. I will say 6" in this example. By changing the variable how can i determine how much longer the hypotenuse than the base expressed in percent or decimal.

Answered by Stephen La Rocque.
1, 3, 6, 10... 2006-12-15
From Lisa:
What is the nth term for the sequence, '1, 3, 6, 10...' and could you say how you got the anwser.
Answered by Stephen La Rocque and Penny Nom.
I want to convert milliliters into grams. 2006-12-14
From Kenchotenzin :
I want to convert milliliters into grams.
Answered by Penny Nom.
Place value, coins and money 2006-12-12
From Emily:
I was wondering what the best way would be to teach place value using coins and money. I have used base ten blocks already and am looking for a way to integrate money, perhaps on a place value chart and perhaps using decimals.
Answered by Diane Hanson.
The average of 3 tests 2006-12-12
From Charles:
If you average 84 on first three test and the first score was 67. The second score was 7 less than the third score. What was the score on the second and third test.
Answered by Stephen La Rocque.
Drilling holes in a cube 2006-12-12
From Liz:
What is the volume of the solid remaining if a unit cube has a hole drilled thru one face to the opposite side and the hole that is drilled is a cylinder of unit diameter. Then the cube is rotated and an identical hole is drilled from an untouched face to the opposite side. Then the cube is rotated and an identical hole is drilled between the last two untouched faces. This will result in the cube vertices and the remaining volume of the cube falling into 8 parts. What is the total volume of these 8 parts. How is this volume calculated?
Answered by Penny Nom.
The sum of the angles in a triangle 2006-12-12
From Becky:
Why does a triangle always equal 180 degrees? We know that all the angles add up to 180 degrees but cant find a valid reason for it.
Answered by Penny Nom.
Circles and polygons 2006-12-11
From Irene:
Can you define a circle as a polygon with an infinite number of sides which are infinitely small? If you can, can you then define a cylinder as a prism? The sides would be rectangles or parallelograms with one length infinitely small?
Answered by Chris Fisher.
The absolute value of imaginary and complex numbers 2006-12-11
From Keith:
i don't get how to find the absolute value of imaginary and complex numbers here is an examples from the text book the answers are given but they don't show the work so i can follow along just show me the work please and explain how it is done

problem
3+4i

Answered by Stephen La Rocque and Penny Nom.
1.26 billion 2006-12-10
From James:
how do you write 1.26 billion
Answered by Penny Nom.
Parallel and perpendicular lines 2006-12-07
From Valeria:
I m having trouble with parallel and perpendicular lines a problem i need help with is y=6 find the slope if al line parallel to the graph of each equation
Answered by Stephen La Rocque.
Rolle's Theorem 2006-12-07
From Erika:
If f(x) = (x^2)(square root of [3-x]) on the interval [0,3] is given, Does Rolle's Theorem apply? If yes, find any values of c such that f '(c)=0
Answered by Penny Nom.
lim x-->infinity cos x 2006-12-07
From Katie:
I was wondering if it was possible to find: lim x-->infinity cos x
Answered by Stephen La Rocque.
Fill in the signs 2006-12-07
From Ujwala:
4$4$4$4=20 Fill in the signs in the place of dollor symbol.
Answered by Stephen La Rocque.
The proof of inequality by mathematical induction 2006-12-07
From Carol:
S(n) = 2^n > 10n+7 and n>=10
Answered by Stephen La Rocque.
The square footage of an irregular lot 2006-12-07
From Lori:
If I have a rectangular irregular sized lot with the following:

A - 110.53'
B - 159.87'
C - 127.28'
D - 165.86'


How do I determine the square footage?

Answered by Penny Nom.
A 15% tip 2006-12-06
From Danielle:
When she eats at a restaurant,Viviana like to leave a 15% tip,multiplying the price of the meal by 0.15. Franklin usually leaves a 20% tip,multiplying the price by 0.20.They both round up to the nearest $0.05. How much tip would Viviana leave for a $24.85 meal?
Answered by Penny Nom.
The volume of an irregular tetrahedron 2006-12-06
From Hai Van:
Could you please show me the way to calculate the volume of an irregular tetrahedron
Answered by Penny Nom.
The length of each side of a regular n-gon 2006-12-06
From Shannen:
The problem is: Find the length of each side of a regular n-gon when a=80ft, n=20ft, and A is approx. 20,000 square feet.
What do "a" and "n" stand for and how do I find the side length of an n-gon?

Answered by Penny Nom.
Star of Bethlehem 2006-12-05
From Stephen:
I need to construct a star for a Christmas play being done at our church. I want it to be a four-pointed star. The top and side points should be 12 inches long and the bottom point 24 inches. It will be flat on the back but raised on the front with lights inside for illumination. Each point will be made of two triangles with a ridge in the center. I would like the width of the rays at the center (widest point or center of star) to be about five inches and the center ridge at this same point to be raised about two inches from the flat back. I need to know the dimensions and angles of each of the 8 triangles that make up the star so that all of them meet properly at the center of the star so as to have the correct height from the flat back.
Answered by Penny Nom.
27 snakes in four cages 2006-12-05
From Sherril:
A zookeeper has to put 27 snakes in four cages. His problem is that he must have an odd number of snakes in each cage. How can he accomplish this? You can put any number of snakes in a cage as long as the total number of snakes in each cage is an odd number.
Answered by Stephen La Rocque and Penny Nom.
The area of a 20 sided polygon 2006-12-02
From Kate:
could you please tell me the strategy of how to work out the area of a 20 sided polygon
Answered by Stephen La Rocque and Penny Nom.
Area of a circle 2006-12-01
From Lisa:
i have 24m of fence how would i find out the area of a circle using this 24m of fence?
Answered by Stephen La Rocque.
Linear inequalities 2006-12-01
From Susan:
how would I graph a linear inequality of y>3.
Answered by Stephen La Rocque.
The height of a triangle 2006-12-01
From Jeni:
I need to know how to figure out the height of a triangle when all I know are the sides and base. The sides are 5 and the base is 8 What do I do?
Answered by Penny Nom.
A scale model 2006-11-30
From Adrianna:
1) A model of a storage compartment is to a scale of 1:40. If the surface area of the model is 25 square centimeters, what is the surface area of the actual storage compartment?

a) The storage compartment in the last question has an actual volume of 8 cubic metres. What is the volume of the model?

Answered by Penny Nom.
Sectors of a circle 2006-11-30
From Maithreyi :
A circle of diameter 21m is divided into three sectors with central angles 60degree,120degree and 180degree. Find the area of each sector?
Answered by Stephen La Rocque.
A Norman window 2006-11-30
From Joe:
a norman window is a rectangle with a semicircle on top. If a norman window has a perimeter of 28, what must the dimensions be to find the maximum possible area the window can have?
Answered by Stephen La Rocque.
I have to design a lettering on corrugated plate 2006-11-29
From Tim:
For a client of mine I have to design a lettering on corrugated plate. The lettering has to appear "normal" when viewed from the front (100%) I am so far now to consider the shape of the plate to be two halves of an oval/ellipse. What I would like to know is a way to calculate the percentage in which I have to "stretch" my design in order to let it appear "normal".
Answered by Stephen La Rocque and Penny Nom.
BEDMAS? 2006-11-29
From john:
I won a contest and they have a confusing format for the STQ. here it is:
11 (+) 44 (x) 2 (-) 10 (%) 25
I've never seen brackets around the operands and wondering how to treat it. I fear they want me to answer the question left to right as the answer is 4 if I use BEDMAS I get 98.6 (which clearly sounds wrong for what they want)
any thoughts?

Answered by Stephen La Rocque.
A triangle problem 2006-11-29
From Ann:
Triangle FUN has A) 72.8
B) 65.2
C) 72.7
D) 65.3

Answered by Chris Fisher.
What is the expected number of ripe and ready to eat watermelons 2006-11-29
From James:
An agricultural cooperative claims 95 percent of the watermelons shipped out are ripe and ready to eat. If 20 watermelons are shipped out , what is the probability that the number of watermelon that are ripe and ready to is (i) exactly 14 (ii) more than 18 (iii) of the 20 watermelons that are shipped what is the expected number of ripe and ready to eat watermelons
Answered by Penny Nom.
Creating a triangle in a circle 2006-11-28
From Dirk:
My daughter has a school project where she must draw a circle and then draw an equilateral triangle inside the circle. She said you have to identify six points on the circle to correctly draw the triangle. How do you accomplish this?
Answered by Penny Nom.
What proportion of cups of coffee have at least one defect 2006-11-28
From Ayman:
coffee machine may be defective since it dispenses the wrong amount of coffee (C) & or the wrong amount of suger ( S ) the probability of these defects are P(C) = 0.05, P(S) = 0.04 & P( C&S ) = 0.01 find
1- what proportion of cups of coffee have at least one defect
2- what proportion of cups of coffee have no defect

Answered by Penny Nom.
solve; 2006-11-28
From Monique:
solve; f(f(x)) = 2x + 4 ; if f(x) = 1
Answered by Penny Nom.
2x > -6 and x - 4 < 3 2006-11-27
From Alyssa:
2x > -6 and x - 4 < 3 , i need to know how to solve it and how to graph it.
Answered by Stephen La Rocque.
A spherical bowl 2006-11-26
From Lynne:
A spherical bowl of internal radius 15cm is partly filled with liquid. If the radius of the horizontal liquid surface is 12cm, calculate (taking the value of pi to be 3.142), the height of this liquid surface above the center of the bowl.
Answered by Penny Nom.
A problem involving logs 2006-11-26
From Beth:
any help would be appreciated on how to solve without using the change of base formula for logarithms in the solution and check of the solution!!!

log256 (x) + log16 (x) + log4 (x) + log 2 (x) = 7/4

Answered by Stephen La Rocque and Penny Nom.
The area of a semicircle 2006-11-25
From Melinda:
If the radius of a semi-circle is 9ft what is the area?
Answered by Karen McIver and Penny Nom.
How many licence plates are possible? 2006-11-24
From Naomi:
The state of New York has licence plates with five numbers followed by either a letter A or a letter B (eg. 36457A ,59972B ). How many licence plates are possible?
Answered by Penny Nom.
(x - 5)/3 - 3/2 = (4x + 7)/6 2006-11-23
From Terri:
I have never had algebra before and I am finding it difficult.Can you please help me work this problem out?

(x - 5)/3 - 3/2 = (4x + 7)/6

Answered by Penny Nom.
Dimensions of a rectangle 2006-11-22
From Terri:
How can you find the dimensions of a rectangle that has a perimeter of 18 feet?
Answered by Stephen La Rocque.
A right angled triangle 2006-11-22
From Siddhartha:
area of a right angled triangle OAB where OA = 5 and OB = 3
Answered by Penny Nom.
Building a corral 2006-11-22
From Dennis:
I'm building a corral and need to know how many holes 6"x24'' in a yard of concrete
Answered by Stephen La Rocque and Penny Nom.
Sally plays roulette 2006-11-21
From Sherry:
Sally plans to bet 100 bets of $1.00 each on red roulette. the probability of the ball landing on red is 18/38 what is the probability that Sally will win at least half the time
Answered by Penny Nom.
The Fibonacci sequence 2006-11-21
From Ross:
Let f0 = 0; f1 = 1,... be the Fibonacci sequence where for all n greater than or equal to 2 fn = fn-1 + fn-2. Let Q = (1+square root of 5)/2. Show that for all positive n greater than or equal to 0, fn less than or equal to Q^(n-1).
Answered by Penny Nom.
Pick any prime number greater than 3,square it ,then ... 2006-11-20
From Eliseo:
I was ask to pick any prime number greater than 3,square it ,then subtract 4, then divide the new result by 12 and record the remainder. He told me the remainder was 9. How could he be sure that the remainder was 9 without knowing which prime I picked?
Answered by Stephen La Rocque.
A cyclic quadrilateral 2006-11-20
From Namrata:
If two sides of a cyclic quadrilateral are parallel, prove that
(1) remaining two sides are equal,
(2) both diagonals are equal.

Answered by Penny Nom.
Circle geometry 2006-11-19
From Namrata:
AB is a diameter and AC IS A CHORD OF A CIRCLE SUCH THAT angle=30. the tangent at C intersect AB produced in a pointD. Prove that BC=BD.
Answered by Stephen La Rocque.
The height of a building 2006-11-19
From Sweetie:
I have to figure out the measure of the water towers antenna on my schools campus using up to two items created by me and a manual. I don't know far away we are going to have to be. I need to have a fool proof way to figure out the distance to the tower. I have an idea using trigonometry but its really roughly estimated. Do you have any suggestions?
Answered by Stephen La Rocque.
Composition of functions 2006-11-19
From RJ:
Let f0(x) = 2/2-x and fn+1 = f0 o fn for n greater than or equal to 0. Find a formula for fn and prove it by mathematical induction. Recall that o represents function composition. i.e., (f o g)(x) = f(g(x)).
Answered by Stephen La Rocque.
Conic sections 2006-11-19
From Joyce:
My son has a project on conic sections. I need the following information on Parabola, Circle, ellipse,and hyperbola. He can't find the following information for each conic section: equations with explanations, four uses for each shape and Shape explanation.
Answered by Penny Nom.
Scale factors and ratios 2006-11-19
From Maggie:
I have a BIG math test coming up, and it's about scale factors and ratios, but i don't understand them at all. How can you tell the difference between scale factors and ratios? and How are they used in the everyday world?
Answered by Penny Nom.
Two trains left the station at the same time and traveled in opposite directions. 2006-11-18
From Tori:
Two trains left the station at the same time and traveled in opposite directions. The E train averaged 130 km/h. The A train's speed was 110 km/h. In how many hours were they 480 km apart?
Answered by Stephen La Rocque.
An aircraft and a missile 2006-11-18
From Sarah:
an aircraft is flying at a constant altitude with a constant speed of 600mph. an antiaircraft missile is fired on a straight line perpendicular to the flight path of the aircraft so that it will hit the aircraft at a point P. at that instant the aircraft is 2 miles from the impact point P the missile is 4 miles from P and flying at 1200 mph. at that instant, how rapidly is the distance between missile and aircraft decreasing?
Answered by Stephen La Rocque.
Wheat is poured on a conical pile 2006-11-17
From Rachel:
wheat is poured through a chute at the rate of 10 cubic feet per minute and falls in a conical pile whose bottom radius is always half the altitude. how fast will the circumference of the base be increasing when the pile is 8 feet high?
Answered by Penny Nom.
The volume of a box 2006-11-16
From Sam:
I have this box to mail and i would like to find out the cubic foot of the box.The W-52 -D-13- H- 37 inch cubic
Answered by Penny Nom.
Induction 2006-11-16
From John:
Find a formula for
1/(1x3)+1/(2x4)+1/(3x5)...+1/(n(n+2))
by examining the values of this expression for small values of n. Use mathematical induction to prove your result.

Answered by Penny Nom.
Converting cubic meters to gallons and liters 2006-11-16
From Lorne:
I have a large tank and I would like to know the formula for the conversion. And do you have any other info on web sites that have conversion tools as most of the tools are in u.s and I live in Canada and need the metric conversions.
Answered by Stephen La Rocque.
Find the distance function s(t) given the acceleration function a(t) 2006-11-16
From David:
Find the distance function s(t) of the acceleration function: a(t)= sin(0.1t)/ cos^3(0.1t)
Answered by Penny Nom.
An equation for a line 2006-11-16
From Jessica:
(-3,4) m=undefined write an equation for each line given the slope and a point on the line. show your work
Answered by Stephen La Rocque.
How can half of 12 be 7? 2006-11-16
From Liz:
How can half of 12 be 7?
Answered by Steve La Rocque, Chris Fisher and Penny Nom.
What is the highest number possible with 3 digits? 2006-11-16
From Liz:
What is the highest number possible with 3 digits? The teacher said the answer is not 999 and that the question did not say "3-digit number." My answer then is 9 to the 99th power (written mathematically of course) as this uses "3 digits" and is obviously a huge number. Is this correct?
Answered by Stephen La Rocque.
"Less than" and "less than or equal to". 2006-11-15
From Ross:

7 ≤ 10 and 7 ≤ 7 are both true statements.

How can 7 be equal to 10, and 7 be less than 7? The book doesn't explain WHY!


Answered by Penny Nom.
The grade of a highway 2006-11-14
From Nancy:
If a portion of highway has a grade or slope of 12%, and the length of this portion is approximately 1.1 miles, what is the change in elevation from the top of the grade to the bottom of the grade in feet?
Answered by Stephen La Rocque and Penny Nom.
How to show my work when I simplify this 2 - 2[4 - 9] ÷ (-5)? 2006-11-14
From Remada:
How to show my work when I simplify this 2 - 2[4 - 9] ÷ (-5)
Answered by Penny Nom.
Motorcycle expansion chamber design 2006-11-14
From David:
I'm interested in calculating cone information regarding motorcycle expansion chamber design for example. I guess it's called a truncated cone, from what I've read so far. If I know the center line height, small radius, and large radius of a truncated cone then, how can I calculate the angle (included angle?) the cone forms? I'd like to know the variations of the formula so I can calculate for angle, or length, or one of the diameters if I know the other two measurements.
Answered by Stephen La Rocque.
How high is the satellite above the earth? 2006-11-13
From Naomi:
If the linear speed of a satellite in synchronous orbit is 1000 mi/h, how high is the satellite above the earth?
Answered by Stephen La Rocque.
Apples and plums 2006-11-13
From Saif:
the problem i am stuck with is, nicole buys 2.3kg apples 1.8kg plums she pays £7.18 total plums cost £2.20per kg cost 1kg apples what is the cost of 1kg of apples
Answered by Stephen La Rocque.
The name of an equation 2006-11-13
From Lee:
I am struggling to find the name of the following equation. I remember watching a TV programme introduced by Arthur C Clarke. The programme focused on an equation that gave a wonderful pattern that went on into infinity. Two maths professors discussed this equation.It was described as 'Gods equation' as it was compared to the shape of trees and other shapes in nature. It was named after the inventor/discoverer. If you have any ideas i would be most grateful.
Answered by Penny Nom.
Some applications of conic sections 2006-11-13
From Burt:
how are circles, ellipses, and hyperbolas used in everyday life
Answered by Penny Nom.
15g/50g^3 2006-11-12
From Nicole:
How do u simplify the variable expression? 15g/50g^3
Answered by Penny Nom.
ml to grams 2006-11-11
From Lisa:
We have 900mL and you have to multiply by 1.18 grams do you just multiply and get 1062 grams or am I missing a step?
Answered by Stephen La Rocque.
The name of a shape 2006-11-11
From Valentina:
I have a shape of 4 sides 3 are the same length (2cm each) and the other smaller (1.5cm)
Answered by Chris Fisher.
A trillion sugar cubes 2006-11-10
From Leeza:
How many dump trucks (I believe the standard bed size is 16'L x 8'W x 4'D) would it take to hold one trillion sugar cubes (which I believe are approximately 2cm in L, W and D)?
Answered by Penny Nom.
After how many hours will the class have been notified? 2006-11-10
From Brittany:
A college math class consists of 32 students. On Monday at 9:00 a.m., the teacher tells one student to notify other students that the test scheduled for Wednesday at 9:00 a.m. has been cancelled. The model for the number of students in the class who will have heard this information after t hours is N=32-32e^-0.02t. After how many hours will the class have been notified?
Answered by Stephen La Rocque.
Tangent lines 2006-11-09
From Melissa:
let f be a function with f(1)=4 such that for all points (x,y) on the graph of f the slope is given by (3x^(2)+1)/(2y)

a.)Find the slope of the graph of f at the point where x=1.
b.)Write an equation for the line tangent to the graph of f at x=1 and use it to approximate f(1.2)
c.) Find whether f is concave up or concave down when x=1. Is your answer in part b an overestimate or an underestimate?

Answered by Stephen La Rocque.
Least common multiple 2006-11-08
From Melenie:
I have a daughter in 5th grade she can't seem to grasp the concept of finding LCM's does anyone have a quick easy way for her to learn?????
Answered by Stephen La Rocque.
The slope-intercept form 2006-11-08
From Kilihea:
I need to graph this equation but first need to turn it into a slope-intercept form. 6x - 3y = -3 I would really appreciate it if you could help me turn it into a slope-intercept form.
Answered by Penny Nom.
A container to hold 600 gallons of water 2006-11-07
From Cissy:
What size rectangular tank would I need to hold about 500-600 gallons of water.
Answered by Penny Nom.
Simplifying exprressions 2006-11-06
From Jessica:
-21(a+2b)+14a-9b
Answered by Stephen La Rocque.
A proof by induction 2006-11-06
From Zamira:
i have a problem with this mathematical induction: (1^5)+(2^5)+(3^5)+...+(n^5) = ((n^2)*((n+1)^2)*((2n^2)+2n-1))/12
Answered by Penny Nom.
A melting snowball 2006-11-06
From Jay:
A snowball melts at a rate proportional to its surface area. Show that its radius shrinks at a constant rate. If it melts to 8/27 of its original volume in 20 minutes, how long will it take to melt completely? Please I need your help.
Answered by Stephen La Rocque.
Simultaneous equations 2006-11-06
From An other:
e^2y-x+2=0
ln(x+3)-2y-1=0

Answered by Penny Nom.
Direct variation 2006-11-05
From Johann:
The perimeter of regular octagon varies directly with the length of one side of the octagon
Answered by Stephen La Rocque.
A mixture problem 2006-11-05
From Lynne Rae:
How many kg of 30% salt solution by mass and 40% salt solution by mass should be mixed to form 200kg of 37% salt solution by mass?
Answered by Penny Nom.
Bill has four more dimes than nickels ... 2006-11-05
From Allison:
bill has four more dimes than nickels, and seven fewer nickels than pennies.he has a total of $3.35. how many of each kind of coin has he?
Answered by Karen McIver.
Newton meters and ft-lbs 2006-11-04
From Michael:
What is the correlation between Newton meters and ft-lbs of torque? If an engine has 100ft-lbs of torque,what would that equate to in Newton meters?
Answered by Stephen La Rocque.
4/7x+3=15 2006-11-04
From Ashley:
Solve 4/7x+3=15
Answered by Penny Nom.
ESL and math-speak 2006-11-03
From Matt:
My name is Matt and I am an ESL teacher in Korea. Recently I have begun tutoring a student who will be going to Canada soon as a 9th grade student. She is a very capable student and is concerned that her mathematical abilities outstrip her ability to communicate in English. I am writing mainly to inquire what level of math-speak I should teach to this student. For instance: should I be explaining geometric terms, or simply algebra? And if algebra, should I include complex numbers and the quadratic formula? If there are things that she will be taught in Canada, I'd prefer her to learn them firsthand from a math teacher. However, knowledge that she will be presumed to have I would like to teach.
Answered by Karen McIver.
The sum of two numbers is 30 2006-11-03
From GG:
The sum of two numbers is 30 and the product of the same two numbers is 221. Find the two numbers.
Answered by Stephen La Rocque.
y = |x| - 10 2006-11-03
From Pam:
what is the process for finding the vertex point for an equation such as this, y= |x| - 10.
Answered by Penny Nom.
Where did the extra unit come from? 2006-11-03
From Daniel:
There are two triangles each with a base of 8, a height of 3 and an area of 12 units squared and two trapezoids, each with a one parallel side that is 3 and one that is 5 and a height of making it have an area of 20 units squared. When arranged in a 8x8 square then the total area is 64 units squared, but when arranged in a 5x13 rectangle the total area is 65 units squared, where did the extra unit come from?
Answered by Haley Ess.
3x+4y+7x+9+2 2006-11-03
From Chavon:
3x+4y+7x+9+2 I don't get it please help
Answered by Stephen La Rocque.
Kilowatts and Horsepower 2006-11-03
From Michael:
How many Kilowatts=1 Horsepower or Horsepower=1 Kilowatt?
Answered by Stephen La Rocque.
A dinner group of 16 couples 2006-11-02
From Nancy:
This is a real-world problem. I should know the answer but I don't. A friend is starting a dinner group of 16 couples, to be distributed across 4 houses. Each month she wants to have a different set of host houses (no problem) AND she doesn't want repeats among pairs. That is, if the Smiths and Joneses are together one month, they should not be together another month until all the combinations have been exhausted. How many valid combinations are there? Is there a formula that I can convert into a computer program? I will have to get the names from my friend and give her back the combinations as she would not be able to deal with the math formulation.
Answered by Denis Hanson.
The dimensions of a rectangle 2006-11-02
From Ben:
A rectangle's length is 8 cm. greater than its width. Its perimeter is 128 cm. Find its dimensions.
Answered by Penny Nom.
Chord length 2006-11-01
From Tony:
how do i find a chord length when all i have is arc length (13.633m) and the radius (89.500m)
Answered by Stephen La Rocque.
How many M&Ms are in a container? 2006-11-01
From Jill:
How do we algebraically figure out how many M&Ms are in a container?
Answered by Stephen La Rocque and Penny Nom.
What is the probability that the sum of the two numbers is even? 2006-11-01
From Ryan:
10 balls numbered 1-10 are in a jar. George reaches into the jar and pulls out a ball. Then Wally pulls out another ball. What is the probability that the sum of the two numbers is even?
Answered by Haley Ess.
The area of a quadrilateral 2006-10-31
From Georgina:
I would really like to know and be shown how to figure out the area of a quadrilateral. I already know that it is side x matching height but i need to be shown what to do. Could you please help?
Answered by Penny Nom.
Paddeling on the Alleman River 2006-10-31
From Anita:
Len is planning a three-hour trip down the Alleman River and back to his starting point. He knows that he can paddle in still water at 3 mph and that the rate of the current is 2 mph. How much time can he spend going downstream? How far downstream can he travel?
Answered by Penny Nom.
3.253 cubic meters of water 2006-10-31
From Elaine:
a tank full of water that has a volume of 3.253 cubic meters would have a mass of?
Answered by Penny Nom.
Pulleys and belts 2006-10-31
From Lynn:

A belt has a linear speed of 32 mph. It is spinning three pulleys.
Pulley A has diameter 14 in. radius = 7 inches
Pulley B has diameter 26 in. radius = 13 inches
Pulley C has radius 3 ft. radius = 36 inches

Angular speed in Radians per min for
I got
Pulley A= .08 rads/min Pulley B= .04 rads/min Pulley C=.01 rads/min

How many cm would pulley C travel in 14 seconds?


Answered by Penny Nom.
Induction 2006-10-31
From Ross:
Suppose that A and B are square matrices with the property AB= BA. Show that AB^n = B^n A for every positive integer n.
Answered by Stephen La Rocque and Penny Nom.
The area of an irregular hexagon 2006-10-30
From Ashish:
I want to find out area of an irregular HEXAGON in square feet, whose sides are as follows,
Answered by Stephen La Rocque.
The surface area of a triangular prism? 2006-10-29
From Amanda:
What is the formula for the surface area of a triangular prism?
Answered by Haley Ess.
The real numbers with decimal representations consisting of all 1s. 2006-10-29
From Ivessa:
Determine if the following set is countable or uncountable : the real numbers with decimal representations consisting of all 1s.
Answered by Steve La Rocque and Walter Whiteley.
The distance from a point to a line in 3 space 2006-10-29
From Marie:
find the distance from the point (1, -2, -3) to the line x = y = z-2
Answered by Penny Nom.
An octagonal deck 2006-10-29
From R.M.:
I am building an octagon deck and I want to know what length to cut the outside boards.The overall is 12ft. wide.
Answered by Penny Nom.
How much labor should the firm employ? 2006-10-28
From Christy:
A dressmaking firm has a production function of Q=L-L(squared)/800. Q is the number of dresses per week and L is the number of labor hours per week. Additional cost of hiring an extra hour of labor is $20. The fixed selling price is P=$40. How much labor should the firm employ? What is the resulting output and profit? I am having a difficult time with this, HELP!
Answered by Stephen La Rocque.
LCM and HCF 2006-10-28
From Henry:
Is there a unique solution to the question: If the LCM and HCF of two numbers are 180 and 15 respectively, what are the two numbers? I got 45 and 60. I got a feeling there are others.
Answered by Stephen La Rocque.
Balancing equations 2006-10-26
From Della:

With the following equations, how do you know whether you need to apply the multiplication or division rule of equality?

75% t = 18

120% w = 72


Answered by Penny Nom.
Ratios, proportions and medicine 2006-10-25
From Steven:
I don't understand why we need to learn about ratios and proportions because i want to become a doctor and i don't know if i'll ever use this type of math.
Answered by Penny Nom, Claude Tardif and Walter Whiteley.
3-6+4X3= 2006-10-25
From Tom:
3-6+4X3=
Answered by Penny Nom.
(2/3)^2/(4/2)/1/3 2006-10-25
From Jena:
how do i simplify this problem? (2/3)^2/(4/2)/1/3
Answered by Stephen La Rocque.
How much smaller is this one in percentage? 2006-10-24
From Michael:
I have two different squares. One of them is smaller than the other. I need to know how to calculate how much smaller it is in percentage. 11.95mm x 6.72mm 11.2mm x 6.3mm How much smaller is this one compared to the one above in percentage?
Answered by Haley Ess.
Water is being pumped into the pool 2006-10-24
From Jon:
A swimming pool is 12 meters long, 6 meters wide, 1 meter deep at the shallow end, and 3 meters deeps at the deep end. Water is being pumped into the pool at 1/4 cubic meters per minute, an there is 1 meter of water at the deep end.

a) what percent of the pool is filled?

b) at what rate is the water level rising?

Answered by Stephen La Rocque.
Simplifying radicals 2006-10-23
From Christin:
i would like for you to solve this problem?? 6 square root 8
Answered by Stephen La Rocque.
Octagonal Stop Sign 2006-10-23
From Phil:
My colleagues and I are in a discussion about how many true "sides" an octagonal stop sign has. I say that it has 10 true sides because of the front back sides of a stop sign (3D octagon). Is this correct terminology or should the front and back "sides" be referred to as faces -- equaling 2?
Answered by Paul Betts, Steve La Rocque and Harley Weston.
Ridingcap's daughter 2006-10-23
From Marcus:
Ridingcap's daughter,whose real name was Coleen, set out for grandma's house again. This time she was delivering jam and jellies. First she stopped to see Mrs. Parsley,who took in grandma's mail while grandma was @ the hospital recovering from the wolf. Colleen gave Mrs. Parsley one third of the jars in her basket. Then Colleen set out again,and suddenly the wolf jumped out behind the bush and grubbed one half of the jars in Colleen's basket. When the wolf ran off, Colleen went on and gave Mr .Woodsman two thirds of what was left in her basket. Colleen had six jars of jam and jellies left for grandma. HOW MANY JARS DID COLLEEN LEAVE HOME WITH
Answered by Stephen La Rocque.
Concrete tubes 2006-10-23
From Rick:
I have 36 18” concrete tubes to fill with 36” of concrete. How much concrete will it take to do this job
Answered by Stephen La Rocque.
Word equations 2006-10-22
From Samira:
One half of a number is subtracted from the sum of one-third and one-quarter of the same number. If the result is 16, find the number.
Answered by Stephen La Rocque.
Word problem 2006-10-22
From Sabrina:
Thomas bought a badminton racquet and paid for it in quarters. If Thomas had used nickels and dimes, he would have needed 20 more dimes than quarters and one more than twice as many nickels as quarters. How much much did the badminton racquet cost?
Answered by Stephen La Rocque.
The area of a regular octagon 2006-10-22
From Farhana:
I have a regular octagon with sides of 0.8m and I need to find the area,
Answered by Karen McIver and Penny Nom.
An approximation 2006-10-22
From Ellen:
consider the curve -8x^2 +5xy+y^3 +149 =0 Write an equation for the line tangent to the curve at (4, -1) use this equation to approximate the value of K at the point (4.2, K)
Answered by Penny Nom.
The height of a triangle as a function 2006-10-19
From Ryan:
Let 2s denote the length of the side of an equilateral triangle. Express the height of the triangle as a function of s
Answered by Penny Nom.
How do we use fractions in every day life? 2006-10-19
From Tori:
I am doing fractions in class and we need to write a paper about them. So my question is:"how do we use fractions in every day life?"
Answered by Claude Tardif.
How fast is the water level rising when the water is 1 meter deep? 2006-10-19
From Don:
The cross section of a 5-meter trough is an isosceles trapezoid with a 2-meter lower base, a 3-meter upper base and an altitude of 2 meters. Water is running into the trough at a rate of 1 cubic meter per minute. How fast is the water level rising when the water is 1 meter deep?
Answered by Stephen La Rocque.
An octagonal room 2006-10-18
From Rick:
I am a cabinetmaker, and I have a client who has an octagonal room which he would like to be used as a walk-in closet. Each side of the octagon is 60" in length. Each wall section is to have a 24" deep cabinet installed on it. I am trying to figure out the width of each cabinet allowing for a 3" space between cabinets at the front corner.
Answered by Stephen La Rocque.
Putting things in exponent form 2006-10-18
From Sheila:
Write in exponential form 2 sq. root a with an exponent 5.
I do not understand on how to do it.

Answered by Stephen La Rocque.
Lattice multiplication 2006-10-16
From Patti:
I am a parent of a 5 th grader, and am a bit loss in the "new" math. I know how to multiply using the "old fashion" method. But, could you please explain lattice multiplication.
Answered by Diane Hanson.
A trillion seconds? 2006-10-16
From A student:
How many years, days, hours, minutes, and seconds in a trillion seconds?
Answered by Stephen La Rocque.
How do you find the length and width of a rectangle if you know ... 2006-10-15
From Anne:
How do you find the length and width of a rectangle if you know the perimeter of the rectangle is 48 m and the area is 144 m squared?
Answered by Stephen La Rocque and Penny Nom.
How do you solve for variables in the denominator? 2006-10-15
From Donna:
How do you solve for variables in the denominator?

178 = 17/R
Answered by Stephen La Rocque.

how many ounces are in 750 cc's. 2006-10-14
From Not a teacher:
how many ounces are in 750 cc's.
Answered by Stephen La Rocque.
How many feet tall is the tunnel 2006-10-14
From Charles:
A tunnel in the Smoky Mountains is semicircular. At a distance of 12 feet from the center of the tunnel, the tunnel has a height of 16 feet. How many feet tall is the tunnel at its center?
Answered by Penny Nom.
A specific 8 digit number in a 11 digit mobile phone number 2006-10-13
From James:
I need to know the chance of finding a specific 8 digit number in a 11 digit mobile phone number. the number must be 29571596 and the 11 digit phone number must end in this. All numbers 0-9 are used in a mobile number but the first 2 digits will always be 07. Is there a way to work this out??
Answered by Stephen La Rocque.
A problem involving a quadrilateral 2006-10-13
From Jeff:
In the quadrilateral above, AB=AC=AD. If angle BCA = 65, then x=???
Answered by Penny Nom.
Replace the blanks with + or - to make the statement correct 2006-10-13
From Molly:
Hi my name is molly, and I have a question I can't answer and need your help. I am a student. This is the qustion:
Try to replace the blanks below with + or - to make the statement correct
1_2_3_4_5_6_7_8_9 = 1

Answered by Claude Tardif.
Solving a linear equation 2006-10-12
From Eriona:
6p-5=17. I have to solve the equation.
Answered by Penny Nom.
Place value 2006-10-12
From Sue:
We have extended the place value chart out 14 places and want to know what comes after hundred trillions?
Answered by Stephen La Rocque.
Determine the width and height 2006-10-12
From Roshawn:
If I am given the length of 73" and the girth of 11" and am trying to determine the width and height. How would I do this?
Answered by Stephen La Rocque.
How many 1 square 3/8 thick pieces of steel can you get from one cubic foot of steel? 2006-10-12
From Jana:
How many 1 square 3/8 thick pieces of steel can you get from one cubic foot of steel?
Answered by Stephen La Rocque.
The area of regular pentagon inscribed in a circle 2006-10-12
From Admire:
i need help on how to find area of regular pentagon inscribed in a circle of radius 8cm
Answered by Stephen La Rocque and Penny Nom.
How do you write a variable expression 2006-10-11
From Josh:
How do you write a variable expression for the following:

The amount of money Waldo has if he has $10 more than Joe and

How much weight Kirk can lift if he lifts 30 lbs more than his brother.

Answered by Stephen La Rocque.
Is a circle a regular shape? 2006-10-10
From David:
Is a circle a regular shape or an irregular shape?
Answered by Stephen La Rocque.
I have a 3000 sq ft house ... 2006-10-10
From Ann:
I have a 3000 sq ft house and need to use flea foggers. Each fogger treats 6000 cubic feet. How many do I need?
Answered by Stephen La Rocque.
Plotting a linear equation 2006-10-09
From Jennifer:

I am having problems explaining how to do these problems to my child .

graph:2=x-y

graph the equation: 2y+x=-3


Answered by Penny Nom.
Metric conversion 2006-10-08
From Eve:
45.6 microliter to megaliter?
Answered by Stephen La Rocque.
The area of an oval 2006-10-08
From Bruce:
area of an oval: 60" x 120"
Answered by Stephen La Rocque.
Fill in the blanks 2006-10-04
From Justin:
1. To find out about how much, you can-------------
2. The --------- ---------states that the sum is the same no matter who you group the addends.

Answered by Stephen La Rocque.
The volume of a hexahedron. 2006-10-04
From A student:
How do i compute volume of a generic hexahedron with all its faces being quadrilateral?
Answered by Penny Nom.
Filling a swimming pool 2006-10-04
From Jon:
what would be the formula to fill a swimming pool in with dirt that is 15ft wide,36ft long and is 8ft deep to 3ft deep
Answered by Penny Nom.
Least common denominator 2006-10-04
From Paulette:
Why is the LCD of 3/4 and 4/8 not the product of 4 and 8?
Answered by Penny Nom.
Angular speed 2006-10-04
From Jack:
A horse on a merry-go-round is 7m from the center and travels at a speed of 10km/h. What is its angular speed?
Answered by Stephen La Rocque.
How many line segments are necessary? 2006-10-04
From Varun:
If you place 35 points on a piece of paper so that no three are collinear, how many line segments are necessary to connect each point to all the others?
Answered by Stephen La Rocque.
Divisibility by 9 and 11 2006-10-04
From Prakai:
can 818991 divisible by 9, or 11?
Answered by Penny Nom.
Inequalities 2006-10-03
From Maleeha:
a)Use algebra to find the set of values of 'x' for which x(x-5)>36.
b)Using your answer to part a, find the set of values 'y' for which y2(y2-5)>36

Answered by Penny Nom.
A 5x2 rectangle 2006-10-03
From Stanley:
Cut a 5x2 rectangle into 4 pieces, and then put them back into as a square.
Answered by Claude Tardif and Penny Nom.
1_._ x _._ = _ _.5 _ 2006-10-03
From Harold:

1_._ x _._ = _ _.5 _

use numbers 2-9 to fill in the blanks.


Answered by Claude Tardif.
A concrete foundation 2006-10-02
From A homeowner:
I am pouring a concrete foundation 8" X 14" X 23'. How many cubic feet of concrete do I need?
Answered by Penny Nom.
Two word problems 2006-10-02
From Eve:

1. A surveyor's map shows a plan for the rectangular rose garden whose area is
a2+25ab-350b2. Find an algebraic expression for the length and width. If a=20 ft and b=10 feet, find the actual dimensions of the garden.

2.A square is enclosed in a circle. The area of the square is (4r2-32r+64)sq cm and the area of the circle is 484r2 sq cm. Write a polynomial in factored form to represent the difference of the two areas.


Answered by Stephen La Rocque.
The hypotenuse 2006-10-02
From Ashley:
How do you find the hypotenuse of a right triangle? I don't understand how to find c.
Answered by Stephen La Rocque.
The area of a circle 2006-10-02
From A student:
Calculate the area of a circle whose radius is 75 meters?
Answered by Stephen La Rocque.
What is 1.7142857143 as a fraction? 2006-10-01
From Lizzy:
What is 1.7142857143 as a fraction?
Answered by Stephen La Rocque and Penny Nom.
The focus of a parabola 2006-10-01
From Lily:
I have a mathematical assignment which includes applications of parabolas, hyperbolas and ellipses in the real world. I have been searching the internet and now I am ware that most of the applications of parabolas have a connection with what people call "the focus". However, I do not think I clearly understand what "the focus" of a parabola is. Would you please explain it to me?
Answered by Penny Nom.
Prove that 2nCn is less than 4n, for all positive integers n? 2006-10-01
From Anna:
How can I prove that 2nCn is less than 4n, for all positive integers n?
Answered by Penny Nom.
30+7(x-1)=3(2x+7) 2006-09-29
From Art:
30+7(x-1)=3(2x+7)

I know with the distributive law I multiply 7 by x and 7 by -1 and the same thing to the right, only 3 by 2 and 7 than I am lost?
Answered by Penny Nom.

The volume of a cube 2006-09-29
From Greg:
Nicolai said that 6 billion can represent the volume of a cube. How many units long is the cube? How do you know?"
Answered by Stephen La Rocque.
Family of lines 2006-09-29
From Sasha:
could a "Family of Lines" have both parallel and intersecting equations that aren't related directly but otherwise related by each other to create a curve
Answered by Chris Fisher.
Computing gravity 2006-09-27
From Patricia:
What is the gravity of planet z in inch/hr squared if the weight of an element with mass of 45000 oz on planet z is 2N?
Answered by Stephen La Rocque.
A volume 2006-09-27
From jc:
If I have to pay storage charges on 45 cu ft. What size is that area L X W X H
Answered by Stephen La Rocque.
X^1/3-X^1/9=60 2006-09-26
From Harini:
I need the answer for

The value of X in X^1/3-X^1/9=60.
Answered by Penny Nom.

We need to know how many cubic yards of concrete to order for the job. 2006-09-25
From Linda:
We are pouring an additional drive way and shop in our back yard. The total square foot of all the concrete is 1200 and the concrete will be 3" deep. We need to know how many cubic yards of concrete to order for the job.
Answered by Penny Nom.
An expansion and a translation 2006-09-25
From meghan:

Write the equations for f(x) = squareroot(4 - (x - 2)^2) after:

a) a horizontal expansion by a factor of 2
Answer: f(x) = squareroot(4 - (1/2x - 2)^2)
b) a horizontal translation 3 units left
Answer: f(x) = squareroot(4 - (x + 1)^2)
c) the expansion in part a), then the translation in part b)
d) the translation in part b), then the expansion in part a)

I understand how to do a) and b), but I'm not sure what I'm supposed to do for the equations in a specific order (expansion, then translation vs. translation, then expansion).


Answered by Penny Nom.
Least common multiple 2006-09-25
From A student:
What is the LCM of 10,45 and 38?
Answered by Stephen La Rocque.
Equivalent fractions 2006-09-25
From Orlanda:
How do I write two equivalent decimals for 5.8?
Answered by Penny.
How many items must the company produce to begin to make a profit? 2006-09-24
From Devon:
The profit a manufacturing company makes can be found using the formula P=120n - n^2 - 2200 How many items must the company produce to begin to make a profit?
Answered by Stephen La Rocque.
a _________________ line forms a 90 degree angle with another line 2006-09-24
From Lisa:
a _________________ line forms a 90 degree angle with another line is this to it what fills in the blank? first letter is E third letter is R and has 13 letters
Answered by Stephen La Rocque.
Help me prove that the square root of 6 is irrational 2006-09-23
From Sylvia:
Could you please help me prove that the square root of 6 is irrational ?
Answered by Stephen La Rocque.
I need to cut an octagon 2006-09-23
From Freddie:
I have a 48 inch square piece of wood that I need to cut into an octagon, help. What's an easy way to just measure and cut it.
Answered by Penny Nom.
How do I determine the length of the lot? 2006-09-22
From Dylan:
If a rectangular lot's perimeter is 340 feet, the width is 70 feet. How do I determine the length of the lot?
Answered by Penny Nom.
An equilateral triangle has been wedged in between two circles. 2006-09-22
From Kim:

An equilateral triangle has been wedged in between two circles. How does the diameter of the smaller circle compare to the diameter of the larger circle.

image: circle inside of an equilateral triangle touching all sides of the triangle; both the triangle and the circle inside are placed into a larger circle where the triangle vertices all touch the circle


Answered by Penny Nom.
How long will it be before the two snails meet? 2006-09-21
From Brett:
Two snails are 8.5 feet apart. They start to crawl toward each other at the same time. One of them travels at the speed of 6 in. per minute, while the other zips along at 11 inches per minute. How long will it be before the two snails meet?
Answered by Penny Nom.
Linear systems 2006-09-21
From Alexander:
Hello, i have a math assignment due tonight at midnight and i need some help please

It is using the addition method to solve linear systems
Answered by Penny.

Constructing a cone 2006-09-20
From Suresh:
i want to know the required size of plate for cone rolling,sizes are 2950mm is bottom dia,894 is top dia and 600 is height.I have already read u r answers but i little bit confused ,harely and sue have given useful answers but when i worked both method the required plate size is different. so i like to know which method is easy and correct.and also i like know whether it can be rolled without segment my rolling machine width is 1500.
Answered by Penny Nom.
Cubic yards of sand 2006-09-20
From Michael:
How many compacted cubic yards of sand are there in 40 acres of land at a depth of 20 feet?
Answered by Stephen La Rocque.
How do I determine the acreage? 2006-09-20
From Charlene:
I have a survey of property with various points on it; how do I determine the acreage?
Answered by Penny Nom.
How much water it will hold? 2006-09-20
From Doug:
I have thirty feet of piping that is two inches in diameter and I want to know how much water it will hold and the equation for solving the answer.
Answered by Stephen La Rocque.
The length of the third side of a 45 degree Isosceles triangle 2006-09-20
From Rusty:
what is the formula to determine the length of the third side of a 45 degree Isosceles triangle?
Answered by Penny Nom.
How large is the property? 2006-09-20
From Jahn:
a triangular lot has 750 feet on one side of a 90-degree angle and 500 feet on the other side of the angle. how large is the property?
Answered by Penny Nom.
Equivalent fractions 2006-09-20
From Sarah:
I am the aunt of a 5th grader and we were doing math homework and the question was What is the equivalent decimal for .05 and 2.875? I am college education and was completely stumped. Please Help. Thought the question should have been what is the equivalent fraction.
Answered by Penny Nom.
Is this unsolvable? 2006-09-19
From Al:
Is this unsolvable???

2xy + 3x^2 - 3y + y^3 = 4x + 3 where y=3.
Answered by Penny Nom and Claude Tardif.

A rectangular tank 2006-09-19
From Milton:
What size rectangular tank will I need to hold 841.5 gallons of water. The water level is 3'-0" from the bottom of tank.
Answered by Penny Nom.
The square footage of my property 2006-09-17
From Jack:
I have been trying to calculate the square footage of my property, without success. The measurements are below. Is there a simple formula that can be used based upon the outside parameters? Or, is there a simple way to calculate the square footage?
Answered by Stephen La Rocque and Penny Nom.
Parameters 2006-09-15
From Chase:
What is the meaning of the word "parameters" when used in reference to Algebra.
Answered by Penny Nom.
The length of 2 sides of a triangle 2006-09-15
From Lonnie:
I need to know how to figure the length of 2 sides of a triangle, as the following example:

The length of the bottom is 12' and the angles are 45, 45 I need to know how long the other 2 sides must be to get an angle of 90 at the top.
Answered by Stephen La Rocque.

The area of a quadrilateral 2006-09-14
From A student:
How can i find the area of a quadrilateral?
Answered by Penny Nom.
Convert a standard form to a slope intercept form 2006-09-14
From Maiko:
I need to know step by step procedures to convert a standard form to a slope intercept form?
Answered by Penny Nom.
10,000 Million sandwiches 2006-09-12
From A business person:
One of our customers said that the UK sells "10,000 Million" sandwiches annually in their take away restaurants... How many is this exactly?
Answered by Stephen La Rocque.
Two characters 2006-09-12
From Robert:
I am looking to find out how many possible unique combinations I can have if at any time i can use only two characters of numbers or the alphabet...For ex., i can have 1-99 or i can have a1, a2, z1, z2 ... or i can have aa, ab, ac... i came up with possible combinations of 1295 or so... can you help?
Answered by Penny.
The volume of a triangular box 2006-09-12
From Irene:
How do I find the volume of a triangular box?
Answered by Penny Nom.
Square roots in a binomial expansion 2006-09-11
From Sydney:
(√x + 5)4 expanded using the binomial theorem
Answered by Penny Nom.
A square palindrome 2006-09-11
From Liz:
What is the least three-digit palindrome that is a square number?
Answered by Stephen La Rocque.
What was the total cost of the dinners he served 2006-09-10
From Connie:
Walter is a waiter at the town diner. He earns a daily wage of $100 plus tips that equal to .15 of the total cost of the dinners he serves. What was the total cost of the dinners he served if he earned $170 on Tuesday?
Answered by Stephen La Rocque.
How much did I spend on the first trip? 2006-09-10
From Alexander:
I have $600 to spend.
I take 10 shopping trips.
Each trip I spend $10 more than the last trip.

How much did I spend on the first trip?
Answered by Stephen La Rocque.

Your birthdays until your one hundredth 2006-09-09
From Alan:
which of your birthdays until your one hundredth, fall on the same day of the week as the day you were born?
Answered by Chris Fisher.
A hexagonal concrete form 2006-09-09
From Lewis:
How many cubic yards of cement will be required to fill a hexagonal concrete form with equal 12 foot sides that will be 3 feet thick?
Answered by Penny Nom.
Plotting co-ordinates on a number plane 2006-09-08
From Victor:
I need to plot co-ordinates (0,2), (2,-2), (-2,0), and (0,0) on a number or co-ordinates plane, I am not sure how to do this, and get lost after the first point, can you please help?
Answered by Penny Nom.
How much cement would I need? 2006-09-07
From Lou:
At a five to one mix how much sand/gravel and how much cement would I need for a 71/2 yard pour?

What formula would I use for this calculation. My sand/gravel is in a cone shaped pile and I know how to calculate that. I need to know if I have enough and how many bags of cement I need to buy. I will be using a engine operated cement mixer
Answered by Harley Weston.

The velocity of a pendulum, part II 2006-09-07
From Erin:
We saw the question in your database about the velocity of a pendulum swinging.....It is the same exact question....but there is another question......it says....

"estimate the instantaneous rate of change of d with respect to t when t = 1.5. At this time, is the pendulum moving toward or away from the wall? Explain."

Answered by Harley Weston.
Cubic feet 2006-09-06
From Tom:
How do I calculate cubic feet ? What is the formula?
Answered by Penny Nom.
Converting a decimal into a fraction 2006-09-06
From Andres:
How do i convert a decimal into a fraction?
Answered by Penny Nom.
Metric Conversions 2006-09-02
From Patricia:
I have a new difficulty, metric conversion when it involves the prefixes. Some problems I need help with are:
Answered by Stephen La Rocque.
The area of a triangle 2006-09-01
From Anthony:
I WOULD LIKE TO KNOW THE AREA OF A TRIANGLE THAT IS 42" AT THE BASE WITH EQUAL SIDES OF 29" .
Answered by Stephen La Rocque.
Rays 2006-08-31
From Natasha and daughter:
My daughter, Natasha, is in 4th grade. She had the following homework question about rays that confused us: Name as many rays as you can in the figure below.
Answered by Penny Nom.
Making and using equations 2006-08-30
From Brooke:

write an equation to model the relationship between these variables. Define the variables you use, and refer to your notes for a variety of formats of equations we have studied this year.

week 1: 3 birds infected
week 2: 15 birds infected
week 3: 75 birds infected
week 4: 375 birds infected

Answered by Stephen La Rocque.
The lower half of a sphere 2006-08-30
From Yvette:
I am interested in finding out how to represent the lower half of a sphere in the form of z=f(x,y) with r=2 and a centre point (3,1).
Answered by Stephen La Rocque and Penny Nom.
How many marbles are there? 2006-08-30
From Cathey:
Four children are playing with marbles. At the end of the day, one child has four less than half the marbles. The second child has six more than one-fifth the marbles. The third child has one third of what the first child has and the fourth child has one less than the third child. How many marbles are there?
Answered by Stephen La Rocque.
How do i get the height of an isosceles triangle? 2006-08-29
From Luis:
How do i get the height of an isosceles triangle?
Answered by Penny Nom.
Brackets and more brackets 2006-08-29
From Michelle:
Feeling stupid asking but it's been awhile ... {{{2}}} ...what is this really saying ....are the outer brackets = null?
Answered by Stephen La Rocque.
How high (in feet) is the mountain? 2006-08-29
From Briana:
A survey team is trying to estimate the height of a mountain above a level plain. From one point on the plain, they observe that the angle of elevation to the top of the mountain is 29 degrees. From a point 2000 feet closer to the mountain along the plain, they find that the angle of elevation is 31 degrees. How high (in feet) is the mountain?
Answered by Stephen La Rocque.
What is the total cost of a driveway ... 2006-08-29
From Ismael:
What is the total cost of a driveway 15' wide, 40' long and 4" thick if the concrete costs $60.00 per cubic yard and the labor costs $1.25 per square foot?
Answered by Stephen La Rocque.
How many microliters (ul) are there in 0.5 ml? 2006-08-29
From Monica:
How many microliters (ul) are there in 0.5 ml?
Answered by Stephen La Rocque.
Grossing up payroll wages 2006-08-27
From Katie:
I really need to understand (spelled out step by step) how to gross up payroll wages.
Answered by Penny Nom.
How many total worker are in the factory? 2006-08-26
From Sherine:
One quarter of the workers at the factory are clerical, one fifth are technical, half are are administrative,and the remaining 25 are managerial. How many total worker are in the factory?
Answered by Penny Nom.
How many thousands are in ten million. 2006-08-24
From Echoe:
How many thousands are in ten million.
Answered by Stephen La Rocque.
The area of an irregular octagon 2006-08-23
From Billy:
How do you figure the area of an irregular octagon? Measurements are 4 equal sides of =BD", and 4 equal sides of =BE". It measures 1 1/2" = from flat to flat.
Answered by Penny Nom.
Differentiate Y= sin3x + cos7x 2006-08-22
From james:
Differentiate the function of x using the basic rules.

Y= sin3x + cos7x

Answered by Stephen La Rocque.
How many nickels would it take to fill a room? 2006-08-22
From Diana:
How many nickels would it take to fill a room that measures 20 feet x 20 feet x 10 feet?
Answered by Stephen La Rocque.
I am pouring cement 2006-08-22
From David:

I AM POURING CEMENT

8FT. X 8 FT. X 4 INCHES

8FT. X 8FT. X 5 INCHES.

8FT. X 8FT. X 5 1/2 INCHES.

WHAT IS THE FORMULA?


Answered by Penny Nom.
How many milliliters of solution should be administered 2006-08-22
From Traci:
If a solution contains 4 percent , how many milliliters of solution should be administered to give a patent 80 grams?
Answered by Stephen La Rocque.
The reciprocal of a decimal 2006-08-21
From A student:
I need help with finding the reciprocal of a decimal
Answered by Leanne Boehm.
10 friends run into each other at a bar 2006-08-19
From Ken:
10 friends run into each other at a bar. We will name them A,B,C,D,E,F,G,H,I AND J. They all leave at the same time heading for seperate bars. However, A runs into B, C runs into D, E runs into F, G runs into H and I runs into J. These five groups leave again and the same thing keeps happening, but each time running in to someone they have not run into before. How many bars will it take, with each person meeting the others only once, until all of them have met each other again?
Answered by Chris Fisher.
How many miles has he walked? 2006-08-19
From Ed:
If a man is mowing a 100' x 100' yard with a 20" push mower, how many miles has he walked when the yard is completely mowed?
Answered by Stephen La Rocque and Penny Nom.
If 5x + 4y = 6 then x + y = ? 2006-08-18
From Vijai:
If 5x+4y=6 then x+y=?
Answered by Stephen La Rocque and Penny Nom.
A golf schedule for three days 2006-08-18
From Jon:

I was wondering if I could trouble you to assist me with the following: I am working on a playing schedule for 28 golfers. 7 groups of 4. I have 3 golf days to schedule.

Ideally, I would like to schedule all 28 golfers in 7 different groups of 4 on each day. Here is the catch.....no golfer in any group can be grouped togther more than once. Every group of 4 each day will have 4 new golfers that have never played together before. Is this possible?


Answered by Penny Nom.
Annual percentage yield 2006-08-17
From Joe:
If you have $360,767 in a money market fund that has accumulated a total of $2,127 in year to date dividends (an eight month period January to August) what is the annual percentage yield.
Answered by Penny Nom.
Approximately what is the acreage? 2006-08-17
From Lisa:
An ad describing a lot size of 73x150. Approximately what is the acreage?
Answered by Stephen La Rocque.
Mini Golf 2006-08-17
From Sarah:
I am a sixth grade teacher in Minnesota. I want to have my students explore mini golf and calculate the reflections and angles so that they can figure out how to hit a hole in one. I know that my daughter had various problems like this in eighth grade geometry, but I can't seem to find any internet activities of the appropriate level.
Answered by Stephen La Rocque.
Litres of water in my swimming pool 2006-08-16
From A student:
I need to calculate how many litres in my swiming pool, it is a circle shape, 5.5 m in diametre by 1.07 m deep.
Answered by Stephen La Rocque.
How many cubic yards of soil is needed 2006-08-16
From John:
how many cubic yards of soil is needed to cover 750 square feet at a height of 3 inches?
Answered by Penny Nom.
The development of trigonometry 2006-08-15
From Eugene:
Can you please give the exact time line of trigonometry.
Answered by Penny Nom.
A concrete semicircle 2006-08-15
From John:
I have to figure out how many yards of concrete I need for a semicircle. I have a radius of 21 feet and the concrete is 4" thick. I took 3.14 x (21)^2 then divided by 40 to get how many yards of concrete but that doesn't seem right.
Answered by Penny Nom.
An angle in a parallelogram 2006-08-13
From Sam:
Parallelogram ABCD has diagonal AC equal in length to side AB. CD is produced to E so that D is between E and C. If angle BAC =30 degrees find the size of angle ADE.
Answered by Stephen La Rocque.
How fast is the water level rising 2006-08-12
From Erin:
Water runs into a conical tank at the rate of 9ft3/min. The tank stands point down and has a height of 10 ft. and a base radius of 5 ft. How fast is the water level rising when the water is 6 ft. deep? (V=1/3 pi r2 h).
Answered by Penny Nom.
A problem with exponents 2006-08-09
From A student:
(8a to the negative 2 b cube c to the negative 4/4a squared b to the negative 3 c squared) to the negative 2
Answered by Stephen La Rocque.
How old would i be in minutes 2006-08-09
From Mariah:
i would like to know how old would i be in minutes if i was thirteen years old including leap years
Answered by Stephen La Rocque.
A triangle problem 2006-08-06
From A student:
A triangle is such that its medium side is twice as long as its shortest side, and its longest side is three times as long as its shortest side. The perimeter of the triangle is 54 yards. What are the lengths of the three sides?
Answered by Penny Nom.
The area of a lot 2006-08-05
From Lyn:
I have no idea how to find the square feet of an uneven polygon. I am in the process of making a land agreement and need to know.
Please tell me how to figure out the footage and acreage of land as follows.
road-front 360'
back 360'
left side 220'
right side 180'

Answered by Penny Nom.
HCF and LCM 2006-08-05
From Bharath:
The HCF and LCM of polynomials p(x) and q(x) are h and l respectively and p(x) + q(x) = h + l, show that
[p(x)]2 + [q(x)]2 = h2 + l2

Answered by Stephen La Rocque.
x+ 6 / 3 = x+ 6 / x 2006-08-01
From Lynne:
x+ 6 / 3 = x+ 6 / x
How do I solve this equation?

Answered by Penny Nom.
A point on a bicycle rim 2006-08-01
From Sonu:
Find the speed in m/s of a point on a bicycle rim of diameter 88cm making 200 revolutions per minute.
Answered by Penny Nom.
Gallons of water in a well 2006-07-31
From Martha:
I have a shallow water well that is 30 feet in depth.

I have 8 feet of water in the well.

The water well is a 30 inch in diameter.

My question : How many gallons of water is in my tank?

Answered by Penny Nom.
Power walking 2006-07-30
From Wendy:
I have an exercise routine of 'power walking', approx. 5-6 days per week. I carry a pedometer with me during these walks so that I can keep track of the distance of each walk. I've created a weekly walking chart on my computer, so each time I get back from a walk, I enter my stats, such as kilometers and miles walked that day, as well as my start and finish time. At the end of each week, before I printed out my completed chart, I would calculate out what my average walking distance was, for that particular week.

At the end of each month, I would calculate:
[1] I'd add up my total kilometers and miles walked for that month. (a no brainer ~ I could do that one!)
**But also, I would work out the percentages of the following:
[A] What my average daily distance p/walk was, for that month
[B] What my average time spent, per walk was, for that month

Answered by Penny Nom.
An Integral 2006-07-30
From Aniket:

I am Aniket studing in 12 th standard At Mumbai
I have following integration problem please give me a solution

integral of 1/under root of (5x2 - 2x) dx


Answered by Penny Nom.
Grams and milliliters 2006-07-27
From Poh:
We would like to know the conversion of 1 gm of cream/powder to ml.
Answered by Stephen La Rocque and Penny Nom.
I want to calibrate a glucose meter 2006-07-27
From John:
If I have 3 glucose control solutions having:
Low - 50 mg/dl
Normal - 100 mg/dl
High - 330 mg/dl

I want to calibrate a glucose meter but I need more than these 3 data points. How can I mix these up to get different concentrations in between 50 and 100, 100 and 330. I would like to mix up different concentrations and put them in test tubes or whatever to do some calibration work. Like 70 mg/dl, 90 mg/dl, etc. More data points should be in between say 50 and 100 since this is normal, 90 mg/dl is considered close to normal.


Answered by Stephen La Rocque.
The capacity of our hexagon fish tank 2006-07-26
From Doug and Andrea:
My name is Andrea and my son, Doug, and I were wondering what the capacity of our hexagon fish tank was but it’s been so long since I’ve been to school that I’ve forgotten the formula to do so. Would you please help us? The dimensions are: 27 ¾ W x 24L x 29H or one side (glass panel) is 14W x 29H.
Answered by Penny Nom.
Minimizing a cost 2006-07-25
From Edward:
The cost of running a car at an average speed of V km/h is given by c= 100 + (V2 / 75) cents per hour. Find the average speed (to the nearest km/h) at which the cost of a 1000 km trip is a minimum.
Answered by Stephen La Rocque.
Converting fractions to decimals 2006-07-25
From Diane:
HOW DO I CONVERT A FRACTION TO A DECIMAL? (E.G,. 8/14 to ? )

And HOW DO I DIVIDE A FRACTION (e.g., 1/14 divided by 3) and then convert that answer to a decimal?

Answered by Penny Nom.
A hamiltonian circuit 2006-07-24
From Tom:

1)Prove that every simple not directed graph with 21 vertices and 208 edges has
a hamiltonian circuit but not an Euler circuit.

(I have proved that if we give the 2 edges for each vertices then after a few
steps I have 21 vertices of odd degree and 19 edges.So in the end I have 2
vertices of odd degree.So I have not an Euler circuit.But how can I prove that
there is a Hamiltonian one?)

2)If G is a tree and has a virtex k degree,prove that it also has at least k
vertices degree 1.


Answered by Penny Nom.
My wife's recent pregnancy, 2006-07-24
From Tom:
During my wife's recent pregnancy, it so happened that my wife's 29th birthday fell on the exact same day that her unborn child was 29 weeks old (i.e. it was 29 x 7 days from the date of conception as advised by the doctor) I would like to know what the probability is of the above event occurring for a randomly chosen pregnant woman i.e. that the pregnant mum's x'th birthday falls on the same day that the unborn child is x weeks old EXACTLY.
Answered by Stephen La Rocque.
The last two digits 2006-07-22
From Sam:
You have answered this question already but you only answered to one digit.

What are the last TWO digits of:
3 to the power of 1994
7 to the power of 1994
3 to the power of 1994 + 7 to the power of 1994
7 to the power of 1994 - 3 to the power of 1994


Answered by Paul Betts and Penny Nom.
How many miles are there in 30 furlongs? 2006-07-22
From Jennifer:
If there are 8 furlongs in a mile, how many miles are there in 30 furlongs?
Answered by Steve La Rocque and Paul Betts.
How do I draw the arc? 2006-07-22
From Dani:
How do I draw the arc (calculate the included angle) when I know only the arc length?
Answered by Stephen La Rocque.
A perpendicular intersection of two barrel vaults 2006-07-21
From Neal:
I'm wanting to build a series of architectural models of different roman and medieval buildings out of cardboard. Once I have perfected the models I want to print them out on card stock so that school kids (or anyone else) can make the buildings.
A feature of many of these models is the cross or groin vault (a perpendicular intersection of two barrel vaults).
A single barrel vault is easy to imagine as a plane (a rectangular piece of cardboard) that will be folded into a semi-circular arch.
The intersection of a second barrel vault and this one is presenting me with problems. The second plane needs to have an ellipse cut into it so that when it is folded into the arch, it will mate up with the curve of the first barrel vault.
Given that the two pieces of card have identical widths (and therefore identical arcs in cross section) is there a way to calculate the ellipse that needs to be cut so that it can be cut before the second arch is folded?

Answered by Edward Doolittle.
How many bags of concrete will we need to use? 2006-07-20
From Sue:
We are trying to pour some concrete
The area measures 26 ft long x 20 in wide. We want to pour the concrete 3 in thick.
One bag covers .41 cu. ft.
How many bags of concrete will we need to use?

Answered by Penny Nom.
The lot is a very odd shape 2006-07-20
From Liz:
I have a square footage question. I'm looking at buying a piece of land and would like to know how to calculate the square footage of it. The lot is a very odd shape and I have attached a drawing of the lot:
Answered by Stephen La Rocque.
What is the area of the circle? 2006-07-19
From Nana:
the circle is tangent to sides BC and AD of the 8 by 12 rectangle, ABCD. What is the area of the circle?
Answered by Stephen La Rocque.
Mini Golf geometry 2006-07-18
From Sarah:
I want to have my students explore mini golf and calculate the reflections and angles so that they can figure out how to hit a hole in one. I know that my daughter had various problems like this in eighth grade geometry, but I can't seem to find any internet activities of the appropriate level. If you can steer me towards any resources, I'd be most grateful.
Answered by Natasha Glydon.
Volume of rainfall 2006-07-18
From Randy:
Calculate the amount (in gallons)of rainfall .98 inches will put over an area of 3.76 acres.
Answered by Stephen La Rocque.
Calculating the belt length of a three pulley system 2006-07-16
From Mark:
I have a 3 pulley system with sides abc and pulleys ABC. Pulley A has radius of 10cm, pulley B has radius of 20cm, and pulley C has radius of 3cm. The side lengths are: (center to center of pulleys) between pulleys AB = 75cm, between pulleys BC = 100cm, and between pulleys AC = 50cm. I set these side lengths up as (according to law of sines and cosines) a = 100cm, b = 50cm, and c = 75cm. What is the length of the belt required for this system? I need to know how I would set this problem up and solve.
Answered by Stephen La Rocque.
Bike combination lock 2006-07-16
From Alejandra:
I have a bike lock. It has three columns of the digets 1-9 and zero. I have forgotten the combination. Three numbers need to be imputed. What are the possible lock cominations, so that i can try to break the lock?
Answered by Stephen La Rocque.
Ounces of water to grams 2006-07-16
From Liza:
How many grams is in 10 ounces? I'm asking this question because i just bought a sea monkey and it says add 10 ounces of water and I don't know what that means. I have asked my parents but neither of them know.
Answered by Stephen La Rocque.
Finding square footage from lengths in inches 2006-07-14
From Ann:
How do I find out the square footage of a window 45" by 22" and 43" by 35.5"?
Answered by Stephen La Rocque.
Concrete for a chicken coop floor 2006-07-13
From Chelsea:
How much concrete for a chicken coop floor? The dimentions are 3 slabs, 30 feet long and 4 inches thick, 6 feet 6 inches wide. One slab is 41 feet long, 6 feet 6 inches wide, and 4 inches thick.
Answered by Stephen La Rocque.
The product of polynomials 2006-07-13
From Lynne:
If you multiply two polynomials, what can you say about the degree of their product? What exponent applies here?
Answered by Stephen La Rocque.
A board is cut into two pieces... 2006-07-11
From DeAnna:
A 102" board is cut into 2 pieces. One piece is two times the length of the other. Find the lengths of the two pieces.
Answered by Stephen La Rocque.
The dimensions of a rectangle 2006-07-09
From Lori:
The length of a rectangle is 1 cm more than 3 times its width. If the area of the rectangle is 61 cm2, find the dimensions of the rectangle to the nearest thousandth.
Answered by Penny Nom.
Isoperimetric quotients 2006-07-07
From Jessica:
How do you work out an algebraic equation for the IQ of an isosceles triangle.
Answered by Penny Nom.
Flying with and against the wind 2006-07-06
From Tim:
When a plane flies with the wind, it travels 800 miles in 4 hours. Against the wind it takes 5 hours to cover the same distance. Find the rate of the plane in still air and the rate of the wind.
Answered by Stephen La Rocque.
How old is y = mx + b? 2006-07-05
From Beckie:
I am taking intermediate algebra this summer (Math 95), and have a question for you. My instructor will award extra credit points to any of us who find out how old the formula y = mx+b is. Would you happen to know the answer?
Answered by Penny Nom.
Two water jugs 2006-07-05
From Kisha:
suppose that you have two water jugs, one which holds three gallons and one which holds five gallons. in additions, you have a water faucet at your disposal. your job is to find a method that produces exactly one gallon of water, another method that produces exactly two gallons of water, another method that produces exactly three etc up to exactly eight gallons of water. you may fill jugs from the faucet or from one another. you may not rough estimates???
Answered by Penny Nom.
A tangent to a circle 2006-07-05
From Izumi:

I have problems finding the point of tangency between the circle C

x2 + y2 + 4x - 6y - 12 = 0

and the tangential line that passes through the point P (6, -1).


Answered by Stephen La Rocque.
The lowest denominator 2006-07-02
From Lisa:

What is the lowest denominator for the following numbers:

a) 26
b) 21
c) 47


Answered by Stephen La Rocque.
A 9 foot diameter circle 2006-07-02
From Lisa:
How do I calculate 9 foot diameter measurement in total feet??
Answered by Stephen La Rocque.
Two algebra questions 2006-07-02
From Majeedah:

I'm upgrading thru correspondence and haven't been in school for a long while, so I have no class or teacher to explain the basics to me.

I have a couple of questions:

The problem is this:

  1. f(x) = x2 - 2, Find the expression.

    Q f(-x)
    A x2 - 2

    Why is the answer not -x2 - 2?

  2. A relation f, is given by f(x) = x - 2/x. Find the expression.

    Q 2/f(3)
    A 6

    How do you get the answer.


Answered by Stephen La Rocque.
[(400/n)-5](n+ 4)=400 2006-07-02
From Maria and George:

Can somebody solve for n in the following equation please?

[(400/n)-5](n+ 4)=400


Answered by Penny Nom.
Some plumbing calculations 2006-07-02
From Steve:

I am preparing to write my journeyman plumber ticket and would appreciate answer/solutions to the following math questions along with the formula to solve them

  1. A range boiler is 16 in. in diameter and 4 ft high. How many gallons will it hold ?
  2. What would the weight in Kg be if the pipe was .4 of a meter in diameter and it was 8.6 m in length
  3. An oil storage tank is 4 m in diameter and 6 m high. How , many liters of oil can be stored

I thank you in advance for your help ... all math books I have are US and nothing is Metric


Answered by Stephen La Rocque and Penny Nom.
A number puzzle 2006-06-30
From Atitya:
The diagram given below consist of seven line segments and six circles,place the numbers 1 to 13 on either a line or a circle in such a way that the numbers on each segment is the difference of the two at its end points.
Answered by Stephen La Rocque and Penny Nom.
Some word problems 2006-06-30
From Oliver:

Question: I am a father of a 10 years old boy and sending this question from Bangkok, Thailand. I need solutions for the following:

1
Joseph and Peter spent $45 altogether.
Joseph and Harold spent $65 altogether.
If Harold spent 3 times as much as Peter, how much did Joseph spent?

I know the answer is $35 but do not know how to get to it.

2
.
.
.


Answered by Penny Nom.
Order of operations 2006-06-29
From Jaden:

(1) A triangle has vertices at (-3,-3) (7,-3) and (-2,5). What is the area of the triangle?

Is there a way of doing this so that you dont have to graph it? If the x axis is horizontal can i say it is the base and the y-axis is horizontal meaning the height. Would I be able to do this...-3 to 7-->9 spaces, -3 to 5-->7 spaces-----9(7)63/2=31.5

(2) A cube has a surface area of 54 square centimeters. What is the volume of the cube in cubic centimeters?


Answered by Stephen La Rocque.
Marking out a circle 2006-06-28
From Peter:
given a straight line. how do i work out the off sets ( at right angles) at several intermediate points. to set out a 5.0m arc that has a 18.0m radius.
Answered by Stephen La Rocque and Penny Nom.
The volume of a building 2006-06-28
From A student:
a building is 8 feet wide and 16 feet long. it has a flat roof that is 12 feet high at one corner, and 10 feet high at each of the adjacent corners. what is the volume of the building?
Answered by Penny Nom.
livinia the housefly finds herself caught in the oven 2006-06-27
From Danielle:

livinia the housefly finds herself caught in the oven at the point (0,0,1). the temperature at the points in the oven is given by the function

T(x,y,z) = 10 (xe(-y2) + ze(-x2))

where the units are in degrees celsius.

(i.)so if livinia begins to move towards the point (2,3,1) at what rate in deg/cm does she find the temp. changing?
(ii.)in what direction should she move in order to cool off as rapidly as possible?
(iii.)suppose that livinia can fly at a speed of the square root of 2 (cm/sec.) if she moves in the direction of part (ii) at what rate will she find the temp. to be changing?


Answered by Penny Nom.
I have a ditch how many tons of dirt will it take 2006-06-25
From Scott:
I have a ditch 200 feet long 10 feet wide and 5 feet deep.How many tons of dirt will it take to fill it?
Answered by Stephen La Rocque.
An octagon shape flower bed 2006-06-24
From Brandy:
hello my name is brandy my husband and I would like to build an octagon shape flower bed to put around the tree in the front yard we would hope to have the whole shape about 4 ft-5 ft around the tree. what would be the way to find out how to cut each side to that they fit together equally
Answered by Penny Nom.
A side segment of an octagon 2006-06-24
From Ed:
I would like to know how to find a side segment of an octagon when I know that my octagon is 9 feet from side to side and top to bottom?
Answered by Stephen La Rocque.
A trig problem 2006-06-24
From Greg:
A and B are two towers, B being 4 km due east of A. The true bearings of a flagpole, C, from A and B are α east of north and α west of north respectively. The true bearings of a second flagpole, D, from A and B are (α + β) east of north and (α - β) west of north respectively. Assuming A, B, C, and D are on level ground, and that α = 25, β = 10, find the distance between C and D.
Answered by Penny Nom.
The area of a sector and a triangle 2006-06-23
From Howard:
I thought of the following problem which is similar but much simpler than the tethered goat problem: What is the angle(it is more illustrative in degrees)of arc of a unit circle so that the area between the chord it subtends and the arc length is equal to the area of the triangle with opposite side the subtended chord.
Answered by Stephen La Rocque and Penny Nom.
As close to 841 as possible 2006-06-21
From Alan:
using these numbers by either add subtract division multiply come up with the answer as close to 841
100 25 10 7 6 3

Answered by Paul Betts.
simplify 7x + 4y + 2x - 6y 2006-06-21
From Alan:
simplify 7x + 4y + 2x - 6y
Answered by Paul Betts.
Gallons in a pool 2006-06-21
From Dave:
is it possible to find out how many gallons of water is in my 18 foot x 12 foot pool it is also about 4 and 1/2 feet deep?
Answered by Penny Nom.
A 5 digit, push-button door 2006-06-20
From George:
We have a 5 digit, push-button door look (1-5). How many combinations are possible without repeating a number in a 3 digit entry code?
Answered by Stephen La Rocque.
What should the length of the other sides? 2006-06-20
From Kevin:
if a property is basically square for 5 acres and one side is 340 feet what should the length of the other sides?
Answered by Stephen La Rocque.
Factor 81x^4 - 72x^2 + 16 2006-06-19
From Jesse:

how would i factor this?

81x4 - 72x2 + 16


Answered by Penny.
What are the 3rd and 4th terms of (2x-y)^7? 2006-06-18
From April:
What are the 3rd and 4th terms of this sequence: (2x-y)7?
I'm having an issue with this...is there any easier way to get it without completely factoring the whoooole thing out?

Answered by Penny Nom.
What will the graph of y=x2 - 2x - 3 be? 2006-06-17
From Byrony:
What will the graph of y=x2 - 2x - 3 be?
Answered by Steve La Rocque.
express the number 100 using the same figure 6x 2006-06-17
From A student:
express the number 100 using the same figure 6x
Answered by Stephen La Rocque.
How much did I have in my wallet when I lost it? 2006-06-15
From Marangeo:
I recently lost my wallet and when it came to completing the police report I couldn't remember how much money I had in there. I do remember thinking that the first time I bought something that it cost me 10 per cent of what I had. Then I noticed that the second purchase had also cost me exactly 10 per cents of what I had left. My receipts totalled $19. How much did I have in my wallet when I lost it?
Answered by Stephen La Rocque.
How many gallons of water are in a 24' X 4' round swimming pool? 2006-06-15
From Nicole:
How many gallons of water are in a 24' X 4' round swimming pool?
Answered by Stephen La Rocque.
What is the probability that the policeman will shadow the correct person? 2006-06-14
From Kolby:
Through an informer from the underworld, the police know the meeting place of a gang. The identity of the different gang members, however, is unknown. it is the duty of a policeman to shadow the leader of the gang. The policeman knows that the leader is the tallest of the five persons, all of whom have different heights. after the meeting, the gangsters, as a safety measure, leave the building separately at intervals of 5 minutes. As the policeman cannot see who is the tallest, he decides to let the first two gangsters go and shadow the one after that who is taller than all those who left before.Through an informer from the underworld, the police know the meeting place of a gang. The identity of the different gang members, however, is unknown. it is the duty of a policeman to shadow the leader of the gang. The policeman knows that the leader is the tallest of the five persons, all of whom have different heights. after the meeting, the gangsters, as a safety measure, leave the building separately at intervals of 5 minutes. As the policeman cannot see who is the tallest, he decides to let the first two gangsters go and shadow the one after that who is taller than all those who left before. What is the probability that the policeman will shadow the correct person?
Answered by Steve La Rocque and Claude Tardif.
How many digits are in the number 2^64? 2006-06-14
From Trang:
How many digits are in the number 264?
Answered by Stephen La Rocque.
How many bags will it take to cover the area? 2006-06-14
From Ken:
I have area of 160 sq ft. I want to put mulch in the area 2 inches thick. The bag of mulch comes with 2 cu ft of mulch per bag. How many bags will it take to cover the area?
Answered by Penny Nom.
A 52 cubic foot box 2006-06-14
From Konstanze:
I need to figure out what LXHXW I need to create a 1.5 cubic meter or 52 cubic foot volume--there is an answer in your database that relates to this..but it does not give the formula to go from cubic feet/inches to a measurement for a box.

Empirically I can figure out that 3 x 3 x 3 equals 27 and that 3.5 feet (42") x 3.5 x 3.5 equals 42.87 and 3.75 x 3.75 x 3.75 equals 52.73 which is about 1.5 cubic meters (1cubic meter = 52.971643 cubic feet) i.e a box 45 x 45x 45 "

There must be an easier way.

Answered by Penny Nom.
The chord length of a polygon 2006-06-14
From Krishna:
I have to find out the chord length of a polygon - Tetradecagon ! The Radius of the Circle is 11.5 Cms. The Circle is intersepted by 14 arcs. Then how to find out the chord length?
Answered by Stephen La Rocque.
A jar contains 36.25$ 2006-06-12
From Brunia:
A jar contains 36.25$ in pieces of 0.10 and of 0.25$. If it there has 250 change pieces in all, how much there he has pieces of 0.25$ in the jar.
Answered by Penny Nom.
Fahrenheit and Celsius 2006-06-12
From Doris:
I just know how to do problems with fahrenheit or celcius. Could you show me step by step how to do these? Then I can do it if you can show me each step of the way.
Answered by Stephen La Rocque and Penny Nom.
how do i find i^22? 2006-06-12
From Sky:
how do i find i22?
Sky

Answered by Stephen La Rocque.
Square feet and linear feet 2006-06-10
From Richard:
Need to know how many linear ft are in 2800 sq.ft
Answered by Penny Nom.
The volume of a roll 2006-06-09
From Yogesh:
I have a roll which height is 63 inches & width is 11 inches i want to know that how to calculate the cubic meter of this roll
Answered by Penny Nom.
The hardest algebra question you know 2006-06-08
From Steven:
could you send me the hardest algebra question you know and the work with it , i want to test a tutor. he a friend.
Answered by Stephen La Rocque.
Flipping a coin 2006-06-08
From John:
If a coin is flipped n times, where H is the number of heads after n flips, and T the number of tails, then will the quantity (H-T) change signs infinitely often as n goes to infinity?
Answered by Chris Fisher.
Designing a garage 2006-06-08
From A builder:
I'm currently designing a garage and came upon this interesting math problem. I've tried using various methods to solve it but have so far been unsuccessful. I've included a picture as its far easier to show you my question than explain it verbally. I realize it could be done by trial and error but i'm looking for a real solution.
Answered by Stephen La Rocque and Penny Nom.
Tiling a floor 2006-06-08
From Marguerite:
A builder is adding a 20' X 28' addition to a house. He has two choices: he can use 8" X 8" tile @ $.98 a tile or 12" X 12" tile @ $1.98 a tile. How much money would he save by using the least expensive choice?
Answered by Stephen La Rocque and Penny Nom.
A volume of revolution 2006-06-07
From Colleen:
Find the exact volume in cubic units generated by rotating a region, R, around the
y-axis, given that R is the region bounded by the curve y = x3 and the lines x = 1 and
y = 8.

Answered by Penny Nom.
How many yards will I need? 2006-06-06
From Melba:
I'm am trying going to put mulch around the tree in my front yard. I am going for 6" deep and 4069.44 sq. in. around.
How many yards of dirt will I need?

Answered by Penny Nom.
Excluded values 2006-06-05
From Mike:

i need to find the excluded values of:

2n2-9n+4 / 2n2-5n-12


Answered by Penny Nom.
The perimeter of a pool 2006-06-05
From Troy:
How many Linear feet would a pool be if the pool was 18 by 38 foot oval
Answered by Penny Nom.
Putting in a sidewalk 2006-05-31
From Sandra:
I am putting in a sidewalk the measurements are 12 feet by22 inches ,by 4inchs how many bags of cement will I need?
Answered by Penny Nom.
What is the sum of the first 100 whole numbers? 2006-05-31
From Jo:
what is the sum of the first 100 whole numbers?
Answered by Natasha Glydon, Paul Betts and Penny Nom.
The width & depth of a 1/4 acre pie shaped lot 2006-05-30
From Linda:
Can you tell me what the approximate width & depth of a &frac; acre pie shaped lot might be?
Answered by Penny Nom.
Three people go to a store where there is a sale 2006-05-30
From Geocee:
Three people go to a store where there is a sale. One person buys two shirts, one pair of pants and one sweater. The second person buys one shirt, two pairs of pants and two sweaters. The third person buys three shirts and four pairs of pants. Find the price of each item if the first person spent $155 and the second and third person spent $235 each.
Answered by Stephen La Rocque.
Math manipulatives 2006-05-29
From Mike:
Our school is looking to order a large amount of math manipulatives. Do you have a reliable, reasonably priced source that is not producing 'kits' that are intended for specific math programs like Math Makes Sense?
Answered by Diane Hanson and Walter Whiteley.
The area of part of a circle 2006-05-29
From Larry:
need to find area of a circle between a given line (cord) to the circumference of the circle (see attachment). I often review blue prints of homes and many times have to know the area the home.
Answered by Stephen La Rocque and Penny Nom.
A truncated cone 2006-05-28
From Phil:
Hi, I am an art student and I am trying to make a "truncated cone" (ie: a cone with the top cut off) out of sheet metal. I need to design a template first and am having problems working out the angles. The truncated cone is 250mm high, 550mm wide at the bottom and 290mm wide at the top. Can you help?
Answered by Stephen La Rocque.
what is the cubic feet of 18... 2006-05-27
From A student:
what is the cubic feet of 18"x18"x24"?
Answered by Penny Nom.
Cubic yards of sand 2006-05-26
From Steve:
I need to cover an area with sand that is 30 feet wide and 60 feet long and I need the depth of the sand to be 1/4 inch thick. How many cubic yards of sand will I need?
Answered by Penny Nom.
Simultaneous Equations 2006-05-24
From Angie:

Question: solve the equations
2x-3y-z=0
3x-2y+z=-5
x+3y-2z=14

for x,y,z


Answered by Stephen La Rocque.
differentiate the volume of a cylinder with V respect to h 2006-05-24
From A student:
differentiate the volume of a cylinder with V respect to h
Answered by Stephen La Rocque.
The area of a 30, 60 90 triangle 2006-05-24
From Willetta:
if there is a triangle with a 30, 60, and 90 degree angle and the shortest side is 6cm how do you find the area?
Answered by Stephen La Rocque and Penny Nom.
How do you graph 3x + 6y = -9 by axis intercepts? 2006-05-22
From Crystal:
How do you graph 3x + 6y = -9 by axis intercepts?
Answered by Penny Nom.
Three purses 2006-05-22
From Marcee:
If you take $20 from the first and put it into the second of three purses, the second would then contain 4 times as much as remains in the first. If $60 of what is now in the second is put into the third, the third will contain twice what is in the first and second together. Now, if $40 be removed from the third and put into the first, there will be half as much as in the third. What did each purse originally hold?
Answered by Penny Nom.
The volume of a cylinder 2006-05-22
From A student:
Show that a volume of a cylinder equals to v = A2 / (4πh)
where h= height of the cylinder
A = area of a original paper

Answered by Penny Nom.
A concrete driveway 2006-05-21
From Diane:
I am trying to estimate how many cubic yards of concrete I will need to pave my driveway. I have an area that is 30 ft wide by 25 ft long and then goes into an area that is 10 ft wide by 219 foot long. I would like to make this 4 inches thick. How many cubic yards of concrete will it take to do this?
Answered by Penny Nom.
The area of a triangle 2006-05-21
From A student:
how do you find the area of a triangle?
Answered by Penny Nom.
The interior angles of a right triangle 2006-05-20
From Greg:
I am wondering if there is a way to figure out the interior angles of a right triangle if we know ONLY the side lengths, and the trick is, we CANNOT use arctangent!
Answered by Leeanne Boehm and Penny Nom.
The coefficient of variation 2006-05-20
From Glenn:
What is the correct formula for coefficient of variation for a binomial distribution?
Answered by Penny Nom.
A pole 22 5/6 feet long is broken in two 2006-05-19
From Kenneth:
A pole 22 5/6 feet long is broken in two. One piece is 2 3/4 feet longer than the other. What is the length of each piece?
Answered by Penny Nom.
A box with an interior of 1.26 cu. feet 2006-05-19
From Maria:
I am trying to build a box with an interior of 1.26 cu. feet with all sides equal.
Answered by Stephen La Rocque and Penny Nom.
The length of the diameter 2006-05-18
From Steven:
A circle has a circumference of 312in. Find the length of its diameter to the nearest integer?
Answered by Penny Nom.
A bag contains 9 red, 6 white, 3 blue, and 8 green marbles 2006-05-18
From Micheael:
A bag contains 9 red, 6 white, 3 blue, and 8 green marbles. Two marbles are drawn, but the first marble drawn is not replaced. Find P(white, then white). Make into one reduced fraction.
Answered by Paul Betts and Steve La Rocque.
A triangle problem 2006-05-18
From Jim:
Right angle triangle with a hypotenuse of 20 units. Square inside the triangle with sides of 4 units, the square shares two sides with both legs of the triangle, and the corner touches the hypotenuse limiting the triangles size.
Answered by Penny Nom.
Three questions 2006-05-17
From Diane:

Question:

I

What is the largest real number less than 10?
1- there is no largest real number less than 10 or
2- 9
or
3- 9.999...

II

Given that not (a and b) implies c.What does
(not c) imply ?
1-a and b
2- (not a and b) or (a and not b)
3- (not a and b) or (a and not b)
4- a or b

III

What are the x and y intercepts of the equation
2x + 3y -6 =0 ?
1- x=3 and y=2
2- x=6 and y=-2
3- -3x + 2y =0


Answered by Paul Betts.
What is the radius of the sphere? 2006-05-17
From Jimmy:
The surface area and volume of a sphere are both equal to the product of 4-digit whole numbers and pi. What is the radius of the sphere?
Answered by Stephen La Rocque.
integral of tan^4 x 2006-05-14
From Aqil:
integral of tan4 x
Answered by Penny Nom.
What are the dimensions of the field? 2006-05-14
From Angie:
The length of fence required to enclose a rectangular field is 3000 metres. What are the dimensions of the field if it is known that the difference between the length and width is 50 metres?
Answered by Penny Nom.
The volume of water in a sphere shaped water tower 2006-05-14
From Dan:
I work in the water industry, providing automatic controls for water systems. I was wondering if there was a formula for calculating the volume of water in a sphere shaped water tower? With the use of a pressure gauge we know the elevation of zero water {the bottom of sphere) and the overflow elev. (near the top of sphere) I would like to calculate how many gallons are at any elevation in between.
Answered by Stephen La Rocque.
I want to calculate 0.00353 to the power 1.3 2006-05-14
From Richard:
I want to calculate 0.00353 to the power 1.3 How do I do it?
Answered by Stephen La Rocque.
How many children were there? 2006-05-14
From Geecee:
At a local village gala, the entire population turned up, 500 people. The event raised £3,000. Tickets were priced as follows: £7.48 men, £7.12 women and £0.45 children. How many children were there?
Answered by Stephen La Rocque.
Landscaping 2006-05-11
From Luke:
HOW MANY CUBIC YARDS OF CONCRETE WILL IT TAKE TO FILL UP A BOX 20FT.X20FT.X6IN DEEP?


HOW MANY CUBIC YARDS OF CONCRETE WILL IT TAKE; I HAVE A 12FT.IN DIAMETER POND AND I WANT TO PUT A 3FT.SIDEWALK AROUND THE OUTSIDE OF IT?

Answered by Stephen La Rocque.
A person is given 3 true or false questions 2006-05-11
From Larry:
A person is given 3 true or false questions. They have no idea of the answers. What is the probability of the person getting them all correct? Could you show me a tree for this.
Answered by Penny Nom.
A bicycle ride around the local lakes 2006-05-11
From Geecee:
Yesterday I went for a long bicycle ride around the local lakes. As the ride was quite long, I rode in various stages. In the first stage I rode half of the overall distance. Stage two saw half of the remaining distance plus 35 metres covered. Stage three covered three-quarters of the remaining distance. Stage four completed half of the remaining distance plus 75 metres. Stage five completed the journey with a final burst of 150 metres. How far did I cycle in total?
Answered by Stephen La Rocque.
How many thousands make 1million? 2006-05-10
From Raj:
How many thousands make 1million?
Answered by Penny Nom.
What is the measure of the interior angles of a decagon? 2006-05-10
From Malissa:
What is the measure of the interior angles of a decagon? And what is the measure of each interior angle of a regular decagon?
Answered by Stephen La Rocque.
A box with volume 0.78 cu ft 2006-05-10
From Sherrina:
i need to build a box that equals .78 cu ft any ideas on how i do this
Answered by Penny Nom.
Twenty golfers 2006-05-08
From Mona:
We have twenty golfers and four rounds of golf. We would like to figure out how to make foursomes so that each golfer plays with as many different golfers as possible.
Answered by Penny Nom.
A hallway in an old house 2006-05-06
From Sabrina:
I am trying to determine how many square feet is in a room that is not square or rectangular. It has basically 5 walls of different lengths, it is acutely considered a hallway in an old house, how can I figure out the footage, the Length times Width requires to much wasted materials
Answered by Penny Nom.
The radius of a soccor ball 2006-05-05
From Jacqui:
A soccer ball is made up of hexagons and pentagons with the same side lengths. A manufacturer wants to produce a ball of certain diameter. The questions that follows is what side length for the polygons will produce a ball of certain diameter.
Answered by Chris Fisher.
5 square miles or a 5-mile square? 2006-05-03
From Rosemary:
I am an interpreter in our Historical Museum. From what I received, I have always told my guests that when the Connecticut Land Company sold the Western Reserve land, 5 square miles constituted a township. I was corrected yesterday by someone who said it should have been a 5-mile square. What is the difference? Hope you can help.
Answered by Penny Nom.
Cutting a piece of PVC pipe 2006-05-03
From Shonda:
A plumber has a 10' piece of PVC pipe. How many 9/5 foot pieces can be cut from the 10' piece?
Answered by Penny Nom.
A fountain of water jets forms parabolic arches 2006-05-03
From Jennifer:
Let's say in you have a fountain and the water jets form parabolic arches. The center of the fountain, being the origin of the coordinate system, it is elevated 5 feet off the ground, . The equation formed the water arch is y= -x2+4x, what is the radius of the basin needed to catch the water at ground level?
Answered by Stephen La Rocque.
A parabolic arch 2006-05-02
From Mike:
A bridge over a river is supported by a parabolic arch...arch is 200 m wide at water level...the maximum height of the arch is 80 m..what is the height of the arch measured from a point on the water 40m from the center of the arch?
Answered by Stephen La Rocque.
A limit involving trigonometry 2006-05-02
From Allie:
My question is how do you solve.
lim as t goes to 0 [sin squared *3t] / t squared?

Answered by Penny Nom.
Simplifying with negative exponents 2006-05-02
From Kristine:

I am having a hard time understanding factoring and multiplying polynomials. I have a couple of problems I just can't get, can you please help me?

Simplify, do not use negatives in the answer:
(5a-1b-7)(-2a4b2)

and:
(5x2y-7z)(-4xy-3z-4)


Answered by Stephen La Rocque.
A word problem with fractions 2006-05-02
From Stuart:
I have 54 1/4 yards of material. i need to cut as many pieces as possible out of it that are 3 1/12 long. how many pieces will you get and how much is left over?
Answered by Stephen La Rocque.
Solving an equation without using algebra 2006-05-01
From Kenneth:
How can the following be solved without using algebra? 10 times some number is 3 times the same number plus 14.
Answered by Paul Betts.
Rate of ladder falling 2006-04-30
From Harsh:
A ladder 4 m long rests against a vertical wall. If the bottom of the ladder slides away from the wall at a speed of 30 cm/s, how quickly is the top of the ladder sliding down the wall when the bottom of the ladder is 2 m from the wall?
Answered by Stephen La Rocque.
How much does 1300cc of liquid weigh? 2006-04-28
From Debra:
How much does 1300cc of liquid weigh, and how is the formula equated?
Answered by Stephen La Rocque.
Adding polynomials and counting the terms 2006-04-28
From Kris:
Suppose that two polynomials, each containing 3 terms, are added. Is it possible for the sum to contain more than 3 terms? fewer than 3 terms? exactly 3 terms? Explain.
Answered by Leeanne Boehm and Steve La Rocque.
School bus reliability - a probability question 2006-04-27
From Peggy:
The school bus arrives at Janet's stop on time on 75% of school mornings. What is the probability it will arrive on time each day in a 5-day week?
Answered by Stephen La Rocque.
The last slice of pizza 2006-04-27
From Lori:
I have a pizza. The radius is 10 inches long.The pizza was cut into 16 equal slices. When one slice was left, my sister and I both wanted it, so we agreed to cut it in half, but I like the crust more than she does, so we decided to cut it the "other way." In other words, the two pieces would not be symmetrical. The inside piece would contain all topping, and the outer piece would contain some topping and some crust. How far up the radius from the center of the circle will I need to cut so we will both have an equal area of pizza?
Answered by Stephen La Rocque.
Ladder and wall 2006-04-27
From Lori:
A 12-foot ladder is leaning across a fence and is touching a higher wall located 3 feet behind the fence. The ladder makes an angle of 60 degrees with the ground. Find the distance from the base of the ladder to the bottom of the fence.
Answered by Stephen La Rocque.
How wide is this runway? 2006-04-27
From Sam:
A runway is 3.75 miles long and covers a total of 5.6 acres. What is the width of the runway?
Answered by Stephen La Rocque.
Odd-man out coin tossing and probabilities 2006-04-27
From Kalyan:
When four people toss fair coins, what is the probability that in a given toss there will be one "odd man". That is one person whose coin does not have the same out comes as that of any other members?
Answered by Stephen La Rocque.
Percentage of weight 2006-04-26
From Karen:
How do you calculate percentage of weight lost?
Answered by Stephen La Rocque.
Marine shipping - fuel consumption 2006-04-26
From Sandy:
When a ship is 800 nautical miles from port its speed is reduced by 20%, thereby reducing the daily fuel consumption by 42 tonnes and arriving in port with 50 tonnes onboard. If the fuel consumption per hour is given by the expression (0.136 + 0.001V^3) where V is speed in knots; Estimate a) reduced consumption per day b) fuel onboard when speed reduced c) normal consumption for 800 nautical mile voyage d) increase in steaming time
Answered by Stephen La Rocque.
Absolute Value vs. Standard Deviation ? 2006-04-24
From Sonia:
Why we don't just take the average of the absolute value of difference scores (use the mean deviation) to describe variability instead of calculating the standard deviation?
Answered by Claude Tardif.
How many gallons in a cubic foot? 2006-04-24
From Jana:
How many gallons in a cubic foot?
Answered by Stephen La Rocque.
Geometry proof 2006-04-23
From Jade:
From a point P outside a circle with centre O, tangents are drawn to meet the circle at A and B. a) Prove that PO is the right bisector of the chord AB. b) Prove that
Answered by Stephen La Rocque.
Finding the supplementary angles 2006-04-22
From Kendra:
Angles ABC and DBA are supplementary. If m
Answered by Stephen La Rocque.
The perimeter of a regular octagon 2006-04-20
From Martin:
I would like to make an octagon out of 2x4 lumber. I know that the lumber needs to be cut at 67.5 degree angles, but how do I determine the length of each piece if I want to make, say, a 2.5 ft diameter octagon?
Answered by Stephen La Rocque.
Geometric sequence and basic functions (graphs) 2006-04-20
From Marlene:
Which of the basic functions is related to the geometric sequence: Linear, Quadratic, Rational, or Exponential? Can you give me an example of how it would be used in normal life?
Answered by Stephen La Rocque.
Equation of a line in Ax + By = C form 2006-04-19
From Christopher:
How do you write the equation of a line with a slope of 3/8 and passing through (-4,6) in Ax+By=C form?
Answered by Stephen La Rocque.
How much money can I borrow if I have $5 000 for a deposit? 2006-04-19
From Chris:
I have a sum of cash that I want to use as a deposit ($ 5 000) and a lender will lend me 65% of the purchase price. How do I calculate the maximum value I can borrow?
Answered by Stephen La Rocque.
Which has more pizza? 2006-04-19
From Kristine:
Which has more area: a round pizza that is 16 inches in diameter or three square pieces of pizza that are 8.5 inches on each side?
Answered by Stephen La Rocque.
How many numbers are relatively prime with 250? 2006-04-19
From David:
How many positive integers less than or equal to 250 are relatively prime with 250?
Answered by Stephen La Rocque.
Finding c in 4x=10-cy 2006-04-19
From Jennifer:
A solution of the equation 4x=10-cy is (5,2). What is the value of c?
Answered by Stephen La Rocque.
Cubic yards of landscape stone 2006-04-19
From Janet:
I need to spread landscape stone around my pool. A landscaper said I need 16 yards. What is the area of a cubic yard if I want the stone 3" deep?
Answered by Stephen La Rocque.
Three circles inside a larger circle 2006-04-16
From Meghan:
Given three congruent circles tangent to one another (radii = 1), what is the radius of a circle circumscribed around them?
Answered by Stephen La Rocque.
Mayan multiplication 2006-04-16
From Marca:
I'm researching the Mayans, and my advisor, Dr. McDonald, stated that you may know whether the Mayans knew how to multiply. I'm finding conflicting information on the internet, and anything you could provide would be greatly appreciated.
Answered by Chris Fisher.
Volume of a tennis ball can 2006-04-16
From Jon:
Tennis balls are generally packaged in circular cylinders that hold 3 balls each. The diameter of a tennis ball is 6.5 cm. Find the volume of a can of tennis balls.
Answered by Stephen La Rocque.
How many arrangements are there? 2006-04-16
From Jim:
Can you tell me how many different combinations there are using 3 letters and 4 numbers. Letters first then numbers, example -aaa0001, aaa0002, aaa0003,-----------baa0001, baa0002, ------zzz9999.
Answered by Stephen La Rocque.
Overlapping area of two circles 2006-04-15
From Jade:
Given two identical circles where the radius (6 units) is the distance between the centers, what is the area of the overlapping region?
Answered by Stephen La Rocque.
What is the cost price? 2006-04-14
From Purushotham:
If the selling price is 120 and the profit is 20%, what is the cost price?
Answered by Stephen La Rocque.
Price percentages 2006-04-13
From Kristine:
An item that is no longer on sale at "25% off" receives a price tag that is 33 1/3% more than the sale price. Has the item price been restored to its original price? Why or Why not?
Answered by Stephen La Rocque.
Perimeter of a triangle atop a square 2006-04-13
From Mary:
There is an (irregular) pentagon that is made up of a square and a triangle. The triangle is directly on top of the square. The triangle is a right triangle. One of the angles is 45 degrees. One of the sides of the triangle is 4. What is the perimeter of the pentagon?
Answered by Penny Nom.
How to find the odds for a lottery jackpot 2006-04-12
From Harvey:
Is it best to use factorials to calculate the odds of winning a lottery, such as the MegaMillions that is popular in the US, or is there a better way?
Answered by Stephen La Rocque.
Finding the inverse of a function 2006-04-12
From Sam:
If g(x) = 2x+3/5 then how do I find the inverse?
Answered by Walter Whiteley.
Probability of a committee with 3 parents and 3 teachers 2006-04-12
From Javier:
A committee consisting of 6 people is to be selected from 8 parents and 4 teachers. Find the probability of selecting 3 parents and 3 teachers.
Answered by Stephen La Rocque.
Finding square roots 2006-04-12
From Fehmida:
I would like to know about tricks or formulas to do complicated square roots. ex. square root of 1029
Answered by Penny Nom and Steve La Rocque.
An epicycloid 2006-04-10
From Sharon:
What is the name of the curve formed by a point on the circumference of a circle that rolls on the outside of a fixed circle? This curve is used in the study of gears.
Answered by Stephen La Rocque and Penny Nom.
Given three angles and a side 2006-04-09
From Jon:
How do you figure out the length of all sides of a scalene triangle if given the measure of all angles, and one side?
Answered by Stephen La Rocque.
How many billions make a trillion? 2006-04-08
From Bill:
How many billions make a trillion?
Answered by Penny Nom.
Two circles 2006-04-07
From Louisa:
One circle of radius 7cm is touching another circle of radius 4cm. These circles are on a line and the problem is to find the length AB where A is the point marking the bottom of the radius of one circle and B is the point marking the bottom of the radius of the other circle.
Answered by Stephen La Rocque.
How many people are in the race? 2006-04-07
From Marty:
You're in a race with 1/5 of the racers ahead of you and 5/6 of them behind you. How many people are in the race?
Answered by Penny Nom.
Finding the side of a triangle 2006-04-06
From Carole:
okays, well, im having difficulty finding the side of a triangle. It is a right triangle and the information given is that the hypotonous is 24 and the angle adjacent to the 90 degree is 32. Im trying to find X, which is placed on the bottom leg of the triangle, and have no idea how to do it. can you explain to me how to get the answer, please? i'd like to know for future reference, please. thank you.
Answered by Stephen La Rocque.
A region described by the lengths of four sides 2006-04-06
From Mark:
I have an area that is 77ft in length at the top, 83ft at the base. One side is 46ft and the other side is 20ft. What will be the square feet for this area?
Answered by Stephen La Rocque.
x^2+2x-3 is a factor f(x)=x^4+2x^3-7x^2+ax+b 2006-04-06
From Maha:
Given that x2+2x-3 is a factor f(x)=x4+2x3-7x2+ax+b, find a and b and hence factor f(x) completely.
Answered by Penny Nom.
Equations of two lines 2006-04-05
From Wael:
Determine a cartesian equation of the line passing through A and V as a direction vector?

1- A (2;1) and V = j

2- A (2;3) and V = i

Answered by Penny Nom.
A square in a circle 2006-04-05
From Lisa:
A square is inscribed in a circle. Determine the percent of the circle's area that is outside the square.
Answered by Stephen La Rocque.
A 20 foot container 2006-04-05
From Karen:
can you tell me how many cubic square meters are in a 20 foot container
Answered by Stephen La Rocque.
Probability of getting their stories straight 2006-04-05
From Amy:
Four students drive to school in the same car. They claim they were late because they had a flat tire. Assuming they did not have a flat tire what is the probability that they all would choose the same tire if asked which one was flat.
Answered by Stephen La Rocque.
x-7=20 and 2x+5=17 2006-04-04
From Janette:
I'm trying to help out my daughter, it has been much too long since I've been in school. The question is x-7=20 and the other question is 2x+5=17. What are the formulas?
Answered by Penny Nom.
Mastering the multiplication tables 2006-04-04
From Ellie:
I need a detailed study plan fro helping my son master his multiplication tables.
Grade: 3 elementary

Answered by Paul Betts.
2x+5y=3 And -x+3y=-7 2006-04-03
From Lloyd:
simplify 2x+5y=3 And -x+3y=-7
Answered by Penny Nom.
List price plus 10% 2006-04-03
From Heather:
We shipped 2 pieces of an item with a unit price of $578.74 for a total amount of $1157.48. My task is to bill the customer at LIST + 10%. What formula do I need to use to calculate this?
Answered by penny Nom.
An exponential function 2006-04-03
From Meadow:
The graph of an exponential function passes through the points (0,1) and (4,10) find the constant of growth or decay. State the function.
Answered by Stephen La Rocque.
The size of a piece of property 2006-04-02
From Nancy:
My name is Nancy and we purchased a piece of property and I am trying to determine whether it is an acre or less. Could you figure this out for me.
Here are the measurements of the property:
The front is 128.02'
The back is 210'
The left side is 220'
The right side is 251.61'
I am very anxious to find out if it is over an acre.
Thank you for any help you can give.

Answered by Penny Nom.
The centre and radius of a circle 2006-04-02
From Kaye:
I need to calculate Dimension E and F. I am given A, B, C, (or over all A+B+C), D, G. The radius is one continuous unknown radius. Example: A = 23.50
B = 35.50
C = 0.50
D = 11.50
G = 23.50
I have calculated this for angles but my mind is drawing a blank for the radius calculation. I can draw it but I need to put into Excel spreadsheet.

Answered by Harley Weston.
Some probability questions 2006-04-02
From Amy:
I am a student and need help on some probability questions I really need your help on some problems dealing with probability. If you will show me how to do the following:
Answered by Harley Weston.
The Mitchell Park Domes 2006-04-01
From Nicole:
I would like to know how I could find the volume underneath the Mitchell Park Domes? The Diameter of the Domes is 140ft, 85ft high and 15,000 square feet.
Answered by Stephen La Rocque.
The height of an isosceles triangle 2006-04-01
From Chris:
how do you work out the height of an isosceles triangle if i know the length of the base but i don't know any angles or the lengths of the sides?
Answered by Penny Nom.
Find the point of inflexion for the curve y = e^x/(x^2-1) 2006-03-31
From Sam:
Hi, i am trying to find the point of inflexion for the curve y = ex/(x2-1) and i got a really complex expression for y". I can't seem to solve x4-4x3+4x2+4x+3=0 so does that mean there is no point of inflexion?
Answered by Penny Nom.
Can one divide a circle into 4.5 parts 2006-03-30
From Chris:
If it is possible, can one divide a circle into say, 4.5 parts (with 4 equal parts and a half part)?
Or for that matter, for any integer, n, into n/2 parts as above?]

Answered by Walter Whiteley.
A plant increases its height by 1/2 the first day,... 2006-03-30
From Andy:
A plant increases its height by 1/2 the first day, 1/3 the second day, 1/4 the third day and so on. How many days did it take to grow to 100 times its height?
Answered by Stephen La Rocque.
A fence around a pen 2006-03-30
From Daryl:
I hope you can help me out with the attached problem, It has been driving me crazy.
Answered by Stephen La Rocque and Penny Nom.
Superimposing images 2006-03-30
From Laurie:
I am a parent and trying to find real world applications for the math strand that includes superimposing one item on another to determine identical. The Alberta strand is match size and shape of figures by superimposing one on top of the other. I am trying to find a way to put this into context for everyday life. Any ideas?
Answered by Penny Nom.
cos(3X) 2006-03-29
From Joshua:
I'm having trouble proving that cos(3X)=cos3X -(cosX)(sin2 X)
Answered by Penny Nom.
Exponential decay 2006-03-29
From Casey:
Fifty milligrams of a drug was injected into a patient at 6am. The drug is known to be eliminated according to the law of exponential decay. At 8.30am it was determined that 60% of the drug remained in the body. How many milligrams will remain in the body at midday?
Answered by Penny Nom.
Angular velocity 2006-03-28
From Ben:
How do you (a) express angular speed in terms of linear speed and (b) express linear speed in terms of angular speed?
Answered by Stephen La Rocque.
A table of values 2006-03-28
From Cindy:
Create a table of values for each of the following functions: y=5x, y=52, and y=5x. I dont know to do this.
Answered by Stephen La Rocque.
A line parallel to a plane 2006-03-28
From Ryan:

I was wondering how I would prove the following theorem. I am completely lost at this point and all my other ideas have been extremely complicated and somewhat blurry.

"Through a point outside a given plane, there is at least one line parallel to the given plane."


Answered by Chris Fisher.
Permutations and probability 2006-03-27
From Alyssa:
I am a high school student, my teacher gave the class a worksheet on permutations and probability and told us to do independent work when we have not covered the material yet and he will not answer any questions. I am lost and don’t know where to begin. Can you help?

1. Find the number of 6-letter permutations of all the letters in EUCLID that end with either the letter E or the letter D?

Aluminum chips A, B. C, and D weigh 1g. 5g. 10g. and 20g. respectively. How many different masses can be measured by using one or more of the 4 weighs on a balance scale?

Answered by Stephen La Rocque.
The area of a triangle 2006-03-26
From Colby:
the length of one side of a triangle is 2cm less than twice the length of the altitude to that side. the area of the triangle is 30 cm squared. find the length of the altitude.
first of all, what is an altitude? and how do i find it?

Answered by Penny Nom.
Adding fractions 2006-03-26
From Barbara:
I know that to subtract 1/4 from 2/3 I must find a common denom. Now the 2/3 becomes 8/12.....i understand the 12, but where does the 8 come from?
Answered by Penny Nom.
Solve the equation cos x = sin 20 where x is acute. 2006-03-26
From Elle:
Solve the equation cos x = sin 20 where x is acute.
Answered by Stephen La Rocque.
The area of a block of land 2006-03-26
From Ronald:

I have a building block of land with four unequal sides and only one right angle. I want to know the total area (in metres) and how the calculations were carried out.

The four sides are: Rear of property: 9.14 metres
left side: 36.9 metres
Right side: 32.61 Metres
front to street: 27.43 Metres

The front to street and right side constitute a right angle. but there are no others.


Answered by Penny Nom.
Can an equilateral triangle have an obtuse angle? 2006-03-26
From Chris:
Can an equilateral triangle have an obtuse angle?

I'm thinking not, because all sides must be equal, but
does that also imply that all angles are equal?


Answered by Stephen La Rocque.
A scale factor 2006-03-25
From Jackie:
I need a formula or step by step help to show my child how to find missing dimensions for example: The scale factor for a model is 10 inches = ________ feet
Model 39.7 in
Actual 11.9 ft

Answered by Penny.
what is 9x3h for h = 2 2006-03-25
From Dillon:
what is 9x3h for h = 2 just don't get it.
Answered by Penny Nom.
The cartesian product of a countably infinite collection of countably infinite sets 2006-03-25
From Geetha:
Is the cartesian product of a countably infinite collection of countably infinite sets countable infinite?
Answered by Penny Nom.
Two squares 2006-03-25
From Debbie:
A small square is constructed. Then a new square is made by increasing each side by 2 meters. The perimeter of the new square is 3 meters shorter than 5 times the length of one side of the original square. Find the dimension of the original square
Answered by Stephen La Rocque.
A 25 foot ladder is leaning against a building. 2006-03-24
From Ali:
A 25 foot ladder is leaning against a building. The base of the ladder is 7 feet from the building. How high up the building does the ladder reach?
Answered by Stephen La Rocque.
A number puzzle 2006-03-22
From A teacher:
Students brought this website to my attention and asked why this puzzle worked... I'm not sure. The url is: http://digicc.com/fido/ and it tells you to choose a 3 or 4 digit random number with different digits. Write it down, rearrange, subtract the smaller from the larger. then circle a nonzero digit, type the remaining digits into the space provided and they will tell you the number you circled. Can you provide the reason that this works.
Answered by Claude Tardif and Penny Nom.
Piecewise functions 2006-03-21
From Kris:
First Problem:
Southeast Electric charges .09 cents per kilowatt-hour for the first 200 kWh.The company charges .11 cents per kilowatt-hour for all electrical usage in excess of 200 kWh. How many kilowatt-hours were used if a monthly electric bill was $57.06? The answer I came up with is 360, is that right? and also how do I set it up in an equation form?

Second Problem:
A construction worker earned $17 per hour for the first 40 hr of work and $25.50 per hour for work in excess of 40 hr. One week she earned $896.75. How much overtime did she work? I came up with 8.5 hrs over-time worked. Again I don't know how to set up the equation to come up with the answer.

I need some pointers on how to figure out story problems! If you have any suggestions that would help me out I would be very grateful.

Answered by Penny Nom.
The volume of water in a cone 2006-03-21
From Ghulam:
A vessel has the shape of an inverted cone.The radius of the top is 8 cm and the height is 20 cm. Water is poured in to a height of x cm.Show that if the volume of the water is V cubic cm,then V=(4/75)pi x3.
Answered by Penny Nom.
How far from Earth is Neptune? 2006-03-21
From Scott:
A laser beam travelling at the speed of light bounces off the planet Neptune and returns to Earth in 29 960.72 s. The speed of light is 299 792.5 km/s. How far from Earth is Neptune?
Answered by Penny.
The waterway between Lake Huron and Lake Superior 2006-03-21
From Trenae:
the waterway between lake huron and lake superior separates the u.s and canada.it is usually 13 feet above the water when its closed and each section is 210 feet long if the angle of elevation is 70 degrees then what is the distance from the top of the drawbridge to the water and the width of the gap created by the 2 sections of the bridge
Answered by Penny Nom.
Three towns are located at the vertices of an equilateral triangle 2006-03-20
From A student:
three towns are located at the vertices of an equilateral triangle. The towns are 8, 5, and 3 miles, respectively, from a store. How far apart are the towns?
Answered by Chris Fisher.
A manufacturer of cotton pins 2006-03-20
From Nirmal:
A manufacturer of cotton pins knows that 5% of his products are defective. If he sells cotton pins in boxes of 100 and guarantees that not more than 10 pins will be defective, what is the approximate probability that a box will have the guaranteed quantity?
Answered by Penny Nom.
What percentage of the seniors polled yes? 2006-03-20
From Jean:
An equal number of juniors and seniors responded to the question, "Do you like math?" Each respondent said yes or no. If 70% of those who said yes were seniors and 80% of those who said no were juniors, what percentage of the seniors polled yes?
Answered by Penny Nom.
A bill with a 15% tip and a 10% discount 2006-03-20
From Tracy:
A $4500 bill is presented, without tax. 10% discount coupon. Add 15% tip with 3 people spitting for the sever.
Answered by Penny Nom.
Linear feet on a paper roll 2006-03-20
From Vishal:
I WOULD WISH TO KNOW OF A STANDARD FORMULA BY WHICH I CAN CALCULATE THE LINEAR FEET OF A PAPER ROLL AS PER THE FOLLOWING DETAILS:-

THICKNESS: 150 GSM (GRAMS PER SQUARE METER)
REEL WIDTH: 85 CM
REEL DIAMETER: 140 CM
REEL INNER CORE: 10 CM
REEL WEIGHT: 946 KG's


Answered by Stephen La Rocque.
Quarts, liters cubic yards and soil 2006-03-19
From Kathy:
I am trying to find out how many quarts of dirt there are in a yard of dirt. The local flower store sells bags of dirt that are 20 quarts or 22 liters for $1.25 and another store sells a yard of dirt for $25.00. Which is the better deal?
Answered by Penny Nom.
The day of the week 2006-03-19
From Neil:

A friend of mine has an excellent memory for dates and events that occurred on that date, but his party piece is the ability to tell you the day of the week that fell of any given date. This talent is all the more extraordinary because the answer comes back in less than 2 seconds (often under 1 second). By his own admission he is no mathematician.

Now his memory and knowledge are without question, but I challenged him that because of its diversity this party piece could not be based upon memory, but on mathematics. I believe that with the correct mathematical approach and the use of a common algorithm, anyone with a basic mathematical mind can do this in their head.


Answered by Chris Fisher.
The square footage of an area in my backyard 2006-03-17
From Kim:
I need to find out how to calculate the square footage of an area in my backyard that is in the shape of a "slice of pie". There are two sides that are straight lines that come together at the top to form a point, and then at the bottom is a curved line that joins the two other lines together. I need to figure out how to calculate the square footage that is inside the area.
Answered by Penny Nom.
Building a flower bed 2006-03-17
From Bobby:
I am building a flower bed 60 ft long by 8 ft wide by 3 ft deep. How much dirt will it take to fill it with top soil/dirt.
Answered by Penny Nom.
If a circle is elongated into an oval .. 2006-03-16
From Bruce:
If a circle is elongated into an oval, with constant circumference, does the area remain constant?
Answered by Walter Whiteley.
Back to the nines 2006-03-15
From Victoria:

Can you answer this problem, does an answer exist?

  • Get a set of numbers 1-9 !
  • Using the whole set of nine number tiles (digits 1-9), try to arrange them to make three 3-digit numbers so that the sum of the first two is the third.
Can this be done without carrying over? If not can it be done without carrying over into the hundreds column?

Answered by Claude Tardif.
A 3-dimensional star 2006-03-15
From Rachel:
I am trying to figure out the name of a figure that consists of 8 (I think) square pyramids. There is a net drawing of this figure somewhere and I can't find it because I don't know the name. When all 8 pyramids are connected, they form a '3-dimensional star'. We did this project when I was 14 years old and I am now a 27 year old teacher and I would love to do this project with my kids.
Answered by chris Fisher.
Rectangles with the same perimeter 2006-03-14
From Kristine:
If two rectangles have the same perimeter, will they also have the same area? Why
Answered by Penny Nom.
2x+3y=0 and 3x-y=0 2006-03-14
From Lisa:
my name is lisa I am doing math for work and i have a math problem to solve and i need help with it here is the question 2x+3y=0 and 3x-y=0 this is one question can you help me please
Answered by Penny Nom.
Adding a cellar 2006-03-14
From Spencer:

You want to add a cellar to your store. The construction company told you it would cost $5.90 per cubic yard to dig the cellar plus $20.00 per square ft. to finish it.

How much will it cost to add a cellar 36 feet long, 14 feet wide, and 8 feet deep?


Answered by Stephen La Rocque.
A circle containing two points 2006-03-13
From Skye:
Write the equation of a circle having area 15pi and containing the points (1, 5) and (1, 9).
Answered by Penny Nom.
A fifth degree polynomial 2006-03-13
From Forrest:
When f(x) = x5 - 3x4 - 6x3 + 3ax2 - 24 is divided by x2 - 2 the remainder is bx.
Find the values of a and b.
Hence solve the equation f(x) = -8x

Answered by Penny Nom.
f(x) is a polynomial of degree 3. 2006-03-13
From Meadow:
f(x) is a polynomial of degree 3. It leaves a remainder of 10 and 4 when divided by x + 1 and x - 2 respectively. Given also that f(1) = f(-2) = 0, find the remainder when f(x) is divided by x - 3.
Answered by Penny Nom.
The kangaroo high school band 2006-03-12
From Cristy:
Two hundred students joined the kangaroo high school band. Based on statistics from previous years, 1/3 of the students will drop out. of the remaining members, only 6% will qualify for the state band. How many of the original members can be expected to qualify for the state band?
Answered by Stephen La Rocque.
The area of a parallelogram 2006-03-12
From Shayne:
Is there a method of finding the area of a trapezium or/and parallelogram even if the height is not given? Could you also explain how to find the height?
Answered by Penny Nom.
On the blueprint, the scale indicates that 2 cm represents 5 ft. 2006-03-11
From Teresa:
On the blueprint, the scale indicates that 2 cm represents 5 ft. What are the actual dimensions of a room that is 4.8 cm by 6 cm on the blueprint?
Answered by Penny Nom.
How many students and how many cookies are there? 2006-03-10
From Stephanie:
A group of students is trying to evenly divide the cookies remaining from a bake sale. If each student takes three cookies, there are five cookies left. If each student takes four cookies, they are two cookies short. How many students and how many cookies are there?
Answered by Penny Nom.
What is the height of the pot? 2006-03-10
From Michelle:
If a pot is 11 inches in diameter and holds 5.8 litres what is the height of the pot? - assuming it is a cylinder.
Answered by Stephen La Rocque.
George and his car 2006-03-10
From Catrina:
George's car gets 3 more miles per gallon during highway driving that it does during city driving. Recently he drove 112 miles on a highway and 150 miles in the city and used exactly 10 gallons of gas. How many miles per gallon does his car get during city driving?
Answered by Stephen La Rocque.
The number of gallons in a tank 2006-03-09
From Doug:
We have a round tank that is 216 inches long 64 inches wide across ; how do we find the number of gallons it could hold ?
Answered by Stephen La Rocque.
A 4-digit Combination Lock 2006-03-09
From Adam:
I have a 4-digit Combination Lock but i have forgotten the code. i do know that it is an even number, all the digits are different, the second digit is zero and the largest digit is 6. what are all the possibilities.
Answered by Stephen La Rocque.
8,_,4,9,1,_,10,3,_,0 2006-03-08
From Rachel:
I cant figure out this sequence if someone could help me 8,_,4,9,1,_,10,3,_,0
Answered by Claude Tardif.
Factors of a cubic 2006-03-08
From Skye:
Show that (x+2) is a factor of f(x) where:

f(x) = px3 + (4p+2)x2 + (5p+4)x + 2p

Find the range of values for which the equation f(x) = 0 has 3 distinct real roots

Answered by Penny Nom.
The dimensions of a poster 2006-03-08
From Sonu:
poster measures 150cm by 180cm is enlarged in the ratio 8:5 find the length and breath of the enlarged poster.
Answered by Stephen La Rocque.
What names are known for the quarter circle shape? 2006-03-06
From Christina:
What names are known for the quarter circle shape?
Answered by Stephen La Rocque and Penny Nom.
A nine digit number 2006-03-06
From Ryan:
What is the total number of possible combinations of a nine digit number (ie., social security number) including repeating numbers?
Answered by Stephen La Rocque and Penny Nom.
2,4,9,6,5,6,____,____,____,... 2006-03-06
From Mike:
I am having problems figuring out the following sequence:

2,4,9,6,5,6,____,____,____,...

We were able to guess that the pattern simply started to reverse itself, but I was wondering if there were other possibilities.


Answered by Claude Tardif.
A hat contains between 10 and 25 marbles 2006-03-06
From Kerry:
A hat contains between 10 and 25 marbles. Some marbles are green, and the rest are yellow. Without looking you are to reach into the hat and pull out a marble. The probability of pulling out a green marble is 2/9. How many marbles are in the hat and explain?
Answered by Stephen La Rocque and Penny Nom.
Heights and shadows 2006-03-06
From Debra:
A person is 7 ft tall and his shadow is 10 ft tall. using the same info what is the shadow of a person who is 5ft tall
Answered by Stephen La Rocque.
The nth term 2006-03-05
From Umar:
my question is what is the nth term for a house of cards 4 stories high if you use the following numbers:
2 7 15 26

Answered by Penny Nom.
An equation involving logarithms 2006-03-05
From Chris:
Given that loge y = 0.3logex+1.2 show that y=3.32 x0.3

Answered by Penny Nom.
1/infinity and 1/0 2006-03-04
From Evan:
I was thinking the other day when i was in math class that when you divide 1 by say n you'll get 1/n. As the value of n increases the smaller the number you get. So if you divide 1/infinity would that equal zero? And if that is true then would 1/0=infinity be true also?
Answered by Penny Nom.
Balls in a cylinder 2006-03-03
From Roxanne:
If a cylindrical can holds three balls with no room to spare, explain how to find the volume of the air that surrounds the balls in the can. Yet the balls have a diameter of six inches
Answered by Stephen La Rocque.
The quadratic formula 2006-03-03
From Miriam:
Please show work and indicate "answer", using the quadratic formula -

(x+2) (2x+3) = 6

Answered by penny Nom.
Volume 2006-03-02
From Irene:
how do you find the volume in cubic units. i need the basic formula to help my child.
Answered by Penny Nom.
25+36+12=52+m, m= 2006-03-02
From Jenny:
25+36+12=52+m, m=
Answered by Stephen La Rocque.
A cone with an oval as base 2006-03-01
From Richard:
I am trying to find the volume of a cone that is not round but oval.
Answered by Penn Nom.
Separate 35 into two parts such that 4 times the larger is 4 less than five times the smaller 2006-03-01
From Barbara:
Separate 35 into two parts such that 4 times the larger is 4 less than five times the smaller
Answered by Stephen La Rocque.
An equilateral triangle and a regular hexagon 2006-02-28
From Trevor:
An equilateral triangle and a regular hexagon have equal length perimeters. What is the ratio of their areas?
Answered by Penny Nom and Stephen La Rocque.
The volume & surface area of a rectangular pyramid. 2006-02-27
From Cheryl:
My daughter is working on a math project & we are having a hard time finding the volume & surface area of a rectangular pyramid. Can you help us with this?
Answered by Penny Nom.
Luke and Slim have only one horse. 2006-02-26
From Emily:
Luke and Slim have only one horse. Luke rides for the agreed on distance and then ties up the horse for Slim, who has been walking. Meanwhile, Luke walks on ahead. They alternate walking and riding. If they walk 4 miles per hour, and ride 12 miles per hour, what part of the time is the horse resting?
Answered by Penny Nom.
The path of a submarine 2006-02-26
From Meadow:
Suppose that a submarine has been ordered to follow a path that keeps it equidistant from a circular island of radius r and a straight line shoreline that is 2 units from the edge of the island. Derive an equation of the submarine path, assuming that the shoreline has equation x = -p and that the center of the island is on the x-axis.
Answered by Penny Nom.
Diluting an acid solution 2006-02-25
From Alex:
IF YOU HAVE 1 Litre OF A 70% ACID SOLUTION, AT WHAT RATIO DO YOU DILUTE IT WITH WATER TO ACHIEVE A 30% AND 50% ACID SOLUTION?
Answered by Penny Nom.
Select a month at random 2006-02-24
From Josh:
Find the probability that the name of a month of the year,chosen at random, begins with a consonant.
Answered by Penny Nom.
A bag contains 6 red marbles,9 blue marbles,5 green marbles. 2006-02-24
From Jen:
A bag contains 6 red marbles,9 blue marbles,5 green marbles. You withdraw one marble,replace it,and then withdraw another marble. What is the probability that you do not pick two green marbles?
Answered by Stephen La Rocque.
The area of a flag 2006-02-24
From Kim:
I have a flag that's very similar to the canadian flag. Two smaller red areas and i larger white area. If I am given the length is 3 and the height is 1.4 how do I determine what the area of the two red areas is?

Here's the answer but I can't figure out why they are multiplying 1.4 x 1.4?

(3x1.4-1.4x1.4)divided by 2= 2.24 divided by 2 = 1.12

Answered by Penny Nom.
The graph of y = 2x + 1 2006-02-23
From Geneva:
I'm having trouble with graphing equations like y=2x+1 which is my homework I'm not sure how to graph it. I'm in the 8th grade taking a 9th grade course algebra I.
Answered by Penny Nom.
Cutting off the top of a triangle 2006-02-22
From Ken:
I'm a creative type who needs to find how you can determine the horizontal, relative percentages within a triangle. Attached is a pdf showing approximately the
50% line, but I'd like to know how to determine any percentage (horizontally) within a triangle . I know that if I put a horizontal line though the height of the triangle I won't be showing 50% above and 50% below. The bottom part has to be larger (correct me if I'm wrong).

I'm not concerned about height as I am wanting to break down the percentages into 60% at the bottom, 25% in the middle and 15% at the top.


Answered by Penny Nom.
A triangle of 50p pieces 2006-02-22
From Stuart:

Ok, so i am collecting 50p pieces and arranging them on my desk in the shape of a triangle.
eg
50p
50p 50p
50p 50p 50p
50p 50p 50p 50p

I want to work out how much money I'm saving just by knowing how many rows of coins there are. If i can work out how many coins there are just by knowing how many rows I have I can just divide by 2 to find out the amount in dollars.


Answered by Penny Nom.
Daddy Warbucks' cash 2006-02-22
From Grace:
Daddy Warbucks always carries a specific number of $100 and $500 bills for impulse purchases and $1 bills for tips. If he has 500 bills in his briefcase and they total $50,000, how many bills of each denomination does he carry?
Answered by Stephen La Rocque.
The nth term of a sequence 2006-02-21
From Mike:
I'm having trouble finding the nth term in an equation, can you help me?
Answered by Stephen La Rocque.
The Least Common Multiple of two primes 2006-02-21
From Kristine:
Is the Least Common Multiple to two prime numbers always their product? Why or Why not?
Answered by Stephen La Rocque.
Stem-and-leaf plots, box-and-whisker plots and medians 2006-02-21
From A student:
explain which plot would be easiest to use to find the median....box-and-whiskers plot or stem-and-leaf plot
Answered by Penny Nom.
Find the xy equation of the curve on which Brian is located. 2006-02-20
From Skye:
Andrew, located at (0, -2200) fired a rifle. The sound echoed off a cliff at (0,2200) to Brian, located at point (x,y). Brian heard the echo 6 seconds after he heard the original shot. Find the xy equation of the curve on which Brian is located. Assume the distances are in feet and that sound travels at 1100 feet/second. (hint: find the equation of the hyperbola)
Answered by Stephen La Rocque.
Translate and magnify a triangle 2006-02-19
From A student:
Triangle CAT has vertices located at C(-2,5), A(-5,1), T(1,1)

Translate triangle CAT 4 units right and magnify the triangle by 3. List the new coordinates and explain the process of computing the new coordinates.

Graph triangle CAT and the new triangle form part A on the same coordinate plane....

Will the area of the new triangle be 3 times as large as the original?? Explain why or why not

Answered by Penny Nom.
Outliers in a box and whisker plot 2006-02-19
From A student:
i need help on determining if their is an outlier...i know how to find the median and the lower quartile and the upper quartile..but i don't understand about the outliers....please tell me if their is an outlier in this problem....the numbers are...63,88,89,89,95,98,99,99,100,100
Answered by Penny Nom.
Whose photo is he looking at? 2006-02-19
From Adeyeri:
Looking at a picture a man says, that the father of the person (in photo) is the son of my father. whose photo is he looking at?
Answered by Penny Nom.
Another normal distribution problem 2006-02-18
From Mary:
Assume that blood pressure readings are normally distributed with a mean of 120 and standard deviation of 8. A researcher wishes to select people for a study but wants to exclude the top and bottom 10 percent. What would be the upper & lower readings to qualify people to participate in the study?
Answered by Penny Nom.
Potting soil 2006-02-17
From Cheryl:
I need to buy some potting soil for a raised planting bed that is 6'x24'x1'. How do you figure how many 2 cu. ft. bags of potting soil that I will need?
Answered by Stephen La Rocque.
One acre 20 feet deep 2006-02-16
From Bob:
How many cubic yards of material are in one acre of land approximately 20 feet deep? What is the approximate weight of one cubic yard of Black dirt, Gravel, and Sand?
Answered by Penny Nom.
Broken-line graphs and histograms 2006-02-16
From George:

1. What is the main difference between a broken-line graph and a histogram? Both represent continuous variables.

2. What is the correct way to read a multiplication array: x-axis first and then y-axis, other way around or it doesn't matter?


Answered by Penny Nom.
The height of a cylinder 2006-02-15
From Autumn:
the surface area of a cylinder with circumference 24pi cm. is 300pi cm2 . what is the height???
Answered by Penny Nom.
Orbital velocity 2006-02-15
From Lizzie:

Assume the world is a spherical planet with a diameter of 1600 KM. with a uniform density or 5200 KM.

It the moon is in a circular orbit of 630 KM. above the surface of the world,what is it's orbital velocity in metres per second?Round to the nearest WHOLE number.


Answered by Stephen La Rocque.
8 - 5r / 6 = 3 2006-02-15
From Scott:
I was trying to help my daughter with the following problem:

8 - 5r / 6 = 3

Can you give me a quick refresher.


Answered by Stephen La Rocque.
A normal distribution problem 2006-02-15
From Mary:
In a certain normal distribution, find the mean when the standard deviation is 5 and 5.48% of the area lies to the left of 78.
Answered by Penny Nom.
How many flat surfaces does a cylinder have? 2006-02-14
From Kimia:
How many flat surfaces does a cylinder have?
Answered by Stephen La Rocque.
Zhi Wei was born on christmas day in 1983. 2006-02-14
From Clement:
Zhi Wei was born on christmas day in 1983. His father shares the same birthday, but he was born in 1951. In which year will Zhi Wei's age be exactly 1/3 of his father's age?
Answered by Stephen La Rocque.
Golfing with 8 or 12 2006-02-13
From Carolyn:
I have a couple of questions that my dad asked me and I do not think it can be done in the manner he wants it to come out to.

He is going on a golf trip in June. There are going to be either 8 or 12 players playing 4 rounds of golf.

The questions that he says will not work and I do not know how to figure it out either is

4 Rounds of golf is for sure. 18 hole Rounds each

4 Players to a group

Either 8 or 12 players can every player play with every player in a group at least once during those 4 rounds.

If this can be done can you give me the combinations for both 8 and 12 , so I can help out my dad. Along with how you you figure this out.

Answered by Penny Nom.
A concrete slab 2006-02-13
From Cheryl:
A concrete area that will be used for picnic benches will measure 20 ft on each side. The concrete slab will be 8 inches deep. How many cubic feet of concrete will be needed to construct the patio?
Answered by Penny Nom.
Related rates and an oil spill 2006-02-12
From Brandon:

An Oil Tanker Spills 100,000 cubic meters of oil, which forms a slick that spreads on the water surface in a shape best modeled by a circular disc is increasing at a rate of 3m/min (it doesn't state what is increasing at 3m/min, so I'm assuming Radius until I can ask my teacher.) At t=T, the area of the "circular" slick reaches 100pi Sq. meters.

A) how fast is the area of the slick increasing at t=T
B)How fast is the thickness of the slick decreasing at t=T
C)Find the rate of change of the area of the slick with respect to the thickness at t=T.


Answered by Penny Nom.
X^2 + 4X + 6 is a factor of X^4 + rX^2 + s 2006-02-11
From Katie:
If X2 + 4X + 6 is a factor of X4 + rX2 + s, then find the value of r+s?
Answered by Steve La Rocque and Penny Nom.
A lampshade shape 2006-02-11
From Rose:
Is there a solid shaped like a lampshade? Or do you just call it the bottom portion of a cone?
Answered by Penny Nom.
Is there a solid shape with 5 faces, 8 edges, and 4 vertices? 2006-02-11
From Annie:
Is there a solid shape with 5 faces, 8 edges, and 4 vertices?
Answered by Penny Nom.
Equal fractions 2006-02-10
From Dylan:
write three equal ratios

3/4 _________________________


Answered by Penny Nom.
sinh(i/2) 2006-02-09
From Louis:
How can you set up an equation to find sinh(i/2)
Answered by Penny Nom.
Degree 2006-02-09
From Jessica:
I am a 9th grade home school student and have started doing degrees of terms in my math book. The following is some examples they give:

25a to the 4th power is a 4th degree term

67c to the 9th power is a 9th degree term

x is really x to the 1st power so it is a 1st degree term

10 is really 10x to the 0 power so it is a 0 degree term

They go on to say that every constant is a zero degree term.

My question is why isn't a constant, like 10, simply to the 1st power (making is a 1st degree term) like x.

Answered by Claude Tardif.
Simplify the variable expression: 3g^2h over 12gh 2006-02-09
From Andrew:
okay the question says simplify the variable expression: 3g2h over 12gh
how would u do this?

Answered by Penny Nom.
The third side of a triangle 2006-02-09
From Clayton:
How do I find the length of the third side of a triangle if a=30m, b=30m and I need to find c?
Answered by Steve La Rocque and Penny Nom.
How many 16 foot 2 by 4's are there in a 1000 board feet of lumber. 2006-02-08
From Fred:
how many 16 foot 2 by 4's are there in a 1000 board feet of lumber.
Answered by Penny Nom.
Grade 3 Math Puzzle 2006-02-08
From Jayne:
The question is to use the numbers 1 through 9. They are to be placed on lines, and each group is to add up to 11. The trick is that the first 2 numbers used add to 11, then lines 2, 3 and 4; then lines 4,5 & 6; then lines 6,7 & 8; and then lines 8 and nine (it is really hard to describe. In the book, is shows circles making each group). The lines are all horizontal, and lined up to each other like:_ _ _ _ _ _ _ _ _
Answered by Claude Tardif.
Concrete in a foundation 2006-02-08
From Philippe:
Can you please help me determine how many cubic yard of concrete I need to purchase for a foundation that is 50' by 25.5 by 8' height and how many cubic yard would I need for the slab that is 50' by 25.5 by 0.5
Answered by Penny Nom.
A locus problem 2006-02-08
From Jason:
If the distance from p(1,5) is 3/4 that of the distance of a(4,-3)? find the equation of a locus.
Answered by Penny Nom.
6^(x+1) = 12^(x-3) 2006-02-05
From BB:
6x+1 =12x-3
Answered by Penny Nom.
Find the ground speed and the planes true heading. 2006-02-05
From Jimmie:
An aircraft going from city A to city B on a bearing of S69E (degrees) is traveling at a speed of 430 mph. The wind is blowing out of the north to south at a speed of 25 mph. Find the ground speed and the planes true heading.
Answered by Penny Nom.
An irregular quadrilateral 2006-02-04
From Christopher:
Is there such a shape as an irregular quadrilateral with 4 equal sides?
Answered by Penny Nom.
Mixing 2006-02-03
From Caren:
If I am 75% flippino and 25% white and the guy is full white what will be the whole percentage of the flippino and white nationality in the baby?
Answered by Penny Nom.
Why 1/3 in the volume formula for a pyramid? 2006-02-03
From Becky:
why is the formula for finding the volume of a pyramid divide by 3?
Answered by Penny Nom.
How long and how wide is the table? 2006-02-02
From A student:
the perimeter of a table is 24 feet. the table is twice as long as it is wide. how long and how wide is the table?
Answered by Penny Nom.
Prove that p^n >= (p!)/(p-n)! 2006-02-02
From Rhydian:

PROVE:

pn >= (p!)/(p-n)!


Answered by Penny Nom.
More on the game of 24 2006-02-02
From Cathy:
10,16,9,4 this numbers have to equal up to 24. we can't figure it out.
Answered by Claude Tardif.
The box of maximum volume 2006-02-01
From Elizabeth:
A box factory has a large stack of unused rectangular cardboard sheets with the dimensions of 26 cm length and 20 cm width.
The question was to figure what size squares to remove from each corner to create the box with the largest volume.
I began by using a piece of graph paper and taking squares out. I knew that the formula L X W X H would give me volume. After trial and error of trying different sizes I found that a 4cm X 4cm square was the largest amount you can take out to get the largest volume. My question for you is two parts

First: Why does L X H X W work? And second, is their a formula that one could use, knowing the length and width of a piece of any material to find out what the largest possible volume it can hold is without just trying a bunch of different numbers until you get it. If there is, can you explain how and why it works.

Answered by Penny Nom.
The sum of two numbers is 21. 2006-01-31
From Dan:
The sum of two numbers is 21. One number is three less than the other. Find the numbers
Answered by Penny Nom.
Make 2 rows of 4 circles with only 6 circles 2006-01-31
From Sarah:
Moving as many circles as you need, make 2 rows of 4 circles only having 6 circles?
Answered by Chris Fisher.
Percentage increase in sound intensity 2006-01-29
From David:
I am doing some emotion response mapping. Being a composer I know how to push the heart beat tempo by a % increase of BPM in my music. I want to know how to calculate an increase in intensity of sound. Now I have 60 decibels. I want an increase of intensity by a factor n% with out going over 156 decibels.
Answered by Penny Nom.
Cubic yards of dirt 2006-01-28
From Greg:
How many yards of dirt do i need to cover an area of 1000 ft long, 20 ft wide and 3 inches deep
Answered by Penny Nom.
Word problems 2006-01-27
From Duk:
One kind of candy sells for 90cents more a pound than another kind. A pound of each sells for a total of $5.10. What is the selling price of each kind?


A soft drink sells for 20 cents more than the deposit required on the bottle. The cost of the soft drink and the bottle deposit totals 70cents. How much is the soft drink? The bottle deposit?

Answered by Penny Nom.
How do you find the angles in a triangle? 2006-01-27
From Keith:
How do you find the angles in a triangle if you know the lengths of the sides?
Answered by Chris Fisher and Penny Nom.
The median 2006-01-27
From Wael:
" median"; what does it mean and how do we calculate it?
Answered by Penny Nom.
how can i find the height of a triangle if i have the base and the hypotenuse 2006-01-27
From Kelsey:
how can i find the height of a triangle if i have the base and the hypotenuse
Answered by Penny Nom.
Four people are in a race 2006-01-26
From Tammy:
If 4 people are in a race, how many different placements, i.e., 1st, 2nd, 3rd, 4th, can there be and what is the equation?
Answered by Penny Nom.
Subdividing a polygon into triangles 2006-01-26
From Adam:
is there an algorithm to divide a regular polygon into N equilateral triangles having the same area (no limit on N), or if not, an algorithm to divide a regular polygon into N triangles having the same shape and size?
Answered by Chris Fisher.
The third side of a triangle 2006-01-25
From Bob:
Is there any way to obtain the third side of a triangle when 2 sides are the same length (2.18 inches). I also need to find the angles.
Answered by Penny Nom.
Sectors and arcs 2006-01-25
From Wael:
How is the area of an arc (alpha*pi*r squared/360) derived?
How is the length of an arc (alpha*pi*r/180) derived?

Answered by Penny Nom.
Right angles 2006-01-25
From AshLee:
I was recently given a challenge in my Algebra class. My teacher wanted to know about a right angle. He said he would give five bonus points to the person that could bring in information. (I know five bonus points may not seem like a lot but in this class, they are.) I looked on this site and I found out why it was called a right triangle, but my teacher want to know where did that theory come from. I was wondering too... not just because of the bonus points.
Answered by Penny Nom.
Composite triples 2006-01-24
From Laeah:
question 1 Find the smallest integer n such that n+1, n+2,and n+3 are all composites.

question 2 If n = 5! +1, show that n+1, n+2, and n+3 are all composite.

question 3 Find the sequence of 1000 consecutive composite numbers.

Answered by Penny Nom.
A circle problem 2006-01-23
From Matyan:

Find the standard equation of the circle that passes through the points (0,4) and (3,7) with center on the line
2x-y+4=0.

I've tried substituting the two points to the general equation of a circle, but I can't solve it without a third equation. The problem here is I don't really know how to use the given line of 2x-y+4=0. Please help. Thank you.


Answered by Penny Nom.
The three angles in a triangle 2006-01-23
From A student:
the measure of the 2nd angle in a triangle is 4 more than the measure of the 1st angle. the measure of the 3rd angle is eight more than twice the measure of the 1st angle. find the measure of each angle.
Answered by Penny Nom.
Two parallel lines 2006-01-22
From Marbara:
On my math homework I have a problem where it says find the value of x so that L is parallel to M

on one of the there are three lines. the top one is horizontal which is L. the bottom one is also horizontal directly under L which is M. there is another line that is diagonal going through the middle sections of L and M.

To the right of the diagonal line and right under line L is an equation 5x-10

to the right of the diagonal line and right above line M is an equation 8X-5

I tried to find what X is but I can't because it never matches both equations no matter what way I try to do it.

Answered by Penny Nom.
The angles in a hexagon 2006-01-22
From Linda:
My problem is in relation to wood and making a six sided object from it. On my saw, there is a place to set the angle to which you wish to cut. I cannot for the life of me, figure this out. I am starting with a piece of plywood (1/4" x 6" x 18") and need to know what the angle degree would be to make each of the sides match perfectly to form a hexagon. Trial and error just is not working. Can someone help me?
Answered by Penny Nom.
A binomial distribution exercise 2006-01-21
From Belinda:
In a survey of 15 manufacturing firms, the number of firms that use LIFO (a last-in first-out accounting procedure for inventory) is a binomial random variable x with n=15 and p=0.2. a) What is the probability that five or fewer firms will be found to use LIFO? Is it unlikely that more than 10 firms will be found to use LIFO? Comment.
Answered by Penny Nom.
A confidence interval 2006-01-21
From Jonathan:

I am attempting to calculate how my confidence interval will widen at the 95% confidence level if my response universe increases from 100 to 150 or to 200.

There is a universe of 54,000. I take a 5% sample for a test universe of 2,700

If my "yes" universe is 100, at the 95% confidence level, what is my +/- range? (i.e +/- 3? +/-5?)

Historically, 6.6% of the 2,700 you say "yes". I am trying to determine how the confidence interval would change if the number of "yes" responders increased to 150 or to 200.


Answered by Penny Nom.
The circumference of a larger circle is twice the circumference of a smaller circle. 2006-01-20
From Amanda:
The circumference of a larger circle is twice the circumference of a smaller circle. What is the ratio of the radius of the smaller circle to the diameter of the larger circle?
Answered by Penny Nom.
One boundary of a pond is parabolic in shape. 2006-01-20
From Glenn:
One boundary of a pond is parabolic in shape. The boundary passes through the points A(-20,45), B(40,40) and E(30,35). The equation of the parabola is of the form y=ax2+bx+c. Find the equation of the parabola and the coordinates of the vertex of the parabola. Any assistance you could provide would be greatly appreciated.
Answered by Penny Nom.
An irregular octagon 2006-01-20
From Robert:
I am building a poker table which is in the shape of an irregular octagon. I know the table measures 72 inches long and 48 inches wide with two parallel straight sides of equal length and six smaller sides of equal length ( three at each end of the table), what I don't know are the lengths of the any of the sides.
Answered by Harley Weston.
A 3 by 3 block on a calendar 2006-01-18
From A student:
Choose two different blocks of nine numbers arranged 3 across and 3 down on a calendar, and check that this rule holds: For any block of nine numbers the average of the four corner numbers is equal to the middle number. Show your work and explain why the rule works.
Answered by Penny Nom.
The centroid of a triangle 2006-01-18
From Andrea:
I am trying to find the centriod of a triangle. I have been given the three vertices of the triangle: A (-25, -20), B (-5, -40), and C (15, 30). I believe I calculated the correct midpoints: AB(E) (-5, 5), BC(D) (5, -5), and AC(F) (-15, -30). When I graphed these points I came up with the solution (-5, -10), but I can't seem to write the answer out (equations of the lines, etc) correctly. Any help would be greatly appreciated!
Answered by Penny Nom.
Interior and external angles of a polygon 2006-01-17
From Anthony:

In a regular polygon, the ratio of the interior angles to the exterior angles is 3:1.

(a) Find the measure of the interior angle.
(b) How many sides are there


Answered by Penny Nom.
A sequence of circles and tangents 2006-01-16
From Paul:
Consider a circle whose center is (2,2) and whose radius is 1, and the straight line that goes through the origin and that is tangent to this circle so that the intersection between them is as shown in the attached picture. With this new point we make a new circle whose radius is half of the first one, and we calculate the corresponding intersection point with the same suppositions as in the first case. We repeat the process to the infinite. Find the distance between the center of the circle in the infinite and the origin (point (0,0)).
Answered by Chris Fisher.
The height of a triangle from the lengths of the sides 2006-01-16
From A student:
How do you figure out the height of a triangle when all you have is the length of the sides of the triangle?
Answered by Claude Tardif.
Intercepts and slope 2006-01-15
From Anthony:
Given -1.7x + 0.8y=7.1:

(a) state the intercepts and slope for the graph

(b) Draw the graph

Answered by Penny Nom.
51 acres 2006-01-15
From A landowner:
i just wanna know how many miles are surrounding my 51 acres? a simple answer will do. thanks:)
Answered by Penny Nom.
Polar to Cartesian coordinates 2006-01-15
From Linden:
How do you convert six units at 30 degrees Polar Coordinate into a Cartesian Coordinate?
Answered by Penny Nom.
12100 sq ft is how many acres? 2006-01-14
From Sydney:
If there are 12100 Sq. Ft in property what would that be acre wise?
Answered by Penny Nom.
Who is lying? 2006-01-14
From Diane:
A business man was working in his home office when he realized he had left a five-dollar bill in the book he had been reading. He called his butler to bring him the book from the library. When he got the book, the bill was no longer there. He then questioned the maid and the butler. The maid remembered seeing the bill between pages 99 and 100 in a book to the left of a business book. The butler did not recall seeing the bill, but was sure the book was to the right of the business book, because to the left of it there was a statistics book. Who is lying?
Answered by Penny Nom.
Concrete tubes 2006-01-14
From Paul:
I have 15 tubes. Each tube measures 12in.x 48in. I have to fill each one with concrete. How do I figure how many cubic yards of concrete I will need?
Answered by Penny Nom.
What initial velocity is required to double the maximum height 2006-01-14
From A photographer:
A body is thrown into the air with an initial velocity V ft./sec. What initial velocity is required to double the maximum height previously obtained?
Answered by Penny Nom.
5x^2 - 27x - 18 2006-01-13
From Katy:
How would you factor 5x2 - 27x - 18?
Answered by Penny Nom.
Convert to decimal form 2006-01-12
From A student:
I would like to know the full answer on how to turn the fraction 3 3/8 into a decimal please.
Answered by Penny Nom.
The volume of a piece of furnature 2006-01-10
From Kimberly:
What is the cubic feet for a piece of furniture that is 38" wide X 45" tall X 18" deep
Answered by Penny Nom.
How do I find the area with only one sides measurement? 2006-01-10
From Amy:
If you have a right triangle with the 90 degree angle located in the bottom left corner and the triangles most sloped line goes from the top left down to the right bottom. And the length of the slope line is 10(radical sign)with the radicand of 2. How do i find the area with only one sides measurement.
Answered by Penny Nom.
Barrels of oil in a tank 2006-01-09
From Francis:
How many barrels of oil (42 gallons of oil in one oil tank) would there be in an oil tank that was 15 feet tall and 10 feet across?
Answered by Penny Nom.
y - y1 = m(x - x1) 2006-01-08
From Greg:
How do you change 2x + 3y = 7 into y - y1 = m(x-x1)?
Answered by Penny Nom.
GCF, LCM, primes and the ladder method 2006-01-07
From Linda:
How would I teach both finding the GCF and LCM with prime factors...I recall the ladder method vaguely.???
Answered by Penny Nom.
How many millimeters are in 12 feet? 2006-01-06
From Hunter:
How many millimeters are in 12 feet?
Answered by Penny Nom.
Differentiation, powers and logs 2006-01-06
From Claudia:

Question: how do I find the derivative of

x* ln(x+(e^2))^2

x^lnx

x^(e^(-x^2))


Answered by Penny Nom.
Liters and milliliters 2006-01-05
From Cara:
if you had 1.5 liters of water, how many milliliters would you have
Answered by Penny Nom.
The volume of a cylinder 2006-01-04
From Doug:
I have a Cylinder which is 34" x 72" how many gallons will it hold?
Answered by Penny Nom.
Milliliters, centiliters and hogsheads 2006-01-03
From A worried teen:
How many milliliters are in a centiliter? and What is an equivalent to a hogshead in liquid measuring? Your help is greatly appreciated. Sincerely worried teen
Answered by Penny Nom.
The area of an octagon 2006-01-03
From Nikki:
I want to figure out the square footage of an octagon. i have 8 panels that are 24" wide. Its for my dogs and i wanna know how much room they'll have.
Answered by Penny Nom.
The area of a sector of a circle 2006-01-02
From Natashia:
How do you find the area of a shaded sector of a circle?
Answered by Penny Nom.
A train model is created using a scale of 1:60 2006-01-01
From Jennifer:
An "n gauge " train model is created using a scale of 1:60. If an actual train engine, the "K-4" an early twentieth-century engine,is 45 feet long, what is the length of an "N gauge" model of the engine in inches?
Answered by Penny Nom.
Extraneous solutions 2006-01-01
From Liz:

Question: solve and check for extraneous solution

3(w + 1)1/2 = 6


Answered by Penny Nom.
A probability question 2005-12-31
From Ling:

Question: If an integer is randomly selected from all positive 2-digit integers (i.e., the integers 10, 11, 12, . . . , 99), find the probability that the integer chosen has

(a) a 4 in the tens place
(b) at least one 4
(c) no 4 in either place


Answered by Penny Nom.
The area of a polygon 2005-12-31
From Rahul:
I have problem in finding area of polygons. I don't know the formula to find area of pentagon, hexagon and so on.
Answered by Penny Nom.
A probability question which resulted from a game of Yahtzee 2005-12-31
From Robert:
Could someone please assist me with this probability question which resulted from a game of Yahtzee we were playing in Melbourne, Australia on our holidays.

The object on this turn was to throw a “large straight” which is 5 numbers in sequence from 5 dice numbered 1 – 6. A player initially throws all 5 dice and then selects those dice they want to throw again for a further two more times. In this instance the player on their first throw, threw a 1,2,3,4 and 6.

Question:- What is the respective probabilities of gaining a straight if they were to –

a) put back say the 6 and try and throw a 5 on the two further throws or…..
b) put back the 1 and 6 and try and throw a 1 and 5 or 5 and 6 on the two further throws bearing in mind that if one of the numbers was a 5 on the second throw they could hold that number and try for a 1 or 6 on the third throw.

I would be most appreciative if someone could assist in showing me how to calculate the probabilities particularly in the second instance (b).

Answered by Penny Nom.
Two related rates problems 2005-12-29
From Shimaera:

#1. A manufacturer determines that the cost of producing x of an item is C(x)=0.015x2+12x+1000 and the price function is p(x)=250+2x/10. Find the actual and marginal profits when 500 items are produced.

#2. At 9 a.m a car is 10km directly east of Marytown and is traveling north at 100 km/h. At the same time, a truck leaves Marytown traveling east at 70 km/h. At 10 a.m, how is the distance between the car and the truck changing?


Answered by Penny Nom.
Joey had 8 drinking glasses, all of different sizes. 2005-12-29
From Danielle:
Joey had 8 drinking glasses, all of different sizes. he has lined them up from smallest to largest and numbered them #1 (smallest) to #8 (largest). he knows that glass #4 holds 10 ounces and that glass #7 holds a pint, or 16 ounces. Now he wants to figure out how much all the others hold!

He experiments by filling up different glasses with water and pouring the water back and forth into other glasses. Her are his results:

Answered by Penny Nom.
Milliliters to grams 2005-12-29
From Kaitlyn:
If I'm using baking soda, how do I convert milliliters to grams?
Answered by Harley Weston.
One car leaves a spot traveling at 100 km per hour 2005-12-28
From Jason:
One car leaves a spot traveling at 100 km per hour. The second car leaves the same spot 15 minutes later and traveling at 120 km per hour. How long does it take for the second car to catch up to the first car?
Answered by Penny Nom.
Twelve golf balls 2005-12-28
From Todd:
You have 12 golf balls and 11 of them are the exact same weight, but one of them is either a little heavier or a little lighter. You only have three attempts to weigh the balls. How can you determine which ball weighs more/less than the rest?
Answered by Penny Nom.
If an acre of land is covered with one inch of water 2005-12-27
From Patti:
If an acre of land (43560 sq ft) is covered with one inch of water, how many gallons of water cover that acre.
Answered by Penny Nom.
A goat is tied to the corner of a 50 ft square outbuilding 2005-12-24
From Danielle:
Topic: A goat is tied to the corner of a 50 ft square outbuilding with a 40 ft. rope.
a) What is the measure of the partial circumference created as the goat walks at the full length of the rope?

b) Since the goat is trimming the grass from part of the outbuilding, how much of the perimeter of the outbuilding will the building owner have to trim?

Answered by Penny Nom.
Why do we bother learning pre-algebra and algebra 1? 2005-12-23
From Priya:
My students always ask "Why do we bother learning pre-algebra and algebra 1?" and I haven't found an answer to satisfy them yet. Can you help me? My students are from grade 9 to 11. I have tried giving them real life examples in each topic but it just feels like they are not satisfied!!!
Answered by Penny Nom.
Cubic yards of fill 2005-12-23
From Mike:
And I have a question in regards to the amount of fill needed to bring a vacant lot up to "grade level". The lot is 112' x 87' and I need to bring in fill to elevate the finished level up 4'.
Answered by Penny Nom.
Percentages in our daily life 2005-12-23
From Naina:
what are the uses of percentages in our daily life
Answered by Penny Nom.
Two dogs and a flea 2005-12-23
From Michelle:
Two dogs, each traveling at 10 ft/sec, run toward each other from 500 feet apart. As they run, a flea flies from the nose of one dog to the nose of the other at 25 ft/sec. The flea flies between the dogs in this manner until it is crush when the dogs collide. How far did the flea fly?
Answered by Penny Nom.
Mean and average 2005-12-22
From Jerry:
This is just a question from a degreed engineer that has a "understanding" problem. please go to http://www.metrika.com/3medical/hemoglobin-m.html

At the end of the page they talk about "mean" vrs "average" there does appear to be a difference. What is it?


Answered by Harley Weston.
The Mean Value Theorem 2005-12-22
From Candace:
Verify that the function satisfies the hypotheses of the Mean Value Theorem on the given interval. Find all numbers "c" that satisfy the Mean Value Theorem. 11. f(x)=3x2 + 2x +5 [-1, 1]
Answered by Penny Nom.
$24,000 investment at 5% daily interest 2005-12-21
From Gary:
My friend is a professional day trader on the stock exchange. He claims that he can take a $24,000 investment at 5% daily interest and at the end of 3 years have approximately 34 million dollars. Can this be correct?
Answered by Penny Nom.
The height of a right triangle 2005-12-21
From Sanmantha:
I am trying to solve for the height of a right triangle. The base is .05 mm, and the apex is 0.5 degrees. I vaguely recall from high school that this should be enough information to solve for height, but I can't remember what equation(s) to use.
Answered by Penny Nom.
A rectangle has a length that is 7cm more than twice the width 2005-12-21
From Stephanie:
A rectangle has a length that is 7cm more than twice the width. The area of the rectangle is 60cm squared. Find the dimensions of the rectangle.
Answered by Penny Nom.
Simultaneous Equations 2005-12-21
From Matt:
I have these two equations,
336 = 60a + 10b
and
432 = 84a + 6b
Am I right in saying both a and b are 4.8?

Answered by Penny Nom.
Skeeball 2005-12-21
From Jenny:
Kira and Dajuan decided to have a Skeeball contest. They each played one game to see who would get the highest score. A description of their contest follows:
Answered by Denis Hanson.
A word problem 2005-12-19
From Karen:
A package designer wants to protect an expensive book with a 5 cm wide cardboard rim. The area of the rim is equal to the area of the book. What is the length of the book? (The book is square)
Answered by Penny Nom.
If 7 times a number is decreased by 8 ... 2005-12-17
From Joyce:
If 7 times a number is decreased by 8, the result is the same as when 3 times the number is increased by 6. Find the number.
Answered by Penny Nom.
A max-min problem 2005-12-16
From Julie:
A car travels west at 24 km/h. at the instant it passes a tree, a horse and buggy heading north at 7 km/h is 25 km south of the tree. Calculate the positions of the vessels when there is a minimum distance between them.
Answered by Penny Nom.
Josh and John were both exposed to the flu 2005-12-16
From Dudley:
Josh and John were both exposed to the flu. John has a 25% chance of getting it, while Josh has a 75% chance of getting it. What are the chances that at least one of them has the flu?
Answered by Penny Nom.
Folding a sheet of paper 2005-12-15
From Victoria:
The current problem is to take a normal 8 1/2 x 11 sheet of paper, take a corner and fold it to meet the opposite corner, and (without actually measuring) produce a formula to describe the result fold/crease.
Answered by Penny Nom.
Mrs. Faria lives on an island 2005-12-15
From Julie:
Mrs. Faria lives on an island 1 km from the mainland. She paddles her canoe at 3 km/h and jogs at 5 km/h. the nearest drug store is 3 km along the shore from the point on the shore closest to the island. Where should she land to reach the drug store in minimum time?
Answered by Penny Nom.
5(x-3)/4 =x+1 2005-12-14
From Jennifer:

I have two problems i am stumped on I don't know how to help my daughter.

5(x-3)/4 =x+1
3(x+12)/5 =x +2


Answered by Penny Nom.
Pat invested a total of $3000 dollars 2005-12-14
From Duane:
Pat invested a total of $3000 dollars. part of the money yields 10 percent interest per year and the rest yields 8 percent interest per year if the total yearly interest is $256 how much did pat invest at 10 per cent and how much at 8 percent.
Answered by Penny Nom.
A 24 sided polygon 2005-12-14
From Matt:
I would like to know if there is a name for a 24 sided shape
Answered by Penny Nom.
A loan of $50,000 2005-12-14
From Fre:
A loan of $50,000 taken today is payable within five years.
a. determine the annual payments within to be made to repay the loan if interest is charged at a rate of 10% compounded annually
b. show the amortization schedule

Answered by Penny Nom.
Inclusive definitions 2005-12-14
From Layla:

recently the solvable quandary of 5+5+5=550 came up (the question says that you have to put 1 straight line somewhere in the equation to make it true with out turning the "=" into a "not=" sign).

So two answers were put forward:
545+5=550 (the use of a line converting a + into a 4)
AND
5+5+5(less than or equal to)550

There is currently an argument about the second solution. The disagreement is about whether this sign can be used. One person is arguing that the "less than or equal to" sign defines that the number on the left is in the range 550 and below. The other is saying that since the number (which is clearly defined with no variables) can never equal 550, then the "less than or equal to" sign cannot be used in this case.

Which one is the correct definition?


Answered by Walter Whiteley.
I want to place four sticks equal distance apart into a tennis ball. 2005-12-13
From Bev:
I want to place four sticks equal distance apart into a 3 inch tennis ball. The circumference of the ball is 9 inches. Where should I place the sticks and how many inches apart.
Answered by Chris Fisher.
A regular octagon is inscribed in a circle 2005-12-13
From Carlin:
A regular octagon is inscribed in a circle of radius 15.8 cm. What is the perimeter of the octagon?
Answered by Penny Nom.
LCM 2005-12-12
From Alex:
what is the LCM of 210 and 54 and the LCM of 42 and 126
Answered by Penny Nom.
The temperature in June 2005-12-12
From Farina:

Question: Hi, I was having problem doing this problem.
Question:
Suppose the temperature during June is normally distributed with the mean= 20 degrees C and standard deviation= 3033 deg. A i the event that temperature is between 21.11 deg C. B is the event that the temperature is over 25Ged C. Event C is the temperature is below freezing point. Find the following probabilities:
A) P(A)
b) P(B)
C) P(C)
D) P(C given B)
E) P(neither A nor B)
F) find the expected number of days in June when event B will occur.

I know how to find questions A-D
I am only having problem with question E and F. Can you please help me. Thank You.


Answered by Penny Nom.
2/n +4 = 10- 4/3n 2005-12-11
From Jennifer:

2/n +4 = 10- 4/3n

5(x-3/4) = x+1


Answered by Penny Nom.
Complete the array 2005-12-11
From Ginger:
4 0
2

Make the sum of these number add up to six.

Going across, down, corner to corner. Using the number 4, 2, 0 only.


Answered by Penny Nom.
A point is twice the distance from y = 5 + 2x as it is from y = 5 - 2x 2005-12-09
From Hazel:
A point moves so that its distance from the line y=5+2x is twice its distance from the line y=5-2x. Find the general form of the equation of its locus.
Answered by Penny Nom.
Four tangent circles 2005-12-06
From Ananth:

I have one bigger circle A with radius 15.

Inside this bigger circle i have another circle B with radius 3 which touch this bigger circle. Have another circle C with radius 4 which touches A and B. I would like to draw a biggest circle which touches A,B and C.


Answered by Chris Fisher.
A concrete patio 2005-12-06
From Anne:
If you are making a concrete patio 4 inches thick, what is the area of the largest patio you could make if you ordered 2 cubic yards of concrete?
Answered by Penny Nom.
how do you convert ml to oz? 2005-12-05
From Sara:
how do you convert ml to oz?
Answered by Penny Nom.
A sheet metal cone 2005-12-05
From Laura:
I am an art student and in the process of making a cone out of sheet metal. I am unable to work out the template I need to produce my final cone. The dimensions I have are that the final cone will be 58mm high and will have a diameter of 102mm.
Answered by Penny Nom.
A long distance call 2005-12-05
From A student:
The Small World long distance phone company charges 55cents for the first minute of a long distance call and 23cents for each additional minute.

1. How much would a 10- minute long-distance call cost?

2. If a call costs $4.55. how long did the call last?

3. Write an equation for the total cost, C, of an m-minute long-distance call.

Answered by Penny Nom.
Fill in the blanks 2005-12-05
From Bonnie:
Distribute the numbers 1 through 9 on the lines below so that two equations are true.

_ _ x _ = _ _

_ x _ = _ _

Use each digit only once-

Answered by Penny Nom.
How do you simplify a fraction if one of the numbers is negative? 2005-12-05
From Stephanie:

Question: How do you simplify a fraction if one of the numbers is negative?

Ex. -32/40


Answered by Penny Nom.
Vertical asymptotes 2005-12-05
From Kelly:

X2 - 49
Y= -------------------
X2 + 9x + 20

We are trying to solve this equation to find the vertical asymptote.


Answered by Penny Nom.
Solve for x 2005-12-04
From Lisa:
5(x-4)=3(2-3x)
Answered by Penny Nom.
How is trigonometry applied to everyday life? 2005-12-03
From Yadira:
My question is how is trigonometry applied to everyday lives and functions. Ex: Builders use it but how and what are some examples of the trig-functions or formulas that they use?
Answered by Harley Weston.
Area of a lot 2005-12-03
From Ben:

How do you calculate the area of the following Lot?

I figured the following angles from the deed, which read:

N 86 degrees, 45 minutes E, for 322 feet.

S 10 degrees, 30 minutes W for 113 feet.

N 84 degrees, 30 minutes W for 368 feet.

N 50 degrees, 42 minutes E for 76 feet.

N 40 degrees, 40 minutes E for 15 feet.

There is a discrepancy between two surveyors and I'd like to figure out how to calculate the Area of such a shape.


Answered by Harley Weston.
A sample size estimation 2005-12-03
From Ivonne:
I have to do a research about the behavior of library users. We are going to apply a survey to a population of 1280 students (Management an Economics students) but of course we have to do it to a sample....I need to know the size of my sample.
Answered by Penny Nom.
An integer equation 2005-12-01
From Henry:
3x = 2y + 7

I know the answer: x=9 and y=10, but what are the algebraic steps to solve the equation?

Answered by Penny Nom.
Express t as a function of d 2005-11-30
From Matthew:

I have a problem on my homework I cant seem to figure out. Basically it gives me this Linear Regression Equation:

ln t=1.5 ln d-0.8954

and it wants be to transform the equation expressing t as a function of d. For some reason I keep getting it wrong. Anyone got an idea of how to do this?


Answered by Penny Nom.
Blocks in a mile in Phoenix Arizona 2005-11-30
From Rita:
How many blocks are there in a mile in our city of Phoenix, Az.?
Answered by Penny Nom.
The square root of i 2005-11-30
From Kevin:

If the square root of -1 is i, what is the square root of i?

How can you find the log of a negative number?

What is the log of -1?


Answered by Claude Tardif.
Resizing a polygon 2005-11-26
From Anthony:
I am currently working on a mapping program in which polygons, rectangles, circles, etc... are rotated, moved, and resized. Recently, I used the rotation conversion formula to rotate the items n degrees/radians by using the point and the center point of the shape. Currently, I am working on resizing the shape (length and/or width) while still trying to maintain the integrity of the shape. I basically want to decrease or increase the y value by 1 in order to resize the length and I want to decrease or increase the x value by 1 in order to resize the width. The corresponding x and y values either increase by 1, stay the same, or decrease by 1. Is it possible to use just the point and center of the shape in order to calculate this resizing? What can I use to accomplish this task?
Answered by Chris Fisher.
Common fractions to decimals 2005-11-25
From A student:
how do u turn 8 over23 into a decimal and how do u turn 8 over 29 into a decimal
Answered by Penny Nom.
The sum of the angels in a triangle 2005-11-25
From Rachel:
how do you prove, without knowing any of the measurements or degrees, that the three angles of a triangle equal 180? what are the steps for proving that?
Answered by Penny Nom.
Find the measure of each angle 2005-11-25
From Bev:
in triangle abc, angle a is four times as large as angle b, angle c measures 20 degrees less than angle b. find the measure of each angle.
Answered by Penny Nom.
Adding improper fractions 2005-11-25
From Paula:
I would like a simple step by step explanation on how to add improper fractions.
Answered by Penny Nom.
Four 25-sided dice 2005-11-24
From dan:
Simply put, I am curious to find the likelyhood of each possibility (4-100) if I were to roll 4 25-sided die. It seems to start off simply, as 1, 4, 10, 20...following the inside diagonal of Pascal's triangle. But as soon as i reach the point where the sum is greater than the number of sides, Pascal's no longer seems to work. I would be greatly appreciated if I could be informed of the proper formula to find this answer, or perhaps if my theory of doing it is wrong.
Answered by Andrei Volodin and Shaun Fallat.
Graphing two lines 2005-11-20
From A student:
About y = 5x + 24 and y = -3x - 8
1. Graph both equations on the same set of axes. When you made your graph, how did you choose the range of x and y values?


2.Find the point of intersection for the graphs.


3. Test the point of intersection you found by substituting its coordinates into the equations. Do the coordinates fit the equations exactly?

Answered by Penny Nom.
A standard normal probability 2005-11-19
From Pat:
Find the probability of Z when it lies between -1.10 and -0.36
Answered by Penny Nom.
Gallons and cubic feet 2005-11-19
From Just interested:
I have found that 1 cubic foot will hold 7.48 gallons of gasoline, but I would like to know how to come by this answer.
Answered by Penny Nom.
I = PRT 2005-11-16
From Ryan:
Use the formula to find the value of the variable that is not given:
I=PRT;I=$2880, R=0.08, P=$12,000

Answered by Penny Nom.
Rational expressions 2005-11-15
From Zach:
I can solve easy problems such as (x/2)+3=2+(3x/4). That is easy because the Lowest Common Denominator is 4. But what really gets me stuck is a problem like this one. (6/x-2) = ( 21/(x-2)(x+2) )+ 1.
Answered by Penny Nom.
What shape do I say a football is? 2005-11-14
From Peter:
I am a 43 year old man. I am no longer in school, I have a bachelors degree. I like to tell stories about sports in ways to make people think about the amount of time they devote to the sports. For example, while attending a church in Chicago on the north side, I would pass by Wrigley field. I began to call it the church of the flying orb or sphere. Now if I want to tell a similar story about football (not soccer), what shape do I say a football is?
Answered by Chris Fisher.
Two word problems 2005-11-14
From Jennifer:

1. Every other person on a school's parent advisory committee is surveyed to determine how many people support passage of a school bond to build a new elementary school. Is this a good sample? Why or why not?

2.What is the difference in elevation between the highest point in California,Mount Whitney,which towers 4421 meters above sea level, the lowest point in California,Death Valley, which lies 86 meters below sea level?


Answered by Harley Weston.
A picture window 2005-11-14
From Marcos:
The length of a picture window is 2 feet longer than twice its width. If the diagonal of the window is 13 feet, find the dimensions of the window.
Answered by Penny Nom.
An isosceles triangle 2005-11-14
From Chris:
PX and QY are attitudes of acute triangle PQR, and Z is the midpoint of PQ. Can you write a proof that triangle XYZ is isosceles?
Answered by Chri Fisher.
Add one 1 at the beginning and at the end of a number 2005-11-13
From Samuel:
If we add one 1 at the beginning and at the end of a number, this number is increased by 14789. What is the sum of the digits of the number?
Answered by Penny Nom.
The elevation of the top of the house 2005-11-13
From Chloe:
Karen is standing 23 metres away from the base of a 23 metre high house. Assume that Karen's eyes are 1.5 metres above ground. Find the elevation of the top of the house from Karen's eye line.
Answered by Penny Nom.
Shapes 2005-11-12
From Tonya and Hailey:
MY THIRD GRADERS TEACHER HAS SENT HOME HOMEWORK ASKING FOR THEM TO FIND REAL-WORLD OBJECT IN MANY DIFFERENTS SHAPES A FEW HAVE US STUMPED
Answered by Penny Nom.
The diagonals of a trapezoid 2005-11-11
From Chris:
Is there ever a case where the diagonals of a trapezoid bisect each other?
Answered by Walter Whiteley.
Quadratic Equations & Given Roots 2005-11-11
From Lindsey:

I need to find the quadratic equation in the form:
f(x)=a(x2 - (-b/a)x + (c/a))

The vertex is (1, -7), the roots are (4, ?)

I need to find the other root but I don't know where to begin. My answer key says the other root is (-2). How is this possible?


Answered by Penny Nom.
Solving a pair of equations 2005-11-10
From Steven:
what is answer to
2 x + y = 1
y = -1/3 x - 4

Answered by Penny Nom.
Solve for g 2005-11-09
From Cassandra:
solve each equation or formula for the variable specified

5g+h=g, for g

Answered by Penny Nom.
Notation for the second derivative 2005-11-08
From Mussawar:
my question is d/dx( dy/dx) = d2y/dx2. why it is not equal to d2y/d2x.
Answered by Penny Nom.
A variable rectangle 2005-11-08
From Mussawar:
find the lengths of the sides of a variable rectangle having area 36 cm2 when its perimeter is minimum i do not want solution of this question. i would like to know what is mean by variable rectangle.and what is difference between rectangle and variable rectangle.also what is mean by when its perimeter is minimum.
Answered by Penny Nom.
The height of a tower 2005-11-08
From Vinita:
Observers at point A and B, who Stand on level ground on opposite sides of a tower, measure the angle of elevation to the top of the tower to be 33 degrees and 49 degrees respectively. Another point C is 120 m from point B, Triangle ABC =67 degrees and BAC = 31 degrees. Find the height of the tower to the nearest metre.
Answered by Penny Nom.
Mario is 16 years older than his youngest sister, Mercedes. 2005-11-08
From Jennifer:
Mario is 16 years older than his youngest sister, Mercedes. Together, their ages total 36 years . How old is each person?
Answered by Paenny Nom.
Real Pay 2005-11-07
From Judy:
Is there a formula for computing "real pay"? Example - you are offered a position in your home town for $A, and another position in another town for $B. To work at $B job there would be so many hours in commute time involved (z), and a real daily commute cost (either train, bus, or auto mileage-wear, etc.) (x). How would you calculate the real pay of job $B in relation to job $A?
Answered by Claude Tardif.
The number is increased by the sum of its digits 2005-11-07
From Ernesto:
The number 1 is written on a blackboard. After each second the number on the blackboard is increased by the sum of its digits. is it possible that at some moment the number 123456 will be written on the blackboard?
Answered by Claude Tardif.
Divide a circle into eleven parts 2005-11-07
From Dean:
I would like to know how to divide a circle into eleven parts using only four lines.
Answered by Claude Tardif.
Cubic yards of concrete 2005-11-06
From Cyndi:
I have a carport that is 20' x 31' and a driveway that is 12' x 18' that requires a 3" depth of concrete. I need to now how many cubic yard it will need to complete these two areas.
Answered by Penny.
Triangles with integer sides 2005-11-04
From Tammy:
I am trying to find another pair of integer sided isosceles triangles, not the same as the ones listed below, with equal areas. (5,5,8) (5,5,6)
Answered by Chri Fisher.
Percent or percentage 2005-11-03
From Kenneth:

Which word should be used in the following?

Change a (percent or percentage) to a decimal.

Should the word percent be used only when a number precedes it as in 45 percent?


Answered by Harley Weston and Chris Fisher.
The length of a chord 2005-11-03
From Sue:
How do you determine the length of a chord when given the diameter of the circle (1.6m) and that the angle = 7π/8
Answered by Penny Nom.
How many passengers boarded the plane in Phoenix? 2005-11-03
From Vish:
Passengers boarded a plane in Phoenix. In Dallas 1/2 of the passengers got off and 4/5 of the original number got on. In Memphis 3/4 of the passengers got off and twice the number of remaining passengers got on. In Chicago 2/3 of the passengers got off the plane leaving 39 passengers. How many passengers originally boarded the plane in Phoenix?
Answered by Penny Nom.
The sides of an octagon 2005-11-02
From Royce:
I understand there is a simple calculation to determine the sides of an octagon when you know the distance across the parallel flats. something like .447 . can you help?
Answered by Penny Nom.
A tangent to a parabola 2005-11-02
From A student:
Find the point on the curve y=x2 where the tangent to the curve is parallel to the secant line connecting (-1,1) and (2,4)
Answered by Penny Nom.
Writing an equation 2005-11-01
From Richie:
A man's electric bill is $23 less than his gas bill. The two utilities cost him a total of $109. Find the amount of his gas bill.
Answered by Penny Nom.
Volumes 2005-10-31
From Diane:
My name is Diane and I am a returning student to a vocational technical school.

As a reference point to see what I did/didn't remember from my HS math days, I was sent home with a 75 problem math packet. I was fine until I got to computing:


1. the volume of a cylinder-is it pi r2 h?

2. the volume of a cone- is it pi/3 r2 h?

3. the volume of a sphere- I can't even hazard a guess.

Help- my last classroom was 23 years ago, and I've forgotten far too much!

Answered by Penny Nom.
A two cubic foot box 2005-10-27
From Michael:
I need to make a box that measures two cubic feet. It will be used to items in to determine if an inmate has to much property. They can only possess 2 cubic feet of property. How can I convert that into measurements in inches for feet so I can make a box with a lid that is 2 cubic foot. I sure hope you can help me
Answered by Penny Nom.
Velocity and acceleration 2005-10-27
From Candace:
When taking the integral of the position function, you get the velocity function, and the same for velocity to acceleration. So when you do each of these, you get a function. But when you integrate on a graph, you get an area under a curve. The area is un units squared- where do the units go when you make it an equation? How can a function be an area?
Answered by Harley Weston.
A slump cone 2005-10-27
From Wendy:
we are trying to make a slump cone (used to measure the slump in concrete). It has dimensions of 8" diameter on the bottom, 4" diameter hole on the top and a height of 12". Please help, it is getting frustrating.
Answered by Penny Nom.
How many cubic yards of dirt will be needed? 2005-10-27
From Mike:
We are filling in a hole left by the removal of an aboveground pool that was dug into the ground. The diameter is 25' across and the depth is 5'. I wondered how many cubic yards of dirt will be needed to fill this hole.
Answered by Penny Nom.
What number am I? 2005-10-27
From Samantha:
First 8 is added to me, then I am multiplied by 6. Then 40 is subtracted from me. Finally, I am divided by 10. The result is 11. What number am I?
Answered by Penny Nom.
Cubic feet and imperial gallons 2005-10-27
From Dave:
What is the volume in cubic feet which is occupied by 1080 Canadian gallons of water?
Answered by Harley Weston.
(x/1+x)^x 2005-10-23
From A student:
LIMIT x-.->0 (x/1+x)x how to solve it.
Answered by Penny Nom.
Least common multiple 2005-10-22
From Dayna:
I'm stumped! I can't seem to come up with a least common multiple of 22 and 25. Is there one?
Answered by Penny Nom.
A triangular pyramid 2005-10-22
From Stacee:
When stacking oranges to form a triangular pyramid, each orange touching 3 oranges below it. In order to have a pyramid with 6 layers, how many oranges should be on the bottom layer?
Answered by Penny Nom.
A conical hat 2005-10-22
From Manish:
I need to make a conical hat for my daughter's upcoming fancy dress, the circumference of the base(hollow) is 50 cms,the height of the cone is 30 cms,what should be the dimensions of the paper which will make a cone of the beforementioned dimensions?
Answered by Penny Nom.
Improper fractions 2005-10-21
From Joe:
Is a whole number an improper fraction. For example, is 4 an improper fraction? Is 4 an improper fraction? Is 4 the simplest form for 20/5? Are 4 and 20/5 equivalent fractions?
Answered by Penny Nom.
sin(kx) = x 2005-10-20
From David:
What is an integer value for k so that sin(kx) = x has exactly 2005 solutions? How does one arrive at the answer?
Answered by Harley Weston.
Coefficient of variation 2005-10-19
From Jan:
I am currently teaching the coefficient of variation and am wondering if there are some guidelines as to the interpretation of this statistic. I understand that it measures the variation in a variable relative to the mean - but what is the cut off for "too much" variation expressed in this way???
Answered by Andrei Volodin and Penny Nom.
Can we take the derivative of independent variable 2005-10-18
From Mussawar:
why we take derivative of dependent variable with respect to independent variable .can we take the derivative of independent with respect to dependent.if not why.
Answered by Walter Whiteley.
A line that intersects a circle 2005-10-18
From Bruce:
I would like to solve the following problem illustrated below. How do you calculate the length of a line that intersects a circle.
Answered by Penny Nom.
What is the angle between 2 non-collinear parallel lines? 2005-10-17
From Ben:
What is the angle (if any) between 2 non-collinear parallel lines?
Answered by Chris Fisher and Walter Whiteley.
A boat weighs 1500 lb more than its motor ... 2005-10-17
From Dimitri:
A boat weighs 1500 lb more than its motor and 1900 lb more than its trailer. Together the boat and motor weigh five times as much as the trailer. How much does the boat weigh?
Answered by Penny Nom.
Divisibility by each of the first ten counting numbers 2005-10-17
From Simon:
determine smallest positive integer that is divisible by each of the first ten counting numbers
Answered by Penny Nom.
A random sample 2005-10-16
From Stu:
If I have a set of data points (14 to be exact)of unknown pedigree from a large population, what tests can I apply to see if they constitute a random sample from the large population?
Answered by Andrei Volodin and Penny Nom.
2x - 5x + 6.3 = -14.4 2005-10-16
From Leslie:
How do we solve 2x - 5x + 6.3 = -14.4
Answered by Penny Nom.
How many gallons will it hold? 2005-10-16
From Scott:
I am building a pond and need to convert the cubic dimensions into gallons in order to know what size fountain pump to purchase.

My pond is 1 foot deep x eight feet long x three feet wide.

How many gallons will it hold?

Answered by Penny Nom.
What is the sum of the measures of the angles of a decagon? 2005-10-15
From Dianna:
What is the sum of the measures of the angles of a decagon?
Answered by Penny Nom.
How many pixels are there to an inch? 2005-10-15
From Fred:
I want to know how many pixels there are to an inch.
Answered by Penny.
The area of an irregular semi-circle 2005-10-14
From Bob:
Is there a way to compute the area of an irregular semi-circle i.e. one in which the arc length is not determined by the diameter; and is therefore not technically any part circular - yet still possessing an arched side?
Answered by Penny Nom.
The coefficient of thermal expansion for steel 2005-10-14
From Jim:

Is the following statement true?

The coefficient of thermal expansion for steel is 0.00000645in/in/deg. Doesn't sound like much but when you run out the numbers it comes to .405504 ft/mile/deg. Still doesn't sound like much, only about 5". Then multiply by 40 degrees and you get a piece of rail that has grown by 16.22 feet in that one mile. It's not at all unusual for the rail temp to go from say, 40 deg to 80 deg on a spring or fall day. Remember that on a sunny day, the rail temp can be significantly higher than the air temp as well."

I ran the math and came up with an answer closer to 16 inches, instead of 16 feet. Which is closer to being correct?


Answered by Penny Nom.
Simultaneous equations 2005-10-13
From Daniel:
5x + 3y = 22 4x - 7y = -20
Answered by Penny Nom.
The bathtub curve 2005-10-13
From David:

My father asked me to submit a question about the so-called 'bathtub
curve'. If you cut a bathtub in half lengthwise down it's middle, the
edge of the tub would describe the 'bathtub curve' which can be used
to demonstrate typical failure rates of products. This curve is
characterised by high initial (infant mortality) failure rates at
it's beginning, which drop quickly to a very low level. Failures then
increase gradually to the "end of life" stage where the failure rate
takes off dramatically again.

If anyone in the math department knows about the so-called 'bathtub
curve' my father would really appreciate the equation.


Answered by Chris Fisher and Edward Doolittle.
Means of sample means 2005-10-12
From Fareeha:
A population consists of six numbers 1,2,3,4,5,6
a) How many different samples of two numbers can be selected with replacement? and list them.
b) Calculate the mean and standard deviation of the population
b2) Calculate the mean and standard deviation of the mean of the samples.
c) Find the standard error of your estimated mean in question b2 and compare with the estimated standard deviation in b2.

Answered by Penny Nom.
What is the number? 2005-10-10
From Jess:
One eighth of the square root of 7 less than a number is 2. What is the number?
Answered by Penny Nom.
Whole numbers 2005-10-10
From Kirk:
Can the sum of two or more whole numbers be less than any of those numbers?
Answered by Penny Nom.
Inches and millimeters 2005-10-08
From Ron:
You know they say 14mm = 9/16" or something of that nature, i need to know exactly how this is done.
Answered by Chris Fisher.
Constructing a quadrilateral 2005-10-07
From Jayasri:

Can we construct a unique quadrilateral with the following information:

The lenghts of all the sides of a quadrilateral are known.

The angle between one of the sides and the X-axis is known.


Answered by Chris Fisher.
Coefficients, constants and like terms 2005-10-05
From Elizabeth:
In the equation -8y+6ab+7-3ab what are the coefficients; the like terms and constants?
Answered by Penny.
A combination lock 2005-10-03
From A bicycle owner:
What are the number of permutations (Combinations?) of a combination lock of a cylinder with four separate movable rings. Each ring has the numbers from one to six (six "permutations" per ring).

This is a bicycle lock.

I was told that it is 62 (6x6x6x6) = 1296.

Is this correct?.


Answered by Penny Nom.
3z-5+2z=25-5z 2005-10-02
From Davis:
3z-5+2z=25-5z
Answered by Penny Nom.
Prove that a rhombus' diagonals are perpendicular 2005-10-02
From Tania:
How do you prove that a rhombus' diagonals are perpendicular using the 2 column proof method?
Answered by Walter Whiteley.
Inches and millimeters 2005-09-30
From A student:
how many inches are there in mm? For example 30x40 mm = how many inches.
Answered by Penny Nom.
6 trillion x 50 million 2005-09-30
From Roly:
There is a realm of the richest galaxies in the in the night sky. They are 50 million light years away. That is 6 trillion x 50 million???. What math equivalent is that?
Answered by Penny.
Two groups that have equal sums 2005-09-30
From Anita:
using the numbers 1, 2, 3, 4, 5, 6, 7,and 8. how do we divide them into two groups so that they have equal sums?
Answered by Penny.
Calculer la clé 2005-09-29
From Un eleve:
Chaque individu a un numéro INSEE de 13 chiffres auquels est adjointe une clé de deux chiffres. voici comment est calculée cette clé :
Answered by Claude Tardif.
Open dots and closed dots 2005-09-29
From Cynthia:
When graphing the solutions of an inequality, what is the difference between an open dot and a closed dot?
Answered by Penny.
The length of a circle's chord 2005-09-28
From A homebuilder:
find the length of a circle's chord with a known arc length and radius/diameter lengths.
Answered by Penny Nom.
A multiple-choice and true-false exam 2005-09-28
From Kiat:

Harry, Neville, Ron, Hermione and Malfoy each answered five questions on O.W.L. exam consisting of two multiple-choice questions (A, B or C) and three True-False questions. They answered the questions as follows:

Student Q1 Q2 Q3 Q4 Q5
Harry A A T T T
Neville B B T F T
Ron A B T T F
Hermione B C T T F
Malfoy C A F T T

If no two students got the same number of correct answers, who got the most correct answers? And what is the correct answer for each question?


Answered by Denis Hanson.
2x/x^2-9 - 6/x^2+x-12 2005-09-27
From Emily Ann:
2x/x2-9 - 6/x2+x-12
Answered by Penny Nom.
6x+7=8x-13 2005-09-27
From A student:
show me how to do 6x+7=8x-13
Answered by Penny Nom.
14p - 8 = 22 + 20p 2005-09-27
From Tiffany:
14p - 8 = 22 + 20p
Answered by Penny Nom.
A cone with vertex (1,1,2) 2005-09-26
From Brandon:
Find the equation of a double cone with vertex (1,1,2) and which intersects the xy plane in a circle of radius 4.
Answered by Penny Nom.
U'(X) - U(X) = 0; U(0) = 2 2005-09-23
From David:
Out of interest could you please answer the following questions?

U'(X) - U(X) = 0; U(0) = 2

and

U''(X) - U'(X) = 0; U'(0) = U(0) = 2


Answered by Harley Weston.
The length of the side of an octagon 2005-09-22
From Billy:
How can you find the length of the side of an octagon when all that you know are the long sides of one of the eight perfect isc. triangles inside the octagon that share the same center point?
Answered by Penny Nom.
Isn't 1/3 written as a decimal .33? 2005-09-21
From Lisa:
Isn't 1/3 written as a decimal .33? My son informs me I'm wrong, but doesn't know the answer
Answered by Chris Fisher and Penny Nom.
Constructing figures 2005-09-20
From Kim:
I would like to know how to draw different shapes:
Regular Octagon with sides of length, 1 unit
Equilateral Triangles, with sides of length 1 unit
Regular Hexagons, with sides of length 1 unit
Isoseles Triangles, with hypotenuse of length 1 unit

Answered by Penny Nom.
Cubic yards is in a box trailer 2005-09-20
From barbara:
i need to know how many cubic yards is in a box trailer that is 96" wide 13' 6" high and 40" long
Answered by Penny Nom.
Use the numbers and symbols only once to come up with the answer 2005-09-19
From Kaitlyn:

Question: I have math problem that no one in my class was able to solve but the teacher said it does have an answer. We need to use the numbers and symbols only once to come up with the answer:

36 18 12 6 + - ( )

______________________________ = 41


Answered by Claude Tardif.
A 3-dimensional pie shape 2005-09-17
From Bill:
Your site appeared in my search for the name of a 3-dimensional pie shape. 2-d would be a sector of a circle. As it it curved, I don't believe it is in a the polyhedra family. Can you help me find the mathematical term for it?
Answered by Chris Fisher.
A point is moving on the graph of x^3 + y^2 = 1 in such a way that 2005-09-17
From Gina:
A point is moving on the graph of x3 + y2 = 1 in such a way that its y coordinate is always increasing at a rate of 2 units per second. At which point(s) is the x coordinate increasing at a rate of 1 unit per second.
Answered by Penny Nom.
The trinity concept says three is one. 2005-09-17
From Kevin:

The trinity concept says three is one. Mathematically, I will attempt to proof this.

Let G be God and T trinity. We start with
G = T
2G = 2T Double both sides
G + 2G = G + 2T Add G on both sides
3G = G + 2T Simplify
3G - 3T = G + 2T - 3T Subtract 3T on both sides
3(G - T) = G - T Factor 3 on the left, simplify on the right
3 = 1 Divide (G - T) on both sides.

But how can three equals one? Can it?


Answered by Leeanne Boehm.
What is my number? 2005-09-16
From Bob:
I have 6 digits. 100,000th digit is one less than my ones digit, eight greater than my 1,000's digit, twice my tens digit and four times my 100s digit. My 10,000s digit is 0 What is my number?
Answered by Penny Nom.
Elliptic trigonometry 2005-09-15
From Krystal:
I'm currently searching for a science project topic and i have the idea of deriving elliptic trigonometry analogous to circular trigonometry. My questions are: Is this project "possible" to do?
Answered by Chris Fisher and Harley Weston.
Converting from grams to milliliters 2005-09-15
From A student:
What if you don't know the liquid that you are converting from grams to milliliters?
Answered by Penny Nom.
How do you solve 3x^2 + x - 2 = 0 2005-09-15
From Lizzy:
How do you solve 3x2 + x - 2 = 0
Answered by Penny Nom.
A piecewise function 2005-09-15
From Duncan:
A child is assigned to your care and she has a headache. The parent has authorized the administration of Children's Tylenol. Read a box of Children's Tylenol (or any other similar drug) and find the directions for administration. Note the child's weight and the corresponding dosage. (Be sure the drug you choose has a minimum of 4 weight intervals.)
Answered by Penny Nom.
What is 2,500 X 6 trillion miles? 2005-09-15
From Roly:
ANDROMEDA Galaxy is 2,500 light years away. A light year is 6 trillion miles. What is 2,500 X 6 trillion miles in a mathematical number?
Answered by Penny Nom.
A rational arithmetic expression 2005-09-14
From Vijay:
How to simplify following Grade 9 math problem:

(4/-9) X (-21/-32) X (-3/14)

Answered by Penny Nom.
Who is taller: John or Mary? 2005-09-14
From Ulises:
All the students in a school are arranged in a rectangular array. After that, the tallest student in each row was chosen, and then among these John Smith happened to be the shortest.Then, in each column, the shortest student was chosen, and Mary Brown was the tallest of these. Who is taller: John or Mary?
Answered by Penny Nom and Claude Tardif.
How do you differentiate y=(x)^(x^x)? 2005-09-14
From Calebius:
How do you differentiate y=(x)(xx)?
Answered by Penny Nom.
Whats the value of the digit 7 in 756,809? 2005-09-14
From Anastasia:
Whats the value of the digit 7 in 756,809?
Answered by Penny Nom.
Equivalent fractions 2005-09-13
From Cindy:

I am a parent and aunt of a 5th grader.

My nephew has asked me what the equivalent decimal is of 0.4. Does he change it to a fraction or just add a zero either before or after?


Answered by Chris Fisher and Harley Weston.
Volume of a tank 2005-09-11
From Jim:
We own an Rv and need to actually know the volume of our fresh water tank. I think I'm getting the run around from the manufacturer. I'm unable to locate my chemistry books from from collage. I know there is a formula for the volume of a rectangle some place.
Answered by Penny Nom.
A 30-60-90 triangle 2005-09-11
From Gary:
I have the length of only 1 side of triangle with angles of 30-60-90 degrees. How can I find the length of the other 2 sides?
Answered by Penny Nom.
One square mile 2005-09-11
From Rudy:
I know that there are 27,878,400 sq ft in a square mile and that there are 640 acres in a sq mile. But how do I figure out the measurement of each acre. If I could make every acre a square what would it's measurement be?
Answered by Penny Nom.
Cows, chickens and pigs 2005-09-10
From Markequetta:
I was asked a question by a friend who refuses to give the right answer to a question that I answered wrong. So if you can help me that would be great! The level of my question is secondary I believe.

Question:

Cows cost: $10
Chickens cost: $0.50
Pigs cost: $3

You have to buy 100 animals and spend $100, so how many cows, chickens, and pigs do you have on your farm?

Answered by Claude Tardif.
Cubic yards of topsoil 2005-09-08
From Ken:
How many cubic yards of topsoil would there be on 100 acres of land if you averaged a 2' depth of material.
Answered by Penny Nom.
The length of a chord 2005-09-08
From A student:
how do you find the length of a chord given the angle and radius of the circle
Answered by Penny Nom.
n^2+n-1 has no divisors ending with 3 or 7 2005-09-08
From Arne:
at least it seems like for any integers n and k,
10k+3 and 10k+7 do not divide n²+n-1
I tested this for every n from 0 to 3200 (which means same for the numbers from -3201 to -1)
could this be true, or is it just coincidence, or am I just totally wrong?

Answered by Richard McIntosh.
Given only the length of an arc & the length of its chord find the radius? 2005-09-07
From Robert:
Given only the length of an arc (72) & the length of its cord (71), how to find the radius?
Answered by Penny Nom.
x = (y^2 - b^2)/2b 2005-09-04
From Bob:
while looking at x=(y2-b2)/2b on a table I was preparing for a graph, I saw that there is a point where x and y get closer to each other's value. By trial and error I tried to find the point where x=y. I saw a number which looked familiar and I realized it was the square root of 2. and so, if y=b(1+square root of 2) then x=b(1+square root of 2) also, or as x=y.
My question is: How can this result x=y when y=b(1+square root of 2) be algebraically derived from the equation x=(y2-b2)/2b without pre knowing it is so?

Answered by Penny Nom.
Seven consecutive reds in roulette 2005-09-04
From Joe:
I know that the odds are a little less than 50% for either black or red comming up on a spin of the roullette wheel. My question is what are the odds of red or black comming up 7 times in a row?
Answered by Penny Nom.
A typical farm 2005-09-01
From Pagedi:
A typical farm is about 700 acres. How many square miles is this? Please show me the work so, that I will know I to perform this problem myself.
Answered by Penny Nom.
cos x * cos 2x * cos 4x * cos 8x 2005-08-29
From Leandro:

A = cos x * cos 2x * cos 4x * cos 8x

What's the value of log A at base 2?


Answered by Chris Fisher and Penny Nom.
The area of a lot 2005-08-29
From Richard:
My wife and I are interested in buying property in Idaho but the owner can't give us a square footage of the lot. The dimensions are as follows:

121.0 on the left side
157.0 on the right side
135.0 on the bottom
162.0 on the top

The bottom right corner of the lot is a true right angle, the rest are not.

Answered by Penny Nom.
The length of a lot 2005-08-28
From Billy:
If I have 3 acres and the front is 300 ft across how many feet would I go down the sides to equal 3 acres?
Answered by Penny Nom.
1_2_3_4_5_6_7_8_9 = 1: Fill in the blanks 2005-08-26
From James:
Try to replace the blanks below with + or - to make the statement correct
1_2_3_4_5_6_7_8_9 = 1

Answered by Penny Nom.
Graphing a linear inequality 2005-08-26
From Gina:
When graphing a linear inequality, how do you know if the inequality represents the area above the line?
Answered by Penny Nom.
y = log(x) + x. Solve for x. 2005-08-26
From Alain:

I have the following equation:

y = ln(x) + x

How do I solve for x?


Answered by Penny Nom.
The area of a triangle 2005-08-26
From Martha:
I have a triangle that with a base of 60' and the two sides of 37'. I know the formula for area is A=1/2 (b*h) but how do I find the height??
Answered by Penny Nom.
Labour efficiency 2005-08-23
From Rob:

The problem, on the surface, seems very simple and yet has created some controversy among a group of accountants. The problem itself has to do with labour efficiency rates and only involves two variables; standard working hours, and actual working hours. The difficulty lies in deriving an efficiency % from these two numbers.

Standard working hours or the targeted number of labour hours required to produce one widget, which I will represent as "s". Actual working hour or the actual number of labour hours require to produce one widget, which I will represent as "a". Labour efficiency I will represent with "E". The prevailing calculation with which I have a problem with is this:

s/a=E or if s=3000, and a=4000 then 3000/4000=75%

What bothers me about the calculation is that the standard hours get represented as a percentage of the actual hours and in my opinion changes the focus of the calculation from standard or target, where it should be, to the actual hours. I cannot define why, but this just seems inherently wrong to me.
The calculation that I use:

(1+((s-a)/s))=E or if s=3000, and a=4000 then (1+((3000-4000)/3000))=66.67%

My calculation is like a %change from standard calculation. However, there is something that also concerns me about my calculation.

If you substitute 100 for a and 50 for s, then you come to a quandary, because if you plug those numbers into the second equation the result is of course zero % efficient which doesn't sit right with me either. If you plug them into the first calculation you get 50% efficiency which doesn't really seem to work either, because you require 100% more hours to do the same work in this case. ???

Is the first calculation correct? Am I missing something altogether? Are both calculations off base?


Answered by Harley Weston.
3A/4 +1=(2A-1)/3 2005-08-21
From Rachael:

I'm doing a practice test question and the answer I keep turning out is different from the answer on the answer sheet. I think I might be forgetting a step. The problem is:

3A/4 +1=(2A-1)/3

The answer is -16. If anyone could explain the steps to me to getting the answer I would be grateful. Whenever I do it I get something completely different.


Answered by Penny Nom.
The radius of an arc 2005-08-19
From Jared:
I need to be able to calculate the radius of an arc on an existing structure (Supports for Fifth Ave Dimensional Text). I have the overall length of this structure, but, the complication I believe lies in the fact that it is a curve mounted onto a flat section of wall on either side of a very mild curve, therefore, measuring the middle of the wall to the top of the arc does me little good...The reason I need this is to be able to replicate the above curve on a similar structure to be mounted below it...Also, because the client wants the structure mounted on the inside of the opening below the existing, the beginning and end point of the new structure would be lower than the existing, so, I do not believe that it is possible to exactly replicate the radius of the above curve?
Answered by Penny.
Which number is greater? 2005-08-18
From Dante:
Which number is greater: 888....88 X 333...33 or 444.......44 x 666........67 (each of the numbers has 1989 digits)?
Answered by Claude Tardif.
The volume of the earth in gallons 2005-08-18
From Daniel:
I would like to know how to find the volume of the Earth in gallons.
Answered by Penny Nom.
Sample variance 2005-08-17
From Rosalie:
To give an unbiased estimate of the population variance, the denominator of the sample variance should be (n-1) instead of n. I tried to convince myself by comparing the population variance and sample variance (with denominator n and n-1):
Answered by Penny Nom.
My fantasy football league 2005-08-16
From Jim:
I have to make a schedule for my fantasy football league. There are 10 teams in the league and we play a 13 week season. The challenge is we want to play each team only once during the first 10 weeks. This way no team will play another team twice without having played each team at least once. Is this possible?
Answered by Penny Nom.
The diagonals of a hexagon 2005-08-15
From Brittany:
are all of diagonals in a hexagon has same length
Answered by Penny Nom.
Jim is travelling from Toronto to Ottawa 2005-08-15
From Eleanor:
Hi! I am looking for an easy way to solve this type of question: Jim is travelling from Toronto to Ottawa, a distance of 435 miles. He travels part of the way at a speed of 100 km/h and because of an accident, he uses a secondary road and travels at 70 km/h. If the trip took him 6.5 hours, how far did he travel on each of the two roads.
Answered by Penny Nom.
Is 360 Really the correct value? 2005-08-15
From Jack:
Considering the circumference of a "Perfect Circle" with a Diameter of 1 meter would be something like 3.14 meters, why do we use the number 360 to represent the number of degrees within that circumference?

Would it not make more sense to express the degrees in reference to the relationship to the diameter as related to pi?

That is, let's just say our "Perfect Circle" has a circumference of 3.14 meters, therefore, what we now consider as due east would change from 90 Degrees to 78.5 Degrees.

Answered by Penny Nom.
Framing an arched wall 2005-08-12
From Mike:
I'm framing a building wall with a curved (arcing) top section. The radius of the section is 74'6" with a height above finish floor of 16'0". The horizontal run of the arced section is 23' 1 1/2" with a low height above finish floor of 12'4". If I start with a 16' stud at the high end how long are the subsequent studs if they are on 16" centers? Short of laying this out on a tennis court how can I work out the lengths of the studs?
Answered by Penny Nom.
A triangular lot 2005-08-10
From Elizabeth:
If a piece of property was 100 ft on the west, 133 ft on the east and 100 on the north, how many acres is this or what fraction of an acre is this property?
Answered by Penny Nom.
The Maclaurin series generated by f(x)=x^ cosx + 1 2005-08-10
From Latto:
f(x)=x3·cosx + 1. but when I take the derivatives, I couldn't see a pattern. Can you help?
Answered by Penny Nom.
The volume of soil needed to fill a planter 2005-08-10
From Phil:
I need to calculate the volume of soil needed to fill a planter box that measures - 880mm W x 6080mm L x 900mm D.
Answered by Penny Nom.
Cubic feet 2005-08-10
From Selene:
This is my math question I need to convert these inches to cubic feet 34"W x 34"H x 34"L
Answered by Penny Nom.
At what rate is the circumference of the circle increasing? 2005-08-08
From John:

A mathematics professor is knitting a sweater. The main part of the sweater is knit in a large spiral, ending up with a diameter of 30 inches. She knits at a constant rate of 6/7 square inches per minute.

1. At what rate is the circumference of the circle increasing when the diameter is 2 inches?

2. How long will it take her to finish this piece of the sweater?


Answered by Penny Nom.
A normal distribution problem 2005-08-08
From Brad:
The life of a toy is normally distributed. Suppose 92.51% of the items lives exceeding 2,160 hours and 3.92% have lives exceeding 17,040 hours. Find the mean and the standard deviation.
Answered by Penny Nom.
Area of a lot 2005-08-07
From John:
We hope this copy of our survey map will help you to determine the acreage of our land.
Answered by Harley Weston.
Converting rational numbers 2005-08-07
From Joe:

I am helping my son with his converting rational
numbers in the form of A/B where A and B are integers and
B not equal to zero

I think I am doing this right but I am not sure so
below are the problems and our answers are beside
them, please let me know if these answerers are correct


Answered by Penny Nom.
A golf outing for 16 golfers 2005-08-05
From Bob:
I have a golf outing fast approaching and I’m having a problem matching the golfers so that everyone plays a round of golf with every other golfer in the group.

My problem is this: I have 16 golfers playing 5 rounds of golf. I would like a different foursome for every round of golf, that is each round (5 total) an individual is playing with 3 other players he hasn’t played with as of yet. Can you crunch the numbers and get back to me?

The way I see it, we have 20 combinations (5 rounds x 4 golfers) with numbers 1 thru 16.

Answered by Chris Fisher.
What is the radius of this planet 2005-08-05
From Kelly:
Assuming that a North-South line has been established, you set up two camps that are 67 miles apart. You now set up poles at each camp perpendicular to the ground. On a certain day at "noon" the pole at the South camp casts no shadow, while at the North camp a shadow is cast. The shadow makes an angle of 89 degrees with the horizontal. What is the radius, diameter, and circumference of this planet?
Answered by Chris Fisher.
Cubic yards of sand 2005-08-04
From Beth:
I am trying to figure out how much sand I will need for ground cover under my children's playset. The area to cover is 30' x 30' and I want 4 inches of sand.
Answered by Penny Nom.
A concrete slab 2005-07-29
From Armand:
I am planning to pour a concrete slab over an existing slab in my garage
which is very uneven, with a depth variation from 2 1/2 inches to 5 1/2
inches from level. I'm afraid to over estimate concrete since it will be
wasted but worse, I'm afraid to underestimate. I've broken the slab down
into four sections with the following being the four dimensions, can you
tell me how much concrete I will need please, I would very much
appreciate your help.
1- 6ft/8in x 20ft x 4 1/2 in. thick
2- 16ft/10in. x 20ft x 3 1/2 in. thick
3- 6ft/6in. x 16in. x 5 1/2 in. thick
4- 6ft/6in. x 8ft. x 4in. thick

Answered by Penny Nom.
Speed on a bicycle 2005-07-27
From Deborah:
We are trying to figure out how many miles per hour we rode in a bike race. If the distance was l5 miles and the time it took was was 55 minutes, what is the speed?
Answered by Penny Nom.
Cubic feet in a mobile home 2005-07-22
From Jeanne:
my mobile home is 24 x 56. I'm trying to find out how many cubic feet this is. I'm going to set of some bug bombs and it says 1 can will cover 4000 cubic feet. I don't know how to figure my cubic feet. the ceilings are 8 ft high everywhere but the living and dining room. They are vaulted and approximately 10 feet at the peak. I just need an estimate.
Answered by Penny Nom.
A set of numbers whose sum is 1 2005-07-21
From Santos:
Does there exist a set of numbers whose sum is 1, and the sum of whose squares is less than 0,01?
Answered by Paul Betts.
Solve for x 2005-07-20
From Ed:
(x-1)/3 - 1 = (x + 2)/2
Answered by Penny.
How many tiles do I need? 2005-07-20
From Jeannette:
I am putting up backsplash tile in my kitchen and I need to know how many tiles I would need to use for about 45 square feet using 4x4 tiles.
Answered by Penny Nom.
Three prime numbers p,q and r, all greater than 3, form an arithmetic progression: 2005-07-18
From Ladis:
Three prime numbers p,q and r, all greater than 3, form an arithmetic progression: p=p, q=p+d and r= p+2d. Prove that d is divisible by 6.
Answered by Chris Fisher.
The equation of an ellipse 2005-07-17
From Allan:
I working on a problem that asks me to give the equation of an ellipse when only the location of the directrix and the length of the latus rectum are given. No other points on the ellipse are given. Again, the only "givens" are:

Length of latus rectum = 12
Location of directrix is x = 16

If I could determine the eccentricity, I could proceed from there by taking the ratio of the distance from a focus to the latus rectum point to the distance of the point from the directrix, but I lack the x coordinate of c. I've searched the text, and feel I've "missed something" somewhere! I note that the latus rectum segment is unique in one respect in that it is parallel to the directrix, where any other line segment on the ellipse to the focus would not be. Please indicate where I'm going wrong.

Answered by Chris Fisher.
The area of a lot 2005-07-17
From John:
A plot of land has the following dimensions: 391 ft. x 757 ft. x 208 ft. x 788 ft. Q: How many acres is this?
Answered by Harley Weston.
A lighthouse is located on a small island,... 2005-07-14
From Brittnee:
A lighthouse is located on a small island, 3 km away from the nearest point P on a straight shoreline, and its light makes four revolutions per minute. How fast is the beam of light moving along the shoreline when it is 1 km from P?
Answered by Penny Nom.
Cubic yards of topsoil 2005-07-12
From Maggie:
How much topsoil in cubic yards i need to order to cover 7500sq. ft. 3 inches deep?
Answered by Penny Nom.
The value of n*tan*(180/n) tends to pi 2005-07-11
From Daniel:
I am 14 and i have been given a piece of maths coursework whereby a farmer has to fence off a piece of land as large as possible using 1000m of fence. I already know that the formula for working out the area of any shape of a 1000m perimeter = 5002/ n*tan*(180/n), however, after some research I have found out that as the number of sides (n), tends to infinity, the n*tan*(180/n) tends to pi. Why is this?
Answered by Penny Nom.
A melting ice sphere 2005-07-09
From A student:
A 2 meter diameter sphere of ice melts at 1.5 millimeters per minute. How many minutes will it take to fill up a hexagonal container measuring 1.5 meters on a side, and 0.8 meters deep?
Answered by Penny Nom.
The volume of a pool 2005-07-09
From Douglas:
I have a pond that I need to treat with an anti algae solution which needs to applied based on the volume of water in the pond (in gallons).

The rough diameter of the surface of the pond = 25 ft
The depth of the pond = 8 ft
The pond shape is conical

How many gallons of water are in my pond?

Answered by Penny Nom.
The substitution method 2005-06-28
From Addie:
how do i know where to began the substitution method? how do i solve y = x + 1 2x - y = 1
Answered by Penny Nom.
Two tangents to a circle 2005-06-18
From Tej:
The tangents drawn from points M and N of a circle having centre O intersect each other at point P. If angle MPN=60 degrees, NM=10, then find the radius of the circle and Area of quadrilateral OMPN.
Answered by Penny Nom.
The sum of the digits of 2^100 2005-06-11
From Richard:
The sum of the digits was calculated for the number 2100, then the sum of the digits was calculated for the resulting number and so on, until a single digit is left.
Answered by Penny Nom.
The size of a lot 2005-06-09
From Bob:
Is it possible to determine the square footage and acreage of a residential lot that's straight on 3 sides, and triangular on the 4th side?

As shown in the crude sketch below, the two legs of the triangle at the back end of the lot are 61' and 62'.

Answered by Penny.
Digging a hole for a pool 2005-06-08
From Debra:
I need to have dirt hauled away from my built in pool and the haulers are asking me how many yards of dirt there is. How would I figure that out? It is a 24 ft round pool and 3 inches of soil depth was dug out.
Answered by Penny.
An octagonal deck 2005-06-07
From Scott:
I want to build a octagonal deck. The wood I am using are cut in 8 foot lenghts. What I want to know is if the sides of the octagonal are 8 feet, what is the diameter. Also what are the angles of each side?
Answered by Penny Nom.
How many digits? 2005-06-07
From Floren:
The numbers 21989 and 51989 are written one after another. How many digits in all are there?
Answered by Claude Tardif.
The length of a whale 2005-06-04
From Judy:
A whale's head is 72 inches long... its tail is as long as its head plus half the length of its body, and its body is half the entire length. Who long is the whale?
Answered by Penny Nom.
The volume of a sphere. Why 4/3? 2005-05-30
From Lauren:
You know when you find the volume of sphere? I know the formula is V= 4/3 pi r3 but why do they use 4/3?
Answered by Penny Nom.
Square feet and cubic feet 2005-05-30
From Kim:
Is there a relationship between square feet and cubic feet?
Answered by Penny Nom.
5+5+5=550 2005-05-30
From Bill:

5+5+5=550

This is a math equation that is not true. How can it be made to be true by using only one line?
Answered by Leeanne Boehm.

Which is larger, 31^11 or 17^14? 2005-05-29
From Linda:
Which number is greater 3111 or 1714 (without using calculator).
Answered by Chris Fisher and Paul Betts.
The volume of a hopper 2005-05-28
From Brian:
I would like to know the volume of this rectangular hopper. can you help
Answered by Penny Nom.
Cost price 2005-05-25
From Sunitha:
What is (how to calculate) the cost price of a product which is marked-up by 15% and retailed at $25?
Answered by Penny Nom.
A parabolic mirror 2005-05-24
From Nathan:
i am trying to find the equation for a mirror for a laser experiment. the mirror is parabolic but my question is how do you find the equation when you know only the focus and the diameter the mirror diameter is 520 mm and the focus is at 1024 mm. would you just use the measurements in the equation instead of "nice numbers" or what.
Answered by Penny Nom.
Logarithmic differentiation 2005-05-23
From Richard:
I need to convince myself that I understand the process of differentiating y=xx.
The specific question is that if I have to take the logarithm of both sides of the equation how can differentiate the following?
y= {(x+2)(x+2)}/{(x+1)(x+1)} - {(x+1)(x+1)}/(xx),
I have an idea that the differential of this fairly complex function is itself ... am I right or wrong.

Answered by Penny Nom.
50^99 and 99! 2005-05-23
From Romel:
Which number is greater, 5099 or 99!
Answered by Claude Tardif.
A triangular lot 2005-05-21
From Linda:
I am looking to purchase a piece of property, and I need some help trying to figure out the square footage of the land. I know that normally if I multiply the length and width I could get my answer. However, this piece of land is an odd shape and has 3 measurements. Can you please help me.
The measurements for this piece of land is:

97.8'
201.2'
139.7'

Answered by Penny Nom.
A right triangle 2005-05-18
From Bill:
If I know the base and the slanted side of a right triangle, how do I figure out the height?
Answered by Penny Nom.
The circumference of an oval pool 2005-05-18
From John:
I have an oval pool of which I am trying to find the circumference. it is 38 feet long and 19 feet wide
Answered by Penny Nom.
Which is smaller 2^175 or 5^75? 2005-05-18
From Sarah:
Which is smaller 2175 or 575? without using a calculator.
Answered by Claude Tardif.
Jobs relating to the topics of ratios and proportions 2005-05-17
From Alexa:
What are some jobs relating to the topics of ratios and proportions other than architects and map makers?
Answered by Walter Whiteley.
Divisibility of a^2 + b^2 2005-05-16
From Ampa:
given natural numbers a and b such that a2+b2 is divisible by 21, prove that the same sum of squares is also divisible by 441.
Answered by Penny Nom.
What is my acreage? 2005-05-16
From Vonis:
My property is 65.5 feet by 130 feet. what is my acreage?
Answered by Penny Nom.
L'hopital's rule 2005-05-15
From Abraham:
Find the limit of [(1/(x+4))-(1/4)]/x as x approaches zero. How do you use l"hopital's rule to find this limit. I know how to do it with multiplying everything by 4(x+4), and getting the answer, -1/16.But how do you apply derivatives with l'hopitals rule to this type of problem?
Answered by Penny.
Angle of incline 2005-05-15
From Kyle:
What is the degree of incline of a 12 foot plank that goes from 10.5 inches on one end to zero inches on the other?
Answered by Penny Nom.
Surface areas 2005-05-11
From Jessica:
How can I demonstrate to my high school students the reason for the formulas for the surface area of a prism, right cylinder, and regular pyramid, and right cone?
Answered by Penny Nom.
sin x + sin 2x + sin 3x + sin 4x = 0 2005-05-10
From Elia:
I tried many times, but can't get to solve the following question:
sin x + sin 2x + sin 3x + sin 4x = 0

Answered by Chris Fisher.
Some triangle problems 2005-05-06
From A student:
1.use the heron's formula to find area of triangle ABC to nerest tenth

A.a=21 b=28 c=11

B.a=23 b=28 c=12

2.Find the nearest tenth the altitude of the longest side if a=3 b=3 c=5

3.the length of each side of a rhombus is 10 cm the length of one diagonal is 12 cm find the area to the nearest square centermeter

Answered by Penny Nom.
y = x+2 and y = 2x - 1 2005-05-03
From Kimberley:
Question: y=x+2 y=2x-1
Answered by Penny Nom.
Graphing an inequality 2005-05-02
From Janice:
I need to know how to graph inequalities. I need to know just abot everything. From graphing an equation to shading.
Answered by Leeanne Boehm.
Volume and surface area of a prism 2005-04-29
From Melissa:
Can you please tell me if the volume and surface area of a prism can ever be the same? Could you please explain why or why not.
Answered by Walter Whiteley.
Area of a region on a map 2005-04-28
From Balachandar:
Pl. find the attached map.

1. Can u pl. find out the total square are of WLMZ
2. Can u pl .find out the total square are of WXYZ -- { A }
3. Can u pl .find out the total square are of XLMY -- { B }

Pl .help me to find out the above.

.....


Answered by Chris Fisher and Penny Nom.
Probability and smoking 2005-04-24
From Sean:
20% of a certain population smoke. For smokers the probability of dieing of lung cancer is ten times what it is for non-smokers. Over all the probability of dying of lung cancer is .006. Find the probability for smokers and nonsmokers.
Answered by Chris Fisher.
An isosceles triangle...with 2005-04-23
From Shannon:
If given an isosceles triangle...with
Is there any possible way to do this, without knowing a side, if so, please explain in detail.

Answered by Chris Fisher.
Centimeters, millimeters and milliliters 2005-04-19
From A student:
How many milliliters in a centimeter?
Answered by Penny Nom.
A Taylor series for ln(x) 2005-04-16
From Anood:
i have to represent ln(x) as a power series about 2

i`m not getting the final answer which is ln 2+ sigma (((-1)(n+1)/ (n*2n))*(x-2)n). i don`t get the ln 2 part

i show you my trial

f(x)= ln x.

f-(x)=(1/x) .

f--(x)= (-1/x2)*1/2!

f---(x)= (2/x3)*1/3!

f----(x)= (-6/x4)* 1/4!

so the pattern shows me that f(n)= ((-1)(n+1))/xn *n)

so f(2)= sigma ((-1)(n+1))/2n *n) *(x-2)n

so as you see i don`t get ln 2


Answered by Penny Nom.
How much sand is in a pile? 2005-04-16
From Larry:
I was wondering if there is a formula for determining how much sand would be in a pile. I am a student in medical school, and this is a bonus question for a test. I hope you can help me.
Answered by Harley Weston.
A roll of paper 2005-04-16
From Tuomas:
How would you calculate a diameter of a paper reel when the paper is 0,04 millimeters thick and for example 5000 meters of it is winded on a 76 mm (3inch) core.
Answered by Penny Nom.
A flaw in a problem 2005-04-15
From Bryce:

Question:

(x2-x2) = (x2-x2)
x(x-x) = (x+x)(x-x) [divide both sides by (x-x)]
x = x + x
x = 2x [divide both sides by x]
2 = x/x = 1

Where is the flaw in this problem?


Answered by Paul Betts.
The limit as x approaches 3 of (sqrt(x+1)-2)/(x-3) 2005-04-14
From Abraham:
What is the limit as x approaches 3 of (sqrt(x+1)-2)/(x-3)
Answered by Penny.
cos(2x) = sin(3x) 2005-04-14
From A student:
Show that if x= 18 degrees, then cos2x =sin 3x. HENCE find the exact value of sin 18 degrees, and prove that cos 36 - sin 18 =1/2.
Answered by Andrei Volodin, Claude Tardif and Penny Nom.
The centroid of a triangle 2005-04-07
From Maria:
Q3)find the coordinates of the centroid of triangle ABC i want your help here to solve the 3rd question i got stuck.
Answered by Penny Nom.
Difference in latitude 2005-04-05
From Gretel:
Assuming the the earth is a sphere of radius 6378 kilometers, what is the difference in latitude of two cities, one of which is 400 kilometers due north of the other? 500 kilometers?
Answered by Penny Nom.
Rational functions 2005-04-05
From Nicole:
My name is Nicole and I am a teacher at Weyburn Comprehensive School. I am currently teaching both Math B30 and Calculus 30 at the school and I have a question about rational functions. I know that if a rational function (by definition) has common factors in the numerator and the denominator then it is not a rational function (math b30) however in calculus this common factor creates a hole in our graph. Can you explain to me why a common factor or constant does not give us a rational function?
Answered by Penny Nom and Leeanne Boehm.
Ax + By = C 2005-04-03
From Farzana:
I was wondering if you could please explain to me how I would write the equation of line through the given points [(-3,7), (0,5)] in Ax+By=C form.
Answered by Penny Nom.
Some liquid in a cone 2005-04-03
From Vasuki:
There are 2 right cones, height is X. one of them is filled from the bottom up (round side) = x/2, when you invert the cone and add the same amount of liquid inside the second cone, what is the HEIGHT of the liquid?
Answered by Penny Nom.
Bayes Theorem 2005-04-03
From wei:

In the January 11,1988,issue of the Oil&Gas Journal, R.A.Baker describes how the Bayesian approach can be used to revise probabilities that a prospect field will produce oil. In one case he describes, geological assessment indicates a 25% chance the field will produce oil. Further,there is an 80% chance that a particular well will strike oil given that oil is present on the prospect field.

  1. Suppose that one well is drilled on the field and it comes up dry. What is the probability the prospect field will produce oil?
  2. If two wells come up dry, what is the probability the field will produce oil?
  3. The oil company would like to keep looking as long as the chances of finding oil are greater than 1%. How many dry wells must be drilled before the field will be abandoned
  4. If the first well produces oil,what is the chance the field will produce oil?

Answered by Andrei Volodin and Penny Nom.
Gallons in a tank 2005-04-02
From Jim:
Would you be so kind as to help me with a problem? My memory has failed me in remembering how to calculate how much liquid in gallons a tank will hold. The tank is 8 feet long and 44 inches in diameter.
Answered by Penny Nom.
An asphalt driveway 2005-04-02
From Bob:
I'm trying to convert square feet to cubic yard.
I have an asphalt drive way I'd like to replace with concrete. Concrete is sold by the cubic yard and I keep telling myself that their must be an easy conversion table out there somewhere.

My driveway is 18 feet wide and 47 feet long with a proposed depth of 4 inches.

Answered by Penny Nom.
An inequality 2005-03-31
From Ambross:
demostrate the following inequality:
2.Sqrt(K+1)-2.Sqrt(k)<1/Sqrst(K)<2.Sqrt(K)-2.Sqrt(k-1)

Answered by Penny Nom.
The integral of tan^4x 2005-03-31
From Nicole:
college student. need help with tough integral.

the integral of 2tan4xdx


Answered by Penny Nom.
Two matrix problems 2005-03-30
From Sue:
Question 1
Suppose all matrices in the equation below are square and invertible. Solve for x .

BA-1XB-1 + 2BA + In = 0 (the symbol "0" here denotes the matrix of all 0's in it)

Also, A-1 or B-1 is indicating inverse and "In" = for example, A-1 times A
I hope you understand the above. I have to show all the steps.

Question 2
Suppose we consider the set of all 2x2 matrices along with the operations of matrix addition and multiplication. Do they form a field? Why or why not?
I think the answer is no because under multiplication it is not commutative and not all square matrices are invertible. I not positive so I'd like some help.

Answered by Penny Nom.
The shortest distance from a point to a line 2005-03-30
From carly:
Find the shortest distance from the point (-15, 2.5) to the line 5x+6y=30.
Answered by Penny Nom.
A wishing well 2005-03-28
From Don:

I am building a wishing-well out of pieces of 2-by-4. I have included a picture of a miniature version of what I want. There are to be ten 2-by-4 pieces around the well and I want the circle around the outside of the structure to have a diameter of approximately 3 feet. How long to I cut the 2-by-4's to build the wishing-well.

Thanks,
Don


Answered by Harley Weston.
Explain why 3(x+2) = 3x+2 is incorrect 2005-03-28
From Cynthia:
An algebra student incorrectly used the distributive property and wrote 3(x+2) = 3x+2. How would you explain to him the correct result, without the use of the distributive law?

Explain why the square of the sum of two numbers is different from the sum of the squares of two numbers.

Answered by Penny Nom.
A product equals a sum 2005-03-27
From Tor:
Can you please help me with a general formula for the following system:
2+2=2*2
3+3/2=3*3/2
4+4/3=4*4/3

and how will you prove the formula?


Answered by Penny Nom.
A torus and a sphere 2005-03-27
From Tony:
Is it possible to shrink a torus into a sphere?
Answered by Andrei Volodin and Penny Nom.
Gasoline in a cylindrical tank 2005-03-23
From Jennifer:

Gasoline is stored in a tank which is a cylinder on its side. Height of fuel is "h" meters and the diameter is "d". The length is "l".

I need to find the amount of gas in the tank when the height is h and also to calculate the fraction of how full it is.

Also, the part I am really confused on is this one,
E(h/d) is the error of the function of h/d, when h/d is used to measure how full the tank is. For what value of h/d is the error maximal?


Answered by Penny Nom.
Expanding (x+2)(x+3) 2005-03-23
From Rick:
The problem involves writing expressions, one in factored form and one in expanded form, for the area of a rectangle of sides x+2 and x+3. We've gotten as far as area = (x+2)(x+3). But I can't come up with the other expression.
Answered by Penny Nom.
Find the height of the pole 2005-03-22
From Dorothy Jean:
From the top of a building 70ft.high, the angle of elevation of the top of the pole is 11.2 degrees. From the bottom of the building the angle of elevation of the top of the pole is 33.4 degrees. Find the (a) height of the pole and (b) the distance from the building.
Answered by Penny Nom.
|ax+b| = cx + d 2005-03-22
From Angel:

|ax+b| = cx + d

how to show sometimes the above has 2 roots, sometimes 1 root and sometimes no roots. Hopelessly lost on how to start...


Answered by Penny Nom.
Two overlapping circles 2005-03-20
From Safi:
I have a problem to calculate the area of two overlapping circles because two circles are overlap then how i calculate the overlap area to subtract from the area of both circle.
Answered by Penny Nom.
Tangents to a circle 2005-03-19
From Sue:

You're given the equation to a circle (x-3)2 + (y-3)2 = 4 and you need to do 3 things:
1. Find a point on the circle
2. Construct an equation for a tangent line to the circle and through the point
3. Plot the circle, point and the tangent line on one graph

Answered by Penny Nom.
A matrix construction problem 2005-03-14
From Marcelo:
I want to know if is it possible to solve this problem:
I have an empty NxM matrix and I know totals (sum) by rows and totals by column.
Is there any algorithm to fill the matrix so that the summary of columns and rows gives the original values I have?

Answered by Harley Weston.
If the mixture is 1 pint to 6 gal 2005-03-13
From Trey:
If the mixture is 1 pint to 6 gal. Do i use 4 pints to 24 gallons of gas
Answered by Penny.
The volume of a stock tank 2005-03-13
From Cord:
I am a farmer and I have recently installed a new stock tank which is 1650" in circumference and 10' deep. How many gallons of water will it hold?
Answered by Penny Nom.
Doubling time 2005-03-12
From Nick:
Looking for the equation to how many years it will take to double the population given the growth rate is 7%.
Answered by Penny Nom.
21yd 2ft divided by 5=what? 2005-03-10
From A student:
1gallon 3qt divided by 6= what?
21yd 2ft divided by 5=what?

Answered by Penny Nom.
Discrimination based on gender? 2005-03-10
From A student:
After being rejected for employment, Kim learns that the Bellevue office has hired only two women among the last 20 new employees. She also learns that the pool of applicants is very large, with an approximately equal number of qualified men and women. Help her address the charge of gender discrimination by finding he probability of getting two or few women when 20 people are hired, assuming that there is no discrimination based on gender. Does the resulting probability really support such a charge?
Answered by Penny Nom.
A wrestling schedule 2005-03-08
From Ed:
Hello my name is Ed and I am a couch of a wrestling team in New Jersey. I was doing searches for formulas on how to make a matrix for team sports and it seems to be hard to find. I came some how to your site and thought I'll give it a try. Here is what we need or are trying.
Answered by Claude Tardif.
A nine week schedule 2005-03-08
From Greg:
I'm trying to build a nine week schedule where 10 teams play each other once throughout the season.

I make it all the way to week nine, then I start having all sorts of conflicts.

HELP!!


Answered by Claude Tardif.
Biodiesel 2005-03-08
From William:
My son is doing an demonstration for science class about how to make biodiesel. we are having problems with the conversion. for instance we think there are approx 190 liters in 50 gallons. Is this correct? Also if you needed 3.5 grams of lye per liter, would that be 662g of lye.
Answered by Penny Nom.
Perfecting an ideal gambling system 2005-03-06
From Gaz:
I am a screenwriter, currently in the fortunate position of having the development of a Screenplay funded by the South Australian Film Corporation. The (anti)hero of this screenplay is a statistician whose life is falling apart around him, thanks in part to his obsession with perfecting an "ideal" gambling system.
Answered by Andrei Volodin.
I need to fill an area with rock 2005-03-04
From Kelly:
if I need to fill an area with rock that is 64 feet long by 4 feet wide and 1 foot 2 inches in depth
how many tons of rock will this job require?
it is also sold by cubic yard how many of these cubic yards will I need?

Answered by Penny Nom.
How much of a acre is this? 2005-03-02
From Ken:
my property is 11,914 square ft. How much of a acre is this?
Answered by Penny Nom.
Reverse pecentage 2005-03-01
From Nathan:
Question for you. If I have spent $10.00, what is the mathamatical equation to
figure out what the G.S.T (7%) was?

Also if I were to spend $80.00 on a hotel, and I would like to know how much
G.S.T (7%) and how much provincial tax (6%) I spent how would I go about this.


Answered by Penny Nom.
The area of a quadrilateral 2005-02-27
From Jonathan:
I want to know how to find the area of a quadrilateral.
Answered by Penny Nom.
A triangle question 2005-02-26
From John:
H=10' 7and3/4",angleX=30 degrees solve for the adjacent side
Answered by Penny Nom.
A countably infinite collection of countably infinite sets 2005-02-26
From Feroz:
Suppose a set can be divided into a countably infinite number of countably infinite sets.Then can the original set be considered as a countably infinite set?
Answered by Penny Nom.
The volume of a sphere 2005-02-25
From Stephanie:
find the volume of a sphere that has a diameter of 12 cm. please show me how to work this problem
Answered by Penny Nom.
Trading fish for bread 2005-02-25
From Mike:
Byron lives where people trade goods they produce for other things they need. He has some fish and wants to trade them for bananas. He finds the following:
5 fish = 2 loaves of bread
6 oranges = 2 melons
1 loaf of bread = 1 banana and 3 oranges
4 loaves of bread = 14 oranges
How many bananas can Byron get with 5 fish?

Answered by Penny Nom.
The capacity of a dryer 2005-02-23
From Darlene:
I am trying to figure out the capacity of my dryer. I need to know the size in order to buy a new one. The drum within measures 27" in diameter and 18" deep. I know that I am looking for an answer in cubic feet but am not sure how to determine this.
Answered by Penny Nom.
Each interior angle of a particular polygon is an obtuse angle... 2005-02-22
From Victoria:
Each interior angle of a particular polygon is an obtuse angle which is a whole number of degrees. What is the greatest number of sides the polygon could have?
Answered by Walter Whiteley.
Grams per liter 2005-02-20
From A student:
A litre and a half of water has 0.045 grams of herbicide added for spraying roses. How many grams of herbicide per litre is this?
Answered by Penny Nom.
1 pied linéaire 2005-02-18
From Christiane:
Pouvez-vous m'indiquer 1 pied linéaire égale quoi? 3 pied....?
Answered by Claude Tardif.
Graphing inequalities 2005-02-15
From Melanie:
Solve and Graph the following variable inequalities:

y<2x+4

y>-2x+6

3x+4y<-24

4x-6y>-24

Answered by Penny.
The weight of some concrete 2005-02-15
From Jordan:
I have 6 acres of concrete that is 3 feet thick the aprox. weight of 1 cubic yard of concrete is 4024 pounds I am trying to find out how much my 6 acres weighs. fortunately my concrete is 3 feet deep.
Answered by Penny Nom.
One million times one billion 2005-02-15
From A student:
How much is ONE MILLION TIMES ONE BILLION in the American system ?
Answered by Penny Nom.
What is the least three-digit palindrome that is a square number? 2005-02-12
From Ben:
What is the least three-digit palindrome that is a square number?
Answered by Chris Fisher and Penny Nom.
Understanding fractions 2005-02-11
From C.J.:
I have some questions about the lesson, "Understanding Fractions" by Diane Hanson, Regina Catholic Schools. How successful was the lesson? Are there any changes you would recommend?
Answered by Diane Hanson.
arcsech x 2005-02-10
From Monica:
Prove that arcsech x = ln[(1 + (1-x2)(1/2)) / x ]
Answered by Penny Nom.
Th dimensions of a garden 2005-02-09
From Rebecca:
Nate's garden has a perimeter of 34 meters and an area of 72 square meters. What are the dimensions of the garden?
Answered by Penny Nom.
Sales before tax 2005-02-09
From Lynda:
I know my total sales for the month, which includes sales tax. How do I get to the sales dollars before the sales tax?
Answered by Penny Nom.
A combination lock has 60 positions... 2005-02-07
From Kathy:
A combination lock has 60 positions arranged in a circle to which a pointer may be set. Three settings are required to open it and no one setting may be closer than 5 positions from the immediately preceding setting. How many different locks may be manufactured?
Answered by Penny Nom.
Extraneous solutions 2005-02-04
From Heather:
My teacher wants to know why there are extraneous solutions in logarithms?
Answered by Penny Nom.
A seating arrangement 2005-02-03
From Kay:
If you have 5 married couples, how many arrangements can be made if they must sit across from each other?
Answered by Penny Nom.
The diaonals of an isosceles trapezoid are congruent 2005-02-02
From Daniel:
I am currently unable to find a proof that the diaonals of an isosceles trapezoid are congruent. Might you happen to have one?
Answered by Walter Whiteley.
Solve for x 2005-02-02
From Christie:
Solve for x

.387 = (.40 - .265x)/(sqrt(1-x2))

Answered by Penny Nom.
A line segment connecting two vertices of a polygon 2005-02-02
From James:
What is a line segment (not a side) connecting two vertices of a polygon called?
Answered by Penny Nom.
The volume of an aquarium 2005-02-01
From Selena:
Im trying to figure out the amount of gallons from a rectangle 48x20x12?
Answered by Penny Nom.
Are the vetices of this quadrilateral concyclic? 2005-01-30
From A student:
Show that the peints A(-2,0),B(6,6),C(-1,7) and D(-2,6) are concyclic. Also find its circum radius.
Answered by Chris Fisher.
Some logarithm problems 2005-01-27
From Jo:
Given ln x=7.1; ln y=8.2, Find ln(x^3y^7)
Answered by Penny Nom.
By how much is 100 multiplied or divided,... 2005-01-27
From Benedict:
By how much is 100 multiplied or divided, so that whether the number obtained is divided by 5 or 7, there will be a remainder of 1 and so that at the same time, the first quotient is greater by 4 than the second?
Answered by Penny Nom.
Approximating an integral 2005-01-27
From A student:
i used excel to find the area under a graph instead of integrating the function the analytical way. These are the different answers i got with different width of the x-values, the smaller the x-increment gets the more precise the area is when compared to the answer obtained the numerical way. I have trouble seeing why the answers i get is getting bigger and bigger and not smaller can someone plz explain?
Answered by Penny Nom.
Three tangent circles 2005-01-25
From Kate:
Two circles, C1 and C2, touch each other externally; and the line l is a common tangent. The line m is parallel to l and touches the two circls C1 and C3. The three circles are mutually tangent. If the radius of C2 is 9 and if the radius of C3 is 4, what is the radius of C1?
Answered by Chris Fisher.
Differentiating F(x,y) = 0 2005-01-23
From Jacob:
In calculus, we often mention to the students that if F(x,y) = 0, then we can differentiate both sides and still get an equality. The problem is that we can't perform the same operation on F(x) = 0, say x = 0, otherwise 1 = 0, which is absurd. What is the reason?
Answered by Walyer Whileley and Harley Weston.
The diameter of a 800 km circle 2005-01-22
From Mechelle:
What is the diameter of a 800 km circle?
Answered by Penny Nom.
A 6 sided (hexagonal) pyramid 2005-01-22
From Steve:
im trying to make a 6 sided (hexagon) pyramid, from 6 triangles of 12mm plywood, i know all the angles to cut apart from the one one to join all 6 triangles together. Rough measurements are outer edge (A) of each triangle is 13cm's, length of other 2 sides (B&C) of triangle outside to center is 14cm's with a height of the whole thing together about 6cm's.
Answered by Chris Fisher and Harley Weston.
The area of a triangle 2005-01-21
From Amy:
In tirangle ABC, AB=6, BC=9, BC is the angle bisector of angle ABC and M is the midpoint of BD. If the area of ABC is 30, what is the area of ABM? The height comes out greater than the hypotenuse, but the teacher said that there's an answer for it. Can you show me how? Thanks
Answered by Chris Fisher.
Finding the missing side of a triangle 2005-01-20
From Jason:
I found a geometry problem that reads as follows:In triangle ABC,
Answered by Penny Nom.
The angle between two forces 2005-01-19
From Abraham:
One force of 20 pounds and one force of 15 pounds act on a body at the same point so that the resultant force is 19 pounds. Find, to the nearest degree, the angle between the two original forces.
Answered by Penny Nom.
The radius of a circle 2005-01-18
From A student:
find the radius of a circle whose area is 1256sq cm.Use pi as an appoximation for pi.
Answered by Penny Nom.
A non-rerctangular lot 2005-01-18
From EM:
One corner of a 60X120 foot lot, otherwise rectangular, is a curve with a radius of 20 feet and a central angle of 90 degrees. What is the area?
Answered by Penny Nom.
The side of a cube 2005-01-17
From Jake:
I have one for you that's got me stumped. Is it possible to calculate the dimensions of a square if I have the cubic feet? Say I know the package to be 180 cubic feet.
Answered by Penny Nom.
The length of a chord 2005-01-13
From A parent:
Does anyone have a formula for calculating the chord length for a segment of a circle when you know the radius and the enclosed angle or radian ?
Answered by Penny Nom.
Profit 2005-01-10
From Abraham:
The profit a coat manufacturer makes each day is modeled by the equation P(x)=-x2+120x-2000, where P is the profit and x is the price for each coat sold.For what values of x does the company make a profit?
I don 't understand this problem(how to do it) and hope you can help me.

Answered by Penny Nom.
Arcs and chords 2005-01-09
From Aniesha:
A chord of a circle is 48 centimeters long and is 10 centimeters from the center of the circle. Find the radius?
Answered by Penny Nom.
Acres 2005-01-08
From Jim:
What size is this lot?
And is there a location on the web where you can plug in lot dimensions to figure acreage?

North = 180ft
South = 210ft
West = 336ft
East = 244 ft

Answered by Harley Weston.
ln(hx+n) 2005-01-07
From Anibal:
if I have the eq. y=hx+n and I want to know ln(y), how do I process ln(hx+n)?
Answered by Penny Nom.
Quadratics 2005-01-05
From Usman:
Hi, in my Grade 11 Functions math class we have been assigned the task of finding jobs and careers related to quadratics, I have done many searches but have been unsuccessful, then I saw your website and e-mailed. I also have to use an example of a math problem that the job uses, then solve it, this will all compile on bristol board for a presentation. I would greatly appreciate it if you could send me some links and references of sources that refer to this subject.
Answered by Harley Weston.
Express the answer in dollars 2005-01-03
From Abraham:
My question on this question is if I wrote just 5280 because they say they want it in dollars, is that correct or is the only answer, 5279.61 as their answer key says? How should I have known to give it with cents? Should I give myself full credit?
Answered by Harley Weston.
An isosceles triangle 2005-01-03
From Abraham:
The question is,"Triangle ABC is not isosceles.Prove that if altitude BD were drawn, it would not bisect AC."My question is If an altitude is drawn wouldn\'t that mean automatically its isosceles because, In a triangle the sides opposite congruent angles(in this case the right angles)are congruent? What am I thinking wrong?
Answered by Harley Weston.
A sequence of transformations 2005-01-03
From Abraham:
Please go to this web site and look at number 31.

http://www.nysedregents.org/testing/mathre/matharch/mathbtestau02.pdf

That is the New York state education web site. Can you tell me if my answer would be marked correct? My answer was Transformation of 4,-3 (T 4,-3) followed by a reflection in the x-axis.If I graphed the question and wrote that, would I get full credit

Answered by Harley Weston.
An elliptical table 2005-01-03
From Roger:
Want to make an elliptical table, say the long (major) axis is 4 feet, and the short (minor) axis is 3 feet. I can construct this figure, but I'm trying to figure out what the exact dimension of a rectangle within this ellipse will be if I make the table a drop leaf type where the drop dimensions are equal for each end of both the long and short axes. Intuitively, it looks like there is one and only one solution.
Answered by Penny Nom.
A 4-digit access code 2005-01-02
From Missi:
I need to find the 4 digit number to access my voice mail. These numbers will be 0-9 & numbers can be repeated. Could you provide me with a list of possibly combinations? If not, do you know anywhere that I could get a list of combinations so I don't have to write it all down?
Answered by Harley Weston.
A line from the center of the patch to the periphery 2005-01-01
From Sandrine:
I am currently researching a patch disease of grasses. These patches are roughly circular. I need a term for a line from the center of the patch to the periphery. Since the patches are not perfectly circular, my supervisors tell me I cannot use the word 'radius'. What else could I use?
Answered by Denis Hanson and Harley Weston.
A geometry problem 2005-01-01
From Alexandra:
In triangle ABC, b=40, and angle A= 30 degres. What values of BC will give two solution for angle B?
Answered by Penny Nom.
The golden ratio 2004-12-31
From Cristina:
let x represent the longer segment. to find the golden ratio, write a proportion such that the longer of the two segments is the geometric mean between the shorter segment and the entire segment.Use the quadratic Formula to solve the proportion for X. Find the value in both radical and decimal form.
Answered by Penny Nom.
The area of a triangle 2004-12-30
From Perry:
What is the area of a triangle with dimensions 3"X5"X7"? Could you provide the formula?
Answered by Penny Nom.
Gallons in a cylinder 2004-12-29
From Dean:
I have 2 cylinders:
Cylinder A = 8'D x 12'H
Cylinder B = 10'D x 15'H


Each 1ft represents how many gallons of fluid?

Cylinder A=?

Cylinder B=?

Answered by Penny Nom.
Half of five 2004-12-26
From Pauline:
Why is four half of Five? Hint: It's the middle half?
Answered by Penny Nom.
Modelling monthly temperature with a cosine 2004-12-25
From Regis:
The average monthly temperature for a location in Ontario as a function of month number can be modelled using the equation y = a cos[k(t + b)] + d.
Answered by Harley Weston.
Irrigation and a sector of a circle 2004-12-23
From Chuck:
A friend of mine is a farmer and uses Pivots to irrigate portions of his land. The crop rows are in straight lines that all form chords of a large circle. The intent is to determine area between any two "boundary" rows expressed in acres.
Answered by Harley Weston.
A Reuleaux triangle 2004-12-22
From Bob:
I am trying to remember the name given to the regular figure constructed from three arcs. The figure is like an equilateral triangle except with arcs for each side.
Answered by Chri Fisher.
Divisibility by 15 2004-12-19
From Lisa:
My son was asked to find divisiblity rules for 15. We have been unable to find the answer. Does it exist?
Answered by Leeanne Boehm and Denis Hanson.
The gradiant of a hill is 9% 2004-12-18
From Jim:
The gradiant of a hill is 9%. What angle is created by the run/rise of the hill and 0 degrees?
Answered by Penny Nom.
Two intersecting lines 2004-12-18
From James:
Imagine a vertical line 1.107' tall. Leaving the top of this line, sloping down to the right at a 4:1, at what horizontal distance will the line strike a second line, which leaves the the bottom of the vertical line sloping down to the right at a rate of .02 ft/ft (or 2%)? There is a fairly easy solution to this, but I have lost my notes!
Answered by Harley Weston.
A two equation fact family 2004-12-17
From Tony and Hailey:
My Daughter Hailey had this question as part of her 4th grade math homework. Name 2 fact families that have only two equations. For the life of me I can't figure it out. Any ideas?
Answered by Penny Nom.
0.999..., asymptotes and infinity 2004-12-17
From Mike:
My Name is Mike and I teach high school. I had a student ask me to explain why .9 repeating is equal to 1. Then he asked me about an asymptote, or why a parabola or any other curve for that matter can continually approach a value (like 1) and yet never attain a value of 1. He is thinking that these two should represent the same concept and yet they contradict each other. Do you have a solid explanation for him? Of by the way he is a 7th grader. Great little thinker!!!!!
Answered by Claude Tardif and Harley Weston.
2^10 = -1/2^x - 1 2004-12-13
From Randy:
Could you explain to me how the following equation is solved please.

2^10 = -1/2^x - 1

Answered by Harley Weston.
Three calculus problems 2004-12-09
From Ashley:
Hi, I am having a lot of trouble with three calculus questions and was wondering if you could help
Answered by Penny Nom.
A regular hexagon is inscribed in a circle. 2004-12-08
From Abraham:
A regular hexagon is inscribed in a circle. What is the ratio of the length of a side of the hexagon to the minor arc that it intercepts?
(1) pi/6
(2) 3/6
(3) 3/pi (This is the correct answer.)
(4) 6/pi
I found the length of the minor arc to be (pi)(r)/3 by doing a sixth of the circumference(2pi r).But I can't find the length of the radius to finish off the problem. If I knew the radius I would then plug it into the above and then use the radius again to be the length of the side because the triangle(one of the six of a hexagon) is equilateral. But can you show me how to get the radius to be 3? Thank you so much.

Answered by Walter Whiteley.
A belt around two pulleys 2004-12-07
From Ian:
a belt is stretched around two pulleys whose centers are d units apart and whose radii are R and r respectively (obviously R+r<d). the challenge is to find the length of the belt, l as a formula in terms of R, r, and d only.
Answered by Penny Nom.
An arc of a circle 2004-12-05
From Ruben:
i have an arc 55 inches wide, 12 inches high at the centerline of the arc. how can i determine the diameter of the circle that would correspond to the arc.
Answered by Penny Nom.
Probability 2004-12-04
From A parent:
Consider a 30 sided polygon. If three diagonal are selected at random, what is the PROBABILITY that they share a common endpoint?
Answered by Denis Hanson.
Select a card from the deck. 2004-12-02
From Heidi:
Select a card from the deck. What is the probability that this card will be red? Show the number of expected outcomes versus the number of total possible outcomes. What type of event does this represent?
Answered by Penny.
1/x + 1/y = 2/31 2004-12-01
From matt:
I have this problem which I think is a bit more tricky than usual. x and y are positive integers.

1/x + 1/y = 2/31

Answered by Claude Tardif.
e = ln(b/a) solve for a 2004-12-01
From Daniel:
I am trying to make "a" the subject of this equation "e = ln(b/a)" but am not sure if i'm doing it right. Any help would be appriciated,
Answered by Claude Tardif.
The volume of a sphere 2004-11-30
From Lyndsay:
twenty cubic metres of air are pumped into a spherical hot air balloon with each given diameter: 1.0m; 2.0m; 3.0m; 4.0m; 5.0m;6.0m
a)calculate the new diameter, new suface area, and new volume of each balloon.

Answered by Penny Nom.
The tangent line at an inflection point 2004-11-28
From Louise:
the equation of the tangent line to the curve y = x3 - 6x2 at its point of inflection is...
Answered by Penny Nom.
Computing confidence intervals 2004-11-26
From Christie:
I was given a question with N=100, sample proportion is 0.1- compute the 95% confidence interval for P? I have tried this several ways but do not know how to do without means, standard deviations, standard error of the mean? I asked my teacher and she said I have all the info I need. Can you help????
Answered by Penny Nom.
A right triangle 2004-11-24
From Bruce:
In triangle ABC.
Angle ABC is 90 degrees.
Side AB measures 34.
Side BC measures 31.
What does side AC measure?

Answered by Penny Nom.
Slicing cubes 2004-11-23
From Anthony:
You are working with a power saw and wish to cut a wooden cube 3-inches on aside into 27 1-inch cubes. You can do this by making six cuts through the cube keeping the pieces together in the cube shape. Can you reduce the number of necessary cuts by rearranging the pieces after each cut?
Answered by Chris Fisher.
A graph with certain properties 2004-11-22
From A student:
i was asked as a question in coursework to sketch the graph with the following characteristics:
a double root at -3
a pair of imaginary roots
an x-intercept at 6
a root at 4 which is not a double root

Answered by Penny Nom.
The length of an arc 2004-11-21
From Daryl:
I am tying to find the arc length of a line 6' and the vertex of the arc 1' off of the line. that is all that is known
Answered by Penny Nom.
Is a square a rectangle? 2004-11-21
From Carol:
I am a teacher. In an FCAT sixth grade review test, there was a question to the students to draw a square and then they referred to it as a rectangle.

What is the definition that makes a rectangle a square that can be taught to the students without confusing them.

Answered by Walter Whiteley.
Proof by induction 2004-11-20
From Vic:
Problem: Find the first 4 terms and the nth term of the infinite sequence defined recursively as follows:

a(1) = 3 and a(k+1) = 2a(k) for k -> 1.

Note: Quantities in brackets are subscripts
-> means 'equal to or greater than'.

Using the recursive formula, the first 4 terms are; a(1) = 3, a(2) = 6, a(3) = 12, a(4) = 24

The nth term a(n) = 2n-1 x 3 (equation 1)

Equation 1 must be proven using mathematical induction. This is where I am having a problem.

Answered by Penny Nom.
Solving quadratics 2004-11-20
From Aaron:
Can you solve the following equation (quote the value of x,y and t to 3 sig figs)

((i have changed figures so that i can work through my own example))

x2 + x - 6 = 0

5y2 +19y = 3 10cos2 t + 3cos t = 4

Answered by Penny Nom.
Construction of a cone 2004-11-20
From John:
I am a builder working on a project where I need to make a cone. It's a right circular cone with 15" base radius and slant angle of 30 degrees. I want to cut it out of flat sheet metal then bring the edges together to form the cone. Is this enough information?
Answered by Penny Nom.
Volume of a box 2004-11-18
From Catherine:
I need to find a formula to calculate the amount of liquid would hold. maybe the volume or something. its a square box used to hold liquid on a farm . we want to measure how much liquid it can hold.
Answered by Penny Nom.
1/4 tsp salt = ??? grams 2004-11-16
From Julie:
if i have 1/4 teaspoon of salt how many grams does that equal
Answered by Penny Nom.
Equations involving radicals 2004-11-16
From Merin:

I am quite confused about solving radical equations. This is what I understand:
___
V x-3 = 3

Then you would square each side to get ride of the radical sign and then you solve the problem from there. (x=12 right?)

However, what about a problem with two radicals? For example:
___ ____
V x-2 + V 3x-3 = 3

No matter what I try I cant get the right answer!! Please help!!!


Answered by Penny Nom.
Interior and exterior angles of a polygon 2004-11-16
From Aaron:
How do you find the measure of interior and exterior angles of a regular polygon when you are given the number of sides?
Answered by Walter Whiteley.
Planting trees 2004-11-15
From Heather:
Design a plan of how it would be possible to plant 10 trees in 5 rows, and have 4 trees in each row.
Answered by Penny Nom.
An ODE 2004-11-10
From David:
I have a question that i really cant do, it is as follows:

The ODE dy/dx + 0.5y = 0.5e^(-1.5x) ; y(5) = 2

Solve the ODE subject to the given condition using exact methods and evaluate the solution y for x = 5 x=5.2 x=5.4 x=5.6 x=5.8 x=6

Answered by Harley Weston.
A 30-60-90 triangle 2004-11-09
From Amy:
How do you find the length of the hypotenuse of a 30-60-90 triangle when the side opposite the 60 degree angle is 6 inches?
Answered by Penny Nom.
The lcm of 5, 3, 4, 7 and 2 2004-11-05
From Christine:
what is the lcm of 5,3,4,7,2
Answered by Penny Nom.
Probabilities in poker 2004-11-04
From Ben:
Please can someone tell me the odds of getting a royal flush while playing texas hold em.
Please can you also let us know the odds of a royal flush while playing texas hold em if you must be dealt two of the needed cards.

Answered by Chris Fisher.
The least common denominator 2004-11-03
From A student:
Write the LCD for each pair of fractions.

13. 1/3,1/5

14. 2/7,1/4

15. 3/4,3/5

Answered by Penny Nom.
The lcm of 1 and -1 2004-11-03
From Nayiyan:
I would like to know what is the LCM of (-1) and (+1) is.
Answered by Penny Nom.
Tennis balls and probability 2004-11-01
From Jeremy:
I have a hopper of tennis balls that contains 25 new tennis balls and 10 old tennis balls. In the hopper the tennis balls are mixed up randomly. If i take one ball out of the hopper and hit it and then put it back and then take another ball out of the hopper and hit it and put it back, what is the probability that i hit two new tennis balls? Also a tennis ball becomes old after being hit just once.
Answered by Penny Nom.
Multiplying two algebraic fractions 2004-10-31
From A parent:
Multiply:

x+2 x²-4
----- x -------
x-2 x²+x-2


Answered by Harley Weston.
Imaginary roots of a ploynomial 2004-10-31
From Jennifer:
how to find the roots of a polynomial equation if it would be imaginary?
Answered by Harley Weston.
GCD and LCM 2004-10-31
From Cartalina:
how do you calculate the "positive difference between the GCF and LCM of two numbers"?
Answered by Penny Nom.
The combined force of two vectors 2004-10-30
From Brian:
Two teams are playing push ball with a large 8 foot diameter ball. One team exerts a force represented by the vector a = 2i + -5j, and the other team exerts a force represented by the vector b = -8i-3j.

1.Determine the direction of movement of the ball if the i axis is due east.

2.Determine the combined force magnitude.

Answered by Harley Weston.
Solving triangles 2004-10-30
From Allen:
Solve the following triangles.

Given

1. B = 20 Degrees, a = 25, b = 16
2. A = 35 Degrees, b = 2, c = 3
3. A = 32 Degrees, C = 44, c = 20

Answered by Harley Weston.
The force of a 5000 tonne lorry 2004-10-27
From Aaron:
A 50 tonne lorry is parked on a slope with a gradient of 4%. Due to the self weight of the lorry, what is the force

* acting down the slope
* acting normal (at right angles) to the slope

Answered by Penny Nom.
20 Golfers 2004-10-26
From Ian:
I have 20 golfers playing over 6 days I would like to schedule all 20 golfers in 5 groups of 4 on each day here is the catch.... I would like each player to play with each other once during the 6 days.
Answered by Chris Fisher.
The point slope form of a line 2004-10-26
From Jack:
Given a set of ordered pairs, ie (1,1) (2,4) (3,7), how does one determine the rule f(n) other than by trial and error
Answered by Penny Nom.
Largest square inside a circle 2004-10-25
From Bob:
my granddaughter asked
what is the largest size square in inches would fit in a 60 inch circle?
I believe it to be around 42.3 inches but would like to teach her how to do it mathematically.

Answered by Penny Nom.
Square roots and inequalities 2004-10-25
From Waheed:
Q1. What is the simplest way of finding a square root of any number using just a pen and paper? (I am asking this question because I browsed a few sites a didn't find any method that is simpler than the one I use. so I am just curious.)

Q2. Is it possible that you take an equation and turn it into an inequality by performing same mathematical operations on both sides?

Answered by Claude Tardif and Penny Nom.
Converting coordinates 2004-10-25
From Allen:
I am trying to help my son with his home, but I don't remember these conversions. Please help.

1. Convert the following coordinates Cartesian x = 7, y = -24

2. Convert the following coordinates Polar r = 4, theta = pi / 3

Answered by Penny Nom.
Implicit differentiation 2004-10-24
From Emily:
If x^3+3xy+2y^3=17, then in terms of x and y, dy/dx =
Answered by Penny Nom.
Maximize income 2004-10-24
From Connie:
A company that sells x units of a product generates an income (I, in dollars) which is a function of x. The income generated is described by the equation

I = (-1/2)x^2 + 100x.

Discuss how to determine the number of units that must be sold so that the company can maximize its income. What is the maximum income?

Answered by Penny Nom.
Practical applications: parabolas and Pythagoras 2004-10-24
From Connie:
Provide two examples of real life objects that incorporate parabolic shapes. Explain the reason why the parabolic shape was used in each object.

I need at least one practical application of the Pythagorean Theorem.

Answered by Penny Nom.
Solve for a and t 2004-10-23
From Justin:
How do I solve for "a" and "t" in the equations:

1000t= -4000 + 2000t + (1/2)at^2
1000=2000 + at

Answered by Penny Nom.
An aircraft is flying directly from airport A to airport B 2004-10-22
From A student:
In this question, i is a unit vector due east and j is a unit vector due north. An aircraft is flying directly from airport A to airport B, which is 2000 km from A. The velocity in still air of the aircraft is (150i+50j)km/h and the ground speed is (200i-30j)km/h. Calculate

(i) the time of flight, to the nearest min
(ii) the direction of the wind.

Answered by Penny Nom.
The last digit of a large exponent 2004-10-20
From Landon:
How can one determine the last digit of a large exponent by hand. I have seen several examples of how to do this by breaking it up into smaller exponents, but is there a formula or some common things to look for?
Answered by Penny Nom.
sin(3A) 2004-10-20
From A student:
Express sin3A in terms of sinA and cosA.
Answered by Penny Nom.
A square footage problem 2004-10-19
From Jerome:
I have a peice of property that is almost a triangle but not exactly about 280 frontage with the side lines at 228 and 237

I was told I had about one acre plus a little, a developer says I have 33,000 sq ft. and I would like to know what the real result is

Answered by Penny Nom.
The third derivative 2004-10-15
From Holly:
Why would you ever take the third derivative?
Answered by Harley Weston.
Two gears 2004-10-14
From Lindsay:
"There are two gears, a small one on the left and a larger one on the right. The gear on the right makes 1 revolution. The gear on the left makes two revolutions. Suppose the gear on the right is turned through an obtuse angle. Will the gear on the left make a full turn?"
Answered by Penny Nom.
Divisibility by 7 and 11 2004-10-13
From Tammy:
I'm stuck in class in Yr 7 And I'm finding it hard on our new topic Divisibility! When I try to find out what this means on Internet sites i can not understand the used symbols like algebra and so on. I'm stuck on the divisibility rules for the number 11!
Answered by Penny.
Pythagoras in everyday life 2004-10-13
From Tiffany:
I was wondering if you have any real-life uses of the pythagorean theorem that you use in your everyday life.
Answered by Penny Nom.
Cycling in the wind 2004-10-11
From A student:
A cyclist is travelling in the direction of 12i+5j at a speed of 39 km/h where i is a unit vector due east and j is a unit vector due north. The wind appears to the cyclist to be blowing with a velocity of (-4i+j) km/h. Find, correct to 1 d.p
(a) the true speed of the wind and
(b) the bearing from which the wind is blowing

Answered by Penny Nom.
Accelerating to the speed of light 2004-09-30
From Lars:
How long time would it take to accelerate up to the speed of light with an acceleration speed similar to 0-100 seconds in 5 seconds?
Answered by Penny Nom.
A mixture problem 2004-09-30
From Abraham:
A chemist has one solution that is 35% pure acid and another solution that is 75% pure acid. How many cubic centimeters of each solution must be used to produce 80 cubic centimeters of solution that is 50% acid?
Answered by Penny Nom.
A theorem involving a trapezoid 2004-09-29
From Abraham:
Given:Trapezoid ROSE with diagonals RS and EO intersecting at point M
Prove:Diagonals RS and EO do not bisect each other.

Answered by Harley Weston.
The area of a lot 2004-09-29
From Deb:
I am trying to figure out how many square feet are in a piece of property. Start at Point A-then go 140 feet north-then 100 feet due east-then 300 feet at an angle southeast so that connecting to point A would be a straight line (right angle to first line north.)
Answered by Penny Nom.
Cubic yards of concrete 2004-09-29
From Tanya:
We are starting up a new business and for the business we will be required to order concrete for various shapes. Right now we have a job that requires a concrete slab that a decagon it is 140-3/4" from flat side to flat side, 74" from point to center and each flat side is 46" The pad should be 4" thick with 12"x12" continuous footing.
Answered by Harley Weston.
The area of a lot 2004-09-28
From Stan:
I own a parcel of land (lot 20) that is assessed at 9127.00 square feet and has a frontage of 99.90 feet according to the tax roll. I don't know how large my property actually is since I feel that the tax assessment and the survey map don't jive
Answered by Harley Weston.
A palindrome question 2004-09-28
From Lilly:
12, 21, 13, and 31 are the only double-digit numbers whose squares are the palindromes of the squares of the palindromes of double-digit numbers. I was wondering why it works for these numbers, and if only these numbers work this way.
Answered by Chris Fisher and Penny Nom.
The length of a rectangle 2004-09-26
From Rebecca:
My question is each side of a square S is 18 inches long and the area of S is at least 3 times the area of a rectangle that is 9 inches wide. What is the greatest possible length in inches of the rectangle?
Answered by Penny Nom.
Grams to milliliters 2004-09-25
From Emily:
I am trying to convert 19.5g into mL assuming that it's water; density=1g/mL. I don't know where to begin.
Answered by Penny Nom.
A parallelepiped 2004-09-25
From Aman:
" If OA, OB, and OC are three edges of a parallelepiped where O is (0,0,0), A is (2,4,-2), B is (3,6,1), and C is (4,0,-1), find the coordinates of the other vertices of the parallelepiped.)"
Answered by Penny Nom.
The last digit of 2^28 2004-09-25
From A student:
I am currently enrolled in a math reasoning class and a math question I have to answer is determine the units digit in 3 28power. When I set up a table and keep cubing the numbers, I find a pattern with the last digits of the numbers that follow 1,3,9,7. My calculator gets to about the 19th power, and then I get an error. If I continued the pattern, would the units digit then be the last digit of 3 28th power, which would be a 7? Please let me know if I am solving this correctly and what a unit's digit actually is.
Answered by Penny Nom.
Rational expressions 2004-09-24
From A student:
In general, I understand rational expressions except when it comes to solving problems such as:

x+y/2x-y - 2x/y-2x or m-4/3m-4 + 3m+2/4-3m

I am confounded by the issue of having to find a common denominator. For example, if I tried to solve these problems by multiplying both denominators they would still be uncommon.

Answered by Claude Tardif.
Supplementary angles 2004-09-23
From Rosemary:
Is the term supplementary angles only applicable to 2 angles (ie. a pair of angles) or can it be used when talking about 3 or more angles that add to make 180 degrees?
Answered by Penny Nom.
Incremental variance 2004-09-23
From Carlos:
I must keep statistical data (mean and variance) in 3 granularity levels depending on the age of the data (daily for older than 1 year, hourly for older than 1 month and quarter-hour for older than 1 day). How can I calculate the resulting variance from a set of variances previously calculated supposing I have the count and mean for each member of the set?
Answered by Andrei Volodin and Penny Nom.
The volume of a pile of mulch 2004-09-22
From Sam:
Is there a formula to determine the cubic feet of something in a pile. IE- I need to determine the cubic feet of a pile of mulch. The pile comes to a peak, so the length and width decrease as the pile increases.
Answered by Penny Nom.
Four and five digit numbers 2004-09-21
From Josy:
The question I have now in front of me is....Is a 4-digit number always less than a 5-digit number? Hi Josy, The digits in order are 0, 1, 2, 3, 4, 5, 6, 7, 8 and 9. Thus the largest four digit number is 9999. The smallet 5 digit number is 10000. Penny Go to Math Central
Answered by Penny Nom.
Wages for one month 2004-09-20
From Janet:
if offered $10,000 or 1 cent on the first day and then doubled daily for one month which would you take ?
Answered by Harley Weston.
B={A,{A}} 2004-09-20
From Muhammad:
Let A be a set and let B = {A,{A}}.

(a) Explain the elements of set B (with some example)

(b) Prove that A is not a subset of B.

Answered by Penny.
The hypotenuse of a right triangle 2004-09-20
From Shannon:
I am trying to find the hypotenuse of a right triangle with only the length of the opposite side. What is the formula as I don't have the length of the adjacent side? Can I compute it without knowing what the other two angles are?
Answered by Penny Nom.
Fabricating with pipe 2004-09-19
From Gil:
I need to fabricate a 10 section, 4" pipe circle with an inside diameter of 40". I would like to know what angles would apply and how to find them,
Answered by Penny Nom.
A binomial squared 2004-09-19
From John:
Can you explain the steps for me, that get you from (a - b)2 to a2 - 2ab + b2
Answered by Penny Nom.
Perimeter and area of a semi_circle 2004-09-18
From A student:
I want to know how to find the perimeter and area of a semi-circle and a quarter-circle.
Answered by Penny Nom.
Driving me crazy 2004-09-18
From Jerry:
This is driving me crazy!!! Someone told me of a number that can be divided EQUALLY by each of the following individual numbers: 1,2,3,4,5,6,7,8,9. what is that number?
Answered by Penny.
The price of a book 2004-09-18
From Dorly:
If the number of toonies required to buy a book is nine more than the number of five-dollar bills required to buy the same book, determine the cost of the book.
Answered by Penny.
A challenge 2004-09-17
From Lasse:
xy
xx
xyx
xxx
xyxx


"x" and "y" each represents a number. Find out the system to make the next line

Answered by Penny Nom.
Division with and without units 2004-09-17
From Kenneth:
What does it indicate when the divisor has units (dollars) but not the dividend, as in 500/$10.00 or 500 is divided by $10.00? Are the units understood to be there? Does 500/$10.00 make sense?
Answered by Penny Nom.
The length of a cut 2004-09-17
From Florita:
My daughhter, who is a 9th grader is attempting to cut a piece of wood after determining the length of the cut for the hypotenuse. These are the measures: a=4squared, b=6squared.

She determined that c should equal 52. But when she measured the actual piece to be cut, c measured 39.5 inches! Can you offer any insight as to what she is doing wrong? I have suggested that she may be working with an Acute rather than a Right angle . But she insists that it is a Right angle after using a "framing square".

Answered by Claude Tardif.
Pizza for Jack? 2004-09-16
From Grace:
Jack is playing pool with Jim for $1 a game. He has only $2 and decides to play until he goes broke or has $5, at which point he will quit and go out for a pizza with Jim(Dutch treat). Jack knows from past experience that he beats Jim 60% of the time. What is the probability that Jack will get to eat pizza? Hints: Let A be the 6x6 matrix defined by A=[aij], where aij is the probability that Jack will have $(i-1)after one game is he starts with $(j-1). For example, a23 - .40 since there is a 40% probability that Jack will end up with $1 after a game is he starts the game with $2 (If Jack wins 605 of the time, he must lose 40% of the time). Also, for example, a52 = 0 since there is no way jack can have $4 after one game if he had $1 at the beginning of the game. Since Jack will stop if he goes broke or accumulates $5, a11 and a66 are both 1.

Let x0 = [0 0 1 0 0 0 ] transposed, which we interpret as saying that initially Jack has $2 with a probability 1. Then Ax0 will represent the porbability of each amount of money, $0-$5, after one game. What is the probability that Jack will be able to eat pizza by computing Akx0 for large k and finding a limiting value.

Answered by Penny Nom.
Volume 2004-09-16
From Jonathan:
I have an area of 22ft x 43 ft, 6 inches high
Sq. ft..946
sq. yds...105.1

What is the cubic feet and cubic yards for this dimension.

Answered by Penny Nom.
Finding the height of a triangle 2004-09-14
From A student:
I have to calculate the height of a triangle. The base is 6 (units)say ab One of the top sides is 4 (units)say ca The other side is 2 (units) say cb I don't know the angles.

How to find the height?

Answered by Chris Fisher and Penny Nom.
Payment plus interest 2004-09-14
From Penny:
Brian wants to borrow some money from Margaret.
After three years he is to pay X back plus one interest sum of 25%. If he is pays back a total of $3800, what was the original sum borrowed?

Answered by Penny Nom.
A copper sphere 2004-09-12
From Irfan:
The radius of a copper sphere is 3cm. If a wire of 0.4 diameter is drawn by melting it,what is the length of this wire?
Answered by Penny Nom.
Three word problems 2004-09-12
From Adrian:

1. Increasing area of a field: Julia's soybean field is 3m longer than it is wide. To increase her production, she plans to increase her to increase both the length and width by 2m. If the new field is 46m2(2nd power) larger than the old field, then what are the dimensions of the old field?

2. Width of a Football Field: If the perimeter of a football field in the NFL including the end zones is 1040 ft and the field is 120 yd long, then what is the width of the field in feet?

3. Fencing dog pens Clint is constructing two adjacent rectangular dog pens. Each pen will be three times as long as it is wide, and the pens will share a common long side. If Clint has 65 ft of fencing, what are the dimensions of each pen?


Answered by Penny Nom.
A schedule for 6 teams 2004-09-10
From Greg:
I have a schedule I need to make of 6 teams. They must play each other once however no team can be the away team twice in a row. There must be 3 home games and 3 away games for each team.
Answered by Penny Nom.
Safe cracker 2004-09-09
From Adam:

Franklin's friend had taken an item from him, and put it in his family's safe, when franklin went to retrieve it he came to a combination lock on the safe, with the dial numbers going from 0 to 59. Unfortunately, he wasn't sure whether there were three or four numbers in the combination, or even which direction to turn the wheel

If it takes him 15 seconds to try a single combination, how many days will it take him to to try every possible combination? Please round to the nearest day.
Answered by Penny Nom.

A travelling salesman 2004-09-09
From Liz:
A salesman traveled due west from city A to city B. The distance he traveled, that is the ditance from A to B, was X miles. He returned from B to A and found that he had traveled half the distance, X/2 miles. How can that be?
Answered by Chris Fisher and Penny Nom.
Crossing a river 2004-09-09
From Barb:
Nine men and two boys, trekking through the jungle, need to cross a river. They have a small inflatable boat and it's easy enough to row it across the river. The boat, however, can hold no more than one man or the two boys. How can they all get across? (Hint. suppose there was only one man and two boys) Does it make math sense and what would the answer be
Answered by Penny Nom and Claude Tardif.
Half way between 2004-09-08
From Ben:
Find the number halfway between the number shown

751,843

Answered by Penny Nom.
The intersection of planes 2004-09-07
From Joshua:
I was wondering about the intersection of planes. Can planes intersect?
Answered by Penny Nom.
The bar over repeating digits. 2004-09-07
From Debbie:
I would like to know the name given to the bar that is written over the repeating digits of a decimal.
Answered by Penny Nom.
How to dry matches 2004-09-06
From Liz:
The neighbors of a young man who needed seclusion for a few days offered him the keys to their cabin in Maine. Arriving there on a damp and cold autumn evening he saw that while the cabin had normal running water and electric lights, for heat it had only a pot-bellied stove. Fortunately, there was plenty of wood, kindling, newspaper, and matches. Being an experienced hand at making a fire, he opened the damper, put into the stove crumpled newspaper, and properly stacked the kindling and logs. The matches, however, wouldn't light. They were too damp from the humidty. The nearest town was 30 miles away, and, anyway, it was late. What next?
Answered by Claude Tardif.
(2x+5y-6) + (3x-4y+12) 2004-09-04
From GG:
I am a grade 9 student and i do not understand a question. Plz help! OK here it is........ (2x+5y-6) + (3x-4y+12)
Answered by Penny Nom.
The distance formula 2004-09-04
From Joe:
Why do we use square roots and squares in the distance formula Wholdnt they cancel each other out?
Answered by Claude Tardif.
Nine minutes 2004-09-02
From A student:
You have two hour glasses-one measures 7 minutes and one measures 4 minutes.How can you time 9 minutes?
Answered by Penny Nom.
Cubic yards 2004-09-01
From Garnet:
I have the dimensions of 16 feet by 10 ft and 3 1/2 inches deep how much concrete do I need.
Answered by Penny Nom.
An isosceles trapezoid 2004-08-31
From Bruce:
An isosceles trapezoid with bases of lengths 12 and 16 is inscribed in a circle with a radius of 10. The center of the circle lies in the interior of the trapezoid. Find the area of the trapezoid.
Answered by Penny Nom.
Solve for z 2004-08-30
From Ana:
Solve for z when 4x + 10yz = 0

solve for z if y2 + 3yz - 8z - 4x = 0?

Do I have to solve all of the variables in order to get z?

Answered by Penny Nom.
A big number 2004-08-26
From Mark:
If you have $999,999,999,999,999,999,999,999,999,999.99 and you find a pennie how much is that and can you give me the name of it.
Answered by Penny Nom.
The area of a quadrilateral 2004-08-25
From Rich:
I would like to know how to measure the area (the formula) of a quadrilateral.
Answered by Penny Nom.
The radius of a circle 2004-08-24
From Peter:
If you slice any circle with a line, and call the distance of the line between intersections the "y" length and the perpendicular length to the shorter side of the curve the "x" length, what is the resulting equation for the radius?
Answered by Penny Nom.
The area of a lot 2004-08-22
From Cassie:
I'm in the process of purchasing some land and I'm trying to figure out the square footage of the property. Unfortunately, it's be a while since I had to remember this type of formula. The property is 140 ft on left side, across the back it is 220.61 ft, the right side is 167.52 ft, the frontage is 105 ft.
Answered by Penny Nom.
15 cubic yards of dirt 2004-08-22
From Barry:
My question...what is the equivalent of 15 cubic yards of dirt?
Answered by Penny Nom.
130 cc. is ? 2004-08-20
From Ayrn:
130 cc. is ? I have to put 130cc. of petroleum in a bucket how much exactly is that? Also 110 cc.? I have to also put 110cc. wood alcohol in a bucket aswell!
Answered by Penny Nom.
Golfing with Norm 2004-08-19
From Norm:
I am organizing a golf tournament with 6 American and 6 Canadian players. We play 3 rounds of golf. Is there any way to arrange that everyone plays without playing with the same person twice. Each foursome must consist of 2 Americans and 2 Canadians with one of each in a cart. Your help would be much appreciated.
Answered by Claude Tardif.
Golfing with Sally 2004-08-19
From Sally:
We are a group of 8 golfers (2 foursomes) and want to golf 3 games together. How can we arrange it so that we all get to golf with each other at least once?

At another venue, we have 18 golfers (3 foursomes, 2 threesomes) and want to golf 6 games together. Same question, how can we arrange it so that we all get to golf with each other at least once?

Answered by Claude Tardif.
A division symbol 2004-08-18
From William:
i was wondering what the mathematical name for this division sign (÷).
Answered by Penny Nom.
Area of a triangle 2004-08-17
From Barkie:
I'm going into tenth grade and I have a triangle who's sides measure 3ft, 4ft, and 5ft. I need to find the area however I don't know the height, or how to get it.
Answered by Chris Fisher.
Two water jugs 2004-08-16
From A student:
You go to a source of water with two jugs and only two jugs. One jug has a capacity of exactly three pints and the other five pints. Both jugs are opaque (you can`t see inside them) and irregularly shaped. You have no other water containers. You do not have a scale. How can you use the two jugs and only the two jugs to measure exactly four pints of water?
Answered by Penny Nom.
Percentiles 2004-08-15
From Gary:
Table 1 Selected percentiles for family income in the US in 1992 1 $1,300 10 $10,200 25 $20,100 50 $36,800 75 $58,100 90 $85,000 99 $151,800

Q. The percentage of families in Table 1 with incomes below $58,100 was about?

Answered by Penny Nom.
A probability question 2004-08-15
From Gary:
In a law school class, the entering students averaged about 160 on the last LSAT; the standard deviation was about 8. The historgram of the lLSAT scores follwed the normal curve reasonable well.

Q. About what percentage of the class scored below 166?

Q. One student was 0.5 above average on the last LSAT, about what percentage of the students had lower scores than he did?

Answered by Penny Nom.
Cubic yards of dirt 2004-08-11
From A parent:
How many cubic yards of dirt would it take to fill an area that is 325 feet by 325 feet and is 3 feet deep?
Answered by Penny Nom.
Acres and miles 2004-08-11
From A student:
How many acres are in 1 mile?
Answered by Penny Nom.
A conic 2004-08-10
From A student:
My question is about this equation 32x*2 -18y*2 -64x +72y +248 =0 explain why as x goes to the infinity the graph of the conic looks like y= (3/4)X.

What I did to try to find the solution of this problem was to graph and then I thought that maybe trying to find the equation of the asymptotes I could do it, but it was useless -because the equation of the asymptotes is y= +4/3(X-1) -2 and that does not explain why tho conic looks like y=(3/4) x . I would really appreciate your help on this.

Answered by Penny Nom.
Constructing a triangle 2004-08-08
From KV:
HOW TO CONSTRUCT A TRIANGLE ABC, WHEN ITS ALTITUDES AD,BE,CF ARE GIVEN
Answered by Chris Fisher.
A big number 2004-08-08
From Carter:
Can you tell me what number this is?
10,000,000,000,000,000,000,000,000,000,000,000,000

Where can I find the names for big numbers?

Answered by Penny Nom.
Limits and composite functions 2004-08-07
From Sue:
I have two questions, one about a limit and the other about a composite function. If you could help me, I'd really appreciate it.

1. Find the limit:

lim[x->0] (x*csc(x))

I converted csc(x) to cos(x)/sin(x), but I didn't know what to do after that.

2. f(g(x)) = ln(x^2 + 4), f(x) = ln(x^2) and g(x) > 0 for all real x, find g(x):

I'm having trouble with this one because x^2 + 4 isn't a perfect square.

--Sue

Answered by Penny Nom.
The integrating factor method 2004-08-05
From A student:
Whilst using the integrating factor method, I am required to integrate a function multipled by another function.

say f(t) = exp(kt) and some other function g(t); where exp = exponential and k is some constant.

Integral f(t)*g(t) dt or Integral exp(kt)*g(t) dt

What would the result of this integral be? I have never met an integral like this before. Would it simply be exp(kt)*g(t)/k? More specifically, the problem and my attempted answer is in PDF format:

In my attempted solution, I am unsure about the last two lines I have written out, as it relates to integrating a function multipled by another function.

Answered by Harley Weston.
Integrating e^sin(x) 2004-08-04
From A student:
I need to know that how to solve the integral " e^sin x",
Answered by Harley Weston.
Differentiation 2004-08-04
From A parent:
I am a parent trying to understand higher level of maths and would be very grateful if you could help with differentiating the following functions, identifying general rules of calculus:

a) f(x)=e^2^xIn(cos(8x))

b) f(x)=secx/SQRTx^4+1

Answered by Penny Nom.
When is the limit of f(x) undefined? 2004-08-03
From Nicolasa:
When is the limit of f(x) undefined?
Answered by Penny Nom.
an integer with three factors 2004-08-03
From A student:
What is the probability that a randomly chosen 3 digit number has exactly 3 factors
Answered by Penny Nom.
sin^2x-5sinx-3=0 2004-08-02
From Kerri:
hey my name is kerri i'm a college students and i was wondering if you could help in with this problems

sin^2x-5sinx-3=0

Answered by Penny Nom.
A tangent to a hyperbola 2004-08-02
From A student:
The equation of a hyperbola is 32x*2 -18Y*2-64x +72y +248=0. The equation of a tangent line to this hyperbola is y= (16/15)X + 10/3 I have been trying to find the point where this line intersects the graph. What I did was solve for x and then plugged in the result into the equation of the hyperbola, but I am getting two answers and I am supposed to get only one because the line is tangent to the graph. For this reason, I would like to know what I am doing wrong or what I have to do to know which answer is correct.
Answered by Penny Nom.
Two problems 2004-08-02
From JJ:
A question on my homework assignment about logs says "suppose that x=logA and y=logB & write the expression in terms of x and y"

log(A - B)

Is this even possible? It seems like a typo.

Another is: "solve for t."

12(1.221)^t = t + 3

Is this one possible? If it is, I cant figure it out.

Answered by Penny Nom.
The law of sines 2004-08-01
From Joy:
How do you solve this? Do you solve this triangle using the law of sines of the law of cosines? (ASA)

A=120DEG. B=40DEG c=35 cm I keep getting different answers.

Answered by Penny Nom.
Combien de litre y a t'il dans un M3 2004-07-30
From Un eleve:
j'aimerais savoir combien de litre y a t'il dans un M3. car je dois mettre un produit spécial dans une piscine.
Answered by Claude Tardif.
A parallelogram 2004-07-30
From Keith:
I'm a high school student and have been having trouble with this problem , which is based on one pair of parallel sides of a parallelogram: If AD = 6x-5 and BC = 3x+7 find x. Of course both sides have to be equal. Now I know the value of x is 4 So my question is how do you get 4 from the given information.
Answered by Penny Nom.
Extraneous solutions 2004-07-28
From Nicole:
When I have a problem like (2x + 3)/5 = (x + 1)/6 and the question asks to check for extraneous solutions, how do you solve that?
Answered by Penny Nom.
Water in a cone 2004-07-28
From A student:
A vertically inverted cone( i.e. vertex down) has a radius 7 inches and height 24 inches. Water is filled to one third of its height .Find the ht of water when cone is turned upside down
Answered by Penny Nom.
The third side of a triangle 2004-07-28
From Annette:
How do you find the length of a triangle side if you know two sides?
Answered by Penny Nom.
x^2 = x + 2 2004-07-26
From A parent:
x2 = x+2

My daughter came home with this equation today and although I know the answer is x=2, i have no way of proving it by showing any working out.

Answered by Penny Nom.
The railing around a pool 2004-07-26
From Bob:
I have a 15' circular above ground pool. Around the perimeter of the pool are eleven (11) sections of railing. Each rail has 5 slots at each end for pins. I have calculated that the length of the arc under the railing to be 51.4". what I am trying to determine is the distance between the end points of the arc so that I can figure out which slot to use in the rails without going round and round the pool moving and removing the rails until they finally fit. Been there, done that, no fun.
Answered by Penny Nom.
The p-value 2004-07-25
From Kathy:
Ms. Lisa Monnin is the budget director for the New Process Company. She would like to compare the daily travel expenses for the sales staff and the audit staff. She collected the following sample information.

At the .10 significance level, can she conclude that the mean daily expenses are greater for the sales staff than the audit staff? What is the p-value?

Having problems finding the p-value & unsure of the formula. Kathy

Answered by Penny Nom.
Row echelon form 2004-07-24
From Michael:
My name is Michael and I'm in the 11th grade. I have a Math question that I can't solve. The problem is system of equations that I need to do in augmented matrix form, find the row echelon form, and solve it by using back substitution.

2x + 3y + 7z =13
3x + 2y - 5z = -22
5x+ 7y - 3z = -28

Answered by Penny Nom.
The number of blocks in a mile 2004-07-18
From Valerie:
I don't know if you can help me but I was wondering how many blocks are in a mile in Ocean City, New Jersey?
Answered by Penny Nom.
Solving some quadratic equations 2004-07-15
From A student:
Could you please solve the following for x

x2+6=5x
x2+16=8x
2x+21=3x2

Answered by Penny Nom.
Two vector problems 2004-07-14
From Jaye:
Two force, 120lb and 200lb, act on a body and make a 52 degree angle w/ ea. other. What's the magnitude of the resultant of the forces and what is the measure, to the nearest degree, of the angle that it makes with the 200lb force?
Answered by Penny Nom.
12(1.221)^t = t + 3 2004-07-14
From Jay:
This problem has me stumped since i cannot separate the t and the 3. Here it is:

12(1.221)t = t + 3

It's on a logarithm worksheet, but how do i figure it out?!

Answered by Harley Weston.
Two puzzles 2004-07-13
From Fred:
A young man's car developed a flat tire while he was driving along a deserted street. He pulled over to the curb and did all the usual things; removed the hub cap; unscrewed the lugs and rested them carefully in the hub cap, jacked up the car. As he was putting the spare tire onto the axle he accidentally kicked the hub cap. The lugs rolled out, and all five of them rolled down a nearby grate. Peering through the bars of the grate the man thought he could see the lugs about 6 feet below in a shallow water puddle. He had a problem, how do you think he solved it?

It is noon, your lunch hour, but you can not go out because there is a terrific hailstorm. Turning on your radio you hear the weathercaster predict that the hail will change to rain and that it will pour all day today. How can you determine the sun will be shining in 36 hours? Justify your answer.

Answered by Penny Nom.
An Octagonal playhouse 2004-07-13
From Levi:
I'm building an octagon playhouse for my son that is 8 feet wide. what would be the measurements of each of the eight sides.
Answered by Harley Weston.
x = 2^x 2004-07-12
From A student:
If the graphs of y=x and y=2x are drawn on the same set of axes, they will intersect when x is equal to what?

(1)1
(2)2
(3)0
(4)They won't intersect.

I know the answer is number 4 by graphing which is permitted; however, I would like to know if there is any algebraic way to do this type of problem.If yes, please show me how.Thanks.

Answered by Penny Nom.
The tangent of theta 2004-07-10
From Jacob:
P is a point on a unit circle with coordinates(0.6,0.8). Find tan of theta. My book shows me how to do it,"tan of theta=opp./adj.=0.8/0.6=4/3,"and leaves it as that's the answer(4/3).When do we know from a problem to find the angle measure (in this case, the angle measure of theta) and how do we know when to give something like 4/3 without converting it to the angle measure?
Answered by Penny Nom.
The circle through three points 2004-07-06
From Jim:
I am a student trying to solve math problem. I'd like to calculate the radius of the circle that exactly fits any three points. If the points are (X1,Y1), (X2,Y2), and (X3,Y3), what is the radius of the circle that contains those three points?
Answered by Penny Nom.
An ineqlality with absolute values 2004-07-05
From Uneeza:
abs(x2 - 4x) > 3
Answered by Claude Tardif.
Factoring integers 2004-07-02
From A student:
After looking at all the info I could get about NFS, I still have some questions that are unsolved:

First of all: If someone found an algorithm that has a worst case running time of N*Log(N) to factor an integer n into his divisors, would it be quicker or slower then the number field sieve algorithm?

secondly, what exactly is the time complexity of the Number Field Sieve algorithm, if I would factor an integer n?

Answered by Claude Tardif.
Choosing 5 numbers from 1,...,36 2004-06-30
From A lottery player?:
how many combinations are there for the numbers 1thru 36 when only using each number once in groups of five
Answered by Penny Nom.
Simplify 2004-06-27
From Tiff:
Simplify:

-3x (x2 + 2)

(x + 2) (x + 3)

Answered by Penny Nom.
Aunt Lucy's letter 2004-06-27
From Olivia:
"Dear bob, now that i am getting on (70 today). i want to give you some of my money, i shall give you a sum each year, starting now. you can choose which of the following schemes you would like me to use.

a) £100 now, £90 next year,£80 the yaer after and so on.
b) £10 now, £20 next year, £ 30 the next year and so on.
c) £10 now, 1.5 times as much next year, 1.5 times as much the year after that and so on.
d)£1 now, £2 next year, £ 4 the year after, £8 the year after and so on.

of course these schemes will only operate while i am alive. i look forward to hearing which scheme you choose and why! Best wishes, Aunt Lucy."

Answered by Penny Nom and Claude Tardif.
Undercapatalized small businesses 2004-06-26
From Greg:
Suppose that 30% of all small businesses are undercapitalized. 40% of all undercapitalized small businesses fail and 20% of all small businesses that are not undercapitalized fail. A small business is chosen at random. The probability that the small business succeeds if it is undercapitalized is? Greg
Answered by Penny Nom.
Pick a number greater than 1 2004-06-25
From A student:
I understand that when you pick a number greater than 1 and less than 10; multiply it by 7 and add 23, then add the digits of that number until you get a one digit number. Then multiply that number by 9, add the digits of that number until you get a one digit number, subtract 3 from that number and divide the difference by 3; that this process will always give you the result of 2. Does this have a name or theory for it as to why the answer will always be 2?
Answered by Penny Nom.
(a+b) + 5i = 9 + ai 2004-06-25
From Josh:
The question which someone gave you (a+b) + 5i = 9 + ai question) gave me trouble.
Answered by Penny Nom.
(ln(50/x)) / (ln(40/x)) = a 2004-06-21
From Un eleve:
(ln(50/x)) / (ln(40/x)) = a = constante

comment extraire la variable x ???

Answered by Claude Tardif.
How much loam to fill a hole? 2004-06-19
From Harry:
Can you tell me how much loam I would need to fill a hole 15 feet in dia. x 6 inches deep.
Answered by Penny Nom.
Forty balls are placed in a bag 2004-06-19
From Sheila:
Forty balls numbered 1-10 are placed in a bag and four are drawn at random. What is the probability that the first ball drawn is between 1-10 (inclusive) the second is between 11-20, the third is between 21-30 and the last is between 31-40?
Answered by Penny Nom.
3+4i abd |3+4i| 2004-06-17
From Sandy:
how would you do a question like |3+4i|? is that different than just doing 3+4i?
Answered by Penny Nom.
An equation with rational terms 2004-06-17
From Louise:
simplify the expression (a2+n2)/2n = a2 + (a2-n2)/2n
Answered by Penny Nom.
Area of a lot 2004-06-17
From Mike:
Could you please figure out the square footage and acreage of my lot? One side is 228 feet, another is 150 feet, another is 80 feet, and the last side is 164 feet. My house is in the middle and it is 62 feet by 37 feet. Is that subtracted from the total? I would like to know this when I have to buy fertilizer.
Answered by Penny Nom.
A garden space 2004-06-16
From Carol:
We have a 17' x 19' garden space. If we want it a foot high raised bed, how many cubic yards of topsoil do we need?
Answered by Penny Nom.
The height of a building 2004-06-16
From Lauren:
I have a homework assignment to measure a building on my school's campus. I have to stand at least 40 ft away from the building and find the angle of elevation from my feet to the top of the building. Then I have to walk at least 40 ft form that point, record the distance as X, and find the angle of elevation from my feet to the top of the building. Using X and the angles, I have to determine the height of the building. I used a protractor and a weighted string to find the angle of elevation from my eyes to the top of the building. But I haven't been able to find the way to measure the angle of elevation from my feet to the top.
Answered by Penny Nom.
Solving an equation 2004-06-14
From Anthony:
I was hoping that you could help me with this problem. I believe that this is a high school level problem but as an adult who has forgotten the "rules", I am stuck.

the equation is this: x=t2/(t1-t2)

we know that x=5.73
t1=303

we must solve for t2

Answered by Penny Nom.
The Number of the Beast 2004-06-13
From A Heinlein reader:
Most people who have an interest in mathmatics are familiar with Robert Heinlein's novel "The Number of the Beast", where 666, or ((6 to the 6th power) to the 6th power) equals 1.0315 times 10 to the 28th power, which in the novel is the number of parallel universes in the cosmos. My question is what would the number of parallel universes be if you grouped 666 the other way: (6 to the power of (6 to 6th power)). I have tried this on my calculator, but it won"t register that high.
Answered by Penny Nom.
Integers 2004-06-10
From A parent:
I have an odd 3 digit product. My factors are a 1-digit number and either or 2- or 3-digit number. My factors are odd, but neither is 1. Find my factors and my product.
There is one answer. What is it?

Answered by Penny Nom.
Jack's social security number 2004-06-08
From A student:
Jack's social security number contains each of the nonzero digits exactly once. By examining the digits from left to right, he also found that 1 divides the first digit evenly, 2 divides the sum of the first two digirs evenly, 3 divides the sum of the first three digits evenly, and so on, until 9 divides the sum of all the digits evenly. What is Jack's social security number?
Answered by Penny Nom.
What is the name of a 3D rectangle? 2004-06-07
From Jack:
if a cube is a 3D square, what is the name of a 3D rectangle?
Answered by Penny Nom.
The area of a pentagon 2004-06-06
From Dana:
HELLO, I NEED HELP IN DETERMINING THE SIZE OF A PENTAGON.

one side is 280',from both ends of the 280 'section,we have 300' lengths,from one of these 300' sections we have a 118' length,which goes over to a length of 228',will this be enough info. or should i provide more. thanks

Answered by Penny Nom.
Cubic yards in a pool 2004-05-30
From Jerry:
HAVE A AREA WHERE I HAD A 18 FOOT ROUND POOL.
THE DEEPEST PART IT WAS IN THE GROUND WAS AROUND 7 INCHES TAPERING DOWN TO AROUND 4 TO FIVE INCHES AND A LITTLE OF THE ONE SIDE ALMOST AT GROUND LEVEL...HOW MANY YARDS OF DIRT WOULD I NEED TO FILL IT IN???? IS THERE A FORMULA FOR THAT.......COULD YOU FIGURE THAT FOR ME?

Answered by Penny Mon.
The Parliament of the land of Achronia 2004-05-29
From Christine:
The Parliament of the land of Achronia consists of two houses. The parliament was elected in 2003 for a period of six years beginning on Thursday, the 1st of January 2004, when the two houses had their first sessions. According to the rules, the meetings of the first house must occur every twelve days for the duration of the term, and the meetings of the second house must occur every eighteen days. For example, the second meetings of the first and the second houses were held on the 13th and 19th of January respectively. A new law can be passed on any day when both houses meet, except on a Thursday. On how many days can the parliament members pass new laws during this six year term?
Answered by Penny Nom.
112! 2004-05-28
From Beatriz:
1) A bus driver collects identical sets of 5 coins from each passenger. If the totoal colledted was $21.83, how many pennies did the driver get??

2) How many terminal zero in the base 10 expression of 112! (factional) N! means N(N-1)(N-2) .....(2) (1).

Answered by Penny Nom.
English 2004-05-27
From Chang:
well my question is i was told by most of the teachers that ''think in english'' is the best way to learn english but how can i do like that do i need to know about the stucture first or do i need to learn the western culture ?

here is a sentence i have translate directly from my mother language.'' i was given an admiration of sincerity from this client on the follow up action i have done in the complaint.

Answered by Claude Tardif.
The horizontal fraction bar 2004-05-27
From Kashia:
What is the name of the bar that separates a numerator and that denominator?
Answered by Penny Nom.
The center of a circle 2004-05-26
From Wan:
I am trying to find the radius of an arc. The only things i know about the arc is all referenced from the line of tangency to the arc. on both sides i have a differnt horizontal perpendicular distance to the point of tangency. left side |-------*---------------| right side (*=point of tangency). Then i have 2 difference vertical perpendicular distance of the end points to the line of tangency. I know it sounds very bad in text but this is all i know about the arc. Can you help me find the radius?
Answered by Penny Nom.
The base of a triangle 2004-05-25
From Ralph:
With a 30 degree angle at the top of a triangle and a height of 15 what is the base?
Answered by Penny Nom.
Programming without trig functions 2004-05-25
From Derek:
I am a programmer trying to calculate the following.

What is the formula to find the cross-sectional area of a cylinder with out using any trig functions? or better yet, how can you calculate any given volume in a cylindrical tank with spherical heads with out trig functions?

I am using a PLC (programmable logic controller) to do this and trig functions are not available.

Answered by Harley Weston.
Finding bearings 2004-05-24
From James:
This question is about finding bearings. A boat race starts from point A, goes North to Point B, a distance of 1000 meters. The course is triangular. The bearing from point B to point C is South 70degrees West. The distance from Point B to point C is 1500 meters. Find the course bearing from C to A.
Answered by Penny Nom.
Some trig expressions 2004-05-23
From A student:
Prove:

sin A + sin B = 2sin(A+B/2)cos(A-B/2)

cos A - cos B = -2sin(A+B/2)sin(A-B/2)

cos A + cos B = 2cos(A+B/2)cos(A-B/2)

sin A - sin B = 2cos(A+B/2)sin(A+B/2)

Answered by Penny Nom.
Volume of a box 2004-05-23
From A student:
The area of the top of a closed rectangular box is 252cm2, the area of the front of the box is 105cm2, and the surface of the box is 834in2 (or 5380.64cm2). What is the volume of the box?
Answered by Penny Nom.
33 3/4 doubled 2004-05-22
From A tailor:
need to know how to add 33 3/4 doubled. How do I come up with the solution? I need to know for measuring garments. Usually I measure on the half meaning on the front of the garment and then double it with the measuring tape but I always get stuck when I need to measure 3/4 such as 66 3/4 doubled and etc. Can you help me figure this out easily.
Answered by Penny Nom.
The perimeter of a triangle 2004-05-21
From A student:
In a triangle i have the length of a line and it's opposite angle.how can i calculate perimeter?(the angles are not right)
Answered by Penny Nom.
Word problems 2004-05-19
From A student:
why can't 2nd graders have a easy time doing word problems and drawing pictures with it
Answered by Diane Hanson.
The dimsenions of a room 2004-05-18
From Estella:
I have a room that is rectangular the area is 159.5 feet, the perimeter is 51 feet what would the dimensions of the room.
Answered by Penny Nom.
Angles of elevation and depression 2004-05-18
From Anjum:
what is the difference between an angle of elevation and angle of depression?
Answered by Penny Nom.
The combinatiion for a keybox 2004-05-17
From Darrin:
I have a puzzle question based on reality. I have a real-estate type keybox (Supra) that has 10 digits, 0-9. I am trying to figure out a methodical way to test all number combinations to it to open it. (totally legal- it was taken from the trash at a jobsite since they couldn't open it!) The combination could be any or all of the digits, but any digit is only used once. This is not like a simplex lock problem where a combination of keypresses would be different- i.e. 1,2,3 is the same as 1+2,3. Any ideas?
Answered by Penny Nom.
Diagonals in a regular dodecahedron 2004-05-15
From Joshua:
A dodecahedron has twelve faces, all of which are regular pentagons. Three edges meet at each vertex of the dodecahedron. An interior diagonal is a segment connecting two vertices such that the segment is not an edge or along a face of the dodecagedron. How many interior diagonals does a regular dodecahedron have?
Answered by Penny Nom.
Maximizing the angle to the goal mouth 2004-05-15
From Yogendra:
You are running down the boundary line dribbling the ball in soccer or hockey. Investigate where in your run the angle the goal mouth makes with your position is at a maximum.
Answered by Penny Nom.
Subtracting fractions 2004-05-11
From Filipe:

Question:
_5_ - __7__
6ab 8a


Answered by Penny Nom.
Area of a lot 2004-05-11
From Dana:
HELLO MY NAME IS DANA ,I AM AN AVERAGE JOE WITH A QUESTION THAT I CANNOT ANSWER MYSELF SO I CAME TO YOU, I HAVE A PIECE OF PROPERTY IN THE SHAPE OF AN ISOSCELES TRAPEZOID THAT IS 250 FEET ACROSS THE BACK, 100 FEET ACROSS THE FRONT AND BOTH SIDES ARE 600 FEET LONG. HOW MANY SQUARE FEET ARE IN THIS PROPERTY.?
Answered by Penny Nom.
Three dice 2004-05-10
From A student:
If one has 3, 6 sided dice what is the probability of the numbers that are rolled to total 4 through 10 inclusively?

Subsequent to this, what is the probability to do this consecutively...say 3 times?

Answered by Peny Nom.
Pi 2004-05-10
From Kim:
I'm a 7th grader at Lakeside Middle School. And I was wondering if you can answer a question for me for my math project. I'm doing a project on pi, and my teacher said to ask an expert like you to ask about how you would use the subject, pi, in real life usage. How would you use pi in real life usage?
Answered by Penny Nom.
Why the word combinations? 2004-05-05
From Megan:
Why are locker combinations called locker combinations not locker permutations. The order of the numbers matter so why aren't they called locker permutations.
Answered by Claude Tardif and Penny Nom.
Chi-Squared 2004-05-05
From Gillian:
OF75 PEOPLE, 43 ARE EXPOSED TO A VIRUS AND BECOME ILL. 11 PEOPLE ARE EXPOSED AND DO NOT BECOME ILL. 3 PEOPLE ARE NOT EXPOSED AND BECOME ILL. 18 PEOPLE ARE NOT EXPOSED AND DO NOT BECOME ILL. THE ATTACK RATE IS 80% FOR THOSE EXPOSED AND 14% FOR THOSE NOT EXPOSED. THE RELATIVE RISK IS 5.7. CAN YOU EXPLAIN TO ME HOW TO CALCULATE A CHI SQUARE TEST FROM THIS.
Answered by Penny Nom.
3x squared - 27 / x + 3 2004-05-04
From Stef:
3x squared - 27 / x + 3
Answered by Penny Nom.
A 3 digit number divisible by 7 2004-05-03
From A student:
We need to arrange 1,3 and 6 to form a 3 digit number that is divisible by 7.
Answered by Penny Nom.
Solving inequalities 2004-05-03
From Joe:
Let x be a real number find and graph the solution to the following sentances:

2x-10+9 4X+2<15 5X+3>(x-1)

Answered by Peny Nom.
A clock chimes 7 times in 7seconds 2004-05-02
From Anna:
A clock chimes 7 times in 7seconds. how long does it take the clock to chime 10 times?
Answered by Penny Nom.
The equation of a line 2004-04-30
From Amy:
Write the equation of the line that contains the point (-4,6) and is perpendicular to the line y=5x+100.
Answered by Penny Nom.
Setting up an equation 2004-04-30
From Matt:
I am trying to determine the dimensions of a 22 megapixel digital camera chip that has a 3:2 ratio. We know then, that: 3x = 2y (or 1.5x = y)

and that

xy = 22

Beyond this, I don't know how to hook those 2 seemingly unrelated facts together.

Answered by Penny Nom.
Yards in a concrete driveway 2004-04-27
From Bobbi:
I am wondering how many yards of cement I need for a 49 by 45 area, 5 inches thick. I am a stay at home mom who is not very good at math and I want to pave my drive way. Also what would the yardage be for a 6 inch thick drive way with the same area.
Answered by Penny Nom.
An oil-drilling platform 2004-04-27
From Alie:
An oil-drilling platform is located in the Gulf of Mexico 3.25 miles from the nearest point on shore. From a point B on the shore due east of A the bearing of the platform is S51.2W. How far is it from B to the platform?
Answered by Penny Nom.
The height of a triangle 2004-04-27
From Danielle:
what is the definition of height of a triangle?
Answered by Penny Nom.
Related rates and baseball 2004-04-26
From Bethany:
A baseball diamond is the shape of a square with sides 90 feet long. A player running from second to third base at a speed of 28 feet/ second is 30 feet from second base. At what rate is the player's distance from home plate changing?
Answered by Penny Nom.
The problem of Apollonius 2004-04-25
From Mitja:
There are given 2 circles lying one out of another and one point out of both circles. How to construct a circle passing through a given point and internally tangent to one and externally tangent to the other cirlce?
Answered by Chris Fisher.
A Fibonacci triangle 2004-04-25
From Marcelle:
Is it possible to construct a triangle with sides that are three consecutive Fibonacci numbers?
Answered by Walter Whiteley.
Water in a pipe 2004-04-25
From Marlean:
How much water would be in a 480 foot one inch inside diameter pipe?? In imperial gallons please. thanking you in advance for your help in this matter.
Answered by Penny Nom.
Place Value 2004-04-25
From Santosh:
Q. Write the Place value of the digits in the following numerals.

1. 5 in 54
2. 3 in 63

Answered by Penny Nom.
Marching legion 2004-04-24
From Art:
A column of soldiers is 100 meters long. Their sargeant at the rear of the column gives the order to march. The sargeant marches alongside the column to its head and then back to the rear, at which point he gives the order to halt. In all, the column of soldiers has marched 100 meters. How far has the sargeant marched? I say 187.5 meters, but those who teach or who have taught mathematics tell me no. Some say much more and some different. What do you say?
Answered by Penny Nom.
Three parabolas 2004-04-24
From A tutor:

I am a maths tutor. One of my year 12 students has given me this assignment. Many parts are ambiguous. I am trying to determine the equation of the line segment AB. Given that A has an incline of 10 degrees below the horizontal, I am assuming that this parabola (although you can only see part of it) has been rotated 10 degree clockwise. Am I right in assuming this? If so, how do I derive the equation of the parabola as it will then not fit the general form y = ax squared +bx +c of a parabola.

Also, how is one expected to find the x coordinate of D without the equation of this parabola. To find the equation you need the x-coordinate and therefore be able to find another point on the parabola in order to derive the equation using simultaneous equations.


Answered by Penny Nom.
square root 125 x 16 2004-04-23
From Mike:
how do you simplify a square root of "- square root 125 x16" when x is a variable... i don't really get how to do this....
Answered by Penny Nom.
The "22" puzzle and the "1089" puzzle 2004-04-22
From Marcelle:

1. Choose 3 digits from 1-9
2. Make all the 2 digit numbers you can from these (6)
3. Add the 3 original digits and divide them into the sum from step 2.
The answer is always '22'. I just can't understand why. Can you please help.

Another one related to this is it:
1. Choose a three digit number ensuring the first and third digit are differnt by at least two.
2. Make the reverse three digit number and subtract the smaller one from the larger of these.
3. Take this answer and reverse it and add these two 3 digit numbers .

eg:
643 - 346 = 297
297 + 792 = 1089

it doesn't matter what numbers are used, the results are alwasy the same. eg 22 or 1089


Answered by Paul Betts.
Surface area of a cylinder 2004-04-17
From Kathryn:
How do you find the radius of a cylinder when you are given only the surface area and the height of the cylinder?
Answered by Penny Nom.
The line through (4,2) and (4,-8) 2004-04-17
From Kim:
Write an equation for the line.
Given points (4, -8) and (4,2) are on the line.
I believe m=undefined.

How do I solve for b when m is undefined, and then plug in the values for m, b, and y?

Answered by Penny Nom.
A probability density function 2004-04-14
From A student:
The label on a bottle of liquid detergent shows contents to be 12 ounces per bottle. The production operation fills the bottle uniformly according to the following probability density function:

f(x) = 8 for 11.975 <= x <= 12.10
and
f(x) = 0 elsewhere

a. What is the probability that a bottle will be filled with 12.02 or more ounces? b. What is the probability that a bottle will be filled between 12 and 12.05 ounces? c. Quality control accepts production that is within .002 ounces of number of ounces shown on the container label. What is the probability that a bottle of this liquid detergent will fail to meet the quality control standard?

Answered by Penny Nom.
Circles in a hexagon 2004-04-11
From Crystal:
step by step can you show me how to calculate the area of the region inside the hexagon but outside the seven circles. given the radius of each circle is one inch
Answered by Penny Nom.
Rational expressions 2004-04-06
From Shivani:
I am in 11th grade Honors Algebra II. We are currently learning about adding and subtracting rational expressions. I looked at the other questions that were similar to that of mine, but they weren't quite answering my question. I have been working on many problems to practice and see if I can come up with the correct answer, but I can't, I got like 2 out of every 7 right on my school homework. I get the easy ones, where when you have to find the LCD, it's just the other denominator. But, on others you have to really think about it. Those are hard. Here are some of the problems that I cannot get at all: (x2 - 3x + 1)/(x2 - 4) - (x2 + 2x +4)/(2 - x) - (x - 4) /(x - 2)

The correct answer is: (x3 + 4x2 + 7x + 17)/(x2 - 4)

Answered by Penny Nom.
The sum of angles in a triangle 2004-04-06
From A student:
How can u prove sum of angles in a triangle equal to 180 degrees?
Answered by Penny Nom.
Trisecting an arbitrary angle 2004-04-06
From Joe:
Where can I submit my effort on trisecting an arbitrary angle with only a straightedge and a compass? I can do it but I do not have the smarts to prove it.S
Answered by Chris Fisher.
x^2/3 - 7 x^1/3 + 12 = 0 2004-04-05
From Jackie:
I am having trouble solving this question for x:

1.) x^2/3 - 7 x^1/3 + 12 = 0

Answered by Penny Nom.
A changing rectangle 2004-04-03
From A student:
The width x of a rectangle is decreasing at 3 cm/s, and its length y is increasing at 5 cm/s. At what rate is its area A changing when x=10 and y=15?
Answered by Penny Nom.
Ratio and proportion 2004-04-02
From Kenneth:
The terms of a ratio in a proportion are often expressed as a is to b as c is to d.

Example: 2/4 = 6/12 this proportion represents that 2 is to 4 as 6 is to 12.

What does the "a is to b as c is to d" really represent or indicate in ratios?

Answered by Claude Tardif.
Some calculus problems 2004-04-01
From Weisu:

I have questions about three word problems and one
regular problem, all dealing with derivatives.

  1. Find all points on xy=exy where the tangent line
    is horizontal.
  2. The width x of a rectangle is decreasing at 3 cm/s,
    and its length y is increasing at 5 cm/s. At what rate
    is its area A changing when x=10 and y=15?
  3. A car and a truck leave the same intersection, the
    truck heading north at 60 mph and the car heading west
    at 55 mph. At what rate is the distance between the
    car and the truck changing when the car and the truck
    are 30 miles and 40 miles from the intersection,
    respectively?
  4. The production P of a company satisfies the
    equation P=x2 + 0.1xy + y2, where x and y are
    the inputs. At a certain period x=10 units and y=8
    units. Estimate the change in y that should be made to
    set up a decrease of 0.5 in the input x so that the
    production remains the same.

If you could just give me some hints on these
questions, I'd really appreciate it. Thanks!


Answered by Penny Nom.
The intersection of two graphs 2004-03-31
From Benjanim:
How do I find the points of intersection of the two functions:

1) y = 2 - (e^-x)
2) y = 1 + (x^2)

I know that I have to set them equal to each other, but I can't solve for x. Please help :-)

Answered by Penny Nom.
Algebraic simplification 2004-03-31
From Chris:
Simplifying Exponents

Question

b^5d^2/b^3d^8

Answered by Penny Nom.
Replacement times for TV sets? 2004-03-31
From Barb:
Replacement times for TV sets are normally distributed with a mean of 8.2 years and a standard deviation of 1.1 years. Estimate the probability that for 250 randomly selected TV sets, at least 15 of them have replacement times greater than 10.0 years.e
Answered by Andrei Volodin and Penny Nom.
n! > n^2 2004-03-30
From Jose:
How can you prove by mathematical induction that:

n! > n2.

Answered by Penny Nom.
A stained glass window 2004-03-29
From Kay:
I'm doing a stain glass project and it's on a 4 foot across octagonal window...and I'm trying to set up the pattern and I don't know how long the sides are!
Answered by Penny Nom.
Numbers around a circle 2004-03-28
From Rebecca:
my maths question is use the numbers 1,2,3,4,5,6 and 7 place each number in a circle so each line adds up to 12. There are seven circles, six on the outside and one in the middle. Each number lines up with the middle number and the outside numbers line up with the one directly across from it as if a line was going through the middle number circle.
Answered by Penny Nom.
Maximizing the area 2004-03-27
From Petey:
Please could you tell me why for my coursework (where I have to find the largest area that a fence 1000m long can cover) why I should only test equilateral and isoceles triangles? We were told NOT to do right angled triangles but I was wondering why not?
Answered by Penny Nom.
The volume of a sphere 2004-03-26
From Mike:
I was in class today and I learned how to calculate the volume of a sphere. I understood it easily but can't find the logic behind it. So here are my questions, in the formula why is the radius cubic and not squared? And what does 4/3 have to do with it?
Answered by Penny Nom.
z^2 = 3 - 4i 2004-03-26
From John:
Solve: Z^2 = 3 - 4i
Answered by Harley Weston.
Large exponents 2004-03-26
From John:
I have encountered problem with the lack of memory every calculator seems to have. No calculator, on or off the computer, I've found has the amount of memory or writing space to calculate the sums I want to solve. The sums are in great importance for my continued progress. The sums are following:

16777216^1310270, 16777215^995328, 16777215^786432 and 16777215^480000

Answered by Claude Tardif.
Four marbles on a box 2004-03-22
From Karyn:
Suppose 2 solid color marbles and 2 striped marbles are placed in a box. All are the same size. If one marble is randomly drawn from the box and replaced, then a second marble is randomly drawn, what is the probability that the marble drawn both times will be striped?

I know there is a simple formula for working this out but I can't remember how.

Answered by Penny Nom.
A partial derivative 2004-03-19
From Penny Nom:
Is it possible to differentiate the following equation, if so could you please explain.

S=SQRT(T-(5/X^2))

I would like the derivative of S with respect to X.

Answered by Harley Weston.
A water lily 2004-03-17
From Inba:
A water lily with a rigid stem extends one foot above the surface of the water. When pulled over, it disappears beneath the water at a distance 3 feet from where the stem originally entered the water. How deep is the pond?
Answered by Penny Nom.
Factoring x^2 + 5x + 4 2004-03-14
From Keenan:
how do I factor x^+5X+4?
Answered by Penny Nom.
Johann Muller 2004-03-14
From A student:
What major contributions to spherical trigonometry did Johann Muller make?
Answered by Penny Nom.
Baseball games 2004-03-14
From Fabian:
Let's say I have 12 different Baseball games. There is one of only 2 possible outcomes for each of the 12 games. How would I fugure out how many combinations there are for the 12 games and for 11,10, and so fourth.
Answered by Penny Nom.
Billions in Australia 2004-03-12
From An Australian:
how many million in one billion ?
Answered by Penny Nom.
Area and perimeter 2004-03-12
From Sandy:
The rectange has an area of M square units and a perimeter of 2M units. What is the value of x?

The length of the rectange is 3, while the width of the rectange is x-1.

Answered by Penny Nom.
Sum of the angles in a pentagon 2004-03-11
From Ashish:
What is the sum of all the measures of the angles of a Pentagon
Answered by Penny Nom.
The area of a lot 2004-03-09
From Dave:

If our lot is 192.71 x 138.79 x 158.74 x 103.08

Question is what percentage of an acre is our lot?


Answered by Penny Nom.
A metric prefix table 2004-03-07
From April:
Can you tell me what the scale is for nano, micro, mega, kilo, etc.... I know that mega is 10 to the sixth power but I can't remember the other ones.
Answered by Penny Nom.
Equivalent sets 2004-03-06
From A student:
If A=(1,2,3,4,...) and B=(5,10,15,20,...), is A equivalent to B. Why or Why not ?
Answered by Penny Nom.
The radius of a circle 2004-03-06
From A student:
what is the radius of a circle with the circumference of 12 inches?
Answered by Penny Nom.
A fifteen sided polygon 2004-03-05
From Joann:
WHAT IS THE NAME OF A FIFTEEN SIDED SHAPE?
Answered by Penny Nom.
A geometry problem 2004-03-04
From Jennifer:
I need help with this problem: Square ABCD has side length 2. A semicircle with diameter AB is constructed inside the square, and the tangent to the semicircle from C intersects side AD at E. What is the exact length of CE?o
Answered by Chris Fisher.
Cosine of 35 degrees 2004-03-03
From Jason:
How do you find the exact solution to cosine 35 degrees.
Answered by Chris Fisher.
Volume of a fuel tank 2004-03-01
From Mike:
I have a fuel ank for my big truck that has a radius of 24inches and a length of 65 inches. I am trying to compute the volume of fuel in the tank. I tried pie x the radius squared times the length but no usable results. Can you help me please?
Answered by Penny Nom.
Logs 2004-02-29
From Sandy:
find X
3/2logb4-1/2logb8+logb2=logbx

my book uses prop 7 to rewrite as logb4raised3/2-logb8raised2/3+logb2=logbx; then writes logb8-logb4+logb2=logbx how did it get here?

Answered by Claude Tardif.
Napoleon's theorem 2004-02-27
From David:
How do i prove this : For any triangle, if you make 3 equillateral triangles using the sides of the the original triangle, the central points of the 3 tringles another triangle that is equillateral.z
Answered by Chris Fisher and Penny Nom.
Three jugs 2004-02-26
From Shelly:
You have three jugs: A 6 Liter, 4 Liter and a 3 Liter. Your goal is to get exactly 5 Liters of liquid in the 6 Liter jug. Your supply of liquid is unlimited and there are NO markings on the containers. You cannot do halves.
Answered by Claude Tardif.
Rearranging an expression 2004-02-24
From Corey:

use the following formula to answer the question F=9c divided by 5 plus 32

rearange the formula to solve for c
how many degrees fahrenheit would it be it it was:
0 degrees celcius
-10 degrees celcius
-46 degrees celcius


Answered by Penny Nom.
An elliptic cone 2004-02-24
From Ben:

I am building a model for my architecture class. I need to build a elliptic cone out of chipboard and i have no idea how to do this.

The cone needs to be 20in tall and the ellipse has a max radius of 10in and a min radius of 8in.

So my question is how do i lay this out on a piece of paper so that i can form the cone after i cut it out.


Answered by Penny Nom.
A problem with logs 2004-02-22
From Caitlyn:
problem: ln x + ln (x-2) = 1

I can solve this on the calculator, but the book wants 1+ [(sqroot)1+e]. How can I get the e?

Answered by Penny Nom.
Selecting balls from a golf bag 2004-02-21
From A student:
There are 5 white and 5 yellow balls in a golf bag. Two are selected randomly, and simultaneously. What is the probability that the first one or the second one will be white?
Answered by Penny Nom.
The dimensions of a rectangle 2004-02-21
From Jane:
A rectangular label has an area of 176cm2 and a perimeter of 54cm. Find the dimensions of the label.
Answered by Penny Nom.
Gallons in a cylinder 2004-02-20
From Jim:
Please advise how to use the formula I saw for determining the number of gallons in a cylinder? This is how it was written V = pi r2 h . does this mean pie, or 3.14 X radius squared, X the height? If so please use a standard 55 gal barrel (1' R & 3' H) for example?
Answered by Penny Nom.
A worm crawling home 2004-02-18
From Cindy:
A worm is crawling to his home which is one meter away. The longer he crawls the weaker he gets and the less he can crawl the next day. If he crawls within 1/3000 of a meter of his home, he will find food. He must eat within twelve days. The first day he crawls 1/2 meter. The second day he crawls 1/4 meter. The third day he crawls 1/8 of meter. This pattern continues for twelve days. Make a Chart that shows the distance he has covered at the end of each day and the total he has covered at the end of each day. Does he make it to the Food in time?
Answered by Penny Nom.
Nonconvex shapes 2004-02-18
From Merrill:
Draw 4, 6 and 8 sided shapes so that ONE straight line can be drawn through each side
Answered by Chris Fisher and Penny Nom.
Acres and square miles 2004-02-17
From Richard:

How many acres are in .17 square miles? and How many acres are in .6 square miles?

The two areas I am requesting information about are Vatican City and San Marino, the two smallest countries in the world. If I can transfer the sq. miles into acres , I can relate the size of these countries to our school grounds and the students will better understand their sizes.


Answered by Penny Nom.
Grams and Milliliters 2004-02-17
From Chantal:
We have to do a "thing", and I need to convert grams into mililitres (g into mL). how can I do this?
Answered by Penny Nom.
Areas, perimeters and derivatives 2004-02-15
From Geoff:
A recent lecture in my calculus class led me to realize that the derivative of the area of a circle, with respect to the radius is equal to the circumfrence. This also holds true for the relationship between the volume of a sphere and the surface area of that sphere:
why do these hold true? why is it only for circular objects?

Answered by Harley Weston.
An arc on a train track 2004-02-15
From A student:
I'm trying to build a route in a train simulator program. I have a curve to the right (an arc, in other words) with a length of 25 meters and the radius is 1,500 meters. Let's say point P is the beginning of the curve (arc) and point Q is the end point of the arc. Then picture a tangent to point P. I need to find the length of a line perpendicular to that tangent that runs to point Q (the end of the curve/arc).
Answered by Penny Nom.
The derivative of x to the x 2004-02-14
From Cher:
what about the derivative of x to the power x?
Answered by Penny Nom.
Probabilities and olivies 2004-02-14
From Annetta:
A bowl contains five green olives and eight black ones. Wilma sticks a fork into a bowl and brings out two olives. What is the probability that she gets one green and one black olive?
Answered by Harley Weston.
Red riding hood 2004-02-14
From Frank:
Red riding hood had a number of jars to take to grandma's house. As she traveled she gave away 1/3 of them to an old lady. Then later As she walked through the forest a wolf stole 1/2 of her remaining jars. When she arrived at grandma's house she gave her 2/3 of the remaining jars. She kept 6 jars for herself. How many jars did she start out with.
Answered by Diane Hanson.
A pyramid-shaped tank 2004-02-13
From Annette:
The base of a pyramid-shaped tank is a square with sides of length 9 feet, and the vertex of the pyramid is 12 feet above the base. The tank is filled to a depth of 4 feet, and water is flowing into the tank at a rate of 3 cubic feet per second. Find the rate of change of the depth of water in the tank. (Hint: the volume of a pyramid is V = 1/3 B h , where B is the base area and h is the height of the pyramid.)
Answered by Harley Weston.
Common fractions to decimals 2004-02-12
From A parent:
I'm trying to help my son work with turning fractions into decimals and this seems foreigh to me after 20 years of not being in school. Can you give me a few illustrations and/or examples of how to do that??
Answered by Penny Nom.
Three numbers are added in pairs 2004-02-12
From Jane:
When three numbers are added in pairs, the sums of the pairs are 22, 39, and 45. What are the three numbers?
Answered by Penny Nom.
Digits in the decimal expansion 2004-02-11
From Leslie:
In the decimal expansion of 1/17 what digit is in the 1997th place?
Answered by Penny Nom.
Three times a year 2004-02-09
From Graham:
What is the term for a report that is submitted three time a year or at the very least a prefix term for one third
Answered by Harley Weston.
Will be shinning in 36 hours? 2004-02-08
From Kim:
It is noon, you lunch hour, but you cannot go out because there is a terrific hailstorm. Turning on you radio you hear that weathercaster predicts that the hail will change to rain and that it will pour all day today. How can you determine whether the sun will be shinning in 36 hours? Justify your answer.
Answered by Penny Nom.
Graphing lines 2004-02-08
From A student:
how do you solve y=1\3x+3 and y=1\3x-3 by graphing?
Answered by Penny Nom.
A quadratic word problem 2004-02-04
From Carl:
A walkway of uniform has 72m2 and surrounds a swimming pool that is 8m wide and 10m long. Find the width of the walkway.
Answered by Penny Nom.
$20,000 at 25% compounded daily 2004-02-03
From A student:
Hi there, I'm tying to figure out how much interest I would pay on 20,000 if it was 25% compounded daily.
Answered by Penny Nom.
The volume of a tetrahedron 2004-02-03
From Joseph:

How do I calculate the volume of a bird tetrahedron. I'm trying to find out to get extra credit. I came with this idea while making a origami tetrahedron.

Joseph

P.S. Do you by any chance have instructions for orig ami figures. If you do, please send me some. Thanks!


Answered by Penny Nom.
The incline of an exercise machine 2004-02-02
From An exerciser:
This is a question about incline percentage. On an exercise machine if the elevation is raised 3 and one half inches from a level position then what would be the incline percentage?
Answered by Penny Nom.
Angular speed 2004-02-01
From Rachel:
A 4 inch pulley on a motor makes 1500 revolutions/minute. It is connected to an 8 inch saw blade. a. Find the angular speed of the pulley in radians/minute

b. Find the angular speed of the saw in radians/ minute

Answered by Harley Weston.
Logarithms 2004-02-01
From A student:
Where did the word logarithmic come from? Where/whom did it originate?
Answered by Harley Weston.
Intersecting a line and a curve 2004-01-29
From Senthil:
between line and curve how can i find intersection point? could you write me the formula and explanation also sir.
Answered by Penny Nom.
Is a cylinder a type of prism? 2004-01-29
From Rebecca:
Is a cylinder a type of prism?
Answered by Penny Nom.
Sin(3x), cos(3x) and tan(3x) 2004-01-28
From Jon:
What is the identity for cos3x, sin3x, and tan3x? In class, we learned double angel identities and were asked to find out the identity to these three trig functions. If you can help, please do. Also, i know that the cos4x- sin4x is the same as cos2x. Is cos8x-sin8x = cos2x also true? Thank you.s
Answered by Chris Fisher.
The London marathon 2004-01-27
From Nina:
After a years training, Minnie Midriffe increased her average speed in the london marathon by 25%. By what percentage did her time decrease?o
Answered by Penny Nom.
Lead weights 2004-01-26
From Jeff:
I am making a mold for my decoy anchors. I need 10 - 12 lead ounce weights. How would I determine the liquid measurement for that weight? For example if I melted down 10 ounces of lead how many cups would that be or quarter cups etc?

I would like the formula if I could and good instructions on how to use the formula.

Answered by Penny Nom.
Polynomials 2004-01-25
From Bruce:

A polynomial is defined as
Polynomial functions are functions that have this form:
f(x) = anxn + an-1xn-1 + ... + a1x + a0
The value of n must be an nonnegative integer. That is, it must be whole number; it is equal to zero or a positive integer.
The coefficients, as they are called, are an, an-1, ..., a1, a0. These are real numbers.

Questions:

  1. why must n be positive?
  2. what are some historical facts about the evolution of the definition?

Answered by Harley Weston.
|x^2 - 12x| = 8 2004-01-24
From Sherre:
|x2 - 12x| = 8
Answered by Penny Nom.
Bundles of asphalt shingles 2004-01-24
From Larry:
According to my study material 4:12 multiplying factor for shingles is 1.054. The question reads as follows: A building with a floor plan of 3350 sq. ft. and a roof slope of 4:12 will require _______ bundles of standard asphalt shingles.
Answered by Harley Weston.
A counterfeit coin 2004-01-23
From Kim:
You are considering buying 45 silver coins that look alike, but you have been told that one of the coins is a lightweight counterfeit. Find the least number of weighings on a balance scale that you can use to be certain you have found the counterfeit coin?
Answered by Penny Nom.
g on Mars 2004-01-22
From RC:
To measure the acceleration on Mars an object is dropped from a height of 160m. It takes 10s for this object to hit the ground. What is the value for g on Mars?g
Answered by Penny Nom.
A game with some strings 2004-01-22
From Trudy:
A boy holds 6 pieces of strings in his hands with the ends protruding above and below. The top ends are tied together in pairs and then the lower ends are tied together in pairs. What is the probabilty that the pieces of string are all joined in one loop?

What is the probability of obtaining two loops? Can you generalise this to solve the problem for 2n blades of grass?

Answered by Andrei Volodin and Penny Nom.
A problem with sets 2004-01-20
From Jason:

Given that the universal set S is the set of all sports fans, and

F={x|x is a football fan}
B={x|x is a basketball fan}
H={x|x is a hockey fan}
a)Describe (F^B)' (f intersect b)' in words
b)Draw a Venn Diagram and shade the region that represents the set of football fans or both basketball and hockey fans.


Answered by Penny Nom.
What is the speed of the automobile? 2004-01-20
From Rita:
An automobile travels toward Nashville from Cookeville. It takes 33 minutes to travel from Cookeville to Manchester, which is a 36 mile distance. If the driver continues at this pace, what is the speed of the automobile?
Answered by Penny Nom.
A parabolic arch 2004-01-19
From Teresa and Robyn:
A bridge over a river is supported by a parabolic arch...arch is 200 m wide at water level...the maximum height of the arch is 80 m..what is the height of the arch measured from a point on the water 40m from the centre of the arch
Answered by Penny Nom.
Cubic yards in a concrete wall 2004-01-18
From Danny:
I am studing for my contractors exam. I am haveing truble with a few questions. I have the answer to the problems but I dont know how to get them my self.
please help

How many cu. Yds. Of concrete are in a wall that is 150' long and 8'4" high an d 8' thick

Answered by Penny Nom.
Some trig problems 2004-01-18
From Weisu:

I have some questions about pre-calculus.

(1) (2(cos(x))^2)+3sin(x)-1=0
(2) sin(x)cos(x)=(1/2)
(3) 3sin(x)=1+cos(2x)
(4) tan(x)*csc(x)=csc(x)+1
(5) sin(arccsc(8/5))
(6) tan(arcsin(24/25))
(7) arccos(cos(11pi/6))
the last problem uses radian measure.


Answered by Penny Nom.
Volume of a tube 2004-01-14
From Mike:
where can you find how to get volume of a tube: 3 in across x 3 inch high
5 in across x 5 inch high
same formula for both. ONE will get the other.

Answered by Penny Nom.
3/8 as a decimal 2004-01-13
From A student:
My Math Midterm is tommorow and I am stumped... how do you turn 1 and 3/8 into a decimal... I know the answer is 1.375 but I just copied down what the teacher wrote on the board.
Oh, and what are the three cases of percents?

Answered by Penny Nom.
What is a Fact Family? 2004-01-13
From Keisha:
I am helping my child with his homework. The worksheet we are trying to do ask the child to choose a number from inside of a circle and then one from inside of a triangle and use the number to build a fact family. The title of the worksheet is "Building a Fact Family House". I noticed that Silver Burdett Ginn,Inc. was the publisher of the book the worksheet came from so I was on the website trying to find an answer to my question. My question is "What is a Fact Family?"
Answered by Judi McDonald.
Polynomial division 2004-01-13
From Efrat:

How do we divide the following polynomials?

(x3 + x2 - x + 2) / (x + 2)


Answered by Penny Nom.
Twelve acres 2004-01-13
From Heather:
I am looking at buying a property that includes 12 Acres of land and I am trying to visualize what this size is equivalent to. I have searched your site and figured out how many square feet it is but I was hoping that you could apply 12 Acres to something tangible, like a football field!
Answered by Penny Nom.
Why is the letter J used for the integers? 2004-01-13
From Amanda:
Why is the letter J used to represent intergers as a symbol?
Answered by Penny Nom.
Cubic furlongs 2004-01-11
From A student:
1 mile=8 furlongs, how many cubic furlongs in a cubic mile?
Answered by Penny Nom.
Reflex angles 2004-01-09
From Sonya:
My daughter is searching for examples of reflex angles. We already have the hands of a clock but still need another example. Can you help us.
Answered by Chris Fisher.
Geoboards 2004-01-09
From Elissa:
What is a geoboard? I have a young girl in grade eight math who has a math question using a geoboard.
Answered by Diane Hanson.
The area of a lot 2004-01-09
From Lea:

I have a problem. I am interested in purchasing a piece of property which has the following dimensions. It is 259.10 Feet on the north side x 61.20 Feet on the east side x 217.80 Feet on the south side x 196.70 Feet on the west side. It looks like an irregular rectangle shape. How do you figure out the square footage on this irregular sized property, and what is the correct square footage answer.

I have attached a survey map.


Answered by Harley Weston.
Inequalities 2004-01-08
From Michael:
How do you use inequalities in your job and in your everyday life?
Answered by Penny Nom.
Unusual occurances 2004-01-08
From Martin:
My wife and I have a question about the probability of something that happened to us a few years ago. So far, no one has been able to give me even an approximate answer. On my 32nd birthday, my wife and I went out to eat at local Japanese hibachi style restaurant. At the restaurant, couples/families are sat together around the hibachi where the cook performs a show. There was a fifteen minute or so wait, so my wife and I sat in the lounge waiting for our name to be called. When they called our names for the reservation, this is what happened. The first group called was the Martin family. Then they called the Francis family. We were the next family to be called, the Ashton family. My full name is Martin Francis Ashton! I think the odds of that happening to someone are very unlikely, but it did, and there is more. Next, we were all sat at the same table in that order, "Martin" family, "Francis" family, then us, the "Ashton" family. Again, it formed my full name!
Answered by Penny Nom.
The sides of a circle 2004-01-07
From Helena:
My name is Helena and I am 10 years old. On a resent math exam I was asked the question" How many sides does a circle have?" and I wrote down none. The teacher said the answer was one side.
Answered by Chris Fisher.
Laying decorative bricks 2004-01-07
From Duk:
Loren is laying decorative brick along both edges of the 21-meter walkway up to his house. Each brick is 0.26 meters long. He is placing the bricks end to end. How many bricks does he need to do the job?
Answered by Penny Nom.
Factoring expressions 2004-01-03
From A student:

I'm having trouble factoring expressions that aren't monic. I can do things like x2-9x+8, but problems like 12x2+5x-3 have me stumped.

I also have a question about factoring out common factors. In a problem like x2-18x+81, wouldn't you divide by 9? But what happens to the x2, is it x2/9?
Answered by Penny Nom.

Solving an equation 2004-01-03
From A student:
the question is to solve and check equations and the question is
2n + 3n + 7 = -41

Answered by Penny Nom.
Making a windmill 2004-01-02
From Matthew:
I am a farmer in Ontario. It has been almost 20 years since high school. I am toying with making a windmill. The output chart for the the old generator I have is shown below. Before I tear it appart I would like to develop a formula from the chart that can predict the output at various speeds.
Answered by Penny Nom.
A new way to measure randomness 2003-12-31
From Stephanie:

Last year, I did a science project in which I asked, "Which shuffles better, an automatic card shuffler or shuffling by hand?" To measure this I decided the "best" shuffler was the one to become random first. Last year, to measure randomness, I numbered cards 1-52 and had the subjects shuffle them until they broke up the rising sequences or reached 10 shuffles. (Usually 10 shuffles came first...) Anyway, I did the same thing with the automatic card shuffler, and, as hypothesized, the automatic card shuffler randomized the deck first.

This year, I have decided to continue the project. The problem is, I need a new way to measure randomness without the use of fancy computers or something. I have searched the Internet, I have posted my query on websites based on math, and I have searched the local library.

I have found many useful things on the Internet, but none of them can tell me a new way to measure randomness. I cannot do a perfect shuffle, and I am not terribly gifted in the art of using computers. If you have any information (anything will help) or advice, I would be greatly obliged.


Answered by Andrei Volodin.
The trapezoidal rule 2003-12-31
From Prashanth:
I have a set of XY co-ordinates(say 49 points),I want to find the area enclosed by the curve joining them and the axes.I do not have an equation for the curve and I am not able to fit a curve given the points.I use the trapezoidal rule (Y1+Y2/2)*(X2-X1), Is it right? can I do this with any other method?
Answered by Penny Nom.
Ratios and proportions 2003-12-31
From Frank:
I would like to know how you use 'ratios and proportions' in everyday life.
Answered by Penny Nom.
The angles in a regular polygon 2003-12-21
From Ernie:
If i have a measure of one interior angle of a polygon, how can i find the number of sides it has?
Answered by Penny Nom.
The area of a plot of land 2003-12-19
From David:
I have a plot of land that I cannot find the square footage of. Actually my home sits on a large lot. Local building codes require that a home is seated on no less than 7500 square feet of land and have no less than 50ft of road frontage (the rear of the lot has 65ft of frontage on another road). I need to know if I have enough square footage to build a second home on the rear lot. The lot has five sides. Only one angle is a 90 deg. angle. The others I cannot determine. Of the 4 remaining angles, all 4 angles are obtuse, or greater than 90 deg. The dementions are 89.19' x 130' x 118.52' x 65' x 170.94'. The two legs that make the 90 deg angle are 89.19' and 170.94'. The measurements fall in order using the 90 deg angle as the point of origin and proceeding clockwise, 89.19', 130', 118.52', 65', 170.94'. I have used all the skill I can muster, however, it has been a long time since school and regretably I cannot find an answer.
Answered by Penny Nom.
Multiplier and multiplicand 2003-12-17
From Karen:
The K-5 curriculum states that at a grade 5 level students should "understand and use the terms multiplier, multiplicand" The curriculum defines 'multiplicand' as " a number being multiplied by another number. A factor." There is no definition given for 'multiplier' Question: Define these words- are they just other words for 'factors'. If so, how important is it that students use these terms? It would seem like 'factors' would be the better choice of words to use. If multiplcand refers to eg. the first factor in the equation and the 'multiplier' the seond term, (if in fact it does)- is this not confusing for students when we teach them that multiplication is commutative?
Answered by Penny Nom.
Binomial distribution 2003-12-17
From Lesley:
my daughter is having difficulty with the following formula P(X=x) = ( n over x) px (1-p) n-x
The teacher has given them the formula but not taught them how to apply it or understand it.

Answered by Penny Nom.
Factoring in 2003-12-16
From Priscilla:
How can you use solving by factoring in real life applications?
Answered by Harley Weston.
An elliptical race track 2003-12-16
From Judy:
the inner rail of a race track is a perfect ellipse. the track is a standard width all the way around. how can i prove that the outer rail is a ellipse?
Answered by Penny Nom.
How far can you see? 2003-12-15
From Judy:

How far apart, assuming no obstacles, can two people stand and still see each other?

i know this deals with the curvature of earth, but i can't figure out the formulas involved.


Answered by Chris Fisher.
(2^ 20,966,011) - 1 2003-12-12
From Reid:
Can someone please explain, answer and/or solve this: (2^ 20,966,011) - 1

I don't know what it means, my boss sent it out, I'm assuming it is to be solved but don't know what it is, any prompt help would be appreciated, very much so, in fact, thanks,...........


Answered by Penny Nom.
A snowball melts 2003-12-11
From Adam:
A spherical snowball has diameter 10cm and density 0.75g/cm3. It is placed in a cylindrical can with diameer 12cm. After the snow melts it turns to water with density 1.0g/cm3. What will be the depth of the water in the can (to 1 decimal place)?
Answered by Penny Nom.
Diamonds and Pearls 2003-12-11
From Lauren:
A crown holds a total of 3080 jewels.
It is made up of diamonds and pearls.
There are 10 times as many diamonds as there are pearls.
How many pearls are there?

Answered by Penny Nom.
Non-euclidean geometry 2003-12-08
From Geoffrey:
How can you use non-euclidean geometry to navigate on a sphere? What geometers did work in this area?
Answered by Chris Fisher.
Non-Euclidean geometry 2003-12-03
From Geoffrey:
What are the applications of Non-Euclidean geometry (especially hyperbolic and spherical)?
Answered by Walter Whiteley.
A locus 2003-12-02
From Tash:

Question:

a)Find the equation of the locus of the point P which moves so that its distance from A(1,2) is always three times its distance from B(5,6)

b) Show that this locus is a circle and states the coordinates of its centre and the length of its radius


Answered by Penny Nom.
Finding angles 2003-12-02
From Jason:
I AM TRYING TO SOLVE A TRIG PROBLEM AND HAVE FORGOT HOW TO DO IT. WHAT I HAVE IS A RIGHT TRIANGLE WITH SIDE A BEING 14 FEET AND SIDE B BEING 3 FEET, USING PYTHAGOREAMS THEOREM SIDE C SHOULD EQUAL 14.318 FEET ON A RIGHT TRIANGLE BUT I AM TRYING TO REMEMBER HOW TO FIND MY ANGLES OTHER THAN THE ONE THAT IS 90 DEGREES.
Answered by Penny Nom.
The area of a triangle 2003-12-01
From A student:
Find the Area of Triangle ABC
A(-3,2)
B(4,0)
C(0,8)

Answered by Penny Nom.
5.75 acres of land 2003-11-28
From Susan:

We own 5.75 acres of land here in El Paso Tx. Some one wants to buy all or some of our land...to build homes on...but.. I can't figure a few things

1. our land is Almost exactly 500x 500x500 x500 ft square proprty based on this can you tell me the approx perimiter measurements of EACH acre we wish to sell approx 1/2 of this 5.75 acres ...or a tad bit less.. these are all approx measurements what I also need to know is.. how many 50ft x 100ft lots are in each (1 )Acre???


Answered by Penny Nom.
A fish tank 2003-11-26
From Bill:
How do I work out the volume and weight of water in a fish tank that is 36in x 14in x 12in?
Answered by Penny Nom.
Mr.Carter is very cautious 2003-11-26
From Bob:
Mr.Carter is very cautious. He decides to invest in only three stocks: one low stock, one high stock, one medium stock. Given that the expected annual yields are 6% for low stock, 7% for medium stock, and 8% for high stock, he wants his investment in medium stock to be half of his total investment in low and high stock. How much should I invest in each type fo stocks to expect a total annual return fo $650 form my investments?
Answered by Harley Weston.
Area of an equilateral triangle 2003-11-25
From Jared:
Can you help me understand why the area of a equilateral triangle is the square root of 3 divided by 4 times the lenght of the side squared?
Answered by Penny Nom.
An inequality whose solution is all points 2003-11-25
From Katie:

Hi my name is Katie and I am in the 9th grade and I would like help on this question. Is it possible to have a system of inequalities whose solution is all points?

1.I need to know why (explanation)

2.how to do it(example)


Answered by Penny Nom.
The volume of an irregular tetrahedron 2003-11-24
From Peter:
How do I calculate the volume of an irregular shaped tetrahedron where:-
side a = 1.4 m
side b = 1.4 m
side c = 1.2 m
and height = 0.75 m at the junction of sides a and b.

Answered by Penny Nom.
Difference of squares 2003-11-24
From Susie:

Factor assuming that n is a positive #

Problem: (I will give it to you in words beacuse I don't know how to do exponents on the computer.) Forty-five r to the 2n power minus five s to the 4n power. I was hoping you could walk me through it not just give me the answer.


Answered by Penny Nom.
An octagon shaped bed frame 2003-11-23
From Trish:
My son and I are making an octagon shaped bed frame. We are going nuts trying to figure out what angle to cut the boards to make an outline of an octagon. It seems that the 8 inside angles of the 8 "corners" are 120*, but what is the angle that the 2x6 wood should be cut so that they will angle together to form the outline of the octagon?
Answered by Penny Nom.
Laws of sines and cosines 2003-11-23
From A parent:
On the one side of a stream lines PA= 586.3 feet, PB = 751.6 feet are measures, angle APB being 167 degrees and 36 min. Q is a point on the opposite side of the stream. Angle PAQ=63 degress and 18 min and PBQ=49 degrees and 24 min. Find PQ.
Answered by Penny Nom.
Aristotle 2003-11-21
From John:

I would like to know how aristotle was related to math.

In all my searches I have found only philosophy etc. But not any math.


Answered by Penny Nom.
AB/AP=AP/PB 2003-11-20
From James:
My name is James McBride. I'm having a difficult time with a pre calculus problem, which goes as follows: "show that AB/AP=AP/PB is equal to (1+5^1/2)/2 (one plus the sqaure root of five with the sum divided by two. I can't do the square root sign, sorry.) I have tried to solve for PB in terms of the other varialbles and then work the quadratic equation. THAT DOES NOT WORK!!!! I am befuddled. Please help me. I am a student of secondary level.
Answered by Penny Nom.
Four digit numbers 2003-11-20
From Rob:
Hello, I sure hope you can answer this question. I got into a discussion at work about how many possible combinations there are for a four digit number using the numbers 0 through 9. This was in reference to a discussion about the lottery. I said there was only 10,000 possible combinations. My friend said that there was many times more, but he could not remember how to figure it out. The answer and the formula would be greatly appreciated.
Answered by Penny Nom.
4(r-2) = r(2+8) 2003-11-20
From Angela:
I not going to give you the problem that I have, but a problem like it. 4(r-2) = r(2+8) I get mix up trying to solve a problem like this could you help me. So that I will be able to handle the ones on my homework.
Answered by Penny Nom.
A riddle 2003-11-19
From Sarah:
Ok, our teacher gave us this riddle, and I cannot for the life of me figure it out. He said that there are three problems with the following proof:
Answered by Penny Nom.
Systems of equations 2003-11-19
From Scott:

I hope that u can help me....I am a college student taking a class in Pre Calculus.....I have homework due this Friday and it counts a BIG Percentage on my FINAL grade.....I am getting mixed up and can not figure out a few problems.....Please help me.....

Method Of Subsitution

Problem 1. y- 8x = -5
x(squared) + y(squared) = 25

Problem 2. y = x(squared) - 2x - 6
Y = x(squared) - 4

Answered by Penny Nom.

A point and a line 2003-11-19
From Meenakshi:

Given a line segment L(x1,y1) to (x2,y2) and a Point P(x3,y3).
I need the formula to find a point on the line L that is closest to the point (x3,y3).


Answered by Penny Nom.
Unmatched socks 2003-11-17
From Shannon:

i have 6 pairs of socks lying unmatched in my dresser drawer. Each pair is a differnt color (brown, black, red, yellow, green, purple) When i wake up in the morning, I randomly grab 2 of the loose socks and put them on without looking.

If two socks are randomly selected from the drawer, what is the probability that they match?


Answered by Andrei Volodin.
Three problems 2003-11-16
From Megan:
My name is Megan and I am a junior in high school. Our teacher gave us a few xtra credit questions and I need some help.
Answered by Penny Nom.
Divisibility by 7 2003-11-14
From A student:
how do you test a number to see if it is divisible by 7 or not?
Answered by Penny Nom.
Lines 2003-11-14
From A student:
What is a name that a group of lines pass through?
Answered by Chris Fisher.
root24 = 2 root6 2003-11-13
From Burke:
I know that all algerbra can be represented visually, for example: Y=9x+3 is a line on a two dimentional plane. Could Math Central please send me a visual representation of the proccess of simplifying square root like root24 equals 2root6
Answered by Penny Nom.
Diagonals in an octagon 2003-11-13
From Glenn:
Question 1.
What is the probability of choosing the longest diagonal in an octagon?

Question 2.
Probability of choosing the shortest?
Answered by Andrei Volodin and Penny Nom.

Mixing Brazil nuts and cashews 2003-11-09
From Bob:
Cashews are sold for 1.20 per 1/4 pound and Brazil nuts are sold for $1.50 per 1/4 pound. If you mix 20lbs of Brazil nuts, how many cahsews would it take to make a mix that sells for $1.30 per 1/4 pound?
Answered by Penny Nom.
A least squares line 2003-11-09
From Michelle:
Hooke's Law asserts that the magnitude of the force required to hold a spring is a linear function of the extension e of the spring. That is, f = e0 + ke where k and e0 are constants depending only on the spring. The following data was collected for a spring;

e: 9 , 11 , 12 , 16 , 19
f : 33 , 38 , 43 , 54 , 61

FIND the least square line f= B0 + B1x approximating this data and use it to approximate k.
Answered by Penny Nom.

Equal ratios 2003-11-07
From A student:
In the ratio 3:8=17__, can__be a whole number?
Answered by Penny Nom.
Intercepts 2003-11-04
From A student:
I am totally confused on how to determine intercepts of an equation and then to graph the line. For example, how to determine the intercepts of 2x-3y-12=0.
Answered by Penny Nom.
X bar 2003-11-04
From mary:
Remind me what this means:

    _    
    X 

Answered by Andrei Volodin.
The length of a chord 2003-11-03
From David:
When Radius=400.00' and Arc=130.58' what is the Cord distance in feet?
Answered by Penny Nom.
A circle around an irregular polygon 2003-11-03
From Dale:
How do I find the properties of a circle that is drawn around an irregular polygon of (n) sides with the lenghts of each side given and all end points of the polygon lye on the circumferance of the circle?
Answered by Chris Fisher.
Solve for Q 2003-11-03
From Anwaar:

The problem is:

QT - X
________ + H = HT
QS


Answered by Peny Nom.
Divisibility by 2 or 5 or both 2003-10-30
From Abdu:
How many positive integers less than 1,001 are divisible by either 2 or 5 or both?
Answered by Penny Nom.
12 cookies 2003-10-29
From Joel:
there are 12 chocolate, sugar and cinnamon cookies. the probability of choosing a chocolate cookie is 1/3. the probability of choosing a sugar cookie is 1/4 what is the probability of choosing a cinnamon cookie
Answered by Penny Nom.
A rectangle on a disk 2003-10-29
From Arthur:
How do I go about solving the following problem: What is the width of the largest rectangle with a length of 16 inches you can cut from a circular piece of cardboard having a 10 inch radius?
Answered by Penny Nom.
How much time? 2003-10-29
From Lori:
I am a elementary teacher 3-6 math. We have just implemented the Houghton Mifflin Math curriculum. It seems to be quite difficult for most of our students. In order to get through all the material we need to do a lesson a day. Although the students are not getting the material that fast. Should we move on without all the students understanding the material or should we wait and remediate until they get it and be behind for the year. This isn't just a few kids its most of the kids. We are also suppose to teach to all the standards so if we don't get to them the students will not do well on the standardized tests.
Answered by Diane Hanson.
Trees 2003-10-28
From Ali:
Is there any formula for counting the number of trees (graphs)?
Answered by Denis Hanson.
Solve for x 2003-10-27
From Lonnie:
i need help with solving equations and formulas such as -3x+b=6x, for x. I do not understand how you find x.
Answered by Penny Nom.
Knitting gauge and felting 2003-10-27
From Sara:

I wish to create a pattern to knit an item that will then be felted, thus, shrink as a result. In order to determine the proper gauge to use in creating the item, I produced a gauge swatch, noted the original properties and then the post-felting properties. They are as follow:

Before Felting:

20 stitches(width) x 20 rows(height)yielded:
Gauge: 3.2 stitches(width) and 4 rows(height) = 1"
Dimensions: 6.25"(width) x 5.25"(height)

After Felting:

20 stitches(width) x 20 rows(height)
Dimensions: 4"(width) x 3"(height)

How do I calculate the percentage change in size to accurately determine the number of stitches and rows needed to produce the desired dimensions for the finished (felted) item?


Answered by Penny Nom.
A rock on a string 2003-10-26
From A student:
a rock on a 4' string is rotated at 80 rpm. what is the linear speed in feet per second? in miles per hour?
Answered by Penny Nom.
Maine and Nevada 2003-10-24
From Jimmy:

From 1989 to 1990, the population of Nevada increased b y 157,000, and that of Maine increased by 30,000. In 1990, the population of Nevada was 1,206,152, and that of Maine was 1,233,223. If the populations of the two states continue to increase at the same rates, when will the populations of Nevada and Maine be the same?

Write a verbal model for this problem
Write an equation for the model
Solve the equation and answer the question.


Answered by Penny Nom.
A parabola 2003-10-24
From Delores:
Given the vertex (4, -2) y intercept = -6 find if/where the parabola crosses the x axis?
Answered by Penny Nom.
0.810 and 0.801 2003-10-24
From Ahmed:

could you please tell me what is the greatest decimal fraction formed of the digits 0, 1 , 8?

Is it 0.810 or 0.801, please give me the reason of the answer in details and the references if it possible.


Answered by Penny Nom.
Odd Pythagorean triples 2003-10-23
From Kathleen:
in a triple can a and b be odd numbers
Answered by Penny Nom.
Squares in a rectangle 2003-10-21
From Raj:

Draw a rectangle with sides of 3 and 4. Divide the sides into 3 and 4 equal parts respectively. Draw squares joining the points on the sides of the rectangle. You will have 12 small squares inside the 3 x 4 rectangle.

If you draw a diagonal of the rectangle, it will intersect 6 of the the 12 smaller squares.

Similarly, if you have a 4 x 10 rectangle, the diagonal would intersect 12 of the 40 squares inside the rectangle.

Is there an algebric equation that determines the number of squares that will be intersected by the diagonal of a rectangle?


Answered by Chris Fisher.
0.3 trillion 2003-10-20
From Bill:
What is .3 of one trillion?
Answered by Penny Nom.
Domain of a function 2003-10-14
From Karim:
Find the Domain of the Function
H(x) = Log3 (4x + 7)-10
H(x) = log base 3 times 4X plus 7 minus 10
i am having problem solving these kinds of problems.

Answered by Penny Nom.
5x - y 2003-10-11
From Candace:
math problem 5x-y for x=12 and y=14 do you mulitilpy 5x12 and than -the answer from 14 which gives you 46? is this right
Answered by Penny Nom.
Points inside a triangle 2003-10-10
From Laura:

is there a way to establish, if a given point at coordinates (x,y) is within an area defined by three other sets of coordinates forming a triangle ABC?

is T(x,y) within triangle A(x,y)B(x,y)C(x,y)


Answered by Chris Fisher.
10,000 2003-10-08
From Nathan:
what two numbers make the product of 10,000. Neither number can end in a zero
Answered by Penny Nom.
Interest compounded daily 2003-10-08
From Terie:
If I owe $2000.00 @ 6.25% compounded daily for 54 days. How much do I owe in interest?
Answered by Penny Nom.
Standard Deviation 2003-10-07
From Rebecca:

I have a task to complete, which is to calculate the mean and standard deviation of something. I have done this but am then asked to write a short explanation of my findings.

I know what the mean is about, and I thought I knew what the standard deviation meant too - shows the variation from the mean. However, on a task I completed earlier the feedback I got said 'you need to tell us that it is talking about the middle 66% of the data' - that has thrown me, I don't understand that. Can anyone help me get my head round this???


Answered by Penny Nom.
Two chords 2003-10-07
From Lori:
Chords AB and CD of circle O intersect at E. If AE=4, AB=5, CE=2, Find ED.
Answered by Penny Nom.
The sketch of a graph 2003-10-07
From A student:
I was wondering how do you figure out if a graph has a horizontal tangent line. One of my homework problem was to sketch the graph of the following function; (4/3)x3-2x2+x. I set f''(x) ( the second derivative) of the function equal to zero and got the inflection point:(1/2,1/6). Also i am having trouble finding the concavity for x>1/2 and x<1/2, i am getting a different answer from the back of the book, the graph i draw looks completely different from the correct answer.
Answered by Penny Nom.
Arrangements around a circular table 2003-10-07
From Lori:
How many arrangements can 6 people be seated round a circular table?
Answered by Penny Nom.
Indeterminate forms 2003-10-06
From A teacher:
Is it possible for me to find any geometrical interpretation without using calculus to explain indeterminate forms?
Answered by Chris Fisher.
Functions, graphs and derivatives 2003-10-05
From Jathiyah:
I wanted to know how would you tell (on a graph diplaying two funtions), which funtion is the derivative of the other?
Answered by Walter Whiteley.
The slope of a tangent 2003-10-01
From A student:

find the slope of the tangent to each curve at the given point

f(x)=square root 16-x, where y=5


Answered by Penny Nom.
A rectangle in a circle 2003-09-27
From Abdu:
A rectangle ABCD is inscribed in a circle. If the length of AB is 5 and length of BC is 12, what is the area of circle outside the rectanlge ABCD?
Answered by Penny Nom.
Relatively prime 2003-09-27
From James:
what is the definition a relative prime numbers?
Answered by Penny Nom.
Absolute value 2003-09-23
From Crystal:
I am trying very hard to understand Absolute Value, and I cannot seem to grasp the idea. I do not know why we use it, what the point of it is. I do not know how to use it. I know that it is always positive or zero, and I assume it relates to distance, but how can you use it in equations. Please explain it to me as simply as possible, thank you.
Answered by Walter Whiteley.
The golden ratio 2003-09-23
From Phillip:
The Golden Section can be made from an equilatereral triangle inscribed within a circle. The Golden Section is achieved by joining the mid points of two arms of the triangle to the circumference. I can prove this by erecting a perpendicular to the line outside the circle, but am interested to see how it can be proved from within the circle.
Answered by Chris Fisher.
Applications of logarithms 2003-09-16
From A student:
I have a project due in my algebra two class dealing with logs and where they are used in life, but I am having trouble finding websites that relate. So I would really appreciate it if you couldhelp me.
Answered by Penny Nom.
Substitution method 2003-09-14
From John:

Hello, my name is john and while I was finishing up my work I stumbled into these problems. a/4-b=-1
a+b=11
and
6a-b=-5
4a-3b=-8

Thanks a lot. I really need some help :)
John


Answered by Penny Nom.
The volume of an earthen pit 2003-09-12
From Bruce:
This shape would occur at the four corners of a rectangular shaped earthen pit with sloping sides (1:1.5). Depth is 14'. Top dimensions are 170' by 158'. After calculating the easy volume components of this shape, we are left with the end corner pieces. 21' base, 14' height, side hypotenuse 25.24' and corner diagonal 32.83'. We're confused. Thank you for any help you may provide.
Answered by Penny Nom.
The digits in 349 2003-09-12
From Franik:
in the number 349, which number has the least value, and why?
Answered by Penny Nom.
Two line segments and a plane 2003-09-11
From Laura:
Do all figures made up of two segments lie in a plane?
Answered by Penny Nom.
Area of a rectangular room 2003-09-11
From Kelly:
The area of a rectangular room is 238 square feet. The width is 3 feet less than the length. What are the dimensions?
Answered by Penny Nom.
The general equation for a sphere 2003-09-11
From Jaidev:
Is there any general equation for a sphere?
Answered by Penny Nom.
The mean house price 2003-09-10
From Carol:

Question:
I have to find the mean from the following example:

Price Range £000 No of Houses
55 and under 60 3
60 and under 65 6
65 and under 70 13
70 and under 80 21
80 and under 100 15
100 and under 130 7
130 upwards 1

I know when calculating the mean you use the mid points of the classes, but how does this work for the 130 upwards class? Also, does this still work given the difference in the classes (ie. 1st class is 5, 5th class is 20, etc). Any help to get me started would be greatly appreciated.


Answered by Penny Nom.
How high does the sequence of numbers go 2003-09-07
From Shayna:
My quetion is "How high to the sequence of numbers go"?
e.g. one, ten, hundred, thousand, million, billion, trillion, and from there is my queston.

Answered by Penny Nom.
7,473,000,000 divided by 52000 2003-09-06
From A student:
I am having trouble with a question. 7,473,000,000 divided by 52000. Our calculater gets 147311.5385. I come up with 143711.53846, and then the number starts to repeat itself
Answered by Penny Nom.
Can 2 vertical planes intersect? 2003-09-06
From Erin:
My question is can 2 vertical planes intersect?
Answered by Penny Nom.
One million seconds 2003-09-04
From Vince:
If you can write a number each second for one million seconds, how many hours would it take?
Answered by Penny Nom.
A 16 foot ladder 2003-09-04
From Kelly:
Two buildings are separated by a three-foot alleyway. Fatima wants to use a 16 foot ladder to reach a window in the wall of one of these buildings. If she places the foot of the ladder against the base of the other building, how far up the wall will the top of the ladder reach??
Answered by Penny Nom.
One percent of a billion 2003-09-04
From Charles:
What is one percent of one billion?
Answered by Penny Nom.
A boat travels 12 km upstream... 2003-09-03
From Sohaib:
A boat travels 12 km upstream and back in 1 hour 45 minutes. If the speed of the current is 3km/hr throughout, find the speed of the boat in still water, giving your answerÝcorrect to 3 decimal places.
Answered by Penny Nom.
Why percentage? 2003-09-03
From Vicky:
My cousin aske me a question about the use of percentages and I'm not sure how to answer it. He wants to know why when there is a sale in a shop we use percentages and not fractions or decimals.
Answered by Penny Nom.
A helicopter rises vertically 2003-09-02
From Kate:
A helicopter rises vertically and t seconds after leaving hte ground its velocity is given in feet per second by v(t) = 8t + 40 / (t+2)2 How far above the ground will the helicopter be after 3 seconds?
Answered by Penny Nom.
Water in a tank 2003-09-02
From A parent:
if a tank is 48 inches long 14 inches wide and 18 inches deep if a yardstick was inserted into the tank and measured 2 inches of water in the tank how many gallons of water would be in the tank.
Answered by Penny Nom.
A theorem in geometry 2003-09-02
From Diego:
Please refer to figure in attached file. P is a point on the chord AB of a circle such that the tangent PT which touches the circle at T is equal to AB. How do we prove that PT2 = AP x BP.
Answered by Dieter Ruoff and Penny Nom.
The units digit of 57 to the 28th power 2003-09-01
From Maria:

If 57 is raised to the 28th power, what must be the units digit in the result?

If you raise 7 to the 500th power, what is the numeral that will appear in the units digit position?


Answered by Penny Nom.
A 6 team schedule 2003-08-31
From A scheduler:
I saw that someone put on your web site a team schedule and you helped them figure it out.Ý I have 6 teams that want to play each other once, and believe it or not, I cannot figure it out.
Answered by Penny Nom.
Terminology 2003-08-31
From Maria:
My daughter Veronica is a rising 6th grader and has to complete some Summer Math
assignments and would like to ask you three questions:
  1. ___________ are number pairs that have a product of 1.

  2. You can name any point on a plane with two numbers. These two numbers are called _____________.

  3. A _______________ is the size of a cube that is exactly 1 inch on each edge.
Thanks,

Answered by Penny Nom.
The quadratic formula 2003-08-31
From Alex:
I am using Houghton Mifflin's Precalculus with Limits book, 2nd edition. However, the first chapter encompasses Algebra review, and I am stuck on a problem. All that's required is to solve the following and verify using a calculator:
3y2+6y+2=0
I have solved the problem using the quadratic formula, but from what I remember, the quadratic formula is used in the case of equations following the AX2+BX+C=0 pattern. As the problem I am attempting uses a y-variable, can I still use the quadratic formula? Since I am not sure what route to take in solving this problem, I am hoping you can assist me.

Answered by Harley Weston.
Newton's binomial theorem 2003-08-30
From William:
According to page 126 of Murtha & Willard's "Statistics and Calculus" (Prentice-Hall, 1973), Newton's binomial theorem can proved inductively. I suppose that was his method, which I would like to see.
Answered by Penny Nom.
I can Guess your birthday 2003-08-29
From Michelle:
Hi I am trying to explain to my children how this problem works. It was sent to me on the internet and I can not figure it out. They keep asking me how it works and I can not tell them.

The problem is: "I can Guess your birthday::


Answered by Penny Nom.
How many cubic yards must I purchase? 2003-08-29
From Walt:

I am trying to determine how much top soil I must purchase (by the yard).

The area is 80' x 15' and I want it 3" thick. How many cubic yards must I purchase?


Answered by Penny Nom.
Simplify 2003-08-24
From Jessica:
I have been asked to simplify the expression (5a2)(-2b2)(3b)+(7a2)(2b2)+(-3a)(-5a)(4b) and the answer is apparently 44(a2)(b2). For the life of me I can't figure out how they derive this.
Answered by Penny Nom.
A rule with a variable 2003-08-24
From Crystal:
what is a rule with a variable
Answered by Penny Nom.
X.9999... and X+1 2003-08-23
From David:
I have read your answers to the questions on rational numbers, esp. 6.9999... = ? and still have a question: The simple algebraic stunt of converting repeating decimals to rational numbers seems to work for all numbers except X.999999.... where X is any integer. The fact that the method yields the integer X+1 in each case seems to violate the completeness axiom of the real numbers, namely that there is no space on the number line which does not have an number and conversely that every geometric point on the number line is associated with a unique real number. In the case of 3.999... for example, it seems that both the number 4 and the number 3.9999.... occupy the same point on the number line. How is this possible???
Answered by Penny Nom.
A golf tournament 2003-08-23
From Dale:
I am running a golf tournament with twelve golfers. We will be playing four rounds of golf. I don't want to schedule the same two players together more than twice (if possible). How would I make a schedule that each player would play at least once with every other player? I would appreciate any help that you can give me.
Answered by Denis Hanson.
A worm on a hill 2003-08-21
From Lisa and Samantha:
A WORM IS AT THE BOTTOM OF A 10 FOOT HILL. HE CRAWLS UP THE HILL 4 1/2 FEET A DAY. AT NIGHT WHEN HE REST HE SLIDES DOWN 2 1/2. HOW LONG DOES IT TAKE THE WORM TO CRAWL UP THE HILL.
Answered by Penny Nom.
Two equations 2003-08-21
From Jennifer:

The following problems have me stuck and any help would be greatly appreciated seeing my text book is doing me no favours:

32a+1 - 283a + 9=0
and
52m - 5m=20


Answered by Penny Nom.
39 consecutive natural numbers 2003-08-19
From A student:
Prove that among any 39 consecutive natural numbers it is always possible to find one whose sum of digits is divisible by 11.
Answered by Penny Nom.
The volume of a fuel tank 2003-08-16
From Phil:
I hope you can help me out with a small problem. My boat has a fuel tank measuring 1680mm long x 630mm wide x 130mm high. Can you tell me how much fuel it will hold in both litres and imperial gallons.
Answered by Penny Nom.
The number of three digit combinations 2003-08-16
From Fred:
There are four close friends having a heated discussion concerning the the number of three digit combinations using the numbers 0 through 9. The discussionÝinvolves the daily lottery.Ý Any assistance you can lend would be greatly appreciated.
Answered by Penny Nom.
Making a blanket 2003-08-13
From Liz:
Please help me figure this out. I am making a blanket the directions call for a piece of material measuring 72"X45". Instead of one piece I'd like to make a patchwork quilt but the squares need to equal 72"X45". Centered on the bottom of the longest edge is an 18" square.
Answered by Harley Weston.
Water in a cone 2003-08-12
From Adrienne:

Water is poured into a tank in the shape of an inverted right circular cone.ð The height of the tank is 8 m and its radius at the top is 4 m.

a. Draw and label a picture to represent this situation.ð (I know how to do this)

b. Identify all variable quantities. (h = 8m, r = 4m)

c. Find an equation that relates the variable quantities, and reduce the number of variable quantities to two.

I was thinking about the equation V = 1/3 pi r2 h, which is the Volume of a cone, but I am stumped as to how I am supposed to "reduce the number of variable quantities to two." Can you point me in the right direction?


Answered by Penny Nom.
Yards and cubic feet 2003-08-11
From Sonya:
How many cubic feet are in a yard?
Answered by Penny Nom.
The side length of a regular octagon 2003-08-09
From Rich:
If I know the distance between the parallel lines of an octagon and want to find the length of the sides, can I just divide that distance by 2.414 to reveal the length of the sides? I work construction and just curious if I am figuring it right.
Answered by Penny Nom.
Liters in a pool 2003-08-05
From A parent:
I need to know how many litres in a 12 foot diameter, 3 foot deep pool? Any help much appreciated as I need to add the correct amount of chemicals
Answered by Penny Nom.
Diagonals in an octagon 2003-08-04
From Amanda:
How many diagonals are in an octagon??? What would be a drawing of those diagonals???
Answered by Penny Nom.
Two precalculus problems 2003-08-04
From Kate:

Please help me verify the identity:
cos2x(sec2x-1)=sin2x

Also I am having trouble withdetermining whether f(x) is odd, even, or neither
f(x)=x3-x


Answered by Penny Nom.
The incline of a road 2003-08-03
From A driver:
incline- % grade as to degree. ie 6% hwy grade= what degree of incline?
Answered by Penny Nom.
Cutting some wood 2003-07-25
From Betty:
My husband is building a six sided wood circle and would like to know the angle to cut the pieces.
Answered by Penny Nom.
The area of an odd shaped piece of property 2003-07-24
From Barry:
What is the formula for figuring the square footage of an odd shaped piece of property?
Answered by Penny Nom.
Natural logarithms 2003-07-22
From Amanda:
I'm going into my senior year of high school. I will be taking AP calculus, and my teacher gave us some homework over the summer. However, there are two things that I do not understand how to do. The first is, she wants us to be able to generate a unit circle by hand using 30, 60 and 90 degree triangles. I have used the unit circle in trigonometry, however I was never taught how to draw it. Secondly, I need to know how to do natural logarithms without a calculator. I was not taught how to do this, and the worksheet I was given only showed me how to complete them using a calculator.
Answered by Claude Tardif.
Slope 2003-07-20
From Brian:

You are placing a pipe 60 feet long at a slope of 1.5%. How would you figure the amount of drop from the inlet of the pipe tot he outlet of the pipe? and what is the answer .

If you are trying to keep a shoulder slope of a road between 3/4'' per foot and 11/2'' per foot . If the shoulder is 10 feet wide, how would you figure how much lower should the outside of the shoulder be than the edge of pavement and what is the answer ?


Answered by Penny Nom.
|x+3| = -15x 2003-07-19
From Nathan:
Without the use of trial and error, is there a way of solving for x where |x + 3| = -15x? I know that x = -0.1875, but how do I find whether x + 3 is positive or not if I did not have this information (In other words, how would I solve whether the x-value was -3 or less, or if it was more than -3)?
Answered by Claude Tardif and Penny Nom.
"cubic feet" of a refrigerator 2003-07-16
From Ron:

Is the volume of a home appliance (e.g. fridge or freezer) calculated in a different way? I am planning to replace both my fridge and my freezer but I'm unable to determine what size replacement to get.

For example, one company descibes their fridge as being 32"(L) x 30"(W) x 66" (H). I converted those measurements to feet (2.67 x 2.5 x 5.5) and I calculate the fridge to be 36.7 cubic feet. However, the company says the fridge is 18 cf.


Answered by Chris Fisher.
Cathy's Clothing 2003-07-15
From Cathy:
I am supposed to write a practical and useful word problem that would use the following algebriac inequality to solve it: 12x + 25 > 15x + 10. State the quanity that represents x and solve the inequality.
Answered by Penny Nom.
Preparing 100ml of a 30% alcohol solution 2003-07-13
From Justin:
My question is how do you prepare a 100ml of a 30% alcohol solution using 95% alcohol?
Answered by Penny Nom.
A problem with lugs 2003-07-11
From Lori:
A young man's car developed a flat tire as he was driving along a deserted street. He pulled over to the curb and did all the usual things: removed the hub cap, unscrewed the lugs and rested them carefully in the hub cap, jacked up the car. As he was putting the spare tire onto the axle he accidentally kicked the hub cap. The lugs rolled out, and all five of them fell down a nearby grate. Peering through the bars of the grate the man thought that he could see the lugs about 6 feet below in a shallow water puddle. He had a problem. How do you think he solved it?
Answered by Penny Nom.
A mixture problem 2003-07-10
From A student:
Having a problem with word problems. example A chemist mixes a 10% acid solution with a 50% acid solution to form 400mL of a 40% solution. How much of each solution should be used.
Answered by Penny Nom.
A triangle in the complex plane 2003-07-10
From Scott:
The vertices O and A of an EQUILATERAL triangle OAB in the complex plane are located at the origin and 3 + 3i. Find all possible values for the complex number representing the vertex B. Give the location of B in both polar and cartesian form (to 2.d.p)
Answered by Penny Nom.
E(X + Y) and V(X + Y) 2003-07-10
From Reuben:

I'm a teacher trying to find a way to make some stats work from first principles. The topic is expectation algebra and it is for the top age level in high school.

By using a set of data I can show how the mean is the same as the expected value ie 2,2,3,4,4 is 15 / 5 = 3 also, 2x0.4+3x0.2+4x0.4=3. I can also show this to work for the variance in the same style but using Sum(x-mean)2/n and the Var(X) version of squaring x then multiplying by the probability ( all this for random independant samples).

Now comes the tricky bit.....when I try to show E(X+Y) = E(X) + E(Y) from setting up two data sets I get it to work only if I add each item from X to each item from Y. However, I can't get it to work for V(X+Y)=V(X) + V(Y) from two sets of data.

There must be something missing in my knowledge of how the sets are required to add together or my knowledge of expectation algebra. I am familiar with proofs but still want to show my students that the basic formulae work from groups of data whether done the long way or by use of formulae. Can you help please?


Answered by Penny Nom.
An onsale DVD player 2003-07-09
From Kim:
I have a DVD player with a regular selling price of $175 and percentage of discount is 10 percent. How to solve for discount amount and sale price? Also, when you figure out the sale price how to figure out commission amount when percentage of commission is 5 percent?
Answered by Penny Nom.
A wooden deck around a pool 2003-07-09
From Dat:
A pond is enclosed by a wooden deck that is 3 feet wide. The fence surrounding the deck is 100 feet long.
  1. If the pond is square, what are its dimensions?

  2. If the pond is rectangular and the length of the is three times its width, what are the dimensions of the pond?

  3. If the pond is circular, what is the diameter of the pond?

  4. Which pond has the most area?

Answered by Penny Nom.
Is 3/5 CLOSEST to 0, 1/2 or 1? 2003-07-09
From A student:
I would like to know is 3/5 CLOSEST to 0, 1/2 or 1. And can 3/10 go either way as to be closest to 0 or 1/2.
Answered by Penny Nom.
A geometry problem 2003-07-08
From Chris:
My name is Chris, I work for a custom fabricator company. I am needing a formula for the Height (H) shown in the attached picture. The picture shows dimensions for my current application. If you could please, assign variables to the dimensions.
Answered by Harley Weston.
Volume of water in a pool 2003-07-06
From An other:
How many litres of water are in a pool that is 16 feet in diameter and 35 inches deep?
Answered by Penny Nom.
The length of fabric in a roll 2003-07-03
From Paul:
We receive rolled fabric as part of our process and I would like to know is there a formula that could be used to estimate the overall length of the roll with out rolling the fabric out and measuring its length.
Answered by Harley Weston.
Contradiction in the Procedural Precedence and the Distributive property 2003-07-03
From Arthur:
When complicated expressions exist on both sides of a fraction, and both these expressions contain like and unlike terms with "literal" coefficients, the procedural precedence suddenly gets extremely confusing and seems impossible to simplify. This seems confusing and impossible to simplify because there is a HUGE contradiction in the way Procedural Precedence and the Distributive property deal with Parentheses. The Procedure says you MUST deal with what's INSIDE the parenthesis FIRST, BEFORE dealing with ANYTHING on the outside where as the Distributive property contradicts that by saying to go ahead and ignore the Precedence and use a factor on the outside of the Parenthesis.
Answered by Claude Tardif.
C is 10 more then the square of... 2003-07-03
From Carolina:

C is 10 more then the square of the quotient of the sum of x and y , and the product of x and y. Which equation expresses this relationship?

A. C=(x+y +10)^2        ___        xy  
B. C=(x+y)^2 + 10        ___        xy  
C. C=(xy)^2+10           ___       x+y  
D. C=(x+y)^2 + 10        ___        xy 


Answered by Penny Nom.
The domain of 1/g(x) - 5 2003-07-03
From Barbara:
If the range of g(x) is ( neg. infinity,4] and the domain of g(x) is ( neg. infinity, infinity), how do I find the domain of 1/g(x) - 5?
Answered by Penny Nom.
How much soil will be removed? 2003-07-02
From Amy:
My site is 1230' (east-west) by 1300' (north-south) and we need to remove 2 feet of soil from the entire site, how much soil will be removed?
Answered by Penny Nom.
Setting up a word problem 2003-06-30
From Christie:
A rectangular corral is forty feet longer than it is wide. If the perimeter is 220 feet, determine the length and width of the corral.
Answered by Penny Nom.
A ruler and compass construction 2003-06-29
From Mandy:
I have been searching for instructions on how to do this using a compass. Can you please send instructions step by step on how to bisect a segment into any given number of segments. (example, a way that it will work for bisecting segment into 3, 4, or even 5 congruent parts)
Answered by Penny Nom.
Solve x = y^z for z 2003-06-28
From Nathan:
If x = y^z is there a way to solve for z, if x and y are given, without guessing or already knowing the answer?

ie: 64 = 2^z
6 is the obvious value for z. Is there a way I could solve for this variable using a formula, which would also apply when more complex values were substituted for x and y?

Answered by Penny Nom.
Polynomials degree 4 and bigger 2003-06-26
From Stephen:

How do I show kids how to find all the zeros for polynomials degree 4 and bigger.

For examples:
r(x) = x5-11x3-7x2+77 = (x2-11)(x3-7) and s(x) = x4-121 = (x2-11)(x2+11)

Also am I correct when I say that the following are irreducible?

f(x) = x2-11
g(x) = x3-7
h(x) = x4-5


Answered by Harley Weston.
Odd powers of sine and cosine 2003-06-25
From Antonio:
Can you please tell me how to integrate a trig function involving sine and cosine? I know if the powers of both the sine and cosine are even and nonnegative, then I can make repeated use of the power-reducing formulas. But for the question I have on my hand, the powers of both sine and cosine are odd: ( sin3x + cos7x ) dx.
Answered by Harley Weston.
Sesame Street on Stage 2003-06-25
From Christie:
Sesame Street on Stage has different ticket prices for adults and for children under 6. Beth and her two children, ages 3 and 4, paid $15 to gain entrance, while Jamie and John Jacobs and their 2-year old daughter got in for $19.50. What is the ticket price for adults?
Answered by Penny Nom.
Interest compounded daily 2003-06-24
From Jeff:
What would be the amount of interest charged or accrued and how is it calculated on principal balance of $209.12 @ 6% interest rate compounded daily for 70 days?
Answered by Penny Nom.
pst and gst 2003-06-24
From Robin:
I need to find an easy solution to remembering how to calculated the gst and pst once I have the total amount. ex: my total is $154.40, I have to find the gst and then the pst. I live in bc so the taxes are 7% for gst and 7.5% for pst.
Answered by Penny Nom.
Flim 2003-06-24
From Dan:
I have a 4th grader and the word (Flims) came up in her summer text,
Answered by Penny Nom.
An inch and a millimeter 2003-06-21
From Greg:
The conversion factor for inches to millimeters 25.4 It strikes me as very odd that it's not some infinite decimal. I could understand this if one system was derived from the other but as I understand it they were derived independently. Is the conversion 25.4 or 25.4000XXXXXXX? If it's just 25.4 then how did that happen?
Answered by Harley Weston.
The sum of two numbers is 5 and their difference is 2003-06-16
From Akhil:
The sum of two numbers is 5 and their difference is 11. What is the product of the two numbers?
Answered by Penny Nom.
Two trig problems 2003-06-10
From Bett:

I have this ongoing trouble with trig and solving triangles with laws of cosines and sines!! For example if it asks to solve triangle FGH, given angle G=102.7 , side f=14.2, and h=18.6. Now do I use law of cosines because I don't have the measure of an angle and length of the opposite side??I don't know where to go from here,I am totally confused!!!

I also have a problem with this word problem I have been doing. It asks: An airplane flies 847.5 km at a bearing of 237.3 degrees. How far south and west fo its original position is it? Huh? Please help!


Answered by Penny Nom.
How many gallons are in 83 liters? 2003-06-09
From Anita:
How many gallons are in 83 liters?
Answered by Penny Nom.
Definitions and descriptions 2003-06-08
From Tammy:
MY DAUGHTERS TEACHER ASKED HER TO GIVE BOTH A DESCRIPTION AND A DEFINITION OF THE FOLLOWING ... CIRCLE, SQUARE, TRIANGLE,HEXAGON...... THE LIST GOES ON. WHAT IS THE DIFFERENCE BETWEEN DEFINITION AND DESCRIPTION ? DO A CIRCLE FOR AN EXAMPLE PLEASE.
Answered by Penny Nom.
The metric and Imperial systems 2003-06-08
From Ashlee:
i am a year 8 student, and i am doing my Maths talent quest on the Metric and Imperial number system.

i was wondering, where did the 2 systems orginate and if possible, a little bit or information.

I also need to know the conversions of ounce, foot, mile..etc..from imperial to metric.

Answered by Penny Nom.
A radio controled car 2003-06-08
From Robert:
I have a radio controled car (scale size 1/43). This car can travel 490 ft. per minute. I would like to know how fast that is at that scale size in miles per hour (and Kilometers per hour)?
Answered by Penny Nom.
Spacing the spindles for a railing 2003-06-06
From Jennifer:

Scott is a homebuilder. He builds railings in which he places spindles. Spindles ar evertical posts taht are equally spaced beneath a horizontal bar. Scott would like a mathematical model to help him determine the amount of space to put between each spindle. The railing must follow the following criteria: The spaces between each spindle must be equal except for the ones at either end. Theses spaces are smaller. They are half the width of a spindle less than the other spaces. The number of spindles needs to be minimized since spindles are costly.

Help Scott determine what width of space to use between the spindles. Create a mathematical model to determine the width of space between each spindle in termso f the number of spindles. Use an 8' (96") railing that has 2 1/2" wide spindles. Explain your thinking. For safety reasons the maximum width of a space is 4"

Generalize you model to determine the width of spaces for total railing of length L and spindle width s


Answered by Penny Nom.
The square footage of a section of land 2003-06-04
From Diane:
We are trying to calculate the square footage of a section of land with the following dimensions:

One end is 100 feet. The other end is 105 feet.. One side is 350 feet and the other side is 372 feet. I calculated the area as 35,875 sq. feet, however, the brochure for the land says the area is 36,200 + sq. feet.

Answered by Leeanne Boehm.
$1/week for week 1 and then double each week 2003-05-28
From A student:
I AM DOING A PROJECT FOR MATH I AM TRYING TO CREATE A SEQUENCE BASED ON: MAKING A SALARY OF $1/WK FOR THE FIRST WEEK WITH IT DOUBLING EVERY WEEK FOR 52 WEEKS, SO MY SEQUENCE LOOKS LIKE THIS: 1,2,4,8,16,32,64,128,256,512,1024 ETC. THERE WOULD BE 52#'s TOTAL IN MY SEQUENCE.I BELIEVE IT IS CALLED A FINITE SEQUENCE. MY QUESTION IS HOW DO I CREATE A FORMULA/RULE FOR THIS PARTICULAR SEQUENCE? MY GOAL BEING TO FIND THE 52nd TERM IN THE SEQUENCE.
Answered by Penny Nom.
Millimeters to inches 2003-05-28
From Brian:
I am a music instructor just trying to get a horn repaired. I have been trying to get replacement pearls for a keyboard of a horn. The mechanic needs the measurement in inches. The calibers say that the size is .115 mm can you tell me what that is in inches, I tried your formula of 2.54 mm per inch but this is a smaller size and I cannot seem to figure it out.
Answered by Penny Nom.
George and his goat herd 2003-05-27
From A student:

George the bulgarian goatherd drives his father's goats into a valley each morning and lets them browse there all day before driving them home in the evening.

he notices that each morning the goats immediately separate into groups and begin to feed the number and sizes of the initial groups vary some days there are nine and so days there are three or fewer. there can be groups of one or the whole group.

about every five minites one goat breaks away from each feeding group and these breakaway goats form into a new group.

George has noticed that by the afternoon, even though the goats continue their regrouping the sizes of the groups have stabilised, and there is always seven feeding groups.

How many goats are there in the herd? What are the sizes of the feeding groups once they have stabilised?


Answered by Penny Nom.
The cross-section of a football field 2003-05-25
From Francis:
Have you ever walked on a football field covered with artificial turf? If so, you probably noticed that the field is not flat. The profile of the surface is arched and highest in the centre, permitting rainwater to drain away quickly.

height from base to highest point- 45.75 centimetres distance of the field- 50 metres

a) The diagram shows the profile of an actual field, viewed from the end of the field. Assuming that the cross-section is a parabola, find the algebraic model that describes this shape.

b) Use your equation to determine the distance from the sidelines where the field surface is 20 cm above the base line.


Answered by Harley Weston.
The weight of a fish tank 2003-05-20
From A ponderer:
I had a question pertaining conversion units..I have a fish tank at home and the dimensions are 24, 44, and 48 inches, I want to find out how much it weighs in pounds when filled up with water...knowing water is 62.4 pounds per cubic feet...do I just divide the 62.4 pounds to 12 because I"m trying to find out through cubic inches.
Answered by Penny Nom.
The HCF and LCM of polynomials 2003-05-20
From Charanpal:

Question:
Find the HCF and LCM of the polynomials given below. Verify that he productof these HCF and LCM differs from the product of the polynomials, if at all, by a factor of -1

  1. 1-x2 and x3 -1
  2. 1 - x2 and x4 - 1

Answered by Penny Nom.
A trig identity 2003-05-20
From Patty:
Please help with the following

1/ tanx + cotx = sinxcosx

Answered by Penny Nom.
R= 5.3lnx + x. 2003-05-11
From A student:
Reaction R to a dose x is given by: R= 5.3lnx + x. For a certain drug, R must not exceed 21. Show that a dose between 8 and 12 units satisfies this requirement & find, correct to 5 decimal places, the greates value of x which satisfies this condition?
Answered by Harley Weston.
Subtracting rational expressions 2003-05-10
From Simone:
hi, i'm totally lost. i understand that you need to find a lowest common denominator to subtract two fractions (rational expressions) with different denominators. but what if the denominators are "x-1" and "x". is x the common denominator? if so what happens to the "-1"? do you know of any live online help i can get with the following:

3/(x-1) - (1-2x)/x

i've looked through my notes and have no examples that quite match that i can follow to get through it. please help!
Answered by Penny Nom.

x-6square root of x +8=0 2003-05-10
From Elizabeth:
x-6square root of x +8=0
Answered by Penny Nom.
Circumference 2003-05-09
From A parent:
Find the circumference use 3 1/7 for pi

1. r= 28 ft.
2. D=98 cm

Answered by Penny Nom.
Real numbers 2003-05-09
From Sirena:
what is a "real" number
Answered by Penny Nom.
ABCD*4 = DCBA 2003-05-07
From Dee:

In the following problem, if the letters A, B, C and D represent some number, what numbers would they represent so that:

ABCD*4 = DCBA


Answered by Penny Nom.
Rules of exponents 2003-05-05
From Carl:
Hi, I am a student who would like to recall how to multiply exponents. Here is such an equation:
6.02569 X 1025 X 5.254 =?

Also, adding exponents. Don't I just add subtract the exponents separately?
Such as 523 +15-12 =??

Answered by Penny Nom.
Write sin(3x) in terms of sin(x) 2003-05-05
From A student:
Write sin 2x in terms of sin x
Answered by Penny Nom.
Four digit palindromes 2003-05-05
From Heather:
How many palindromes were in between the numbers 1000 and 9999?
Answered by Penny Nom.
Integrating e^x sin(x) 2003-05-03
From Lech:
I am having trouble integrating the following expression by parts:

ex sin(x)

I used the integrator at http://www.integrals.com/ to find the solution,

? 1/2 ex cos(x) + 1/2 ex sin(x).

This is easy to confirm by differentiation, however I am confounded as how to arrive at the answer.


Answered by Penny Nom and Claude Tardif.
The volume of air flowing in windpipes 2003-05-02
From James:
The volume of air flowing in windpipes is given by V=kpR4, where k is a constant, p is the pressure difference at each end, R is the radius. The radius will decrease with increased pressure, according to the formula: Ro - R = cp, where Ro is the windpipe radius when p=0 & c is a positive constant. R is restricted such that:
0 < 0.5*Ro < R < Ro,
find the factor by which the radius of the windpipe contracts to give maximum flow?

Answered by Penny Nom.
A circle, tangent to two circles and a line 2003-04-30
From Keith:
I have a horizontal line (that is treated as a datum line or the X axis), with two circles having their center points at different heights from that line (X1,Y1 & X2,Y2). The two circles are also at different diameters (R1 & R2). Both circles and the line (X-Axis) do not intersect nor are they tangent. My goal is to determine the maximum diameter of an inscribed circle that will fit between all three.
Answered by Chris Fisher and Harley Weston.
A binomial probability 2003-04-30
From Erica:
In a biathlon, athletes shot at 20 targets. A particular athlete had a probability of 0.18 of missing a target. Assuming a binomial distribution, what is the probability, to the nearest thousandth, that the athlete hit exactly 16 targets?
Answered by Andrei Volodin.
Some shapes 2003-04-29
From Suzanne:
  1. What shape has 12 edges all the same length?

  2. What shape has 3 surfaces and 1 is curved?

Answered by Penny Nom.
A skill testing question 2003-04-27
From Allan:
it is a contest Labatt Blue can you please give me the answer some people are saying 154 and some 34
Answered by Leeanne Boehm and Harley Weston.
Gallons of rain 2003-04-25
From Jocelyn:
If you have a surface of 6500 square feet and 1 inch of rain falls on the surface, how many gallons of water does this equal?
Answered by Penny Nom.
38 minus 10 plus 12 divided by 4 times 16 2003-04-23
From James:
please help me work this math problem 38 minus 10 plus 12 divided by 4 times 16
Answered by Harley Weston.
Two intersecting graphs 2003-04-23
From Patty:

a) graph the equation

x2 - y - 4 = 0

x2 + y2 = 9
on the same set of coordinate axes.

I did not have a problem with this.

The problem is part (b) of the question ask: Find all solutions of the system in part (a) algebraically. Express answers in decimal form, accurate to two decimal places.


Answered by Penny Nom.
8 faces, 12 vertices, and 18 edges 2003-04-22
From Thomas:
I would like to know the proper name for this geometrical solid. It has 8 faces, 12 vertices, and 18 edges?
Answered by Penny Nom.
The effectiveness of a drug 2003-04-22
From A student:
A certain drug is found to be effective 80% of the time. Find the probability of successful treatment in two out of four cases.
Answered by Andrei Volodin.
A tangent to a circle 2003-04-18
From Lech:
The line with equation y=mx is a tangent to the circle with equation x2+y2-6x-6y+17=0. Find the possible values of m.
Answered by Harley Weston.
Interest compounded daily 2003-04-18
From Bruce:
I recently received $7,987.68. This amount consisted of $7,543.61 invested at 6% per annum, compounded daily. I am trying to find out how many days this investment accrued interest.
Answered by Penny Nom.
The substitution method 2003-04-14
From Patty:

I need help with the following:

5x - 4y = 13

2y + 3x = -1


Answered by Penny Nom.
The square of my age was the same as the year 2003-04-14
From Pat:
Augustus de Morgan wrote in 1864, "At some point in my life, the square of my age was the same as the year." When was he born?
Answered by Penny Nom.
Area of a trapezoid 2003-04-13
From A road builder:
My husband works with asphalt building roads. There are times when one end of the road will be (for example) 100ft wide the other end would be( for example) 200ft wide and he must figure the area in square feet. So far it has been a guessing game because he dosen't have the formula to figure the square feet.
Answered by Penny Nom.
A fractional inequality 2003-04-08
From Jessica:
Solve for x. Write in interval notation.

[(x2)-9]/[x-5] >= 0

Answered by Penny Nom.
The surface area of a cube 2003-04-07
From Sarah:
if the volume of a cube is 64 cm, then what's the total surface area? I also want to know how to show my work.
Answered by Penny Nom.
Jo's books 2003-04-07
From Miguel:
Jo announces, "I have more than 999 books."
Jean says: "No Jo! You have fewer than 1,000 books."
Mary says: "Jo has at least 1 book."
Only one of these statements is true. How many books does Jo own?

Answered by Penny Nom.
The area of a parallelogram 2003-04-06
From A student:
Find the area of the parallelogram with vertices (0,0),(7,2),(10,7),(3,5)
Answered by Penny Nom.
The height of an equilateral triangle 2003-04-06
From Rosa:
If Each side of an equilateral triangle is 10 m. What is the height?
Answered by Penny Nom.
How many hits? 2003-04-06
From Jack:

My name is Jack. I'm a uncle. Student is in the 5th grade email is above.

If a baseball player at sping training had a good season with the following:

one seventh of his hits were doubles.
12.5% of his hits were home runs.
But didn't have any triples.
How many hits did he have?

Can you give me an explanation of you solved the problem.


Answered by Penny Nom.
Divide a circle in 8 equal pieces 2003-04-04
From Naomi:
I have to divide a circle in 8 equal pieces but can only cut 3 times
can you please help me

Answered by Penny Nom.
Uses of conic sections 2003-04-01
From William:
My name is William and I am doing a research paper on conic sections for my 12th grade math class. Part of the project is to find two conic sections in our world today and explain what there purpose is. I really need help in this area because I've been searching the internet for where conic sections are used in our world today and I really can't find anything. If you can tell me specific building or a pyramid that contains conic sections that would be great. Or even something in the universe would be helpful.
Answered by Leeanne Boehm.
Doubling the size of an object 2003-04-01
From Dave:
If I have a known surface area and volume of an unknown object and I want to double the size of the object, how do I find the new area and volume?
Answered by Walter Whiteley.
A handshake problem 2003-03-31
From Jaylan:
Suppose you and your partner attend a party with n other couples. Several handshakes took place. No one shook hands with himself (or herself) or with their partner, and no one shook hands with the same person more than once. After all the handshaking was completed, suppose you asked each person, including your partner, how many hands they had shaken. Each person gave a different answer.

Determine how many hands did you shake and how many hands did your partner shake when n=5

Answered by Penny Nom.
Quadratic word problems 2003-03-30
From A student:
Question:
i think i know the basics of how to solve direct quadratic equations,but how do i go about solving those word problems? i have tried them times and again, but nothing seems to work at all!they are driving me crazy! pls help!:):):):):)

  1. bob & nick earned $1260 each. bob's hourly wage is $3 more than nick's hourly wage. he worked 2h less compared to nick. find the number of hours nick worked.
  2. there are 2 tanks in henry's house. in tank A, the fish are all goldfish. they are bought at a total cost of $259. in tank B, there are only guppies. the guppies were bought at a cost of $1.20 each. a day later,henry bought 3 more goldfish and put them into tank A. the number of guppies in tank B is now twice the number of goldfish in tank A.the total cost of all the fish that henry has presently,i.e. goldfish & guppies, is $355.30. given that the cost of each goldfish is the same,
    a)find the initial number of goldfish in tank A
    b)find the total cost of all the guppies.
  3. a particular disc can be made to rotate at 2 different speeds. if it is made to rotate slower by 135 revolutions per hour, it wld require an additional 2.7 hour to make 315 revolutions. find the higher speed at which it can operate.

Answered by Penny Nom.
12*nC2 = n! 2003-03-30
From A tutor:
Solve for "N"

12*nC2=N!

Answered by Walter Whiteley.
A royal flush 2003-03-24
From Vikki:

A poker hand consists of 5 cards selected randomly from an ordinary deck of cards: find the probability of a ROYAL FLUSH : the 10 , jack, queen,king and ace of the same suit.

I was thinking somewhere along the lines of:

*the number of ways to get the suit is 4C1
*the number of ways to get a 10 out of the 13 cards etc....
...but Im not sure I am going about this the right way, could you help?


Answered by Andrei Volodin.
Friends and enemies 2003-03-24
From Becky:

Consider a room that contains six people. Any two people are either friends of each other, or they are enemies.

A. Argue why there are three people, all who are friends, or there are at least three people, all who are enemies

B. Rephrase the situation using graph terminology, using all of these terms correctly: vertex, edge, graph, complement, clique, independent set, and bipartite.


Answered by Penny Nom.
A triangle and a circle 2003-03-21
From Jynks:
We need a formula that we can use to figure this out for work. We aren't math wiz's or students. Basically we know 3 points in space of a triangle, we know the length of each side and the length of the line from apex to base line. Each point of the base line ends upon the circumference of a circle. IS three a way to work out the radius of that circle.
Answered by Penny Nom.
Gallons in a tank 2003-03-20
From Leslie:
I am trying to figure the liquid gallons in a water tank that is 19" wide by 29.25" long and is 12.25" high. Could you tell me the formula to figure this out.
Answered by Penny Nom.
A recurrence relation 2003-03-19
From Becky:
Solve this recurrence relation, with the initial conditions.

A1 = 10
A2 = 100
An = 10a n-1 + 29a n-2

Answered by Penny Nom.
The origin of integers 2003-03-19
From Travis:
What are the real life usages of integers? Also what is the origin of integers?
Answered by Harley Weston.
1 followed by a million zeros 2003-03-19
From David:
What do you call the number represented by the numeral '1' followed by one million zeros?
Answered by Penny Nom.
Two algebra problems 2003-03-18
From Kayla:
1st question- If a+b divided by a equals 6 and b+c divided by c equals 9, compute the numerical value of a divided by c. ( Your answer will be mumbers only, no variables.)

2nd question- Three adjacent faces of a box (a rectangular prism) have areas of 7, 14, and 18 square inches. Find the volume of the box. (Hint: Choose variables to represent the three sides of the box and then figure out the problem.)

Answered by Peny Nom.
An equilateral triangle 2003-03-17
From Shirley:
An equilateral triangle is one in which all three sides are of equal length. If two vertices of an equilateral triangle are (0,4) and (0,0), find the third vertex. How many of these triangles are possible?
Answered by Penny Nom.
A sequence that converges to e 2003-03-16
From Dane:
Something I noticed fooling around with a calculator about 30 years ago.
Considering e = 2.718281828459045....
Using Window's Calculator you will find


1.111 = 2.8531167...
1.01101 = 2.731861...
1.0011001 = 2.71964085...
1.000110001 = 2.71841774...
1.00001100001 = 2.7182954...
1.00000110000011 = 2.178231875...
1.000000110000001 = 2.178219643...


There apears to be a pattern. My conjecture is:


1.'infinite number of zeros'11'infinite number of zeros'1 = e.


Answered by Penny Nom.
Distributing r identical balls into n distinct boxes 2003-03-16
From Diana:
How many ways to distribute r identical balls into n distinct boxes with exactly m boxes empty?
Answered by Penny Nom.
An arc of a circle 2003-03-12
From Melissa:
A strip of wood is 16 ft. long and is bent in the arc of a circle. Two radii, from the center of the circle to the ends of the arc, form a right angle. What is the approximate distance from one end of the wooden arc to the other?
Answered by Penny Nom.
Harmonic numbers 2003-03-12
From Becky:
What can you tell me about the limit of harmonic numbers as it reaches infinity?
Answered by Penny Nom.
Monomials 2003-03-11
From Roxy:
Explain why (x+y)z is not equal to xz+yz? P.S. Z is an exponent
Answered by Penny Nom.
Division names 2003-03-10
From A parent:
what is the answer to a division problem called
Answered by Penny Nom.
A question on combinations 2003-03-06
From Jose:

I'm an architect student and have a question on combinations. I have a grid of 3 x 3, hence a total of 9 spaces. I have 3 elements to place in this grid.

How many possible ways are there of arranging this elements on this grid ? (order, orientation not important)

First putting the elements each in its own space and secondly allowing the elements at a given moment to "share" one space.

Since I got kind of obsessed with this I went ahead and graphically did all the combinations allowing "sharing", a grand total of 729. How could I have known this before hand ?


Answered by Penny Nom and Claude Tardif.
(n+17)(n+7) = 0 2003-03-05
From Janea:

What is the answer to 1. (n+17)(n+7)=0

What is the answer to:2. (y-15)(y-100)=0


Answered by Penny Nom.
Can a square be a rhombus? 2003-03-04
From Beth:
Can a square be a rhombus? Some sources say yes, some say no. Some sources define a rhombus as a quadrilateral and parallelogram with equal sides, but without right angles. Some sources say a square is a special case of a rhombus. Clarity, please!
Answered by Walter Whiteley.
y < 2x + 1 2003-03-01
From Erika:
how can I solve and graph y<2x+1 and which side in the graph should i shade after i finish solving my inequality?
Answered by Penny Nom.
______cm=0.048km 2003-02-27
From Antonette:
______cm=0.048km
Answered by Penny Nom.
Can a square be considered a rectangle? 2003-02-27
From Carla:

Can a square be considered a rectangle? (since opposite sides are same length and parallel)

Would a regular hexagon or octagon be considered a parallelogram since its opposite sides are parallel? or does a parallelogram HAVE to have only 4 sides?


Answered by Penny Nom.
A number line 2003-02-27
From Shery:

My seventh grader problem of the month

0__________1______________5____>

This is a arrow, the number should be below

A.Mrs Decker created an arrow representing a number line shown above. She wanted to find points and label them with a heart (G) for Valentine's Day so that the fraction 5/g is less than 1. (be sure to mark the G and not the fraction 5/g). She pondered, "Are there any other locations for G?" Is so help her description the location of all these points. If not why not?


Answered by Penny Nom.
A regular polygon 2003-02-26
From Melissa:
The measure of each interior angle of a regular polygon is eight times that of an exterior angle of the polygon. How many sides does the polygon have?
Answered by Penny Nom.
X times Y equals ten million 2003-02-26
From Baneen:
Would you be able to tell me which two numbers would compute ten million without any number having zeros in it?
Answered by Penny Nom.
A section of land 2003-02-25
From Bev:
How many acres in a section of land? How many square miles is in a secion of land?
Answered by Penny Nom.
The length of an arc 2003-02-24
From Gavin:
does anyone have a formula for calculating the length of an arc if I have the circle radius and the cord length of the arc??
Answered by Penny Nom.
Excluded values 2003-02-22
From Josh:
Why do you think it is necessary to include the "excluded values" when you write your answers to rational expressions?
Answered by Penny Nom.
The number of possible outcomes with 8 games 2003-02-21
From Gianni:
If 8 basketball games being played(no ties), which means a total of 16 teams, what are the total number of possible outcomes that can occur.
Answered by Penny Nom.
Percentage change in completion rate 2003-02-21
From Remo:
In the year 2000 300 out of a possible 600 completed a phone survey for a total of 50% completion rate.

In the year 2001 20 out of 100 possible people completed the survey when it was re-done. This is a completion rate of 20%

To figure out the percentage change between the years can just use the difference between the percentage figures I already have? Or can I calculate it the same as you would for non-percentage numbers. For example you gave an example here of a salary increase from $20 to $95 being an increase of 375%.

Would I solve my problem the same way expect substituting my percentages (.5 and .2) in place of $20 and $95?


Answered by Penny Nom.
Hundreds, thousands, millions,... 2003-02-19
From Karissa and Jasmeen:
hundreds, thousands, millions, - can you help us with the rest of this sequence - we are trying to find the largest number
Answered by Penny Nom.
Sasha's candies 2003-02-18
From Pat:
Tick-tac-toe

Sasha and Trudy are playing tick-tac-toe. Sasha agrees to give trudy two pieces of candy for each game Sasha loses, and Trudy agrees to give Sasha three pieces of candy for each game that Trudy loses. After playing thirty games, Sasha has the amount of candy that he started with. If no ties occurred, how many games did Sasha win?

Answered by Paul Betts.
A determinant 2003-02-13
From A student:

I have to find the determinant of the following matrix

-2 3 1 2 4
-3 0 -2 5 1
4 2 1 -3 5
2 3 4 -1 2
6 0 3 2 -4

Answered by Penny Nom.
Multiples of all four of the numbers 2, 3, 4, and 5 2003-02-11
From Stephanie:
How many of the first 1000 positive integers are multiples of all four of the numbers 2, 3, 4, and 5?
Answered by Penny Nom.
Storyteller figurines 2003-02-10
From A student:
It takes 3/4 of an hour to bake a storyteller figurine. If only one figurine can be baked at a time, how many can be baked in 6 hours?
Answered by Penny Nom.
Splitting weight 2003-02-10
From Jin Ho:
A balance scale has only two weights, 1 ounce and 4 ounces. In only three weighings split 180 ounces of seed in two bags of 40 and 140 ounces.
Answered by Penny Nom.
Finite differences 2003-02-10
From Jenny:

I need to find a formula that will work with any number.
I am finding the volume of a 3d cross- shape. Here are my results so far:

Term Number      0   1    2    3     4      5
nth term         1    7   25   63   129    231
1rst diff           6   18   38   66     102
2nd diff              12    20   28   36
3rd diff                  8     8    8

I can't seem to find a formula that will work with any number. Any help would be much appreciated.

Answered by Penny Nom.

Three consecutive positive intergers 2003-02-09
From Yew:
Prove that when we multiply any consecutive positive intergers, the result is always divisible by 6.

ex. (7)(8)(9) = 504 = 6 (84)

Answered by Penny Nom.
Millilitres and millimeters 2003-02-09
From Iwan:
I was just wondering whether a millilitre was equal to a millimetre, if not, what is there difference.
Answered by Penny Nom.
Two investments 2003-02-07
From A student:
you have $7200 in principal and invest different sums @ 10% and 14%. You receive 920 at end of term. (one year)What amounts are invested at what rate?
Answered by Penny Nom.
The area of an isoceles triangle 2003-02-07
From A student:
I have to find the area of an isoceles triangle with one angle side of 30 degrees, and length of base 5. Could you please help me solve this problem?
Answered by Penny Nom.
Two equations in two unknowns 2003-02-06
From Patrick:
What are all ordered pairs of real numbers (x, y) for which: yx2 - 7x + 12 = 1 and x + y = 6?
Answered by Penny Nom.
Transformations of pattern blocks 2003-02-06
From Cheryl:
I am trying to help my 8th grade son with Transformations - use of patttern blocks. He has to trace a figure and reflect it across the x-axis, which I can do, but then it indicates include ordered pairs - that confuses me.
Answered by Penny Nom.
Two airplanes leave Dallas 2003-02-06
From A student:
TWO AIRPLANES LEAVE DALLAS AT THE SAME TIME AND FLY IN OPPOSITE DIRECTIONS. ONE AIRPLANE TRAVELS 80 MILES PER HOUR FASTER THAN THE OTHER. AFTER THREE HOURS, THEY ARE 2940 MILES APART. WHAT IS THE RATE OF EACH AIRPLANE?
Answered by Penny Nom.
Milliliters 2003-02-04
From James:
What is a milliliter?
Answered by Penny Nom.
Filling in the back yard with dirt 2003-02-04
From Joe:

We just bought a house and need to start filling in the back yard with dirt to level it out.

I measure 37'x 56' by 2' deep on an angle.

I will be building a retaining wall at the 2' deep portion. The slop will go from the 2' to nothing at the top or near the house. I can't remember how to get the Sq. foot or cubic yards.


Answered by Penny Nom.
A train with 2 cars 2003-02-04
From Michael:
A train with 2 cars is traveling at a speed of 80 km/hr from town X to town y, located 800 km from each other. At the same moment that the train departed, a passenger started to walk back and forth from one end of car B to the other at a speed of 100cm/sec. Arriving in town Y, the passenger had already gone and returned 720 times. The length of car A is that of car B plus one fourth of the length of the locomotive, and the length of the locomotive equals the length of Car A plus one fifth of the length of car B. What is the total length of the train?
Answered by Penny Nom.
Problem solving 2003-02-03
From Brigitte:
My daughter is having lot's of problem with problem solving she is in grade 6. she is using mathematics in action from macmillan/ mcgraw- hill. Do you know of any way she can get help, and if there is any tests on line
Answered by Diane Hanson.
Arithmitic sequence 2003-02-01
From A student:
I am having problems solving this arithmetic sequence... 1, 5, 10, ___, 50, 1.00, ___, 10.00, ... I believe the answers to be 25 and 5.00 but I can't figure why.
Answered by Claude Tardif.
Triangle perimeter 2003-01-30
From An Aunt:

Hi I'm the aunt of a middle school child. He need help with two problems and I don't understand how to show he how to do them. Can you help me.

  1. How do you find the perimeter for X=4, X=0.7, and X=5/6?

  2. And an acute triangle that bottom is (3X+1) cm, left side is (X+4) cm and the right is (2X+5)cm.

Can you show me not only the answer but also the steps that you used.


Answered by Penny Nom.
I have three circles... 2003-01-30
From Tony:
I HAVE THREE CIRCLE THAT IS CIRCLE TOGETHER: IN CIRCLE A, THE NUMBERS ARE: 11 I KNOW IS IN CIRCLE A, BUT I HAVE THE: 5 THAT IN A AND C, I HAVE THE 2 IN THE CIRCLE C AND B AND AND A, THE CIRCLE C I KNOW THAT 10 IS IN THE CIRCLE THE 4 IN CIRCLE A: AND B: IN CIRCLE B, I KNOW NUMBER 13 IS IN CIRCLE B; BUT I HAVE THE 3 IN CIRCLE B AND C AND I HAVE THE 2 IN CIRCLE B AND C AND A ,THE 4 IN CIRCLE B AND A.
HOW DO I FIND THE SUM IN CIRCLE C AND IN B IN BOTH CIRCLE A AND B AND B AND C NOT IN CIRCLE B, AND NOT CIRCLE C.

Answered by Penny Nom.
Fibonacci exam 2003-01-28
From Mike:

My son will be participating in the national math contest. He is a grade 4 student but will be writing the Fibonacci exam for grade 5. He has been given last years exam for practice. Parents have been ask to assist for this part. I was therfore hoping for a little "inside" help on several questions with answers & short explanations. This with a would be greatly appreciated.(from last years exam)

  1. With 3 weights, Mathusalem can weigh any object that has a weight which is smaller or equal to 13 kg, but that weighs a whole number of kilograms. He wants to weigh an object of 7 kilograms. To do so, he must place two of the weights on the left side of the scale, and the third one on the right side. Which one will he have to place on the right side

    a)4 b)3 c)6 d)2 e)5
  2. Mathilda uses a rectangular prisms to build stairs. The cross section of each prism is a square which has a side of 1 dm. Mathilda has already put in place 4 steps of a staircase that must have a height of 10 dm. When finished, the staircase will have a volume of 550 dm3. What will the width of the staicase in dm be?

    a)10 b)55 c)11 d)1 e)100
  3. The year 1991 was the last palindrome year of the second millennium. The year 2002 is the first palindrome year of the third millennium. Including 2002, how many palindrome years will there be, altogether, in the third millennium ?
    a)8 b)2 c)10 d)182 e) 9


Greatly appreciated,
Mike


Answered by Penny Nom.
Factoring a trinomial 2003-01-26
From A student:
how do you factor: Xsquared + 8x + 12
Answered by Penny Nom.
Extraneous solutions 2003-01-24
From Paul:

What is an extraneous solution and in what cases do you get one?

How do you know it is extraneous?


Answered by Penny Nom.
Some 5th grade problems 2003-01-24
From Sabrina:

[What is the greatest common factor of 20,28, and 36?] next problem is

[5k+7(k+8)= ] next problem

[Curtis simplified 0.8 divided by 0.2 x 0.04 and got 10. Latoya Simplified and got 1.6. is either correct? Can you explaine how you got the answer please.]


Answered by Penny Nom.
Dorothy and the wizard 2003-01-24
From Ken:

dorothy was going to see the wizard of oz...she stopped at the bakery and bought a box of cookies.

she met the scarecrow and gave him half the cookies. she ate half of the remaining cookies and threw one half cookie away.

she met the tin man and gave him half the cookies. she ate half the remaining cookies and threw one half cookie away.

she met the lion. she gave him half of her cookies. she ate half the remaining cookies and threw the LAST half cookie away.

how many cookies did dorothy leave the bakery with?


Answered by Penny Nom.
6 digit numbers from 0,0,2,2,4,4 2003-01-23
From Amanda:
How many 6 digit numbers can you make from the numbers 0,0,2,2,4,4, giving that 0 cannot come first. The number has to contain 2 twos, 2 fours, and 2 zeros.
Answered by Penny Nom and Claude Tardif.
The cousin of Sally's sister's boyfriend 2003-01-23
From Michael:
Sally went to a farm to buy eggs. Returning home, she gave half of them to her sister who, in turn, gave a third of those she had gotten to her boyfriend. The latter, after eating one third of the eggs that he had gotten, gave the rest to his cousin. Given that each egg weighs 70 grams, that Sally cannot carry more than 2.5kg, and that the eggs were raw, calculate how many eggs the cousin of Sally's sister's boyfriend received.
Answered by Penny Nom.
The area of a regular n-gon 2003-01-22
From Sophie:

We have been given a piece of maths coursework. A farmer has exactly 1000 metres of fencing and wants to fence off a plot of level land. she does not mind what the shape is but it must have a perimeter of 1000m. She wants to fence off the plot of land which contains the maximum area. Investigate the shape, or shapes that could be used to fence in the maximum area using exactly 1000 metres of fencing each time.

I have investigated many shapes, and I feel that a circle will have the biggest area. However we have also been asked to investigate shapes with 20 and 30 sides. My dad said that there is a formula for finding out any area of land. Do you know of this formula, if so I would be very grateful if you were to email it to me.


Answered by Penny Nom.
Rational expressions 2003-01-22
From A student:

// = the main fraction line

1.) a+1/a-1 + a-1/a+1 // a+1/a-1 - a-1/a+1

2.) 2/a(squared)-3a+2 + 2/a(squared)-a-2 // 2/a(squared)-1 + 2/ a(squared)+4a+3


Answered by Penny Nom.
The least common denominator 2003-01-21
From Brittan:
Hi there I need help! My name is Brittany and i am in the 6th grade.

I need help finding the least common denominator(LCD), and the book says Find the LCM of the denominators and i've done that and then it says write equivalent fractions,using the LCM as the least commonn denominator.The directions say Use the LCD to write each pair as like fractions. and the problem is 1/8 and 5/40. Could u explain how in the word u do this? Thanks a lot

Brittany

Answered by Penny Nom.
A penny, a nickel, a dime, a quarter, and a loonie 2003-01-21
From Blaine:
I have 5 coins: a penny, a nickel, a dime, a quarter, and a loonie. How many different amounts of money could I pay using any combination of these coins?
Answered by Penny Nom.
Jo's books 2003-01-21
From debra:
Jo announces: I have no more than 999 books.
Jean says: No Jo, You have fewer than 1,000 books.
Mary says: Jo has at least 1 book.

Only one of these statements is true, How many books does Jo own?

I do not know HOW to approach this problem and solve it.

Answered by Penny Nom.
Why quadratic? 2003-01-21
From Dan:
Why are equations of the form aX2 + bX + c =0 call quadratic? The quad indicates 4 but the power is 2.
Answered by Penny Nom.
Radians 2003-01-16
From Erikson:
I am a student in the 10th grade and attending advanced math at my high school. I was assign to do a report about the unit circle and the radian. But there seems to be no information available about the history of the radian; who first found out about them, which civilizations used it if any. Well, hopefully you'll assist me in this troubling question. Thank you for your kind consideration.
Answered by Penny Nom.
NO RED-HATTED DWARVES ALLOWED 2003-01-14
From Dulce:

There are 1000 dwarves in Dwarfland. Now as you know, dwarves are born with either red or white hats on.

The dwarves in Dwarfland love to party. Every night, they all flock to the same club to dance the night away.

One day, a sign is posted at the club: "NO RED-HATTED DWARVES ALLOWED". A certain number of days pass, after which all of the white-hatted dwarves are in attendance,and none of the reds.

How did the red hats know not to come?

Assumptions:

  1. The number of red and white-hatted dwarves need not be equal. There are at least 1 of each color.

  2. There are no reflections or mirrors in Dwarfland (i.e. the dwarves can't see the color of their own hat), and no dwarf can reveal through direct or indirect means the color of another dwarf's hat.

  3. There is NO COMMUNICATION among dwarves

  4. No white-hatted dwarf ever misses a night of partying


Answered by Claude Tardif.
Collinear Points 2003-01-13
From Gary:

Which of the 4 points are collinear when you construct the following concurrent lines or rays of a triangle?

  1. P(1), the point where the angle bisectors intersect.
  2. P(2), the point where the altitudes (or extensions) intersect (inside or outside of the triangle).
  3. P(3), the point where the medians intersect.
  4. P(4), the point where the perpendicular bisectors (or extensions) of the three sides of a triangle intersect.


This is for my 9-12 high school class in geometry.
My name is Gary


Thanks for your help.
Gary


Answered by Harley Weston and Chris Fisher.
Would it be worth paying $20K for 4 chances... 2003-01-13
From Steve:

If I've got 4 chances to to achieve X goal, and in each chance that I have I've got a 65% chance of achieving this goal, what is my overall probablity of achieving this goal? Also, what would it be after each of my 4 chances (i.e., for my 2nd chance, would it be (.65 * (.65*.65)? I've forgotten some basic probablity theories, and would appreciate any help on this.

With the probablities above, would it be worth paying $20K for 4 chances to achieve the goal or $8.5K for only 1 chance to acheive the goal? (at 65% probability)


Answered by Andrei Volodin.
What is larger than infinity? 2003-01-12
From Dana:
What is larger than infinity?
Answered by Claude Tardif and Harley Weston.
Cubic yards in a gravel pile 2003-01-10
From Ron:
I am looking to find a way to measure gravel piles to get yards. They are generaly not square.it would be like pouring sand out of a bag into a pile. they generaly are concave in dimension. can you help. I have attached a picture to show a small scale what I am working on is in much larger volume.
Answered by Harley Weston.
A rectangular prism 2003-01-09
From Julie:
How many faces on a rectangular prism and how many bases? Can the base also be a face and can a face also be a base?
Answered by Diane Hanson.
Repeating decimals 2003-01-08
From A student:
If k=.9repeating, and 10k=9.9repeating then 10k-k=9k, k=1 therefore .9repeating=1 and 1/3=.3repeating 3x1/3=.3repeatingx3, 3/3=.9repeating, therefore 1=.9repeating

It would seem to me that .9repeating approaches one but never quite makes it. Can you clarify?


Answered by Penny Nom.
The area of my lot 2003-01-07
From Linda:
I have a lot that is 210 feet in the front, 240 feet in the back and the sides each measure 150 feet. How many square feet is this all together and how close to an acre is it?
Answered by Penny Nom.
Integration of 1/(2+cos(x)) 2003-01-07
From A student:
integral from pi to 0 of

dx/(2+cos x)

i used the substitution t=tan(x/2) and i ended up with

integral from +infinity to 0 of

2dt/(t2+3)

which looks like an inverse tan function , and i ended up with sqr(27)/2 pi , which is not the same as my calculator's answer , so i suspct i am doing some thing wrong. can some one tell me where i am going wrong please.
Answered by Penny Nom.

Constructions of polygons 2003-01-03
From Garrett:
Our teacher just finished the constructions unit, and he mentioned briefly about odd sided figures such as pentagons and septagons, only that they're very hard. My question is, how do you draw, with a compass and a straight edge, a pentagon and septagon?
Answered by Chris Fisher.
Two numbers have a ratio of 5:3 2003-01-01
From Patricia:
Two numbers have a ratio of 5:3. The larger number is 36 more than half the difference of the two numbers. What are the two numbers?
Answered by Penny Nom.
Augmented matrix 2002-12-31
From Michelle:
I am trying to augment a matrix so that i can find the values of the variables a, b, and c. For the life of me i can't find a solution to the matrix: 3 -5 2 ' 22 2 3 -1 ' -9 4 3 3 ' 1 I thank you for your help, Michelle
Answered by Penny Nom.
Successive coefficients in Pascal's Triangle 2002-12-27
From Quincy:
There is a formula connecting any (k+1) successive coefficients in the nth row of the Pascal Triangle with a coefficient in the (n+k)th row. Find this formula
Answered by Penny Nom and Walter Whiteley.
The intersection of conics 2002-12-19
From Glenda:
We are studying systems of equations where two conic sections are the two equations that we are solving simultaneously. We were studying the number of solutions that are possible if you have an ellipse and a parabola. We all agree that there can be none, one, two, three or four solutions. The question that the students had for me was whether or not a portion of an ellipse and a parabola can overlap and thereby allow an infinite number of solutions. What should I tell them?
Answered by Chris Fisher and Harley Weston.
Calculations 2002-12-15
From Mahdi:
This problem is rather general, but it usually makes a lot of problems. I almost have no difficulty in math and physics questions, but unfortunately I'm not at all accurate in simple one or two digit calculation. I even sometimes make silly mistakes in simple sums like 7+4=12 or 4+5=11! Is there any effective way to reduce these mistakes?
Answered by Walter Whiteley.
A bouncing ball 2002-12-14
From Eman:

Q : When a childís ball is dropped from a height h metres on to a hard, flat floor, it rebounds to a height of 3/5h metres. The ball is dropped initially from a height of 1.2m.

  1. Find the maximum height to which the ball rises after two bounces.
  2. Find the total distance that the ball has traveled when it hits the floor for the tenth time.
  3. Assuming that the ball continues to bounce in the same way indefinitely, find the total distance that the ball travels.

Answered by Penny Nom.
Two modes 2002-12-12
From ali:
I have a math assignment to pick the mode in a set of data. 12 and 21 both appear twice in the data set. What is the mode?
Answered by Penny Nom.
Construct a pentagon 2002-12-12
From Mary:
We were wondering what the instructions/formula, step-by-step, is to make a pentagon.
Answered by Chris Fisher and Penny Nom.
A sphere with a hole - cylindrical shells 2002-12-11
From Kathy:
Suppose you make napkin rings by drilling holes with different diameters through two wooden balls (which also have different diameters.) You discover that both napkin rings have the same height, h.
  1. Guess which ring has more wood in it and why.
  2. Check you guess: Use cylindrical shells to compute the volume of a napkin ring created by drilling a hole with radius r through the center of a sphere of radius R and express the answer in terms of h.

Answered by Penny Nom.
Find a, b and c 2002-12-10
From Sameer:
Question:

a+b+c=180
b=a+(120/2-5)
c=b-(130-10)/6
a=?
b=?
c=?

Answered by Penny Nom.
Quadratic equations 2002-12-10
From Katie:
just bugging the hell outta me and I cannot figure it out for the life of me.

1)x-2x-1 =0
Find both roots. You must simplify both of your roots. This equation requires the use of the Quadratic Formula. I have tried everything for this and I am still either getting the wrong answers, and end up getting really frustrated. If you could find the answers and tell me how you did it I would greatly appreciate it! Then the second problem is

2) The square of a certain negative number is equal to 70 decreased by 3 times the number. Find the answer! On this one I set it up but still came out with the wrong answer. So if you could help me on this one I would greatly appreciate it!

Answered by Penny Nom.
y = 1 - sin(x + 60) 2002-12-10
From Eman:
Sketch the graph of y = 1 - sin(x+60). for 0 <= x<= 360, giving the coordinates of the maximum and minimum points and the pints where the curves crosses the y axis.
Answered by Penny Nom.
How many billions equal one trillion? 2002-12-07
From Ryan and Aylah:
How many billions equal one trillion?
We believe that the answer is one thousand times one billion equals one trillion. Please help us with the answer to this question.

Answered by Penny Nom.
A rope trick 2002-12-05
From Jerry:
My name is Jerry and I am a math teacher at the secondary school level in the greater Toronto area. I posed the following problem to my enriched grade 10 math class that I found in the book called "Mathematics and the Imagination" by William Kastner. According to the author, it is posible for two people linked to each other with two ropes around their hands, to separate themselves without letting go of the rope. The students had lots of fun trying to accomplish this problem using skipping ropes from the Phys Ed department, but with no success.
Answered by Penny Nom.
The dimensions of a box 2002-12-03
From Paula:
My son, who is in secondary school, needs to make a box with the inside dimensions of 1.25 cubic feet. It's supposed to be more in the shape of a rectangle than a square.

I would like to know the equation to use to find the inside dimensions of a box if I know how many cubic feet (or inches) I need the inside to be.


Answered by Penny Nom.
A Normal probability problem 2002-12-03
From A student:
The height of married men is approximately normal with mean 70 and standard deviation 3. The height of married women is approximately normal with mean 65 and standard deviation 2.5. What is the probability that a random married woman is taller than a random married man?
Answered by Andrei Volodin.
How do you estimate the equation of a parabola? 2002-12-01
From Audrey:
How do you estimate the equation of a parabola? I have a project where a must find parabolas in magazines and then estimate their equation.
Answered by Penny Nom.
The height of a triangle 2002-11-29
From Dean:
Could you please tell me the formular for me to calculate the height of a triangle. I have the angles and side lengths. I am trying to calculate the height of an isosceles triangle, does this make a difference from a normal triangle or is the formular the same.
Answered by Penny Nom.
Constructing a tangent to two circles 2002-11-28
From Tom:
I have two circles, different sizes a known distance from each other. We know the radii of the circles. How do I construct a line that is tangent to both circles relative to the segment that connects the centers of both circles?
Answered by Chris Fisher and Penny Nom.
x + y=80 and x + z=100 2002-11-28
From PJ:
x + y=80
x + z=100

can you solve for x, y or z?


Answered by Penny Nom.
Miles per hour 2002-11-28
From Liz:
If a car has traveled 16 miles in 30 minutes, how many miles per hour did they go?
Answered by Penny Nom.
The length of an arc 2002-11-27
From Nancy:
If all I have is the length between 2 ends of an arc (72"), how do I find the length of the arc at its apex and the radius?
Answered by Penny Nom.
[x*(x+2)] + 1 = (x+1)(x+1) 2002-11-27
From Melissa:
While teaching the multiplication table to my daughter I noticed an interesting pattern. It goes something like ...Take a whole number
...add two to it
...multiply the two numbers together
...add one to the resulting number
...it will now be equal to the original number plus one, squared.

Answered by Claude Tardif.
Making 7 2002-11-26
From Bill:
At one time I had the answer to this math equation but I have lost it and can't seem to regain it. I may be going at it wrong but as I recall there were the numbers 1 through 4, and the std operators, +, -, /, *, and parentheses. The object was to make the four numbers with the operators equal to 7. Each number and operator may be used only once. My 9 year old loves these quizzes but I can't give it to her if I can't provide the correct answer.
Answered by Claude Tardif.
A two stage rocket 2002-11-26
From Hoda:
a two stage rocket accelerates in free space by ejecting fuel at a constant relative speed , v(ex). the full fuel load makes up 80% of the initial mass of the entire two stage rocket . the rocket accelerates from rest until at the end of the first stage when 75% of its fuel has been burnt. find an expression for the speed of the rocket at the end of the first stage in terms of v(ex).
Answered by Claude Tardif.
A lampshade from a cone 2002-11-26
From Ellsie:
I need to make a pattern to cover an old lampshade. This is actually the bottom portion of a cone. Please help me figure out how to draw this pattern, so that we can complete our project.
Answered by Penny Nom.
Decimals in everyday life 2002-11-24
From Fritz:
How do you use decimals in your every day life?
Answered by Penny Nom.
Filling A swimming pool 2002-11-21
From Sarah:
A swimming pool is being filled by three pumps. Alone pump A would take 6 hours, pump B would take 3 hours, and pump C would take 3 hours. If all three pumps are used to fill the pool, what fraction of the process is pump A.
Answered by Penny Nom.
The volume of a juice jug 2002-11-21
From Edward:
I try to find the measurement for a juice jug. I have the height of the jug and need the jug to hold 2.5 Liter of juice. Can you help me out with the formula.
Answered by Penny Nom.
Performance reviews 2002-11-21
From Lara:
Perhaps you can help.

He's doing performance reviews at work.

The average performance review is 3.92 on a 5.0 scale. This person would receive a 3.5% raise for next year.

The highest performance review is 4.9 and this person would recieve a 6.0% raise for next year.

How do I solve for the other performance review numbers.


Answered by Penny Nom.
Differentiating inverses 2002-11-20
From Amy:
f(x)= x3+x+1, a=1 find g'(a) (g = f -1). I am having trouble finding g(a).
Answered by Penny Nom.
4 x abcd = dcba 2002-11-20
From A student:
Say you have a four digit number (e.g. abcd) and you multiply the number by 4. The answer you get will be the reverse order of the number you started with (dcba). What is the number? The four digits (a,b,c,d) cannot be the same number or cannot be repeated.
Answered by Penny Nom and Claude Tardif.
abc,abc 2002-11-20
From Pam:
Prove or disprove that "every number of the form abc,abc (where a, b, and c represent digits) is divisible by 7,11, and 13"
Answered by Penny Nom.
Round to hundredths 2002-11-19
From Brittany:
Can u tell me how to do a problem like this:

35 divded by 4.8= Round to hundreths.

Answered by Penny Nom.
Why is 5 divided by 1/7 greater than 5 divided by 2/3? 2002-11-19
From Elizabeth:
  1. Without performing the division, explain why 5 divided by 1/7 is a number greater than 5 divided by 2/3.
  2. Is the least common multiple of two prime numbers always their product? Why or why not?

Answered by Diane Hanson and Penny Nom.
The product of two consecutive numbers is 41 more than their sum 2002-11-15
From A mother:
THE PRODUCT OF TWO CONSECUTIVE NUMBERS IS 41 MORE THAN THE SUM OF THE NUMBERS. WHAT ARE THE NUMBERS?
Answered by Penny Nom.
The substitution method 2002-11-15
From A student:
Solve by using the substitution method.

x = 3y - 1
x + 2y = 9


Answered by Penny Nom.
A pattern in a table 2002-11-14
From Thamar:
how do you solve for pattern in a table like

the top row is261217z
the bottom row is9131924blank

question ask in each blank space write a rule to find the next number in the table

Answered by Penny Nom.
Algebra 2002-11-13
From Carol:
I am a confused parent trying to help my dau hgter solve some math problems and I need help.

3[x-4]=27


Answered by Penny Nom.
The volume of water in a hose 2002-11-13
From David:
I trying to find out how much water 500 ft of 3/4 inch hose will hold. I'd also like to know how you figured the answer.
Answered by Penny Nom.
Subdividing a circle 2002-11-11
From David:
Say you have a cirlcle.

Then you draw 2 dots on the circle.

Then you connect the dots with lines.

The circle is divided into 2 parts.

If you do the same with 3 dots and connect each dot to each dot with a line then you get a circle with 4 parts.

4 dots with lines connecting all (6 lines) = 8 parts....


Answered by Claude Tardif.
Fuel consumption 2002-11-11
From Guy:
When a car is driven 'normally' the feul consumption is 7.5 per litres per 100km. On a particular journey the car was driven in 'hard' and the consumption changed to 9.1 litres per 100km. Fuel cost 80p per litre. Calculate:

a) The extra feul consumed.
b) The extra cost of feul, in pounds, for the journey when the car was driven in 'hard'.

Answered by Penny Nom.
Radicals with fractional radicands 2002-11-11
From A student:
how do I simplify radicals with fractional radicands?

ex. 7sqrt(2) + sqrt(50) - 2sqrt(18)

10sqrt(3/5) - 24sqrt(5/3)

3sqrt(2/9) + 1/2sqrt(32) + sqrt(9/8)


Answered by Penny Nom.
x intercepts 2002-11-10
From Laura:
Find the x intercepts for the graph of the equation.

x 2 - x - 12 = 0


Answered by Penny Nom.
(93+n)+158=93+(28+158) 2002-11-10
From Parents:
(93+n)+158=93+(28+158)
Answered by Penny Nom.
Three bags of marbles 2002-11-08
From A student:
I have 3 bags of marbles

1 bag is labeled blue; 2nd bag is labeled red; 3rd bag is labeled blue & red

all the bags are mismarked

your job is to take one marble from 1 bag look at it and correctly label all the bags


Answered by Penny Nom.
Cubic centimeters 2002-11-07
From Rita:
Was in emergency room yesterday-MD aspirated 40cc of fluid from my knee--what does a "cc" equate to?
Answered by Penny Nom.
G = 3H - 5 2002-11-07
From Kash:
G=3H-5, for H solve for the given variable
Answered by Penny Nom.
Determining a parabola 2002-11-06
From Shelley:
let (0,3) and (1,9)and (-1,1) be given points in a parablola. Determine a b and c
Answered by Penny Nom.
18m-11+ -7 2002-11-06
From Sophia:
what is 18m-11+ -7 (m = to 12)?
Answered by Penny Nom.
A house's selling price 2002-11-05
From Tanisha:
a CERTAIN REAL-ESTATE AGENT RECEIVES $6 FOR EVERY $100 OF A HOUSE'S SELLING PRICE. HOW MUCH WAS A HOUSE SOLD FOR IF THE AGENT $10,725._
Answered by Penny Nom.
The percentage grade of that hill 2002-11-05
From Cathy:
If there is an 80ft climb over a kilometer(about 3280ft) what is the percentage grade of that hill?
Answered by Penny Nom.
An octagon of perimeter 1,000 meters 2002-11-05
From Stephen:
my son has to work out the area of a octagon with a outside perimeter of 1,000 metres,
Answered by Penny Nom.
Clown has eight steps to climb 2002-11-05
From A mom:
I'm a parent of a grade two student. What is the best way to help my son figure out a math comprehension problem?

Clown has eight steps to climb, is shown in a picture with front foot on third step and behind foot on first step. Each time he climbs three steps, he falls back two. How many times did he have to climb up to reach the top.

Should I get him to do a line graph or math equations. On his own he came up with the answer seven. I think the answer is six. I did 0+3=3, 3-2=1, 1+3=4 and so on, In think my why is alittle confusing to him. Is there a simplier why to explain it to him.


Answered by Diane Hanson.
A twelve-volume set of encyclopedias 2002-11-04
From Ron:
A novice librarian shelved a twelve-volume set of encyclopedias in the following order from left to right. Volumes 8, 11, 5, 4, 9, 1, 7, 6, 10, 3, 12, and 2. Using her system, where will volume 13 go?
Answered by Penny Nom.
Rounding 27.27 2002-11-03
From A parent:
If you have a problem 27.27 and you need to round to the first 7 the answer would be 27

Would 27.0 be acceptable as well or is it completely wrong.


Answered by Penny Nom.
How would you find the length of the chord? 2002-10-31
From A draftsperson:
If given the length of an arc and the distance from the midpoint of the arc to the midpoint of a chord, how would you find the length of the chord and the radius of the arc. The chords endpoints are the same as the the arcs endpoints.
Answered by Penny Nom.
A coin is tossed 11 times 2002-10-31
From A student:
There are 2 people that are playing a game in which a coin is tossed 11 times. The first player gets a point for a toss of heads. The 2nd player gets a point for a toss of tails. whoever gets 6 points wins. Suppose that so far the first player has 2 points and the second player has 4 points. What is the probability that the first player wins the game?
Answered by Andrei Volodin and Penny Nom.
Mrs. Klein has been losing 1 pound a week for the last year 2002-10-30
From Brett:
On her diet, Mrs. Klein has been losing 1 pound a week for the last year. Her husband weighs 110 pounds more than she does now. If his weight is twice what her weight was 6 weeks ago, how much does mrs. Klein weigh now?
Answered by Penny Nom.
An isosceles triangle 2002-10-30
From Stan:
What two different base lengths can an isosceles triangle have with sides on both remaining at 13 inches? How do I show this?
Answered by Paul Betts and Peny Nom.
A triangular prism 2002-10-30
From Daniel:
I was wondering what a triangular prism was.
Answered by Diane Hanson.
Simplifying Variable Expressions 2002-10-28
From Erika:
i would like to ask you a mathematical question on Simplifying Variable Expressions. The question is 2(n-4)+3.
Answered by Penny Nom.
Jimmy's hits 2002-10-27
From Patty:
One seventh of Jimmy's hits were doubles
12.5% of his hits were homeruns
Jimmy had 82 singles
He did not have any triples

How many hits did Jimmy have? (Be sure to include the equation that represents the number of hits)

Answered by Penny Nom.
10 hours before 7:15 2002-10-27
From Jessica:
It is evening 7:15. What time would it be 10 hours earlier?
Answered by Penny Nom.
The four colour theorem 2002-10-27
From Rhonda:
is there an actual equation for the four colour map theorem??? i cannot find it anywhere!! if so can you give a breif description on how it works!!
Answered by Penny Nom.
What are the digits? 2002-10-24
From Ebenezer:
There are three digits. The middle digit is zero and the sum of the first and last digits are is 13. When the number is reversed it is increased by 297. What are the digits?
Answered by Penny Nom.
How many trees were planted in March? 2002-10-20
From Allison:
The number of trees planted by the Alpine Nursery in April was 3 more than twice the number of trees planted in March. If 71 trees were planted in april, write and solve an equation to find how many trees were planted in March.
Answered by Penny Nom.
Is a square a rectangle? 2002-10-20
From Nona:
Are squares rectangles? Third grade math teacher told a student this was true. Grandmother needs confirmation on this!
Answered by Claude Tardif and Harley Weston.
At the zoo 2002-10-20
From A student:
The number of adults at the zoo was 2/9 the number of children. there are 700 more children than adults. The number of men was four times the number of women. The number of girls was four times the number of boys. Find the total nymber of men and boys at the zoo.
Answered by Penny Nom.
Simplify 2002-10-19
From A student:
well i need to simplify this problems out and i cant do it so here is one
 3x^2+6x-45 ------------ = 3x^2+21x+30 

Answered by Penny Nom.
Exponential notation 2002-10-19
From Ivy:
I understand how to figure out for example 4 to the power of 3,what I don't understand is how to figure it out using both negative numbers and negative exponents,as an exaple -4 to the power of 6 and what makes this different that (-5) to the power of 7,please help me if you can.I am an adult taking correspondence and finding it quite frustrating.
Answered by Penny Nom.
Use each digit 1-9 only one time each,... 2002-10-19
From Megan:
Use each digit 1-9 only one time each, divide the numbers into 3 groups, make one 2 digit number, one 3 digit number, and one 4 digit number. There are 6 combinations of these numbers in which you can multiply the 2 digit and 3 digit number together to get the 4 digit number (using each digit 1-9 one time each). What are they!!!!!!!!!????????????
Answered by Claude Tardif.
Integers in real life 2002-10-15
From Rica:
What are the real life usages of integers?
Answered by Peny Nom.
A sale price 2002-10-15
From Chester:
A jacket is on sale for 32% off. Find the original if the sale price is $89.00
Answered by Penny Nom.
6% grade 2002-10-15
From Micheal:
When I am driving down the road and see a sign that says for example steep grade ahead 6% grade etc etc what does this mean?
Answered by Penny Nom.
Two problems 2002-10-14
From Seçkin:
i am from turkey i am a teacher in a collage i have two difficult question which i havent solve yet these are very important for me....
Answered by Claude Tardif.
A functional equation 2002-10-14
From Rob:
Let f be a function whose domain is a set of all positive integers and whose range is a subset of the set of all positive integers with these conditions: a) f(n+1)>f(n)

b) f(f(n))=3(n)

Answered by Claude Tardif.
Two problems 2002-10-14
From Eva:

a) How many different equivalence relations can be defined on the set X={a,b,c,d}?

b)Show that 6 divides the product of any 3 consecutive integers. I know it is true that 6 divides the product of any 3 consecutive integers. However, i have problem showing the proof.


Answered by Leeanne Boehm and Penny Nom.
Three algebra problems 2002-10-13
From Veronica:
Solve the following inequality:

5(xsqured-4)/(xto the 5th(2x-5)to the 3rd) < or equal 0

Solve the following equations for all roots

square root of x+2=-1+square root of 2x+3

what's ZERO FACTOR PROPERTY???

The hypotenuse of an isosceles right triangle is 7cm. long. Determine the lenghts of the other sides.


Answered by Leeanne Boehm.
Kilometers to miles 2002-10-09
From A student:
how many miles are in 16 kilometers?
Answered by Leeanne Boehm.
Nixon, Jefferson, and Madison 2002-10-08
From Lisa:
The longest-lived US presidents are John Adams(age90), Herbert Hoover (also90), and Harry Truman (88). Behind them are James Madison, Thomas Jefferson, and Richard Nixon. The lattter three men lived a total of 249 years, and their ages at the time of death form consecutive odd integers. For how long did Nixon, Jefferson, and Madison live?
Answered by Penny Nom.
8 squares from 12 sticks 2002-10-08
From A student:
If you have 12 sticks the same size, how do you make them into 8 squares?
Answered by Claude Tardif.
Baseball, basketball and football 2002-10-08
From Debbie:
The school newspaper is interviewing 6th grade students to see what sports they follow regularly on TV. Of the 70 students interviewed; 40 enjoyed basketball; 40 enjoyed baseball; 40 enjoyed football; 20 enjoyed basketball and football; 22 enjoyed baseball and basketball; 27 enjoyed football and baseball; and 12 enjoyed all three sports. How many students out of the 70 interviewed didn't follow any one of the three sports?
Answered by Leeanne Boehm, Penny Nom and Walter Whiteley.
A complex quadratic 2002-10-06
From Michael:
I would like to know, how to solve this Complex number: quadratic equation. ix 2 + x -i = 0
Answered by Harley Weston.
Relatively prime 2002-10-04
From Natasha:
I really need help with this middle level math question. My little brother is asking me and I have no clue what the answer is. Explain what it means when two numbers are negatively prime. (?)
Answered by Kathy Nolan and Penny Nom.
$100 is split between four guys. 2002-10-03
From Stephan:
$100 is split between four guys. A,B, C, and D. How much money does if person get if B has four more dollars than A, C has 8 more dollars than B, and D has twice as much money as C?
Answered by Penny Nom.
Rational expressions 2002-10-03
From Ashley:
1/x(squared) + 5/xy
Answered by Penny Nom.
The interrelations of the propositions in Euclid I 2002-10-01
From Rob:
I'm a student at St. John's College in Santa Fe New Mexico, are school is devoted to the great books and we're now reading Euclid's Elements in english and Attic Greek. We have an assignment to show a frequency or flow chart of how all the propositions of book one are interrelated.
Answered by Chris Fisher.
Two rate problems 2002-09-30
From Rebecca:
  1. There are two small holes in the bottom of a tub filled with water. If opened, one hole will empty the tub in three hours; the other will empty it in six hours. If both holes are opened at the same time, how long will it take to empty the tub?

  2. An airplane flies 400 miles per hour in calm air. It can cover 900 miles flying with the wind in the same time that it can cover 700 miles against the wind. What is the speed of the wind?

Answered by Penny Nom.
A geometry problem 2002-09-29
From Sonia:
Given AB||CD; m a. Find the measures of
b. Is there enough information for you to conclude that

Answered by Penny Nom.
0.99999... 2002-09-26
From Erica:
Yesterday in my 8th grade math class we were being taught how to convert a Repeating Decimal into a fraction. Since I, for some odd reason, seem to understand math better than the rest of my classmates, i began to drown out my teachers explaination for the rule. While she was about half way through with explaining mixed decimals i came up with an unsolvable question. Like I said before, I understand how to turn a repeating decimal into a fraction, but how would I turn a repeating .9 into a fraction? We all know it would equal 9/9, but doesn't 9 over 9 also equal 1? Even though it comes very close to one, it never really equals one. I'm very confused about this and i would love it if you could clear this up for me.
Answered by Penny Nom.
Number play 2002-09-24
From Julie:
Organize these numbers and the operations so that each equation is correct. Use each coice only once per equation.

36 18 12 6 - + ( ) /

_________________________ = 22

_________________________ = 41

Answered by Claude Tardif.
A cone that is cut off at the top 2002-09-23
From Stuart:
I have to work out the dimensions and arcs of a cone that is cut off at the top.

I.e Top diameter is 33mm to bottom diameter is 43mm and the depth is 80mm

Are you able to work what the flat of this cone would be as I need to design within the flat area and then have it cut out.

I really need to know what the flat of it is before it is cut and curled to form the above cone.


Answered by Walter Whiteley.
Solving an equation 2002-09-22
From Emanuel:
how do u solve these problems?:

30/x=7+ 18/2x

3m+1/4 = 2- 3-2m/3

Answered by Penny Nom.
Ceiling 2002-09-22
From Joe:
Does the ceiling of a number mean the ceiling of 1.3 is 2 and
the ceiling of 1.5 is 2 and
the ceiling of 1.8 is 2.

Or the ceiling of 1.3 is 1.5?


Answered by Penny Nom.
A max/min problem 2002-09-21
From Evelina:
A window is the shape of a rectangle with an equilateral triangle on top. The perimeter of the window is 300 cm. Find the width that will let the maximum light to enter.
Answered by Penny Nom.
Fractional exponents 2002-09-20
From Jill:
The problem is with fractional exponants:
10 1/3 mult. by 10,000 The 1/3 is an exponant of 10.

Answered by Penny Nom.
The entire earths' population would fit in the state of Texas 2002-09-18
From Roz:
I have been told that the entire earths' population would fit in the state of Texas and each person would have 1/2 acre. Is this true.
Answered by Chris Fisher.
How long will it be before you reach your friend? 2002-09-16
From Margaret:
Your friend runs at 8 miles per hour and you drive at 40 miles per hour, hence the diatance between you is decreasing at 32 miles per hour. You firend left 2 hours before you so how far has he gone in that 2 hours? How long will it take for the distance between you to decrease to 0 miles if the distance is decreasing at 32 miles per hour?
Answered by Penny Nom.
A Circle is evenly divided into six equal triangles 2002-09-16
From Marilynn:
A Circle is evenly divided into six equal triangles leaving an area between the outside of the circle and the one side of the triangle. This area is measured as 3.14. What is the length of the radius, one line on the triangle?
Answered by Paul Betts.
A million sheets of 30 cm long paper 2002-09-15
From Jennifer:
if you put a million sheets of 30 cm long paper end to end, how many kilometer long would the paper be from begining to end?
Answered by Penny Nom.
Formulating equtions 2002-09-14
From Theresa:
Hello. In math class we have to formulate equations for certin questions for extra credit. We are allowed to ask anyone and anyone can do it for us and explain it to us. Could you please do that? Here are the questions
  1. Jordan has 3 times as much money as Reva. Together they have $44. How much money does each have?

  2. The Tigers played 48 games. They won twice as many as they lost. How many did they win?

  3. There are 900 students in Sewickley Academy. There are 20 more girls than boys. How many girls are there?

  4. A board 400 cm long is cut into 2 pieces. One piece is 66 cm longer than the other. Find the lenght of the shorter piece.


Answered by Penny Nom.
Mixing solutions 2002-09-13
From Margaret:
In a chemistry laboratory the concentration of one solution is 10% hydrocholoric acid (HCl) and that of a second soulution is 60% HCl. How many mL of each should be mixed to obtain 50 mL of a 30% HCl solution??
Answered by Penny Nom.
The number of decimal places in 1 over a power of 2 2002-09-12
From Allan:
Does anyone notice that the maximum number of decimal place of the number 2 dividing 1 and its increment (4, 8, 16...etc) is the same as the power of number 2? eg. 22=4, thus the max number of decimal of 1/4=0.25 which is 2 decimal place and 2 is the number of power of 2 take 64 as example: 26=64, and take 1/64=0.015625 which has 6 decimal place (and is the power 6)

Is there such a law in math? If yes, can you tell me what it is? Or is this my discovery?


Answered by Paul Betts.
Use a 4 four times 2002-09-12
From Amanda:
The question is what is if you can only use a 4 four times and get the answer 12, 10, and 6
Answered by Claude tardif.
The quadratic formula 2002-09-11
From Kathleen:
im having a problem trying to understand quadratic formulas can u please help me

i have problems like

3x2 - 4x - 11 = 0 how do i work out that problem? i have the formula written down but it doesnt seem to register for me.

Answered by Penny Nom.
A fish weighs 10 pounds more than half its weight. 2002-09-10
From Rebecca:
A fish weighs 10 pounds more than half its weight. How much does it weigh?
Answered by Penny Nom.
The factor theorem 2002-09-10
From Yael:
Prove (x - a - b) is a factor of x3 - a3 - b3 - 3ab (a + b)
Answered by Penny Nom.
A linear function 2002-09-09
From Chad:
If y = 9x - 13 does it represent a linear function? And if it does what is its slope and is it a direct variation?
Answered by Peny Nom.
The community garden 2002-09-08
From Michele:
Bryce, Duanne, Juliana and Sonya share a 40ft by 48 ft rectangular portion of the community garden in the neighborhood. With this space each has their own rectangular garden plot.

-Bryce is the only one with a square plot. its are is 1/2 the area of Duanne's -The area of Bryce's plot is 2/3 the area of Julianna's plot. Their plots have one side in common.

-Sonya's garden has an area twice that of Julianna's garden.

What are the dimensions of each person's garden plot?


Answered by Leeanne Boehm.
Algebra 2002-09-08
From Heather:
This question is from an 8th grade Algebra classs. I am a parent asking for help to explain to my child. Q: d=25t2.
Answered by Leeanne Boehm.
The area of a triangle 2002-09-07
From Phill:
How do you find the area of a equilateral and other triangles?
Answered by Penny Nom.
Sanding a wooden block 2002-09-07
From Wanda:
If a wooden block, 2in height, 3in width, and 6in length is sanded and 1/8 of an inch is removed from the top and from the bottom, how much is the volume of the block decreased?
Answered by Penny Nom.
Subsets of the Real Number System 2002-09-07
From Christine:
In looking for a French equivalent of the word "integer", I found that the word seems not to actually exist in French, and that Canadian schools use the term "natural number" to describe what we have been trained to call "whole numbers," while using the term "whole number" to describe what we have been trained to call "integers."
Answered by Claude Tardif.
Social security numbers 2002-09-06
From Chris:
How many different number combinations are there for the social security system? We know that there are only a certain ammount of combinations available and the current US population is at 281 million. The number system should include as many nine digit combinations as possible using the numbers 0-9 in multiple different spots.
Answered by Penny Nom.
I am a 3 digit number. 2002-09-06
From Jasmine:
I am a 3 digit number. My last digit is 3 times my first digit. My firat digit is twice my middle digit. What numbers am I? There are at least 2 correct answers?
Answered by Paul Betts.
What really is pi? 2002-09-05
From Rachel:
what really is pi? 3.1444
Answered by Penny Nom.
Dan is 4 inches taller than Mike. 2002-09-04
From A parent:
Dan is 4 inches taller than Mike. Together they are 8 feet 8 inches tall. How tall is each boy in feet and inches.
Answered by Penny Nom.
1/8 in decimal form 2002-09-04
From A student:
explain how you put 1/8 in a decimal form
Answered by Penny Nom.
Intercepts 2002-09-04
From Tamara:
Why is it that the x intercept the value of y is 0 and at the y intercept the value of x is 0?
Answered by Leeanne Boehm.
Monthly rent 2002-09-04
From Al:
How do you figure monthly rent if the charge is 11.00 per sq. ft. and there are 1,000 sq. feet?
Answered by Leeanne Boehm.
A limit 2002-09-03
From Circe:

Lim 3x 3+ax+a+3 /(x 2+x-2), X->-2


Answered by Penny Nom.
Addends can be grouped differently but the sum does not change 2002-09-03
From Jodia:
I have been searching the web for over an hour & a half now for the answer to the following question:

The _____ states that addends can be grouped differently but the sum does not change.


Answered by Penny Nom.
A schedule for a fantasy football league 2002-09-01
From Steve:
i am in a ten(10) team league, split evenly into two five team divisions. we play a thirteen(13) game schedule and would like to play each team in your own division twice, and each team in the other division once. according to my rudimentary mathematics that equals 13 games. a good schedule should look somthing like this: team 1-5 = division 1 team 6-10 = division 2 team 1 should play teams 2-5 twice and teams 6-10 once.
Answered by Claude Tardif.
A group of 9 2002-08-31
From William:
what is the mathmatical name for a group of nine like a group of 3 is called 'triplets'
Answered by Penny Nom.
How many cubic feet are in a cubic mile? 2002-08-30
From William:
How many cubic feet are in a cubic mile?
Answered by Penny Nom.
A square of tiles 2002-08-30
From Rosa:
How do I go about finding a formula for the number of tiles I would need to add to an arbitrary square to get to the next sized square?
Answered by Penny Nom.
38-10+12divided by4multiplied by 16 2002-08-30
From Brenda:
my math question is as follows: 38-10+12divided by4multiplied by 16=?
Answered by Penny Nom.
< and > 2002-08-30
From Kelsey:
What is the origin of the greater (>) than and less (<) than signs?
Answered by Harley Weston.
A solution to an equation 2002-08-28
From A student:
How do you determine whether a pair of numbers is a solution for an equation?

If you could identify the specifics steps I need to take to determine whether it is a solution or not would be appreciated! Thanks

i.e. x-4=0 Pairs of numbers as possible solutions (4,7) (3,4) (0,-4).


Answered by Penny Nom.
Mixing gas and oil 2002-08-27
From Sherry:
how do I calculate the correct fuel ratio for a boat motor? 50:1 the oil containers only say how many ounces to add to 5 gallons of fuel to make a 50:1 ratio.(13 ounces to 5 gallons of fuel) I have a 20 gallon tank so what if I add 11.6 gallons of fuel, what formula would I apply to get the correct measurement of oil to add?
Answered by Penny Nom.
The tangent to a curve and the tangent of an angle 2002-08-26
From A teacher:
Is there a relationship between the tangent of a curve(line touching the curve at one point) and tangent (the trigonometric function)?
Answered by Chris Fisher.
Divisibility of 5 2002 2002-08-25
From Simon:
I need to ask you a question if 5 2002 and 3 2002 are divisible by 26.
Answered by Penny Nom.
Express 5120 as a sum of consecutive numbers 2002-08-25
From Todd:
Express 5120 as a sum of more than 1 consecutive number.
Answered by Leeanne Boehm.
When does an inequality represent the area above the line? 2002-08-25
From Tamara:
How do you determine that an inequality represents the area above the line and when do you include the line in the solution?
Answered by Penny Nom.
Cubic yards of concrete 2002-08-24
From Michael:
if you have a 10" dia pipe 8' long how many cubic yards would it take to fill with concrete. how how would you go about figuring it out.

and also if you would how many cubic feet, cubic inches in a cubic yard. i know you don't need to answer, seeing i am not in school, but would appreaciate it.


Answered by Penny Nom.
Sets and elements 2002-08-22
From Dianne:
I want to know why its okay to say that, for example, 6 is an element of the set of integers, but you get counted off for saying that the set of 6 is an element of the set of integers. How come?
Answered by Judi McDonald.
Gallons in a cylindrical tube 2002-08-21
From Heidi:
IF I HAVE A 12" DIA TUBE WITH 1/4" WALL AND 72" IN LENGTH HOW MANY GALLONS OF WATER WILL THIS HOLD
Answered by Penny Nom.
Parallelograms 2002-08-21
From Moorthy:
11 parallel lines are intersected by another 9 parallel lines.Then how many parallelograms are formed?
Answered by Walter Whiteley.
The 20 kilometer walk 2002-08-20
From A student:
THE 1996 OLYMPIC GOLD METAL WINNER FOR THE 20 KILOMETER WALK WAS JEFERSON. HIS TIME WAS 1HOUR, 20 MIN,AND 7SECONDS. HIS TIME WAS NOT GOOD ENOUGH TO BEAT THE WINNER IN 1988 BY JOSEF. HIS TIME WAS 1 HOUR,19 MIN, 57 SECONDS. WHAT WAS THE WALKING RATE OF EACH PERSON?
Answered by Penny Nom.
When is 1! + 2! + 3! + ... + x! a square? 2002-08-19
From Sarathy:
Solve :

1! + 2! + 3! + ... + x! = y 2

How do i find the solutions ?


Answered by Claude tardif.
Combination locks 2002-08-17
From John:
How many combinations of three number codes can occur with a standard combination lock? The range of possible numbers is 0-39 which allows 40 different possible choices each turn of the dial. Each combination requires exactly three numbers (i.e. three turns of the dial). E.g. 35-40-5. Allow for reuse of the same number three times e.g.10-10-10. I don't think there is any preference shown for certain number combinations. Can you explain how the total is derived? Someone at the YMCA this week put this question to me and I gave an answer but I am not sure I am right--so I have come to you.
Answered by Penny Nom.
Volume of a sphere 2002-08-16
From Rahul:
How do i find the volume of a sphere without the knowledge of calculus?
Answered by Walter Whiteley.
Distance in 3-space 2002-08-16
From David:
The question is: how do I figure out the distance of one object in 3D space to another object in 3D space? I have an object at say x = 5.872, y = 2.876, and z = 7.290; and the other object is at x = 1.129, y = -8.213, and z = -11.127. I have been suggested to use the pythagorean theory on this, but since there are three variables, I don't understand how.
Answered by Penny Nom.
Probability 2002-08-16
From Chris:

There are two possible outcomes for a random event, A and B.

The probability of A being the outcome is 63%, and B 37%.

What is the likelihood that B will be chosen twice, consecutively? Three times, ten?


Answered by Andrei Volodin.
Variable expressions 2002-08-15
From Brittany:
I am just learning variable expressions. My homework is to write a variable expression for each word phrase;

example :

the number of minutes in s seconds
the cost of x cartons of juice at $.75 each
And etc.

I don't understand it

Answered by Penny Nom.
A paper model of a cone 2002-08-14
From Bruce:
I have made a paper model of a cone, cut a sloping section, and removed the top. I have drawn the major and minor axis on the paper surface of the section. The major axis is not symmetrical about the minor axis. To me, this is not an ellipse. To me, an ellipse is a tubular section, because this gives a symmetrical major axis. What is your opinion?
Answered by Walter Whiteley and Chris Fisher.
The circumference of a 72 2002-08-14
From Linda:
What is the circumference of a 72" diameter circle?
Answered by Penny Nom.
Two equations 2002-07-26
From Derek:
1. 3x + 2y = 4
2. -7x + 2y = 24

finding x and y.


Answered by Penny Nom.
Some inequalities 2002-07-25
From A student:

1)-5 <= -3X+1 < 1

2)4X-2 < 6 OR X+2 > 9


Answered by Penny Nom.
Musical Scales 2002-07-24
From Terence:
Given that there are 12 notes in a musical octave, what is the maximum number of musical scales possible within that octave, if each scale has a minimum of 5 notes and a maximum of 9 and we start all the scales from the same note?

In case you don't know anything about music, a scale is a progression of notes where you start on a specific note and end on that same note an octave higher. There are twelve different notes between these two similar notes. Which notes you choose to play determine the sound of the scale. Anything less than five notes would not make for a very interesting scale. Anything more than nine and you would be playing almost 'every' note in the scale, not leaving much room for distinction in how you organize these notes.

I assume you first have to figure out the maximum number of variations possible in a 5-note scale (with 12 notes at your disposal). Then do the same for a 6-note scale, then a 7-note, then an 8-note, and so on. Then add up the results. How to find this maximum number of variations for each scale size though is what I don't know.


Answered by Leeanne Boehm.
20 men dig 40 holes in 60 days 2002-07-24
From Lindsey:
20 men dig 40 holes in 60 days. So, 10 men can dig 10 holes in how many days?
Answered by Penny Nom.
x + 5 + x, if x = 4 2002-07-23
From Elisa:
x + 5 + x, if x = 4
Answered by Leeanne Boehm.
7th grade math lesson plans 2002-07-23
From Peter:
Where can I get samples of 7th grade math lesson plans and curriculum for the whole year?
Answered by Leeanne Boehm.
As much greater than 47 as it is less than 105 2002-07-23
From Joe:
To find the number in question you need to find the number exactly half-way between 47 and 105. Another way of thinking about the number that is half-way between two other numbers is as the average of the two numbers given.
Answered by Leenaan Boehm.
Place six numbers around a triangle 2002-07-19
From Monika:
I need to place six numbers around a triangle, as such that one number is on each corner, and one number on each side making three numbers in one line, adding up to two hundred exactly. The number I have to use are, 40, 50, 60, 70, 80, 90. I have to use each one once, there for, each number cannot be repeated.
Answered by Claude Tardif and Chris Fisher.
Can you make up a word that adds up to 1000000? 2002-07-19
From Adrie:
My question is elementary and I am a student Please help me with the following conundrum, as it is driving me crazy.

'When the letters in the alphabet are given values from 1 to 26 for A to Z, can you make up a word that adds up to 1000000 ( one million )-used as a multiplication?'

Like 'cat' would be 3*1*20 = 60

Answered by Claude Tardif.
A rope around a post 2002-07-15
From Nic:
My daughter who is in the 10th grade asked me this ?, you have a pole 45 inches high, 22inches around. You have a rope 1/2 thick, how many feet of rope would you use to go from top bottom of pole to top.
Answered by Penny Nom.
A probability problem 2002-07-12
From Yvonne:
What is the probability of 38 people choosing the same five options, given a choice of 15?

Perhaps you could just give me the formula with an explanation.

I am an English teacher doing educctional research, but am stuck because I don't have the math experience to help me.


Answered by Andrei Volodin.
The Praxis II test 2002-07-09
From A student:
I am taking the Praxis II test and I can't seem to answer certain styles of questions. Please help!!!!!!
  1. In a class of 29 children, each of 20 children has a dog and each of 15 has a cat. How many of the children have both a dog and a cat?

  2. If there are exactly 5 times as many children as adults at a show, which of the following CANNOT be the number of people at the show? a. 102 b. 80 c. 36 d. 30

  3. In order to estimate the population of snails in a certain woodland, a biologist captured and marked 84 snails there were then released back into the woodland. Fifteen days later the biologist captured 90 snails from the woodland, 12 of which bore the markings of the previously captured snails. If all of the marked snails were still active in the woodland when the second group of snails were captured, what should the biologist estimate the snail population to be based on the probabilities suggested by this experiment? a. 630 b. 1,010 c. 1040 d. 1080.

Answered by Paul Betts.
A polygon inscribed within an ellipse - Part 2 2002-07-08
From Steven:
I recently sought your advice about a problem that I have been working on for eight years or so concerning a polygon inscribed within an ellipse. I think that I may have confused matters by the way in which I put the question and hope that the enclosed diagram will clear matters up. In the ellipse below I have drawn three chords inscribed within one quadrant ( this would pertain to a twelve sided figure within the whole ellipse). These chords are exactly the same length as each other, for example if the major axis of the ellipse was 360 and the minor axis 240 I have worked out that a twelve sided figure would have sides of 78.2487. However I worked this out empirically with a method that could only be described as gruelling I would be most grateful if you could tell me of a method that would work for any ellipse and any number of sides.
Answered by Chris Fisher.
The base 10 multipliction table 2002-07-07
From A student:
These are two questions from Math for Elementary Teachers and they have me stumped.

You have two coins that are worth 30 cents. One of the coins is not a nickel. What are the two coins?

The product of the diagonals of any 2x2 matrix in the base 10 multiplication table are equal. Why?


Answered by Penny Nom.
A box that holds 22.4 liters 2002-07-07
From Jon:
How do I find the measurements required to make a box that holds 22.4 liters of something?
Answered by Penny Nom.
Equivalence relations on a set of cardinality n 2002-07-06
From Siddhartha:
what is the no. of equivalence and transitive relations on a set of cardinality n?
Answered by Penny Nom.
An equalateral polygon inscribed within an ellipse 2002-06-30
From Steven:
How would you calculate the length of one of the sides of an equalateral polygon (of n sides) inscribed within an ellipse ( of any eccentricity ) where all of the vertices exactly touch the perimeter of the ellipse?

I know that when the eccentricity is zero ( i.e a circle ) the formula: r * (sin(180/n) * 2) will suffice. But what about when the eccentricity is greater than zero?


Answered by Chris Fisher.
Future value 2002-06-27
From Susan:
If one were to invest $115 a month for 20 years and expect a 4% annual return, what is the value of this money in 20 years?
Answered by Leeanne Boehm.
Jack and Jill wrote letters 2002-06-19
From Angel:
jack and jill each bought a stationary
# of sheets of paper in each box were the same
# of envelopes were the same
jill wrote letters consisting of 3 pages
jack wrote letters consisting of 1 page

when they wrote all letters jill had 50 envelopes left and jack had 50 sheets of paper left how many pieces of paper and how many envelopes?

Answered by Leeanne Boehm.
Integrating x^x 2002-06-18
From Jeremy:
I am a student at the University of Kansas and I am wondering if there is a general anti-derivative for x x (i.e. the integral of x x dx)? I've looked in a bunch of Table of Integrals and have found nothing (can you guys find it?), so I'm sort of wondering if this may be a research type question.
Answered by Claude Tardif.
A good rule of thumb when driving 2002-06-13
From Lisa:
A good rule of thumb when driving is that you should be about one car length away from the car in front of you for every 10 miles per hour that you are travelling. Suppose you follow this rule perfectly (so you are exactly the correct distance away). You are waiting at a stop light with your front bumper just touching the car in front of you. The light turns green and the car in front accelerates at a constant rate "r". Calculate how you should accelerate in order to follow the rule.
Answered by Penny Nom.
The cross country team 2002-06-12
From Denae:
In cross country, a team's score is the sum of the first five finishers on the team. The captain of the team finished 2nd in the meet. The next four finishers on the team placed in consecutive order. The team score was 40. in what places did the other members finish?
Answered by Penny Nom.
An equilateral triangle 2002-06-11
From Sarah:
Hi. My name is Sarah. I'm a secondary student taking a Math 30C course by correspondence. The question has two parts.

The first part is: Draw an equilateral triangle XYZ. Draw the altitude from X to YZ. Choose any point P inside the triangle or on the triangle. Draw perpendiculars from P to the sides of the triangle. The Second part is:

Measure the altitude h and the 3 perpendiculars s, t, and u to the nearest mm. Repeat as many time as is necessary until you can state a generalization concerning h, s, t, and u. If you could help me, it would be greatly appreciated.

Answered by Penny Nom.
Drawing cards 2002-06-11
From Ed:
What is the probability of drawing without replacement from a standard deck of 52 cards the following 5 card hand ...... the ace of spades, 2 tens and 2 face cards

Solution 1: 1/52 x 4/51 x 3/50 x 12/49 x 11/48

Solution 2: Using Combination theory ..... (1C1 x 4C2 x 12C2) divided by (52C5)

Can you help us understand which answer is correct and why the other is not?


Answered by Penny Nom.
Successive coefficients in the nth row of Pascal's Triangle 2002-06-10
From Tim:
There is a formula connecting any (k+1) successive coefficients in the nth row of Pascal's Triangle with a coefficient in the (n+k)th row. find this formula.
Answered by Penny Nom.
Regular polyhedra 2002-06-07
From Sandra:
The other day a colleague and I were talking about polyhedra. Is regular a term applied to polyhedra or just polygons? If so, then what would define a regular polyhedron? Would it mean all faces are regular or would it mean that all faces are identical and regular? That is, could a pyramid with equilateral triangles for lateral faces and a square base be considered regular or must the base also be an equilateral triangle?
Answered by Chris Fisher.
Constructing the square root of 3 2002-06-07
From Allan:
I am a Math 7/8 teacher. I was wondering how you would show a student how to find the exact location of the square root of three on the number line using just a compass and a straight edge.
Answered by Penny Nom.
A polynomial 2002-06-05
From Melissa:
I'd like to know what is a polynomial( the definition and an explication)? And is 7x a polynomial? and why?
Answered by Penny Nom.
The area of a circle 2002-06-03
From Jessica:
I am doing a maths assigment for university, which is aimed towrds primary school students(k-6). I was wondering if you could give me some information as to how I could describe to students the rule for finding the area of circle, using a circle cut up into equal sectors (like a pizza). I know it has something to do with the fact that you can make these shapes into a parallelogram, but I am a bit uncertain as to how I can express this idea clearly and articulately to students.
Answered by Penny Nom.
Conics 2002-05-29
From Brooke:
Which conic cannot be generated by an intersection of a plane and a double napped cone?
Answered by Chris Fisher.
Overlapping circles 2002-05-29
From Naman:
There are two circles, big circle with radius R and small one with radius r. They intersect and overlap in such a way that the common area formed is 1/2 pi r 2 (half the area of the small circle) If r=1, find the Radius of the big circle (R)?
Answered by Harley Weston.
The sum of the areas of two regular decagons 2002-05-27
From A parent:
The sum of the areas of two regular decagons is 39 square inches, and their radii are in the ratio of 2:3. Find the area of the larger decagon.
Answered by Paul Betts.
Linear programming 2002-05-27
From Jes:
A machine shop makes two parts, I and II, each requiring the use of three machines, A, B, C. Each Part I requires 4 minutes on Machine A, four minutes on Machine B and five minutes on machine C. Each Part II requires five minutes on Machine A, one minutes on Machine B and six minutes on Machine C. The shop makes a profit of $8 on each Part I and $5 on each Part II. However, the number of units of Part II produced must not be less than half the number of Part I. Also each day the shop has only 120 minutes of machine A, 72 minutes of Machine B, and 180 minutes of Machine C available for the production of the two parts. What should be the daily production of each part to maximize the shop's profit?
Answered by Claude Tardif.
Cubic feet and cubic yards 2002-05-27
From John:
how do you find the cubic feet/cubic yard of area 10 feet wide by 15 ft long by 2 inches high
Answered by Penny Nom.
One-fourth of a number is added to one-third of the same number 2002-05-26
From A student:
When one-fourth of a number is added to one-third of the same number, the result is 28. What is the number?
Answered by Penny Nom.
A spotlight shines on a wall 2002-05-25
From Barb:
A spotlight on the ground shines on a wall 12m away. If a man 2m tall walks from the spotlight toward the bldg at a speed of 1.6 m/s, how fast is his shadow on the bldg decreasing when he is 4m from the bldg?
Answered by Penny Nom.
A schedule for 24 golfers 2002-05-25
From John:
I am working on a schedule for 24 golfers. 6 groups of 4. I have 8 golf days (twice per week for a month).

Ideally, I would like to schedule all 24 golfers in 6 different groups of 4 on each day. Here is the catch.....no golfer in any group can be grouped togther more than once. Every group of 4 each day will have 4 new golfers that have never played together before. Is this possible?


Answered by Chris Fisher.
| n + 4 | + | 3 - 2n | = 16 2002-05-24
From Randall:
I don't seem to be able to solve this: | n + 4 | + | 3 - 2n | = 16
Answered by Penny Nom.
Blood donations 2002-05-23
From Fiza:
One particular high school encourages students to donate blood. the high school gym is set up for purpose. the distribution of blood type in north america is as follows.

Type O:44%
Type B:10%
Type A:42%
Type AB:4%

(a) what is the probability that the first two people in the line up have the same type of blood?

ANSWER: I donn't know how many people are in the line. so how can I answer this question.

(b) what us the probability thar none of the first five people in the line up have type AB blood?


Answered by Andrei Volodin.
3 radians subtends an arc of 27 meters 2002-05-22
From Kyle:
In circle O, a central angle of 3 radians intercepts an arc of 27 meters. Find the number of meters in the length of the radius.
Answered by Penny Nom.
Bob swam across a river 2002-05-22
From Torri:
Bob swam across a river 420 ft wide. A strong current carried him 580ft downstream as he swam. Find x, the distance bob actually swam.
Answered by Penny Nom.
Telephone banking 2002-05-22
From Fiza:
To use telephone banking to pay bills, the customer has to enter the last three digits of each bill. the numbers 0 to 9 can be used. if the number happened to be the same on more tan one bill, the customer has to enter the first three digits as well.

(a) what is the probability that a person has 2 bills to register with the same last three digits?
ANSWER:P(2 bills to register)=1/10P3 1/(10!/7!), 1/(10*9*8)=.001389

(b) what is the probability that a person with 10 bills to register has at least 2 bills with the same last three digits?
ANSWER:P=1/(10*10*10) =0.001


Answered by Andrei Volodin.
Probability and investment 2002-05-19
From Bally:
Investors do not invest all their money in the investment predicted to be the best beuause it is too risky. an investor holds shares in three independent cmpanies. company A, Company B, compnay C. an analyst predicts the probability of profit increases for each company as follows. Company A:0.70 Compamy B:0.50 Compnay C:0.65

(a) If the analyst's predictions are true, what is the probability that all three comapanies show a profit incresse?

ANSWER: I try using vin diagram but it did'nt work, I dont know any other ways to solve this question

(b) if the analyst's predictions are true, what is the probability that at least one comany shows a profit increase?


Answered by Andrei Volodin.
Conditional probability 2002-05-19
From Manny:
In a certain school, it is known that 80% of the students use the internet for school projects, 60% use e-mail on a regular basis, and 90% use the internet for school projects or for e-mail on a regular basis. a student from this school is selected at random

Determine thge probability that the student used e-mail, given that the student used the internet for school projects.
ANS: how can i solve this question by useing the vin diaagram


Answered by Andrei Volodin.
Flying with the wind 2002-05-19
From SaraJean:
It takes 6 hours for a plane to travel 720 km with a tail wind and 8 hours to make the return trip with a head wind. Find the air speed of the plane and the speed of the wind current.
Answered by Penny Nom.
How far apart are the transmitters? 2002-05-18
From Jeff:
A ship at sea is 70 miles from one transmitter and 130 miles from another. The measurement of the angle between the signals is 130 degrees. How far apart are the transmitters?
Answered by Penny Nom.
Find the angle measures 2002-05-18
From Amanda:
In triangle ABC; the measure of angle A is 20 degrees more than twice the angle B. The measure of angle C Is five times angle B. Find the angle measures.
Answered by Penny Nom.
Chord length 2002-05-17
From Ashlie:
How do you find the chord length of one section of a chord if you only have the diameter length and the other whole chord length.

WV is the diameter and equals 16. XY is perpendicular to it, and equals 10. They intersect at pt. Z. I need to know what WZ equals. Please help!


Answered by Penny Nom.
On which assignment did I do better 2002-05-17
From Denise:
I have 2 writing assignments in class. The first assignment, which had a mean of 10 and a standard deviation of 2, I got a score of 12. The second assignment had a mean of 18 and an s.d. of 3, I got a 21. I need to know which assignment did I do better, relative to my classmates?
Answered by Andrei Volodin.
Day care 2002-05-13
From Sonam:
In many familes, both parents work. as a result, there is increasing need for day care. data was collected; and in one year in Canada, approximately 32% of children aged 0 to 11 years were in day care for at least 20h per week.

(a) what is the probability, in a random poll of 60 children form the age of 0 to 11, that more than 15 children are in day care at least 20 h per week? nearest tenth of one %

ANSWER:
P(children are in daycare at least 20h)= 60/60C14 = to the answer

(b) what is the probability, in a random pool of 60 children that fewer than 20 are in day care at least 20 h per week?

ANSWER:
P= 20/60= 33.3% stay in day care for 20h per week,

I dont know if these answers are right please help me out.


Answered by Andrei Volodin.
What is Calculus About? 2002-05-13
From A student:
I am a high school senior and have to write an essay answering the question "What is Calculus?" I need some ideas. Thanks
Answered by Walter Whiteley.
A game of dice 2002-05-13
From Mika:
To play a game a dice is rolled to see who plays first. four players are going to play the game. what is the probability that at least two people roll the same number?

ANSWER: P(least 2 people roll the same #)=number of player/total outcome

P = 4!/4C2 = 24/4C2 = 4 the answer is 40%


Answered by Andrei Volodin.
An award ceremony 2002-05-13
From Mika:
Five students are equally qualifide their school at an award ceremony. two students will be randomly chosen to go to the ceremony.

(a) if Tony is one of the five student, what is the probability that he will be chosen?
Ans: 2/5C2=0.2 which is 20%

(b)Tony's friend, Allecia, who was also eligible to go, has been chosen. what is the Tony's probability of being chosen after he learns that allecica is going:
Ans: 1/5C1=0.2

I think the answes are wrong please help me with these questions


Answered by Andrei Volodin.
Some algebra 2002-05-11
From A student:

/x - 2 ^4/x = 8

/ = square root sign
^ = 4th root of x

also, dividing
2x3 -3 -6x
__________
4 + 2x

the 2x3 is 2x to the third


Answered by Penny Nom.
The law of cosines and obtuse angles 2002-05-09
From Bryant:
The question that I am pondering is that I need to derive the law of cosines for a case in which angle C is an obtuse angle.
Answered by Penny Nom.
A rectangular marquee 2002-05-07
From Alyaa:
a marquee with rectangular sides on a square base with a flat roof is to be constructed from 250 meters square of canvas. find the maximum volume of the marquee. i find this topic so hard
Answered by Harley Weston.
Forming a committee 2002-05-06
From Navi:
A committee is to be formed to investigate what activites teenagers have available in small communities. the committee is to have 7 members, chosen randomly form interested community members. there are 10 parents, 5 teenagers, and 4 adult without children who have all expressed an interest in serving on the committee.

what is the probability that the adults without children are all on the committee?


Answered by Andrei Volodin and Penny Nom.
Oven lights 2002-05-06
From Manny:
Q.A manufacturer of stoves has to buy oven lights from from two diffrent companies because one company alone cannot meet its demand. the manufacturer purchases 60% of the oven lighs from comany A and the rest form Company B. Past experiecnce shown that 1% of Company A's oven are defective oven and 2.5% of Company B's oven light are defective.

Determine the probability that a defective oven light is supplied by company A.


Answered by Andrei Volodin.
How will I use calculus in my career? 2002-05-06
From Meridith:
How will I, hopefully a future secondary mathematics teacher, use calculus in my career if I'm not teaching calculus?
Answered by Walter Whiteley.
3 = -2x 2002-05-05
From Timothy:
My question is 3 = -2x

How do I isolate the variable here?


Answered by Penny Nom.
A probability tree 2002-05-04
From A student:
Karl and Naomi roll three dice. Karl gives $10 to Naomi if the number 5 turns up once, twice or three times. Naomi gives Karl $3 in all other cases. A) construct the probability tree for this situation.

B) Does Karl have an advantage in this game? Justify your answer.
My problem is I am unfamiliar on how to draw this probability tree and seem to be having a lot of difficulties, when you have a chance please answer this math problem and get back to me with the results.

Answered by Penny Nom.
Factor 12x^2 - 20x - 8 2002-04-30
From Crystal:
Factor Completely: 12x2 - 20x - 8
Answered by Penny Nom.
Trains and cracker boxes 2002-04-29
From Lindsay:
1)Two trains are 250 miles apart on the same track heading towards eaxh other at 7AM- The first train travels at 55mph, the second at 75mph. At what time would they crash?

2)A cracker box measures 12 by 2 by 18 inches. The company wants to double the amount of crackers, but keep the shapes the same[similar] Find the new length[nearest tenth of an inch]


Answered by Penny Nom.
Conversion of units 2002-04-29
From Matt:
I am having trouble finding a workable equation for the following problem, particularly the conversion of two systems of measurement.

The question states that there are 27 grams of salt in a kilogram of seawater. And it asks how many tonnes (1 tonne = 1000 kilograms) of salt are there in one cubic kilometer of ocean. It also says that the density of seawater is 1.1 g/cm3.

What is the equation and how do I figure out how many kilograms there are in a cubic kilometre?


Answered by Harley Weston.
A 6,000 gallon tank 2002-04-28
From Someone with a big tank:
I was wondering if you can find out information about how much liquid fuel can fit in a 6,000 GAL. tank. Right now I do not have the exact dimensions of this tank but I will have it the 29th of April. Could you please E-mail me and let me know if you can answer this question after I have the correct specifications.
Answered by Penny Nom.
A triangular prism 2002-04-28
From A student:
Need the formula for the surface area of a triangular prisim?
Answered by Penny Nom.
a+b=10 and ab=40 2002-04-27
From April:
What two numbers add to ten and multiply to forty?

(a+b=10, a*b=40)

I think the answer includes radicals and/or imaginary numbers.


Answered by Penny Nom.
Designing a ballot 2002-04-26
From Kelley:
I want to design a ballot for four elections. Actually all the candidate races are on 1 ballot. I need to know how many different ballot styles would be needed for all of the candidates to be in each rotation an equal number of times.

For example:

  • A,B,C & D are running for mayor

  • E,F,G & H are running for congress

  • I,J & K are running for senate

  • L,M & N are running for governor

They are all on the same ballot. But in each race their name (for instance A) has to be in the #1 rotation, #2 rotation, #3 rotation, and #4 rotation for his race on this ballot an equal number of times as B,C and D.

The same goes for the other candidates for their perspective races.

How many total ballot styles will there be?


Answered by Penny Nom.
Triangular numbers 2002-04-26
From Anika:
Can you please tell me what a triangular number is?
Answered by Penny Nom.
Functions and relations 2002-04-25
From Erin:
Here's a few math problems that might drive a person insane ;)
  1. If 2f(x) - 3f(1/x) = x^2 what is f(2)?

  2. What are the domain and range of the following relations?
    • x^2 - 4y^2 = 25
    • 4x^2 + 9y^2 = 36

  3. The port of Swan Harbor is 200km away from Merry Town Inlet on a bearing of N50E from Merry Town. A ship leaves Merry Town at 8am and sails N15W at 15 km/h. At the same time, a second ship leaves Swan Harbour on a course of S80W at 20km/hour. How close, to the nearest km, are the two ships at 13:00?

  4. Prove the identity. 1+sinx + cosx/ 1-sinx + cosx = 1+sinx / cosx

Answered by Penny Nom.
An Olympiad problem 2002-04-23
From Aurora:
I wrote a mathematical Olympiad the other day and there was one question that I could not work out.

The question was If ab = 1, bc =2, cd =3, de =4 and ea = 6, what does a + b + c + d +e =

It was a multiple choice and the answers were : 43/6; 47/6; 49/6; 53/6; 61/6 The correct answer was 61/6, but how can do you get to that answer?


Answered by Anrei Volodin.
A test drive 2002-04-23
From Jennie:
At 9 A.M. a test car driving at a constant speed passes a marker 50 miles from it's starting point. At noon the car is about 130 miles from the marker. If the test drive ends at 1:30 P.M., how far will the car be from its starting point?
Answered by Penny Nom.
25m = 100 2002-04-22
From Megan:
If 25m = 100, Then m equals what number?
Answered by Penny Nom.
Lines of symmetry 2002-04-22
From Cindie:
How many lines of symmetry do the following figures contain?

trapezoid:

rhombus:

hexagon:

pentagon:


Answered by Walter Whiteley.
Problem solving 2002-04-20
From Kathy:
  1. Zachary wanted to run the shore-to shore marathon next month, a 26 mile race from one side of the island where he lived to the other. He decided to work up to 26 miles by running five days a week for the next five weeks. Each day he'd run one mile longer than he did the day before, except for the first day of the week, when he'd run the same number of miles he ran on the last day of the previous week. He wanted his last two practice runs to be 26 and 27 miles long.

    How many miles would he have to run on his first practice run?

  2. A school just bought a new set of encyclopedias. The front and back covers are 1/8 inch thick, and the pages are 1/100 inch thick (in other words, there are 100 pages per inch). the 26 volume set has a total of 5,250 pages. If you assume there is no extra space left between books, then .........

    What length of bookshelf will the encyclopedias take up?

Answered by Penny Nom.
Solve for x 2002-04-20
From A student:
solve for x.
  c-x      2x+b ----- = --------   a        c 

Answered by Peny Nom.
Cubic yards 2002-04-20
From Pat:
I HAVE AN AREA 10'x6'I WOULD LIKE TO PUT LOAM ON IT 5" DEEP HOW MANY CUBIC YARDS OF LOAM DO I NEED?
Answered by Harley Weston.
An augmented matrix 2002-04-20
From A student:
Hi my math teacher asked us to solve an augmented matrix. I am in twelfth grade and need help. The book we are working on is college algebra. Here it is

a+2b+c=0
2a+5b+4c=-1
a-b-9c=-5


Answered by Penny Nom.
Solving for x 2002-04-19
From Susan:
I'm having a little trouble solving these equations for x. I can't seem to separate x fully from the other numbers. Please help! Here are the problems:

(2x-1)/(x-2)(x 2+3) = 0 and

y/(x+1)=z/x
Answered by Penny Nom.

Moving a triangle 2002-04-18
From A student:
find the verticles of a triangle after it is translated 2 units to the left and then is reflected across the graph of y=x+2. The original verticles of the triangle are (2,0), (3,2), and (6,2).
Answered by Peny Nom.
Arc length 2002-04-17
From Vix:
Find the point on the curve r(t)=(12sint)i-(12cost)j+5tk at a distance 13pi units along the curve from the point (0,-12,0) when t=0 in the direction opposite to the direction of increasing arc length.
Answered by Harley Weston.
Ratios 2002-04-17
From A student:
  1. Medication is givin at 1cc. per 20 pounds of body weight. A dog weighs 16 pounds how many ccs. of medication does the dog get?

  2. Dr. Lee wants you to prepare 1000 ml. of a 10% dextrose solution in lactaed ringers for diuresis or a patient you have a 1 L bag of lactated ringers and 500 ml of 50% dextrose for injection. How would you prpare the solution?

Answered by Penny Nom.
Related rates 2002-04-17
From Molly:
A tanker spilled 30 ft cubed of chemicals into a river, causing a circular slick whose area is expanding while its thickness is decreasing. If the radius of the slick expands at the rate of 1 foot per hour, how fast is them thickness of the slick decreasing when the area is 100 feet squared?
Answered by Penny Nom.
Two algebra problems 2002-04-14
From Ashley:
Question 1: For all values of a, let < a > be defined as < a > = 4a - 4 . Which of the following equals < 6 > - < 5 >? A: < 2 >
B: < 3 >
C: < 4 >
D: < 5 >
E: < 6 >

Question 2: If 5n + p = 3 and 2m - 10n = 2, what is the value of m + p?


Answered by Penny Nom.
A polynomial 2002-04-14
From A student:
A polynomial function is described by the following conditions: f(x) has two real zeros at x= -2 and x= 1, each of multiplicity 2 f(x) has two complex zeros as x goes to -infinity, f(x) goes to +infinity as x goes to +infinity, f(x) goes to +infinity f(x) has a y-intercept at (0,2)

Sketch a polynomial function that satisfies the above conditions.

Write an equation for function f(x)

Write another function g(x) that also satisfies the above parameters.


Answered by Penny Nom.
How many gallons of water in a cubic foot? 2002-04-14
From Richard:
I am tring to find out how many gallons of water in a cubic foot.
Answered by Penny Nom.
The bleep test 2002-04-13
From Lorraine:
I am currently undergoing training and have asked about a fitness test called the bleep test. The bleep test involves running continuously between two points that are 20m apart. These runs are done in time to a pre-recorded bleep sounds on a pre-recorded audio cassette. The time between the recorded bleeps decrease after each minute. I would like to do my own personal version for training but I have to work out the following before doing this:

I need to find out the time in-between the beeps at least for the first level so I can work out the difference for the other levels:

The information given to me for the first level is :

A 20 meter run at 8.5km/hr (how long would it take?)

Please let me know if there is any way in working out this query with the information given.


Answered by Leeanne Boehm.
Diagonals in a polygon 2002-04-10
From Murray:
If you have a regular polygon with n sides and you draw all (n-3)n/2 diagonals how many intersection will they form with each other and how many sections will they devide the polygon into.
Answered by Caude Tardif and Chris Fisher.
The Proof in the Pudding 2002-04-09
From A student:
The host at a party turned to a guest and said, "I have three daughters and I will tell you how old they are. The product of their ages is 72. The sum of their ages is my house number. How old is each?" The guest rushed to the door, looked at the house number, and informed the host that he needed more information. The host then added, "The oldest likes strawberry pudding." The guest then announced the ages of the three girls. What are the ages of the three daughters?
Answered by Claude Tardif.
Area of a triangle 2002-04-07
From Puzzled:
What is the surface area of a triangle 13 ft x 13 ft x 18 ft?
Answered by Penny Nom.
3/4n-6=12 2002-04-07
From Diane:
I am in 6th grade advanced math and I just started Algebra. My teacher tries to explain it to me but I just can't get it. I am having difficulty understanding it. Please HELP!!! I will type a problem, if you could answer it and show your work so i can understand it that

would be great. Please explain in simple terms.

here is a couple problems I am having trouble with,

PROBLEM #1

3/4n-6=12


Answered by Penny Nom.
Pairs of equations 2002-04-04
From A student:

high school level
student is asking

y=4x x=-4y 
x+y=5 3x+2y=20   

y=x-1 3x-y=4 
x+y=3 2x-3y=-9   

x+5y=4 
3x+15y=-1 

. . . 

Answered by Penny Nom.
Two triangles 2002-04-03
From Scott:
Consider 2 triangles: Triangle PMB and Triangle PLA.

Triangle PLA is contained within Triangle PMB.

Side LA is parallel to Side MB.

Point L is located on Side PM. Point A is located on Side PB.

If the ratio PL:LM = 5, then what is PB:PA ??


Answered by harley Weston.
Take It! 2002-04-03
From Bryan:
You are playing Take It! for $180,00 with a total stranger. There are 180 identical balls in a big vase. Each player in his turn, reaches into the vase and pulls out 1,5,or8 balls. These balls are discarded. The player who takes the last ball from the vase wins the $180,000. A flip of the coin determines that you will go first. Are you glad? How many will you take out on the first move, and how will you proceed to win the prize?
Answered by Claude Tardif.
Common fractions to decimal fractions 2002-04-02
From Natalie:
My questions are: "how can I change 23/60 into a decimal. and give my answer to 2dp."??????????

2nd question:for each of these pairs, which is bigger?

4/7 or 36%____________
3/11or 0.29____________
14/24 or 0.57___________


Answered by Penny Nom.
Pascal's Triangle 2002-04-02
From Brian:
It's about (a+b)x. I remember there a triangle with numbers to remember for a faster solution. Can you please teach me?
Answered by Penny Nom.
If you conect all the vertices of a regular n-gon... 2002-04-01
From Murray:
If you conect all the vertices of a regular n-gon with lines you will have (n-3)(n/2) lines inside the polygon. I want to find out how many sections these lines divide the polygon into and how many intersections they have with each other.
Answered by Claude Tardif.
All four eights in three consecutive hands 2002-04-01
From Blair:
What are the odds against the same person holding all four eights in three consecutive hands.
Answered by Brian Alspach.
Meg's biscuits 2002-03-29
From Tami:
Meg's biscuits recipe calls for 4 cups of flour for every 1.5 cups of milk. If Meg uses 6 cups of flour, how many cups of milk will she need?
Answered by Penny Nom.
The reciprocal of a decimal mixed number 2002-03-29
From A student:
How do I find the reciprocal of a decimal mixed number?
Answered by Penny Nom.
Some 5 card hands 2002-03-28
From A student:
From a standard deck of cards how many 5 card hands are possible consisting of a. exactly 4 hearts

b. two cards of one kind and three of another(like a full house).

Answered by Penny Nom.
100 from four 9's 2002-03-27
From A student:
My teacher gave us a math problem to try and figure out and I am stumped. The problem was this.... by only using four nines, how can you get an answer of 100? You can add, subtract, multiply or divide...anything you need to do ,but you can only use four nines.
Answered by Leeanne Boehm.
If the matrix A is inverible and AB =AC, then B = C 2002-03-27
From Vikki:
i hope you can help i am soooo stuck here goes:

a)

 A= 0 1 B= 1 1 c= 2 5    0 2    3 4    3 4 
A,B and c are matrices

Evaluate AB and Ac (which I can do)

then

b)
I need to prove that if the matrix A is inverible and AB =AC, then B = C. Why does this not contradict what happened in part a)?
Answered by Leeanne Boehm.

A triangle in a circle of radius 6 2002-03-26
From Marko:
In a circle of radius 6, a triangle PQR is drawn having QR = 8 and PQ = 10. Determine the length of PR
Answered by Chris Fisher.
A lottery question 2002-03-25
From Noel:
I want to take all numbers in a Lotto game from 1 to 45 and ditribute them between 9 boxes each containing 5 numbers. Each number is used only once.
  1. What is the probability that I derive the correct combination of 5 numbers in the draw?

  2. What is the probability if I take a systems game and now distribute 9 numbers into each of 5 boxes? Once again, the numbers are used only once.

  3. Can you explain what formulae are used and how the answer is derived? Is this a distribution or combination problem? Or is it a combination of both distribution and combination?

Answered by Andrei Volodin and Penny Nom.
A hamburger and a soft drink 2002-03-23
From A student:
A GROUP OF FRIENDS WENT OUT TO LUNCH. EACH BROUGHT A HAMBURGER AND A SOFT DRINK. TOGETHER A HAMBURGER AND A SOFT DRINK COST MORE THAN A DOLLAR. THE TOTAL COST FOR THE GROUP WAS $17.81. THERE WAS NO TAX OR TIP INCLUDED.

IF A HAMBUGER COSTS 2 CENT MORE THAN TWICE A SODA, FIND THE COST OF THE HAMBURGER.


Answered by Leeanne Boehm.
A probability problem 2002-03-23
From Kate:
If I have a spinner and then spinner has the numbers 1,2,3,4 on it and it is spun 3 times, what is the probability that the spinner will stop on 4,3,and 1 ? In that order?
Answered by Leeanne Boehm.
What fraction of the world's motor vehicles are built in Canada? 2002-03-20
From A student:
About 1/4 of the world's motor vehicles is built in Canada or the United States. About 1/5 of the world's motor vehicles are built in the United States. What fraction of the world's motor vehicles are built in Canada?
Answered by Claude Tardif.
Adding algebraic fractions 2002-03-20
From Dolores:
I get totally confused with this problem. I get confused with the getting the lowest terms.

4/x-5 + -2/x -10/x^2-5x


Answered by Penny Nom.
A cone in 3 space 2002-03-20
From Matthew:
Let C in R3 be the cone defined by x2 + y2 - z2 = 0

(A) Let P be the plane described by x + 2z = 1

(i) Find a description of P in terms of two parameters s and t

.
.
.

Answered by Walter Whiteley.
Interest owing 2002-03-17
From Dean:
I am an investor. If I lend to someone on January 1, 1999, $1000 to be paid back interest at 2% per year (simple interest, ie not compounded), and on July 15, 1999, I lend the same person $1,500 more (total loan is now $2,500) but at 18%, and once again on October 15, 1999, I lend to the same person another $1,500, and the person pays me back the principle($4,000) and interest on January 1,2001: What is the total (principle plus interest) due me? The tougher question is: what was my blended rate of return on my total outlay of money? Is there a formula you can give me so that I can perform my own calculations?
Answered by Harley Weston.
Combinations? 2002-03-16
From Sara:
A combination lock has three dials numbered 0 - 9. Give a mathematical reason why "combination" is a bad name for this type of lock.
Answered by Penny Nom and Chris Fisher.
How much olive oil? 2002-03-15
From Annie:
Joe used 1/3 cup of olive oil, which was 3/4 of the amount that he had. How much olive oil did he have!
Answered by Penny Nom.
The square root of i 2002-03-14
From Arlene:
what is the square root of i, if i=x+yi?

what is the square root of 1-i? i'm getting problems like these in which I do not understand.


Answered by Harley Weston.
My 1978 Chevy has a 350 cubic inch engine 2002-03-11
From Sharon:
If my 1978 Chevy has a 350 cubic inch engine, about what size engine would that be in liter?
Answered by Penny Nom.
x and y-intercepts 2002-03-10
From A student:
I have the problem- f(x)= X-5/X2+X-6 and I have to find the vertical asymtope, horizontal asymtope, x-intercept, and y-intercept. And graph the problem. I am having problems finding the y-intercept.
Answered by Harley Weston.
Multiplying decimals 2002-03-10
From Kaitlin:
I just recently had a test on multiplying decimals and I did not do really well on it. Only 5 kids out of 19 got a passing grade. My teacher says that she will give us a different test and we can take it again, but I'm still not sure of it. My teacher has taught me a lot and she had done a wonderful job of it but I'm just confused with where to put the decimals and stuff like that. I have a math practice workbook that I do for fun and they have nothing on decimals in it. I have checked on websites to try to find worksheets but I really couldn't find a good one. Can you please help me either give me some tips or clues or send me a worksheet.
Answered by Penny Nom.
The isosceles triangle of smallest area 2002-03-08
From Lettie:
can you find the isosceles triangle of smallest area that circumscribes a circle of radius of one?
Answered by Walter Whiteley.
Factor completely 2002-03-07
From Taylor:
I'm supposed to completely factor this but I don't know what to do with all the variables.

a3b5 - a2b5 - 12ab5.

Does it have anything to do with b5 being a common factor or am I completely off?
Answered by Penny Nom.

A sequence 2002-03-07
From Caroline:
hi, here is the problem,

1
11
21
1211
111221
_ _ 2_ _ _

we can't get the sequence of the last line ,with the 3rd number being 2,thanks


Answered by Walter Whiteley.
The square root of 2 2002-03-05
From Roger:
Does two (2) have a square root or do the numbers just keep going? Are there any other numbers that behave like two when it comes to extracting the square root?
Answered by Penny Nom.
A circular wading pool 2002-03-04
From Patrick:
The community of melfort is planning to build a circular wading pool in the park. The pool will cover an area of 1000m2. The building committee has decided to put a 5m cement pad around it. How much additional area will the cement pad take up?
Answered by Harley Weston.
The slope of a tangent line 2002-03-04
From Ridley:
Suppose a function f(x) has the line 3x+4y=2 as its tangent line at x=5. Find f'(5).
Answered by Harley Weston.
Mixed numbers, whole numbers and improper fractions which are less than 2. 2002-03-03
From Dawn:
Change each mixed number to a whole number and an improper fraction which is less than 2. 3 1/2 = 2 /
Answered by Penny Nom.
What number am I? 2002-03-03
From Samantha:
I am an even number that has more ones than tens. My thousands digit is the quotient of 9 divided by 3 and my tens digit is the sum of 3 and 3. I have no hundreds. What number am I?......Please answer this for me...The answer I come up with is 3,060...Is this right?..
Answered by Penny Nom.
The median with ties 2002-02-27
From Marcel:
What, exactly, is the proper way to determine the median of a set of numbers when doubles or triples of a number are part of that set? Do the doubles count as two and the triples three, or does each count only as one toward determining the median.
Answered by Harley Wston.
Two circles inscribed in a rectangle 2002-02-27
From Amina:
Given a rectangle with dimensions L=6, H=5. Two circles are inscribed such that they touch each other(circles are adjacent to each other) and also their circumferences touch 2 sides of the rectangle. One of the circles has radius=4. Find the radius of the other circle.
Answered by Penny Nom.
Simplify this expression 2002-02-26
From Francine:
2X power 5 Y power 3 times 18 X power -5 Y power -3 divided by X power 1/3 Y power 1/4.

I came up with
     36 ---------- X 1/3 Y 1/4 
The book says 36 x1/3 y1/4.
I don't get it.

Answered by Penny Nom.
sin 2x = cos 3x 2002-02-25
From Allan:
solve:

sin 2x = cos 3x

Primary question: how do you handle the cos 3x?


Answered by Paul Betts and Chris Fisher.
The substitution method 2002-02-24
From Joe:
whats the answer to this question? 3x+y=11
x+2y=-3

its substitution method i am having alot of trouble figuring it out. send the answer as soon as possible. thank you


Answered by Penny Nom.
A hat of a different color 2002-02-24
From Kirstin:
A teacher made a deal with 3 of his students. He said that if you can guess what color hat you have on your head without looking, I will pass you. There were 2 red hats and 3 blue hats. The deal worked like this: The three students would close there eyes, and the teacher would put a hat on each of their heads and then hide the other 2. Then one at a time, the students would open their eyes and try to figure out what color hat was on their head. The student could guess or pass. This is what happened when they put their deal to the test: A boy named Arturo was first and opened his eyes but wasn't sure so he passed so he didn't get it wrong. Belicia was next and she passed too because she wasn't sure. Also she thought about the fact that Arturo didn't know. Carletta was last and without even opening her eyes, she knew for sure what color hat she was wearing and her answer was right. So i have to figure out what color hat she's wearing for sure.
Answered by Claude Tardif.
Alfredos house number 2002-02-21
From Aunt Patty:
Alfredos house number is between 20 and 35. The sum of the digits is less than 5. If you subtract 1 from it you would get a multiple of 3. If you add three, you get a multiple of 5. What is Alfredos house number.
Answered by Penny Nom.
Proof by induction 2002-02-20
From Tamaswati:
How do I prove the assertion that "the determinant of an upper triangular matrix is the product of the diagonal entries" by mathematical induction? (Before I check this assertion for a few values of n how do I rephrase the assertion slightly so that n appears explicitly in the assertion?)
Answered by Penny Nom.
An octagon-shaped deck 2002-02-20
From An instructor:
How can you solve for finding the side measurements of an octagon-shaped deck that is 10 feet long and 10 feet wide.
Answered by Penny Nom.
When will the calendar be the same as 2002? 2002-02-20
From John:
When will the calendar be the same as 2002? I know the answer is 2013, but I don't know how to answer this mathematically, or if it is possible to do so. This is a ninth grade level algebra question my daughter received from her teacher. Any ideas?
Answered by Chris Fisher, Patrick Maidorn and Penny Nom.
Shipping charges 2002-02-19
From Sohail:
A mail order company charges a fixed fee for shipping merchandise that weighs 1 pound or less, plus an additional fee for each pound over 1 pound. If there shipping charge for 5 pounds is $4.80, and for 12 pounds is $6.20, find the fixed fee and the additional fee.
Answered by Penny Nom.
A farmer takes 100 animals to the market to sell. 2002-02-19
From Ann:
A farmer takes 100 animals to the market to sell. All are sold and $100 is earned. The prices were $5.00 per cow, $1.00 per sheep and $.05 per pig. At least one of each kind of animal was sold. How many of each kind of animal did the farmer take to market?
Answered by Penny Nom.
Sampling distributions 2002-02-18
From A student:
  1. given: n = 40, standard deviation is not known, population of individual observations not normal. does the central limit theorem apply in this case? why or why not?

  2. for an estimation problem, list two ways of reducing the magnitude of sampling error?

  3. What will happen to the magnitude of sampling error if the confidence level is raised all other things remaining the same? justify your answer?

Answered by Harley Weston.
Double square roots 2002-02-17
From Ali:
i have a question about how to do double square roots with variables and powers.

example : v/"" v/"" 81y8


Answered by Harley Weston.
A degree 5 polynomial 2002-02-17
From Carla:
Write an equation for a polynomial of degree 5 given the following zeros:
-7+3i, -2+sqrt(13), 6

Answered by Penny Nom.
Diameter of a pipe 2002-02-16
From Landry:
I am trying to calculate the dia. of a pipe 60 inches long that will hold a gallon of water. What is the formula?
Answered by Penny Nom.
The perimeter of an ellipse 2002-02-14
From Harry:
I am planning to build a coffe table with an ellipse of 24x36 for the top. I wish to decorate the edge and need to know the lenght of the perimeter for lay out purposes. Is there an easy way to approximate this figure with out using intergal calculus?
Answered by Penny Nom.
36 is 20% less than _____? 2002-02-13
From Lori:
36 is 20% less than _____?
Answered by Walter Whiteley.
A man and his wife walk up a moving escalator 2002-02-13
From Monty:
A man and his wife walk up a moving escalator. The man walks twice as fast as his wife. When he arrives at the top, he has taken 28 steps. When she arrives at the top, she has taken 21 steps. How many steps are visible in the escalator at any one time.
Answered by Peeny Nom and Claude Tardif.
The slopes of the sides of the Great Pyramid 2002-02-09
From Christina:
The Great Pyramid is the largest of the Egyptian pryamids. When it was built, it was 481 feet tall and had a square base with 755-foot sides. The pyramid has two different slopes-one along its sides and the other along its edges. Which slope is steeper?
Answered by Penny Nom.
The number of hidden cubes 2002-02-05
From Katie:
This problem is about finding the number of cubes visible and hidden in a cube.

In a cube that is 3x3, 19 cubes can be seen. 8 are hidden.
In a cube that is 4x4, 37 cubes can be seen. 27 are hidden.
In a cube that is 5x5, 61 cubes can be seen. 64 are hidden.
In a cube that is 6x6, 91 cubes can be seen. 125 are hidden.

The question is:
Explain how you could find the number of small cubes that are visible and hidden in a cube of any size.


Answered by Paul Betts and Penny Nom.
Parabolas 2002-02-03
From Kuang:
-Who is credited for working with or studying the Parabola?

-What is a conic section?

-What does a parabola look like?

-How is a parabola formed?

-Where and how are parabolas used today in the real world?


Answered by Harley Weston.
Square arrangements of clovers 2002-02-03
From Cassie:
Clyde had a strange fascination with numbers. One day he decided to mount his 4 leaf clover collection in groups of square numbers.he took a long piece of butcher paper and glue and began this arduous task.There was 1 clover,4 clovers.and then 9 clovers in the third set. what would be in the seventh set of 4 leaf clovers?
Answered by Penny Nom.
I am a fraction 2002-02-01
From Anthony:
I am a fraction that is greater than 1 but less than 2. The sum of my numerator and denominator is 11. My denominator subtracted from my numerator is 1. What fraction am I?
Answered by Paul Betts.
The Euclidean Algorithm 2002-01-31
From Murray:
Can you please explain to me why the euclidean algorithm works?
Answered by Penny Nom.
Baseball and sit ups 2002-01-31
From A student:
A baseball team won 8 more games than it lost. If it played 56 games, how many games did it win?

On the first day of practice, Paul did 3 sit ups. On the second day he did 5, and on the the third day he did 8, and on the fourth day he did 12. If this pattern continued, how many sit-ups did ihe do on the sixth day of practice?


Answered by Penny Nom.
The size of a lot 2002-01-26
From Claudia:
I own a piece of property that I need to know the square feet for assessment purposes. The figure they came up with is wrong. They measured from one point to another and halved the sums but that means I own the cul de sac and we don't. My lot is 55 feet wide and one side is 108.96 feet and the other side is 146.04 that extends all the way to a circle. The front of the lot on the cul de sac is stated on the survey like this. 78.21 feet where R=40 feet. This large arc is taken off the size of our land. How many square feet is our lot.
Answered by Harley Weston.
Logico mathematical knowledge 2002-01-23
From A teacher:
How does young children's logico mathematical knowledge develop?
Answered by Walter Whitely.
Adding algebraic fractions 2002-01-23
From Francine:

Hello, I've come up with an answer for the following problem but it's not the same as the book's answer. However, the book has a lot of mistakes in it and I don't know if I'm doing it properly or not. Your help would be soooo appreciated. It's driving me crazy.The problem is:

   
  2             + 3          + 4 
---------  +  ---------  +  ---------- 
(x-1)^3        (x-1)^2       (x-1) 

[(x-1)^3 is (x-1) to power of 3 etc.]


Answered by Penny Nom.
Miles and kilometers 2002-01-22
From Dennis:
I'm in the middle of purchasng a vehicle and it was built in Canada. Therefore it is in kilometers and not in miles. I forgot how many kilometers are in a mile. It registers as 183,049 kilometers.
Answered by Penny Nom and Judi McDonald.
Normal distribution 2002-01-21
From Danielle:
A teacher gave a test on which the students' marks were normally distributed, but the results were pathetic. The mean was 52% and the standart deviation was 12%. The teacher decided that the top 10% of the students should get A's, the next 20% should get B's, the next 40% should get C's, the next 20% should get D's, and the bottom 10% should get F's. To the nearest percent, what are the cutoff marks that will result in an A, B, C, D, and F?
Answered by Penny Nom.
Rolling 5 sevens before rolling a six or an eight 2002-01-20
From Tony:
When rolling 2 dice, what is the probability of rolling 5 sevens before rolling a six or an eight?
Answered by Andrei Volodin and Penny Nom.
The distance across a circle 2002-01-18
From Douglas:
If you know how far around a circle is (say earth) 25000 miles how do you calculate the distance across?
Answered by Penny Nom.
Making 24 2002-01-17
From Renee:
My 4th grade daughter and I need to find a simple math sentence using 5, 5, 3, & 7 to equal 24. You can add, subtract, divide or multiple.
Answered by Penny Nom.
A sale on eggs 2002-01-17
From A student:
A store had a sale on eggs, selling 13 eggs for the usual price of a dozen eggs. As a result, the price of the eggs was reduced by 4 cents a dozen. What is the original price for a dozen eggs?
Answered by Penny Nom.
My salary is doubled everyday for 30 days 2002-01-17
From Kanishk:
I recieve 1 penny the 1st day, 2 pennies the 2nd day, and my salary is doubled everyday for 30 days. How much money will I have by the end of the 30 day time period? (Is there a way of solving this problem without a chart?)
Answered by Penny Nom.
Children and probability 2002-01-16
From Bill:
If a family has four children, what is the probability that all 4 are the same sex?
Answered by Vanrei Volodin.
Linear regression 2002-01-16
From Murray:
If you have a set of coordinates (x[1],y[1]),(x[2],y[2]),...,(x[n],y[n]),find the value of m and b for which SIGMA[from 1 to m=n]AbsoluteValue(y[m]-m*x[m]-b) is at its absolute minimum.
Answered by Harley Weston.
What is a group of three numbers called in a large number? 2002-01-15
From Laurie:
What is a group of three numbers called in a large number?

My son has this question on his 4th grade worksheet. I've taught middle school math for 7 years and nver heard of it.


Answered by Paul Betts.
Algebra review questions 2002-01-14
From A student:
  1. A wildebeast was observed racing a distance of 86.4m in 4.8 seconds. Find the speed of the wildebeast in m/s.

  2. 8x - 7 - 5x = 23

  3. 7 z - 3 (z + 2) = 26

Answered by Penny Nom.
Adding vectors 2002-01-12
From Lena:
how do you add vectors together?
If you are given the length and angles of both vectors and are asked to add/subtract them, how do you do it? I know you are supposed to do the head to tail method, but whenever i try it i get the wrong answer. I need help setting it up.

example: A is 2.7cm, and 60 degrees, B is 1.6cm and 135 degrees, find the magnitude and amplitude


Answered by Penny Nom.
The famous "train problem" 2002-01-12
From Karen:
How do you solve the famous "train problem"? For example, 2 trains leave different stations at the same time. One is traveling at 50 mph and the other at 40 mph. How long does it take them to meet?
Answered by Penny Nom.
Why do integers have the symbol of Z? 2002-01-11
From Stephanie:
Why do integers have the symbol of Z? Natural numbers have the symbol of N, whole numbers have the symbol of W. Our professor has asked us to find out why the letter Z is represented for integers.
Answered by Penny Nom.
A simplification problem 2002-01-10
From A Parent:
FIND

(A / B) . (C / D)
where
A = Y squared + 3 . Y cubed
B = Y squared + 4
C = 2 . Y + Y squared
D = Y + 4 Y squared + 3 Y cubed


Answered by Penny Nom.
An octagon inscribed in a circle 2002-01-10
From Kent:
A circle of 30 in. diameter has an octagon (8 equal chords) inscribed in it. What is the length of each chord?
Answered by Chris Fisher.
Nickles, dimes, quarters and fifty cent pieces 2002-01-08
From A parent:
The total for all coins counted is $4,564.50 The last coin added to the pile is a 50 cent piece There are 8 times as many 50 cent pieces as there are quarters There are 6 times as many dimes as nickles How many of each are there?
Answered by Penny Nom.
Maple trees and pine trees 2002-01-06
From A student:
A team of scientists found that in a forest there was four maple trees for every nine pine trees.How many maple were there if they counted 45 more pine than maples?
Answered by Penny Nom.
Catenary 2002-01-02
From Jason:
I am a high school math teacher. I was asked by a friend who is in architectural design for a method for determining the volume of what he called a Catenary.

The Catenary curve is modeled by the equation y=a cosh(x/a). I ran into a mess when I tried to compute the volume of the solid formed by revolving that curve around the y-axis.

Any help you can provide would be greatly appreciated.


Answered by Harley Weston.
Isoperimetric quotients 2002-01-02
From A student:
I'm stuck on my GCSE Maths coursework, what do isoperimetric quotients measure?
Answered by Penny Nom.
A hyperbola centered at (0,0) 2001-12-28
From Carla:
  1. Find the equation of the hyperbola centered at (0,0) with vertices at (0,3) and (0,-3) that is asymptotic to the lines y=+-5x.

  2. Find the equation of a hyperbola in standard position foci (1,5) and (1,7) and eccentricity 2.

Answered by Peny Nom.
Two algebra problems 2001-12-24
From Cass:
if the graph of a equation is 2x2 - y2 = 8 passes through point (6,k), find the positive value of k.

if 5x2 - 2 x = 1, find positive value of x


Answered by Penny Nom.
The probability that the area of a triangle is smaller that 1/8 2001-12-21
From Hoda:
Given unit square pick two points on adjacent sides of the square , what is the probability that the area of the resulting triangle is smaller that 1/8? I tried to tackle the problem by first sketching a square with the four vertices at (0,0), (1,1), (0,1) and (1,1). The answer seems to be the area under the curve y=0.25/x between 1 and 0, but how can you integrate such a function when ln0 is undefined? any help would be highly appreciated.
Answered by Andrei Volodin.
Feeding Jumbo 2001-12-21
From Lupe:
There are 2 dogs one named major and the other jumbo. major weighs 100 pounds and jumbo 150 pounds. they eat dry dog food which comes in a 40-pound bag that cost $30. Together, major and jumbo eat a whole bag a week. major eats two fifths of the bag while jumbo eats the rest.How much would it cost to feed jumbo in february, which has 28 days?
Answered by Penny Nom.
Getting to B in the shortest time 2001-12-19
From Nancy:
A motorist in a desert 5 mi. from point A, which is the nearest point on a long, straight road, wishes to get to point B on the road. If the car can travel 15 mi/hr on the desert and 39 mi/hr on the road to get to B, in the shortest possible time if......

A.) B is 5 mi. from A

B.) B is 10 mi. from A

C.) B is 1 mi. from A


Answered by Penny Nom.
Simultaneous equations 2001-12-17
From Matthew:
4x + y = 4
2x - 3y = 5

what is x and y


Answered by Penny Nom.
Making toys 2001-12-17
From Karen:
Jonathan and Morgan are two of Santa's elves. Jonathan can make a toy in eleven hours. Morgan can make the same toy in nine hours. How long it take for both Jonathan and Morgan to make the toy if they were working together?
Answered by Claude Tardif.
Composition of functions 2001-12-16
From Paula:
  1. if f(x)= 3x-1 and g(x)= 1/2x + 3 find fog(2)

  2. find the values of x for which tanx=0

Answered by Penny Nom.
A 1 m by 1 m square box in the xy-plane 2001-12-15
From Murray:
Inside a 1 m by 1 m square box in the xy-plane, there are finitely many line segments, whose lengths sum to exactly 10 m. Show that there exists a straight line in the plane which crosses at least six of these line segments. (Hint: first, show that there exists a straight line in the plane which crosses at least five of these line segments.)
Answered by Claude Tardif.
A right-angled triangle 2001-12-13
From Pamela, Jeff, Kayleigh and Jacob:
A right-angled triangle has one side that is 8 cm shorter that the hypotenuse. The other side is 1 cm shorter than the hypotenuse. Find the length of the sides.
Answered by Penny nom.
Algabra 2001-12-13
From Stephanie:
My name is Stephanie. My grade 7 class is doing some strange form of algebra which I've never done nor seen before. We are doing equations like these: 3y - 6 + 7 - 4y = 6y - 6. I don't really understand them and was wondering if you could help me do my homework and explain how to at the same time. It would be really great if you could!!!!! Thank you so much! -Stephanie
  1. -9x - 5 - 8 + x = ?

  2. 5 + 6x - 3y + x + 8y = ?

  3. -x - 3 + 5x + 6y + 8x - 9= ?

Answered by Penny Nom.
Reduce 79,537/3,623,420 2001-12-13
From Eliana:
How do you reduce 79,537/3,623,420.
Answered by Claude Tardif.
For 7 the multiplicand is 143 2001-12-12
From Cory:
For 7 the multiplicand is 143

I am asked to solve this, if you could help me it would be greatly appreciated!


Answered by Penny Nom.
A quadrilateral with 0 sets of parallel sides 2001-12-12
From Jess:
Ok well for quadrilaterals a parallelogram is a 4 sided figure with 2 sets of parallel sides and a trapezoid is a 4 sided figure with 1 set of parallel sides.

So is there a name for a quadrilateral with 0 sets of parallel side?


Answered by Chris Fisher.
Painting walls 2001-12-12
From Rizwan:
3 sixth graders painted 4 walls in 3.5hrs. At that rate, how many walls would they paint in 7 hrs.
Answered by Penny Nom.
Division symbol 2001-12-12
From Amanda:
What is the name of the symbole used in a long divison problem that seperates the two factors? It looks like a little house.
Answered by Penny Nom.
Exponential form 2001-12-11
From Crystal:
Hi, I am asking a question on behalf of a homeschooling grade 11 student. She is having trouble with changing something like (3-4)2 into a exponential form.

First of all she needs to know what the term means "exponential form" and then the process of it.


Answered by Penny Nom.
y = zx - x - c 2001-12-11
From Andrew:
Could you please help me work out the value of x where: y = zx - x - c
Answered by Penny Nom.
Normal lines 2001-12-11
From Kristie:
Why are perpendicular lines called normal lines?
Answered by Chris Fisher.
Cinderella clothes 2001-12-11
From A student:
If cinderella clothes, inc. has determined that 0.5% of all incoming phone calls involve complaints, what is the probability that in 200 incoming calls there are more than one complaint?
Answered by Andrei Volodin.
y= (3x+2)/(4x-5) 2001-12-05
From Courtney:
y= (3x+2)/(4x-5)
solve for x

Answered by Penny Nom.
Piles of coins 2001-12-05
From A student:
Sharon has less than 20 coins. When she puts them in piles of 5, she has 1 left over. When she put them in piles of 3, she also has 1 left over. How many coins does Sharon have?
Answered by Penny Nom.
Fractions in lowest terms 2001-12-05
From Thomas:
how to change decimal to a fraction in the lowest terms .7 .65 .019 .0111 .225 .1225 .625 .25 .025 .26 .875 .5 .75 .750 .33 .09 .44 .15 .12 .90 .044 .2500 .95 .500 .02 .40 .125 .1276 .00009 .45 .125 .1276 .375 .975 .054 .0005 .20 .2 .0001 .85 .80 .9 .0007 9.8 10.17 25.75 48.4 80.35 19.64 276.20 31.042 18.1034
Answered by Penny Nom.
Two equations in two unknowns 2001-12-04
From Courtney:
y = 3x + 2
y = 4x - 5

solve for x

Answered by Penny Nom.
Integers 2001-12-02
From Alison:
How do you use integers in your everday life?
Answered by Harley Weston.
Solving x^2-3x+5=0 2001-11-30
From Katy:
How do you find the solution to x2-3x+5=0?
Answered by Penny Nom.
Can a infinite set be smaller than another infinite set? 2001-11-29
From Carlos:
Can a infinite set be smaller than another infinite set? If so why?
Answered by Chris Fisher and Penny Nom.
A lighthouse and related rates 2001-11-29
From Melissa:
A lighthouse is located on a small island 3 km away from the nearest point P on a straight shoreline, and its light makes 4 revolutions per minute. How fast is the beam of light moving along the shoreline when it is 1 km from P?
Answered by Penny Nom.
Multinomial theorem 2001-11-28
From Murray:
Could you please state and explain the multinomial theorem (I already know the binomial theorem etc, to give you an idea of where i am)
Answered by Harley Weston.
Slope intercept form 2001-11-27
From A student:
what is slope intercept form?
Answered by Penny Nom.
Numerical problems 2001-11-27
From Yossi:
I am preparing a 45 minutes class for K-12 students. The problem is to show that algebraic equivalent expressions are not always numerically equivalent. In particular I would like to show one of these dangers: cancellation that occurs during the subtraction of nearly equal quantities. Do you have a good reference I could use to prepare my class. In particular to be able to show some examples and how one can avoid this type of error. I also would like to show examples with practical use. I tried to look up in the web but did not find anyhting appropriate.
Answered by Chris Fisher.
Dual spaces 2001-11-26
From Hoda:
In the book "Linear Functional Analysis" from Epstein there is a proof (page 116) of the following theorem:

The dual space of Lp is equal to Lq.

Later, it is stated as a theorem without proof that

for 1 <= p < +infinite, The dual space of lp is equal to lq.


Answered by Doug Farenick.
Ratios and proportions 2001-11-26
From Adam:
I am doing a project where I come to this website and ask a math expert about a certain topic. My topic is ratios and proportions. Can you please tell me what your job is and how you use ratio's and proportions in your job. Can you also please explain what ratios and proportions are.
Answered by Harley Weston.
A health club 2001-11-25
From Maria:
A health club with a membership of 650 people operates a running track and an indoor swimming pool. A survey of the membership indicates that 68% use the running track, 44% use the swimming pool, and 8% use neither. If a member is chosen at random, what is the prbability that the member uses:

a) Both the track and the pool?

b) Only the track?


Answered by Penny Nom.
L'Hopital's Rule 2001-11-25
From Mike:
I'm a high school senior. I'm in ap calculus and was wondering if you know the proof of L'Hopital's Rule? Our teacher said he has never been able to find it in a math book and was unable to derive it himself.
Answered by Claude Tardif.
Undetermined coefficients 2001-11-22
From Hoda:
The equation is:

y" - 2y' + y = t et + 4

We need to use The method of Undetermined coefficients. I have tried assuming that the solution is Atet+Bet+C, but all I get is C=4 and I tried (At2+Bt+C)et+D, but again I get 0=0 when I calculate the first and second derivatives, so i get no information on the constants. Any suggestions?


Answered by Harley Weston.
Matrix 2001-11-21
From Hoda:
I have a question about matrix multiplication; well, actually, matrix division. I am assuming that matrices are divided the same as they are multiplied; that is, row by column. But what happens if you have to divide by zero? How does this affect the resulting matrix?
Answered by Patrick Maidorn and Penny Nom.
A tangent line 2001-11-21
From A student:
write an equation of the line tangent to the graph of

ey + ln(xy) = 1 + e at (e,1)

Answered by Harley Weston.
Rewriting and evaluating formulas 2001-11-19
From A conserned mother:
Due to the wide range of temperatures experienced in Canada, engineers who construct roads must allow for expansion and contraction of the road surface. The following formula is used to calculate the amount of expansion E to allow for: E= kL(T-t), where k is the constant of expansion for the road surface. L is the length of the section of highway in metres. T is the temperature of the air in degrees Celsius. t is the temperature, in degress Celsius, at which the section of the highway was constructed.
Answered by Walter Whiteley.
Cardinality of sets 2001-11-19
From Tania:
  1. Show that the cardinality of P(X) (the power set of X) is equal to the cardinality of the set of all functions from X into {0,1}.

  2. Show that (the cardinality of the natural numbers set) |N| = |NxNxN|.

  3. Show that the cardinality of the set of prime numbers is the same as the cardinality of N+

Answered by Walter Whiteley.
Gallons 2001-11-19
From Shawn:
Do we use British or American gallons in Canada?
Answered by Chris Fisher.
Translated by the vector (-5,2) 2001-11-19
From Jennifer:
Give the image of (p,q) when translated by the vector (-5,2).
Answered by Penny Nom.
The "pi" of a circle in hyperbolic space 2001-11-19
From Alex:
How can you find the "pi" of a circle in hyperbolic space (or is it the same). I would like to know because our environment is hyperbolic and if the "pi" of hyperbolic space is irrational, it would follow that space is non discrete. I would greatly appreciate any help in this question.
Answered by Walter Whiteley.
Box and Whisker plots 2001-11-19
From Rod:
In our Prealgebra course, we have been studying Box and Whisker plots. Recently, we learned how to decide whether a data point is an outlier or not. The book (Math Thematics, McDougall Littell) gave a process by which we find the interquartile range, then multiply by 1.5. We add this number to the upper quartile, and any points above this are considered to be outliers. We also subtract the number from the lower quartile for the same effect.

My question: where does this 1.5 originate? Is this the standard for locating outliers, or can we choose any number (that seems reasonable, like 2 or 1.8 for example) to multiply with the Interquartile range? If it is a standard, were outliers simply defined via this process, or did statisticians use empirical evidence to suggest that 1.5 is somehow optimal for deciding whether data points are valid or not?


Answered by Penny Nom.
Renee, Paulo and Peter 2001-11-18
From Sue:
Renee is 5 years older than Paulo. she is also 7 years older than Peter. The sum of their ages is 21. How old is each person.
Answered by Penny Nom.
A box of pants and shirts 2001-11-17
From Debbie:
I need to determine the volume of a box that is 4'HX4'WX4'L. The box contains uniform pants and uniform shirts. I need to know how many shirts and pants are in the box. When folded, the uniform shirt is .5 inches thick. The pants are also .5 inches thick.
Answered by Penny Nom.
How do you get to 100 by using 6 nines? 2001-11-17
From A student:
How do you get to 100 by using 6 nines?
Answered by Claude Tardif.
A clap of thunder 2001-11-15
From Frustrated Mom:
While getting a recipe for the Thanksgiving feast. The teacher was talking on the phone with a friend who lives four miles north of her. She saw a flash of lightning through the window: fifteen seconds later, she heard a clap of thunder. Ten seconds after that she heard the thunder over the phone. Where did the lightning strike in relation to the teacher's house. (There are two possible answers. Sound travels about 1/5 mile per second. Some people say it's not good to be on the phone in a thunderstorm).
Answered by Claude Tardif and Penny Nom.
what number doubled plus 20 will give you 4/3 the original number 2001-11-15
From Hank:
what number doubled plus 20 will give you 4/3 the original number

2 x__ + 20 = 4/3 of the original number

Answered by Penny Nom.
(a+b) squared and a squared plus b squared 2001-11-14
From Kristen:

(a+b)2 = a2 + b2

Is this true for all whole numbers, and why?


Answered by Penny Nom.
Subsets of a countably infinite set 2001-11-14
From Tania:
How could I show (and explain to my son) that any countably infinite set has uncontably many infinite subsets of which any two have only a finite number of elements in common?
Answered by Claude Tardif.
3x-1/2(x-4)+5= -3 2001-11-13
From Tausa:
3x-1/2(x-4)+5= -3
Answered by Penny Nom.
Inequalities 2001-11-13
From Pat:
When writing the solution to an inequality, why is the solution written in set notation? {x/ }
Answered by Penny Nom.
Prove that the medians of triangle PQR meet in a central point G. 2001-11-11
From Jessica:
Use a scalene triancle with one point (0,b) on the y axis, another point (2c, 0) on the x-axis and the last point, (2a,0) on the x-axis. Prove that the medians of triangle PQR meet in a central point G.
Answered by Penny Nom.
A cubic 2001-11-10
From Louise:
x3 + 9x2 - 7x - 63
Answered by Penny Nom.
1 minus 5(y) equals negative 2 2001-11-10
From Judy:
1 minus 5(y) equals negative 2
Answered by Penny Nom.
A circle and triangle overlap 2001-11-09
From Tara:
A circle and triangle overlap as shown.the area of the circle is three times the area of the triangle.If the common region is removed,then the area of the rest of the circle would be 14 sq cm more than the area of the rest of the triangle.How many sq cm are in the area of the complete triangle.
Answered by Penny Nom.
A circle and a triangle 2001-11-09
From Tasha:
I have a circle that has an equalateral triangle inscribed in it. The tip of the triangle (B) is at the center of the circle with the other corners (A & C) extending to the sides of the circle. I need to know the equation to find the linear length of AC. I also need to find the cordial length of the circle from point C to A.
Answered by Penny Nom.
Third side of Acute Triangle 2001-11-09
From Don:
I gave this problem to my children, 9th and 12th grade and they are not(me too!) sure of the answer. I am trying to determine the length of the third side of an acute triangle. I know two of the lengths but I do not know the angles. Is there a proper formula to use to find the length of the third side? Thank you.
Answered by Chris Fisher and Penny Nom.
12 pairs playing bridge 2001-11-09
From Diana:
I have 12 pairs playing bridge against one another for 12 games. I need to have each pair partnered with another pair -- but only once. I'm looking for a schedule for play for all 12 games. They should only be able to play against another team only once also. (ex: 1/2 v 3/4 then 4/2 v 3/1) Thank you for this opportunity to solve my dilemma.
Answered by Claude Tardif.
Asymptotes 2001-11-09
From Frank:

given the function:

f(x) = (x2) / (x-1)

the correct answer to the limit of f(x) as x approaches infinity is:

y = x+1

all math references point to this answer and the method they all use is long division of x-1 into x2

however if one were to multiply both the numerator and denominator by 1/x and then take the limit, one gets:

y=x

how can the descrepency between the two answers be explained?


Answered by Chris Fisher and Penny Nom.
A 3 dimensional 5 pointed star 2001-11-08
From Kent:
I am looking for a formula that will give me a layout for a 3 dimensional 5 pointed star. I want to form it out of sheet metal, using 5 polygons and soldering them at the apex. Can you please help me with this? I would like to be able to give the formula the height of the star from the bottom two points to the top point and also how deep the star is. Thank you very much!
Answered by Judi McDonald.
Manufactures golf balls 2001-11-06
From Kevin:
A company manufactures golf balls. Golf balls are each numbered consecutively 0 to 4, and then the numbering starts over with zero again. Every 12th golf ball is yellow. Every 8th golf ball is tested for defects. The company produces 120,000 golf balls in an eight-hour shift.

A. How many yellow golf balls are tested for defects during the day? Explain your reasoning.

B. How many of these yellow balls that are tested are imprinted with the numeral 2? Explain your reasoning.


Answered by Penny Nom.
Applied geometry 2001-11-02
From Jenny:
Where can I find some hands-on activities for my Applied Geometry classes? I want to do more activities with them that allow us to get out of the classroom. However, I want to use activities that use only inexpensive equipment because I usually buy the equipment myself.
Answered by Walter Whiteley.
A lighthouse problem 2001-11-02
From A student:
A lighthouse at apoint P is 3 miles offshore from the nearest point O of a straight beach. A store is located 5 miles down the beach from O. The lighthouse keeper can row at 4 mph and walk at 3.25 mph.

a)How far doen the beach from O should the lighthouse keeper land in order to minimize the time from the lighthouse to the store?

b)What is the minimum rowing speed the makes it faster to row all the way?

Answered by Harley Weston.
8w-15 = 3w-10 2001-11-01
From A student:
8w-15=3w-10, w =?????
Answered by Paul Betts.
m - 8 2001-10-30
From Danita:
Can you please e-mail me the answer to m -8 for m = 20 and how you got the answer
Answered by Penny Nom.
A sample size problem 2001-10-28
From Charles:
The U.S Transportation Dept. will randomly sample traffic reports to estimate the proportion of accidents involving people over the age of 70. The Dept. has no advance estimate of this proportion. how many reports should the dept select to be atleast 97% confident that the estimate is within .01 of the true proportion?
Answered by Harley Weston.
Three problems 2001-10-28
From Brenda:
  1. Joseph is planting bushes around the perimeter of his lawn. If the bushes must be planted 4 feet apart and Joseph's lawn is 64 feet wide and 124 feet long, how many bushes will Joseph need to purchase?

  2. The cost of a long distance phone call is $1.50 for the first two minutes and $0.60 for each additional minute. How much will Maria pay for a 24 minute phone call?

  3. Find the next three numbers in the pattern. 1,3,7,15,31,___,____,___.

Answered by Penny Nom.
Is the tree a pine tree? 2001-10-27
From Kate:
Suppose Stock Brokers tell the truth 2/5 of the time, and 1/3 of the trees in a given forest are Pine. If 4 Stock Brokers say that a certain tree in that forest is Pine, what is the probability that the tree is indeed a Pine tree?
Answered by Penny Nom.
Negative times negative is positive 2001-10-26
From Mary:
I have a question about adding and multiplying positive and negative numbers. When we add two negative numbers the answer is negative BUT when we multiply two negative numbers the answer is positive. I don't understand. Why?
Answered by Penny Nom.
Why do you flip the inequality sign? 2001-10-26
From Sean:
I have a question about an inequality problem. Here it is: Solve and graph 5 - 3x => 17. (=> is greater than or equal). Please tell me why you flip the inequality sign when divinding by a negative number.
Answered by Penny Nom.
Concavity of f(g) 2001-10-25
From Troy:
Suppose f & g are both concave upward on (-infinity,infinity). Under what condition on f will the composite function h(x)= f(g(x)) be concave upward?
Answered by Walter Whiteley.
An algebraic fraction 2001-10-25
From brandi:
4x - 5
_____ =4
3-7x

Answered by Penny Nom.
Metric units 2001-10-25
From Kelly:
i am helping my kids out with school work and don't remember meters,centimeters decimeters and all those. We have to change meters into cm.....and so on..........is there a place on the web to look for some kind of help in these questions
Answered by Penny Nom.
Where is the fourth point? 2001-10-24
From Mike:
Four points are placed at random on a piece of paper. Connect the three points of the triangle of the largest area. What is the possibility that the fourth point is in the triangle?
Answered by Penny Nom.
A right angled triangle 2001-10-23
From Carissa:
How do you : Triangle ABC is right-angled at A. Calculate the length of the third side if b = 3 and c = 6, express the answer as a surd in its simplest form.
Answered by Harley Weston.
The distributive law 2001-10-23
From Carissa:
How do you simplify the following using the distributive law :

5 squareroot 2+3 squareroot 2?

Answered by Penny Nom.
Squaring the circle 2001-10-23
From Margaret:
I have the following question that I cannot answer on my own (I don't know where to begin): Is it possible construct a square whose area will equal the area of a given circle? Please Explain why or why not.
Answered by Penny Nom.
The average of two polygons 2001-10-23
From Irene:
How can I prove that the average of two polygons will give me another one?
Answered by Walter Whiteley.
Multiplying vectors 2001-10-22
From Murray:
Could you please explain why a vector times a vector is a scalar and how to derive the formula vector a * vector b = ab cos(a,b)
Answered by Harley Weston.
Lines in the plane 2001-10-21
From Greg:

*If the line with equation 3x-4ky= -16 has y-intercept -2, find k.

*Write the equation of the line passing through (2,5) and perpendicular to the line passing through (2,-4) and (6,1)


Answered by Penny Nom.
Water in a conical tank 2001-10-20
From Sarah:
The problem: Water flows into a conical funnel at a continuous rate of one gallon per minute (One gallon = 231 Cu.In.). The height of the funnel is 5" and the diameter is 8".

The 1st formula: I need to develop a formula that will give the volume, in cubic inches, of the water in the funnel at any time t (in seconds). V = f(t).

The 2nd formula: I need to develop a formula that will give the height of the water in the funnel at any time t (in seconds). h = f(t).


Answered by Penny Nom.
A function or not a function? 2001-10-18
From Christian:
Which are the following are not functions?
  1. y = x2

  2. y = x3

  3. x = y2

  4. y = 3

Answered by Leeanne Boehm.
A line in 3 dimensions 2001-10-17
From Murray:
I'm working on a complicated proof and i need the equation for a line in 3 dimensions.
Answered by Claude Tardif.
Dividing a circle 2001-10-17
From Ahmeen:
I am having a hard time figuring out how a circle can be divided into 11 equal parts with only 4 cut allowed? My teacher gave this to us and I still can't cut my pie into eleven equal parts with only four cuts.
Answered by Walter Whiteley.
Dimensions of a frame 2001-10-16
From Rachel:
A rectangular picture frame has a perimiter of 44.2 cm.The width of the frame is seven tenths of its length what are the dimensions of the frame?
Answered by Penny Nom.
Proof by induction 2001-10-16
From John:
Can you help me with any of these?
  1. For any natural number n > 1, prove that

    (4n) / (n + 1) < [(2n)!] / [(n!)2].

  2. For any natural number n > 1, prove that

    1/sqrt(1) + 1/sqrt(2) + 1/sqrt(3) + ... + 1/sqrt(n) > sqrt(n).

  3. For any natural number n and any x > 0, prove that

    xn + xn - 2 + xn - 4 + ... + x-n >= n + 1.

Answered by Penny Nom.
Some algebra 2001-10-15
From James:
I cannot figure these out I was wondering if you could help me? I have no one to answer my questions.
  1. (7x2 – 3yz)2 – (7x2 + 3yz)2

  2. Use Pascal’s triangle to expand (2x – y)4

  3. 8x3 y - x3 y4

  4. (m + 3n)2 – 144

  5. 12x4 y – 16x3 y2 – 60x2 y3

  6. p3 q2 – 9p3 + 27q2 – 243

Answered by Peny Nom.
Rules of exponents 2001-10-14
From Carissa:
how do you work this out? Investigate the relationship between a,b,c and d if 2a*2b=4c/4d?
Answered by Leeanne Boehm.
Absolute Value 2001-10-14
From Susan:
I am a homebound teacher, and my nineth grade algebra I student is having trouble understanding absolute value. Please send me an idea about how to best help her. She may be out of the classroom the entire year.
Answered by Chris Fisher.
Odd plus even is odd 2001-10-14
From James:
Why is the sum of an odd number and an even number always odd?
Answered by Peny Nom.
Area of a quilt 2001-10-14
From Jack:
  1. how would you find the area of the quilt? which is a square

  2. how would you fined the area of each square?

Answered by Penny Nom.
Theme day 2001-10-14
From A teacher:
I woulld like a math theme for a theme day in a high school, grades 9-12. Our last theme was fractal fest.
Answered by Penny Nom and Claude Tardif.
Maximize the area 2001-10-13
From Mike:

I have no clue how to do this problem. Here is what the professor gave to us:

A=LW

C=E(2L+2W) + I(PL)

Where P = # of partitions

E= cost of exterior of fence

I = cost of interior of fence

C = total cost of fence

.
.
.

Answered by Harley Weston.
Mathematical & conventional meaning of a word 2001-10-12
From A student:
What is the mathematical & conventional meaning of a word? Like the word Rational or Median.
Answered by Penny Nom.
60 seconds in a minute 2001-10-11
From Andy:
I am a fourth grade teacher. Yesterday my students asked "Why are there 60 seconds in a minute?" Which also led to 60 minutes in an hour? I have had trouble determining why the number 60? Any help would be appreciated.
Answered by Penny Nom.
4 sinx cosy = 1 2001-10-10
From A student:
How would i differentiate the following example in terms of t (x and y are functions of t)

4 sinx cosy = 1


Answered by Claude Tardif.
Acres 2001-10-10
From Allison:
how many feet are there in an acre?
Answered by Chris Fisher.
eix = cosx + isinx 2001-10-10
From Peter:
Given: eix = cosx + isinx
  1. substitute -x for x to find e-ix, simplifying your answer

  2. use the given and part a to find an identity for cosx making no reference to trig functions

  3. find an identity for sinx
  4. .
  5. .

Answered by Penny Nom.
Ratio and proportion 2001-10-10
From Steve:
Where would you use a proportion and/or a ratio in a real life job or problem.
Answered by Leeanne Boehm and Walter Whiteley.
Pythagoras & magic squares 2001-10-09
From John:
My grandson became intrigued when he recently 'did' Pythagoras at elementary school. He was particularly interested in the 3-4-5 triangle, and the fact that his teacher told him there was also a 5-12-13 triangle, i.e. both right-angled triangles with whole numbers for all three sides. He noticed that the shortest sides in the two triangles were consecutive odd numbers, 3 & 5, and he asked me if other right angled triangles existed, perhaps 'built' on 7, 9, 11 and so on.

I didn't know where to start on this, but, after trying all sorts of ideas, we discovered that the centre number in a 3-order 'magic square' was 5, i.e. (1+9)/2, and that 4 was 'one less'. Since the centre number in a 5-order 'magic square' was 13 and that 12 was 'one less' he reckoned that he should test whether a 7-order square would also generate a right-angled triangle for him. He found that 7-24-25, arrived at by the above process, also worked! He tried a few more at random, and they all worked. He then asked me two questions I can't begin to answer ...

  1. Is there a right-angled triangle whose sides are whole numbers for every triangle whose shortest side is a whole odd number? and

  2. Is each triangle unique (or, as he put it, can you only have one whole-number-sided right-angled triangle for each triangle whose shortest side is an odd number)?

Answered by Chris Fisher.
Lucas' theorem 2001-10-09
From Tania:
How could I demonstrate: nCp is congruent to floor(n/p) (modulo p)? where rCk is a binomial coefficient, rCk = r(r-1)...(r-k+1)/k(k-1)...1, and p is a prime number
Answered by Richard McIntosh.
Binomial probabilities 2001-10-08
From Amna:
I had a few binomial probability questions which I can not use from the tables as instructed:
  1. If 60 % of television viewers are watching a certain program, what is the probability tha tmore than half of those selected in a random sample of five will be watching the specified program?

  2. If it is known according to Mendel's Law, that we can expect in teh long run to have 3 white, 1 brown rabbits in every 4 rabbits of a certain type, what is the probability that 2 in a litter of 3 will be white?

  3. On the average, 2% of the items sold in a department store are returned for refunds. what is the probability that of its next five items sold, at most two will be returned for refunds?

Answered by Leeanne Boehm.
Locusts 2001-10-08
From A parent:
JOHN CONJECTURED THAT BOTH 13-YR.&17-YR. LOCUSTS CAME OUT THE SAME YEAR. ASSUME IT IS CORRECT. HOW MANY YEARS PASS BETWEEN THE YRS. WHEN BOTH 13 YR. & 17YR. LOCUSTS ARE OUT AT THE SAME TIME? EXPLAIN. NEXT, SUPPOSE THERE WERE 12YR.,14 YR.,& 16YR. LO- CUSTS,& THEY ALL COME OUT THIS YR. HOW MANY YRS. WILL IT BE BEFORE THEY ALL COME OUT TOGETHER AGAIN?
Answered by Leeanne Boehm.
Simplify the expression 2001-10-08
From Natalia:
simplify expression

4(x-7)+(-5x)=-10


Answered by Leeanne Boehm.
Percentage in our daily lives 2001-10-08
From Natasha:
What are the uses of percentage in our daily lives?
Answered by Leeanne Boehm.
The amount of gravel needed to fill a hole 2001-10-06
From Rhett:
My name is Rhett and my problem is...I am a contractor and I am having problems determining the amount of gravel(in tonnage) needed to fill this hole,the measurements are 60' in length,22' in width,and 14' in height....the problem is that it is not a usual box shaped hole or in this case a rectangle shaped hole,but more of a triangular shaped hole.I refer to the triangular shape as the interior shape of the crater not the exterior,which is shaped like the above dimensions.If you could help it would be greatly appreciated.P.s.This was a retaining wall project,built by myself, in front of a sloped end of a yard,so if you could imagine a 14" high hill with a landslide looking face,then build in your mind a three sided rectangle in front of it using the hill side that slopes as the fourth side of the rectangle,then you may be able to imagine what I mean as a triangular shaped interior of the hole.
Answered by Penny Nom.
Subtracting 5 from 25 2001-10-05
From Melissa:
How many times can you subtract the number 5 from 25?
Answered by Leeanne Boehm.
A circle and a triangle 2001-10-04
From Christina:
The points (3,4), (9.-2), and (-3,-2) define a circle and a triangle.
  1. find the areas of the circle and the triangle. Find the difference between their areas.

  2. Find the length of a side of a square with the same area as the triangle.

  3. Find the length of a side of a Square with the same are as the circle.

Answered by Penny Nom.
Who owns what pet? 2001-10-03
From A parent:
Don, Frank, Jenny, and Ken each come from one state, either Alaska, Maine, Montana, or Oklahoma. They each speak one primary language, either English, french, Russian, or Spanish. And they each have one of four pets, a chinchilla, a dog, a hamster, or a turtle.

Figure out which language and what pet each of the four people have, using the following clues.
.
.
.


Answered by Judi McDonald.
A polygon with five sides 2001-10-03
From A student:
I was asked a question in 5th grade today that goes: i am a polygon with five sides and 2 right angles what do i look like and what is my name
Answered by Penny Nom.
The height of the lamppost 2001-10-02
From Werner:
I am working on question 51,section 3.7 ,page 191 of Stewart's Single Variable Calculus. The question involves a lamp post which is casting a shadow around the eliipse whose formula is x2 + 4*y2 = 5. I have found the derivative of the elllipse both explicitly: x/4(((5-x2)/4)0.5) and implicitly : y' = - x/(4*y).
Answered by Harley Weston.
(x^2-5x-6)/(x-6) 2001-10-02
From Bill:
given f(x) = (x2-5x-6)/(x-6) find f'(6).
Answered by Harley Weston.
Proof by induction 2001-09-30
From Kyle:
I'm trying to learn induction and I need to see how this done please help with this problem...

20 + 21 + 22 +... + 2n = 2n+1 -1 is true whenever n is a positive integer.


Answered by Penny Nom.
Solitaire 2001-09-27
From Babahadj:
Il s'agit du probleme du solitaire avec des cases numerotees 1, 2 et 3. Un pion se trouve initialement sur chacune des cases, sauf la centrale qui est vide. Au depart, il ya 15 pions sur les cases avec le numero 1 , 15 pions sur des cases avec le numero 2, et 14 pions sur des cases avec le numero 3. Le seul mouvement pour un pion P consiste a manger un pion P' en sautant par dessus ce dernier , a condition que le pion P' soit situe sur une case voisine (horizontalement / verticalement) et que la case dans laquelle aboutit le pion P soit vide. En partant de cet etat initial , est - il possible d'arriver a un etat final avec un seul pion se trouvant sur une case portant le numero 1. Pourquoi? Merci de me repondre.
      1 2 3       
      2 3 1       
      3 1 2 
1 2 3 1 2 3 1 2 3 
2 3 1 2 3 1 2 3 1 
3 1 2 3 1 2 3 1 2       
      1 2 3       
      2 3 1       
      3 1 2 

Answered by Claude Tardif.
A bag with 3 red marbles and 2001-09-27
From Mike:
In a bag, there are 3 red marbles and "B" blue marbles. Two marbles are randomly selected from the bag without replacement. The probability that the two marbles are the same color is 0.5. Calculate the sum of all possible values of B.
Answered by Claude Tardif.
The triangle inequality 2001-09-27
From A student:
    
           __   __   __ 
Prove that AX + XB = AB for any X on AB.  

Answered by Chris fisher.
Joan and Jan work together 2001-09-26
From Karyna:
Joan and Jan work together and they set up a bandstand in just 24 minutes, but if Joan does it by herself she takes 1 hour. How long would it take Jan to set up the bandstand by himself?
Answered by Claude Tardif.
Bicycles and tricycles 2001-09-26
From Sally:
A bicycle shop has 5 bicycles and tricycles to repair. They have 12 wheels. What strategy would you use to determine how many bicycles and tricycles need to be repaired?
Answered by Claude Tardif, Diane Hanson and Penny Nom.
Scoring a college placement test 2001-09-26
From Carmen:
A college placement test consist of 75 multiple-choice questions, each having 5 possible answers. The exam is scored by subtracting from the number of correct answers one-quarter of a point for each question answered incorrectly. If a person answers every question on the exam and scored 45, how many questions were answered wrong?
Answered by Penny Nom.
Investing in a yacht 2001-09-26
From Junior:
Marvin and his four friends wish to invest in a yacht by sharing equally in the cost. As the date of sale approaches, Marvin realizes that if three more people join the partnership, each partner's share would decrease by $18,000. How much does the yacht cost?
Answered by Penny Nom.
Julia goes to work 2001-09-25
From Karyna:
On one October morning, Julia rides her bike (at 12 miles per hour) from her home to her friend Ida's house; then the two of them walk (at 6 miles per hour) to work. If it takes an hour for Julia to go the 10 miles from her home to work, how far does she walk?
Answered by Penny Nom.
GCD and LCM 2001-09-24
From David:
Find the GCD adn LCM of the following numbers. Assume p 2 to the power p, 3 to power q, 5 to power r

and

2 to power q, 3 to power r, 7 to power p


Answered by Penny Nom.
Four sequences 2001-09-24
From Nicole:
I have a few questions I couldn't figure it out. So any help will be great to find the next three numbers or letters. Here's the patters...
  1. DEGHNORY, EHILMU, HIILMTU, BEILLMRUY,...

  2. 50, 33, 25, 20, 16, 14, 12,...

  3. 1777, 1795, 1818, 1819, 1820, 1822, 1836, 1837..

  4. A,H,I,M,O,T,...

Answered by Chris Fisher and Penny Nom.
What are the two numbers? 2001-09-24
From A student:
What are the two numbers.
  1. the sum of two numbers is 15. 6 times the smaller number is 4 times the larger number .find the two numbers.
  2. the sum of the two numbers is 28. 2 times the larger number is 5 times the smaller number minus 7. find the two numbers.

Answered by Claude Tardif.
David's wages 2001-09-24
From Kimberlee:
My math question is David earns $6.00 per hour plus $9.00 for every extra hour he works over 40 hours per week. If David earned $285.00 last week, how many extra hours did he work?
Answered by Penny Nom.
An acute scalene triangle 2001-09-24
From Kim:
Hi- I'm on a mission--I'm suppose to have a group of children write up a hypotheses about the altitudes and angle biscetors of the follwing 3 types of triangle--equilateral, isosceles and acute scalene triangle.

Is there such a thing as a acute scalene triangle and why or why not.


Answered by Chris Fisher.
Solving trig equations 2001-09-22
From Asad:
Can you please explain to me how to solve trig Equations,e.g sin(x)=x4+12/2+cos(x)=x6+9/3= (if this can be solved)
Answered by Claude Tardif.
Standard deviation 2001-09-22
From Doug:
I was wondering if anybody knows who the founder of standard deviation was?
Answered by Penny Nom.
An egg shaped island 2001-09-22
From Karen:
I am a civil engineering designer trying to design an egg shaped island. I want a 30' radius at the top and a 40' radius at the bottom and the longest length of the egg to be 125'. Is there standard geometry for an egg shape? I am not held to exact radii or the length given.
Answered by Chris Fisher.
Milliters and grams 2001-09-20
From Betty-jo:
500 ml is what in grams
Answered by Pnny Nom.
The square root of 20 2001-09-18
From Dianna:
How do you simplify a square root? My daughter tells me that the square root of 20 simplified is 5root4
Answered by Penny Nom.
Solve for h 2001-09-17
From Kim:
s=2lw+2wh+2hl solve for h
Answered by Penny Nom.
A linear inequality 2001-09-17
From Eric:
How can I solve |x + y| + |x - y|< =6?
Answered by Penny Nom.
Counterclockwise 2001-09-14
From Rolanda:
When Descartes invented the coordinate system he decided to number them counter-clockwise. Why?
Answered by Chris Fisher.
300 pounds 2001-09-12
From Cassie:
HOW MANY KILOS IS IN 300 POUNDS?
Answered by Penny Nom.
A polygon 2001-09-11
From Sueling:
what is the smallest polygon. what is a polygon.
Answered by Penny Nom.
27 2001-09-10
From Kacy:
In class we were given this problem. We have the numbers 30, 29, 19,11,26. these numbers = 27 in some way but how? the numbers can be applied any way using + - / X ( ) but can only be used 1 time each. can you please help me.
Answered by Claude Tardif.
Three fractions 2001-09-10
From Kathleen:
a/bc + d/ef + g/hi = 1

Each of three fractions has a one-digit numerator and a two digit denominator. The three fractions together add up to one. Place the nine digits 1-9 into the fractions to make the equation correct.


Answered by Claude Tardif.
Rolling two dice 2001-09-10
From Owen:
This probability question has been bugging me for a while. Two ordinary dice are rolled. If it is known that one shows a 5, what is the probability that they total 8?

I have two different but (seemingly) correct solutions.


Answered by Andrei Volodin.
A 91-gon 2001-09-10
From Ashley:
Hi! My name is Ashley and I am an 8th grader and I couldn't find the name of a polygon with 91 sides. i was a question from my math teacher.
Answered by Penny Nom.
Double bar graphs 2001-09-09
From Leslie:
My daughter has a homework problem in which she is graphing data from two frequency tables onto a double bar graph. The graph shows how many students worked in a program in two different years, grouped by age. In the first age group, one of the years has no students working, while there were students in that age group in the other year.
Answered by Penny Nom.
Similar triangles 2001-09-08
From Dave:
I am standing on the bank of a river ( whose banks are parallel here) directly opposite a boathouse, B, on the opposite bank. I walk along the bank of the river past a signpost, S, until I reach a point C distant 60 metres from where I started walking. I then walk away from the bank, at right angles to the bank, until I reach a shady tree at D. Attached to teh tree is a sign stating that this spot is 45 metres from the signpost. C is 36 metres beyond S and B and S are in line from D.

(a) How far did I walk away from the bank of the river??

(b) Calculate the width of the river?


Answered by Penny Nom.
Multiplying binomials 2001-09-08
From A student:
(x+1)(x+2)=?
(2x-1)(x-4)=0
please tell me this question!!

Answered by Leeanne Boehm.
The best of 7 2001-09-08
From Michelle:
Professional basketball, hockey, and baseball championships are decided on the best 4 out of 7 games. The first team to win four games wins the championship. In how many possible ways could the team win the championship series after winning the first game?

 Example:  
Game   1    2    3    4    5     6     7         
Result W    L    L    W    W     W 

Answered by Leeanne Boehm.
Divisibility rules 2001-09-07
From A student:
Why is it that when you add the digits of a number you can tell what the multiples of that number are. Example: 12131313111,

1+2+1+2+1+3+1+1+1=18,

therefore 12131313111 is divisble by 2, 9, 18, & 3 because those numbers are divisble by 18.

Answered by Penny Nom.
Don't worry too much about your difficulties in math... 2001-09-07
From Matt:
Do you know who said,"Don't worry too much about your difficulties in math, I assure you that mine are still greater"? Thanks for your help. this for extra credit and I can't find it ANYWHERE!
Answered by Penny Nom.
0.999999=1? 2001-09-06
From Catherine:
Hi! My teacher told us that 0.9 repeating equals one. We discussed how this is true. But, I was wondering if there is a proof that this is true. If so what is this called? I was trying to find information, but, it's hard when you don't know the name.
Answered by Walter Whiteley.
Commas in numbers 2001-09-06
From Mike:
When did the Ontario Education system drop the use of commas in large numbers and replace with a space? Why?
Answered by Walter Whiteley.
Earthquake warning 2001-09-03
From Dillon:
In the city of Skangai there are 6 million people.Each person has to tell 2 more people that theres going to be an earthquake. After they do they leave the city. There will be no duplications.It takes 3 minutes to tell each person. How long will it take for the whole city to know about the earthquake?
Answered by Penny Nom.
72 cards 2001-09-03
From A student:
What would be the probability of dealing a deck of 72 cards out in the exact orderly sequence that they were in when they were packaged after they have been throughly shuffled?
Answered by Andrei Volodin.
Two snakes 2001-08-31
From A student:
Two snakes(pythons,actually)of same size and shape start swallowing each other by the tail with the same speed. What will happen ultimately?
Answered by Claude Tardif.
Water jars 2001-08-27
From Pe3te:
Stacie and Kacie each have a jar. Stacies jar holds 5 quarts of water and Kacies jar holds 3 quarts. The girls dad ask them to go get him some water from the pump and to get him 1 gallon of water (NO MORE NO LESS) The jars were not marked in any way. In ten minutes the girls where back with 1 gallon. How did they get exactly 1 gallon ?

1) there are two ways to do it I only no one. 1st answer is.....


Answered by Penny Nom.
Bisecting angles 2001-08-27
From Monica:
Ray QS is the bisector for angle PQR. Find the measure of angle PQS and PQR if the measure of angle SQR is 52 degrees.
Answered by Penny Nom.
Fourier transform 2001-08-07
From Adbul:
  1. Sir, we have the Dirichlet's condition for the Fourier transform : " The function should be integral over the real line " But why we are we neglecting this for example when we take the Fourier transform of an impulse train?

  2. Suppose we want to travel from one corner of a square of side 'a' to the diagonally opposite corner. We can travel along the sides which gives a pah length of '2a'. We can also do it in steps as shown below:

      _ | |_PATH |   |_ |_____| 


    Suppose The step size =DELTA x Then the path length will be again '2a'. Now in the limit DELTA x -->0 again we get '2a' But when we take the limit we get the straight line diagonal whose length is 'SQRT(2)X a' Where did I go wrong?

Answered by Chris Fisher.
How far does the fly fly? 2001-08-07
From Harold:
 
6 MPH                            4 MPH 
Rachel  ----------------------     Eli                        
                  10 Miles apart  

The fly is on Rachels handlebars. The fly is scared so it flys back and forth at 20 MP H. How far has the fly flown when Rachel and Eli meet?

f

Answered by Penny Nom.
Standard angles 2001-08-05
From Nagaraj:
Why 0o , 30o , 45o , 60o ,and 90o are taken as standard angles in Trigonometry? Why can't we take some other angles as standard angles?
Answered by Chris Fisher.
Conformable matrices 2001-08-05
From Wayne:
Can someone explain the concept of conformable matrices in a way that is easy to understand ? One definition says to multiply matrices rows and columns must conform, ie, 5 x 3 matrix times a 3 x 5 matrix. In the next example, however a 8 x 1 matrix and a 8 x 3 matrix are said to be conformable!
Answered by Steve Kirkland.
Linear regression 2001-08-01
From A student and a mother:
My mom is taking a correspondence course in Simple Linear Regression and Correlation Analysis and we've been arguing about the relationship between the correlation -rxy and standard estimate of error-Sy.x. I took statistics last year in high school and I remember something about the Sy.x being proportionate to the r. Are they inversely related, directly related, not related, or can only range from 0 to 1.00? Her book doesn't say very much and I believe they are inversely related. She says they're directly related.
Answered by Chris Fisher and Penny Nom.
Intersection of perpendicular cylindrical surfaces 2001-07-31
From Charlie:
Please consider two right circular cylinders, perpendicular one to the other, and of unlike radii in a 3 dimensional Cartesian space with mutually perpendicular x,y,z axes. If one cylinder is centered on the y axis with radius ra, and the other on the z axis with radius rb, then the expression for the first surface would be x2 + z2 = ra2, y = any number. Likewise, the second cylinder's surface would be x2 + y2 = rb2, z = any number.

It is my intent to define the curve at the intersection of these two cylindrical surfaces. From sketching the conditions it appears that this intersection resembles an ellipse folded about its minor axis.


Answered by Chris Fisher.
The radius of a planet 2001-07-30
From Jessica:
A satellite is orbiting the earth at an altitude of 100 miles. If the angle of depression from the satellite to the horizon is 50 degrees, what is the radius (to the nearest mile) of the planet?
Answered by Harley Weston.
1996^1999 and 1999^1996 2001-07-29
From Rajesh:
what is greater 19991997 or 19971999?
Answered by Chris Fisher.
The best linear model 2001-07-29
From Rebeca:
I am a student (senior) summer hire working for Dept of the Army Resource Management. I've been assisting the Analyst in developing a model to predict training costs. The technique we used was simple linear regression. Now I've been asked for my opinion in choosing the best model I feel is better. I've chosen #1. Garrison Training but I want a second opinion before I give my choice. I've taken basic statistics last year in high school and will probably be taking more this year. Could you look over the information and give me a reason my choice would not be best? The dep variable-y, is Training costs in dolars per day.

ModelInd Variable- Xabrsy.x
1.Garrison Training126.310.9820.9055117.2
2.Local Training212.161.370.7937142.7
3.Man Rig Area116.382.470.6481175.6
4.Major Training346.124.670.7280167.2

Answered by Andrei Volodin.
e^pi > pi^e 2001-07-27
From Dusty:
What book(s) contain a proof that ePi > Pie? I think it might be in Problems in Analysis published by Springer-Verlag but I have not been able to check.
Answered by Chris Fisher.
Radian measure 2001-07-26
From Amy:
i have to find out what is meant by the radian measure of an angle and compare it to the measure of an angle in degrees.
Answered by Harley Weston.
The Mean Value Theorem 2001-07-23
From Corrie:
I need to find if the mean value theorem exists. and if so, find all values c guaranteed by the theorem.

f(x) = |x2-25| on the interval [-10,0]


Answered by Harley Weston.
What did the girls score on average? 2001-07-20
From Rajesh:
The students of two schools were sitting the same test. The students attending the first school scored 74 on average; within this the average scores of the boys and the girls were 71 and 76, respectively. The corresponding scores in the other school were 84, 81 and 90, respectively. If the average score of all boys was 79, what did the girls score on average?
Answered by Penny Nom.
Matrix reconstruction 2001-07-19
From Guy:
Is there a way to get the sums of rows, columns and diagonals of an n x n matrix to reconstruct the original matrix?
Answered by Walter Whiteley and Patrick Maidorn.
Hours worked 2001-07-18
From Carlos:
I have a simple question:whats the formula to transform time into decimals? more exactly im working on a spreadsheet and i want to be able to type in for example hours worked: 11- 11:30 pm and a formula would tell me 12.5 hours how do i do it?
Answered by Patrick Maidorn.
Rhombus 2001-07-16
From William:
Calculate the internal angles of a rhombus given measurments of all four sides only.
Answered by Walter Whiteley.
6,000,000,000,000,000,000 2001-07-12
From James:
How would I express 6,000,000,000,000,000,000 in words?
Answered by Leeanne Boehm.
The speed of the boat 2001-07-12
From Sharon:
A motor boat is travelling in a southeasterly direction in water that is flowing from the south at 2km per hour. Show that the speed of the boat is (6 times the square root of 2) km per hour, given that it can travel at 10km per hour in still water.
Answered by Penny Nom.
Investing $5,000 2001-07-09
From A student:
A principal amount of $5,000 was invested in a savings account for 5 years.The interest earned was $500.Use the simple interest formula to find the annual rate of interest.
Answered by Penny Nom.
Some irrational numbers 2001-07-03
From Kellie:
  1. Is it possible to write pi as a fraction?

  2. Is it possible to write the square root of 2 as a fraction.. Explain why?

Answered by Harley Weston.
Arclength of an ellipse 2001-07-03
From A hobbyist:
What is the equation (with the length of the arc as a variable) for one quadrant of the ellipse,...
Answered by Claude tardif.
A confidence interval 2001-06-28
From Murray:
An investigator wants to find out of there are any difference in "skills" between full and part time students. Records show the following:
Student            Mean Score         Std Dev        Number
----------      -----------------     ----------     -----------
Full time              83                   12          45
Part time               70                   15          55
Compute a 95% confidence interval for the difference in mean scores.

Answered by Andrei Volodin.
Three chords 2001-06-28
From Paul:
AE is a diameter of a circle and AC, CD and DE are chords of lengths 1, 2 and 3 respectively. (See the diagram.) Find the ridius of the circle.
Answered by Harley Weston.
Three tangents to a circle 2001-06-27
From Stephanie:
The three lines PS, PT, and RQ are tangents to the circle. The points S, X, and T are the three points of tangency. Prove that the perimeter of triangle PQR is equal to 2PT.
Answered by Chris Fisher.
A three legged stool 2001-06-27
From Teri:
I wanted to know why a three legged stool is always steady, and why a four legged stool is not. I am wanting to know the mathematical reasoning behind this.
Answered by Walter Whiteley.
A phone bill 2001-06-18
From Janet:
What is the formuala to calculate cost per minute?

Here is the data below

# of calls - 238
# of minutes - 443
cost - $70.06


Answered by Penny Nom.
Two log problems 2001-06-16
From A student:
Hi..this is one of 9th grade student in Fort Worth TX. well..I am doing EPGY stuffs in my school right now..and.. me and my teacher had problems to solve some advanced logarithmic thing. so I searched some sites to solve these two questions
  1. (log base 7 * 10) * (log base 10 * X ) = 2

  2. log base 10 * (3x-4) + log base 12 * X
    ---------------------------------------------------
    (log base 12 * 2) + (log base 12 * 5)

    = log base 10 *4

Answered by Harley Weston.
A calculation with 6 numbers 2001-06-16
From Edwin:
I'm asked to come with, and program (in Ansi -C) an algorithm that calculates all the possible results of a calculation with 6 numbers and one result. For example: I want all calculations with the numbers 3, 3, 8, 8, 2, 9, and with a result of 786. all numbers may be used once, arithmetical operations allowed are + - / *, fractions are not allowed. The problem here is what is a fast method to do this (i.e. what's algorithm that can to this).
Answered by Claude Tardif.
Euclid and Pythagoras 2001-06-14
From Scott:
Question 1. In about 300 BC Euclid recorded a proof of Pythagoras rule. Disscuss Euclid's contribution to developing the theroem.

Question 2.

Why was it named after Pyhagoras if he did not orginally discover it?


Answered by Chris Fisher.
Area between curves 2001-06-13
From Phil:

question 1

find the area bound by the curves y = x2 + 2x + 3 and y = 2x + 4

question 2

Find the volume generated by rotating the curve x2 + y2 = 9 about the x-axis

Answered by Harley Weston.
An inequality involving triangles 2001-06-12
From Sandra:
The triangle inequality guarantees that the sum of the lengths of two sides of a triangle is greater than the length of the third. As a consequence, if x and y are legs of a right triangle, with x less than or equal to y, and z the hypotenuse, then x + y is greater than z, so x is greater than z - y. Under what circumstances will x is greater than 2(z - y) be true?
Answered by Chris Fisher and Penny Nom.
Monthly payments 2001-06-12
From Anthony:
This question is base on my interest. I would like to know the formula for calculating this example: If you borrow $10,000 from a bank with an APR of 11.7% to be paid off in 5 years, what is your monthly payment?
Answered by Penny Nom.
5+5+5=550 2001-06-11
From Tom:
I am in algebra and my teacher gave us an equation that was not true. she told us that we could only use one line segment(it can't bend turn has to be straight) to make the equation true. here is the equation: 5+5+5=550. i have not figured it out but have tried many things and believe it is not mathmatical but cross a # or sign out.also i forgot you can't put a slash mark through the equals sign.
Answered by Penny Nom.
Geometry problems involving triangles 2001-06-07
From Sandi:
Find the radius of the largest circle contained in a right triangle whose legs are 8 and 15 and hypotenuse is 17. If the right triangle has legs a and b and hypotenuse c, find an expression for the radius of the circle.
Answered by Penny Nom.
Election in Angletown 2001-06-07
From Zoe:
At the last school election in Angletown 4620 votes were cast. Candidate Acute received 236 more votes than Candidate Obtuse. Candidate Right received 698 votes more than Candidate Acute. Candidate Straight received 256 votes less than Candidate Right. How many votes did each receive?
Answered by Penny Nom.
Danging couples 2001-06-06
From Danielle:
How many boy-girl dancing couples could be formed if 85 boys and 102 girls attend a school dance?
Answered by Penny Nom.
Exponential form 2001-06-06
From A student:
Hello can u please answer my question what is the exponent form of 8x8x8x8
Answered by Penny Nom.
Securing a plank in four dimensions 2001-06-06
From Becca:
If it takes two nails to secure a plank from rotating on a wall in the third spatial dimension, how many nails would it take if you were attempting to secure the board from rotating in the fourth spatial dimension?
Answered by Walter Whiteley.
Mutually exclusive 2001-06-05
From Marje:
What does the mathmatical term "mutually exclusive" mean. Pleas diagram if possible.
Answered by Penny Nom.
The shortest distance from a point to a line 2001-06-05
From Kat:
Find the shortest distance from the given line. Round to the nearest tenth. (-2,-1) and x+5y+20=0
Answered by Penny Nom.
Where is the other $1? 2001-06-05
From Samantha:
3 people go to a motel. The room is $30. They split it $10 each. While in the room the Mgr. tells the clerk the room was only $25, so he gives her $5 to take back to the 3 people. On the way she cant decide how to split the $5 between the three people so she puts $2 in her pocket and gives them $1 each.....Therefore, the room now only cost the three people $9 each, which is $27 plus the $2 the clerk pocketed,making a total of $29.....Where is the other $1?
Answered by Leeanne Boehm.
Two jugs 2001-06-04
From Heather:
One goes to a source of water with two jugs and only two jugs. One jug had a capacity of exactly three pints and the other five pints. Both jugs are opaque (you canít see inside them) and irregularly shaped. How can you use the two jugs and only the two jugs to measure exactly four pints of water?
Answered by Harley Weston.
Conservation, consumption and population growth 2001-06-04
From Steve:
I'm trying to quantify the relation between conservation/consumption and population growth. For instance let's consider California:

The 2000 census states that California's population grew from 29,760* in 4-1990 to 33,871 in 4-2000. I want to find r or rate of growth per year. Based on the exponential growth formula for population growth:
.
.
.


Answered by Penny Nom.
Common solution 2001-06-02
From Samantha:
  1. Solve for common solution: x+y=6 2x-3y=2

  2. Solve for y in terms of x: 3x-y=4

Answered by Penny Nom.
Three goldfish 2001-05-30
From Nathan:
A man has three goldfish. When the youngest goldfish was born, the oldest fish was three times the middle fish's age. Nine years ago the oldest fish's age was the sum of the two other fish's ages. How old are the three goldfish?
Answered by Penny Nom.
Multiplication 2001-05-30
From Lindsay:
Who came up with multipling? its so easy
Answered by Claude Tardif and Penny Nom.
Your DJ Business 2001-05-30
From Linda:
Your DJ Business has 6 rap, 10 rock, 6 alternative, 8 oldies, and 5 country CD singles. How many different 10-song sets can the DJ play, if she plays 2 singles from each category? and How many different 10-song sets if she plays exactly 3 rap singles and 4 rock singles in each set?
Answered by Leeanne Boehm.
SPRT 2001-05-24
From A researcher:
I have a medical-pharmaceutical study that says:

SPRT (something to do with Secuential and Truncated).

Do You know what it means S.P.R.T. and what it is?


Answered by Ejaz Ahmed and Penny Nom.
The diameter of an oval 2001-05-23
From Tim:
Is there a such thing as a diameter of a oval? If not, is there a way to get the circumference?
Answered by Claude Tardif and Penny Nom.
How many sides does a circle have? 2001-05-16
From Georganne:
How many sides does a circle have?
My son answered "infinate" on a test and was corrected. The school insists the answer is 0.

Answered by Chris Fisher and Denis Hanson.
A 6 character field 2001-05-16
From A parent:
I have a 6 character field that can use any combination of alpha-numeric characters (A-Z, 0-9). How many total combinations are possible?
Answered by Penny Nom.
Quadratics 2001-05-16
From John:
I am in the final stages of a math project and I need to interview an expert for the last part. Please try to answer at least a few of these questions.
  1. How do you use quadratic equations in your everyday life?
  2. Do you find being a math expert very helpful in life?
  3. Is the quadratic equation useful to you?
  4. Why did you decide to become a math expert?
  5. What do you think is the most important function of the quadratic equation?

Answered by Harley Weston.
25 y^2 + 20 y z - 4 z^2 2001-05-15
From Kimmy:
i don't know how to factor the trinomial 25y2 + 20yz - 4z2. I can get it, but i get messed up on getting the minus 4 at the end. all i get is a plus 4. Can it even be factored.
Answered by Penny Nom.
A pile of sand 2001-05-14
From Gul:
  1. Sand for use on icy roads is stored in a conical pile 14.2 m high and with a base diameter of 34.4 m
    • calculate the volume of the pile

    • if one sander can take 6.9 m of sand, how many sanders can be filled from the pile?

Answered by Penny Nom.
A triangular pyramid 2001-05-14
From Kumar:
In a Pyramid OABC, all the three sides of the base: a, b, c are known. Also known are the angles between the three sides from the vertex O: Angles AOB, BOC, COA.

With this information, is it possible to calculate the lengths of the three sides: OA, OB & OC.
Answered by Chris Fisher.

Manifolds 2001-05-14
From Thierry:
Maybe you could help me, because I have serious problems for the translations of an English mathematic word which is "MANIFOLD". It's impossible to have a clear translation in French. Do you have a solution?
Answered by Claude Tardif.
Toy bricks 2001-05-13
From Sarah:
A regular-sized brick weighs 4 kilograms. How much will a similar toy brick, made of the same material, but all of the same dimensions of which are four times smaller, weigh?

The 300-metre tall Eiffel tower is made of steel and weighs 8 000 000 kg. An exact replica, made of the same material, weighs 1 kg. How high is the replica


Answered by Penny Nom.
Simplifying algebraic expressions 2001-05-12
From A student:
How do you simplify algebra expressions?
Answered by Penny Nom.
The angles in a triangle 2001-05-11
From Nikki:
Find the measure, to the nearest degree, of each angle of a triangle with sides of the given lengths.

26, 35, 40


Answered by Penny Nom.
Circles, ellipses, parabolas and hyperbolas 2001-05-09
From Colleen:
How is an ellipse like a circle?
In what way does an ellipse have a center?
How is a hyperbola similar and different to an ellipse?
How is a parabola similar a different to a circle ellipse and parabola?

Answered by Pnny Nom.
Marginal Propensity to Consume 2001-05-09
From Esther:
The derivative of the national consumption function C = f(y) is called the ____________________.
Answered by Judi McDonald.
National consumption function 2001-05-09
From Brian:
If consumption is $11 billion when disposable income is 0 and the marginal propensity to consume is dC/dy = 1/(2y+4)1/2+0.3(in billions of dollars), find the national consumption function.
Answered by Harley Weston.
Maximize profit 2001-05-09
From Brian:
The marginal cost for a certain product is given by MC = 6x+60 and the fixed costs are $100. The marginal revenue is given by MR = 180-2x. Find the level of production that will maximize profit and find the profit or loss at that level.
Answered by Harley Weston.
Dividing fractions 2001-05-09
From Rina:
I just wanted to ask if you could help me in math. See I'm having a test soon and its on Dividing Fractions and I just don't get it. My math teacher says that I'll be just fine but I failed my math quiz. I went to ask eric but they could help me so they told me to go to you. So here I am asking you if you could help me.
Answered by Penny Nom.
The average value of a continuous function 2001-05-08
From Esther:
The average value of a continuous function y = f(x) on the interval [a,b] is given by ________________?
Answered by Harley Weston.
24 from 2, 2, 14, and 21 2001-05-08
From Kimberly:
using the numbers 2, 2, 14, and 21. how can you get 24 using + - X or division
Answered by Claude Tardif.
Taking matches 2001-05-05
From Mark:
Two piles of matches are on a table. A player can remove a match from either pile or a match from both piles. The player who takes the last match loses. If there are two players, how should you play?
Answered by Claude Tardif.
Probabilities and dice. 2001-05-04
From Carmen:
Hi! My name is Carmen! I am a member of a grade 5 class in Edmonton, AB. We were talking about probability in math, and I was wanted to know what the probability (odds) of rolling two sixes would be rolling two dice? If it is not to diffucult could you tell me what the probability (odds) of rolling any number with two dice were?
Answered by Andrei Volodin and Penny Nom.
How many acres are in a square mile? 2001-05-04
From Terri:
How many acres are in a square mile?
Answered by Judi McDonald.
Adding and subtracting rational expressions 2001-05-03
From Donna:
Adding and subtracting Rational expressions. I am in grade 10 and I am a student here is an example of the questions:

1/(x+1) - 1/(x-1) = ?


Answered by Penny Nom.
Tiling a floor 2001-05-01
From Sarah:
Cut out of paper or cardboard a quadrilateral having no two sides parallel, no two sides of equal length and no indentations. Can an endless floor be tiled with copies of such a figure?
Answered by Claude Tardif.
Area of a regular polygon 2001-05-01
From Carl:
What is the formula for finding the area of a regular Hexagon and a regular Pentagon
Answered by Penny Nom.
Population growth 2001-05-01
From Gina:
Suppose the population of a country increases at a steady rate of 3% a year. If the population is 50 million at a certain time, what will it be 25 years later? Define the recurrence relation that solves this problem.
Answered by Penny Nom.
A sequence defined recursively 2001-05-01
From A student:
A sequence s is defined recursively as follows:

s0=1
s1=2
sk=2sk-2 for all integers

- Compute s2,s3,s4... to guess an explicit formula for the sequence sk.
Answered by Penny Nom.

A sequence of even terms 2001-04-29
From A student:
A sequence c is defined recursively as follows:

c0 = 2
c1 = 4
c2 = 6

ck= 5ck-3 for all integers

Prove that cn is even for all integers.


Answered by Leeanne Boehm and Penny Nom.
Choosing a car 2001-04-28
From Ashley:
THE CAR DEALERSHIP IN TOWN OFFERS 32 DIFFERENT MODELS OF VIHICLES.EACH MODEL HAS A CHOICE OF EIGHT INTERIOR COLORS,EIGHT EXTERIOR COLORS,AND ALSO THE OPTION OF AUTOMATIC OR MANUAL TRANSMISSION. HOW MANY COMBINATIONS ARE POSSIBLE?
Answered by Penny Nom and Andrei Volodin.
A Taylor series 2001-04-27
From Karan:
Given the following information of the function
  1. f''(x) = 2f(x) for every value of x

  2. f(0) = 1

  3. f(0) = 0
what is the complete Taylor series for f(x) at a = 0

Answered by Harley Weston.
Triangles and fractions 2001-04-27
From Constance:
My name is Constance and I am thirteen years old (I am a student). The question that I am queering about I don't understand why you do ONE HALF x the base x the width WHEN YOU WANT TO FIND the area of a triangle? My second question is if you multiply one half and 10 together why does it come out as 5?
Answered by Penny Nom.
A confidence interval 2001-04-26
From Kim:
A poll asked 1528 adults if they were in favor of the death penalty, 1238 said yes, find 99% confidence level for percent of all adult who are in favor of the death penalty.
Answered by Andrei Volodin.
Working together 2001-04-26
From Stephanie:
Tom takes 10 hours to piant a mural on the wall of Evergreen School. Carol takes 6 hours to do the same job. If they work together, how long will it take them to paint the mural?
Answered by Claude Tardif and Penny Nom.
A lemon and a recall 2001-04-25
From Katie:
A Man has two cars, a recall and a lemon. The probability that the recall starts is 10%. The proabability that the lemon will start is 5%.

What is the proabability that both cars will start?


Answered by Andrei Volodin.
Expected value 2001-04-24
From Cindy:
A game consists of rolling a single fair die. If a number great than 4 is rolled you win the number of dollars showing on the die. If any other number is rolled, you receive $1.00. What is the expected value of this game?
Answered by Andrei Volodin.
Writing in the math curriculum 2001-04-23
From Wendy:
I am trying to incorporate more writing into my math curriculum. I am currently using journals and activities which involve written explanations. Any new ideas on how to involve my students more in the writing process in math class?
Answered by Walter Whiteley.
A few problems 2001-04-22
From Carol:
Hello I am a College student trying get through my elmentry concepts class. I am also working with the middle and elementry school student with their work. I was given a few problems to work on and i am having a hard time explaining them to students. DO you think you can help with a coupld of these questions!
Answered by Penny Nom.
Oil revenue 2001-04-21
From Brian:
Suppose that t months from now an oil well will be producing crude oil at the rate of r(t), not a constant, barrels per month and that the price of crude oil will be p(t), not a constant, dollars per barrel. Assume that the oil is sold as soon as it is extracted from the ground.
  1. Find an expression for the total revenue from the oil well, R(t).

  2. A certain oil well that currently yields 400 barrels of crude oil a month will run dry in 2 years. The price of crude oil is currently $18 per barrel and is expected to rise at a constant rate of 3 cents per barrel per month. What will be the total revenue from this well? {Hint: Model the degraded production rate with the equation:
    r(t) = (A-Bt)e0.04t}

Answered by Harley Weston.
Five 2001-04-21
From Billie:
If One is single, two is twin, three is triple, four is quadruple, five is ????? what is five and where would I find the rest of the numbers,six, seven, eight, nine, etc.
Answered by Penny Nom.
Repeating decimals 2001-04-21
From Sarah:
Hi, I'm working on a project for school. The theory I choose was "When turned into a fraction, a repeating decimal has a denometor that is a multiple of three." I have a couple of questions about this topic. My first question is, have you ever heard of this, and what can you tell me about it? My second question is, when I was testing this theory I came across .999... now, when this is a fraction it is 9/9 which is equal to one. The denometor is a multiple of 3, but it's a whole number. I don't understand how a decimal can be equal to a whole number since a decimal is a piece of a whole number. Please don't just show me a math problem, I don't want to see a math problem. I want to see an explanation of this theory and the decimal .999...
Answered by Penny Nom.
Two boats 2001-04-19
From Pat:
Two boats head directly toward each other, one of them traveling 12 miles per hour and the other traveling 17 miles per hour. They begin at a distance of 20 miles from each other. How far apart are they one minute before they collide?
Answered by Penny Nom.
Pyramids 2001-04-18
From Kelly:
I'm in tenth grade and I know that there's some formula for finding out the slope of the side of a pyramid. I thought it could be the pythagorean therom, but I'm not sure if this works. Please send me the formula as soon as possible, I need it for a project.
Answered by Penny Nom.
A geometry proof 2001-04-18
From Melissa:
Extend the bisectors of angle A, angle B, and angle C of triangle ABC to meet the circumcircle at points X, Y, and Z respectively. Show that I is the orthocenter of triangle XYZ.
Answered by Chris Fisher.
The number of occurrences of 0 is __, of 1 is __, of... 2001-04-17
From Martyanne:
"In this sentence the number of occurrences of 0 is __, of 1 is __, of 2 is __, of 3 is __, of 4 is __, of 5 is __, of 6 is __, if 7 is __, of 8 is __, of 9 is __."

Each blank is to be filled with a numeral of one or more digits, written in decimal notation.

Answered by Claude Tardif.
Isoscles and scalene 2001-04-17
From Autumn:
explain where the term isoscles and scalene came from?
Answered by Chris Fisher.
Differentiation 2001-04-17
From Esther:
Could you please tell me what the first derivative is of the following:

y = 2/(2x+e2x)

Is it (1+xe2x)/(2x+e2x)2 or perhaps -4(1+e2x)/(2x+e2x)2 ? I am a little confused between the two!


Answered by Harley Weston.
Solving some equations 2001-04-16
From Justin:
Solve the following equations for x
  1. x+7=10
    x=

  2. x-7=10
    x=

  3. 3x=7
    x=

  4. x/3=10
    x=

  5. .

  6. .

  7. .

  8. .

Answered by Penny Nom.
Leukemia 2001-04-15
From Don:
Assume: Leukemia occurs at an incidence of 1 in 10,000.

Benzene is a known carcinogen which causes Leukemia and has been found in toxic levels in the homes of a community of 1200 people.

Question: How many cases of Leukemia would be necessary to reject the null hypothesis. That is, to suggest that statistically, the cases are more likely due to benzene exposure rather than chance.


Answered by Chris Fisher and Penny Nom.
Constructing an equilateral triangle 2001-04-14
From Peggy:
Please send directions to make an equilateral triangle in plane geometry. I want each student to draw two, cut them out, and place them together to form a Jewish Star.
Answered by Penny Nom.
The sides of an octagon 2001-04-13
From Craig:
How do I figure the length of the sides of an octagon when all I know is the diameter (4.375).
Answered by Penny Nom.
1 = 0.999... 2001-04-13
From Joan:
I have a middle grade math question for you. I would like to know why .9999... = 1 ? I can not use algebra to show this or the following: We agree that 2 = 2 and that 2-2 = 0, so

1.00000......
-0.99999.....
----------------
0.000000......
and
0.000... = 0 therefore 0.9 = 1
----------OR--------------- 1/3 = 0.333333 and 3 X 1/3 = 1, so if 3 X 0.333... = 0.999... then 0.999... = 1

My teacher says that I can not use the above example to show why this is true, and that I must use a couple different examples. He says that there are several other ways. Do you know any? I could really use the help because I can't think of any other ways to show this is true. Thanks for any help you can give.


Answered by Penny Nom.
Squares on a chess board 2001-04-11
From Tom:
It was once claimed that there are 204 squares on an ordinary chessboard (8sq. x 8sq.) Can you justify this claim? "PLEASE" include pictures.

How many rectangles are there on an ordinary chessboard? (8sq. x 8sq.) "PLEASE" include pictures.


Answered by Penny Nom.
Prisms 2001-04-10
From A fourth grade class:
A fourth grade class is curious about the names that have been given to various solid geometric figures based on their shapes: are there names for solids based on a rhombus, a trapezoid, a parallelogram, a decagon, and/or a quadralateral?
Answered by Chris Fisher.
Parabola problems 2001-04-10
From Kathleen:
  1. Graph each function and state its domain and range. y = 3x2 + 4

  2. For each parabola find: i) the direction of opening
    ii) the coordinates of the vertex
    iii) the y-intercept
    iv) the x-intercepts
    y = x2 + 3

  3. Find the equation of each parabola vertex at (0, -2) and passing through the point (3,7)

Answered by Harley Weston.
Integration by parts 2001-04-09
From A student:
how do you integrate x tan-1x dx, i know it can be done by integration by parts maybe, but i'm not sure....
Answered by Claude Tardif and Harley Weston.
The domain of a function 2001-04-08
From Mina:
Let f(x) = (2x2+3x-17)/(x+5)
What is the domain of f? What are the values of x for which f'(x) does not = 0?

Answered by Harley Weston.
The normal to a curve 2001-04-08
From Varenne:
I am having SO much trouble tackling this question and don't know what the right answer is... can you help me out? The question is

Find the equation of the normal to the curve y=(x-2)2/(1-x)2 that is parallel to the line x+4y+7=0

Answered by Harley Weston.
Equivalent fractions 2001-04-08
From A student:
which of the following pairs of fractions are equivalent?
questions a-f

(a) 1/5 and 6/30
(b) 4/9 and 16/27
(c) 6/10 and 30/50
(d) 15/20 and 48/64
(e) 2/3 and 33/100
(f) 12/32 and 3/16


please show how u got the answer thank you.

Answered by Penny Nom.
Common tangents 2001-04-08
From Anne:
I have been working on this problem for a while but I'm not sure I'm getting the right answer: Find the common tangents of 2y=x2 and 2y=-x2-16 Thanks for the help. :)
Answered by Harley Weston`.
Where do the lines y=2x-4 and y=x-1 intesect? 2001-04-06
From Bryce:
solve the following problem by setting them equal to each other. Solve for x and y. Where do the lines y=2x-4 and y=x-1 intesect?
Answered by Penny Nom.
The unit circle and trigonometry 2001-04-05
From Ashley:
"My teacher wants us to find out what a unit circle is, which I found out, a circle with the radius of 1, but the problem is he wants us to show the relationship between the unit circle and the sine(30,45,60 degrees), cosine(30,45,60 degrees),and tangent ratios(30,45,60 degrees). I need help with this and my teacher will not help us out. Thanks very much ...
Answered by Penny Nom.
Hexominos 2001-04-05
From Tom:
What is a hexomino and how many different shapes are possible?
Answered by Harley Weston.
A combination lock 2001-04-05
From Ken:
What is total number of combinations possible on a combination lock which contains 3 wheels each numbered 0-9?
Answered by Penny Nom.
f(x), f(x) + 2, f(x +2) 2001-04-04
From Monica:
Could you explain to me how one should go about graphing functions such as f(x), f(x+2), and so on. Also, how should you explain things such as constants and relationships among functions?
Answered by Penny Nom.
Two integrals 2001-04-03
From Jim:
I'm having trouble with these integrals. Can you help me out?

1)the integral of:

x5 arctan x dx

2)the integral of:

2x5 + 9x4 + 19x3 + 13x2 - 5x - 25
----------------------------------------------- dx
x4 + 4x3 + 5x2


Answered by Claude Tardif.
(8 + 5)5 = (y + 4)4 2001-04-01
From Kelley:
(8 + 5)5 = (y + 4)4
What is the value of y?

Answered by Penny Nom.
Sigma 2001-03-31
From A student:
What does the Sigma symbol mean in mathematics?
Answered by Penny Nom.
Volume and surface area of a cylinder and a triangular prism 2001-03-31
From Kevin:
I was wondering how do you find the volume and surface area of a cylinder and a triangular prism.
Answered by Penny Nom.
Why K? 2001-03-30
From Molly:
I was wondering why K is used as the dilation constant for transformation geometry?
Answered by Chris Fisher.
Stem-and-leaf plot 2001-03-29
From Kelly:
I'm tutoring my 5th grade nephew, and he needs to correct test questions regarding a stem - and - leaf plot. He left his book at school and I'm not familiar with the term. Can you explain it to me so that I can help him?
Answered by Penny Nom.
Cash advance 2001-03-28
From A student:
dulani has a new credit card. it says: you can get cash advances wherever you are. whenever you want. also if you pay off your balance in full each month for a small transaction fee, the cash is interest free. (see important information on reverse side. On reverese side it says:

cash advance transaction fee:

$500 or less 2.5%;
$500.01 to $1000.00-2%;
$1000.01 or more -1.5%;
$2.00 minumum.

Annual percentage rater for cash advances 19.8%.
dulani wants to know what a cash advance will really cost. Analyze the cash advance terms given here. how much would he pay for a $20 cash advance? What about a $450 cash advance? What annual interest rate are these charges equivalent to? choose other amounts. determine cost and annual ratees for these as well. make recommendations to dulani.

Answered by Penny Nom.
The diagonal 2001-03-28
From Candace:
Building measuring 40 feet 3 inches by 50 feet 3inches What is the measurement of the diagonal of the building? What was method of obtainin answer?
Answered by Andrei Volodin.
Solving an equation 2001-03-27
From Jennifer:
How would I solve this equation? 1.6 = 2x/(.1-x) - the equation in parenthesises is raised to the second power. I know you take the square root of both sides, but there is where I get lost.
Answered by Leeanne Boehm.
12 RTV's 2001-03-27
From Christine:
  1. In order to promote the 2000 Census and encourage participation, 12 Road Tour Vehicles(RTV's) set out from 12 locations across the US for a 10 week promotion tour last February. The RTV's reportedly traveled a total combined distance of 97,911 miles. What was the aberage number of miles traveled in a day by each RTV?

  2. 2. The 12 RTV's mentioned above traveled a total combined distance equivalent to traveling 12.35 times around the earth at the equator. To the closest mile, what is the radius of the earth? Use 3.14 as your approximation for pi.

Answered by Leeanne Boehm.
Finite non-periodical numbers 2001-03-27
From Wouter:
Is there anyone who knows the official name for decimal finite non-periodical numbers such as 0.4 or 0.25 as opposite of numbers like 0.3333333... or 0.28571428571428...?
Answered by Penny Nom.
Mistie, Tammy, and Jennifer 2001-03-27
From Renee:
Mistie, Tammy, and Jennifer audition for parts in a big-budget remake of Bedtime for Bonzo. On the basis of their past experience and the caliber of competition they face, Mistie has a 40% chance of being hired, Tammy has a 50% chance, and Jennifer has a 30% chance. If exactly two of the three are cast, what is the probability that Mistie was rejected?
Answered by Andrei Volodin.
A box with an interior of 1.25 cubic feet 2001-03-26
From Gabe:
I need a box with an interior of 1.25 cubic feet. I am using 3/4 inch material to make the box. What would the exterior dimensions be in inches?
Answered by Harley Weston.
Matrices 2001-03-26
From Peg:
What are some applications of matrices, basic trigonometry, and linear systems in the real world? I'm writing and Algebra report about where these topics are used outside of the classroom.
Answered by Judi McDonald and Walter Whiteley.
Two ferry boats 2001-03-25
From Gil:
Two ferry boats leave from opposite shores. One is faster than the other. They meet 720 yards from the nearest shore. They proceed to destination and upon returning they meet 400 yards from the other shore. What is the exact width of the river.
Answered by Penny Nom.
Airflow in windpipes 2001-03-25
From Ena:
The volume of air flowing in windpipes is given by V=kpR4, where k is a constant, p is the pressure difference at each end, R is the radius. The radius will decrease with increased pressure, according to the formula: Ro - R = cp, where Ro is the windpipe radius when p=0 & c is a positive constant. R is restricted such that:
0 < 0.5*Ro < R < Ro,
find the factor by which the radius of the windpipe contracts to give maximum flow?

Answered by Harley Weston.
Cube roots on a calculator 2001-03-24
From Will:
Hi my name is Will.I have a question about the calculator TI-83 Plus or the use of a scientific calculator. It is about using them to turn a cubed root to a decimal. When using that for a rational and irrational number. Rational numbers is a number that terminates or repeates. A irrational number goes on and on and uses ... (like pie). Like when you type the cube root of 8 it gives you 2, and that is a rational number. The squre root of 2 is 1.41421356... that is irrational. So why is it when you put the cube root of 16 in the calculator it says 2.5198421 that is rational it terminates at the ninth digit and my float is set for the tenth? But why when you do it by hand you get the 2 times the cube root of 2 and that is irrational? Why is that?
Answered by Judi McDonald.
An elliptic tunnel 2001-03-24
From Janna:
A tunnel is built under a river for a road 12m wide with a 2m sidewalk on either side. The top of the tunnel is semi-elliptical. A local bylaw stipulates that there must be a clearance of at least 3.6m at all points on the road. If the smallest possible ellipse is used, find the clearance at the center of the road.
Answered by Harley Weston.
A suspension bridge 2001-03-24
From Janna:
The cables of a suspension bridge hang in a curve which approximates a parabola. The road bed passed through the vertex. If the supporting towers are 720m apart and 60m high, find:

a) an equation of the parabola (it's y = 1/2160x2)

b)the height of the cables at a point 30m from the vertex. I substituted 30 in for the x value and got 0.42 and the answer is 42. What did I do wrong?
Answered by Denis Hanson and Claude Tardif.

A famous landmark 2001-03-23
From Corinne:
A family is traveling due west on a road that passes a famous landmark. At a given time the bearing to the landmark is N 62 degrees W, and after the family travels 5 miles farther the bearing is N 38 degrees W. What is the closest the family will come to the landmark while on the road?
Answered by Harley Weston.
Genealogy 2001-03-22
From Brian:
In considering tracing genealogy the problem of a large and growing number of direct (grandparent) ancestors arose. Since the number of grandparents in any given generation doubles, for a person born in 2000 and assuming 25 year generations, in the year 1000 that person would have over one trillion direct ancestors in that year alone. Since estimates I have read place the total world population in 1000 at 250-300 million, the one trillion figure cannot be correct. I must be missing something in my calculations. Can you help provide the answer? While I am unlikely to be able to trace back to 1000, there is reason to believe 1250 is possible. Even in that year there would exist over one billion direct ancestors. That is certainly rather daunting and unlikely to be achieved if correct.
Answered by Penny Nom.
Camille Jordan 2001-03-22
From Missy:
PLEASE SEND ME INFORMATION ON CAMILLE JORDAN AND HIS WORK IN ABSTRACT ALGEBRA
Answered by Rick Seaman.
Factoring 2001-03-22
From Kaleena:
Factor this equation: 5x7-10x5+4x3-8x
Answered by Harley Weston.
The repetend in repeating decimals 2001-03-21
From Sharon:
What is the name for the bar over the repetend in repeating decimals? Also, what is the name of the long division "house"?
Answered by Penny Nom.
A bargin with the devil 2001-03-21
From DJ:
There once was a boy who spent all his time dreaming about getting rich. " I would do anything to be rich". The devil, hearing this, appeared before the boy and offered to make him rich. "See that bridge?" said the devil. Just walk across and I will double your money you have now. In fact, each time you cross I will double your money. There is just one thing you must give me $24 after each crossing. The boy agreed. He crossed the bridge, stopped to count his money and, and sure enough, it had doubled. He paid the devil $24 and crossed again. Again his money doubled. He paid another $24 and crossed a third time. Again his money had doubled, but this time there was only $24 left which he had to pay the devil so he was left with nothing. The devil laughed and vanished.
  1. How much money did the boy start with?

Answered by Penny Nom.
Timing with hour glasses 2001-03-20
From Nathan:
How can a chef use an 11 minute hour glass and a seven minute hour glass to time a vegtable that needs to be steamed for 15 minutes.
Answered by Leeanne Boehm.
Radicals 2001-03-18
From Nikki:
3) squareroot of x2 + 10x + 25

4) sixthroot of (m + 4)6

5) cuberoot of -64r6w15

.
.
.
.

Answered by Penny Nom.
Systems of equations 2001-03-16
From joy:
How do u solve problems using systems of equations?
~ finding x and y~

ex:

26 = 3x - 2y
42 = 4x + y


Answered by Penny Nom.
How can you prove a quadrilateral to be a parallelogram? 2001-03-16
From Joy:
How can you prove a quadrilateral to be a parallelogram?
Answered by Walter Whiteley.
Probability and a three digit number 2001-03-15
From Glenn:
A THREE DIGIT NUMBER IS SELECTED AT RANDOM. WHAT IS THE PROBABILITY THAT NON OF THE THREE DIGITS ARE PRIME.
Answered by Harley Weston.
Names of the polygons 2001-03-15
From Sami:
I was asked to find out what the names for different polygons were. The only two I cannot find are the names for a polygon with 30 sides and a polygon with 40 sides. I would really appreciate it if you could help me. Thank you very much.
Answered by Harley Weston.
The bond angles of a tetrahedral polygon 2001-03-14
From Nishi:
how do i prove (a simply as possible) why the bond angles of a tetrahedral polygon are 109.5 degrees? *i already have two explanations that i don't understand. one is about "theory of dot products" and "vectors" and a hook-like symbol w/ a cosine, and the other has an incomprhensible diagram w/ difficult notation- PLEASE BE SIMPLE! thanks sooo much
Answered by Harley Weston.
A jogger 2001-03-12
From Bill:
At time t=0 a jogger is running at a velocity of 300 meters per minute. The jogger is slowing down with a negative acceleration that is directly propotional to time t. This brings the jogger to a stop in 10 minutes.

a) write an expression for the velocity of the jogger at time t.

b) what is the total distance traveled by the jogger in that 10-minute interval.


Answered by Harley Weston.
Quadratic equations 2001-03-11
From Philip:
I am a student in the UK and i am in secondary school and i am doing my GCSE next year.

here are my questions:

solve the following equations correct to 2 decimal each time.

[1] 2xsquared + 5x = 1

[2] 3xsquared(2x+1)=1

[3] xsquared + 7x + 2 = 2x + 4

[4] 5xsquared - 3x = xsquared + 4


Answered by Penny Nom.
Math Puzzle 2001-03-10
From Vanessa:
I have a math puzzle. I have been working on it for over 2 weeks. Here it is: use the numbers 1-25 each only once. The rows across are an equation and the columns down are an equation. There are 25 boxes. You have to use the order of operation x and / first then = and -. Some of the answers are -numbers.
.
.
.

Answered by Claude Tardif.
Two locus problems 2001-03-08
From Janna:
A point P moves such that it is always equidistant from the point G(2,5) and the line defined by y=3. Find the equation of the locus. I got as far as the equation: 3y2 -4y = -x2 + 4x - 16 and didn't know what to do from there. Of, course that whole equation could be wrong.

Question 2: P is always twice as far from A(8,0) as it is from B(2,0). Find the equation of the locus. Once again, I got as far as y2 = -x2 -8x -56, and got stuck.


Answered by Harley Weston.
The angle of elevation 2001-03-08
From Jeffrey:
At a Certain time, a vertical pole 3m tall cast a 4m shadow. What is the angle of elevation of the sun?
Answered by Harley Weston.
problem of the week 2001-03-06
From Peggy Allan:
My son has been challenged with "the problem of the week" and I am unable to assist him in finding a reasonable solution.

Problem 1,

Julio needs to draw a line segment 15cm long. He does not have a ruler. He does have some sheets of letter size paper 28 X 21.5 cm. Describe how Julio can use the paper to measur 15 cm.

Answered by Claude Tardif.
The substitution method 2001-03-05
From A student:
Solve each system of equations by the substitution method. Show your work.
  1. y = 8
    7x = 1 - y

  2. y = x - 1
    4x - y = 19

Answered by Penny Nom.
Powers 2001-03-04
From A student:
Hey, can you show me how you do ..

(2xy)to the 3rd power (x) to the 2nd power?


Answered by Penny Nom.
Coins and monomials 2001-03-04
From A student:
Tokola has 5 coins with a total value of $1.05. He canot make change for a dollar, 5 cents, 10 cents, 25 cent, or 50 cents. What 5 coins does he have?

and

Determine whether 6c-d is a monmial. Explain why or why not.


Answered by Penny Nom.
Arc of a sphere 2001-03-04
From Some students:
My friends and i have a geometry project and we cannot seem to figure out how to find the arc of a sphere.
Answered by Harley Weston.
What is the value of n? 2001-03-03
From Adam:
When I add n to 7 and then divide my answer by 4,I get 5. What is the value of n?
Answered by Penny Nom.
Spins of a roulette wheel 2001-03-02
From Bob:
Here's a problem I'm working on myself.....If you look at six consecutive spins of a roulette wheel, how many combinations of red and black are possible? I.E. BRRBRB, BBBBBR, BBRRRB......ETC.....
Answered by Claude Tardif.
How tall is the tree? 2001-03-02
From Ronda:
a tree's shadow is 42 ft. long. There is a stop sign that is right next to it and it is 18 ft. tall and it's shadow is 12 ft. long. How tall is the tree?
Answered by Penny Nom.
P vs NP problem 2001-03-01
From Roy:
What math journals would publish a proposed solution to the P vs NP problem when the work has been done by an amateur mathematician who has no degree in math?
Answered by Claude Tardif.
Converting percents to decimals 2001-03-01
From Elliot:
My math question is about Converting Percents to Decimals, here it is-

Convert 37 1/2 = ?


Answered by Penny Nom.
Solve for the variable 2001-02-28
From Ashley:
I've got 2 questions. We (my dad and I) can not figure them out. please help me.
  1. 21 = 9 - 2 (4a + 2) and I am supposed to find out what the variable is.

  2. 2/3 n + 3/8 n = 15/16 (those are fractions). Again, I need to find out what the variable is.

Answered by Penny Nom.
Three sheep 2001-02-28
From A student:
A farm sold three sheep. the weight of the three sheep combine wae 152 lbs. The smallest sheep weighted 7 lbs less than the middle size sheep. The largest sheep weighted three times the smallest sheep. what is the weight of each sheep?
Answered by Penny Nom.
T-numbers an T-totals 2001-02-27
From Mr. M. Wiseman:
Investigate the relationship between the T-total and the T-number.
Answered by Penny Nom.
Lotto 6-49 2001-02-27
From A student:
I am a secondary student and I was wondering if there was a better way to find out how many combinations and what they are in the lotto 6/49 than writing them out on a piece of paper. Any help you can provide me with would be greatly appreciated.
Answered by Penny Mon.
The n-th place of pi 2001-02-27
From Andrew:
I have always wondered; is it possible to find the value of an irrational number, such as (phi) at it's nth decimal place???

You would plug the decimal place into the formula and the value would be given at the specified decimal place.

When we look at the expression { (sqr 5 + 1)/2 } we are in a sense visualising the number in it's entirety, so the formula may include elements of the above expression in some form.
Answered by Claude Tardif.

Solve for two variables 2001-02-25
From A student:
How do I solve for %1 and %2 in the following formula when T1, T2 and T3 are known? %1 and %2 are ratios of the same element, therefore %1 + %2 = 100%

(%1 x T1) + (%2 x T2) = T3

Answered by Penny Nom.
Shape, symmetry, measurement and space 2001-02-24
From Marilyn:
I have to write in detail about how young children learn about shape, symmetry, measurement and space. It is not activities I need it is how the children learn. How/why their brain takes it all in
Answered by Walter Whiteley.
Solving Equations 2001-02-23
From Stephanie:
Do you know who came up with solving equations?
Do you have any web-sites that can give me good information on solving equations?
Do you know any history on solving equations?
Do you know what solving equations is used for?
And finally............Sorry about all of the questions :)
Has the form of solving equations changed from the time it came out to now?

Answered by Juci McDonald.
What is my name? 2001-02-23
From Julia:
My perimeter is 12 cm. My sides are all equal. The sum of half my sides is equal to 6 cm. How many sides do I have? What is my name?
Answered by Penny Nom.
Difference of Squares 2001-02-22
From Bruno-Pierre:
I noticed the other day that if you substract two consecutive squared positive numbers, you end up with the same result as if you add up the two numbers.

Ex. 5 and 6 (2 consecutive positive numbers)
52 = 25
62 = 36
36 - 25 = 11 (Substraction of the squared numbers)
5 + 6 = 11 (Sum of the numbers)

A more algebric view:
a2 - b2 = a + b where a and b are consecutive positive positive numbers. (b = a + 1)

I wondered if this rule had a name, and who discovered it.


Answered by Penny Nom.
The domain of the derivative 2001-02-22
From Wayne:
I know that the domain of f'(x) is a subset of the domain of f(x). Is it necessarily true that the subset will have at most one less element than the domain of the original function?
Answered by Harley Wesston.
Mr. Moser's roof 2001-02-21
From Michelle:
Mr. Moser is planning to replace the roof of his home. He needs to order a pack of shingles. Each pack covers 100 sq. ft. of roof. Without a ladder, Mr. Moser can not climb to the roof to measure it. Instead, he measures his attic and finds it to be 40 ft. long, 24 ft. wide, and 5 ft. high at the peak of the roof which is in the center of the house. Although the roof is even with the side walls, he estimates the roof line continues 1.5 ft. beyond the front and back walls. How many full packs of shingles should Mr. Moser order to cover his roof?
Answered by Penny Nom.
Winning percentages 2001-02-21
From Mike:
TEAM A WINS 55% OF GAMES PLAYED

TEAM B WINS 40% OF GAMES PLAYED AGAINST THE SAME OPPONENTS

WHAT WINNING PERCENTAGE SHOULD BE EXPECTED WHEN TEAM A PLAYS TEAM B (NEUTRAL SITE)?


Answered by Harley Weston.
Faces 2001-02-21
From Sandy:
How many faces are there on a sphere?

What are the faces of a cone?

What is the definition of a "face" of a 3D object?


Answered by Walter Whiteley.
The law of cosines in the real world 2001-02-21
From Hope:
Do you have any examles and/or labs that show how the law of cosines is used in the real world?
Answered by Harley Weston.
Undecagon 2001-02-21
From Karen:
I need to know what kind of a polygon an undecagon is and how many sides it has. This is a problem in my math book.
Answered by Penny Nom and Michael Tsatsomeros.
A remainder of 1 2001-02-20
From A student:
what is the smallest positive integer which has a remainder of one when divided by 2,3,4,5 and 6
Answered by Penny Nom.
A pole next to a building 2001-02-20
From Robert L. Haun:
A pole 100 feet tall is erected next to a building 10' X 10' X 10' . Object is to cut off the pole so the upper end remains touching the part cut off, while the cut-off portion touches the cutoff point, the edge of the building and the ground. Note assume the pole is touching the building and is "very thin" What is/are the cutoff points?
Answered by Penny Nom.
Cubic feet 2001-02-20
From Frances:
I'm trying to calculate cubic feet from the dimensions of a machine given in inches.

Width = 32"
Height = 79.5"
Length = 91.5"
Weight = 3920 lbs

Please help? Obviously I've been away from school too long!!


Answered by Penny Nom.
Law of cosines 2001-02-20
From Emily:
I missed a few days of class and I can't figure out how to solve Law of Cosines problems. I have the notes and formulas but I can't figure out how to do the math involved to answer the problems. I am also getting confused about how to use degrees and seconds in the Law of Sine and Cosine. I can't seem to get the correct answers and I don't really know how to solve them and enter them into my TI 86.
Answered by Harley Weston.
Difference of squares 2001-02-20
From Janna:
Hi! I was just wondering how you would factor x2 - 9y2.
Answered by Harley Weston.
g(g+5)=0 2001-02-20
From Jenna:
g(g + 5) = 0
Answered by Penny Nom.
The area of platts 2001-02-20
From Steve:
I have to calculate the areas of different "platts" of land from a map, and the only thing I have are the four side measures. There are some cases where no sides are parallel, no lengths the same, and no angles are given, and I was wondering how to calculate the area of such figures. I've attached a rough sketch of a typical platt's shape.
Answered by Walter Whiteley.
Dividing polynomials 2001-02-19
From Janna:
I have two questions involving dividing polynomials by polynomials.

Here's the first one:

Two factors of 12a4 -39a2 + 8a - 8a3 + 12 are a - 2 and 2a + 1. Find the other factors. The other question I'm stuck on is: When 10x3 + mx2 - x + 10 is divided by 5x-3, the quotient is 2x2 + nx - 2 and the remainder is 4. Find the values of m and n.


Answered by Penny Nom.
Adding functions 2001-02-19
From Jackie:
f(x) = x2 + 1; g(x) = x2 - 1

find
a) ((f+g) of h)) (x)
b) ((f-g) of h)) (x)
c) ((f of h) - (g of h)) (x)


Answered by Penny Nom.
Working on commission 2001-02-17
From Jay:
Ron the realtor is offered a job directly out of real-estate school. He has a choice as to which way he will receive his salary the first year.

Salary Plan 1: He would receive a base pay of $2000 per month plus a 3% commission on each sale.

Salary Plan 2: No base pay but a 6% commission on each sale.


Write an equation to determine when it would be better to switch from the first plan to the second plan. Give a one- or two-sentence answer that includes Ron`s sales in a month. Round to the nearest dollar, if necessary.

Answered by Leeanne Boehm and Penny Nom.
Angles in a polygon 2001-02-17
From Joan:
How many sides does a polygon have if its smallest interior angle is 120 degrees and each sucessive angle is 5 degrees greater than the predecessor?
Answered by Penny Nom.
Differentiation of y = x n 2001-02-17
From Jashan:
i am studying differentation at the moment i have drawn some graphs such as y=x 2. i have found the formula for the gradient of this curve, this being 2x obtained by using differentation, but i need to know the general case for the formula where y=xn in order for me to understand this topic more throughly, i would also like to know how u derived this general formula
Answered by Harley Weston.
A four digit number 2001-02-17
From Dean:
Need to find a 4 digit number that can be divided by 2, 3, 4, 5, 6, 7, 8, 9, and will always have a remainder of 1 in the answer.
Answered by Penny Nom.
A quartic equation 2001-02-15
From George:
Let P(x) = x4 + ax3 + bx2 + cx + d. The graph of y = P(x) is symmetric with respect to the y-axis, has a relative max. at (0,1) and has an absolute min. at (q, -3)

a) determine the values for a, b c, and d using these values, write an equation for P(x)
b) find all possible values for q.


Answered by Harley Weston.
A four dimensional object 2001-02-14
From A student:
Can you give me some examples of a four dimensional object you can find around your house?
Answered by Harley Weston.
Linear equations 2001-02-13
From Jamie:
I have a question how do you do linear equation?
Answered by Penny Nom.
Rewrite as an addition problem and Simplify 2001-02-13
From A student:
On my homework sheet it says "Rewrite as an addition problem and Simplify" and the gives this problem "16 - 120 ="
how do i do that problem?

Answered by Penny Nom.
Books and cars 2001-02-13
From Aquilya:
  1. A pile of books is 96.08cm. If each book is 0.8cm thick. How many books are in th pile?

  2. 2. A car travels 9.84km on 0.6 litres of petrol. How far will the car travel on 3.7 litres of petrol?

Answered by Penny Nom.
L - R 2001-02-12
From A student:
If L = (a-b)-c and R=a- ( b-c), then L-R = ???
Answered by Penny Nom.
An order relation 2001-02-12
From Amy:
What is an order relation?
Answered by Claude Tardif.
Two inequalities 2001-02-12
From A student:
If 3
Answered by Leeanne Boehm.
Chris' cards 2001-02-11
From A student:
Chris gave Jane x cards. He gave Bety one card more than he gave Jane and he gave Paul two cards fewer than he gave Betty. In terms of x, how many cards did Chris give Bety, Jane, and Paul together?
Answered by Penny Nom.
Multiplying one and two digit numbers 2001-02-11
From Marty:
When multiplying do you put the number you are multiplying by on the top or the bottom. Example if you are multiplying by 6's would you write the problem 6
x 2
or 2
x 6
Maybe it just doesn't matter. But when you get to double digits, the double digit always goes on the top?

Answered by Penny Nom.
Lucky dip 2001-02-09
From Andrew:
I buy 8 lucky dip (random number) panels, each panel has 6 numbers chosen at random from 49. I never seem to cover more than 32 different numbers on average, over the total 48 numbers chosen, which seems low? How can I calculate the probability of 32 different numbers or 33 diferent numbers?
Answered by Claude Tardif.
A dog and a rabbit 2001-02-09
From Josh:

1. A dog and a rabbit are 160 meters a part for every 9 meters that the dog runs , the rabbit jumps 7 meters. How many meters must the dog run in order to vertake the rabbit???

2. A cars odometer reads 15951 KM. Find the least number of kilomaters to travel until the palindromic number appears???


Answered by Leeanne Boehm.
What is wrong with these probelms? 2001-02-09
From Cindy:
Hi, I am a planning on becomming a teacher and i am asked to find out what is wrong with these probelms and how i would go about showing a student what is wrong with them!!

Error in patterns:
13/35=1/5; 27/73=2/3; 16/64=1/4

4/5+2/3=6/8; 2/5+3/4=5/9; 7/8+1/3=8/11

2/3*3=6/9; 1/4*6=6/24; 4/5*2=8/10


Answered by Leeanne Boehm and Penny Nom.
The odds of winning 2001-02-09
From Anna:
The odds of winning in a game are 3 out of 10. How many times would one have to play the game in order to be sure they would win?
Answered by Leeanne Boehm.
Kinds of lines 2001-02-09
From Robyn:
What are the different kinds/types of lines?
Answered by Leeanne Boehm.
An irregular polygon 2001-02-09
From Jason:
I have a 5 sided irregular polygon I am trying to figure out the area of. There are no right angles in the polygon as far as I can tell. I do not know any angles.
Answered by Chris Fisher.
Partitioning of an arbitrary line segment 2001-02-08
From David:
Did Euclid's Geometry include a construction for the regular partitioning of an arbitrary line segment?
Answered by Chris Fisher.
Bicycles and phone calls 2001-02-08
From Sarah:
  1. A bicycle has a diameter of 66 centimeters. How many times must the tire rotate to travel 1 kilometer?explain answer.

  2. Becky want to make a long distance call to her friend Sarah from a pay telephone.She has $5.00 in change.The call costs $0.90 for the first three minutes and $0.24 for each additional minute.How long can Becky talk to Sarah?

Answered by Leeanne Boehm.
Buckets of golf balls 2001-02-08
From A student:
A bucket containing 40 golf balls weighs 135 ounces. The same bucket with 20 golf balls weighs 75 ounces. What is the weight of the bucket?
Answered by Claude Tardif.
Find an exprression for f(x) 2001-02-07
From A 12th grade AP Calc student:
Let f be the function defined for all x > -5 and having the following properties. Find an expression for f(x).

i) f ''(x) = 1/ (x+5)1/3 for all x in the domain of f

ii) the line tangent to the graph of f at (4,2) has an angle of inclination of 45 degress.


Answered by Harley Weston.
The "goesinta" box 2001-02-07
From Bridget:
We are looking for the "proper" name of the "box" that is drawn around the dividend on a division problem:

  ___ 3)12 

It has always been a family joke to call it the "goesinta" box, but a search of several reference sources hasn't yielded an answer --

This is a question of curiosity rather than a question for homework.


Answered by Claude Tardif.
(-2b+3)(-b-1) 2001-02-06
From Melissa:
Alright, lets say you are multiplying (-2b+3)(-b-1) and you are using "FOIL" first you would get : 2b2+2b-3b-3 then you add like-terms. when multiplying,if the bases are the same, add the exponents...does the same thing apply when you are adding the results of the multiplication even though it's addition?

would the answer be 2b3-3b-3 or would it be 2b2-b-3?


Answered by Penny Nom.
23 + 17 * 2 / 8 + 11 = 13 2001-02-02
From David:
My child is working through basic operations, and gave this problem:

23 + 17 * 2 / 8 + 11 = 13

The teacher wants the parenthesis to be put in the right places to come up with 13.
Answered by Claude Tardif.

Which fraction is greater? 2001-02-01
From Carol:
Hello My name is Carol I am a student teacher! I am asked to aproach these students on their problems. I am unsure of how to explain to them that these are wrong. Can you help? Thanks.

Iris claims that if we have two positive rational numbers, the one with the greatest numerator is the greatest.

Shirly claim that if we have two positive rational numbers, the one with the greatest denominator is the least. Thanks!


Answered by Claude Tardif.
The solutions of a quadratic 2001-01-30
From A student:
  1. Show that the product of the solutions of ax2 + bx + c = 0 is c/a.

  2. For each equation under the given condition, (a) find k and
    (b) find the other solution.
    kx2 - 2x + k = 0; one solution is -3

Answered by Penny Nom.
Why is 0! = 1? 2001-01-30
From Diane:
Every math book always claims that 1!=1 and 0!=1 are givens, and that we should just memorize it. i understand the 1! part, but where is the basis for claiming that 0!=1????
Answered by Walter Whiteley.
Unscramble these words 2001-01-30
From Myosha:
Hi my name is Myosha. I would be part of the secondary(6~9) class. I need you to unscramble these words & define them for me.

  1. edicnreeff

  2. rtgeare

  3. itfrnoca

  4. lmicdae

  5. sdvioini

  6. imrretepe


Answered by Claude Tardif.
Two number theory questions 2001-01-30
From Richie:
i have two questions and they are:
  1. the product of three consecutive prime numbers is 7429. Find the product of the smallest and largest of these three integers.

  2. the least common multiple of two relatively prime positive integers, a and b, is 144. if a (a>or=2) is as small as possible, compute the value of b.

Answered by Penny Nom.
Subsets of the natural numbers 2001-01-30
From Christina:
How do I explain why the set of natural numbers (N) cannot be equivalent to one of its finite subsets?
Answered by Penny Nom.
Fractions of fractions 2001-01-30
From Katie:
When you multiply two positive fractions less than 1, how does the answer compare to the size of the fraction? Why?
Answered by Penny Nom.
Crossing the dessert 2001-01-29
From Zach:
An Explorer needed to make a 5 day trip across a dessert. Each man with him could carry exactly 3 days food and water. For one man how many men will it take to cross the dessert?
Answered by Claude Tardif.
Equilateral 2001-01-25
From A student:
I AM 9 YEARS OLD AND I HAVE HOMEWORK. I WOULD LIKE TO KNOW IF THERE IS SUCH A THING AS A EQUILATERAL SQUARE.
Answered by Penny Nom.
Rotation 2001-01-25
From A student:
If I have a rectangle 2inches long and 4 inches wide and I rotate the rectangle 45 degrees will the length still be 2 inches?
Answered by Penny Nom.
Conversions 2001-01-24
From Tanya:
I have 2 questions
  1. covert 628 kilometers to miles.(round off your answer to two decimal places.)

  2. a new clerk in your office is to earn $200 per week. If she works 30.5 hours each week, what is her hourly rate? (round off your answer to the nearest hundreth, that is, to the nearest cent.)

Answered by Penny Nom.
Building a circular silo 2001-01-24
From Natasha:
We wish to build a circular silo with internal diameter 10 feet. How much concrete will we need to pour the foundation, if we only need a 1 foot wide and 1 foot deep ring on which the silo walls will sit? Assume the 4 inch thick silo wall rests on the middle of the ring.
Answered by Penny Nom.
Factoring a trinomial 2001-01-24
From Kyle:
I am an eighth grade student.I can not figure out the following problem. 4x2 + 2x - 20
Answered by Leeanne Boehm.
Cincinnati to Winston-Salem 2001-01-23
From Belinda:
Joan Bedney travels from Cincinnati to Winston-Salem at an average rate of 40 miles per hour and returns at an average rate of 60 miles per hour. What is her average speed for the trip?
Answered by Leeanne Boehm and Claude Tardif.
1 + 1 = 1 2001-01-23
From Stephanie:
My friend has this as a bonus question the other day and I want to figure it out. I don't know how 1+1 in any form could equal 1. Please let me know how you come about geting that.
Answered by Claude Tardif.
7x6 by doubling 2001-01-23
From A student:
Can you think of a way to use doubling to multiply 6x7?
Answered by Allen Herman.
Some algebra 2001-01-23
From Thomas:
If a is 4 more than the quotient of b and 3, and if b=2, then what is the value of a?
Answered by Penny Nom.
The sum of the angles in a polygon 2001-01-23
From A student:
What is the sum of the measures (interior angles) in an octagon ... heptagon ... decagon.
Answered by Penny Nom.
The hypotenuse of a right triangle 2001-01-22
From Phillipe:
How do you find the hypotenuse of a right triangle?
Answered by Penny Nom.
Conditional probabiity 2001-01-22
From Wallace Yang:
A pair of six-sided fair dice is thrown. Find the probability that the sum is 10 or greater if it is given that a 5 appears on at least one of the dice.
Answered by Penny Nom and Claude Tardif.
Square feet and cubic feet 2001-01-21
From Jenny:
How do I convert square feet to cubic feet? If I have 2000 square feet then how many cubic feet do I have?
Answered by Harley Weston.
Proportionality: direct and indirect 2001-01-18
From Nancy Ellis:
What is the difference between directly proportional and indirectly proportional?
Answered by Leeanne Boehm.
More repeating decimals 2001-01-17
From Alan:
I am neither a math teacher nor a student, but I hope you will consider my question anyway. I recently was discussing repeated decimals with a friend, and went on the web to find out more about a pattern was looking for years ago. In doing so I came upon your correspondence on repeating decimals.
.
.
.

Answered by Chris Fisher and Penny Nom.
Motivators 2001-01-16
From Michelle Stapley:
Do you know of any (or where I can find) student motivators for math at the secondary level? Basically any way I can motivate my students to WANT to learn math.
Answered by Penny Nom.
A problem with distance 2001-01-16
From A student:
For what values of a is the distance between P(a, 3) and Q(5, 2a) greater than the square root of 26.
Answered by Penny Nom.
Some complex problems 2001-01-15
From Nick:
I am having enormous difficulty with one question in my maths homework. The question is shown below. If anybody out there can find the answers and show the workings and help me to understand.
Answered by Harley Weston.
Polynomials and exponents 2001-01-15
From A student:
I am duing a project in math on polynomials and exponents. I need a real life usage of polynomials and exponents for my project.
Answered by Penny Nom.
The path of a cannonball 2001-01-15
From A student:
Artillerymen on a hillside are trying to hit a target behind a mountain on the other side of a river. Their cannon is at (x, y) = (3, 250) where x is in kilometers and y is in meters. The target is at (x, y) = (-2, 50). In order to avoid hitting the mountain on the other side of the river, the projectile from the cannon must go through the point (x, y) = (-1, 410).

Write the equation for the problem.


Answered by Penny Nom.
Vitamins A and B 2001-01-14
From Sara:
A diet is to include at least 140 mg of Vitamin A and at least 145 mg of vitamin B. these requirements are to be obtained from two types of food. type X contains 10 mg of vitamin A and 20 mg of vitamin B per pound. Type Y contains 30 mg of vitamin A and 15 mg of vitamin B per pound. if type X food costs $12 and Type Y $8 per pound, how many pounds of each type of food should be purchased to satisfy the requirements at the minimum cost?
Answered by Claude Tardif and Harley Weston.
Completing the square 2001-01-12
From Jacky Lam:
A friend told me that completing square not only can find the vertex but also the roots of a quadratic equation. Is it possible? How? Can you show me how to find the roots of the following equation using completing the square?

8x2 + 8x + 4


Answered by Penny Nom.
Blood flow and age 2001-01-11
From Gayle:
I don't understand the second part of this problem Here is the first part that was correct. A 35yr old has 100% blood flow. A 65yr old has 64% blood flow. Find the rate of change per yr in the percent of blood flow. (This part is correct) m=(64-100)/(35-65)= -36/30= -1.2 per year of blood flow.

Now, here is the second part I DON'T UNDERSTAND! .Let b represent the percent of blood flow and a represent a person's age. Write an equation that you could use to predict the percent of blood flow for a person of any age over 35.


Answered by Penny Nom.
Place value chart 2001-01-11
From Charles:
Can you send me a pictorial example of a place value chart?
Answered by Harley Weston.
8, 4, 2, 18, 1, 9 2001-01-11
From Neyra:
Place each number below in one of the blanks to create the most meaningful and realistic story possible.
Answered by Claude Tardif.
A 1 from 35 lottery 2001-01-11
From Jamie:
A number is randomly drawn from an urn containing 35 balls numbered 00 to 34.

How to win

Division 1 Match the number drawn
Division 2 Match the last digit of the number drawn

Calculation of probabilities for this lottery:...


Answered by Claude Tardif and Penny Nom.
Is n^2 - 2 a multiple of n - 4? 2001-01-10
From John:
Find all positive integers n so that n2 - 2 is a multiple of n - 4.
Answered by Sukanta Pati.
Four crayons 2001-01-10
From Neyra Espinoza:
You have four crayons (red, blue, yellow, green). If you line them up, how many different combinations can you get?
Answered by Patrick Maidorn.
The Pythagorean Theorem 2001-01-08
From Megan:
Why the Pythagorean Theorem so important in our lives and what is it's history?
Answered by Penny Nom.
Multiplication of fractions 2001-01-08
From Angela:
Questions:

1. represent multiplication of fractions via an area model

2. describe why, not just how, to round decimals


Answered by Penny Nom.
Women mathematicians 2001-01-07
From Kisha:
Can you please tell me names of at least five female math teachers who changed history back then?
Answered by Penny Nom.
Derfs, Enajs and Sivads 2001-01-07
From John and Norman:
All Derfs are Enajs. One-third of all Enajs are Derfs. Half of all Sivads are Enajs. One Sivad is a Derf. Eight Sivads are Enajs. The number of Enajs is 90. How many Enajs are neither Derf nor Sivad?
Answered by Penny Nom.
A dollar, quarter, dime, nickle and penny 2001-01-07
From Sarah:
Arnold has a dollar coin, one dime, one quarter, one nickel, and a penny. The number of different sums of money that can be formed using three coins is...
Answered by Penny Nom.
The pythagorean theorem in everyday life 2001-01-06
From Josh:
What are some ways that we use the pythagorean theorem in jobs, or even in everyday life?
Answered by Claude tardif.
The laws of sines and cosines 2001-01-02
From Faydene:
Can the sine /cosine rule be applied to a right -angle triangle to find a particular solution or are these 'rules' applied only when the triangle is not right angled?
Answered by Penny Nom.
Filling a tank 2001-01-02
From Liting:
Water flows into a rectangle tank which measures 4 m by 3 m by 2.8 m through a cylindrical pipe of radius 2.4 cm. If the rate at which the water flows into the tank is 8 km/h, find how long it takes to fill the tank.
Answered by Penny Nom.
Polite numbers 2001-01-02
From A student:
A positive whole number, greater than 1, is said to be a polite number if it can be expressed as a sum of consecutive whole numbers

Find all the polite numbers less than 21. Find a pattern in the numbers that are not polite. Explain why every odd number greater than one must be polite.


Answered by Penny Nom.
A quarter-circle and two semi-circles 2000-12-31
From Christopher:
Inside the quarter-circle are two semi-circles with the same radius, (r). Which has a greater area, G or L?
Answered by Penny Nom.
Height of the lamp 2000-12-31
From Joey:
The figure shows a lamp located three units to the right of the y-axis and a shadow created by the elliptical region x2 + 4y2 < 5. If the point (-5,0) is on the edge of the shadow, how far above the x-axis is the lamp located?
Answered by Harley Weston.
Repeating decimal notation 2000-12-31
From Clarende Duby:
I have seen single repeating decimals for ex. 1/3 = 0.3 with the dot above the 3 to represent the repeating decimal. Then, there is the more common form of the bar over the top of the repeating number or group of numbers (called the period?). Which notation is correct?
Answered by Penny Nom.
Bush fractals 2000-12-30
From Anita Wisecup:
My son has a report due on fractals. He needs information on bush fractals, but we cannot seem to find anything out about them.
Answered by Penny Nom.
Solving x - sin(x) = constant 2000-12-29
From Keith Roble:
If x is in radians, how do you solve for x, where: x-sin(x) = constant?
Answered by Harley Weston.
Solving an equation 2000-12-29
From Bob:
Would you be able to help me solve this math equation..

Solve for b, 2 [b - (q + s)] / q = s


Answered by Penny Nom.
A lampost and its shadow 2000-12-24
From Laura:
A lamppost line EC casts a shadow line AC. A 30 cm ruler line DB has been moved from A so that it's shadow falls just within the shadow of the lamppost.
  1. Suppose the length of the ruler's shadow is 42 cm. What is the slope of the imaginary line AE?

  2. Suppose the lamppost's shadow is 15 m. long. How tall is the lamppost?

Answered by Penny Nom.
How do you integrate secant(theta)? 2000-12-22
From Robert Williamson:
How do you integrate secant(theta)?

I know the answer is ln [sec(theta) + tan(theta)] but how do you get there?


Answered by Claude tardif.
When did the man go out? 2000-12-14
From Rajesh:
A man goes out in time between 5 and 6 and when he comes back he observes that two hands have interchanged position. Find when the man did go out?
Answered by Penny Nom.
Three problems 2000-12-14
From Sarah:
Hi, my name is Sarah, and I'm asking this question on behalf of niece who is in secondary two. Thank you.

Q1. What is the angle formed by the hour hand and the minute hand of a 12 hour clock at 5.50pm.

.
.
.

Answered by Penny Nom.
What is the difference between Mean and Average? 2000-12-13
From Julie:
What is the difference between Mean and Average? My thinking is the average, is the equal to the sum of all numbers divided by the number of numbers added together. But the mean, I think should be calculated by adding the largest and smallest numbers in the set and them dividing by 2. (which is the point where 1/2 the numbers are higher and 1/2 the numbers are lower)
Answered by Patrick Maidorn and Penny Nom.
A limit using l'hopital's rule 2000-12-13
From Wassim:
I need to know how to solve the:
limits of (x ( to the power lamda) -1 )/LAMDA when lamda tends to zero ( the answer is that the functional form is ln x) and I still don't know how using hopital rule leads to this answer.

Answered by Harley Weston.
Divisibility by 16 2000-12-12
From Shiling:
A number can be divided by 16 if and only if its 1st four digits can be divided by 16. How can you prove that?
Answered by Penny Nom.
A chemist had 8 flasks 2000-12-10
From Jimmy:
A chemist had 8 flasks capable of holding 12, 15, 27, 35, 37, 40, 53 and 69 fluid ounces respectively. He filled some with water and then filled all the rest except one with alcohol. He used exactly one and a half times as much alcohol as water. Which flask was left empty and which were left with water and which with alcohol?
Answered by Claude Tardif.
Volume of a trough 2000-12-09
From Matt:
I have been given the information of the length of the trough, and the end pieces are isosceles triangles, and two equal sides of the triangle are given. I cannot seem to find the answer, nor anything in my book relating to the question.
Answered by Harley Weston.
Mixed Fractions 2000-12-09
From Tim Finch:
My son is having problems with mixed decimals and mixed fractions.. I am unable to find any information or examples on this subject so I can show him how to do it.. for example how do you make a mixed decimal or fraction out of 7/16
Answered by Penny Nom.
Is this a right triangle? 2000-12-08
From Alicia:
How would you set-up and answer a problem like these one? Triangle ABC has vertices A(-2,2), B(1,-2), and C (1,2). Use slopes to determine if the triangle is a right triangle.
Answered by Penny Nom.
Networks of satellites and linear spaces 2000-12-08
From David:
Let us suppose some companies have collaborated to place several satellites in orbit. Let us call the set of all satellites that a given company helped place in orbit a network. Finally let us assume the following 4 rules.
  1. There are at least two distinct satellites.


  2. For each pair of satellites there is exactly one network containing them.


  3. Each network contains at least two distinct satellites.


  4. For each network, there is a satellite not in it.

What is the least number of satellites.

what is the least number of networks?


Answered by Penny Nom.
Kai's bike 2000-12-07
From Fumiko Muhammad:
The wheels on Kai's bike are 27 inches in diameter. His little sister, Masako, has a bike with wheels that are 20 inches in diameter.

a. How far will Kai go in one complete turn of his wheels?

b. How far will Masacko go in one complete turn of her wheels?

c. How far will Kai go in 500 turns of his wheels?

d. How far will Masako go in 500 turns of her wheels?


Answered by Penny Nom.
Pizza delivery 2000-12-07
From Meghan:
3 men give 10 dollars each to a pizza deliver driver and when he gets back to his work the manager says to the pizza driver the bill was actually 25 dollar so he has to returns 5 dollars. on his way to the mens house he the driver decides to kept 2 dollars for himself and he gives 1 dollar back to each of the men. So each one of the men actually paid 9 dollars. but 9 times 3 is 27 plus the other 2 that the driver took equals 29 so where did the other dollar go.
Answered by Harley Weston.
Optical illusions 2000-12-06
From Jessica:
Hi, my name is Jessica, 7th grade, and Im doing a Math Fair project on optical illusions. As one of the required factors, we need a "mathmatical significance" paragraph. Unfortunatley, I can only think of one way that optical illusions have to do with math, and thats time because some optical illusions tell you to look at the picture for a certain amount of time.
Answered by Penny Nom.
< and > Which one is which? 2000-12-06
From Alice:
This grandmother forgot and wants to know the correct for greater and the one for lesser.....

the two are < and > Which one is which?


Answered by Penny Nom.
Scheduling a golf vacation 2000-12-05
From Michael:
I'm having a problem scheduling matches for a golf vacation. We have 12 people playing 7 rounds of golf in 7 days. We play 2 man teams vs. 2 man teams everyday.Is there a formula so that you play WITH a different partner everyday and AGAINST as many different people as possible? Thank You for any help you may be.
Answered by Penny Nom.
The aspect ratio of a rectangle 2000-12-04
From Ron Delavigne:
The aspect ratio of this rectangle is 4:3. That is A to B is 3. And B to C is 4. If I know the lenght of A to C is 19 inches, how can I find the length of A to B, and B to C.
Answered by Penny Nom.
Parabolas in life 2000-12-03
From Ashley:
I am a student and my teacher recently gave us the assignment of writing a portfolio on parabolas in life and finding examples, three to be in fact, only we have to go into detail about only one. We have been instructed to include such terms as: axis of symmetry, completing the square, parabola, quadratic formula, standard form (vertex form) and vertex. We also must include in our detailed example an equation of the parabola and very specific details, PLEASE HELP!
Answered by Harley Weston.
Equivalent fractions 2000-12-03
From A student:
please explain to us how to find the answer to this problem, we need three fractions that name the same amount as 2
Answered by Penny Nom.
A non-integrable function 2000-12-03
From Mark Spilker:
I have a proof that I cannot do here it goes.
Let F(x)= 1 if x is a rational number 0 if x is an irrational number Prove the function is not intregrable on the interval (0,1). Hint: Show that no matter how small the norm of the partition, the Riemann Sum for the SigmaNotation F(wi) DeltaiX is not unique.

Answered by Harley Weston.
Triangles and trigonometry 2000-11-30
From Mose:
If I have a right triangle, and I know the lengths of all three sides, is there a formula that will allow me to determine the measurements of the 2 non right angles?
Answered by Harley Weston.
An integration problem 2000-11-30
From A student:
If a>0 and the integral from b to 0 of 1/(1+x) equals 1/2 the integral from a to 0 of 1/(1+x), express "b" in terms of "a"
Answered by Claude tardif.
The astronomers' calendar 2000-11-29
From Show_Li:
The calendar that astronomers use began on Jan. 1, 4713 B.C. on that day, the julian calendar, the lunar calendar, and roman tax system calendar all coincided. This won't happen again until 3267 A.D.
(a). How many years ago was the astronomer's calendar started?
(b). Find the number of years between times that the three calendars coincided.

Answered by Penny Nom.
Vector Problem 2000-11-27
From Ben:
An aircraft can fly 260km/h in still air and the wind is blowing at 70km/h towards the West. In what direction should the aircraft head so that its actual velocity is on a bearing of 030 degrees?
Answered by Harley Weston.
Order of operations 2000-11-26
From Margaret Pratt:
My daughter has a math question and I am afraid I am of no help. Can you help? 2x5/2+1-5= She arrives at 8 as the answer and has been told this is incorrect. Any help you can give would be appreciated.
Answered by Penny Nom.
A complex calculation 2000-11-24
From Angie:
Multiply (3-2i)2=32-2(3)(2i)+(2i)2
Answered by Penny Nom.
A piecewise linear equation 2000-11-24
From Jacky:
There is a light bulb and it is given that the light bulb cost $0.75 and the cost of operating it is $0.0081 per hour. From the information give, I came up with the linear equation: Let c be total cost and Let h be hours used. Therefore: c = 0.0081h + 0.75 represents the total cost of the light bulb and the electricity. However, the second part of the question added the fact that the light bulb will only last for 800 hours. If the light bulb is replaced as soon as it burns out exactly after every 800 hours, how can I write an equation that represents that? Is it possible? What would it look like on the graph.
Answered by Harley Weston.
What are adjacent angles that equal 360 called? 2000-11-22
From David:
I know that supplementary angles add to 180 degrees and that commplementary angles add up to be 90 degrees, but what are adjacent angles that equal 360 degrees called?
Answered by Chris Fisher.
Comparing an integral and a sum 2000-11-21
From Douglas Norberg:
A fellow teacher asked me about a problem she wanted to give to her students. It involved whether to take a million dollars or a penny doubled a number of times. I was able to determine the number must have been .01 * 230 which is about $10 million and a lot more than $1 million. To check that I was right I used a spreadsheet and did a Riemann sum.

When I finished I reasoned that I had done the task in several steps and I could have done it in 1 step. Thus I integrated .01 * 2x from 0 through 30 but the number I got was $15,490,820.0324. Why the difference?


Answered by Harley Weston.
Integral solutions 2000-11-20
From David:
determine all pairs of integers (x,y) which satisfy the equation

6x2 - 3xy - 13x + 5y = -11

I got

y= (6x2-13x+11)/(3x-5)

and i don't konw how to continue


Answered by Claude Tardif.
Bridges and parabolas 2000-11-18
From Lauren:
My name is Lauren, and Im a secondary school student in Ontario. For my gr11 advanced math class I have to find out how and why parabolics are used in arch bridges and write 3 paragraphs on it. People who cohse satelites and whatnot are lucky - I've found a ton of info, but for arch bridges there seems to be nothing.
Answered by Harley Weston.
Find the 3D angle 2000-11-18
From Jacky:
Included is the diagram. I am trying to find out the angle of ABC. Is it possible? How?
Answered by Penny Nom.
Inscribing a circle in a rhombus 2000-11-16
From Jacky:
A rhombus ABCD is drawn in which the diagonals are 12 and 20 units long. A circle is inscribed in the quadrilateral with the centre of the circle right on the intersection point of the 2 diagonals. The circumference of the circle touches all 4 sides of the rhombus. Is it possible to find the radius of the inscribed circle? If so, how and what is it?
Answered by Chris Fisher.
Volume of a box 2000-11-15
From A student:
A box 3" wide and 2" deep is to have a volume of 2 cubic feet. How long must the box be?

How many pieces of lumber are stacked in a pile 2" high if each board is 3/8" thick?


Answered by Penny Nom.
Dividing with decimals 2000-11-15
From Alex:
I teach math and my class and I were discussing the dividing of decimals. I explained that if there is a decimal in the divisor, it needs to be moved and so does the decimal in the dividend. My question is why is it necessary to move the decimal point in the divisor before dividing?
Answered by Penny Nom.
Henri Leon Lebesgue 2000-11-14
From Mandy:
I have a report that I have to do on Henri Leon Lebesgue for my Foundations of College Math class. I have searched for him on erols, google, yahoo and a few other search sites and I have only come up with one page that offered some information. I was wondering if anyone knew anything about him, or a web page that I could get some info.
Answered by Harley Weston.
Solving an equation 2000-11-14
From Heather:
4x-3 = 3x+4 these kinds of problems i dont get but everything else in my math class i have got. i just dont understand it.
Answered by Penny Nom.
Five digit palindromes 2000-11-12
From Jacky:
Positive integers such as 1287821 and 4554, in which the number is unchanged when the digits are reversed, are called palindromes. The number of five-digit integers larger than or equal to 10,000 which are not palindromes is... a. 10 000 b. 81 000 c. 89100 d. 90 000 e. 99 100
Answered by Penny Mom.
Static electricity on an airplane 2000-11-12
From John:
What discharges an airplane of static electricity in a electric storm when it is flying. I assume the charge is on the out side of the plane but when does it get discharged, or does it occur at landing?
Answered by Ted Mathie.
Euler's Formula 2000-11-11
From Denise Roberts:
I'm trying to design a unit (possibly involving a Webquest) on Polyhedra and I cannot find a formula I once used. The formula involved the number of vertices (V), edges (E), and faces (F) of the polyhedra.
Answered by Walter Whiteley.
The magnitude of a complex number 2000-11-11
From Jeremy:
Recently, we started studying how to graph complex numbers. Our math teacher said to use what would normally be the x-axis as the real-axis and to use the y-axis as the imaginary-axis. However, when he started talking about how to calculate magnitude, that's when I became confused. For instance...
Answered by Walter Whiteley.
How big is a trillion? 2000-11-07
From A teacher:
How can you explain how much a trillion is to a 5th grade class?
Answered by Penny Nom.
How many tens are in 400? 2000-11-07
From A student:
How many tens are in 400?
Answered by Penny Nom.
Crown molding mitre cuts 2000-11-06
From Jim Tomfohrde:
My question has to do with making mitre cuts when installing crown molding. Crown molding is the trim that is put up at the top of walls with one edge on the wall and the other edge on the ceiling. To make a mitre cut on your mitre saw for a 90-degree corner you can lay the molding flat on the saw base, set the bevel of the blade to 34 degrees and the mitre to 31.5 degrees (these may be slightly appoximate). Of course depending on which piece of molding you're cutting you will cut one end or the other, or use the left or right end. These angles allow the cuts to line up and form a seamless corner when they're put in place on the wall/ceiling at 90 degrees. My question is this - is there some mathematical formula from which the 34 degrees and 31.5 degrees are derived. I want to know this because in many cases the corner is not 90 degrees but can be more or less, and in these instances I would like to know if I can calculate the bevel and mitre to use based on the angle of the corner.
Answered by Harley Weston.
Graphing F(x) = 3x 2000-11-06
From Jose:
graph the exponential problem F(x)=3x
Answered by Harley Weston.
Banana yogurt 2000-11-03
From James:
A grocery store has 100 cartons of banana yogurt in stock.Each carton contains 12 cup of banana yogurts.The probability that a cup has fewer than 20 banana chunks in it is 10 %. So,What is the probability that between 15 and 25 (inclusive) cartons out of the 100 cartons have exactly 3 cups with fewer than 20 banana chunks?
Answered by Harley Weston.
Expected number 2000-11-02
From James:
1)There are 40,25,50 students in the traditional,reformed,and reformed traditional Calculas sections taught by three teachers. On one fine day the three sections congregate together to watch the NOVA video tape on Fermat's Last Theorem.

a) A student is randomly selected from he crowed of students.What is the expected number of students in the same section of this student?

****this question which i did not have any idea to do as follow for the part b
b) If one of the three teachers is selected at random, what is the expected number of students taught by is teacher?


Answered by Harley Weston.
Length of a shadow 2000-11-01
From Jessy:
A man who is six feet tall is walking away from a street light that is fifteen feet tall. How long is the man's shadow when he is ten feet away from the light?
Answered by Penny Nom.
Lobbing a tennis ball 2000-10-31
From Alex:
If a tennis ball is lobbed into the air with an initial upward velocity of 12 meters per second, its velocity and height will be functions of time in flight described by the following rules:

V=12-9.8T and H=1+12T-4.9T 2

Find the maximum height of the tennis ball and the time it takes to reach that height.


Answered by Penny Nom.
False Positives 2000-10-29
From James:
A rare disease infected 1 in 1000 people in the population. A test for the disease is accurate 99% of the time when given to an infected person and also when given to a heathy person.
  1. Fill out a two-tier tree diagram and find the probability of the false positive(i.e the conditional probabily of being healty even when tested postive by the test) Comment on the result?

  2. ...

Answered by Penny Nom.
Square roots 2000-10-29
From Pamela:
HERE GOES(I WILL USE Q AS THE SYMBOL FOR SQUARE ROOT):

8(Q2) - 5(Q2) + Q2

SECOND PROBELM IS

(1 + Q2)2

LAST ONE

3/(2-Q5)


Answered by Claude Tardif.
Overlapping a circle and a square 2000-10-28
From Jacky:
A square with a dimension 20 by 20cm. and a quarter of the circle with the radius of 25cm (A quater of a circle is created by 2 cuts that are perpendicular bisectors of each other where the intersecting point is at the centre of the circle). With these 2 pieces, the 2 pieces are placed over each other in which the 90o angle of the quarter circle matches with one of the right angles on the square. Now, calculate the overlapping area of the 2 figures.
Answered by Chris Fisher and Harley Weston.
Find the rule 2000-10-27
From Kelsa and Linda:
Find the rule: It had 13 in a circle then 2 blank circles then 31 in a circle then two more blank circles. Could you please help us.
Answered by Penny Nom.
Developing a community garden 2000-10-26
From adam:
Robin's class is developing a community garden. There are 15 students in Robin's class. They volunteer to work in a community garden at various times during the school week. Robin's teacher says it will take about 3 weeks to complete the project if 3 students work 1 hour each day. Some students can work on more than one day. The following table lists the number of students who can work on various days.
.
.
.

Answered by Penny Nom.
Cutting the lawn 2000-10-26
From Kacie:
Ellen can mow a lawn in 2 hours. Mary can mow the same lawn in 1.5 hours. How long would it take them to mow the lawn together?
Answered by Penny Nom.
The ages of three daughters 2000-10-25
From Andrea:
An encyclopedia saleman call at a home. The woman who answers the door says she will buy something from him if he can give the ages of her three children. The first clue, she says, is the three ages multiplied together equal 36. he responds that he needs more infomration, so she says that the threee ages add up to the number of the bus that passed by (the prof did not give us this number) He thinks for a while and says he needs one more clue. So she says, my youngest child has red hair, and he is able to answer and make the sale. what are the ages of the three children? (hint: the salesman needed all three clues to get the answer).
Answered by Penny Nom.
Assessment and portfolios relating to math 2000-10-24
From Jackie:
I will be assisting in a presentation for grades 4-6 on assessment and portfolios relating to math. If anyone has some ideas will love to hear them. I have checked out a few sites on the net
Answered by Diane Hanson.
Divisibility by 9 2000-10-24
From Kelera:
If the sum of the digits of a number is divisible by 9, then the number itself it divisible by 9. Why is that? How do you explain this?
Answered by Penny Nom.
Working together on a job 2000-10-23
From Nicole:
WORKING TOGETHER ON A JOB: Patrice, by himself can paint 4 rooms in 10 hours. If he hires April to help they can do the same job together in 6 hours. If he lets April work alone , how long will it take her to paint 4 rooms?
Answered by Claude Tardif.
Concavity 2000-10-22
From Alex:
the question is: on what interval is f(x)=(x2)(ex)?

ive found the 2nd derivative which is ex(x2+4x+2) and i did the quadratic to get -2-20.5 and -2+20.5, but i dont know what the interval is.
Answered by Harley Weston.

Tapes and CD's 2000-10-22
From Arnold:
Nicole has 25 CDs and tapes altogether. When she tries to pair them up, she has 4 CDs left over. How many tapes does Nicole have?
Answered by Penny Nom.
The scale on a map 2000-10-21
From Caroline:
On a map scale of 1:15000 the distance between two buildings is 31.4cm what is the actual distance in km.
Answered by Penny Nom.
Connecting to a water line 2000-10-20
From Vanja:
My question is...A house is to be connected to a new water main that runs along the line y=2/3x-1. The connection point at the house has coordinates (2,9), where the units represent metres. What lenght of plastic pipe is needed to connect to the water main at the closest point?
Answered by Penny Nom.
The perimeter of a football field 2000-10-18
From Paige:
The width of a football field is 67ft greater than the length. The perimeter is 480ft What is the width and length?
Answered by Penny Nom.
Dividing fractions 2000-10-18
From Paula:
Why do you have to change the division sign to a multiplication sign and invert the fraction that follows the division sign in order to get the answer to a division problem when you're working with fractions?
Answered by Penny Nom.
Transporting bananas 2000-10-18
From Krystie:
A farm has 45 bananas, a man has to take a truck and transport 15 bananas to a market that is 15 miles away. Every mile he travels, he must eat a banana. I have to get at least one banana to the market
Answered by Penny Nom.
A chord length 2000-10-17
From Al Paas:
How to determine the length of a chord given the diameter of the circle and the maximum distance from the chord to The circle?
Answered by Chris Fisher.
Multiplying decimals 2000-10-16
From Lily:
I need help with mutiplying decimals (example) 2.28*0.48( * = times).
Answered by Harley Weston.
Area of shapes 2000-10-14
From Melanie:
What is the formula to calculate the area of different shapes?
Answered by Harley Weston.
2-step equations 2000-10-11
From Jen:
i don't get how you find the answer to these equations

140= -4 -12e

x/-2 + 6 = 4


Answered by Penny Nom.
Where will we use this in the real world? 2000-10-11
From Jane Ann Musgrove:
As a teacher of mathematics, I am always asked "Where will we use this in the real world?". I am seeking ideas/sites via the internet where students can find answers to this type of question. Can you help me?

To be more specific, right now I am interested in finding careers where the employees would use the concepts of "Radicals", "Matrices", and "Logarithms". This information will be used by students to make presentations to the class on their findings from internet searches.


Answered by Harley Weston.
Larger and smaller 2000-10-10
From Nicole:
which one of these arrows < , > points to the greater number? which arrows points to the smaller number.
Answered by Penny Nom.
Hendecagon 2000-10-09
From Jillian:
I need an example of a object that is in the shape of a hendecagon. I know what the shape is but I cannot come up with a real life example of an object that is this shape.
Answered by Chris Fisher.
Sunflower seeds 2000-10-09
From A parent:
Find the number of 3/4 pound of sunflower seeds that can be filled with 225 pounds of sunflower seeds
Answered by Penny Nom.
Independent tests 2000-10-07
From A student:
If the false-positive rate of each test in a battery of tests is 0.05, how many independent tests can be included in the battery if we want the probability of obtaining at least one false-positive result to be at most 0.2?
Answered by Harley Weston.
Repeating decimals 2000-10-06
From Mary O'Sullivan:
I understand that with repeating decimals (those with a pattern), the number of digits repeated is put into fraction form with the same number of 9's

ex. 0.4444 = 4/9 0.145145145 = 145/999

How can I explain why this is so? Some inquisitive 8th graders are anxious to find out!
Answered by Penny Nom and Walter Whiteley.

Cutting the cake 2000-10-05
From Cherie:
How can you divide a cake (circle) into 14 pieces by using only 4 straight lines?
Answered by Penny Nom.
Pillows and Cushions 2000-09-27
From Fiona:

The following problem was given to grade eleven algebra students as a homework assignment. To manufacture cushions and pillows, a firm uses two machines A and B. The time required on each machine is shown. Machine A is available for one full shift of 9.6 hours. Machine B is available for parts of two shifts for a total of 10.5 hours each day.
Answered by Harley Weston.

Isolating an exponent 2000-09-24
From C. Scott:
Suppose you invest $500.00 in an account that pays 10% interest compounded annually. How long will it take for this value to triple?

A=p(1 + i)a A=1500 i=0.1
1500=500(1.1)a p=500 a=unknown

How do you solve this problem algebraicly? How do you isolate the variable (a) when it is an exponent?


Answered by Harley Weston.
Maclaurin series again 2000-09-23
From Jason Rasmussen:
I suppose my confusion comes into play when I am trying to figure out where the xn term comes from. I know that the Power Series notation is directly related to the Geometric Series of the form sigma [ brn ] where the limit is b/(1-r) for convergence at | r | <1. Therefore, the function f(x) needs to somehow take the form of b/(1-(x-a)), which may take some manipulation, and by setting r = (x-a) and b = Cn, we get the Geometric Series converted to the Power Series. Taking the nth order derivative of the Power Series gives Cn = fn(a)/n!. There must be a gap in my knowledge somewhere because I cannot seem to make f(x) = ex take the form of f(x) = b/(1-(x-a)). Maybe I should have labelled my question as "middle" because it may be more of a personal problem with algebra and logarithms. Or, am I to assume that all functions can be represented by sigma [fn(a) * (x-a)n / n!]?
Answered by Harley Weston.
Mode 2000-09-22
From James Barton:
I have always been told that a mode is the "one" number that appears most in the set of numbers: ex.{1,3,4,6,3,2} the mode is 3. What if you have {1,1,3,4,5,5}is there a mode. I was taught long ago that there is no mode, Not i am having to teach there is two modes. 1 and 5. If this is the case if we have {1,1,2,2,3,3,4,4,5,5} that every number is the mode. True or false. This is being ambigiuous if we say all are the mode. Because no one number is used more than the others.
Answered by Claue Tardif and Harley Weston.
Permutations and Multiplication Principle 2000-09-22
From Candice:
A forester selects 4 pink and 4 white dogwoods. The trees are to be planted in row. If a tree is distinguished by color only, in how many ways can the eight dogwoods be planted? How many of these arrangements have at least two trees of the same color side by side?
Answered by Denis Hanson and Claude Tardif.
Dividing a region in half 2000-09-21
From Kerry:
There is a line through the origin that divides the region bounded by the parabola y=x-x2 and the x-axis into two regions with equal area. What is the slope of the line?
Answered by Penny Nom.
A Maclaurin series 2000-09-21
From Jason Rasmussen:
I have a question regarding power series notation for certain functions within the interval of convergence.
Answered by Harley Weston.
Leap Year 2000-09-20
From Jennifer:
The earth revolves around the sun in 365.24 days. How does our calendar deals with the extra 0.24 day?
Answered by Penny Nom.
A cycloid in Cartesian form 2000-09-20
From Billy:
The parametric equation of cycloid is given:
x=r(t-sint)
y=r(1-cost)

How to eliminate t?


Answered by Harley Weston.
A proof that 1=2 2000-09-19
From sporky:
Why does the proof for 1=2 not work?

x = 1
x2 = 1
x = x2
1 = 2x (derivitive)
1 = 2(1)
1 = 2 ???

please tell me where the false logic is.


Answered by Walter Whiteley.
Circular Permutations 2000-09-18
From J. Stuart:
I am able to understand the simple theory behind circular permutations (that is, there are (n-1)! ways to arrange n people in a circle. One question, however, is giving me some difficulty. It reads: In how many ways can four married couples be arranged around a circular table if each man must sit beside his wife? Any suggestions you have would be greatly appreaciated.
Answered by Penny Nom.
Buying art supplies 2000-09-18
From Travis:
An art teacher buys exactly 100 items at this sale, and the total cost before tax is $100. She buys at least one of each item. How many of each item does she buy?

Markers $0.50
Sketch Pads $3.00
Paint Sets $10.00


Answered by Denis Hanson.
Probability of a square 2000-09-18
From Nick:
What is the probability that four different points chosen at random from the fifteen equally-spaced points shown are the vertices of a square? I need to have my answer expressed as a common fraction. Help! I don't know how to write it.

 . . . . . . . . . . . . . . . 

Answered by Penny Nom.
Review problems 2000-09-18
From Dee:
A card is selected from an ordinary deck of cards. What is the probability of. . . selecting 2 aces in a row?

If you toss two dice, what would be your probability of the following?
.
.
.


Answered by Penny Nom.
Mr. Edgecomb's math exam 2000-09-17
From Rachel:
In Mr. Edgecomb's math class, 30 students took an exam on statistics. If the average passing grade was 84, the average failing grade was 60, and the overall average was 80, how many students passed the test?
Answered by Penny Nom.
Tessellations 2000-09-17
From Lindsay:
What is the word that means a shape repeated over and over again to make something like a quilt pattern?

Note: I'm pretty sure it is either a fractal or tesselation. Could it be that the pattern itself is a fractal but the entire quilt would be a tesselation?


Answered by Chris Fisher.
6-49 2000-09-14
From Steve:
In our state lottery you must choose 6 numbers (1-49). How many different combinations are there? They can be in any order.
Answered by Harley Weston.
Rounding 2000-09-14
From MaryAnn:
How many whole numbers rounded to the hundreds place, round to 400? Can you also explain why?
Answered by Penny Nom.
Two linear equations 2000-09-14
From David Dean:
2a + 1b = 3.39 3a + 3b = 6.59
What formula do I use to find what a = ?

Answered by Harley Weston.
The smaller of a and b 2000-09-14
From Jenna:
For any two real numbers, a and b, give a mathematical expression in terms of a and b that will yield the smaller of the two numbers. Your expression should work regardless of whether a>b, a
Answered by Penny Nom.
Rational Numbers 2000-09-14
From Josh Kuhar:
How can you tell a number is rational?
Answered by Harley Weston.
1 + 2 + 3 + ... + 50 2000-09-14
From Vicki Charron:
How can you calculate the total of the numbers one through fifty, without adding up the individual numbers?
Answered by Penny Nom.
7 - - - - 77 2000-09-13
From Peter:
Does anyone know how to solve the following:
7 _ _ _ _ 77 ?

I have to find the missing values.


Answered by Chris Fisher and Walter Whiteley.
Manipulating powers 2000-09-13
From Phil:
21/5 x 42/5 = ?
Answered by Penny Nom.
Flim 2000-09-12
From Jodi and Ari:
Can you tell us the definition of the word FLIM? My daughter is in 4th grade, and this came up. Could not find it in the Websters Dictionary.
Answered by Chris Fisher.
Selling chickens 2000-09-12
From Rick:
A man raised chickens. He sold all but 8 of them. The first buyer bought 1/3 of all the chickens + 2/3 of a single chicken. The second buyer bought 1/3 of the remaining chickens + 1/3 of a single chicken. The third buyer bought 1/3 of the remainder + 2/3 of a chicken. How many chickens did the seller own before selling any? All chickens were alive and whole when the buyers received them.
Answered by Penny Nom.
Logs and trig functions 2000-09-12
From Becky:
How do they get these answers?
  1. log 8 + log 2 = ?
    Answer is: log 16

  2. For 0 degree < x < 90 degree, how many solutions are there for the equation 2sin x = cos x?
    Answer: 1


Answered by Penny Nom.
Two geometry problems 2000-09-09
From Becky:
What are all the real values of x that are solutions for the inequality [x-2] < 6? ( it's less than or equal to)

What is the distance between the points with (x,y) coordinates (3,-2) and (-3,-1)?


Answered by Peny Nom.
Induction 2000-09-07
From Joe Peterson:
How do I prove by the principal of mathematical induction?
1.n+2.(n-1)+3.(n-2)+.....+(n-2).3+(n-1).2+n.1=(n(n+1)(n+2))/6

Answered by Paul Betts.
Roots of x^2 + 22 x + 112 2000-09-07
From Brian R.:
I need to find the root of the quadratic equation of a square. The problem is x2 + 22x + 112 = 0.
Answered by Penny Nom.
Trinomial 2000-09-06
From Lea:
I need to find the trinomial value of k which makes a perfect square. The problem is x2 - 6x + k
Answered by Penny Nom.
Derivatives, there must be an easier way 2000-09-06
From Brad Goorman:
The direction read: Take the derivative of each expression.

y = {1+[x+(x2 +x3)4]5}6


Answered by Harley Weston.
Euclidean distance 2000-09-06
From Velma Zahirovic-Herbert:
If d(x,y) is euclidean distance between x and y Prove that
d(x,y)>=0
if d(x,y)=0 than x=y
and d(x,y)=d(y,x)

Answered by Paul Betts.
Gravity 2000-09-06
From Robert Broomfield:
If they dropped a 1lb object from 40,000 feet in the air, at what speed would it it the ground?
Answered by Harley Weston.
Making milkshakes 2000-09-04
From Linda:
To make a chocolate milkshake, 3 parts chocolate syrup are mixed with 2 parts malt. To make a vanilla milkshake, 2 parts vanilla syrup are mixed with 1 part malt. If equal parts chocolate and vanilla milkshakes are mixed, what is the proportion of malt to milkshake?
Answered by Penny Nom.
A equation with a square root 2000-09-03
From Lea:
Please help with 2 plus the square root ( and under it has x plus 1) = 6
Thank you

Answered by Penny Nom.
Inequalities 2000-09-02
From Jennifer:
I would like some help with equations and inequalities. My first inequality is 3p-6>21 and 3q+11+8q>99.
Answered by Paul Betts.
A number problem 2000-09-02
From Lalita Balu:
Can you arrange 1234567890 to add up to 100?
Answered by Penny Nom.
Trigonometry 2000-09-02
From david:
determine the sum of the angles A,B where 0 <= A , B <= 180 (degrees)

sinA + sinB = sqr(3/2) , cosA + cosB = sqr(1/2)


Answered by Chris Fisher.
How many planes contain a line and a point? 2000-08-30
From Harold:
How many planes contain each line and point?
Answered by Harley Weston.
Adriana's earnings 2000-08-28
From Adriana:
On Friday Adriana earns half as much as she did on Thursday. On Saturday she earns twice as much as she did on Thursday, if her total for 3 days is $63.00 how much did she earn each day?
Answered by Penny Nom.
Velocity of a pendulum 2000-08-28
From Mekca:
A pendulum hangs from the ceiling. as the pendulum swings, its distance,d cm, form one wall of the room depends on the number of seconds,t, since it was set in motion. assume that the equation for d as a function of t is: d=80+30cos3.14/3t, t>0.

estimate the instantaneous rate of change of d at t=5 by finding the average rates for t=5 to 5.1, t=5 to 5.01, and t=5 to 5.001.


Answered by Harley Weston.
Solving an equation 2000-08-26
From Patrick:
My question is how would u do a problem like this..........
[2x-37] =15?....u have to solve it for absolute value equations....????

Answered by Harley Weston.
Subtracting fractions 2000-08-26
From Sarah Stanczyk:
how is this problem solved??

3/ x+1 - 5/x
( 3 divided by x +1 subtracted by 5 divided by x)


Answered by Penny Nom.
Three points on a line 2000-08-25
From Casey:
I am trying to find the slope and y-intercept of an equation. I have THREE x values and THREE y values. How do you do it? Help!
Answered by Penny Nom.
Three pattern questions 2000-08-22
From Michael Rodriguez:
I have 3 pattern questions that I need help answering, please.
  1. 6 and 7 = 42
    2 and 5 = 10
    4 and 8 = 8
    5 and 9 = ?

  2. 7 and 5 = 1
    9 and 12 = 3
    10 and 15 = 5
    12 and 16 = ?

  3. 1st = 1
    3rd = DEF
    10th = *
    6th = ?


Answered by Claude Tardif.
Slope 2000-08-19
From John K. DePonte:
My question is to why the variable "m" was chosen to represent the slope of a linear equation. I always told my class to think of "m" for "m"ountain slope. But what is the real reason for this?
Answered by Penny Nom.
Six typists 2000-08-19
From Paula Duncan:
If it takes a typist 4.5 hours to type 860 words, how many hours would it take 6 typist to do 16800 words?
Answered by Penny Nom.
Three consecutive odd integers 2000-08-18
From Wallace:
A six-digit integer XYXYXY, where X and Y are digits is equal to five times the product of three consecutive odd integers. Determine these three odd integers.
Answered by Paul Betts.
Two word problems 2000-08-16
From Bridget:
A whale's head is 72 inches long... its tail is as long as its head plus half the length of its body, and its body is half the entire length. How long is the whale?

Also, An author published a book every two years. When the seventh one was published, the sum of the years of publication was 13,804. In what year was the author forst published?


Answered by Penny Nom.
Some trigonometry 2000-08-11
From Angela:
I have some PreCal questions. I am a student at the secondary level. I would be very grateful for your help.

Solve the equation for theta (0 <= theta < 2pi).

tan2(theta) = 3 I know sec2(theta) -1 = tan2(theta)

.
.
.
Answered by Harley Weston.

PreCalculus 2000-08-09
From Angela:
Use absolute values to define the interval or pair of intervals on the real line.

 
<--|--- |--- ]--- |--- |--- |--- [---| ---|-->    
   18  19  20  21  22  23  24  25  26    

A car is moving at the rate of 50 miles per hour, and the diameter of its wheels is 2.5 feet.
a) Find the number of revolutions per minute that the wheels are rotating.
b) Find the angular speed of the wheels in radians per minute.


Answered by Harley Weston.
The angle of rotation 2000-08-03
From Jay:
I have the following information Given.

(X1, Y1) Origional Point
(X2, Y2) Origional Point After a Rotation
(Xa, Xb) Center of Rotation

What formula would I use to figure out the angle the point was rotated?


Answered by Chris Fisher and Harley Weston.
Area of a circle 2000-08-03
From Larry:
I know the formula is-- pi r squared--. For a circle 4 inches in diameter, do I multiply pi (3.1416) by the radius (2") then square the answer to that ie: 3.1416 X 2 squared or do I square the radius (2 X 2") then multiply by pi (3.1416) ??
Answered by Penny Nom.
Graphing an inequality 2000-08-02
From Lori:
How do I find x and y and graph this problem

3x + y < 5


Answered by Penny Nom.
Two linear systems 2000-08-01
From A student:
please help me with this homework question. 5/x + 6/y= 19/6 3/x + 4/y =2
hint: let a = 1/x and b = 1/y substitute these expressions into the system to find a and b. Then find x and y.

This is a question I answered wrong on a test: solve the given system for x, y, and z. Express your solutions in terms of a, b, and c.


Answered by Penny Nom.
The circumference of a circle 2000-07-30
From Not a student:
An equalateral triangle is enclosed in a circle. The three corners touch the edges of the circle. One side of the triangle is 12. What is the circumference of the circle?
Answered by Penny Nom.
A semi-circle and a triangle 2000-07-28
From Ben:
A semi-circle and an isosceles triangle ABC have the same base AB and the same area. The equal angles in the triangle are BAC and CAB. I have to find the value of each of these angles.
Answered by Harley Weston.
LOG(LN(x)) = 1 2000-07-28
From An algebra student:
LET F(x)=LOG X AND G(x)= LN X. SOLVE (f *G)(x)= 1
SHOW COMPOSITION AND USE DEFINITION OF LOGS.

Answered by Harley Weston.
(mn + m + n)/(m^2 + n^2) = 1 2000-07-24
From William:
Find all pair of positive integers m and n such that mn + m + n divides m2 + n2 = 1.
Answered by Peny Nom.
An equation involving x to the x 2000-07-22
From Joy Peter:
I am joy, a teacher teaching Maths at the secondary level and while solving a sum came to this stage when i got xx (1 + log x) = 0, by which we can conclude that xx = 0 or 1 + log x = 0. If xx = 0, than what should be the value of x? I feel that the value of x should then be 0 (zero) but then how do I explain this to the students as we also know that anything to the power of 0 is 1 but here 0 raised to 0 is 1. If this is not defined then how do I explain this?
Answered by Penny Nom.
n! = 42(n-2)! 2000-07-21
From Damon Bailey:
Solve n! = 42(n-2)!
Answered by Paul Betts.
10,000! 2000-07-21
From Lauren:
Hi I was just wondering if you could tell me how many zeros are in 100,000! (factorial.)
Answered by Denis Hanson.
L'Hospital's Rule 2000-07-19
From Dan Krymkowski:
The limit of the following as x goes to infinity is 2*y. Y is a constant.

lim 2*x*log(x/(x-y)) = 2*y


Answered by Harley Weston.
Completing a square 2000-07-10
From Lexa Michaels:
How do you complete a square? the textbook question is: x 4 + 64
Answered by Penny Nom.
Word addition 2000-07-05
From Harman Chaudhry:
What is the value of DISHES if SEE + THE + WAITER + WASHES = DISHES. In addition, I know there is a name for these types of problems--what are they actually called?
Answered by Chris Fisher.
Simplify 21+2[3z+5(3z+8)] 2000-07-02
From Dennis Dyer:
My daughter has a math problem that I can't help her with. The problem is 21+2[3z+5(3z+8)] I can get the answer 101 +36z but I can't show her the correct way to write it out. Would you please show the correct way to show her work...
Answered by Penny Nom.
A trig question 2000-07-01
From Will:
An open rectangular tank a units deep and b units wide holds water and is tilted so that the base BC makes an angle theta with the horizontal. When BC is returned to the horizontal, who that the depth of the water is

(a squared) * cot theta div 2b units


Answered by Harley Weston.
Parallel tangents 2000-06-30
From Ebony Indalecio:
I need to prove the theroem: Tangents to a circle at the end points of a diameter are parallel.
Answered by Walter Whiteley.
Tiling a floor 2000-06-27
From Carolyn Clarkston:
A square tile measures 6 inches by 6 inches. What is the least number of tiles needed to cover a rectangular floor area of 9 feet by 12 feet?
Answered by Walter Whiteley.
Calculator and telephones keypads phones 2000-06-27
From Thomas Smith and Veronica Yates-Riley:
On a keyboard, why is the number "7" on the top of the keypad as opposed to the number "1" at the top a telephone keypad?
Answered by Penny Nom.
Central Limit Theorem and Law of Large Numbers 2000-06-26
From Jonathan Yam:
The Central limit Theorem states that when sample size tends to infinity, the sample mean will be normally distributed. The Law of Large Number states that when sample size tends to infinity, the sample mean equals to population mean. Is the two statements contradictory?
Answered by Paul Betts and Harley Weston.
A bar over a Roman numeral 2000-06-21
From Charles Fontaine:
Is there any significance to the bar placed over some Roman numerals?
Answered by Penny Nom.
Cairo tesselation and Archimedean duals 2000-06-21
From Joyce DuVall:
I am looking for a picture of the Cairo tesselation, and pictures of the Archimedean duals. Do you know of any good web sites or books?
Answered by Penny Nom.
Lines on an oscilloscope screen 2000-06-20
From Janis Watson:
What are the proper names for the vertical and horizontal lines on a graph? I am trying to explain an oscilloscope screen, and I don't know what to call these, except "vertical lines" and "horizontal lines."
Answered by Penny Nom.
The speed of light 2000-06-19
From Benny Lau:
I have a personal questions. In physics,if a person is running on a train, his relative speed on the ground will be the addition of the speed of train and the person. If u have a train inside another train and inside another train, so on...... can u reach the speed of light? It would need a very long distance. But if we make all the train travel in circles. Then we don't need that much distance. So can we achieve the speed of Light?
Answered by Robert Coleman and Harley Weston.
Numerator and denominator 2000-06-18
From Maureen Beard:
What is the origin of the terms numerator and denominater?
Answered by Penny Nom.
Names for large numbers 2000-06-16
From Mireille Carthier:
I am trying to find the common names (in English and French) for the big numbers as follows:
English French
106 million million
109 billion milliard?
1012 trillion billion?
1015 ?? trillion?

Thank you very much


Answered by Claude Tardif and Harley Weston.
Simplifing 2000-06-15
From Angie Herbert:
My daughter is having problems with her algebra homework. She is a year 7 pupil at high school in the UK. She has been given homework on simplifying formulae and she doesn't understand how to do them. Here is an example of one of the questions :

9s+7t+5c+5s-3t

can you help her to understand how to do these and perhaps me too.
Answered by Penny Nom and Claude Tardif.

Rectangles and algebra 2000-06-13
From Kirstin:
A rectangle's length is 4 more than twice its width. The area of the rectangle is 336m squared. What is its length?
Answered by Paul Betts and Penny Nom.
Two problems 2000-06-12
From Sharon:
If f(4)=0 and f(6)=6, which of the following could represent f (x)?

A. 2/3x-4 B. x+2 C. x-4 D. 3/2x+6 E. 3x-12

these are problems to study for a test so I need to know the answer and how it was solved!

I have one more question

If 180o < theta < 270o and tan theta = 4/3, then sin theta =?

A. 5/4 B. -4/5 C. 3/5 D. 12/5 E. -3/5


Answered by Harley Weston.
Projecting a line segment onto a plane 2000-06-08
From Monica:
What is the measure of the angle determined by a 14 inch segment and its projection into a plane if the length,in inches, of the projection into the plane is 7 inches?
Answered by Penny Nom.
Surface area of a contact lens 2000-06-06
From Evie Contreras:
I would like to know how to calculate the surface area of a contact lens with a radius of 7mm? I know that the area of a circle is pi R squared, but a contact lens has a dome.
Answered by Harley Weston.
Divisors of 2000 2000-06-06
From Amanda Semi :
  1. find the product of all the divisors of 2000
  2. dog trainer time has 100m of fencing to enclose a rectangular exercise yard. One side of the yard can include all or part of one side of his building. iff the side of his building is 30 m, determine the maximum area he can enclose

Answered by Claude Tardif.
Problems 2000-06-06
From Debbie Cummins:
I am a Mum of a 12 yr.boy needing help with some math problems. I need not only the answers but how it is worked out.
  1. Both the leftmost digit & the rightmost digit of a 4 digit number N are equal to 1. When these digits are removed, the 2 digit number thus obtained is N div by 21 Find N.
  2. Find all 3 digit even numbers N such that 693xN is a perfect square, that is, 693x N = k x k where k is an integer.
.
.
.

Answered by Paul Betts and Claude Tardif.
Writing in Math 2000-06-05
From Laurel:
How can I incorporate writing into my math curriculum?
Answered by Penny Nom.
A derivative problem 2000-06-04
From Jeff Ellis:
If F(x)=(4+x)(3+2x2)2(2+3x3)3, find F'(0)
Answered by Harley Weston.
Pi to 5 decimal places 2000-06-03
From Emily:
The question is find the value for pi to 5 decimal places????
Answered by Penny Nom.
A centroid problem 2000-06-02
From Kerstin:
An isoceles triangle has sides measuring 13 cm, 13 cm, and 10 cm. Find the distance from the centroid to the vertex of a base angle.
Answered by Harley Weston.
4 Color map Theorem 2000-06-02
From Rick Slen:
I am challenging my students to find a map that can not be coloured using only 4 colours. I know that it is not possible, but one boy drew a map with a "country" totally surrounding all the others so of course it touched all other colours. How do I explain that this type of map is not permissable?
Answered by Denis Hanson.
1+4+9+16+...n^2 = n(n+1)(2n+1)/6 2000-06-01
From Shamus O'Toole:
How do you derive that for the series 1+4+9+16+25.. that

S(n)=(n(n+1)(2n+1))/6


Answered by Penny Nom.
Using the inverse sine function 2000-05-31
From Nelson Rothermel:
This has me completely baffled. I have to use the laws of sine or cosine to find the angles of a triangle when I have 3 sides, so I can't go 180-x-y when I have 2 angles. Now, I have a triangle with values of 3, 7, and 9. Here are the steps I used (A,B,C are angles; a,b,c are opposite sides):

angle A (16.1951 degrees): cos-1*((b2+c2-a2)/(2*b*c))
angle B (40.6011 degrees): sin-1*(b*sin(A)/a)
angle C (56.7962 degrees): sin-1*(c*sin(A)/a)

If you notice, A+B+C does not equal 180. According to the book, A and B are correct, but C is supposed to be 123.2038 degrees. Why doesn't it work???


Answered by Harley Weston.
Powers of i 2000-05-24
From Paul Fieldhouse :
What is the result of raising i to the googol power? is there a rule or pattern to raising i by increasing powers of 10?
Answered by Penny Nom.
Why does division start from the left? 2000-05-24
From Salil Dave:
Addition, subtraction and multiplications start with right most digit and proceed left, but division starts from left-most digits and goes right ... why?
Answered by Harley Weston.
A 100-gon 2000-05-24
From Chris Mohr:
What is a 100-gon called?
Answered by Harley Weston.
Angular velocity 2000-05-24
From Ashley Milliman:
The crankshaft of a particular automobile engine has an angular velocity of 1,500 rpm at 30 mph. The crankshaft pulley has a diameter of 10 cm, and it's attached to an air conditioner compressor pulley with a 7 cm diameter and an alternator pulley with a 5 cm diamter

At what angular velocities do the compressor and alternator turn?

.
.
.

Answered by Harley Weston.
The equation of a parabola 2000-05-22
From Ian Forsyth:
Given the points A(0,0) B(60,10) C(24,d) find the equation of the parabola. leave the equation in terms of x, y and d if the general form of a quadratic is y = ax2 + bx + c.
Answered by Penny Nom.
Calculus Research Questions 2000-05-22
From William Wright:
I am a Calculus Teacher, and me and my class ran into these two problems without solutions in my manual, we got answers, but are unable to check them. If anyone gets this email and can respond to this with the solutions it be greatly appreciated.
.
.
.

Answered by Harley Weston.
Adjacent Angles 2000-05-21
From Katherine Keys :
Can a straight angle be an adjacent angle to another angle?
Answered by Chris Fisher.
Volume of a sphere 2000-05-21
From Kevin Partridge:
Does anyone have a way to physically demonstrate how to explain the volume formula for a sphere? Or perhaps how to derive the formula without calculus?
Answered by Harley Weston.
The pythagorean theorem 2000-05-19
From Lauren Fitzgerald:
how do you find the length of th hipotnuse( or however you spell that word). i understand you have to add the two sides. but when i do add i always end up with this way off answer. i donot understand at all.
Answered by Paul Betts.
The square root of -1 2000-05-19
From Gary:
i am not a student i am just some one that heard something and i can't be sure on the answer...my ? is what is the square root of -1? i think it is -1 but not sure can you let me know please thank you
Answered by Harley Weston.
A conic in standard form 2000-05-18
From Tara McConkey:
Im havign trouble converting the following conic to standard form, i know that the conic is a hyperbola but that is all 16x2-9y2-160x-18y+247=0
Answered by Harley Weston.
Radioactive decay 2000-05-18
From Catherine Sullivan:
Please help me with the following: The radioactive isotope carbon-14 is present in small quantities in all life forms, and it is constantly replenished until the organism dies, after which it decays to carbon-12 at a rate proportional to the amount of C-14 present, with a half life of 5730 years. Suppose C(t) is the amount of C-14 at time t.
  1. Find the value of the constant k in the differential equation: C'=-kC
  2. In 1988 3 teams of scientists found that the Shroud of Turin, which was reputed to be the burial cloth of Jesus, contained 91% of the amount of C-14 contained in freshly made cloth of the same material. How old is the Shroud according to the data?

Answered by Harley Weston.
Graphing a linear function 2000-05-17
From Chelsea:
I need help with grahing linear functions.If you could e-mail me back the basics and how tos I would be much appriciative.
Answered by Penny Nom.
Six letter words 2000-05-15
From Karl Freitag:
An anthroplogist discovers an isolated tribe whose written alphabet contains only six letters (call the letters A,B,C,D,E, and F). The tribe has a taboo against using the same letter twice in the same word. It is never done. If each different sequence of letters constitutes a different word in the language, what is the maximum number of six-letter words that the language can employ?
Answered by Penny Nom.
Multiplying fractions 2000-05-14
From A student:
75/2 x 1/100 = 3/8

Please show how to came up with the answer?
Answered by Penny Nom.

A matrix equation 2000-05-14
From A student:
Right now, we are dealing with matrices and we are supposed to solve the following problem on our graphing-calculators: 2a+3b-4c+d=20
a-2b+3c-5d=-14
3a+4b-2c+3d=19
5a-b+6c+4d=-5

Answered by Penny Nom.
Subtracting polynomials 2000-05-13
From Teresa:
I am suppose to subtract the lower polynomial from the one above it.
-3a+11b+2f
-3a- 6b+8f
________________

Answered by Penny Nom.
4x - [2x -(3x - 2)] = 13 2000-05-13
From Teresa:
I need to find the following equation.
4x - [2x -(3x - 2)] = 13

Answered by Penny Nom.
Reflection off a sphere 2000-05-12
From Mark Adami:
Given two points P,E and a sphere. Find a point on the sphere, T, so that a line from P that bounces off the sphere at T goes through point E.
Answered by Chris Fisher and Harley Weston.
Odds and combinations 2000-05-10
From Lucy:
If the odds are 1/15 of winning, what is the probability?

if c 5! 3! How does it equal 10?


Answered by Harley Weston.
The Village of Truth 2000-05-10
From Sean Pearson:
Our teacher gave us a logic problem involving two villages: the village of truth, and the village of lies. The problem is in asking two guards the right question to gain entrance to the fenced compound the villages are in without being shot. Have you heard of this problem ?
Answered by Harley Weston.
Trig functions 2000-05-09
From Melissa:
Find all solutions in the interval (0,2pi)
2cos2x-3cosx-4=0

Answered by Paul Betts and Harley Weston.
Supplementary angles 2000-05-09
From Suzanne:
We know that: Supplementary angles are two angles whose sum equals 180 degrees and complementary angles are two angles whose sum equals 90 degrees. Are supplementary and complementary angles necessarily adjacent? or can they be non-adjacent?
Answered by Chris Fisher.
Parallel and perpendicular lines 2000-05-08
From Jimmy Lumpkins:
Need a method for solving the following problem: Find the equation of a line through point P that is parallel or perpendicular to another line.
Answered by Paul Betts.
Related Rates 2000-05-07
From Derek:
How can you show that if the volume of a balloon is decreasing at a rate proportional to its surface area, the radius of the balloon is shrinking at a constant rate.
Answered by Harley Weston.
Probability and odds 2000-05-05
From Leah:
The chances of rain on friday, saturday, and sunday are 20%, 30%, and 50% respectively. what are the odds it will rain all three days?
Answered by Harley Weston.
Finding a formula 2000-05-05
From Erica Hildebrandt:
If a farmer has a field and his plots are laid out in the following grid where each # represents a plot:
4 5 12 13 20
3 6 11 14 19
2 7 10 15 18
1 8 9 16 17

Of course the plot numbers aren't meaningful as I have described above. In fact they may not be numbers at all. The only constants I have are the total number of rows and columns. Using the total number of rows and columns and my current position row and column, how can I write a formula that tells me column 3 row 3 = 10, column 4 row 2 = 14, etc. I can see the pattern but can't quite get the formula. I believe I will need 2 different formulas one for even and one for odd rows.
Answered by Paul Betts and Penny Nom.

An improper integral 2000-05-04
From A high school senior:
Hi, I am a high school senior and I need help stugying for a final. I am stuck on one of the questions on my review sheet. Does the improper integral from 5 to infinity of (38/97)x converge or diverge? If it converges I also need to know how to find the approximate value accurate to .01 of its actual value.
Answered by Harley Weston.
sin(7pi/12) 2000-05-04
From Kristel:
What is the exact value of sin 7pi/12?
Answered by Chris Fisher and Paul Betts.
Thearcius Functionius 2000-05-03
From Kevin Palmer:
With the Olympics fast approaching the networks are focusing in ona new and exciting runner from Greece. Thearcius Functionius has astounded the world with his speed. He has already established new world records in the 100 meter dash and looks to improve on those times at the 2000 Summer Olympics.

Thearcius Functionius stands a full 2 meters tall and the networks plan on placing a camera on the ground at some location after the finish line(in his lane) to film the history making run. The camera is set to film him from his knees(0.5 meters up from the ground) to 0.5 meters above his head at the instant he finishes the race. This is a total distance of two meters(the distance shown by the camera's lens).
Answered by Harley Weston.

An indefinite integral 2000-05-03
From Bonnie Null:
I am to find the indefinite integral of: (ex - e-x)2 dx
Answered by Claude Tardif.
Mary Fairfax Greig Somerville 2000-05-03
From Joshua Garrett:
I'm giving a report in class on her life. Could i recieve some sort of information about her. thank you
Answered by Claude Tardif.
Minimizing the metal in a can 2000-05-02
From May Thin Zar Han:
A can is to be made to hold 1 L of oil. Find the dimensions that will minimize the cost of the metal to manufacture the can.
Answered by Harley Weston.
Area of an ellipse 2000-05-02
From Kaushal Shah:
How do I Calculate the area of a ellipse known the length of any related thing. Example, suppose if I know the length of latus rectum, major&minor axis etc.
Answered by Walter Whiteley.
Two calculus problems 2000-05-01
From Kaushal Shah:
How Do WE Integrate the following Functions,
  1. Integral xtanx dx
  2. How was natural base "e" discovered and why e=2.7.......

Answered by Claude Tardif.
Fractals 2000-04-29
From Rachel Maginn:
What are Fractals? I am doing a research project. Any information would be great. I need to know the history behind fractals, and how they are used. I would like some examples like the snowflake and a fractal tree. Any other examples would be appriciated greatly.
Answered by Harley Weston.
Monica's geometry problem 2000-04-27
From Monica:
Given: ABCD is a square; AX is perpendicular to BY
Prove: Angle 1 is congruent to Angle 3

Answered by Chris Fisher.
Graphing cube roots 2000-04-27
From Heather Jones:
What do the graphs of cube roots look like?
Answered by Walter Whiteley.
Searching for socks 2000-04-26
From Luther Jackson:
There are 10 red socks and 10 blue socks in a drawer. Aside from their color the are identical. All of the socks in the drawer are mixed up. If the room was pitch black, How many socks would you need to pull out of the drawer to be certain that you have a pair that matches? Why?
Answered by Penny Nom.
Paying with silver 2000-04-26
From Luther Jackson:
A silver prospector is unable to pay his March rent in advance. He owned a bar of pure silver, 31 inches long, so he made the following arrangement with his landlady. He would cut the bar, he said into smaller pieces. On the first day of March he would give her and inch of the bar, and on each succeeding day he would add another inch to her amount of silver. She would keep this silver as security. At the end of the month, when the prospector expected to be able to pay his rent in full, she would return the pieces to him.
.
.
.

Answered by Claude Tardif and Penny Nom.
The weight of a watermelon 2000-04-25
From Lori Speir:
If I have a 100 pound watermelon and it is 99% water. If it sits out in the sun all day and is now only 98% water what does it weigh now?
Answered by Penny Nom.
An expected value 2000-04-24
From Carl Pride:
Suppose that in a statistics class of size 23, each student has a probability of passing of 73 percent.

sample:

What is the expected number of students who will pass??


Answered by Harley Weston.
Conversions 2000-04-23
From Tara:
750 lbs = ___ tons

How do I figure this out!? I know that 1 ton is 2000 lbs, how do I figure out this answer?
Answered by Penny Nom.

A colony of bacteria 2000-04-22
From Nicci Demmin:
Suppose that the bacteria in a colony can grow unchecked, by the law of exponential change. The colony starts with one bacterium and doubles every half hour. How many bacteria will the colony contain at the end of 50 hours?
Answered by Penny Nom.
Side length of an octagon 2000-04-22
From Unknown:
If I have an octagon that is 12 feet across (side to opposite side) how can I find the length of the sides?
Answered by Harley Weston.
Circumference = Area 2000-04-19
From Scot George:
The area and circumference of a circle has the same measurement. Find the radius.
Answered by Chris Fisher.
Factoring trinomials that are squared 2000-04-17
From A student:
I have a question that I really need an answer to. I need help with factoring trinomials that are squared.
Answered by Penny Nom.
Uniting algebra and geometry 2000-04-16
From Beth:
Who is the mathmatician that united algebra and geometry???
Answered by Claude Tardif.
Finding a Quadratic Equation given the roots 2000-04-16
From A secondary student:
How do I find the equation for the quadratic with roots at 2-i and its complex conjugate?
Answered by Harley Weston.
Sample variance 2000-04-16
From Jonathan Freeman:
I was just reading your article entitled "A Note on Standard Deviation" I'm now teaching a unit on s.d. and my students were wondering why one uses a denominator of n for a population and n-1 for a sample. I saw in your article that this is because "[the quantity] tends to underestimate sigma... and other technical reasons." To which my students again asked... "Why?" Could you please elaborate a bit on the "other technical reasons" perhaps in terms a high school senior (or their teacher...) could understand?
Answered by Harley Weston.
All the roots of x^6 - 64 2000-04-15
From Dakota:
Find ALL zeros of P(x) = x 6 - 64
Answered by Walter Whiteley.
The area of a triangle using calculus 2000-04-15
From Todd Bowie :
Hi, I am not a student but am reviewing calculus for an upcoming interview. I would like to know how to derive the area of a triangle using calculus. Thanks!
Answered by Patrick Maidorn.
Rounding 2000-04-15
From Tawana Brown:
I was uncertain as to whether a number such as 85.25 rounding to the nearest tenth would be 85.3 or 85.2. I thought I had heard somewhere that if 5 is the last number that you don't round up. But I have also heard that anything 5 and above you do round up. It's been awhile since I've done rounding, so I need a refresher course.
Answered by Walter Whiteley and Harley Weston.
Logs 2000-04-12
From Jessica:
Write as a single log 3ln x - 2ln(y+e) + 5ln(x-12) - 4ln y
Answered by Harley Weston.
Infinite sets 2000-04-12
From Brian Provost:
Here's the deal: There are an infinite amount of integers (1,2,3...). Agreed? There are an infinite amount of even integers, too (2,4,6...). Agreed? By convention, infinity equals infinity. Yet common sense tells us there are obviously more integers than there are even integers. Prove this to be true mathematically.
Answered by Harley Weston.
Natural numbers 2000-04-12
From Sara:
What is a natural number???
Answered by Penny Nom.
Magic triangles 2000-04-11
From Sandy:
My tutoring student brought math homework today in the form of a "magic triangle". There are three spaces along each side for missing numbers. The sums of the numbers along each of the 3 sides should be the same. Use the numbers 4 through 9. Don't use any number more than once. The sum of the numbers on each side should be 20. What is the logic behind solving a problem of this kind?
Answered by Claude Tardif and Harley Weston.
White and blue paint 2000-04-09
From Lauren Emerson :
A truck full of cans of blue and white paint flips over on the road. There are dots of blue and white paint everywhere. Prove that two dots of the same color paint are exactly pi feet apart.
Answered by Penny Nom.
2^32 + 1 2000-04-08
From Un Etudiant:
232 +1 can be divided by 641 , why?
Answered by Claude Tradif.
Star-multiplication 2000-04-07
From Greg Potts:
I have this question to answer and I don't know where to start. 1*9=0, 9*8=72 and 2*8=9, then 9*9 =?
Answered by Penny Nom.
Surface area of a cylinder 2000-04-07
From Jim Campbell:
I am going to cover the outside of a shaft with some material. How do I figure the square inches of the outside surface of the shaft? Shaft is 6" in diameter and 24" long
Answered by Harley Weston.
History of a meter 2000-04-07
From Jarod Fischer :
The length of a meter has been determined in three different ways. The first and original way was determined in 1798. The second way was determined in 1960. The third and current way was determined in 1983. What were the three ways and why were the first two abandoned?
Answered by Harley Weston.
Why are equivalent fractions equal? 2000-04-05
From An elementary student:
What is the mathematical reason equivalent fractions are equal?
Answered by Penny Nom.
y = x^x^x^x... 2000-04-05
From Michael Hackman:
Find the derivative of: y = x^x^x^x... on to infinity.
Answered by Claude Tardif.
Manipulatives 2000-04-04
From Jodi Stuker:
I am a student looking for a good lesson plan using manipulatives to teach multiplication to LD students. They are currently functioning on approximately 3rd grade level in math but are age 12-14.
Answered by Penny Nom.
The square root of 3 2000-04-04
From Mr. William:
Prove that root 3 is irrational
Answered by Harley Weston.
The side length ratios of some triangles 2000-04-04
From Alexis Lockwood:
I am doing a project for my Math 30B class regarding the side length ratios of 45-45-90 degree and 30-60-90 degree triangles. I would really appreciate any assistance in answering the following questions, or even direction to an appropriate web site or resource on the matter.
Answered by Harley Weston.
Interest compounded daily 2000-04-03
From Debbie Bracken:
How do I figure interest that is compounded daily? 8% compounded daily.
Answered by Penny nom.
Why a Right angle? 2000-04-03
From Joseph Mizerek:
I was wondering why a 90 degree angle is called a Right angle. I mean why isn't called a left angle.
Answered by Haragauri Gupta.
Running Through a Train Tunnel 2000-04-01
From Eugene Chan:
A man is running through a train tunnel. When he is 2/3 of the way through, he hears a train that is approaching the tunnel from behind him at a speed of 60 mph. Whether he runs ahead or back, he will reach an end of the tunnel at the same time the train reaches that end. At what rate, in miles per hour, is he running? (Assume he runs at a constant rate.)

I think the answer (12 mph) is wrong. Also, I believe it should read 1/3 of the way through, but don't know how to prove it. Could you come up with some way to prove it, please? I would really appreciate it.


Answered by Penny Nom.
Filling a hole 2000-04-01
From John McNeill:
I have an area 20 feet long and 12 feet wide It starts out at a depth of 4 inches and ends up at 0 inches. How much sand do I need to fill the excavation.
Answered by Penny Nom.
Careers after a math degree 2000-03-31
From Jeanne Hyer:
What type of careers can a person have with a math degree, and what is the closest thing that you have to an undergraduate degree in financial mathematics? (Administration, math, actuarial science, etc.)
Answered by Harley Weston.
Word problems 2000-03-31
From Amy:
Question:
  1. a north bound train left at noon. four hours later a south bound train left the same station. at 5 o'clook they were 160 miles apart. what was the rate of both if the north bound train was going 20 miles faster than the other one.
  2. clark's final exam was given 5 times the weight of the weight of a weekly test. the avgerage of 15 weekly test was 91. the overall weighted average was 85. what did he make on the final.

Answered by Penny Nom.
Hats, jackets and gloves 2000-03-30
From Max:
Find the one that does not belong 66 333 99 111 88

also if you have 3 hats and 3 jackets and 3 gloves, how many different combinations of a hat and a jacket and gloves can you wear?
Answered by Claude Tardif and Penny Nom.

Riemann sums 2000-03-30
From Joshua D. Parham:
If n is a positive integer, then
 
     lim   (1/n)[1/(1+1/n) + 1/(1+(2/n) + ... + 1/(1+n/n)] 
n->infinity 

can be expressed as the integral from 1 to 2 of 1/x dx
Answered by Penny Nom.

Making fruit salad 2000-03-29
From A grade 5 student:
Miss.Muffet males a fruit salad by adding 31/2 pounds of grapes and 4 1/4 pounds of apples to every 2 1/4 pounds of melon. How many pounds of grapes does she use in 50 pounds of fruit salad?
Answered by Claude Tardif.
Taxis in Chicago 2000-03-27
From A high school student high school student:
Suppose that taxis pick up and deliver passengers in Chicago, which is divided into three zones. Records kept by the drivers show that of the passengers picked up in Zone 1, 50% are taken to a destination in Zone 1, 40% to Zone 2, and 10% to Zone 3. OF the passengers picked up in Zone 2, 40% go to Zone 1, 30% to Zone 2, and 30% to Zone 3. Of the passengers picked up in Zone 3, 20% go to Zone 1, 60% to Zone 2 and 20% to Zone 3. Suppose that at the beginning of the day, 600 of the taxis are in Zone 1, 100 in Zone 2, and 300 in Zone 3. What is the distribution of taxis in the various zones after all have had two riders?
Answered by Harley Weston.
Divisibility by 3 2000-03-24
From Pat Walsh:
W hy does it work when you add the digits of a number then divid by three to see if the number is divisible by three
Answered by Penny Nom.
Increasing the weight 2000-03-23
From Lee Millard:
A weight of dimension a, b is to have its weight increased by w% by bolting plates of the same material along the top and one side (but not at both ends). The top plate and side plate have the thickness, x. Find the thickness of the plates for a given weight increase of w%.
Answered by Penny Nom.
Sylvester's Problem 2000-03-23
From Sheryl Webb:
I am trying to find a proof of the theorem: Given n points in a plane, there is a line that contains exactly two points.
Answered by Chris Fisher.
A parabola problem 2000-03-23
From Morin:
I need to prove that if parabola x2=4py has a chord (not necessarily a focal chord) intersecting it at points A and B, with tangents to the parabola at points A and B that intersect at C, then a line drawn through C and the midpoint of the chord M is parallel to the y-axis. Further, prove that the point D where this line intersects the parabola is the midpoint of line CM.
Answered by Penny Nom.
Reflex angles 2000-03-22
From D. Reed:
What is the name of an angle that exceeds 180 degrees?
Answered by Penny Nom.
Rule of 78 2000-03-22
From Dan Baumgarten:
Can you explain the rule of 78 and the reverse rule of 78? Thanks.
Answered by Claude Tardif.
Surface area of a sphere 2000-03-22
From Gina Wilkie:
How can I demonstrate to my middle school students the reason for the formula for the surface area of a sphere?
Answered by Walter Whiteley and Chris Fisher.
I am in lowest terms. 2000-03-21
From Travis Fox:
I am in lowest terms. When you double my numerator and my denominator the result is 10/15.
Answered by Penny Nom.
Compounding continuously 2000-03-21
From Gina:
You deposit $1500 in an account that pays 6.5% annual interest, compounded continuously. Find the balance after 10 years.

I'm not sure what to do with the "compounded continuously" part.


Answered by Penny Nom.
Binomial coefficients 2000-03-21
From Howard Lutz:
How do you find each successive numerical term in this equation y+dy=(x+dx)5 =x5+5*x4dx+10*x3(dx)2+10*x^2(dx)3+5*x(dx)4+(dx)5 I would appreciate an explanation of the method to find the numeric coefficient in a binomial expansion
Answered by Penny Nom.
A system of equations in five unknowns 2000-03-20
From Will:
I have been having some problem with the following question for some time. I would appreciate any help on solving the problem or a solution.

Q: Assume that a system of equations in the unknowns x1, x2, x3, x4 and x5 when converted to row echelon form gives

.
.
.

Answered by Penny Nom.
root(-1)* root(-1) 2000-03-20
From Michael Moran:
i squared = -1

but

i squared = root(-1)* root(-1)
= root( -1*-1)
= root(1)
= 1
-1 doesn't = 1

can you help me with my question


Answered by Claude Tardif.
Travelling from A to B 2000-03-20
From Matt:
A car is traveling from point A to point B. Point A and point B are 30 miles apart. A car travels 30 miles an hour from point A to point B and then goes back to point A. How fast does the car have to go back to average 60 miles an hour?
Answered by Claude Tardif.
Six digit numbers using 1,2,5,6,7, and 9 2000-03-20
From Rachel:
How many different six-digit numbers can you make using the digits 1,2,5,6,7, and 9? How many of these six digit numbers are divisible by six?
Answered by Claude Tardif and Denis Hanson.
A quantitative methods course 2000-03-19
From Mark Haberman:
Question:
Last semester, the grade distribution in a quantitative methods course had the following distribution: 10% A, 25% B, 35% C, 10% D, and 15% W (withdrew).
  1. If this grade distribution does not change this semester, what is the probability that a randomly selected student will make at least a D?
  2. If this grade distribution does not change this semester, what is the probability that a randomly selected student will fail the course?
  3. If this grade distribution does not change this semester, what is the probability that a randomly selected student who finished that course (did not withdraw) made a grade of D or better?

Answered by Harley Weston.
The Terror Run 2000-03-18
From Danny Mclean:
A fairground's most popular attraction is a roller coaster ride known as the Terror Run. One stretch of track is called the Missile Path and is in the form of a parabolic curve. B is 180m horizontally from A and the highest point of the curve is 100m above A and B.

A.. The owner works out a quadratic expression to describe the Missile Path. What is the expression He found?

B.. A safety Engineer examined the structure and observed that points A and B were likely to be damaged due to the steepness of the Missile path near these points. The owner can see no way to make the Missile path less steep near A and B and to keep the height of the ride the same. HOW CAN IT BE DONE?


Answered by Harley Weston.
Four questions 2000-03-17
From Ibrahim Bin kasmin:
  1. What is a hexahedron?(please show a picture of a hexahedron).
  2. How do we make a cube out of three pyramids?(show me the picture).
  3. How do we find the approximate perimeter and area of a hibiscus leaf?
  4. What is a Pascal triangle?

Answered by Penny Nom.
The Kennedy Pep Club 2000-03-17
From David Grimmer:
The Kennedy Pep Club rented a bus to take all of its members to a baseball game. The cost per person was to be $18.00. However, those making the trip had to pay $24.00 each because 10 members cancelled at the last minute. How many pep club members took the bus to the game?
Answered by Penny Nom.
Induction 2000-03-16
From William Tsang:
I am trying to prove a induction question

Sigam r=1 n (2r -1)cube = n square (2 n square - 1)


Answered by Harley Weston.
Functions that satisfy f' = f 2000-03-16
From Kevin Palmer :
Recently my calculus teacher asked his students to try and find any functions whose derivatives where the exact same as the original function.

The only function then I have determined that statement to be accurate in is all the natural exponential functions. Ex. f(x) = ex, f'(x) = ex

If possible could you please email me all the functions that you can find in which the original function and its derivative is identical.


Answered by Claude Tardif.
Putting in a pool 2000-03-16
From Katie:
If your digging a hole for a pool and the pool is:

Length= 11m
Width= 6m
Sallow Depth= 1.1m graduating to
'Deep' Depth= 1.8m

What is the volume of soil that will be taken out? And if a bobcat can excavate and remove 10m3 (qubed) of soil an hour how long will it take him to excavate the soil.


Answered by Penny Nom.
Geometrical solids 2000-03-15
From Sarah:
  1. What geometrical solid has 8 edges and 5 vertices?
  2. What geometrical solid has 12 edges that are all the same length?

Answered by Walter Whiteley.
Rationals, irrationals and integers 2000-03-14
From Erin McKeon:
Why does the letter J represent the set of integers, the letter Q represent a set of rational numbers and the letter P represent a set of irrational numbers? What do each of these letters stand for?
Answered by Harley Weston.
Characteristic of a logarithm 2000-03-14
From Andrew Strauss:
What is the characteristic of log base 18 of 1980 ?
Answered by Harley Weston.
Two problems 2000-03-14
From Teresa Bostic :
  1. A man's age at death was 1/29 of the year of his birth. How old was he in 1949?
  2. At a contest, 2 persons were asked their ages. Then to test their ability mathematically, they were asked to give the ages together. One gave 44 and the other gave 1280. The first had subtracted one age from another, and the second had multiplied the two together. What were their ages?

Answered by Penny Nom.
An integer max-min problem 2000-03-13
From Paul Servic:
Maximize Q = xy 2 where x and y are positive integers such that x + y 2 = 4
Answered by Penny Nom.
Maximize 2000-03-12
From Tara Doucet:
My question is Maximize Q=xy^2 (y is to the exponent 2) where x and y are positive integers such that x + y^2 ( y is to the exponent 2)=4
Answered by Harley Weston.
An equilateral triangle in a circle 2000-03-11
From Michael Setlik:
An equilateral triangle is drawn within a circle such that all three points of the triangle just touch the inside of the circle. Given the diameter of the cicle as six inches what is the length of the sides of the triangle?
Answered by Harley Weston.
Simultaneous equations 2000-03-11
From Laura Molck:
My name is Laura Molck and I am in Year 11 in Australia. Please help me with the following. I know that they are all simultaneous equations which I can do but I have trouble with the formulae to work the equations. Can you please help!!

1. A tent manufacturer produces 2 models, Outback and Bushwalker. From earlier sales records it is known that 20% more of the Outback model is sold than the Bushwalker. A profit of $200 is made on each Outback sold, but $350 is made on each Bushwalker. If during the next year a profit of $177,000 is planned how many of each model must be sold?


Answered by Penny Nom.
Completing the square 2000-03-11
From Sandra:
How do i complete the square and leave in simple radical form? x squared + 18x - 17=0
Answered by Penny Nom.
Congruent parts of congruent figures 2000-03-10
From Erica:
Yesterday, I recieved a test problem asking to prove two line segments equal. Here is the problem as I was given it:

Given: paralleogram ABCD
AE is perpendicular to DB
CF is perpendicular to DB
Prove: AE is equal to CF

I answered the problem as follows:
.
.
.
Answered by Walter Whiteley.

A goat problem 2000-03-10
From Nancy:
A farmer has a fenced-in, circular field with an area of about one acre. A goat in the field is tethered to one of the fence posts with a rope which gives the goat approximately one-half acre of grazing area. To the nearest foot, how long is the rope?
Answered by Chris Fisher.
Systems of linear equations 2000-03-10
From Ann Marie Devereux:
hi there!!, I guess I have a problem!!!
  • 3x+4y=10 (over)
    4x+y=9

  • 2x=5y+3 (over)
    x=3y+1


Answered by Penny Nom.
Grazing area for a goat 2000-03-10
From Amy:
A goat is tied in the middle of a side of a square building whose sides are 2 yards long. The rope is 4 yards long. What is the grazing area for the goat?
Answered by Harley Weston.
Box and whisker plots 2000-03-09
From Brett Blake:
Do you have any information on Box and whisker graphs?
Answered by Harley Weston.
Some equations 2000-03-08
From Maria:
Hi, could you help me with these questions:

Find x & solve

  • 7-2x/3=9
  • 4x+3/5=-2
  • 6-2x=14
  • -1/7x=-4/3+1/6

Answered by Penny Nom.
Mathematical deduction and mathematical induction 2000-03-07
From Espera Pax:
What are mathematical deduction and mathematical induction, and what is the difference between them?
Answered by Harley Weston.
Making dolls 2000-03-07
From A 6th grader:
How many small dolls can be made from 3 lbs. of dough if each doll uses 7/8 lbs
same question if each doll takes 3/8 lbs.

Answered by Penny Nom.
A mixture problem 2000-03-06
From Rebecca Edwards:
A tank in which cholocate milk is being mixed contains a mixture of 460 liters of milk and 40 liters of chocolate syrup initially. Syrup and milk are then added to the tank at the rate of 2 liters per minute of syrup and 8 liters of milk per minute. Simultaneously the mixture is withdrawn at the rate of 10 liters per minute. Find the function giving the amount of syrup in the tank at time t.
Answered by Harley Weston.
Two calculus problems 2000-03-03
From Tara Doucet:
  1. The height of a cylinder with a radius of 4 cm is increasing at rate of 2 cm per minute. Find the rate of change of the volume of the cylinder with respect to time when the height is 10 cm.

  2. A 24 cm piece of string is cut in two pieces. One piece is used to form a circle and the other to form a square. How should the string be cut so the sum of the areas is a maximum?


Answered by Harley Weston.
Angle of Intersection of Two Lines 2000-03-02
From Veronica Patterson:
I am having a real hard time trying to figure out this problem. Could you please help me! The homework question says to find the acute angle of intersection between the two lines y=3x+1 and y=(1/2)x-1. (It also says to use the results of a problem I had already figured out.) That problem was to use information from a picture shown that tan(theta sub1-theta sub2)= ((m sub2- m sub1)/(1+(m sub1 * m sub2))). I used the difference identity of tangent to figure out the answer. Any help on this problem would be greatly appreciated.
Answered by Harley Weston.
Triangular numbers and square numbers 2000-03-02
From Emily McCallum:
Would someone find me some math activities using triangular numbers and square number. Especially acitivities that actually form the shapes. They need to be at the 5th grade or 6th grade level. But you can not find anything that can be tauht at these level, that fine. I just need to be able to teach this new subject or kind of math to my kids.
Answered by Rick Seaman and Penny Nom.
Pythagorean triples 2000-03-01
From Bob Ross :
Could you please tell me what pythagoria triad is.I am a year 10 student.
Answered by Chris Fisher.
Proportionality in a triangle 2000-03-01
From Courtney Smith:
I would appreciate assistance with the following problem In triangle ABC,segment MN divides sides(segment)AC and (segment)AB proportionally. If the coordinates are A(3,7),M(0,10) and N(8,22) and if AM:MC = 3:1, find the coordinates of B and C.
Answered by Penny Nom.
Folding a page 2000-03-01
From Krista Bischoff:
One corner of a page of width a is folded over and just reaches the opposite side. Express L, the length of the crease, in terms of x and a.

I can't get the picture to copy to this form so I guess I will have to try and describe the picture the best that I can. The top right hand corner is folded to the left side, almost half way down. The width of the paper is a ( the width of the bottom part which is not folded.) The creased side is L and the part shorter part of the folded area is x (the part that would have been the top right of the original piece.)


Answered by Chris Fisher.
What's the next term? 2000-02-29
From Meredith O'Neill:
At this point, the whole family wants to know the answer. if possible an explanation. thanks
a sequence ?

16,37,58,89,145 ??


Answered by Chris Fisher.
Area of a circle 2000-02-29
From Michelle Buboltz:
My name is michelle and I just need to convert 25 ft across a circle into how many square feet.
Answered by Penny Nom.
Fractions in algebra 2000-02-27
From Leslie:
Question:
add (4x+1)/(x-8) + (3x+2)/(x+4) + (49x+4)/(x^2-4x-32) and

solve x - 6/(x-3) = 2x(x-3)

Answered by Penny Nom.
Factoring, primes, GCF and LCM 2000-02-27
From Ruth Kroek:
My son is in grade seven, he has to do a Factoring Booklet the areas covered are:
  • Prime #'s
  • Composite #'s
  • Rainbow Factoring
  • Finding Multiples (consecutive multiples)
  • Finding GCF of 2-3 numbers uning Rainbow factoring
  • Finding LCM of 2 numbers using consecutive multiples
  • prime factor trees
  • prime factor ladders
  • finding GCF of 2 numbers using Prime number Method
Although his text 'Math Power' gives some information, we are at a loss ..

Answered by Penny Nom.
Building a pyramid 2000-02-26
From Francis X. Hines Jr.:
I am presently trying to build a pyramid. I can understand that the base has 90 degree angles on the first plane which is the outline of the square that makes up the floor.

As close as I can figure the slope of each wall face is 35 degrees or 35.7 to be exact if I am correct by using 360 as the total of the three interior angles.Now , I run into a compound angle where the corners meet what would be the angle created by the two 35 degree angles that would allow for the 90 degree edge to continue.

Because I'm working in three dimensions I also need to be sure that my math would be correct when I substract 35 from 90 to aquire the angle of the narrow edge as to allow for a 90 degree surface to be present ..to allow for another level to be added with only the base line being shortened. I hope you can understand what it is that I'm asking assistance with.I would greatly appericate your help.


Answered by Chris Fisher.
Polar coordinates 2000-02-24
From Tran Trung:
How do I convert r = 2/(2sin(theta) - 3cos(theta)) to rectangular and x2 + y2 -2y = 0 to polar form?
Answered by Harley Weston.
Slant height of a cone 2000-02-24
From Jocelyn Wozney :
I need help with this problem for my high school calculus class. Any help you can give me will be greatly appreciated-I am pretty stumped. "Express the volume of a cone in terms of the slant height 'e' and the semi-vertical angle 'x' and find the value of 'x' for which the volume is a maximum if 'e' is constant.
Answered by Harley Weston.
The isoperimetric theorem 2000-02-24
From Raj Bobal:
How can you prove Mathematically that the maximum area enclosed by a given length is a circle?
Answered by Chris Fisher.
More on googols 2000-02-23
From Kevin Brennan:
Could you please give me a couple of examples of when you MATH guys would use a google. Do youu measure, like, galaxies with it ? Or was it created to keep track of Mr Gate's bank account.
Answered by Penny Nom.
Triple angle formula 2000-02-23
From Sara:
Can one derive a triple angle formula for sine and cosine? If so, how?
Answered by Chris Fisher.
A triangle and a circle 2000-02-23
From wendy:
If a triangle of base 6 has the same area as a circle of radius 6, what is the altitude of the triangle? I am having trouble with altitude.
Answered by Penny Nom.
Some integration problems 2000-02-23
From Tim Valentine:
I am having a great deal of difficulty with the following integrals, can you help? I think they need the use of trig substitution or integration by parts but I cannot figure out how to begin. Thanks!

The integral of 1/(2+3x2) dx.

and

The integral of x * square root of (4x+5) dx.


Answered by Harley Weston.
Party favors 2000-02-22
From Krystina Fernandez:
Luanne was making party favors for her little sister's birthday party. Each party favor was in the shape of a cube. Luanne had pink and purple paint to paint the cubes and she could paint each face only solid pink or solid purple (no dots,stripes,ect.).For example, one cube may be all purple, another may have two purple faces and four pink faces. Her little sister wanted to have a different cube for each guest.(A cube is not considered different if it can be turned so that all it's sides match the corresponding sides of another cube.)How many different cubes was it possible for Luanne to make?
Answered by Claude Tardif.
A moving point on the graph of y=sinx 2000-02-22
From Veronica Patterson:
Find the rate of change of the distance between the origin and a moving point on the graph of y=sinx if dx/dt=2 centimeters per second.
Answered by Harley Weston.
Pi 2000-02-22
From Shelley Collier:
Numbers that can be written in the form p/q where p and q are integers, (q not 0), are known as rational numbers. What you are missing is that p and q must be integers. The fact that Pi is irrational means that you can't have a circle with both the circumference and diameter being integers. In fact you can't even have the circumference and diameter both rational since the quotient of two rationals is again a rational.
Answered by Claude Tardif and Penny Nom.
A valentines project 2000-02-21
From Courtney Dohoney:
I was assigned a valentines project. Our job is to make a valentines card that has a math theme. On my valentines card I would like to include the set of equations that when graphed look like a heart. If you can figure out a set of equations and get it back to me I would be very happy.
Answered by Harley Weston.
The quotient rule 2000-02-21
From Charlene Anderson:
Question: I came across a question in our book that states: Let Q(x) = N(x) / D(x) Then re-write Q(x) in a form that can utilize the Power and Product Rules. Use this rearranged form to derive the Quotient Rule.

The Quotient Rule can be derived from the Power Rule and the Product Rule.

One must also use the chain rule too, right?


Answered by Harley Weston.
Why counterclockwise? 2000-02-21
From Drew:
Why do the numbers of quadrants in a graph increase in a counter clockwise pattern? Why aren't they numbered increasingly in a clockwise pattern?
Answered by Chris Fisher.
Irrational algebraic functions 2000-02-21
From Bucky Cadena:
Here is the multipart problem:

Given f(x) = x-3* squareroot of x + 4

What does the f(x) intercept equal Find the two values for which f(x) = -5 Find the one value for which f(x) = -3


Answered by Harley Weston.
Perimeter of an ellipse 2000-02-21
From Kurtis Proffit:
What is the formula for the perimeter or circumference of an ellipse?
Answered by Chris Fisher.
Jogging up a hill 2000-02-17
From Aaron Williams:
A man jogs down the hill at 6 miles per hour and back up at 4 miles per hour. The total time that he travels is 5/4 hours. What is the total distance that he jogged.

The solution is supposedely 3 miles, but i believe that it is 6. Can you help me? Please.


Answered by Claude Tardif.
The length of a cylinder 2000-02-17
From Kelly Wichmann:
What is the formula to figure the volume of a cylinder? We have a cylinder that is 2 3/4" in inside diameter and a gallon of oil that needs to fill it. We are trying to figure out how long the cylinder should be. Can you help us?
Answered by Chris Fisher.
Cross multiplication 2000-02-16
From J E Swinton:
Why does cross multiplication work?

How come canceling work?


Answered by Penny Nom.
Least Common Multiple 2000-02-16
From Kathy Heath:
The way I understand it is for 2,3,9 the least common multiple is 18. Am I right? If so, I guess the least common multiple for 3,4,7 is 105. Please let me know if I'm right so I can tell my daughter.
Answered by Denis Hanson.
Change for a dollar 2000-02-16
From Rachelle:
What is the least positive number of coins that is impossible to give out change for a dollar? It is higher than 50 i know and you can us dimes, nickels, pennies, quarters, 50 cent pieaces and silver dollars.
Answered by Claude Tardif.
2 = 1 2000-02-16
From Chuck Kennedy:
Question:
  1. Assume a=b
  2. Multiply both sides by a, a2=ab
  3. Subtract b2, a2-b2=ab-b2
  4. Factor (a-b)(a+b)=b(a-b)
  5. Cancel like factors a+b=b
  6. Substitue b for a b+b=b
  7. Then 2b=b
  8. Therefore 2=1
Question; Were is the mistake?

Answered by Claude Tardif.
Midpoints and endpoints 2000-02-15
From Jessica Sipes:
I need to know how to find and endpoint using the midpoint and the other endpoint.
Answered by Penny Nom.
Weighing bales 2000-02-15
From Thinh Than:
You have 5 bales of hay. and they were weighed but they didn't weigh them individually, they were weighed in pairs. The pairs were 1&2, 1&3, 1&4, 1&5, 2&3, 2&4, 2&5, and so on. The weights of the pairs were 80,82,83,84,85,86,87,88,90, and 91. Can you tell me how much the bales weigh individually.
Answered by Claude Tardif.
Is a square a rectangle? 2000-02-15
From Jaireh:
This is something that aroused a debate in class: A rectangle was defined as a parallelogram with 4 right angles. A square was defined as a parallelogram with 4 congruent sides and 4 right angles.

I need written and conclusive proof that some rectangles can or cannot be squares. I tried insisting that some of them can.. but without proof nobody will listen.


Answered by Walter Whiteley.
The sum of repeating decimals 2000-02-15
From Caitlin Harris:
Express 0.111... + 0.121212... + 0.123123123... as a repeating decimal and its equivalent fraction. Show work. Also, are there any extensions to this problem? In other words, are there any questions that we could ask that may be related to this problem?
Answered by Penny Nom.
Counting to a trillion 2000-02-06
From Art Keck:
If I counted one dollar every second, would it take 31,000 years to count to a trillion? If not, could you please explain how long it yould take?
Answered by Penny Nom.
Ten stools 2000-02-06
From Haim:
In a cafe with ten stools, three customers want to be seated, so that no two are next to each other. How many ways can this be done? Do not consider it to be a separate seating if two customers switch seats.
Answered by Denis Hanson and Walter Whiteley.
Tennis doubles 2000-02-04
From Brittany Allinson:
Cheri, Beth and Jacinta are daughters of Mr. Sullivan, Mr. Marchand, and Mr. Benoit. Four of these people are playing tennis doubles. Mr. Benoit's daughter and Mr. Sullivan are partners. Cheri's father and Mr. Marchand's daughter are also partners. There aren't any father/daughter combinations. Who is Cheri's father?
Answered by Penny Nom.
Filbert Family Circus 2000-02-04
From Sarah:
As Clyde moves his broom around the circus ring, he thinks that he has finally found a job where he can make a clean sweep of things. Clyde is sweeping the ring where the lions perform in the Filbert Family Circus. The ring is 76 feet across and Clyde is using a broom 3 feet wide. He starts at the outside edge and works his way to the middle, making circles around the ring. After sweeping 3/4 of the ring, Clyde sees the lions coming with their trainer and scurries out of the ring. How many trips around the ring did he make?
Answered by Penny Nom.
Play ball 2000-02-03
From Jessie:
Here's a calc question that is probably a lot easier than I am making it. If you have a legendary "baseball problem" for the related rates section of Calc I, and you are given that the runner is running from 2nd to 3rd base at a given rate, and the umpire is standing at home plate, and you are given the distance between the bases on the field, how do you find the rate of change of the angle between the third base line (from the point of the umpire) and the runner? Here is a sample prob: Runner is moving from 2nd to 3rd base at a rate of 24 feet per second. Distance between the bases is 90 feet. What is the rate of change for the angle (theta, as described previously) when the runner is 30 feet from 3rd base?
Answered by Harley Weston.
A problem with a radius. 2000-02-01
From Howard B Davis:
We start a Line that goes up 1 unit, then it goes to the Right for 5 units long, and then goes down 1 unit which is the end point. If we draw a circle that is tangent to both ends as well as the mid-point of the horizontal line: How do we find the radius of the arc; in Mathematics, with only this information?
Answered by Chris Fisher.
100% on two tests 2000-02-01
From Craig and Chelsea Bruzzone:
A class of 35 students took a math test and a science test. 12 students got 100% on the math test. 9 students got 100% on the science test. There were 19 students who made less than 100% on both tests. How many students made 100% on both tests?
Answered by Penny Nom.
Mark's life 2000-01-30
From Jean and Larry Brandemuehl:
Mark passed 1/6 of his life in childhood, 1/12 in youth, and 1/7 more as a bachelor. Five years after his marriage, there was born a son who died four years before his father, at half his father's (final) age. How old was Mark when he died?
Answered by Penny Nom.
Can't make an equation! 2000-01-30
From Maggie Clarke:
An estate is to be distributed among the wife, 3 children and 2 grandchildren. The children will each receive 2x as much as each grandchild, and the wife will receive 4x as much as each child. If the estate amounted to $115,000, how much will each person receive?
Answered by Penny Nom.
Names of the polygons 2000-01-29
From Janemath:
I have been doing research on the names of polygons with 11 to 20 sides. I have found conflicting names for some of them . for example 18 sided-octadecagon and octokaidecagon. 11sided-undecagon and hendecagon. Why is this? Is one more correct than another? Or are they equally acceptable?
Answered by Harley Weston.
Two problems 2000-01-29
From D. J. Anthony:

Hi, I am taking a course "math for elementary teachers" and I have two problems.

  1. How thick is a sheet of paper in your textbook.
  2. A little island has a population of 1,000 people. It doubles every 30 years. What will the population be in 30 years?, in 60 years?, when will it be over one million?

Answered by Peny Nom and Claude Tardif.
Compatible numbers 2000-01-27
From Angie:
Use compatible numbers to estimate each product and quotient.
23*1/2
1/3*11

Answered by Penny Nom.
Probability, graph theory and VLSI testing 2000-01-27
From Valentin Muresan:
I am postgraduate student of Dublin City University, Ireland, in the field of VLSI Testing. I am currently working on some heuristics and I need a probability formula. I want to get an expert's point of view in my matter. Say there is an initial list of so called tests: T1, T2, T3, ..., Tn Every test element Ti of the above list has a list of compatibility (incompatibility), which includes all the elements (tests) from the initial list, the Ti test is compatible (incompatible) with. For example, say the initial list is: {T1, T2, T3, T4, T5, T6, T7, T8, T9, T10} and the compatibility lists of these elements (tests) are:
.
.
.

Answered by Denis Hanson.
Quadratics 2000-01-27
From Lori:

Hi my name is Lori. I'm a student, in the 11th grade.I have a problem in quadratic functions. I don't know how to get the numbers into the problem and then how to do the problem. here is one problem the rest are sort of similar to it... y=2x squared. If I were to find out how to get the (x,y) answers then I could graph it but as I said I can't find the numbers to go into the problem.

Also in the equation g: x arrow to y = -x + 4..... how would you know weather it is or isn't a quadratic equation????


Answered by Penny Nom.
Quadratic 2000-01-26
From Karen Johnson:
What is the reasoning behind the title quadratic equation?
Answered by Penny Nom.
Mixing oil and gas 2000-01-25
From Karen:
My hubby is ready to use his snow blower and is supposed to use 50:1 measurement....50 parts gas to 1 part oil......how does he do this if he has 1 gallon of gas? How much oil do we use...need info asap.....hopefully you are there!
Answered by Harley Weston.
Complex Roots 2000-01-24
From Jess Rutherford:
How do I find the value of k when 5x2 + k = 3x and has complex roots ?
Answered by Penny Nom.
Euclidean Math puzzle 2000-01-24
From Margaret Matthews:

(Check out this web-site: Simeon's Triangle Puzzle )

I have tried to figure out how this could be, because everything I know about it tells me it can't be. However, I can't seem to make it NOT work.

Two right angle triangles. They are each cut up into four identical pieces. In the first, all the pieces fit together so that there are NO empty spaces; in the second, presumed to be identical in size to the first, the pieces are slightly rearranged, and now, there IS a space in the triangle.


Answered by Patrick Maidorn and Claude Tardif.
Word problem 2000-01-24
From April:

Ok, my algerbra teacher gave me this problem and by just looking at it I can tell what the answer is but he wants me to show how I got the answer in an equation form.

The tempature at 7am on a cold day was -4 degrees Celcius. At 3pm the tempature was 1 degree Celcius. Write and solve an equation to find out how many degrees the tempature rose between 7am and and 3pm.


Answered by Penny Nom.
Functions 2000-01-23
From Tara:
Hi my name is Tara, I have two math problems that I need help with in my calculus math class.

  1. If f(x)= x - 2 show that (x+3)f(x)-(x+2)f(x+1)+4=0

  2. Graph this function and use the graph to determine the range y=2x2 - 8x - 3

Answered by Harley Weston.
Frogs and lily pads 2000-01-23
From Nick:
There are 5 lily pads and 4 frogs, 2 Red and 2 Green, how many moves does it take for them to swap sides?
Answer: 8

I have a formula which will tell you how many moves it will take with different total numbers of frogs, it is [(F*F)/4]+F Where F is the number of red frogs add the number of Green frogs. This formula only works with numbers of frogs that are the same on each side. What I would like to know is why this formula works and why you have to divide it by 4?


Answered by Penny Nom.
The limit of f(x)/x 2000-01-22
From Laurent Jullien:
I would appreciate help to prove that a twice continuously differentiable convex function from R+ to R has the property that f(x)/x has a limit when x tends to infinity.
Answered by Claude Tardif.
Domain of a function 2000-01-19
From Stacey Hutchison:

What is the domain of:

f(x) = -1/(x + 4)


Answered by Patrick Maidorn.
Arclength of a circle 2000-01-19
From Holly:
What is the formoula for finding the arc length of a central angle of a circle??
Answered by Harley Weston.
Logarithms 2000-01-18
From Jennifer:

Hi I have a test tomorrow and I was wondering if you could help me with these two problems.

  1. Explain how to express y in terms of x if log3y=4log3(2X)

  2. The LN key on your scientific calculator is not working, but the ex and LOG keys are. Write directions for calculating a natural logarithm using the ex and LOG keys. Explain why these directions will get you the right answer.


Answered by Harley Weston.
Pyramids and prisms 2000-01-18
From Tyler:
What's the definition of a Triangular Prism and a Triangular pyramid.
Answered by Penny Nom.
Algebra 2000-01-18
From Candis:
I want to know what exactly is Algebra?
Not how to do it, or methods. Just what is it?

Answered by Allen Herman.
Ax + By = C 2000-01-18
From Fatiha Khanboubi:
I was wondering if you could please explain to me how I would write the equation of line through the given points [(4,1), (6,3)] in Ax+By=C form.
Answered by Harley Weston.
Solving an equation 2000-01-17
From Shelley's Mom:

My daughter has an assignment in math that includes this equation:

-6b+5= -10

I've tried to help her with a scale. We've figured that you subtract 5 from each equation so its not -6b = -15. Is this correct and can you help us figure out the next step?


Answered by Penny Nom.
An unbelievable story 2000-01-17
From Melissa:
good morning-i am taking a course "math for elementary teachers" and i have a take home problem that i really need help on. here is the question
-the million dollar problem
a recent item in the newspaper described a robbery in which the robber had held up a local bank for 1 million dollars in small bills, had the tellers put the money in a briefcase which he gave them, and fled the scene on foot. is this story believable? explain your answer!

Answered by Harley Weston.
Taxicab geometry 2000-01-16
From Jack:
Im doing an investigation entitled 'taxicab geometry', ive attempted it and have done most of the practical part of it. But i cant seem to see the equation behind it all. please guide me in the right direction.

They provide a regular grid with regular horizontal and vertical lines. The lines are roads and the actual squares are blocks of hoouses/ buildings.

In taxicab geometry the distance between place A & B is worked out by adding together the horizontal and vertical distances. each square is counted as i unit.

Part 1 (I've done this)

A taxix cab firm is based at A. the posistion B is 7 units away from A. investigate all the posistions of B at 7 units. investigate for different distances.

.
.
.

Answered by Penny Nom.
A roll of paper 2000-01-15
From Richard:
I have a roll of paper, wrapped around a corrugate core, whos diameter is 10.750 in. The outer diameter of the roll is approx. 60 in. The thickness of the paper is .014 in. I am trying to find out how much linear feet of paper is left on the roll, given only the diameter of paper remaining on the core.
Answered by Chris Fisher and Harley Weston.
A three digit number 2000-01-13
From Manali Shah:
My son is in 5th grade, he has a math problem. There is three digit number, which is greater than 500, can be divided by 7,9 and 11, but cannot be divided by 2,4,6, and 5.
Answered by Harley Weston.
Algebra 2000-01-13
From Tim Merrill:
"please help me out with these four algebra questions"

  1. Rewrite the following using positive exponents only.

    xy -3
    -------
    x -2

  2. Find the following quotients.

    9a3+3a2+6a
    --------------
    3a


.
.
.

Answered by Penny Nom.
Parallel lines 2000-01-12
From Lori:
Find an equation of the line parallel to the given line containing the given point. x + y = 2; (1,2) We know parallel line have the same slope and we know the answer y = -x + 3.
Answered by Penny Nom.
Three dimensional rectangle 2000-01-11
From Dennis Murphy:
I would like to find out the name of a Three dimensional rectangle.
Answered by Harley Weston.
Parallel planes 2000-01-10
From Hugo Alvarez:
When two parallel planes are cut by a third plane, the lines of the intersection are?
Answered by Claude Tardif.
Phone Number Possibilities 2000-01-09
From Hamilton Weston:
How do phone companies arrive at the possible number of phone numbers that can be generated for customers?
Answered by Harley Weston.
Capacitor discharge rate 2000-01-09
From Bill Phillips:
I need to be solve for t in the following rate problem for an electrical capacitor. Vr = E(e^-t/RC), e=nat. log 2.718 raised to the -t/RC power.
Answered by Harley Weston.
How many coins is one calculator? 2000-01-09
From Mandy Brockmann:
Tom weighed 1 pen and his calculator with 4 coins. Then he weighed his calculator and 3 pens with 12 coins. How many coins is one calculator?
Answered by Claude Tardif.
Functions 2000-01-06
From Tori Morris:
Consider the function f(x)=x2-3. Which of the following are true?

  1. f(1)>f(0)
  2. f(2)>f(-3)
  3. f(2)=f(-2)
  4. f(-1)=f(3).
More than one answer can be true.

Answered by Penny Nom.
Angles 2000-01-06
From Rayna:
I am doing a presentation report on angles which has to be fun and entertaining as well as educational. I am having problems locating resources on angles that give me ideas of fun entertaining projects. My lecture is to be about 20 minutes long infront of a class of 11th and 12th graders. Please Help if you can.
Answered by Claude Tardif and Walter Whiteley.
A probability experiment 2000-01-05
From Vanessa:
Duels in the town of Discretion are rarely fatal. There, each contestant comes at a random moment between 5 a.m. and 6 a.m. on the appointed day and leaves exactly 5 minutes later, honor served, unless his opponent arrives within the time interval and then they fight. What fraction of duels lead to violence?

There must be a minimum number of 100 trials and things like graphing calculator, dice, spinners, and whatever are allowed.
Answered by Harley Weston.

Why study calculus? 2000-01-05
From Trlpal:
I am a high school senior enrolled in a pre-calculus class. Could you tell me what the benefits of taking calculus are and why it would be important to take the class.
Answered by Walter Whiteley and Harley Weston.
Y-intercept 2000-01-03
From Kayla Boylan:
How do you use and find the y-intercept?
Answered by Penny Nom.
Derive the quadratic equation 2000-01-01
From Thuy:
My teacher asked us this question in which i can't understand when she asked to derive the quadratic equation. What does this mean?
Answered by Penny Nom.
Solving a cubic equation 1999-12-30
From Natalia:
How can I solve:
2 x3 - x2 + x - 2 = 0 ???

Answered by Penny Nom.
Absolute value 1999-12-29
From Alex Waller:
What are some practical applications for absolute value?
Answered by Harley Weston.
Parachute Problem 1999-12-28
From Stephanie Zinsli:
A parachutist jumps from an airplane and lands in a square field that is 2 kilometers on each side. In each corner of the field there is a large tree. The parachutist's ropes will get tangled in the tree if she lands within 1/11 kilometer of its trunk. What is the probability that she will land in the field without getting caught in a tree?
Answered by Penny Nom.
Three algebra problems 1999-12-28
From Stephanie Branton:
  1. If P represents the product of all prime numbers less than 1000, what is the value of the unit's digit of P?

  2. Do any real numbers a and b exist such that: ln(a+b)=ln a + ln b? if so, what are they?

  3. Define a function by: f(x)=1/1-x where x is not equal to 0,1. what is f(f(f(a)))?

Answered by Harley Weston.
Climbing stairs 1999-12-22
From Harman Chaudhry:
Paul, Michelle and Michael all climb up 54 steps. Paul climbs the steps one at a time i.e. 1, 2, 3, etc. Michelle Climbs the stairs 2 at a time i.e. 2, 4, 6, etc. Michael climbs the stairs 3 at a time i.e. 3, 6, 9, etc. How many steps are used by exactly two people?
Answered by Penny Nom.
Specific heat 1999-12-22
From Irena:
In transferring 1260J of heat to water, the temperature of 0.20 kg of mercury falls 50 K. Calculate the specific heat capacity of mercury using the equation C= Q/(m)(delta t)

i know that Q= -1260J because it's loosing heat and m= 0.2kg and since i have to use 'C i have to convert Kelvins into Celsius (i also know that 'C has to be negative since it looses heat)
.
.
.
Answered by Harley Weston.

Comma as decimal separator 1999-12-18
From Franck Menuge:
Could somebody tell me the origin of the use of a 'comma' as a decimal separator and of a 'dot' for thousands in the French numeric system, ex.1.234,56 Is it only used in France or in other european coutries? Why is it different in the UK?
Answered by Patrick Maidorn and Penny Nom.
Two algebra problems 1999-12-17
From Michael Standfest:
If x+4 is a factor of 2x4+kx3-3kx2+6x-40, find k

and

Prove that n2-n is even for all n, using the proof of contradiction
Answered by Penny Nom.

Logarithms 1999-12-16
From Pierre:
Can you please answer those three question

  1. inverse or the exponential function.

    logx 81 = 4,
    x expo4 = 81,
    x = +or- 4square root 81,
    x = +or- 3
    s.s = {3}
    Why is -3 not acceptable.

  2. .
    .
    .

Answered by Harley Weston.
Rectangular hyperbola 1999-12-15
From Aarti Chand:
Why do they call a rectangular hyperbola, rectangular and where the normal hyperbola looks like a rectangle and the rectangular hyperbola looks like a sqaure?
Answered by Chris Fisher.
A decreasing ellipsoid 1999-12-15
From A student instructor:
The volume of an ellipsoid whose semiaxes are of the lengths a,b,and c is 4/3 *pi*abc. Suppose semiaxes a is changing at a rate of A cm/s , the semiaxes b is changing at B cm/s and the semiaxes c is changing at C cm/s . If the volume of the ellipsoid is decreasing when a=b=c what can you say about A,B,C? Justify.
Answered by Harley Weston.
A model area 1999-12-15
From John Ost:
Hello, I am a college student taking an elementary math course I need if possible assistance to developing an area model 36 x 25 that shows each of the four separate partial products. I must know how to do the computation 36x25 showing each of the four partial products separately, and indicate how each corresponds to the drawing that is required of the area model.
Answered by Penny Nom.
Cutting a carpet 1999-12-15
From Heather:
A rectanglular piece of carpeting is 90 inches long and 90 inches wide. How can the carpet be cut into two pieces of equal sides and shape to cover an area of 100inches? There can only be one cut and no scraps. Please show me how.
Answered by Penny Nom.
.400 in Baseball 1999-12-15
From Nieve Nielson:
I have several questions to ask about the probability of getting a .400 in baseball:
  1. What is the probability of a baseball player hitting a .400 in one game, considering that the hitter is up to bat four times?

  2. What is the probability of a baseball player hitting a .400 in a season, considering that there are 100 games in a season? With the first two questions in mind:

  3. Considering that baseball has been around for about 100 years, about how many people should hit .400 inthat time?

Answered by Chris Fisher.
Fibonacci Numbers 1999-12-15
From Gary Nelb:
I'm doing a project on fibonacci numbers and I'm using different starting values and finding out if different starting values to see whether or not the ratios still get closer to phi. I was wondering, what numbers should I use. Should I use two of the same # like 2 and 2, or numbers like 1 and 2, or even something totally different.
Answered by Denis Hanson.
Two calculus problems 1999-12-13
From Alan:
I have 2 questions that are very new to me, they were included on a quiz and the material was never covered. Our teacher never explained the purpose and detailed explanation of how to solve the problem. Could you help? Thanks.

Question 1:
A ball is falling 30 feet from a light that is 50 feet high. After 1 sec. How fast is the shadow of the ball moving towards the light post. Note that a ball moves according to the formula S=16t^2

Question 2:
How many trapezoids must one use in order for the error to be less than 10^-8 if we want to find the area under the curve Y=1/X from 1 to 2. Find the exact area, Graph the function and use the trap rule for the "N" that you found.


Answered by Harley Weston.

Passing trains 1999-12-10
From Nancy:
Two trains, each of 80 cars, must pass on a single track which has a deadend siding. How can they pass if the siding can only hold a locomotive and 40 cars?

I do not understand this...help. It will be due monday. I thought it would be simple if the siding could hold 80 cars.
Answered by Penny Nom.

Sines & cosine laws 1999-12-10
From Pierre Boivin:
Triangle LMN, angle L=71 degree , LM= 7.2 , MN=8.3 , ln= 5.9

The questiion was to find angle M. Using the cosine law I found the answer to be 44 degree. It is also the book answer. Using the sines law I found the answer to be 42.2 degree. why can't I use the sines law.
Answered by Chris Fisher.

Cubic meters to gallons 1999-12-09
From Chris Compton:
I am trying to convert cubic meters to gallons. Do you have the conversion formula? The problem I am trying to solve is:

59.62 cubic meters to gallons.


Answered by Harley Weston.

Solve for v 1999-12-09
From Beth:
1-3/4(v+2)=-5
Answered by Penny Nom.
A calculus problem 1999-12-08
From JT Wilkins:
These are the questions:

  1. Show that there exists a unique function that meets the following requirements:

    a) f is differentiable everywhere
    b) f(0)= f'(0)= 0
    c) f(x+y)= f(x)+ f(y), for all real values of x,y

  2. Consider the function F: R-->R (All Reals)

    F(x) = 0, for x irrational & 1/q, x=p/q gcd(p,q)=1 q > 0

    a)determine the values x where f is continuous, respectively discontinuous.
    b)determine the values x when f is differentiable and for each of these values compute f'(x).


Answered by Penny Nom.
Advanced Calculus 1999-12-07
From Kay:
Hi, my name is Kay. Please help---these problems are driving me crazzzzy!!!! Your help would be greatly appreciated!
  1. Let a,b be contained in R, a
  2. .
    .
    .

Answered by Claude Tardif.
Inequalities and absolute values 1999-12-07
From Austin Cline:
Solve for x: The absolute value of x-1 is less than or equal to the absolute value of x-2
Answered by Harley Weston.
Two conics 1999-12-07
From Quinn:
I know the formula to find the axis of symmetry of a conic section (I'm not sure what shape - circle for the first one??) is (-D/2A,-E/2C) but I obviously don't get how to calculate it, because when I check the answer it's wrong, but I'm so close!! For the following equations my teacher suggested to "divide the x term coefficient, D, by the x squared term coefficient before...do the same for y."

2x2+2y2-8x+12y+16=0
.
.
.
Answered by Penny Nom.

Bases other than 10 1999-12-06
From Garret Magin:
We are doing a lesson on numbers of other bases than 10. We are working with binary, octal, and Hexadecimal. I was wondering what is used to represent number of different bases other then 16? Does it just continue on with the alphabet and if so what happens when you get to Z. It would be a help if you could answer this because it is really bugging me. And none of the math teachers at my school could let me know.
Answered by Claude Tardif and Patrick Maidorn.
Systems of equations 1999-12-06
From Roger Hornbaker:
I am having problem figuring out x and y solutions.
  1. 5x + y = 4
    3x - y = 4

  2. 3x + 2y = 6
    - 3x + y = 0

Answered by Penny Nom.
order 4+ determinants 1999-12-06
From Joe Kron:
Why is it never shown how to calculate the value of 4x4 (or larger size) deteminants by the diagonal multiply methods that are generally shown for 2x2 and 3x3 determinants? The method I'm talking about is called Cramer's Rule??? Is this method not extensible to order 4+ and if not why not? Anyway the method always shown for order 4+ is called "reduction by minors" which is not the answer to this question.
Answered by Walter Whiteley.
Volume of an ellipsoid 1999-12-06
From Michael Nowicki:
I am a physician at the University of Mississippi. I am trying to compare the size of polyps (abnormal growth of tissue in the colon) between 2 groups of individuals. The department of pathology gives the size as length X width X depth - how do I calculate the volume? Polyps are nearly ovoid in shape.
Answered by Walter Whiteley.
Multinomial coefficients 1999-12-03
From Suraj Das:
Is there a formula for the expansion of (a+b+c) to the nth power? Does it have to do with Pascal's triangle?
Answered by Penny Nom.
The chain rule 1999-12-03
From Jennifer Stanley:
This problem is making me dizzy. I would greatly appreciate a little help!

Express the derivative dy/dx in terms of x.

y=u^2(u-u^4)^3 and u=1/x^2
Answered by Harley Weston.

Log Cancel 1999-12-03
From Justin Murray:
Ok I feel like a goof because I know this and I cant think of an answer But I want to isolate the x in the function y=2^x how do I do this?
Answered by Harley Weston.
Whole numbers 1999-12-02
From Patty:
I'm in a 5th grade class, please explaine what whole numbers are.....
Answered by Penny Nom.
The elimination method 1999-12-02
From Jennifer:
Could I get an answer to this one:

2x+5y=36
3x+2y=32

I have to use the Elimination method, as I already know how to do Substitution. How do I begin and show my work? I'm attempting to eliminate the values for y.
Answered by Penny Nom.

Zero to the zero 1999-12-02
From A middle school teacher:
I am a middle school teacher who is looking for a precise explanation of why zero raised to the zero power is undefined. I am hoping to get an explanation using something other than the fact that diividing by zero is undefined.
Answered by Penny Nom.
Camels and bananas 1999-12-02
From Marie Rich:
Corey Camel's harvest, worth its weight in gold, consists of 3000 bananas. The market place where the stash can be cashed in is 1000 miles away. However, Corey must walk to the market, and can only carry up to 1000 bananas at a time. Furthermore, being a camel, Corey eats one banana during each and every mile she walks (so Corey can never walk anywhere without bananas). How many bananas can Corey get to the market?
Answered by Claude Tardif.
Get rich quick 1999-12-02
From Holly Hutson:
If one cent were placed in an investment that doubles in value every day, what would be the value of the investment at the end of 30 days?
Answered by Penny Nom.
Two calculus problems 1999-12-01
From O'Sullivan:
Question #1 Assume that a snowball melts so that its volume decreases at a rate proportional to its surface area. If it takes three hours for the snowball to decrease to half its original volume, how much longer will it take for the snowball to melt completely? It's under the chain rule section of differentiation if that any help.

I've set up a ratio and tried to find the constant but am stuck.

Question #2 The figure shows a lamp located three units to the right of the y-axis and a shadow created by the elliptical region x^2 + 4y^2 < or= 5. If the point (-5,0) is on the edge of the shadow, how far above the x axis is the lamp located?

The picture shows an x and y axis with only the points -5 and 3 written on the x axis. the lamp is on the upper right quadrant shining down diagonally to the left. There's an ellipse around the origin creating the shadow. It's formula is given as x^2 + 4y^2=5.


Answered by Harley Weston.
Cos x = -1/2 1999-12-01
From Pierre Boivin:
When I factor[ 2cos (square) - 5cos -3], I get (2cos + 1)(cos - 3). 2cos + 1 = 0, 2cos = -1, cos = -0.5,. Using inv cos on calculator, I get 120 degree related angle.

When I graph I get two values, between 90 and 180 degree and between 180 and 270 degrees. How do I find those two values. How do use 120 degree in relation with the x axis.
Answered by Penny Nom.

An area problem 1999-11-29
From Vicky Birch:
The length of a rectangle is three times it's breadth. If the length were 6 meters less, and the breadth were 6 meters more, the rectangle would be a square. Make a mathematical model and solve the resulting equation to find the breadth of the original rectangle. What is the area of the square?
Answered by Penny Nom.
Two candles 1999-11-24
From Skip Simpson:
You have two candles the same length. They are lit at the same time. One burns down in 4 hours; the other in 5 hours. How long does it take before one candle is three times the length of the other candle?
Answered by Walter Whiteley.
The number of city blocks in a country mile 1999-11-24
From Gloria Hearst:
For years my family has had an on going debate on the number of city blocks in a country mile. We vary from a minmum 8 blocks per mile to a maximum of 12 blocks per mile.
Answered by Nadine Griffiths and Penny Nom.
How wide is the well? 1999-11-24
From Chris Baranski:
There is a well and in the well there are 2 sticks one is 2 meters long the other is 3 meters long and they are opposite to each other. They are leaning against the wall of the well. The place where they touch is 1 meter of the bottom of the well. How wide is the well?
Answered by Chris Fisher.
Repeating decimals 1999-11-22
From Andrew:
Is 1.9 repeating the same as 2?
Answered by Walter Whiteley.
Age in minutes 1999-11-21
From Mesha Jackson:
How old would I be in minutes if I were 12
Answered by Penny Nom.
A goofy clock 1999-11-20
From Kate:
While repairing a watch, a jeweler removed the hands and inadvertently replaced the hour hand on the min. spindle and vice versa. he set the hands to read 2:00pm, which was the correct local time when I picked up the watch. A few minutes later, I noticed that the hands were taking goofy positions. What was the first time thereafter that the watch would show the correct local time?
Answered by Chris Fisher.
A matching quiz 1999-11-20
From William J. Ricciardi:
I gave a matching quiz the other day and one of my students got all 9 incorrect. That got me thinking......What is the probability that someone could get all 9 questions incorrect?
Answered by Peny Nom.
Selecting some players 1999-11-20
From Korbin Brown:
Roger Craig, during his term as team manager of the San Francisco Giants, received a strange communication from the team general manager, Al Rosen. Mr. Rosen told him to select 25 players according to this formula: 1/2 of the team had to be outfielders and infielders 1/4 of the team had to be starting pitchers 1/6 of the team had to be relief pitchers 1/8 of the team had to be catchers Roger was a bit confused by Al's request, yet complied anyway. How did he do it?
Answered by Claude Tardif.
Area of a quadrilateral 1999-11-19
From Zane Cram:
I need the formula to calculate the area of an irregular sided rectangle. Each side has a different measurement or length.
Answered by Walter Whiteley.
polynomial functions 1999-11-19
From Quinn:
  1. Without fully factoring the following show that they all have the same zeros:

    I: x4-x2+2x+6
    II: -x4+x2-2x-6
    III: -4x4+4x2-8x-24
    IV: 10x4-10x2+20x+60

  2. When P(x)=x3-3x2+5x+1 and G(x)=x3-2x2-x+10 are each divided by(x-a) the remainders are equal. At what coordinate point does the graph of P(x) intersect G(x)?

Answered by Walter Whiteley.
Critical numbers 1999-11-17
From Sarah:
My question is this:

If f(x)=2x3+4x2-9x+8

and the derivative is f'(x)=6x2+8x-9,

how do I find the critical points if f'(x) is not factorable??
Answered by Penny Nom.

Two derivatives 1999-11-16
From Gina Renicker:
The derivative of:

y=e(xlnx) and y=x2arctan(x1/2)
Answered by Harley Weston.

The sixth game 1999-11-16
From Stephanie Sprunger:
Janice scored 18,21,27,16,and 19 points in five basketball games. How many points must she score in the next game to raise her average to 22 points per game?
Answered by Penny Nom.
Estimating the population mean 1999-11-13
From John Barekman:
Statitistics: Estimating the population mean when the standard deviation is known:

I am not sure which n to use in the formula for the confidence interval equation:

x +/- z*(standard deviation/sqrt(n))

If we have data of ten people, and if we have the data of ten sets of ten people each, what is the difference in the n that we use? What is the difference between the standard deviation and the standard error? Are we using the number of sampling means or just the number of samples?
Answered by Harley Weston.

Body part measurements 1999-11-10
From V Bailey:
I am a kindergarten teacher and mother of a 6th grader who wants to do a science project on the correlations of body part measurements. We heard this on the radio and it sparked our interest, but now I cannot find any information about it. Examples are: your height is supposed to equal the distance from fingertip to fingertip when your arms are outstretched. Your foot size is supposed to equal the distance from your wrist to your elbow.

Please help me find more information on this topic.
Answered by Penny Nom.

Linear models 1999-11-09
From Helen Moreno:
What is a linear model?
Answered by Harley Weston.
Multiple step problems 1999-11-09
From Maria Rainsdon:
I don't understand how to solve these multiple step problems. I wasn't there when my teacher explained it and I am really lost. Here is one of the probs I don't understand: 3m+6=-m-6.
Answered by Penny Nom.
Parabolic mirrors 1999-11-07
From Andy White:
I am working on a project concerning parabolic mirrors. I need to create a mirror to focus sunlight on a focal point, but I don't know how to do it. Is there some equation that tells where a focal point will be in relation to a parabola?

What is a directrix?
Answered by Penny Nom.

Mean, median and mode 1999-11-06
From Rick:
I have five number places and the mean and median are both "0". The mode is greater than the mean. The sum of the absolute values of all of the numbers is 20. The smallest number is 4 less than the next smallest number. What are the five numbers. (negative integers can be used) All of the numbers are integers.
Answered by Penny Nom.
Transforming a triangle 1999-11-02
From Wilder Maldonado:
The height, H, of a triangle is increased by a lenth M, How much must be subtracted from the corresponding base,b, so that the area of the new triangle is one-half that of the original triangle.
Answered by Penny Nom.
Trolls and Gargoyles 1999-11-02
From TexGrimm:
How can you seat 6 monsters - 3 Trolls and 3 Gargoyles- at a circular table if the trolls look alike and the gargoyles look alike? Does your formula work for 9 monsters - 4 trolls and 5 gargoyles?
Answered by Penny Nom.
Area of a circle and an inequality 1999-10-30
From Adam Anderson:
I have two problems.

The first: prove that the area of a cirlce is pi times radius squared without using calculus.

The second: show that ln(x) < x - 1 for all x > 0.


Answered by Harley Weston.

Calculating a square root 1999-10-28
From Jonathan:
What is the formula for calculating a square root?
Answered by Penny Nom.
Clockwise or Counterclockwise? 1999-10-27
From Tim:
A particle moves around the circle x2 + y2 = 1 with an x-velocity component dx/dt = y
  1. Find dy/dt

  2. Does the particle travel clockwise or counterclockwise around the circle? Why?

Answered by Harley Weston.
Derivatives with logs 1999-10-26
From Kate:
What is the derivative of 5 to the 5x-2 at x equals 0.8?
Answered by Harley Weston.
-log(a) 1999-10-22
From Brenda Miskimmin:
I need to know the mercury concentration in mg/L or ng/L for the following:

-log M (Hg) = 8.5

where mw of Hg=200.59

(it's the negative sign in front of log that confuses me).
Answered by Harley Weston.

Cutting a board into 8 equal pieces 1999-10-21
From Brittany:
Jamie wanted a board cut into 8 equal pieces. The Lucas Lumber Co. charges 60 cents for cutting a board into 4 equal pieces. How much will it charge for cutting Jamie's board?
Answered by Claude Tardif.
0! + 1! + 2! + 3! + ... + 2000! 1999-10-21
From Melissa:
My name is melissa. I am a 9th grade student I am having trouble finding out how to do this: What is the tens digit of 0! + 1! + 2! + 3! + ... + 2000! I know how to find these, but my calculator cant go any higher than 69! Is there any way i can do this problem?
Answered by Penny Nom.
|6-2x|<4 1999-10-20
From Tchilco:
Solve |6-2x|<4
Answered by Harley Weston.
Broken line graphs 1999-10-19
From Lori Samara, Kristi Cameron and the 4th grade students:
Students in Ms. Cameron's grade 4 class were wondering if a broken line can be used anywhere in the range on the y - axis when creating a broken line graph? They understand that the broken line can be used at the 0 on the broken line but what if there is a large gap in the data from 100 - 200? What do we do? The text book and mathematics dictionary do not answer these questions.
Answered by Penny Nom.
l'Hospital's Rule 1999-10-18
From Yannick Gigandet:
How can I solve these two limits :
  1. lim when n approches 1 of n[a1/n -1]

  2. lim when x approches 0 of (eax - ebx) / x
Thanks for the answer!

Answered by Harley Weston.
Working Backwards 1999-10-16
From Linda:
I am having problems finding examples of problems that require "Working Backwards" used as a strategy for solving. We are required to give a presentation on Monday, October 25, 1999 in our school districts math class. We are trying to become better problem solvers and how to teach problem solving in the elementary classroom. Help! I can't find anything in my web searches.
Answered by Penny Nom.
An equivalent decimal 1999-10-14
From Cheryl Wendlandt:
I have a question in my book that is one and five sixths and you are to put them in an equivalent decimal. I'm sick at home or I would be able to ask my instructor. I would appreciate any help.
Answered by Penny Nom.
(-5)^2, -5^2 and -(5)^2 1999-10-13
From Jennifer Brown:
What is the difference between the following problems:

(-5)2, -52 and -(5)2

Our text book (Beginning Algebra, fourth edition, published by McGraw Hill, by Streeter, Huthison and Hoetzle) says the second and third problem are exactly the same. I don't see how that can be. Is there a mathematical rule that explains this?
Answered by Penny Nom.

Find the missing numbers 1999-10-12
From Bonnie:
I must find the missing numbers and function rules

noutput
710
1220
3940
2430
16b
87c

we are totally confused can someone help
Answered by Denis Hanson.

A famous mathematician 1999-10-12
From Yvette Perez:
Another way to write 3/15. Remove 0 add a line, unscramble, you have the name of a famous mathematician.
Answered by Claude Tardif.
Isosceles triangles 1999-10-12
From Amber:
In defining the types of triangles, our class was stumped by a question asked by one of the student. Maybe you could help. The definition of an equilateral triangle is a triangle with three congruent sides. The definiton of an isosceles triangle is a triangle with at LEAST two congruent sides. The question is, if an isosceles triangle only requires at Least two of the sides to be congruent, could an equilateral triangle be called an isosceles triangle?
Answered by Penny Nom, Walter Whiteley and Chris Fisher.
Solving using logs 1999-10-11
From Rich Bolton:
Here's my question:

$541.39(1+i)15=784.09

Can you please show me how to do this?
Answered by Penny Nom.

Equations with fractions 1999-10-10
From Lori:
I'm a secondary student. How would you figure out a problem such as t-2/t =14/3t - 1/3. The book gave one example but it is really easy. The work YOU have to do is totally different. You don't have to tell me the answer just how to do such problems like these.
Answered by Penny Nom.
Length of a line 1999-10-10
From Dagmara Sarudi:
My question has to do with the length of a diagonal. This problem came up when I thought about the shortest distance between two points, for example walking from one point to another in my neighborhood. I can choose a zig zag route and assuming the blocks I walk are exactly the same length, it shouldn't matter what route I took, the distance I travel should still be the same when I reached my goal. If, on the other hand I could travel in a diagonal line, the distance would be shorter. But what if, in my zig zag motion, the sections get so small the route approaches a diagonal. Shouldn't it be that each separate section added together equals the value of the two original sides? Or would it suddenly equal the value of the diagonal (which, of course was shorter than the two sides added together)?
What gives?


Answered by Chris Fisher and Harley Weston.

Two Quadratics 1999-10-08
From Jason On:
Solve the following equations,expressing solutions in the form p + iq.
  1. sqr(x) + 2x - 1 = 0,
  2. sqr(x) + 2x - 3 = 0.

Answered by Harley Weston.
sin x = x/10 1999-10-07
From Amandeep Grover:
Solve the equation sin x = x/10
Answered by Harley Weston.
A trig limit 1999-10-06
From Yannick Gigandet:
What is the limit, as x approaches pi/3, of (1-2cosx) / sin(x-(pi/3)) ?
Answered by Penny Nom.
Palindrome 1999-10-06
From Chris Ware:
What is a non paladromic number?
Answered by Claude Tardif.
Logic and mathematical logic 1999-10-06
From Polly Mackenzie:
What is the difference between logic and math logic?
Answered by Walter Whiteley.
A googol 1999-10-06
From Kevin Brennan:
describe to me a guegoll is that how u spell it?

it sounds like that
Answered by Penny Nom.

Common factors 1999-10-06
From Tommygirl98:
Ok I have a question about common factoring I'm not to sure if you could help me but it's worth a try here it goes. Ok I have a test on Friday and i'm having trouble understanding how to do it here's an example,

Factor,
-10 + 2b how do you do it and what do they mean by factor?
Answered by Penny Nom.

The circumference of a circle 1999-10-05
From Mara Frost:
what is the formula to find the circumference of a circle, or if there is no formula, how do you find the circumference of a circle?
Answered by Penny Nom.
towers of cubes 1999-10-05
From Sanker:
I need help to solve this Rules for bulding towers of cubes
rule 1 The number of cubes on the bottom layer is always one less than the number of squares on the grid
rule 2 Each new layer is made with one cube less than the layer underneath it.
  1. Investigate how many different arrangements there are of 4 cubes on top of 5 cubes on a two by three grid

  2. investigate the number of different arrangements of six cubes on top of seven cubes on a two by four grid

  3. investigate the relation between the number of arrangements of cubes and the size of the grid
    • when there are two layers of cubes
    • when there are more than two layers of cubes

Answered by Walter Whiteley.
Regular and irregular shapes 1999-10-03
From Samuel Tighe:
What is the difference between a regular shape and an irregular shape? Are a rectangle and a triangle regular or irregular shapes?
Answered by Walter Whiteley.
Two limits 1999-10-02
From Jennifer:
How do I find

lim (1-cosx)/(x^2) as x-> 0

and

lim (tan3x)/x as x->0
Answered by Harley Weston.

A donkey and his carrots - part 2 1999-10-02
From Sarah Klink and Brenda Gamble:
Does anyone know the solution of the donkey who had to carry 3000 carrots to the market which was 1000 miles away? The donkey can only carry 1000 carrots at a time and he eats one carrot every mile he walks but this is only when he is carrying carrots. How many carrots can get to the market with the donkey carrying them?
Answered by Claude Tardif.
Temperatures 1999-09-27
From Eula:
How do you cahnge farenheit degrees to celsius degrees?
Answered by Penny Nom.
Dollar Bills 1999-09-27
From NRQT:
Why are 1999 American dollar bills worth more than 1997 American dollars bills? This has to be put into a mathematical problem???
Answered by Penny Nom and Claude Tardif.
Jose and Arthur 1999-09-26
From Joel Hammer:
Jose can run around the track in 40 seconds. Arthur runs with Jose but in the opposite direction. They meet every 15 second. How long will it take Arthur to make one trip around the track?
Answered by Denis Hanson.
Two 12-sided polygons 1999-09-25
From Kelly Boulton:
Two 12-sided polygons are similar. A side of the larger polygon is 3 times as long as the corresponding side of the smaller polygon. wHAT IS the ratio of the area of the larger polygon to the area of the smaller polygon.
Answered by Walter Whiteley.
English to french maths words translators 1999-09-22
From Steph Keeley:
Can you foward be any information re English to french maths words translators, as i am having problems finding one.
Answered by Penny Nom.
-8*-(-5a)=15 1999-09-22
From Shelby:
-8*-(-5a)=15

i cannot get this question!! please help me!!
Answered by Harley Weston.

Numbers with the digit 2 in 1...1000 1999-09-20
From Jessica:
Is there a trick to finding out how many numbers containing the digit two is there from 1 to 1000?
Answered by Walter Whiteley.
Distance between the windows 1999-09-19
From Lawrence:
An observer on level ground is at distance d from a building. The angles of elevation to the bottom of the windows on the second and third floors are a and b respectively. Find the distance h between the bottoms of the windows in terms of a b and d
Answered by Harley Weston.
Fractions, percents and decimals 1999-09-19
From Sue Farr:
My son is in 5th grade, and I can't seem to find any papers that have a simple explanation of how to convert decimals into fractions, percents into decimals, and vice-versa.
Answered by Harley Weston.
Repeating decimals 1999-09-18
From Kavoos Bybordi:
I dont know how to change a reccuring decimal to a fraction please could you tell me the method.
Answered by Penny Nom.
The greatest possible difference 1999-09-17
From Denea Hugunin:
Use each of the digits 1 to 5 once to make a 2 digit number and a three digit number that the greatest possible difference.
Answered by Penny Nom.
2 to the x and x squared 1999-09-17
From John:
For what values of x is 2 to the exponent x greater than x squared?
Answered by Harely Weston.
Bob and his dad 1999-09-14
From Yvette:
"Bobs dad is 27 yrs older then Bob. The sum of their ages 5 yrs ago was 45 yrs. How old is Bob now?"
Answered by Penny Nom.
Stem and Leaf Plot 1999-09-14
From Jeanette Sovick:
My 5th grade son brought home a math paper, the title of which reads, Reading Stem-and-Leaf Plots...can you explain this so I can explain it to him...There is no book, his teacher just sent this practice sheet home for him to complete and I have no clue!
Answered by Penny Nom.
Six toothpicks 1999-09-14
From Maddie:
The book asks: Using 6 toothpicks represent the number 17. I am studying expressions to represent a number. ex.) (2+4)4x4=96.
Answered by Penny Nom.
Two Jugs 1999-09-14
From Kynadi Kemp:
If you have and 8 liter and a 3 liter jug and you want exactly 4 liters how do you get it if there is a water hose near by but thats all you have to get the 4 liters how do you do it ?????
Answered by Penny Nom.
Help with three mathematicans! 1999-09-14
From Ashley Tacheira:
I am a sophomore in high school and have been assigned a research paper on over thirty mathematicians. I was able to find information on all but three. I need to know the birth and death dates and major contributions of these three mathematicians. Any suggestions on where to look would be appreciated. George Stigler, T.C. Koopmans and Thomas Guthrie are the three that I cannot seem to find.
Answered by Chris Fisher.
Bales of hay 1999-09-13
From Ivy:
You are given 5 bales of hay. Two bales are weighed at a time, which equal the following weights:
110, 112, 113,114,115,116,117,118,120,121. What does each individual bale weigh?

Answered by Harley Weston.
Roman Numerals 1999-09-13
From April:
Write MCDXCVII in arabic numbers
Answered by Penny Nom.
Rolling Circles 1999-09-12
From Craig Ellis:
We have a circle of radius 3. inside the circle and tangent to the circle of radius 3 at one point is a circleof radius 1. The question is if we could roll the smaller circle around the inside of the larger circle how many revolutions would it take to get around to where we started.
Answered by Chris Fisher and Walter Whiteley.
2,5,8,1 yields 24 1999-09-12
From Terry Hill:
I was wondering how do I make the numbers 2,5,8,and 1 equal 24?
Answered by Penny Nom.
Degrees and triangles 1999-09-09
From Sandra Mills:
Are there any triangles which are not 180 degrees?

I am also in need of information on the history of degree measure for an angle.
Answered by Walter Whiteley.

Four fours 1999-09-09
From Roger:
I need help with a math problem my child asked me about I guess in her 7th grade math class they were told to come up with a answer from 1-10 only using four 4's and she got stuck on the problem that needs to be equal to ten, she asked me and I couldn't help her it's sad so if you could give me the problem and and answer so i could explain it to her I would really appreciate it.
Answered by Penny Nom.
Using Factoring in Problem Solving 1999-09-09
From Lori:
I have a problem with a problem. The Problem is....The sum of the squares of two consecutive positive even integers is 340. Find the integers.
Answered by Penny Nom.
Towers of Hanoi 1999-09-03
From Jeni:
In the puzzle called the Towers of Hanoi there are three peg and you are asked to move the rings from one peg and stack them in order on another peg. You can make as many moves as you want, but each move must consist of moving exactly one ring. Moreover, no ring may be placed on top of a smaller ring. The minimum number of moves required to move n rings is 1 for 1, 3 for 2 rings, 7 for 3 rings, 15 for 4 rings, and 31 for 5 rings. Find a formula for this sequence. What is the minimum number of moves required to move 6 rings?
Answered by Penny Nom.
Notation 1999-09-03
From Judy Gray:
My name is Judy Gray. I guess the level on this would be about middle school, and I am an other!! I've been out of school for alot of years. I have a couple of questions, I guess.
  1. What does an equal (=) with a slash through it mean? I put intersects, but I think I'm wrong.

  2. What is the symbol of a small n? Only the symbol is wider. Example: If A = {1.2} B = {2,3} then what is A (n) B? Can you make sense out of that?

  3. What is the equation of the y-axis?

Answered by Penny Nom.
The three jugs problem 1999-09-02
From Kent Lane:
I wonder if you could help me figure out this secondary Discrete Math Problem. I cannot figure out where this comes from.

Here's the question:

You have three containers. Container 1 is a three (3) liter container. Container 2 is a five (5) liter container. Container 3 is an eight (8) liter container that is full of liquid. The goal is to get 4 liters in one of the containers.

Stipulations: There are no marks on the containers to measure out the liquid.

All you know is that 1 is 3 l., 2 is 5 l., and 3 is 8 l. full of liquid.
Answered by Chris Fisher.

A double negative 1999-09-01
From Dennis:
If b = -2 what does -b = ? As in (a + 8.5) - [(-b) + |c|] a = 1.5, c = -1.7
Answered by Penny Nom.
A Multiple Choice Test 1999-08-31
From Cintra Ramnarine:
I have ten questions. There are two multiple choice answers to each question. What are the chances of answering all questions correctly.
Answered by Harley Weston.
The Bellhop Problem 1999-08-29
From Gerard O'Neill:
Three fellows going to a Math Conference book into a motel. "That will be $30.00" says the bellhop. Each fellow gives $10.00 and they go off to their room. The manager enters and says "That was a $25.00 room. Go and give them $5.00 back."
The bellhop heads off but thinks....."how am I going to split $5.00 between 3 fellows? Hey I will keep two dollars and give each guy a dollar." He does just that.......each fellow now paid $9.00 and the bellhop kept two. 3 X 9= 27 plus the bellhops two = $29.00 Where is the missing dollar?

Answered by Harley Weston.
A Trigonometry Question 1999-08-28
From Diane Simms:
My question is can the following be factored. I am a teacher who needs the factors to this right away. 2 Sin2X + 2 SinX CosX - 1= 0
Answered by Harley Weston.
Invert and multiply 1999-08-28
From Debbie Walter:
I have another question please, WHY do we invert and multiply when dividing fractions? I know that's what we do but WHY? What is the reasoning behind it?
Answered by Penny Nom.
Crossing the Bridge 1999-08-27
From Debbie Walter:
If you have four people who need to cross a bridge in the dark with only one flashlight and only two of them can cross at a time and their speeds of crossing are as follows:
  • one person can cross it in one minute
  • one person can cross it in two minutes
  • one person can cross it in five minutes
  • and the slowest person can cross it in ten minutes
what is the shortest amount of time it would take to get all four across the bridge?

Answered by Penny Nom.
The sum of the cubes is the square of the sum 1999-08-25
From Bernard Yuen:
How to prove 13 + 23 + 33 + 43 + ... n3 is equal to (1+2+3+...n)2? (for n is positive integer)
Answered by Harley Weston.
A gazillion 1999-08-21
From Leah:
Does a gazillion really exist?
Answered by Penny Nom.
Why is slope designated m? 1999-08-18
From Peter Komlos:
Why is the slope of a line is designated by the letter "m"?
Answered by Penny Nom.
Perimeter 1999-08-15
From Mel Nordan:
The length of a rectangle is IS 3 1/2 times its width. Its perimeter is 108cm. Find its length and width. Choices for the width: 12,18,24
Answered by Penny Nom.
111...1222...2 1999-08-11
From Brad Goorman:
Let N = 111...1222...2, where there are 1999 digits of 1 followed by 1999 digits of 2.
Express N as the product of four integers, each of them greater than 1.

Answered by Penny Nom.
Parametric Equations 1999-08-06
From Nicholas Lawton:
Show that an equation of the normal to the curve with parametric equations x=ct y=c/t t not equal to 0, at the point (cp, c/p) is :

y-c/p=xp^2-cp^3
Answered by Harley Weston.

From an airport control tower 1999-08-04
From Pammy:
Hi I am a 30 yo mature age student doing my HSC but am having difficulty understanding this, if you can help me.

From an airport control tower, a Cessna bears 023 degrees T and is 27km away. At the same time, a Boeing 767 bears 051 degrees T and is 61km from the tower. Both planes are at the same height.

i) What is the size of angle ATB?

ii) Using the cosine rule to calculate the distance the planes are apart, to nearest kilometre.

I figured out and drew the triangular diagram but can't figure out the rest and which formula to use. sorry about this, thankyou kindly
Answered by Harley Weston.

Cognitive development as related to learning mathematics 1999-08-04
From Chris Ahlman:
I am doing an independent study for my Math theory course at Evergreen State College, WA. I am looking into teaching Math to elementary students "as a language". I need information on cognitive development to tie to this idea of language development. I only know of Piaget. There must be others who are more recent. Can you help?
Answered by Walter Whiteley.
Cleaning an Ellipse 1999-07-29
From Mark Salter:
Hello hope some one can help. We need to clean an elipse and then paint it. We need to know the square foot of the job and the job is an elipse which rises 2 ft. is 12 ft wide and is 36 ft long.
Answered by Harley Weston.
A calculus problem 1999-07-22
From Nicholas Lawton:
The curve y= e^x(px^2+qx+r) is such that the tangents at x=1 and x=3 are parallel to the x-axis. the point (0,9) is on the curve. Find the values of p,q and r.
Answered by Harley Weston.
Infinity Symbol 1999-07-13
From Mark E. Kelly:
There is a symbol that looks like a sideways 8 that is used to represent infinity. Does it have a name?
Answered by Doug Farenick and Penny Nom.
Standard Deviation 1999-07-11
From Anthony Fama:
I have seen several answers to this question: If one standard deviation represents 68% of the population, what does two, three, four and five sigma [std deviation] represent? As stated, I have seen several different answers and thus, the impetus for my question.
Answered by Harley Weston.
The number of acres in a square mile 1999-07-09
From Rita Murphy:
What is the # of acres in square mile
Answered by Harley Weston.
Compound Interest 1999-07-01
From Kim Palmer:
In early 1997, my son borrowed $4831 at 7.5%. He has made 30 monthly payments of $130 each. He is now in a position to pay off the balance. What is his remaining principal?
Answered by Penny Nom.
The shortest ladder 1999-06-26
From Nicholas:
A vertical wall, 2.7m high, runs parallel to the wall of a house and is at a horizontal distance of 6.4m from the house. An extending ladder is placed to rest on the top B of the wall with one end C against the house and the other end, A, resting on horizontal ground. The points A, B, and C are in a vertical plane at right angles to the wall and the ladder makes an angle@, where 0<@
Answered by Harley Weston.
Scheduling Meetings 1999-06-25
From Beth Carver:
We have 6 groups meeting at three different places simultaneously. The meetings take place once a month each year. Is there a way to have each group meet with each of the other groups at least twice in a year, four times in each place?
Answered by Chris Fisher and Harley Weston.
Computer use in mathematics classrooms. 1999-06-23
From Rosemary Greer:
Can you give me any information, either articles or the names of books, concerning the use of computers in the math classroom?
Answered by Lillian Perivolaris.
Sin 4A 1999-06-22
From Ryan Cochrane:
If sinA = 4/5, and A is a first quadrant angle, find sin4A
Answered by Harley Weston.
A Weighty Problem 1999-06-17
From A parent:
When using a balance scale, weights can be placed on either side of the scale. For example, if a 10 pound weight provides a counter balance to an object and a 7 pound weight, then the object must weigh 3 pounds. What four weights can be used to weigh objects of 1, 2, 3 ... 38, 39, 40 pounds?
Answered by Penny Nom.
Even and Odd Function 1999-06-17
From Kent:
There is one function with the domain of all real numbers that is both even and odd. Please give me the answer to this question before I go insane.
Answered by Penny Nom.
Liquid capacity 1999-06-15
From Bernard Antill:
Would you please tell me how many litres of water in a swimming pool that is 24 feet long, 16 feet wide & 4 feet deep. I would appreciate you showing the formula for this calculation. I am a 74 year old male and I cannot find a formula in which I have any faith! I would also appreciate the answer in Imperial gallons.
Answered by Penny Nom.
Slopes of curved lines 1999-06-09
From Stephen Ehrler:
When one plots the graphs of y=2x, y=3x, y=xx When each of these graphs pass through point (0,1) do they have the same slope? I know they are different lines but is it possable that they have the same slope at point (0,1).
Answered by Harley Weston.
Measuring the fourth dimension 1999-06-07
From Suzanne Hall:
Hi! The other day, two of my students asked a very interesting question: Is the fourth dimension measured with hypercubes? Their reasoning went like this: Lines are 1D and are measured with line segments, which are part of a line. Planes are 2D and are measured with squares, which are part of a plane. Space is 3D and is measured with cubes, which are part of space. So, logically, hypercubes would be used to measure the fourth dimension.
Answered by Chris Fisher.
The three cousins 1999-06-01
From Irena:
I have three cousins on my dad's side. The product of their ages is 84. Next year the youngest will be 1/4 the age of the middle cousin. In 2 years from now, the oldest will be twice the current age of the middle. What is the age of my oldest cousin?
Answered by Harley Weston.
A problem with powers 1999-06-01
From Bill:
solve for the variable:

5-x = 25x+6
Answered by Penny Nom.

How to carpet a room 1999-05-31
From Appleby:
A room which is 9X12 is to be covered with carpet but the carpet has been provided in one 8X1 piece and one 10X10 piece. The larger piece is to be cut into two pieces so that the room can be covered in carpet.
Answered by Stacey Wagner.
An Invalid Argument 1999-05-31
From Rod Redding:
Can an invalid argument have a true conclusion? If yes then why?
Answered by Walter Whiteley.
Graphing a linear equation 1999-05-27
From Dylan Bradley:
Im in Grade ten and am doing linear equations, I cann't figure out how to make a chart for questions like y = -2x + 3
Answered by Penny Nom.
A circle in a square 1999-05-26
From Jose V Peris:
A circle is inscribed in a square. The circumference of the circle is increasing at a constant rate of 6 inches per second. As the circle expands, the square expands to maintain the condition of tangency.

find the rate at which the perimeter of the square is increasing.

find the rate of increase in the area enclosed between the circle and the square at the instant when the area of the circle is 25(pi) square inches.
Answered by Harley Weston.

Girth 1999-05-26
From Carolyn Bulkley:
I am trying to explain to my son (who is in the 8th grade) how to figure girth. I'm afraid I have just confused him. Is there a simply formula to figure the girth of a box.

for example: I have a box that is 27" L X 22" W X 21" H.
Answered by Penny Nom.

Repeating decimals 1999-05-21
From Stan:
Hi, I am in Honors Math, and have confronted everyone, including teachers, about repeating decimals. What interests me is the number 0.9... and 1. Everyone says that since there is no number between 0.9...(repeating) and 1, that 0.9... = 1. However, isn't a repeating number a representation of a number, and not a real number? Let's look at it this way. 0.9 is close to 1. 0.99 is closer. 0.99999999999999 is even closer. so, 0.9... is a representation of it's closeness to 1. it's an active number... I don't understand how 0.9... is equal to 1. Please help me prove that 0.9... does NOT = 1.
Answered by Penny Nom.
Introductory Algebra 1999-05-19
From Pat:
(2 + sq. root of 3) x (2 - sq. root of 3) = 1

Please show me the work.
Answered by Harley Weston.

Three Daughters 1999-05-18
From Norm Campbell:
I recently received a beat up copy of the three daughters question. I have only some of the question. It ends after the statement "the oldest one looks....... It does not finish. Is there any chance that someone can finish it off for me?
Answered by Rick Seaman and Harley Weston.
Circles, cirmcuference and area 1999-05-16
From Stephen Ehrler:
I would appreciate if you could please tell me if what I discovered here is something or my ignorance? I noticed that a circle with r radii has the folling characteristic.

r = [2 * ( pi * r2 / pi * 2r)]

The equation states that the ratio of a circles area over its circumfrence = 1/2 that of the circles radii. It works every time. Did you know this ? Is it some kind of therom and can it be used for any thing? I thought this was intresting and would appreciate any input you may have.
Thank you.
Answered by Chris Fisher.

Related rates 1999-05-13
From Tammy:
The sides of a rectangle increase in such a way that dz/dt=1 and dx/dt=3*dy/dt. At the instant when x=4 and y=3, what is the value of dx/dt? (there is a picture of a rectangle with sides x and y, and they are connected by z, which cuts the rectangle in half)
Answered by Harley Weston.
Missing the boat 1999-05-11
From Judith Moe:
Our daughter is in Grade 3 and seems stubbornly determined NOT to learn how to add and subtract. She's been assessed as quite intelligent in reading and writing,...Do you know of any alternative approaches to teaching/learning math that are proving useful in such circumstances.
Answered by Diane Hanson.
A log problem 1999-05-08
From N. Lam:
I don't understand how to do this type of question:

log(base absolute value x)y = log(base abs.val.x)2 + log(base abs.val.x)(Cos x)

How do you graph the equation also, if -2(3.14)<=x<=2(3.14)
Answered by Harley Weston.

Equivalence Relations 1999-05-06
From Megan:
I am tutoring a boy who got this assignment from his teacher and I have no clue how to do it because I don't even know what the questions is asking! I need some help. Hereit is: "Give five examples of relations which are not equivalent relations and five examples of equivalent relations and explain why they are equivalent relations." This is seventh grade and I read about it in his book but it is not coming together for me.
Answered by Chris Fisher.
A Polar Plot 1999-05-06
From Irene:
Consider the polar equation r=2-3Cos(theta/2) In the interval [o, 4Pi], how would you find the area of one of the leaves and also the length of one of the edges of a leaf?
Answered by Harley Weston.
Shape-preserving transformations 1999-05-04
From J McAndrew:
A shape preserves its shape if a rotation, translation or scaling is performed on it. Are these the only continuous transformations which have this property? These transformations if performed on the parts and then summed have the same effect as the transformation being applied to the whole; are these linear transformations? Who, and what area of mathematics has classified all transformations of this type completely?
Answered by Chris Fisher.
Parabolic shapes 1999-05-04
From Justin Ailor:
Can you give me some parabolic shapes?
Answered by Penny Nom.
A 1999-05-02
From Leah:
a=b
a^2=ab
a^2+b^2=ab-b^2
(a-b)(a+b)=b(a-b)
a+b=b
b
2=1

why is this proof wrong?
Answered by Penny Nom.

Roman Numerals 1999-04-29
From Michelle Jenkinson:
Someone proposed this question to me and I do not know the answer, so I was wondering if you could help. How, using Roman Numeral, did people add, subtract, multiply, and divide with no zero or negative numbers?
Answered by Penny Nom.
An equilateral triangle on a square 1999-04-26
From Ed:
My Grade 8 class and I were discussing the solution to the following problem:

What is the area of the largest equilateral triangle that can be drawn on a 5 cm square.

We used 5 cm as the base of our triangle and then drew the other two legs of 5 cm each to make the equilateral triangle. We then drew an altitude from the upper vertex to the base of the triangle. Using the law of Pythagoras with side a of 2.5 and side c of 5 we calculated side b to be 4.3 cm (the altitude). Therefore the area of the triangle would be 5 x 4.3 divided by 2 or 10.75 square cm.

The answer key to this resource says I am wrong.

What do you think? Have we interpreted the question incorrectly?
Answered by Chris Fisher and Harley Weston.

Question about 3rd degree polynomials 1999-04-23
From Patrick Bryan:
What is the general solution to the equation with the form:

a*x^3 + b*x^2 + c*x + d = 0

I have once seen a solution to this a few years ago, but I do not recall if it was a general solution. What I do know, is that you could simplify this equation to:

a*x'^3 + p*x' + q = 0...
Answered by Doug Farenick.

Parallel and perpendicular lines 1999-04-23
From Crystal Pilling:
My name is Crystal Pilling and I am in 9th grade algebra. We are currently studying parallel and perpendicular lines. I am having trouble with this problem: 3/4x - 5y= 16, (5,-6) I have to find a line that is perpindicular to this line on a graph.
HELP ME PLEASE!!!!!

Answered by Penny Nom.
Bill and Sam at the Casino 1999-04-23
From Rham Stewart:
Bill and Sam went off to bet at the casino. Each started with the same number of dollars. At the end of the first hour, Bill had won 20$ and sam had lost 20$. At the end of the second hour, BIll had lost two thirds of his money, and Sam had won the same amount that Bill had lost. At that point, sam had four times as much money as BIll. How much did each one start with?
Answered by Penny Nom.
Radius of an arc 1999-04-22
From Rusty Riddleberger:
I need to find the equation for finding the radius of an arc; I know the length of the arc (i.e the distance of the line connecting the two ends of the arc) and the height; (i.e the rise of the arc at its apex,) I had the formula years ago but it has lost me; this would be invaluable for work in new homes i.e. where we need to build an "arch" with a rise of 21" between two columns 11 feet apart
Answered by Chris Fisher.
Shopping at Wegman's 1999-04-22
From Stan:
Joanne, Steve, Pat, Alice, Joan, and Bill go to Wegman's at the same time. Joanne buys 2 gal of milk, 1 dozen oranges, 8 apples, and 2 lb. of ground beef, paying a total of $13.24. Steve buys 3 qts. of milk, 5 lbs. of ground beef, 10 lb. of potatoes, and 2 bags of mixed vegetables, paying a total of $16.95. Pat buys 3 gal. of milk, 2 dozen oranges, 1 dozen apples, 5 lb. of potatoes, and 5 bags of mixed vegetables, paying a total of $25.09.
....
Find the cost of: 1 qt. of milk, one orange, one apple one lb. of ground beef, one lb. of potatoes, and one bag of mixed vegetables.

Answered by Penny Nom.
A ladder problem 1999-04-22
From Michael Blade:
There is a cube box 3feet x 3feet x 3ft resting against a vertical wall on level ground. Resting against the outside corner of the box is a ladder 10 feet tall, this ladder is of course resting on the ground but also against the outside corner of the box and rests on the wall.

The question- the ladder is divided into two unequal section bounded by the box to the ground and the box to the wall. what are those dimensions?
Answered by Penny Nom.

Area of a triangle from vertex coordinates 1999-04-21
From Mark Tyler:
I'm no schoolkid, but I liked your answers about triangles. You might enjoy a quick look at this, the kids may too.

I was working on a Voronoi dual where I had to calculate the areas of very many triangles expressed as vertex coordinates, so I derived the following very direct formula:

A = abs((x1-x2)*(y1-y3)-(y1-y2)*(x1-x3)) for triangle (x1,y1)(x2,y2)(x3,y3)

I've never seen this in a textbook. Is it original? I doubt it, the proof is only a few lines long.

Regardless, it may be fun for the kids, even if it's not on the curriculum.
Answered by Walter Whitley.

Radius of convergence 1999-04-21
From Nowl Stave:
Why is the radius of convergence of the first 6 terms of the power series expansion of x^(1/2) centered at 4 less than 6?
Answered by Harley Weston.
Circles 1999-04-21
From Alex Elkins:
How do you find the circumference of a circle if you only know the radius and the square feet or inches of the circle if the radius is 18 inches, If done in inches do you multiply by 12 to get the square feet?
Answered by Jack Lesage and Harley Weston.
The average rate of change of a function 1999-04-20
From Tammy:
Suppose that the average rate of change of a function f over the interval from x=3 to x=3+h is given by 5e^h-4cos(2h). what is f'(3)? I would appreciate any help with this question.
Answered by Harley Weston.
Unit conversions 1999-04-20
From Deanna:
My question has to do with unit conversions with English and metric systems. I need to know how to use conversion factors with powers of 10. Which way do i move my decimal point. ex. .48kl to liters. Thank you.
Answered by Jack LeSage.
A Series 1999-04-20
From Deepak Shrestha:
Given the sequence an=e^(-n*Ln(n)), does the series converge and why?
Answered by Harley Weston.
The halfway mark 1999-04-20
From Sherry & Tom McGee:
How do you figure the halfway mark between 1/3 and 2/3 as a fraction and a decimal point?
Answered by Penny Nom.
The birthday problem 1999-04-19
From Gordon Cooke:
How do I explain the rapid rise in the probability that at least two people in a group of n have the same birthday. We have derived the formula for p(n) and have graphed it and have seen how the results are counter-intuitive. At around n=23 p(n)=.5 and at n=50 p(n) is very close to 1. It does not help to simplify the problem (eg use months instead of days) because then our intuition does correspond more closely to reality. Is there some way we can see how the probabiltiy of a "collision" increases with n? It makes me think of data storage problems and hash tables in computer science.
Answered by Harley Weston.
Work jobs 1999-04-19
From Cindy Miller:
Hi. I'm a college student, majoring in Elementary Education. This semester I am taking a math class for elementary school teachers.

I'm not sure if you can help out, but here is what I need. For the class, I am supposed to make 10 work jobs for students. Simple things that teach math that students can work on individually, for example after finishing a test.

If you have any suggestions, I would really appreciate it.
Answered by Penny Nom.

Volume of oil in a tank 1999-04-17
From Lars Waldemarsson:
My problem is to get an equation for the depth of the oil in a gastank formed like a cylinder. The cylinder is in a horizontal position and by a stick you will be able to get the depth of the oil in the tank. All I need is an exmaple which I can build on. By this equation you will be able to get the volume of the oil if you know the depth.
Answered by Harley Weston.
Ounces and milliliters 1999-04-17
From Tony Rizzi:
I'm mixing a formula of:

48 oz distilled water.
15 grams of Hydroxy Ethyl Cellulose GR (powder)
30 grams of Hydroxy Ethyl Cellulose MR (powder)
200 grams of Barium Sulfate (powder)
1000 grams of Calcium Hydroxide (powder)

After mixing, I need to estimate the amount of each ingredient that fits in a 3ml syringe. And how many syringes I can fill?
Answered by Harley weston.

y=mx+b 1999-04-15
From Selena Royle:
When solving a problem on slopes,and in the formula y=mx+b what do the M and B stand for? Why M and B?
Answered by Penny Nom.
Extra point questions 1999-04-15
From W Troy Council:
My teacher has given me 5 extra point questions. I have figured out two, but i need a little assistance with 3 others, could you help me with:
  1. A stablehand agreed to work for 1 year. At the end of that time, she was to receive $240 and one horse. After 7 months she quit the job, but still received the horse and $100. What was the value of the horse?

  2. If the roots of x2 + bx + c are pi and square root 2, then find "b" and "c."

  3. ...

Answered by Penny Nom.
The Binomial Theorem for rational exponents 1999-04-15
From Angela Evans:
The full question is this: Isaac Newton generalized the Binomial Theorem to rational exponents. That is, he derived series of expansions for such expressions as (x+y)-3 (x+y)2/3 (x+y)5/6 What did Newton find? What are the first four terms of the series expansions of binomials above? How can this extended Binomial Thrm. be used to aid in calculations?
Answered by Penny Nom.
Two contest problems 1999-04-14
From Bruce Baldwin:
We have students that are preparing for the Pythagoras Contest which is a nation wide Grade 6 math challenge.

In the preparatory tests we have run into several questions that we can not understand.

Is there anyone who can help us?

  1. If 1 * 9 = 0, 9 * 8 = 72, 2 * 8 = 9, then 9 * 9 = ?

  2. ...

Answered by Judi McDonald and Walter Whiteley.
Dividing a Circle 1999-04-12
From Mike Kenedy:
I am having trouble with a homework question for bonus marks. A Circle is continually divided by lines that do not intersect the center so that they produce the most pieces of circle. For example
  • 1 line divides the circle into 2.
  • 2 into 4.
  • 3, however into 7.
  • 4 into11
  • 5 into 16
  • 6 into 22
  • 7 into 29
  • 8 into 37
  • etc...
I am stumped and cannot figure out the equation, though I'm sure it involves squares. Can you help?

Answered by Penny Nom.
Cross-sectional area of a cylinder 1999-04-12
From Rebecca Connor:
What is the formula to find the cross-sectional area of a cylinder?
Answered by Chris Fisher.
Linear programming and optimization 1999-04-09
From Shams:
What is Linear programming and optimization?
Answered by Jack LeSage and Penny Nom.
Grade 10 math 1999-04-08
From Ron:
the type of question i'm having trouble with is 5(2x-1)=3(x-4)
Answered by Penny Nom.
Dotted graph paper 1999-04-08
From Bridget Winward:
A teacher at our school is trying to locate dotted graph paper online or in print. His class would like to make three dimensional, geometerical drawings.
Please let us know if you have a good source.

Answered by Jack LeSage.
Converting Metric Measurements 1999-04-08
From Rick:
Do you have any simple explanations for converting Metric measurements to other Metric measurements?? What helpful hints do you also have for a 5th Grader...
Answered by Penny Nom.
Universal Area 1999-04-08
From Karen Richardson:
I need a formula for area that works for a square, a rectangle, a parallelogram, a trapezoid, and a triangle.
Answered by Jack LeSage, Chris Fisher and Harley Weston.
Instant Winner 1999-04-07
From FSSTAN:
The probabilities of being an "instant winner" of $25 or $50 in a lottery are 1/600 and 1/1200, respectively. The mathematical expectation of being an "instant winner" of $25 or $50 is?
Answered by Jack LeSage and Penny Nom.
Area of an irregular shaped objects. 1999-04-07
From Jesse Townsend:
How do I find the surface area of an irregulary shaped object such as someone's knee from the thigh to shin?

Thanks.
Answered by Walter Whiteley.

Large Numbers 1999-04-05
From CK:
Hello. I need a lesson plan for grades 4 to 7 dealing with the concept of large numbers. Specifically, how to teach scientific notation. The lesson plan has to deal with the following: the size of the universe is so huge that is is almost beyond the ability of the mind to comprehend. One way is to measure...
Answered by Chris Fisher.
Volume of a cube. 1999-04-04
From Alex Elkins:
How do you find the volume cubic matter of a cube or space, example how many cubic feet in an example 6" high 4' X 6' rectangle

Thank You
Answered by Penny Nom.

Finding Excluded Values 1999-04-01
From Lorraine Le Cam:
The question asks me to find the value of the variable that must be excluded in each rational expressionion. The operation is:

a
______
a - 2

How do I work this out?
Answered by Harley Weston.

Quadratic equations 1999-03-31
From Ken Niebergall:
#1 a2 - 10a = 2a - 36

#2 t(t - 5) = 5 (t - 5)

#3. (x + 4)(x - 3) = 8

Thanks.
Answered by Jack LeSage and Penny Nom.

Factoring 1999-03-30
From Maggie Stephens:
I don't know anything about factoring would you plese help me.

3x4 - 48

54x6 + 16y3

125-8x3

12x2 - 36x + 27

9 - 81x2

a3 + b3c3

I would greatly appreciate any help you can give me thanks.
Answered by Jack LeSage.

Factoring polynomials 1999-03-29
From Karen:
I have been unable to factorise a polynomial equation and was wondering if you could please help. It is level (10-12) maths. The polynomial is x3 + x2 - 24x + 36
I have tried a few factorisation methods such as foctorisation by grouping but it won't work this polynomial. Please help.

Answered by Jeff Walters and Jack LeSage.
Resources for real-world math activities 1999-03-26
From Kate O'Brien:
Where is there a collection of math acitivities or projects to use in high school Algebra I, Algebra II, or Trigonometry that tie concepts to real-world careers?
Answered by Jack LeSage.
Cubic feet and gallons 1999-03-26
From Karen Coheley:
How many gallons in a cubic ft. ??
Answered by Harley Weston.
Complex numbers/polar coordinates 1999-03-25
From Kate Cegelis:
What is the relationship between complex numbers and polar coordinates?
Answered by Harley Weston.
A word problem 1999-03-25
From Anna Gariss:
A second number is 4 less than a first number. The product of the second number and 3 more than the first number is 18. Find the pair of numbers.
Answered by Jack LeSage and Penny Nom.
Probability and Statistics 1999-03-25
From Karrie Waller:
Hi my name is Karrie and I am an elementary ed. student at Florida State University in Tallahassee, Florida. I am doing a report on how children learn probability and statistics. I am having trouble locating information that will tell me specifically how students learn. I am wondering if you can answer this question for me or direct me in the right direction.
Answered by Jack LeSage.
A field trip 1999-03-18
From Jennifer Rudd:
This word problem is driving me nuts! I could really use your help. Its a secondary level question. Here it is:

A group of students planned to charter a bus for a school field trip. Each student was to share the $810 cost equally. However, 3 people were unable to go and this increased the share of each person by $3. How many went on the trip?

Thanks!
Answered by Jack LeSage and Penny Nom.

0/11 1999-03-09
From Jacob:
What is the term applied to 0 divided by a number.

In calculating slope problem my answer was 0 divided by 11. I don't know what to do with this answer or how to interpret it.
Answered by Jack LeSage and Penny Nom.

Factoring 1999-03-08
From L. Sivad:
Question:
m2+6m+9-n2

Answered by Penny Nom.
An airplane problem 1999-03-08
From B.M.R.:
A plane left New York and headed East to its destination 3600 miles away across the Atlantic. On the way back its speed was boosted by a 50 mph tail wind and it arrived an hour early. What was its normal speed?
Answered by Jack LeSage.
Metric Conversion 1999-03-07
From TDestra:
Please help to answer this question: Is a cubic centimeter equivalent to milligrams or milliliters, and how many? Also, is it a liquid measure?
Answered by Penny Nom.
Where three sequences meet. 1999-03-06
From Ali:
My name is Ali and I am in the 5th grade.

I have a math question:

What is the least positive integer meeting each of the following conditions:

  • Dividing by 7 gives a remainder of 4
  • Dividing by 8 gives a remainder of 5
  • Dividing by 9 gives a remainder of 6
Help!

Answered by Denis Hanson.
Reversing the direction of an inequality. 1999-03-06
From Mallory White:
If the Problem was -4a plus -5 is less than or equal to 14, why would you change the sign to greater than or equal to?
Answered by Jack LeSage and Harley Weston.
Enlarging a Rink 1999-03-06
From Jennifer Rudd:
I've been having difficulty with this one question involving area. The level of the question is Grade 11/12. Here it is!
A rink is 40 m long by 20 m wide. There are plans to enlarge it by 700m2 by adding a strip at one end and a strip of the same width along one side. Find the width of the strip. (Let the width of the strip by x meters.)

Answered by Jack LeSage.
The square root of two is never supposed to end 1999-03-06
From a wondering student:
i am algebra II and am in the 9th grade. today we were talking about rational and irrational numbers. When we were talking about square roots my friend and i were talking and we thought of something. if you have a square with sides of length one then the diagonal of the square is the square root of 2. Now the square root of two is never supposed to end. But the diagonal of the square ends so therefore doesn't the square root of 2 end. our math teacher did not really answer our question because it was not in the lesson plan and not to many people would see where we were coming from. the answer is really bugging me and i would like to have your input.
Answered by Jack LeSage and Penny Nom.
Converting mm's to inches 1999-02-22
From Paul White:
It has been a long time since high school and I do not remember how to convert mm to inches. Could you please tell me what this would convert to in inches? 210 X 254 mm.
Answered by Chris Fisher.
Divisibility by 9 1999-02-21
From Razzi:
I've been having a hard time trying to solve the following problem and I was wondering if you could help me.

For any positive integer a let S(a) be the sum of its digits. Prove that a is divisible by 9 if and only if there exist a positive integer b such that S(a)=S(b)=S(a+b).
Answered by Chris Fisher and Harley Weston.

Finding a rule for a sequence 1999-02-17
From Lindsey Masters:
I'm doing a maths investigation and i have a sequence which goes:-

13,16,25,32,45,56,73.

Our teacher told us we have to find a rule by looking at the differences of the terms until we find a constant. The first differences are:-

3,9,7,13,11,17.

The differences of these are:- ......

Please could you tell me how to work it out so that I could work out the rules of similar sequences.
Answered by Penny Nom.

A palindrome 1999-02-16
From panajoti:
Find the smallest number that must be added to 70808 so that the digits would read the same backward or forward.
Answered by Penny Nom.
Four Corners Maths Problem 1999-02-16
From Helen Williams:
I am currently a student teacher in the UK and I have to write a 1000 word report on the following maths problem which I am completely stuck on! PLease HELP!! Choose and 3 by 3 section of the hundred square. Add the total of the four corners. How many different groups of four numbers can you find that add up to that number? eg,
123
111213
212223
Total of 4 corners add up to 48.

Adding 2, 13, 22, 11 also make 48 etc..

How many different groups of 4 numbers would add up to 48?

How would these results compare with thoses obtained from a 3 by 3 square in which the numbers are consective? eg,
123
456
789

PLEASE HELP AS I AM COMPLETELY STUCK? WHY DO ALL THESE DIFFERENT WAYS ADD UP TO THE SAME NUMBER??
Answered by Harley Weston.

Circumference and Area 1999-02-16
From Natalie:
finding the circumference of a circle? formula
finding the area of a parallelogram? formula

Answered by Penny Nom.
The Board Problem 1999-02-15
From Avery:
Mr. Avery has 3-foot boards and 4-foot boards. If he puts the 3-foot boards in a line, they have the same length as the 4-foot boards put in a line. Altogether he has between 16 and 25 boards. How many 3-foot boards does he have?
Answered by Jack LeSage and Penny Nom.
Dig digs in the garden 1999-02-11
From Katherine Shaw:
A circular garden has an a radius of 8m. Dig, the dog, is tied up to a fence that runs round the outside of the garden. Dig was able to dig up all the garden, apart from an area of 64 square metres, which he couldn't reach. How long was his lead?
Answered by Chris Fisher and Harley Weston.
Satellite dishes 1999-02-10
From Katherine Shaw:
I have read your information on 'Why are satellite dishes parabolic", and I know the reciever should be placed at the focus of the parabola. Could you test this with lights beams and a parabolic mirror, or would light beams behave differently. Thanks.
Answered by Jack LeSage and Harley Weston.
Non-Euclidean Geometry 1999-02-10
From Robert Smith:
Is non-euclidean geometry necessary for the college bound student? I have students that are inerested in teaching math one day. My school is restricted to Euclidean Geometry.
Answered by Walter Whiteley and Jack LeSage.
Word problems 1999-02-09
From Brian Rodriquez:
Please answer these problems with a formula. Also explain in English step by step how these problems should be solved. I know the answer I just don't know how the answer was achieved Thank you very much for trying to help
  1. A ski lift carries a skier up a slope at the rate of 120 feet per minute and he returns from the top to the bottom on a path parallel to the lift at an average rate of 2640 feet per minute. How long is the lift if the round trip traveling time is 20 minutes?
  2. An airplane on a search mission flies due east from an airport, turns and flies due west back to the airport. The plane cruises at 200 miles per hour when flying east, and 250 miles per hour when flying west. What is the farthest point from the airport the plane can reach if it can remain in the air for 9 hours?

Answered by Jack LeSage and Penny Nom.
Grade 4 curriculum 1999-02-09
From John:
My daughter is in grade 4 and her math teacher has asked her to write out the number 1 to 10,000. She is currently at 4567 and is sick of it. When I complained that this was not a usefull exercise she said that the Ministry of Education curriculum requires this and gave a photocopy of the page. It states: "read and write whole number to 10,000 in standard, expanded, and written forms (eg., 9367=9000+300+60+7 = nine thousand three hundred sixty-seven). Has the teacher interpreted this the wrong way?

Thanks John
PS. Do you have a web address for Ontario Math and Science Teachers organization?
Answered by Jack LeSage.

John Napier and e 1999-02-06
From Shimin:
I would like to ask about the exact definition of e, its history (like how John Napier came about discovering it) and its applications in problems and real life situations. Thank you!
Answered by Chris Fisher.
Dividing Decimals 1999-02-06
From Melanie Campbell:
i ugently need to know how to divide a decimal by a decimal eg:6.0 divided by 2.4 i need to show all working on a basic skills exam please help!!!!

mel
Answered by Jack LeSage.

Lunes 1999-02-04
From Kai G. Gauer:
A prof once told me that a certain type of lune is quadrable given that the diameter is an integer. She used the construction of a right isosceles triangle within a semicircle and later constructed another semicircle on the base of the first semicircle and used area subtraction to show equality to a smaller triangle with quadrable area. What happens when the original inscribed triangle is no longer isosceles? She mentioned something about other lunes also being quadrable; but not all. What are the dimensions of other such lunes? Note: I'm not certain if I still have my hercules account; please simply post on Q&Q.
Thanks!

Answered by Chris Fisher.
Modular Arithmetic 1999-02-04
From Leslie Kupper:
I am trying to do a project on modular arithmetic. I was wondering if there were any websites that include a sample lesson plan on modular arithmetic for any grade level. Let me know where and how to find them. Thanks.
Answered by Harley Weston.
Cannonballs 1999-01-27
From Roger King:
How many cannonballs can be stacked in a triangular pyramid?
Answered by Penny Nom.
Simplifying Radicals 1999-01-26
From Mary:
I would like to know how to simplify this question:
   
4 __________________ squareroot7 + squareroot3 
I know the answer is (sqrt7 - sqrt3) but i would really love to know how to get that answer!! Thanks.

Answered by Jack LeSage and Penny Nom.
Simplifying Radicals 1999-01-25
From Randy:
Find the perimeter of a rectangle in simplified radical form L=root80 W=root45. I would appreciate a detailed explination of the process for solving.
Answered by Jack LeSage and Penny Nom.
The Quadratic Formula 1999-01-22
From Eric Morgan:
Hi my name is Eric Morgan and I m in 8th Grade honors math (middle) and I'd like to know how to prove the quadratic formula -b±sqrt of b(a)(c)over 2(a)
Answered by Penny Nom.
Angles in Polygons 1999-01-21
From Jen:
  • How do u find the interior angles of a pentagon when you are given 4 of the angles and you need to find the fifth?

  • If you are given the measure of each exterior angle of a regular polygon, how do you figure out how many sides the polygon has?

  • i need all information on polygons and how to find their angles!!!

Answered by Jack LeSage.
Root 17 is Irrational 1999-01-21
From John Murdock:
If you could help me out with this I would appreciate it. Prove that the square root of 17 is irrational.
Answered by Harley Weston.
Math and Writing 1999-01-21
From Nancy Waite:
This may not be an inquiry for this particular service, but we are searching for teacher resources incorporating writing into mathematics. With the new state math frameworks, our district is emphasizing using the writing process to develop thinking skills in math. We are gathering available resources in this area so we can begin formulating a plan to incorporate writing skills into the mathematics curriculum.
Answered by Walter Whiteley and Rick Seaman.
Math Olympics 1999-01-19
From Kim Brandson:
I am trying to put on a Math Olympics for K-5 students in my district. Any ideas? Resources? Suggestions?
Answered by Jack LeSage.
Fitting a Curve 1999-01-19
From Kirk:
Hello my name is Kirk from Scarborough, Ontario.

I have been out of a formal education system for thirty years. I program microcontrollers in my spare time. I have built a temperature sensing device ready to go but, thermistors are very non-linear. I do know that there is a way to calculate the input condition of the thermistor and display the correct temperature in degrees C. I am sending a file to show my progression so far.
Answered by Harley Weston.

Graphing the Derivative 1999-01-18
From Milena Ghebre:
This question has been nagging me for sometime now.

Is there a way of finding out the derivative of a function, just by looking at the graph of it?
Answered by Walter Whiteley.

Calculus 1999-01-16
From Kaylea Rankin:
Differentiate the following.

y = 1 /(2+3/x)
Answered by Jack LeSage and Penny Nom.

Linear Equations 1999-01-16
From Casie Tomney:
My dad and I have been trying to figure, how to solve for x on any of these problems my teacher gives us. The problem is:

2h/3 + 1/2 = 5h/6 - 3/4

Thanks!!!!!!!!
Answered by Jack LeSage.

Turning Fractions into Decimals 1999-01-16
From Lynn:
How do you do it step by step
Answered by Jack LeSage.
Levers 1999-01-13
From Lane Campbell:
1. Solve for x or y.

(a)

<-----4m------><------4m-----><---------xm ---------> 
_____________________________________________________
^             ^               ^                     ^
25N          5N               ^                    15N 

Answered by Jack LeSage.
Absolute value of i 1999-01-06
From Wayne Bagley:
I would like to know what is the absolute value of i. I need an answer suitable for the secondary level.
Answered by Harley Weston.
Place Value 1999-01-05
From Reg:
I work in a Section 27 classroom with various grades. I am looking for assistance on how to deliver a lesson on place value:decimal numbers keeping in mind that they need to see it (visual) to comprehend. I am looking for grade 4-6 level.
Answered by Jack LeSage.
Geometry patterns lesson plans 1998-12-31
From Vicki:
hi,,, my name is Vicki and I am a new 5th grade teacher....

Anyway, I'm supposed to come up with a lesson plan to

  • Explore patterns that result from cominations of "reflections, rotations, and translations of geometric figures.
The plan is to include:
  • writing/metacognition, assessment strategies, interdisciplanary connections, supplemental materials, or textbook, and Bloom's taxonomy level.
Any help you could give would be greatly appreciated.

Thanks
Answered by Jack LeSage.

The area of a triangle 1998-12-26
From Elysia:
How do you figure the square inches of a triangle?
Answered by Harley Weston.
Complex numbers and the quadratic formula 1998-12-25
From Richard Peter:
My age is 16, and my name is Richard. My question relates to the topic complex numbers & the quadratic formula.

I would like to know how to solve quadratic equations in which the discriminant is less than 0 (i.e. we get two complex solutions to the quadratic)

3x2+2x+5 = 0

and how mathematicians like euler contributed to this field. If it would be possible I would also like to know how this type of quadratic equations can be graphed?
Answered by Harley Weston.

Complex Numbers 1998-12-23
From Wayne Bagley:
I would like to know what is the square root of i , and i squared? I am looking for a response appropriate for secondary level students.
Answered by Harley Weston.
Some Word Problems 1998-12-17
From Jennifer Rudd:
To Whom It May Concern,
I'm having some difficulty with these word problems. Maybe you could help. Thanks.
  1. A matte of uniform width is placed around a painting so that the area of the matted surface is twice the area of the painting. If the outside dimensions of the matte are 40cm and 60cm, find the width of the matte.

  2. The Joneses start out A 1520km car trip to Walker Park. On the first day they cover 960km. On the second day they complete the trip, but a rainstorm causes them reduce their average speed by 10km/h.If the 2-day trip took a total of 20 h, what was the average speed on each day?

  3. Granny stood up at her grandson's wedding reception and announced " My age now is a perfect square and it is equal to the difference between the square of my grandson's father's age and the square of his mother's age. Come to think of it the difference my age and the square of my grandson's age is seven times the age of his mother, my daughter in law." The grandson's wife remarked to her new husband," That means the difference between the squares of our ages is three times the age of your father!" How old was the bride?

  4. The volume of a retangular box 4cm high is 144 cubic cm. If the perimeter of the base is 24cm, find the dimensions of the box.
P.S. the first two questions are the most important ones that need to be solved.

Thanks for your help!!
Answered by Jack LeSage and Penny Nom.

Volume in gallons 1998-12-09
From Hill Hermit:
Please tell me the formula for computing the volume, in gallons, of a cylinder. Thanks from a parent. American gallons, please.
Answered by Harley Weston.
6.99999... = ? 1998-12-05
From Tom:
I have had a rather heated arguement with my students. Please settle this for me.

Solve <,>, =
6.99999... __ 7

Thank you.
Answered by Penny Nom.

Intersection of Planes 1998-12-03
From Lindsay Fear:
My name is Lindsay Fear. I am an OAC student (which is the Ontario equivalent to Grade 12 in most other states and provinces). I am in an Algebra and Geometry course and am currently studying a unit on equations of planes. Our teacher has given us this question that my friend and I have attempted several times, but we are still unable to solve it. My teacher has also suggested using the internet as a resource. The question is:

Prove that a necessary condition that the three planes

 -x + ay + bz = 0 ax -  y + cz = 0 bx + cy -  z = 0 
have a line in common is that
a^2 + b^2 + c^2 + 2abc = 1

Answered by Walter Whiteley.
Platonic Solids Surface Area 1998-12-02
From Rachel Bidwell:
How do you find the surface area of the 5 platonic solids, when they each have a volume of one cubic inch
Answered by Harley Weston.
Duplation method of multiplication 1998-11-26
From Sara Whitford:
I am wondering why 16 and multiples of 16 do not work in the ancient duplation method of multiplication used by the Egyptians. I discovered the method in the Journeys math text gr 7 level. Just curious. Am I doing something wrong??
Answered by Jack LeSage and Harley Weston.
Three keys 1998-11-26
From Karen Chan:
A man has a bunch of three keys, only one of which fits the lock of his front door.
When he comes home in the dark he tries the keys at random until he finds the one fits. Find the probability that in a week of five nights, he tries the right key first on at least one night.

Answered by Penny Nom.
Algebra 1998-11-25
From Casey:
yeah my son brought home a math worksheet and he was wondering and me too if u could help him and me out . he has problems like this 4n+9=7n+2= ??? and he has to show work so if u could help us out that would be great
Answered by Jack LeSage.
The bricklayers formula 1998-11-24
From Rachel Kaplan:
I have to do a report on the bricklayers formula N = 7LH. Can you give me any information on this.
Answered by Harley Weston.
Intersection of planes 1998-11-22
From Dave Rasmussen:
I am a teacher of secondary mathematics with a question about the uses of Three Dimensional Co-ordinate Geometry. I have been teaching my students to write equations of planes and lines, - to find the intersection of these and the distance between them. What I am having difficulty finding are good applications of these techniques to "real world" situations. Can anybody help me?
Answered by Walter Whiteley and Harley Weston.
Three dice 1998-11-17
From Ryan Allinson:
Rolling three regular dice how many different ways could a sum of 10 be rolled?

Hope you can help. Can you please show me how this is done?
Answered by Penny Nom.

Linear systems of equations 1998-11-16
From Crystal Girvan:
My name is Crystal Girvan.Im in grade 11. I have a question.We are solving linear systems of equations: comparison method. My questions is. it says

a)use the method of comparison to solve this system a=b-1, 3a+ b=3

b) verify your solutin in a

I dont understand it. please help
Answered by Jack LeSage.

Terminating decimals 1998-11-16
From Debra Karr:
A college student studying elementary education asked me a question that I could not think of the correct answer. How can you look at a fraction and tell if is a terminating or non terminating decimal?
Answered by Jack LeSage and Penny Nom.
A trig limit 1998-11-14
From Amy Atwell:
what is the limit of of tanx / x + sin x as x approaches 0
Answered by Harley Weston.
Golf Problem - The Sequel 1998-11-13
From Bob:
Sixteen golfers, seven rounds. What are optimum combinations for each golfer to play with as many different golfers as possible?

Many thanks,
Bob Payson
Answered by Denis Hanson.

What's the next term? 1998-11-12
From Ilia:
What the formula and explonation for formula for next patterns:

1) 1, 4, 10, 20, 35, 56, 84,...

2) 0, 6, 30, 90, 210, 420,...

Thanks!
Answered by Penny Nom.

Two sides and a bisectrix. 1998-11-11
From Victor Grinshtein:
I am looking for someone who can tell me how to construct a triangle by 2 sides and a bisectrix using a compass and a ruler.
Answered by Chris Fisher.
Kumon 1998-11-09
From Kathy Steele:
Perhaps I am not spelling this correctly,since there is nothing to be found on Kumin Math, but my search for info on this topic led to this site. My understanding is that it is a program of repeated drills. Do you have any additional information? I know that this will come up during Parent conferences since some of my Jr. High students are talking about it.
Answered by Jack LeSage and Harley Weston.
Triangular Numbers 1998-10-30
From Matt:
i would like to know about triangular numbers and it history i would also like to know about the history of prime numbers thank you
Answered by Chris Fisher.
Divisibility by 11 1998-10-28
From Pat Duggleby:
I am an upgrading instructor at a drop-in program in Regina. One of my students is taking General Math 30 through correspondence, and we have run into some confusing instructions. The section is about divisibility rules, and we did just fine up until the rule for Divisibility by 11. The statement is as follows:
If the difference between the sum of the odd-numbered digits and the sum of the even-numbered digits, counted from right to left, is divisible by 11, then the number is divisible by 11.
.
.
.

Answered by Penny Nom.
A Puzzle 1998-10-27
From Bruce Bragnu:
We are trying to solve what SHOULD BE a simple 7th grade problem and have run into a wall.
It is:
ABCD
ABCD
ABCD
ABCD
-------
EBEA

It is an addition problem. Each particular letter represents the same digit. We are looking for what number "B" represents. We have tried a multitude of solutions and have come up empty.
Answered by Penny Nom.

Fractions 1998-10-27
From Jessica Braimoh:
I have a problem that I just can't figure out!!

Put into a fraction a) -1.157... b) 0.046... c) 0.032... d) -0.3234... Please show how you got the answer and how you came to it(step by step)
Thanks Jessica Braimoh grade 9 Ontario
Answered by Harley Weston.

A Number Trick 1998-10-26
From Brenda Meagher:
I'm a 37 year old mother and i have returned to school for personal reasons. I was faced with this question.
Choose any number less than 10, multiply it by 8547, then take your answer and multiply it by 13. I chose the number 8.

8547
x 8
--------
68376

and

68376
x 13
---------
205128
68376
---------
888888

Any number chosen from 1 to 9 will result in the same form of answer.If the number two is used you will end up with 222,222.etc So my question is, is there another number that will results in the answers.
Answered by Harley Weston.

Logs 1998-10-21
From Benny:
What does log means? How come we have to use log
Answered by Chris Fisher.
Complementary and Supplementary Angles 1998-10-21
From Christina Saunders:
I am in 9th grade and my math teacher wanted us to find out why complimentary angles are called complimentary and why supplimentary angles are called supplimentary. I have looked everywhere and asked numerous people, but I have yet to find an answer. My math teacher said it had something to do with trigonometry. Do you have an answer for me?
Answered by Chris Fisher and Penny Nom.
The Left Side of a Parabola. 1998-10-20
From Shay:
Find the parametrized equation for the left half of the parabola with the equation: Y=x^2-4x+3
Answered by Chris Fisher.
Adding Fractions 1998-10-15
From Lindsay:
please help me!!!!
Answered by Harley Weston.
Problem Solving 1998-10-15
From Pamela Fisher:
Can you give me a comprehensive list of problem solving stategies. I teach grade one in a K to 8 school and we are working on improving problem solving at all grade levels. I have heard that there is a list of various strategies that we could adapt to different grade levels. Any help you give me would be appreciated.
Thank you. Pamela Fisher

Answered by Walter Whiteley.
A Linear System 1998-10-08
From Jason Bussey:
The linear systems with the following pattern all have the same solution. Is their some kind of reasoning as to why this is so?

2x+3y=4
3x+4y=5

4x+5y=6
5x+6y=7

The solution is always x=-1, y=2

Thank you
Jason
Answered by Penny Nom.

Operations Research 1998-10-08
From Lisa Barrett:
What is the history of operations research and the study of linear programming?
Answered by Judi McDonald.
A Kite 1998-10-07
From Paul Scott:
What is the mathematical term for the kite shape?
Answered by Walter Whiteley.
Number Systems 1998-10-07
From Frans:
What other number system are there besides the real number system and the complex number system?
Interested adult
thanks

Answered by Penny Nom.
Adding Fractions 1998-10-03
From Pam Bailey:
Can you help me simplfy this?

(1/2a + 1/3b) - (1/4a - 1/5b) + (1/6a - 1/7)

thanx
Answered by Harley Weston.

Students and Lockers 1998-10-02
From Mike:
There is a row of 1000 lockers.
There is a line of 1000 students.

Student number 1 starts at the first locker and opens all 1000. Student number 2 starts at the second locker and closes every other one. Student number 3 starts at the third locker and goes to every third one, closing the open ones and opening the closed ones. Student number 4 does the same with every fourth locker and so on down the line... After all 1000 students have gone how many lockers are open and which ones are they?

Please help! There is proboly a simple solution but we couldnt figure it out for the life of us. Please let us know how you solve it.
Answered by Patrick Maidorn and Penny Nom.

French Francs 1998-10-01
From Richie Franklin:
1.what did one dollar equal in France francs ten years ago, twenty years ago, and fifty years ago?
Answered by Chris Fisher.
Repeating Decimals 1998-10-01
From Chris Norton:
Could you PLEASE give the formula to find out the number of digits in a repeating decimal before it repeats. I have been trying to get it for weeks from Math sites on the Internet. Can you please, please help me ?
Chris Norton

Answered by Chris Fisher and Penny Nom.
Two Chords 1998-09-29
From Jennifer Cane:
Question:
two parallel chords of a circle, AB and CD, have lengths of 10 cm and 17 cm respectively. The diameter of the circle is 21 cm. Find the shortest distance between the chords.

Answered by Harley Weston.
An Implication 1998-09-27
From Danielle Evans:
i am in my second year of math for the liberal arts and a problem has be terribly confused..

can you answer this

[(p --> q) ^ (q --> p)] --> (p <--> q)

is this an implication??

i would love to hear your answer

thank you...

danielle evans
Answered by Chris Fisher.

A Horizontal Line 1998-09-23
From Tess Warer:
Find the equations of a line through (-1,-5) and parallel to the X-axis.
Answered by Harley Weston.
Nickles on the Highway 1998-09-22
From Ronnie Mac Donald:
How many nickels would it take to cover 226km of highway,each nickel is 2cm in diameter?
Answered by Penny Nom.
A Hexagon 1998-09-22
From Lee Curtis:
Could someone calculate this for me?
If a hexagon is 38 feet 8 inches across, how wide would each of the six sides be?

Thank you,
Lee Curtis
Answered by Chris Fisher and Harley Weston.

Rational and Irrational Numbers 1998-09-19
From Ri:
I am trying to explain rational & irrational numbers to my niece who is grade 7 and am having difficulties. Could you please explain the difference between rational & irrational numbers.

Thank you
Ri
Answered by Penny Nom.

Simplifying Roots 1998-09-18
From Lauren Lattimer:
1. 2root27 divided by root 12

2. 3 root 48-root 75
Answered by Penny Nom.

Probability 1998-09-17
From Chris:
Six marbles are placed in one of three boxes. What is the probability that each box contains two marbles? What is the formula used?
Answered by Penny Nom and Chris Fisher.
Random 1998-09-15
From Hugh Ballantyne:
I am an occasional teacher.
Here is my question:
Does the word "random" have a technical meaning in mathematics?

Answered by Harley Weston.
Radicals 1998-09-15
From Lana Sabo:
Question:
fifteen times the square root of twenty, divided by the square root of 2.

nine subtract the square root of forty-five, divided by 3.

the square root of 18 plus the square root of 12, divided by the square root of 3.
Answered by Harley Weston.

Human Calculator 1998-09-14
From Pam Sloan:
I recently saw a special on NBC Dateline about a man known as the human calculator. They called him Mr. Math. I want to know more about his books, seminars, and activities. Also, what is his real name?
Answered by Patrick Maidorn.
(1+z+z^2)^2 1998-09-10
From A.R.:
These questions deal with factoring.

(1+z+z^2)^2 and (p^2-pq+q^2)^2

I partially understand the second line which would be 1+z^2+z^4+2z+2z^2+2z^3 but I don't understand where the 2 comes from in the second half of that line.

p.s.(^2 means squared or cubed etc.)

Please help me out A.S.A.P.!
I have a quiz on this tomorrow!
Thank-you,
A.R.
Answered by Harley Weston.

Triminoes 1998-09-09
From Roxanne Hale:
I am doing an investigation about a game called triminoes (like dominoes). The game is played using triangular pieces of card. Each card has 3 numbers on it. I have to investigate the relationship between the number of trimino cards in a set and the largest number on the cards. I found;

largest no. used 0 1 2 3 4
no. of trimino cards 1 4 10 20 35

I was ginen the formula for this which is:
UN= UN - 1 + 1/2 (n + 1 ) (n+2)

UN=no. of trimino cards n= largest no.

I don't know how to get to this equation I think it has something to do with triangle numbers!
Answered by Penny Nom.

Rational Numbers 1998-09-05
From Kristin Mckenzie:
Iam a secondary student with a math question I don't know how to do.
This is the question:

Explain whether each decimal number represents a rational number.

(a) 0.16

(b) 0.12

(c) -3.125

(d) -0.27

(e) -0,212 112 111 2 ....

(f) 0. 457 92

If it wouldn't be a problem i would really appreciate it if you sent back the instructions on how to do the question.

My name is Kristin Mckenzie and my return e-mail is lexus1999@hotmail.com

Thank-you for you time
Answered by Jack LeSage and Harley Weston.

Volumes 1998-08-29
From Lorraine Wall:
Consider the region in the first quadrant bounded by the x and y axes, the vertical line x=3 and the curve y = 1 / (x squared + 3). Determine the volume of the solid by rotating this region about the x-axis. Now that is the first part.
I then have to find the coordinates of the centroid of the solid by rotating this region about the x-axis.

Thanks.
Lorraine
Answered by Harley Weston.

The area and the circumference of a circle. 1998-08-27
From Jason Wright:
I was looking at the relationship of the area of a circle and the circumference when I realized that 2*pi*r is the derivative of pi*r^2. I was wondering if there is any connective deep dark meaning as to why this appears to be related. Thanks for any help you can give me!
Answered by Walter Whiteley.
Golf problem 1998-08-25
From Bob:
Twelve golfers plan to play seven rounds of golf. Each would like to play as many rounds with different golfers as possible. What are the optimum combinations to accomplish that goal?
Answered by Denis Hanson.
Frieze Patterns 1998-08-19
From Brian Bairstow:
I am doing a research project on frieze patterns (also called band patterns or border patterns). I know that there are exactly seven different types of frieze patterns, but I have been unable to find a proof for this. If you could tell me this proof, or tell me some internet sites on which I can find material on this, I would be very grateful.
Answered by Chris Fisher.
Concurrent Lines in a Triangle 1998-08-10
From Chris Woolf:
The question is Name four types of concurrent lines, rays, or segments that are associated with triangles.
Answered by Chris Fisher.
Two Problems 1998-07-28
From James Pulver:
How do you solve these problem? If log abc=16 and log ac=12 , find b. (The logs are log base 10.)
and
If a and b are real numbers, i^2 = -1 and (a+b)+5i=9+ai what is the value of b?

Answered by Jack LeSage.
Area and Volume 1998-07-28
From James Pulver:
I am going into 12th grade and am practicing for the SAT II. I have come across a problem that I cannot solve. It states that the front, side , and bottom faces of a retangular solid have areas of 24 square centimeters, 8 square centimeters, and 3 square centimeters, respectively. What is the volume of the solid. I need to know how to solve similar problems so is there a formula to go from area to volume?
Answered by Jack LeSage.
Parabolas 1998-07-24
From Danica:
how do you find the focus, vertex, and directrix of 4x-y^2-2y-33=0
Answered by Penny Nom.
Volumes of Revolution 1998-07-24
From Lorraine Wall:
I'm on the section fpr The Computation of Volumes of Solids of Revolution and the following question is giving me problems:

-Consider the region in the first quadrant bounded by the x-and y-axes, the vertical line x=3, and the curve y=1/(xsquared + 3) I can determine the volume of the solid by rotating the region about the y-axis using the shell method but I can't seem to be able to get started with the volume when rotated about the x-axis.
Answered by Harley Weston.

Snake Eyes 1998-07-20
Would you please advise me what the probability is of rolling snake eyes on a pair of dice is? My recollection is 1 out of 6 multiplied by 1 out of 6 = 1 out of 36 is this correct?
Answered by Jack LeSage and Penny Nom.
Solving Quadratics 1998-07-16
From Fred Goodrich:
I need a step by step lesson on solving quadratic equations.

Thank you,
Answered by Penny Nom.

Calculus problems 1998-07-13
From Lorraine:
I'm stuck again. Can you help?

This involves integration using the method of partial fractions

the integral of:
7x(to the 5th) - 2x(cubed) + 3 dx
--------------------------------------
x(to the fourth) - 81

Do I have to do long division to reduce the numerator to the fourth power?

the integral of:
4- 16x +21x(squared) + 6x(cubed) - 3x(fourth) dx
----------------------------------------------------
x(cubed)(x - 2)(squared)

Lorraine
Answered by Harley Weston.

Social Security Numbers 1998-07-07
From Valerie Lucas:
How many social security numbers are possible?

the answer is 10^9

why isn't the answer 9^9?
Answered by Harley Weston.

A Calculus Problem 1998-06-28
From Lorraine:
I'm a post-secondary student taking calculus by correspondence. I'm stuck on the following question (and similar ones) Can you help?

Evaluate the following indefinite integral:

d(theta)
----------
1 + sin (theta)

(It says to multiply both numerator and denominator by: 1 - sin(theta)

Thanks
Lorraine
Answered by Harley Weston.

The Birthday Problem 1998-06-12
From Josh Skolnick:
if you are at a party what is the least amount of people that have to be there to have at least a 50% chance of having 2 people with the same birthday? and how do you get the answer thank you in advance

josh
Answered by Harley Weston.

Rolling a seven 1998-06-11
From Bruce Thompson:
What is the probablity of rolling two dice and it coming out as seven?

a) 1/6
b) 1/36

please give me the correct awnser and explain please.
Answered by Harley Weston.

Grade 9 Algebra 1998-06-07
From Tarah Kostenko:
I am a grade 9 student and I cannot figure out how to solve these two similar problems. can you please help?
-3(y-1/2)=1/2 also -2/3(x + 1)=6
I don't come up with the same answers as the book and I can't figure out what I'm doing wrong.

Thank you,

Answered by Harley Weston.

A Logic Problem 1998-06-07
From Anthony Bacigalupo:
My name is Anthony Bacigalupo and I take Sequential ][ Math and am taking a practice regents. When doing a logic problem, I encountered the following statements, where I am trying to prove P ( I left out steps unrelated to the question)....
Answered by Chris Fisher.
A trig limit 1998-05-28
From Ann:
This problem is a calculus 1 limit problem-high school level. I'm teaching myself calc over the summer and I'm already stumped.

find the limit

 lim sec^(2)[(sqrt2)(p)]-1 p-->0 --------------------- 1-sec^(2)[(sqrt3)(p)] 
I'm Ann.

Answered by Harley Weston.
A Place Value Curiosity 1998-05-25
From Ed:
I was visiting with an elderly gentleman this afternoon. He showed me this curiosity and then asked if I could explain it to him. Can you provide an explanation of why the 9 or multiple of 9 keeps occurring in this procedure? Choose any number, say 125 and add the digits to get 8. subtract the 8 from the 125 and the result is 117. Add the digits in 117 to get 9. Subtract the 9 from the 117 to get 108. Add the digits in 108 to get 9. If this procedure continues a 9 or a multiple of 9 reoccurs. What is the mathematical explanation behind this happening?
Answered by Denis Hanson.
Graph question 1998-05-12
From Rose Seminary:
Why is the point of intersection of two lines the solution to the corresponding system of equations?
Answered by Penny Nom.
Isosceles trapezoid formula 1998-05-12
From Donna McMullin:
The teacher of Gifted and Talented Math has been trying to locate the formula for anisosceles trapezoid and we can't find it anywhere. Could it be the same formula for that of a parallelogram ? Please advise.
Answered by Walter Whiteley.
Multiplying by Nine - Chismbop Style 1998-04-27
From Noria Jones:
About a year ago a grade 5 teacher at my son's school taught the children how to multiply the 9 times table on their fingers quickly.

It was part of a kind of finger math kind of thing...
Answered by Patrick Maidorn.

Logic and Matrix Instruction 1998-04-24
From Robin Booker:
I need assistance with instruction strategies to teach the construction of a matrix, solving a logic problem. Providing instruction in the construction of a simple matrix , no problem. However, I stumpted on this one.

Five players were chosen as All Stars at the basketball banquet. Based on the following clues, find the player's name, team, uniform color and number of points scored....
Answered by Harley Weston.

Hour Hand Movement 1998-04-19
From Chris and Angela Bradley:
My wife and I are haveing an argument over the answer to this question, what is the total number of degrees through which the hour hand of the clock moves in 3 hours and 18 minutes?
Thanks in advance.

Answered by Chris Fisher.
Seed Problem 1998-03-26
From Jody A. Johnson:
Seed Problem: The height of an amazing plant grown from a seed varies linearly with the time it has been in the ground. After six days it is 6cm tall and after 60 days it is 15.8m (1580 cm) tall.
  1. Write the particular equation giving the height as a function of time.
  2. Find the intercepts and explain them.
  3. What would be the height in one year?
  4. How long would it take for the plant to grow to a height of 1km?
I'm having a problem with this equation. Could you please help. Thank you.

Answered by Penny Nom.
The Largest Factor 1998-03-15
From Senthuran Nadarajah:
When each expression is evaluated for different values of n, the answers will differ. For each expression, find largest natural number that will divide the result for all natural number values of n.

n^5 - 5n^3 + 4n
Answered by Harley Weston.

Clock Arithmetic. 1998-03-09
From Joann Dixon:
What is clock mathematics?
Answered by Patrick Maidorn.
Area of a Triangle. 1998-03-05
From Amanda:
How do you figure out the area of a triangle? You already have the perimeter and height
Answered by Penny Nom.
Y-intercept 1998-02-27
From Don Trumpet:
What is the Y-intercept for the problem:Y = -2(x+2) + 9
Answered by Harley Weston.
Pay Phone Problem 1998-02-26
From Shameq:
Hi, I've been given a problem that I'm having some trouble with. I'd really appreciate any help. Here's the question (it's called the Pay Phone Problem)

A pay phone will take only 10p, 20p, 50p, and £1 coins"(It's British).

A woman has plenty of 10p and 20p coins. She has no other coins. She can put the coins into the pay phone in any order.

INVESTIGATE the number of different ways, she could put the 10p and 20p coins into the pay phone.
Answered by Penny Nom.

The origin of angles 1998-02-24
From Marc Poulin:
I'm currently teaching angles to students in grade 10 and I've been asked what's the origin of the terms degrees, radians and gradians.

I know that the radians come from the sexagesimal numerical system of the Babylonians but my kids wanted to know dates and persons who would have brought these terms first.
Answered by Harley Weston.

Tessellations of non-polygons 1998-02-24
From Ellen Goldwasser:
Hi! My name is Ellen Goldwasser. I'm a seventh grade student and I'm doing a prodject on tessellation. My question is: why will certain shapes (not polygons) tessellate? Thanks for your help!
Answered by Penny Nom.
Tangrams 1998-02-24
From Allison:
Do you know the story behind the tangram? I know that it is a Chinese puzzle about 200 years old. If there is a story about its creation, I would love to share it with my 7th graders. Thanks.
Answered by Diane Hanson and Penny Nom.
(50^20)(20^50) 1998-02-24
From fion:
50 power of 20 X 20 power of 50?

How many zero can be found in the answer and why?
Answered by Penny Nom.

A Tightrope Walker. 1998-02-19
From Amy Zitron:
A tightrope is stretched 30 feet above the ground between the Jay and the Tee buildings, which are 50 feet apart. A tightrope walker, walking at a constant rate of 2 feet per second from point A to point B, is illuminated by a spotlight 70 feet above point A....
Answered by Harley Weston.
The sum of the angles of a triangle is 180 degrees 1998-02-19
From Quin Liu:
How do you prove that the sum of the angles of a triangle is 180 degrees? Is there a proof? what is it?
Answered by Chris Fisher.
Graph Distortion 1998-02-19
From Dana Steffan:
What exactly is graph distortion? I have to explain it for a project and I can't seem to find anything on it.
Answered by Penny Nom.
Intersecting Lines. 1998-02-10
From Erik Heppler:
My typewriter frequently makes mistakes. Suppose I gave you the following system of equations to solve:

35.26X + 14.95Y = 28.35
187.3X + 79.43Y = 83.29

When I looked back, I realized that I meant to use 14.96 as the "Y" coefficient in the first equation instead of 14.95. Solving the system using 14.95 and then solving another system using 14.96 with all other values the same both times results in the intersection points (1776, -4186) and (-770, 1816) respectively.
How can that be?
Answered by Penny nom.

The Earth's Arc 1998-02-06
From Robert Dyck:
How do I find the arc/mile of the earths surface? What is the arc?
Answered by Harley Weston.
Roman Numerals 1998-02-05
From Dawn Hilgenberg:
How would I read a Roman Numeral such as LXII? Does the L in front make the X equal a lesser amount? We have been discussing how to read these and I can find no information about how to read an extended Roman Numeral problem. Help!!
Answered by Harley Weston.
Area of a triangle. 1998-02-01
From Jodi Blucher:
Is there a formula for the area of an equilateral triangle knowing the length of the sides?
Answered by Chris Fisher and Harley Weston.
Two Inscribed Trapezoids 1998-01-27
From James:
A hexagon inscribed in a circle has three consecutive sides each of length 3 and three consecutive sides each of length 5. The chord of the circle that divides the hexagon into two trapezoids, one with three sides each of length 3 and the other with three sides each of length 5, has length equal to m/n, where m and n are relatively prime positive integers. Find m+n.
Answered by Haragauri Gupta.
The two-digit numbers from 10-99 1998-01-21
From Alexis Riddle:
My name is Alexis. I'm a student in 8th grade and I'm taking Algebra I Honors. My question is: In a string of numbers, two adjacent digits are considered as a two-digit number. For instance, the string 11012 contains the numbers 10, 11, and 12. What is the number of digits in the smallest string that contains all of the two-digit numbers from 10-99? Please help and thank you for any assistance you can give me.
Answered by Penny Nom.
The Factor Theorem 1998-01-18
From Dhruv Bansal:
Hi, I'm a ninth grade student doing a math project for school. I'm trying to learn about various ways of solving 3rd and 4th degree polynomial equations using the remainder theorem, the factor theorem, and synthetic division. The books I have all mention x - r, which I know nothing about, not even how to get it.

I would really appreciate any information on this. Thanks.

Dhruv Bansal
Answered by Penny Nom.

Polynomials 1998-01-13
From Sarah Storkey:
Hi, My name is Sarah, My question is at the junior level.

My question is, what is a polynomial?
Answered by Harley Weston.

Quadratic Graphs 1998-01-13
From Simon Batten:
How does varying the values a,b and c affect the graph of
y=ax2 + bx + c?

Answered by Harley Weston.
Greatest Common Factor. 1998-01-09
From Percy Stribling:
how do you figure out what the greater common factor is?
Answered by Penny Nom.
Find the Numbers. 1997-12-28
From Sandy C:
The sum of the squares of two numbers is 53. Twice the greater, minus the lesser is 11. Find the numbers.
Answered by Penny Nom.
Trigonometric functions 1997-12-21
From Calvin Cheng:
My name is Calvin and I have a year 12 question for you to help me with.

From a point S, the angle of elevation of the top of a tower due north of it is 20 degrees. From R, due east of the tower, the angle of elevation is 18 degrees. S and R are 100m apart. Find the height of the tower.
Answered by Harley Weston.

(-2)^x 1997-12-12
From Ken Bokinac:
During our math class last week we came accross an equation that we could not graph we were wondering if you could help us try and graph it. The question is:

y=(-2)^x
Answered by Harley Weston.

Calculating the Intersection Point. 1997-12-10
From Milton Parsons:
Given 2 lines through (x1,y1),(x2,y2),and (x3,y3),(x4,y4). Can you give me a good formula for calculating the intersection point. I would greatly appreciate it.Thanks.
Answered by Chris Fisher and Penny Nom.
Pythagorean Triples. 1997-12-04
From Shameq Sayeed:
I've got a couple of problems which I hope you'll be able to solve for me.

I'm investigating pythagorean triples, and I have found a trend for the triples themselves, and thus have been able to form a general equation, i.e. a=2x+1, b=2x^2+2x, and c=b+1. Now, I sure this equation works, because I've tried it out and have come up with triples that adhere to a^2 + b^2 = c^2. But I was wondering WHY c=b+1. Is it possible to have c=b+2, and if not why not? THAT is the first problem.
Answered by Chris Fisher.

Shimin's Geometry Problem 1997-12-02
From Ong Shimin:
ABCD is a rectangle. X and Y are the midpoints of BC and CD respectively. W is a point on AB such that AW : WB = 2 : 1. Z is a point on AD such that AZ : ZD = 2 : 1.

WY intersects XZ at O. If the area of triangle WOZ is 84 centimeters squared, find the area of triangle XOY.
Answered by Chris Fisher.

Limited area and unlimited perimeter. 1997-11-28
From Rosa:
There is a figure, it has unlimited perimeter but has limited area , what is the figure and how to draw it ?

Thank you very much!
Answered by Harley Weston.

A Problem with Logs. 1997-11-26
From Herman:
Given log 24=a, log 25=b and log 26=c, express log 39 in terms of a, b and c.
Answered by Penny Nom.
Guess and Check 1997-11-25
From Don Ramey:
Hope you can help !!!! my daughter is in the 4th grade.. she has brought home a homework paper and the subject is problem solving. she does not understand what is expected of her. I have tried to help, but I do not understand either.

She is to use the Guess and check Strategy.

Ben knows 100 of his neighbors by name. Ten are fish. The rest are frogs and turtles. He knows the names of twice as many turtles as frogs. How many turtles does he know by name.?

Please help by explaining the guess and check strategy for a problem like this.
Answered by Diane Hanson.

Division by a fraction. 1997-11-21
From Roger Marchegiano:
I am interested in a solution to the following extra credit problem for my son in Geometry. We have been unable to produce a satisfactory response:

Show a geometric representation of the division of fractions algorithm, (When you divide fractions you multiply the numerator by the reciprocal of the denominator).
Answered by Penny Nom.

A geometry problem 1997-11-20
From Herman:
When produced, two equal chords AB and CD of a circle meet at P in an angle of 24 degrees. If H is the mid-point of AB and K is the mid-point of CD, calculate the size of angle HKD.
Answered by Penny Nom.
Multiplying imaginary numbers. 1997-11-03
From Jim Catton:
Here is the question:

(square root -2) x (square root -8)

My algebra suggests two possibilities .
.
.

Answered by Walter Whiteley, Chris Fisfer and Harley Weston.

Pi 1997-10-31
From Ryan McKinnon:
What Is Pi?
Answered by Chris Fisher.
Roman Numerals 1997-10-31
From Mark Curts:
I am looking for some resources for teaching Roman Numerals. I would like to expand upon the basic concepts, by locating some addition, subtraction, multiplication or division problems written with Roman Numerals.

--
Mark Curts
Answered by Chris Fisher.

Some Calculus Problems. 1997-10-30
From Roger Hung:
  1. What real number exceeds its square by the greatest possible amount?

  2. The sum of two numbers is k. show that the sum of their squares is at least 1/2 k^2.

  3. .
    .
    .

Answered by Penny Nom.
Two Questions 1997-10-28
From Melissa Kelley:
I would appreciate any help you could give me.
  1. Why isn't 1 a prime number?

  2. How can the absolute value of a number be negative?

Answered by Penny Nom.
How many intersections? 1997-10-08
From James:
(a) A collection of eight points, no three collinear. If lines are drawn between each pair of these points, how many points of intersection would there be?

(b) what would your answer have been in part (a) if there had been n points to start with?
Answered by Chris Fisher.

Polynomials 1997-10-07
From Sheryl and Jeff:
I'm a math teacher in Jerusalem, Israel.

I'm teaching about graphing polynomial functions in a pre-calc class. A student asked me what they're good for. I couldn't give her a good example. Do you have one. Thanks.
Answered by Penny Nom.

Cos(x) Cos(2x) Cos(4x)=1/8 1997-09-24
From Tan Wang:
How many distinct acute angles x are there for which cosx cos2x cos4x=1/8?
Answered by Chris Fisher Harley Weston and Haragauri Gupta.
A Geometry Problem 1997-09-18
From Rebecca Henry:
A circle is centered at the vertex of the right angle of an isosceles triangle. The cirlce passes through both trisection points of the hypotenuse of the triangle. If the length of a radius of the circle is 10, find the area of the triangle.
Answered by Chris Fisher Harley Weston.
A Trigonometric Limit 1997-09-18
From Brian Ray:
What is the limit, as x approaches 0, or tan^23x/x^2? (read, tan squared 3x over...)?
Answered by Harley Weston.
Imperial System of Measurement. 1997-09-17
From Judy Pardilla:
Please also give the solution to the answers. Thank you.
  1. If a room is 12 feet by 15, how many square yards are there?

  2. If a single roll of wallpaper will cover 30 square feet, how many single rolls will it take to cover a wall 8' x 15'?

  3. If ceramic tiles are 8" x 8", how many tiles would it take to cover an area 4' x 6'?
If possible, can you e-mail me a table for imperial measurements. Thank you!

Answered by Penny Nom.
Rowing on the Charles River. 1997-09-17
From Fawwaz Muraisi:
On the Charles River in Boston, the Harvard bridge and the Longfellow bridge are 1 mile apart. The MIT crew starts rowing upstream at the Longfellow bridge. As the crew passes under the Harvard bridge, the coxswain's hat falls into the river. Ten minutes later, the coxswain notices and turns the boat around instantaneously. He has t he crew row back to get it, rowing at the same constant rate. By the time the team reaches the hat, they are back at the Longfellow bridge.

How fast is the river flowing?
Answered by Harley Weston.

A Limit Problem 1997-09-16
From Robert Reny:
what is the limit, as x approaches 0, of 3x/2x-[x]? [] means absolute value.
Answered by Harley Weston.
Triangular Numbers. 1997-09-08
From Rachel:
Show visually that the square number 16 is the sum of two triangular numbers. Which two?
Answered by Penny Nom.
Two Questions 1997-08-28
From Faye Tan:
I am a trainee teacher. I would appreciate it very much if you could help me solve the following two problems which I think are meant for students at middle grade.

1. If a hen and a half lays an egg and a half in a day and a half, how many hens are needed to lay a dozen eggs in one day?

2. There are fewer than 200 passengers on a train. If they get off in pairs, one passenger will be without a partner. If they get off in groups of 3 or 4, there will still be one passenger left by himself. However, if they get off in groups of 5, no one will be left by themselves. How many passengers are there on the train?
Answered by Penny Nom.

The Length of a Chord. 1997-07-26
From Nathan Arthur:
Picture a 9 inch diameter circle. Inside that circle is a 6 inch diameter circle tangent to it. Then, tangent to both circles is a 3 inch diameter circle. So there are three circles, two smaller ones inside a big one, all of them just touching but not overlapping.

Now picture a chord on the 9 inch circle that is created by making a line that is tangent to both the 6 and the 3 inch circles and extending it to the edge of the 9 inch circle.

I need the length of that cord.
Answered by Chris Fisher.

A Question About Pi. 1997-07-08
From Mike Chan:
I have read the section Repeating Decimals in your data base. It mention that 1/17 has at most only 16 repeating digits. But, why does "pi" have an infinite number of digits (and not repeating ).
Answered by Penny Nom.
Finding the Mine 1997-06-23
From Billy Law:
Tom is gold prospector. On his last trip out from town, he headed 35 degree South of West to a lake where he had lunch. The lake was 24 km out of Town. He then headed due East for 35 km before Doubling back on bearing of 15 degree South of west for 20 km to reach his mine. By converting to Cartesian coordinate before doing vector additions do the following:

a) Calculate the position of the mine from town in term of a distance and a direction.
...
Answered by Harley Weston.

The angle between two tangents. 1997-06-09
From Felix Ho:
Two tangents are drawn from the origin to the circle (x)(x)+(y)(y)-4x-6y+9=0. If the angle between the tangents is m, fine the value of tan(m).

P.S. (x)(x)=square x
Answered by Harley Weston.

The General Equation of a Parabola 1997-05-28
From Michelle:
My name is Michelle and I am a 10th grade student in algebra 2 w/ analysis. I am doing a report on parabolas and I need to know what the general equation is. I've looked in books and keep finding different ones! I also need to know how they can be used in nature.

Thank you so much for your time. I really appreciate it!

- Michelle
Answered by Harley Weston.

The Area of a Trapezoid. 1997-05-07
From Mary George:
I am doing the area of trapezoids and mixed polygons and I was wondering if you can help me figure out this problem.

A line segment drawn parallel to a leg of a right triangle divides the other leg into segments of 3cm and 6 cm and the hypotenuse into segments of 5cm and 10 cm. The two figures formed are a triangle and a trapezoid. Find the area of each.

I would appreciate if you would email me back the solution.
Answered by Harley Weston.

Surface Area 1997-04-30
From Amber Nobile:
The height of a cylinder is twice the diameter. Express the total surface area as a function of the height h.
Answered by Harley Weston.
A proof that e is Irrational. 1997-04-30
From Peter Hall and Jenny:
We have a little mathematical problem... we need some help proving e is an irrational number!

We don't feel very confident in our formulas, so if You have the time to give us a little explanation we would be very grateful!!!
Answered by Doug Farenick and Penny Nom.

Solving a Trig Equation. 1997-04-28
From Susan Harvey:
Hi I am a teacher and have a calculus problem that I have a solution to but it seems so involved that I would be interested to see if their were other solutions.

Solve for x, if x is from -90 to 90 degrees

tan2x = 8cos{squared}x - cotx
Answered by Chris Fisher Denis Hanson and Harley Weston.

A Monte Carlo Procedure 1997-04-23
From Donna Hall:
A irregularly shaped object of unknown area A is located in the unit square 0<=x<=1. Consider a random point uniformly distributed over the square. Let X = 1 if the point lies inside the object and X = 0 otherwise. Show that E(X) = A. How could A be estimated from a sequence of n independent points uniformly distributed over the square? How would you use the central limit theorem to gauge the probable size of the error of the estimate.
Answered by Harley Weston.
Natural Logarithm Functions 1997-04-23
From Rickson:
The following two questions are some of my son's homework that he is having trouble with......any advice or assistance would be appreciated.

(eX)5=1000.............the X and 5 are exponents

lnx + ln(x+3) = ln10

In each question the problem is to find x.
Answered by Harley Weston.

Square Roots and Functions. 1997-04-23
From Ed:
1. In most texts the solution to a question such as square root x = -6 is

x is undefined.

Yet when teaching to solve
xsquared = 36
x = +6 or -6

There appears to be a contradiction here. My question is when, where and why do we use the principle square root, not both + and -? This often occurs as the extraneous root in the solution of radical equations and in stating the domain and range of functions involving square roots.

2. Are there any simple rules for determining whether equations are functions without graphing them and doing a vertical line test?
Answered by Harley Weston.

The Central Limit Theorem 1997-04-21
From Donna Hall:
A skeptic gives the following argument to show that there must be a flaw in the central limit theorem:
We know that the sum of independent Poisson random variables follows a Poisson distribution with a parameter that is the sum of the parameters of the summands. In particular, if n independent Poisson random variables, each with parameter 1/n, are summed, the sum has a Poisson distribution with parameter 1. The central limit theoren says the sum tends to a normal distribution, but Poisson distribution with parameter 1 is not normal.

What do you think of this argument?
Answered by Neal Madras.

The Division Bracket. 1997-04-09
From Judy Riley:
A fellow teacher recently asked if I remembered the exact word for a division bracket (not the symbol with dots, the horizontal line in a fraction, or a solidus). I couldn't. Can you help?
Answered by Walter Whiteley and Harley Weston.
A Geometry Problem 1997-04-09
From Gina M. Pisco and Rebecca Henry:
Three segments of 3, 4, and 5 inches long, one from each vertex of an equilateral triangle, meet at an interior point P. How long is the side of the triangle?
Answered by Richard McIntosh.
Equation of a line 1997-04-08
From Susan Gregson:
I am a secondary school teacher. My students and I would like to know why the letters m and b are traditionally used to stand for slope and Y-intercept in the standard form of an equation. Was this an arbitrary choice? Who made it? Are the letters from Greek ot Latin words?
Answered by Harley Weston.
Graphing Inequalities of Conic Sections 1997-03-24
From James Sheldon:
I'm trying to graph Systems of Conic Sections with inequalities, but I'm running into problems on which area to shade:
x^2+y^2 is greater than or equal to 16
xy > 4

So I graph these two equations, and then my teacher said to substitute a point into it but I'm still not sure how to do it...
Answered by Penny Nom.

The normal distribution. 1997-03-21
From Donna D.Hall:
I am looking for a proof for the normal distribution.

I suppose "proof" was not a good choice of words. What I am looking for is a way to "derive" the normal distribution in simple terms so that the most average teenager can see the logic. Can you help me?
Answered by Harley Weston.

Mathematical Induction and the Derivative 1997-03-18
From Shuling Chong:
"Obtain a formula for the nth derivative of the product of two functions, and prove the formula by induction on n."

Any educated tries are appreciated.
Answered by Penny Nom.

A Presidential Proof 1997-03-18
From Greg Smith:
Which US president developed a proof for the Pythagorean Theorem?

Where can a copy of the proof be located?
Answered by Chris Fisher and Harley Weston.

Ajax, Beverley, Canton and Dilltown 1997-03-14
From S. Johnson:
The following towns are placed on a coordinate system. Ajax at (-x,z), Dilltown at (-10,0), Canton at (0,0) and Beverly at (0,10). The roads from Beverly to Canton and from Canton to Dilltown are perpendiculat to each other and are each 10 miles in length. A car traveling at all times at a constant rate, would take 30 minutes to travel straight from Ajax to Canton, 35 minutes to travel from Ajax to Canton via Beverly, and 40 minutes to travel from Ajax to Canton via Dilltown. What is the constant rate of the car, to the nearest tenth of a mile per hour.
Answered by Chris Fisher and Harley Weston.
Names of the Polygons 1997-03-04
From Manuel:
What do you call a 13 sided polygon?
Is there a list of the different names?

Answered by Walter Whiteley and Harley Weston.
0! 1997-03-02
From Donna D. Hall:
I am looking for a quick and easy explanation as to why 0! is 1.
Answered by Walter Whiteley and Denis Hanson.
Area Between Two Sectors 1997-03-02
From Rebecca Henry:
Points A,B,C,D,E,F are equally spaced on a unit circle. Arc CGE has a center A. Find the number of square units of area in the shaded region.
Answered by Walter Whiteley.
A Functional Equation. 1997-02-26
From Ronald Lui:
Find all functions f:S --> S statisfying the two conditions:

1)f(x+f(y)+xf(y))=y+f(x)+yf(x) for all x and y in S

2)f(x)/x is strictly increasing on each of the interval -1
Answered by Saroop Kaul and Chris Fisher.

Equivalent Expressions 1997-02-26
From Karyn Jones:
Josie says that s=n/(n+1) and s/(1-s)=n are two ways to write the same formula. Describe whether or not you agree with Joise. Explain how you made your decision.
Answered by Walter Whiteley and Penny Nom.
Some Circle Questions. 1997-02-25
From Staci Vawser:
A circle with a radius of 10m is drawn. A chord is drawn across the circle. How is the area that is formed between the chord and the arc calculated?
Answered by Harley Weston.
Triangles, The Pythagorean Theorem and Pizzas. 1997-02-23
From Sherryle Mathis:
I am a graduating senior presently teaching geometry as part of my student teaching. I will do my CUP on Right Triangles and Pythagorean theorem. I am looking for a fun activity as part of my unit plan.
Answered by Walter Whiteley.
Circular Permutations 1997-02-05
From Ed Varjassy:
I have an understanding of simple problems involving circular permutations but do not understand them when they start to have restrictions. Is there any advice you can give for these more complicated permutations.
Answered by Penny Nom.
Deriving the Quadratic Formula 1997-02-04
From James:
How do you derive the quadratic formula? I know what it is, but the textbook doesn't say how to derive it.
Answered by Penny Nom.
Parabolic Mirrors 1997-01-28
From Megan Wennberg:
Consider a ray of light that passes through a chord of a parabola (the chord is above the focus and parallel to the directrix), hits the parabola at a point (x,y) and is reflected through the focus. If d1 is the distance from the chord to the point of incidence (x,y) and d2 is the distance from (x,y) to the focus, can you prove that the sum of the distances d1+d2 is constant, independent of the particular point of incidence.
Answered by Penny Nom.
A donkey and his carrots 1997-01-28
From Emily Lind:
There is a donkey who carries carrots. A farmer has 3,000 carrots to carry to the market. The market is 1,000 miles away. The donkey can only carry 1,000 carrots at a time and he eats 1 carrot every mile that he walks but this is only when he is carrying carrots. How many carrots can the farmer get to market by having the donkey carrying them?
Answered by Penny Nom.
Repeating Decimals 1997-01-24
From Grant Reed:
Is there a way to tell that the repeating decimal for 1/17 has no more than 16 repeating digits?
Answered by Penny Nom.
Foci of an Ellipse 1997-01-22
From David Gilliam:
How do I find the focii of the following equation? 4x^2 + 9y^2 = 36
Answered by Harley Weston.
Mathematics of Schedules 1997-01-16
From Byron Krull:
I was asked if there was a mathematical method to work with schedules. The problem is this. There are 24 teams playing weekly on 4 sheets at 3 different times of the day as follows...
Answered by Denis Hanson.
A triangle problem 1996-12-19
From S. Johnson:
Given that Triangle ABC is a right triangle and Circle O is inscribed in it find the radius of Circle O, totally in terms of a, b, and c.
Answered by Penny Nom.
Roots of a Polynomial 1996-12-06
From Paula Miller:
Why is the solution called a "root"? Why not just the "x-intercepts"?
Answered by Chris Fisher and Harley Weston.
Sides in a Regular Polygon 1996-12-06
From Rick Moss:
If you are given the measure of each interior angle (162 degrees) of a regular polygon. How many sides does the polygon have?
Answered by Penny Nom.
A balance problem 1996-11-18
From Jack N. Bussell:
There are 12 coins which look exactly the same, however one of them is heavier or lighter than the rest. Using a pointer balance scale, can you identify the odd coin and whether it is heavy or light in 3 weighings?
Answered by Harley Weston.
Pentominoes 1996-11-14
From Sam Maraldo:
What is a pentominoe? I need to understand the concept and how/why it is used?
Answered by Penny Nom.
Smith, Rodriguez and Jones 1996-11-07
From Rafayel Ambartsumyan:
On a train, Smith, Rodriguez, Jones are the fireman, brakeman, and engineer, but not in that order. Also aboard the train are three businessman who have the same names, a Mr. Smith, a Mr. Rodriguez, and a Mr. Jones. ..... Who is the engineer?
Answered by Penny Nom.
Height of a Hotel 1996-11-07
From Irene:
"Irene" is to determine the # of floors in a hotel 500 feet up the street. Irene is on the 10th floor of an office building and can measure the angle of elevation to the top of the hotel, 57 degrees. Her view of the entire building is obstructed. If the street rises at an angle of 8 degrees from the office building to the hotel and the average distance between floors is 11 feet, how many floors are on the hotel?
Answered by Penny Nom.
Common Area of two sectors. 1996-11-06
From Lynda Mow:
How do I find the area common to two intersecting circles of radii 8 ft and 10 ft if their common chord is 10 ft long?
Answered by Penny Nom.
Priority of operations 1996-11-04
From Wallace:
What is the answer to this problem? Let x=-2 and y=3. Evaluate 12x / 2y (if the old division sign is used).
Answered by Chris Fisher.
Approximating roots. 1996-11-04
From Ben Dixon:
How do you calculate a square root? eg the square root of 2.

There is obviously some sort of successive approximation type algorithm for doing it to however many decimal places is required, but what is the algorithm?
Answered by Harley Weston.

A tangent to a circle is perpendicular to the radius at the point of contact. 1996-10-22
From Rita Leung:
I wonder if there is any proof for this theorem - A tangent to a circle is perpendicular to the radius at the point of contact. If there is any proof for that, can you tell me please?
Answered by Chris Fisher and Harley Weston.
Could you tell me the name for the bar in a division problem? 1996-10-21
From Linda:
Could you tell me the name for the bar in a division problem. Not the line with dots on either side but the line that divides the two numbers? My name is Linda. I am asking for my niece who is in 8th grade.
Answered by Chris Fisher.
What is the volume of a regular tetrahedron? 1996-10-21
From Paul Bleier:
How does one calculate the volume and the height of a three sided pyramid formed by equilateral triangles?
Answered by Bruce Gilligan.
How many digits are there in 2^2520? 1996-10-07
From Rita Leung:
A)The number 64 is a sq., a cube, and a sixth power because 64=8*8, 4*4*4, 2*2*2*2*2*2. Find the smallest integer greater than 1 that is a sq., a cube, a 4th, 5th, 6th, 7th, 8th, 9th and 10th power. I have found that the answer is 2 to the power 2520th.
B) How many digits are in the correct answer to part A? The answer is uncertain. It is either 758 or 759. Can you give me any ideas?

Answered by Penny Nom.
Why is a circle divided into 360 degrees? 1996-09-30
From Kurtis Kredo:
I was recently wondering why a circle has been divided in to 360 degrees. When I asked my physics teacher he could not think of an answer. His guess is that it probably has to do with people long ago using the base 6 number system. I have a small inkling that it has to do with easy conversion or usage with radians or grads.
Answered by Chris Fisher.
Divisibility of 2n choose n. 1996-09-24
From Kathy Doan:
Can you prove that "2n choose n" is not divisible by 3, 5, and 7 for infinitely many n?
Answered by Penny Nom.
Repeating decimals 1996-09-09
From Alice:
What is the line called that is placed over the decimal to show that it is infinitely repeating?
Answered by The Centralizer.
Will this always yield a palindrome? 1996-08-28
From Andy Golden:
My friend Justin Skywatcher, said that when his children were in 6th grade, they were taught that if you take any number, reverse it and add the two together, a palindrome will result. If not, continue performing the above operations, and a palindrome will eventually result. Will this always yield a palindrome?
Answered by Chris Fisher.
Thousands, millions, billions and trillions 1996-08-22
From Blaine:
What are the names of the periods in groups of numbers like 123,456,789. I need to know the names of them for school. I already know the first ones: units thousands millions billions trillions.
Answered by Penny Nom and Diane Hanson.
How do you raise a number to an imaginary/complex power? 1996-07-03
From Andy Golden:
How do you raise a number to an imaginary/complex power? I know how you raise "e" to a complex power, like e^(pi*i): cos pi + i * sin pi But what about numbers other than "e"? What if I want to raise 5 to the 2i power? How is that done?
Answered by Chris Fisher.
What are fractals and are they of any practical use? 1996-06-26
From Ron:
What are fractals and are they of any practical use?
Answered by Chris Fisher.
Diagonals in a polygon 1996-06-18
From Steven:
How many diagonals are there in a polygon?
Answered by Denis Hanson.
Prove that these two lines are parallel. 1996-05-29
From Adam Piekarzewski:
Using parallel line theorems and side angle side etc. in triangle acd a line is drawn from point b on ac and point i on ad. another triangle fliped vertically and joined at d is named fdh, a line is drawn from point e on fd and point g on fh. ab = fg, ai = fe, ac || fh prove that bi || eg /- -/
Answered by Harley Weston.
How do you graph these inequalities? 1996-05-17
From Robert:
How do you graph the inequalities 5+gg, and g+h>6 on one coordinate plane?
Answered by Harley Weston and Maxine Stinka.
How do you graph these equations? 1996-04-30
From David Gamble:
How do you graph the equation x=3 on a x,y graph? Also how do you graph f(x)=-3x + 2 on the same graph?
Answered by Harley Weston and Maxine Stinka.
A coin problem 1996-04-16
From Julie Hebert and Simon Hamel:
How many weighings would you need to find out which pile is fraudulent?
Answered by Penny Nom.
An application of Pythagoras' theorem 1996-04-09
From Mike:
We'd like to know what practical applications there may be for the Pythagorean theorem.
Answered by Penny Nom and Maxine Stinka.
Area of an annulus 1996-04-04
From Ryan Levering:
What is the area of this annulus?
Answered by Penny Nom.
Area=Perimeter 1996-04-02
From Jason Pelowski:
When will the area and the perimeter of a right triangle be numerically equal?
Answered by Penny Nom and Maxine Stinka.
Cardano's method 1996-03-11
From Mike Gorski:
What is Cardano's method for finding the roots of a cubic equation?
Answered by Harley Weston.
Grade nine problem solving 1996-03-11
From Azmat Hussain:
Can math problems be actually fun and engaging?
Answered by Diane Hanson.
Pronunciation 1996-03-05
From Todd Stienhauser:
Is 123.4 pronounced "one hundred and twenty three and four tenths"or "one hundred twenty three and four tenths"?
Answered by Penny Nom.
Algorithms for roots 1996-02-22
From Charles Hewitt:
I have seen an algorithm for finding the square root of numbers. Are there similar such algorithms for higher roots?
Answered by Harley Weston.
Prime numbers 1996-02-22
From Charles Hewitt:
I would apprecaite it if you could share with me any algorithm to determine whether a number is prime.
Answered by Penny Nom.
Queue length 1996-02-22
From Frank Fronhofer II:
How long is the toll booth queue?
Answered by Harley Weston.
Pascal's Triangle 1996-02-19
From Richard:
Do you know of any resources that might help us make use of the numeric relationships in Pascal's triangle on a fairly simple basis?
Answered by Denis Hanson.
Euclid's Pythagorean proof 1996-02-14
From Sean:
What is Euclid's proof of Pythagoras' theorem?
Answered by Harley Weston.
What is the variance of the difference of two binomials? 1996-02-08
From Chris Johnson:
My co-workers and I have come up with different estimates of the z-statistic, and are in particular disagreement over the calculation of Variance for this problem. I am trying to find out, with a five percent level of significance, whether the new form yields a higher rate of return than the old form. Any thoughts, comments, or solutions?
Answered by Harley Weston.
(-3)x(-2) 1995-10-25
From Azmat:
Why is (-3)x(-2) = 6?
Answered by Herley Weston and Ed Giesbrecht.
Complex numbers 1995-10-22
From Jacquie:
Why should we teach complex numbers in high school?
Answered by Harley Weston.
Magic Square 1995-10-20
From Marianne and Carrie:
How can an 8 by 8 square have the same area as a 5 by 13 rectangle?
Answered by Denis Hanson.
What is the golden section? 1995-09-17
From Cindy:
What is the golden section of a line?
Answered by Denis Hanson.
Solides d’Archimède 2014-04-29
From Clémentine:
Pourquoi une pyramide a base carrée n'est elle pas un polyèdre archimedien ? J'ai pourtant essayé d'en construire un avec tout ses cotés égaux et ça fonctionne ! Aidez moi je n'en dors plus la nuit ? :S
Answered by Chris Fisher.
calcul de durée 2012-09-03
From laila:
un bus démarre de paris à 8h 45mn s'il fait 42h 20 mn de trajet entre paris et la ville suivante à quelle heure il arrivera à la ville suivante. est ce qu'il ya un shema pour calculer ou comment procéder. je sais qu'il faut additionner 8h 45 mn à 42h 20mn, mais comment on trouve le résultat rapidement. merci
Answered by Claude Tardif.
Les dérivées partielles 2012-05-16
From Haddouche:
Bonjour,
Je vous demande de bien vouloir m'éclaircir comment on calcule les dérivées partielles généralement étudiées dans les modèles de la concurrence fiscale à titre d'exemple celui de Wildasin ""NASH EQUILIBRIA IN MODELS OF TAX COMPETITION "", dont j'ai essayé toutes les méthodes et aucune d'elle n'est arrivée à trouver le résultat des dérivées de ce dérnier papier, à cet effet, je vous serais reconnaissant de m'éclaicir la méthode afin de résoudre ce problème qui pour moi un vrai mistère à percer. Je vous remercie infiniment à l'avance. Et comptant sur votre compréhension, mes salutations les meilleures. HA

Answered by Claude Tardif.
Combien de bonds aura-t-il besoin 2012-04-10
From zachary:
Un lapin fait toujours un bond de 3m suivi d'un bond de 5m. S'il maintient ce rythme de combien de bonds aura-t-il besoin pour franchir une distance de 584m ??
Answered by Claude Tardif.
un triangle 2011-11-07
From Haelita:
La corde élastique a une longueur de 60cm au repos . Quelle est la nouvelle longueur de la corde si on l'écarte de 11 cm en la tirant par son milieu ?
Answered by Claude Tardif.
Les calcules avec les heures 2011-09-09
From johanna:
bonjour je voudrais pouvoir expliquer a mon fils ces calcules avec les heures, mais franchement je n'y comprend rien, pouvez vous m'aider.
voilà le premier calcul:
1h35min29s - 46min37s

deuxiéme exercice:
calculer la durée d'une emission télévisé qui débute à 20h40min et se termine à 22h17min.
troisiéme exercie:
je suis partie en vacances le 25 juillet. Mes vacances ont durées 2 semaines. Quel jours suis-je revenu?
quatroéme exercice:
je pars de st-pierre, je prend le bus. je veut arrivé à st-denis à 10h et demi. je sais que la durée du trajet st-pierre /st-denis est de 1h50min. a quel heure dois-je partir de st-pierre.

Answered by Claude Tardif.
La conjecture de Goldbach 2011-02-18
From ahmedbenmoussa:
montrez que tout nombre entier paire supérieur à 4 s'écrit somme de deux nombres premiers
Answered by Claude Tardif.
une écriture plus simple des expressions 2011-01-13
From jessie:

trouver une écriture plus simple des expressions suivantes sachant que : x + y = 3 et x - y =-2
A=x-1+y+2             B=x-1+y-2
C=x+1+y+2            C=x+1+y-2


Answered by Claude Tardif.
(5+3)/2-3= 2010-12-10
From chantal:
(5+3)/2-3=
Answered by Claude Tardif.
Terminologie mathematique 2010-10-31
From Adil:
Bonjour ,

Pourriez-vous m'indiquer les titre et auteur d'un bon dictionnaire francais anglais de terminologie mathematique ? Auriez-vous egalement l'adresse d'un site web traduisant du francais a l'anglais les termes ert expressions mathematiques ?

Merci par avance,
Adil

Answered by Pierre-Louis Gagnon.
Un polynome de degré 3 2010-10-09
From ait:
comment determiner n polynome P de degré 3 verifiant pour tout reel x la relation p(x+1)-p(x)=x²
Answered by Claude Tardif.
Quel est le 6e nombre dans la serie suivante 2010-08-13
From jeff:
Quel est le 6e nombre dans la serie suivante: 1,2,6,42,1806,...
Answered by Stephen La Rocque and Claude Tardif.
l'orientation d'un plan dans l'espace 2010-03-22
From gianno:
comment calculer l'orientation d'un plan dans l'espace
Answered by Claude Tardif.
Résolution d'une équation 2010-02-25
From sellin:
Il faut que j'exprime x en fonction de m dans l'équation suivante et je n'y arrive pas: -3x + 1 = (m-1) / 2 * x + (2m + 3) / 5
Answered by Claude Tardif.
Volume d'une sphere entre 4 autres spheres 2010-01-18
From Bonfils:
Bonjour,

Je souhaiterais en fait calculer le rayon de la sphère max qui pourrait s'insérer entre 4 sphères de rayon connu. Merci par avance pour votre réponse. Sincères salutations

Answered by Chris Fisher.
L'aire d'un hexagon, octagon etc 2010-01-07
From danie:
C'est quoi la formule pour l'aire d'un hexagon, octagon etc. est-ce que c'est Nombre de cotes x ((L'apothem x la mesure d'un cote tout divise par 2)???
Answered by Claude Tardif.
Les polyèdres 2009-11-12
From Paul:
Parmi tous les polyèdres dont les sommets sont sur une sphère et dont les faces sont isométriques (pas nécessairement régulières, ni nécessairement triangulaires),
"(2,3,5)" (voir définition ci-après) est-il celui qui présente le plus grand rapport entre les rayons des sphères inscrite et circonscrite?

"(2,3,5)" désigne le polyèdre (de centre O) dont les faces sont des triangles isométriques tel que: si ABC est une face, les mesures des dièdres d'arêtes OA, OB, OC sont pi/2, pi/3 et pi/5.
Merci pour ce site.
Paul

Answered by Robert Dawson and Claude Tardif.
Le plus grand commun diviseur 2009-11-11
From Katie:
Question: Pendant mon cours de MAT 1300, mon enseignant m'a posé la question suivante:

(a, b) = 12
(a², b) = 24
(b, 72) = ?

Pouvez-vous m'aider s'il-vous-plaît?

Answered by Claude Tardif.
Le volume d'une boule 2009-10-22
From ve.lu.ch:
Pour calculer le volume d'une boule, il faut utiliser la formule :

4/3 x Pi x R au cube

Pourquoi 4/3 ?

Merci d'avance

Answered by Pierre-Louis Gagnon et Claude Tardif.
Un triangle isocèle 2009-10-08
From lauwick:
ABC un triangle isocèle en A de périmètre 16 cm. De plus son aire est égale au quart de l'aire du carré construit sur sa base bc. Quelles sont les longueurs des cotés de ce trangle? Merci
Answered by Claude Tardif.
Polyèdres 2009-10-07
From DEPASSE:
Si un polyèdre est inscriptible et circonscriptible, et si ses faces sont isométriques(non nécessairement polygones réguliers) , le dièdre entre deux quelconques de ses faces adjacentes est il toujours le même ? Si non, un exemple?
Answered by Claude Tardif.
Preuve du volume d'une boule 2009-04-06
From Geneviève:
J'aimerais avoir l'explication du volume de la boule, qui est (4pi R au cube) divisé par 3. J'aimerais savoir le pourquoi de l'équation, la preuve de celle-ci. Merci!
Answered by Pierre-Louis Gagnon.
Une multiplication 2009-04-03
From josee:
24/6x5+40-10=

le x est une multiplication et il n'y as pas de parenthèses.

Merci Josée

Answered by Antoine Letarte and Claude Tardif.
Algebre 2009-03-04
From Jonnathan:
Pour entrer sur le site d'une exposition, il en coute 41$ pour 2 enfants et 2 adultes mais 37.50$ pour 3 enfants et un adulte. Quel est le prix de la rentree pour l'enfant et quel est le prix de la rentree pour l'adulte? il faut utiliser la methode de comparaison ou de substitution
Answered by Claude Tardif.
Calculs de minutes en heures 2009-02-23
From Denis:
Je suis en train de suivre un cours en navigation maritime et je dois changer souvent des minutes en heures. ex: 495 minutes = ?h??

Je désire avoir le cheminement le plus simple a faire pour ce type de calcul. Je ne travail pas avec excell. Je veux une formule a faire seulement avec une calculatrice élémentaire. Merci pour votre aide, j'apprécie beaucoup votre coup de main. Denis.

Answered by Claude Tardif.
Un système de plusieurs équations et plusieurs inconnues 2009-01-22
From Cédrick:
J'ai un problème écrit à résoudre est-ce que vous pouvez m'aider ?

Le premier est le tiers de la somme des deux autres.
La somme du premier et du deuxième est 13.
Le produit du deuxième et du troisième est 56.
Réponse: Le premier nombre est___________________
Le deuxième nombre est _________________
Le troisième nombre est _________________

Answered by Pierre-Louis Gagnon, Antoine Letarte at Claude Tardif.
Un volume d'une capacité minimal 2009-01-05
From Pauline:
je doit faire un volume d'une capacité minimal de 2500 cm 3 en quell matiere que ce soit
* un prisme a base pentagonale non réguliere
* une sphere
* stella octangula

Answered by Pierre-Louis Gagnon.
La longueur de la corde 2008-12-24
From Lucile:
Voici l'énoncé :

On appelle (C) un cercle de centre O et de rayon 6cm. A et B sont deux points du cercle (C) tels que AOB = 50°

Calculer la longueur de la corde [AB] (arrondi au millimètre). On justifiera soigneusement toutes les étapes.

Cela fait déja 1 semaine que je trouve pas la réponse a cet énoncé .

Pouvez vous m'aider ? Merci d'avance .

Answered by Pierre-Louis Gagnon.
Probleme de trigonométrie 2008-12-06
From Maryne:
On donne AB=15cm, AC=30,5cm et A=35°. Calcule BC à 0;01cm près. Calcule B et C à 0,01° près. Calcule l'aire du triangle à 0,01cm² près. Sachant que le triangle n'est pas rectangle. Merci
Answered by Pierre-Louis Gagnon.
J'ai l'équation X^n=Y Je cherche à sortir n. 2008-11-05
From Jean-Philippe:
Bonjour.
J'ai l'équation X^n=Y Je cherche à sortir n.
Existe-t-il une équation donnant n=f(X,Y) ?

Answered by Pierre-Louis Gagnon et Émile Létourneau.
Le volume du tas 2008-05-28
From ribot:
je cherche la formule mathematique du volume d un tas de cailloux formule employée dans le bâtiment
Answered by Thierry Moisan.
Un tetraèdre régulier 2008-05-05
From delabroye:
La figure ci-contre est un tetraèdre régulier :
AB = AC = BC = AD = 6cm

1) Représente un patron de ce tetraèdre
2) Calcule le volume de ce tétraèdre

Answered by Thierry Moisan.
méthode pour trouver l'air d'un carré 2008-04-08
From Mélodie:
je doit trouver au moins 4 manieres différentes de calculer l'air d'un carré. j'en ai déja trouvée 2 mais il m'en faut le plus possible...
1. longueur x largeur
2. division en triangles, puis air des triangles et multipler par l'air des triangles

Answered by Thierry Moisan.
Énigme 2008-04-06
From Frédérique:
Tu es dans une pièce ou il y a 3 interrupteurs reliés a 3 lumières dans une autre pièce que tu ne vois pas.Comment fais-tu pour savoir a quelle lumière chacun des interrupteurs est relié si tu ne peux aller voir dans l'autre pièce qu'une seule fois!Merci a l'avance!!!
Answered by Claude Tardif.
Combien d'éleves fréquentent cette école? 2008-04-04
From Jeff:
Une ecole compte sept fois plus d'éleve que d'enseignants et enseignantes .De plus , le nombre de membres du personnel non enseignant correspond a la moitié du nombre d'enseignats et enseignantes . Si l'on tient compte de l'ensemble des éleves , du personnel enseignant et du personnel non enseignante , on dénombre 714 personnes . Combien d'éleves fréquentent cette école?
Answered by Antoine Letarte.
La dérivée de f(x)=ln[(1/16)(-x^2+6x+16)] 2008-03-31
From Mathilde:
Je viens de reprendre mes études et j'ai quelques lacunes, pourriez - vous m'aider à trouver la dérivée de cette fonction.
Comme ça je pourrais vérifier si mon résultats est juste avant de faire l'étude de fonction.
f(x)=ln[(1/16)(-x²+6x+16)]
Merci
Mathilde

Answered by Maxime Fortier Bourque et Pierre-Louis Gagnon.
Le terme suivants 2008-03-30
From jacques:
voici 2 suites. Explique la logique qui permet de passer d'un terme a l'autre dans chacune des suites et trouve le terme suivants. 1)douze,5,trente,6,neuf,4,seize,... no.2) 26,vx,18,dt,36,tx,51,...
Answered by Maxime Fortier Bourque.
la capcite en metre cube de ma remorque 2008-03-22
From pages:
je voudrai connaitre la capcite en metre cube de ma remorque les dimensions font 124-95 -34 merci
Answered by Pierre-Louis Gagnon.
Un bénéfice de 5% 2008-03-11
From mahiques:
Un propriétaire decide de vendre deux parcelles de terrains pour un montant total de 141750 F. Il fait un bénéfice de 15% sur la première et une perte de 10% sur la deuxième. L'ensemble de la transaction lui a rapporté un bénéfice de 5%.combien a-t-il vendu chacune des parcelles ?
Answered by Pierre-Louis Gagnon.
Logarithmique et semi logarithmique 2008-03-08
From azaar:
Je veux savoir comment passer d'un échelle linière à l'échelle logarithmique ou semi logarithmique ?
Answered by Lierre-Louis Gagnon.
La valeur absolue 2008-02-09
From Elodie:
que signifie la notation x entre 2 traits verticaux?
Answered by Claude Tardif.
Je voulais savoir d'où provient le signe # ?! 2008-01-17
From Dimitri:
Je voulais savoir d'où provient le signe # ?! On l'appelle diez, sur les téléphones surtout, mais j'aimerai savoir plus précisément à quoi sert ce signe (autre que sur nos appareils) ! Et pourquoi, en anglais, s'il est placé avant un chiffre, on dit 'number' (par exemple #1, 'Number one'). Son origine m'interesse tout autant que son utilité !
Answered by Claude Tardif.
Arithmétique modulaire 2008-01-10
From hausknecht:
ma grand mère a entre 60 et 80 ans.il y a un an son âge était un multiple de 3.dans deux ans son âge sera un multiple de 2,de 3 et de 9.cette année , c'est un multiple de 5.quel est l'âge de ma grand mère?
Answered by Claude Tardif.
800 poules pondent 800 oeufs en 8 jours 2007-12-17
From LINDA:
800 POULES EN 8 JOURS PONDENT 800 OEUFS, COMBIEN 400 POULES VONT-ELLES PONDENT D'OEUFS EN 4 JOURS?
Answered by Claude Tardif.
Une sphere 2007-12-03
From karine:
voici ce que je dois trouver ; j'aiune boule disco mesurant 48'' de diametre je veux savoir combien de petit miroir il y a sur cette boule si les petits miroires on 1'' carré ?
Answered by Claude Tardif.
F=4a-3c+6b 2007-11-22
From olive:
comment calcule ton ?
F=4a-3c+6b
mon fils et en 4ème et je suis dépassée, je ne peut pas l'aider merci de le faire à ma place.

Answered by Claude Tardif.
La somme de deux fonctions 2007-11-19
From maud:
Consigne : Ecrire la fonction f comme somme de deux fonctions u et v définies sur I. Donner le sens de variation de u et de v sur I. En déduire le sens de variation de la fonction f sur l'intervalle I indiqué. f(x)=-2x+(1sur x) I=]0;+infini[

Correction : Sens de variation de f sur I=]0;+infini[ On a f(x)=u(x) + v(x), avec {u(x) = -2x et v(x) = 1sur x La fonction u est strictement décroissante sur R, donc sur I ( droite avec coefficient directeur -2 négatif). La fonction v qui est la fonction inverse est stricyement décroissante sur [0;+infini[. Donc, la fonction f = u+v est strictement décroissante sur [0;+infini[.

Ma question : Pourquoi la fonction v et la fonction f ne sont pas définies sur le même intervalle que la fonction u c'est-à-dire sur l'intervalle I indiqué ?

Answered by Claude Tardif.
La somme de deux entiers naturels est égale à 83. 2007-11-06
From abraham:
La somme de deux entiers naturels est égale à 83. Si on divise le plus grand par le plus petit , on obtient 4 comme quotient et 17 comme reste. Quels sont les deux nombres ? Justifier clairement la réponse. Merci de me répondre au plus vite.
Answered by Claude Tardif.
Le symbole ∈ epsilon 2007-10-10
From Cora:
Je veux aider ma fille avec ses devoirs. C'est quoi le symbole qui semble un E, mais c'est un demi-circle? ie:X ∈ N
Answered by Claude Tardif.
un jardinier plantent 10 arbres 2007-09-30
From yannick:
un jardinier plantent 10 arbres.il a reussi a les disposer en formant 5 ligne de 4 arbres. retrouver une disposition possible
Answered by Claude Tardif.
Quel est ce nombre 2007-09-30
From fred:
Un groupe de 7 enfants vient de recevoir une grosse quantité de billes. ils font le partage : toutes les billes sont distribuées et ils en ont chacun le même nombre. le nombre total de billes s'écrit avec les mêmes chiffres que 1991. Quel est ce nombre
Answered by Claude Tardif.
pente nulle 2007-08-31
From stephanie:
si la pente dune droite est nulle alors cette droite est quoi ???
Answered by Claude Tardif.
Exemples d'équations a résoudre 2007-08-17
From Stéphanie:
j'aimerais connaitre les terme pour désigner pour les chiffres dans l'algèbre et pouvoir avoir quelques exemples d'équations a résoudre!!
Answered by Claude Tardif.
Combien vais je avoir de surface ? 2007-05-22
From Maëlle:
J'ai 5m3 de terre et veux en mettre 5 cm partout pour faire un terrain de jeux. Combien vais je avoir de surface ?
Answered by Claude Tardif.
Mesure/volume 2007-05-09
From Ashley:
Calcule le volume des 2 contenants et quels est le plus grands.
Answered by Claude Tardif.
Le pourcentage en +ou en- que représent la diférence de 2 nombres 2007-04-29
From Jean:
pouvez vous me donner la formule pour trouver le pourcentage en +ou en- que représent la diférence de 2 nombres ex: 8594 et 7634
Answered by Claude Tardif.
Illusion géométrique 2007-04-04
From Dominique:
Je n'arrive pas expliquer la modofocation de surface.
Answered by Claude Tardif.
Le perimetre d un cercle 2007-03-15
From Martins:
Comment calculer le perimetre d un cercle sachant que le diametre et de 25 de plus que le perimetre
Answered by Claude Tardif.
Dérivée logarithmique 2007-03-04
From Albert:
Déterminer la dérivée première des fonctions logarithmiques suivantes : 1) f (x) = x exposant x 2) f (x) = x exposant e exposant 2x
Answered by Claude Tardif.
Qui suis-je? 2007-02-28
From un parent:
bonjour, le problème est le suivant : Qui suis-je? Mon tiers augmenté de mes deux cinquième est égal à la moitié de mon triple Serez t'il possible d'avoir la solution à ce problème et que je puisse y comprendre un peu quelque chose quand même Je vous remercie d'avance et espère bientôt vous lire
Answered by Claude Tardif.
algèbre et fractions 2007-02-05
From marion:
Une certaine somme était inscrite sur un livret de caisse d'épargne au 1er janvier. Le 1er mars, on retire le tiers de ce qui était placé au 1er janvier. Le 1er juin, on retire le quart de ce qui était placé au 1er janvier. Il reste sur ce livret une somme égale à 1/7 des sommes retirées plus 1 600 Quel était le montant du livret d'épargne au 1er janvier?
Answered by Claude Tardif.
boulier français 2007-01-19
From Stéphanie:
Je prépare une exposition sur le centième anniversaire du vieux couvent de Château-Richer dans la province du Québec qui s'ouvrira en mai 2007. Dans ce couvent les soeurs de la Congréagation du Perpétuel Secours ont enseigné pendant de nombreuses années. Pour mettre en valeur, le site et l'exposition, nous souhaitons mettre les visiteurs à l'épreuve dans l'ambiance d'une classe d'antan (début 20ème siècle) avec, entre autres, les bouliers français que nous possédons et leur faire faire des exercices de mathématiques avec. Malheureusement, nous ne savons pas comment faire les quatre opérations de base (addition, soustraction, multiplication et division) avec. Pouvez-vous nous l'expliquer de façon détaillé, s'il vous plaît?
Answered by Claude Tardif.
Des grains de blé 2007-01-06
From Renaud:
Remplir un échiquier avec des grains de blé de la façon suivante un grain dans la 1ère case, deux grains dans la 2ème case, ainsi de suite en doublant jusqu'à la 64e case Combien de grains de blé il y a dans la 64e case?
Answered by Claude Tardif.
45 000 litres d'eau a 15 degres 2006-12-21
From Troussiere:
j'ai 45 000 litres d'eau a 15 degres combien de litres d'eau a 50 degres doit on rajouter pour obtenir une eau a 25 degres?
Answered by Claude Tardif.
volume d'un cône 2006-11-29
From Emmanuel:
On peut diviser un cube en 3 pyramides et ainsi en déduire le volume d'une pyramide. Mais comment peut-on déduire le volume d'un cône de celui d'un cylindre? Comment visualise-t-on la division d'un cylindre en trois?
Answered by Claude Tardif.
la somme des aires des deux dommaines 2006-11-06
From Nina:
Une ficelle de longeur L = 1m est coupée en deux morceaux. Avec l'un des morceaux, de longeur x, on forme un carré, et avec l'autre un cercle. A quel endroit doit on couper la ficelle pour que la somme des aires des deux dommaines obtenus soit minimale?
Answered by Claude Tardif.
Maximiser l'aire d'un triangle isocèle 2006-10-31
From Flav:
ABC est un triangle isocèle en A tel que AB= 1 pour quelles valeurs de BC, l'aire de ce triangle est-elle maximale?
Answered by Claude Tardif.
logarithme 2006-10-24
From Julie:
Loga(x^2-4) - loga(x-2)
Answered by Claude Tardif.
Un commis temporaire reclame 1200$ 2006-06-10
From Suzanne:
Un commis temporaire reclame 1200$ pour 2 semaines de travail (5 jrs semaine) a raison de 15$ de l'heure. Au cours de ces 2 semaines, le commis a inclus par erreur dans ses heures de travail la periode de 30 min pour la pause de diner. Quel est le montant a payer...J'ai essaye de trouver une reponse et je n'y arrive pas.
Answered by Claude Tardif.
Questions de Probabilités 2006-05-18
From Guy:

Le directeur d'un cirque a retenu cinq numéros pour composer son spectacle : les chiens savants, l'illusioniste, les clowns, les tapézistes et les acrobates.

  1. de combien de façons peut il composer son spectacles?
  2. En supposant qu'il y ait équiprobabilité dans le choix de la composition du programme, calculer la probabilité de chacun des événements suivants:
    1. les clowns terminent le spectacle
    2. les acrobates débutent ou terminent le spectacle
    3. les trapézites suivent immédiatement les clowns
    4. les clowns sont séparés des chiens savants par les trapézistes et seulement par eux.

Answered by Claude Tardif.
Ordre des opérations 2006-05-15
From Chantal:
2324/4-153+619= je sais pas comment calculer merci
Answered by Claude Tardif.
Volume d'un cône 2006-04-24
From Christelle:
Caroline décide de se servir de ses connaissances mathématiques pour "rouler" son petit frère: "Arthur, dit-elle, je te propose que nous mettions chaucun un euro dans l'achat d'un cône glacé. Je mangerai la première, jusqu'à mi-hauteur, et toi, tu auras la seconde moitié." Combien la part de Caroline représenterai-elle par rapport à celle de son petit frère s'il acceptait ?
Answered by Claude Tardif.
ABCD est un carré de 4 cm de côté 2006-03-06
From Elodie:
ABCD est un carré de 4 cm de côté. On découpe un peit carré AEFG d côté x. Que vaut x si l'aire de AEFG est la moitié de l'aire de ABCD?
Answered by Claude Tardif.
Une énigme échelle/échafaudage(ou boite) 2005-11-08
From Christian:
J'ai un problème & souhaite demander votre aide pour résoudre une énigme échelle/échafaudage(ou boite)
Answered by Claude Tardif.
problème de trigonométrie 2005-11-01
From Manon:
Soit un triangle ABC, de cotes a , b et c, avec A=45° et B=60°.
Apres avoir demontre que 3a²-2b²=0 (que jai fais en utilisant la regle du sinus) et a²-2c²+2ac=0 (que jai fait en utilisant la regle du cosinus), vous montrerez que laire du triangle ABC peut sexprimer: { [ 3 + racine carre 3 ] / 12 } b² unites.

Je ne sais pas comment arriver a cette expression de laire. Je narrive pas a exprimer c² et ac en fonction de b².. et je pense que ca pourrait vraiment aider..
puisque je suis arrive a letape:
ac[(racine carre 3) / 4]
avec ac = a² + c² - b²
et a² = 2/3 b²
ms c²= 1/3 b² + ac

Answered by Claude Tardif.
exercice! 2005-09-28
From Un eleve:
soit x et y deux nomres réels telque:
1 ≤ x2-x*y+y2 ≤ 2
1) Montrer que: 2/9 ≤ x4+y4 ≤ 8
2) Montrer que: x2*n+y2*n ≥ 2/32*n telque n est un nombre entier naturel et n≥3.

Answered by Chris Fisher.
Mon problème (né d'un retour aux conclusions du prix Nobel d'économie ) revisité 2005-09-27
From Nzitat:
Soit L un sensemble de distributions de probabilités. Désignons par p une distribution de probabilité quelconque. En clair p=(p0,..., pm) où m >0 est fini. Appelons Vi, i=1,...,n, une fonction définie sur L et à valeurs dans l'ensemble des réels. De même Wj, j=0,...n est une fonction définie sur L et à valeurs dans l'ensemble des réels. Vi, Wj sont définies ainsi pour tout p:....
Answered by Claude Tardif.
Problème avec rapport point, prix... 2005-09-08
From Nico:

Le prix d'un diamant est proportionnel au carré de son poinds. Un diamant de 0.45 g vaut 7500euros.

1. Combie coute un diamant de 0.693 g ?

2. Quel est le poids d'une diamant valant 45000 euros ?

Je vous demande pas de le raisoudre à ma place mais j'aimeré comprendre comment je peut calculé le rapport entre le point et le prix.


Answered by Claude Tardif.
Le prix d'un diamant est proportionnel au carré de son poinds 2005-09-07
From Nico:

Le prix d'un diamant est proportionnel au carré de son poinds. Un diamant de 0.45 g vaut 7500euros.

1. Combie coute un diamant de 0.693 g ?

2. Quel est le poids d'une diamant valant 45000 euros ?

Je vous demande pas de le raisoudre à ma place mais j'aimeré comprendre comment je peut calculé le rapport entre le point et le prix.


Answered by Claude Tardif.
quel sont les plus grands diviseurs communs? 2005-09-07
From Sylvain:
quel sont les plus grands diviseurs communs?
Answered by Claude Tardif.
problème de math 2005-06-03
From Nadia:
1* La somme de 3 multiples de 6 consécutifs est 72.Quels sont ces nombres?

2* Dans la fabrication d’un alliage, on utilise 5g de manganèse pour 32g de fer.
a) Quelle quantité de manganèse retrouve-t-on dans un morceau de cet alliage s’il contient 512g de fer? 00015166

b) Quelle est la masse totale de ce morceau si le fer et le manganèse en sont les seules composantes?

3*...

Answered by Claude Tardif.
Problème de géométrie 2005-04-29
From Christian:
C un cercle de centre I et de rayon 3 cm. SORT est un carré inscrit dans le cercle C M est un point quelconque de C Calculer la somme des carrés des distances de M aux sommets du carré (MS2 + MO2 +MR2 + MT2) Cette somme dépend-elle de la position de M sur le cercle ?
Answered by Claude Tardif.
Un repère orthonormal 2005-04-26
From Ylène:

Dans un repère orthonormal, le point A a pour coordonnées (-2;3) et le point B a pour (4;-5). A partir des coordonnées des points A et B on propose les calculs suivants:

a/ (-2+4 sur 2 ; 3-5 sur 2)
b/ (4+2; -5-3)
c/ racine carré (4+2)²+(-5-3)²

dans chaque cas, quelle est la notion géométrique ainsi mise en évidence?


Answered by Claude Tardif.
Avec les chiffres 1 3 4 6... 2005-04-14
From Ambre et Miel:
avec les chiffres 1 3 4 6 utilisé une seule fois chacun soit en div multipl addit ousoustr je dois avoir 24 comme résultat
Answered by Claude Tardif.
1 ellipsoïde et une sphère 2005-04-04
From Tihon:
Dans le cadre de mon mémoire réalisé à l'université de Liège (Belgique), je suis amené à savoir si 1 ellipsoïde et une sphère ont ou n'ont pas d'intersection...Quelqu'un peut-il m'aider??
Answered by Claude Tardif.
La limite de l'aire du domaine colorié 2005-03-21
From Rita:
bonjour un petit problème qui m'a l'air bien compliqué et je n'est pour le moment aucune pistes pour demarré un carré de coté 1 est divisé en 9 carrés identiques et le carré central est colorié chacun des 8 carré qui restent est à son tour divisé et colorié suivant le meme procédé et on continue indéfiniment quelle est la limite de l'aire du domaine colorié?
Answered by Claude Tardif.
6 poules pondent 6 oeufs en 6 jours 2005-03-01
From Nathalie:
SACHANT QUE 6 POULES PONDENT 6 OEUFS EN 6 JOURS, COMBIEN 12 POULES PONDENT D'OEUFS EN 12 JOURS?
Answered by Claude Tardif.
l'aire d'un carré 2005-02-24
From Un eleve:
calculer l'aire d'un carré parfait sachant que les diagonales mesurent 8 mètres

Je ne sais pas comment lui expliquer.

Answered by Claude Tardif.
le système RSA 2005-02-19
From Un eleve:
Je suis actuellement en classe préparatoire MP au lycée Henri Poincaré de Nancy et je souhaiterais obtenir des informations sur l'exponentiation modulaire car je réalise un TIPE sur la cryptographie et plus particulièrement le système RSA. pourriez-vous m'indiquez en quoi consiste l'exponentiation modulaire et comment cela fonctionne.
Answered by Claude Tardif.
il y a 1/6 de voitures bleues, 2005-01-24
From Facrice:
il y a 1/6 de voitures bleues, 2/9 de voitures rouges, 5/12 de voitures blanches et 1/8 de voitures vertes. Sachant que 15 voitures sont d'une autre couleur. Combien y a t il de voitures de chaque couleur.
Answered by Claude Tardif.
J'ai deux fois l'âge que tu avais quand 2004-12-03
From Un eleve:
Julien dit à Nicolas:"J'ai deux fois l'âge que tu avais quand j'avais l'âge que tu as.Quand tu auras l'âge que j'ai la somme sera égale à 63".A toi de trouver l'âge de Julien et de Nicolas.
Answered by Claude Tardif.
Un problème de math 2004-09-10
From Sophie:
(V est en fait le signe de la racine carrée)

(3 / V27) x ( V75 ) = ???

Answered by Claude Tardif.
(x+y)^n = x^n + y^n + Z 2004-03-18
From Jean-Sébastien:
Bonjour, je me m’appelle Jean-Sébastien et je suis étudiant en secondaire 4. Je me demandais s’il est possible de développer le terme (x+y)n, lorsque n est un entier positif. Il est évident que xn+yn fera partie de la réponse mais comment définir le reste (Z dans l’exemple ci-dessous)?

(x+y)2 = x2+y2+2xy

(x+y)n = xn+yn+ Z

P.S. : Une preuve mathématique démontrant la possibilité ou l’impossibilité du problème serait grandement appréciée, merci.


Answered by Claude Tardif.
Marie achète un croissant et une tartelette 2004-02-01
From M. et Mme Simoneton:
Marie achète un croissant et une tartelette. Elle paie 1,64 €
Isabelle achète un croissant et un pain au chocolat. Elle paie 1,51 €
Marc achète un poain au chocolat et une tartelette. Elle paie 1,77 €
Trouver le prix d'un croissant, d'une tartelette et d'un pain au chocolat.

Answered by Claude Tardif.
Les 3 maisons 2003-11-16
From Amandine:
Sur la figure ci-contre on peut voir 3 maisons A, B et C nouvellement construites. Toutes les 3 doivent être reliées au gaz, à l'eau et à l'électricité. Cependant, tous ces raccordements doivent impérativement avoir lieu dans un même plan et les câbles et autres canalisations ne peuvent pas se croiser. Comment doit-on s'y prendre pour relier les 3 maisons à l'eau, au gaz et à l'électricité?
Answered by Claude Tardif.
Nombre d'or 2003-10-31
From Claude:

Comment démontrer que si a/b est égal au nombre d'or alors a+b/a est égal aussi au nombre d'or

Comment faut t il choisir a et b pour que le puzzle de lewis caroll soit réalisable? on sait déjà que les nombres 8 et 5 ainsi que 6 et 3 ne sont pas valables.


Answered by Claude Tardif.
Hauteur d'un tétraèdre irrégulier 2003-08-29
From Thibault:
Je cherche à connaître comment calculer la hauteur d'un tétraèdre irrégulier (aucune arêtes égales) en ne connaissant que la longueur des arêtes.
Answered by Claude Tardif.
Fractions 2003-05-10
From Jacques:
Voici la question qu'il pose: 2 x (1/2+1/4) + 3 + 2 x (1/3-1/4). La reponse est 4 2/3. Pourriez m'expliquer etapes par etapes ce que je dois faire.
Answered by Claude Tardif et Diane Hanson.
Un polyedre convexe 2003-04-07
From Hubert:
quelle sont les conditions nécessaire pour former un polyedre convexe avec un seul type de polygone?
Answered by Claude Tardif.
Le terme 2003-04-01
From Francine:
Je suis une enseignante de 5e/6e et j'aimerais savoir comment on traduit le terme "array" en français.
Answered by Claude Tardif.
les formules d'aire et de volume de les solides 2003-02-01
From Annie:
Je cherche le nom des mathématiciens qui ont découvert les différentes formules d'aire et de volume de tous les solides ( boule, cube, les prismes, cylindre, cône, pyramide, polyèdres réguliers). Je cherche aussi à trouver comment ils ont démontré l'exactitude de ces formules. L'important, c'est de connaître le nom des mathématiciens qui ont découvert ces formules.
Answered by Claude Tardif.
La longueur d'une corde 2003-01-22
From Un eleve:
Existe-t-il une formule permettant de calculer la longueur d'une corde quelquonque d'un cercle connaissant uniquement le rayon de celui-ci? Si oui quelle est-elle?
Answered by Claude Tardif.
le matricule 2003-01-12
From Sirkka:
"Dans un pays totalitaire, tous les habitants ont un matricule composé de 5 chiffres. Les policiers se reconnaissent au fait que le premier chiffre de leur matricule est strictement supérieur à la somme des quatre autres. ex: 90124 9>0+1+2+4=7 Combien y a-t-il (au plus) de policiers?"
Answered by Claude Tardif.
Volume d'une calotte ellipsoidale 2002-12-11
From Fabien:
Quel est le volume d'une calotte d'ellipsoide de hauteur h (ellipse de longueurs d'axes a, b, et c)? Je souhaite une formule exacte, un développement limité, voire une intégrale à estimer numériquement. Quelle méthode pour le calculer ?
Answered by Claude Tardif.
Une plantation d'arbres 2002-12-10
From Eric:
créer une plantation d'arbres de 6 rangées de 5 arbres, pour un total de 15 arbres . Je n'arrive pas à trouver la forme géométrique.Merci de me répondre que j'ai l'air moins bête envers mon gamin.
Answered by Claude Tardif.
100 centimètres 2002-11-13
From Un élève:
je voudrais savoir la contenance de 100 centimètres cube en litre?
Answered by Diane Hanson.
Les unités linéaires 2002-11-12
From Sandy:
je me demande qu'est ce qu'une unité linéaire, mesures de surface et des mesures de volume. je voudrais aussi savoir quelle est la règle de transformation pour les unités linéaires, les mesures de surface et les mesures de volume
Answered by Claude Tardif.
Un triangle équilatéral 2002-10-27
From Un eleve:
Soit ABC un triangle équilatéral construit dans le sens direct. Le point D est symétrique de A par rapport à la droite (BC), et le point E est symétrique de B par rapport au point C. L'intersection des droites (AD) et (BC)est notée H. On pose AB=a.
  1. Je dois calculer les longueurs AD et AE en fonction de a.

  2. Je dois montrer que le triangle ADE est équilatéral. J'arrive a prouver qu'il est isocèle en E mais j'aimerai trouver que AD=DE.

Answered by Claude Tardif.
La limite de (x(n)) 2002-10-01
From Une étudiante:
3) Soit L, le point ayant pour abscisse la limite de (x(n))

a) Exprimer la distance M(n)L en fonction de n

b) Déterminer le plus petit entier naturel n(o) pour lequel la distance Mn(o)L est inférieur à 0,01

c) Justifier alors qu'à partir du rang n(o), tous les points M(n) sont situés dans le segment de centre L et de rayon 0,0 1


Answered by Claude Tardif.
3 maisons 2002-08-23
From Wilfrid:
voila j'ai une petite énigme a vous proposez il y a 3 maisons chaque maisons doit être reliées par une arrivées d'eau , d'électricité et par le gaz sans que les lignes ne se croisses pouvez vous me dire si c'est possible car moi j'ai essayé et ne suis pas arrivé je vous joint un exemple pour que vous compreniez mieux merci d'avance
Answered by Claude Tardif.
Espaces vectoriels 2002-08-18
From Bacar:
Comment montrer que trois (ou plus que trois) sous espaces vectoriels sont en somme directe.
Answered by Claude Tardif.
Un hectolitre 2002-06-12
From Un eleve:
Pouvez-vous me dire combien d'hectolitres y a t-il dans un mètre cube (m3)?
Answered by Diane Hanson.
La traduction anglais 2002-05-27
From Une etudiant:
je voudrais savoir la traduction anglais de :

homothétie (mot de niveau intermédiaire (6-9))
apothème (mot de niveau internédiaire (6-9))

parce que je ne les trouve pas.

Answered by Chris Fisher at Claude Tardif.
Un problème mathématique 2002-04-23
From Mylène:
En me servant de ces chiffres 1 2 3 4 5 6 je dois les utiliser seulement qu'une fois en faisant l'équation suivante.
? X ?   X  ? __    __    ?   ? X ?   = 2 

Answered by Claude Tardif.
Un empilement de spheres 2002-04-16
From Sébastien:
Lorsque j'etais encore a l'universite je me souviens qu'un prof nous avez dit, sans le demontrer (ce n'est pas bien du tout de sa part) que l'espace minimal qui existe dans un empilement de spheres est de l'ordre de 26% Voici plusieurs jours que je cherche a resoudre ce probleme de geometrie, et je n'arrive meme pas a trouver comment l'aborder.
Answered by Claude Tardif.
Nombres algebriques sur Q 2002-04-11
From Un eleve:
On sait que le corps des nombres algebriques sur Q est algebriquement clos. A priori partant d'un polynome f a coefficients algebriques sur Q on devrait pouvoir un trouver un polynome g a coefficients rationnels dont une partie des racines est constituee de racines du premier. y-at-il un algorithme qui permet qui permet le passage de f a g ?
Answered by Claude Tardif.
Les quantités d'aliments 2002-03-09
From Benhacine:
soient les aliments A1, A2, A3.....An contenant respectivement le nutriment "a" aux quantités a1, a2, a3, et an sachant qu'un organisme a besoin d'une quantité connue de nutriment "a" égale à "Be",

Ayant: A1.a1 + A2.a2 +.....+ An.an = 100.Be

et

A1 + A2 + A3 +....+ An = 100

Quelles seront les quantités d'aliments A1, A2, et An qui satisferont le besoin défini "Be" de l'organisme?


Answered by Claude Tardif.
Une autre bille de rayon différent 2002-02-27
From Sarah:
une bille de 6 cm repose au fond d'1 cylindre droit dont la base est un disque de rayon 10 cm. On verse de l'eau dans le cylindre de façon à recouvrir exactement la bille. Démontrez que l'on peut remplacer la bille par une autre bille de rayon différent (mais supérieur) de sorte que l'eau initialement versée recouvre exactement cette nouvelle bille
Answered by Claude Tardif.
Ses points de discontinuités est les irrationnelles 2002-01-08
From Un eleve:
Je voudrait montrer qu'il n'existe pas de fonction de R--->R tels que ses points de discontinuités est les irrationnelles, en utilisant la methode de "baire".
Answered by Claude Tardif.
Fraction rationnelle 2001-12-11
From Audrey:
réduire en une seule fraction et simplifier.

(4-2x+x2)/(2+x) - (x+2)

Answered by Claude Tardif.
Minesweeper 2001-10-24
From Un eleve:
J'espere que vous pourrez m'aider. Est ce que vous connaissez un algorithme ou une strategie de resoulutiom pour le jeu du demineur (8x8 avec 10 mines)(Minesweeper).
Answered by Claude Tardif.
Un nombre entier relatif ou reel 2001-06-13
From Omar:
J'aimerais savoir c'est quoi un nombre entier relatif ou réel?
Answered by Claude Tardif.
Les tableaux de conversions 2001-05-03
From Jean-Jacques:
Sur quel site est il possible de trouver tous les tableaux de conversions de mètres,kg,litre,m2,m3

Quelle est la relation entre les m3 et les litres 1 m3 = 1000 litres = 1000 kg?


Answered by Claude Tardif.
Les dimensions d'un rectangle 2001-04-09
From Un elévè:
1. Les dimensions d'un rectangle sont a et b Si on augmente a de 3cm et b de 2cm, l'aire du rectangle augmente de 37 cm2, mais si on diminue a de 2cm et b de 1cm, l'aire du rectangle diminue de 16cm2. Quelles sont les valeurs de a et b?

2. Quelle est la valeur numerique de l'expression (ac+bd)+(ad-bc) sachant que a+b=1 et que c+d=1? Quelles sont les regles essentielles qui me manquent pour resoudre ceci?


Answered by Claude Tardif.
Le volume d'un ellipsoïde 2001-03-17
From Lise:
Je cherche à calculer le volume d'un ovoide, avez-vous une formule?
Answered by Claude Tardif.
Recherhce sur pi et exponentielle 2001-03-17
From Cyril:
étudiant a la fac de sciences de montpellier je cherche des informations concernant la demonstration suivante: "comment prouver que pi et exponentielle sont irrationnels?"
Answered by Claude Tardif.
Solides 2001-02-26
From Joanie:
J'ai présentement un projet à faire, je dois le remettre avant vendredi.

Je dois trouver entre 31 ou 35 solides: prismes et pyramides

J'ai a donner : le nom de chaque solide; son dessin; et le dessin de chaque solide décomposé en toute ses parties.


Answered by Claude Tardif.
Un lièvre et un chien 2001-02-20
From Ganay Alain:
Un lièvre à 9 sauts d'avance sur un chien. 7 sauts du chien égalent 11 sauts de lièvre. Quand le chien fait 4 sauts , le lièvre en fait 6. En combien de sauts le chien prend-il le lièvre?
Answered by Claude Tardif.
Boulier 2001-02-13
From Un élève:
Comment se sert-on d'un boulier (non asiatique) comportant 10 rangées de 10 boules chacune?
Answered by Claude Tardif.
Radical de n 2001-01-03
From Un eleve:
j'aimerai savoir ( d.Levy), élève de 1éS ce que signifie la phrase: radical de 2 + radical de 3 est un nombre négatif ( niveau intérmédiaire).
Answered by Claude Tardif.
Calculer la hauteur de chute de cette pierre 2000-12-16
From Marcus:
Une pierre tombe au fond d'un puit, à 5 secondes on entend un son (de la pierre). Calculer la hauteur de chute de cette pierre en tenant compte de la vitesse du son = 340m.s-1

Il faut savoir que pour la chute libre v= g.t ; h = 1/2 g.t2 ; v = 2g.h

g intensité du champs de pesanteur = 10 ; h hauteur de chute.


Answered by Claude tardif.
Le volume d'une sphère 2000-09-17
From Pitmill:
merci de me donner la formule pour trouver le volume d'une sphère.
Answered by Harley Weston.
isomorphisme 2000-08-10
From Romain Kroes:
Pour les beoins d'un ouvrage d'économie que je suis en train de terminer, pouvez-vous me dire qui est (sont) l'inventeur de l' "isomorphisme" en mathématiques (calcul tensoriel)?
Answered by Claude Tardif.
Un polyedre ayant comme face 20 triangles 2000-08-02
From Sonia:
J'aimerai savoir comment s'appelle un polyedre ayant comme face 20 triangles équilatéraux égaux.
Answered by Chris Fisher.
Exposant à la 0 2000-07-04
From Sylvie:
J'aimerais savoir combien fait 0 exposant 0.
Answered by Claude Tardif.
le nombre d'or 2000-06-14
From Belhaj Saad:
quel est le nombre d'or?
Answered by Claude Tardif.
Sens d'un mot 2000-06-01
From Edward Brisse:
Serait-il possible de connaître le sens précis du mot "PORISME". Exemple : "le porisme de Poncelet".
Answered by Claude Tradif.
la dimension fractale 2000-05-01
From François RIVOIRE, Julien CHOLLET et Alexandre DECOSTER:
nous sommes étudiants en première année MPSI en école d'ingénieur ISEN ( Institut supérieur d'électronique du Nord ). Nous avons choisi les FRACTALES ( ou plus précisément sur la dimension fractale et topologique ainsi que sur les suites logistiques) comme sujet de TIPE; serait-il possible que vous nous envoyez des démonstrations mathématiques ou des conseils car jusqu'ici nous avons eût beaucoup de mal à trouver des démonstrations se rapportant à ce sujet,
Answered by Claude Tardif.
Resolution of the equation f(x)=0 2000-04-01
From Naoufal:
How can i evalf the equation f(x)=0 with f function . Can you ansear me in french thank you very much
Answered by Claude Tardif.
Explications 2000-03-05
From Jean Barbe:
Comment expliquer l'algèbre à un adolescent du Sec. 111
Answered by Claude Tardif.
Point de partage 2000-02-21
From Sebastian Murciano:
J'aurais besoin de savoir où je peux trouver de l'informations, ou est-ce que vous pouvez me donner de l'information sur : Point de partage d'un segment étudié en Secondaire 4.
Answered by Claude Tardif.
La bissectrice de l'angle ABC 2000-02-16
From Laurent Gauthier:
Est-ce redondant d'ecrire "angle ABC" avec un accent circonflexe sur le B? Peut-on ou devrait-on se contenter d'ecrire soit ABC (avec l'accent), soit l'angle ABC (sans accent) ? Le contexte dans lequel ceci apparait est le syntagme "la bissectrice de l'angle ABC", c'est-a-dire que je me demande si on devrait ecrire "la bissectrice ABC " (avec accent), "la bissectrice de l'angle ABC" (sans accent) ou "la bissectrice de l'angle ABC" (avec accent).
Answered by Claude Tardif.
Septante 1999-12-18
From Guillaume Bessis:
Pourqoui et depuis quand en France emploie-t-on l'expression 'soixante-dix' et non septante pour désigner 70 ? (idem pour 90 et nonante)
Answered by Claude Tardif.
Les equations 1999-12-09
From Enciso Alberto:
j'ai les equations suivantes:

X1=Rs + A1/(B+W^2)
X2=Rs + A2/(B+W^2)
X3=Rd + A3/(B+W^2)
X4=Rd + A4/(B+W^2)
X5=Rg + A5/(B+W^2)
X6=Rg + A6/(B+W^2)

dont les coefficients Ai (i=1,2,..) et B sont constants (mais on ne conais pas leur valeur) et Rs, Rd, et Rg sont inconnues. Nous pouvons constater que la dependence de ces differentes expessions en fonction de W^2 est identique. Le tracé de l'une de ces expressions en fonction d'une autre est donc une driote.
.
.
.
Answered by Claude Tardif.

ln(x+3)+ln(x+2)=ln(x+11) 1999-11-24
From Thibault Portevin:
"Résolvez l'équation : ln(x+3)+ln(x+2)=ln(x+11)" Je suis bloqué, pourriez-vous m'aider?
Answered by Claude Tardif.
la pantographie 1999-11-18
From Erick et Christine:
Nous recherchons la définition de pantographie voltaïque, que représente cette "figure" ? est-ce qu'on peut en définir une unité de mesure ?
Answered by Claude Tardif.
logarithme neperien 1999-11-10
From Louise Kieffer:
D'où provient la valeur e ( 2,7....) des logarithmes népériens ?
Answered by Claude Tardif.
15 films 1999-11-09
From Hebert:
Un touriste revient de vacnces avec 15 films:
  • 2 films de photos d 'Italie
  • 8 films de photos de Grèce
  • 5 films de photos de Turquie.
Aucune marque distinctive ne permet d'identifier les films. Pour des raisons financières le touriste ne fait développer à son retour que 11 de ces 15 films qu'il choisit au hasard.NB : on donnera les résultats sous forme décimale approchée à 10-4 près.
  1. Combien y a t il de choix différents possibles de 11 films parmi les 15 ?
  2. Quelle est la probabilité que , parmi les 11 films développés, il y ait:
  3. ...

Answered by Claude Tardif.
Derivées partielle 1999-10-19
From Arnaud Flandin:
Quel est la definition des derivées partielle
Answered by Claude Tardif.
La longueur des segments formant un triangle 1999-10-05
From Michel Provencher:
Connaissant la longueur des segments formant un triangle,comment détermine-t-on chacun de ses angles si:
  1. il S'agit d'un triangle rectangle

    Sachant que la somme des angles d'un triangle est de 180 degrés et sachant par conséquant qu'un des angle est de 90 degrés (triangle rectangle) il reste donc, 90 degrés à partager entre les 2 angles restant. Si les 2 segments formant l'angle droit sont de même longueur on obtient un angle de 45 degrés pour les angles restant soit 1/2 angle droit ce qui ne me pose évidement aucun problème. Quel relation, S'il y en a une, y a t-il entre la longueur de ces 2 segments et les angles restants.

  2. il S'agit d'un triangle quelconque

Answered by Claude Tardif.
Application surjective 1999-09-30
From Charass:
je suis étudiant en premiere année de deug et je cherche apres une definition si vous pouviez m'aider, ça serait sympa. qu'est ce qu une application subjective?
Answered by Claude Tardif.
Traduction 1999-09-27
From Beatrice:
Pouvez-vous me traduire en anglais les terms ci-dessous?
  • Intrégration ensembliste
  • Espaces vectoriels normés
  • Traitement statistique du signal
  • Equations aux dérivées partielles
  • ...

Answered by Claude Tardif.
Le salinon d'Archimèdre 1999-03-11
From Don Craig:
I am trying to find the English translation of "Le salinon d'Archimèdre" and would appreciate any help. This is a figure, presumably studied by Archimedes, created from 4 semi-circles. Since I can't draw it for you, I will try to describe it with the help of the 5 collinear, horizontal points below.

. . . . . A B C D E 

A semi-circle is constructed on AE as diameter (let's say above AE).

Two more semi-circles are then constructed with diameters AB and DE on the same side of the line AE as the first semi-circle (above it). Finally, a fourth semi-circle is constructed on diameter BD, this time on the opposite side of the line AE from the others (i.e. below the line).

These semi-circles and the region enclosed by them constitute what is called in French "Le salinon d'Archimèdre".

If you know the English name of this curve I would appreciate it if you let me know.
Answered by Harley Weston.

Maths 1999-01-11
From Stephane Roissard:
Soit ABC un triangle dans lequel les trois médianes sont de meme longueur. Montrer que ce triangle est quilatéral.
Answered by Jack LeSage.
Le sang humain 1998-10-06
From Golden:
Le sang humain contient approximativement 2,5 x globules rouges. Chaque globule a un rayon de 0,004 mm. Si on le place ces globules bout a bout, quelle est la longeur de la chaine en millimetres? En kilometres? Compare toi avec la circonference de la terre (24 000 km) (Travail avec exposants)
Answered by Claude Tradif.
Un angle solide 1998-05-06
From Giol:
Qu'est ce qu'un angle solide ?? J'ai beau chercher dans mes documents, je ne trouve rien de bien convaincant sur le sujet si ce n'est une définition qui me semble bien vague et creuse ... En vous remerciant de votre attention ( puissiez vous illustrer votre réponse par un exemple, s.v.p...)

Bonjour,

Answered by Chris Fisher.

les fractions 1998-02-24
From Colette Huguenin:
Bonjour je révise mes math de tout les secondaire et le livre louer la bibliothèque n'est pas entier voici le genre de problêmes qui me bloque

(5 4/5+1/2)divisé 1/3

je sais que je dois commencer par la parenthèse mais...je fait quoi comme opération avec le 5? dois-je le multiplier ou l'additionner ou.......????? si je pouvais seulement avoir la base des fractions je redébloquerais surement
Answered by Diane Hanson.

Volume tetraedre 1998-02-13
From Solecki Yannick:
j`aimerais savoir s`il existe une formule de calcul simplifie pour trouver le volume du tetraedre en fonction de ses cotes et sans utiliser une expression matricielle.
Answered by Chris Fisher.
Question de trigonométrie 1997-12-11
From Jean-Pierre Quesnel:
Je suis dans le désert et je parcours 1000 km à partir du point "A" jusqu'au point "B". Si je reviens au point "A" et fais une rotation de 8 degrés en faisant un autre 1000 km, quelle sera la distance en km entre les points "B" et "C".
Answered by Diane Hanson et Penny Nom.
 
Page
1/1

 

 


Math Central is supported by the University of Regina and The Pacific Institute for the Mathematical Sciences.

CMS
.

 

Home Resource Room Home Resource Room Quandaries and Queries Mathematics with a Human Face About Math Central Problem of the Month Math Beyond School Outreach Activities Teacher's Bulletin Board Canadian Mathematical Society University of Regina PIMS